Você está na página 1de 1044

MATEMÁTICAS-

DISCRETA
. Y
COMBINATORIA
Wna introducción con aplicaciones'
TERCERA-EDICIÓN

RALPH P. GRIMALDI
Rose-Hulman lnstítute ofTechnology

~ Addison-Wesley lberoamertcana
Argentina• Cl'le • Colombia• Espalla • - Unidos
México • Perú • PIJOtlo Rico . Venezuela
\l,rsión enespaiioldelaollra Dis<:me and Combinatoria/ M - . A n app/iedinmxb,ctim,
de Ralpb P. Grimaldi, publicada originalmente en inglés por Addison-Wesley Publishing
Company, !ne., Reading. Massachusens @ 1994, por Addison-Wcsley Company, !ne.

Esta edición en español es la wúca autorizada

ADDISON-WESLEY IBEROAMERICANA
Ma1abia 2363--rG. Buenos Aires 1425, AlgeDtina
Cruz 1469 DqKo 21 1ndcpmdcocia. Santiago, Chile
Apartado Ai:n:o 241-943 Smta fe de Bogod.. Colombia
- 3 bajo. Madrid 28014, -
l Ja:::ob Way, Readi:D&, Mass. 01867, E.U.A.
...,.- Paml 22-012. M!xko D.F. 14000. México
k. San ADtoaio Este. 11.11.m.. 658. depl:o. D. Urb. Veanua Rossi, Lima lS, Pu4
El Moate Mall 'Z' pd(), oficina 19-B, Ave. Mdoz River:a Hato Rey, 00918, Puerto Rico
. . . , . - Postal S14S4, C..- IOSO-A. Vcnomcla

@ 1997 por AoolsoN-WESLEV ilEaoAMDJCANA, S-A.


Wilmington, Delaware, E.U.A.

Impreso en Estados Unidos, Prinud in U.S.A.

ISBN 0-201-65376-1

123456789 J().DQC-OI 00999897


1
A mia nu:ulre
e alta memoria di mio padre
con affetto e stima
Prefacio

L os a ~ tecnológicos de los ú]timos veinticinco años han producido varios cambios


en el curriculum de licenciatura. Estos cambios han apoyado el des.arrollo de muchos
CUISOS de uno o varios semestres en )os que se presenta lo siguiente:

l. Mtrodos discn:tos que subrayan la naturalez:a finita inh<Rnte a muchos problemas


y estructuras;
2. La combinatoria: el álgebra de la enumeración o las túnicas para contar;
3. La teoría de grafos con sus aplicaciones e interrelaciones con áreas como las cstruc-
tuns de datos y los mtlodos de optimización; y.
4. Las estructuras algebraicas finiw que surgen ju.oto con disciplinas como la teoría
de códigos, los mtlodos de enumeración, las~ de puertas y los diseilos combinatorios.

Una de las principales ra:zones para el estudio de las materias de cualquiera de esos
cuatro grandes temas es la abundancia de aplicaciones que se encuentran en las ciencias de
la computación; en particular, en las ireas de las estructuras de datos, la teoría de los
lenguajes de computación y el análisis de algoritmos. Tambi~n existen ap1icaciones en
ingenieña y en las ciencias físicas y biológicas, as{ como en la estadística y las ciencias
sociales. En consecuencia, las matem.fticas discreta y COID.binatoria proporcionan un va-
lioso material para los estudiantes de ottas áreas, no sólo para quienes se especializan en
matenw:icas o en ciencias de la computación.
El propósito principal de esta nueva edición es seguir ofrecieodo una introducción a las
matemáticu discreta y combinatoria. El material incluido~ dirigido a los principiantes,
po< lo que se ofrece una gran cantidad de ejemplos con explicaciones detalladas. (Los
ejemplos se numeran por separado y se ha utilizado una línea gruesa para denotar el final
de cada ejemplo.)Además, cuando se dan demosu-aciooes. tstas tambitn se presentan con
el suficiente detalle (pensando en los principiantes).
El texto tiene los siguientes objetivos:

l. Presentar al estudiante de primer o segundo alio de licenciatura. o de niveles pre-


vios. los temas y t6c:o.icas de los ~odos disactos y el razonamiento combinatorio.
Los problemas del conteo, o la enumeración, necesitan un análisis cuidadoso de la
estructura (por ejemplo, el hecho de que sea necesario o no el orden o la repetición) y
Prefacio

las posibilidades lógicas. loclu.so podría tratarse de una cuestión de existencia en algu•
nos casos. A1 seguir ese análisis cuidadoso. veremos con frecuencia que la solución de
un problema necesita t«:nicas sencillas para contar los resultados posibles que surgen
de la descomposición del problema dado en subproblemas menores.
2. Presentar una amplia gama de aplicaciones. En este aspecto, donde se necesiten las
estructuras del álgebra abstracta, sólo se desarrollará la leoría básica necesaria para la
aplicación. Además. las soluciones de algunas aplicaciones conducen por sí mismas a
procedimientos iterativos que llevan a algoritmos espec:fficos. La aplicación del enfo-
que algorítmico a la solución de los problemas es fundamental en la matemática discre-
ta, además de que refuerza los estrechos lazos entre esta disciplina y el área de ciencias
de la computación.
3. Desarrollar la madurez matemática del estudiante a travts del estudio de un área
muy diferente de lo que tradicionalmente se incluye en el cálculo y las ecuaciones
diferenciales. Aquf, por ejemplo. se presenta la opon.unidad de establecer re sultados al
contar cieno grupo de objetos en más de una forma. Esro proporciona lo que se conoce
como identidades combinatorias; tambi~n se introduce una nueva ~cnica de demostra-
ción. En esta edición, la naturaleza de la demostración, junto con lo que constituye un
argumento válido, se desarrolla en el capírulo 2, que tambi~n incluye las leyes de la lógica
y las reglas de inferencia. El análisis es en este caso más amplio que el de la segunda
edición. Las demostraciones por inducción matemática (junto con las definiciones
recursivas) se presentan en el capítulo 4 y desputs se utilizan en todos los capfrulos
posteriores.
Respecto a los teoremas y sus demostraciones, en muchos casos se ha intentado
derivarlos a partir de la observación de ejemplos específicos. Así mismo, cuando una
situación finita da como resultado aJgo que no es cieno en el caso infinito, se ha hecbo
hincapit en ella para que se examine con atención. Se han omitido las demostraciones
demasiado largas o muy especializadas; sin embargo, se proporcionan referencias para
el lector interesado en revisar la validación de los resultados de las escasas demostra-
ciones omitidas. (El infasis puesto en las demostraciones dependerá de los objetivos de
cada profesor y de sus estudiantes.)
4 . Presentar un análisis adecuado de los temas para el estudiante de ciencias de la
computación, quien tomará cursos más avanzados en áreas como las de estrucruras de
datos, teoría de 1enguajes de computación y análisis de algoritmos. E1 estudio de los
grupos, anillos, cuerpos y álgebras booleanas también proporcionará una introducción
aplicada para los estudiantes de la carrera de matemáticas que deseen continuar su
estudio del álgebra abstracta.

El lector de este libro debe contar, en primera instancia, con una base sólida en matemá•
ticas de nivel bachillerato y un inte~s por abordar y resolver distintos tipos de problemas.
Si bien no se requiere una capacidad particular para la programación, en la obra aparecen
varios segmentos de programas (presentados principalmente en Pascal). diseñados y ex-
plicados para reforzar algunos ejemplos particulares. Respecto a1 cálculo. más adelante en
este prefacio mencionaremos su alcance en los capítulos 9 y 10.
Mi principal motivación para escribir la primera y segunda ediciones de este libro fue
el apoyo recibido, durante varios años. de mis alumnos y colegas, así como de los estu-
diantes y profesores que usaron la primera edición del texto en diferentes universidades.
Esas dos ediciones reflejaban mis intereses y los de mis alumnos, así como las recomenda-
Prefacio ..,;¡

ciones del Comminec on tbe Uodergraduate Program in Matbematics y de la Associatioo


of Computing Machinery. Esta tercera edición sigue la misma línea y refleja ahora las
recomendaciones tanto de los profesores como, en particular, de los estudiantes que han
utilizado o estin utilizando la segunda edición.

Características

A continuación se describen brevemente algunas de las principales características de la


nueva edición, las cuales tienen por objeto ayudar al lector (estudiante, profesor, etc.) a
aprender los fundamentos de las matemáticas discreta y combinatoria

tn&sis e.o algoritmos y aplicaciones. En todo el texto se presentan algoritmos y apli-


caciones e n muchas áreas. Por ejemplo:

1. El capítulo l incluye varias situacioncs particulares en las que se necesitan los


temas introductorios de enumeración; en particular, un nuevo ejemplo estudia e l tema
del recuento excesivo.
2. La sección 7 del capítulo 5 proporciona. una focroducción a la complejidad
computacional. Este material se usa des~ en la sección 8 de dicho capítulo para
analizar los tiempos de ejecución de algunos programas elementales, uno de los cua-
les trata de la generación de los números de Fibonacci.
3. El material del capítulo 6 estudia los lenguajes y las máquinas de CSlados finitos.
Esto introduce al lector a un área importante de las ciencias de la computación: la
teoría de los lenguajes de computación.
4. Los capítulos 7 y 12 incluyen análisis de las aplicaciones y algonnnos que tratan
de la ordenación topológica y las tk nicas de búsqueda conocidas como búsqueda e n
profundidad y búsqueda en anchura.
S. En el capítulo 10 se analiza el tema de las relaciones de rccurrencia~ se tratan
aplicaciones sobre (a) la ordenación por el método de la burbuja, (b) la búsqueda
binaria, (c) los números de Fíbonacci, (d) la curva copo de nieve de Koch, (e) las
redes de resistencia lineal, (f) la estructura de datos llamada pila, y (g) los árboles
binarios.
6. El capítulo 16 pn:senta las propiedades fundamentales de la estructura algebraica
llamada grupo. Aquí se muestra cómo se usa esta estructura en el estudio de la teoría
algebraica de códigos y en los problemas de conteo que requieren el ~todo de enu-
meración de Polya.

Explicadones detalladas. Ya sea e n un ejemplo o e n la demostración de un teorema.


las explicaciones pretenden ser cuidadosas y completas. La presentación busca princi-
palmente que el lector que se inicia en el estudio de estas materias las comprenda mejor.

Ejercicios. El papel de los ejercicios en cualquier texto de matemáticas es primordial. La


cantidad de tiempo invertida en los eje,cicios influye en gran medida en el rittno del curso.
El profesor podrá comprobar que el tiempo de clase utilizado en el análisis de los ejerci-
cios variará, dependiendo del intcr~s y los conocimientos matemáticos de los alumnos.
.,¡¡¡ Prefacio

En los 17 capítulos hay más de 1700 ejercicios. Los que aparecen al final de cada
sección siguen, por lo general, el orden en que se desarrolló el material de la sección.
Estos ejercicios están diseñados para (a) repasar los conceptos básicos de la sección:
(b) enlazar las ideas preseoiadas ea las primeras se<:eiooes del capítulo; y (e) presentar
conceptos adicionales relacionados con los temas de la sección. Algunos ejercicios pi-
den el desarrollo de un algoritmo, o la escritura de un programa de computador; con
frcc:ueocia, para resolver cierta particularización de un problema general. Nonnalmen-
te esto sólo requiere un mínimo de experiencia en programación.
Cada capítulo concluye con un conjunto de ejercicios complementarios. Estos pro-
porcionan un repaso adicional de las ideas presentadas en el c apítulo y tambiin utilizan
material desarrollado e n capítulos anteriores.
Al final del texto se proporcionan las soluciones de casi todas las panes de los ejer-
c icios de número impar.

Resúmenes de capítulo. La óltima sección numerada de cada capítulo presenta un


resumen y un repaso histórico de las ideas principales de dicho capítulo. Esto tiene por
objeto dar al lector un panorama del contenido del capítulo, así como información para
un estudio más profundo y sobre otras aplicaciones. Dicha profundización puede ba-
sarse sin ningún problema en la lista de referencias prOP.(lrcionada..
En particular, los resúmenes que aparecen al final de los capítulos 1, S y 9 incluyen
tablas de las fórmu1as de enumeración desarrolladas dentro de cada uno de estos capí-
tulos. En algunos casos, estas tablas incluyen resultados de capítulos anteriores para
compararlos y mostrar la forma en que los nuevos resultados amplían los anteriores.

Organización

Las áreas de matemáticas discreta y combinatoria son en ciena medida nuevas para el
currículum de l icenci~ de modo que existen varias opciones acerca de los temas que
deben estudiarse en los cursos. Cada profesor y estudiante puede tener diferentes intere-
ses. En consecuencia, los aspectos que se abarcan en esta obra son bastante amplios, como
corresponde a un curso general Aun así, siempre habrá más temas que algunos lectores
desearían incluir; así mismo, habrá diferencias de opinión respecto al orden en q ue se
presentan algunos temas en este texto.
En todo el texto se hace ~nfasis en la naturaleu e importancia del enfoque a lgorítmico
para la solución de problemas. Las ideas y puntos de vista acerca de la resolución de
problemas se ven reforzados aún más por las interrelaciones entre la enumeración y la
estructu~ dos de los temas principales que proporcionan un enlace para e l material desa-
rrollado en el libro.
El material se subdivide en cuatro áreas principales. Los primeros siete capítulos for-
man el nt1cleo del libro y presentan los fundamentos de las matemáticas discretas.Aquí se
presenta el material suficiente para un curso de matemáticas discretas con duración de un
trimestre o un semestre. El material del capítulo 2 puede ser revisado por los estudiantes
que ya tengan conocimientos de lógica. Para los interesados en el desarro11o y escritura de
demostraciones, este material debe examinarse con cuidado. Un segundo curso, que haga
hincapi~ en la combinatoria. debe incluir los capítulos 8, 9 y JO (y si el tiempo lo pennite.
las secciones 1, 2. 3, 9, lOy 11 del capítulo 16). Enel capítu1o9 se usan algunos resultados
del cálculo; a saber, los fundamentos de la diferenciación y la descomposición en fracdo-
Prefacio ix

nes simples. Sin embargo, aquellos que deseen sal~este capítulo, podrán estudiar las
secciones 1, 2, 3, 6 y 7 del capítulo 10. A partir de los capítulos 11, 12 y 13 puede desarro-
llarse un curso que ponga el ,fofasis en la teoóa y las aplicaciones de los grafos finitos.
Esos capítulos forman la tercera subdivisión principal del texto. Para un curso de álgebra
aplicada, los capítulos 14, 15, 16 y 17 (la cuana y llltim.a subdivisión) versan sobre las
estructuras algebraicas (grupo, anillo, álgebra booleana y cuerpo) e incluyen aplicaciones
relativas a las funciones de conmutación, la teoría a1gebraica de códigos. y los disei\os
combinatorios. Por dltimo, un curso acerca del papel de las estructwas discretas en la
ciencia de la computación puede desarrollarse con el material de los capítulos J 1, 12, 13 y
15, y las secciones I a 8 del capítulo 16. En esta parte hay aplicaciones acerca de las
funciones de conmutación y la teoría algebraica de códigos. así como una introducción a
la teoría de grafos y los árboles, y su papel en la optimización.
Otros posibles cursos pueden desarrollarse con base en las siguientes relaciones de
dependencia enue los capículos.

Capítulo Dependencia de capítulos anterions


1 Sin dependencia
2 Sin dependencia (Por lo tanto, un instructor puede iniciar un curSO en
matemáticas discretas con el esrudio de la lógica o con una introducción a
la enumeración.)
3 1, 2
4 1, 2, 3
5 1, 2, 3
6 1, 2, 3, 5 (La sección 6.1 tiene una dependencia mc:norde las secciones4. I .
4.2)
7 1, 2, 3, 5. 6 (La sección 7.2 tiene una dependencia menor de las secciones
4.1,4.2)
1, 3 (El ejemplo 8.3 de la sección 8. 1 tiene una dependencia menor de la
sección 5.3) •
9 1, 3
10 1, 3, 4, 5, 9
11 1, 2, 3, 4, S (Aunque algunas ideas de la teoría de grafos se mencionan en
los capítulos S, 6, 7, 8 y 10, el material de este capítulo se desarrotla con
independencia del material de teoría de grafos dado en estos resultados
anteriores.)
12 1,2,3,4,5, 11
13 3, 5, 11, 12
14 2, 3, 4, 5, 7 (La función phi de Eulcr (♦) se usa en la sección 14.3. Esta
función se obtiene en ia sección 8.1 pero el resultado puede usarse aquí, en
el capítulo 14. sin estudiar el capítulo 8.)
15 2, 3, 5, 7
16 1, 2, 3, 4, 5, 7
17 2, 3, 4, 5, 7, 14

Además, el índice de materias se ha disei.\ado cuidadosamente para hacer que el texto


sea aún más flexible. Los términos se presentan con listas principales y varias listas sccun•
dañas. Tambifo hay una buena cantidad de referencias cruzadas. Esto se ha hecho con el
fin de ayudar al profesor que desee modificar el orden de presentación y elegir uno propio.
Pretaft0

Cambios en la tercera edición

Los cambios a la tercera edición de Matemáticas discreta y combinatoria varían de lo


mcxlerado a lo amplio. Sin embargo, el tono y el propósito del texto se mantienen. Sigue
siendo un objetivo del autor proporcionar. dentro de estas páginas, una introducción sóli-
da, legible y comprensible a las matemáticas discreta y combinatoria para el estudiante
que comienza. Con base en los comentarios y críticas de los profesores y estudiantes que
usaron las dos primeras ediciones, se han hecho algunos cambios en la tercera edición:

• Esta edición incluye una introducción al uso de la notación para la suma (sigma) en el
capítulo l. así como para la notación del producto (pi) en el capítulo 4. Las ediciones
anteriores presuponían que el lector estaba familiarizado con este material.
• Se ha ampliado el tratamiento de la escritura de demosuaciones en el capítulo 2. En
particular, la nueva sección 2.5 incluye material acerca de las reglas de especificación
universal y generalización universal y su papel en la escritura de demostraciones. Esto
continúa luego en la primera sección del capírulo 3, donde se aplica para demostrar
teorem~ n::lativos a conjuntos y subconjuntos.

• En la segunda edición, las definiciones recursivas se desarrollaron en la sección 4.1,


donde se introdujo el concepto de inducción matemática. Aquí hemos dedicado la sec-
ción 4.2 al desarrollo del concepto de definición recursiva Esto nos ha permitido pre-
sentar los números de Fibonacci y los números de Lucas relacionados con ellos antes
(del capítulo JO).

• Los números armónicos aparecen (por primera vez) en la sección 4.1 y se ha amplia-
do e l estudio de los números de Catalan en la sección 10.5.

• Se han incluido tres apéndices como ayuda al lector: los dos primeros son un repaso
de las funciones exponencial y logarítmica. y una introducción a las matrices. El lector
verá que la exposición en esta pane es adecuada para comprender el material del texto
donde surgen esos coaceptos. El último apéndice estudia los conjuntos numerables y
no numerables, para los que quieren algo más de conjuntos y funciones, es decir, quie-
nes deseen ampliar las ideas presentadas en los capírulos 3 y 5.
• Desde la publicación de la segunda edición, el autor ha tenido oportunidad de hablar
con alumnos que han estudiado las matemáticas en este libro. Estas charlas. junto con
revisiones de los profesores que han usado la segunda edición, han llevado a añadir 87
nuevos ejemplos. El propósito de estos ejemplos es ayudar al estudiante en situaciones
donde, tal vez. sienta la necesidad de otro ejemplo.

• Hay más de 1700 ejercicios en esta nueva edición. Esto representa un incremento de
más de 300 respecto de la segunda edición.

• Muchos de los resúmenes y repasos históricos que aparecen al final de los capítulos
han sido ampliados, y la lista de referencias ha sido actualizada.Además, cada resumen
y repaso histórico incluye ahora al menos una ilustración del matemático mencionado
en el repaso.
Prefacio xi

Agradecimientos

Si el espacio lo permitiera. me guswía mencionar a cada uno de los estudiantes que me


han ayudado y animado mientras escribía las tres ediciones de este libro. Sus sugerencias
me sirvieron mucho para eliminar algunos errores y ambigüedades. y mejorar así la expo-
sición. Los que más me ayudaron en este aspecto fueron Paul Griffith, Mereditb Vannauker,
Paul Barloon, Byron Bishop, Lee Bccl<harn, Bn:U Hunsaker, Tcm Vandcrlaan, Michael
Bryan, John Breitenbach, Dan Johnson, Brian Wilson,Allen Schneider, John Dowell, Cha,.
les Wilson, Richard Nichols, Charles Brads, Jonathan Atkins, Kenneth Schmidt, Oonald
Stanton, Mark Stremler, Stcphen Smalley, Anthony Hinrichs y Kevin O'Bryant
Agradezco a Lany Alldrcdge, Claude Anderson y Martin Rivers sus comentarios rela-
tivos al matcria1 de ciencias de la computación. y a Barry Farbrothcr, Paul Hogan, Den nis
Lewis, Charles Kyker, Keith Hoover y Jerome Wagner sus instructivas observaciones so-
bre algunas de las apHcaciones.
Agradezco el entusiasmo y apoyo persistentes del equipo de Addison-Wesley (tanto el
actual como el anterior); en particular, a Waync Yuhasz.. Tbomas Taylor, Micbacl Payne,
Charles Glaser, Mary Criuenden, Herb Merritt, Maria Szmauz,Adeline Ruggles, S1ephanie
Botvin, Jack Castecl y Oeborah Schneider. Laurie Rosatone y Juliet Silveri merecen un
reconocimiento especial por su notable conttibución a esta tercera edición.
Tambifn estoy en deuda con mis colegas John Kinncy, Roben Lopez, Nacer Abrouk.
Gary Shcnnan, Georgc Berzsenyi y. particularmente, AJfred Schm.idt, por su interés y
apoyo durante la elaboración de esta obra y sus ediciones anteriores.
Doy mi agradecimiento y reconocimiento a los siguientes revisores de la primera. se-
gunda y tercera ediciones.

NonnaE.Abel Digital F.quipment Ccrporation


Larry Alldredge Alsys, /ne.
Oaude W. Anderson m Rose-Hu/man Jnstituu ofTtchncltJgy
V. K. Balakrishnan Universiry o/Maine a1 Orono
Roben Bamhill Univtniry of Utah
Dale Bedgood E.ast Texas Statt Univusity
Jerry Beehler Tri-Stau Univusity
Kata1in Bencsath Manluman Col/eg,
AllanBishop Western Jllinois Univtrsiry
Monte Boisen Virginia Polyttchnic lnstitute
Samuel Councilman California State Universiry at Long Btach
Roben Crawfonl Western Kellfllek:y Univtrsity
Ellen Cunningham, SP Sailll Mary-ofrh,.111,ods Colleg,
Carl DeVi10 Naval Postgraduau School
Car! Eckberg San Diego Stau Universiry
RobenGeitz O/Nrlin Coll,g,
James A. Glasenapp Rochester lnsrituu o/Technolbgy
David S.Han Rochesur /nstítute o/Technology
Maryann Hastings Marymount Co/lege
W. MackHill Worcestu Srau Col~ge
!Uchanl lltis Willameru Univusity
Akihiro Kanamori &,ston Univtrsity
John Konvalina University o/Nebraska at Omaha
xii Prefacio

Rochelle Lcibowitz Whearon College


James T. Lcwis Universiry of Rhode Island
Y-Hsin Liu Universiry ofNebraska ar Omaha
Hugh Montgomery University ofMichigan
Richard Orr Rochu ter lnstitute o/Technology
Edwio P. Oxford Baylor Universiry
Jobo Rausen New Jersey lnstitute ofTechnolcgy
Manin R.ivers Lexmark lnternational, !ne.
James H. Scbmerl Universiry ofConnecticut
Paul S. Schnare Eastern Kentucky University
Debra Diny Scott University of ll-ísconsin ar Grun Bay
Daltoo Tarwater Texas Tech Universiry
JeffTecosky-Feldman Harvard Universiry
W. L Terwilliger Bowling Green Stare Universiry
Dooald Thompsoo Pepperdine University
Thomas Upsoo Rochesrer lnstiture ofTechnology
W. D. Wallis Sourhern lllinois University
LarryWest Virginia Commonwealth University
YIXÍD Zhaog Universiry ofHebras/ca '!,l Omaha

Agradezco particulannente a Douglas Shier de Oemson University el extraordinario tra-


bajo que realizó revisando los manuscritos de las tres ediciones.
La traducción de la dedicatoria es de la Dra. Yvonne Panaro de Nonhern Virgina
Com.munity College. Gracias Yvonne. y gracias a Patter (Patricia Wickes Thurston) por su
labor para consegujr la traducción.
Un texto de esta magnitud requi7re el uso de muchas referencias. Los miembros del
personal de la biblioteca del Rose-Hui.man lnstirute of Technology siempre estuvieron
disponibles cuando se necesitaban libros y anícuJos, así que debo expresar mi aprecio por
el esfuerzo de John Robson. Sondra Nelson y en especial Margaret Ymg.
Por último, y con seguridad, la más importante nota de agradecimiento penenece de
nuevo a la siempre pacieote y animosa secretaria del departamento de matemáticas de
Rose-Hulman, la señora Mary Lou McCuJlougb. Gracias por tercera vez. Mary Lou, ¡por
todo tu trabajo!
Desgraciadamente, los errores, ambigüedades y come ntarios equivocados restantes son
responsabilidad exclusiva del autor.

R.P.G.
Terre Haute, Indiana
Contenido

PART E 1
Fundamentos de las
matemáticas discretas

Principios fundamentales d• I conteo 3


1.1 Las reglas de la suma y del producto 3
1.2 Permutaciones 6
1.3 Combinaciones: El teorema del binomio 19
1.4 Combinaciones con repetición: Distribuciones 33
1.5 Una aplicación a las ciencias físicas (Opcional) 43
1.6 Resumen y repaso histórico 44

2 Fundamentos de lógica 51
2.1 Conectivas básicas y tablas de verdad 51
2.2 Equivalencia lógica: Las leyes de la lógica 61
2.3 Implicación lógica: Reglas de inferencia n
2.4 El uso de cuantificadores 98
2.5 Cuantificadores, definiciones y la demosrración de teoremas 121
2.6 Resumen y repaso histórico 137

3 Teoría de conjuntos 143


3.1 Conjuntos y subconjuntos 143
3.2 Operaciones de conjuntos y las leyes de la teoría de conjuntos 156
3.3 T~oicas de conteo y diagramas de Venn 169
3.4 Unas palabras en cuanto a la probabilidad 172
3.5 Resumen y repaso histórico 176

4 Propiedades de los enteros: Inducción matemática 183


4.1 El principio del buen orden: Inducción matemática 183
4.2 Definiciones recursivas 201
4.3 El algoritmo de la división: Nómeros primos 213
4.4 El máximo comón divisor. El algoritmo de Euclides 225
4.5 El teorema fundamental de la aritmética 232
4.6 Resumen y repaso histórico 238

xiii
Contenido

5 Relaciones y funciones 245


5.1 Productos cartesianos y relaciones 246
5.2 Funciones: en general e inyectivas 251
5.3 Funciones sobreyectivas: N6meros de Stirling del segundo tipo 260
5.4 Funciones especiales 267
5.5 El principio del palomar 275
5.6 Composición de funciones y funciones inversas 280
5.7 Complejidad computacional 293
5.8 Análisis de algoritmos 297
5.9 Resumen y repaso histórico 308

6 Lenguajes: Máquinas de estados finitos 315


6.1 Lenguaje: La teoría de conjuntos de las cadenas 316
6.2 Máquinas de estado finito: Un primer encuentro 327
6.3 Máquinas de estado finito: Un segundo encuentro 335
6.4 Resumen y repaso histórico 343

7 Relaciones: La segunda vuelta 349


7 .1 Repaso de relaciones: Propiedades de las relaciones 349
7.2 R«onocimiento por computador: Matrices cero-uno y grafos dirigidos 357
7 .3 Órdenes parciales: Diagramas de Hasse 371
7.4 Relaciones de equivalencia y particiones 382
7 .5 Máquinas de estado finito: El proceso de minimización 388
7.6 Resumen y repaso histórico 394

PARTE 2
Temas adicionales
de conteo 401

8 El principio de inclusión y exclusión 403


8.1 El principio de inclusión y exclusión 403
8.2 Generalizaciones del principio 413
8.3 Desórdenes: Nada está en el lugar correcto 418
8.4 Polinomios de torre 420
8.5 Disposiciones con posiciones prohibidas 424
8.6 Resumen y repaso histórico 428

9 Funciones generatrices 433


9. 1 Ejemplos introductorios 433
9.2 Definiciones y ejemplos: Tknicas de cálculo 436
9.3 Particiones de entef9S 445
9 .4 La función generatriz exponencial 449
9.5 El operador de suma 454
9.6 Resumen y repaso histórico 456
Contenido

10 Relaciones de recurrencia 461


10.1 La relación de recum,ncia lineal de primer orden 461
10.2 La relación de recurrencia lineal homogénea de segundo orden con
coeficientes constantes 471
10.3 La relación de recurrencia no homogénea 482
10.4 El método de las funciones generatrices 493
10.5 Un tipo especial de relació n de recurrencia no lineal (Opcional) 499
10.6 Algoritmos divide y vencerás (Opcional) 511
10.7 Resumen y repaso histórico 521

PARTE 3
Teoría de grafos y
aplicaciones 527

11 Una introducción a la teoría de grafos 529


11.1 Definiciones y ejemplos 529
11 .2 Subgrafos, complementos e isomorfismos de grafos 537
11.3 Grado de un vátice: recorridos y circuitos eulerianos 550
11.4 Gnúos planos 560
11.5 Caminos y ciclos hamiltonianos 578
11.6 Coloración de grafos y polinomios cromáticos 588
11.7 Resumen y repaso histórico 598

12 Árboles 607
12.1 Definiciones, propiedades y ejemplos 607
12.2 Árboles con raíz 614
12.3 Árboles y ordenaciones 634
12.4 Árboles ponderados y códigos prefijo 638
12.5 Componentes biconexas y puntos de articulación 644
12.6 Resumen y repaso histórico 650

13 Optimización y emparejamiento 657


13.1 Algoritmo del camino más corto de Dijkstra 657
13.2 Árboles recubridores minimales: Los algoritmos de Kruskal y Prim 665
13.3 Redes de transporte: El teorema de flujo mwmo y corte mínimo 671
13.4 Teoría de emparejamiento 683
13.5 Resumen y repaso histórico 694

PARTE 4
gebni moderna aplicada 699

14 Anillos y aritmética modula r 701


14.1 La estructura de anillo: definición y ejemplos 701
14.2 Propiedades y subestructuras de un anillo 709
xvi Contenido

14.3 Los enteros módulo n 117


14.4 Homomorfismos e isomorfismos de anillo 722
14.5 Resumen y repaso histórico 730

15 Álgebra booleana y funciones de conmutación 735


15.1 Funciones de intercambio: Formas normales d.isjuntiva y conjuntiva 735
15.2 Redes de puertas: Suma mínima! de productos y mapas de Kamaugh 745
15.3 Aplicaciones adicionales: Condiciones de indiferencia 756
15.4 La estructura de un álgebra booleana (Opcional) 762
15.5 Resumen y repaso histórico TT2

16 Grupos, teoría de la codificación y método de enumeración de Polya 777


16.1 Definiciones. ejemplos y propiedades elementales 777
16.2 Homomorfismos, isomorfismos y grupos cíclicos 784
16.3 Clases laterales y teorema de Lagrange 791
16.4 Elementos de la teoría de la codifieación 793
16.5 La métrica de Hamnúng 798
16.6 La verificación de paridad y matrices generadoras 801
16.7 Códigos de grupo: Decodificación con líderes de clase 806
16.8 Matrices de Hamming 810
16.9 Enumeración y equivalencia: Teorema de Bumside 812
I 6.10 El índice de ciclo 820
16.11 El inventario de patrones: Método de enumeración de Polya 824
16.12 Resumen y repaso histórico 829

17 Cuerpos finitos y diseños combinatorios 835


17. l Anillos de polinomios 835
17.2 Polinomios irreducibles: Cuerpos finitos 843
17.3 Cuadrados latinos 853
17.4 Grometrfas finitas y planos afines 859
17.5 Diseños de bloques y planos proyectivos 865
17.6 Resumen y repaso histórico 871

Apéndice 1 Funciones exponenciales y logarítmicas A-1


Apéndice 2 Matrices, operaciones con matrices y determinantes A·13
Apéndice 3 Conjuntos numerables y no numerables A-27
Soluciones 5-1
Índice de materias 1·1
PARTE

1
FUNDAMENTOS
DELAS
MATEMÁTICAS
DISCRETAS
1
Principios
fundamentales
del conteo*

L a enumeración, o conteo, puede parecer un proceso obvio que un estudiante aprende al


estudiar aritm,tica por primera vez. Pero luego. scglln parece. se presta pQCa atención
en lo que se refiere a un desarrollo más amplio del conteo conforme el estudiante pasa a
'1eas "'lmS dillcilcs.. de las matemáticas, como el álgebra, la geometría, la trigonometría y
el cálculo. En consecuencia, este primer capítulo deberá servir como advertencia acerca
de la seriedad y dificultad del "mero" conteo.
La enumeración no termina con la ariunética. TambiW tiene aplicaciones en áreas como
la teoría de códigos. la probabilidad y estadística (en matemáticas). y el análisis de algotjttnos
(en ciencias de la computación). Los capítulos posteriores mostrarán algunos ejemplos
específicos de esw aplicaciones.
A medida que vayamos entrando en este fascinante campo de las matemáticas, nos
encontraremos con muchos problemas que se pueden enunciaren forma sencilla pero que
son ..duros" de resolver. Así, asegúrese de aprender y comprender las fórmulas básicas,
pero ,w confíe demasiado en ellas, ya que. sin el análisis de cada problema, el mero
conocimiento de )as fórmulas es casi inútil. En ,-ez de ello, acep1e el reto de resolver
problemas poco usuales o diferentes de los problemas que ha visto en el pasado. Busque
soluciones con base en su propio análisis sin importar si es exactamente la que proporcio-
na el autor. Con frecuencia existen varias vías para resolver un problema dado.

1.1
Reglas de la suma y del producto

Nuestro estudio de las matemáticas discreta y combinatoria comienza con dos principios
básicos del conteo: las reglas de la suma y del producto. Los enunciados y aplicac:ioncs
iniciales de estas reglas parecen sencillos. Al analizar problemas más complejos, a menudo
podemos descomponerlos en panes que puc:den resolverse mediante estos principios bási•
cos. Queremos desarrollar la capacidad de -descomponer" dichos problemas y acomodar

• E.a cscc te.no se bu utilizado }os &ámiaol- ·cooceo·. ""reicoen10., y -contar para traducir d tbmioo
co1U1rinr. (N. dtl T.)
(.apítulo 1 Principios fundamentales del conteo

nuestras soluciones parciales para llegar a la respuesta final. Una buena fonna de h;icerlo
consiste en analizar y resolver muchos problemas distintos de enumeración, tomando nota,
todo el tiempo, de los principios utilizados en la solución. Éste es el método que seguiremos.
Nuestro prime r principio del conteo puede expresarse de la forma siguiente:

Regla dela _Si_¡,riaaa_pa,li

............
aaasoguada...,.,...miliDneclear..-,
_,,.........,__,
aoalqaieracle•+n formas.

Observe que cuando decimos que una ocurrencia particular. como una primera tarea, pue-
de realizarse de m formas, se supone que estas m formas son distintas, a menos que se
indique lo contrario. Esto será así a lo largo de tcxJo el texto.

La biblioteca de una universidad tiene 40 libros de texto de sociología y 50 de antropolo-


gía. Por la regla de la suma, un estudiante de esta universidad puede elegir entre 40 + 50 =e
90 libros de texto para aprender acerca de alguno de estos dos temas.

La regla puede amplíarse a más de dos tareas, siempre que ninguna pareja de tareas pue~
ocurrir en forma simultánea. Por ejemplo, un instructor de ciencias de la computación que
tiene, digamos, cinco libros de nivel introductorio acerca de APL, BASIC, FORTRAN y
Pascal puede recomendar cualquiera de estos 20 libros a un estudiante interesado en aprender
un primer lenguaje de programación.

El instructor de ciencias de la computación del ejemplo 1.2 tiene dos colegas. Uno de ellos ·
tiene tres libros de texto acerca del análisis de algoritmos y el otro cuenta con cinco libros.
Si n denota el número máximo de libros diferentes sobre el tema que el instructor puede
pedirles prestados, entonces 5 .s n .s 8, ya que ambos profesores podrían tener copias del
mismo libro (o libros).

El siguiente ejemplo presenta nuestro segundo principio de conteo.

A1 tratar de tomar una decisión acerca de la ampliacíón de una planta, un administrador


organiza a 12 de sus empleados en dos comités. El comité A está formado por 5 miembros
y está encargado de investigar los resultados favorables posibles de dicha ampliación. Los
otros siete empleados, del comité B. revisarán todas las posibles repercusiones desfavora-
bles. Si el administrador decide hablar sólo con un comité antes de tomar su decisión,
entonces, por la regla de la suma, existirán 12 empleados a los que puede llamar. Sin
embargo. para tener menor sesgo, antes de tomar una decisión, decide hablar con un miem-
1.1 Reglas de la suma y del p,oducto

bro del comité A el lunes y el martes con un miembro del comité B. Mediante el siguiente
principio, vemos que puede elegir a esos dos empleados de S x 7 • 35 formas.

Esta regla se conoce tambi~n como el principio th elecci6n.

El club de teatro de la Universidad Ceotral realiza ensayos para una obra que se montará
en primavera. Si seis hombres y ocho mujeres ensayan para los papeles principales (mas-
culino y femenino). por la regla del producto, el director puede elegir a la pareja principal
de 6 x 8 • 48 formas.

Ahora ejemplificaremos varias extensiones de la regla, examinando la fabricación de pla-


cas de automóvil que constan de dos letras seguidas por cuatro dígitos.
a) Si ninguna letra o dlgito se puede repetir, habrá 26 x 25 X 10 x 9 x 8 x 7 =
3.276.000 placas posibles difen:ntes.
b) Sisepermiterepetirlasletrasylosdlgitos,seráposibletener26 x 26 x 10 x 10 x
10 x 10 = 6,760,000 placas diferentes.
e) Si se permiten las repeticiones. como en la parte (b), ¿cuántas placas tendrán sola-
mente vocales (A, E, l. O, U) y dlgitos paros? (O es un entero par.)

En la memoria principal de un computador, la información se almacena en celdas de me--


moria. Para ideoti.f,c.ar las celdas de memoria principal del computador, cada celda tiene
asignado un nombre ónico cooocido como su dirección. En algunos computadores, una
dirección se representa mediante una lista ordenada de ocho símbolos. en la que cada
símbolo es uno de los birs (de binary digirs, dlgi1os binarios) O o 1. Esta lista de ocho birs
se denomina byre. Mediante la regla del producto, vemos que existe o 2 x 2 X 2 x 2 X 2
X 2 x 2 x 2 = 2' = 256 de esos bytes. Por lo tan10, tenemos 256 direcciones para las celdas
de memoria en las cuales es posible almacenar información.
Algunas tMquioas (incluyendo la familia de la POP 11 t utilizan direcciones de dos
bytes. Tal dirección se forma con dos bytes consecutivos. o 16 bits consecutivos, de modo
que es posible almacenar hasta 256 x 256 =21 X 21 = 216 = 65,536 piezas de infonnación.
Otros computadores (incluyendo el IBM PC/RT*) utilizan sistemas de direccionamiento
de cuatro bytes. En este caso, se dispone de hasta 2' x 2• x 2• x 2• = 2" =4,294,967,296
direcciones para almacenar información en las celdas de memoria de la máquina.

t La familia de c o m ~ PDP-11 es Wl producto de Digital E.quipment Corporation.


*EJ proocsador" IBM PC/RT ad fabricado por Inieraaiooal Bmioess Machi.aes.
6 úpitulo 1 Principios fundamentalM del conteo

A veces es necesario combinar varios tipos diferentes de principios de conteo en la solu-


ción de un problema. Aqui veremos que necesitamos ambas reglas, la de la suma y la del
producto, para obtener el resultado.
En algunas de las primeras versiones del lenguaje de programación BASIC, e l nombre
de una variable consta de una sola letra (A, B, C. ...) o una sola letra seguida de un solo
dígito. Como el computador no distingue entre las letras mayúsculas y minúsculas, a y A
se consideran como el mismo nombre de variable, así como también E7 y e 7. Por la regla
del producto, existen 26 x 10 • 260 nombres de variables que constan de una letra segui-
da por un dígito; y como hay 26 nombres de variables que constan de una sola letra. por la
regla de la suma existen 260 + 26 = 286 nombres de variables en este lenguaje de progra-
mación.

1.2
Permutaciones

Para seguir con el análisis de las aplicaciones de la regla del producto, contaremos ahora
disposiciones lineaJes de objc1os. tambi~n conocidas como¡Hrmutaciorus cuando los otr
jetos son distintos. Desarrollaremos algunos máodos sistemáticos para el estudio de las
disposiciones Uneales, partiendo de un ejemplo bastante común.

En un grupo de IO estudiantes, se escogerá a cinco y se les sentará en fila para una foto.
¿Cuántas disposiciones lineales son posibles?
La palabra clave aquí esdisposici6n. que indica la importancia del orthn. Si A, B, C, ...,
I, J denotan a los I Oestudiantes, entonces BCEFI. CEFIB y ABCFG son tres disposiciones
diferentes, aunque las dos primeras están formadas por los mismos cinco estudiantes.
Para responder la pregunta. analizamos tas posiciones y el número posible de esrudian-
tes que podemos elegir para ocupar cada posición. La ocupación de una posición es una
etapa de nuesuo procedimiento.

- -
!O X 9 X X 7 X 6

"""''"

Cualquiera de los 1Oestudiantes puede ocupar la primera posición de la fila. Puesto que
aquí no son posibles las repeticiones, sólo podemos elegir a uno de los demás estudiantes
para que ocupe la segunda posición. Continuando de esta manera, sólo tenemos seis estu-
diantes de donde elegir para que ocupen la quinta y última posición. Esto produce un total
de 30,240 disposiciones posibles de cinco estudiantes seleccionados del grupo de 10.
Se obtiene exactamCnte la misma respuesta si las posiciones se ocupan en el orden
inverso (6 x 7 x 8 x 9 x 10). Si el orden seguido es 3•, 1•, 4•, s• y 2• cesdecir. se ocupa
primero la tercera posición, luego la primera, luego la cuarta. luego la quinta y por último
la segunda), entonces la respuesta es 9 x 6 x 1O x 8 x 7.
1.2 Permutaciones

Como en el ejemplo 1.9. con frecuencia el producto de cienos enteros p0sitivos cense•
cutivos interviene en los problemas de enumeración. En consecuencia, la siguiente nota•
ción resultará útil al trabajar con dichos proble'mas, ya que a menudo nos permitirá expre•
sar las respue stas en forma más conveniente.

Definición 1.1 Para un entero n 2: O. n factorial (que se denota con n!) se define como

O! = 1,
n! = (n)(n - l)(n -2)· · ·(3)(2)(1), para n 2: l.

Así, 1! = 1, 2! • 2, 3! = 6, 4! : 24 y 5! = 120. Además, para cada n ~ O. ( n + 1}! =


(n + 1) (n!) .

Debemos tomar nota de la rapidez con que crecen los valores den!. Así que, antes de
proseguir, intentaremos tener una idea más clara de la velocidad con que crecen!. Pode•
mos calcular que 10! • 3.628,800, y éste es precisamente el número de segundos que hay
en seis semanas. En consecuencia, 11 ! es superior al número de segundos que tiene un
año, 12! supera el número que hay en 12 años, y 13! sobrepasa la cantidad de segundos
que tiene un siglo.

Y ahora, si utilizamos la notación factorial, veremos que la respuesta del eje mplo 1.9 se
puede expresar en forma más compacta:

10 X 9X 8 X 7 X6 = 10 X 9 X 8 X 7 X 6 X S X 4 X 3 X 2 X J = !O! .
5 X 4X3X2Xl 5!

Definición 1.2 Dada una colección den objetos distintos, cualquier disposición (lineal) de estos objetos
se denomina permutación de la co1ecci6n.

Si partimos de las letras a, b, e, veremos que hay seis formas de disponerlas, o pennu•
tartas: abe. acb, bac, bca. cab, cba. Si sólo estamos interesados en colocar dos letras a la
vez, habrá seis permutaciones de tamaño dos para la colección: ab, ba, ac, ca, be, cb.

En general, si existen n objetos distintos. que se denotan con ª" a1, ••• , a., y res un
entero, con I S r s n. entonces, por la regla del producto. el número de permutaciones
de tamaño r para los n objetos es

--- -n X (n-1) X (n - 2) X··· X (n -r+ !)=


primera Sll!gUnda tercera r--ésima

(11 - ,)(11 - r - 1) · · · (3)(2)(1) 11!


(11)(11 - l)(n - 2) · .. (n - r + 1) x (11 - 1'(11 - r - 1) · .. (3)(2)(1) - (11 - r)! .
capitulo 1 Principios fundamentales del conteo

Denotamos este número con P<.n, r). Para r= O, P<.n, O) = 1 = n!/(n - O)!, de modo que
P<,n, r} =n!l(n- r)!, donde Os rS n. Un casopanículardeesteresultado esel ejemplo 1.9. 1
d onden = 10,r= 5 y P(IO, 5) = 30,240. Cuando permutamos 1osn objetos de la colección,
=
tenemos r n y vemos que P(n. n} = n!/0! n !. =
Observe, por ejemplo, que sin ~ 2, entonces P(n, 2) = n!/(n - 2)! =n(n - 1). Cuando
= =
n > 3, se tiene que P(n, n - 3) = n!/ln - (n -3)]1 n!/3! (nXn - IXn - 2)·· ·(5)(4).
El ntímero de permutaciones de tamafto r, donde O s r s n, de una colección de n
objetos es P(n, r} = n!/(n - r}!. (Recuerde que P(n, r) cuenta disposiciones (lineales} en las
que los objetos no pueden repetirse.) Sin embargo, si se permiten las repeticiones, enton-
ces, por la regla del producto. existen n' disposiciones posibles, con r ~ O.

EJemp--,}~~O El número de permutaciones en la palabraCOMPUTER es 8!. Si sólo se utilizan cuatro de


las letras, el número de pennutaciones (de camaño cuatro) es P(8, 4) = 8!/(8 -4)! = 8!/4! =
1680. Si se permiten repeticiones de las letras. el número de secuencias posibles de 12
=
letras es 8" 6.872 X 10".t

Ejamplo 1_11 A diferencia del ejemplo 1.1O. el número de disposiciones Oinealcs} de las cuatro letras de
la palabra BALL es 12, no 4!, o 24. La razón es que no tenemos que ordenar cuatro letras
dis1intas. Para obtener las 12 disposiciones, podernos enumerarlas como se muestra en la
tabla l.l (a).
Si las dos letras L se distinguen como Li. Lz. entonces p<xlemos utilizar nuestras ideas
an1eriores relaüvas a las permutaciones de objetos distintos; con los cuatro símbolos dis-
tintos B, A, L 1, I,..z, tenemos 4!:: 24 permutaciones. Éstas se enumeran en la tabla 1.l {b).

tabla 1. 1

ABLL A B L, L, A B L, L,
ALBL A L, B L, A L, B L 1
ALLB A L, L, B A L, L, B
BALL B A L, L, B A L, L 1
BLAL B L, A L, B L, A L¡
BLLA B L, L, A B L, L, A
LABL L, A B L, L, A B L,
LALB L, A L, B L, A L, B
LBAL L1 B AL, L,BAL¡
LBLA L, B L, A L,BL,A
LLAB L,L,AB L,L,AB
LLBA L,L,BA L,L,BA
w (b)

t EJ $ímbolo .. ::: .. se Ice '"es aproximadamente i¡ual a".


1.2 Permutaciones

La tabla 1.1 revela que a cada disposición en ta que las letras L son indistinguibles le
corresponde una pareja de permutaciones con letras L distintas. En consecuencia.

2 X (Número de disposiciones de las letras B. A. L. L)


• (Número de permutaciones de los símbolos B. A. L 1• L:),

y la respuesta al problema original de encontrar todas las disposiciones de las cuatro letnis
que hay en BALL es 4!/2 ; 12.

Con la idea desarrollada en el ejemplo 1. 11, analizaremos ahora las d isposiciones de las
seis letras de PEPPER.
Existen 3! :: 6 disposiciones con las letras P distinguidas para cada disposición en las
que las letras P no se distinguen. Por ejemplo, P,EP2 P~R, P1EPJ>2 ER. P2EP1P~ER.
P: EP3 P1ER. P3EP1PiER y P3EP:P1ER corresponden a PEPPER cuando eliminamos los
subíndices de las letras P. Además, a la disposición P1EP?P~ le corresponde la pareja de
pennutaciones P 1E1 P:PsEiR, P 1 E:P2P~1 R. cuando se distinguen las letras l:.. En conse-
cuencia.

(2!X3')(Número de disposiciones de las letras de PEPPER)


:: (Número de pennutaciones de los símbolos P 1• E1• Pz. P!-> E2 • R).

de modo que el número de disposiciones de tas seis letras de PEPPER es 6 !/(2! 3!) = 60.

Antes de enunciar un principio general para disposiciones con símbolos repetidos. ob-
SCr\'e que en los dos ejemplos previos resolvimos un nue vo lipode problema relacionándo-
lo con los princ ipios de enumeración anteriores. Esta prktica es común en las matemát1•
cas e n general y aparece con frecuencia en la deducción de fórmulas discretas y
combinatorias.

En general, si existen n objetos con n 1 de un primer tipo. "2 de un segundo tipo. . . , y n,


n!
de un r-ésimo tipo. donde 111 + 112 + ··· + n,:: 11, entonces exislen - ,- ,-_.-.- 1 disposi•
cioocs (lineales) de losn objetos dados. (Los objetos del mismo tipol~.:iru.i;guibles.)

~ 1~1 . La MASSASAUGA es una serpiente venenosa marrón y blanca originaria de América del
None. Al ordenar todas las letraS de MASSASAUGA, vemos que existen
_ _I_O_!- =25 200
4! 3! 1! 1! 1! '
disposiciones posibles. Entre ellas. hay
7!
3!1!1!1!1!=840
10 Capitulo 1 Principios fundamentales del conteo

en las que están juntas las cuatro letras A. Para obtener este último resultado, considera-
mos todas las disposiciones de los siete símbolosAAAA (un símbolo), S, S, S, M , U. G.

Determine el número de trayectorias (escalonadas) del plano.tydc (2, 1) a (7, 4); cada tra-
yectoria está formada por escalones individuales que van una unidad hacia la derecha (R)
o una unidad hacia arriba (U). Las líneas azules de la figura l. 1 muestran dos de estas
trayectorias.
Debajo de cada trayectoria de la figura 1. 1 enumeramos cada escalón. Por ejemplo. en
la parte (a), la lista R, U, R, R, U, R, R. U indica que a partir del punto (2, 1), primero nos
movemos una unidad hacia la derecha [a (3, l)], luego una unidad hacia arriba [a (3, 2)),
luego dos unidades a la derecha (a (5, 2)], ctc .. hasta alcanzar el punto (7, 4). La trayectoria
consta de 5 letras R para los movimientos a la derecha y 3 letras U para los movimientos
hacia arriba.

234567 234567
(al R,U,R,R.U.R.R.U lbl U.R,R,R,U,U,R,R

Figura 1.1

La trayectoria de la parte (b) de la figura también está formada por 5 letras R y 3 letras
U. En general, el recorrido de (2. J) a (7, 4) necesita de 7 - 2 .:. 5 movimientos horizontales
a la derecha y 4 - l .:. 3 movimientos verticales hacia arriba. En consecuencia, cada trayec•
toria corresponde a una lista de 5 letras R y 3 leuas U, y la solución para el número de
trayectorias resulta ser el número de disposiciones de estas letras, que es 8!/(5! 3 !) = 56.

Haremos ahora aJgo más absu-acto y demosuaremos que sin y le son enteros positivos con
n:;; 2k, entonces n!/2' es un entero. Como nuestro argumento se basa en la enumeración, es
un ejemplo de demostración combirulloria.
Consideremos los n símbolos xi, xi, x 1, x 2 , ••• , x,, Xt, El número de formas en que pode•
mos ordenar estos n:::: 2k símbolos es un entero igual a
n! n!
2!2'- · -2, - 2'°·
~

kfactor~de2!
1.2 Permutadones 11

Por último. aplicaremos lo desarrollado hasta ahora a una situación en la que las dispo-
siciones no son line ales.

Si seis personas, designadas como A. B, ..., F, se sientan en torno de una mesa redonda.
¿cuántas disposiciones circulares diferentes son posibles. si las disposiciones se conside-
ran iguales cuando una puede obtenerse de otra mediante una rotación? (En la figura 1.12.
las disposiciones (a) y (b) se consideran idénticas, mientras que (b), (c) y (d) son tres
disposiciones distintas.)

'O·
C
A

F
E
·o, ·o,·o·
E
e

8
A E
A

F
C F
o

8
C

(a) (b) (e) (d)

Figur~ 1.2

Como en muchas circunstancias nuevas. hemos intentado relacionar este problema con
OlJ"OS anteriores con los que ya nos hemos topado. Paniendo de las figuras 1.2 (a) y (b).
desde la parte superior de la circunferencia y moviéndonos en el sentido de las manecillas
del reloj. enumeramos las disposiciones lineales diferentesABEFCD y CDABEF. que co-
rresponden a la misma disposición circular. Además de estas dos, ocras cuatro disposicio-
nes lineales (BEFCDA, DABEFC, EFCDAB y FCDABE) también corresponden a la mis•
ma disposición circular, como en (a) y (b). Así, puesto que cada disposición circular co-
rresponde a seis disposiciones lineales, tenemos 6 x (Número de disposic iones circulares
= =
de A, B • ...• F) (Número de disposiciones lineales de A, B• .... F) 6 !.
En consecuencia. existen 6!/6= S! = 120disposiciones de A, B, .... F en torno a la mesa
redonda.

~ 1.17 Supongamos ahora que las seis personas del ejemplo 1.16 son tres parejas casadas y que
A, By C son las mujeres. Deseamos colocar a las seis personas en tomo a la mesa redonda
de modo que los sexos se alternen. (De nuevo, las disposiciones se consideran idénticas si
una se puede obtener de la otra mediante una rotación.)
Antes de ocupamos de este problema, resolveremos el ejemplo 1.16 mediante un méto-
do alternativo. el cual nos ayudará en la solución de nueslrO problema actual. Si coloca-
mos a A en la mesa corno se muestra en la figura 1.3(a), faltan por llenar cinco lugares (en
el scntidp de las manecillas del reloj a partir de A). El hecho de ocupar estos lugares con B.
C...., F, esel problema de permutar B, c •..., F de manera lineal, y esto puede hacerse de 5!
= 120 formas.
12 capitulo 1 Principios fundamentales del conteo

50 1M30 M1
A A

4 2 F3 F2

M2

(a) (b)
Figura 1.3

Para resolver el nuevo problema de alternar los sexos, consideremos el método que se
muestra en la figura 1.3(b). A (una mujer) se coloca como antes. La siguiente posición, en
el sentido de las manecillas del reloj a partir de A, se marca como M I (hombre 1) y puede
ocuparse de tres formas. Si continuamos en la misma dirca:ión, a partir de A, la posición
F2 (mujer 2) puede ocupa= de dos formas. Siguiendo de esca forma y, por la regla del
producto, existen 3 X 2 x 2 x 1 x 1 .. 12 fonnas en las que estas seis personas pueden
ordenarse sin que dos hombres o dos mujeres se sienten juntos.

EJERCICIOS 1 .1 1. Durante una campafta local. ocho candidatos republicanos y cinco demóctaW se nominan
Y 1.2 para presidentes del consejo escolar.
a ) Si el presidente va a ser alguno de estos candidatos. ¿cuámas posibilidades hay para el
posible ganador?
b ) ¿Cu4ntas posibilidades hay para que una pareja de candidatos (uno de cada partido) s,e
opongan entre sf en la elc.cci6n final?
e) ¿Qu~ principio del conteo se usó en la parte (a)?, ¿en la parte (b)?
2. Responda la parte (e) del ejemplo 1.6.
3. U>$ automóviles Buic:t se fabriQn en 4 modelos. 12 colore$, 3 tamaftos de motor y 2 tipos de
transmisión.
a) ¿Cu4ntos Buict distintos se pueden fabricar?
b) Si uno de 10$ coloresdisponjbles es el azul. ¿cu!ntos Buict arulcs diferentes se pueden fabricar?
4 . a) El consejo directiOJO de u.na empresa íarmactutica ticoc 10 miembros. Se~ programado
una próxima reunión de accionistas para aprobar una nue,..a lista de ejecutivos (elegidos
entre los 10 miembros del consejo). ¿Cuántas listas diferentes, formadas por un presidente,
un vicepresidente. un secretario y un tcSOrcro, puede presentar el consejo a los accionistas
para su aprobación?
b) Tres miembros del consejo de directores (de la parte a) son m6dicos. ¿Cuántas listas de la
panc (a) tienen
i) un m6iico nominado para la presidencia?
ii) cx.actamcnte un m&lico en la lista?
iü) al menos un ~co en la lista?
5. Un sábado, cuando iban de compras, J uana y Teresa OJieron a dos hombres alejarse en automó-
vil de la fachada de una joyeria. justo antes de que sonara una alarma contra robos. Aunque
todo ocunió muy rfpido. cu.aodo fueron interrogadas las dos jóvenes, pudieron dar a la policía
la siguiente información acerca de la placa (que constaba de dos letras seguidas de cuatro
1.2 Permutaciones 13

dí¡itos) del automóvil que huyó. Teresa estaba segura de que la segunda letra de la placa era
una O o una Q. y que el último dígito era un 3 o un 8. Juana dijo que la primera letra de la placa
era una C o una G y que el primer dí¡ito era definitivamente un 7. ¿Cuántas placas diferentes
tendn que \'erificar la policía?
6. Con el fin de juntar fondos para una nueva alberca municipal, la c:imara de comercio de cietta
ciudad patroCina una carrera. Cada panicipante paga una cuota de inscripción de S5 y tiene la
probabilidad de ganar uno de los trofeos de distinto wna6o que se cntre¡arán a los primeros
ocho corredores que lleguen a la meta
a) Si 30 personas entran a ta carrera, ¿de cuá..ntas formas será posible entregar los tro♦
feos?
b) Si Robcrta y Clara $00 dos de los participantes en la carrera, ¿de cuántas formas se pueden
otorgar los trofeos de modo que ellas queden entre los tres primeros lugares?
7. Un anuncio de hamburguesas indica que un clicn1e puede ordenar su hamburguesa con alguno.
con ninguno de los siguientes ingredientes o con todos: catsup, mostaza. mayonesa. lechuga.
toma1e. ceboUa. pepinillos, queso o champinoncs. ¿Cuffltas órdenes diferentes de hamburgue•
sa se pueden servir?
8. Madas trabaja como operador de computador m una pcqucfla uni\'ersidad. Una tarde. él ,·e
que durante el día se han enviado 12 programas para su procesamiento por loccs. ¿,De cuánw
formas puede ordenar Matías el procesamiento de estos programas si: (a) no existen restriccio-
nes? (b) él considera que cuatro de los programas tienen Pf!Oridad sobre los ouo ocho y desea
procesarlos antes? (e) primero separa los programas en los cuatro de múi ma prioridad, cinco
de menor prioridad y tres de mínima prioridad, y desea procesar los 12 pro¡.ramasde modo que
los de mbima prioridad se: proc:cscn primero y los tres programas de mínima prioridad se
procesen al final?
9. La cafetería Paty tiene ocho tipos diferentes de pas1elcs y seis tipos diferentes de bollos.
Además de las piew de pastelería, es posible adquirir vasos pequelios, medianos o grandes
de las siguientes bebidas: café (negro. con crema. con azúcar, o con crema y azúcar). té
(solo. con crema, con núcar, con aema y azúcar. con limón, o con limón y az1kar). choco-
late caliente y jugo de naranja. Cuando Carolina va a la cafetería Paty, ¿de cuámas formas
puede ordenar
a) una pieza de pasi:clcña y una bebida mediana para ella?
b) una pieza de pastelería y un vaso de café para ella. y un bollo y un vaso de té para su jefe,
la señora Dueñas?
e) una pieza de pastelería y un ,·aso de té para ella, un bollo y un vaso de jugo de naranja para
la señora Dueñas y una pieza de pasteleóa y un vaso de café para cada uno de sus asistentes.
el señor Torres y la sei\ora Gm
1 O. Pamela tiene 15 libros distintos. ¿De cuántas formas puede colocar sus libros en dos repisas de
modo que haya al menos un libro en cada una? (Tenga en cuenta que los libros, en cualquier
disposición, están ordenados uno junto a otro, y el primer libro de cada repis.aqueda en el lado
izquierdo de la misma.)
11. Tres pueblos, designados como A, B y C. están intercomunicados por un sistema de carreteras
de doble sentido, como se muestra en la figura 1.4.
a) ¿De cuántas fonnas puede Elena ir del pueblo A al pueblo C1
b) ¿Cuántos trayectos puede hacer Elena del pueblo A al pueblo C yde regreso al pueblo A'!
e) ¿Cuántos de los 1rayectos completos de la pane (b) son tales que el viaje de regreso (del
pueblo Cal pueblo A) es diferente. al menos parcialmente. de la ruta que toma Elena del
pueblo A al pueblo C'! (Por ejemplo, si Elena viaja del pueblo A al pueblo C por las
carreteras R1 y R., para re¡rcsar podría tomar las carreteras R. y R,. o las carreteras R, y
R1, o la R, y la R:. o la R.. entre otras posibilidades. pero no viajando por las carreteras~
y R,.)
14 Capítulo 1 Prinppios fundamentales del conteo

Ra
e

Figura 1.4

12. Enumere todas las permutaciones de las letras a, c, L


13. a) ¿ Cuántas-permutaciones existen para las ocho letras a, c, f, g, i, t, w, x?
b) ¿ Cuántas de las permutaciones de la parte (a) comienzan con la letra t?
c) ¿Cllántas de las permutaciones de la parte (a) comienzan con la letra t y terminan con la
letra c?
14. Evalúe cada uno de los siguientes casos.
a) P(7, 2) b) P(8, 4) e) POO; 7) d) P (l2, 3)
15. ¿De cuántas formas es posible ordenar los símbolos a, b, c, d. e. e. e, e, e de modo que ninguna
e quede junto a otta?
16. Para la transmisión de mensajes en un sistema de comunicación se usa un alfabeto de 40
símbolos. ¿Cuántos mensajes.distintos (lista de símbolos) de 25 símbolos puede generar el
tranSmisor si los símbolos se pueden repetir en el mensaje? ¿Cuántos. si JO de los 40 símbolos
sólo pueden aparecer como el primero o el último símbolo del mensaje, o en ambas posiciones
a la vez, los restantes 30 símbolos pueden aparecer en cualquier parte, y las repeticiones de
iodos los símbolos están permitidas?
17. En una implementación del lenguaje de programación Pascal, un identificador consta de una
sola letra. o de una sola letra seguida de hasta siete símbolos, que pueden ser leiras o dígitos.
(Supongamos que el computador no distingue entre las letras mayúsculas y mim1sculas; hay
26 letras y 1Odígitos). Sin embargo. ciertas palabras clave están reservadas para los coman-
dos; en consecuencia. estas palabras clave no pueden usarse como identificadores. Si esta
implementación tiene 36 palabras reservadas. ¿cuántos iqentificadores diferentes son posibles
en esta versión de Pascal?
18. La producción de una pieza de una máquina consta de cuatro etapas. Hay seis líneas de ensam-
ble disponibles para la primera etapa, cuatro líneas para la segunda etapa, cinco para la tercera
y cinco para la última. Determine la cantidad de formas diferentes en que dicha pieza puede
quedar totalmente ensamblada en este proceso de producción.
19. Un profesor de ciencias de la computación tiene siete l.ibros de programación diferentes en una
es.t antería Tres de los libros son de FO.RTRAN; los otros cuatro de BASIC. ¿De cuántas for-
mas puede ordenar el profesor estos libros; (a) si no hay restricciones?, (b) si los lenguajes se
1.2 Ptmlutaciones 15

deben alternar?, (e) si todos los libros de FORTRAN deben estar juntos?, (d) si todos los libros
de FORTRAN deben estar juntos y los libros de BASIC tambitn?
20. ¿Qut nombre de eMado implica más disposiciones de las letras de w nombre; PENI"-SYLVANIA
o MASSACHUSETTS?
21. a) ¿De cuántas maneras se pueden colocar las letras de VISITJNG?
b) Para las disposiciones de la parte (a). ¿cutntas de ellas úenen las tres letras J juntas?
22. ¿Decuintas formas se pucdcD colocar las leuas de la paJabra POLYUNSATURATED de modo
que se manlCnga el ocden en que apareceo las vocales?
23. a ) ¿Owitas dis posiciones h.ay de todas las letras de la palabra SOCIOLOGlCAL?
b) ¿En cuántas de las disposiciones de la parte (a) están juntas la A y la G?
e) ¿En cuántas de las disposiciones de la parte (a) estin juntas todas las vocales?
2 4. ¿C\Lintos enteros positivosn se pueden formar con losdígjtos 3. 4, 4, S, S. 6. 1 si queremos que
n sea mayor que 5,000,000?
25. Doce platillos (con forma idintica) seor-
denan en cuatro columnas. verticales, como
se mucsm c:n la fi¡ura 1.5. Hay cuatro de
color rojo en la primera columna. ttcs de
color azul en la segunda columna, dos gri-
ses en la tercera columna y tres blancos
en la cuarta. Para entrar al equipo de tiro
de su universidad, Dora debe romper los
12 platillos (con su pistola y sólo 12 ba-
las) y, para esto, siempre debe romper el
platillo que queda en la parte inferior de
la columna. En estas coodictoncs, ¿de
cuintas formas puede disparar (y romper)
los 12 platillos? Figura 1.5
26. Muestre que, para cualquier pareja d e enteros n , r ~ O, sin+ 1 > r ,cnlonces

P(n + !,,) - - n -+ -I ) P(a,,~


( n +1- r

27. Detcnninecl valor (olos valort$)dencncadau.nodclos si¡uicnces casos: (a)P(n. 2) =90.(b)


Pr_n, 3) = 3/'f_n. 2). y (e) 2P(n. 2) + 50 = Pí_2n. 2).
28. ¿Cu4nw trayectorias distinw hay de (0, O) a (7, 7) en el plano xy si una trayectoria se constru•
ye paso a paso, yendo ya sea un espacio a la dere.cha (R) o un espacio h.acia arriba (U)? ¿Cuin-
w de estas trayectorias hay de (l.7) a (9. 14)? ¿Puede hacerse un enunciado general que incor-
pore estos dos resultados?
29. a ) ¿Cu.intas trayectorias diferentes hay de (- 1, 2, O) a ( 1, 3, 7) en el espacio eucHdeo
tridimensional si cada movimiento es de uno de los si¡uiemes tipos?

(H): (z,y, z)-(z + 1,y, z); (V): (z, y, z) - (z,y + ! , z);


(A): (z,y,z) - (.t,y,z + ! ).

b ) ¿Cu.mtas de aw trayectorias existen dc ( I. O, 5) a (8. l. 7)?


e) Ceneralice los resultados de las panes (a) y (b).
16 Capítulo 1 Principios fundamentales del conteo

'
30. a) Determine el valor de la variable entera counter después de la ejecución del siguiente seg-
mento de programa en Pascal. {Aquí, í,j y k sen variables enteras.)

counter := O;
For i := 1 to 12 do
counter : = counter + l;
For j : = 5 to 10 do
counter := counter + 2;
For k : = 15 downto 8 do
counter := counter + 3;

b) ¿ Qué principio de conteo está en juego en la pane (a)?


31. Considere el siguiente segmento de programa en Pascal, donde i,j y k son variables enteras.

For i : = 1 to 12 do
For j : = 5 to 10 do
For k := 15 downto 8 do
Writeln ((i - j)*k );

a) ¿Cuántas veces se ejecuta la proposición Wrire/n?


b) ¿Qué principio de conteo se usó en.la parte (a)?
32. Una serie de letras de la forma abcba, en la que la expresión no cambia al invenir su orden, es
un ejemplo de palínaromo (de cinco letras).
a) Si una letra puede aparecer más de una vez, ¿cuántos palíndromos de-cinco letras se pueden
formar? ¿de seis le1tas?
b) Repita la pane (a) con la condición de que ninguna letra aparezca más de dos veces.
33. a) Determine el número de enteros de seis dígitos (que no comiencen con cero) en los que (a)
ningún dígito se pueda repetir; (b) se pueden repetir los dígitos. Responda las partes (a) y
(b) con la condición adicional deque ei entero de seis dígitos sea (i) par; (ü) divisible emre
5; (üi) divisible emre 4.
34. a) Proporcione un argumento combinatorio para mostrar que sin y k son enteros positivos con
n·= 3k, entonces n!/(3!)' es ·u n entero.
b) Generalice el resultado de la parle (a).
35. a) ¿De cuánc:as formas puede un estudiante responder un examen de I O preguntas de verdade-
ro-falso?
b) ¿De cuántas fortnas puede el estudiante responder el examen de la pane (a) si es posible
dejar una pregunta sin respuesta para evitar que se penalice una respuesta equivocada?
36. ¿Cuántos enteros distintos de cuatro dígitos se pueden formar con los números 1, 3. 3, 7, 7 y
8?
37. a) ¿De cuántas formas se pueden semar siete personas en 1omo a una mesa circular?
b) Si dos de las personas insisten en sentarse j untas. ¿cuántas disposiciones son posibles?
38. a) ¿De cuántas formas se pueden sentar ocho personas, A, B, ... , H, alrededor de la mesa
cuadrada de la figura 1.6, donde las figuras I.6(a) y l .6(b) se consideran iguales pero
distintas de la figura 1.6(c)?
b) Si dos de las ocho personas. digamos, A y B, no se llevan bien. ¿cuántas disposiciones
diferentes en las que A y B no se sienlen j untos son posibles?
e) ¿Cuántas de las disposiciones de la parte (b) evi1an que A y B se sien ten uno frente al
otro?
1.2 Permutaciones 17

A 8 G H F G

·□' ·□· ·□·


G

F E
O E

o e
8 O

e 8
A

(a) (b) (e)

Figura 1.6

39. Una implementación particular del sistema e>perativo UNIX t proporciona la estrucrura de
arcluvos que se modela en la fi¡un I.7. En este caso. el' i•nodo del archivo contiene. entre
ouas informaciones. los permisos de acceso pan el archivo. Esto va seguido de datos que
contienen información acerca de la posición del archivo en el dispositivo de almacenamiento.
Las primeras 1O entradas son las direcciones de los bloques donde se almacenan los datos
reales del archivo. Si un bloque contiene S 12 bytes de infonnación. estos 10 bloques dirt!cto$
pueden almacenar hasta 512 x 10=5120 byteS de datos.
C\landoel tamaftodel archivo es mayor que los lObloques.cl dalo ni1mero 11 proporciona
el acceso, o apunta, a un bloqiu indiucro. Este bloque indirecto contiene las direcciones de
o<ros 128 bloques donde se guardan los datos. Si se usa este bloque indirecto. el tamano del
archivo puede ser de hasta 10 + 128 =138 bloques y contener hasta 51 2 x 138 =70,656 bytes
de información.
a ) Se necesita una entrada ndmcro 12 si hay que utilizar mis de 138 bloques para el atchh·o.
Esta entrada contiene la dirección de un bloque doblemente indirecto que, a su vez. contie-
ne las direcciones de 128 bloques indirectos. Como ya se scftaJó, cada uno de estos bloques
indirectos contiene las direcciones de 128 bloques donde se almacenan los datos. Si un
archivo usa 12 entradas en su i-ruxlo, ¿cu'1 es su wnafto máximo en tmninos de bloques y
bytes?
b) Si el archivo debe ser mayor que el espacio proporcionado por 12 entradas, se necesita una
entrada mlmero 13. Esta proporciona la dirección de un bloque triplemcntc indirecto que
apunta a 128 bloques dobl~tc indirtc:tos, como se muestra cola figura 1.7. Determine
cJ tamaiio mbimo de archivo, en ténninos de bloques y bytes, J>Ma la implantación de este
sistema operativo UNIX en estas condiciones.
40. Escriba un procrama (o desarrolle un a1goritmo) para calcular n! para cualquier entero n ~ O.
41. Escriba un pro¡rama (o desarrolle un algoritmo) para calcular Pf.n. r) para cualquier pareja de
enteros n. r e!: O.

42 . Escriba un programa (o desarrolle un algoritmo) para determinar si existe un entero de tres


dígitos abe ( =100a + IOb + e) donde abe= a ! + b! + e!

t UNIX~ una Jl\4fCa rt¡istrada de UNIX Software Laboratory.


18 capítulo 1 Principios funda~tales del conteo

Figura t.7
1!:i
1.3 C:ombinaciones: El teorema del binomio

1.3
1
Combinaciones: El teorema
del binomio
La baraja normal consta de 52 carras con cuatro palos: tréboles, diamantes, corazone,s y
espadas. Cada palo tiene 13 carras: as, 2, 3, . . . , 9, 10, sota, reina y rey.
Si nos piden sacar tres cartas de una baraja normal, una tras otra y sin sustituidas,
entonces, por la regla del producto, existen

52 X 51 X
,
-21.
50 =- =P(52 3)
49! '

posibilidades, una de las cuales es AC (as de corazones), 9T (nueve de tréboles), RD (rey


de diamantes). Si en vez de esto seleccionarnos u-es cartas a )a vez de modo que el orden de
selección de las cartas no sea importante, entonces las seis permutaciones AC-9T- RD,
AC-RD-9T, 9T- ...\C--RD, 9T-RD-AC, RD-AC-9T, RD-9T-AC corresponden sólo a una
forma (no ordenada) de selección. En consecuencia. cada selección o combinación de tres
cartas, sin tener en cuenia el orden, corresponde a 3 ! permutaciones de tres cartas. En
forma de ecuación, esto se traduce como

(3 !) x (Número de selecciones de tamaño tres de una baraja de 52)


= Número de permutaciones de ~I!l;año 3 para las 52 cartas
.,
-21.
= P (52 3) = - .
' 49!

Por lo tanto, de una baraja estándar se pueden sacar u-es cartas, sin reemplazo, de 52!/
(3!49!) = 22,100 formas.

En general. si partimos de n objetos distintos, cada sekcción, o combinación, de r de


estoS objetos. sin hacer referencia al ord,en. corresponde ar! permutaciones de tamaño
r de los n objetos'. Así, el número de combinaciones de tamaño r de una colección de
tamaño 11, que se denota C(n, 1C), donde O:S r:S n satisface (r!) x C(n, r) = P(n, r) y

C(n r) = P(~r) = n! Osr :s.n.


' r! r!(n - r)! '
Además de C(n, r), también se usa con frecuencia el símbolo(~)- Tanto C(n, r)
como (~) se leen como "cGlllbinaciones den en r''. Observe que C(n, O)= l .

¡Un buen consejo! Cuando se u-ata de un problema de conteo, debemos preguntarnos


acerca de la importancia del orden en el problema. Cuando el orden es necesario, pensa-
mos en términos de permutaciones y disposiciones y en la regla del producto. Cuando el
orden no sea necesario, las combinaciones podrían tener un papel importante en la solu-
ción del probléma.
20 Capítulo 1 Principio1 fundamentales del conteo

~ 1.18 Miriarn quiere dar una fiesta para algunos miembros de su comitt de caridad. Debido al
tamaño de su casa. sólo puede inviw a 11 de los 20 miembros de su comit~. El orden no es
importante, de modo que puede invitar a los "11 afortunados" deC(20, 11) = (l'/) =20!(1 I !9!)
= 167.960 formas. Sin embargo. una vez que lleguen los 11. la forma en que ella los siente
en torno de su mesa rectangular es un problema de disposiciones. Por desgracia, ninguna
parte de la teoría de combinaciones y permutaciones puede ayudar a la anfitriona con los
"'9 ofendidos"' que no fueron invitados.

E)lin'IPio 1.11 a) Un estudiante que rea1iza un examen de historia recibe la instrucción de responder
siete de 10 prcguntaS. Aquí no impona el orden. por lo que el estudiante puede
responder el examen de

(10)
7
= ~ - 10 x 9x 8 _ 120 formas.
7!3! 3 X2 X 1
b) Si el estudiante debe responder tres preguntas de las primeras cinco y cuatro de las
dltimas cinco, puede elegir tres preguotas de las primeras cinco de <l) = 10 formas.
y elegir las otras cuatro preguntas de (l) = 5 formas. De modo que, por la regla del
producto, el estudiante puede realizar el examen 0)(1) = 10 x 5 = 50 fonnas.
e) Por último, si las indicaciones del examen dicen que debe responder siete de las 10
preguntas. de las cuales al menos tres deberán ser de las primeras cinco. hay tres
casos por considerar.
i) El estudiante responde tres de las primeras cinco preguntas y cuatro de las
Qltimas cinco: por la regla del producto. esto puede ocurrir de (l)(l) = 1O x 5 =
50 formas, como en la parte (b).
il) El estudiante elige cuatro de las primeras cinco preguntas y tres de las últimas
cinco: esto puede hacerse de {l)G) = 5 x 10 = 50 formas, de nuevo, usando la
regla del producto.
iil1 El estudiante decide responder las cinco primeras preguntas y dos de las últi-
mas cinco: la regla del producto indica que este último caso puede ocurrir (~)(f)
= =
1 x 10 10 formas.

Si combinamos los l"C$ultados de los casos (i). (ii) y (iii). por la regla de la suma tene-
mos que el estudiante puede hacer (l)(l) + (lXll + (l}(l) = 50 + 50 + 10 = 110 selecciones
de siete (de 10) preguntas. cada una de las cua1cs incluye al menos tres de las primeras
cinco preguntas.

E)lin'IPlo 1.20 a ) En una escuela secundaria. la macsua de gimnasia debe elegir a nueve estudiantes
de segundo y tercer afta para el equipo de voleibol femenino. Si hay 28 jóvenes en
segundo y 25 eo ten:ero, ella puede hacer la elección de(~) = 4.431 ,613,550 formas.
b) Si dos estudiantes de segundo y una de tercero son las mejores rcmatadoras y deben
estar en el equipo, entonces el resto del equipo podrá elegir,< de(':) = 15,890.700
fonnas.

1.3 Combinaciones: El teorema del binomio 21

e) Para cieno torneo, el equipo debe tener cuatro estudiantes de segundo y c1nco de
tercero. La maestra pued_e elegir a las cuatro estudiantes de segundo de (2}) formas.
Para cada una de estas selecciones, ella tiene (2t) formas de elegir a las cinco estu-
diantes de tercero. Por lo tanto, por la regla del producto, puede elegir su equipo de
(~X2/) "' 1,087,836,750 formas para ese torneo particular.

Algunos problemas se pueden analizar desde el punto de vista de las disposiciones o de


las combinaciones, según la forma en que se examine la situación. El siguiente ejemplo
demuestra esto.

La maestra del ejemplo 1.20 debe formar cuatro equipos de voleibol, de nueve mujeres
cada uno, con las 36 estudiant~s de primer año de su curso de educación física. ¿De cuán-
tas formas puede elegir esos cuatro equipos? Los equipos se llaman A. B, C y D.
a) Para formar el equipo A, puede elegir a nueve mujeres de las 36, de (3;) formas.
Para el equipo B, el proceso de selección proporciona (1¡) posibilidades. Esto deja
(1) y (~ formas posibles de seleccionar los equipos C y D, respectivamente. Así,
por la regla del producto, los cuatro equipos se pueden elegir de

(~)(~)(~)(!) =·(9~~,)~~~,)(9~~,)(9~~,)
36
= , , !! =2.l45 X 1019 formas.
9 9 9 91
b) .Para una solución alternativa, consideremos que las 36 estudiantes están alineadas
.
como sigue:

l" 2· 3• 35" 36ª


estudiante estudiante estudiante estudiante estudiante

Para seleccionar los cuatro equipos, debemos distribuir nueve letras A, nueve B, nueve C
y nueve D en los 36 espacios. El número de formas en que se puede hacer esto es el número ,
de disposicio!}es de 36 letras que comprende nueve de cada una de las letras A, B, C y D. Este
es ahora el conocido problema de disposiciones de objetos no distintos, y la respuesta es

36!
como en la parte (a).
9! 9! 9! 9! '

Nuestro siguiente ejemplo muestra que algunos problemas necesitan los conceptos de
disposiciones y combinaciones para obtener las soluciones.

El número de disposiciones de las letras de TALLAHASSEE es


11!
3!2!2!2! 1! 1! = S3l,600.
¿Cuántas de estas disposiciones no tienen letras A adyacentes?
22 Capítulo 1 Principios fundamentales del conteo

Cuando no tomamos en cuenta las letras A, existen

8! = 5040
2!2!2!1!1!
formas de arreglar las letras restantes. Una de estas 5040 formas se muestra en la siguiente
figura, donde las flechas que van hacia arriba indican nueve posiciones posibles para las
tres letras A.

Tres de estas posiciones se pueden elegir de (t) =84 formas; y debido a que esto también
es posible para las restantes 5039 disposiciones de E, E, S, T, L, L, S, H, por la regla del
producto existen 5040 x 84 = 423,360 disposiciones de las letras de TALLAHASSEE sin
letras A consecutivas.

Antes de continuar necesitamos introducir una fo.nna concisa para escribir la suma de
una lista den + 1 términos como a.,, a.,+., a,,.. 2, ••• , a., ••, donde m y n son enteros y 11 > O.
Esta notación se conoce como notación sigma debido a que utiliza la letra mayúscula
griega l:; usamos dicha notación para representar una suma de la siguiente manera:

a,,, + ª m+1+ a.,+z + · • • + a,.•• =


...._¿ a,.
i =-m

Aquí, la letra i se denomina índice de la suma y representa todos los enteros que comien-
.zan con el límite inferior m y continúan hasta el líniite superior m + n (inclusive).
Podemos usar esta notación como sig.ue.

7 '
1) ¿ a, =a3 + ª•+as+(¼ + a1 =¿
i•3 j•3
a1 , ya que la letra i no tiene nada de especial.
4 •
2 2
2) ¿i = 1 +22 +32 +42 =30=¿k2, ya que 02 = 0.
i•l
100 101 ~
3 3
3) ¿ i3= 11 + 12 + 133 + · · · + 1003 = ¿ (j - 1)3 = ¿ (k + 1)3 •
i=-11 j=l.2 .t-10
JO 10
4) ¿ 2i = 2(7) +2(8) + 2(9) + 2(10) = 68 =2(34) = 2(7 + 8 + 9 + 10) = 2¿ i.
i-=7 1-1
3 4 2
S) La1= a;=La,- 1= L a;+1•
i =3 i-=4 l=2.
10 3 • 3
6) ¿ a¡ = a,+ a8 + a9 + a10 =¿ a¡+ 1 == ¿ a6+¡ = ¿ 010-;•
i=1 ¡~o j= l jaO
5 4 7
7) ¿bu = b2.3 + hz.• + hz.s = b6 + bs + bio =
i-=3
¿ b2;+2 == ¿
j= 2 k =5
b21c-,.
1.3 Combinaciones: El teorema del binomio 23

5
8) ¿a=a + a+a + a+a = 5a.
i =-1

Además, usando la notación de suma vemos que podemos expresar la respuesta de la


parte (c) del ejemplo l.19 como

Veremos el uso de esta nueva notación en el siguiente ejemplo, así como en muchas
otras partes del resto del libro.

En los estudios de la teoría algebraica de códigos y la teoría de lenguajes de computación.


consideramos cienas disposiciones, llamadas cadenas,• fonnadas a partir de un alfabeto
prescrito de símbolos. Si el alfabeto ya detenninado consta de los símbolos O, l y 2, por
ejemplo, entonces O1, 11, 2 I, I 2 y 20 son cinco de las nueve cadenas de longitud dos.
Entre las 27 cadenas de longitud tres están 000, 012, 202 y 110.
En general, si n es un entero positivo, por la regla del producto, existen 3• cadenas de
longitud n para el alfabeto O, l y 2. Si x = x 1 x2 x3 • • • x. es una de esas cadenas, definimos
el peso dex, que se denota wt(x), como wt(x) =x, +x2 +x3 + · · · +x•. Por ejemplo, wt( l2)
= 3 y wt(22) = 4, sin = 2; wt( lOl) = 2, wt(210) = 3 y wt(222) = 6 paran= 3.

Entre las 31º cadenas de longitud l O, queremos determinar el número de ellas que tengan
peso par. Una cadena tendrá peso par exactamente cuando el número de unos que tenga sea par.
Tenemos que considerar seis casos diferentes. Si la cadenax no contiene unos, entonces
cada una de las l O posiciones dex puede ser ocupada con O o 2 y, por la regla del producto,
hay 2 10 de esas cadenas. Cuando la cadena contiene dos números 1. sus posiciones pueden
elegirse de (~) fonnas. Una vez elegidas las dos posiciones, hay 2 8 formas de ·c olocar el O
o el 2 en las ocho posiciones restantes. Por lo tanto, (~)28 cadenas de peso par que contie-
nen dos números 1. El número de cadenas de los otros cuatro casos aparece en la tabla 1.2.

Tabla 1.2
· Número de unos Número de cadenas Número de unos Número de cadenas

4 (lf)l6 8 (f)22
6 (12)'.24 10 (l8)

En consecuencia, por la regla de la. suma, el número de ca1enas de longitud I O que


tienen peso par es 2'º + (~)28 + (12)26 + <12)2• + (t )2 2 + Cg) = L.. o
Ü~)2 10 - 2-.

Con frecuencia, debemos tener cuidado con el recuento excesivo, situación que parece
surgir en problemas de conreo aparentemente sencillos. El siguiente ejemplo demuestra la
forma en que puede surgir el recuento excesivo.

• El término string también se ttaduoe como "cuerda". (N. del E.)


24 Capitulo 1 E'rincipiós fundamentales del conteo

a) Supongamos que Elena saca cinco canas de una baraja estándar de 52 canas. ¿De
cuántas formas puede .resultar que su selección no tenga tréboles? Aquí estamos
interesados en contar todas las selecciones de cinco cartas como
i) as de corazones, tres de espadas, cuatro de espadas, seis de diamantes y la sota
de diamantes.
ü) cinco de espadas, siete de espadas, diez de espadas, siete de diamantes y el rey
de diamantes.
ili) dos de diamantes, tres de diamantes, seis de diamantes, d iez de diamantes y la
sota de diamantes.
Si analizamos esto más de cerca, veremos que Elena debe elegir sus ci neo cartas de
las 39 cartas de la baraja que no son tréboles. En consecuencia, puede hacer su
selección de (3t) formas.
b) Supongamos ahora que queremos contar el número de selecciones qué hace Elena
de cinco cartas que contienen al menos un trébol. Éstas son precisamente las selec-
ciones que 110 se contaron en la parte (a). Y como existen ( 5; ) manos posibles de
cinco cartas en total, tenem.o s que

( 52) (39) _
5 - 5 = 2,598,960 - 575,7.:,7 = 2,023,203
de todas las manos de cinco canas que contienen al menos un trébol.
e) ¿Podemos obtener el resultado de la parte (b) de otra manera? Por ejemplo, como
Elena desea tener al menos un trébol en la mano de cinco cartas, hagamos que ella
seleccione primero un trébol. Esto puede hacerse de('?) formas. Y ahora ella no tiene
que preocuparse por lo que tenga en las otras cuatro cartas. Así, después de eliminar
el trébol elegido de su baraja normal, puede seleccionar las otras cuatro canas de (~1)
formas, Por lo tanto, por la regla del producto, el número de selecciones es

¡Algo aquí está definitivamente mal! Esta cantidad es mayor que la de la pane (b),
por más de un millón de manos. ¿Cometimos un error en la pane (b)? ¿ O hay algo
incorrecto en nuestro razonamiento actual?
Por ejemplo, supongamos que Elena selecciona primero

el tres de tréboles

y después elige

el cinco de tréboles,
el rey de tréboles,
el siete de corazones
y la sota de espadas.

Si, por el contrario, elige primero


1.3 Combinaciones: El teorema del binomio 25

y después selecciona

el tres de tréboles,
rey de tréboles,
el siete de corazones
y la sota de espadas,

¿es esta selección realmente diferente de la anterior? Por desgracia, ¡no! Y el caso
en que eUa elige primero

el rey de tréboles

y después selecciona

el tres de tréboles,
el cinco de tréboles,.
el siete de corazones
y la sota de espadas

no es distinto de las otras dos selecciones mencionadas antes.


En consecuencia, este punto de vista es incorrecto ya que estamos contando en
exceso, es decir, considerando selecciones semejantes como si fueran disúntas.

d) ¿Pero existe otra forma de llegar a la respuesta de la parte (b)? ¡Sí! Como las manos
de . deben contener al menos un trébol, debemos considerar cinco casos.
cinco cartas
,
Estos aparecen en la tabla 1.3.
A partir de los resultados de la tabla vemos, por
ejemplo, que hay ('.,3)(3:) manos de cinco cartas que contienen exactamente dos tré-
boles. Si estamos interesados en tener exactamente tres tréboles en la mano, enton-
ces los resultados de la tabla indican que hay ('J)(3f) de tales manos.

Tabla 1.3
Número Número de formas Número de Número de formas
de de Séléccio!lar este cartas que d.e seleccionar este
tréboles número de tréboles no soo tréboles número de n~tréboles

1 (1:) 4 (3:)
2 (~) 3 (3:)
3 (1:) 2 (3;)
4 (~) 1 (3:)
5 (~) o (3:)
26 Capítulo 1 Principios fundamentales del conteo

Puesto que ninguna pareja de los casos de la tabla 1.3 tiene una mano de c inco
cartas en común, el mlmero de manos que Elena puede seleccionar con a1 menos un
trébol es

(1?)(3:) + (1¡)(3;) + (~)(3;) + (~)(3;) + (1;)(3:) =!(1:)(/~;)


= (13)(82,251) + (78)(9139) + (286)(741) + (715)(39) + (1287)(1)
= 2,023,203.

Cerraremos esta sección con tres resultados relativos al concepto de combinaciones.


Primero observemos que para n, r enteros con n ~ r ~ O. (:) = (. ~,). Esto puede
establecerse algebraicamcnle a partir de la fónnula para (!), pero preferimos hacer notar
que, al analizar una selección de tamaño r de una colección de n objetos distintos, el
proceso de selección deja atrás n - r objetos. En consecuencia. (!) = (.. ~,) afirma la exis-
tencia de una correspondencia entre las selecciones de tamaño r Oos objetos elegidos) y
las selecciones de tamaño n - r (los objetos dejados de lado). Un ejemplo de esta corres-
pondencia aparece en la tabla 1.4, donde n = 5, r = 2 y los objetos distintos son 1. 2, 3, 4 y
5. Este tipo de correspondencia se definirá más form.almenle en el capíluJo 5 y se usará en
ouos casos de conteo.
Tabla 1.4
SdecdoDa de tamaño r • 2 Selttdona d~ tamaño 11 - r = J
(ob.l<... ........) (objttOS dejadof de lado)

l. 1, 2 6. 2,4 l. 3,4, 5 6. 1, 3, 5
2. 1,3 7. 2,5 2. 2,4,5 7. 1,3,4
3. 1,4 8. 3,4 3. 2,3,5 8. 1,2,5
4. 1,5 9. 3,5 4. 2,3, 4 9. 1, 2, 4
5. 2,3 10. 4,5 5. 1,4,5 10. 1,2,3

Nuestro segundo resultado es un teorema relativo a nuestra experiencia anterior en álgebra.

TEOREMA 1.1 (El teonma dll binomio) Si x y y son variables y n es un entero positivo, enlences

(x +y)•• (~)x°y" + (~)x',--, +(;y,--•+·••


+ (n: J•-•y• + (:►•y• • i(:}r,--•.
Antes de revisar la demostración general, analizaremos un caso particular. Si n = 4, el
coeficiente de x'y en el desarrollo del producto

(x + y)(x + y)(x + y)(x + y)


l)(im« ~ ttfcer CUMto
f.Kto, facto, fKtot factor

es el número de formas en que podemos seleccionar dos de las cuatro x, una de las cuales
está disponible en cada factor. (Aunque las x son iguales en apariencia. las distinguimos
1.3 Combinaciones: El teorema del binomio 27

como la x del primer factor, la x del segundo factor, ..., y la x del cuarto factor. También
observarnos que, cuando seleccionamos dos x, usamos dos factores, lo que nos deja otros
dos factores de los que podemos seleccionar las dos y necesarias.) Por ejemplo, entre las
posibilidades, podemos seleccionar (1) x de los dos primeros factores y y de los dos últi-
mos, o (2) x de los factores primer.o y tercero y y del segundo y cuarto. La tabla 1.5 resume
las seis opciones posibles.

Tabla 1.5
Faetol'éS sel'eccionados Factores seleccionados
para x pan y

(1) 1,2 (1) 3,4


(2) 1,3 (2) 2,4
(3) 1, 4 (3) 2, 3 •
(4) 2,3 (4) 1, 4
(5) 2,4 (5) 1, 3
(6) 3,4 (6) 1,2

En con.s ecuencia, el coeficiente dex'y en el desarrollo de (x + y)4 es (~) = 6, el número


de formas de elegir dos objetos distintos de una colección de cuatro objetos distintos.
Ahora pasaremos a la demostración del caso general.
Demostración: En el desarrollo del producto

(x + y)(x + y)(x +y)•• • (x + y)


primer segundo tercer n-ésimo
factor factor factor factor

el coeficiente der)'"'-•, donde O !é k < n, es el número de formas distintas en que podemos


elegir k letras x (y en consecuencia (n - k) letras y) de los n factores disponibles. (Por
ejemplo, una forma es elegir x de los primeros k factores y y de los últimos n - k factores.)
La cantidad total de selecciones de tamaño k de una colección de tamañon es C(n. k) = (í:),
de lo que se sigue el teorema del binomio.

En vista de este teorema, G) se conoce con frecuencia como coeficiente binomial. Observe
que también es posible expresar el resultado del teorema l . l como

(x +y)" = ±( n.
k=O n- k
)x"y•-*.

a) Del teorema del binomio se sigue que el coeficiente dery2 en el desarrollo de (x + y)7
es (~) = (i) = 2 l.
b) El coeficiente dea5b2 ene! desarrollo (2a -3b)' es <n(2)5(- 3)2. Obtenemos esto del
teorema del binomio, con x = 2a y y = -3b.
28 capítulo.1 Principios funda mentales del conteo

COROLARIO 1 .1 Para cualquier entero n > O,

a) (o) + (1) + G) + · · · + ~) =2", Y


b) (~) - (1) + (;) - · · · + (- 1)" (~) = O.
Demostración: La parte (a) se sigue del teorema del binomio cuando x =y= 1. Cuando x = - 1
y y = 1, se obtiene la parte (b).

Nuestro tercero y último resultado generaliza el teorema del binomio y se conoce como
teorema multinomial.

TEOREMA 1.2 Para enteros positivos n, t, el coeficiente de x~ x;' x;' · · · x;1 en el desarrollo de (xi+ x 1 +
X3 + ··· + x,)" es

n!

donde cada n, es un entero con O s; n; s; n, para todo l s; i < t y n, + n 2 + n3 + ·· · + n, = n.


Demostración: Como en la demostración del teorema del binomio, el coeficiente de
x~' x;' x;• · · · x;• es el número de formas en que podemos elegir x i den, de los n factores,
-½ de 11,z de los n-ni factores restantes, x3 de n3 de los n - n, -11,z factores restantes ahora, . .
. , y x, den, de los últimos n - ni - n2 - n 3 - • • • - n,.., = n, factores restantes. Esto se puede·
realizar, como en la parte (a) del ejemplo 1.21, de

(:J(n :2n,)(n -::-n2) ·..(n -ni-nn, ···-n,_1) 2

formas. Dejamos al lector los detalles de la demostración de que esto es igual a


n!

que también se escribe como

(n1 ,n2,n:, .. .,n)


y se denomina coeficiente multinomial.

a) En el desarrollo de (x +y+ z)', se sigue, del teorema multinomial, que el coeficiente


3
J;
de x2 y2 z es (2,;_3) = 2 3! = 210, mientras que el coeficiente de xyt es ( 1} 5 ) = 42 y
el de x2 y4 es (3.J.4 ) = 3!¿;,, = 35.
b) Supongamos que tiene la necesidad de conocer el coeficiente de a2 b3 c2 d 5 en el
desarrollo de (a+ 2b - 3c + 2d + 5)16• Si reemplazamos a con v, 2b con w, - 3c con
x, 2d con y y 5 con z, entonces podemos aplicar el teorema multinomial a (v + w +
x + y + z) 16 y determi_n ar el coeficiente de v 2 w3 x2 f z4 como ( 2./f.5 •4 ) = 302,702,400.
2
Pero C2.3 .~ 5.,)(a) (2h)3(- 3c)2(2d)5 (5)' = (2s.~~ 5.,)(1)2(2) 3(-3)2(2)5(5)'(a2 b3 c2 d 5) =
435,891 ,456,000 a2 b 3 c2 d 5 •
1.3 Combinaciones:: El teorema del binomio 29

EJERCICIOS 1.3 1. Calcule(~) y verifique su respuesta enumerando todas las selecciones de tamaño dos que se
pueden hacer con las letras a, b. c, d, e y f.
2 . Como Diana debe hacer un viaje de cuatro horas en autobús de regreso a su escuela, decide
llevar consigo cinco revistas de las 12 que su hermana Ana María acaba de adquirir. ¿De
cuántas formas puede Diana hacer su selección?
3. Evalúe cada uno de los siguientes casos.
a) C(lO, 4) b)- ('l) e:) C(14, 12) d) 00
4. En el sistema Braille, un símbolo. como una letra minúscula, un signo de puntuación, un
sufijo, etc., se escñbe resaltando al menos uno de los puntos de la disposición de seis puntos
que aparece en la parte (a) de la figura 1.8. (Las seis posiciones Braille se enumeran en esta
parte de la figura.) Por ejemplo, en la parte (b) los puntos de las posiciones I y 4 están resalta-
dos y esta disposición de seis puntos representa la letra c. En las partes (c) y (d) de la figura
tenemos las representaciones de las letraS m y t, respectivamente. El artículo definido "the" se
muestra en la parte (e), mientras que 1a parte (f) contiene la forma para el sufijo ..ow'º. Por
último, el punto y coma (;) aparece en la disposición de seis puntos de la parte (g), donde los
puntos de las posiciones 2 y 3 aparecen en relieve.

1• •4
•• •• •
• •
• • • • •

2• .s • • • •
• • • • • • • •

3• •6 • •
• • • • •• • • • •

(a}
.
(b) · e· (e) · m· (d) ·t· (e) "the" (f} "CJN" (g)
...•
Figura 1.8

a) ¿Cuántos símbolos diferentes podemos represen.t aren el sistema Braille?


b) ¿Cuántos símbolos tienen exactamente tres puntos en relieve?
c) ¿Cuántos símbolos tienen un número par de puntos en relieve?
d) ¿Cuántos símbolos tienen al menos cuatro puntos en relieve?
S. a) ¿Cuántas permutaciones de tamaño 3 pueden producirse con las letraS m. r, a, f y t?
b) Enumere todas las combinaciones de tamaño 3 que resultan de las letras m, r, a, f y t.
1
6. Sin es un entero positivo y n > I, demuestre que G) +("; ) es un cuadrado perfecto.
7. Un comité de 12 personas será elegido entre 10 hombres y 10 mujeres. ¿De cuántas forrnas se
puede hacer la selección si (a) no hay restricciones? (b) debe haber seis hombres y seis muje-
res? (c) debe haber un número par de mujeres? (d) debe haber más mujeres que hombres? (e}
debe haber al menos ocho hombres?
8 . ¿De cuántas formas puede un jugador extraer cin.co cartas de una baraja común y obtener (a}
una corrida (cinco cartas del mismo palo)? (b) cuatro ases? (c) cuatro canas del mismo tipo?
(d) tres ases y dos sotas? (e) tres ases y un par? (f) un full (una tema y un par)? (g) una terna?
(b) dos pares?
9. ¿Cuántos bytes contienen (a) exactamente dos unos? (b) exactamente cuatro unos? (c) exacta-
mente seis unos? (d) al menos seis unos?
1 O. ¿De cuántas formas se puede formar un equipo de baloncesto de cinco personas con 12 posi-
bles jugadores? ¿Cuántas opciones incluyen al jugador más débil y al más fuene?
30 Capítulo 1 Principios fundamentales del conteo

11 . Un estudiante debe responder siete de las l O preguntas de un examen. ¿De cuán1as formas
puede hacer su selección si (a) no hay restricciones'? (b) debe contestar las dos primeras pre-
guntas? (e) debe responder al menos cuatro de las primeras seis preguntas?
12. Al ordenar la comida del día, un cliente puede elegir entre tres entradas y dos de seis verduras
disponibles.
a ) ¿Cuántas comidas diferentes puede elegir si {i) debe seleccionar dos verduras diferentes?
(ii) se le permite tener dos porciones de la misma verdura?
b) Responda las partes (i) y (ii) de la parte (a) si también puede elegir en1re jugo de toma1e,
jugo de naranja o sopa de lentejas como aperilivo.
13. ¿De cuántas fonnas es posible distribuir 12 libros diferentes entre cuatro niftos de modo que
(a) cada nilío reciba tres libros? (b) los dos niños mayores reciban cuatro libros cada uno y los
dos menores reciban dos libros cada uno?
14. En la pizze ría de Pedro, las pi:u:as se sirven en cua1ro 1amaños: pequeña. mediana. grande y
colosal. Un cliente puede ordenar una pizza sencilla de queso o pedir cualquier combinación
de los siguientes siete ingredientes adicionales; anchoas, pepinillos, champii."iones, aceitunas.
cebolla. pepperoni y salchicha. Determine la cantidad de pizzas diferentes (a) de tamaño me-
diano y que tengan exactamente dos ingredientes adiciona1es; (b) exactamen1e con dos ingre-
dientes adicionaJes: (e) grandes o colosales y exactamente con eres ingrediemes adicionales.
15. ¿Cuántas disposiciones de las letras de MJSSlSSIPPI no tie~en letras S consecutivas?
16. Un entrenador debe elegir 11 estudiantes de tercer año para jugar en un equipo de futbol. Si
puede elegir entre 12.376
formas, ¿cuántos estudiantes de tercero son elegibles?
17. a) En un plano se tienen quince puntos. de los cuales no hay ues que estén alineados. ¿Cuán-
tas rectas determinan?
b) Se tienen veintincínco puntos en el espacio. de forma que cuatro de ellos no son coplanares
¿Cuántos triángulos determinan? ¿Cuántos planos? ¿Cuántos tetraedros (sólidos piramidales
con cuatro caras triangulares)?
18. Determine el valor de cada una de las sumas siguientes.

a) I(i'+l)
,_, b) L'
¡- - l
(i' - 1) e) fr1+(-1J'J

,.
d) L(Y- 2')
,., ..
•l I(-1J'
,
f ) .,~.. ( - 1)*, donde n es un e nlero posilivo impar.

19. Exprese lo siguiente mediante la notación de suma (o sigma). En las pan.es (b). (e). (f) y (g). n
denota u n entero positivo.

•> 1+!+!+!+!+!+···+2..
2 3 4 5 6 17
b) .!.+..!..+..!..+···+_!_
2! 3! 4! n! '
n 2:2
e) 1+4 + 9 +16+25 + 36+49
d) 13 - 23 +33 -41 +5'-63 +73
e) !+_2_+_3_+···+~
n n +l n+2 2n
n+l n+2 n+3 2n
r) ,:r+2!+4!+&+···+ (2n)!
g) n -(!!..:!:1)+("
2!
+2) - (• 6'+3)+ ··. +(-1)"(~)
4! (2n)!
13 Combinaciones: El teorema del binomio 31

20. Para las cadenas de longitud lOdel ejemplo 1.23. ¿cuántas tienen (a) cuatro ceros, tres unos y
tres doses?; (b) al menos ocho unos?; (e) peso 4?
21. Considere la colección de todas las cadenas de longitud 10 que se forman con el alfabeto O, 1, 2
y 3. ¿Cuántas de estas cadenas tienen peso 3? ¿Cuántas tienen peso 4? ¿Cuántas tienen peso par?
22. En las cuatro partes de la figura 1.9, se han marcado ocho puntos equidistantes sobre la circun•
ferencia de un círculo dado.
a) Para las partes (a) y (b) de la figura 1.9, tenemos dos triángulos diferentes (aunque con-
gruentes). Estos dos triángulos (que se distinguen mediante sus vértices) surgen de dos
selecciones de tamaño tres de los vértices A, B, C, D, E, F, G, H. ¿Cuántos triángulos
diferentes (congruentes o no) podemos inscribir de esca forma en el círculo?
b) ¿Cuántos de los triángulos de la parte (a) son isósceles?
e) ¿Cuántos cuadriláteros diferentes podemos inscribir en el círculo usando los vértices mar-
cados? (Uno de tales cuadriláteros aparece en la parte (e) de la figura 1.9.)

A A A A
8 H 8 H 8

G e G e G e G e
F D F D F
E E E E

(a) (b) (e) (d)

Figura 1.9

d) ¿Cuántos de los cuadriláteros de la parte (e) son cuadrados? ¿Cuántos de ellos son rectán•
gulos (no cuadrados)?
e) En la parte (d) de la figura J.9 tenemos un pentágono inscrito en nuestro círculo. ¿Cuántos
pentágonos podemos inscribir en el círculo dado usando los vértices marcados?
f) ¿Cuántos polfgonos diferentes. de tres o más lados, podemos inscribir en el círculo dado
usando tres o m~ de los vértices marcados?
23. ¿Cuántos triángulos quedan determinados por los vértices de un polígono regular den lados?
¿Cuántos si ninguno de los lados del polígono debe ser un.lado de alguno de los triángulos?
24. a) En ~I desarrollo completo de (a + b + c + d)(e + f + g + hXu + v + w + x +y+ z), obtenemos
la suma de términos como agw, cfx y dgv. ¿Cuántos de esos términos aparecen en este
desarrollo compléto?
b) ¿Cuáles de los términos siguientes no aparecen en el desarrollo completo de la parte (a)?
i) afr, ii) bur, üi) chz; iv) cgw; v) egu; vi) dft.:
25. Determine el coeficientedex"y'en los desarroUos de (a)~+ y)", (b) (x + 2y)" y (c) (2x -3y) 12•
26. Complete los detalles de la demostración del teorema multinomial.
27. Determine el coeficiente de
a) xyi en (x + y + z)'
b) xyz;-en (w +x +y+ z)'
e) xyi en (2x - y - z)'
d) ;r;yt:2 en (x - 2y + 3z·')'
e) w'.ryr en (2w - x + 3y- 2z)8•
28. Determine el coeficiente de w'x'y'z' en el desarrollo de (a) (w +x +y+ z + l)'º, (b) (2w-x +
3y + z - 2) 12; y (c) (v + w- 2x + y+ Sz + 3)12•
32 Capítulo 1 Prin~ipios fundamentales del conteo

29. Determine la suma de todos los coeficientes de los desarrollos de


a) (x + y)3 b) (x + y)'º
e) (x +y+ z)'º d) (w + x +y+ z)'
e) (2s-3t+5u+6u-11w+3x+2y)'º·
• .1
30. Muestre que para todos los enteros n > 2, ( ; ) = (;) + n.
31. Muestre que si n es un entero positivo, entonces

32. Para cualquier n entero positivo, deteanine


• 1
.í: .,(.n-z.
a> r-oi ">' b) ¿• ~
(-1)'
~-
,-o i!(n - i)!
33. Muestre que para todos los enteros positivos n y m. n(",;•) = (m + IX'.:::::) .
34. Con n un entero positivo, evalúe la suma

(~) + 2(;) + z2(;) + -- - + 2•(;) + -- - + r(:)-


35. Para x un ntímero real y n un entero positivo, muestre que

a) 1= (1 + x)" -(~)x'(l + x)•-t + (;)x'(I + x)"- 2


- • - - + (-1)•(:)x".
b) 1 = (2 + x)" - (;)(x + 1)(2 + xy-• + (;)<x + 1) (2 + x)"-
2 2
- - •-

·+ (-1)"(:)<x + 1)".

e) 2" = (2 + x)"- (;)x'(2 + xy-• + (;)x2 (2 + x)•-•- · · · + (-t)•(:)x-.


36. Determine x si I :0 (f) 8; = x""· ;
37. a) Si ao, ª" a,, a, es una lista de cuatro números reales, ¿a qué es igual L (a; - a;_,)?
;=-1
b) Dada una lista ao, a ,. a, . ...• a. ele n + 1 números reales, donde n es un encero positivo.
determine¿~ (a, - a;_,).
r•l 100
e) Determine el valor de .L..r;-1
" (.,..L
,+2
- .,L)
t+1
.
7
38. El desarrollo "' - "'' ij es un ejemplo de una doble suma (o doble sumatoria). Aquí vemos
L,¡F3 L.,,=
que"' ~ ii="'' (~ ii)= " ~ (j +2j+ 3j +4j) = " '. IOj, despuésdedesarrollar
LJF3 4=t .L..,j=-3 4 •t LJ,•3 L-1=3
la suma interior en la variable i. Luego desarrollamos la suma exterior en la variable j y vemos
que"''
.L..r¡:3
IOj = 10 L 7
1=~
, j = 10{3 + 4 + 5 + 6 + 7) =250. Por lo tanto, ¿
7
;=3L-;.¡
"'" ij =250.·

• 7

a> .í: .í: ij b)


.
Determine el valor de cada una de las siguientes dobles sumas.
'
L L (i + j + I)
¡-o;-1
4 3

e> ;-.í:1;-.í:o ¡
39. Escriba un programa (o desarrolle un algoritmo) para calcular C(n, r) para cualesquiera enteros
n, r > O.
40. a) Escriba un programa (o desarrolle un algoritmo) que enumere todas las selecciones de
tamaño 2 de los objetos l. 2, 3. 4, 5, 6.
1.4 Combinaciones con repetición: Distribuciones 33

1.4
Combinaciones con repetición:
Dist ribuciones

Cuando se permiten las repeticiones, hemos visto que, paran objetos distintos, una dispo-
sición de tamaño r de estos objetos puede obtenerse de n' formas, para un entero r ~ O.
Ahora analizaremos el problema comparable para las combinaciones. y de nuevo obten-
dremos un problema relacionado con el anterior cuya soluc-ión se sigue de nuestros princi-
pios de conteo anteriores.

Al regresar a casa después de una práctica de-carrera en pista, siete estudiantes de bachille-
rato se detienen en un Testaurante de comida rápida, donde cada uno puede comer lo si-
guiente: una hamburguesa con queso, un hot dog, un taco o un sandwich de atún. ¿Cuántas
compras diferentes son posibles?
Sean q, h, t y p la hamburguesa con queso, el hot dog, el taco y el sandwich de atún,
respectivamente. Aquí nos interesa el número de artículos comprados y no el orden en que
se adquirieron, de modo que el problema es de selecciones o combinaciones con repetición.
En la tabla 1.6 enumeramos algunas compras posibles en la columna (a) y otro medio
de representar cada forma en la columna (b).

Tabla 1.6

l. q, q, .h. h, t, t, p l. xx lxx lxx lx


2. q, q,q,q, h, t, p 2. xxxx Ixixlx
3. q, q,q, q, q, q, p 3. xxxxxx lll x
4. h, t, t,P,P,P,P 4. lx lxx jxxxx
5. t, t, t, t, t,p, p 5. llxx xxxlxx
6. t, t, t, t , t, t , t 6. llxxxxxxx l
7. P,P,V,P,P,P,P 7. 11lxxxxxxx
(o} (b}

Para una compra de la columna (b) de la tabla 1.6, vemos que cada x a la izquierda de
la primera barra ( 1) representa unaq, cada x entre la primera y la segunda barras representa
una h, las x. entre la segunda y tercera barras representan t y todo x a la derecha de la
tercera barra representa una p. Por ejemplo, la tercera compra tiene tres barras consecuti-
vas, porque nadie compró un bot dog ni un taco; la barra que está al principio de la cuarta
compra indica que no se adquirieron hamburguesas con queso.
De nuevo, se ha establecido una correspondencia entre dos colecciones de objetos, en
la que sabem.o s cómo contar el número de una colección. Para las representaciones de la
columna (b) de la tabla 1.6, estamos enumerando todas las disposiciones de 10 símbolos
que constan de siete letras x y tres barras 1, de modo que, por nuestra correspondencia, el
número de órdenes diferentes para la columna (a) es

10! (1º)
7!3! = 7 .
34 Capítulo 1 Principios fundamentales del conteo
.
En este ejemplo observam.os que las siete letras x(una para cada estudiante) correspon-
den al tamaño de la selección y que las tres barras son necesarias para separar los 3 + 1 =
4 alimentos que se pueden elegir.

En general, cuando queremos elegir, con repeticiór,. r de n objetos distiatos, vemos e


(como en la tabla 1.6) estamos considerando todas- las disposiciones de r leb»:r~
n-1 l,yquesunúmeroes

(n +r-1)! =(n + r -t).


r!(n- 1)! r
En consecuencia:, el número de combinaciones de n objétos tomados de r ea r, c.
repeticiones, es C(n + r - I, ,).

(En et ejemplo 1.27, n = 4, r = 7, de modo que es posible que r sea mayor que n cuando se
permiten las repeticiones.)

Una tienda ofrece 20 tipos de donas(donuts). Si suponemos que al menos hay una docena
de cada tipo cuando entramos a la tienda, podemos elegir una docena de donas de C(20 +
12 - l, 12) = C(31, 12) = 141,120,525 formas. (En este caso, n =20, r = 12.)

La presidenta Elena tiene cuatro vicepresidentas: ( 1) Beatriz, (2) Carmen, (3) María Luisa
y (4) Mónica. Elena desea distribuir entre ellas $1000 en cheques de graúficación en navi-
dad; cada cbeque será expedido por un m.últiplo de $ l 00.
a) Si se admite el caso en que una o más de las vicepresidentas no obtenga nada, la
presidenta Elena hace una selección de tamaño 10 (uno por cada unidad de $'100)
de una colección de tamaño 4 (cuatro vicepresidentas), con repetición. Esto puede
hacerse de C(4 + 10- l, 10) = C(l3, 10) = 286 formas.
b) Si desea que no haya resentimientos, cada vicepresidenta debe recibir al menos $100.
Con. esta restricción, la presidenta Elena debe hacer ahora una selección de tamaño 6
(las restantes seis unidades de $100) de la misma colección de tamaño 4, y las opciones
son ahora C(4 + 6-1, 6) = C(9, 6) = 84. [Por ejemplo, en este caso, la opción 2, 3, 3, 4,
4, 4 se interpreta así: Beatriz no recibe nada adicional, ya que no hay ningún 1 en la
opción. El 2 de la selección indica que Carmen recibe $100 adicionales. María Luisa
recibe $200 adicionales, $100 por cada uno de los dos 3 que hay en la selección. Debido
a los tres 4, el cheque de gratificación de Mónica es de $100 + 3($100) =$400.J
e) Si cada vicepresidenta debe recibir al menos $100 y Mónica, vicepresidenta ejecu-
tiva, recibe al menos $500, entonces el número de formas en que la presidenta
Elena puede distribuir los cheques de gratificación es
C(3 + 2-1, 2) + C(3 + 1-1, 1) + C(3+ 0-1,0) = 10= C(4+2-1,2)
Mónica recibe M6.nka recibe M 6ni<.a recibe Usando la
exactamente SSOO. exactamente S600. exactamente $700. técnica de la parte (b).

1.4 Combinacionés con repetición: Distribuciones 35·

Habiendo trabajado ya con estos ejemplos de combinaciones con repetición, analizare-


mos ahora dos ejemplos relativos a otros principios de conteo.

¿De cuántas formas podemos distribuir siete manzanas y seis naranjas entre cuatro niños.
de modo que cada uno reciba al menos una manzana?
Al dar a cada niño una manzana, tenemos C(4 + 3 - l, 3) = 20 formas de distribuir las
otras tres manzanas y C(4 + 6 - 1, 6) =84 formas de distribuir las seis naranjas. Así, por la
regla del producto, hay 20 x 84 = 1680 formas de distribuir la fruta en las condiciones
dadas.

Un mensaje está formado por 12 símbolos diferentes y se va a transmitir a través de un canal


de comunicación. Además de los 12 símbolos, el transmisor también enviará un total de
45 espacios en blanco entre los símbolos, usando al menos tres espacios entre cada par
de símbolos consecutivos. ¿De cuántas formas puede el transmisor enviar ese mensaje?
Hay 12! formas de colocar los 12 símbolos diferentes y para cualquiera de estas dispo-
siciones hay 11 posiciones entre los 12 símbolo~. Debido a que debe haber al menos tres
espacios entre los símbolos consecutivos, utilizamos hasta 33 de fos 45 espacios y distri-
buimos los 12 espacios restantes. Ésta es entonces una selección, con repetición, de tama-
ño 12 (los espacios) de una colección de tamaño 11 (las posiciones), y puede realizarse de
C(ll + 12- 1, 12) = 646,646 formas.
En consecuencia, por la regla del producto, el tran.smisor puede enviar tales mensajes
=
con el espacio necesario de ( 12 !)(H) 3097 x 1O'" formas.

En el siguiente ejemplo se introduce una idea que parece tener más que ver con la teoría
de números que con las combinaciones o las disposiciones. Sin embargo. la solución de
este ejemplo será equivalente al conteo de las combinaciones con repetición.

Determine todas las soluciones enteras de la ecuación

dondex1 > O para toda I ~ i < 4.

Una solución de la ecuación es x 1 = 3, x2 = 3, x 3 = O, x, = l. (Esto es di ferente de una


solución del tipox, = l,xl = O, x 3 =3,x, =3, aunque se utilicen los mismos cuatro enteros.)
Una posible interpretación de la solución x, = 3, x, = 3, x 3 = O, x, = 1 es que estan1os
distribuyendo siete monedas (objetos idénticos) entre cuatro niños (recipientes diferen-
tes), y que hemos dado tres monedas a cada uno de los dos primeros niños. nada al tercero
y la última m oneda al cuarto niño. Si seguimos con esta in terpretación, vemos que cada
solución entera no negativa de la ecuación corresponde a una selección, con repetición, de
tamaño 7 (las monedas idénticas) de una colección de tarn.año 4 (los niños distintos), de
modo que hay C(4 + 7 - 1, 7) = 120 soluciones.

36 Capítulo 1 Prinápios fundamentales del conteo

En este momento es imprescindible n:conocer la •

a) El número de soluciones enteras de ).o, ecuaci6a.

X1 +x2+ · · · + x.=r,

b) El número de seleccioi,es.. CQII iepetieido, cft>•w-


llio,,:
e) El número de f(lffl)3S en~ ,objeto!¡ c:mrintm&
pientes distintos.

En términos de distribuciones, la parte (c) es válida sólo cuando los r objetos que se
distribuyen son idénticos y los n recipientes son distintos. Cuando los r objetos y los n
recipientes son distintos, podemos elegir cualquiera de los n recipientes para cada uno de
los objetos y obtener n' distribuciones mediante la regla del producto.
Cuando los objetos son distintos pero l os recipientes son idénticos, resolveremos el
problema mediante los números de Stirling de segundo tipo (cap. 5). Para el último caso,
cuando los objetos y los recipientes son idénticos, la teoría de particiones de los enteros
(cap, 9) nos dará algunos resultados necesarios.

¿De cuántas formas se puede distribuir 10 canicas blancas (idénticas) ea seis recipientes
diferentes?
La solución de este problema es equivalente a determinar el número de soluciones
enteras no negativas de la ecuación x 1 + x2 + ,.. + x6 = 10. Ese número es la cantidad de
selecciones de tamaño l O, con repetición, de una colección de tamaño 6. Por lo tanto, la
respuesta es C(6 + 10 - l, 10) =3003.

Ahora analizaremos dos ejemplos más relacionados con el tema de esta sección.

A partir del ejemplo l ,33, sabemos que hay 3003 soluciones enteras no negativas para la
ecuación x, + Xz + •·· + x6 = 10. ¿C uántas sol uciones enteras no negativas corresponden a
la desigualdad x 1 + x 2 + ·· · + x6 < l O?
Un método que podría ser adecuado para abordar esta desi gualdad consi ste en detenni-
nar el número de soluciones enteras no negativas de x 1 + x 2 + ·· · + ~ = k, donde k es un
entero y O :S k < 9. Aunque factible, la técnica es poco realista si se reemplaza 10 coa un
número un tanto mayor, como 100. Sin embargo, en el ejemplo 3.11 del capítulo 3 estable-
ceremos una identidad combinatoria que nos ayudará a obtener una solución alternativa al
problema mediante este enfoque.
Por el momento transformaremos el problema observandoJa correspondencia entre las
soluciones enteras no negativas de

X¡ +Xi + • • · + ~ < 10 (1)


y las soluciones enteras de

X¡+ Xi+ • · • + ~ + X1 = 10, Ü :S X¡ , 1:Si S6, O<x7 • (2)


, .4 1..omoinac1ones con repeuc,on: l.ilstrrouaones 37

El número de soluciones de (2) es el mismo que el número de soluciones enteras no


negativas dey, +y2 + ·· · + y6 +y, = 9, donde y; =x; para l :S. i :f 6, y y7 =x7 - l , es decir, C(7
+ 9- 1,9) = 5005.

Nuestro siguiente resultado regresa a los desarrollos binomiales y multinomiales.

En el desarrollo binom.ial de (x + y'f, cada término es de la forma (r.>ry,-,,, de modo que el


número total de térm.inos que hay en el desarrollo es el número de soluciones enteras no
negativas de n 1 + n 2 = n (n1es el exponente·dex, n2 es el exponente de y) . Este número es
C(2+n-l,n)=n+l.
Tal vez parezca que hemos utilizado un argumento muy rebuscado para obtener este
resultado. Muchos de nosotros estaríamos dispuestos a creer en este resultado con base en
nuestra experiencia al desarrollar (x + y)• para varios valores pequeños den.
Aunque la experiencia es útil en el reconocimiento de patrones, no siempre es suficien-
te para determ.inar un principio general. En este caso sería de poco valor si quisiéramos
determinar la cantidad de términos que hay en el desarrollo de (w + x + y+ z) 10 .
Cada té.r mino distinto es de la forma (.,.,,',º.,,;.,,,)w"1 x"' y"' z"', donde O !f n; para 1 s i S 4
y n 1 + n2 + n 3 + n, = 10. Esta última ecuación puede resol verse de C( 4 + 1O- l, l O) = 286
formas, de modo que hay 286 térirunos en el desarrollo de (w + x + y + z)'º·

Y ahora entrará de nuevo en juego el desarrollo binomial, ya que usaremos la parte (a)
del corolario l. l.

a) Determinemos todas las formas posibles.en las que podemos escribir el número 4
como una suma de enteros positivos, en la que el orden de los sumandos se conside-
ra necesario. Estas representaciones se denominan con1posiciones de 4 y pueden
enumerarse como Sigue:

1) 4 S) 2+1 +1
2) 3+1 6) 1+2+1
3) 1+3 7) 1 + 1 +2
4) 2+2 8) l + l+l + l

Aquí incluimos la suma que consta de un solo sumando, a saber, 4. Veremos que
para el número 4 hay ocho composiciones·en total. (Si el orden de los sumandos no
nos preocupa, entonces las representaciones en (2) y (3) ya no se considerarán dife-
rentes, así como tampoco las representaciones de (5), (6) y (7). En estas circunstan-
cias, veremos que hay cinco particiones del número 4; a saber, 4; 3 + 1; 2 + 2; 2 +
1 + 1 y l + 1 + 1 + l. Aprenderemos más acerca de las paniciones de enteros
positivos en la sección 3 del capítulo 9.)
b) Supongamos ahora que queremos contar el número de composiciones para el nú-
mero 7. En este caso no queremos enumerar todas las posibilidades, que incluyen el
7; 6 + 1, 1 + 6; 5 + 2, 1 + 2 + 4 ; 2 + 4 + l y 3 + l + 2 + l. Para contar todas estas
composiciones, consideremos el número de sumandos posibles.
38 Capítulo 1 Principios fundamentales del conteo

i) ·Para un sumando sólo existe una composición, a saber, 7.


ü) Si existen dos sumandos (positivos), queremos contar el número de soluciones
enteras de

Esto es igual al número de soluciones enteras de

X1 + X2 = 5, donde X¡, X2 ~ o.
El número de soluciones de este tipo es (2 • J- 1) = ct).
ili) En nuestro siguiente caso analizaremos las composiciones con tres sumandos
(positivos). Así que ahora queremos contar el número de soluciones enteras
positivas de
Y1+y2+y3=7.
,
Este es igual al número de soluciones enteras no negativas de

Zt + Z2 + Z3 =4,
y ese número es (3 + t - 1) = (! ).
Resumimos los casos (í), (ii) y (iii), y los otros cuatro casos, en la tabla 1.7, en
la que recordam.o s que 1 = (i) para ti caso (i).

Tabla 1.7

n = número de sumandos Número de composiciones


tn una composición de 7 de 7 con II sumandos

(i) n=l (i)


(!)
(ii) n =2 (ii)
(!)
(iü) n=3 (iii)
(!)
(iv) n=4 (iv) (:)
(v) n=S (v)
(~)
(vi) n=6 (vi)
(~)
(vii) n=7 (vii)
(~)
En consecuencia, los resultados del lado derecho de la tabla indican que el
número (total) de composiciones de 7 es
1.4 Combinaciones con repetición: Distribuciones 39

(~) + (~) + (!) + (~) + (~) + (~) + (~) = k*O(:).


Y, a partir de la parte (a) del corolario 1.1, esto se reduce a 2 6 •

En general, para cada entero positivo m, ex_isten ¿;',::cm ; 1) = 2m•1 composiciones.

Los dos últimos ejemplos de esta sección son aplicaciones del área de ciencias de la
computaeión. Además, el último ejemplo conduce a una importante fórmula para la suma
que utilizaremos en muchos capítulos posteriores.

Consideremos el siguiente segmento de un programa en Pascal, donde i, j y k son variables


enteras.

Fo.r i : = 1 t o 2 0 do
For j : = 1 to i do
For k := 1 to j do
Writeln (i * j + k );
¿Cuántas veces se ejecuta la proposición Writeln en este segmento de programa?
Entre las opciones posibles de i,j y k (en el orden i pcimero,j segundo y k tercero) que
nos conducen a la ejecución de la proposición lVriteln, enumeramos (1) 1, 1, l; (2) 2, l, 1;
(3) 15, 10, l y (4) 15, 10, 7. Observamos que i = 10, j = 12, k = 5 no es una de las
selecciones que -deberán considerarse, ya que j = =
12 > 10 í; esto viola la condición
establecida en el segundo ciclo For. Cada una de las cuatro selecciones anteriores en las
que se ejecuta la proposición Writeln satisface la condición l s k < j s i S 20. De hecho,
cualquier selección a, b, c (a s b s e) de tamaño 3, con repeticiones, de la lista l , 2, 3, . .. ,
20 produce una de las selecciones correctas: en este caso, k = a, j = b, i = c. En consecuen-
cia. el enunciado Writeln se ejecuta

20 2
( +: - l) = (~ ) = 1540 veces.

Si hubiera r (.:::: l ) cíclos Foren vez de rres, la proposición Writeln se habría ejecutado
(2°•:-1 ) veces.

Aquí usaremos el siguiente segmento de programa en Pascal para obtener una fórmula
para la suma. En este segmento de programa, las variables i, j, n, y coun1er son variables
enteras. Hemos supuesto que, en una sección anterior del programa. el usuario proporcio-
nó un entero positivo; este dato establece el valor den.

counter : = O;
For i : = 1 ton d o
For j : = 1 to ido
counter : = counter + l;
40 Capítulo 1 Princioios fundamentales del conteo

A partir los resultados del ejemplo 1.37. después de que se ejecute este segmento, el
valor de counter será('-• ?- 1) = (" i 1), (Esto es igual al número de veces en que la proposi-
ción

(*) counter · = counter + l

se ejecuta.)
Este resultado también puede obtenerse como sigue: cuando i: = l ,j varía de I a I y
(*) se ejecuta una vez; cuando i recibe el valor de 2, entonces j varía de I a 2 y (*) se
ejecuta dos veces ; j vaóa de l a 3 cuando i tiene el valor 3 y (") se ejecuta tres veces; en
general, para 1 < k < n, cuando i : = k, entonces j varía de 1 a k y (*) se ejecuta k veces. En
total, la variablecounter se incrementa [y la proposición (*) se ejecuta] 1 + 2 + 3 + ••• + 11
veces.
En consecuencia.

"
Í.:i = 1+2+3+ .. ·+n= (n + 1) = -'--
n(n + 1)
~ .
~· 2 2
La d ~rivación el~ esta fórmula para la suma, obtenida al contar el mismo resultado de
dos fo= as diferent es, constituye una demostración combinatoria.

EJERCICIOS 1.4 1. ¿De cuántas formas es posible distribuir l Omonedas (idénticas) entre cinco niñ.os si (a) no hay
restácciones? (b) cada niño recibe al menos uoa moneda? (c) el niño mayor recibe al menos
dos monedas?
2. ¿De cuántas formas es posible distábuir 15 barras de chocolate (idénticas) entre cinco niños.
de modo que el más pequeño sólo reciba una o dos?
3. Determine las formas en que se pueden elegir 20 monedas de cuatro grandes recipientes que
contienen mon.edas de diferente denominación. (Cada recipiente contiene un solo tipo de mo-
neda.)
4. Una tienda de helados tiene disponibles 31 sabores de helado. ¿De cuántas formas se puede
ordenar una docena de conos de helado si (a) no queremos el mismo sabor más de una vez?
(b) un sabor puede ordenarse hasta 12 veces? (c) un sabor no puede ordenarse más de 11
veces?
5. a) ¿Decuáncas formas es posible seleccionar cinco monedas de una colección de 10. formada
por una moneda de l centavo. una moneda de 5 centavos, una de 10 centavos. una de 25
centavos, otea de 50 centavos y cinco dólares (idénticos)?
b) ¿De cuáncas formas podemos seleccionar n objeros de una colección de tamaño 211 que
consta den objetos distintos y n objetos idénticos?
6. Resuelva el ejemplo 1.31 pero, en este caso. los 12 símbolos que se van a transmitir son cuatro
letras A, cuatro B y cuatro C.
7. Determine el número de soluciones enteras no negativas de x, + xi + x 3 + x. = 32. donde
3 ) X¡ ->O , 1<"<4
- 1- . b) X;> O, 1 < t. S 4 •
e) X1, X22:5, x,, x.>7. d) x,2:8, I:s:is4.
e) x, > - 2, I s is4. f ) x.,x,,x, > 0, O<x.s25.
1.4 Combinaciones con repetición: Distribuciones 41

8. ¿De cuántas fonnas puede distribuiI un maestro ocho donas de chocolate y siete donas glaseadas
entre tres esrudi antes si cada uno desea al menos una dona de cada ú po?
9. Columba tiene dos docenas de colecciones de n piedrecillas de colores diferentes. Si puede
seleccionar 20 (penniúendorepeticiones de colores) de 230,230 formas, ¿cuál es el valor de 11?
1 O. ¿De cuán1as formas puede tirar Lisa 100 dados (idénticos), de modo que caigan al menos tres
dados de cada cara diferente?
11. Dos enteros den dígioos (se permiten ceros al principio) se consideran equivalentes si uno e s
una redisposición de ocro. {Por ejemplo. los enteros de cinco dígitos 12033, 20331 y 01332 se
consideran equivalentes.)
a) ¿Cuántos enteros de cinco dígitos no son equivalentes?
b) Si los dígitos 1, 3 y 7 pueden aparecer cuando mucho una vez. ¿cuántos enteros no equiva-
Jences de cinco dígitos e xisten?
12. Determine el número de soluciones enteras de x, + x, + x, + x, + x; < 40, donde
a) X¡ ie 0, } :SÍ S 5. b) x.-> -3, 1 <i<S.
13. ¿De cuántas formas se pueden distribuir ocho bolas blancas idénticas en cuatro recipie01es
distintos de modo que (a) ningtín recipiente quede vacío? (b) el cuarto recipiente contenga un
número impar de bolas?
14. a) Encuentre el coeficiente de v'w'xz en el desarrollo de (3v + 2w + x + y + z)8•
b) ¿Cuántos términos distintos aparecen en e l de~rrollo de la parte (a)?
15. ¿De cuántas formas puede colocar Beto 24 libros diferentes en cuatro repisas de modo que
haya al menos un li bro en cada repisa? (Para cualquiera de esias disposiciones. considere que
e n cada repisa los libros deben ser colocados uno jumo al otro. y el primer libro a la izquierda.)
16. ¿Para qué entero positivo n se cumple que las ecuaciones

(1) x 1 + x 2 + x 3+ •· • +x;~=n, y
(2) y, + Y• + y, + · · · + y.. = n
tengan el mismo n6mcro de soluciones enteras positivas?
17. ¿De cuántas fonnas se pueden colocar 12 ca.-iicas del mismo iamaño en cinco recipientes dis-
tintos si (a) todas las canicas son negras? (b) cada canica es de distinto color?
18. Un maestro de química tiene siete cajas, cada una con 36 rubos de ensayo con sustancias
'"desconocidas" para un experimento de laboratorio. Las 36 sustancias de la prim.era caja contie-
nen cuatrocompuC$tOS diferentes que aparecen 5, 12, 7 y 12 veces. respecúvamente. ¿De cuántas
formas se puede distribuir el contenido de esta caja entre cinco laboratorios de química'.'
19. ¿ De cuántas formas es posible distribuir una moneda de 25 cemavos, otra de 10. otra de 5 y 25
monedas de un ceniavo entre cinco niños (a) sin restricciones? (b) de modo que el niñ.o más
grande reciba 20 o 2~ centavos?
20. a) ¿Cuántas soluciones en1eras no negativas tiene la pareja de ecuaciones
X 1 + X2 + X3 + ·•• + X1 = 31, X i + X1 + X3 = 6[
b) ¿Cuántas de las soluciones de la parte (a) tienen x 1• x,, Xi > O?
21 . ¿Cuántas veces se ejecuta la proposición \Vriteln en el siguiente segmento de programa e n
Pascal? (En este caso, i,j, k y m son variables enteras.)

Por i := 1 to 20 do
Por j : = 1 to ido
For k : = 1 to j do
Po r m : = 1 to k do
Writeln (( i*j ) + ( k * m)) :
Capitulo 1 Principios fundamentales del conteo

22. En el siguiente segmento de programa en Pascal. i,j. k. y counrer son variables enteras. Deter-
mine el valor de counter despu~s de ejecutarse el segmento de programa.

counter : = 10;
For i : = l to 15 do
For j : = i to 15 do
For k : = j to 15 do
counter : = counter + l;
23. Determine el valor de la variables1D11 después de ejecutarse el siguiente segmento de programa
e n Pascal. (En este caso, i, j, k., inc~rrumt y sum son variables enteras.)

increment : O: =
sum : = O:
For i : = 1 to 10 do
For j : = 1 t o i do
For k : = l to j do
Begin
increment : = increment + l;
sum : = su.m + increment
End;

24 . Considere el siguiente segmento de programa en Pascal, dondei,j, k, n y counterson variables


enteras. En una parte anterior del programa. el usuario ha dado el entero positivo que establea:
el valor de n para esa ejecución particular del programa.

counter : = O:
For i : = l to n do
For j : = 1 to i do
For k , - l to J do
counter : = counter + 1:
Dctcmtinaremos, de dos modos distintos, el número de veces que la proposición

counter : = counter + l
se ejecuta. (Éste es tambitn el valor de counUr después de la ejecución del segmento de progra-
1
ma..) A partir de1 resultado del ejemplo 1.37, sabemos que la proposición se ejecuta (-j ) =
2
( "; ) veces. Para un valor fijo dei, los ciclosForrelativos ajy k. producen Cit')
ejecuciones de
la proposición donde se incrementa counter. En consecuencia,()2 )= L;_
Cit). Use este
k . .t
1
resultado para deducir una fórmula para la suma de 11 + 22 + 3i + •·· + n 2 =
25. a) Use las ideas del ejemplo 1.38 y el ejercicio 24 para explicar por quf (~3)= ¿~1<'~2 ).
b) Use el resultado de la pane (a) para obtener una f6nnula para la suma

ll+2'+ 3l+ . . +nl= f ¡l_


i- 1

26. Mucsue que el número de formas de colocar n objetos distintos en r recipientes diferentes, con
los objetos ordenados en cada recipiente, es P(r + n - J. r - l ).
27. a) Dados m,n enteros positivos con m 2:: n, muestre que el número de fonnas de distribuir m
objetos idénticos en n recipientes distintos, sin que quede un recipiente vacío, es C(m - I ,
m-n)::C(m- l , n-1) .
1.5 una aplicación o las ciencias físicas (opcional) 43

b) Muestre que el nómcro de disttibuciones de la parte (a), doode cada recipiente contiene al
menosrobjetos (m ~ nr), es C(m-1 + (1-r)n, n-1).
28. Escriba un programa (o desarrolle un aJgoritmo) para calcular las soluciones e nteras de

29. Escriba un programa (o desarrolle un algoritmo) para calcular las soluciones e nteras de

x1+ .x1 + x, +x,=15, Os.xi, lsis4.

3 0 . Escriba un programa (o desarrolle un algoritmo) para calcular las soluciones enteras de

x1+x2 +x3 + x.. =4, -2sx,, 1::si::s4.

1.5
Una aplicación a las ciencias
f!sicas (opcional)

En esta sección analizaremos una aplicación de las técnicas para contar desarrolladas en
este capítulo. Esta aplicación surge en mecánica estadística y termodinámica estadística.
En estas áreas, nos hemos interesado en el número de formas en querpartículas subatómicas
pueden distribuirse entren estados de energía distintos.
En el modelo de Maxwell-Boltzmann se supone que las partículas son distintas y que
cualquier cantidad de ellas puede estar en cualquier estado de e nergía. Obtenemos rf dis-
tribuciones posibles debido a que existen n estados de energía posibles para cada una de
las r partículas. (La teoría moderna de mecánica cuántica ha demostrado que este modelo
no es adecuado para las partículas subatómicas conocidas hasta el momento.)
Otros dos modelos con más éxito son los de Bose-Einstein y Ferm.i-Dirac, que se basa-
ron en un principio en los momentos angulares intrínsecos de las partículas.
El modelo de Bose-Einstein requiere que las r partículas sean idénticas, y cualquier
cantidad de ellas puede estar en Cualquier estado de energía. El número de distribuciones
diferentes en este caso es el número de soluciones enteras no negativas dex1 + x?+ ••• + x,,
= r. igual a C(n + r - 1, r ). Las partículas con espín entero (en unidades de (hlm), donde h
es la constante de Planck) siguen este modelo. Dichas partículas, llamadas bosones, inclu-
yen a los fotones, los mesones pi y las parejas de electrones de conducción en el material
superconductor, como el plomo o el estaño.
Para las partículas con espín scmíentero. como los protones, electrones y neutrones, el
modelo de Fermi-Dirac es más adecuado, y las parúcuJas se conocen comofermiones. En
este modelo, las r partículas son de nuevo idénticas, pero un estado de energía puede
contener al menos una partícula En consecuencia. e n este caso r :s.: n y el número de
distribuciones posibles es (;'). (Este modelo es bastante útil en el estudio de la teoría de
bandas semiconductoras.)
44 Capítulo J Principios fundamentales del conteo

1.6
Resumen y repaso histórico

En este primer capítulo presentamos los fundamentos del conteo de combinaciones,


permutaciones y disposiciones en muchos tipos de problemas. La descomposición de los
problemas en componentes que requieren fórmulas iguales o diferentes para su solución
proporcionó un acercamiento clave a las áreas de matemáticas discreta y combinatoria
Esto es similar al enfoque descendente para el desarrollo de algoritmos en un lenguaje de
programación ·estructurada como Pascal. En este caso, se desarrolla el algoritmo para re-
solver un problema difícil abordando primero algunos subproblemas principales que de-
ben resolverse. Después, estos subproblemas se refinan (se subdividen en tareas de pro-
gramación más manejables). Cada nivel de refinamiento mejora la claridad, precisión y el
objetivo del algoritmo, hasta que se puede traducir fácilmente al código del lenguaje de
programación. (Hablaremos más de los algoritmos en capítulos posteriores.)
La tabla 1.8 resume las principales fórmulas para contar desarrolladas hasta ah~ra En
este caso utilizamos una colección den objetos distintos. Las fórmulas cuentan el número
de formas para seleccionar, u ordenar, con o sin repeticiones, r de estos n objetos. Los
resúmenes de los capítulos 5 y 9 incluyen otras tablas similares que aparecerán conforme
Vayamos ampliando nuestro estudio de otros métodos de conteo.

Tabla 1.8
El orden es ·se permiten
significativo . repeticiones Posidóa en
Tipo de resultado Fórmula el luto
Sí No Permutación P(n, r) =n!/(n - r)!, Página8
Osr<n
Sí Sí Disposición n' n,r>O
' Página.8
No No '
Combinación C(n, r) =n !l[r!(n - r)!J = (;), Página 19
Osrsn
No Sí Combinación con (n +;-1), n,r~O Página 34
repetición
.

Al continuar nuestro estudio sobre otros principios de enumeración, así como sobre las
estructuras matemáticas discretas para aplicaciones en la teoría de códigos, la enumera-
ción, la optimización y los esquemas de ordenamiento en ciencias de la computación, nos
basaremos en las ideas fundamentales presentadas en este capítulo.
El concepto de permutación se puede encontrar en la obra hebrea Sefer Yetzirah (El
libro de la creación), ·un manuscrito escrito por un místico entre el año 200 y el 600. Sin
embargo, es interesante subrayar que ya antes un resultado de Xenócrates de Calcedonia
(396-314 a.C.) podría contener "el pri.m er intento por registrar la solución.de un problema
difícil de permutaciones y combinaciones" . Para más detalles, véase la página 319 del
texto deT.L. Heatb. [3] , así como la página 113 del artículo de N. L. Biggs [1), una valiosa
fuente de información acerca de la historia de la enumeración. El primer libro de texto que
1.6 Resumen y repaso h~tórico 45

estudia algo del material analizado en este capítulo fue An Conjectandi, del matemático
su.izoJakob Bemoulli (1654-1705), uno de los ocho destacados matemáticos de la familia
Bcmoulli. El texto fue publicado en forma póstuma. en 1713. y contiene una reimpresión
del primer traUodo formal de probabilidad. Este tratado fue escrito en 1657 por Christiaan
Huygens ( 16~1695), físico, matemático y astrónomo bol~ que descubrió los anillos
de Saturno.
El teorema binomial para n 2 2 aparece en la obra de Euclides (300 aC.); no obstante, el
t~nnino "coeficiente binomial" fue introducido en realidad por Michcl Stifcr (1486-
1567) en el siglo XVL En su Arizhmerica Integra (1544), Stifcr da Jos coeficientes
binomiales basta el orden n = 17.). En sus investigaciones acerca de la probabilidad,
Blaisc Pascal (1623-1662) publicó, en la d~cada de 1650, un tratado acerca de las relacio-
nes entre los coefÍcicntcs binomiales, las combinaciones y los polinomios. Estos resulta-
dos fueron utilizados por Jakob Bernoulli para demostrar la forma general del leorema
del binomio, en una forma similar a la presentada en este capítulo. El uso del símbolo
C:) se inició en el siglo XIX, cuando fue utilizado por Andreas voo Ettinghausen ( 1796-
1878).

Bloi"' Pascal (1623-1662)

Sin embargo, sólo en el siglo XX, con el advenimiento de los computadores. fue posible
el anflisis sistemático" de los procesos y aJgoritmos utilizados para generar permutaciones
y combinaciones. En la sección 10.1 examinaremos uno de esos aJgorianos.
El primer libro de texto que trata ampliamente los temas de combinaciones y
permutaciones fue escrito por W. A. Whitworth [10). El material de este capítulo también
se analiza en el capítulo 2 de D. Cobcn [2], el capítulo I de C. L Liu [4], el capítulo 2 de
F. S. Robens [6], el capítulo 4 de K. H. Roseo m, elcapítulo I de H. J. Ryser [8] y el
capítulo 5 de A. Tucker (9). Para mis información acerca de las ideas de mecánica estadís-
tica y termodinámica estadística, consulte el libro de texto de R. Reed y R. Roy (5].
46 Capítulo 1 Principios fundamentales del conteo
,

BIBLIOGRAFÍA

l. Biggs, Normao L., ''Toe Roots of Combinatorics", Historia Marhematica 6, 1979, págs. 109-
136.
2. Coben, Daniel l. A., Basic Techniques o/Combinatoria[ Theory, Nueva York, Wtley, 1978.
3. Heatb, Thoroas Little, A History of Greek Marhematics, vol. 1. reimpresión de la edición de
1921, Nuev.a York, Dover Publications, 1981.
4. Liu. C. L., Introduction to Combinatoria/ Mathematics, Nueva York, McGraw-Hill, 1968.
5. Reed, Robert D. y R. R. Roy, Statisrical Physics for Srudenrs of Science and Engineering.
Scramon, Pa., lntext Educational Publisbers, 1971.
6. Robens, Fred S. , Applied Combinarorics, Englewood Cliffs, N.J., Prentice-Hall. 1984.
7. Rosen, Kennetb H., Discrere Mathemarics and lts Applicarions, 2' ed., Nueva York, McGraw-
Hill, 1991.
8. Ryser. H. J., CombinatorialMarhemarics. publicado por la MathematicalAssociation ofAmerica.
Nueva York, \Vi!ey, 1963.
9. Tucker, Alao, Applied Combinarorics, 2' ed., Nueva York, Wiley, 1985.
1O. \Vhítwortll, W. A., Choice and Chance, reimpresión de la edición de 1901. Nueva York, Hafner.
1965.

nombres de archivo, el computador lee todas las letras como


EJERCICIOS · mayúsculas, sin importar si se escriben como mayúsculas o
COMPLEMENTARIOS minúsculas.) También es posible añadir al, nombre del ar-
chivo una extensión de arclúvo opcional. Esta se obtiene al
escribir un punto y de uno a tres caracteres alfanuméricos
1 . En la fabricación de cieno tipo de automóvil, pueden
más. Por lo tanto, seis ejemplos de nombres de archivo son
aparecer cuatro tipos de defectos mayores y siete tipos de (1) TAXI, (2) PRESPTO, (3) A 138478 J, (4) lNGRES0.87,
defectos menores. Para las situaciones en las que realmente (5) CALIF.JUN, y (6) Z35.P72. Los últimos tres ejemplos
se presentan los defectos, ¿ de cuántas formas puede haber utilizan la extensión de arclúvo opcional.
el doble de defectos menores que de mayores? a) ¿Cuántos nombres de archivo utilizan solamente
2. Una máquina tiene nueve diferentes discos, cada uno los 36 carac1eres alfanuméricos, sin extensión?
con cinco parámetros etiquetados como O. 1, 2, 3 y 4. b) ¿Cuántos de los nombres de archivo de la parte (a)
a ) ¿De cuántas formas se pueden configurar todos los comienzan con dos letras A?
discos de la máquina? e) ¿Cuántos nombres de archivo usan extensiones con
b) Si los nueve discos se ordenan en una línea en la tres caracteres alfanuméricos exactamente?
parte superior de la máquina, ¿cuántas de las con- 4. La directora de un coro debe elegir seis himnos para el
figuraciones de la máquina no tienen dos discos servicio reügioso dominical. Ella tiene tres libros de lúm-
adyacentes con el mismo parámetro? nos, cada uno de los cuales contiene 25 himnos (e n total
e) ¿Cuántos de los parámetros de la parte (b) sólo uti- hay 75 himnos distintos). ¿De cuántas formas puede elegir
lizan O. 2 y 4 como parámetros en los discos? los lúmnos si desea elegir (a) dos himnos de cada libro? ( b)
3 . El software para procesamiento de palabras en cierto al menos un himno de cada libro?
microcomputador permite arcllivos cuyos nombres tengan S. ¿De cuántas fortnas es posible colocar 25 banderas di-
de uno a ocho caracteres. Cada carácter puede ser alguno de ferentes e-n I O astas numeradas si el orden de las banderas
los 36 caracteres alfanuméricos (26 letras y I O dígitos) o en un as1a (a) no es significativo? (b) sí lo es? (c) sí es signi-
cualquiera de otros 15 sfmbolos determinados. (Para los ficativo y en cada asta ondea.al menos una bandera?
Ejercicios Complementarios 47

6. Una moneda se lanza 60 veces, dando como resulta- la selección de modo que e l producto de los cuatro núme-
do 45 caras y 15 cruces. ¿De cuántas formas podña haber ros sea positivo y (i) los números sean distintos? (ii) cada
ocurrido esto de modo que no hubiera cruces consecuti- número pueda elegirse hasta cuatro veces? (iii) cada nú-
vas? mero pueda elegirse cuando mucho tres veces? (b) Res-
7. Hay 12 hombres en un baile. (a) ¿De cuántas fonnas ponda la pane (a) cuando e l producto de los cualro núme-
pueden ser elegidos ocho de ellos para formar un grupo de ros es negativo.
limpieza? (b) ¿De cuántas formas se pueden formar parejas 16. Una casa para esrudiantes de una universidad funciona
con ocho mujeres que hay en el baile y ocho de estos 12 bajo la supervisión del señor Morales. La casa tiene tres
hombres? pisos, cada uno de los cuales está dividido en cuatro seccio-
8. ¿Cuántas secuencias cuaternarias den-dígitos (0,1,2,3) nes. El próximo año, el señor Morales tendrá 12 estudiantes
tienen exactamente r unos? (uno para cada una de las 12 secciones). Entre los 12 estu-
diantes hay cuatro de tercer año (Daniel, Femando. Hugo y
9. ¿De cuántas formas se pueden colocar las letras de
Toño. (Los otros ocho esrudiantes serán nuevos en el curso
WONDERING de modo que tengan exactamente dos voca- de otoño y se designan como de primer año.) ¿De cuántas
les consecutivas?
formas puede asignar el señor Morales las secciones a los
10. Un disolvente orgánico se hace mezclando seis com- 12 estudiantes si
puestos líquidos diferentes. Después de que un primer com- a) no hay restricciones?
puesto se viene en un recipiente. los demás com¡mestos se b) Daniel y Femando deben ser asignados juntos al
agregan en un orden predeterminado. Se prueban todos los primer piso?
órdenes posibles para determinar el mejor resultado. ¿ Cuán- c) Hugo y Toño deben ser asignados a pisos diferen-
tas pruebas son necesarias? tes?
11. David tiene un conjunto de 180 bloques distintos. d) Toño .debe ser asignado a un piso que quede arriba
Cada uno de estos bloques está hecho de madera o plástico del asignado a Hugo?
y viene en alguno de tres tamaños (pequeño, mediano, gran- e) Daniel, Femando y Hugo deben ser asignados a
de), cinco colores (rojo, blanco, azul, amarillo. verde) y pisos diferemes?
seis formas (triangular, cuadrado, rectangular. hexagonal, 17. a) ¿ Cuántos de los 9000 enteros de cuatro dígitos
octagonal. circular). ¿ Cuántos de los bloques de este con- 1000, 1001. 1002, ...• 9998. 9999 tienen cuatro
junto difieren dígitos dife.rentes que son crecientes (como en
a) del bloque pequeño, rojo, de madera, cuadrado en 1347 y 6789) o decrecientes (como en 6421 y
exactamente una forma? (Por ejemplo, el bloque 8653)?
pequeño, rojo, de plástico, cuadrado es uno de esos b) ¿ Cuántos de los 9000 enteros de cuatro dígitos
bloques.) 1000, 1001, I002• . ... 9998, 9999 tienen cuatro
b) del bloque grande, azul, de plástico, hexagonal en dígitos que son no decrecientes (como en ]347,
exactamente dos formas? (Por ejemplo, el bloque 1226 y 7778) o no crecientes (como en 6421, 6622
pequeño, rojo. de plásrico, hexagonal es uno de y 9888)?
esos bloques.)
18. a) Determine el coeficiente de x1yz' e n el desarrollo
12. E señor y la sei\ora Rodñguez desean que su hija re- de [(x/2) + y - 3zJ2
cién nadda tenga sus tres iniciales (nombre, segundo nom- b) ¿Cuántos lérminos distintos hay en el desarrollo
bre y apellido) en orden alfabético sin que se repita una iru- complewde
cial. ¿ Cuántas de estas temas de iniciales pueden aparecer
en estas circunstancias?
13. ¿De cuántas formas pueden pintarse los IO caballos
idénticos de un carrusel de modo que tres de ellos sean ma-
rrón, tres blancos y cuatro negros?
14. ¿De cuántas formas puede distribuir un maestro 12 li- e) ¿ Cuál es la suma de todos los coeficiemes del de-
bros diferentes de ciencia entre 16 estudiantes de modo que sarrollo complelo?
(a) _n ingún estudiante reciba más de un libro? (b) el estu-
diante de más edad reciba dos libros pero ningún otro estu- 19. a) ¿De cuántas formas se pueden sentar IO personas.
diante.reciba más de un libro? denotadas como A, B, .. .. Y, J, en romo de la
mesa rectangular que se muestra en la figura 1. 10.
15. De la siguiente lista se eligen cuatro números: - 5, -4, si las figuras 1.1 O(a) y l. 1O(b) se consideran igua-
-3, -2. - 1, 1, 2, 3, 4. (a) ¿De cuántas formas se puede hacer les enlre sí pero diferentes de la figura I. IO(c)?
48 Capítulo 1 Principios fundamentales del conteo

A 8 F G 1 J

H
·□, ·□" "□·
1 0

E
O

e
1

J
G 8

F C

G F 8 A E o
(a) (b) (e)

Figura 1.10

b) ¿En cuántas de las disposiciones de la parte {a) que- 23. Sean un número impar. ¿De cuántas formas podemos
dan A y 8 sentados en los lados más largos de la ordenar n unos y r ceros con una fila (lista de símbolos
mesa, uno enfrente del otro? consecu1ivos idénticos) de exactamente k unos, conk. :S n <
20. a) Detemtine el número de soluciones enteras no ne- 2k.?
gativas de la pareja de ecuaciones 24. Dados n objetos distintos. determine de cuámas for.
mas es posible ordenar rde estos objetos en un círculo; dos
disposiciones se consideran iguales si una se puede obtener
X1+X2+x,=6, X1+x2+·· · +X.s""'l5, de la otra mediante una rotación.
X;=!!: 0, l:Si:S5. 25. Para cualquier entero positivo n, muestre que

b) Responda la par1e (a), reemplazando las dos


ecuaciones por las dos desigualdades
26. a) ¿De cuántas formas se pueden ordenar las letras de
UNUSUAL?
x 1 + :c2 + x, s6, x1+x2+· ·· + x.s:S l 5, b) Para las disposiciones de la parte (a). ¿cuámos dit
X¡¡¡:Q, 1 :Si::S S, ellos tienen las tres letras U juntas?
c) ¿Cuántas de 13s disposiciones de la parte (a) no
lienen leiras U consecutivas?
21. ¿De cuántas formas es posible distribuir una docena de 27. Francisca tiene 20 libros diferentes pero e n la repisa de
manunas entre cinco niños de modo que ningún niño tenga su dormitorio sólo caben 12 de ellos.
más de siete manzanas? .a) ¿De cuántas formas puede colocar Francisca 12 de
22. En cualquier set de un torneo de tenis, el oponente A estos libros en la repis3?
puede vencer a B de siete fonnas diferentes. (En 6-6 juegan b) ¿Cuántas de las disposiciones de la parte (a) inclu-
a muen e súbita.) El primer opone me en ganar tres sets gana yen los tres libros de tenis de Francisca?
el torneo. (a) ¿De cuántas foonas se pueden registrar los 28. Determine el va1or de la variable en1er.a counur des•
marcadores en que A gana en cinco sets? (b) ¿De cuántas pués de la ejecución del siguiente segmento de programa de
formas pueden registrarse marcadores en que el tomro ne- Pascal. (En este caso, i, j, k, l, m y n son variables enteras.
cesite al menos cuatro sets? Lls variables,; sy ttambiér. son variables enteras: sus valo-
Ejercicios Complementarios 49

res (f > l, s .:: 5 y r .:: 7) se han asignado antes de la ejecución y


de este segmento de programa.)
4
1 1 1 1 1
counte r : = 10; 3
For i := 1 to 12 do 1 1 1
2
For j : = 1 tordo
1
counte r : = counter + 2; 1
For k := 5 to s do X
For 1 : = 3 to k do 1 2 3 4 5 6 7
c ounte r : = counter + 4 ;
Figura 1.11
For m := 3 to 12 do
counte r : = counter + 6 ;
For n : = t downt o 7 do
counter := counter + 8 ;
e) Responda las p arres (a) y (b) si se permite un ter-
29. a) Encuentre el número de formas en que se puede cer tipo de movimiento (D): (x, y) ➔ (x + 1, y+ 1).
escribir 17 como una suma de unos y doses si el
orden es significativo. 34. El siguiente ejercicio muestra un importante método para
b) Responda la parte (a) para 18 en vez de 17. contar, conocido corno principio de reflexión. En este caso,
e) Generalice los resultados de las panes (a) y (b) para una panícula se mueve en el plano xy de acuerdo con los
n impar y para n par. signientes movimientos:
30. a) ¿De cuántas formas es posible escribir 17 como
una suma de doses y treses si el orden de los U: (m,n)-+ (m + l , n + 1);
sumandos (i) no es significativo? (ii) sí es signifi- L: (m, n) -> (m + 1, n - 1);
cativo?
b) Responda la parte (a) para 18 en vez de 17.
donde m y n son enteros , m , n .:: O. En las figuras 1.12 (a)
31. a) Sin y r son enteros positivos .con n .:: r, ¿cuántas y 1.12 (b), tenemos dos de tales trayectorias, de (0. 3) a
soluciones tiene (7. 2).
Xi + X 2 + · · · + x, = n, a) ¿Cuánras de estas trayectorias hay de (O. 3) a (7 , 2)
·con estas restricciones?
donde cadax1 es un entero positivo, para 1 < j s r?
b) Las figuras 1. l2(c)y l.l2(d) demuestran lasiguien-
b) ¿De cuántas formas es posible escribir un entero
te idea acerca de las trayectorias de las partes (a) y
positivo n como la suma de r sumandos enteros (b) de la figura. respectivamente. Cuando una tra-
positivos (1 < r < n) si el orden de los sumandos es yectoria de (0, 3) a (7, 2) toca o cruza el eje .r. hay
significativo? una trayectocia c-orresporidiente de (0, -3) a (7. 2)
32. a) ¿De cuántas formas es posible recorrer el plano xy obtenida al reflejar el segmento inicial de la tra-
desde(] , 2) a (5. 9) si cada movimiento es de algu- yectoria antes de que toque o cruce el eje x. Use
no de los sigiúentes tipos: esta observación para dete rminar el número de tra-
yectorias de (0, 3) a (7, 2) que tocan o cruzan el eje
(H): (x,y)-. (x + 1,y); (V): (x,y) -> (x ,y + 1)?
x al menos una vez.
b) Responda la parte (a) si también es posible un tercer e) ¿ Cuántas trayectorias de (O. 3) a (7, 2) nunca tocan
movimiento diagonal (D): (x, y) ➔ (x + 1, y+ 1). o cruzan e l eje x?
33. a) ¿De cuántas formas se puede mover una panícula d) ¿ Cuántas de estas trayectorias de (l , l.) a (8, 2) nun-
en e l plano xy desde el origen al punto (7,4) si los ca tocan o cruzan el eje x?
movimientos permitidos son de la forma e) Debido a sus extraordinarias notas, Diana y Cata-
lina son las finalistas para obtener el título de estu-
(H): (x,y)-> (x + l,y); (V): (x,y) -> (x,y + l)? diante sobresaliente de física (en s u clase). Un co-
mité de 14 profesores ·s eleccionará a una de las
b) ¿Cuántas de las trayectorias de la parte (a) no utili- candidatas para ser la ganadora y colocará su voto
zan la trayectoria de (2. 2) a (3. 2) a (4. 2) a (4. 3) en una urna Supongamos que Catalina recibe 9
que se muestra en la figura J. 11? votos y Diana recibe 5. ¿De cuántas formas pue-
50 capítulo 1 Principi9s fundamentales del conteo

y y
4 4
/
3

2 '' /
3
2
' lf'.. f'..1 /
1

- 1
1 2
' "'
1
3 4 5
1
/
6
1
7
)(
1

- 1
1 2
1/
f'-4/5 6

7
)(

-2 -2
1
-3 -3
-4 1 1 - 4

(a) (b)

y y
4 4 .
1
3 3

2 2

1
/ 1
/
1 / X
/ )(
• • • •
1 2 3 4/ "s 6 7 1 2/"~"'-4/~ 6 7
-1 • -1
/ /.
/ , 1
-2 -2 ,
✓"
_.,
-3 • -3
' '· ./
-4 -4

.
(e) (d)

Figura 1.12

den extraerse los votos de la urna, de uno en uno, ma general llamado, en forma adecuada, el proble-
de modo que siempre haya más votos en favor de ma de la urna. Este problema fue resuelto por
Catalina? Éste es un caso particular de un proble- Joseph Louis Fran~is Bertrand (1822- 1900).
2
Fundamentos
de lógica

E n el ejemplo 1.38 (sección 1.4) del primer capítulo obtuvimos una fórmula para la suma.
Esta fónnula se obtuvo contando la misma colección de objetos (las proposiciones que
ejecutamos en cierto segmento de programa) de dos formas distinw e igualando luego los
resultados. En consecuencia. decimos que la fórmula fue establecida mediante una demos-
tración combinatoria. Ésta es una de las múltiples técnicas para obtener una demostración
con las que trabajaremos en todo el texto. ·
En este capítulo daremos un vistazo a Jo que es un argumento válido y una demostra-
ción más convencional Cuando un matemático desea ofrecer una demostración de una
situación dada. debe utilizar un sistema de lógica. Esto tambi6n es cieno para un científico
de la computación que desarrolla los algoritmos necesarios para un programa o sistema de
programas. La lógica de las matemáticas se aplica para decidir si una proposición se sigue
o es consecuencia lógica de una o más proposiciones.
Algunas de las reglas que rigen este proceso se describen en este capítulo. Usaremos
des.pu& estas reglas en las demostraciones (proporcionadas en el texto y solicitadas en tos
ejercicios) que aparcc:en en los siguientes capítulos. Sin embargo. en ningún momento
hemos pensado llegar a un punto en el que apliquemos las reglas de modo automático.
Como ha ocurrido con la aplicación de las ideas de la enumeración analizadas en el capí•
tulo 1. siempre deberemos analizar y tratar de comprender la situación dada. Esto pide
atributos que no podemos aprender en un libro, com<> la inspiración o la creatividad. El
solo he.cho de aplicar las fórmulas o utilizar las reglas no nos permitirá avanzar en la
demostración de resultados (como es el caso de losteorcmas) ni resolver problemas de
enumeración.

2. 1
Conectivas básicas
y tablas de verdad

En el desarrollo de cualquier teoría, se hacen afirmaciones en forma de oraciones. Tales


afirmaciones. llamadas enunciados (o proposicionu), son oraciones declarativas que son
verdaderas o falsas (pero no ambas). Por ejemplo, las siguientes son proposiciones y, para
representarlas, utilizaremos las letras minúsculas (como p, q y r).

51
52 Capitulo 2 Fundamentos de lógica

p: La combinatoria es un curso obligatorio para el segundo año


de bachillerato.
q: Margaret Mitcbcll escribió lo que el viento se /kvó.
r: 2+3=5.
Por otro lado. no consideramos como proposiciones algo como la exclamación
¡Qué bonita tarde!
o el mandatot
Levántate y haz tus ejercicios.
Las proposiciones anteriores, representadas por las letras p, q y ,; se consideran propo-
sicíones primitivas, ya que no hay forma de descomponerlas en algo más sencillo. Es
posible obtener nuevas proposiciones, a partir de otras e xistentes, de dos maneras.
1) Transformando una proposición p en la proposición -.p. que denota su negación y
se lee como ..no p".
Para la proposición anterior p, ....pes la proposición "'La combinatoria no es un curso
obligatorio para el segundo año de bachillerato". (No consideramos la negación de una
proposición primitiva como una proposición primitiva.) ·
2) Combinando dos o mti proposiciones en una proposición compuesla mediante las
siguientes conectívas Mgícas.
a) Conjunción: La conjunción de dos proposiciones p, q se denota como p /\ q,
que se lee como "p y q'. En nuestro ejemplo, la proposición compuestap /\ q
se lee como "La combinatoria es un curso obligatorio para el segundo año de
bachillerato y Margarct Mitchell escribió Le que el viento se llev6".
b) Disyunción: La expresión p V q denota la disyunción de cualquier par de pro-
posiciones p, q y se lec como "p o q"'. Por lo tanto, "La combinatoria es un
curso obligatorio para el segundo año de bachiUerato o Margaret Mitchell es-
cribió Lo que el víento se llevó" es la traducción verbal de p V q cuando p, q
son las proposiciones ya mencionadas. Usamos la palabra ..o .. en el sentido
ínclusívo. En consecuencia. p V q es verdadero si una o la otra o ambas propo-
siciones p, q son verdaderas. En español, a veces se escribe ..y/o" para subrayar
se
este he.cho. La ""o" exclusiva denota como pyq. La proposición compuesta
pY..q es verdadera si una u otra, pero no ambas proposiciones son verdaderas.
Una f01ma de expresar p Y..q para este ejemplo es "La combinatoria es un curso
obligatorio para el segundo año de bachiUcrato o Margaret Mitchell escribió
Lo que el viento se llevó, pero no ambos".
e) Implicación: Decimos que "p implica q" y escribimos p---+ q para designar la
proposición que es la ímplicación de q por p. En forma alternativa, podemos
decir (i) Si p, entonces q; (ii) pes suficíente para q; (iii) pes una condicíón
suficiente para q; (iv) p sólo si q; (v) q es necesario para p; y (vi) q es una
condici.én necesaria para p. Una.traducción verbal de p---+ q usando nuestro
ejemplo es ..Si la combinatoria es un curso obligatorio para el segundo año de
bachillerato, entonces Margaret Mitchell escribió Lcque el viento se llevó''. La

t El término commandtambitn se traduc:c como ~comando". (N lkl E.)


2.1 Conec!Nas básicas y tibias de ve<dad S3

proposición p se conoce como la hipótesis de la implicación, y q como la con-


c/usi6TL Cuando se combinan las proposiciones de esta forma. no es necesario
que haya una relación causal entre las proposiciones para que la implicación
sea verdadera.
d ) Bicondicional: Por 6Jtimo, la bicondicional de dos proposiciones p. q se deno-
ta como p +-+ q, lo cua1 se lec: como "psi y sólo si q", o p es necesario y
04

suficiente para q". Par.a las proposiciones p, q dadas en el ejemplo, ..La


combinatoria es un curso obligatorio para el segundo año de bachillerato, si y
sólo si Margaret Mitchell escribió lo que d vitnto se llev6" tiene el significado
de p H q. A veces abreviamos "psi y sólo s i q" como ..P sii q".
En nuestro análisis posterior de lógica debemos tener presente que un enunciado como
El número x es un entero
no es una proposición ya que su valordt verdad (verdadero o falso) no puede determinar-
se si no se asigna un valor numérico ax. Si asignamos ax el valor de 7, el resultado sería
una proposición verdadera. Sin embargo, si a ;r le asignamos valores como ½, Ji
o .:r,
tendríamos una proposición falsa. En las secciones 2.4 y 2.5 de este capítulo encontrare•
mos de nuevo este tipo de situación.
En el análisis anterior, mencionamos las circunstancias Cn las cuales las proposiciones
compustas p V q, p X q se consideran verdaderas, con base en la verdad de sus compo--
nentcs p. q. Esta idea de que la verdad o falsedad de una proposición compuesta sólo
depende de los valores de verdad de sus componentes requiere un estudio más profundo.
Las tablas 2.1 y 2.2 resumen los valores de verdad de la negación de los diferentes tipos de
proposiciones compuestas, con base en los valores de verdad de sus componcn1cs. Al
construir estas tablas de verdad, hemos usado el ..O.. para falso y el ''I" para verdadero.

,:t. ,,. .. ,,...


tabla 2.2

,, • ""• ,v.
o o o o o 1 1
o 1 o 1 1 1 o
1 o o 1 1 o o
1 1 1 1 o 1 1

Las cuatro asignaciones posibles de verdad para p, q pueden enumerarse en cualquier


orden. El orden particular que presentamos en este caso sen1 útil en nuestro trabajo posterior.
Vemos que las columnas de los valores de verdad de p y -.p son opuestas entre sí. La
proposición p A q es verdadera sólo cuando p, q son verdaderas, mientras que p V q es
falsa solamente cuando las dos proposiciones componentes son falsas. Como ya habíamos
observado, p X q es verdadera exactamente cuando una de las dos es verdadera
Para la implicación p ➔ q, el resultado es verdadero en tcxlos los casos, excepto cuandop es
verdadero y q es falso. No queremos que una p-opooicióo venladera nos conduzca a pensar en
algo que es falso. Sin embargo, consideramos oomo verdaderu las proposiciooes "Si 2 + 3 =6,
entonces 2 + 4 = r, aunque las proposiciones ''2 + 3 =6" y º2 + 4 = T' sean ambas falsas.
Por último, la bicondicionaJ p H q es verdadera cuando las proposiciones p. q tieoen el
mismo valor de verdad. y falsa en los demás casos.
54 capítulo 2 Ftindamentos de lógica

Ahora que hemos presentado algunos conceptos, estudiaremos con más detalle estas
ideas iniciales acerca de las conectivas. Nuestros primeros dos ejemplos senln útiles para
nuestro estudio.

Sean s, t y u las siguíentes proposiciones prim,jtivas:


s: Felipe sale a dar un paseo.
r. La luna eslá brillando.
u: Está nevando.

Las siguientes oraciones ofrecen algunas traducciones posibles para las proposiciones
compuestas simbólicas dadas.

a ) (t I\ -,u ) ➔ s: Si la lunaestábriJlando y no está nevando,cntonccs Felipe sale a dar


un paseo.
b) t ➔ (-.u ➔ s): Si la luna está brillando, entonces si no está nevando, Felípe sale a
dar un paseo. [Así,...,,, ➔ s se entiende como (...,u) ➔ s y no como ...,(u ➔ s).)
e) ~ (s H (u V 1)): No ocurre que Felipe salga a dar un paseo si y sólo si eslá nevando
o la luna está brillando.

Ahora trabajaremos en orden inverso y examinaremos la notación lógica (o simbólica)


para las siguientes tres frases:

d) "Felipe saldrá a dar un paseo si y sólo si la lunaeslá brillando". Aquí, las palabras
"si y sólo si" indican que estamos trabajando con una bicondiciona1. En forma sim-
bólica. esto se convierte en s H l .
e) "Si está nevando y la luna no está brillando, entonces FcÍipe no saldrá a dar un
paseo... Esta proposición compuesta es una implicación, en la que la hipótesis es a
su vez una proposición compuesta. Podríamos expresar esta pr<>posición en forma
simbólica como (w A ....,) ➔ -.s.
O "'Está nevando pero, aun asf, Felipe saldrá a dar un paseo... Ahora nos encontramos
con una nueva conectiva: pero. En nucstrO estudio de la lógica. seguiremos la con-
vención de que las conectivas pero e y tienen el mismo significado. En consecuen-
cia, esta frase puede representarse como u A s.

Ahora regresemos a los resultados de la tabla 2.2; en panicular, la sexta columna. Por-
que si es ~sea la primera vez que el lector se encuentra con la tabla de verdad de la implicación
p ➔ q, entonces podría ser dificil que acepte los datos de dicha tabla, particularmente los
resultados de los dos primeros casos (donde p tiene el valor de verdad 0). El siguiente
ejemplo puede ayudarle a comprender y aceptar estas asignaciones de vaJores de verdad.

Consideremos la siguiente situación. Estamos a casi una semana antes de Navidad y


Penélope irá a varias fiestas en esta semana. Como está preocupada por su peso, ha decidi-
do no pesarse hasta el día desputs de Navidad. Pensando en las consecuencias que esas
2.1 Conectivas básicas y tablas de verdad 55

fiestas podrían tener con su figura. ha decidido lo siguiente para el día 26: "Si peso más de
60 kilogramos, e ntonces me inscribiré en una clase de educación física".
· Aquí haremos que p y q denoten las proposiciones (primitivas)

p: Peso más de 60 kg.


q: Me inscribiré en la clase de educación física.

Entonces la posición de Penélope (una implicación) está dada por p ➔ q.


Consideraremos los valores de verdad de este ejemplo particular de p -+ q respecto a
las filas de la tabla 2.2; en primer lugar. los casos más sencillos, las filas 4 y 3.

• Fila 4: p y q tiene el vaJor de verdad l. El 26 de diciembre, Penélope pesa más de 60


kilogramos y se inscribe rápidamente en la clase de educación física, justo como lo
había dicho. Aquí, p ➔ q es verdadera y se le asigna el valor de verdad l.
• Fila 3: p tiene el valor de verdad 1, q tiene e l valor de verdad O. El 26 de diciembre.
Penélope pesa más de 60 kilogramos, pero decide no entrar al curso de educación físi-
ca. En este caso, vemos que Penélope ha roto su promesa; en otras palabras, la implicación
p ➔ q es falsa (y tiene el valor de verdad O).

Los casos de las filas 1 y 2 podrían no coincidir inmediatamente con nuestra intuición.
pero el ejemplo nos ayudará a aceptar los resultados.

• Fila l: p y q tienen el valor de verdad O. El 26 de diciembre Penélope tiene un peso


menor o igual que 60 kilogramos y no se inscribe en el curso de educación física. Ella
no ha violado su resolución; consideramos entonces que la proposición p ➔ q es verda-
dera y le asignamos el valor de verdad l.
• Fila 2: p tiene el valor de verdad O, q tiene el valor de verdad l. En este último caso,
Penélope pesa 60 kilogramos o menos el 26 de diciembre, pero aún así se inscribe en el
curso. Es probable que pese 59 o 60 kilos y siente que esto es demasiado. O bien, es
probable que desee inscribirse porque piensa que es bueno para su salud. Sin imponar
la razón. ella no va en contra de su resolución p -+ q. Una vez más, aceptamos esta
proposición compuesta como verdadera, y le asignamos el valor de verdad 1.

Nuestro siguiente ejemplo analiza un concepto relacionado con lo anterior: la estructu-


ra de decisi6n (o sekcción) en la programación de computadores.

En las ciencias de la computación. aparecen las estructuras de decisión si-entonces y si-


entonccs-o en Jenguaj~ como BASIC y Pascal. La hipótesis pes con frecuencia una ex-
presión relacional como x > 2. Esta expresión se conviene entonces en una proposición
(lógica) que tiene el valor de verdad O o 1, según el valor de la variablex en ese.punto del
programa La conclusión q podría ser un ..enunciado ejecutable" para que el programa
tome otra dirección o para una impresión. (Así, q no es una de las proposiciones lógicas
que hemos estado analizando.) Al trabajar con ..Si p entonces q". en este contexto, el
56 C.,pítulo 2 Fundamentos de lógic.,

computador ejecuta q sólo en el caso de que p sea verdadero. Si p es falso. el computador


pasa a la siguiente instrucción en la secuencia del programa. En el caso de la eslructura de
decisión ..Si p entonces q. o r', q se ejecuta cuando p es verdadera.. y r se ejec:uta cuando
pes falsa.

Antes de continuar, una advertencia: tenga cuidado al usar los símbolos ➔ y H. La


implicación y la bicondiciona1 no son iguales, como lo muestran las últimas dos columnas
de la tabla 2.2.
Sin embargo, en el lenguaje cotidiano, con frecuencia se utiliza una implicación con la
intención real de una bicoodicionaJ. Por ejemplo, consideremos las siguientes implicaciones
que un padre dirige a su hijo.

s ➔ r. Si haces ru tarea, entonces irás al juego de béisbol.


t ➔ .t: Irás al juego de béisbol sólo si haces la tarea.

• Caso l: La implicación .r ➔ t. Cuando el padre le dice al hijo ..Si haces cu tarea.


entonces irás al juego de béisbol", intenta darle un punto de vista positivo haciendo
hincapié en la djversión de ver un juego de ~isbol. .
• Caso 2: La implicación r ➔ s. AquJ encontramos el punto de vista negativo y el padre
que advierte al hijo al decir "lrú al juego de b!isbol sólo si haces la tarea". Este padre
pone énfasis en el castigo (carencia de diversión) en que se puede incurrir.

Sin embargo, en ambos casos, el padre desea que su implicación, ya seas ➔ to t ➔ s. se


entienda como la bicondiciona1 s H t. Ya que en el primer caso, el padre da indicios del
castigo a la vez que promete un premio; en el caso 2, en el que se utíliza el castigo (tal vez
para amenazar), si el chico realmente hace la tarea, entonces definitivamente tendrá la
oportunidad de disfrutar el juego de bfübol.
En los escritos científicos, debemos hacer el máximo esfuerzo para no ser ambiguo:
cuando se da una implteación, generalmente no puede, ni debe, interpretarse como una
bicondicional. Las definiciones son una notable excepción que analizaremos en la sección
2.S.

Antes de continuar daremos un paso atrás. Al resumir el material que produjo las tablas
2.t y 2.2, tal vez no pusimos suficiente énfasis en que los resultados son ciertos para
cualquier par de proposiciones p, q, y n.o sólo para proposiciones primitivas p, q. Los
ejemplos 2.4 a 2.6 nos ayudarán a rcfonar esto.

Examinemos la tabla de verdad de la proposición compuesta: "Margare1 Mitchell escribió


Lo qiu tl viento se llevó y si 2 + 3 ~ 5, entonces la combinatoria es un curso obligatorio
para el segundo año de bachillerato". En notación simbólica, esta proposíción se escribe
comoq /\ (-ir ➔ p), donde p. q y r representan las proposiciones primitivas que introdu-
jimos al principio de esta sección. La tl.Jtima columna de Ja tabla 2.3 contiene los valores
de verdad de este resultado. Obtuvimos estos valores de verdad recurriendo al hecho de
que la conjunción de dos proposiciones es verdadera si y sólo si ambas proposiciones son
verdaderas. Esto es lo que dijimos antes, en la tabla 2.2; ahora una de nuestras proposicio-
57

ncs (la implicación -., ➔ p) es definitivamente u.na proposición compuesta y no una


primitiva. Las columnas 4, S y 6 de esta tabla muestran la forma de construir la tabla de
verdad. considerando partes más pequeñas de la proposición compuesta y usando los rc-
sullados de las tablas 2.1 y 2.2.

Tablo 2.3

p q , ,, 7 r----,.p 9 /1(,r➔ p)

o o o 1 o o
o o 1 o 1 o
o 1 o 1 o o
o 1 1 o 1 1
1 o o 1 1 o
1 o 1 o 1 o
1 1 o 1 1 1
1 1 1 o 1 1

En ]a tabla 2.4 desarrollamos las tablas de verdad de las proposiciones compuestas p V


(q /\ , )(columna 5) y (p V q) /1 r(columna 7).

'Dbl.i 2.4

p q r q l\ r p V (q l\ r) PVf 1, v,>11,
o o o o o o o
o o 1 o o o o
o 1 o o o 1 o
o 1 1 1 1 1 1
1 o o o 1 1 o
1 o 1 o 1 1 1
1 1 o o 1 1 o
1 1 1 1 1 1 1

Como los valores de verdad de las columnas S y 7 difieren (en las filas S y 7), debemos
evitar escribir una proposíción compuesta como p V q A r. Si no disponemos de los
panfotesis que indique n cuáJ de las conectivas lógicas A u V debe aplicarse primero, no
tendremos idea de si estamos trabajandoconp V (q /\ r)ocon (p V q) Ar.

El 6.ltimo ejemplo de esta sección ilustra dos tipos particulares de proJ)OSiciones.

Los resultados de las columnas 4 y 7 de la tabla 2.5 muestran que la proposiciónp ➔ (p V q)


es verdadera y que la proposiciónp A ( -.p A q) es falsa en el caso de todas las asignaciones
de valores de verdad para las proposiciones componentes p, q.
58 Capitulo 2 ftJndamentos de lógica

1:lbl• 2.5
p q ,v, ,-<,v,> .,, , p /\ 9 p /\(,p /\ q)
o o o 1 1 o o
o 1 1 1 1 1 o
1 o 1 1 o o o
1 1 1 1 o o o

Defink16n 2.1 Una proposición compuesta es una lautolog(a si es verdadera para todas las asignaciones
de valores de verdad para sus proposiciones componentes. Si una proposición compuesta
es falsa para todas estas asignaciones, entonces es una contradicción.

En este capítulo usaremos el símbolo 7 0 para denotar una tautología y el símbolo F0


para denotar una contradicción.
Podemos usar las ideas de taUtologfa e implicación para describir lo que entendemos
por un argumento válido. Esto tendrá un inte~ primordial para nosotros en la sección 2.3,
y nos ayudad a desarrollar la capacidad necesaria para demostrar teoremas matemáticos.
En general, un ar¡umentocom.ienz.acon una lista de proposiciones dadas llamadaspnmisas
y una proposición que se conoce como la conclusión del argumento. Debemos examinar
estas premisas, p 1,p2 , p 3, ••• , p,. e intentar demostrar que la conclusión q se sigue lógica-
mente de esw proposiciones dadas; es decir, intentamos demostrar que si cada una de las
premisas pi, Pi, p,. .•. , p. es una proposición verdadera. entonces la proposición q tam-
bi~n es verdadera. Una forma de hacer esto consiste en analiZM la implicación

( p,l\p, /\p,11· · •llp.)t - q,

donde la hipótesis es la conjunción de las n premisas. Si cualquiera de las premisas p 1• P2t


Ps, .. .• p. es falsa, entonces no importa el valor de verdad ele q. pues, en este caso, la
implicación (p1 /\ P2 /\ p 3 /\ • • • /\p.) ➔ q es verdadera. En consecuencia, si panimos de
las premisas p., Pi, p 3, ••• , p. (cada una con va1or de verdad 1) y vemos que en estas
circunstancias q tambii n tiene el valor 1, entonces la implicación

( p,llp,/\p, I\ ... /\p.) ➔ q

es una tauiologfa y tenemos un argum~nto válido.

:JER000S2.1 1. Determine cuiles de las si¡uiente.s oraci0De$ Jon proposiciones.


a) F.n 1990, George Bush eta el presidente de Estados Unidos:.
b) x + 3 es un entero positivo.
e) ¡Si todas las maiianas fueran tan soleadas y despejadas como 6sta!

t Ea esre momtDIO, sólo tnlbajamm con la ot"Gjvnci6a de dos proposicioms, a que debemos SCJbla- ~
lacoajancióo p l A Pl A p, I+. • • • I+. p.den proposiciones es verdadera Ji y sólo si Clldap¡. 1 Si :!ió "· ts
verdadm.. AAaliz.aremo$ con dc:Wk esta conjww::ióo ¡cneraliz:ada ca d ejemplo 4.14 de la sección 4.2.
2.1 Conectivas básicas y tablas de verdad 59

d ) Quince es un número par.


e) Si Josefina tarda en llegar a la fiesta. su primo Za.carías podña enojarse.
O ¿Qut hora es'?
g) De la corte de Moctez.uma a las playas de Tñpoli.
h) Hasta el 30 de juniode 1986. Christine Mari e Even había ganado el abierto de Francia siete
veces.
2. Identifique las proposiciones primitivas en el ejercicio l.
3. Sean p, q proposiciones primitivas para las que la implicación p --+ q es falsa. Determine los
valores de verdad de
a) p l\q b) ,pyq d) ,q-+-ip

4. Sean p, q. r, s las siguientes proposiciones: p: Termino de escribir mi programa de computa-


ción antes de la comida; q: Jugaré tenis en la tarde; r. El sol está brillando; s: La humedad es
baja. Escriba lo siguiente en forma simbólica.
a) Si el sol está brillando. jugaré tenis esta tarde.
b ) Terminar de escribir mi programa antes de la comida es nooesario para que juegue tenis esta tarde.
e) La humedad baja y el sol brillante son suficientes para que juegue tenis esta tarde.
s. Seanp, q. r las siguientes proposiciones acerca de un triánguloABC particular. p: El triángulo
ABC es isósceles; q: El triángulo ABC es equilátero; r: El triángulo ABC es equiangular. Tra-
duzca cada una de las siguientes proposiciones en una frase.en español.
a) q-+p b) -.p - -.q e) q - r
d ) p/\-,q t) r-+p
6. Determine el valor de verdad de cada un.a de las siguientes implicaciones.
a) Si 3 + 4 = 12, entonces 3 +2 = 6.
b) Si3+3=6,entonccs3+6 = 9.
e) Si3+3=6.entonccs3+4=9.
d) Si Thomas Jefferson fue el tercer presidente de Estados Unidos, entonces 2 + 3 = 5.
7 . Vuelva a escribir cada una de las siguientes proposiciones como una implicación de la forma
si-entonces.
a) La práctica diaria de su servicio es una condición suficiente para que Oaniela tenga una
buena posibilidad de ganar el torneo de tenis.
b ) Arregle mi aire acondicionado o no pagaré la renta.
e) Maria puede subir a la motocicleta de Luis sólo si usa el casco.
8. Construya una tabla de verdad para cada de las siguientes proposiciones compuestas; p. q, r
denotan proposiciones primitivas
a) -.(p y-.q) - -.p b) p-(q-r) e) (p-q)-,
d) (p-q)-(q-p) e) [pl\(p-q)J-q f ) (p/\q) -p
g) q++(-.p y-.q) b) [(p-q)/\(q-r)J-(p-r)
9. ¿Cuáles de las proposiciones compuestas del ejercicio 8 son tautologías'?
10. Verifique que [p ➔ (q ➔ r)] ➔ [(p ➔ q) ➔ (p ➔ r)] es una tautología.
11. a) ¿Cuántas filas se necesitan para la tabla de verdad de la proposición compue sta (p V -.q)+-+
[(~r As) ➔ r], donde p. q, r, sy rson proposiciones primitivas'?
b) Seanp,,pz, ... ,p. proposiciones primitivas. Scap una proposición compuesta que contie-
ne al menos una ocurrencia de cada p,, para 1 :!i i :s n (y p no contiene otra proposición
primitiva). ¿Cuántas filas se necesitan para construir la tabla de verdad de p'?
12. Determine todas las asignaciones de valores de verdad. si es que existen. para las proposiciones
prill'Utivasp, q, r, s. r que hacen que todas las siguientes proposiciones compuestas sean falsas.
a) [(p /\ q) /\r)-(svt)
b) [p /\(q/\r)J-(s Yt)
60 capitulo 2 F,,,nda mentos de lógica

13. a) Si la proposición q tiene el valor de verdad 1, determine todas las asignaciones de valores
de verdad para las proposiciones primitivas p. r y s para laJ que c:I valor de verdad de la
proposición
(q-[(-,py7'A-,,D/\[-u-(-,,/\q)]
es igual a l.
b) Responda la panc (a) si q tiene el valor de verdad O.
14. AJ inicio de cierto programa en Pascal, la variable enteran recibe el valor de 7. Deten:nine el
valor de n des~ de encontrar cada uno de los siguientes enunciados suce.sivos durante la
ejecución del programa. [En este caso, el valor de n después de la ejecución del enunciado de
la parte (a) se convierte en el valor de n para el enunciado de la parte (b), ~ hasta el
enunciado de la parte (e). La operación Oiv en Pascal devuc1ve la parte entera de un cociente;
por ejemplo, 6 Div 2 • 3, 7 Oiv 2 = 3 y 8 Oiv 3 = 2.]
a) It n>S tben n : • o+2:
b) If ((n+2=8) or (n-3•6)) then o: • 2• n+l:
e) Ir ( ( n - 3 - 16) and ( n Div 6=1)) tben n :,. n+3:
d) Ir {(n<>21 ) and (n - 7=15 )) tben n := n - 4;
t) If { (n Div 5 • 2 ) or ( n+l = 20 )) then n : = n+l;
15. Las variables enteras m y n reciben los valores de 3 y 8, respectivamente. durante la ejecución
de cierto programa en Pascal. Durante la ejecución del programa. se eocuc:ntran los siguientes
enunciados sucesivos. [Aquí. los valores de m, n desputs de la ejecución del enunciado de la
parte (a) se convierten en los valores dem, n para el enunciado de la pane (b). ~ . hasta el
enunciado de la parte (g).] ¿Cutles son los valores de m, n: despu~ de encontrar cada uno de
estos enunciados'?
a) I r n-•=5 the n n :- n - 2;
b) Ir (( 2• a a.n ) and ( nOiv ◄ • l )) t benn : • 4•• -3 1·
e) Ir (( o<8) or (• Div 2 - 2)) tben n :- 2 • •
else ■ := 2• o;
d) Ir (( ■ <20 } and { n Div 6=1)) the n • := • -n- 5;
e) Ir (( a. • 2 • • ) or ( n Div 2 = 5 )) then ■ : = • +2;
f) Ir ( ( D Oh 3 -3) and I• Oiv 3 <> l )) then ■ :• n;
a) Ir ■ •n <> 35 then o :• 3• ■ +7;
16. En el siguiente segmento de un programa en Pascal, i,j, m y n son variables enteras. El usuario
proporciona los valores de m y n en una parte anterior de la ejecución (total) del programa.

For 1 : - l to • do
ForJ:=ltoado
Ir i <> j then
Writeln ( ' The su or i and J is • , i + J ):

¿CUántas veces aparece el enunciado Writeln en el segmento ejecutado cuando (a) m := l O, n := l O:


(b) m = 20.n = 20; (e) m = 10, n = 20; (d) m = 20, n = 10?
17. Para el siguiente programa en BASIC. ¿cuántas veces se ejecuta la proposición PRINT de la
lfnea40?

10 x - 10
20 FOR I • l TO 7
30 F0RJ - 1TOI+3
40 IF ((X>8 ) OR 11 > 5 AliD J<lO )) n®I PRIHT X
50 NEXT J
60 x- x- 1
70 NElT I
80 EN!)
de la lógica 61

1 a. Un segmento de un programa co Pascal cootieoe un ciclo Repeat-Unti.1 estructurado de la


forma siguiente:
Repeat

UntU (( x <> O) aad ( 7>0)) or (aot ((•>O) a.cd (t • J)));

Eo el caso de cada una de las siguientes asignaciones para las variablesx. y, w y,. detennine si
termina O DO el ciclo.
a) z :a 7,y•2, w=S,t • 3 b) x•O,y - 2,w•-3.t=3
e) z=O,y•-1,w=-=1,t•3 4) z •l,y• - 1,w•1,t=3
19. Dcspu& de bornear un pastel para sus dos sobrin~ y dos sobrinos que vienen a visitarla. la tía
Natalia deja el pastel en la mesa de la cocina para que se enfríe. Luego, ella va al centro
comercial para cerrar su tienda durante eJ resto del d!a. Al n:gresar. descubre que alguien se ha
comido una cuarta parte del pastel (e incluso tuvo el descaro de dejar su plato sucio junto al
resto del pa.stel). Puesto que nadie estuvo co su casa ese dfa (exoeptO los cuatro visitantes), la
tía Natalia se prcpnta aá1 de sus sobrinos se comcña esa parte del pastel. Los cuauo "sospe-
chosos" le dicen lo si¡uiente:
Carlos: Jimena se comjó d troz.o de pastel.
Delia: Yo no me lo comí.
Jimena: To6o se lo comió.
Tollo: Jimcna mintió cuando dijo que yo me babfa comido el pastel

Si sólo una de csw proposiciones es verdadera y sólo uno de ellos cometió el terrible
crimen. ¿quiál es el culpable al que la tía Natalia debe castigar severamente?

2.2
Equivalencia lógica:
Las leyes de la lógica

En todas las áreas de las matemáticas, necesicamos saber cuándo las entidades que estudia-
mos son iguales o esencialmente las mismas. Por ejemplo, en aritmética y á.l¡ebra sabe-
mos que dos nWlleros reales distintos de cero son iguales cuando tienen 1a misma magni-
tud y signo algebraico. Por lo tanto, ~ dos mimeros reales cuaJesquiera.r. y, distintos de
cero, tenemos que z • y si Ix 1:s Iy I y xy > O, y viceversa (es decir, si x = y, cn.tooccs
1 1; 1,1
x y .ry > O). CUando analizamos tri.ángulos en geometría. swgeel concep10 de
congruencia. En esle caso, el triángulo ABC y el tri.ángulo DEF son congruen1ts s~ por
ejemplo, sus lados correspondientes son iguales (e• decir, la longitud del ladot\B es igual
a la longilUddel lado DE, la longilUddel ladoBCesigual a la del ladoEF. y la del ladoOI
es igual a la del lado FD).
Nuestro estudio de la lógica se conoce con frecuencia como 6lgtbra tk proposiciones
(en oposición al álgebra de los ntlmeros reales). Aquí utilizaremos las tablas de verdad de
los enunciados, o proposiciones, para desarrollar una idea acerca de cuándo las dos entida-
des son esencialmente Ja misma. Comencemos con un ejemplo.

Para las proposiciones primitivasp y q, latabla2.6propon:iona las tablas de verdad delas ·


proposiciones compuestas -.p V q y p-+ q. Aquí vemos que las tablas de verdad corres-
pondientes de las dos proposiciones ....,p V q y p -+ q son exactamente iguales.
62 capítulo 2 Fundament~ de lógica

labia 2.6
p q .,, 1 PVf ,-,
o o 1 1 1
o 1 1 1 1
1 o o o o
1 1 o 1 1

Esta situación nos lleva a la idea siguiente.

Definición 2.2 Dos pro¡x,siciones s 1, s 2 son /6gicamente equivalentes, y escribirnos s 1 <=> s 2, cuando 1a
proposición s1 es verdadera (respectivamente, falsa) si y sólo si la proposición s 2 es venia•
dera (respectivamente. falsa).

Observe que cuando J" 1 <=> s2 , las proposiciones s 1 y s1 dan lugar a las mismas tablas de
verdad ya que s1, s2 tienen los mismos valores de verdad para todas las opciones de valores
de verdad de sus componentes primitivas.
Como resultado de este concepto, vemos que podernos expresar la conectiva para la
implicación (de proposiciones primitivas) en términos de la negación y la disyunción; es
decir, (p ➔ q) (:::) -.p V q. De la misma manera, el resultado de la tabla 2. 7 indica que
(p t t q) <=> (p ➔ q) /\ (q ➔ p); esto ayuda a validar el uso del término bicondiciona/. Si
usamos la equivalencia lógica de la tabla2.6. vemos que también podemosescribir(p H q) <:=)
( -.p V q) /\ (-.q V p ).Enconse.cuencia,siasíloqueremos,podemoseliminarlas conectivas
➔ y H de las proposiciones compuestas.
Si examinamos la tabla 2.8, tenemos que la negación y las conectivas /\ y V son todo
lo que necesitamos para reemplazar la conectiva o exclusiva, y. De hecho, podríamos
incluso eliminar una de las. conectivas/\ y V. Sin embargo, para las aplicaciones relaciona-
das con este tema y que queremos estudiar más adelante en este texto, necesitaremos tanto
/\ y V como la negación.

tabla 2.7

p q ,-, q-p (p - 9)/1(9 -p) p+. q


o o 1 1 1 1
o 1 1 o o o
1 o o 1 o o
1 1 1 1 1 1

labia 2.8

p q pYq PV 9 p l\q -i(p /\ q) (p Vq)/1-.(pllf)


o o o o o 1 o
o 1 1 1 o 1 1
1 o 1 1 o 1 1
1 1 o 1 1 o o
2.2 Equivalencia lógica: Las leyes de la lógica 63

Ahora usaremos la idea de equivalencia lógica para examinar algunas de las propieda-
des importantes que se cumplen en el flgebra de las proposiciones.
Para cualquier par de números <eales a, b, sabemos que -(a+ b) ~ (-a)+ (-b). ¿Existe
un resultado similar para las proposiciooes primitivas p, q?

En la tabla 2.9 hemos construido las tablas de verdad para las proposiciones ~ ( p /1 q),
~p V ~q, ~(p V q) y ~p /\ ~q, donde p, q son proposiciooes primitivas. Las columnas

"Dbl.a 2.9

, 7(p /\ 9) ,, ,, 7JIV7f ,v, 7(p V f) , , A,9


o o
9
'"'o 1 1 1 1 o 1 1
o 1 o 1 1 o 1 1 o o
1 o o 1 o 1 1 1 o o
1 1 1 o o o o 1 o o
4y 7 muestr3Jlque ~ (p /\ q)""' ~p V ~q; las columnas 9 y IOmuesttanque - (p V q) c:,
-,p /\ -,q. Estos resultados se conocen como las Leyes de De Morgan. Son similares a la
conocida propiedad conespond.iente de los nWD.eros reales,

- (a +b)- (-a)+ (-b),

ya comentada. que muestra que el negativo de una suma es igual a la suma de los negati-
vos. Sin embargo, aquí surge u.na diferencia crucial: la negación de la conjunci6n de dos
proposiciones primitivas p, q produce la disyunción de sus negaciones -.p, -.q, mientras
que la negación de la disyunción de las mismas proposiciones p, q es lógicamente equiva-
lente a la conju.nci.ón de las negaciones -.p.....q.

Aunque en el ejemplo anterior p, q eran proposiciones primitivas, pronto veremos que


las leyes de De Morgan son válidas para cualquier par de proposiciones.

En la aritm~tica de los m1meros reales, las operaciones de suma y multiplicación están


relacionadas por Ja llamada propiedad distributiva de la multiplicación sobre la suma; si
a. b, e son números reales,

a X (b +c) - (a xb)+(a xc).

EJ siguiente ejemp1o muestra una propiedad similar para las proposiciones primitivas.
llunbitn existe otra ley relacionada con esto (para las proposiciooes primitivas) que oo.
tiene su contrapartida en la aritmttica de )os números reales.

La tabla 2.10 contiene las tablas de verdad de las proposiciones p I\ (q V r), (p I\ q) V


(p /\ r). p V (q /\ r) y (p V q) /1 (p V r). De la tabla se sigue que para p, q y r
proposiciones primitivas,
4 Capitulo 2 Fundamentos de lógica

p /\(q v,)e(p /\q)V(p /\,) La propiedad distributiva de A sobre V


PV(q /\r)e(pVq)/\(pVr) La propiedad distributiva de V sobre A
tobla 2.10
p 9 , p /\(9Vr) (p /\9)V(p/\r) P V(9 /\r) (p V q) /\(¡,Vr)
o o o o o o o
o o 1 o o o o
o l o o o o o
o l l o o 1 l
l o o o o 1 l
l o 1 l 1 1 1
1 1 o l 1 1 1
1 1 1 l J 1 1

La segunda ley distributiva no tiene su contrapartida en la aritmética de los números reales.


Es decir, noescienoqueparatodoslosntlmerosrealesa,byc,a+(b x e)= (a+b) x (a +e).
Por ejemplo, para a = 2,b = 3 y e= 5, a+ (b X e)= 17 pero(a.+ b) X (a+ e)= 35.
Antes de proseguir, observemos que, en genera1, si Si, s 2 son proposkiones y s 1 H s2 es
una tautolo~ entoncess1 y s 2 deben tener los mismos valores de verdad correspondientes
(es decir, para cada asignación de valores de verdad a las proposiciones primitivas en s 1 y
s2, s 1 es verdadera si y sólo si s 2 es verdadera y s 1 es falsa si y sólo si s2 es falsa) y s 1 ~ s 2•
Cuando s1 y s 2 son proposiciones lógicamente equivalentes (es decir, s1 ~ s 2), entonces la
proposición compuesta s 1 +-+ s 2 es una tautología. En este caso, también es cierto que
-.s, ~ -.s2 y -.s, +-+ -.s1 son tautologías.
Sisi, s 2 y s 3 son proposiciones tales quc s 1 ~s2 y s 2 -C:> s3 , entoncess1 <:> S3,. Cuandodos
proposiciones s 1 y -'2 no son lógicamente equivalentes, podemos escribir s 1 <;p s2 para de-
signar esta situación.
Con los conceptos de equivalencia lógica, tautología y contradicción, enunciamos la
siguiente lista de propiedades del álgebra de proposiciones.

... ..,_dela l6gka


,.._ nMsqai,a pn,po.iciow . . - - -p. q. r,c:mlrpiorllalalogfa T0 y .........
eoma6,cci6a F•
1) -np~p
2) ,(pVq)~,p/\,q
,(p/\q)enpv--v¡
3) p y q~qVp I.e,a.-.ec--ño~a
_p /\q~q /\p
4) P V (qVr'J~(pVq)V rt
p /\(q/\r'je(pl\q)/\ r

t Observemos que., debido a las leyes MOCiativas, no hay ambigüedad en las proposiciones de la fonna
pVqVrop/\q/\r.
2.2 Equivalencia lógica: Las leyes de la lógica 65

5) py(qAr)e:,(pVq)A(pV,)
pA(qyr)e:,(pAq)y(pA r)
61 PVP""P
pAp,::,p
1-•if~,,.,.-..
7) pyli,e:,p ,_
p A1",,,::,p
1) p y,p,::,¡¡.
pA,pe:,F0
,, py¡¡.,::,¡¡.
pAF,,e:,F,,
lt) PV(p A q)e:,p
p A (pVq)e:,p

Ahora vamos a demostrar todas esw propiedades. Al hacerlo nos daremos cuenta de
que simplemente podríamos construir las tablas de verdad y comparar los resultados de los
valores de verdad correspondientes en cada caso, como Jo b.icimos en los ejemplos 2.8 y
2.9. Sin emba!¡o, antes de comenzar a escribir, revisemos de nuevo esta lista de 19 leyes,
las cuales, excepto por la ley de la doble ne¡:ación, se agrupan naruralmcntc por pares.
Esta idea del apareamiento nos ayudará des~ de analizar el siguiente concepto.

Definkión 2 .3 Seas una proposición. Sis no contiene conectivas lógicas distintas de /\ y V, entonces el
dual des, que se denota comos', es la proposición que se obtiene de s al reemplazar cada
ocurrencia de A y V con V e A. respectivamente, y cada ocwrencia de 1 0 y f o con F0 y
To, respectivamente.

Si p es una proposición primitiva., entonces p' es igual a p; es decir, el dual de una


proposición primitiva es simplemente la misma proposición primitiva Y (~p}' es igual a
-.p. Las proposiciones p V -.p y p A -..p son duales una de otta cuando pes primitiva;
por lo tanto, tambi~n lo son las proposiciones p V T0 y p A Fo- •
Dadas las proposiciones primitivas p, q, r y la proposición compuesta
s: (p/1,q)V(r ATo),
vemos que el dual de s es
s': (p y ,q)/\(rvli,).
(Observe que - q no cambia al pasar des as'.)
Ahora estableceremos y utilizaremos un te0rema sin demostrarlo. Sin embargo, en el
capítulo 15 justific=mos el resultado que apa=e aquí.

TEOREMA 2 .1 (El principio de dualidad) Sean s y t proposiciones como las descritas en la definición 2.3.
Sis~,. entonces si~ t'.

Como resultado. las propiedades 2 a 10 de nuestra lista pueden ser establecidas mediante
la demostración de una de las propiedades de cada par y recurriendo luego a este principio.
C.apítulo 2 Fundamentos de lógica

También vemos que es posible obtener muchas otras equivalencias lógicas. Por ejem-
plo, si q, r, s son proposiciones primitivas. los resultados de las columnas 5 y 7 de la tabla
2.11 nos muestran que
(r/\s)➔ q~7(r/\s)Vq
o que [(r A s) ➔ q] H [--(r A s) V q] es una tautología. Sin embargo. en vez de construir
cada vez más tablas de verdad (y, por desgracia. cada vez más grandes) sería buena idea

Tabla 2.11

r r /\ s (r/\ s)➔f 7 (r/\,) 7 (r/\s)Vf


f
o
'o o
o 1 1 1
o o 1 o 1 1 1
o 1 o o 1 1 1
o 1 1 1 o o o
1 o o o 1 1 1
1 o 1 o 1 1 1
1 1 o o 1 1 1
1 1 1 1 1 o 1

recordar el ejemplo 2.7, en el cual se establece que para q, p proposiciones primitivas, la


proposición compuesta
(p ➔ q)<->(,pVq)

es una tautología. Si ru mpla7Jl}'Ms cada ocurrencia de esta proposición primitiva p por la


proposición compuesta r /\ s, entonces obtendóamos la tautología anterior
[(r /\s)➔ q]<->b(r /\s)vq].
Lo que ha ocurrido aquí muestra la primera de las siguientes -reglas de sustitución:
1) Suponga que la proposición compuesta Pes una tautología Si pes una proposición
primitiva que aparece en P y reemplazamos cada ocurrencia dep por la misma propo-
sición q, e ntonces la proposición compuesta resultante P 1 también es una tautología.
2) Sea Puna proposición compuesta donde p es una proposición arbiuaria que apare-
ce e n P, y sea q una proposición tal que q <=) p. Supongamos que en P reemplaza-
mos una o más ocurrencias de p por q. Entonces este reemplazo produce la propO-
sición compuesta P 1• En ese caso. P1 ~ P.
Estas reglas se ilustran en los siguientes dos ejemplos.

a) De la primera de las leyes de De Morgan, sabemos que para cualesquiera proposi-


ciones primitivas p , q, la proposición compuesta
P: 7(pVq)<+(7p/\7 q)
es una tautología. Cuando reemplazamos cada ocurrenc ia de p por r A s , se sigue, a
partir de la primera regla de sustitución, que

P,: 7 [(r/\s)v q]<+(7(r /\s)/\7q]


2.2 Equivalencia lógi<,: Las leyes de la lógica 67

tambié n es una tautología. Si extendemos un poco más este resultado, podemos


reemplazar esta ocurrencia de q por t ➔ u. La misma regla de sustitución pnxlucc
ahora la tautología
P,: -,[(r /\s) v (1 .... u)]++[-,(r /\s) /\-,(1-+ u)],
y, por lo tanto, por las observaciones que aparecen un poco dcsputs del ejemplo
2 .9, la equivalencia lógica
-,[(r /\s)v (1--+ u))~b(r /\s) /\-,(1-+ u)].
b) la segunda ley de dominación indica que, para cualquier proposición primitiva p. la
proposición compuesta
P'. (p /\ Fo)++ Fo
es una tautología. Si reemplazamos p por la proposición ((q V r) ➔ s], entonces la
misma primera regla de sustitución produce la nueva tautología

P,: (I(q V r) -+s]/\F0) ++F0 ,


y por lo tanto la equivalencia lógica
[(q V r)-+s] /\ F,~ F,.
e) Para las proposiciones priÍn.itivasp, q, vimos en la última columna de la tabla 2. 12
que la proposición compuesta [p /\ (p ➔ q)} ➔ q es una tautología. En consecuen-
cia. si r, s, 1, u son cualesquiera proposiciones arbitrarias, entonces, por la primera
regla de sustitución obtenemos la nueva tautología
[(r-+ s) /\ [(, .... s) ➔ (-,1 V u )D.... (-,1 V u)
cuando r=nplazamos cada aporicióo dep por r ➔ s y cada aparición de q por ~ 1 V u.

TObl• 2.12

p q ,- , p /\( p - q) 1,/\(p-flJ-,
o o 1 o 1
o 1 1 o 1
1 o o o 1
1 1 1 1 1

a) Como aplicación de Ja segunda regla de sustitución. sea P Ja proposición compues~


· ta (p ➔ q) ➔ r. Como ( p ➔ q) <=> --p V q (como se muestra en el ejemplo 2.7 y en
la tabla2.6).siP 1 denoca la proposicióncompuesta(--p V q) ➔ r,cntonces P 1 <=> P.
(Tombitn 1<nemos que [(p ➔ q) ➔ r] +-+ [(~p V q) -+ r] es una lautología.)
b) AhoraseaPlaproposición compues1a (enrealidad, esuna1autología)p ➔ (p V q).
Como -.--.pe:> p, la proposición compuestaP1:p ➔ c-.--.p V q) se obtiene de Pal
reemplazar únicmtuntt la segunda aparici6n (pero no la primera) de p por --. -.p. La
sc¡undarcglade sustitución todavíaimplicaqucP1 <=> P. (Observe que P 2: -.-.p ➔
Capítulo 2 Fundamentos de lógica

(-o-.p V q), obtenida al reemplazar ambas apariciones de p por -.-.p, también es


lógicamente equivalente a P.]

Nuestro siguiente ejemplo demuestra cómo podemos utilizar la idea de la equivalencia


lógica junto con las leyes de la lógica y las reglas de sustitución.

Niegue y simplifique la proposición compuesta (p V q) ➔ r.


Hemos organizado nuestra exphcación de la manera siguiente:
1) (p V q) -->r= -(p V q) V r[porlaprímeraregladesustirución, ya que (s --H) <-->
(-.s V r) es una tautología para las proposiciones primitivas s, t].
2) Al negarlasproposicionesdelpaso 1, tenemos -[(p V q)--> r]= - [- (p V q) V r].
3) A partir de la primera leydeDeMorgany laprímeraregladesustitución, - [- (p V q)
V rJ=--(p Vq)A -r.
4) La ley de la doble negación y la segunda regla de sustitución nos producen -. -.(p
V q) A -r=(p V q) A -r.

Delos pasos I a4obtenemos -[(p V q) ➔ r] =(p V q) V - r

Cuando quisimos escribir la negación de una implicación, como en el ejemplo 2.12,


vimos que el concepto de equivalencia lógica tuvo un papel central.junto con las leyes de
la lógica y las reglas de sustitución. Esta idea es tan importante que merece revisarse.

- Sean p, q las proposiciones primítivas


p: Juan va al lago George.
q: María paga las compras de Juan.
y consideremos la implicación
p ➔ q: Si Juan va al lago George, entonces María
pagará las compras de Juan.
Aquí queremos escribir la negación dep ➔ q de una forma distinta a --(p ➔ q). Quere-
mos e vitar escribir la negación como "no es cierto que si Juan va al lago Georg.e. entonces
Maria pagará las compras de Juan".
Para hacer esto. consideramos lo siguiente. Como p ➔ q ~ -op V q, se sigue que
-(p ➔ q) = -(-p V q). Entonces, por la leydeDeMorgan, tenemosque-(-p V q)=
-. -.p A -.q y, a panir de la ley de la doble negación y Ja segunda regla de sustitución, se
sigue que-_.-.p A--q<=>p A -.q.Enconsecuencia.

,(p .... q)~,(,pVq)~p/\,q,

y podemos escribir la negación de p ➔q en este caso como

-(p ➔ q): Juan va al lago George, pero Mana no


paga las compras de Juan.
2.2 Equivalencia lógica: Las leyes de la lógica 69

(Nota: La negación de una proposición si~ ntonces no comienza con la palabra si, porque
no es otra impücaci6n.)

En la definición 2.3, et dual s' de una proposición s sólo se definió para proposiciones con
negación y lascooectivas búicas V e /\ . ¿Cómoscdetermioacl dua1 de una proposición
como s: p ➔ q, donde p, q soo primitivas?
Com,,(p-->q)<a> -p V q,.r"es lógicamcniecquivalentealaproposición( - p V q)'.que
es igual a -p /1 q.

La implicación p ➔ q y cicnas proposiciones relacionadas con ella se analizan en el


siguiente ejemplo.

La tabla 2.13 da los valores de verdad para las proposiciones p ➔ q, -.q ➔ -.p, q ➔ p y
.....p ➔ .....q. La tercera y cuarta columnas de la tabla revelan que

(p-+q)~(,q-+,p).
Tobl• 2.13
, f ,-, ..,,--., ,-, -.,--,_,
o o 1 1 1 1
o 1 1 1 o o
1 o o o 1 1
1 1 1 1 1 1

La proposición -.q ➔ -ip se conoce como la contrapositiva de la implicación p ➔ q. Las


columnas 5 y 6 de la tabla muestran que
(q-+p)~(,p--+ -iq).
la proposición q ➔ p se denomina la recfproca de p ➔ q; -,P ➔ -.q se conoce como la
inversa de p --> q. A partir de la tabla 2.13 tambitn podemos ver que
(p-+q)</>(q ➔p) y (,p-+, q)</>(, q-+-ip).
En consecuencia. debemos conservar en orden la implicación y su recíproca. El hecho de
quccicrtaimplicaciónp -4 q sea vcrdadera(en panicular, como en la fila 2 de la tabla)no
exige que la n:c:íproca q ➔ p tambi~n sea verdadera. Sin embargo, sí necesita la verdad de
la contrapositiva -.q ➔ -.p.
Consideremos el caso específico donde p. q representan las proposiciones
p: Hoy es el día de acción de gracias.
q: Mañana es viernes.
Entonces obtenemos
• (La implicación: p ➔ q). Si hoy es el día de acción de gracias. entonces mañana es
viernes. (VERDADERA)
(apítulo 2 Fundament os de lógica

• (La contrapositiva: -.q ➔ -.p). Si mañana no es viernes. entonces hoy no es el día de


acción de gracias. (También es VERDADERA)
• (La recíproca: q -+ p ). Si mañana es viernes, entonces hoy es el día de acción de
gracias.
• (La inversa: -.p ➔ -.q). Si hoy noes el día de acción de gracias, entonces mañana no
es viernes.

Hay que tener cuidado con la recíproca y la inversa. Consideremos cualquierjueves del
mes de mayo. En cada uno de estos días, la proposición p es falsa pero la proposición q es
verdadera y cada una de las proposiciones q ➔ p y -.p ➔ -.q es falsa. Ahora considere-
mos cuaJquier jueves. Para cada uno de estos días, ambas proposiciones p. q son falsas,
pero las proposiciones q ➔ p y -.p ➔ -.q son verdaderas.

El siguiente ejemplo muestra que las proposiciones lógicamente equivalentes pueden


conducir a situaciones diferentes en una aplicación a las ciencias de la computación.

La tabla 2.14 revela que las proposiciones compuestas ( p A q) ➔ r yp ➔ (q ➔ r) son


lógicamente equivalentes.

Tabla 2.14
p f r p /\q (p /\ q)-r ,-, p-+(q- r )

o o o o 1 1 1
o o 1 o 1 1 1
o 1 o o 1 o 1
o 1 1 o 1 1 1
1 o o o 1 1 1
1 o 1 o 1 1 1
1 1 o 1 o o o
1 1 1 1 1 1 1

En los segmentos de programa en Pascal que se muestran en la figura 2.1, x, y, z e i son


variables enteras. La pane (a) de la figura utili:za una estructura de decisión comparable a
una proposición de la forma (p A q) ➔ r. En este caso, al igual que en laparte(b), tenemos
que p: x > O, q: y> O, que se convierten en proposiciones cuando las variables x, y toman
los valores 4 - i (parax) y 4 + 3 • i (paray). ¡Te nga cuidado! la letra rdenotael enunciado
Write/n, un "enunciado ejecutable" que no es realmente un enunciado en el sentido usual
de una frase declarativa, que puede ser calificada como verdadera o falsa.
En el sCgmento que aparece en la parte (a), el número total de comparaciones, (x > O) y
(y> O), que se realizan durante la ejecución del programa, es 10 (para x >O)+ !O (para y
> O) = 20. El segmento que apaccce en la parte (b). por otro lado. utiliza una forma de
enunciado comparable a las implicaciones anidadas p ➔ (q ➔ r). En este caso, la compa-
ración (y> O) no se ejecuta a menos que se ejecute la comparación (x > O) y sea verdadera.
En consecuencia, el número total de comparaciones es ahora de l O (para x > O) + 3 (para
y>O, cuandoi toma los valores 1, 2. 3)= 13. Por lo tanto, en términos del número total de
2.2 Equivalencia lógica: Las leyes de la lógica 71

comparaciones hechas en cada uno de estos dos casos., el segmento de programa que aparece
en la parte (b) es más eficicnle que el scgmen10 de programa que aiw- en la pane (a).

z : = 4;
For 1 : • l to 10 do
Begin
x : - z-1;
y ;e z + 3 • i;
It ( (x > O) and (y > O)) then
Wri teln (' El valor de la suma x + y es X + y)
End;

(a)

z :• -4;
For 1 : • 1 to 10 do
Bagin
x :• z - 1 ;
y:•z+3•i;
Ir x > o then
If y > O lben
Writeln (' El valor de la suma x + y es X + y)
End;

(b)
Figura 2.1

Vcmnos ahora algunos ejemplos dcsimplilicacióu de proposiciones compucsw, así como


una aplicación relacionada con esto para simplificar redes de conmutación. En este caso.
para simplificar la exposición, enumeraremos las principales leyes de la lógica que se hayan
utilizado, pero no mencionaremos la aplicación de nuestra.S dos reglas de sustitución.

Para las proposiciones primitivas p, q, ¿existe una forma más sencill~ de expresar la pro-
posicióo compucsta(p V q) 11 ~(~p II q)?;csdccir,¿podcmosenconrrarunaproposición
más sencilla que sea lógicamente equivalente a la dada'?
Aquí vemos que

(p v q) /\,(,p/\q) Razoocs
<:!>(pVq) /\(,,p v,q) Ley de De Morgan
<:!>(pVq)/\(pV-iq) Ley de la doble negación
<:!>pV(q /\,q) Ley distributiva de V sobre 11
<:!>pVFo Ley de la inversa
<:!>p Ley del neutro

En consecuencia. tenemos que

(p V q)/\,(,p /\ q)<:!>p,
Capítulo 2 Fundamentos de lógica

de modo que podemos expresar la proposición compuesta dada mediante la proposición


lógicamente equivalente, más sencilla. p.

Consideremos la proposición compuesta

7[7[(pVq)/\r] V 7 q],
donde p, q , r son proposiciones primitivas. Esta proposición contiene cuatro apariciones
de proposiciones primitivas. tres símbolos de negación y tres conectivas.
De las Jeyes de la lógica se sigue que

7[7[(pV q) /\ r]V7 q] Razones


~ 77[(pVq) /\ r]f\77q Ley de De Morgan
~ [(p V q )/\r] /\ q Ley de la doble negación
~ ( p y q) /\(r /\ q) Ley asociativa de /\
~(p y q)/\(q/\ r) Ley conmutativa de /\
~ [(p Vq)/\q] /\ r Ley asociativa de /\
~q /\ r Leyes de abS()rción (aJ iguaJ que
las leyes conmutativas de A y V)

En consecuencia, Ja proposición original

es lógicamente equivaJente a la proposición mucho más sencilla


q /\ r,
donde sólo tenemos dos proposiciones primitivas, ningún símbolo de negación y sólo una
conectiva.
Observe además que, a partir del ejemplo 2.7, tenemos
7 [(p V q) /\ r]--> 7 q ]~7[7[(p V q) /\ r]V7q],
de modo que

7[[{p V q) /\ rJ--,7q]~q /\r.

Cerramos esta sección con una aplicación de la forma en que las ideas de los ejemplos
2. 17 a 2.18 pueden utilizarse para simplificar las redes de conmutación.

~ , ~ ,,.,. . .,,. Una red de conmutación está formada por cables e interruptores que conectan dos tenni-
nales 71 y 72• En dicha red, cualquiera de los interruptores puede estar abierto (0), de mcxto
que no pase corrie nte por él, o cerrado (1 ), de modo que la corriente pueda pasar por él. -
En la figura 2.2 tenemos, en la parte (a), una red con un interruptor. Cada una de las
panes (b) y (e) conliene dos interruptores independientes entre sí.
Para la red de la parte (b), la corriente fluye de T1 a T2 si cualquiera de los interruptores
p, q está cerrado. Llamamos a esto una red en paralelo y la representamos como p V q. La
2.2 Equivalencia lógica: Las leyes de la lógica 73

-p-
r,

(a)
r,
{:}
(b)
..,..._ P -
T,

(e)
q ----e
r,

Figura 2.2

red de la parte (e) necesita que cada uno de los interruptores p. q estén cerrados para que la
corriente fluya de T1 a T2 • En este caso, los interruptores están en serie y esta red se repre-
senta como p A q.
Los interruptores de la red no tienen que ser independientes entre sí. Consideremos la
red de la figura 2.3(a). En este caso, los intem.Jptores t y -it no son independientes. Hemos
acoplado los dos interruptores de modo que testé abierto (cerrado) si y sólo si --. r está
cerrado (abierto) en forma símultánea. Lo mismo ocurre con los interruptores que están en
q, ....q. (fampoco, por ejemplo, los tres interruptores p no son independientes.)
Estaredserepre,cntarnc<liantelaproposicióooomspondiente(p V q V r) A (p V t V ~q) A
(p V ...., V r). Por medio de las leyes de la lógica simplificaremos esta proposición, que
representa la red, de la manera siguiente:

<,vqvr)/\(pVtV,q)/\(pv,rv ,)
~pV((qVr)/\(tV,q)/\(,ty r)] --
LeydislribulivadeV

O,V{(qV r)/\(,ry,)/\(t v,q)]


OpV[((q/\,r)yr)/\(tV,q)]

OpV[((q 1\-.t)Vr)/\(.,-,tV ,q)]


¡;,o ,v(((q/\-.i)vr)/\,(,t/\q)J
opvf:,(-.i/\q)/\((,t/\q)vr)J
-
"°""'"
Ley COllllllll8tiva de A
Ley dislribativa de V

"
Ley de la doble aegacíóo
Ley de De Morgaa
LeyaJllllllllalivade"
(do,..~:es)
OpV[(,(-.t/\q)l\(,r/\q))y (,(,t/\q)/\r)) Ley dislributivade A

~V[F,y(-,(,r/\q)/\r)]
"°""'V
-..r A s ~ F., para cualquier
~s
q,,v[(,(,t/\q))/\r) F1 es d neuuo para V
o,v[r/\,(,t/\q)] Ley ooamutaliva de A
OpV[r/\(tV,q)] Ley de De Morgaa y
Ley de la doble aegaci6a
Capitulo 2 Fundamentos de lógica

Figura 2.3

Porlotanto,(p V q V r) /\ (p V t V •q) /\(p V •t V r)<c>p V [r /\ (t V •q)],ylaredque


se muestra en la figura 2.3 (b) es equivalente a la red original. en el sentido de que la
corriente fluye de T 1 a T1 en la red (a) exactamente cuando hace lo mismo en ta red (b).
Pero en (b), la red sólo tiene cuatro interruptores, cinco m enos que en la red (a).

:1c10s 2.2 1. Sean p, q. r proposiciones primitivas.


a) Use las tablas de verdad para. verificar las siguientes equivalencias lógicas.

i) p-(q l\r)<'->(p ->q)l\(p-+r)


ü) [{p V q)->r]~[(p-+r)/\(q-+r)l
ili) [ p -+(q V r)]~[,r-+(p-+q)]

b) Use las reglas de sustitución para ver que [p ➔ (q V r)] ~ [(p /\ -.q) -+ rJ.
2. Verifique la primera ley de absorción mediante una tabla de verdad.
3. Use las reglas de sustitución para verificar que cada una de las siguientes pro¡x,sicioncs es una
tautología. (En este caso p. q y r son proposiciones primitivas.)

a) [p V(q /\ r)] V 7 [P V(q /\r)J


b) [(pVq)-+r] - [,r- •(pVq))
e) [I(pVq)-+r]v(s/\t)J-[II{pVq) -+r)v s)/\[I(p V q)-,)V til

4 . Para las proposiciones primitivas p. q. r y s, simplifique la proposición compuesta

[[I(p /\ q) /\ r)V[(p /\ r) /\-iril v-iq ] ->s.

5. Refute y ~prese cada una de las siguientes proposiciones en español.


a) Karina tendrá una buena educación si pone sus estudios antes que su interés en ser estrella
de cine.
b) Norma está haciendo su tarea de matemáticas y Claudia está practicando sus lecciones de
piano.
e) Si Lorenzo se va de vacaciones, entonces ti se divertirá si no le preocupa viajar en avión.
d ) Si Homero aprueba su curso de Pascal y termina su proyecto de estructura de datos. se
graduará al final del semestre.
2.2 Equivalencia lógica: Las leyes de la lógica 75

6, Refute lo siguiente y simplifique la proposición resultante.

a) p /\(q v r) /\(,pv,qv r) b) (p/\q)- r


e) p-(, q /\ r) d) p V qV(,p /\ ,q/\r)

7. a) Sip,qsonproposicion<sprimítivas,demuestreque{•p V q) t.. {p t.. {p t.. q))<=>(p t.. q).


b) Escriba el dwil de la equivalencia lógica de la parte {a).
B. Escriba el dwil de {a) q --> p, {b) p --> (q t.. r), (e) p H q y {d) p l!'. q, donde p, q y r son
proposiciones primitivas.
9. Escriba la recíproca. la inversa y la contrapasitiva de cada una de las siguientes implicaciones.
Para cada implicación, detenninc su valor de verdad. así como el valor de verdad de la recípro-
ca. la inversa y la contrapOsitiva correspondientes.
a) Si hoy es el día del trabajo. entonces mañana es martes.
b) Si-1 <3 y 3 +7 = 10,entooces (. ,) =-l.
e) Si Paco vive en Nueva Inglaterra, entonces Paco vive en VennonL
1 O. Detenninc si lo siguiente es verdadero o falso. Aquí, p. q y r son proposiciones arbitrarias.
a) Una forma equivalente para expresar la reciproca de ..p es suficiente para q., es ..p es nece-
saria paraq".
b) Una forma eqWvalente para expresar la inversa de "pes necesaria para q" es ··--.q es sufi-
ciente para --.p".
e) Una forma equivalente para expresar la contrapositiva de ·...p es ne<:esaria para q" es ··--.q es
necesaria para --.p".
d) Una forma equivalente para expresarla recíproca dep ➔ (q ➔ r)es (--.q V r) ➔ p.
11. Seanp. qy rproposiciones primitivas. EncuentreunaformadelacontrapoSitivadep --+ (q --+ r)
con (a) sólo una aparición de la conectiva ➔: (b) sin que aparezca la conectiva--+.
12. Escriba la recfproca, la inversa. la conrrapositiva y la negación de la siguiente proposición: ..Si
Sandra termina su u-abajo, entonces irá al juego de baloncesto, a menos que nie ve".
13. Considerclosdosseg;mentosde programa en Pascal de la figura 2.1 (ejemplo 2.16). Determine
el m1mero total de comparaciones ejecutadas en cada segmento si, en vez de asignarle el valor
4, z toma el valor (a) 2; {b) 6; (e) 9; (d) 10; (e) 15; (0 n, un entero mayor que 10.
14. Muestre que para p, q proposiciones primitivas,

p Yq<c>[(p/\, q)V(•P /\ q)]e>,(p .,.q~

15. Verifique que [(p +-+ q) A (q H r) /1.. (r +-+ p ) ~ ( p ➔ q) /\ (q--+ r) /\ (r--+ p)), para las
proposicione s primitivas p. q y r .
16. Para las proposiciones primitivas p. q.
a) verifiqucquep ➔ [q--+ (p /1.. q)] es una tautología.
b) verifique que (p V q) ➔ [q ➔ q] es una tautología. usando el resultado de la pane (a), las
reglas de sustitución y las leyes de la lógica.
e) ¿es (p V q) ➔ (q ➔ ( p /\ q)] una tautología?
17. Defina la conectiva ..Nand" o ..no. . y ..." como (p f q) ~ - (p /\ q). para las proposkionC5
p. q. Represente lo siguiente utilizando solamente esta conectiva.

b) pVq e) p/\q
e) p - q

18. La conectiva ..Nor" o ..No... o . .... se define para las proposiciones p. q como (p J. q) ~
- (p V q). Represente las proposiciones de las panes (a) a (e) del ejercicio 17, utilizando
solamente esta conectiva
CapítlJlo 2 Fundamentos de lógica

19. Para las proposiciones p, q. demuestre que

20. Ol: las razones para cada paso de las siguientes simplificaciones de proposiciones compuestas.
•> [(pVq)/\(pv,qlJv q ·
e>[p V(q /\,q)] V q
e>(pVFo)Vq
e>pV q
b) ,(pVq)V[(,p/\q)V,q]
.,.,(p v q) v[, qv(,p /\q)J
.,., (p v q)v [(, q v,p)/\ (,q V q)]
.,., (pVq)V[(, q v ,p)/\ To]
.,.,(pvq)v(,qv,p)
.,., (pVq)V,(q/\p)
.,.,[(p V q) /\(q /\p)]
.,., [(q/\p)/\(pVq)]
.,., [q /\[p/\(p v q)Il
.,., (q /\p)
e) (p-+q)/\[,q/\(,v, q)J
.,.(p-+q)/\, q
.,.(, p V q) /\,q
.,.,q /\ (,pvq)
.,.(,q/\,p) v (,q/\q)
.,.(,q /\,p)V F,
<:>, q/\,p
.,.,(q v p)
21 . Como en el ejercicio 20, escriba los pasos y las razones para establecer las si¡uientes equiva•
lencias lógicas.
a) P V [p /\(p V q)].,.p
b) PV q V (,p /\ ,q /\ r).,.pV q V r
<) [(, p V ,q),..(p /\ q /\ r)].,.p/\q
d) p /\ [(, q-+ (r /\ r))v,[q V((r /\s)V(r /\,s))Il.,.P
22. Simplifique cada una de las redes que aparecen en la figura 2.4.

-f"'} q p p- r - r

1f:3-.. , f .3C.}·
(a) ,r
,q

(b)
p-q

Figur.a 2.4
2.3 Implicación lógica: Reglas de inferencia 77

2 .3
Implicación lógica:
Reglas de inferencia

Al final de la sección 2.1, mencionamos el concepto de argumento válido. Ahora comen•


zaremos un estudio forma) de lo que para nosotros significa un argumento y de cuándo tal
argumento es válido. Esto, a su vez, nos ayudará cuando analicemos la demostraeión de
teoremas en el resto del texto.
Comenzaremos primero por considerar la forma general de un argumento que quera-
mos demostrar que es válido. Consideremos entonces la implicación
(p,l\p,l\p,I\·. ·l\p.)-q.
Aquí, n es un entero positivo, las proposiciones p 1, Pi, p 3 , •• • ,p,. se denominan premisas
del argumento y la proposición q es la conclusión del argumento.
El argumento anterior es válido si cada vez que las premisas p1, p,_, p3 • • • • , p. sean
verdaderas, entonces Ja conclusión q también lo es. [Observe que si alguna de las premisas
p¡, p 2, p 3, • • • • p,. es falsa, entonces la h.ipótesis p1 A P2 A p., A · · · A p,. es falsa y la
implicación (p1 A P2 A p 3 A · · · A p.) ➔ q automáticamente es verdadera, sin imponar el
valor de verdad deq.] En consecuencia, una vía para establecer la validez de un argumento
dadoesdemostrarquelaproposición(p1 AP2 Ap3 A · · · Ap,.)4-qesunatautología.
Los siguientes ejemplos ilustran este particular método para establecer la validez de un
argumento.

Sean p, q, r las proposiciones primitivas dadas como:


p: Rogelio estudia.
q: Rogelio juega tenis.
r: Rogelio aprueba matemáticas discretas.
Ahora bien. sean p1, !>z, p3 las premisas
p 1: Si Rogelio estudia, entonces aprobará matemáticas discretas.
p,: Si Rogelio no juega tenis, entonces estudiará.
p3 : Rogelio reprobó matemáticas discretas.
Queremos determinar si el argumento
(p,l\p,l\p,) -+ q
es vá1ido. Para para lograrlo, escribimos p,, 1'7., p 3 como
p ¡: p-+r />2: 7q-+p p 3: 7T
y examinamos la tabla de verdad de la implicación
[(p-+ r)l\(,q-+ p) l\-ir]...., q
que ap3(CCC en la tabla 2.15. Como la columna final de la tabla 2. 15 contiene únicamente
unos, la implicación es una tautología. Por lo tanto. podemos decir que (p1 A P2 A p3) ➔ q
es un argumento válido.
78 Capítulo 2 Fundamentos de lógica

Tabla 2.15

p, p, p, (p, /\p,/\p,)➔ q

p q r p➔ r 7 f-P .,,, l(p-r)/\(1q ---,.p)/\7 rJ-q


o o o 1 o 1 1
o o 1 1 o o 1
o 1 o 1 1 1 1
o 1 1 1 1 o 1
1 o o o 1 1 1
1 o 1 1 1 o 1
1 1 o o 1 1 1
1 1 1 1 1 o 1

Consideremos la tabla de verdad de Ja tabla 2.16. El resultado de la última columna de esta


tabla muestra que para cualesquiera proposiciones primitivas p. ,; y s, la imptícación
[ p /\ ((p /\ r)->s)]-+ (,-.·,)
es una tautología. En consecuencia, para las premisas
p 1: p p,: (p /\ r ) -+ s
y la conclusión q:(r ➔ s), sabemos que (p1 /\ p2) ➔ q es un argumento válido; podemos decir
que la verdad de la conclusión q se deduce o infiere de la verdad de las premisas p 1, p2 •

Tabla 2.16

P• P• q (p1 /\p2) - f
p r p /\ r (p l\ r)➔ s [p/\((p/\r) ➔ s)J ➔ (r➔ s)
' r➔s

o o o o 1 1 1
o o 1 o 1 1 1
o 1 o o 1 o 1
o 1 1 o 1 1 1
1 o o o 1 1 1
1 o 1 o 1 1 1
1 1 o 1 o o 1
1 1 1 1 1 1 1

La idea presentada en los dos ejemplos anteriores lleva a la siguiente definición.

Defin ición 2.4 Sip, q son proposiciones arbitrarias tal quep ➔ q es una tautología, entonces decimos que
p implica lógicamente q y escribimos p ;;;;;> q para denotar esta situación.
2.3 Implicación lógica: Reglas d• inferencia 79

Cuando p, q son proposiciones arbitrarias y p =), q, la implicación p ➔ q es una tauto-


logía y decimos qucp-+ q es una implicación lógica. Observe que podemos evitar la idea
de tautología diciendo quep ~ q (es decir, que p implica lógicamente q) si q es verdadera
cuando p es verdadera.
En el ejemplo 2.6 vimos que para las proposiciones prim.itivasp, q, la implicaciónp ➔
(p V q) es una tautología En este caso, p:xlemos decirquep implica J6gicamentcp V q, y
escribirp ⇒ (p V q).Además, por la primera regla de sustitución, tambi6n tenemos que p
= (p V q) para proposiciones arl>itrariasp, q (es dccir,p ➔ (p V q) es una 1au1ología para
cualquier par de proposiciones p, q, sin importar si son primitivas o no).
Sean p, q proposiciones arbitrarias.
1) Si p <=) q, entonces la proposición p H q es una tautología, por lo que las proposicio-
nes p, q tienen los mismos valores de verdad (correspondientes). En estas condiciones.
las proposiciones p ➔ q, q ➔ p son tautologías y tenemos que p => q y q ⇒ p.
2) A la inversa. supongamos que p ⇒ q y q ⇒ p. La implicación lógica p ➔ q indica
que nunca tendremos una proposición p con el valor de verdad 1 y una proposición
q con el valor de verdad O. Pero ¿podremos tener q con et valor de verdad 1 y p con
el valor de verdad O? Si esto ocurre, no podremos tener la implicación lógica q ➔ p.
Por lo tanto, cuando p ~ q y q ~ p, Jas proposiciones p. q tienen tos mismos
valores de verdad (comspoodienlCS) y p ~ q.
Fmalmente, usamos la nOlaciónp -:pq para indicarquep ➔ q no es una tautología; así, la
implicación dada (p ➔ q) no es una implicación lógica.

A partir los resullados del ejemplo 2.8 (Tabla 2.9) y la primera regla de sustitución. sabe-
mosque para cualesquiera proposiciones p , q,
7(p /\ q)<">7pV7q.
En consecuencia.

para cualesquiera proposiciooesp, q. Alternativamente. como cada una de las implicaciones

es una tautología. podemos escribir también

[ 7(p /\ q)➔ (7pV7q)]<"> To [(7p V7q)-+7(p /\ q)l<"> To-

Sigamos analizando las t6cnicas para establecer la validez de un aigumento. Debemos


observar con cuidado el tamal!io de las tablas 2.15 y 2.16. cada tabla tiene ocho filas. En la
tabla 2.15 pudimos expresar las tres premisas p 1, />2 y p,. y la conclusión q. en términos de
las ttC;S proposiciones primitivas p, q y r. Algo similar ocurre con el argumento analizado
en la tabla 2.16, donde teníamos sólo dos premisas. Pero si, por ejemplo, quisiéramos
tratar de establecer que
[(p ➔ r) /\ (r-+ s) /\(I V 7S)/\(71 V u) f\7u]➔ 7p
es una implicación lógica (o que representa un argumento vá]jdo), la tabla necesaria re-
queriría 2s = 32 filas. Como el número de premisas crece y nuestras tablas de verdad llegan
80 Capítulo 2 Fundamentos de lógica

a te ner hasta 64, 128, 256, o más filas, esta primera técnica para establecer la validez de un
argumento pierde rápidamente su encanto.
Además, si observamos de nuevo la tabla 2.15, nos damos cuenta de que para estable-
cer que

es un argumento válído, necesitamos considerar sólo aquellas filas de la tabla donde cada
una de las tres premisas p ➔ q. -.q -4 p y -ir tenga el valor de verdad 1. (Recuerde que si
la hipótesis, formada por la conjunción de todas las premisas, es falsa, entonces la
implicación es verdadera. sin importar el valor de verdad de la conclusión.) Esto sucede
únicamente en la tercera fila, por lo que en realidad no necesitamos gran parte de la tabla
2.15. (No siempre ocurre que una sola fila tenga todas las premisas verdaderas. Observe
que en ]a tabla 2.16 nos interesarían los resultados de 1as fiJas 5, 6 y 8.)
En consecuencia, lo que indican estas observaciones es q ue podríamos prescindir de
gran parte del esfuerzo de construcción de las tablas de verdad que aparecen en las tablas
2.15 y 2.16. Y como no queremos hacer tablas aún más grandes, desarrollaremos una lista
de técnicas llamadas reglas de inferencia que nos ayudarán de la siguiente forma:

1) El uso de estas técnicas nos pennitirá considerar únicamente los casos en que todas
las premisas sean verdaderas. Por lo tanto, analizaremos la conclusión sólo para
aquellas filas de la tabla de verdad donde cada premisa tenga el valor verdadero l.
sin construir dicha tabla de verdad.
2) Las reglas de inferencia son fundamentales en el desarrollo de una justificación
paso por paso de cómo la conclusión q se sigue lógicamente de las premisas p 1• pi,
p 3, ••• , p ... en una implicación de la forma

Dicho desarrollo establecerá la validez del argumento dado, pues mostrará la forma
de deducir la verdad de la conclusión a panir de la verdad de las premisas.

Cada regla de inferencia tiene su origen en una implicación lógica. En algunos casos. la
implicación lógica se establece sin demostración. (Sin embargo, se presentarán varias de-
mostraciones en la sección de ejercicios. )
En el estudio de la lógica surgen muchas reglas de inferencia. Nos concentraremos en
las que nos permiten justificar los argumentos que surgen en nuestro estudio de la lógica.
Estas reglas también nos ayudarán más adelante, cuando estudiemos los métodos para la
demostración de teoremas en el resto del texto. La tabla 2.20 (de la página 88) resume las
regJas que empezaremos a analizar.

Como primer ejemplo, consideremos la regla de inferencia llamada Modus Ponens , o re-
gla de separación. (Modus Ponens viene del latín y puede traducirse como "el método de
afirmación".) En fonna simbólica. podemos e,r;presar esta regla mediante la implicación
lógica

[pl\(p ➔ q)]--> q,
2.3 Implicación lógica: Reglas de inferencia 81

que verificamos en la tabla 2 .17, donde vemos que la cuana fila e s la l'.inica donde ambas
premisas p y p 4 q (y la conclusión q) son verdaderas.

Tabla 2.17
p q p-q p /\ ( p-q) [p /\ ( p-q))-q
o o 1 o 1
o 1 1 o 1
1 o o o 1
1 1 1 1 1

Escribimos la regla en fonna de tabla

p
~
:. q

donde los tres puntos ( .·.) representan las palabras ..por lo tanto" , e indican que q es la
conclusión de las premisas p y p ➔ q, las cuales aparecen ¡x>r encima de la línea horizontal.
Esta regla surge cuando argumentamos que si ( l ) p es verdadera y (2) p ➔ q es verda-
dera (o p ~ q), entonces la conclusión q también debe ser verdadera. (Después de todo, si
q fuera falsa y p fuera verdadera, entonces no podría ocurrir que p ➔ q fuese verdadera.)

Los siguientes argumentos válidos ilustran la aplicación del Modus Ponens.


a) 1) Lidia gana diez millones de dólares en la lotería.
p
2) Si Lidia gana diez millones de dólares en la lotería. e ntonces
José renunciará a su trabajo. _2=.!J.
3) Por lo tanto, J~ renunciará a su trabajo. :. q
b) 1) Si Alejandra se va de paseo a París, entonces tendrá que
ganarse una beca. p-q
2) Alejandra se va de paseo a París. _P __
3) Por lo tanto, Alejandra ganó una beca. :. q

Antes de terminar el análisis de nuestra primera regla de inferencia haremos una última
observación. Los ejemplos (a) y (b) podrían indicar que el argumento válido [p /\ ( p ➔ q))
➔ qesapropiado sóloparaproposicionesprimitivasp, q. Si.n embargo,como [p A (p4q)) -4
q es una tautología para las proposiciones primitivas p, q, la primera regla de sustitución
implica que podemos reemplazar (todas las apariciones de) p o q con proposiciones com-
puestas, y la implicación resultante será también una tautología. En consecuenc ia, si,; s. t
y u son proposicione s, primitivas. entonces

rv s
(r v s)-+(-,1 /\u)
: . -,, /\ u

es unargumentoválido,porModus Ponens; también, [(r V s) /\ [( r V s) ➔ (~1 /\ u]] ➔ (~1


/l. u) es una tautología
Capítulo 2 Fundamentos de lógica

Una situación símilar, en la que podemos aplicar la primera regla de sustitución, se


presentará para cada una de las reglas de inferencia que estudiaremos. Sin embargo. no
mencionaremos esto Wl explícitamente con las demás reglas de inferencia

Una segunda regla de inferencia está dada por la implicación lógica

[(p-+q) l\ (q-+ r))-+ (p-+r),

donde p. q y r son proposiciones. En forma tabular escribimos

p-+q
q-+r
:. p-+r

Esta regla, conocida como la ley del silogismo, surge en muchos argumentos. Por ejemplo,
podemos usarla como sigue:
1) Si el entero 35,244 es divisible entre 396. entonces
e l entero 35,244 es d ivisible entre 66 p-+ q
2) Si el entero 35,244 es divisible entre 66. entonces
el entero 35,244 es divisible entre 3. q-+ r
3) Por lo lallto, si el entero 35,244 es divisible entre 396,
entonces el entero 35.244 es divisible entre 3. : . p-+ r

El siguiente ejemplo tiene un argumento un poco más largo que usa las reglas de
inferencia desarrolladas en los ejemplos 2.23 y 2.24. De hecho, veremos aquí que puede
haber más de una forma de establecer la validez de un argumento.

- - Consideremos el siguiente argumento.


1) Rita está horneando un pastel.
2 ) Si Rita está horneando un paste l, entonces no está practicando la flauta.
3) Si Rita no está practicando la flauta, entonces su padre no pagará el seguro de su
automóvil.
4) Por lo tanto, el padre de Rita no pagará el seguro de su automóvil.
Si nos fijamos en las formas de las proposiciones del argumento anterior. podemos
escribirlo de la siguiente manera:

p
p-1> , q
(*) , q-.,r
:. , r

Ya no necesitamos preocupamos p0r lo que representen realmente las proposiciones. Nuestro


objetivo es usar las dos reglas de inferencia que hemos esrudiado hasta ahora para deducir la
verdad de la proposición -.rapartirde la verdad~ las tres premisasp,p-+ ...,q y ...,q-+ -.r.
2.3 lmplteación I ica: R las de inferencia 83

Establecemos la validez del argumento como sigue:

p...,. Razones
1) p -+,q ~misa
2) , q -+, r ~misa
3) p-+,r Esto se sigue de los pasos (1) y (2)
y de la ley del silogismo
4) p ~misa
5) .-. -,, Esto se sigue de los pasos (3) y
(4) y del Modus Ponen.s.

Antes de pasar a una tercera regla de inferencia. mostraremos que el argumento presen-
tado como ( *) puede justificarse de otra formL En este caso. reduciremos nuestraS "razo-
nes" a la forma que usaremos para el resto de la sección. Sin embargo, siempre enumera-
remos lo necesario para demostrar de dónde surge un argumento o cómo cada paso se
sigue de pasos anteriores.
Una segunda forma de justificar el argumcn10 es la siguiente.

Pasos RuOD<S
1) p ~misa
2) p-+ , q ~misa
3) , q Pasos (1 ) y (2) y Modus Pon,ns
4) ,q-+, r Premisa
5) :. , , Pasos (3) y (4) y Modus Ponen.,

2.JI La regla de inferencia llamada Modus To/1,ns está dada por

Esto se obtiene de la implicación lógica [p ➔ q) /\ -.q] ➔ -.p. Modus Tollens viene del
latín y puede traducirse como "m6todo de negación". Este nombre se debe a que negamos
la conclusión. q. para demostrar -.p. (Observemos que también podemos obtener e sta
regla mediante el Modu$ Pon.eru, usando el hecho de que p ➔ q <=> -.q ➔ ..,p.)
El siguiente es un ejemplo del uso del Modus Tollens para hacer una inferencia válida:

1) Si Concha es elegida presidenta de la asociación femenina Phi


Delta. entonces Elena ingresara a esta asociación. p-.. q
2) Eiena no ingresó a la asociación. ":21!._
3) Por lo tanto, Concha no fue elegida presidenta de la asociación :. , q
femenina Phi Delta.
(apítok> 2 fundamentos de lógica

Ahora usaremos el Modus Tolkns para demostrar que el siguiente argumento es válido
(para las proposiciones primitivas p. r. s. t y u).

p-+r
r-+s
t V7S
7tVU
7U
:. 7p

Aquí utilizamos el Motbu Tolkn.s y la ley del silogismo, junto con la equivalencia
lógica del ejemplo 2.7.

Pasos Razones
1) p-+r, r-+s Premisas
2) p-+s Paso ( 1) y la ley del silogismo
3) IV7S Premisa
4) 7 SVI Paso (3) y la propiedad conmutativa de V
5) s-+ I Paso (4) y el hecho de que -.s V 1-= s ➔ t
6) p-+I Pasos (2) y (5) y la ley del silogismo
7) 7I VU Premisa
8) 1-+u Paso(7) yel hecho deque -1 V u<=> t-> u
9) p-+u Pasos (6) y (8) y la ley del silogismo
10) 7U Premisa
11) ;. ,p Pasos (9) y (10) y Modus Tollens

Antes de pasar a otra regla de inferencia. resumiremos lo que hemos realizado (y lo no


realizado). El argumento anterior muestra que

[(p-+ r) /\ (r-+ s) /\(IV7S) /\(71V u) /\ 7U)~7p.

No hemos usado las leyes de la lógica, como en la sección 2.2. para expresar la proposi-
ción
(p -+ r) /\ (r-+s) /\ (1 V7S)/\ (71V u) /\7u
como una proposición lógicamente equivalente más sencilla. Observemos que

[(p-+ r) /\ (r-+ s) /\ (1V7S) /\ (71 V u)/\ 7U)~7p.


pues cuando p toma el valor de verdad Oy u tiene el valor de verdad 1, el valor de verdad de
-pes 1, mientras que el de -u y (p-> r) 11 (r-> s) 11 (r V -s) 11 (-1 V u) 11 -u es O.

Analizaremos ahora una fonna tabular de las reglas de inferencia Modus Ponens y
Modus Tollens.

Modus Ponen.s: p-+q Modus Tollensc p-+ q


_P_ 7q
:. q :. ,p
23 1 85

La razón por la que queremos hacer esto es que pcxlñan sur¡ir otras formas tabulares en
algún momento. similares en apariencia pero que representan argumentos no válidos, en
los que cada una de las premisas es verdadera pero la conclusión es falsa.

a) Consideremos cada uno de los argumentos siguientes:


1) Si Margaret Ibatcheres presidenta de los Estados p-+q
Unidos. entonces ella tiene al menos 35 años de edad.
2) Marguet 1batcher tiene al menos 35 alios de edad. g__
3) Por lo tanto, Margue! Thatcber es presidenta de :. p
Estados Unidos.

Aqul vemos que [(p ➔ q) /\ q] ➔ p no es una tautología. Ya que s i consideramos


los valo= de verdad p: O y q: 1, entonces cada una de las premisas p ➔ q y q es
verdadera pero la conclusión pes falsa. Este argumento no válido surge de la/alacia
(error en el razonamiento)de tratar de argumentar por la recíproca; es decir, aunque
[{p ➔ q) /\ p] => q. no ocurr< que [(p-+ q) /\ q] ~ p.
b) Un segundo argumento en el que la conclusión no necesariamente se sigue de las
premisas podr{a ser el siguiente:
1) Si 2+ 3 ~6, entonces 2 +4 = 6. p-+q
2) 2+3 ,t 6. ~
3) Por lo tanto, 2 + 4,. 6 :. ,q
En este caso, vemos que [(p ➔ q) /\ -.p] ➔ -iq no es una tautología. De nuevo,
los valores de verdad p: O y q: 1 muestran que las premisas p ➔ q y -p pueden ser
ambas verdaderas pero que la conclusión -.q es falsa. La falacia que subyace en
este argumento no válido viene de nuestro intento de ar¡umentar por la inversa. ya
que efectivamente [(p-+ q) A -q] => - p, pero de aquí no se sigue que [(p -+ q) A
-p]=> - q.
Antes de proseguir, mencionaremos una regla de inferencia sencilla pero imponante.

La siguiente regla de inferencia smgc de la observación de que si p, q son proposiciones


verdaderas. entonces p A q es una proposición verdadera.
Supoe amos que las proposícionesp, q aparecen en el desarrollo de un ar¡umento. Esw
proposiciones podrían ser premisas (dadas) o resultados que se pueden obtener de las
premisas o de los resultados desanollados en una parte anterior del argumento. Entonces,
en estas circunstancias, es posible combinar las dos proposiciones en su conjunciónp A q
y usar esta nueva proposición en pasos posteriores para continuar con el argumento.
&ta es la regla dt conjunci6n, que escribimos en forma de tabla:

~
Prosigamos nuesuo estudio de las reglas de inferencia examinando otra regla muy sen-
cilla pero imponante .
Capítulo 2 Fundamentos de lógica

La siguie nte regla de inferencia, que podría servir para ilustrar el sentido común, es la del
silogismo disyuntivo. Esta regla proviene de la implicación lógica

que podemos obtener mediante Modus Ponens, observando que p V q ~ (-.p ➔ q).
Podemos escribirla en forma de tabla:

pVq
::!E..._
:. q

Esta regla de inferencia se usa cuando hay que analizar exactamente dos posibilidades y
podemos descanar una de ellas si no es verdadera. Entonces la otra posibilidad es la que
tie ne que ser verdadera. El siguiente caso muestra una aplicación de esta regla:
1) La canera de Beto está en su bolsillo o e n la mesa. pVq
2) La canera de Beto no está en su bolsillo ,p
3) Por lo tanto, la cartera de Beto está en la mesa. ~

Hasta el momento hemos analiz.ado cinco reglas de inferencia.Antes de intentarjustifi-


car más argumentos, como el del ejemplo 2.26 (con 11 pasos), analizaremos una regla
más. la cual contiene un método de demostración que se confunde algunas veces con el
método (de demostración} por contrapositiva dado en el Modus Toll~ns. La confusión
surge debido a que ambos métodos implican la negación de una proposición. Sin embargo,
pronto veremos que son dos métodos distintos. (Al fina] de la sección 2.5 compararemos y
contrastaremos de nuevo ambos métodos.)

Sea p una proposición arbitraria y F0 una contradicción. Los resultados de la columna 5 de


la tabla 2.18 muestran que la implicación ( .....p ➔ Fo) ➔ p es una tautología, lo que propor-

"tlbla2.18

p 7p F,
1 o 1
o 1 o

ciona la regla de inferencia llamada regla de contradicción. Podemos escribir esta regla en
forma de tabla:

7p-F,
:.p
2.3 Implicación lógica: R las de inferencia 87

Esta regla indica que si pes una proposición y ...,p-+ F0 es verdadera. entonces -p debe
ser falsa, puesto que F0 es falsa. Así, tenemos que pes verdadera.
La regla de contradicción es la base de un método para establecer la validez de un
argumento: el método de dtmostraci6n por contradicción. o rrducci6n al absurdo. La idea
que estí detrás de este ~todo es demostrar una proposición (la conclusión de un argu-
mento) mostrando que, si esa proposición fuera falsa. entonces Uegaríamos a deducir una
consecuencia imposible. El uso de este ~todo surge en cienos argumentos que describi-
remos a continuación.
En general, cuando queremos establecer la validez del argumento

(p,l\p-,1\- --1\p.)-+q,

podemos establecer la validez del argumento lógicamente equivalente

(p,/\p-,1\- · -1\p. /\,q)-+ Fo.

[Esto se sigue de la tautología de la columna 7 de la tabla 2.19 y de la primera regla de


sustiwción, en la que reemplazamos la proposición primitiva p por la proposición (Pt A Pi
/1 . .. /1 p.)t.J

Tabla 2.19
, 9 F, ¡,/\.,9 (p /\19)-F. ,-9 ( p-f) - [(r /\-,f)-F,J
o o o o 1 1 1
o 1 o o 1 1 1
1 o o 1 o o 1
1 1 o o 1 1 1

Cuando aplicamos el mt!:todo de demostración por contradicción, primero suponemos


que lo que intentamos justificar (o demostrar) es en realidad falso. Despu6s usamos la
hipótesis como uoa premisa adicional para producir una contradicción (o situación impo-
sible) de la forma p /\ -p, para alguna ¡,.-oposición p. Una vez que hemos obtenido esta
contradicción podemos concluir que la proposición dada era verdadera, lo cual justifica el
argumento (o tcnnina la demostración).
Usaremos el m6todo de demostración por contradicción cuando sea más fácil (o parez-
ca serlo) usar --qjunJccon las premisasp1,P1, ...• p. para deducir una contradicción. que
deducir la conclusión q directamente de las premisas p 1, p 1, . . . • p •. Aplicaremos este
mttodode demostración en los últimos ejemplos de esta sección, los ejemplos 2.33 y 2.36.
También lo veremos varias veces en otros capítulos del texto.

f En b. sec:c:ión 4.2 del capítulo 4 <Latcmos la razón por la que sabernos que para eualcsquiua proposic:io-
DCS P1, Pr- ·••,p. y q, se sigue que (p1 A P:. A··· /\p,j/\ --qcp, l'IP:/\ ••· /\p.A--q.
Capítulo 2 Fundamentos de lógica

Ahora que hemos analizado seis reglas de inferencia, haremos un resumen de éstas e
introduciremos otras en la tabla 2.20.

Tabl• 2.20

Regla de inferencia Implicación lógica relacionada Nombre de la regla

1) p [ p/\(p-+q)]-+q Regla de separación


p-+q (Modus poneos)
:. q
2) p-+q [(p-+q)/\(q-+ r)]-+ (p-+r) Ley del silogismo

:.p-+r
3) p-+q [(p-+q)/\,q]-+ 7p Modus Tollens:
"::!L._
: .7p
4) p Regla de la conjunción
'l..__
:. p /\ q
5) p y q [(p y q) /\,p]-+q Regla del silogismo
,p disyuntivo
::q
6) ,p-+ Fo (,p-+F0)-+p Regla de
--:::¡- contradicción
1) p/\q (p/\q)-+ p Regla de simplificación
:.p conjuntiva
8) p p-+ p V q Regla de amplificación
:.pyq disyuntiva
9) p /\ q [(p /\q)/\[p-+ (q-+ r)D-+ r Regla de demostración
p-+(q->r) condicional
:. r
10) p-+r [(p-+ r) /\( q-+r)]-+[(p V q)-+r] Regla de demostración
q-+r por casos
:. (pyq)-+r
11) p-+q [(p-+q)/\ (r-+s)/\(pV ,))-+ (qV s) Regla del
r➔s dilema
PV' constructivo
:. q y s
12) p-+q [(p-+ q) /\(r-+s)/\(,q v,s)]-+ (,p v,,) Regla del
r-+s dilema
destructivo

Los siguientes cinco ejemplos presentan argumentos válidos. Estos ejemplos nos mues-
la forma de aplicar las reglas enumeradas en la tabla 2.20 junto con otros resultados,
tran
como las leyes de la lógica.
2.3 Implicación lógica: Reglas de inferencia 89

Nuestro primer ejemplo muestra la validez del argumento

p -+ r
7p -+ q

~
•♦• 7T-+S

Pasos Razones
1) p -+r Premisa
2) I T-t>7p Paso (1) y p ➔ r~ ~r ➔ -ip
3) ,p-+ q Premisa
4) 7T -t> q Pasos (2) y (3) y la ley del silogismo
S) q -+ s Premisa
6) .'. IT-+S Pasos (4) y (5) y la ley del silogismo

Una segunda forma de justificar el argumento es la siguiente.

Pasos Razones
1) p-+r Premisa
2) q-+s Premísa
3) ,p-+q Premisa
4) pVq Paso (3) y (-p ➔ q) <a> (--p V q) <e> (p V q), donde la
segunda equivalencia lógica se sigue de la ley de la doble
negación
S) rvs Pasos (1), (2) y (4) y la regla del dilema constructivo
6) :. , r-+ s Paso (5) y (r V s) <a> ( - -,V s) <e> (-r ➔ s), donde
usamos la ley de la doble negación en la primera
equivalencia lógica.

El siguiente ejemplo es un poco más complejo.

Establezca la validez del argumento

p-+ q
q -+ (r /\s)
-irV(-itVu)
p/\t
:. u

Pasos Razones
1) p-+q Premisa
2) q-+(r/\s) Premisa
3) p -+(r/\s) Pasos (1) y (2) y la ley del silogismo
4) p/\t Premisa
S) p Paso (4) y la regla de la simplificación conjuntiva
6) r /\s Pasos (5) y (3) y Modus Ponens
90 capítulo 2 Fundamentos de lógica

7) r Paso (6) y la regla de la simplificación conjuntiva


8) 7rV(7I VU) Premisa
9) 7 (r /\ t)V u Paso (8), la propiedad asociativa de V y las leyes de
De Morgan
10) 1 Paso (4) y la ley de la simplificación conjuntiva
11) r/\ t Pasos (7) y (10) y la regla de la conjunción
ll) .-. u Pasos (9) y (11) y la regla del silogismo
disyuntivo

Este ejemplo mostrará que el siguiente argumento es váJido.


Si la banda no pudiera tocar rock: o las bebidas no llegasen a tiempo. en•
tonces la fiesta de Año Nuevo tendóa que cancelarse yAlicia se enojaría Si la
fiesta se cancelara, habría que devolver el dinero. No se devolvió el dinero.
Por lo tanto, la banda pudo tocar rock.
Primero convertimos el argumento dado en una forma simbólica mediante la siguiente
asignación de proposiciones:

p: La banda pudo tocar rock.


q: Las bebidas se entregaron a tiempo.
r: La fiesta de Año Nuevo se canceló.
s: Alicia estaba enojada.
t: Hubo que devolver el dinero.

El argumento anterior se escribe como

r➔ t

7/
:. p

Ahora establezcamos la validez de este argumento como sigue:

Pasos Razones
1) r➔I Premisa
2) 7 / Premisa
3) 7T Pasos (1), (2) y Modus Tollens
4) 7TV7S Paso (3) y la regla de la amplificación disyuntiva
5) 7(T/\ S) Paso (4) y las leyes de De Morgan
6) ( 7 p V7q)➔ (r /\ s) Premisa
7) 7(7pV7q) Pasos (6), (5) y Modus Tollens
8) pi\ q Paso (7), leyes de De Morgan y la ley de la
doble negación
9) .-. p Paso (8) y la regla de la simplificación conjuntiva
2.3 Implicación lógica: Reglas de inferencia 91

En este caso usaremos el método de demostración por contradicción. Consideremos el


argumento

,p++q
q-->r
::JL_
:. p
Para establecer la valide:t de este argumento. hemos supuesto la negación -.p de la
conclusiónp como otra premisa. El objetivo ahora es usar las cuatro premisas para obtener
una contradicción Fo, He aquí una forma de obtenerla.

Pasos Razones
1) ,p<-+q Premisa
2) (,p--+q) A (q....,,,p) Paso(l)y( - p<-+qle>[(-p _. q) A (q4-p))
3) , p--+q Paso (2) y la regla de la simplificación
conjuntiva
4) q --+ r Premisa
S) , p-r Pasos (3), (4) y la ley del silogismo
6) ,p Premisa (que hemos supuesto)
1) r Pasos (5), (6) y Modus Ponens
8) ,r Premisa
9) r /\.r(~fo) Pasos (7), (8) y la regla de conjunción
10) :.p Pasos (6), (9) y el método de demostración
por contradicción

Si analizamos lo que ocurrió en este caso, tenemos que

[( ,p <-+q) /\(q....,, r) 1\-,r /\,p] C? Fo.

Esto requiere que el valor de verdad de [(-,p H q) A (q ➔ r ) A -., A -,p} sea O. Como
--p t t q. q ➔ r y r son las premisas dadas, cada una de estas proposiciones tiene el valor
de verdad l. En consecuencia, para que {(-,p t t q) A (q ➔ r) A -.r A -,p] tenga el valor
de verdad O, la proposición -.p debe tener el valor de verdad O. Por lo tanto, p tiene el
valor de verdad 1 y la conclusión p del argumento es verdadera.

Antes de analizar nuestro siguiente ejemplo, necesitamos recordar el resultado del ejem-
plo 2.16: para las proposiciones primitivas arbitrarias p. q, r,
[p ..... ( q--+ r)]~[(p /\q)--+ r].
Mediante la primera regla de sustitución, reemplazaremos cada aparición de p por la pro-
posición compuesta <,¡,1 /\ p2 A · · · A p,.,). Luego obtenemos el nuevo resultado
[(p 1 /\p2 /\- • -A p.)--+(q--+r)J~[(p 1 Ap,/\ · · -1\p. A q)t --+r].

t En 1a sección 4.2 del capftuJo 4 p¡cscnwemos una demostración fonna1 de por qut (p, A P? A. · · A
p. )Aq~p1 AP"!.A·· · Ap,., A(¡.
92 Capítulo 2 fundamentos de lógica

Este resultado indica que si queremos establecer la validez del argumento (•) podemos
hacerlo estableciendo la validez del argumento correspondiente(º).
(') p, (") P•
p, p,

p. p.
:,q-+r !L
:. r
Después de todo. si queremos mostrar queq ➔ r tiene el valor de verdad l, cuandop1, P?.
.. . , p. tambiin tienen valor 1 y si el valor de verdad de q es O, entonces no hay nada que
hacer. ya que el valor de verdad de q ➔ res 1 en este caso. Entonces, el verdadero proble-
ma es mostrar que q-+ r tiene el valor de verdad 1, cuando p 1, P:, ... ,p. y q también lo
tienen; es decir, necesitamos mostrar que cuandop" Pl, ••• , p,,. q tienen valor de verdad l,
entonces el valor de verdad de res l.
Demostraremos este principio en el siguiente ejemplo.

Para establecer la validez del argumento


(') u-+r
(r/\s)-+(pVt)
-,,
q-+(u/\s)

:.q-+p
consideramos el argumento correspondiente
(") u-+r
(r/\s) ➔ (pVt)
q-+(u/\s)
-,,
q
:.p
[Observe que q es la hipótesis de la conclusión q ➔ p para el argumento (•) y que se
convierte en otra premisa del argumento (**) donde la conclusión es p.]
Para justificar el argumento(**) procederemos de la manera siguiente:
Pasos Razones
1) q Premisa
2) q -+ (u/\s) Premisa
3) u/\s Pasos ( 1), (2) y Mt>d,,s Ponens
4) u Paso (3) y la regla de la simplificación conjuntiva
5) u~r Premisa
6) . r Pasos (4), (S) y Mt>d,,s Ponens
7) s Paso (3) y la regla de la simplificación conjuntiva
8) r /\s Pasos (6), (7) y la regla de conjunción
9) (r/\s)-+(pVt)• Premisa
10) PVI Pasos (8), (9) y Mt>d,,s Ponens
11) -,, Premisa
U) :.p Pasos (10), (11) y la regla del silogismo disyuntivo
2.3 Implicación lógica: Reglas de inferencia 93

Ahora sabemos que para el argumento (H )

[(u-+ r) A[(r As)-+ (p V t)) A[q--+ (u A s)]A7tA q]* p,

y para el argumento (*) se sigue que

[(u --+ r) A[(r As)-+ (p V t)JA[q--+ (u As)) A 71)* (q--+ p).

l.os ejemplos 2.30 a 2.34 nos dan una idea de la forma de establecer la validez de un argu-
mento. Después del ejemplo 2.26 analizamnos dos siruaciones en las que un argumento no es
válido: cuando intentamos argumentar mediante larecfproca o la inversa.Ahora vamos a apren-
der algo más acm:a de la forma de derenninar cuándo un argumento no es válido.
Dado un argumento
p,
p,
p,

J!,_
:. q

decimos que el argumento no es válido si puede ocurrir que cada una de las premisas p 1• p 1,
p)t ... •p. sea verdadera (con valor de verdad I ). y que la conclusión q sea falsa (con valor
de verdad 0).
El siguiente ejemplo ilustra un método indirecto para mostrar que un argumento que
intuimos que no es válido (tal vez porque no podemos encontrar la forma de demostrar
que es válido) realmente no lo t s.

Consideremos las proposiciones primitivas p, q, r, s y r, y el argumento


p
p Vq
q-+(r-+s)
1-+r
.-. ,s~ ,t
Para mostrar que este argumento no es válido, necesitamos una asignación de valores de
verdad para cada una de las proposiciones p. q, r, sy rdc modo que la conclusión ....,s ➔ .....,
sea falsa (que tenga el valor de verdad O) mientras que las cuatro premisas sean verdaderas
(tengan el valor de verdad l). El único caso en que la conclusión ....,s ➔ ....., es falsa se
prese~ta cuando ....,ses verdadera y ....,tes falsa. Esto implica que el valor de verdad des es
O y el valor de verdad de r es 1.
Como p es u.nade las premisas, su valor de verdad debe ser l. Para que la premisap V
qtengael valor de verdad l,qpucdc ser verdadera(l)o falsa(O). Consideremos la pre misa
r 4 ,; donde sabemos que tes verdadera. Si r 4 r debe ser verdade~ entonces r debe ser
verdadera (tener el valor de verdad 1). Ahora bien, si res verdadera (J) y ses falsa (0),
tenemos que r 4 s es falsa (O) y el vaJor de verdad de la premisa q ➔ (r 4 s) será 1
únicamente cuando q sea falsa (0).
Capítulo 2 Fundamentos de lógica

En consecuencia, con la asignación de los valores de verdad


p: q: o ,: 1 s: o 1: 1,

las cuatro premisas


P pVq q .... (,-+s) t -+ r
tienen el valor de verdad 1, mientras que la conclusión
,s-., t
tiene el valor de verdad O. En este caso, hemos mostrado que e l argumento dado no es
válido.

Las asignaciones de valores de verdadp: 1.q: O, r. l,s: O y t: 1 del ejemplo 2.35 muestran
un caso que thsaprueba algo que podríamos haber considerado como un argumento válido.
Debemos observar entonces que. para mostrar que una implicación de la forma

representa un argumento válido, necesitamos considerar todos los casos en que las premisas
p1, p 2 , ••• , p. sean verdaderas. (Cada uno de esos casos es una asignacíón de valores de
verdad para las proposiciones primitivas (queconfonnan las premisas)cnquepi.p 1,p,. ... , p.
son verdaderas.] Para lograr esto (analizar todos los casos sin escribir las tablas de ver•
dad), hemos utilizado las reglas de inferencia junto con las leyes de la lógic~ y otras equi•
valencias lógicas. Para analizar todos los casos necesarios, no pod.emos recurrir a un solo
ejemplo (o caso) específico como medio para establecer la validez. del argumento (para
todos los casos posibles). Sin embargo, cuando queremos mostrar que una implicación (de
la forma anterior) no es una tautología, todo lo que debemos hacer es encontrar un caso
para el que la implicación sea falsa; es decir, un caso e-n el que todas las premisas sean
verdaderas pero que la conclusión sea falsa. Este caso proporciona un contraejemplo para
el argumento y muestra que no es válido.
Veamos un segundo ejemplo en el que utilizaremos el método indirecto del ejemplo 2.35.

¿Es válido o no el siguiente argumento? (En estecaso,p, q, ry s son proposiciones primitivas.)


p-+q
q-+s
r-+7S
7 p Y.. r
: . 7p

¿Podría ser falsa la conclusión ~psi las cuatro premisas fueran verdaderas? La conclusión
-.p es falsa si p tiene el va1or de verdad l. Así, para que la premisa p ➔ q sea verdadera. el
valor de verdad de q debe ser 1. Como la premisa q ➔ s también es verdadera., la verdad de
q implica la verdad des. En consecuencia, las proposicioncsp, qy s tienen e) valor de verdad
l. Si analizamos ahora la premisar ➔ -.s, tenemos que, comos tiene el valor de verdad l,el
valor de verdad derdebeserO. Por lotanto,res falsa. Pero si -.pes falsa y la premisa -.p y
r es verdadera, también debemos tener r verdadera. Por lo tanto, tenemos que p ~ ( --.r /\ r ).
2.3 Implicación lógica: Reglas: de infe<encia 95

No hemos podido encontrar un contraejemplo de la validez del argumento dado. Sin


embargo. esto nos ha mostrado que dicho argumento es válido, y la validez se sigue de la
aplicación del método de demostración por contradicción.

Esta introducción a las reglas de inferencia está lejos de ser exhaustiva. Varios de los
libros citados en la bibliografía del final de este capítulo ofrecen material adicional para el
lector que desee profundizar en el estudio de este tema. En la sección 2.5 aplicaremos las
ideas desarrolladas en esta sección a proposiciones de una naturaleza más matemática, ya
que queremos aprender a desarrollar la demostración de un teorema. En el capítulo 4,
agregaremos otra importante técnica de demostración, la inducción matemática, a nuestro
arsenal para la demostración de teoremas matemáticos. Sin embargo, primero e l lector
deberá resolver cuidadosamente los ejercicios de esta sección.

EJERCIOOS 2.3 1. Los siguientes tres aigUmcntos son válidos. Establezca ta validez de cada uno por medio de
una tabla de verdad. En cada caso, determine las filas de la tabla que son cruciales para evaluar
la validez del aigumento y las que pueden dejarse de lado.

a) [p/\(p-+q)l\ r] -+ [(pVq)-+ r ]
b) [(p /\q)-+ r]/\-.q /\ (p -+-.r )]-+(-.p y-.q)
e) [pV(qVr)]/\-.q]-+(pvr)
2. Use tablas de verdad para verificar que cada una de las siguientes proposiciones es una
implicación lógica:

•) [(p-+ q)/\(q- ,)J➔ (p-,)


b) [( p -+ q)/\-.q]-+-.p
e) [(pyq)/\-.p]- q
d) [(p-+ r)/\(q -+r)]-+[(p Vq)-+r]

3. Verifique que cada una de las síguientes proposiciones es una implicación lógica. mostrando
que es imposible que la conclusión tenga el valor de verdad O mientras la hipótesis tenga el
valor de verdad 1.
a) (p/\q) - p
b) p-+(pyq)
e) [(pyq)/\-.p]-+ q
d) [(p-+q)/\(r -+s)/\(pvr))-+ (qVS)
•l [(p-+q)/\(r -+s)/\(-.qy-.s)]-+ (-.p y-,,¡
4 . Para cada uno de los siguientes pares de proposiciones, use el Modus Ponerzs o el Modus
Tol/ens para completar la Hnea en blanco con un argumento válido.
a) Si Juana tieoe problemas para arrancar su automóvil, entonces su hija Ángela verificará las
bujías.
Juana tiene problemas para arrancar su automóvil.

b) Si Braulio ~vióel primer problema correctamente, entonces la respuesta que obtuvo es 137.
La respuesta de Braulio al primer problema no es 137.

(:) Si ~te es un ciclo rtpeat-wuü, enionces el cuerpo de este ciclo se ejecuta al menos una vez.

:. El cuerpo del ciclo se ejecuta al menos un.a vez.


Capítulo 2 Fundamentos de lógica

d) Si Tomás juega baloncesto después de mediodía. entonces no vera el tele visor por la tarde.

:. Tomás no jugó baloncesto después de mediodía.


e) Si María Luisa no rompe las fotos de Jorge. entonces tendrá que mostrarlas en el tablero de
avisos.
María Luisa no mostró las fotos de Jorge en el tablero.

5. Considere cada uno de los siguientes argumentos. Si el argumento es válido. identifique la


regla de inferencia que establece su validez. Si no, indJque si el error se debe a un intento de
argumentación por la re.cíproca o por la inversa.
a) Andrca puede programar en Pascal y puede programar en FORTRAN.
Por lo tanto, Andrca puede programar en Pascal.
b) Una condición suficiente para que Berta gane el torneo de golf es que su oponeme Mima no
haga un birdie e n el último hoyo.
Mima no hizo un birdu en el último hoyo.
Berta ganó el torneo de golf.
Por to tanto Mima. la oponente de Berta. no hizo un birdú e n el último hoyo.
e) Si el programa de Ronaldo es correcto. entonces podrá terminar su tarea de ciencias de la
computación en menos de dos horas.
Ronaldo tarda más de dos horas en terminar su tarea de ciencias de la computación.
Por lo canco, el programa de Ronaldo es incorrecto.
d ) Las llaves del auto de Elisa están en su bolso o sobre la mesa de la cocina.
Las llaves del auto de Elisa no están sobre la mesa de la cocina.
Por lo tanto. las llaves del auto de Eisa están en su bolso.
e) Si bajan los tipos de interb, entonces subirán las acciones de la bolsa.
Los tipos de interts no están bajando.
Por lo tanto. no subirán las acciones de la bol~.
f) Si Alejandro recibe un aguinaldo, entonces viaj ará al suroeste de Estados Unidos.
Si Alejandro viaja al suroeste de Estados Unidos, entonces visitará el Gran Cañón.
Por lo tanto. si Alejandro recibe un aguinaldo, entonces visitará el Gran Cañón.
6 . Para las proposiciones primitivas p. q y r. sean P la proposición

[p /l(p - q)/1(, v , ) /1(,--.q)]- (sv1)

y Pl la proposición [p /\ (q V r)] V -.[p V (q V r)l.


a) Use las reglas de inferencia para mostrar que q /\ r ~ q V r.
b) ¿Es cieno que P ~ P 1?
7 . Justifique cada uno de los pasos necesarios para mostrar que el siguiente argumento es válido.

[p /l(q 11, )]v,[p V(q /I r)],

Pasos
1) p
2) p ----> q
3) J/
4) ,_.,q
5) q--,,
6) ...,,
7) sv,
8) ,
9) :. ,v,
2.3 Implicación lógica : Reglas de inferencia 97

8. Dt las razones para los pasos que verifican el siguiente argumento.

(,pvqJ- ,
,-(, v r)
,st\-,u
,u_..,,
.-. p

Pasos
1) ,st\,u
2) ,u
J) ..,u_, ,
4) ..,,
5) ""
6) ,s/\,1
7l , -(,vr)
8) -,(svt) -+--ir
9) (,s/\-,1)- , ,
10) ..,,
11¡ (, p v q)-,
IZ) ""\r - ,(,pV q)
13) , , -(p/\,q)
14) p l\,q
15) .-. p

9. a) Dt las razones pan los pasos que justifican el argumento

[(p-q) /\(,, v,)/\(p V r)J-(,q- s).

Pasos
1) ,(,q-s)
l) , q /\,s
3) •s
4) ..,, vs
5) , ,
6) p- q
7) ,q
8) ,p
,¡ p v ,
10) r
11) -.r/\r
12) .-. , q-s
b) Realice una demostración directa del resultado de la pane (a).
<") Realice una demostración directa del resullado del eje mplo 2.33.
1 O. Establezca la vaJ1dez de los sigu.jentes argumentos.

•l [(p /\,q)/\r]-[(p /\r) V q]


b) [pl\(p-q)l\(,q V r)]-r
e) p-q d) p-q t) P-(q-,)
,q ,_,q -..q-•p
,, P
_ _ _
.-. , (p v ,) .-.-..p :. r
capitulo 2 Fundamentos de lógica

C) p/\q b) pyq
p➔ (r/\q) •pVr
r➔ (,vr) ...,
--,, ..., :.q
-_._,-- -
11 . Muestre con un contraejemplo que ninguno de los siguientes argumentos es válido; es decir. dé
una asignación de valores de verdad a las proposiciones primitivas p. q. r y s de modo que
todas las premisas sean verdaderas (tengan el valor de verdad 1) y que la conclusión sea falsa
(tenga el valor de verdad O).
a) [(p/\-.q)/\[p ➔ (q ➔ r)D➔-.r b) [(p/\q)➔ r]/\(-.qyr))➔p
e) p-q d) p
q➔ r p➔ r

rv..,s p ➔ (qy-.r)
•s- q ,qv,s
... s :. s
12. Escriba cada uno de los siguientes argumentos en forma simbólica Establezca desputs la
validez del argumento o dt un contraejemplo para mostrar que no es válido.
a) Si Ros.a María obtiene el puesto de supervisor y trabaja mucho. entonce$ obtendrá un au-
mento. Si obtiene el aumento. entonces comprará un auto nuevo. Ella no ha adquirido un
auto nuevo. Por lo tanto, Rosa María no ha obtenido el puesto de supervisor o no ha traba-
jado mucho.
b) Si Domingo va a la carrera de autos, entonces Elena se enojará.. Si Rafael juega canas toda
la noche. entonces Carmen se enojara. Si Elena o Carmen se enojan, le avisarán a Verónica
(su abogado). Verónica no ha tenido noticias de estas dos clientes. En consecuencia. ni
Domingo fue a las carreras ni Rafael jugó cartas toda la noche.
e) Si Norma va a su reunión del martes por la maruma. entonces deberá levantarse muy tem-
prano ese día Si va al concierto de rock el lunes por la noche, entonces llegará a su casa
desputs de las 11:00 P.M. Si Norma llega a su casa a esa hora y se levanta temprano al día
siguiente, entonces tendrá que ir a trabajar después de dormir menos de siete horas. Por
desgracia. Norma no puede trabajar con menos de siete horas de descanso. Norma no debe-
rá ir al concicno de rock o deberá faltar a su rCWUón del martes por la mai\ana.
d) Si hay cierta probabilidad de lluvia o pierde su cinta roja para el cabello. entonces Lorcta no
cortará el césped Siempre que la temperatura está por arriba de Jos 80ºF. no hay probabili-
dad de lluvia. Hoy la temperatura es de 85ºF y Lorcta está usando su cinta roja. Por lo 1anto
(en algún momento del día). Lorct.a cortará el c6sped.

2.4
El uso de cuantificadores

En la ~ión 2.1 mencionamos el hecho de que los e nunciados que contienen una variable
como x no necesariamente son proposiciones. Por ejemplo, la frase ..E l númerox + 2 es un
entero par" no necesariamente es verdadera o falsa, a menos que conozcamos el valor que
sustituirá ax. Si restringimos nuestra elección a los enteros, entonces, al reemplazar x por
-5, - 1 o 3, por ejemplo, la proposición resultante será falsa. De hecho, es falsa siempre
que sustituyamos x con un entero impar. No obstante, cuando un entero par sustituye ax,
la proposición resultante es verdadera.
2.4 El uso de cuantificadores 99

Nos re feriremos a la frase ..El número x + 2 es un enlero par" como una proposición
abierta, concepto que definimos formalmente como sigue.

Definición 2.5 Una frase declarativa es una prop<>.sici6n abierta si

1) contiene una o más variables, y


2) no es una proposición, pero
3) se convierte en una proposición cuando las variables que aparecen en ella se reem~
plazan por ciertas opciones permisibles.

Cuando analizamos la frase "El número x + 2 es un entero par" a la luz de esta defini•
ción, vemos que es una proposición abierta que contiene una sola variable, x. Respecto al
tercer elemento de la definición, en nuestro análisis anterior restringimos las "ciertas op-
ciones permisibles"' a los enteros. Estas opciones permisibles forman lo que se llama el
universo o universo de discurso para la proposición abierta. El universo comprende las
opciones que queremos considerar o permitir para la variable o variables de la proposición
abierta. (El universo es un ejemplo de un conjunto, concepto que analizaremos con detalle
en el siguiente capítulo.)
Al tratar las proposiciones abienas, usaremos la siguiente notación:
La proposición abierta "'El número x + 2 es un entero par'' se denota con p(x) [o q(x).
r(x ), etcétera]. Entonces --p(x} se podría leer como ··ti númerox+ 2 no es un entero par'".
Usaremos q(x, y) para representar una proposición abierta con dos variables. Por ejem•
plo, consideremos

q(.x, y): Los números y + 2, x - y y x + 2y son enteros pares.

En el caso de q(x,y), cada una de las variablesx, y aparece más de una vez. Se sobreentien•
de que cuando reemplazamos una de las letras x por un elemento de nuestro universo,
reemplazamos la otrax con el mismo valor. De Ja misma forma. cuando se sustituye y (con
un valor de su universo), se hace la misma sustitución para todas las apariciones de la
variabley .
Con p(x) y q(x, y) como antes, y un universo e n el que los enteros siguen siendo las
mismas opciones permisibles, obtenemos los siguientes resultados cuando hacemos algu•
nos reemplazos de las variablesx,y.
p(5): El número 7 (= 5 + 2) es un e ntero par. (FALSO)
~p(7): El número 9 no es un entero par. (VERDADERO)
q(4,2): Los números 4. 2 y 8 son enteros pares. (VERDADERO)

Tam~ién observamos, por ejemplo, que q(.5,2) y q(.4,7) son proposiciones falsas. mientras
que ~ q(5,2) y ~q(4.7) son verdaderas.
En consecuencia. vemos que para ambas expresiones p(x) y q(x. y), según los valores
dados, algunas sustituciones producen proposiciones verdaderas y otras producey propo-
siciones falsas. Po lo tanto, podemos construir las siguientes proposiciones verdaderas.
\...__
1) Para algún x, p(x).
2 ) Para algunos x. y, q(x, y).
DO Capítulo 2 Fundamentos de lógica

Observe que en este caso, las proposiciones ..Para algún x, -.p(_x)" y ..-Para algunos x. y ,
-.q<x, y)" también son verdaderas. [Puesto que las proposiciones "Para algún x, p(x)" y
"Para algúnx, -.p(x)" también son verdaderas, vemos que la segunda proposición no es la
negación de la primera,. aunque la proposición abiena -.p(_x) es la negación de la proposi-
ción abiena p(x). Un resultado similar es verdadero para las proposiciones que implican
q(x, y) y ~ q(x, y).]
Las frases "Para algúnx" y "Para algunos.r. y.. cuantifican las proposiciones abiertas
p(x) y q(x, y), respectivamente. Muchos postulados, definiciones y teoremas de mate-
máticas implican proposiciones que son proposiciones abiertas cuantificadas. Esto sur-
ge de dos tipos de cuantificadores, el cuantificador existencial y el cuantificador uni-
versal.
La proposición (l) utiliza el cuantificador existencial "Para algún x .., que también se
puede expresar como "Para a1 menos un x'' o "Existe un x tal que". En forma simbólica.
este cuantificador se representa como3x. Por lo tanto, la proposición ··Para algún x. p(x)"
se expresa, en forma simbólica, como 3x p(x).
En forma simbólica, la proposición (2) se escribe así: 3x 3y q(x, y). Podemos usar la
notación 3.x, y para abreviar 3x 3 y q(x. y) de mdo que quede como 3x,y q(_x, y).
El cuantificador universal se denota con 'Vx y se lee como "Para todo f, "Para cadax''
o "Para cualquier x''. ••para todox, y" o ...Para todosx y y" se denota con VxVy, que puede
abreviarse como't/x. y.
Si p(x) es como lo definimos antes y usamos el cuantificador universal, podemos cam-
biar la proposición abienap(x) por ta proposición (cuantificada) Vx p(x), una proposición
falsa.
Si consideramos la proposición abierta ,(x): "2x es un entero par'' con el mismo univer-
so (de los enteros). entonces la proposición (cuantificada) 't/x r(x) es una proposición ver-
dadera. Cuando decimos que Vx r(x) es verdadera. queremos decir que no importa con
qué entero (de nuesuo universo) sustituyamos a x en r(x), Ja proposición resultante es
verdadera. También hay que notar que la proposición 3x r(x) es una proposición verdade-
ra, mientras que Vx --, r (x) y 3x ...,,.(x) son falsas.
La variable x de cada una de las proposiciones abiertas p(x) y r(x) es una variable
libre (de la proposición abierta). Si x varía en el unive rso de una proposición abierta, el
valor de ve rdad de la proposición (que se obtiene al reemplazar cada aparición de x)
puede variar. Por ejemplo, en el caso de p(x), vemos que p(S) es falsa. mientras que p(.6)
es una proposición verdadera. Sin embargo. la proposición abierta r(x) se conviene en
una proposición verdadera con cualquier reemplazo (de x) tomado del universo de todos
los enteros. En contraste con la proposición abierta p(x), 3x p(x) tiene un valor de ver-
dad fijo: verdadero. Y en la representación simbólica 3x p(x), la variable x es una varia-
ble acotada, acotada por el cuantificador existencial 3. Esto ocurre también en las pro-
posicionesVx r(x) y 't/x -,r(x); en cada caso, la variablex está acotada por el cuantificador
universal V.
Para la proposición abiertaq(x, y), te nemos dos variables libres. cada una de las cua-
les está acotada por el cuantificador 3 en cualquiera de las proposiciones 3x 3y q(_x, y) o
3x, Y q(x, y).

El siguiente ejemplo muestra la forma en que estas nuevas ideas acerca de los
cuantificadores se pueden usar en conjunción con las conectivas lógicas.
2.4 El uso de cuantificadores 101

En este e.aso, el universo comprende todos los números reales. Las proposiciones abiertas
p(x), q(x). r(x) y s(x) están dadas por
p(x): x2:0
q(x): ,r'a,O
r(x): .r' -3x-4 =0
s(x): .r'-3>0.
Entonces. las siguientes proposiciones son verdaderas.

1) 3x[p(x)/\r(x)]
Esto se debe a que el número 4, por ejemplo. es un miembro del universo ta1 q ue las dos
proposiciones p(4) y r(4) son verdaderas.
2) Vx [ p(x)-+ q(x)]
Si. en p(.r), reemplazamos x por un número real negativo a. entonces p(a) es falsa; pero
p(a) ➔ q(a) es verdadera independientemente del valor de verdad de q(a). Al reemplazar
.r,. en p(x). por un número real no negativo b. vemos que p(b) y q(_b) son ambas verdaderas,
al igual que p(b)-+ q(b). En consecuencia. p(x)--> q(x) es verdadera para todas las susti-
tuciones de x tomadas del universo de todos los números reales y la proposición (cuantifi-
cada) Vx{p(x) --> q(x)] es verdadera.
Esta proposición puede traducirse de las siguientes maneras:

r
a) Para todo número real .:e, si x ~ O, entonces ;;?: O.
b) Todo número real no negativo tiene un c uadrado no negativo.
e) El cuadrado de cualquier número real no negativo es un número real no negativo.

También la proposición 3x(p(x)--> q(x)] es verdadera.

Las siguientes proposiciones son falsas.


1') Vx [q(x)-+ s(x)]
Queremos mostrar que la proposición es falsa. por lo que solamente necesitamos mostrar
Wl con1raejempW; es decir, un valor tk x para el que q(x) ➔ s(x) sea falsa. en lugar de
demostrar algo para todox, como lo hicimos en el caso de ta proposición (2). Si reempla-
zamos x por t. vemos q.;;, q(l) es verdadera y s(l) es falsa. Por lo tanto, q(I)--> s(I) es
falsa. y en consecuencia la proposición (cuantificada) \fx[q(x ) ➔ s(x)] es falsa. [Observe
que.r= 1 no es el único contraejemplo: cualquier número reata cntre - .Jj y .,/3 hará que
q(a) sea verdadera y s(a) sea falsa.]

2') . Vx [r(x)V s(x)]

Aquí hay muchos valores de .:e, e ntre ellos l, ½• - 4, y O. que son contraejemplos de esta
proposición. Sin embargo. al cambiar los cuantificadores, vemos que la proposición 3.x(r(x)
V s(x)] ts verdadera.

3') Vx [r(x)-+ p(x)]


02 Capitulo 2 Fundamentos de lógica

El número real -1 es una solución de la ecuación x?- - 3x - 4 = O, por lo que r(- 1) es


verdadera. mientras que p(- 1) es falsa. Por lo tanto, la elección de -1 proporciona el
contraejemplo que necesitamos para mostrar que esta proposición (cuantificada) es faJsa.
La proposición (3 ') se puede traducir como sigue:
a) Para todo número real x, sir - 3x- 4 = O, e ntonces x ~ O.
b) Para todo númerorealx, si.tes una solución de laecuaci6nx2- 3x-4= 0,entonccs
X ~ Q.

Haremos ahora las siguientes observaciones. Seap(x) cualquier proposición abierta (en
la variable x) con un universo predeterminado no vacío (es decir, eJ universo contiene aJ
menos un miembro). Entonces, si Vx p(_x) es verdadera, también Jo es 3x p(,x), o

Vx p(x) ~ 3,:p(x).

Cuando escribimos V:c p(x) ⇒ 3..x p{x), estamos dicicndo·que la implicación Vx p(.:x)--+
3x- p(x) es una implicación lógica; es decir, 3..x p{x) es verdadera siempre que Vx p(x) sea
verdadera También observamos que la hipótesis de esta implicación es la proposici6n
cuantificada Vx p(,x) y la conclusión es 3x JJ(.x). otra proposici6n cuantificada. Por otro
lado. si 3x p(x) es verdadera, no se sigue que "d'x JJ(.x) deba ser verdadera. Por lo tanto. en
general, 3,: p(x) no implica lógicamente Vx p(x).

Nuestro siguiente ejemplo muestra el hecho de que la cuantificación de una proposi•


ción abierta podría no ser tan explícita como quisiframos.

a) Consideremos el universo de tcxlos los números reales y examinemos las frases:


1) Si un número es racional. entonces es un número real
2) Si x es racionaJ. entonces x es real.
Debeóamos de estar de acuerdo en que estas frases proporcionan la misma infor•
mación. No obstante, también debeñamos preguntarnos si las frases son prOpOSi·
ciones o proposiciones abiertas. En el caso de la frase (2). al menos tenemos la
presencia de la variable x. Pero ninguna frase contiene una expresión como "Para
todo", "Para cualquiera" o "Para cada... La única pista que indica que se trata de
proposiciones cuantificadas universalmente es la presencia del artículo indefinido
"un" en la primera frase. En casos como éste. el uso del cuantificador universal es
implíciro en vez de explícito.
Si p(x), q(x) son las proposiciones abiertas
p(.x): x es un número racional q(x): x es un número real.
entonces debemos reconocer el hecho de que ambas frases son dos modos un tanto
informales de expresar la proposición cuantificada
Vx[p(x)-+q(x)J.
b) Para el universo de todos los triángulos del plano, la frase

Un aiángulo equilátero tiene tres ángulos de 60'1. y viceversa


2.4 El uw de cuantificadores 103

proporciona otro ejemplo de u.na cuantificación implícita. En este caso, el artículo


indefinido "Un" es el único indicio de que podríamos expresar esta frase como una
proposición con un cuantificador universal. Si las proposiciones abiertas

,(t): El triángulo tes equilátero.


a(t): El triángulo t tiene tres ángulos de 60º.
están definidas para este universo, entonces la frase dada se puede escribir en la
fom1a cuantificada explícita
Vr[e(t)<-+a(t)].
También podemos optar por evitar las proposiciones abiertas e(r). a(t) y simple-
mente volver a escribir la frase dada con el cuantificador universal (explícito) ;'Para
todo"como
=
Para todo triángulo ABC, AB BC CA =
si y sólo si cada uno de los ángulos A. B y C es un ángulo de 60º.
e) En un libro de texto de trigonometría común y corriente encontramos con frecuen-
cia la identidad trigonométrica
sen2x + cos2x = l.
Esta identidad no contiene una cuantific.ación explícita y el lector debe comprender (o
habráquellKlicárselo) que está definida para todos los números realesx. Si se especifica
el universo de todos los números reales (o al menos se sobreentiende), entonces esta
identidad puede expresarse mediante la proposición cuantificada (en fonna explícita)

Vx(sen2x + cos2.x = 1].


d) Por 11ltimo, consideremos el universo de todos los enteros positivos y la frase
Et entero 41 es igual a la suma de dos cuadrados perfectos.
Aquí tenemos un ejemplo más en el que la cuantificación es implícita; pero, esta
vez, la cuantificación es existencial. Podemos expresar el resultado de manera más
formal (y simbólica) como

3m 3 n[41 - m'+ n'].

-
El ejemplo siguiente demuestra que el va1or de verdad de una proposición cuantificada
puede depender del universo dado.

Consideremos la proposición abiena p(x): x2 .?: O.


1) Si el universo consta de todos los números rea1es, entonces la proposición cuantifi-
cada Vx p(x) es verdadera.
2) Sin embargo, para el universo de todos los números complejos. ta misma proposi -
ción cuantificada Vx p(x) es falsa. El número complejo i ofrece uno de los muchos
contraejemplos posibles.
No obstante, para cua1quiera de estos universos, la proposición cuantificada3x p(_x)
es verdadera.
104 Capítulo 2 Fundamentos de lógica

En el siguiente ejemplo se ilustra uno de los usos de los cuantificadores en el campo de


las ciencias de la computación.

En el siguiente segmento de programa en PascaJ,n es una variable entera y la variable A es


una tabla A[l], A[2], .. . , A[20] de 20 valores enteros.

For n : = l to 20 do
A[n] : = n•n - n;

Las siguientes proposiciones relativas a la tabla A pueden representarse en forma cuan•


tificada; el universo consta de todos los enteros de 1 a 20, inclusive.

1) Cada entrada de la tabla es no negativa:


'o'n(A[n]2'0).
2) El entero A[20} es la entrada más grande de la tabla:

Vn[(l ,s ns 19)-(A(n] <A(20])].

3) Existen dos entradas consecutivas en A tales que la e~trada mayor es el doble de la


menor:
3n (A(n + 1) =2A[n]).
4) Las entradas de la tabla están ordenadas en fonna (estrictamente) ascendente:
'o'n [(1 s n S !9)--> (A(n] <A[n + 1])].
Nuestra última proposición requiere el uso de dos variables enteras m , n.
5) Las entradas de la tabla son distintas:

'o'm 'o'n [(m • n)--> (A[m] •A(n])], o


'o'm,n [(m < n)--> (A[m] • A [n])].

Antes de continuar, en la tabla 2.21 baremos un resumen y una especie de ampliación


de lo que hemos aprendido acerca de los cuantificadores.
Los resultados de la tabla 2.21 parecerían implicar solamente una proposición abierta.
Sin embaigo, debemos observar que la proposición abiertap(x) de la tabla puede representar
una conjunción de proposiciones abienas, como q(x) I\ r(x). o una implicación de propo-
siciones abiertas. como s(x) ➔ l(x). Si, por ejemplo, queremos determinar cuándo la pro..
posición 3x[s(x) ~ t(x)) es verdadera, observamos la tabla de 3x p(x) y usamos la infor•
mación que apaTece en ella. La tabla indica que 3x[s(x) ➔ t(x)J es verdadera cu:mdo s(a )
~ t(a) es verdadera para (al menos) un a del universo dado.
Analizaremos ahora las proposiciones cuantificadas que tienen más de una proposición
abiena. Sin embaTgO, antes de hacerlo, necesitamos la siguiente definición, comparable a
las definiciones 2.2 y 2.4, relativas a las ideas de proposiciones lógicamente equivalentes
e implicación lógica La definición siguiente se refiere a las mismas cuestiones para el
e.aso de las proposiciones abiertas.
2.4 El uso de cuantificadores 10 5

Tabla 2.21
Proposición ¿Cu.indo es verdadera'? ¿Cuándo es falsa?

3xp(z) Para (al menos) un a del Para cada a del universo,


universo, p(a) es verdadera. p(a) es falsa

Vzp(z) Para cada reemplazo de a del Existe al menos un reemplazo a


universo, p(a) es verdadera. en el universo para el cual p(a)
es falsa
3x , p(z) Para al meoos una elección de Para cada rccmpiazo a del
a del universo, p(a) es falsa,
universo, f'(.a) es verdadera.
de modo que la negación
...,p(a) es verdadera

Vz , p(z) Para cada reemplazo de a del Existe aJ menos un reemplazo a


universo, p(a) es falsa y su en el universo para el cual
negación ~p(a) es verdadera. ~p(a) es falsa y p(a) es
verdadera.

Definición 2.6 Sean p(x), q(x) proposiciones abienas definidas para un universo dado.
Las proposiciones abiertas p(x) y q(x) son (l6gicamenu) equiva/ences, y escribimos
Vz[p(z) <e> q(z)), cuando la bicondícional p(a) +-+ q(a) es verdadera para cada reemplazo
a del universo dado. Si la implicación p(a) ➔ q(a) es verdadera para cada a del uni,,erso,
entonces escribimos V,xfp(x) => q(x)] y decimos que p(x) implica lógicamente q(:c).

Para el universo de todos los ttíángulos del plano, sean p(_;c), q(x) las proposiciones
abiertas

p(z): z es equiangular.
q(x): x es equilátero.

Entonces, para cualquier triingulo particulara (reemplazo dex), sabemos qucp(_a) H q(a)
es verdadera. En consecuencia. V.r{p(z) <e> q(z)].
Observe que aquí, y en general, V.r{p(x) e,, q(z)] si y sólo si V.r{p(x)--> q(z)) y V.r{q(z)
-->p(z)].
También observamos que se puede dar una definición similar a la definición 2.6 para
dos proposiciones abiertas que tengan dos o más variables.

Daremos ahora otro vistazo a la equivalencia lógica de las proposiciones (no proposicier
nes abiertas) conforme analicemos la recíproca, la inversa y ta contraposiliva de una pro-
posición de la fonna v'x [p(z) ➔ q(x)].

Definición 2. 7 Para las proposiciones abiertas p(x), q(x), definidas en un universo dado, y la proposición
cuantificada en forma universal Vx[p(x) ➔ q(z)]. definimos:
06 Capítulo 2 Fundamentos de lógica

1) La contrapositiva de 'lx[p(x) ➔ q(x)] como Vx[ ~q(x) ➔ ~ p(x)].


2) La reciproca de 'lx[p(x) ➔ q(x)J como 'lx[q(x) ➔ p(x)].
3) La foversa de 'lx[p(x) ➔ q(,)] como V.[~ p(x) ➔ ~ q(x)].

Los siguientes dos ejemplos ilustran la definición anterior.

Para el universo de todos los cuadriláteros del plano, sean s(x) y e(x) las proposiciones
abiertas
s(x): x es un cuadrado~ e(x): x es equilá<ero.
a ) La proposición
Vx[s(x)➔ e(x)]

es una proposición verdadera y es lógicamente equivalente a su contrapositiva


Vx[7e(x)-+ 7 s(x)]
ya que [s(a) ➔ e(a)J <e> [ ~ e(a) ➔ ~s(a )] para cada reemplazo a. Por lo tanto,
'lx[s(x)➔ e(x)] ~'lx[7e(x)-> 7S(x)].

b ) La proposición

'lx[e(x)->s(x)]
es una proposición falsa y es la recíproca de la proposición verdadera
'lx[s(x)->e(x)].
La proposición falsa
'lx[7s(x)-> 7 e(x)]
se conoce: como la inversa de la proposición dada Vx[s(x) ➔ e(x)].
Como [e(a) ➔ s(a)] <e> [ ~ s(a) ➔ ~,(a)] para cada cuadrilátero específico a,
tenemos que la recíproca y la inversa son lógícamentc equivalentes; es decir,

'lx[e(x )-> s(x )] ~ 'lx[7 s(x)-+ 7e(x )].

En este caso, p(x) y q(x) son las proposiciones abiertas


p(x): lxl>3 q(x): x>3
y el universo consta de todos los números reales.
a) La proposición Vx[p(x) ➔ q(x)] es una proposición falsa. Por ejemplo, si x ;- -5.
entonces p(_-5) e s verdadera mientras que q(- 5) es falsa. En consecuencia, p(-5) ➔
q(-5) es falsa, al igual que Vx[p(x) ➔ q(x)].
b) Podemos expresar la recíproca de la proposición dada (en la parte a) como sigue:
Todo número rea1 mayor que 3 tiene magnitud
( o valor absoluto) mayor que 3.
2.4 El uso de cuantificadores 107

En forma simbólica, esta proposición verdadera se representa como Vx[q(x) ➔ p(x)].


e:) La inversa de Ja proposición dada también es una proposición verdadera. En forma
simbólica. tenemos Vx[ -.p(_x) ➔ --q(x)), que se puede expresar como

Si la magnítud de un número real es menor o igual que 3,


entonces el propio número es menor o igual que 3.

Y esto es lógicamente equivalente a la proposición (recíproca) dada en la parte (b).


d) En este caso, lacontrapositivade la proposición de la parte (a) está dada porVx[ ~ q(x)
--> ~p(x)]. Esta proposición falsa es lógicamente equivalente a V,jp(x) --> q(x)] y
se puede expresar como sigue:
Si un número es menor o igual que 3, también lo es su magnitud.
e) Consideremos la proposición abiena
r(x): x<-3
que también está definida para el universo de todos los números reales. Las siguien-
tes cuatro proposiciones son verdaderas:
Proposición: Vx[p(x)--+ (r(x) V q(x))]
Conrrapositiva: Vx[7(r(x) V q(x))--+'-ip(x)]
Recíproca: Vx[(r(x) V q(x))--+ p(x)]
Inversa: Vx[7p(x)--+7 (r(x) V q(x))]
En este caso (como la proposición y su recíproca son verdaderas), tenemos que la
proposición
Vx[p(x) ... (,(x) v q(x)}]
es verdadera y también observamos que

Vx p(x)>0>Vx[r(x)v q(x)).

Ahora usaremos de nuevo los resultados de la tabla 2.21 para analizar e l siguiente·
ejemplo.

En este caso, e l universo consta de todos los enteros y las proposiciones abienas r(x), s(x)
eswi dadas por
r(x): 2x+!=5
s(x): x' =9.
Vemos que Ja proposición 3x[r(x) A s(x)] es falsa. ya que no existe un entero a cal que
2a + 1 ::: 5 y a2 = 9. No obstante, e xiste un entero b (== 2) tal que 2b + 1 == 5 y existe un
segundo entero e(= 3 o -3) tal que e' = 9. Por lo tanto, la proposición 3x ~x) A 3x s(x )
es verdadera. En consecuencia. eJ cuantificador existencia] 3.r no distribuye sobre la·
conecti".3 lógica A. Este contraejemplo es suficiente para mostrar que

3x [r(x) l\s(x)]<Í> [3x r(x) l\3xs(x)],


108 Capítulo 2 Fundamentos de lógica

donde~ se lec como "no es lógicamente equivalente a". El ejemplo tambi~n demuestra que

(3.t r(x) /\ 3.ts (x)] ~ 3.t [r(x) /\ s(x)],


donde ,:/;:> se lec como "no implica lógicamente". Así, la proposición
[3.t r(x) /\3.ts(x)]-. 3.t [r(x) /\s(x)]
no es una tautología.
Sin e mbargo. ¿qué podemos decir de la recíproca de una proposición cuanúficada de
esta forma ? En este momento, presentamos un argumento general para cualesquiera pro-
posiciones abiertas (arbitrarias) p(x), q(x) y cualquier universo prescrito (arbitrario).
Si analizamos la proposición

3.t (p(x) /\q(x)J-. [3.tp(x) /\ 3.tq(x)],


vemos que, cuando la hipótesis 3x [p(x) /\ q(x)] es verdadera, entonces existe al menos un
elemento e en el iniverso para el que la proposiciónp(c) A q(c) es verdadera. Por la regla
de simplificación conjuntiva (véase la Sec. 2.3), [p(c) A q(c)) => p(c). Como p(c) es veroa-
dera, obtenemos la proposición verdadera 3x p(.,x). De manera similar, obtenemos 3x q(_x),
otra proposición verdadera. Así, 3.r p(x) A 3x q(x) es una proposición verdadera Como
3x p(_x) A 3:c: q(x) es verdadera siempre que 3:c:{p(x) A q(x)] lo sea, esto implica que

3.t [p(x) /\q(x)]:} [3.tp(x)/\ 3.t q(x)].

Otros argumentos similares a1 del ej emplo 2.43 muestran las equivalencias lógicas y las
implicaciones lógicas enumeradas en la tabla 2.22. Es posible obtener muchas otras equi•
valencias e implicaciones lógicas además de las que aparecen en esta tabla
Nuestro siguiente ejemplo enumera algunas de istas y demuestra cómo se pueden veri•
ficar dos de ellas.
Tabla 2.22
Equivalencias e implicaciones lógicas para proposiciones
cuantifiadas de una variable

Para un universo dado y cuaJesquiera proposiciones abiertas p(x), q(x) en la variablcx,


3.t [p (x)/\ q(x)]:} [3.t p (x) /\ 3.t q(x)]
3.t [p(x) v q(x)]~ [3.t p (x) V 3.t q(x)]
V,; [p(x) /\q(x)] ~ [Vx p(x)/\ Vx q(x)]
[Vx p(x)V Vx q(x)] :} Vx [p(x) V q(x)J

Sean p(.x ), q(x) y r(x) proposiciones abiertas para un universo dado. Enconuamos las si-
guientes equivalencias lógicas. (Son posibles muchas más.)
1) Vx [p(x) /\(q(x) /\ r(x))]~ Vx [(p(x)/\q(x)) /\ r(x)]
Para moscrar que esta proposición es una equiva1encia lógica procedemos de la
mane ra siguiente:
Para cada a del universo, consideramos las proposiciones p(a) /\ (q(a) A r(a)) y
(p(a) A q(a)) A r (a). Por la ley asociati va de A, tenemos que

p (a) /\ (q(a) /\ r (a)) ~ (p(a) /\ q(a)) /\ r (a).


2.4 El uso de cuantificadores 109

En consecuencia, para las proposiciones abienas p(x) A (q(x) A ,(x)) y (p(x) A q(x))
/\ r(x}. se sigue que

Vx [p(x)A (q(x)Ar(x))]<c>Vx [(p(x)/\q(x))/\r(x)].

2) 3x [p(x)-+ q(x)]<c> 3x [,p(x) V q(x)]


Para cada e del universo, se sigue del ejemplo 2.7 que

[ p(c)-+ q(c)]<c> [,p(c) V q(c)].

Por lo tanto, la proposición 3x[p(x) ➔ q(.x)J es verdadera (respectivamente, falsa) si


y sólo si la proposición 3x{-.p(x) V q(x)J es verdadera (respectivamente, falsa). de
modo que

3x [ p(x)-+ q(x)]<c> 3x [,p(x) V q(x)].

3) Otras equivalencias lógicas que encontraremos con frecuencia son las siguientes.
a) Vx-np(x)<c>Vxp(x)
b) Vx ,[p(x) A q(x)]<c>Vx [, p(x) v,q(x)]
e) Vx.[p(x) V q(x)]<c>Vx [ ,p(x) A , q(x)]

4) Los resultados para las equivalencias lógicas de 3(a), (b) y (c) siguen siendo válidos
cuando todos los cuantificadores universales se reemplazan por cuantificadores
existenciales.

Los resultados de las tablas 2.21 y 2.22 y de los ejemplos 2.43 y 2.44 nos ayudarán
ahora con un concepto muy imponante. ¿Cómo negamos las proposiciones que implican
una sola variable?
Consideremos laproposiciónVxp(x). Su negación, -.{Vxp(x)]. puede enunciarse como
"No ocurre que para todox se cumplap(x)". Ésta no es una observación muy útil, así que
volveremos a analizar -.[Vx p(x)]. Cuando esta proposición es verdadera, entonces Vx
p(x) es falsa, por lo que, para algún reemplazo a del universo, -.p(a) es verdadera y 3x
-.p(x) es verdadera. En fonna recíproca, siempre que la proposición 3x -.p(x) sea verda-
dera, sabemos que -.p(b) es verdadera para a1gún elemento b del universo. Por lo tanto, Vx
p(x) es falsa y ~[Vx p(x)] es verdadera. Así, la proposición ~[Vx p(x)] es verdadera si y
sólo si la proposición 3x -.p(_x) es verdadera. (Un análisis similar muestra también que
~[\lx p(x)] es falsa si y sólo si 3x ~p(x) es falsa.)
Esw observaciones conducen a la siguiente regla para negar la proposición Vx p(x):

, [Vx p(x)]<c> 3x ,p(x).

De manera similar, la tabla 2.21 nos muestra que la proposición 3x p(x) es verdadera
(falsa) precisamente cuando la proposición Vx -.p(x) es falsa (verdadera). Esta observa-
ción da lugar a una regla para negar la proposición 3x p(x):

,[3xp(x)]<c>Vx , p (x).
Capítulo 2 Fundamentos de lógica

Estas dos reglas de negación, junto con otras dos que se siguen de ellas, aparecen en la
tabla 2.23 como una referencia que le resultará de utiJidad.

Tabla 2.23
Reglas para negar proposiciones
con un cuantificador

7[Vxp(x)]~ 3.np(x)
7[3xp(x)]~Vnp(x)
7[Vx 7p(x)]~ 3x 77p(x)~ 3x p(x)
7[3x 7p(x)]~Vx 77p(x)~Vx p(x)

Usaremos estas reglas para negar las proposiciones cuantificadas del siguiente ejemplo.

Aquí encontramos la negación de dos proposiciones; el universo comprende todos los


enteros.
1) Sean p(x) y q(x) dadas por

p(x): x es impar.
q(x): x'- I es par.
La proposición "Si x es impar, entonces x2- 1 es par' puede simbolizarse como
Vx(p(x) ➔ q(x)]. (Ésta es una proposición verdadera.)
La negación de esta proposición se determina de la manera siguiente:

7 [Vx (p(x) - q(x))]~ 3x [7(p(x)-+q(x))J


~ 3x [7(7p(x) V q(x))]~ 3x [77p(x)/\ 7q(x)J
~ 3x [p(x) /\ 7q(x)J

En palabras. la negación dice "Existe un entero x tal que x es impar y x2- L es


impar (es decir, no es par)". (Esta pro¡x,sición es falsa.)

2) Como e n el ejemplo 2.43, sean r(x) y s(x) pro¡x,siciones abiertas.

r(x): 2x + 1 = 5.
s(x): x' = 9.
La proposición cuantificada 3:c[r(x) A s(x)] es falsa. ya que asegura la existen•
= =
cia de al menos un entero a tal que 2a + 1 5(a 2) y a1 = 9 (a = 3 o -3). En
consecuencia, su negación

7[3x (r(x)/\s(x))]~ Vx [7(r(x) /\s(x))]~ Vx [7r(x) V7s(x)]


es v~rdadera. Esta negación puede expresarse como "Para cada enterox, 2x + J ~ 5
o x',!, 9".

Como una proposíción matemática puede contener más de un cuantificador, a conti-


nuación presentaremos algunos ejemplos y haremos algunas observaciones acerca de es•
tos tipos de proposiciones.
2.4 El uso de cuantiftcad0<es 111

Aquí tenemos dos variab1es reales x, y, por lo que el universo consiste en todos los mime.
ros reales. La ley conmutativa para la suma de números reales puede expresarse como
VrVy(r + y •y +r).
Esta proposición también puede expresarse como
Vy Vr (r +y = y+ r).
Así mismo, en el caso de la multiplicación de números reales, podemos escribir
Vr Vy (ry = yr) o Vy Vr (ry =yr).
Estos dos ejemplos indican el siguiente resultado general. Sip(x, y) es una proposición
abiena en las dos variablesx, y (con el mismo universo prescrito parax y para y; o bien, un
primer universo para.ry otro paray),entonces la proposición Vx Vy p(x, y) y Vy Vxp(x,y)
son lógicamente equivalentes; es decir, la proposición Vx Vy p(x, y) es verdadera (respec-
tivamente, falsa) si y sólo si la proposición 'tly Vx p(x, y) es verdadera (respectivamente,
falsa). De aquí que
Vx Vy p(x,y)~ Vy Vr p(x,y).

~ Cuando analizamos la ley asociativa para la suma de números reales, cnconu-amos que
para cualesquiera mlmeros reaJcs .x, y, z.

r +(y+ z) = (r + y) + z.
Al usar los cuantificadores universales (con el universo de todos los n\1mcros reales},
podemos expresar esto como

VxVy Vz [r + (y+ z) • (x + y) + z) o
VyVrVz [x +(y+ z) • (r +y)+ z].

De hecho, hay 3! = 6 maneras de ordenar estos tres cuantificadores universales y todas


estas proposiciones cuantificadas son lógicamente equivalentes entre sf.
Esto es cierto también para todas las proposiciones abienas p(x, y. 4} y, para abreviar la
nolación, podríamos escribir, por ejemplo,

Vx,y,z p(x,y,z)~Vy,x,z p(x,y,z)~Vr,z,y p(r,y,z),

para describir la equivalencia lógica para eres de las seis proposiciones.

Los"resultados de los dos ejemplos anteriores aparecen con frecuencia en los textos de
álgebra superior y en muchos libros de cálculo, sin cuantificadores (en palabras o en for-
ma simbólica}. Por lo tanto, si vemos, por ejemplo, la propiedad asociativa de la suma para
los números reaJes, dada simplemente como

r + (y+ z) = (x +y)+ z,
212 Capitulo 4 Propiedades de los enteros: Inducción matemática

10, Para cualquier x E R,lxl=J:;i' •{_:::::~}. y-1•1s x "' 14 Por lo tanto, I>+ 11' =
<.x+if =x'•'2x1•r sx'+x'+21•111l•r • l•I' +21,11,1 • 111'=<1•1• l1i>',1l>+1I'•
<l•I +IJ,I>' =>lx+1J S Jxl •111, para cualesquiera.,. 1 E R.
~ q u e sin E z•,n i!:: 2y.r1,.xi, .. • ,z.E R.entonc:es
lx, +x,+ ·· · +z..j s jx,I+lx~ + · •· +jx..j.
11. ~ una defWcióo rccuniva de cada una de lu siguientes succsioocs de eoteros.
a) 2,4,16,256, ... (o,2,2', (2')'.((2')')', ...)
b) 2,4, 16,65536, ... (o, 2,2',2"'>,z<""'>, ...)
12. Defina la sucesión de n11mcros racionales ao, a1o aba,. ... , en forma recursiva como
l) 0o=0,a1 • 1; y
a.-1+ (n - l)a.-2
2) Sin~2.o.. • n .
Para todo n E N, demuestre que O s a. s 1.
13. Defina. la sucesión de nómeros c:ntc:ros a., a 1, a:. Ot, • .• , en forma recursiva como
1) 0o - l,a1•l,a2=l; y
2) Sin ~3, a,. • o..-, +o..-,.
Demuestre que ...,., cvir para todo, n "'o.
14. Paran ~ O, sea F. el n~mo nllmero de ñboo.ac:c:i Dcmucstrc que

&+~ +fi+···+&•Íft
,__ • &n-L
15. Si n e z·. demuestre que ! .F¡ F... = Fi.,.
16. Paran E z•, demuestre que

.
17. Demuestre que para cualquier entero pootivo n,
. ~!t,,_
-ft X •
F~.
1-7.
18. Como en el ejemplo 4,18, sean 4, L,, L,. .•. los OWDer<>S de Lucas, donde (1) Lo= 2, L, • 1: y
(2) L..i= 1-1 + L. paran e!: O. Sin ;?:; 1, demuestre que

L;+L;+L;+ ... +L!-L..L..., -2.


19. SinEN,demuestrequc~F..z• ~-L,,.
20. Dt u.na definkión recursiva del conjunto de
a) los ef\tel'OS pares positivos. b) los enteros pares no negativos.
21 . Uno de los usos nás comunes de la dcfinicióo recursiva de los conjuntos ocurre en la defini-
ciéo de las fómuJas bim fo,,,,,,¿,,, en distintos sistemas matcI!WÍ<OS. Pl>r ejemplo, en d
esuidio de la lógica póde,nos def<nir las fórmulas bien formada$ como sigue:
1) Cualquier proposición primidvap. la tautología 70 y la 00ntradicci6n F 0 son fórmulas bien
formadas; y,
2) Si p, q son fórmulas bien formadas. ea1ooces tambitn lo son
l) (-,p) 1) (pVq) 11) ( p A q)
lo) (p-q) •l (p ++q)
Mediante esta deftn.ición recursiva vemos que. para las proposiciones primitivas p, q. r, la
2.4 El uso de cuantificadores 113

La traducción de las proposiciones matemáticas (postulados, definiciones o teoremas)


a su forma simbólica puede ser litil por dos imponantes razones.
1) Hacerlo nos obliga a ser muy cuidadosos y precisos con el significado de las propo-
siciones, con el significado de frases como ..Para todo r· y ..Existe un r·. y con el
orden en que aparecen estas frases.
2) Dcsputs de traducir una proposición matemática a su forma simbólica, deberemos
aplicar las reglas aprendidas para detenn.inar proposiciones relacionadas con ella.
como la negación o, en los casos adecuados, la contrapositiva. la recíproca o la
inversa.
Nuestros llltimos dos ejemplos ilustran esto y, al hacerlo. extienden los resultados de la
tabla 2.23.

Sean p(x. y), q(,x. y) y ,{x. y) ltCS proposiciones abicnas, y las variables x. y se n:cmplazan
de cieno(s) univcrn,(s) prcscrito(s). ¿Cuál es la negación de la siguiente proposición?

'!fx 3y [( p(x,y)l\q(x,y))-+ r (x,y)]


Tenemos que

-,['!fx 3y [(p (x,y) l\ q(x,y))-+ r(x,y)Il


~ 3x [-,3y [(p(x,y) /\q(x,y))-+ r(x,y)Il
~ 3x '!fy -,[(p(x,y) /\q(x,y))-+ r (x,y)]
~ 3x '!fy -,¡-,[p(x,y) /\q (x,y)]Vr(x,y)]
~ 3x '!fy ¡-,-,[p(x,y) /\q (x,y)J/\-,r(x,y))
~ 3x '!fy [(p(x,y) /\q (x,y)) 1\-,r(x,y)]

Supongamos que intentamos establecer la validez. de un argumento (o un teorema ma-


temático) para el cual

'!fx 3y [(p (x,y)/\q(x,y))-+ r(x,y)]


sea la conclusión. Si intentamos demostrar el resultado por medio de la demostración por
contradicción, deberemos suponer, como premisa adicional, la negación de esta conclu-
sión. Por ello, nuestra premisa adicional seria la proposición

3x 'lfy [(p (x,y) /\q(x,y)) 1\-,r (x,y)].

Por último. veamos cómo negar la definición de límite, un concepto fundamental en el


cilculo.

2.51 En cáJculo, se estudian las propiedades de las funciones reales de una variable real. (Ana~
lizaremos las funciones en el capítulo 5 de este libro.) Entre estas propiedades está la
existencia de límites; al respecto. encontramos la siguiente definición: lím..,_ /(x) =L si (y
sólo si) para cada f > O ex.is te una 6 > O tal que, para cada x (donde j(x) estt definjda), (0 <
lx - al < Ó) ➔ CIJtx)- LI « ) .
114 Capitulo 2 Fundamentos de lógica

Esto se puede expresar en forma simbólica como

~f(x) = Lec>V,>O 36>0 Vx[(0< lx - a l <6)-(lf(x) -Ll<•)J.


[En este caso, el universo comprende los números reales, y sólo consideramos aquellas
va1orcs reales dcx para los que/(x) esté definida. Además, los cuantificadores VE> Oy
3 0 > O contienen ahora alguna información restrictiva} Entonces, para negar esta defini-
ción, haremos lo siguiente (hemos resumido algunos pasos):

)íP,f(x)*L
ec>,[V, >O 36>0 Vx[(0< lx - al <6)-(lf(x)-LI <•)]]
ec> 3,>0 V6>0 3n[(O< lx-al<6)->(lf(x) - L/<•))
ec> 3,>0 V6>0 3x,[,(O< lx-al<6)V(lf(x) - LI<•))
ec> 3, > O V6 > O 3x [,,(O< lx - al< 8) /\,(1/(x) - LI < <)]
ec> 3,>0 V6>0 3x[(O< lx-al<8)/\(lf(.r)-Ll2:,)]
Traducido en palabras, tenemos que límr--w/(x) ,;J:. L si (y sólo si) existe un número
(real) positivo'i'tal que para cada número (real) positivo ó, existe un valor x [donde/(x)
+
está definida] tal que O< l x - a 1 < 6 (es decir, x a y su1iistancia de a es menor que a)
pero /f(x)-L / 2: • [es decir, el valor de/(x) difiere del de L por al menos '].

EJERCICIOS 2.4 1. Scanp(.x), q(x) las siguientes proposiciones abiertas.

p(x): xs3 q(x): x+l esimpar.

Si el universo consta CM: todos los enteros. ¿cuáles son Jos valores de verdad de las siguientes
proposiciones?
a) p(I) b) q(l) e) -ip(3)
d) q(6) •l p(7)Vq(7) f) p(3)/\q(4)
g) p(4) b) -i(p(-4) V q(-3)) 1) -ip(-4)/\,q(-3)
2. Sean p(x). q(x) las proposiciones definidas en el ejercicio 1. Sea r (x) la proposición abierta
"x > O". De nuevo, el universo está formado por todos los enteros.
a) Determine los valores de verdad de las siguientes proposiciones.
O p(3)V[q(3)v,,(3)] H) -ip(3)/\[q(3)V,(3)]
ili) p (2)-[q(2)- ,(2)] i>) [p(2) /\q(2)J - ,(2)
v) p(O)-[,q(- 1)-,(1)] vi) [p(-1) - q(-2))-,(- 3)

b) Determine todos los valores de x para los cuales (p(:t) A q(.r)] A r {.r) da como resultado
una proposición verdadera.
e) Encuentre los cinco enteros positivos x más pe.queftos para los que la proposición abiena
p(,x) ---t ( --q(x) A r(x)] da como resultado una proposición verdadera.
3. Sea p{x) la proposición abierta ··.t'= 2.r". donde el universo comprende todos los enteros.
Determine si cada una de las siguientes proposiciones es verdadera o falsa.

a) p(O) b) p(l) e) p(2)


d) p(-2) t) 3xp(x) f) Vxp(x)

4. Considere el universo de iodos los polígonos con eres o cuatro lados y defina las siguientes
proposiciones abienas para este universo.
2.4 El uso de cuantifteadores 115

a (x): todos los .ingu.los internos de x son iguales


t'(x): x es un triángulo tquilátero
11(,r): iodos los lados de z son igu.,Jcs
i(,r): x es un trián¡ulo isósceles
p(x): x tiene un án¡ulo interno mayor que 1800
q(x): x es un cuadrilMero
r{x): x es un rcctin.gulo
s(x): x es un cuadrado
l(x): x es uo triángulo

Traduzca cada una de las siguientes proposiciones en una frue en espaftol. y determine si la
proposición es verdadera o falsa.

a) Yz[q(z) Y r(z)J b) Yz[i(z)-•(z)l


<) :1.t(r(z)/\p(z)] d) Yz[o(z ) - •(z)]
•l Yz[(o(.r)/\r(z))-,(z)] r) :1.t(q(z) /1,, (z))
g) :i.t[, (z ) /1-is(z)] b) Yz[h(z)- •(z )]
1) Yz[(h(z) /\ q (z))---n(z)] j) :1.t(q(z)/\p(z)]
k) Yz[r(z) --ip(z)] 1) Yz[o(.r)-(e(z)Yr(z))]
m) Yz[s(.r)-(o(z)/\h(z))] a) Yz[r(z)- (o(z) ++h(z ))J
5. El grupo de mednica cuántica del Profesor Olmedo está fo,mado por 29 estudiantes, de los
cuales exactamente
1) tres estudiantes de física estin en su penúltimo aoo:
2) 005 estudiantes de ingeniería el&:bica estúi en su penúltimo 800;
3) cuatro C$lU<liantes de matemáticas esain en su penúltimo afto;
4) doce estudiantes de física están en su último año;
5) cuatro C$Wdiantcs de in¡enierfa e l6ctrica están en su último año:
6) dos csrudiantcs de ingeniería c16ctric.a son de posgrado; y
7) dos csrucliantes de matcmiticas son de posgrado.

Considere las siguientes proposiciones abiertas.

c(x): El estudiante.x CSU en la clase (es decir, la clase de mecánica cuántica del
profe,or Olmedo ya descrita).
j(x): El estudiante x cstf en w pemlltimo ano.
s(x): El estudiante x está en su último afk>.
,<z): El csnidian1< z es de posgrado.
p(,.r): El esrudiante x esd. en la especialidad de física.
~(..r): El estudiante x est! en la especialidad de ingcnie:rfa el6;trlca.
m(x): El estudian1e x está en la especialidad de matenú.ticas.

Escriba cada una de las siguientes proposiciODC5 en tmnioos de ruantificadores y las proposi-
ciones abiertas c(.r). j(,x). s(.x). g(.r), p(_..r). c(x) y m(x). y detenninc cuáles de las siguientes
proposiciones son verdaderas o falsas. En este caso, el universo está formado por los 12.500
estudiantes imcritos en la uni\.-ersidad donde imparte clases el profesor Olmedo. Además. en
esta universidad eada estudiante tiene solamente una especialidad.
a ) En la clase existe un estudiante de matemfticas que está en w pem11timo al\o.
b ) En la clase existe un estudiante del último afio que no está en la especialidad de matemáiicas.
116 Capítulo 2 Fundamentos de lógica

e) Todo estudiante de la clase está en la especiaHdad de matemáticas o física.


d) Ningún estudiante de posgrado en la clase escá en la especialidad de física.
e) En la clase. todo estudiante del óltimo año está en la especialidad de física o de ingcnieña
eléctrica
f) Algl1n estudiante de posgrado de esta universidad no está en la especialidad de matemáticas
ni en la de física.
6. Seanp(x. y), q(,x. y) las siguientes proposiciones abiertas:
p(x,y): x'2cy q(x ,y): x +2<y
Si el universo para cadax, y está formado por todos los números reales. determine el valor de
verdad de cada una de las siguientes proposiciones.
a) p(2. 4) b) q(l, ,r) e) p ( -3, 8)/\q (l, 3)
d) p(!, l)y-,q(- 2, -3) e) p(2, 2)- q(l , 1) 1) p (l,2)--,q(l,2)
7. Para el wtivcrso de los cntero5, scanp(.:r), q(.r), Tµ), s(x) y l(z) las siguientes proposiciones abiertas.
p(_x): x>O
q(x): x es par
r(x): x es un cuadrado perfecto
s(x): x es (exactamente) divisible entre 4
l(x): x es (exactamente) divisible entre 5

a) Escriba las siguientes proposiciones en forma simbólica·


i) Al menos un entero es par.
U) Existe al menos un entero positivo que es par.
iii) Si x es par. entonces x no es divisible entre 5.
iv) Ning,ln entero par es divisible entre S.
v) Existe al menos un entero par divisible entre S.
vi) Si x es par y x es un cuadrado perfecto. entonces x es divisible entre 4.
b) Determine si cada una de las seis proposiciones de la pane (a) es verdadera o falsa. Para
cada proposición falsa. dé un contraejemplo.
e) Exprese en palabras cada una de las siguientes representaciones simbólicas.
1) Vx(, (x) -p(x)J ü ) Vx[s(x)-q(x)]
W) Vx (s(x)- -,1(x)] iY) ,h [s(x) /\-,,(x)]
•l Vx ¡-,, (x)y-,q(x) V s(x)J
d) Proporcione un contraejemplo para cada proposición falsa de la parte (c).
8 . Sean p(x), q(x) y r(x) las siguientes proposiciones abiertas.
p(x): z2-8x+IS ~ 0
q(x): x es impar
, (x): x>O
Para el universo de los enteros. determine la verdad o falsedad de cada una de las siguientes
proposiciones. Si una proposición es fals.a. dé un contraejemplo.
a) Vx (p(x)--+ q(x)J b) Vx [q(x)- p(x)]
e) ,!x [p(x)-q(x)] d) ,!x [q(x)--+ p (x)]
e) ,!x (,(x)/\p(x)l 1) Vx[p(x)- ,(x)J
e) ,!x [, (x)-p(x)] b) Vx[-,q(x)--,p(x)]
1) ,!x [p(x)-(q(x)/\, (x))] j) Vx[(p(x)yq(x))-, (x)]
9. Seanp(x). q(x) y r(x) las siguientes proposiciones abienas.

p(x); x'-1x + 10~0


q(x); r-Zx - 3~0
r(x): x<O
2.4 El uso de cuantificadores 117

a) Determine la verdad o falsedad de las siguientes proposlciooes, en las qJC el universo csd
formado pe, todo< los cmoros. Si una proposición es falsa, ~ un cootnltjemplo o explk:ación.
1) Yz [p(z)-+,r(.r)] i) 't'z [q(z) -+,(z)J
111) 3.z [q(z)-+,(z)] IY) 3.z [p (z)-+ r(.r)]
b ) Determine las te$pUe:SW de la pane (a) cuando el universo consta de todos los enteros
positivos.
e) Determine las respucsw de la parte (a) cuando el universo consta llnicamcnte de los ente-
ros 2 y 5.
1 O. Para el $iguicnte se¡mcnto de pro¡rama en Pascal,m y n son variables cnteru. La variable A es
un> tabla de dos dimco,ionesA (1.1),A (1,2), ... ,A [I,20), ... , A (10 , I), . . , A [10.20).con
10 füas (indexadas de I a 10) y 20 columnas (indclldas de 1 a 20).
Por • : - 1 to 10 do
ror o : • 1 to 20 do
A(■,D] ; • ■ + 3 •0;

Escriba las siguientes proposiciooes co forma simbólica. (FJ universo de la variable m contic•
ne clnicamente los cnterouiel I al 10 inclmivc: paran, el universo consta de los enteros del 1
al 20 inclusive.)
•) Todas las entradas dc.4 son positivas.
b) Todas las entradas de A son positivas y menores o iguales que 70.
e) Algunas de lu entradas son mayores que 60.
d ) Las entradas de cualquier fila de A tienen un orden (estrictamente) ascendente.
e) Las entradas de cualquier columna de A tienen un orden (estrictamente) uccndcnte.
() Las entradas de lu primeras lCC$ filas de A son distintas.
&) Las eocradas de cualesquiera tres filas consecutivas de A son distinw.
h) Para dos filas consecutivas cuale5qUien de A, la suma de las entradas de la segunda fila
(aquella que tiene el índice de fila mis ¡randc) es 20 unidades mayor que la suma de las
entradas de la fila anterior.
11. Identifique las variables acotad.as y las variables libr'cs de cada una dc,..las si¡uicntes expresio-
nes (o proposiciones). En la parte (a), el universo comprende todos los números reales. excep-
to ... - 5"'2. - 3"2,-.n. "'2. 3"2. 5.n. ... En losdemáseasos.el wúvenoesti formado por
todos los números reales.
a) 't'z 't'y[sec' z - sec' y = tan' z - tan' y J
b) 't'y3z[cos(z +y)• seo(z - z)J
e) 3.% 3y[.r' -y'•z]
d) 3.z[zy = y]
12. a ) Seap(x, y) la proposición abierta '"x divide a y"; el urtiverso para cada una de las variables
x. y es el conjunto de todos tos enteros. (En este contexto, '"divide"' si¡nifica "divide Cll.ac•
ta.mente".) Determine el valor de verdad de cada una de las proposiciones siguientes; si una
proposición cuantificada es (,ah.a, propotciooe una explicación o un contraejemplo.
0 p(3, 7) 1) p(7, 3) 11) p (3, 27)
lv) 't'y p (l, y) v) 't'zp(z , O) rl) 't'zp(z,z)
YII) 't'y 3.zp(z,y) ..ill) 3y 't'z p(z,y)
b ) Yz 't'y[(p(z ,y) /\p(y, z)) - (z • y)]
•l 't'z Yy 't'z[(p(z ,y)l\p(y, z))-+p(z, z)J
b) Determine cuáles de las 10 proposiciones de la parte: (a) cambiarán su valor de verdad si el
universo de cada una de las variables x. y se restringe solamente a los enteros positivos.
e) Oetcnnine e l valor de verdad de cada una de las siguientes proposiciones. Si la proposición
es falsa, proporciooe una explic:ación o contraejemplo. [fJ univcno para cada x. y es como
en la pane (b).J
i) 't'y 3.zp(z,y)
h\ ~ v V-r n(Lv)
118 capítulo 2 Fundamentos de lógica

13. Suponga que p(,x. y) es una proposición abiena en la que el universo para cada .r, )' está forma-
do solamente por tres enteros: 2, 3 y 5. Entonces. la proposición cuantificada 3y JX.2, y) es
lógicamente e.quivalente ap(2. 2) V p(2, 3) V p(2. 5). La proposición cuantificada3x Vy p{x. y)
es lógicamente equivalente a (p(2, 2) t,, p(2, 3) t,, p(2, 5)] V {p(3, 2) t,, p(3, 3) t,, p(3. 5)1 V
[p(5, 2) /\ p(5, 3) /\ p(S, 5 )). Use conjunciones o disyunciones para expresar las siguientes
proposiciones sin cuantificadores.
a) 3xp(x, 5) b) 'fx p(x,3) e) Vyp(2,y)
d) 3x 3y p(x , y) e) \fx'fy p(x,y) f) Vy 3xp(x,y)

14. Sean p(n). q(n) las proposiciones abiertas

p(n): n es impar; q(n): n2 es impar

en el universo de los enteros. ¿Cuáles de las siguientes proposiciones son lógicamente equiva-
lentes entre sí?
a) Si el cuadrado de cualquier entero es impar, entonces el entero es impar.
b) "In (p(n) es necesaria para q(n)]
e) El cuadrado de cualquier entero impar es impar.
d ) Existen a1gunos enteros cuyos cuadrados son impares.
e) Dado cuaJquier entero cuyo cuadrado sea impar. ese entero también es impar.
Í) Vn[ ~p(n) ➔ ~q(n)J
g) Todo entero con un cuadrado impar es impar.
b) Todo entero con un cuadrado par es par.
i) 'vn[p(n) es suficiente para q(n))
1 S. Para cada una de las siguientes parejas de proposiciones, detennine si la negación propuesta es
la correcta Si es correcta. dccermine cuál es verdadera: la proposición original o la negación
propuesta. Si la negación propuesta es incorrecta. escriba una versión corregida de la negación y
determine a continuación si la proposición original o la versión corregida de la negación es
verdadera
a ) Proposición: Para todos los números rcales.x, y, si x2> f . entonces.x> y.
Negación propuesta: Ex.isten números reales .x, y tales que :t'> Y pero x s y .
b) Proposición: Ex.isten números rcaJeu, y tales que.xy y son racionales pcrox + yes irracional.
Negación propuesta: Para lodos los números rcalesx. y. si x +yes racional, entonces x y y
son racionales.
e) Proposición: Para todo número rcalx. six no es O, entoncesx 1ienc un inverso muliplicativo.
Negación propuesta: Existe un número real distinto de cero que no tiene un inverso
multiplicativo.
d ) Proposición: Existen enteros impares cuyo producto es impar.
Negación propuesta: El producto de cualesquiera dos enteros impares es impar.
e) Proposición: El cuadrado de iodo número racional es rae-ional.
Negación propuesta: Existe un número real x tal que si x es irracional. emonces .r es
irracional.
16. Escriba la negación de cada una de las siguientes proposiciones como una frase en español sin
notación simbólica (En es1e caso, el universo cons1a de todos los estudiantes de una universi-
dad donde impane clases el profesor Linares.)
a) Todo estudiame del grupo de Pascal del profesor Linares está en la especialidad de ciencias
de la computación o matemáticas.
b) Al menos un estudiante del grupo de Pascal del profesor Linares est.á en la especialidad de
historia.
e) Un es1udiante del grupo de Pascal del profesor Linares ha leído todos sus artículos de
investigación sobre estructura de datos.
2.4 El uso de cuantificadores 119

17. Escriba la negación de c:ada una de las sigu.íentes proposiciones verdaderas. Para las panes (a),
(b) y (e:), el universo con,ta de todos los enteros; para las partes (d) y (e), el universo abarca
todos los m1meros reales.
a) Pan todo entero n, sin no es (ex.actamcrUe) divisible entre 2. entonces n es impar.
b) Si d cuadrado de un cruero es impar, cntooces el entero es impar.
e) Si t. m. n son enteros tales que k - m y m - n son impares. entonces k - n es par.
d) Si.z: es un mlmcro real tal ~x2> 16, entonces x <-4 ox >4.
e) ParatodomlmcrorcaJx,si lx - 31 <7,cntonces -4 <:x< 10.
18. Niegue y simplifique Jo siguiente.

•l 3.t (p(x) V q(x)] b) Vx[ p(x) /\-,q(x)]


e) Vx [p(x)-+q(x)l d) 3.t[(p(x)Vq(x))-+r(z)]

19. Para cada una de las siguientes prop0$icioncs (y universos) enuncie la recíproca. la im·cna. y la
coott3pOSitivL Determine tambitn el valor de verdad de cada proposición dad.a, así como los
valores de verdad dcsu rcdproca. su inversa y su cootrapositiva. (En este caso, -divide.. signi•
fica ..divide: exactamente.. y ..divisible.. significa -divisible exaaamcnte".)
a) [El universo comprende todos los enteros positivos.)
Si m > n.entonc:es m1 > n1•
b) [FJ u.niveno c:ompreodc todos los enteros.)
Si a> b, entonces ti'-> Ir.
e) [El univeno com¡xende todos los enlffl>$.)
Si m divide a n y n divide a p , entoocesm Wvidc ap.
d) [El universo c:omprcndc todos los números reales.)
Vx{(r > 3) ➔ (x' > 9))
e) [El univeno comprende todos los enteros.)
Todo entero que es divisible entre 12 wnbitn es divisible entre 4.
O [El universo comprende todos los mlmeros reales.)
º·
Para todo mlmero real X, si r + 4.x - 21 > entooccsx> 3 0.1"<-7.
20. Vuelva Aescribir cada una de tu siguientes proposiciones (con los urúvcrsos dados) como una
implicación de la formasi~ntooccs. Escribadesputs larecíproca. lainversa y lacontrapositiva
de I, implicación. Pan cada r<SUlrado de las pan,$ (a) y (d), clt el valor de verdad de la
implicación y los valores de vcrdacl de la recíproca. la inversa y la conuapositivL (En la parte
(a)...divisibilidad" significa tener un resto O.)
a) [El universo comprende todos los enteros positivos.]
La divisibilidad entre 21 es una condición suficiente para la divisibilidad entre 7.
b) (FJ universo abarca todos &os residentes actuales de Estados Unidos.)
Conw con un paquete considerable de acciones en la bolsa es u.na condición ncocsaria pan
que una persona sea feliz.
e) (FJ. univcr$0 comprende todas las serpientes que reptan actualmente en las selvas de Asia.)
El hecho de ser una cobra u una condición ¡ufic:iente para que una serpiente sea peligrosa.
d) (El universo csti formado por todos los na.meros complejos.)
Para cada número complejo z. el hecho de que z sea real C$ necesario para que t sea real.
21. Para las si¡uicntcs proposiciones. el universo abarca todos los enteros distintos de cero. Dccer-
m,inc el valor de verdad de cada proposición.

a) 3.t 3y [.ry • l] b) 3.t Vy [.ry • l] t) Vx3y[.ry • I]


d) VxVy[scn'x +cos'x • sen', +cos'y]
e) 3.t 3y [(2r +y= S) l\ (x - 3y = - 8)]
f ) 3.t 3y [(l< - y= 7) /\ (2< +4y • 3)]

22. Repica el ejercicio 21 para el universo de todos los na.meros reales diferente, de c:ero.
120 Capitulo 2 Fundamentos de lógica

23. En la aritmética de los números reales. existe un número real, O, llamado el neutro de la suma.
puesto que a + O = O + a = a para cada mlmero real a. Esto se puede expresar en forna.
simbólica como

3zVa[a+z=z+a;;;;a].

(En este caso, el universo abarca todos los números reales.)


a) Además de la existencia de un neutro aditivo, existen los inversos aditivos. Escriba una
proposición cuantificada que exprese ""Todo número real tiene un inverso aditivo". (Nosc
debe utilizar el signo menos en la proposición.)
b) Escriba u.na proposición cuantificada que trate de la existencia de un neutro multiplicativo
para la aritmética de los números reales.
e) Escriba una proposición cuantificada relativa a la existencia de inversos mu1tiplicativos
para los números reales diferentes de cero. (No se debe uw el exponente -1 en la propos.i-
ción.)
d ) ¿Cambian de alguna forma los resultados de las partes (b) y (e) cuando el universo se
restringe a los enteros?
24. Considere la proposición cuantificada "dx 3y{x + y = l 7l Detennine si esta proposición es
verdadera o falsa para cada uno de los siguientes universos: (a) los enteros; (b) los enteros
positivos; (c) los enteros para x, los enteros positivos para y; (d) los enteros positivos para z.
los enteros para y.
25. En el caso de las siguientes proposiciones, el universo para cualquiera de sus variables está
formado por los números reaJes. En cada caso, niegue y simplifique la proposición dada.
•>VzVy[(z>y)- (z-y>O)]
b) Vz Vy [[(x > 0)/\(y =log,oz)l-(x = 10')]
e) VzVy[(z<y) -c3z(z<z<y)]
d) Vx Vy [Ozl = IYIJ-(y = ±z)]
<) [Vx Vy ((z >0)/\(y > O))J-[3z (zz > y)]
26. En matemáticas, con frecuencia se desea afirmar no sólo la existencia de un objeto a (ya sea un
m:imero, un tri~lo, etcétera} que satisfaga una proposición abierta p(.x), sino también el
hecho de que este objeto a es el único para el que se satisfacep(x) (es verdadera). Entonces, el
objeto es único. Esto se denota con el cuantificador 3 !xp(x), que se lee como ''Existe un único
x'. Este cuantificador puede definirse en términos de los cuantificadores existencia] y universal:

[3!z p(z)]<ó{[3.tp(x)]/\[Vx Vy [(p(x)/\p(y))-(z = y)]]

Esta definición indica que "una demostración de existencia y unicidad" requiere "una demos-
tración de la existencia-, que con frecuencia se realiza construyendo un ejemplo que satisfaga
JJ(.x), y "'una demostración de la unicidad-.
a) Escriba lo siguiente en forma simbólica, usando este nuevo cuantificador. (El universo
consta de todos los números reales.)
i) Todo número real diferente de cero tiene un dnico inverso multiplicativo.
ü) La suma de dos números reales cualesquiera es ónica.
iü) Para cada coordenada.r, lacoordenadaycorrespondienteen la rectay = 3x+ 7 es única.
b) S_ea p(x, y} la proposición abierta ··y= -2x"; el universo está fonnado por todos los enteros.
Determine cuáles de las proposiciones siguientes son verdaderas o falsas.
1) [Vx 3!y p(z,y)]-[3 !y Vxp(x,y)]
U) [3!y Vzp(z,y)J-(Vx 3!y p(x,y)]
e) Responda la parte (b) para ta proposición abiertap(x. y} : x +yes par.
d) Considere la proposición 31.r (x> 1). ™un ejemplo de un universo en el quep sea vcrda•
dera y un ejemplo de otro univerw donde p sea falsa.
2.5 Cuantificadores, definiciones y la demostración de teoremas 12 1

2 7. En cálculo, la definición del lfmíte L de una sucesión de m1meros reales ri, r:, r 1•• . . pu~
darse como
üm r,.=L

si (y sólo si) para cada f.> O exisce un entero positivo k tal que para todo entero n, si n > k.
entonces lr.,-LI <f..
En fonna simbólíca. esto puede expresarse como

lfm,. -L<>V•>O 3k>O Vn[(n>k)-lr.-L l <•J.

Exprese ~ r. =I=- Len forma simbólica.

2.5
Cuantificadores, definiciones
y la demostración de teoremas

En esta sección combinaremos algunas de las ideas que acabamos de estudiar en las dos
anteriores. Aunque la sección 2.3 presentó reglas y métodos para establecer la validez de
un argumento. por desgracia los argumentos presentados parecen tener poco que ver con
algo matemático. [Las raras excepciones están en el ejemplo 2.24 y el argumento e rróneo
en la parte (b) del material anterior al ejemplo 2.27.] La mayoría de los argumentos trataba
de algunos individuos y predicamentos que estaban a punto de surgir.
Pero una vez aprendidas algunas de las propiedades de los cuantificadores y de las
proposiciones cuantificadas, estamos en mejores condiciones para manejar argumentos
que nos ayudarán a demostrar téo'remas matemáticos. Sin embargo, antes de pasar a los
teoremas, examinaremos la forma usual en que aparecen las definiciones matemáticas en
las obras científicas.
Después del ejemplo 2.3 de la sección 2.1, había un aná1isis relativo a Ja fonna en que
una implicacíón podrfa ser utilizada en vez de una bicondicional en una conversación
cotidiana. Pero se hizo notar que en las obras científicas debíamos evitar las situaciones en
que pudiera surgir una inte rpretación ambigua; en particuJar, no debería usarse una
implicación cuando se necesitara una bicondicional. Sin embargo, hay una excepción fun-
damental a esta regla y se refiere a la forma en que las definiciones matemáticas se presen-
tan comúnmente en los libros de texto de matemáticas y otras obras de carácter cienúfico.
El ejemplo 2.52 demuestra esta excepción.

a) Comencemos con el universo de todos los cuadriláteros que hay en el plano e inten-
temos identificar los que se denominan rectángulos.
· Una persona podría decir que

..Si un cuadrilátero es un rectángulo entonces tiene cuatro ángulos iguales".

Otro podría identificar estos cuadriláteros particulares señalando que

"Si un cuadrilátero tiene cuatro ángulos iguales, entonces es un rectángulo".


122 Capítulo 2 Fundamentos de lógica

(En este caso. ambas personas están haciendo proposiciones cuantificadas en forma
implícita. con un cuantificador universal.)
Dadas las proposiciones abíertas
p{x): x es un rectángulo q(x): x tiene cuatro ángulos iguales,

podemos expresar lo que dice la primera persona como


\/x[p(x)->q(x)],
mientras que para la segunda escribiñamos
\/x(q(x)-> p(x)].
Así, ¿cuál de las proposiciones (cuantificadas) anteriores identifica o define a un
rectángulo? Tal vez nos parezca que ambas lo hacen. Pero ¿cómo puede ser, si una
proposición es la recíproca de la otra y, en general, la recíproca de una implicación
no es lógicamente equivalente a la implicación?
En este caso, el lector debe tener en cuenta lo que se pretendía, no sólo lo que
dijo cada una de las dos personas, o las expresiones simbólicas que hemos escrito
para representar estas proposiciones. En esta sítuacíón, cada persona está usando
una implicación con el sentido de una bicondicionaJ. Ambas pretenden decir (aun-
que no lo establecen)
\/x(p(x) ... q(x)]
es decir, cada una de ellas está diciendo realmente que
"'Un cuadrilátero es un rectángulo si y s6lo si tiene cuatro ángulos iguales".
b ) Dentro del universo de los enteros, podemos distinguir los enteros pares por medio
de cierta propiedad y de este modo definirlos como sigue:
Para cada entero n, decimos que n es par si es divisible entre 2.
(La expresión "divisible entre 2" significa "exactamente divisible entre 2•·; es decir,
no hay resto al dividir el dividendo n entre el divisor 2.)
Si considerarnos las proposiciones abienas
p(_n): n es un entero par q(n): n es divisible entre 2,
entonces pa.rect que la definición anterior se podría escribir en fonna simbólica como
\/n(q(n)-> p(n)].
Después de todo, la proposición cuantificada dada (en la definición anterior) es una
implicación. Sin embargo. esta situación es muy similar a la de la parte (a). Lo que
parece establecerse no es lo que se pretendía. La intención es que el lector interprete
la definición dada como
\/n(q(n) -p(n)],
es decir,
"Para tcxio entero n, decimos que n es par si y s6/o sin es divisible entre 2".
(Nótese que la proposición abiena "n es divisible entre 2" también puede expresar-
se mediante la proposición abit rta ..n"" 2k, para algún entero k". No debe confun-
2.5 Cuantificadores, definiciones y la demostración de teoremas 123

dirnos el cuantificador "para algún entero k", ya que la expresión 3k{n w 2k] sigue
siendo una proposición abiena en la que n es una variable libre.)

Hasta ahora hemos visto el uso de los cuantificadores en los enunciados de las definicio-
nes matemáticas, y que la forma tradicional que adopta dicha definición es lade una impli-
cación. Sin embargo, tenga cuidado y recuerde: solamente en las definicilmes, una implicación
puede leerse (equivocadamente) e interpretarse correctamente como una bicondicional.
Observe ahora la definición del concepto de límite en el ejemplo 2.51. Ahí escribimos
..si (y sólo si)'' ya que queríamos que el lector conociera nuestra intención.Ahora tenemos
la libertad de reemplazar "si (y sólo si)" por un sencillo "si.,.
Una vez desarrollado nuestro análisis acerca de la naturaleza de las definiciones mate-
máticas, continuaremos ahora con el estudio de algunos argumentos relacionados con las
proposiciones cuantificadas.

Supongamos que partimos ckl universo que abarca solamente los 13 enteros 2, 4, 6, 8,.
24, 26. Entonces podemos establece r la proposición:
Para todo n (lo que significan: 2, 4, 6, ... , 26),
podemos escribir n como la suma de cuando mucho tres cuadrados perfectos.
Los resultados de la tabla 2.24 proporcionan una verificación caso por caso que mue s-
tra que la proposición (cuantificada) dada e s verdadera. (Podríamos llamar teorema a esta
proposición.)

lllbla 2.24

2=1 + 1 10 = 9+1 20= 16+4


4=4 12 : 4+4+4 22 : 9+9+4
6=4+1+1 14 = 9+4+1 24= 16+4+ 4
8=4+4 16= 16 26=25+1
18 = 16 + 1 + 1

Esta lista e xhaustiva es un ejemplo de una demostración que usa la técnica que llama-
mos, apropiadamente, método exhaustivo. El uso de este método es razonable cuando
trabajamos con un universo pequeño. Si nos enfrentarnos a una situación en la que el
universo es grande pero dentro del alcance de un computador disponible para nosolros,
entonces podríamos escribir un programa que verifique todos los casos individuales para
no cansarnos con este método de exhaustivo.
(Obsérve que, para algunos casos de la tabla 2.24, se puede dar más de una respuesta.
Por ejemplo, podríamos escribir 18 =9 + 9 y 26 w 16 + 9 + l. Pero esto está bien. Se nos
ha dicho que cada entero positivo par menor o igual que 26 puede escribirse como la suma
de uno, dos o tres cuadrados perfectos.No se nos ha dicho que tal representación tenga que
ser única, por lo que puede haber más de una posibilidad. Lo que teníamos que verificar en
cada caso era que hubiera al menos una posibilidad.)
124 Capitulo 2 Fundamentos de lógica

En el ejemplo anterior mencionamos la palabra teqrema. También en el capítulo 1 en-


contramos este 16rmino; por ejemplo, en resultados como el teorema del binomio y el
teorema mu.ltinomial, donde presentamos algunos tipos de problemas de enumeración.
Para no ser muy técnicos, consideraremos a los teorema.s como proposiciones de interés
matemitico, que se sabe son verdaderas. A veces el término teorema se usa únicamente
para describir resultados imponantes que tienen muchas y variadas consecuencias. Algu-
nas de estas consecuencias, que se siguen inmediatamente de un teorema. se denominan
coro/aries (como en el caso del corolario 1.1 de la sección 1.3). Sin embargo, en este
texto, no utilizaremos de ningún modo especial la palabra teorema.
El ejemplo 2.53 es un buen punto de partida para analizar la demostración de una pro-
posición cuantificada. Por desgracia. a menudo un gran número de proposiciones y teore-
mas matemáticos tratan de universos que no se prestan al uso del método exhaustivo.
Por ejemplo, si queremos establecer o demostrar u.o resultado para Jos enteros o para
los nWlleros reales, no podemos utilizar un m~tcxlo caso por caso como el del ejemplo
2.53. ¿Qué podemos hacer entonces?
Empezaremos por considerar la siguiente regla.

Esta regla indica que la verdad de una proposición abierta en un caso panicular se sigue
(como caso particular) de la verdad más general (para todo el universo} de esa proposición
abierta cuantificada universalmente. Los siguientes ejemplos nos mostrarán cómo aplicar
esta idea.

a) Para el universo de todas las personas, consideremos las proposiciones abiertas

m(x): x es un profesor de matemáticas c(x): x ha estudiado cálculo.

Ahora consideremos el siguiente argumento.

Todos los profesores de matemáticas han estudiado cálculo.


Leona es profesora de matemáticas.
Por lo tanto, Leona ha estudiado cálculo.

Si representamos como la esta mujer en particular (de nuestro universo} llamada


Leona, entonces podemos escribir este argumento en forma simbólica como

Yx(m(x)-->c(x)]
m(/)
:.e(/)
Aquí, las dos proposiciones que quedan arriba de la línea son las premisas del
argumento y la proposición e({) que está abajo de la línea es su conclusión. Esto es
2.5 Cuantifiebdores, definiciones y la demostración de teoremas 125

comparable a lo que se dijo en la sección 2.3, excepto que ahora tenemos una premisa
dada por una proposición cuantificada universalmente. Como en el caso de la sec-
ción 2.3, hemos supuesto que todas las premisas son verdaderas y debemos tratar de
esrablecer que, en estas circuDstancias, la cooclusioo WDbitn es verdadera. Ahora
bien, para esrablecer la validez de un argumento dado, procederemos como sigue.
Paso.. Razones
1) Vx[m(x)-+c(x)) Premisa
2) m([) Premisa
3) m([)-+ e([) Paso (1) y la regla de especifkación universal
4) :.e(/) Paso (2) y (3) y Modus PoMns
Observe que las proposiciones de los pasos (2) y (3) no son proposiciones cuan-
tificadas. Son los tipos de proposiciones que estudiamos al principio del capítulo.
En panicular, podemos aplicar las reglas de inferencia que aprendimos en la sec-
ción 2.3 a esw dos proposiciones para deducir la conclusión del paso (4).
Aquí vemos que la regla de especificación universal nos pennite tomar una
premisa cuantificada universalmente y deducir de 65:ta una proposición ordinaria
(es decir. no cuantificada). Esia proposicioo (ordinaria), llamadam(/) -+ e(/), es un
caso verdadero específico de la premisa verdadera cuantificada universalmente
V.z{m(z)-+ c(z)].
b) Para un ejemplo de naturaleza más matemática. consideremos el unj verso de todos
los triángulos que hay en el plano, junto con las proposiciones abiertas
p(t): t tiene dos lados de igual longitud
q(t): tes un triángulo isósceles
r(t): t tiene dos ángulos de igual medida.
Vamos a centramos en un triángulo específico que no tenga dos ángulos de igual
1.....~da. Este triángulo se llamanlXYZ' y se designará con c. Entonces vemos que el
argumento
En el triángulo X1'Z no hay dos ángulos de igual 7 r(c)
medida.
Si un triángulo tiene dos lados de igual longitud, Vt(p(t)-+q(t))
entonces es un ísósccles.
Si un triángulo es isósceles, entonces tiene dos Vt[q(t)-+ r(t))
ángulos de igual medida.
Por lo ianto, el triángulo X1'Z no tiene dos lados :.7 p(c)
de igual longitud.
es válido, como se muestra a continuación.
Paso.. Rnones
1) Vt(p(t) -+ q(t)) Premisa
2) p(c)-+ q(c) Paso (1) y la regla de especificación universal
3) Vt[q(t)-+ r(t)) Premisa
4) q(c)-+ r(c) Paso (3) y la regla de especificación universal
5) p(c)-+ r(c) Pasos (2)y (4) y la ley del silogismo
6) 7T(C) Premisa
7) :. 7p(c) Pasos (S)y (6) y Modus Tollens
126 capítulo 2 Fundamentos de lógica

Una vez más podemos ver la utilidad de la regla de especificación universal. Ea


este caso se tomaron las proposiciones cuantificadas universalmente en los pasos
(1) y (3) y la regla produjo las proposiciones (ordinarias) de los pasos (2) y (4~
respectivamente. Entonces. en ese momento pudimos aplicar las reglas de inferencia
que aprendimos en ta sección 2.3 (la ley del silogismo y el Modus Tollens) pan.
obtener la conclusión --p(,c) del paso (7).
e) ¡Ahora. el último argumento para que todo quede claro! Consideremos el universo
de todos los estudiantes de una escuela específica. Designaremos a una estudianie
panicular, María, como m.
Para este universo y las proposiciones abiertas

j (x): x está en su penúltimo año s(x): x está en su último año


p(.x): x está inscrita en una clase de educación física

consideremos el siguiente argumento:

Ninguna estudiante de penlJltimo o último año está


inscrita en una clase de educación física.
María está inscrita en una clase de educación física.
Por 1o tanto, María no es una estudiante de último año.

En forma simbólica. este argumento se convierte en

Vx((j(x )v s(x))--ip(x)I
p(m)
:.-,s(m)

Los pasos (y raz.ones) siguientes establecen la va]jdez de este argumento.


Pasos Razones
1) Vx((j(x )vs(x ))-. -,p(x)) Premisa
2) p(m) Premisa
3) (j(m) v s(m))-.-,p(m) Paso ( 1) Lla regla de especificación
universal
4) p(m)-. -i(j(m) v s(m)) Paso (3), (q ➔ t) ~ ( ~t ➔ ~ q) y la ley de la
doble negación
5) p(m)-. (-ij(m)/\-is(m)) Paso (4) y la l,y de De Morgan
6) -ij(m) /\-is(m) Pasos (2) y (5) y la regla de separación
(o Modus Ponens)
1) :. -is(m) Paso (6) y la regla de simplificación
conjuntiva.

En el ejemplo 2.54 tuvimos nuestra primera oportunidad de aplicar la regla de especifi.


cación universal. Usando esta regla junto con las de Modus Ponens (o regla de separación)
y Modiu Tollens, podemos establecer las siguie ntes analogías correspondientes, cada una
de las cuales implica una premisa cuantificada universalmente. En cada caso, considere•
mos un universo fijo con un elemento específico c. y usaremos las proposiciones abiertas
p(x), q(x) definidas para este universo.
2.5 Cuantificadores, definiciontS yla demostración de teoremas 127

(!) Vx(p(x)--+q(x)] (2) Vx(p(x)-+ q(x)]


p(c) ,q(c)
:.q(c) :.,p(c)

Estos dos argumentos vilidos se presentan aquí por la misma razón que los presentamos
para las reglas de inferencia, Modus Ponen..r y Modus Tollens, en la sección 2.3 (durante el
análisis que aparece entre los ejemplos 2.26 y 2.27). Queremos analizar algunos posibles
errores que pueden surgir cuando no se usan correctamente los resultados ( 1) y (2).
Comenzaremos con el univcno óe todos los pollgonos que hay en el plano. Dentro de
este universo, denoramos con e a un poUgono espccffico, el cuadrilátero EFGH, cuyo
ángulo E mide 91 º. Para las proposiciones abiertas

p(x): x es un cuadrado q(x): x tiene c uatro lados,

los siguientes argumentos no son válidos.

(I ') Todos los cuadrados tienen cuatro lados.


El cuadrilátero EFGH tiene cuatro lados.
Por lo tanto, el cuadrilátero EFGH es un cuadrado.

En forma simbólica. esle argumento se traduce en

(!") Vx(p(x)-+q(x)]
q(c)
:.p(c)

Por desgracia. aunque las premisas son verdaderas, la conclusión es falsa. (En un cuadra•
do no hay un ángulo que mida 91°.) Es cieno que podría haber una confusión entre este
argumento y el argumento v'1ido (1) anterior. Ya que en este caso, cuando aplicamos la
regla óe especificaci6n wúversal a la premisa cuantificada ( 1"), obtenemos el argumento
no válido
p(c)-+ q(c)
q(c)
:.p(c)
Y aquí, como en la sección 2.3, el error en el razonamiento se encuentra en el in1en10 de
argumentar mediante el recíproco. .
Podemos dar un segundo argumento no válido, el cual surge del ma1 uso del argumento
(2) anterior, como se muestra a continuación:
Todos los cuadrados tienen cuatro lados.
El cuadrilátero EFGH no es un cuadrado.
Por lo tanto, el cuadrilátero EFGH no tiene cuatro lados.
Al traducir (2 ') a una forma simbólica obtenemos

Vx(p(x)-+q(x)]
,p(c) (2")
:.,q(c)
.:.12::;8:..__ _ _ _ _ capitulo 2 Fundamentos de lógica

Esta vez... la regla de especificación universal produce

p(c)-q(c)
-ip(c)
~
1
que es donde surge la falacia al tratar de argumentar por el inverso.

Observemos de nuevo las tres partes del ejemplo 2.54. Aunque los argumentos presen-
tados ahí tenían como premisas proposiciones cuantificadas universalmente, en ningún
caso apareció como conclusión una proposición cuantificada universalmente. Ahora que-
remos remediar esta situación. ya que muchos teoremas de matemáticas tienen la forma de
una proposición cuantificada universalmente. Para hacer esto necesitamos considerar lo
siguiente.
Empecemos con un universo dado -y una prOJX)sición abierta p(x). Para establecer la
verdad de la proposición "lx p(x), debemos establecer la validez de p(c) para cada elemen-
to e del universo dado. Pero si el universo tiene muchos elementos o, por ejemplo, contieoe
a todos los enteros positivos, entonces esta exhaustiva (o extenuante) tarea de validación
de cadap(c) se toma difícil, si noes que imposible. Para evitar esta situación, demostrare-
mos que p{,c) es verdadera. pero lo baremos para el caso en que e denota un elemento
especfjico pero arbitrario del universo prescrito.
Si la proposición abierta anteriorp(x) tiene la forma q(x) ➔ r(x), para las proposiciones
abiertas q(x) y r(x), entonces debemos suponer, como premisa adicionaJ, que q(c) es ver-
dadera e intentar deducir la verdad de r(c), usando definiciones, axiomas. teoremas de-
mostrados con anterioridad y los principios lógicos que hemos estudiado. Ya que cuando
q(c) es falsa. la implicación q(c) ➔ r(c) es verdadera, independientemente del valor de
verdad de r (c).
La razón por la que el etementoc debe ser arbitrario (o genérico) es para garantizar que
lo que hagamos y demostremos de e sea aplicable a todos los demás elementos del univer-
so. Por ejemplo, si trabajamos con el universo de todos los enteros. no podemos elegir e de
manera arbitraria como 4, o como un entero par. En general, no podemos adoptar hipótesis
acerca de la e lección de e, a menos que esas hipótesis sean válidas para todos los elemen-
tos del universo. Aplicamos la palabra genérico al elemento e para indicar que nuestra
elección (dec) debe compartir todas las características comunes de los elementos del uni-
verso dado.
El principio que hemos descrito en los tres párrafos anteriores es el siguiente.

LaNgladeLy alaad6,,...._Sise--m ··
abatap(<)es,adadolac:aadozserccmplazapcrcaalq,lier._,cdegitJ,,-
ji,,a,""1ilrrrilde_,,,.¡__,,_,.,.1apnlllOlici6a.-ilicada~
- Yzp(.Jt)es >mladon.Adams, la,qlasemieade llcasodemsdem ..._
ble.Ad.pcrejrmpo,.;........,w_m......,.._.allienaq(>-.7)es~.
.. ,_,,...__. ,,.,......___e1egidora,,,,_""1itroriadel,.;..,___,oe
---....,.a;.m.-1a......,.._. ............... , YsYj
q(>-.7)(o'i>;7q(>-.7)Jes>mladon.1la1Jiáasea.pm,_,....silllilmspaa•
CISOSdo-omsYllill,Jes.
2.5 Cuantificadores, definiciones y la demostración de teoremas 129

Antes de demostrar el uso de esta regla en otros ejemplos, quisiéramos regresar a la


parte (1) del ejemplo 2.44 de la sección 2.4. Vemos entonces que la explicación dada ahí
para establecer que

'<fx [p(x) /\ (q(x) /\ r(x))] ~ '<>'x [(p(x ) /\ q (x)) /\ r(x)]

anticipaba lo que ahora hemos descrito con detalle como las reglas de la especificación y
la generalización universales.
Ahora pasaremos a un ejemplo que es estrictamente simbó)jco. Este ejemplo nos ofrece
una oportunidad de aplicar la regla de la generalización universal.

Sean p(x), q(x) y r(x) proposiciones abiertas definidas para un universo dado. Mostrare-
mos que el argumento
'<fx[p(x) --+ q(x)]
'<fx[q(x )--+ r(x )}
:. '<fx[p(x)--+ r(x)]
es válido considerando lo siguiente.
Pasoo Razones
1) '<fx[p(x)--+ q(x)] Premisa
2) p(c)--+ q(c) Paso (1) y la regla de la especificación universal
3) '<fx[q(x) --+ r(x)] Premisa
4) q(c)--+r(c) Paso (3) y la regla de la especificación universal
5) p(c)-n(c) Pasos (2) y (4) y la ley del silogismo
6) : . '<fx[p(x )--+r(x)] Paso (5) y la regla de la generalización universal

En este caso, el elemento e introducido en los pasos (2) y (4) es el mismo elemento
específico pero elegido arbitrariamente del universo. Puesto que este elemento no tiene
propiedades especiales o distinti'Vas sino que comparte todas las características comunes
de cualquier otro elemento del universo. podemos usar la regla de la generalización uni• ·
versal para ir del paso (5) al paso (6) .
De este modo, finalmente tenemos un argumento válido en el que una proposición
cuantificada uni\'ersalmente aparece como conclusión, así como entre las premísas.

La pregunta que podría venir a la mente del lector se referiría al aspecto práctico: ¿cuándo
necesitaríamos utilizar el argumento del ejemplo 2.55? De hecho. ya lo hemos utilizado
(tal vez e n forma inconsciente) en cursos anteriores de álgebra y cálculo, como lo demues-
tra el siguiente ejemplo.

a) Para el universo de los números reales, consideremos las proposiciones abiertas


p(x): 3x - 7=20 q(x): 3x =27 r(x): x =9.
Desarrollaremos la siguiente solución de una ecuación algebraica en forma paralela
al argumento válido del ejemplo 2.55.
10 capítulo 2 Fundamentos de lógica

1) Si 3x- 7 = 20, e ntonces 3x = 27. Vx(p(x)---+q(x))


2) Si 3x = 27, entonces x =9. Vx[q(x)---+r(x)]
3) Por lo tanto. si 3x - 1 = 20. entonces x = 9. :. Vx(p(x)---+r(x))
b) Cuando trabajamos con el universo de todos los cuadriláteros de la geometrfa pla-
na, es probable que relacionemos de la forma siguiente :
·'Como todo cuadrado es un rectángulo y todo rectángulo
es un paralelogramo, se sigue que todo cuadrado es un paralelogramo·•.
En este caso estamos utilizando el argumento del ejemplo 2.55 para las proposicio-
nes abiertas
p(_x): x es un cuadrado q(x): x cs un rectángulo r(x): xes un paralelogramo.

Analizaremos ahora la validez de otro argumento.

Los pasos y razones necesarios para establecer la validez del argumento

Vx[p(x) V q(x))
Vx[(,p(x)/1 q(x))---+ r(x)]
:. Vx[or(x)--+ p(x)]

son los siguientes. [El eleme nto e está en el universo asignado al argumento. Además.
como la conclusión es una implicación cuantificada universalmente, podemos suponer
...., (e) como una premisa adicional, como ya mencionamos al presentar la regla de la
generalización universal.]

Pasos Razones
1) Vx[p(x) V q(x)) Premisa
2) p (c) v q(c) Paso ( 1) y la regla de la especificación
universal
3) Vx[(, p(x) /\q(x))---+r(x)) Premisa
4) [7p(c)/\q(c)]--+ r(c) Paso (3) y la regla de la especificación
universal
5) or(c)--+,[,p(c) /\ q(c)] Paso (4)ys-+r<=> -.r ➔ -.s
6) or(c)--+ [p(c)v ,q(c)] Paso (5), ley de De Morgan y la ley de la
doble negación
7) or(c)
8) p(c)v,q(c) Premisa (supuesta)
9) [p(c)V q(c)) /\[p(c)v ,q(c)) Pasos (7) y (6) y Modus Ponens
Pasos (2) y (8) y la regla de la conjunción
10) p(c)V [q(c) /1,q(c)] Paso (9) y la propiedad distribuúva de
V sobre A
11) p(c) Paso (10), q(c) A ~q(c).,,. Fo
y p(c) V Fo ée> p(c)
12) :. Vx[or(x)---+p(x)] Pasos (7) y ( 11) y la regla de la
generalización universal
2.5 Cuantificadores. definiciones y la demostración de teoremas 131

Antes de proseguir, queremos mencionar un convenio que tal vez no agrade al lector
pero al que deberá acostumbrarse. Se refiere a nuestro tratamiento de las reglas de la
especificación y generalización universales. En el primer caso, partimos de la proposición
Vz p(z) y despu6s trabajamos conp(c) para algún elemento específico e de nuestro univer-
so. Para la regla de la generalización universal, utiliz.amos la proposición verdadera p(c)
para deducir la verdad de Vz p(x) a partir de la de p(c), donde e es un elemento arbitrario
del universo. Por desgracia. usaremos con frecuencia la letra x en vez de e para denotar el
elemento arbitrario, pero mientras comprendamos lo que ocurre, pronto veremos que el
convenio es bastante fácil de usar.

Los resultados del ejemplo 2.55 y, en particuJar, del ejemplo 2.57, nos indican que
podemos utilizar las propOsiciones cuantificadas universalmente y las reglas de inferencia.
incluyendo las reglas de especificación y generalización universal. para fonnaJizar y de-
mostrar muchos tipos de argumentos y (esperamos) teoremas. Cuando hacemos esto, pa-
rece que la validación incluso de argumentos cortos requiere varios pasos, ya que hemos
sido muy meticulosos e incluido todos los pasos y razones; hemos dejado poco. por no
decir nada, a la imaginación. E1 lector puede estar seguro de que al comenzar a demostrar
teoremas matemáticos, presentaremos las demostraciones en un estilo de texto corrido
más con,,encional. Ya no mencionaremos todas y cada una de las aplicaciones de las leyes
de la lógica. el resto de las tautologías o las reglas de inferencia. En algunos casos. subra-
yaremos alguna regla de inferencia. pero nos centraremos principalmente en el uso de
definiciones, axiomas y principios matemáticos (distintos de los que aparecen en el estu-
dio de la lógica) y los demás teoremas (anteriores) que hayamos podido demostrar. ¿Para
qu6 estudiar entonces todo este material de validación de argumentos? Porque esto nos
proporciona un marco de referencia al cual recurrir cuando dudemos de un intento de
demostración. Si surge una duda, tenemos nuestro estudio de la lógica, que nos proporcio-
na los medios (un tanto mecánicos, pero estrictamente objetivos) para ayudarnos a decidir~
Ahora presentaremos aJgunas demostraciones con un estilo de teXto corrido para algu-
nos resultados relativos a los enteros. (Estos re5uJtados pueden considerarse casi ob\·ios;
de hecho, encontraremos algunos que ya hemos visto y usado. Sin embargo. nos propor-
cionan un marco excelente para escribir algunas demostraciones simples.) Las demostra•
cienes utilizan las siguientes ideas, que definiremos fonnalmente. [La primera idea fue
mencionada antes, en la parte (b) del ejemplo 2.52.)

Definición 2.8 Sean un entero. Decimos quen es par sin es divisible entre 2; es decir. si existe un entero
r tal que n = 2r. Sin no es par, entonces decimos que n es impar, para este caso, existe un
enteros tal que n = 2s + 1.

TEOREMA2.2 Para textos los enteros k y /, si k, l son impares, entonces k + les par.
~ d ón: En esta demostración numeraremos los pasos para referimos a ellos en
comentarios posteriores. Dcspu~s ya no los numeraremos.

1) Como k y l son impares, podemos escribirlos como k = 2a + 1y l = 2b + 1, para


aJgunos e nteros a, b. Esto es por la definición 2.8.
132 Capítulo 2 Fundamentos de lógica

2) Entonces

k + 1 = (2a + I) + (2b + 1)
= 2(a + b + 1),
en vinud de las propiedades conmutativa y asociativa de la suma y la propiedad
djsrributiva de la multiplicación sobre la suma, que valen para los enteros.
3) Como a. b son enteros. a + b + 1 = e es un entero; y como k + l = 2c. se sigue de la
definición 2.8 que k +les par.

Comentarios

1) En el paso ( 1) de la demostración anterior, elegimosk y/ de manera arbitraria y por


ello sabemos que el resultado obtenido es verdadero para todos los enteros impares.
por la regla de generalización universal
2) Aunque no nos hayamos dado cuenta. hemos usado la regla de especificación uni-
versal (dos veces)cn el paso (1). El primer argumeQto implícito en este paso se lee
como sigue.
i) Si n es un entero impar, entonces n = 2r + l para aJgún entero r.
ü) El entero k es un entero impar específico (pero elegido en fonna arbitraria).
iii) Por Jo tanto, podemos escribir k = 2a + l para algún entero a (específico).
3) Enet paso (l} no tenemosk-. 2a+ l yl=2a+ l. Comok. lfueron arbitrarios.puede
= = =
ocurrir que k I; y cuando esto sucede, tenemos que 2a + 1 k I = 2b + l. de lo
cual se sigue que a-. b. (Comok podría ser diferente de /, se sigue que (k- IYl = a
podña ser distinto de b =(l - l)/2. Así que debemos usar variables a y b diferentes.]

Antes de proceder con otro teorema, escrito de manera más convencional. examinemos
lo siguiente.

Consjderemos la siguiente proposición para el universo de enteros.


Sin es un entero. entonces n2=n; o bien. Vn[n2 -. n].
Ahora bien, para n = O es cierto que n2 = 02 = O = n. Y si n = 1, también es cieno que ,r1 :=
P = 1 = n. Sin embaJE;o, no podenws concluir que n2= n para cada entero n. La regla de
generalización universa) no se aplica aquí, ya que no podemos considerar la elección de O
(o 1) como si se hubiera realizado arbitrariamente. Sin= 2, tenemos que n 2 = 4 .if: 2 = 11;
este único contraejemplo es suficiente para mostrar que la proposición dada es falsa. Sin
embargo, cada uno de los reemplazos anteriores, n = O o n = 1, es suficiente para establecer
la verdad de la proposición

Para algún entero n, n'= n, o, 3n[n'= n].

Por último, cerramos la sección con tres resultados que muestran cómo escribiremos
las demostraciones en el resto del texto.
2. S Cuantificadores, definiciones y la demostración de teoremas 133

TEOREMA2.3 Para todos los enteros k y l, si J.: y l son impares. entonces también su producto Id es impar.
Demostración: Como k y l son impares, por la definición 2.8. podemos escribirlos como k
= 2a + 1 y I= 2b+ 1, para algunos enteros a, b. Entonces el productokl = (2a + 1)(2b + 1)
=4ab+ 2a+2b + l • 2(2ab+ a +b) + 1, donde 2ab+a +bes un entero. Por lo tanto, una
vez más por la definición 2.8, se sigue que kJ es impar.

La demostración anterior es un ejemplo de demostración directa. En nuestro siguiente


ejemplo demostraremos un resultado por tres vías: primero por un argumento directo (o
demostración). después por el método de la contrapositiva y finalmente por el método de
contradicción. [Para la demostración por (el método de) contradicción añadiremos algu-
nos detalles, ya que es nuestra primera oportunidad de usar esta técnica.] Sin embargo, e l
lector no debe suponer que todos los teoremas pueden demostrarse fácilmente de varias
formas.

TEOREMA2.4 Si m es un entero par, entonces m + 7 es impar.


C>emostr.lción:
1) Como mes par, tenemos que m = 2a para algún entero a. Entonces m + 7 = 2a + 7
-2a + 6 + 1 = 2(a + 3) + l. Como a+ 3 es un entero, tenemos que m + 1 es impar.
2) Supongamos que m + 1 rw es impar. por lo tanto, es par. Entonces m + 7 = lb para
= =
alguna by m 2b-7 =2b- 8 + 1 2(b-4) + 1, donde b-4es un entero. Por lo
tanto, m es impar. [El resultado se sigue del hecho de que las proposiciones de la
forma Vm[p(m) ➔ q(m)) y Vm( - q(m) ➔ -p(m)J son lógicamente equivalentes.)
3) Ahora supongamos que m es par y que también m + 1 es par. (Esta suposición es la
negación de lo que deseamos demostrar.) Entonces, m + 7 par implica que m + 1 _.
2cparaalgúnenteroc. Y, en consccuencia,m= 2c- 7 ~ 2c-8 + l ; 2(c-4) + l con
e - 4 entero, por lo quemes impar. Aquí aparece la contradicción: empezamos con
m par y ahora hemos deducido m impar, una situación imposible, ya que no puede
haber un entero que sea par e impar a la vez. ¿Cómo llegamos a este dilema'? Muy
simple: ¡cometimos un error! Este errores la hipótesis falsa de quem + 7 es par, que
supusimos al comienzo de la demostración. Como la hipótesis es fal~ su negación
es verdadera y m + 1 es impar.

La segunda y tercera demostraciones del teorema 2.4 son muy similares. Esto se debe a
que la contradicción obtenida en la tercera demostración surge de la hipótesis del teorema
y su negación. Más adelante veremos (en el siguiente capítulo) que también podemos
obtener una contradicción al llegar a la negación de un hecho conocido, un hecho que no
sea la hipótesis del teorema que se intenta demostrar. Sin embargo, por ahora, pensemos
un poco más en esta analogía. Supongamos que partimos de las proposiciones abienas
p(_m) y q(m), para un universo dado, y consideremos un teorema de la fonna 'vm[p(m) ➔
q(m)). Si intentamos demostrar este resultado por el método de la contrapOSitiva, entonces
estaremos demostrando la proposición lógicamente equivalente Vm[-,q(m) ➔ --ip(m)).
Para hacer esto, suponemos que --iq(m) es verdadera (para cualquier m específico, pero
elegido en fonna arbitraria del universo) y mostramos que esto implica que ~p(m) es
134 capítulo 2 Fundament05 de lógita

verdadera. Por otro lado, si queremos de mostrar el teorema 'vm[p(m) ➔ q(m)] por el mé-
tcxJ.o de demostración por contradicción, entonces supone mos que la proposición Vm[p(m}
➔ q(m)] es falsa. Esto equivale al hecho de que p(m) ➔ q(m) sea falsa para al menos un
reemplazo de m del universo~ es decir, e xiste algún elemento m en el universo para el que
p(m) es verdadera y q(m) es falsa [o bien, --q(m) es verdadera]. Usamos este hecho Oa
verdad de p(m) y -.q(m)) para obtener una contradicción. [En la tercera demostración del
teorema 2.4 obtenemos p(m) /\ -.p(m).] Estos dos métodos pueden compararse simbóli-
camente como sigue , cuando el m elegido para el mé todo de contradicción es específico
pero elegido arbitrariamente.

Hipótesis Resultado obtenido


Contraposición 7 q(m) 7 p(m)
Contradicción p(m) y 7q(m) Fo

En general, si podemos demostrar un teorema de manera directa o indirecta. resultará


menos enredado el método directo que el indirecto. (Esto parece ser el caso de las tres
demostraciones presentadas en el teorema 2.4.) Cuando no tengamos una dirección fija
para intentar la demostración de cierto teorema, podemos empezar con un método directo.
Si tenemos 6xito. está muy bien. Si no, debemos intentar encontrar un contraejemplo de lo
que creíamos era un teorema Si nuestra búsqueda del contraje mplo falla. entonces debe-
mos tratar con el método indirecto. Podríamos demostrar la contrapositiva del teorema, u
obtener una contradicción, como lo hicimos en la tercera demostración del teorema 2.4,
suponiendo que la hipótesis y la negación de la conclusión (para algún elemento m del
universo) son verdaderas en el teorema dado.
Concluiremos esta sección con otra demostración indirecta por el método de contrapO-
sición.

TEOREMA 2.5 Para todos los números reaJes positivosx y y. si el producto xy es mayor que 25, entonces
x> 5 oy> 5.
Demostración: Consideremos la negación de la conclusión; es decir, supongamos que O<
x :S 5 y O< y s 5. En estas circunstancias, tenemos que O< x • y s 5 • 5 =25, por lo que
el producto .xy no es superior a 25. (Este método indirecto de demostración establece ahora
la proposición dada. ya que sabemos que una implicación es lógicamente equivalente a su
contrapositiva.)

EJERCICIOS 2.5 1. ¿En el ejemplo 2.53, por qué nos detuvimos en 26 y no en 28?
2. ¿Por qué no incluimos los enteros impares entre 2 y 26 en el ejemplo 2.53?
3. Utilice el mttod<> de exaustivo para mostrar que cada entero par entre 30 y 58 (incluyendo 30
y 58) puede escribirse como la suma de no más de tres cuadrados perfectos.
4. Sean un entero positivo mayor que 1. Decimos que n es primo si los llnicos enteros positivos
que dividen (exactamente) a n son J y el propio n. Por ejemplo, los primeros siete primos son
2. 3. 5. 7, 11. J3 y 17. (Aprenderemos más acerca de Jos primos en el capítulo 4.) Utilice el
mttodo exhauSlivo para demostrar que cada entero par, en el universo 4. 6, 8, .. .• 36. 38.
puede escribirse como la suma de dos primos.
2.5 Cuantificadores. definiciones y la demostración de teoremas 135

S. Para cada uno de los siguientes universos y parejas de proposiciones. utilice la regla de espe•
cificación universal, así como el Modus Ponens o Modus Toll~ns. para llenar la línea en blanco
y obtener un argumento válido.
a) [El universo comprende todos los números reales.]
Todos los enteros son m1mcros racionales.
El número real p no es un m1mero racional.

b) [El universo comprende la población actual de Estados Unidos.]


Todos los bibliotecarios conocen el sistema de clasificación de la Biblioteca del Congreso.

.-. Margarita conoce el sistema de clasificación de la Biblioteca del Congreso.


e) [El mismo univcr50 de la parte (b).)

Sandra es directora adminisuativa.


:.Sandra sabe cómo delegar autoridad.
d ) [El universo consta de todos los cuadriláteros del plano. ]
Todos los rectángulos son cquiangularcs.

:.El cuadrilátero MNPQ no es un rectángulo.


e) {El universo es el mismo que en la parte (b).]
Las personas conscientes de los riesgos del colesterol evltan comer hígado.
Greta es una persona consciente de los riesgos del colesterol.

6 . Determine cuáles de los siguientes argumentos son váUdos y cuáles no. Explique cada respues-
ta. (El universo consta de todas las personas que residen acrua1mcnte en Estados Unidos.)
a ) Todos los carteros llevan una lata de aerosol irritante.
FJ señor Beltrán es cartero.
Por lo tanto, el señor Beltrán lleva una lata de aerosol irritante.
b) Todos los ciudadanos respetuosos de la ley pagan sus impuestos.
El señor Pérez paga sus impuestos.
Por lo tanto. el sei\or P&ez es una persona que obedece la ley.
e) Todas las personas que se preocupan por el ambiente reciclan sus recipientes de plástico.
Margarita no se preocupa por el ambiente.
Por lo tanto, Margarita no recicla sus recipientes de plástico.
d ) Ningán estudiante consciente deja las tareas inconclusas.
Antonieta no deja inconclusas sus tareas.
Por lo tanto, Antonieta es una estudiante consciente.
7 . Para un universo dado y cualesquiera proposiciones abiertas p(x), q(x) en la variable x . de-
muestre que
a) 3x [p(x) V q (x)]<>3x p(x)v3x q(x)
b) Yx [p(x) l\ q(x)]<>Yx p(x)I\ Yx q(x)
8. a ) Sean p(,x), q(x) proposiciones abiertas e n la variable x, con un universo dado. Demuestre
que

Yx p(x)v Yx q(x) ⇒ Yx [p(x) V q(x)J.


(Es decir. demuestre que si la proposición Vx p(,x) V Vx q(x) es verdadera. entonces la
proposición Vx[p(x) V q(x)J es verdadera.]
b) Encuentre un contraejemplo para la recíproca de la parte (a). Es decir. encuentre proposi-
ciones abienasp(.x ). q(x) y un universo tal que 'tfx[p(.x) V q(x)J sea verdadera y Vxp(x) V
Yx q(x) falsa.
36 capitulo 2 Fundamentos de lógica

9. Dé las razones para los pasos de verificación del s iguiente argumento. (En este caso. a denota
un elemento arbitrario pero especifico del universo dado.)

Yx[p(x)➔ (q(x) l\r(x))]


Yx[p(x)l\s(x)]
: . Yx[r(x)l\s(x)]
Pasos Razones
1) Yx[p(x) ➔ (q(x) /\r(x))]
2) Yx[p(x) l\s(x)]
3) p(a) ➔ (q(a) l\ r(a))
4) p(a)l\s(a)
5) p(a)
6) q(a)l\r(a)
7) r(a)
8) s(a)
9) r(a) l\s(a)
10) :. Yx[r(x) l\s(x)]

1 O. Escriba las razones que faltan para los pasos de verificación del siguiente argumento:

Yx[p(i)Vq(x)]
3x,p(x)
Yx[,q(x) V r(x)]
Vx[s(x) ➔,r(x)]
:. 3x(-.s(x))

Pasos Razones
1) Yx[p(x)Vq(x)] Premisa
2) 3x ,p(x) Premisa
3) ,p(a) Paso (2) y definición de verdad para
3x -.p(_:x). [En es.te caso. a es un elemento
(reemplazo) del universo para el cual
-.p<.,x) es verdadera.] La razón que justifica
este paso tambiát se conoce como la
regla tk la esp<cificaci6n e.ris1e-ncial.
4) p(a)Vq(a)
S) q(a)
6) Yx[,q(x)vr(x)]
7) ,q(a)vr(a)
8) q(a/ ➔ r(a)
9) ,(a)
10) Yx[s(x)➔,r(x)]
11) s(a)- ,r(a)
12) r(a)➔,s(a)
13) ,s(a)
14) .-. 3x(,s(x)) Paso 13 y la definición de verdad para
3x(-.s(x)). La razón de este paso también
se conoce como ~gla de genera/hación
aisttnciaL

11 . Escriba e l siguiente aigumento en forma simbólica. Verifique entonces la validez. del argumen•
to o c x.pliquc por qué no es válido. [Supongamos que el universo abarca a todos los adultos
2.6 Resumen repaso histótico 137

(mayor-es de J8) que residen actualmente en la ciudad de Las Cruces. Nuevo M~Aico. Dos de
estas personas son Rouna e lnts.]

Todos los empleados de una u.nión de cró:üto deben saber COBOL. Todos los empleados de la
unión de cr6ctito que se encaz¡an de las solicitudes de présWnos deben conocer Quattro. Roxana
trabaja pan la unióo de a6dil0, pero no sabe usar Quatuo. lnb sabe Quatuo pero no COBOL.
Por lo wuo, Roxana no se encarga de solícitudes de pctsaamos e ~ no trabaja para la unión
de cr6cfüo.
12. Dé una demostración directa (como en el te.or-ema 2.3) de lo siguiente.
a) Para todos los enteros k y /, si 1c. I son pares, entonces k + I es par.
b) Para todos los enteros l: y t. si A;. I son pares.. entonces kJ es par.
13. Realice una demostración indircaa (como ea la parte (z) del teorema 2.4] para cada una de liu
siguientes proposiciones; tú.galo estableciendo y demostrando la contrapositi va de cada pro•
posición.
a) Para todos los enteros k y l. si Id es impar, entonces tanto k como l son impares.
b) Para todos los cn1erosk y l. si k +les par, eotonocsk. y I son los dos pares o los dos impares.
14. Demuestre que para cualquier entero n, si n 2 es impar, entonces n es impar.
15. Realice una demostración por contradicción de lo siguiente: Para cualquier entero n. si n 1 es
impar, entonces n es impar.
16. Demuestre que para todo entero n, n 1 es par si y sólo si n ~ par.
17. Dcmuc:strc el siguiente rcsull:3do de tres formas (como en el teorema 2.4): Si n es un entero
impar, entonces n + 11 es par.
18. Sean m. n dos enteros positivos. Demuestre que si m. n son cuadrados pcrfeclos, en1onces el
producto mn tambitn es un cuadrado perfecto.
19. Demuestre. o dcmucsuc que es falso: Si m, n son enteros positivos y m. n son cuadrados per•
rectos, entonces m + n es un cuadrado perfecto.
20. Demuestre, o demuestre que es falso: Existen enteros positivos m, n tales que m, n y m + n son
cuadrados perfectos.
21. Demuestre que para todos los números realesx, y, si z +y~ 100. cntoneesz;;,:: 500 )' ~ 50.

2.6
Resumen y repaso histórico

En este segundo capítulo se han introducido algunos fundamentos de la lógica; en panicu•


lar, algunas de las reglas de inferencia y m~todos de demostración ne.cesarios para estable•
cer teoremas matemáticos.
El primer estudio sistemático del razonamiento lógico se e ncucmra en la obra del filó•
sofo griego Aristóteles (384-322 A.C.). En sus rratados de lógica. Aristóteles presentó una
colección de principios para el razonamiento deductivo. Estos principios tenían por objeto
ofrecer una base para el estudio de todas las ramas del conocimiento. En una fonna modi•
ficada. este tipo de lógica se enseñó hasta y durante la Edad Media.
El matemático alemán Gottfried Wilhelm Leibniz (1646-1716) ha sido considerado
frecuentemente como el primer filósofo que intentó desarrollar la lógica simbólica como
un lenguaje científico universal. Así Jo manifestó en su ensayo De Ant> Combinatoria,
publicado en 1666. Su investigación en el área de la lógica simbólica, realizada de 1679 a
1690, dio un gran impulso a la creación de esta disciplina matemática.
18

Aristóteles (384-322 a.c.)

Después de la obra de Leibniz. hubo pocos cambios hasta el siglo XIX, cuando el mate-
mático inglés George Boole (1815- 1864) creó un sistema de lógica matemática que pre-
sentó en 1847, en el panfleto The MathematicalAnalysis ofLogic, Being an Essay Towards
a Ca/culus of Deductive Reasoning. En el mismo año, el también inglés Augustus De
Morgan (1806--1871) publicó Fonnal Logic; or; the Calculus o/Jnference, Necessaryand
Probable. Su obra extendió considerablemente el trabajo de Boole en varias direcciones.
Después, en 1854, Boole expuso en detalle sus ideas y sus posteriores investigaciones en
su notabk obra An lnvestigation in the Laws of Thought, on Which Are Founded the
Ma1hematical Theories of Logic anti Probability. (Es interesante observar que, además de
su trabajo en el área de lógica, Boole también escribió libros de texto para el estudio de las
ecuaciones diferenciaJes y de las ecuaciones en diferencias. Estos libros de texto se usaron
en Inglaterra hasta el finaJ del siglo XIX.) El lógico noneamericano Charles Sanders Peirce
(1839- 1914), quien también era ingeniero y filósofo, inttodujo el concepto formal de
cuantificador en el estudio de la lógica simbólica.
Los conceptos fonnulados por Boole fueron ampliamente examinados en la obra de
otro estudioso alemán, Emst Schroder ( 1841- 1902). Estos resultados se conocen co-
lectivamente como Vorlesungen über die Algebra thr Logik:. se publicaran entre l 890 y
1895.
Los desarrollos posteriores en el área consideraban un enfoque incluso más moderno
que se vio en el trabajo del lógico alemán Gottlieb Frege (1848-1925) entre 1879 y 1903.
Esta obra influyó en forma significativa sobre los monumentales Principia Mathematica
(1910-1913) de los ingleses Alfred North Whitehead (1861-1947) y Bertrand Russell
( 1872-1970). Aquí rindió sus frutos la obra de Boole. Gracias a este enorme esfuerzo y a
las obras de otros matemáticos y lógicos del siglo xx, en panicular, la amplia Grundlagen
du Mathema1ik ( 1934-1939) de David Hilbert (1862-1943) y Paul Bernays (1888-1 977),
disponemos de las refinadas técnicas de la lógica matemática contemporánea.
2.6 Resumen y repaso histórico 139

Goorge Boole (1815-1864)

Varias secciones de este capítulo han subrayado la importancia de la demostración. En


matemáticas, una demostración confiere autoridad sobre algo que de otra fonna se dese-
charía como una simple opinión. La demostración incorpora el poder y la majestuosidad
del razonamiento puro; pero llega iocluso más allá: da lugar a ideas matemáticas nuevas.
Nuestro concepto de demostración va unido al deteonma (un enunciado matemático cuya
verdad se confirma por medio de un argumento lógico o dtmos1racWn). A quienes creen
que es posible pasar por alto la imponancia de la lógica y de las reglas de inferencia, les
dedicamos las sabias palabras de Aquiles en "Lo que la 1onuga dijo a Aquiles., de Lewis
Carroll: ...¡Entonces la lógica te tomará por el cuello y te obligará a hacerlo!"
Un tratamiento compara.ble del material presentado en este capítulo aparece en los ca-
pítulos 2 y 13 del texto de K.A. Ross y C. R. B. Wright (9Jy en el capítulo I de D. F. Stanat
y D. F. McAllister (1 I]. Los dos primeros capítulos del texto de S. S. Epp (3) proporcionan
muchos ejemplos y algunas aplicaciones a las ciencias de la computación para quienes
desean saber más de la lógica y las demostraciones en un nivel introductorio bastante
comprensible. El texto de H. Delong [2] proporciona un panorama histórico de la lógica
matemática. junto con un examen de la naturaleza de sus resultados y las consecuencias
ftlos6ficas de los mismos. Éste también es el caso de las obras de H. E ves y C. V. Ncwsom
(4), R. R. Stoll (12] y R. L. Wilder (13], en los que se examina la relación e ntre lógica,
demostración y teoría de conjuntos (el tema de nuestro siguiente capítulo) en su papel de
fundamentos de las matemáticas.
El texto de E. Mende:lson (7) proporciona una interesante introducción intermedia para
los lectores que deseen estudiar otros temas de la lógica matemática Un tratamiento poco
más avanzado es el de la obra de S. C. Kleene (6) . Un recuento de las obras recientes de
lógica matemática aparece en el compendio editado por J. Barwise [ l).
El objetivo de las obras de D. Fendcl y D. Rcsck (5] y R. P. Morash (8) es preparar al
estudiante con conocimientos de cálculo para las matemáticas un poco más teóricas que se
basan eo~l flgebra abstracta y el análisis real. Cada uno de estos textos ofrece una excelente
introducción a los métodos básicos de demostración. Por último, el texto dnico de Solow
[101 está dedicado en su totalidad a introducir aJ lector que ya cuenta con bases de mate-
40 Gaprtulo 2 Fundamentos de lógica

máticas de bachiHerato a las técnic.as primarias que se utilizan para desarrollar demostra-
ciones matemáticas.

BIBLIOGRAFÍA

l. Barwise, Jon (editor}, Handhock of Mathematical Logic, Amsterdam. North Holland. 19TI.
2. Delong, Howard, A Profik ofMmhematical Logic, Reading, Mass.• Addison-Wcsley, 1970.
3. Epp. Susanna S •• Discrtte MaJhanatics with Applicarions, Belmont, C'.al.if., Wadsworth, l 990.
4. Eves. Howard y Carroll V. Newsom, An lntroduction to the Foundo.tions an,d Fwuiamental
Concepts ofMathemalics, edición corregida, Nueva Yod:, Holt, 1965.
5. Fendel. Daniel y Diane Resek. Foundations ofHigher Mathematic:s, Reading, Mass.. Addison.
Wesley, 1990.
6. IGccnc, Stcphcn C., Mathemarical Logic, Nueva York, Wiley, 1967.
7 . Mendelson. Elliott, ln.trodJICtion to MathemaJical Logic, 3• ed., Montercy, Calit., Wadswonh
and Brooks/Cole, 1987. .
8. Morash. Ronald P.,_Bridge toAbstraer Ma1hcnatics: Ma1hematical Proofand Strucrures, Nue-
va York. Random House/Birkhaoser, 1987.
9. Ross, Kenneth A., y Charles R.B. Wright, Disc~k Ma1Mmatics. 3• ed.. Eng]ewood CJiffs.
NJ.. Prentice-Hall, 1992.
10. Solow. Daniel. How 10 Read and DoProofs, 2" ed., Nueva York, W'tley, 1990.
11. Stanat, Donald F. y David F. McAllister, Disc"t~ Mathmrarics in Compuur Scimce, Englewood
Oiffs.. N.J., Prentice-Hall, 19TI.
12. Stoll, Robert R., Set T'Mory and Logic, San Francisco, Frceman, 1963.
13. Wilder, Raymond L , lnrrodu.c1i<>rl to tire Fowrdations o/ MathemaJi.cs. 2"' ed., Nueva York.
Wiley, 1965.

EJERCICIOS b ) Traduzca la proposición dela parte (a) en palabras.


de modo que no aparezca la palabra "no'" en la tra-
COMPLEM ENTARIOS ducción.

3 . Sean p, q y r proposiciones primitivas. Demuestre la


1. Construya la tabla de verdad para
verdad o falsedad (con un contraejemplo) de lo siguiente:

•) [p-(q-,))<>[(p -q)- ,]
b) [p-(q-r)J<>[(p - q)-,]
!. a) Construya la tabla de verdad para
4 . Exprese la negación de la proposición p H q en térmi-
nos de los conectivos A y V.
Ejercicios Complementarios 141

S. Escriba la siguiente proposición como implicación, de posiciones primitivas. ( p J, q) ~ -.(p V q) y (pi q) -e=>
111 manent$, cada una de la forma si.entonces: Catalina - (p" q).)
lebert practicar sus lecciones de piano o no irá al cine. a) p ! (q ¡ r), (p ! q) ! r
l. Sean p, q, r proposiciones primitiv as. Escriba la recf. b) p t (q ! r), (p t q) ! (p t r)
aoca. la inversa y la contrapositiva de cada una de las si• <) p ! (q t r), (p ! q) i (p ! r)
~ implicaciones:
a) p ➔ (ql\r) b) (pVq)➔ r 13. La ley dd silogismo indica que para cualesquiera propo-
siciooesp, q, r.laproposición((p ➔q) /\ (q➔r)] ➔(p ➔ r)
J. a} Para las proposiciones primitivas p, q, encuentre
es una tautología. Recordemos que en el ejemplo 2.2 de la
el dual de la proposición (-p A - q) V (T0 A p)
sección 2.1 presentamos una situación hipotética para dar
V p. sentido a la tabla de verdad de la implicación p ➔ q, en
1,) Use las leyes de la lógica para mostrar que el re-
particular para el caso en que p tenía el valor de verdad O.
sultado de la parte (a) es lógicamente equivalente
En la tabla 2.25, tenemos lasocras tres tablas de verdad posi•
ap /\ -.q.
bles para la implicación, determinadas scg\1n las asignacio-
l. Sean p, q. r y s proposiciones primitivas. Escriba el nes de valores de verdad cuando pes falsa. {Así, la tercera y

..
11111 de cada una de las siguientes proposiciones compues•

•l (pV,q)/\(,r vs)
cuarta filas son iguales a las de la tabla 2.2 para la implicación.)
Muestre que para cada una de estas tres tablas de verdad
altcmati.vas, la proposición ({p ➔ q} /\ (q ➔ r)] ➔ (p ➔ r)
ya no es una tautología.
b) p ➔ (ql\ .r l\s)
e) [(pVTo)/\(qVFo)]V[r /\s/\To]
9. &cada uno de los siguientes casos, complete el espacio Tabla 2.25
:a blanco con la palabra~cíprcx:a, inversa o conrrapositiva p4 q p-3.q p-1. q
p q
lt modo que el resultado sea una proposición verdadera.
a) La rccfproca de la inversa de p ➔ q es la o o o 1 o
_ __ _ _ _ _ _ _ _ _ dep ➔ q. o 1 o o 1
b) La rccfproca de ta inversa de p
➔ q es la 1 o o o o
- -- - - -,-- - - -- -de q ➔ p. 1 1 1 1 1
e) La inversa de la recíproca de p ➔ q es la
_ __ _ _ _ _ _ _ _ _ dep ➔ q.

d) La inversa de la recíproca de p ➔ q es la 14. Establezca la validez del argumento


_ __ __ _ _ _ __ _ de,q ➔ p.

e) La recfproca de la contrapositiva de p ➔ q es la [(p--> q)/\[(q l\ r)--> s ] /\ r ]➔ (p -->s).


_ _ _ _ _ _ _ _ _ _ _ dcp ➔ q.

1 5. Pruebe la verdad o falsedad de lo siguiente ( p. q. r son


f) La recíproca de la conuapositiva de p ➔q es la
_ _ _ _ __ _ __ _ _ deq ➔ p.
proposiciones arbitrarias).
g) La inversa de la contrapositiva de p ➔ q es la a) [(p Yq) Y r]~[p Y (q Y r)]
_ _ _ _ _ _ _ _ _ _ _ dep ➔ q. b) [p Y (q➔ r)]~[(p Y q)➔ (p Yr))
e) [p ➔ (qYr)J ⇒ [(p-->q)Y(p --> r)]
10. Para las proposiciones primitivas p. q, r. verifique que
15¡:roposicioncscompuestasp /\ (q ➔ -.r)y -.(-.p V q) 16. Escriba cada uno de los siguientes argumentos en for.
I -.(-.p V r) son lógicamente equivalentes ma simbólica. Estableica dcsputs la validez del argumemo
a) utilizando tablas de verdad o proporcione un contraejemplo para mostrar que no es vá-
b) recurriendo a las leyes de la lógica, las reglas de lido.
sustirución y cualquier otra equivalencia lógica que a) Si hace fño el viernes. entonces Cristóbal utilizará
convenga. su abrigo si los bolsillos están remendados. EJ pro-
nóstico para el viernes es de clima frío. pero los
1. ¿Es asociativa la conectiva naná!, es decir, ¿son Iógica- bolsillos no están remendados. Por lo tanto. Cris•
-.equivalentes las proposicionesp j(q j r) y (p i q) i tóbal no usará su su abrigo este vie rnes.
part todas las proposiciones primitivas p. q. r? [Recordc- b} El contrato se cumplirá si y sólo si las nuevas ven-
>OS qu< (p j q) <a> - (p" q).) tanas se instalan en la casa en junio. Si las nuevas
2. Determine si cada una de las siguientes parejas de pro- ventanas se imtalan en junio.entonces Cristina po-
osicioncs es lógicamente equivalente. (p. q, r son pro- drá mudarse a su nueva casa el primero de julio. Si
12 (apítulo 2 Fundamentos de lógica

no se puede cambiar e l primero de julio, deberá a) p(O,O) b) p(l, 1)


pagar la renta de julio de su departamento. Las ven- <) p(O, 1) d) p(O, 3)
tanas se instalaron en junio o Cristina debe pagar e) Vy p(O,y) f) 3yp(l,y)
la renta de julio de su departamento. Por lo tanto. &) Vx,yp(x,y) b) Vx3yp(x, y)
Cristina no tendrá que pagar la renta de su departa- ;¡ 3yVxp(x, y) j) Vy3xp(x,y)
mento para el mes de julio.
. Consideremos la proposición abierta 18. Exprese lo siguiente en forma simbólica. El universo
está fonnado por los números rea1es positivos.
p(x,y): y-x=y + x 2 a) No existe un námero rea] positivo mínimo.
b) Existe un único número rca1 positivo igual a su cua-
nde el universo de cada una de las variables x, y abarca
drado.
k>s los enteros. Determine el valor de verdad para cada
e) Todo número real positivo tiene un único inverso
a de las siguientes proposicio nes.
multiplicativo.
3
Teoría de conjuntos

ccrás de las matemáticas que estudiamos en álgebra, geometría. combinatoria y casi


D todas las demás áreas de las matemáticas contemporáneas está el concepto de conjunto.
Con mucha frecuencia, este concepto proporciona una cstrucrura subyacente para una
formulación concisa del tema matemático en cuestión. En consecuencia. muchos libros de
matemáticas tienen un capítulo introductorio de teoría de conjuntos. o mencionan en un
~ndice aquellas partes de la teoría necesarias en el texto. En nuestro caso. al abrir el libro
con un capítulo acerca de los fundamentos del conteo. parecería que hemos dejado de lado
la teoría de conjuntos. En realidad, hemos confiado en la intuición; cada vez que aparecía
la palabra colución en el capítulo 1, hablábamos de un conjunto. También en las seccio-
nes 2.4 y 2.5 utilizamos el concepto de conjunto (aunque no el término) Cuando hablamos
del universo (de discurso) para una proposición abiena.
Tratar de definir un conjunto es bastante difícil y con frecuencia da lugar a un uso
circular de sinónimos como ..clase", ..colección"' y wagregado... Cuando comenzamos el
estudio de la geometría, utilizamos nuestra intuición para manejar las ideas de punto. línea
e incidencia. Después empezamos a definír nuevos términos y a demostrar teoremas con
base en estas nociones intuitivas, junto con denos axiomas y postul~dos. En nuestro cstu·
dio sobre la teoría de conjuntos volveremos a apelar a la intuición, esta vez para las ideas
comparables de elemento, conjunto y pertenencia.
Veremos que las ideas de la lógica desarrolladas en el capítulo 2 están íntimamente
ligadas a la teoría de conjuntos, y muchas de las demostraciones que estudiaremos en este
capítulo se basan en ellas. Este capítulo incluye también unos cuantos casos en los que se
aplica el tipo de demostración combinatoria (del capítulo 1).

3.1
Conjuntos y subconjuntos

Tenemos cierta ..noción intuitiva" en el sentido de que un conjunto debe ser una colección
bien definida de objetos. Estos objetos se llaman elementos y se dice que son mitmbros del
conjunto.
El adjetivo bien definid<J implica que para cualquier elemento que consideremos. pode•
mos determinar si está en el conjunto observado. En consecuencia, evitaremos trabajar
con conj untos que dependan de las opiniones, como el conjunto de los mejores lanzadores
de las ligas mayon:s de bfübol co la - de 1980.

143
144 Capítulo 3 Teoría de conjuntos

Utilizaremos letras mayúsculas, como A, B, C, ... , para representar los conjuntos y


letras minúsculas para representar los elementos. Para un conjunto A, escribiremos x E A
si x es un elemento de A; y ~ A indica que y no es miembro de A.

Un conjunto puede designarse enumerando sus elementos dentro de llaves. Por ejemplo,
si A es el conjunto formado por los cinco primeros enteros positivos, escribimos A = { 1, 2,
3, 4, 5}. En este caso, 2 E A pero 6 i A.
Otra notación común para este conjunto es A= {xlx es un entero y I S:x 5 5 }. En este
caso, la línea venica1 J que aparece dentro de las llaves se lee como "tal que•·. Los símbolos
{xi ... } se leen como "el conjunto de todos los x tales que ..... Las propiedades que van
después de I nos ayudan a determinar los elementos del conjunto descrito.
¡Tenga cuidado! La notación {x i 1 ::Sx ::S 5} no es u.na descripción adecuada del conjun-
to A, a menos hayamos acordado previamente que los elementos considerados son ente-
ros. Al adoptar esa convención, decimos que estamos especificando un universo, o unil,,er-
so de discurso, que por lo general se denota comoGU; así. sólo elegiremos e leme ntos de ~l
para fonnar nuestros conjuntos. En este problema particular, si 'ill denota el conjunto de
todos los enteros o el conjunto de todos los enteros positivos, entonces {x I I s; x s; 5} es
una descripción adecuada de A . Sil'.i\t es el conjunto de todos los números reales, {xi 1 S:x
S: 5} contendría todos los números reales entre I y 5 inclusive; si ú\t está formado solamen-
te ¡x,r enteros pares, los únicos miembros de {x i 1 S x S 5} serían 2 y 4.

Si 6ll= { 1, 2, 3, ... } es el conjunto de los enteros positivos. sean

a) A = {1,4, 9, ... ,64, 81} = {x'lxE'IL,x' < 100) = {x'lx E'U /\ x' < 100) =
{xE'ILlx'<IOO}.
b) B = {1, 4, 9, 16} = {y' IY E 'U, y' < 20} = IY'IY E 'IL, y' < 23) =
{y'ly E'IL /\y'< 17} = {y' E'IJ.ly's 16).
e) C = {2, 4,6,8, ... }={2klkE'IL}.

Los conjuntos A y B son ejemplos de conjuntos finitos, mientras que Ces un conjunto
infinito. A l trabajar con conjuntos como A o C, podemos describir los conjuntos en ténni-
nos de las propiedades que deben satisfacer sus elementos o enumerar los elementos sufi-
cientes para indicar, esperemos, un patrón evidente. Para cualquier conjunto finito A, lAI
denota el número de sus elementos y se conoce como el cardinal, o tamaño, de A. En este
=
ejemplo, tenernos que IA 1 9 y IB 1 4. =
En este caso, los conjuntos By A son tales que todo elemento de B es también un
elemento de A. Esta importante relación aparece en toda la teoría de conjuntos y sus apli-
caciones, y conduce a la siguiente definición.

Definición 3.1 Si C, D son conjuntos del universo6l.l , decimos que Ces un subconjunto de D y escribimos
C ~ D, o D ~ C, si cada elemento de C es un elemento de D. Si, además, D contiene un
elemento que no está en C, entonces C e s un subconjunto propio de D y se denota como
C C DoD::JC.
3.1 Conjuntos y subconjuntos 145

Observe que para cualesquiera conjuntos C. D del universoGU, si C ~ D, entonces

V.r[xec ⇒ xeD],

y si Vx[x E e=> X E D], entonces e!: D.


Aquí el cuantificador universal 'rfx indica que debemos considerar cada elemento x del
universo dado :\l. Sin embargo. para cada reemplazo e (elemento de l'.\l) tal que la propo-
sición e E C sea falsa. sabemos que la implicación c E C ➔ e E D es verdadera, indepen-
<tientemente del valor de verdad de la proposición e E D. E n consecuencia, realmente sólo
necesitamos considerar aquellos reemplazos e' (elementos de1ll) en los que la proposición
e' E C sea verdadera Si para cada e' tenemos que la proposición e' E D es verdadera,
entonces sabemos que 'v'x[x E C:;;;;;, x E D] o, en fonna equivalente. C !: D.
Además, para todos los subconjuntos C, D de"ll,

y cuando C, D son finitos,


C!:D ⇒ ICJ s l DI, Y
ccv ⇒ ICl<ID I.

Sin embargo, pan, 'l.l = {I, 2, 3, 4, 5], C = {l. 2] y D = {l, 2}. vemos que Ces un
subconjunto de D (es decir, C !;;; D), pero no es un subconjunto propio de D ( C <t. D ). Así.
en genera], no tenemos que C ~ D :;;;;;, Ce D.

El nombre de una variable en el ANSI FORTRAN (ANSI son las siglas del American
National Standards l nstitute, Instituto Nacional de Estándares de Estados Unidos) consta
de una sola letra seguida a lo sumo de cinco caracteres (letras o dígitos). Si í\l es el conjun-
to de todos estos nombres de variables, entonces, por las reglas de la suma y el producto.
l:l\tl = 26 + 26(36) + 26(36)' + • • • + 26(36)' = 26k°..36' = 1,617,038,306, de modo que
fill es un conjunto grande, aunque finito. Una variable entera en este lenguaje de programa-
ción debe comenzar con una de las letras I. J. K, L. M , N. Así, si A es el subconjunto de
todas las variables enteras en ANSI FORTRAN, entonees IA 1= 6 + 6(36) + 6(36)' + · · · +
= k'.. =
6(36)' 6 36' 373,162,686.

Podemos usar ahora el concepto de subconjunto para desarroJlar la idea de igualdad


entre conjuntos. Consideremos primero el siguiente ejemplo.

3A Para el univerro'U = (1, 2, 3, 4, 5}, consideremos el conjunto A = {l. 2}. Si B= [xjx' E


GU}, entonces los miembros de B son l , 2. En este caso, A y B contienen los mismos
elementos (y ninguno más), lo cual nos lleva a pensar que los conjuntos A y B son igualu.
Sin embargo, también es cien.o que A~ By B !;; A, por lo que preferimos definir la idea
de igualdad entre conjuntos mediante estas relaciones de contenido. Esto nos lleva a la
siguiente definición.

>efinici6n 3 .2 Para un universo dado "'· los conjuntos C y D (tomados de 'JU) son igual~s. y esto se
escribe e= D, cuando e !: D y D !: c.
146 CapítukJ 3 Teoría de conjuntos

A partir de estas ideas de igualdad entre conjuntos, vemos que el orden o la repetición
no son significativos para un conjunto en general. Así, tenemos, por ejemplo: { 1, 2, 3 } :-
{3, 1, 2); {2, 2, 1, 3) ; { 1, 2, !, 3, 1 }.

Ahora que hemos establecido los conceptos de subconjunto e igualdad entre conjuntos,
usaremos los cuantificadores de la sección 2.4 para analizar las negaciones de estas ideas.
Para un universo dado<\(, sean A, B conjuntos tomados de<i\l. Entonces podemos escribir

A <;B~Vx[xeA ⇒xEB].
De la definición ( cuantificada) de A ~ B, tenemos que

A <t. B (es decir, A no es subconjunto de B)


~-iV.r[.reA ⇒ .rEB]
e> 3.r -i[.r EA ⇒ xEB]
~ 3x-i[-,(.r EA) V .r E B]
~ 3x[.r EA /\-i (.r E B)]
~3.r[.rEA /\.r E,B].
Por lo tanto.A g; B si existe al menos un elcmentox en el Ulliverso tal quexes miembro de
A pero x no es miembro de B.
De manera análoga. como A = B <=> A, ~ B /1. B !:: A, entonces

Por Jo tanto, dos conjuntosA y B no son iguales si y sólo si ( l) e xiste al menos un elemento
x E Ci\l tal que x E A pero x ~ B, o (2) existe al menos un elemento y E G\.t tal que y E B
pero y ~A; o tal vez ocurran(!) y (2).
También observamos que para cualesquiera conjuntos C. D ~ <\L (es decir, C ~ "'itl y
D<; G\I),

CCD~Cr;.D I\ C/D.
Ahora que hemos presentado las cuatro ideas de pertenencia. igualdad entre conjuntos,
subconjunto y subconjunto propio, examinaremos un ejemplo más para ver que lo no
indican estos conceptos. Despué.s de este ejemplo, la demostración del primer teorema de
este capfculo será casi directa, ya que se sigue sin dificultad de algunas de estas ideas.

SeaG\1; ( 1, 2, 3,4, S, 6, x, y.{!, 2), { l. 2, 3), ( 1, 2, 3,4)) (donde.r, y son letras minúscu-
las del alfabeto y non:prescnt.an nada más, al igual que 3, 5 o ( !, 2)). Entonces, jill 1; 11.
a) Si A; ( 1, 2, 3, 4), entonces IAI = 4 y tenemos
1) A<;'l.l; H) AC'l.l; Hl) A E'l.l;
iv) {A}<;'l.l; v) {A}C'l.l; pero vi) {A}i'l.l.
b) Ahora sea B; {S. 6,.r, y,A) ; (S. 6, .r, y.(l, 2, 3, 4}}. Entonces IBI; S, no 8. Y
ahora vemos que
i) AEB; ii) {A)r;.B; y, ill) {A)CB.
3. 1 Conjuntos y subconjuntos 147

Pero
iv) {A }éB;
v) A r/;. B (es decir, A oo es subcoojunto de B); y
vi) A t B (es decir, A no es subconjuo10 propio de B).

:<>REMA 3.1 Sean A, B,C ¡;; "11.

a)SiA !:;; B yB !:C,entoncesA !:C. b) Si A C By 8 ¡;; C,entoncesA C C.


c)Si A!:;; 8 y 8 C C, entonces A C C. d) Si A C 8 y B C C, en1oncesA C C.

Antes de demostrar este teorema queremos recordar un comentario de la sección 2.5,


relativo a nuestro tratamiento de las reglas de especificación y geoeralización universales,
que aparcc.ió antes de la definición 2.8. ya que es apropiado en esta nueva área de la teoría
de conjuntos. Por ejemplo, cuando queremos demostrar que x E A ~ x E C. debemos
comenzar considerando cualquier elemento x e n GU, fijo pero elegido en fonna arbitraria;
queremos que este elemento x sea tal que "'x E A" sea una proposición verdadera (no una
proposición abiena). Entonces, debemos mostrar que este mismo elemento x. fijo pero
elegido en forma arbitraria. tambitn está en C. Las demostraciones que presentamos a
continuación se conocen comoargumou.os de penenencia de un elemento. Siempre habrá
que tener presente que, en todas estas demostraciones,x representa un elemento de A, fijo
pero elegido en fonna arbitraria; y aunque x sea genérico (ya que n.o es un elemento espe-
cífico de A), es el mismo durante toda la demostración.
Demostración: nos dedicaremos a las partes(a) y (b) y dejaremos el resto para los ejerci-
cios.

a) Para demostrar que A ~ C, necesitamos verificar que para todo ~ E'>l.l, si x E A


entoncesxEC. Panimos deun elementoxdeA. ComoA ~ B. x E A implicax E B.
Entonces.conB C C,x E BimplicaxE C.Asf, x E A implka.x E C(porla leydel
silogismo, regla 2 de la tabla 2.20, yaquex E A, x E B y x E Cson proposiciones)
y A¡;; C.
b ) ComoA C 8, six EA entoncesx E B.ConB ~ C, sesigueentoncesquexE C, pOr
loqueA S: C. Sin embargo A C B ~ existeunelemcntob E Btal queb (l.A. Como
8 ¡;; C,b E 8 => b E C.Asf, A ~ Cyexiste unelementob E C con b 11. A, por Jo
que A C C.

Nuestro siguiente ejemplo se refiere a varias relaciones de contenido.

=
Sea "ti= {1, 2, 3. 4, 5}. con A• { 1, 2, 3). 8 {3, 4} y C ={
1, 2, 3, 4}. Entonces se
cumplen las siguientes relaciones de contenido de subconjuntos.

a) A!:;;C b) ACC
<) BCC d) A!;;A
e) Br;/;,_A t ) A <i A (esdecir,A noes un subconjunto propio de A)
148 capítulo 3 Teoría de conjuntos

Los conjuntos A, B son sólo dos de los subconjuntos de C. Nos interesa determinar
cuántos subconjuntos tiene C en total. Sin embargo. antes de responder esto, necesitamos
presentar el conjunto sin elementos.

l>efinición 3.3 El conjunto vacío. o nulo, es el ('1nico) conjunto que no contiene elementos. Se denota
como 0 o {l.

Observemos que l 01 = O, pero (O) t, 0. Así mismo, 0 t, ( 0). ya que ( 0 ) es un


conjunto con un elemento, a saber, el conjunto vacío.
El conjunto vacío satisface la siguiente propiedad dada en el teorema 3.2. Para estable•
ccr dicha propiedad usamos el métcxlo de demostración por contradicción (o reducción al
absurdo). Después de la demostración del teorema 2.4 (en la sección 2.5), dijimos que, al
establecer un teorema con este método, habíamos supuesto la negación del resultado y
llegado a una contradicción. En nuestro trabajo anterior (como en el ejemplo 2.33 y la
tercera demostración del teorema 2.4), llegamos a una contradicción de la formar A -.ro
p(_m) A -.p<_m), respectivamente, donde -.r era una premisa del ejemplo 2.33 y /J{.m) un
caso específico de la hipótesis del teorema 2.4. Las cosas varían un poco para la demostra-
ción del teorema 3.2. Esta vez estaremos negando ( o contradiciendo) un resu.lea.do anterior
que aceptamos como verdadero, esto es, la definición del conjunto vacío.

c0REMA3.2 Para cualquier universo 6U, sea A C Gtl. Entonces 0 !: A y si A#. 0, entonces 0 CA.
Demostración; Si el primer resultado no es verdadero, entonces 0 ~ A. por lo que
existe un elementoxdel universo tal que xE 0 perox fJ. A. Pero x E 0es imposible. Así.
rechazamos la hipótesis 0 ~ A y vemos que 0 ~ A. Además, si A '/: 0, entonces existe un
elemento a E A (y a ,t 0). por lo que 0 e A.

Si volvemos al ejemplo 3.6, determinaremos el número de subconjuntos del conjunto


C = { t, 2, 3, 4}. Al construir un subconjunto de C, tenemos dos opciones diferentes para
cada elementoxde C: xestá incluido e n el subconjunto o no está incluido. En consecuen-
cia. existen 2 x 2 x 2 x 2 opciones, lo que produce 24 = 16 subconjuntos de C. Esto incluye
el conjunto vacío 0 y el propio conjunto C. Si necesitamos el número de subconjuntos de
C que tengan exactamente dos elementos, el resultado es igual al número de formas en que
pcxlemos seleccionar dos objetos de un conjunto de cuatro objetos, es decir, C(4, 2) o{;).
Como resultado, el número total de subconjuntos de C, 2 4, es también la suma ( ri) + (:)+ {;)+
(;)+{!),en laque el primer sumando corresponde a1 conjunto vacío, el segundo suman-
do corresponde a los subconjuntos de un elemnto, el tercer sumando a los seis subconjuntos
de tamaño dos, etc.étera. Así. 2" =k. 0
(1)-

lefinició n 3.4 Si A es un conjunto del universovtl, el conjunto pouncia, que se denota~(A), es la colec-
ción (o conjunto) de todos los subconjuntos de A.
3.1 Conjuntos y subconjuntos 149

P>ra el conjunto C del ejemplo 3.7.~(C) = { 0,( I}, {2}, (3}, (4}, ( t, 2}, ( 1, 3}. ( 1, 4}.
(2. 3}, {2, 4}, (3, 4}. ( t. 2, 3}, ( 1.2, 4}, {1.3. 4}, (2. 3, 4}, C}.

Esta identidad ya fue establecida en el corolario t.l(a). Esta ¡nsentación es otro ejem-
plo de una demostración combinatoria,. ya que la identidad se establece contando la misma
colección de objetos Oos subconjuntos de A) de dos fonnas diferentes.
La capacidad para conw algunos, o todos. los subconjuntos de un conjunto dado pro-
porciona un segundo método para la solución de dos de nuestros ejemplos anteriores.

En el ejemplo 1.14, contamos el ntlmero de trayectorias (escalonadas) en el plano ,,y que


van de (2, 1) a (J. 4), donde cada trayectoria está formada por.escalones individuales que van
una unidad aladcr<eba(R) o una unidad hacia arriba (U). Lafigura3.t es igual ala figwa
1.1, donde se muestran dos trayectorias posibles.
La trayectoria de la figura 3.l(a) tiene tres movimientos hacia arriba (U) localizados en
las posiciones 2. S y 8 de la lista que aparece en la parte inferior de la figura. En conse-
cuencia. esta trayectoria determina el subconjunto de eres elementos {2. s. 8} del conjunto
{ 1, 2, 3, ... , 8}. En la figura 3.l(b), la trayectoria determina el subconjunto de tres
elementos { 1, 5, 6}. En forma recíproca. si partimos, por ejemplo, del subconjunto ( 1, 3, 7}
de { 1, 2, 3, ... , 8}, entonces la trayectoria que determina este subconjunto está dada por
U, R. U, R. R. R, U, R.
Por lo tanto. el m1mcrodc trayectorias buscada es igual al número de subconjuntos A de
{ 1, 2, 3, . . . , 8}, donde IAI = 3. Existen (~)=3~!,•56 de esas trayectorias (y
subconjuntos), como vimos en el ejemplo 1.14.

234S67 234567
(a) R.U.R.R.U,R.R.U (b) U,R.R.R.U.U.R.R

Figura 3.1
capítulo 3 Teoría de conjuntos

Si hubiésemos considerado los movímientos R hacia la derecha, en vez de los movi-


mientos hacia arriba U , tendríamos que la respuesta sería el nilmero de subconjuntos B de
= 8
( 1, 2, 3, ... , 8), donde IBI S. Existen ( 5 )=~=
5!3!
56 de tales subconjuntos. (Esta idea
fue analizada antes para el resultado desarrollado en la tabla 1.4.)

En la parte (b) del ejemplo 1.36 de la sección 1.4, aprendimos que e xisten 26 composicio-
nes del entero 7; es decir, hay ~ formas de escribir 7 como una suma de uno o más enteros
positivos, donde el orden de los sumandos es significativo. El resultado obtenido utilizaba
el teorema del binomio junto con las respuestas de siete casos resumidos en la tabla l .7.
Ahora podemos obtener este resultado en una forma un tanto diferente y más sencilla.
Primero consideremos la siguiente composición de 7:

1 + 1 + 1 + 1 + 1 + 1 + 1
' i ¡ \,

Aquí tenemos siete sumandos, cada uno de los cuales es l, y seis signos más(+).
Para el conjunto { 1, 2, 3, 4. 5, 6}, existen 2 6 subconjuntos. ¿Pero qué tiene estoque ver
con las composiciones de 7'!
Consideremos un subconjunto de ( 1, 2, 3, 4, 5, 6}, digamos ( 1, 4, 6}. Formemos ahora
la siguiente composición de 7:

(! + !) + 1 + (1 + 1) + (1 + 1)
¡ ~ ~
1... signo 4º si;no 6º signo

'"" """ """


En este caso, el subconjunto { 1, 4, 6} indica que debemos colocar paréntesis entre los
unos que aparecen a los lados del primero, cuarto y sexto signos más. Esto produce la
composicíón

2+1+2+2.

De la misma forma, vemos que el subconjunto { 1, 2, 5, 6} indica el uso del primero, segun-
do, quinto y sexto signos más, lo que da

(! / 1 t 1) + 1 + (! / 1 t 1)

o la composición 3 + 1 + 3.
En sentido contrario, vemos que la composición 1 + 1 + 5 proviene de

1 + 1 + (! + 1 + 1 + 1 + !)
3.1 Conjuntos y subconjuntos 151

y eslá de1erminada por el subconjunlo {3, 4, 5, 6) de{!, 2, 3, 4, 5, 6}. En la tabla 3.1


tenemos enwneradas ocho composiciones de 7, junto con el subconjunto correspondiente
de {l, 2, 3, 4, 5, 6} de1erminado porcada una de ellas.

Tabla 3.1
Compooldón de 7 Subcoyulto del,rminado de {I, 2, 3, ', 5,6)
(i) 1+1+1+1+1+1+1 (i) e
(ü) 2 +1+1+1+1+1 (ü) {1}
(iü) 1+2+1+1+1+1 (iü) {2}
(iv) 1+1+3+1+1 (iv) {3,4)
(v) 1+2+1+3 (v) {2,5,6)
(vi) 2+3+2 (vi) {1,3,4,6}
(vü) 4+3 (vü) {1, 2, 3, 5, 6)
(viü) 7 (viü) {J,2,3,4,5,6}

Los ejemplos obtenidos hasta ahora muestran u.na correspondencia entre las composicio-
nes de 7 ylos subconjuntos de {1, 2, 3, 4, s. 6}. Por lo tanto, de nuevo tenemos que existen
2' composiciones de 7.

Nuestro siguiente ejemplo genera otra identidad combinatoria imponante.

&íí¡,l,'1 1 Para los enleros n, r con n ~ r > 1,

Aunque podemos establecer es1C resultado en forma algebraica a partir de la definición


ª"
de {:) como n!/(r!(n - r)!). usaremos un enfoque combinatorio. SeaA = {x, a 2, • •• , a ..}
y consideremos todos los subconjuntos de A que contienen r elementos. Hay (-;•) de tales
subconjuntos. Cada u.no de ellos cae exactamente en uno de los dos casos siguientes:
aquellos subconjuntos que contienen el elemento.r y los que oo lo contienen. Para obtener
un subconjunto C de A, donde .r E C y ICJ = r , colocamos x en C y despu~s elegimos
r-1 elementos de losa1. ~ ••• ,a_. Esto se puede hacer de (,=1) formas. Parael otro caso.
queremos un subconjuntoB de A con IBI ;;; r y .r E 8.Así, podemos elegir relementos de
los a 1, Qi, . .• , a.. lo que podemos hacer de(:) formas. De la regla de la suma se sigue que
(·;•)= (:)+ (,:,}

Antes de continuar, volvamos a analizar el resultado del ejemplo 3.11, pero esta vez a la
luz de lo aprendido en el ejemplo 3.9.
De nuevo, sean n. r enteros positivos tales que n ~ r e!: l. Entonces ( •:1 ) cuenta el
mlmero de ttayectorias (escalooadas) eo el planoiy, de (O. O) a (n + 1 - r, r), donde, como
en el ejemplo 3.9, cada una de las trayectorias tiene

(n + 1)- r movimientos horizontales de la forma (x, y) -+ (x + 1, y), y


r movimientos verticales de la forma (.r. y) ➔ (.r, y+ 1).
52 capítulo 3 Teoría de conjuntos

La óltima arista de cada una de estas trayectorias (escalonadas) termina en el punto


(n + 1-r. r ) y comienza en (i) el punto (n- r , r) o(ii) el punto (n + 1-r, r- 1).
En el caso (i), tenemos la OJtima arista horizontal, es decir, (n - r, r) 4 (n + l - r , r); el
número de trayectorias (escalonadas) de (O, O) a (n - r, r ) es {(n-:►•r) =(:} Para el caso
(ii), la última arista es vertical, (n + l - r, r- 1) ➔ (n + 1 - ,, r); el número de trayectorias
(e scalonadas) de (O, O) hasta (n + 1- , , r- I) es (c11 ♦i-;2t<'-n )= (,: 1 ). Como estos dos
casos abarcan todas las posibilidades y no tienen nada en común, se sigue que

Analizaremos ahora la forma en que la identidad del ejemplo 3.11 nos puede ayudar a
resolver el ejemplo 1.34, donde buscábamos el número de soluciones enteras no negativas
de la dcsigualdadx1 +Xi+· ··+ x6 < 10.
Para cada entero k, O S k $ 9, el número de soluciones de x 1 + x, + · · · + x 6 = k es
(
6
"":-
1
)=(SZ"). Así que el número de soluciones enteras no negativas parax + x
1 2 + · ·· +
x.s< 10 es

(~) + mmm + +
4
+ ---+ ( ~ )

= l(~)+ (1)J+ m+ m+. •• + (~ 4


)

= lm +m J+ m+ ... + (~ = lm+ mJ+ m+ ---+ (~


4 4
) )

= [(;) + (!)] + . .. + (194) = ... = (~4) + (~4) = (~) ='5005.

En la figura 3.2 tenemos parte de una útil e interesante disposición de números llamada el
triángulo de Pascal.

(n - O) (g)
(n • 1) {~) (:)
(n : 2) @ m m
(n : 3) @ m m
(n - 4) (6) (i) ~)
(n - 5) @ m m
Figura 3.2
3.1 Conjuntos y subconjuntos 153

Observe que, en esta lista parcial, los dos ttiángulos que se muestran satisfacen la condi-
ción de que el coeficiente binomial de la parte inferior del triángulo invertido es la suma
de los otros dos términos del ttiángulo. Este resultado se sigue de la identidad del ejemplo
3.11.
Cuando reemplazamos cada uno de los coeficientes binomiales por su valor numérico,
el ttiángulo de Pascal toma la apariencia que se muestra en la figura 3.3.

(n = O)

(n = 1)

(n - 2)

(n - 3)

(n - 4 )

(n = 5) 10

Figura 3.3

Existen algunos conjuntos de números que aparecen con frecuencia en todo el libro. En
consecuencia, cerraremos esta sección asignándoles los siguientes nombres.
154 Capítulo 3 Teoria de conjuntos

EJERCIOOS 3.1 1 . ¿Oláles de los siguientes conjuntos son iguales?


a) {1,2, 3) b) {3, 2,1,3) <) {3,1,2,3) d) {1,2, 2,3)
2. Sea. A = { l, { 1}, 2 }}. ¿C\l.áles de las siguientes proposiciones son verdaderas?
a) lEA b) {l}EA e) {l}c;;A d) {{l}}c;;A
<) {2}EA f) {2}c;;A &) {{2}}c;;A b) {{2}}CA
3. Para A :; ( 1.2,{2} }, ¿cuáles de las ocho proposiciones del ejen:icio 2 son verdaderas?
4. ¿Cuáles de las siguientes proposiciones son verdaderas?
•> eee
d) 0E{8}
b)
<)
6c6
6C{ll}
<)
o
•e:•
ec;;¡e¡
5. Determine todos los e lementos de cada uno de los siguientes conjuntos.
a) {l+(- lrlnEN} b) {n+(l/n)lnE{l,2,3,5,7))
e) {n'+n'lnE{0,1, 2,3,4)}
d) {1/(nl+ n) ln es un entero positivo impar y n S 11}
6. Consideremos los siguientes seis subconjuntos de Z:
A={2m+l lmEZ}; B={2n+3lnEZ}; C={:1p-3lpEZ};
D•{3r + l lrEZ}; E={3s+2lsE Z}; . F={31 - 2 ltE Z}.
¿CUáles de las siguientes proposiciones son verdaderas y cuáles falsas?
a)A =B b)A =C <)B=C
d) D=E e) D=F f) E=F

7. Sean A, B conjuntos de un universo GU.


a) Escriba una proposición cuantificada para expresar la ttlación de contenido propia A C B.
b) Niegue el resultado de la parte (a) para determinar cuándo A <t B.
8. J>ara,1 = {!, 2, 3. 4. S. 6. 7).detcmúne el nómero de
a) subconjuntos de A. b) subconjuntos no vacíos de A.
e) subconjuntos propios de A. d) subconjuntos propios no vados deA.
e) subconjuntos de A que contienen tres elementos. f) subcoojurtosdeA. que contienen l. 2.
e) subconjuntos de A que contienen cinco elementos, incluyendo 1, 2.
b ) subconjuntos propios de A que contienen 1. 2.
O subconjuntos de A con un mlmero par de elementos.
D subconjuntos de A con un mimero impar de elementos.
k) subconjuntos de A con un o6mero impar de elementos y que incluyen d-elemento 3.
9. a) Si un conjunto A tiene 63 subconjuntos propios, ¿cuánto vale IA I?
b) Si un conjunto B tiene 64 subconjuntos de cardinal impar, ¿cuánto vale IBI?
e) Geoeralice d resultado de la parte (b).
1 O. ¿Cuáles de los siguientes conjuntos son no vacíos?
a) lx lzEN,2':+7=3} b) {xEZllr+5 • 9}
<) lx lzEQ,.r+4=6} d) {,ERl.r+4•6}
•l {x lzER,.r + 5=4} f) {zERl.r+lr+3- O}
a) lx lzEC,.r+lr+3=0}
11. Cuando est4 a punto de salir de un restaurante, un hombre nota que tiene una moneda de 1
centavo. oua de S, una de 10. una de 25 y una de SO centavos de dólar. ¿De cuántas formas
puede dejar una (al meoos una) de sus monedas para la propína si a) no bay restricciones? b)
quiere quedarse con algo de cambio? e) quiere dejar al mmos 10 centavos?
12. Para'\1 = Z•.s,aA c;;C\1 el conjunto A= (l. 2. 3.4, S. 1. 8. 10, 11.14.17. 18).
a) ¿Oliotos subconjuntos de A contienen seis elementos?
3.1 Conjuntos y subconjuntos 155

b) ¿CUántos wbconjuntos de seis elementos (de A) contienen cuatro enteros pares y dos ente-
ros impares7
e) ¿Cuántos subconjuntos de A sólo conticocn enteros impares?
d ) ¿Cuintos de los subconjUDlos de la parte (e) contienen los enteros 3 y 7?
13. ScaS= {l. 2. 3, .. . , 29, JOJ. ¿C:Uán1os subcoojuruosA ele Ssalisfac<n
• llAl = 5?
b) IAI = S y que el mínimo elemento de A sea S?
e) IAI = S y que el mínimo elemento de A sea menor que S?
14. a) ¿Cuántos subconjuntos de ( l. 2. 3, ...• 11} contienen al mcn0$ un entero par?
b) ¿Cuántos subconjuntos de (1 , 2. 3•. .. , 12} conticncD al menos un entero par?
e) Generalice los t=tltados ele las partes (a) y (b).
1 S. ot un ejemplo de tres conjuntos W, X. Y tales que W E X y X e Y pero W 11. Y.
16. Escriba las siguientes ues filas del triángu]o de Pascal de la figura 3.3.
17. Complete la demostración del l<Orcma 3. l.
18. Para los conjuntos A.. B. C C Ci\.l, demuestre la verdad o falsedad (con un contraejemplo) de lo
siguiente: Si A C: B, B g;; C.entooccsA ~ C.
19. F.o la parte (i) de la figura 3.4, tenemos las primeras seis filas del triángulo de Pascal. en los
que aparece un beúgooo centrado en 4. en las llltimas tres füu. Si consideramos los seis
números (que rodean a 4) como los ~ del heúgono, tenemos que las dos temas aJtcrna-
das (3, 1, l0y 1, 5,6)satisfacenquc3-1 • 10•30= 1. 5 • 6. Laparte (ü) elela figuiacooticoe
las filas 4 a 7 del mismo tri1n¡ulo, donde vemos un bellgono cuyo centro está en 1O; las temas
al1emad.u de los vtnices (4, 10, 15 y 6, 20, 5) satisfacen 4-1().15 = 60() = 6 · 20 • 5.
a) Conjeture el resultado general al que apuntan estos dos ejemplos.
b ) Verifique la conjetura de la parte (a).

301
10
6

10
4

5
1

1
1 3 3 1

í•)

Figura 3 .4

20. a) Algunas de las sucesiones estrictamente cre<:ientes de enteros que comienzan con l y termi-
nan con 7 son:
0 1, 7; iO 1,2, 7; iü) 1, 4, 7;
i.J 1,3,4, 7; •l 1,2,3, 4, 7; ,o 1,2, 4,5, 6, 7.
¿Olámasdeestas,sucesionesesuictamtncaec:ic:ucsdeemcroscormcmaoc.on I ytcnninanen T!
b) ¿Cuántas de estas sucesiones estrictamente crecientes de enteros comienzan con 3 y ternti-
nan cn 9?
156 capitulo 3 leoría de conjuntos

e) ¿Cuántas de estas sucesiones estrictamente crecientes de enteros comienzan con 1 y termi•


nan en 37? ¿CUántas comienzan con 62 y terminan en 98?
d ) Generalice los resultados de las panes (a) a (e).
21. Un.a cuarta parte de los subconjuntos de cinco elementos de {l, 2. 3, ... , n} contienen el
elemento 7. Determinen (n ~ 5).
22. Establezca ta identidad del ejemplo 3.11 en forma algebrajca.
23. ~ un argumento combinatorio para mostrar que para los enteros n. r con n ~ r ~ 2,

(n ;2)-(;) +{: i) +(,: 2).


24. Para los enleros positiv0$ n. r, muestre que

(n +;+l)~(n;') +(n ;~/)+...+(n;2) +(n ;1) +(~)-


25. F.n la teoría abstracta geDCW de conjuntos, formulada por Georg Cantor (1845--1918), un
conjunto se definía como ..cualquier colección de un todo de objetos definidos y separados en
nuestra inblición o pensamiento... Por desgracia. en 1901, esta definición condujo a Bertrand
Russell (1872-1970) al descubrimiento de una contradicción. un resultado que se conoce aho-
ra como la paradoja tk Rwu ll, lo cual fue un golpe al ccntto de la teoría de conjuntos. (Pero,
desde entonces, se han encontrado varias vías para definir: las ideas ~icas de la teoñ a de
conjuntos de modo que esta contradicción ya no aparezca..)
La paradoja de Russell surge cuando nos preguntamos si un conjunto puede ser un ciernen+
to de st mismo. Por ejemplo, el conjunto de todo$ los enteros positivos no es un entero positi~
vo; es decir. z• fl. z•. Pero e1 conjunto de todas las abstracciones es una abstracción.
Ahora bien. para de$arrollar la paradoja de Russell, seas el conjunto de todos los conjuntos
A que.no son miembros de sf mismos; es decir, S= {A JA es un conjunto /\A~ A}.
a) Muestre que si S E S, entonces S r,. S.
b) Muestre que si S f. S, entonces SE S.
Los resultados de las partes (a) y (b) muestran que debemos evitar definir conjuntos como S.
Para hacer esto. debemos restriogír los tipos de elementos que pueden ser miembros de un
conjunto. (Hablaremos más de esto en el resumen y repaso histórico de la sección 3.5.)
26. SeaA=(l,2,3, ... ,39,40}.
a) Escriba un programa (o desarrolle un algoritmo) que genere los subconjuntos de seis ele-
mentos de A.
b) ParaB = (2, 3,6, 7, 11, 13, 17, 19, 23, 29, 31. 37},escriba un programa de romputadora(o
desarrolle un aJgoritmo) quegeoerc un subconjunto de seis elementos deA. y desputsdeter-
mine si es un subconjunto de B.
27. SeaA- {1, 2,3, .. . , 7}. Escriba un programa (o desanolle un algoribno) que enumere todos
los subconjuntos B de A, tales que IBI =4.
28. Escriba un programa (o desarrolle un algoritmo) que imprima todos lo$ subconjuntos de { 1. 2.
3 .... , n}. donde l S n S 10. (El valor den debe proporcionarse durante la ejecución del
programa.)

3 .2
Operaciones de conjuntos y las
leyes de la teoría de conjuntos

Después de aprender a contar, el esrudiante por Jo regular se enfrenta a los mélodos para
combinar los 06.meros contados. _El primer método es la suma. Generalmente, el mundo
3.2 Operaciones de con·untos las leyes de la teoria de con·untos 157

arittmtico de los estudiantes gira en torno al conjunto Z • (o un subconjunto de Z • del que


puedan hablar y escribir, así como teclearlo en una calculadora de bolsillo). donde la adi•
ción de dos elementos de z• produce un tercer elemento de z·. Uamadosuma. Por lo tanto,
los estudiantes pueden concentrarse en la adición sin tellCf que ampliar su mundo aritmé•
tico más allá de z•. Esto tambifn es cieno para la operación de mu1tiplicación.
La adición y la multiplicación de enteros positivos son operaáoMs binarias cerradas
en z ·. Por ejemplo. cuando calculamos a+ b, para a. b E z •, hay dos o~randos. a y b;
por ello, la operación se llama binaria. Como a + b E z· si a. b E z•, decimos que la
operación binaria a:lición (en Z•) es ctmida. Sin embargo, la operación binaria de la divi-
z.,
sión (coo divisor distinto de cero) no es cenada en z•, ya que, por ejemplo, 1/2(: 1 + 2) ~
aunque 1, 2 E z•. Pero esta operación sí es cerrada cuando consideramos el conjunto Q•
en lugar del conjunto z •. (Veremos los coocepc:os generales de operación binaria y opera•
ción binaria cerrada en la sección 5.4.)
Ahora presentamos algunas operaciones binarias para conjuntos.

Dofinici6n 3.5 Para A, B ~ G\l definimos lo siguentc:


a) AUB(launi6ndeAyB) : {xlxEA V x EB).
b) A n B (la inttrs,cci6n de A y B): (xlx E A /\ x E B).
e) A ó C (la difemu:ia simitrica de A y B): (xl(x E A V x E 8 ) /\ x ~ A n B):
(xlxEA U B II x~A n B).

Observe que si A, B !; 'll , entonces A n B, A u B,A 6. B !; 1\1. En consecuencia. n, U


y 6 son operaciones binarias cerradas en~ ('ll), y tambifo podemos decir que lf 01.l) es
cerrada. en estas operaciones (binarias).

~3.1-t' Si"U: (1.2,3, . . , 9, 10),Az {1,2,3, 4,5),B:{3,4, 5, 6, 7) yC: (7, 8,9),tenemos:

a) A n B = {3,4,S} b) A U B = {l,2,3, 4,S,6,7}


e) B nC = {7} d) Anc = e
=
e) A 6B {1,2,6, 7) C) A UC = {l,2,3, 4,S,7,8, 9}
e) A óC = {l,2,3,4,S,7,8,9}

En el ejemplo 3.14 vemos que A n B !;;A ~A U B. Este resultado noes algo panicular
de este ejemplo, sino que es verdadero en general y nos referiremos a fl con frecucnc:ia. El
resultado se obtiene de

xEA n B ~(.tEA Á .tEB)~.tEA

(por la regla de simplificación conjuntiva. regla 7 de la tabla 2.20) y

(donde la primera implicación lógica es un resultado de la regla de amplificación disyuntiva.


regla 8 de la tabla 2.20).
158 Capítulo 3 Teoría de conjuntos

De las partes (d), (f) y (g) del ejemplo 3.14, presentamos las siguientes ideas generales.

[)efinici6n 3.6 Sean S, T ~ G\L. Los conjuntos S y T son disjumos o mutuamente disjuntos si S n T"" 0-

rEOREMA3.3 Si S, T ~ Gtl • entonces S y T son dísjuntos si y sólo si S U T = S ó T.


Demostración: Panímos de S, T disjuntos. (Para demostrar que S U T • S ó T utilizamos
la definición 3.2. En panicular, daremos dos argumentos de pertenencia, uno para cada
inclusión. Puesto que ésta e s nuestra primera experiencia en la demostración de la igual-
dad de dos conjuntos, seremos cuidadosos y muy detallistas.) Considere mos cualquier
x E Gt.l. Six ES U T, entoncesx ES ox E T(o tal vez ambos). Pero como S y Tson
disjuntos, x ~ S n T, por lo quex E S ~ T. En consecuencia, comox ES U T implicax E
S 6. T, tenemos S U T ~ S /l. T. Para la inclusión opuesta. si y E S 6 T, entonces y E S o y
E T. (Pero y ~S n T; nopodemosusarcsto.)Así,y ES U T. Porlotanto, S 6 T<;;; SU T.
Y ahoraque tenemosS U T~ S 6 Ty S 6 T~ SU T, se sigue de la definición 3.2que S
é. T=S UT.
Demostramos la recíproca por el método de demostración por contradicción. Para esto,
considerarnosS,T~ ~ arbitrarios.conservamos la hipótesis (es decir, S U T - S 6 n.
pero
suponemos la negación de la conclusión (es decir, suponemos que S y T no son disjuntos).
Así, si S (l T:;:. 0, seax ES n T. Entonces x E Sy x ET, por lo quex ES U Ty

xESt;T(=sun.

Pero cuando x E S u r y x E S n T, entonces

Esta contradicción (x E S 6 T A x ,t S 6 7) indica que nuestra hipótesis original era


incorrecta. En consecuencia, tenemos que S y T son disjuntos.

Al demostrar la primera pane del teorema 3.3, mostramos que si S, T son conjuntos
cualesquiera, entoncesS6 Tk SU T. El hecho de que los conjuntos fue.ran disjuntos sólo
fue necesario para la inclusión opuesta.

Después de adquirir la capacidad de sumar, se pasa a la resta. En este caso, el conjunto


N preserita algo de dificultad. Por ejemplo, N contiene a 2 y 5 pero 2 - 5 ~ - 3 y -3 f1. N.
Por lo tanto, la operación binaria de sustracción no es cerrada para N, aunque es cerrada
para el superconju.n.to Z de N. Así, para Z podemos intrcxlucir la operación unaria. o
monaria, de la negación, en la que tomamos el "menos.. o ..negativo" de un número como
el 3, y obtenemos - 3. (La definición ge neral de una operación unaria, o monaria, aparece
en la sección 5.4.)
Ahora presentamos una operación unaria comparable para los conjuntos.
3.2 Operaciones de conjuntos ylas leyes de la teoría de conjuntos 159

fin ici6n 3.7 Para un conjunto A ~ <i\l, el complt~nto de A, qu.t se denota con <í\.l. - A o A, está dado
por{xlxE'U /\xl=A).

Para los conjuntos del ejemplo 3.14,A = {6, 7, 8, 9, 10), .8 = { 1, 2, 8, 9, 10) y C = {l. 2,
3, 4, s. 6, 10).

Para cualquier universol\l y cualquier conjunto A ~ <i\.l., tenemos que A~ , 1. Por lo


tanto, W>(Gll) es cerrado en la operación unaria definida por el complemento.
El siguiente concepto se relaciona con el de complemento.

!finici6n 3.8 ParaA, B C<í\.l., elcompltmtnto(nlativo)deA eoB, que sedenotaconB - A está dado por
{xlxEB/\x~A).

M íe'•-... ~
jllw~lJ
.-e;¡ Para los conjuntos del ejemp1o 3.14 tenemos:

=
a) 8 - A {6, 7) b) A-B ={l,2} e) A - C= A
d) C-A =C e) A -A = @ f ) '\i-A=A

Nuestro siguiente resultado, que hará uso nuevamente de la definición 3.2. proporciona
un enlace entre las nociones de subconjunto, unión, intersección y complemento.

EOREMA3.4 Para cualquier universo Gll y cualesquiera conjuntos A, B !:; G\t, las siguientes proposicio--
nes son equivalentes:

a) A\; B b) AUB = B
e) AnB-A ti) 11-:X

Demostraci6n: Demostraremos que (a) => (b), (b) ,.. (c), (c) => (d) y (d) => (a). [La
razón por la que esto basta para dcmosttar este teorema se basa en la idea presentada en el
ejercicio 15 que apa,ccc al final de la sección 2.2.]

1) (a) => (b) Si A, B soo conjuntos cualesquiera, entooces 8 S: A U B. Para la inclu-


sión opuesta. si x E A U B, entonces x E A o x E B, pero como A ~ B, en ambos
casos tenemos que x E B. Así, A U B ~ B y. como tenemos ambas inclusiones. se
sigue (de nuevo de la definición 3.2) que A U 8 = B.
=
ü) (b) (c) Dados los conjuntos A, B. siempre tenemos A ¿ A n B (como se mencio-
nó despukdclejcmplo 3.14). Para la inclusión opuesta, sea y E A. Con A U B= 8,
y E A,_. y EA U 8=> y E B (como A U B=B)=>y EA U 8, por lo que AS: A n
B 'y concluimos que A = A n B.
n n
iil) (c) =0 (d) Aqul tenemos que z E .8 => z 1= 8 ,_. z 1= A 8, ya que A B !; B. De
A n 8 =A se sigue que z \= A n 8,.. z I= A,.. z E A, por lo que ii \; A.
160 Capítulo 3 Teoría de conj untos

iv) (d) ⇒ (a) Por último, w E A ⇒ w ,t Ay como B ~ A, w <t A=> w <t B. Entonces
w ,t B ⇒ w E B,porloque A 1: B.

Com:x::iendo ya algo dedemosuaci6n de reoremas, presentaremos ahora algunas de las prin-


cipales propiedades que rigen la teoría de conjuntos, las cuales tienen una marcada semejanza
con las leyes de la lógica dadas en la sección 2.2. En muchos casas, estas propiedades de la
teoría de conjuntos son similares a las propiedades aritméticas de los números reales, donde
"U.. desempeña el papel de ..+", e ••n·•el de ''x'. Sin embargo, existen algunas diferencias.

Propiedad es de la teoría d e conjuntos


Para cualesquiera conj untos A, By C tomados de un universo i'iij, :

1) A =A Ley del doble complemento


2) AUB=AnB Leyes de De Mol1Jan
A n B=A U B
3) AUB=BUA Propiedades conmutativas
AnB=B n A
4) A U(BUC)= (A U B) UC Propiedades asociativas
A n (BnC) = (A nB)nC
5) A U (B n C) = (A UB)n(A UC) Propiedades distributivas
A n (B UC) = (A nB)U(A ne)
6) A U A=A Propiedades idempotentes
A n A=A
7) A U 0=A Propiedades del neutro
A n 'U=A
8) AUA='U Propiedades del inverso
AnA=0
9) A U 'U='U Propiedades de dominación
A n0=0
10) A U (A n B)=A Propiedades de absorción
A n(A UB) = A

Tcxlas estas propiedades pueden establecerse mediante argumentos de penenencia, como


en la primera parte de la demosuación del teorema 3.3. Demostramos esto estableciendo
la primera de las leyes de De Morgan y la segunda propiedad distributiva de la intersec-
ción sobre la uníón.
Demostración: Sea x E GtJ. . Entonces

xEAUB,}x~AUB
⇒ x~Ayx~B
⇒ xEAyxEB
⇒ xeAnB,

por lo que A U B ~ A n B. Para establecer )a inclusión opuesta, podemos verificar que la


recíproca de cada implicación lógíca es también una implicación lógica (es decir. que cada
implicación lógica es de hecho una eq_uivalencia lógica). Como resultado tenemos que
3.2 Operaciones de conjuntos y las leyes de la teoría de conjuntos 161

xEAn B ~xEAy xEB


~x$A y x$B
~ x$AUB
~ xEAU B.

Por lo 1an10,A n ii !;; A U B . En consecuencia, como A U B s;; ii n ii y ii n ii !;; A U B,


se sigue de la definición 3.2 que A U B = A n ii.
En nuestra segunda demostración estableceremos ambas relaciones de contenido en
forma simultánea, usando la equivalencia lógjca ((::)) en Jugar de las implicaciones lógicas
(=> y<=).
Demostración: Para cada .x. E ~.

xEA í1(8UC)<:c>(xEA) y(xE B UC}


<:c>(xEA) y (xE Box E C)
<:c>(xEAyxEB)o(xEAyxEC)
<:c>(xEA n B)o(xEA ne¡
<:c>xE(A nB)U(A ne).

Como todas las proposiciones son equivalentes. hemos establecido ambas relacíones de
con1enido en forma simullánea. por lo que A n (B U C) = (A n B) U (A n C). (La
equivalencia de la tercera y cuana proposiciones se sigue del principio comparable en las
leyes de la lógica dado por la propiedad distributiva de la conjunción sobre la dis)'unción.)

Sin duda. el lector espera que el emparejamiento de las propiedades en los puntos 2 a 1O
tenga alguna importancia Como con las leyes de la lógica. estas parejas de proposiciones
se llaman duo.les. Una proposición puede obtenerse de la otra al reemplazar todas las
ocurrencias de U por n y viceversa, y todas las ocurrencias de Gll por 0 yviceversa.
Esto nos lleva a la siguiente idea formal.

Definición 3 .9 Seas una proposición (general) que trata de la igualdad de dos expresiones con conjuntos.
Cada una de estas expresiones puede contener una o más ocurrencias de conjuntos (como
A, A, B. lí. eu:étera). una o más ocurrencias de e y '11 y solamente los símbolos de las
operaciones con conjuntos n y U. El dual de s, que se denota con ~. se obtiene de s al
reemplazar ( l ) cada ocurrencia de 9 y oU. (en s) por 'íll y 0, respectivamente; y (2) cada
ocurrencia de n y U (en s) por U e n, respectivamente.

Como en la sección 2.2. estableceremos y usaremos el siguiente teorema. Demostraremos


un resultado más general en el capítulo 15.

TEOREMA 3 .5 El principio de dualidad. Sea s un teorema relativo a la igualdad de dos expresiones con
conjuntos (como en la definición 3.9). Entonces~. el dual des, es tambi~n un teorema.

El uso de este principio reduce nuestro trabajo en forma considerable. Para cada pareja
de proposiciones en los puntos 2 al 1O, sólo hay que demos1rar una de las proposiciones y
util.izar entonces el principio para obtener la otra proposición del par.
Capítulo 3 Teoría de conjuntos

Debemos tener cuidado al aplicar el teorema 3.5. Este resultado no se puede aplicar a
situaciones particulares, sino a resultados (teoremas) relativos a conjuntos en lo general.
Por ejcmplo, consideremos lasituaciónpanicu/ardondeG\I ; { 1, 2, 3, 4, 5) yA; {l, 2, 3, 4),
8; {l. 2, 3, 5), C ; {! , 2) y D ; {l. 3). En este caso,
A í1 B- {1,2, 3} -C UD.

Sin embargo, nopodemosconcluirque.s:A n B=CU D ⇒ ~:A U B-cn D, ya que en este


caso.A u 8; {!, 2, 3,4,5), mieouasqueCn D; { I}.Larazónporlaqueel teo= 35 noes
aplicable es este ca.so es que. aunque A n B = CU Den este ejemplo particular, no es verda-
dera en general (es decir, para cualesquiera conjuntos A, B. C. D tomados de un universo :,1 ).

Dado que en la definjción 3.9 y el teorema 3.5 no mencionarnos nada acerca de los
subconjuntos, ¿podemos encontrar un dual de la proposición A ~ B (donde A, B ~ 1-l )?
Aquí tene mos la oportunidad de usar alguno de los resultados del teorema 3.4. En
panicular, podemos trabajar con la proposición A s;;: B mediante la proposición equivalen-
te A U 8;8.
El dual deA U 8 ;8 produce A() 8 ;8. PeroAn 8 ;8 <=>8 !:; A. En consecuencia, el
dual de la proposición As;;: Bes la proposición B ~ A. (Tambi~n podríamos haber obtenido
este resultado usando A~ B ~ An B - A. En los ejercicios de esta sección pediremos al
le.ctor que verifique este caso.)

Cuando analizamos las relaciones que pueden existir entre los conjuntos implicados en
una proposición de igualdad o contenido entre conjuntos, podemos estudiar la situación de
manera gráfica.
Un diagrama ,u Venn (llamado así en honor del lógico inglés John Venn, 1834-1923)
se construye de la manera siguiente : till aparece como el interior de un rectángulo, mien-
tras que los subconjuntos de Gtl se representan mediante los interiores de círculos y otras
curvas cerradas. La figura 3.5 muestra dos diagramas de Venn. La región sombreada de la
figura 3.5(a) representa el conjunto A, mientras que A queda representado por el área no
sombreada. La región sombreada de la figura 3.5(b) representa A U 8. El conjunto A (18
es el área cuadriculada de la figura.

(a) (b)

Figura 3.5

En la figura 3.6, usamos los diagramas de Venn para establecer la segunda ley de De Morgan.
La figura 3.6(a) tiene sombreado todo excepto el áreaA () B. de modo que la panesombreada
representa A n 8. Ahora utilizaremos undiagramadeVennparamostrarA U B. En la figura
3 .2 Operaciones d e conjuntos y las leyes de la teoría de conjuntos 163

3.6(b). A es la región representada por las líneas que van desde la pane inferior izquierda
a la superior derecha; las Jíneas que van de la parte superior izquierda a la inferior derecha
sombrean la región que representa B. Por lo tanto,A U 8 está dada por la región sombreada
de la figura 3.6(b). Puesto que el área sombreada de la parte (b) es la misma que la de la
parte (a), se sigue que A n B = A U 8.

(]J (]J
<•> (b)

Figura 3.6

Podemos seguir ilustrando el uso de estos diagramas para mostrar que si A. B. C ~ 111 .

(A u s) n e= (A n B) u c. ·
En vez de las regiones sombreadas, otro enfoque que también utiliza los d iagramas de
Venn numera las regiones, como se muestra en la figura 3.7, donde, por ejemplo. la región 3
es A n B n Cy la región 7 es A n B n C. Cada región es un conjunto de la forma S 1 ns~
n S3,dondeS1 se reemplaza por A oA.S2 por B oB y S3 por C o C. Por lo tanto, por la regla
del producto, existen ocho regiones posibles.
Al consultar la figura 3.7, vemos que A U B abarca las regiones 2, 3, 5, 6, 7, 8 y que las
regiones 4, 6, 7, 8 conforman el conjunto C. Por lo tanto, (A U B) () C comprende las regiones
comunes a A U By a C; es decir, las regiones 6, 7. 8. En consccueñcia, (A U 8 ) n C está
formado por las regiones 1, 2, 3, 4, 5. El conjunto A consta de las regiones 1, 3, 4, 6.
mientras que las regiones 1, 2, 4, 7 conforman ii. En consecuencia, A n 8 comprende ]as
regiones 1 y 4. Como las regiones 4, 6, 7, 8 comprenden C, el conjunto Eestá formado por
las regiones 1, 2, 3, 5. Al tomarla unión de.A n ii con C. obtenemos las regiones l. 2, 3, 4. 5,
como en el caso de (A U 8 ) n C.

1 A 8

@ 4
6

e
Figura 3.7

Otra técnica para establecer las igualdades entre conjuntos es la tabla de pertenencia.
(Este método es similar al uso de la tabla de verdad presentada en la sección 2. 1.)
54 Capítulo 3 Teoría de conjuntos

Observamos que para los conjuntos A, B ~ Gtl.. un elemento x E G\l satisface exacta-
mente una de las siguientes cuatro situaciones:

a) x1=A,x1=B b) x1=A , xEB


e) xEA,x1=B d) xEA,xEB.

Cuando x es un elemento de un conjunto dado, escribimos un 1 en la columna que repre-


senta ese conjunto en la tabla de penenencia; cuandox no está en el conjunto, escribimos un
O. La tabla 3.2 proporciona las tablas de penenencia para A n B, A U B, Acon esta notación.
Por ejemplo, en este caso, la te rcera fila de la pane (a ) indica que cuando un elemento x E
Gtl está en el conjunto A pero no está en 8, e ntonces no está en A n B pero sí en A U B.
Estas operaciones con ceros y unos son iguales a las de la aritmética ordinaria, excepto
que!Ul;J.

Tabla 3.2

A B AnB A UB
o o o o
o 1 o 1
1 o o 1
1 1 1 1
(a) (b)

Por medio de las tablas de pertenencia podemos establecer la igualdad de dos conjuntos
si comparamos sus columnas respectivas en la tabla. La tabla 3.3 demuestra esto para la
propiedad distributiva de la unión sobre la intersección. Vemos ahora cómo cada una de
las ocho filas corresponde exactamente a una de ]as ocho regiones del diagrama de Vcnn
de 1a figura 3.7. Por ejemplo, la fila 1 corresponde a la región l: Á n jj n C y la fila 6
corresponde a la región 7: A n B n C.

Tabl• 3.3
A B e anc A U (B n C) A UB AUC (A U B )n (A UC)

o o o o o o o o
o o 1 o o o l o
o 1 o o o 1 o o
o 1 1 1 1 1 1 1
1 o o o 1 1 1 1
1 o 1 o 1 1 1 1
1 1 o o 1 1 1 1
1 1 1 1 1 1 1 1
t t
PIJe$1:0 que est.15 columnas son tdéntic.as. con-
duimos qu~A u (8 n Cl .. (A u B) n (A u Cl

Antes de continuar debemos señalar dos cosas: ( 1) un diagrama de Venn es simplemen•


te una representación gráfica de una tabla de pertenencia; (2) el uso de los diagramas de
Venn o las tablas de pertenencia podría ser atractivo, en panicular, para el lector que por el
momento no se interesa por la escritura de demostraciones. Sin embargo, ninguna de estas
3.2 Operaciones de conjuntos y las leyes de la teoría de conjuntos 165

técnicas especifica la lógica y el razonamiento desplegados en los argumentos de perte-


nencia ya presentados, por ejemplo, ?J. demostrar que para cada A, B. C ~ 6ll ,

AUB =An li, y


A n(BUC) =(A nB)U(A nC).

Los diagramas de Venn pueden ayudamos a comprender ciertas situaciones matemáticas,


pero cuando el número de conjuntos implicados es mayor de tres, el diagrama podría ser
difícil de dibujar.
En resumen, el argumento de penenencia (particularmente con sus explicaciones deta•
lladas) es más riguroso que estas otras dos técnicas y es el método preferido para de mos-
trar resultados en la teoría de conjuntos.
Ahora que disponemos de las leyes de la teoría de conjuntos, ¿qué podemos hacer con
ellas? Los siguientes ejemplos demostrarán la forma de utilizar las leyes para simplificar
una cómplicada expresión con conjuntos o para obtener nuevas igualdades entre conjun-
tos. (Cuando se use más de u.na ley en un paso dado, mencionaremos la ley principal como
la razón.)

Simplifique la expresión (A U B) n Cu B.

(A U B) nCUB Razones
=((A UB)nC)nB Ley de De Morgan
=((A UB)n C)nB Ley del doble complemento
=(AUB)n(C n B) Propiedad asociativa de la intersección
= (A UB)n(B nC) Propiedad conmutativa de la intersección
= [(A u B) n B] ne Propiedad asociativa de la intersección
= B nC Ley de absorción
El lector debe notar la analogía entre los pasos y las razones de este ejemplo y los pasos
y razones utilizados para simplificar la proposición

, [7[(p V q)/\r]v ,q]


hasta obtener la proposición

q l\ r

en el ejemplo 2.18.

Exprese A - B en términos de U y - .
De la definición de complementorelativo,A - B= {xlxEA A x(J. B} =A n B. Por lo
tanto,
A-B =AnB Razones
=AUB Ley de De Morgan
=AUB Ley del doble complemento
166 Capítulo 3 Teoría de conjuntos

De la observación hecha en el ejemplo 3.19, tenemos que A6 B = /x lx E A U B Ax (f.A


n B) = (A U 8 ) - (A nB)=(A U B) n (An B), porlogue

A t,. 8 = (A U B) n(A n B) Razones


= (A U B)U{A n B) Ley de De Morgan
= {A U B)U(A n B) Ley del doble complemento
={A n B)U(A U B) Propiedad conmutativa de U
= (A n B)U(.An.li) Ley de De Morgan
=[(A n B) u A] n[(A nB)U B] Propiedad disttibutiva de U sobre n
=[(A U A)n(BUA)]n Propiedad distributiva de U sobre n
[(A u .li) n (Bu.li)]
= [~n (BU.A)] n[(A u.li)n'IL] Propiedad del inverso
=(B U.A) n(A u .li) Propiedad del neutro
=(.AUB)n(A u .ll) Propiedad conmutativa de U
= (.AUB)n(AnB) Ley de De Morgan
=A t. B
= (AU.li)n(A UB) Propiedad conmutativa de n
= (A u .li) n(A n B) Ley de De Morgan·
= A t,. "jj

Para terminar esta sección, extenderemos las operaciones de conjuntos U e n más allá
de tres conjuntos.

lefinidón 3 .10 Sea/un conjunto no vacíoyó\l. un universo. Para cadai E/, seaA.~lltl Entonces/ es un
conjunto índice (o conjunw de índices) y cada i E/ es un índice.'

~A,={x lxEA, paraalmenos uníE/}, y


!:)A, = {x lxEA1 paratodoíE/}.

Podemos reformular la definición 3.10 usando cuantificadores:


xE ~A;~ 3iEI (xEA1)
xE Q,A,~ViEI (xEA1 )

Entonces, x (f. U ,.,A, <a> -.[3í E l (x E A,)J <a> Vi E / (x (f. A,); es decir, x (f. U"' A, si
y s6losi x,t,\1 pararodoín<ticeíE/. En forma análoga, x (f. íl..,A, <a>~[Vi E /(xE A,)] <">
3í E /(x (f. A,); es decir, x (f. U,., A, si y sólo si x ,t A; para al =nos un índice i E /.
Si el conjunto de índices 1 es el conjunto z• . podemos escribir

UA, =A,UA,U ... = ÜA,, nA,= A,nA, n , .. = ñ A,.


rez• f•l ;ez• í •1
3.2 Operaciones de conjuntos y las leyes de la teoría de conjuntos 167

Sea/= {3, 4, 5, 6, 7) y parai E /sea A;= {l. 2, 3, ... , i} s"-l = z·. Entonces U~,A.
= LJ;_,A; = {!, 2, 3, ... , 7) =A,, mientras que íl,e,A. = { ! , 2, 3) =A,.

0
Sean'U =Re/ = R•. Si paracadar E R._A,= [- r, r], entonces U ..,A,= R yn ,.,A,= {O}.

Por desgracia. al trabajar con uniones e intersecciones generalizadas. las tablas de per-
tenencia y los diagramas de Venn son casi inútiles; sin embargo, podemos seguir utilizan-
do la definición rigurosa de pertenencia. como aparece en la primera parte de la demostra-
ción del teorema 3.3.

Leyes de De Morgan generaliuu:/as. Sea/ un conjunto de índices, donde para cada iE/, A;
!::: 'U. Entonces
a) UA,=
ifil
n A,
iE /
b) r"fA;= U A,
iEI. i el

Demostración: Demostraremos la parte (a) y dejaremos la demostración de la parte (b) al


lector. Paracadax E 'U,x EU,.,A; <a>x f U ,.,A; <a>x f A. paracadai E/ = x EA,.
paratodoiE/<a>xE n..,,1;.

EIERClCIOS 3.2 1. Para'\!= (l. 2, 3, . . . , 9, 10) sean A= {l, 2, 3. 4. 5),B= {l, 2,4. 8). C= {), 2, 3. 5. 7) y D =
{2, 4, 6, 8}. Determine lo siguiente:
a) (A U B)nC b) A U (B nC) e) c ul5
d) CnD e) (AUB)-C f) A U(B-C)
e) (B-C)- D b) B - (C-D) i) (A U B) -(Cn D)
2. s; A= {O, 3), B =(2, 7) y '11 =R, dctemúne lo siguiente:
a)AnB b) A UB c)A
d) AL>B e) A-B f) B-A
3. Sean'\!= {a. b, c.... ,.<~ z).A= {a. b.c) y C= (a. b. d.,¡. s;¡A n BJ= 2 y (A n 8 ) e Be
C. detennine B.
4. a) Determine losconjuntosA.BcuandoA-8 = {l. 3, 7, 11 ), B - A = {2, 6. 8) yA n B = {4. 9J
b) Detemúne los conjuntos C.D. donde C-D = (I, 2, 4}, D-C= {7, 81 y CU D = { l. 2. 4.
5, 7, 8, 9).
5. Para'\l = {l. 2. 3..... 29.30}, sean B. e !; '\l con B = (l, 2. 3. 4. 6. 9. 15) y C= {2. 3. 6.
15, 22. 29}. ¿Cuánto vale IB U cr.
6. Sean A, B, C, D, E~ Z definidos como sigue:
A= {2nln E Z}-es decir.A es el conjunto de todos los mllltiplos (enteros) de 2;
B-{3nlnEZ); C={4nlnEZ};
D- {6n lnEZ}; y E -{SnlnE Z}.
a) ¿Cuáles de las siguientes proposiciones son verdaderas y cuáles falsas?
1) E!;C!;A li) A!;C!;E iü) B !; D
iv) D i;, B v) Di;.A "' D i;. A
168 capítulo 3 Teoña de conjuntos

b) Determine cada uno de los siguientes conjuntos.


i) c n E •1 B U D W) AnB
iv) B n D v) X vi) A n E
7 . Sea Cill un Uni\'CfSO finiro con A, 8 C '-t. Ordene la siguiente lista. en orden creciente de
acuerdo con el tamaño.
•> IA u B I , I B I , 10 1, IA n s ¡, i"lll
b) IA-BI , IA I +IB I , 191, I A LI B I, IA UBI , l"lll
e) IA-B1, 191,I A l , IA l + I B I, IAUBl , l"lll
8 . Dado un universo<'.il.l y los conjuntos A, Be; Gll . ¿a qué es igual (A-B) n (B-A)? Demuestre
su afirmación.
9. Determine cuáles de las siguientes proposiciones son verdaderas y cuáles son falsas.
a) z · (;Q. b) z · (;Q e) Q·~R
d) R·~ Q •) Q•nR• =Q· f) Z- u R·= R·
g) R ·n c = R· b) C U R=R 1) Q· nz =Z
j) Z U Q • Z
1 O. Demuestre cada uno de los siguientes resultados sin usar los cfiagrarnas de Venn o las tabla.$ de
pertenencia. (Suponga la existencia de un universo ~l.)
a) Si A C. By CCD,cntonccsA n CCBn DyA U C!: B UD.
b) Si A C C y BCC, cntoncesA n s C CyA U BC C.
e) A !:;;Bsi y sólo si A nll= 0.
d) A C B si y sólo si A U B=< it.l.
11. Demuestre o refute lo siguiente:
a) Para conjuntos A , B, Cc;CU, A n C=BnC=}A =B.
b) Para conjuntos A ,B,C~CU, A UC =B U C=}A =B.
e) Para conjuntos A ,B,C¡;;"ll, [(A n C= B nC)l\(A U C =B UC)] ⇒ A = B.
d) Para conjuntos A , B,C(;'IL, A LIC • B LIC ⇒A = B.
12. Usando tos diagramas de Venn, analice la verdad o falsedad de lo siguiente.. para los conjuntos
A, 8,C !:;;>\l.
a) A Ll(B n C) = (A LIB)n(A LIC)
b) A n(B LIC)=(A n B)Ll(A ne¡
e) A Ll(B U C) = (A LIB)U(A LIC)
d) A- (B UC) =(A-B)n (A-C)
o) A Ll(B LIC) = (A LI B)LIC
13. Si A = fa, b, d}, B = {d, x. y } y C = {x, z} ¿cuántos subconjuntos propios tiene e l conjunto
(A n B) u C? ¿cuántos tiene el conjunto A n (B U C)?
14. Para un universo dado '1.l . cada subconjunto A de l'.il( satisface las leyes de idemporencia para
la unión y la intersección. (a) ¿Existe algón mlmero real que satisfaga la propiedad de
idempotencia para la suma? (Es decir. ¿podemos encontrar alglln nl1mero o mlmeros reales
taJes que .r + x = x1) (b) Responda la parte (a) reemplazando la suma por la multiplicación.
15. Escriba ta proposición dual para cada uno de los siguientes resultados de la teoría de conjuntos.
•>"ll=(AnB)U(An li)U(A n B)U(A n li) b) A = A n(A U B)
tjA un-0n~u0n ~ u ~ n ~ GA= 0u~n0 u ~
16. Use la equivaJenciaA C B
la pi-oposición B C A.
= Á n B =A para de mostrar que la proposición dual de A~ Bes
17. Demuestre o refute lo siguiente, para los conjuntos A, 8 ~ 'i\t.
a) 9'(A U B) -9'(A) U 9'(8) b) 9'(A n B) - 9'(A) n9'(8)
18. Utilice las tablas de pertenencia para establecer lo siguiente:
a) AnB=AUB b) A U A=A e) A U(A n B)=A
d) (An B)U(AnC) = (An li)U(AnC)
3.3 Técnicas de conteo y diagramas de Venn 169

19. a ) ¿Cuántas filas se necesitan para construir la tabla de penenencia para A n (B U C) n


(DU E U F)?
b) ¿Cuántas filas se necc5itan para construir la tabla de pertenencia para un conjunto formado
por los conjuntosA1, A1 , . . ,A., usando n. U y-<'f
e) Dadas las tablas de pertenencia para dos conjuntos A, B. ¿cómo puede reconocerse la rela-
ci6n A~ B?
d) Utilice la tabla de pertenencia para determinar si (A n B} U (BnC) ~ A U B.
20. Proporcione la justificación (a partir de las propiedades de la teoría de conjuntos) de los
pasos necesarios para simplificar el conjun10 (A n B) U [B n((Cn D} U (C n D))]. donde A.
•coc•. ·
Pasos
(A n B) U[B n((C n D) U(C n D))l
•(A nB)U[Bn(Cn(DUD))]
~ (AnB)U[Bn(Cn<IL)]
~ (A n B)U(B nC)
~(B n A)U(BnC)
~sn(A uC)

21. Mediante las leyes de la teoría de conjWllOS. simplifique lo siguiente:


a) An(B-A) b) 1:4n_!l)U(A nB~CnD)U(A nB)
e) (A- B)U(An B) d) AU B U(A n B nC)
e) A U(A n li) u(AnBnC)U(A nBnCnD) U .. .
22. ParacadanEZ · , sea A..= {l. 2, 3, .. , n-1.n}.(Aquíú\.l =Z· ye! conjunto índice es! =Z".)
7 11 ,. •
Determine UA.., ílA., LJA. y ílA•. donde mes un ente ro positivo fij-0.
.., ...i ...i ...
23. Paracadan E Z • sea B. = {n + l.n + 2.n+ 3, ..}. (Enestecaso.~l = z· yel conjumo índice
• ll •
es/ = Z ' .} Detennine ~ B., QB•. ~ B. y Qº••donde m es un entero positivo fijo.
24. Sea ~l = R y sea/= z·. Para cadan E z·, sea A. = [- 2.n, 3n). Oelermine lo siguiente:
a) A 1 b} A, e) A >- A.,. d) A,.ó.A.,.

e) !d A. r) ó A. g) ~ . A. h) ñ A.
25. Dado un universo{!\( y un conjWltO índice/, para cadai E f. seaB, ~Gtl. Demuestteque para
A C <l\l , A n d.J~,B,l = LJ;.,(AnB,) y A U (J,,_,B,) = íl;.,(AUB,J. ( Propiedades
distributivas generalizadas)
26. Proporcione los detalles para la demostración del teorema 3.6(b).

3.3
Técnicas de conteo y diagramas
deVenn

Con todo el trabajo teórico y de demostración de teoremas que realizarnos en la sección


anterior, ahora es un buen mome nto para examinar algunos otros problemas de conteo.
Para losconjuntosA, B de un universo finitoG't.l, los siguientes diagramas de Venn nos ayuda•
rán a obtener fórmulas de conteo para JA J y JA U B I en términos de JA J. JB I y JA n B 1.
Capítulo 3 Teoría de conjuntos

Figura 3.8

Como lo demuestra la figura 3.8, A u A= 'U y A n A= 0. así que, por la regla de la


suma, IA 1+ IAl= l-0\ll o IA I = 1'111- IA 1- LosconjuntosA, Bde la figura3.9 tienen inte=•
ción vacía, por Ioque a panirde la regla de la suma obtenemos que IA U Bl=IA 1 + IB l;es
necesario que A, B sean finitos, pero no se impone una condición en el cardinal de 1lt .

00 Figura 3.9

Regresaremos al caso en que A, B no son disjuntos y derivaremos la fórmula para I A U B 1


con el siguiente ejemplo.

En una clase de 50 aJumnos de primer ingreso. 30 estudian BASIC. 25 Pascal y IOestán


estudiando ambos lenguajes. ¿Cuántos alumnos de primer ingreso estudian algún lenguaje
de computación?
Sea <.\t la clase de: 50 alumnos de primer ingreso, A el subconjunto de estudiantes que
estudian BASJC y B el subconjunto de alumnos que estudian Pascal. Para resp0nder la
pregunta. nece sitamos calcular IA U B 1- En la figura 3.10, los números que aparecen en
las regiones se obtienen de la información dada: IAI = 30, IBI = 25, IA n BI = 10. En
consecuencia, IA U BI = 45;, IAI+ IBI, ya que IAI + IBI cuenta dos veces a los estu-
diantesqueeslánenA n B.Paraevitaresterecuentoexcesivo, restamos IA n BI de IA I + IBI
para obtener la fórmula correcta: IA U B 1 = IA 1 + 1B 1 - IA n B 1-

m Figura 3.10
3.3 Técnicas de conteo y diagramas de Venn 171

. -\. :· :''--~':c-:,:.-'>C-- -

Esta situación se extiende al caso de ttcs conjuntos, como lo ilustra el siguiente ejemplo.

Un chip lógico de 14 pins tiene cuatro puenas AND, cada una con dos entradas y una
salida. (Véase la Fig. 3.11) La primera puena AND (la pucna de los pins l, 2 , 3) puede
tener cualquiera o todos los defectos siguientes:

D1: la primera entrada (pin 1) está fija en O.


D,: la segunda entrada (pin 2) es1á fija en O.
D,: la salida (pin 3) es1á fija en l.

14
13,
12

11
10
9

Figura 3. 11

Para una muestra de 100 de estos chips lógicos, sean A, By Clos subconjuntos que tienen
los defectos D,, D, y D3, respectivamente. Si IA 1=23, IBI = 26, 1CI = 30, IA n BI = 7,
IA n CI =8, IBn CI = !Oy IA n Bn CI = 3,¿cuántoschipsdelamuestratienen una
primera puerta AND con defecto?
Si trabajamos en dirección contraria, de IA n B n CI = 3 a IA 1 = 23, etiquetamos las
regiones como se muestra en la figura 3.12 y vemos que IA U BU CI = 57 = IA 1+ IBI +

A 8

Figura 3.12
Capítulo 3 ~ de conjunto-s

ICI- IA nBI - IA n CI- ¡an e¡+ IA non CI-Así,elejemplomuesua57chipsque


tienen una primera puerta AND con defecto y 100 - 57 = 43 chips en los que Ja primera
puerta AND no tiene defecto.

Cerraremos esta sección con un problema que utiliza este resultado.

Un estudiante visita un salón de juegos al salir de la escuela y juegalASer Man. Millipede


o Space Ccmquerors. ¿De cuántas formas puede jugar una de las opciones al día, de modo
que juegue al menos una vez durante una semana de clases?
Aquí hay un cierto giro. Elconjuntoú\.l consta de todos lasclisposiciones de tamaño cinco
tomadas del conjunto de tres juegos, con repeticiones. El conjuntOA representa el subconjunto
de todas las series de cinco juegos practicados durante la semana sin jugar Laser Man. En
forma anáJoga, los conjuntos B y C se definen dejando fuera Millipttde y Space Conque,ors,
respectivamente.Lasrtcnicasdeconteodelcapítulo I dancomoresulla<lo l~l l =3', IAI =

fórmula anterior. existen IA n B n e,=


IBI = ICI =2', IA na¡ = IA ne¡= ¡sn e¡= l'= !y IA nsn CI =0, así que, porla
=
35 - 3 . 21 + 3 • 1s - o 150 maneras en que el
estudiante puede seleccionar sus juegos diarios durante una semana de clases y jugar cada
opción al menos una vez.
Este ejemplo también puede expresarse en forma de distribución, ya que buscamos el
número de maneras de distribuir cinco objetos distintos ()unes. martes, ... , viernes) entre
tres recipientes distintos (los juegos de computador) sin dejar uno vacío. Diremos más
acerca de esto en el capítulo 5.

3.4
Unas palabras en cuanto a la
probabilidad

Cuando uno realiza un experimento, como tirar un solo dado o seleccionar dos estudiantes
de una clase de 20 para trabajar en un proyecto, la lista de todos los posibles resultados
para esta situación se llama espacio muestra!. En consecuencia, { 1, 2, 3, 4, 5, 6} sirve
como un espacio muestra! para el primer experimento mencionado, mientras que {{a;, a¡ }
11 S j S 20, 1 Sj S 20, i ~j} puede utilizarse para el segundo experimento, si denotamos
con a; el j-ésimo esrudiante, para cada 1 S i S 20.
Por desgrada. un experimento puede tener más de un espacio muestraJ. Si tiramos una
moneda tres veces y analiz.amos los resultados, un p0sible espacio muestra! para nuestro
experimento es {O, 1, 2, 3}, donde el númeroi, para OS iS 3, se refiere a1 m1merode caras
3.4 Unas palabras en cuanto a la probabilidad 173

que aparecen en tres tiradas. Los resultados pueden darse también mediante el espacio
muestral ;:/ = {cara. cara, cara; cruz, cara, cara; cara, cruz, cara; cara, cara, cruz; cruz,
cruz. cara; cruz, cara. cruz; cara, cruz, cruz; y cruz, cruz.. cruz}, donde intuimos que cada
uno de los ocho posibles resultados tienen Ja misma probabilidad de ocurrir. Esto no suce-
de en nuestro primer espacio muestra! {O. l , 2, 3}, donde creemos que hay más oportuni-
dad de obtener una cara en los tres tiros que de no obtenerla.
En este texto usaremos siempre un espacio muestral en e1 que cada elemento tiene la
misma probabilidad de ocurrir. Sobre esta suposición de probabilidad idéntica. usaremos
la definición de probabilidad dada por el matemático francés Pierre•Simon de Laplace
(1749-1827) en su libro 1ñéorie analitique des possibilités.

Demostraremos estas ideas en los ejemplos siguientes.

En una sola tirada de un dado, ¿cuál es la probabilidad de obtener un 5 o un 6?


Aquf;:/ = { 1, 2, 3, 4, 5, 6 } yel sucesoA que queremos considerares {5.6}. De aquí que

Si tiramos una moneda cuatro veces, ¿cuál es la probabilidad de obtener dos caras y dos
cruces?
En este caso, el espacio muestra! está formado por tc:Kias las sucesiones de la forma Xi,
x2 , x3, x,, donde cada X;, l S" i S" 4, se reemplaza por una cruz, o una cara, así que 13'! = 2'
= 16. El suceso A que nos interesa contiene todos las disposiciones de los cuatro símbolos
= = =
cara, cara, cruz, cruz, por lo que IA 1 4Y(2! 2!) 6. En consecuencia, P~A) 6116 3/8. =
Para el ejemplo 3.27, cada tirada es independienu del resultado de cualquier tirada
anterior. Este caso se conoce como prueba de Be 17Wulli. En un primer curso de probabili-
dad, estas pruebas se analizan junto con la distribución binomial, un ejemplo imponante
de distribución de probabilidad discreta Regresaremos a esto en la sección 16.4 del capí-
tulo 16, cuando estudiemos la aplicación de los grupas abelianos en teoría de códigos.
174 capítulo 3 Teoría de conjuntos

E l acrónimo WYSIWYG (siglas en inglés de "lo que ves es lo que obtienes") se usa para
describir una inte rfaz con el usuario. Esta interfaz presenta material en una VDT ( iniciales
en inglés para terminal de pantalla de vídeo) precisamente en el mismo formato en que
aparece el materia] en una copia impresa
Hay 7!/(2! 2!) = 1260 formas en que pueden ordenarse las letras del acrónimo
WYSIWYG. De estas, 120( = 5!)disposicíones tie ne n las letras W y las letras Y consecu-
tivas. Así, si ordenamos las letras de este acrónimo de una manera aleatoria, e ntonces la
probabilidad de que la disposición tenga las letras W yY consecutivas es 120/1260 0.0052. =
La probabilidad de que una disposición aleatoria de estas siete letras comience y tenni-
ne con la letra W es [(S!n!)J/1(7!/(2! 2 !))) ~ 60/1260 :0.0476.

En nuestro ejemplo final usaremos los conceptos de diagramas de Venn y de probabilidad.

En una encuesta realizada a 120 pasajeros, una línea aérc::a descubrió que a 48 les gustaba
el vino (V) con sus alime ntos, a 78 les gustaban las bebidas preparadas (P) y a 66 el té
helado (T). Además, a 36 les gustaba cualquier par de estas bebidas y a 24 pasajeros les
gustaba todo. Si se seleccionan dos pasajeros aleatoriamen,t e de la muestra e xaminada de
120, ¿cuál es la probabilidad de que

a) (suceso A) ambos desee n solame nte té helado con sus alimentos?


b) (suceso B) ambos prefieran exacramente dos de las tres bebidas que se ofrecen?

Con esta infonnación, construimos el diagrama de Ve nn que se muestra en Ja figura


3.13. EJ espacio muestra! Ef está formado por los paces de pasaje ros que podemos selec-
cionardelamuestrade 120, por lo que IEfl = ( f) =7140.EldiagramadeVennindicaque
1

hay 18 pasajeros que únicamente toman té helado, por !oque IAI ; (1) y Pr(A); 5ln380.
El lector deberá verificar que Pr(B):;:;; 3/34.

Figura 3.11

c!EROCIOS3.3 1. ¿Cuántas permutaciones de 26 letras diferentes del alfabeto contienen (a) el patró n ··our· o el
r3.4 patrón ..DIG'' ; (b) ninguno de los patrones "MAN" o '"ANT'?
2. Un nombre de variable de seis caracteres en ANSI FORTRAN empieza con una letra del alfa-
beto. Cualqu iera de los otros cinco caracteres puede ser una letra o un dígito. (Se permiten
3.4 Unas palabras en cuanto a la probabilídad 175

repeticiones.) ¿Cuántos nombres de variables de seis caracteres contienen el patrón ··FUN" o


el patrón •'TIP"?
3. ¿CUántas permutaciones de los dígitos O, l, 2, ... , 9 empiezan con un 3 o terminan con un 7.
o cumplen ambas concticiones?
4. Un profesor tiene dos docenas de libros de introducción a las ciencias de la computación y es1á
interesado en la forma en que tratan los temas (A) compiladores, (B) estructuras de dalos y (C)
int&prctes. Los siguientes datos representan la cantidad de libros que contienen ma1erial rela-
tivo a estos temas:

IAl=8 1B1=13 lct= 13


IA n Bl•S 1Anct=3 1snct•6
IA n BnCl=2

(a) ¿Cuántos libros incluyen el material de exactamente uno de estos temas? (b)¿Cuántos no
tratan ninguno de estos temas? (c)¿Cuántos no tienen material sobre compiladores?
S. Daño tira un dado eres veces. ¿Ciál es la probabilidad de que
a) su segundo y tercer tiros sean ambos mayores que su primer tiro?
b) el resultado de su segundo tiro sea mayor que el de su primer tiro, y el resultado de su tercer
tiro sea mayor que el del segundo? ·
6 . AJ seleccionar un computador nuevo para su centro de cálculo. el responsable del mismo exa-
mina 15 modelos diferentes. coosidcrando: (.4.) el dispositivo para cinta magnética, (B) la terminal
para mostrar graficas. (C) la memoria semiconductora (además de la memoria principal). El
nwnero de computadores oon walqwera o tDdas estas caract<rlsticas es el sigwente: IA 1= 1B 1=
ICI =6, IAn BI = 1Bn CI = 1, IAn CI = 2, IAn B n CI =O.(a)¿Cuántosmodelos ,;enen
exactamente una de estas caracteñsticas? (b)¿ Cuántos no tienen ninguna de estas característi-
cas? (e) Si se selecciona un modelo al azar. ¿cuál es la probabilidad de que tenga e xactamente
dos de C$tas características?
7, En la comunidad fe menina Gamma Kappa Phi, las 15 estudiantes de último año se fonnan de
una manera aleatoria para una fotografía de fin de curso. Dos de ellas son Columba y Paty.
¿Cuál es la probabilidad de que en la fotografía
a) Pacy quede en la posición central de la fila?
b) Pary y Columba queden juntas?
e) queden cinco estudiantes entre Paty y Columba?
d) Columba quede en a1gún lugar a la izquierda de Paty?
8 . El grupo de primer ingreso en una escuela de ingeniería tiene 300 esrudiantes. Se sabe que 180
pueden programar en Pas,cal, 120 en FORTRAN, 30 en APL, 12 en Pascal y APL 18 en
FORJRAN y APL. 12 en Pascal y FOR'IllAN y 6 en los tres lenguajes.
a) Se elige un estudiante al azar. ¿Cuál es la probabilidad de que pueda programar en exacta•
mente dos lenguajes?
b) Seleccionamos dos estudiantes en forma alea1oria. ¿Cuál es la probabilidad de que
i) ambos puedan programar e n Pascal?
ii) ambos puedan programar sólo en Pascal?
9. F.n una estantería hay ocho libros diferentes, tres de física y cinco de ingeniería elé:ctrica.
colocados aleatoriamente. F.ncueotre la probabilidad de que queden juntos los u-es libros de
ffsica.
1 O. Un cargamento de 24 autos nuevos contiene 15 en excelentes condiciones. seis con defectos
mínimos y tres con defectos mayores. Si se seleccionan dos automóviles del cargamento,¿cuál
es la probabilidad de que (a) ambos se encuentren en e xcelentes condiciones? (b) ambos ten-
76 Capítulo 3 leoría de conjuntos

gan defectos menores? (e) a lo más uno tenga un defecto mínimo? (d) al menos uno tenga UD
defecto mínimo? {e) exactamente uno tenga un defecto mínimo? ( f) ninguno tenga un defecto
mínimo?
¿Qué relación tienen los resultados de los apartados (b). (e) y (f)'!
11. De manera aleatoria se selce<:iona un entero cnu-e 3 y 7 inclusive. Si A es el suceso de que sea
elegido un número divisible entre 3 y B es el suceso de que el mlmero sea mayor que 10.
determine Pr(A). Pr(B). Pr(A n 8) yPr(A U B). ¿Cómo se relaciona Pr(A U B) con Pr(A).
Pr(B) y Pr(A n B)?
12. a) Si las letras del acrónimo WYSfWYG se ordenan de manera aleatoria, ¿cuál es la probabi•
lidad de que la disposición comience y termine con la misma letra?
b} ¿Cuál es la probabilidad de que una disposición aleatoria de las letras de WYSIWYG no
tenga un par de letras idénticas consecutivas?
13. a) ¿Cuantas disposiciones de las letras de MISCELLANEOUS no tienen una pareja de letras
idénticas consecutivas?
b) Si se genera una disposición de estas letras en forma aleatoria. ¿cuá1 es la probabilidad de
que no haya parejas de letras idénticas consecutivas?
14. ¿Cuántas disposiciones de las letras de CHEMIST tictlt'.n la H antes de la E. la E antes de la T.
o T antes de la M? (Aquí, -antes.. significa cualquier lugar antes, y no inmediatamente antes.)
1 5. Si las letras de la palabra CORRESPONDENT se arreglan en orden aleatorio, ¿cuál es la
probabilidad de que la disposición comience y termine con la misma letra?

3.5
Resumen y repaso histórico

En este capítulo presentamos algunos de los fundamentos de la teoría de conjuntos. ade•


más de ciertas relaciones con los problemas de conteo y de probabilidad elemental.
El álgebra de la teoría de conjuntos se desarrolló durante los siglos diecinueve y veinte.
En Inglaterra, George Peacock ( 1791-1858} fue un pionero en reformas matemáticas y
uno de los primeros, con su Treatise onAlgebra, en revolucionar el concepto del álgebra y
la aritmética. Sus ideas fueron desarrolladas más tarde por Duncan Gregory ( 1813--1844),
William Rowan Hanúlton (180>-1865) y Augusrus De Morgan (1 806--1871), quien inten-
tó eliminar la ambigüedad del á1gebra elemental para ponerla en fonna de postulados
estrictos. Sin embargo, fue e n 1854, año en que Boole publicó su lnvestigarion of rhe Laws
ofThoughr, cuando se logró formalizar el áJgebra de conjuntos y la lógica y se e xtendió el
trabajo de Peacock y sus contemporáneos.
Nosotros nos hemos concentrado principalmente e n los conjuntos finitos. Sin embargo,
el estudio de los conjuntos infinitos y sus cardinales ha ocupado las mentes de muchos
matemáticos y filósofos. [Veremos más de esto en el apéndice 3. No obstante , quizá el
lector esté interesado en aprender más sobre funciones (tema que aparece en el capítulo 5)
ante s de ver el material de este apéndice.] El enfoque intuitivo de la teoría de conjuntos se
realizó en tiempos del matemático ruso Georg Cantor (1845-1918). quien definió un con-
j unto, en 1895, en una forma comparable a las ·•nociones intuitivas" que mencionamos aJ
comienz.o de la sección 3.1. Sin embargo, su definición fue uno de los obstáculos que
Cantor no fue capaz de eliminar completamente de su teoría de conjuntos.
3.5 RMumen y r~so histórico 177

Georg Canto r (1845-1918)


Reproducción por cortesía de The Gtanger Cotlection, NuP#a York

En la década de 1870, cuando Cantor estaba estudiando las series trigonom~tricas y las
series de números reales, necesitaba una forma para comparar el tamaño de los conjuntos
infinitos de números. Su estudio del infinito como una realidad. que está en el mismo nivel
que lo finito, fue realmente revolucionario. Parte de su trabajo fue rechazado ya que resulcó
ser mucho más abstracto de lo acostumbrado por muchos matemáticos de su 1iempo. Sin
embargo, su trabajo ganó la suficiente aceptación para que en 1890 la teoña de conjuntos,
tanlo finita como infinita, fuera considerada una rama de las matemáticas por derecho
propio.
Si bien, al terminar el siglo. la teoña era aceptada ampliamente, en 1901 la paradoja
ahora conocida como la paradoja de Russell (que fue analizada en el ejercicio 25 de la
sección 3.1) mos1ró que la teoría de conjuntos propuesta originalmente tenía una
inconsistencia interna. La dificultad parecía residir en la falta de restricciones al definir los
conjuntos; la idea de que un conjunto pudiera ser elemento de sf mismo fue considerada
particulannente sospechosa. En su trabajo Principia Mathematica, los matemáticos bri-
tánicos Lord Bcnrand Anhur Wílliam Russcll ( 1872-1970) y Alfred Nonh Whi1ehead
( 1861-1947) desarrollaron una jerarquía en la teoría de conjuntos conocida como Ja teo•
ria de tipos. Esta teoría axiomática de los conjuntos, entre otras formulaciones del siglo
veinte, evitaba la paradoja de Rus.scll.Adcmás de su trabajo en matemáticas. Russcll escri-
bió libros sobre filosofía, física y sobre sus opiniones políticas. Su gran talento literario
fue reconocido en 1950 cuando ganó el premio Nobel de literatura.
Et descubrimiento de la paradoja de Russcll, aun cuando se pudo remediar, tuvo un
profundo impacto en la comunidad matemática. ya que muchos comenzaron a preguntarse
si habría otras contradicciones ocultas. En 1931, el matemático (y lógico) austriaco• Kun
Gtldel (1~1978) formuJó la idea de que ..en una condición de consistencia dada. cual·
quier sistema axiomático formal suficientemente fuerte debe contener una proposición tal
que ni ista ni su negación sea demostrable y tal que cualquier demostración de consisten•
cia del sistema debe usar ideas y m6todos que están más allá de los propios del sistema en

t En realidad. Kun Gadel nació en lo que era Checoslovaquia.


78 Capítulo 3 ~ria de conjuntos

Lord Bertrand Arthur William Russell (t872-1970)

sf'. De esto aprendimos, lamentablemente, que no podemos establecer, de fonna rigurosa


desde el punto de vista matemático, que no existen contradicciones en matemáticas. Pero,
a pesar de esta ·'prueba de Godel", la investigación matemática continúa; de hecho, la
cantidad de investigación realizada de 1931 a la fecha sobrepasa la de cualquier otro pe-
riodo en la historia
El uso del símbolo de pertenencía E (una forma estilizada de la letra griega épsilon) fue
introducido en 1889 por el matemático italiano Giuseppe Peano (1858- 1932). El símbolo
"'e» abrevia la palabra griega "tOTl" que significa "está".
Los diagramas de Venn de la sección 3.2 fueron introducidos por el lógico inglés John
Venn (1834-1923) en 1881. En su líbro Symbolic Logic, Venn aclaró ideas desarrolladas
anterionnente por su compatriota George Boole (1815-1864). Además, Venn contribuyó
al desarrollo de la teoría de la probabilidad, descrita en su ampliamente conocido libro de
texto sobre esta materia.
Si quisiéramos resumir la imponancia del papel de Ja teoría de conjuntos e n el desarro-
llo de las matemáticas del siglo veinte, podríamos recurrir a la siguiente cita atribuida al
matemático alemán David Hilben (1862- 1943): "Nadie podrá expulsarnos del paraíso
que Cantor ha creado para nosotros"
En la sección 3. 1 mencionamos la disposición de números conocida como el triángulo
de Pascal. Podríamos haberla presentado en e l capítulo l , con el teorema del binomio,
pero esperamos hasta contar con algunas identidades combinatorias necesarias para veri•
ficar la forma en que se construye el triángulo. La disposición ya aparece en el trabajo del
algebristachinoChu Shi-kie (1303), aunque su primera aparición en Europa fue en el sigloxvt,
en la ponada del libro de Petrus Apianus (1495-1552). Niccolo Tanaglia (1499-1559)
utilizó este triángulo para calcular las potencias de (x + y). Debido a su obra sobre las
propiedades y aplicaciones de este triángulo, la disposición recibe el nombre del matemá-
tico francés Blaise Pascal (1623-1662).
E. cicios complementarios 179

Por último, aunque la probabilidad surgió con los j uegos de azar y los problemas de
enumeración, la mencionamos en este punto debido a que la teoría de conjuntos evolucio--
oó como el medio necesario para establecer y resolver problemas de esta importante área
contemporánea de las matemáticas aplicadas.
Gran pane de la historia y desarrollo de la teoría de conjuntos aparece en el capítulo 26
de C. Boyer [1]. El desarrollo formal de la teoría de conjuntos, incluyendo los resultados
relativos a los conjuntos infinitos. se puede encontrar en H . Endcnon [3]. P. Halmos [4],
J. Henle [5] y P. Suppes [7]. Una interesante historia de los orígenes de las ideas en proba-
bilidad y estadfstica, hasta la época de Newton, aparece en F. N. David [2]. Los capítulos
1 y 2 de P. Meyer [6] son una fuente excelente para los interesados en aprender más acerca
de la probabilidad discreta.

BIBLIOGRAFÍA

1. Boycr-, Carl B.• HUtory o/M01h.enuui<s. Nueva York, Wilcy. 1968.


2. David, Aorcncc Nightinga]e, Gamts, Gods, and Gambling, Nueva York, Hafner, 1962.
3. Enderton, Herbcrl B.• El~ments o/Stt Theory, Nueva York. Acadcmic Press, 1977.
4. Halmos, Paul R.. Naive s~,
Theory, Nueva Yort, Van Nosuand, 1960.
5. Henle, James M., M 0"1/w ofS,r Th,ory. Nueva Yod<. Sprin¡er-Verlag. 1986.
6. Meyer. Paul L, lntroductory Probability ami S1a1istical Appücations, T ed., Rcading. Mass.•
Addison-W<:$1ey, 1970.
7. Suppes, Patrick C .• Axioma.tic Set Theory, Nueva Yort:. Van NoMrand, 1960.

b) Para un entero positivo n, muestre que


EJEROOOS
COMPLEMENTARIOS
(2n)
n •
i (•)'
/e
.t• O

1. ScanA, B,Cs;;"IJ. DemueSb"eque (,1-8) s;; C si y sólo


5. a) ¿De cdntas formas puede un profesor dividir un
(A-C)s;; B.
grupC> de siete estudiantes en dos t.quipos. de modo
. A, B l:;'11. demuestre que,\ s;; Bsi y sólo si [VC s;; "ll que cada uno contcn¡a al tnenO$ un estudiante?
!:A)=oC!;;B)]. ¿al menos dos estudiantes?
3. SeanA. B,C C.~. Demuestre la verdad o falsedad (con b) Responda la parte (a) reemplazando siete por un
acoatraejemplo) de lo siguiente: entero positivo n ~ 4-.
6. Determine si cada una de las siguiente:$ proposicioOC$
1) ,1 -C =B -C⇒ A-B
b) [(,1nc-BnC)/\(A.-C • B-C)]*A. - B es verdadcnt o falsa. Para cada proposición falsa, dl un
<) [(A UC• B UC)/\(A. -C • B -C)] *A. = B contraejemplo.
4) ~(A - B) • ~(A ) -~(B) a) Si A y B son conjuntos infinitos. entonces A n B
es infinito.
.f. a) Paraloscntcrospositivosm.n.r,conr Smín{m,,n). b) Si Bes infinito y A C, B. entonces A es infinito .
muestre que e) Si A C B con B finito, entonces A es finico.
d) Si A C B con A finito. entonces Bes finito.

(m;•)-(;)(;) +(7)(,.: i) +(;)(,.:2) 7. Un conjunto tiene 128 subconjuntos de cardinal par.


(a) ¿Cuántos subconjuntos de A tienen cardinal impar?

+ ... + (7)(~)- J,(;)(,. : k).


(b) ¿Cuántovale IAI?
8, Sea A ={l,2, 3.. .. ,15 ).
!O Capín.llo 3 Teoría de conjuntos

a) ¿Oláotos subconjuntos de A contienen todos los exactamente iguales, por lo que esta tabla de pertenencia
enteros impares de A? muestra que A C B ~ A U B=B.
b) ¿Cumtos subconjuntos deA contienen exactamente Utilice las tablas de pertenencia para verificar lo siguiente:
tres enteros impares'?
e) ¿Cuintos subconjuntos de A de ocho elementos a) A¡;;B?A n B=A
contienen exactamente tres enteros impares? b) [(AnB=A)/\(BUC=C)]?
d) Escriba un programa (o desarrolle un aJgoritmo) A UBUC =C
para generar un subconjunto de ocho elementos de e) C<;;Bs;;Ac~{AnB)U(BnC)=
A e imprimir el m1mero de estos elementos que sean AnC
impares. d) AAB=C?A AC =B y BAC=A
SeanA, B. C<;;~. Demuestrcquc(A n B) U C=A n
UC)siysólosiCCA. 16. Enuncie d dual de cada teorema del ejercicio 15. (Aquí
t. Para cualesquiera A, B,C C 1.\1. demuestre que (A - B)- se usa el resultado del ejemplo 3.17 junto c:on el teorema
= (A - C)-(B-C). 3.S.)
17. a) Oeterminelacantidaddedisposicioneslinealesdc
l. Para cualesquiera A, B,C ~~. demuestre gue(a)A U
::C\lsi y sólo si AC B; (b);A n B-=·0 si y sólo si A2 B. m unos y r ceros sin que haya unos adyacentes.
(Establezca las condiciones necesarias para m, r.)
t. ScaGl.l.ununiversodadoconA. B Ct\1.. IA n BI = 3, b) Si~={l, 2.3..... n}.¿Cuántos conjuntos A C ,1
, u 81 = 8 y 1~ I =12. son tales que JA 1 = k y A no contiene enteros con-
a) ¿Cuántos subconjuntosC CGI.I. satisfaccnA n BC
secutivos? (Establezca las condiciones necesarias
C !:: A U B? ¿Cuántos de estos subconjuntos C paran. k.) ,
contienen un número par de elementos?
b) ¿Cuántos subconjuntos D C i'i\l satisfacen A u B 18. Sea A= {nin E z.., y
1 :S n :S 100}. Si B ~ A, ningún
e D e Au B? ¿Cuántos de estos subconjuntos D elemento de Bes el triple de otro elemento de B, ¿cuál es el
contienen un ndmero par de elementos? máximo valor posible de IBI?
:. Sea ".iU. = R y consideremos el conjunto índice /"= Q... 19. En una exposición científica de una escuela secunda-
:~:~~ Q'". seanA, = [O, 2q) yB,-=(0, 3q). Determine ria. 34 estudiantes recibieron premios por sus proyectos cien-
tíficos. Se dieron 14 premios por proyectos de biología, 13
de química y 21 de física Si ues esrudiames recibieron prt•
a) A 713 b) B'!I! e:) A 3 -B,. d) A 3 .ó. B,. mios en las tres áreas temáticas, ¿cuántos recibie ron pre•
e) ~A" f) !,t,B, g) QA• b) QB., mios por exactamente (a) un área temática? (b) dos áreas
temáticas?
1-. Para un universo G\l y conjuntos A. B ~ ·~ .demuestre 20. Si las letras de la palabra BOOLEAN se ordenan al
siguiente: azar.¿cuál es la probabilidad de que las dos letras O queden
a) AAB=BAA b) AAA•'lL juntas en la disposición?
e) AA'lL=A 21. Si se distribuyen 16 galletas con chispas de chocolate
d) A.ó. f =A,porloque tJesel neuuopara 6.,al igual entre cuatro niOOs, ¿cuál es la probabilidad de que cada
que para U ni.no reciba (a) al menos una galleta? (b) al me nos dos ga-
f. Consideremos la siguiente tabla de pertenencia (Tabla lletas?
,). Si tenemos oomo condición que A C B. entonces sólo 22.. Cincuenta estudiantes. cada uno con 75 centavos. visi-
bCJnos tcoec en cuenta las filas de la tabla para las que taron el salón de videojuegos del ejemplo 3.25. De ellos. 17
toes verdadero (las filas 1, 2 y 4, que se indican mediante jugaron los tres videojuegos y 37 jugaron al menos dos de
a flecha. Para estas filas, las columnas de B y A U B son ellos. Ningún estudiante practicó ocro del salón. ni tampoco
el mismo juego más de una vez. Cada juego cuesta 25 cen-
Tabla 3.4 tavos y el total obtenido de la visita de los estudiantes fue
de .$24.25. ¿Cuántos estudiantes optaron por ver y no jugar
A B AU B
ninguno de los juegos?
-+ o o o 23. ¿De cuántas formas puede asignarse trabajo a 15 ayu-
-+ o 1 1 dantes de laboratorio en uno, dos o tres experimentos dife-
1 o 1 rentes, de modo que en cada experimento haya al menos
-, 1 1 1 una persona al pendiente de él?
Ejercicios complementarios 181

La profesora Diana puso un examen con tres preguntas a) 'IL={l , 2,3}. b) 'IL = {l , 2, 3,4).
• grupo de química. Hay 21 estudiantes en su clase Y <) 'IL={l,2,3,4,S}. d) 'lt= {l ,2,3, ... , n}.
uno de ellos contestó al menos una pregunta. Cinco e) '\l. ={ai.a2 ,a3 , ••• , a..}, dondes= a1 + a2 +a,+
oo contestaron la primera pregunta, siete falla- · · •+ a,,.
al contestar la segunda, y seis no contestaron la tercera 27. a) En ajedrez. el rey se puede mover una casilla e n
Si nueve estudiantes contestaron las tres prcgun- cualquier dirección. Si suponemos que el rey sólo
¿aiinros contestaron exactamente una pregunta? se puede mover bacía adelante (una casilla hacia
aniba, hacia la derecha o en fonna d.iagonal hacia
Sea~ un wliverso dado, con A. B ~~.A n B = 0.
el noreste), a lo largo de cuántas trayectorias se
1=l2y IBI = 10. Si sceligensieteelementos deA U B, puede mover un rey de la casilla de la esquína in-
es la probabilidad de que la selección contenga cuatro
ferior izquierda basta la de la esquina superior de-
-.SdeAytresdeB?
recha en el tablero comi1n de 8 x 8 ?
a. Para un conjunto finito A. de enteros, sea cr(A) la suma b) Para las trayectorias de la parte: (a), ¿cuál es la pro-
* los elementos de A. Entonces, si~ es un universo finito
...,., de Z♦• rA~,C,a¡O"(A) denota la suma de todos los
babilidad de que u.na de ellas contenga {i) exacta-
mente dosmovimientos diagonales? (ii) exactamen-
dmmtos de todos los subconjuntos de C\I.. Determine te dos movimientos diagonales consc.cutivos? (iii)
I-,.o(A) para un nómcro par de movimientos diagonales?
4
~ledades de los
enteros: Inducción
matemática

a que hemos oído de los enteros desde nuestros primeros encuentros con la aritmética,
Y en este capítulo examinaremos una propiedad panicular que se halla presente en el
subconjunto de los enteros positivos. Esta propiedad nos pennitirá establecer algunas f6<-
mulas y teoremas matemáticos mediante una tknica llamada inducción maremárica. Este
~todo de demostración teodrá un papel central en muchos de los resultados que obten-
dremos en capítulos posteriores de este texto. Además, este capítulo presenta tres conjun-
tos de números que son muy impon.antes en el estudio de las matemáticas discreta y
combinatoria: tos números armónicos, los números de Fibonacci y los nWDCros de Lucas.
Cuando x, y E Z, sabemos quex + y, :,;y, x- y E Z. Así, decimos que el conjunto Z es
cerrado en Oas operaciones binañas de) suma, multiplicación y resta. Sin embargo, aJ
pasar al caso de la división, vemos, por ejemplo, que 2. 3 E Z pero que el m1mcro racional
f no es un elemento de Z. De 01odo que el conjunto Z de los enteros no es cerrado en la
operación binaria de división tntre números distintos U cero. Para enfrentar esta situa•
ción. presentaremos una forma un poco restringida de división para Z y nos concentrare-,
mos en algunos elementos particular<S de z• llamados primos. Estos primos serán como
los "bloques de construcción,. de los enteros, y nos darán el primer ejemplo de un teorema
de representación; en este caso. el teorema fundamental de la aritmética.

4.1
El pñncipio del buen orden:
Inducción matemática

Dados dos enteros díferentesx. y. sabemos que..r <yo y <x. Sin embargo. esto también es
cien.o si, en vez de ser enteros. :e y y son mlmeros racionales o números reales. ¿Qué hace
especial a Z en este caso? Supongamos que tratamos de expresar el subconjunto z· de Z,
mediante los símbolos de desigualdad> y ;a,;_ Vemos que podemos definir el conjunto de
los elementos positivos de Z como

z• -¡.- E Z ~> O}-{r e Z lx;a, !}.


183
No obstante, cuando intentamos hacer lo mismo con los números racionales y reales, ve-
mosque

Q+ = {xE Q!x >O} y R• =µ ER!x>O},


pero no podemos representar Q+ o R• con ~ como lo hicimos con z•.
El conjunto Z• es diferente de los conjuntosQ" y R-- por el hecho de querodo subconjunto
no vacío X de Z .. contiene un entero a E X tal que a s x, para todo x E X; es decir, X
contiene un elemento menor(o mínimo). Esto no ocune para Q+ o R+. Estos conjuntos en sí
mismos no contienen elementos mínimos: no existe un número racional positivo ni un na.
mero real positivo núnimo. Si q es un número racional positivo, entonces, como O< q/2 < q,
tendríamos un número racional positivo más pequeño, q/2.
Estas observaciones dan lugar a la siguiente propiedad del conjunto z• C Z.

Este principio sirve para distinguir a Z.. de Q.. o R•. Pero, ¿conduce a algo que sea
interesante o útil desde el punto de vista matemático? La respuesta es un rotundo "¡Sí!" Es
la base de una técnica de demostración conocida como inducción matemática. Esta técnica
nos ayudará con frecuencia para demostrar una proposición matemática general relacio-
nada con los enteros posi tivos, cuando algunos casos de esa proposición sugieran un pa-
trón general.
Ahora estableceremos la base de esta técnica de inducción.

EOREMA 4 .1 Principio de inducci6nfinita o principio de inducción matemática. Sea S(n) una proposi-
ción matemática abierta ( o un conjunto de tales proposiciones abiertas), en la que aparece
una o varias veces la variable n, que representa a un entero positivo.
a) Si S(l) es verdadera; y
b) siempre que S(k) sea verdadera (para alg~n k e z· particular, pero elegido al azar),
entonces S(k + l)será verdadera;· ·
entonces S(n) es veniadera para todo n E z...
Demostra ción: SeaS(n) una proposición abienacon las condiciones (a) y (b), y seaF= {t
E z• 1S(t) es falsa} . Queremos mostrar que F = 0, así que paraobtenerunacontradiccióo
suponemos que F #= 0. Entonces, por el principio del buen orden, F tiene un elemento
mínimo s. Como S(l ) es verdadera, s #. 1, por lo que s > l y, en consecuencia, s - 1 E z•.
Comos-1 lf; F, tenemosqueS(s - 1) es verdadera.Así, por la condición (b), se sigue que
S((s - 1) + 1) = S(s) es verdadera, lo que contradice que s E F. La contradicción surge de
la hipótesis F ,/a 0. Por lo tanto. F = 0.

Hemos utilizado eJ principio del buen orden en la demostración del principio de inducción
matemática. También es cierto que el principio de inducción matemática nos sirve para demos-
trar el principio del buen orden. Sin embargo, no nos detendremos en este punto por ahora En
esta sección, nuestro principal objetivo es comprender y utilizar el principio de inducción ma-
temática (Sin embargo, en los ejercicios de la sección 42 analizaremos la forma en que se usa
el principio de inducción matemática para demostrar el principio del buen orden.)
En el enunciado del teorema 4.1, la condición de la parte (a) se conoce como la base de
la inducción, mientras que la pane (b) se conoce como el paso inductivo.
La elección de 1 en la primera condición del teorema 4.1 no es obligatoria. Lo único
que se necesita es que 1a proposición abierta S(n) sea verdadera para un primer elemento
no E Z para que el proceso de inducción tenga un lugar de inicio. Necesitamos que S(no)
sea verdadera como base de la inducción. El entero no podría ser 5 o l. Incluso podría ser
O o negativo, puesto que el conjunto z• junto con {O} o cualquier conjunto finito de ente-
ros negativos sigue siendo bien ordenado. (Si hacemos una demostración por inducción y
partimos de no< O, nos fijarnos en el conjunto de todos los enteros negativos consecutivos
.e no, unido con {O} y z ·.)
En estas circunstancias, podemos expresar el principío de inducción finita. usando
cuantificadores, como

[S(no) /\[Vk 2c no[S(k)9S(k + 1)]]9 Vn "'"° S(n).


Podemos comprender mejor la razón de la validez de este método de demostración
usando nuestra intuición, junto con la situación que se presenta en la figura 4.1.
En la parte (a) de la figura vemos las primeras cuatro fichas de una disposición ( ordena•
da) finita de fichas de dominó, cada una puesta en forma vC{tical. El espacio que hay entre
dos fichas consecutivas es siempre el mismo y es tal que si cualquier ficha (digamos, la
k-ésima) se empuja hacia la derecha, entonces golpeará. la siguiente ((k + 1)-ésima Este
proceso se representa en la figura 4.1 (b). Nuestra intuición nos hace pensar que este proce-
so continuará: la (k + 1}-ésima ficha golpeará (a la derecha) la (k + 2)-ésima, etcétera.

(a)
1
11 11
n, n0 + 1 "º + 2 n0 +3
· · • - -- -

(b)
. . ='1 .. - k k+1

(e)
-/1 1 1
1 n, n0 + 1 n0 + 2 n0 + 3
··· - - - -

Figura 4.1
La parte (e) de la figura indica que la verdad deS(no) proporciona el empuje(a la derecha)
de la primera ficha (enfl(J). Esto proporciona la base de la inducción y pone en movimiento
el ~ - Como S(k) es verdadera, S(k + 1) es verdadera, lo que nos proporciona el paso
inductivo y contim1a el proceso de caída de las fichas. Entonces. podemos inferir el hecho
de queS(n) es verdadera para todani?:no si imaginamos todas las fichas sucesivas cayendo
(hacia la derecha)'.

Ahora demostraremos varios resultados que utilizan el teorema 4.1.

- Paracualquiern E z+,¿ ;_1i = 1 +2+3+···+n = (nXn+ 1)/2.


Demostración: Paran= l , la proposición abierta

S(n): ±i=l + 2+3+ ··· +n=(n)(n+l)/2


l• l

se convierte en S(l ): L!..,i = (!XI+ 1)/2. Así, S(l) es verdadera y tenemos nuestra bas,
para la inducción, un punto de inicio para comenzar la inducción. Si suponemos que el
resultado es cieno paran;;;; k (para algúnk E Z•'), queremos establecer nuestropaso inductivo
mostrando que la verdad de S(k) "obliga" a aceptar la verdad de S(k + 1). [La hipótesis de
la verdad de S(k) es nuestra hip6tesis ,k inducción.] Para establecer la verdad de S(k + 1),
necesitamos mostrar que

(k + l)(k + 2)
~i
l•l 2
Hace~os lo siguiente.

ttl (' k(k + 1)


¿i=l+2+···+k+(k+l) = ¿i) +(k+l)=--+(k+l),
~ ~ 2
ya que estamos suponiendo la verdad de S(k). Pero

k(k + 1) ~ (k + l) = k(k + 1) + 2(k + 1) (k + l)(k + 2)


2 2 2 2
lo que establece el paso inductivo [condición (b)] del teorema.
En consecuencia, por el principio de inducción finita.S(n) es verdadera para todon E z•.

Ahora que hemos obtenido la fórmula para la suma L ;:_1¡ de dos formas (véase el ej.
1.38). nos desviaremos un poco del tema principal y estudiaremos un ejemplo que usa esta

____________________________,
fórmula de la suma.

Una ruleta tiene. números del 1 al 36 pintados en ella de manera aleatoria. Mostraremos
que, independientemente de la posición de los números. hay tres números consecutivos
(en la ruleta) que suman 55 o más.
• Seax1 cualqujer m1mero de la ruleta. Contamos en dirección de las manecillas del reloj
a partir de x" y llamamos a los demás .números x2• x3, . . • • x36. Para que el resultado sea
falso, debemos tener x1 + z, + x 3 < 55, XJ + x3 + x., < 55, ... , X34 + X35 + X36 < 55, X35 + X 36 +
x 1 < SS yx36 +x1 +x:i <55.Enestas36desigualdades,cadaunodelos términosx1>X:z, ... ,x36
aparece exactamente tres veces. por lo que cada uno de los enteros 1, 2, ... , 36 aparece tres
veces. Si sumamos las 36 desigualdades, tenemos que 3¿ :!, x, =3 ¿ :, i < 36(55) = 1980.
Pero I :!,i = (36)(37)/2 = 666 y esto nos da la conlradicción 1998 = 3(666) < 1980.

La siguiente fórmula para una suma nos lleva de la primera potencia a los cuadrados.

Demuestre que para cualquier n E z•,¿;./= (nXn + 1x2n + 1)/6.


Demostración: Aquí trabajamos con la proposición abierta

S(n): ¿ i' = (n)(n + 1)(2n + 1)/6.


l• I

Base de la inducción: Comenzamos con la proposición S(l) y vemos que

±
l•I
i' = l' = (1)(1 + 1)(2(1) + 1)/6,

por lo queS(l) es verdadera.


Paso inductivo: Supongamos ahora la verdad de S(k) para un k E z• (particular), es
decir, supongamos que

±=
l• I
i' k(k + 1)(2.k + 1)/6

es una proposición verdadera (al reemplazar n por k). De esta hipótesis queremos deducir
la verdad de

S(k + 1): ~ i' = (k + l )((k + 1) + 1)(2(k + 1) + 1)/6


l•I = (k + l )(k + 2)(2.k + 3)/6.

Si usamos la hipótesis de inducción S(k), vemos que


k+t .t
¿P=~ + ~+-··+~+~+~ = ¿ P+~+~
i• l i•l

= [(k)(k + 1)(2.k + 1)/6) + (k + 1)2 •

De esto tenemos

~ i 2 = (k + l)[(k)(2.k + 1)/6 + (k + 1)) = (k + 1)((2.k' + 1k + 6)/6)


i•I
= (k + l)(k + 2)(2.k + 3)/6,
• y el resultado general se obtiene por inducción matemática.
Antes de presentar más resultados en los que utilizamos el principio de inducción ma-
temática para establecer su validez. observemos el inicio de las demostraciones de los
ejemplos 4.1 y 4.3. En ambos casos, simplemente reemplazamos la variablen por 1, obte-
nemos igualdades sencillas y verificamos si son verdaderas. Si consideramos que era defi-
nitivamente~ complicado establecer el paso inductivo en el resto de estas demostracio-
nes, tal vez nos preguntemos por qué hay que preocuparse por la base de la inducción. Por
ello, vamos a analizar el siguiente ejemplo.

- Sin E z•, establezca la validez de la proposición abierta

..,
S(n): Li=l+2+3+ •· · +n = (n2 +n+2)/2.

Esta vez vamos directamente al paso inductivo. Si suponemos la verdad de la proposi-


ción

S(k): f; = 1 + 2+ 3 + · · · + k = (k'+ k +2)/2.


l •t

para algún k E z• (particular), queremos ver si podemos inferir la verdad de la proposi•


ción

S(k + 1): ~ i= 1 +2+ 3+ · · · + k + (k + 1) =((k + l)'+ (k + 1)+2Yl


l•t

= (k' + 3k + 4)/2.

Como lo hicimos anteriormente, usamos la hipótesis de inducción y hacemos el cálculo


siguiente:
..,
Li=l+2+3+ ... +k+(k+l) =
i•l
~)
Li +(k+l)=(k'+k+2)/2+(k+l)
1

= (k' + k + 2)/2 + (2k + 2)/2 = (k' + 3k + 4)/2.

Por lo tanto, para cualquier k E z•. se tiene que S(k) ⇒ S(k + 1). Pero antes de decidir
si aceptamos la proposición 'vn S(n) como verdadera. reconsideremos el ejemplo4. l, don-
de aprendimos que ~;; = n(n + 1)12, para todo n E z•. Por lo tanto, podemos usar estos
,..
dos resultados (el ejemplo 4.1 y el recién "establecido") para concluir que

\In [ n(n + 1)/2 = t. i = (n' + n + 2)/2).

Así, tenemos que para todo n E z•,


n(n + 1)/2 = (n' + n + 2)/2,

lo que implica que n(n + l)= n'+ n + 2 y O= 2. (¡Algo está mal!)


1
Sin= 1, entoncesI,1= 1, pero (n'+ n + 2)12 = (1 + 1 + 2)12 = 2. Así, S(l) no es
i•l
verdadera
Pero tal vez pensemos que este resultado únicamente indica que e1egimos el punto de
inicio incorrecto. Tal vezS(n)sea verdadera para tcxlon ~ 3,o todon ~ 7,otodon ~ 137.
Sin embargo, si usamos el argumento anterior, sabemos que para cualquier punto inicial
no E z•. si S(no) fuera verdadera. entonces
.,
(n¡ +no+ 2)12 = Li = 1 + 2+ 3 + · · · + no-
,.,
del resultado del ejemplo 4.1, tenemos ~i = nfÁrto + 1Y2, por lo que nuevamente tendria-
mos O = 2; así, no tenemos un punto de inicio.
Este ejemplo debe indicar al lector la necesidad de establecer la base de la inducción;
sin importar lo fácil que sea verificarla

Consideremos ahora los siguientes programas en Pascal. El programa de la figura 4.2


utiliza un ciclo repeat - until para acumular la suma de cuadrados. El segundo programa
(Fig. 4.3) demuestra la forma de usar el resultado del ejemplo 4.3 en vez de tal ciclo. Cada
programa se usa para evaluar la suma de los cuadrados de los primeros 17 enteros positi-
vos. Sin embargo, mientras que el programa de la figura 4.2 implica un total de 2n(= 34)
sumas y n(= 17) aplicaciones del proceso de elevar al cuadrado (sqr}, el programa de la
figw-a 4.3 requiere sólo dos sumas, tres multiplicaciones y una división (entera). El núme-
ro total de sumas, multiplicaciones y divisiones (enteras) sigue siendo 6 aunque el valor de
n sea cada vez mayor. En conse.cuencia., el programa de la figura 4.3 se considera más
eficiente. (Examinaremos esta idea de un programa más eficieltll en las secciones 5.7 y
5.8.)

Progra.m sumo fSquaresl (input, output};


var
i,n,s: integer;
Begin
Writeln ( ' Queremos encontrar la swna de los cuadrados d e los
primeros · } ;
Write ( 'n enteros positivos , cuando n = ') ;
Read(n} ;
s : = O;
J
i := O;
Repeat
i :=i +l;
s := s + sqr(i}
Unt il i" e n;
writeln( 'La suma de los cuadrados de los primeros n: O};
Writel · enteros positivos es ', s : O)
End.

Queremos encontrar l a suma de los cuadrados de los primeros


n enteros positivos, cuando n = 17
La suma de los cuadrados de los pril'tteros 17
enteros positivos es 1785

Figura 4.2
Program SumOfSquaresl (input, output);
var
n, s: integer:
Begin
writeln( ·Quereoos encontrar la suma de los cuadrados de los '):
Wri te ( ' primeros n enteros positi vos, cuando n = · );
Read (n ) ;
s := (n ) •(n + 1)*(2 * n + 1) Div 6;
Writel n('La suma de los cuadra dos de los primeros n:O);
Writel · enteros positivos es •, s: O)
End.

Queremos encontrar la suma de los euadra dos de los primeros


n enteros positivos, cuando n = 17
La suma de los cuadrados de los primeros 17
enteros positivos es 1785

Figura 4.3

~i observamos de nuevo nuestras dos primeras aplicaciones ele la inducción matemáti-


ca (en los ejemplos 4. 1 y 4.3). es posible que nos preguntemos si este principio se aplica
sólo para la verificación de fórmulas para sumas conocidas. Los siguientes seis ejemplos
muestran que la inducción matemática es una herramienta vital en m uchas otras circuns-
tancias.

Consideremos la suma de enteros positivos impares consecutivos.

1) 1 =1 ( = l')
2) 1+3 =4 (=2')
3) 1+3+5 =9 ( =3')
4) 1+3+5+7 = 16 (=4')
5) 1+3+5+7+9 =25 (=S')
6) 1 +3+5 +7 + 9 +11 =36 ( = 6')
De estos primeros seis casos conjeruramosel siguiente resultado: la suma de los prime-
ros n enteros positivos impares consecutivos es n2, es decir, para todon E z·,

S(n): L (2i - 1) = n2•


í• 1

Ahora que hemos desarrollado una posible fórmula correcta para la swna. utilizaremos
el principio de inducción matemática para verificar que es verdadera para rodo n ~ l.
De los cálculos anteriores, vemos que S(I) es verdadera [al igual que 5(2), S(3), 5(4),
S(5) y S(6)], por lo que ya tenemos una base para la inducción. Para el paso inductivo.
suponemos que S(k) es verdadera y tenemos

.
L(2i-1)=k'.
i•1
Deducimos ahoraqueS(k+ 1) es verdadera: }:~ 11 (2i- l)= (k+ 1)2. Como suponemos
que S(k) es verdadera (nuestra hipótesis de jnducción) escribimos

'i
l•l
(2i -1) = ±
l •l
(2i -1) + [2(k + 1)-1) = k' + [2(k + 1)-1)

=k'+2k + 1 =(k + 1)2 •

En consecuencia. el resultado S(n) es verdadero para todo n :?: 1, por el principio de


inducción matemática

Ahora es el momento para analizar algunos resultados que no son fórmulas para sumas.

En la tabla 4.1 hemos enumerado en columnas adyacentes los valores de 4n y n2 - 7 para


los enteros positivos n, donde 1 s n ~ 8.

Tabla 4.1

• ""
1 4
,., _ 7
-6
.
5
""20
JI,_,
18
2 8 -3 6 24 29
3 u 2 7 28 42
4 16 9 8 32 57

A panir de la tabla, vemos que (rr'- 7) < 4n paran = 1, 2, 3, 4, 5; pero cuando n = 6, 7, 8,


tenemos que 4n < (Ir - 7). Estas tres últimas observaciones nos hacen conjeturar que para
todo n ;;e 6, 4n < (n'- 7).
Una vez más, el principio de inducción finita es la técnica de demostración que necesi-
tamos para verificar nuestra conjetura. Denotamos con S(n) la proposición abierta: 4n <
(n'- 7). Entonces la tabla 4.1 confirma que S(6) es verdadera [al igual que S(7) y S(8)], y
tenemos la base de inducción. (Finalmente tenemos un ejemplo donde el punto inicial es
un entero no~ l.)
En este ejemplo, la hipótesis de inducción es S(k): 4k < (k'- 7), donde k E z• y k ;;e 6.
Para establecer el paso inductivo, necesitamos deducir de S(k) que S(k + 1) es verdadera.
Esto es, de 4k< (k'- 7) debemos concluir que 4(k+ 1) < [(k+ J)'- 7]. Éstos son los pasos
necesarios:

4k <(k'-7)=}4k + 4< (k' - 7) +4< (k' -7) + (2k + 1)

(yaqueparak i:!.6.tenemosque2k+ 1 :?: 13>4),y

4k + 4 < (k' - 7) + (2k + 1):}4(k + 1)< (le' +2k + 1)-7 =(k + 1)2 -7.

• Por lo tanto, pocel principio de la inducción matemática,S(n)es verdadera para todon 2 6.


....,,-~~,, Entre las varias sucesiones interesantes de números que aparecen en las matemáticas dis-
creta y combinatoria, están los números armónicos H ., H2• H3 , ••• , donde
H 1 =1
H,= 1 +½
H 3 =1 +½+¼,

y, en general, H,,,= l +½+½+ ···+¼para cualquier n E z•.


Las siguientes dos propiedades de los números armóIDcos proporcionan dos oportuni•
dades para aplicar el principio de inducción matemática.

a) Para todo n EN, afirmamos que Hz" :S 1+ n.


Demostración: En este caso, la proposición abierta S(n) es H'tf s l + n; para la base de
inducción, debemos ver qué ocurre paran= O. Tenemos que H'l" = H p = H 1 = l S 1 +O ;
l + n, por lo que S(n) es verdadera para este primer caso (cuando n = O).
Si suponemos que S(k) es verdadera para algún k E N (no sólo z•). obtenemos la
hipótesis de inducción

S(k): H,>s 1 + k. .

Para el paso inductivo, analizaremos ahora S(n) paran• k + 1. Encontramos que

H,>•• = [1+!+! + . . . +.!.]+[- - ! - + __ ! _ + ... + - -!- ]


2 3 2• (2' + 1) (2' + 2) (2'+ 2')

= H,> + [(2' ~ !) + (2' ~ 2) + ... + (2' ~ 2')].


1 1
eorno (Zt + j) <F. para todo 1 SJ· ::s; 2•, se sigue
· que

H,.., s H,> + (2')(~) = H,. + l.

Y ahora. de la hipótesis de inducción se sigue que

H,.-, sH,> + !s(l +k)+ 1 = !+ (k + !),

por lo que el resultado S(n) es verdadero para todo n E N, en virtud del principio de
inducción matemática..

b) Sin E Z', entonces L, H¡ = (n + 1) H, - n.


j • I
Demostración: Como lo hemos hecho en los ejemplos anteriores (es decir, los ejemplos
4. 1, 4.3 y 4.5), verificamos la base de inducción enn = I para la proposición abiertaS'(n):
ÍH¡=(n + l)H._ - n. Este resultado se sigue fácilmente de
;-,

'
L.H; = H, = 1 = 2 · 1-1 = (!+ l)H,-1.
j• J
Para verificar el paso inductivo, supondremos que S'(k) es verdadera; es decir,

L• H; •
/•1
(k + 1) H, - k.

Esta hipótesis nos conduce a lo siguiente:


l•I ,t
L H;= L H; + H,.1 = [(k + l)H, - k]+ H,.1
1•1 1•1 =(k+ l)H, - k+H,. 1
=(k + l)[H.., - (1/(k + 1)))- k + H..,
=(k + 2) H,. 1 - (k + 1)(1/(k + 1)) - k
=(k + 2) H,. 1 - 1 - k
= (k + 2) H,. 1 - (k + 1).
En consecuencia, sabemos, por el principio de inducción matemática. que la proposición
Vn S'(n) es verdadera para todos los enteros positivos n.

Paran ;;, O, sea A, C R, donde IA,1 : 2• y los elementos de A, se enumeran en orden


ascendente. Sir E R , demostraremos que para determinar si r E A., (por el procedimiento
que se desarrolla e n seguida), debemos comparar r con no más de n + 1 elementos de A•·
Cuando n = O, Ao= {a} y sólo se necesita una comparación. Así, cJ resultado es verda-
dero paran= O (y tenemos la base para la inducción). Paran= 1, A 1= {a,.~}. a1 <~. Para
determinar si , E A 1o debemos hacer dos comparaciones a lo más. Por lo tanto. obtenemos
el =ultado paran: l. Ahora. cuandon: 2, escribimos A,: {b ,_b,_c,.c,}: B,uc,, doode
b1 < b2 < c1 < c'l, donde 8 1={b., b¡} y C,= {c1, c2 }. Si comparamos rcon b 2,pucde ocurrir
alguna de las siguientes posibilidades: (i) r E 8 1; o (ii) r E e,. e,
Como I B, 1 2 1 1 : 2,
cualquiera de las dos posibilidades requiere a lo sumo dos comparaciones (del caso ante-
rior, en quen = 1). Por lo tanto, podemos determinar sir E A 2 haciendo no m'5 de 2 + 1=
n + l comparaciones.
Ahora podemos dar un argumento general. Suponpmos que el resultado es cierto para
algún k :r?: O y consideremos el caso de At.1, donde IAt.1 J = '1!+1• Para establecer nuestro
paso inductivo, sea A,.,= B, U C,, donde IB,I : 1C,I = 2• y los elementos de B,, C,están
en orden ascendente, con el máximo elemenco x de 81,. menor que el mínimo elemento de
c.. Sea r E R. Para determinar si r E A,..., veamos si r E 84 o r E c•.

a) Primero comparamos r y x. (Una comparación)


b) Sir :S x, como IB,I : 2', se sigue de la hipótesis de inducción que podemos deter-
minar si r E B., haciendo no más de k + 1 comparaciones adicionales. Por lo tanto,
no se hacen más de (k + 1) + 1 comparaciones.
e) Si r > x, hacemos algo similar con los elementos de C..,. Hacemos como máximo
k + 1 comparaciones adiciona1es para ver si , E C4•

El resultado general se sigue entonces del principio de inducción finita.


Una preocupación primordial en la evaluación de la calidad de un programa consiste en
ver si el programa hace lo debido. Así como no podemos demostrar un teorema verifican-
do casos particulares, tampoco podemos establecer si un programa es correcto o no pro-
bando con diferentes conjuntos de datos. (Además. esto sería muy diífcil si nuestro ~
grama fuera a formar parte de un paquete de software más grande en el que, probablemen•
te, se generase el conjunto de datos en forma interna.) Puesto que el desarrollo de software
pone ahora un gran énfasis en la programación estructurada. esto ha traído consigo la
necesidad de la vuificaci6n de programas. En este caso. el programador o equipo de
programadores debe mostrar que el programa en desarrollo es correcto independi.enlt·
mente del conjunto de: datos proporcionados. El esfuerzo invertido en esta etapa reduce en
forma considerable el tiempo utilizado en depurar el programa (o paquete de software).
Uno de los métodos que pueden tener un papel fundamental en tal verificacíón de progra-
mas es la inducción matemática. Veamos cómo.
El segmento de programa en Pascal que se muestra en la figura 4.4 supuestamente
produce como respuestax(y") para las variables reales x, y y n es un entero no negativo. (Et
usuario asigna los valores de estas tres variables en una etapa anterior.) Verificaremos si
este segmento de programa es correcto, aplicando la inducción matemática a la proposi-
ción abierta
S(n): Para cualquier x, y E R , si el programa llega a la parte superior del ciclo While con
n E N, después de evitar el ciclo (para n = O) o ejecutar las dos instrucciones del ciclo
n(> O) veces, entonces el valor de: la variable real Answer es .:c:Cy-).

While n <> O do
Begin
X := X*y;
n := n - l
End;
Answer : = x;
Figura 4.4

El diagrama de flujo para este segmento de programa aparece en la figura 4.5. Mientras
desarrollamos nuestra demostración será conveniente hacer referencia a dicho diagrama.
Primero consideremos S(O), la proposición para el cason = O. En este caso, el programa
llega a la parte superior del ciclo Wbile. pero como n = O, se sigue a la rama No en el
diagrama de flujo y se asigna el valor x = x(l) = xó'1) a la variable real Answer. Por lo
tanto, la proposición S(O) es verdadera y se establece la base de inducción de nuestro
argumento.
Ahora supondremos que S(k) es verdadera, para algún entero k no negativo. Esto nos
proporciona la hipótesis de inducción.
S(k): Para cualquier x, y E R, si el programa llega a la parte superior del ciclo While con
k EN, después de evitar el ciclo (parak= O) o ejecutar las dos instrucciones del ciclok(>O)
veces, entonces el valor de la variable real Answer es x(y).
Si seguimos oon el paso inductivo de lademostrllción. al trabajar con la proposición.l'(k + 1i
observamos que, comok + 1 :?: 1, el programa no seguirá la rama No ni evitará las insttuc-
Inicio

Inicia liza !as


variables rHlu
1(,y y la variable-
entera no
nega tiva n

La parte svperior
de-1 ciclo Whi!t>.
No

Answer :a: X
El progra m.i continúa con
la siguiMtt' instrucción
- - - - - t>jKuta ble dt>spués d e la
,instrucción de asignación
p-,ra la va riablf' rea! Answt"r.

Figura4.5

ciones de] cic1o While. Las dos instrucciones (en e1 cic1o Wbile) se ejecularán al menos
una vez. Cuando el programa ]legue a la parte superior del ciclo Wbile por primera vez.
n = k + 1 > O, de modo que las instrucciones del cic1o se ejecutan y el programa regre sa a
la parte superior del ciclo While, donde ahora vemos que

• El valor de y no ha cambiado.
• Elvalorde.xesx1 = x(y1)=.ty.
• El valordenes(k+ 1)-1 = k.

Pero ahora, por nuestra hipótesis de inducción (aplicada a los números reales X i, y). sabe-
mos que después de evitar el cic1o (para k ::::: O) para x 1, y y n. = k, o ejecutar las dos
instrucciones del cic1o k(>O) veces, entonces el valor de la variable real Answer es

x,(y') = (xy)(y') =x(y"').

De la ecuación 14 = 3 + 3 + 8 vemos que podemos expresar el número 14 usando solamente


treses yochos como sumandos. Pero lo que parece sorprendente es que: para cualquier n 2:: 14,

S(n): n se puede escribir como una suma de treses y ochos (sin imponar el orden).

Como comenzaremos a verificar S(n) para todo n 2:: 14, la frase introductoria anterior
muestra que la base de inducción S(14) es verdadera. Para el paso inductivo. suponemos
queS(k)es verdadera para algún k E z•. donde k:::. 14 y veremos lo que ocurre con S(k +
1). Si existe al menos un ocho en la suma (de treses y ochos) que sea igual a*• entonces
podemos reemplazar este ocho por tres treses y obtener k + l como una suma de treses y
ochos. Pero supongamos que 8 no aparece como sumando de k. Entonces, e1 único SI-
mando utilizado es un 3 y, como k .?:: 14. debemos tener al menos cinco treses como
sumandos. Y ahora, si reemplazamos cinco de esos treses por dos ochos, obtenemos la
sumak + 1, donde los t1nicos sumandos son treses y ochos. Por lo tanto, hemos mostrado
que S(k) ⇒ S(k + l); el resultado se sigue para todon.?:: 14 por el principio de induccióa
matemática

Ahora que hemos visto varias aplicaciones del principio de inducción matemática, ce-
rraremos esta sección presentando otra forma de inducción matemática. Esta segunda fm·.
mase conoce a ve.ces como la/orma alternativa de la inducci6n, el método de induccWII
completa o el principio de inducci6n matemática faene.
De nuevo, consideremos una proposición de la forma Vn 2 no S(n), donde no E Z";
ahora estableceremos una base de inducción y un paso inductivo. Sin embargo, esta vez, 1a
base de inducción requiere la demostración de algo más que el primer caso, cuando n = 11t,
En el paso inductivo, supondremos que todas las proposiciones S(no), S(11o + J ), .•. , S(k- 1)
y S(k) son verdaderas, para establecer la verdad de la proposición S(k + 1). En el siguiente
teorema presentamos formalmente este segundo principio Q.e inducción matemática.

lcOREMA4.2 Principio de inducci6n finita, forma alternativa. Sea S(n) una proposición matemática
abierta (o un conjunto d~ tales proposiciones abiertas) donde la variable n, que representa
un entero positivo, aparece una o más veces. Además, sean n0 , n 1 E z• con n 0 S" n,.

a) Si S(11o). S (11o + 1), S(11o + 2), .. . , S(n, - l ) y S(n ,) son verdaderas; y


b) Siempreque S(11o),S(11o+ l ), ... ,S(k-1) yS(f)sean verdaderas para algúnk E z•
(panicular pero elegido al azar), donde k ~ ni. entone.es la proposición S(k + 1)
también es verdadera;

entonces S(n) es verdadera para todo n ~ no.

Como en el teorema 4.1, la condición (a) se denomina base de inducci6n y la condición (b)
paso inductivo.
La demostración del teorema 4.2 es similar a la del teorema 4.1; pediremos al lector que
la realice en los ejercicios de esta sección. También en estos ejercicios veremos que las dos
formas de inducción matemática (dadas en los teoremas 4.1 y 4.2) son equivalentes, ya
que cada una puede ser una técnica de demostración válida si suponemos la verdad de la
otra.
Antes de dar ejemplos de aplicación del teorema4.2, mencionaremos, como lo hicimos
para el caso del teorema 4.1 , que no es necesario que no sea un entero positivo; en realidad,
podría ser O o incluso un entero negativo. Ahora que hemos vuelto a señalar este punto,
veamos cómo aplicar esta nueva técnica de demostración.
Nuestro primer ejemplo resultará conocido ya que simplemente aplicaremos el teorema
4.2 para obtener el resultado del ejemplo 4.10 en otra forma.
Los siguientes cálculos indican que es posible escribir (sin imponar el orden) los enteros
14, 15, 16 usando como sumandos sólo treses y ochos.

14 = 3+3+8
15 = 3+3+3+3+3
16= 8+8

Con base en estos tres resultados. hacemos la conjetura

Para todo n E z•. donde n :. 14,


S{n): n se puede escribir como una suma de treses y ochos.

DemostrKión: Está claro que lasproposiciooesS(l4),S(l5) y S(l6)son verdaderas: esto


establece la base de inducción. (En este caso. noª 14 y n1 ;;;; 16.)
Pan el paso inductivo. suponemos que las proposiciones

S(14), S(JS), ... , S(k - 2), S(k - I), y S(k)

son verdaderas para algún k E z•, donde k:. 16. [La hipótesis de que estas (k- 14) + 1
proposiciones son verdaderas constituye nuestra hipótesis de inducción.] Y ahora. sin= k
+ 1, entoocesn:. 17 y k + 1 = (k-2) + 3. Pero como 14 :s k-2 :s; ky ya que S(k-2) es
verdadera. sabemos que (k- 2) puede escribirse como una suma de tres.es y ochos; así,
(k + 1) = (k-2) + 3 tambifo puede cscribine de esa forma. Por lo tanto, S(n) es verdadera
para todo n ;;?:. 14 por la forma alternativa del principio de inducción matemática.

En el ejemplo 4. 11 vimos que el hccbo de que S(k + 1) sea verdadera se dedujo de la


verdad de un resultado anterior S(_k - 2). Nuestros dos ejemplos siguientes muestran que
hay situaciones en las que se necesita que más de un resultado anterior sea verdadero.

-i] Vamos a considerar la sucesión entera a 0• a,. 4:z. a)' ... , donde

ao-I,a, - 2, a,=3, y
a. -= ª•-t + 0.-:z + ª•->. para todo n E z♦ tal que n ~ 3.

(Entonces. por ejemplo, tcoemosqueaJ;;;;a2 + a 1+ao;;;; 3 + 2 + 1 = 6; a.= a, + a2 + a 1= 6 +


3+2= ll ;ya,=a.+a,+a,= 11 +6+3=20.)
Afirmamos que las entradas de esta sucesión son tales que a. S 3• para todo n E N; es
decir, Vn E N S'(n), donde S~n) es La proposición abiena: a. :,;; 3•.
Para La base de inducción, observamos que

o ao - J-3":,;3";
H) a1 =2:,,3=31; y
111) a,=3:,,9=3'.

Por lo tanto, sabemos que SlQ). S'(l). S'(2) son proposiciones verdaderas.
Ahora analicemos el paso inductivo, en que suponemos que las proposiciones S'(Oi
S'(l),S'(2), ... ,S'(k - 1),S'(k) son verdaderas,paraalgúnk E Z'dondek l!: 2. Para el caso
enquen =k+ 1 ~ 3vemosque

D.t♦ 1 = ªt + D,t-t + Ot- 2


s 3t + 3t-1 + 3t-2
s 3' + 3' + 3• = 3(3') = 3t♦1,
por lo que [S'(k-2)/\S'(k- l) /\S '(k))~S'(k + 1).
Por lo tanto. de la forma alternativa del principio de inducción matemática se sigue que
a., .s 3" para todo n EN.

Antes de analizar el t1ltimo ejemplo de esta sección, revisaremos los dos resultados
anteriores. En los ejemplos 4.1 l y 4.12 establecimos la base de inducción verificando la
verdad de tres proposiciones:S(14),S(15) y S(16), en el ejemplo4.11; y S{O),S'(l) y S{2)en
el ejemplo 4.12. Sin embargo, para deducir la verdad de S(k + 1) en el ejemplo 4.11, en
realidad utilizamossolamenteunadelas(k- 14)+ J proposiciones de la hipótesis de inducción:
a saber, la proposición S(k- 2). Para el ejemplo 4. 12 usamos tres de las k + J proposiciones
de la hipótesis de inducción; en este caso, las proposiciones Hk-2).S{k- 1) y S'(ki

Después de transcurrir n meses en un experimento de invernadero, el nWilero p., de plan-


tas (de un tipo panicular) satisface las ecuaciones

p0 = 3, p, = 1, y p.=3p.- , -2p. - ,, paranEz+donden.e:2.


[Así, por ejemplo,¡,, = 3p,-2p0 = 3(7)-2(3) = 15 yp,= 3p,-2p,= 3(15)-2(7) = 31.)
Usaremos la forma alternativa de la inducción matemática para mostrar que p. = 2-i _ 1
para todo n EN.
Demostración: Aquí tenemos la proposición abierta
S(n): p.= 2"'2 -1,
y para establecer la base de inducción, observamos que las proposiciones

S(O): Po= 3 =2°'2 - 1, y


S(l): p, 1 = =2"2 - 1

son verdaderas.
Si suponemos que las proposiciones S(O), S(l), ...• S(k- 1) y S(k) son verdaderas para
algún k E z· donde k ;?: 1, consideremos a continuación el caso en que n = k + l ;?: 2.
Tenemos que
Pt•I = 3pt - 1.pt-l
= 3(2..2 - 1) - 2[:z<t-1>•2 - 1)
= 3(2"'')- 3 - :z<••I)♦) + 2
= 3(2"'') - (2"'') - 1
= 2(2"'') - l = :z<"•>•'-1.
[En este caso, la sustitución dept y Pt- i se basa en las proposiciones S(k) y S(k- 1), las dos
últimas de 1a lista de k + 1 proposiciones de la hipótesis de inducción.)
Así. por la forma alternativa del principio de inducción matemática. S(n) es verdadera
para todo n ~ O. (En el capítulo 10 baremos una incursión en los métodos para obtener las
soluciones explícitas a dichas ecuaciones.)

EJERCIOOS 4.1 1. Demuestre lo siguiente mediante inducción matemática.


a) 1' + 3' + 5' + · · · + (211 -1)' = (n)(211 - 1)(211 + 1)/3
b) 1•3 +2·4 +3 · 5 + · · · +n(n +2) =(n)(n + 1)(2n + 7)/6
• 1 •
e)¿--=-
1- 1 i(i + 1) n +1
• •'<•
t) ¿i' = --+- 1)'
= ( ¿i
• )' f ) 1'+3'+5'+ · · · +(2n-l)'=n'(211' - 1)
i• t 4 i-1

2 . Establezca lo siguiente mediante inducción matemática

a) ,t i(2'}= 2 +(n-1)2"'' e) ! (i)(i!) = (n + 1)! - 1

3 . Consideremos el siguiente programa en BASIC:

10 FOR I .,.. l TO 123


20 FORJ-lTOI
30 PRINT I•J
40 NEXT J
50 NEXT I
60 END

a) ¿Cuántas veces se ejecuta la proposición PRINT en la línea 30?


b) Reemplace I en la línea 20 por 1º2 y responda la pregunta de la parte (a).
4. Una ruleta~ losenlm)S 1 a 25 colocados en forma aleatoria Muestre que, independientemente de
la posición de los números en la ruleta. existen ~ de ellos adyacentes cuya swna es al menos 39.
5 . Los enteros del l al 100, inclusive. se han colocado al azar sobre una circunferencia. Demues-
tre que, sin importar su colocación, deben existir cuatro de ellos que fonnen tres pares solapa-
dos en la circunferencia, cuya suma es mayor o igual que 202.

6 . Determine el entero positivo n para el que


,.
Li=¿ ;
. 1

o'-1 ;-¡
7 . Evahíe lo siguiente: (a) L! 1 i; (b) L! 1 i2.
s. a) Demuestre que (cos 8 + i sen 8)2 = cos 28 + i sen 28, donde i E C e f = - 1.
b ) Demuestre por inducción que para cualquier n E (cos 8 + i senz·. 8r =cos n8 + i sen n8.
(Este resultado se conoce como el uortma dt De.Moivu.)
e) Verifique que l + i = .,fi.(cos 4~ + i sen 45°) y calcule (1 + ,)100•
9 . Paran E z·, sin > 10, demuestre que
n 2 -n
n-2<--.
12
10. Demuestreque paracualquier ne z •.n>3:::}2"<n!.
2
11. Demuestre que para cualquier n E z+. n >4=>n <2".
12. Demuestre que para cualquier ne z•. n >9=>n'< '1:'.
13. Paran E Z\ sea H.= l +½+½+·· ·+¼. eln-isimo m1mero armónico (definido en el ejem-
plo 4.7).
a) Para todo n E N. demuestre que 1 + (n/2) '!:Hr.

b) Demuestre que para todo n E z•, I,jH1 = [(n + l)(n)/2) H~, - [(n + IXn)/4).
,..
14. Paranez•,dcmucstrcque í'r.'i=H~-(t)H..
1S. Paran e z•. sea S(n) la proposición abierta
f;. (n + (1/2))'.
i• l 2
Demuestre que la verdad de S(k) implica la verdad de S(k + l) para cualquier k E z•. ¿Es
verdadera S(n) para todo n E Z"'?
16. Consideremos las cuatro ecuaciones siguientes:
(1) l=l
(2) 2+3+4=1+8
(3) 5+6+7 +8+9=8 +27
(4) W+U+l2+D+M+e+M =27 + M
Conjeture la fórmula general sugerida por estas cuatro ecuaciones y demu~strela.
17. Consideremos las seis ecuaciones siguientes:
(1) l'+O'=I' (3) 5'+12'=13' (5) 9'+40'=412
(2) 3'+4'=5' (4) 7'+24'•25' (6) 11'+602 =61'
Conjeture la fórmula general sugerida por estas seis ecuaciones y demu~ela.
18. SeanS, yS1talesque !S1 l =m, ls:I
= r, param, rE z• y loselemcntosdcS1.Szestánen orden
ascendente. Se puede demostrar que los elementos de S1 y S1 se pueden agrupar en orden
ascendente sin hacer más dem + r-1 comparaciones. (Véase el lema 12.1) Use este resultado
para esi:ablecer lo siguiente:
IS
Paran ~ O, sea S un conjunto con 1 = 2ª. Demuestre que el nWllero de comparaciones
necesarias para colocar los elementos de Sen orden ascendente está acotado por n · 2ª.
19. Sin E z•, demuestre que si sen 8 ~ O, entonces
9
:) (cos8)(cos28)(cos4a)(cos88)· · -[cos(r-' 8)] =sen(2" ).
Z" sene
scn2n8
b) cos9+cos3e +00058 + · • • +cos(2n - 1)8 = ~-

20. En el segmento de programa en Pascal que se muestra en la figura 4.6, x. y y Answer son
variables reales y n es una variable entera. Anees de la ejecución de este ciclo While. el usuario
proporciona los valores reales dezy y. así como un valor entero no negativo paran. Demuestre
por inducción matemática que para cualesquiera x.. y E R, si el programa Llega a la parte
superior del ciclo Whilc con n e N des~ de evitar el ciclo (paran = O) o ejecutando las dos
instrucciones del ciclo n(;::-0) veces. entonces el valor asignado a Answer es x + ny.

1rb1le o <> O do
Begin
x: • x+y;
n :- n - l
End:
Answer- : = x;
Figura 4.6
21. Durante la ejecución de un programa e n Pascal.. el usuario asigna a las variables enteras x y n
ciertos enteros positivos (posiblemente d.Jferentes). E! segmento en Pascal que se muestra en la
figura 4.7 va inmediatamente después de estas asignaciones. Si el programa llega a la pane
superior del ciclo While, enuncie y demuestre (por inducción matemática) cuál será el valor
asignado a Answer después de ejecutarse las dos instrucciones del ciclo n(:>0) veces.

Wb.1le n <> O do
Begin
x : - x•n:
n := n - 1
End ;
Figura 4.7

22. a) Sea n E z•,


con n .t 1, 3. Demuestre que n puede expresarse como una suma de doses.
cincos o ambos.
b) Parac.ualquicrn E z•.
muestre que sin~ 24,entonccsn puede expresarse como una suma
docincos y/o sietes.
23. Paran e z·. sea p. el nWDero (aproximado) de bacterias que hay en un cultivo, a1 final de n
horas (después de iniciado un experimento). Sip1=1000.p,:= 2000 y p. =p. _1 +v. _1, para todo
n > 2, muestre que

24. Definimos una sucesión de nómeros a1, a:, a3• •• • como


a1=l
a2 =2
a,.=a..-1+ an- 2, n~3.
a) Determine los valores de a3, a., '1J, a. y a1•
b) Demuestre que para todo n 2: l. a.,,< (7/4}"-.
25. Verifique el teorema 4.2.

4.2
Definiciones recursivas

Comenzaremos esta sección analizando la sucesión de enteros bo, bi, b2, b3, . . . , donde h. =
2n para tadon EN. Tenemos queb0 = 2 -0 = 0,b,= 2 • 1 = 2,b,=2 · 2 = 4 y b, = 2· 3 =6.
Si, por ejemplo, necesitamos determinar b6, simplemente calculamos b6 ;;;; 2 · 6 = 12, sin
necesidad de calcular el valor de b11 para cualquier otro n E N. Podemos realizar estos
cálculos ya que te nemos una fórmula txplfcita, b. = 2n, que nos dice cómo determinar b11
conociendo n (solamente).
Sin embargo, en el ejemplo 4.12 de la sección anterior, examinamos la sucesión de
enteros ao. a 1• az, a3♦ ••• , donde
ao= 1. a1 = 2, a2=3, y
a,.= a.,._1 + a,._2 + ª • -l para todo n E z• tal que n ~ 3 .
• Aquí no tenemos una fórmula txplícita que defina cada a,, en términos den, para todo n E
N. Si queremos conocer el valor de a 6 , por ejemplo, necesitamos los valores de a~. a.i y a 3•
Y estos valores (los de a5 , a,.. y a 3) requieren que conozcamos también los valores de ai, a1
y ao, A diferencia de la situación más fácil en que determinamos b, =- 2 • 6 =- 12, para
calcular a6 tendríamos que escribir

~=a5 + a.+a3
=(a,+ a,+ a,) + (a,+ a, +a1) + (a,+a, +ao)
=[(a, +a,+ a,)+ (a,+ a,+ ao) + az]+ [(a,+ a, + ao) + a,+ aJ +(a,+ a, +ao)
= [[(a,+ a,+ ao) + a, +aJ+.(a, + a, + ao) + az]
+ [(a, +a,+ ao) +a,+ a,]+ (a,+a, + ao)
=[(3 +2 + 1) + 3+ 2] + (3 + 2+ 1) + 3]+ [(3+ 2 + 1) +3 +2] + (3 + 2+ 1)
= 37.
o bien, de una manera más sencilla, podríamos haber ido en la dirección opuesta, con estas
consideraciones:

03= a2 + a1+ ao=3+ 2 + 1 =6


a.=a, +a,+ a 1 = 6+ 3 + 2= 11
a, = a, + a, +a, = 11 + 6+ 3 = 20
a.,=a,+a, +a,= 20 + 11 + 6 = 37.
Sin importar cómo lleguemos a '16, vemos que las dos sucesiones de enteros (bo. b., "2, ~
.. . , y ao, ai, ~ . o3 , •• • ) no sólo son numéricamente diferentes. Los enteros bo. bi, "2, bJ.
. .. , se pueden enumerar fácilmente como O, 2, 4, 6, . .. y para cualquier n E N tenemos
la fórmula explícita b,. ;;;; 2n. Por otro lado, tal vez sea dificil (si no es que imposible)
determinar tal fórmula e~plícita para los enteros ao, a1, ~. 01, .•.
Lo que ocurre aquí para esta sucesión de enteros puede ocurrir también para otros
conceptos matemáticos: conjuntos y operaciones binarias, así como funciones (en el capí•
tu.lo 5), lenguajes (en el capítulo 6) y relaciones (en el capítulo 7).A veces es difícil definir
un concepto matemático de manera explícita. Pero, como en el caso de la sucesión a.o, º"
~. o 3, ••• , podóamos definir lo que necesitamos en términos de otros resuJtados anteriores
similares. (Examinaremos lo que esto significa en Jos distintos ejemplos de esta sección.)
Cuando hacemos esto, decimos que el concepto está definido en forma. recursiva, usando
el método o proceso de ncursi6n. De esta manera obtenemos el concepto que nos intere-
saba estudiar, por medio de una definición recursiva. Por lo tanto, aunque no tengamos
una fórmula explícita en el caso de la sucesión 0o, ai, ~. a3, ••• , sí tenemos una forma de
definir Jos enteros o,. paran E N. por recursión. Las asígnaciones

ao= l ,
proporcionan una basépara la- recursión.
La ecuación
(·) a,.= Ra-t + 011-2 + 011-:,, paran E z+tal que n ~ 3,
proporciona el proceso recursivo; indica la forma de obtener nuevos elementos de la suce-
sión a partir de los resultados anteriores ya conocidos (o que se pueden calcular). {Nota:
los enteros calculados a partir de la ecuación(*) también pueden calcularse a partir de la
ecuación a_3 ;;;; 0-2: + a..1 + a,., paran E N.]

Ahora utilizaremos el concepto de definición recursiva_para establecer un aspecto men•


cionado en tres pies de página de las secciones 2.1 y 2.3. Después de estudiar la sección
2.2 del texto, sabíamos (a partir de las leyes de la lógica) que para cualesquiera proposicio-
nes Pi. f'2.. p3, tenemos

p, /\(p,/\p,) <e> (p, /\p,)/\p,,


y, por lo tanto, podemos escribir p 1 A />2 A p3 sin ambigüedad. Esto se debe a que el valor
de verdad de la conjunción de las tres proposiciones no depende de los paréntesis introdu-
cidos para indicar el orden de formación de la conjunción de los pares de proposiciones
(ciadas o resultantes). Pero nos interesaba el significado que podríamos atribuir a una ex-
presión como p 1 A Pz A p 3 I\ p,1. El siguiente ejemplo aclara ahora esta cuestión.

La conectiva lógica A se definió (en la sección 2.1) solamente para dos proposiciones a la
vez. ¿Cómo trabaja uno entonces con una proposición comop1 A p 1 A p 3 I\ p", dondep11
p 1, p 3 y P,1 son proposiciones? Para responder esta pregunta, introducimos la siguiente
definición recursiva, en la que el concepto de cierta etapa [la (n + 1)-ésima] se desarro11a
a partir del concepto comparable de una etapa anterior [la n-ésima].
Dadas cualesquiera proposiciones p;, Pz, . . . , p., p,..1, definimos
1) la conjunción dep1, p1 como p 1 A p 2 (como lo hicimos en la sección 2.1) y
2) la conjunción de pi, p 2, • • • , p. , p_i, paran ~ 2, cqmo

p,/\p,/\ ... /\p./\p• .,<c>(p,/\p,/\ . .. /\p.)/\p• .,.


[El resultado de (1) establece la base de la recursióo, mientras que la equivalencia lógica
de (2) se usa para proporcionar el proceso recursivo. Observe que la proposición que
aparece en el lado del'CCbo de la equivalencia lógica e n (2) es la conjunción de des propo-
síciones: P-+i y la proposición (p1 A P1 A·· - A p . ) determinada con anterioridad.]
Por lo tanto, definimos la conjunción de pi, P2, PJ, P• como

P• /\p,/\p,/\p,<c> ( p , /\p,/\p,) /\p,.


Entonces, por la propiedad asociativa de /\ , tenemos que

(p, /\ p, /\p,) /\ P• <e> [(p, /\p,) /\ p,J /\ P•


<c>(_p,/\p,)/\(p,/\p,)
<e> p, /\[p,/\ (p,/\p,)]
<e> P• /\[(p,/\p,) /\p,]
<e>P• /\ (p, /\p, /\p,).

Estas equivalencias lógicas muestran que el vaJor de verdad de la conjunción de cuatro


proposiciones también es independiente de la forma en que se intr<Xlucen los paréntesis
para indicar la forma de asociar las proposiciones dadas.
Con la definición anterior, extenderemos ahora nuestros resultados a la siguiente ••pro-
piedad asociativa generalizada para A".
Sean E z •,n 2: 3y rE Z.. con 1 :s r <n. Entonces,

S(n): para cualesquiera proposiciones P1oP1, . . . ,p,.pr+i, ... ,P,u


(p,/\p,/\ · .. /\p,)/\(p,.,/\ · .. /\p. )<c>p,/\p,/\ ·. · /\p,/\p,., /\ · . ·/\p•.
De mostración: La verdad de la proposición S(3) se sigue de la propiedad asociativa para
A; esto establece la base de la inducción de nuestra demostración. Para el paso inductivo,
supooemosqueS(k)es verdadera para algúnk 2: 3 y cualquier 1 :S r<k. Es decir, supone-
mos la verdad de

S(k): (p,/\p-,1\ · · ·/\p,)/\(p,.,/\ · · -1\pJ,;,,p,/\p-,/\ · · -1\p,/\p,.,/\- · -1\p,.

Entonces, mostramos que S(k) ~ S(k + 1). CUando consideramos k + 1 proposiciones.


entonces debemos tener en cuenta que 1 S r < k + 1.
1) Sir= k, entonces

(p,/\p-,1\ ·. -1\p,)/\p,., ,;,,p,/\p,/\·. -1\p,/\p,.,.

de nuestra definición recursiva


2) Para 1 .:s: r < k, tenemos

(p,/\p-,1\ · . -1\p,)/\(p,.,/\-. -1\p,/\p,.,)


,;,,(p,/\p-,1\ .. -1\p,)/\[(p,.,/\ .. -1\p,)/\p,.J
,;,,[(p,/\p-,1\- . . /\p,}/\(p,.,/\ .. -1\p,)]/\p,.,
<;!>(p,/\p-,1\·. -1\p,/\p,.,/\-. -1\pJ/\p,.,
,;,,p,/\p-,1\·. -1\p, /\j,,.,/\·. -1\p,/\p,.,.
Así, del principio de inducción matemática (Teorema 4.1), se sigue que la proposición
abierta S(n) es verdadera para tcxlo n E z• tal que n ~ 3.

Nuestro siguiente ejemplo ofrece una segunda oportunidad para generalizar una pnr
piedad asociativa, pero esta vez con conjuntos en vez de proposiciones.

En la definición 3.10 extendimos las operaciones binarias U en¡ un mimero arbitrario


(finito o infinito) de subconjuntos de un universo dado 0\1. Sin embargo, estas definiciones
no se basan en la naturaleza binaria de las operaciones implicadas y no proporcionan una
forma sistemática para determinar la unión e intersección de cualquier mlmero finito de
conjuntos.
Para evitar esta dificultad. consideramos los conjuntos A h A 2• ••• , A•• A_h donde A, ~
(\1. para tcxio 1 s i s n+1 y definimos su unión ~n forma ncursiva como sigue:
1) La unión de A1, A1 es A 1 U A2 • (Ésta es la base de nuestra definición recursiva.)
2) La unión de A1, A1 , ••• , A,., A,..1, paran ~ 2, está dada por

A, UA2 U • · · U A.UA.+1 = (A, UA,U · · • UAJ UA.+1,


donde el conjunto del lado derecho de esta igualdad entre conjuntos es la unión de dos
conjuntos, A1 U A2 U • • · U A. y A,.. 1• (Aquí tenemos el proceso recursivo necesario para
terminar nuestra definición recursiva.)
De esta definición obtenemos la siguiente "propiedad asociativa generalizada de U".
Sin. r E z", con n ,i?: 3 y l s r<n,entonces
S(n): (A, UA,U · · · UA,)U (A,.1 U··· UA.)
=A1UA2U···UA,UA,+1U · ··UA.,
donde A; ¡;; "1.1 para uxlo 1 :S i S n.
Demostración: La verdad de S(n) paran= 3 se sigue de la propiedad asociativa de U. con
lo que obtenemos la base necesaria para esta demostración inductiva Si suponemos la
verdad de S(k) para algún k E Z♦• tal que k 2: 3 y 1 :s r < k , establecemos ahora nuestro
paso inductivo, mostrando que S(k) => S(k + 1). Al trabajar con k + 1(2:4) coojuntos,
necesitamos considerar que 1 s r < k + 1. Tenemos que
1) Para r = k tenemos

(A1 UA2 U · · · UA,) UA.., =A, UA2 U · · · UA,UA,.,.

Esto se sigue de la definición recursiva dada


2) Si 1 s r < k, entonces

(A1 UA 2 U · · · UA,) U(Aff, U·· · U A,UA,.,)


= (A, UA1 U · · · UA,) U[(A,., U ··· UA,) UA 1 ,J
=[(A, UA,U · · · UA,) U(Aff1 U · ·· UA,)]UA,.,
= (A, UA,U · · · U A,UAff, U· · · U A,) UA,.,
=A1 U A 2 U · · · UA,UA,...1 U··· UA.tUA.t+i·

Así, se sigue del principio de inducción matemática que S(n) es verdadera para todos los
enteros n 2 3.

En fonna análoga al resultldo del ejemplo 4.15, la intersección de los n + 1 conjuntos A,.
A2 , ••• ,A..,,A_.1, (cada uno tomado del mismo universoGU)se define de manera recursiva como:

1) La intersección deA 1, A2 esA1 n A2 •


2) Para n 2:: 2, la intersección de A1, A2, • • • , A.. , .A_i, está dada por
A,nA,n •. • nA.n A., 1 = (A, nA,n • • • nA.) nA.,,,
la intersección de los dos conjuntosA 1 n A2 n · · · n A_. yA .. 1-

Vemos que las definiciones recursivas para la unión e intersección de cualquier nllmero
finito de conjuntos prop0rcionan el medio por el cual podemos extender las leyes de De
Morgan de la teoría de conjuntos. Mediante la inducción matemática, estableceremos una
de estas extensiones en el siguiente ejemplo y solicitaremos una demostración de la otra
extensión en los ejercicios de la sección.

■ 1111!fJ Sean E z +,n 2:: 2yA1, ~ , - • • , An ~G\.lparacada l :S i :S n .Entonces

A,nA,n --- nA. =A,UA,U ... UA,..


Demostración: La base de la inducción de esta demostración está dada para n = 2. Se
=
sigue del hecho de que A, nA, A1 UA,, porta segunda ley de DeMorgan (que aparece
en las leyes de la teoría de conjuntos de la sección 3.2).
Si suponemos que el resultado es verdadero para algún k, donde /c. 2:: 2, tenemos

A,nA,n-- - nA, = AtUA, U .. . UA,.


Y cuando consideramos k + 1( 2::3), usamos Ja hipótesis de inducción para obtener la kf'CO-
ra igualdad entre conjuntos en lo siguiente:

A, nA,n • • • nA, nA.., = (A, nA,n • • • n A,) n A..,


= (A,nA,n •. -nA,) u;i:;., = (Ai UA,U ... UA,,) u x ••,
=AiuJ;u ··· u Atu~♦1•
Esto establece entonces el paso inductivo en nuestra demostración, con lo que obtenemos
esta ley generalizada de De Morgan para todo n 2:: 2 por el principio de inducción matem-
tica.

Ahora que hemos visto las dos definiciones recursivas (en los ejemplos 4.14 y 4.15), al
continuar estudiando situaciones en las que aparezca este tipo de definición, generalmente
nos abstendremos de nombrar las partes de la base y la recursión. De la misma forma. no
siempre designaremos la base y los pasos inductivos en una demostración por induccióa
matemática
Si analizamos los ejemplos 4.14 y 4.15, las definiciones recursivas de ambos ejemplos
parecen similares, ya que si intercambiamos la proposición j,; con el conjuntoAn para todo
1 s i s n + 1 y si intercambiamos cada ocurrencia de A con U y reemplazarnos ,e=> con=.
entonces podemos obtener la-definición recursiva en el ejemplo 4.15 mediante la defini-
ción dada en el ejemplo 4 .14.
De manera similar, ~mos definir de manera recursiva la suma y producto de n ml-
meros reales, donde n E z• y n 2:: 2. Entonces, podemos obtener (por el principio de
inducción matemática) propiedades asociativas generalizadas para la suma y multiplica-
ción de números reales. (En los ejercicios de la sección, pediremos al lector que lo haga.)
Queremos estar al tanto de dichas propie.dades asociativas generalizadas ya que las hemos
estado utilizando y seguiremos utilizándolas. El lector podría sorprenderse de ver que ya
hemos utilizado la propiedad asociativa generalizada de la suma. Por ejemplo, en cada uoo
de los ejemplos 4.1 y 4.3 se utilizó la propiedad asociativa generalizada para establecer el
paso inductivo (en la demostración por inducción matemática). Además, ahora que nos
hemos percatado de ella de manera más cabal, podemos utilizar la propiedad asociativa
generalizada de Ja suma (por Jo general, de manera implícita) en las definiciones recursivas;
ya que ahora no habrá ambigüedad si desearnos sumar cuatro o más sumandos. Por ejem•
plo, podnarnos definir la sucesión de números annónicos Hi, H2, H 3, ••• , como

1) H,; !;y

2) Para n 2: 1, H • ., = H. + G~ J
Si pasamos de la suma a la multiplicación, podemos utilizar la propiedad asociativa genera-
lizada de la multiplicacíóo para dar una definición recursiva den!. En este caso, escribimos

1) O!; 1; y,
2) Pa,ra n 2: O. (n + 1)! ; (n + IXn!).

(Esto fue sugerido en el párrafo que sigue a la d~~nición 1.1 en la sección 1.2.)
Así mismo, la sucesión deenterosbo, bh b2, b,, ... , dada explícitamente (al comienzo de esta
seoción) por la fórmula h.= 2n, n EN, puede ser definida ahora en forma recuniva como

1) b,= O; y,
2) Paran 2: O, b~ 1 = b. + 2.
Cuando analicemos las sucesiones de nuestros dos siguientes ejemplos, encontraremos
otra vez definiciones recursivas. Además, estableceremos resultados en que se utilizará la
ley asociativa generalizada de la suma, aunque de manera implícita.

En la sección 4.1 presentamos la sucesión de números racionales llamados números armó-


nicos. Ahora presentamos una sucesión entera que es importante en combinatoria y teoría
de grafos (temas que estudiaremos más adelante en los capítulos 5, 10, 11 y 12). Los
nwneros de Fibonacci se definen en forma recursiva como

1) .ll,=O,F;=l; y
2) F.= F..-, + F.-i, para n E z• con n 2: 2.
Por lo tanto, de la parte recursiva de esta definición. se sigue que

.l'; = F;+F,=1+0=1
Fj=.l';+}j=l+1=2
F,=F;+.1';=2+1=3
F,= F.+ F;=3 + 2 = 5.

También vemos que F, = 8, F1 ;:::; 13, F, = 21, F9 =-34, F10 = 55, F 11 =- 89 y F 12 = 144.
La definición recursiva de los números de Fibonacci se puede utilizar (junto con el
principio de inducción matemática) para establecer muchas de las propiedades interesan•
tes que exhiben estos nWDCTos. Analizaremos ahora dos de estas propiedades.

a) Consideremos los siguientes seis resultados que tratan de la suma de cuadrados de


los números de Fibonacci.
1) Fi+Fl=O'+l'=l=lxl
2) Fi + Fl+Fl=O'+l2 +1'=2=1X2
3) ~+Fl+Fl+Fl=0'+1 2 +12 +22 =6=2X3
4) ~+ F1+ Fl+ Fl+ F¡=O'+ 12 + 12 +22 +3'= 15 =3 x 5
5) Fi+ F/+ Fl + Fl+ F¡ + F;=O'+ 12 + 1' +2'+ 32 + S'= 40= 5 x 8
6) F~+ Ff + Fi+ Fi+ F¡+ Ff+ Fi
=O'+ 12 + 12 + 22 + 32 + S' + 82 = 104 = 8 X 13.
Según estos cálculos, conjeturamos que

Vn E z• ¿, F: = F. X F.♦1-
~

Demostración: Paran= l. el resultado de la ecuación (l) nos demuestra que la conjetura


es verdadera en este primer caso.
Si suponemos que la conjetura es verdadera para algún k ~ 1. se obtiene por la hipóte-
sis de inducción:
.
¿,F} = F, Xfi.1.
' i•l
Si pasamos ahora al caso en que n = k + 1(~2), tenemos que
t+l .t
¿ Fl = ¿;-o Ft + p¡+l = (f. X Fa:+1) + Ff+l = F.t+l X (ri + Fi+1) = .fi:+1 X .fi:♦2-
;-o

Por lo tanto, la verdad del caso n = k + I se sigue del caso n = k. Asf. la conjetura dada es
verdadera para todo n E z.-, por el principio de inducción matemática. (Tal vez el lectoc
observe que el cálculo anterior utiliza la ley asociativa generalizada para la suma.Además.
empleamos la definición recursiva de los números de Fibonacci, que nos permíte reempla-
zar F1 + F,...1 por Fl,+l.)

b) Una segunda propiedad de los números de Fibonacci establece que F,. s (5/3)" para
tcxlo n EN. Para establecer esta desigualdad, utilizamos la forma alternativa del
principio de inducción matemática

Demostración: Paran= O tenemos que F0 = O S 1 = (5/3)º. y paran= I tenemos F 1 = 1 :S


(5/3) • (5/3)1 • En consecuencia., la propiedad dada es verdadera en estos dos primeros
casos (y esto proporciona la base de inducción para la demostración).
Si suponemos que esta propiedad es verdadera paran = O, 1, 2, ... , k - 1, k, donde k :!:. I,
examinamos ahora lo que sucede en n • k + 1. Tenemos que•

fiH = f¡ + fi-1 S (513)' + (513)t-l = (513)-t-1((513) + 1] = (513)'- 1(&/3)


= (513)'- 1(2419) s (513)'-1(2519) = (513)>-1(513)2 = (513)..1 .

Se sigue e ntonces de la foi:;ma alternativa del principio de inducción matemática, que F,. S
(5/3)" para todo n E N.

La sucesión de los númuos de Lucas está estrechamente relacionada con los m1meros de
Fibonacci. Esta sucesión se define en forma recursiva como

1) I.o=2, L,=1; y,
2) L.= L,.-1 + L.-2. para t~o n E z+ con n ~ 2.

Los primeros 12 námeros de Lucas aparee.en en la tabla 4.2.

Tabla 4.2

L. L. • L. • L.
o 2 4 6 18 9 76
1 1 7 7 29 10 123
2 3 11 8 47 11199

Aunque no son tan famosos como los de Fibonacci, los números de Lucas tambi~
poseen muchas propiedades interesantes, de las que ahora presentaremos dos.

a) Para nEN, Lo+ L1 + L,+ ···+ L.=± L,=L.,, - 1.


i•O
Demostración: Si n = O tenemos que

l.,.= ±L,=2= 3-1 = L,_-1 = Lo.,-1,


i-0
por lo que la afirmación es verdadera en este primer caso.
Para algtln k E N, donde k 2' O, suponemos que

I.,.+L,+L,.+ ... +L, =LL, =L..,-1.
;- o

es verdadera. Entonces, para n ;;;; k + 1( 2:: l) tenemos


1 +1 l
(º) L L, = L L¡ + L,., = (L,.,- !) + L.., = (L,., + L,.,)- 1
1-0 i-0

= L1♦3 - 1 = l.cJ:+1)+2 - l,

por lo que el hecho de que la proposición sea verdadera para n = k implica lo mismo para
k + 1. Por lo tanto, la fórmula para la suma es válida para todo n E N por el principio de
inducción matemática [Observe que para las ecuaciones(*), la primera igualdad se sigue
de la propiedad asociativa generalizada de la suma, y la-cuarta igualdad se basa en la
definición recursiva dada de los números de Lucas puesto que k + 3 ~ 3(~2).]

b) Una de las relaciones entre los números de Fibonacci y los m1meros de Lucas es

Demostración: Aquí necesitamos considerar lo que ocurre cuando n = 1 y n = 2. Tenemos


que
L 1 =!=0+1 = .li,+l'j=Fj_1 + Fj.i, y
L,_ = 3 = 1 + 2 = Fj + Fj = ¡;;_, + l'j.¡,

por lo que el resultado es verdadero en estos dos primeros casos.


A continuación, suponemos que L.= F,._ 1 + F.,.1 para los enteros n = 1, 2, 3, . .. , k- 1,
k, donde k :::: 2 y después consideramos el número de Lucas 4..1• Tenemos entonces que

(")' L,., = L, + L,-, = (F,_, + f;.,) + (fi-, + l'j)


= (li- 1 + .Fi-2) + (F1+1 + fi) = J\ + li+2 = Fc1:♦1)-1 + .fi.t+J)+I·
Por lo tanto, se sigue de la forma alternativa del principio de inducción matemática que L.=
F,._ 1 + F,..,1 para todo n Ez·. [El lector debe observar cómo utilizamos las definiciones
recursivas de los números de Lucas y de Fibonacci en los cálculos de la parte (*)'. ]

Para cerrar esta sección, presentaremos la idea de un conjunto X definido en forma


recursiva. Aquí panimos de una colección inicial de elementos que están en X, lo que
proporciona ta base para la recursión. Después proporcionamos una regla o lista de reglas
que nos indican cómo encontrar nuevos elementos de X a partir de otros elementos que ya
• sabemos están en X. Esta regla (o lista de reglas) constituye el proceso recursivo. Pero
ahora (y esta parte es nueva) tenemos también una restricción implícita; es decir, una
no Capítulo 4 Propiedades de los enteros: Inducción matemática

proposición en el sentido de que níngún eleme nto puede estar en X, excepto los dados ea
la colección ínicial o los que se formaron mediante las reglas dadas en el proceso recursil'll.
Demostraremos estas ideas en el siguiente ejemplo.

fj]IJfd Definimos el conjunto X en fonna recursiva como

1) 1 EX;y
2) Para cualquier a E X, a + 2 E X

Entonces afirmamos que X consta (precisamente) de todos los enteros impares positivos.
Demostración: Si Y denota el conjunto de tcxl.os los enteros impares positivos (es decir, Y=
In
{2n + 1 E N},entoncesqueremosdemostrarque Y= X.Estosignifica. comoyaaprea-
dimos en la sección 3.1, que debemos verificar Y ~ Xy X~ Y.
Para establecer que Y~ X, debemos demostrar que todo entero impar positivo está ca
X. Reallz.arcmos esto mediante el principio de inducción matemática. Comencemos pi'
considerar la proposición abierta

S(n): 2n + lEX,

que está definida para el universo N. La base de la inducción (es decir, S(O)] es verdadera,
ya que l = 2(0) + l E X por la parte (1) de la definición recursiva de X. Para el paso
inductivo, suponemos la verdad de S(k) para algún k ?;; O; esto indica que 2k + l es 111
elemento de X. Si 2k + 1 E X, se sigue de la parte (2) de la definición recursiva de X que
= =
( 2k + 1) + 2 (2k + 2)+ i 2(k+ 1) + 1 E X, porloqucS(k + 1) tambifn es verdadcra.PCI'
lo tanto, S(n) es verdadera para uxlo n E N (por el principio de inducción matemática) y
tenemos Y~ X.
Para la demostración de la inclusión opuesta (a saber, X~ ]"),--usamos la definición
recursi va de X. Primero consideramos la pane (l)de la definicíón. Como l (:;;: 2 . O+ l)es
un entero impar positivo, tenemos que 1 E Y. Para terminar la demostración, debemos
verificar que cualquier entero que esté en X y que resulte de la parte (2) de la definicióa
recursiva también esté en Y. Esto se hace mostrando que a + 2 E Y siempre que el elemento
a de X sea también un elementQ de Y. Ya que si a E Y, entonces a = 2r + 1, donde r E N (pcr
la definición de un entero impar positivo). Así, a + 2 = (2r + 1) + 2 = (2r + 2) + 1 =
2(r + 1) + 1, donde r + 1 E N (en realidad, Z•) y por lo tanto a + 2 es un entero impar
positivo. Esto coloca a a + 2 en Y y muestra que X ~ Y.
De las dos inclusiones anteriores (es decir, Y~ X y X~ Y), se sigue que X= Y.

JERCICIOS 4.2 1. La sucesión de.enteros a 1, a 2, a 3 , •••• definida explícitamente por la fórmula a,.= Sn paran E
z• también puede definirse en forma JttUrsiva como
!) a, =5; '1'
2) a...1=a.+S, paran~l.

Para la sucesión de enteros b1• bt, b3, •••• donde b,. = n(n. + 2) para todo n E z., también
podemos dar la siguiente definición recursiva:
4.2 Definiciones recursivas 211

1)' b, • 3; and,
2)' b•• ,•b. +2n +3, for n-=1.
™ una defmkión rccun.iva para cada una de las siguientes sucesiones de enteros c1, ci. e,•. .
donde pan. cada n E z·
tenemos
a) c.•1n b) c,.=T
<) c. = 3n+7 d) c.=lln-8
•> c.• 1 f) c.. =-n2
el c.. • (n+l)(n+2) b) c.=2 - (- 1)"
2. a ) Dtunadefinición recursiva paraladisyuncióo dcproposiciooespi,J>:, .. . •PnP..,- n Z: 1.
b) Muestre que sin. ,ez•,n ;!:: 3y 1 s r <n.entonces

(p, V P, V ·. ·Vp,)V(p,.,v- . ·Vp.)<:>p,VP,V·. ·Vp,V p,., v -. ·Vp•.

3. a) Use el resultado del ejemplo 4. J4 para demostcar que si p. q,. q1, • ••• q. son proposiciones
y n ~ 2, eotooces

PV (q,l\q,1\- · -l\q.)<:>(p y q,)l\(pVq,)I\· · ·/\(pVq.}


b) Paran E z·. n ~ 2. ylas proposicionesp, q1, ql> . ... q~. demuestre que
pl\(q, vq,v · · ·vq.J<:>(p /\q,) v(p/\q,)V· · · v(pl\q.}
4. Paran E z•. n 2:. 2, demuestre que para c:ualesquieraproposiciooes p 1,Pz. ... , p.,
•> (p, VP,V · · ·vp.)<:>p,l\pil\· · -1\1:,.
b) (p,f\p,f\- · ·l\p.)<:>p,VP:,V-- ·Vp,.,
5 . a) ™u.na definición recursiva para la intersección de los conjuntosA1oA1, ••• • A •• A..1 C: "1.
n Z: l.
b) Use el resultado de la parte(&) para mosuw~ pararualC$QWfflln,. rE z♦, n ~ 3 y 1 S r< n.
(A, nA, n ... n A,)n(A~, n •·· nA. ) • A, nA, n ---nA,nA-,n---nA..

6. Paran Z: 2 y cualesquiera conjuntosA 1,A2, ••• • A. ~t\t. demuestre que


A1UA2U ... UA.•Ji";n;¡;n. • -n::t:.
7. a) Use el rw.tltadodeJ ejemplo4.1S para mosuarqucsi los conjuntos 8 1, 8 2, ••• • B. C:Gll.y n
~ 2. entonces

A n (B,UB,u---u B.) • (A n B,)U(A n B,)U--·U(A n s.).

b) Pan• e z·. • a, 2 y los coojuntos A. B., B, .. ... B. !: "'- demuesae que


A U(B,n B,n• ••nB.)•(A U B,)n(A UB,)n•. -n(A u B.}

8 . a ) Desarrolle una definición recursiva para la suma den mlmeros reales .:e,. x:, ... . .:c•• donde
n Z: 2.
b) Para c:ualesquicra números reales Xi, x 2 y x,. la propiedad asociativa de la suma establece
quei, + <.r2+.rJ =<r1 +x,J + x,. Demuestre que sin. TE z·. n Z: 3 y I s r< n. entonces
(z1 + z2 + · · · + x,) + (z,., + · · · + x.) • .r1+ z2 + · · · + x, + x,..1+ · · · + x•.
9 . a) Desarrolle una definición recursiva para la multiplicación de n números reales x 1, .x11 ••••
x_.doode ni!:: 2.
b) Para cualesquiera ndmc:ros reales i h x 2 y x,. la propiedad asociativa de la multiplicación
estab)cce quc..1:iC.r1..1:J • (.r1..1:z).r_,. Demuestre que sin. Er z·.n
~ 3 y l Sr< n. entonces

(x,xz· • ·x,)(x,... , · · ·.x,.)=x1X2· · ·.r,.r,.1 · • •x.,.


43 El a oritmo de la división: Números primos 213

proposicióo compuesta ((p /1 ( ~q)) ➔ (,V T.)) es una fórmula bien formada. Podemos
obteoc:r esta fórmula bien formada como sigue:

Pasos Raz.o,ws
l) p. q,r, To Parte (1) de la defimcióo
2) (,q) Paso (1) y parte (2i) de la de6nicióo
3) (p/\(,q)) Pasos (1), (2) y parto (2iii) de la definición
4J c,vr.> Paso (1) y parto (2ii) de la definicióo
5l ((p/\(,q))➔(,vToll Pasos (3), (4) y parte (2iv) de la definición

Para las proposiciones primitivas p, q, r y 1, desarrolle las derivaciones que muestran que
eada una es una fórmula bien formada.
•l ((pvq) ➔ (To/\(,,)))
b) (((,p) .,. q)➔ (r/\(sV/i,)))
e) ((p-n)/\((pVq)➔s))

22. a) Pann :ae 2, sip,,p,,p,. .. .• p•• p_, sonpropos;ciooes,demucsaequc[(p, ➔ p,) /\ (p,➔ p,)
/1 .. . /1 (p. ➔ p-,)1 => [(p, /1 p,/1,,, /1 ... /1 pJ ➔ p-,J.
b) Demuesttequcd "'°'""'"4.2implicad IOO!ffllll4.I.
e) UJeel teorema4.1 paraestablectr lo siguiente: si t-/,. S C z• yn E S. para aJgúnn e z•.
entonces S contiene un elemento· mínimo.
d) Muestre que el teorema 4.1 implica el leorema 4.2.

4.3
El algoritmo de la división:
Números primos

Aunque el conjunto Z no es cerrado en la división entre nl1meros distintos de cero, existen


muchos casos en los que un entero divide (exactamente) a otro. Por ejemplo, 2 divide a 6,
y 7 divide a 21. En este caso, la divisióo es exacta y no hay resu>. Así, el hecho de que 2
divida a 6 implica la existencia de un cociente (3) tal que 6 = 2 • 3. Formalizaremos esta
idea como sigue.

Dfflnición 4.1 Si a,. b E Z y b #:- O, decimos que b divuk a a, y lo denotamos b Ia, si existe un entero n tal
que a = bn. Cuando esto OCWTe, decimos que b es un divisor de a, o que a es un ,últiplo
deb.

Con esta definición, podemos hablar de la división dentro de Z sin pasar a Q. Ademís,
cuando ab =Opara a. b E Z, entonces a =Oo b =O; decimos entonces que Z no tiene
divisoru propios de O. Esta propiedad nos permite can.celar, como ea el caso 2x:: 2y~ x.=
ypara.r, y E Z , ya que 2x= 2y => 2(.r-y)= O=> 2 = O ox-y =O =>x= y. (Observe que
en ningún momento mencionamos la multiplicación de ambos lados de la ecuación 2t =
2y por ½• El número ½está fuera del sistema Z.)
Ahora m.wniremos algunas propiedades de esta operación de división. Cuando divida-
mos entre un entero a, supondremos que a O. +
14 Capítulo 4 Propiedades de los e nteros: Inducción matemá tica

c0REMA 4.3 Para cualesquiera a. b, e E Z

a) 11a y • 10. b) [(a lb)/\(b la)]~ a =±b.


e) [(a jb)/\(b lc)J ~ •ic- d) alb~al bx para todo x E Z.
e) Si x =y+ z, para x, y, z E Z y a divide a dos de los enterosx, y, z, entonces adi ·
al entero restante.
f) [(a I b) A (a Ic)J => a 1(bx + cy), para todos .r; y E Z. (La expresión bx + ey
denomina combinación lineal de b, c.)
g) Para 1 s i s n, sea e, E Z. Si a divide a cada C;,
entonces a 1(c1 x 1 + CiXz + • • • + c. x,.), donde x;E Z para todo 1 s i s n.
Demostra ció n: Demostraremos la parte (f) y dejaremos el resto al lector.
I I =
Si a by a e, entonces b am y e= an, para algunos m, n E Z . Así, bx + cy (am)I+ =
(an)y :: a(rm + ny) (por la propiedad asociativa de la multiplicación y la propiedal
distributiva de la multiplicación sobre la suma, ya que los elementos de Z satisfacen•
bas propiedades). Como bx + cy = a(mx + ny), se sigue que a 1(bx + cy).

11:
La parte (g) del teorema será lltil cuando consideremos.Jo siguiente.

¿Existen enterosx, y, z(positivos, negativos o cero) tales que 6x+ 9y+ 15z= 1<J7'! Su:pot-
gamos que existen dichos enteros. Entonces, como 3 l 6, 3 j 9 y 3 l 15, la parte (g) del,.._
rema 4.3 implica que 3 es un divisor de 107, lo cual es falso. Por lo tanto, no existen tales
enteros x. y, z.

Las partes (d), (e) y (f) del teorema 4.3 nos ayudarán en la siguiente situación.

Sean a, b E Z ta1es que 2a + 3bsea un múltiplo de 17. (Por ejemplo, podñamos tener que
b =- 1; o bien, a = 4, b = 3.) Demuestre que 17 divide a 9a + 5b.
• a=- 1 y

Demostración: Observarnos que 17 J (2a + 3b) => 171 (-4X2a + 3b).Además, 17 1(17a+
17b). Por lo tanto, 171 (17a + 17b) + ( - 4X2a + 3b). En coosecueoeia, 17 1[(17 - 8)a +
(17 - 12)b], o bien 17 l(9a + Sb). ·

Con esta operación binaria de división entre enteros, estamos de ntro del área de las
matemáticas llamada teoría de níimLro.s. Si seguimos analizando el conjunto z•, observa-
remos que para todon E Z+, n >l, el enteron tiene al menos dos divisores positivos; 1 yel
mismo n. Algunos números, como 2, 3, 5, 7, 11, 13, 17, . .. , tienen e xactamente dos
divisores positivos. Estos ente ros reciben e l nombre de primos. Todos los demás enteros
positivos (mayores que 1 y que no sean primos) se Uaman compuestos. El lema siguiente
expresa una conexión inmediata entre los níimeros primos y los compuestos.
4.3 El a oritmo de la dMsi6n: Núrnef'OS primos 215

4.1 Sin E z• y n es compuesto, entonces existe un primo p tal que p In.


Demostración: Si suponemos lo contrario, sea Sel conjunto de todos los enteros com~
puestos que no tienen divisores primos. Si S #-8, entonces por el principio del buen orden,
S tiene un elemento mínimo m. Pero si m es compuesto, entonces m = m11nz, donde mh mz
E z• y I< m1< m y l< m.,< m. Como m 1 ~ S, m 1 es primo o divisible entre un primo. En
I
consecuencia, existe UD primo p tal que p m y S ; ff.

¿Por qu6 llamamos al resultado anterior lema en vez de teorema? Después de todo,
debía demostrarse al igual que todos los teoremas de este libro vistos basta el momento. La
razón es que aunque un lema es en sí mismo un teorema. su papel principal es el de ayudar
a la demostración de otros teoremas.
Al enumerar los primos, nos inclinamos por pensar que existe una infinidad de tales
nWDCros. Verificaremos ahora que esto es cieno.

4 .4 (Euclides) Existe una infinidad de primos.


Demostración: En caso contrario, seap1,J>2, ••. ,Pt. la lista finita de todos los primos y sea
B• p 1p,.· • •pt.+ l. ComoB > p¡para todo 1 s; s k, B noj:,uede ser un primo. Por lo tanto,
Bes compuesto. Así, por el lcma4.l, cxistc un primopfa 1 Sj s ktal quep¡ ¡s.Como
P; 1By p¡ 1 p 1p,· • • Pn del teorema 4.3(c) scsigucquep; I '- Esta contradicción su,gc dela
hipótesis de que sólo existe un número finito de primos, de donde obtenemos el resultado.

Sí, éste es el mismo Euclides del siglo rv A.c., cuyos E/ementoJ, escritos en 13 rollos de
pergamino, constituyeron el primer análisis organizado de la geometría que estudiamos en
el bachillerato. Sin embargo, uno puede ver que estos 13 libros tambitn abordan la teoría
de m1meros. En particular, los libros Vll, VID y IX tratan estos temas. El teorema anterior
(con demostración) aparece en el libro IX.

Pasemos ahora a la idea principal de esta sección. Este resu1tado nos permite trabajar
con la división entre números distintos de cero en Z cuando esa división no es exacta.

TEOREMA4.5 El algoritmo de la división. Si a, b E z. con b > O, entonces existen q, r E Z únicos tales


que a= qb + r, con Os r<b.

I
Demostración: Si b a, el resultado es válido con r = O, por lo que ahora consideraremos
el caso en que b~a (es decir, b no divide a a).
ScaS;{a-rblr E Z.a -rb>O}. Sia> Oy t;O, entonces a E Sy s,i,e. SiaSO, sea
r; a - l. Entonces a-tb; a-(a - l)b ;a(! - b) + b, con (1 -b) s O, ya que b a: l.Así,
a- tb > O yS t- e. Por lo tanto, para todo a E Z, Ses un subconjunto no vacío de z•. Por
el principio del buen ordcn,Stiencunelemento mfnimor, talque O< r=a-qb, paraaJgúD
q E Z . Sir; b, entonces a; (q + l)b y bla. lo que contradiccbfa. Sir> b, entonces r;
b + e, para algún e E z• y a-qb= r = b+ e ⇒ e =a-(q + l)b E S, 1o que contradice que
r sea el elemento mínimo de S. Por lo 13.DIO, r < b.
Esto establece entonces un cociente q y un resto r, tales que O s r < b para el teorema.
Pero ¿existen otros q y rque también funcionen? En ese caso, sean q 1, q1, rl' r1 E Z tales
216 Capítulo 4 Propiedades de los enteros: Inducción matemát ica

que a =q1b + r 1, con O :s: r 1<b ya= qi + r 2• con O~ r 2 < b. Entoncesq1b + r 1 =q,P+ r2 ~
b Iq 1 - % 1= 1r 2 - r1 1< b. ya que O S r 1, r 2 < b. Si q 1 1- q 2, tenemos la contradicci6t
b Iq 1 - q, 1< b. Porto tanto, q 1 = q2 , r1 = r2 y tanto el cociente como el resto son únicos.

Como Jo mencionamos en la de mostración anterior, cuando a. b E Z con b > O, entoocu


existe un cociente q único y un resUJ rúnico tales que a= qb + r, con O s r < b. Ademá,
en este caso, el entero b se llama divisor y a es el dividen.do.

a) Cuando a; 110 y b; 11 en el algoritmo de la división, tenemos que 170; 15 • 11 +


5, donde Os 5 < 11. Por Jo tanto, al dividir 170entte 11, el cociente es 15 y el~
es 5.
b) Si el dividendo es 98 y el divisores 7, del aJgoritmo de la división tenemos que 98:
14 • 7. Así, en este caso, el cociente es 14 y el resto es O, y 7 divide (exactamente)a
98.
e ) Para el caso en que a; -45 y b; 8 tenemos -45; ( - 6)8 + 3, donde O :s 3 < 8. Ea
consecuencia. el cociente es - 6 y e l resto es 3. cuando el dividendo es -45 y d
divisor es 8.
d) Sean a, b E z•.
1) Si a; qb para algún q E Z+. entonces --a =e ( - q)b. Por lo que, en este caso.
cuando --a(<O) se divide entre b(>O) el cociente es -q(<0) y el resto es O.
2) Sía ;qb + rpara algúnq EN yO< r<b, entonces-a; (-q) b - r; (-q)b -
b + b - r; ( -q- 1) b + (b - r). Paraesteeaso,cuando-a(<O) se divideentte
b(>O), el cociente es -q - l(<O) y el resto es b - r, donde O< b - r < b.

A pesar de la demostración del teorema 4.5 y los resultados del ejemplo 4.22, realmente
11() tenemos una manera sistemática de calcular el cociente q y el resto r cuando dividimos
un entero a (el dividendo) entre el entero positivo b (el divisor). La demostración del
teorema 4.5 garantiza la existencia de tales enteros q y r, pero la demostración n1J es com-
tructiva, esto es, no parece indicar cómo se calculan q y r ni menciona nada acerca de la
capacidad para utilizar tablas de multiplicación o para realizar grandes divisiones.A fin de
remediar esta situación proporcionamos e l programa en Pascal de la figura 4.8. [En este
programa. abs(a) es el valor absoluto de a.] Nuestro siguiente ejemplo ilustra la idea pre-
sentada en parte de este programa.

Dado que es posible ver la multiplicación de enteros positivos como sumas repetidas,
también podemos ver la división (entera) como restas repetidas. Vemos que la resta sí
cumple un papel en la definición del conjunto Sen la demostración del teorema 4.5.
Cuando, por ejemplo, calculamos 5 • 7. podemos pensar en términos de sumas repeti-
das y escribir
Program IntegerOivi:sion ( input,output ) :
Var
a,b,q , r: integer :
Begin
Wri teln ('Queremos determinar el cociente q i") ;
Wr i teln ('y el r esto r cuando el entero a' ) ;
Writeln ( 'se divide entre el entero positivo b. ' );
Wri t e ( 'a= ' ) ;
Read. ( a ) ;
'ilrite ( 'b = ");
Read (b );
Ita=Othen
Writeln( 'En este caso, a= O, por lo que q = O y r = o .')
Else
Begin
r: = abs(a);
q : = O;
While ( r >= b) do
Begin
r := r-b;
q := q+ l
End;
It a>O then
Begin
Writeln(''Cua:ndodividimos'. a:0, 'entre ' , b:O, ' , ' ) ;
l'riteln (' ·el cociente q::. 1 • q:0, ' y ' ) ;
Writeln ( ' ·el resto r= ', r : O. '.')
End
Else i t r = O tben
Begin
Writeln ( 'cuando dividimos'. a:O, ' entre 1 • b:O, ' . ' ) ;
Writeln ( •·el cocient.e q= ', -q:O, ' y ' ) ;
Writeln ( 1 ·e1 resto r = O. ' )
End
Else
Begin
Writeln ( 'Cuandodividimos'. a:O , 'entre,',');
Writeln (' el cocient.e q= '. (-q- 1 ) :O, ' Y' ' ););
Writeln (' ·el resto r • 1 , (b-r ):O, '. ' };
End;
End;
End.

Queremos de t erminar el co·ciente q


y el rest.o r cuando el entero a
se divide entre el entero positivo b.
•=37
b=S
Cuando dividimos 37 entre 8.
el cociente. q = 4 y
el resto r=S.

figura 4.8
217
!18 Capítulo 4 Propiedades de los enteros: Inducción matemática

2·7•7+7=14
3 ·7-• (2+ l ) ·7• 2·7+ 1 -7 = (7 + 7) +7 = 14 +7 = 21
4·7 • (3 + 1)-7•3·7+ 1-7= ((7 + 7) + 7) + 7 = 21 + 7 = 28
5-7 = (4+ 1) -7 &4•7 + 1-7 = (((7 + 7) +7) +7) +7 =28+7 = 35.
S~ por otro lado, deseamos dividir 37 entre 8, entonces debemos pensar que el coci-f
es el número de ochos contenidos en 37. Al eliminar cada uno de estos ochos (esto es. al
restarlos) y cuando ya no se puede eliminar otro 8 sin obtener un resultado negativo,
entonces el entero que queda (restante) es el resto r. As(, podemos calcular q y r en tmli-
nos de restas sucesivas como si¡ue:

37-8&2928,
29-8 = (37-8) - 8= 37 - 2 • 8z 21 28,
21-8= ((37-8)-8) - 8= 37-3 •8 & 1328,
13-8= (((37 - 8) - 8)-8)-8 = 37-4·8 = 5<8.
La última linea muestra que podemos restar cuatro ochos de 37 antes de obtener UD resul-
tado DO negativo (S)que es menor que 8. Por lo tanto, en este ejemplo tenemos queq=4
y r=5.

Usaremos el algoritmo de la cfjvisi6n para analizar algunos resultados relativos a b.


representación de enteros en bases distintas de 1O.

Escribimos 6137 en el siste ma octal (base 8). Aquf bu.scamos enteros no negativos ro.,.,
r1, .• . • r•• con r*> O, tales que 6137 = (r•• .. .• rL r" rJ,.
Con 6137 = ro+ r1· 8 +r1 • 8 1+ • · • +ra· 8ª= ro+ 8(r1+ r1 · 8 + · • · + r•· 8*- 1). r 0 ese1 rcs10
obtenido en el algoritmo de la división al dividir 6137 entre 8.
En consecuencia, ya que 6137 = 1 + 8-767, tenemos r0 = 1 y 767 "= r1 + r1 • 8 + • • • +
ra· gt- 1 = r 1 + 8( r1 + r,· 8 + • • •+ ra· s•-1). Esto implica quer1= 7 (e l resto de la división de
767 entre 8) y 95 = r1 + r,• 8 + • • •+ r,• s•- 1 • Si continuamos de esta manen veremos que
r1 = 7, r 3 = 3, r,= 1 y r ;= Opara todo i ~ S, por lo que

6137 = l ·8' + 3·8'+7 · 8'+ 7-8+ l • (13771),.


Podemos ordenar las cfjvisiones sucesivas entre 8 como s igue:

Restos
8~
8 l1fil 1 (ro)
8~ 7(r1)
8 l!1 7 (r,)
8 I! 3 (r,)

-
O 1 (r,)

En el campo de las ciencias de la oomputacióo, el sistema de nllmeros binarios (base 2) es muy


importante. En ti, los únicos símbolos que se pueden utilizar son los bits Oy 1. En la tabla 4.3
liemos enumerado la represeniación binaria de los enteros (base 10) desde el O hasta el IS.
4.3 El algoritmo de la división: Números primos 219

Tabla 4 3
llas<IO llas< IO
- 2 - 2
o 0000 8 1000
1 0001 9 1 OO 1
2 0010 !O 1 O1 O
3 0011 11 1 O1 1
4 0100 12 1 1 OO
5 0101 13 11 O 1
6 0110 14 11 1 O
7 O 1 11 15 1111

Hemos incluido los ceros no significativos y encontramos que se necesitan cuatro bits
debido al primer 1 de las representaciones de los enteros de 8 a 15. Podemos continuar con
cinco bits hasta el 31 (= 32 - l = 2~- l); se necesitan seis bits para llegar al 63 (= 64 - 1 =
26 - 1). En general, six E Z y O ::s: x<2•, paran E z•, entonces podemos escribirxen base
2 usando n bits. Los ceros no significativos aparecen cuando O ::s: x .::s: 2• · 1 - 1 y para
2• - 1 :s. x :s. 2• - 1 el primer bit (más significativo) es el 1.
Generalmente. la información de las máquinas se almacena en unidades de ocho bits
llamadas bytes; así, para las máquinas con celdas de memoria de un byte, podemos al-
macenar en una sola celda cualquiera de los equivalentes binarios de los enteros desde O
a 2*- 1 =255. Para una máquina con celdas de dos bytes, cualquiera de los enteros entre
O y 2 16 - 1 = 65.535 puede almacenarse en forma binaria en cada celda. Una máquina con
celdas de cuatro byte s puede proporcionamos basta 2"-1 = 4,294,967,295.
Cuando un humano trabaja con secuencias grandes de ceros y unos, el trabajo se vuelve
muy tedioso y la probabilidad de error se incrementa con el tedio. En consecuencia. es
común (especial.mente en el estudio de lenguajes de máquina y ensamblador) representar
esas secuencias grandes de bits en otra notación. Una de estas notaciones es la notación
he.xaduimal (base 16). Aquí necesitamos 16 símbolos; como sólo tenemos 10 símbolos
en e l sistema estándar en base 1O, introducimos los siguientes símbolos adicionales:

A (Able) C (Charlie) E (Echo)


B (Baker) D (Dog) F (Fox)
En la tabla 4.4, los enteros entre O y 15 están dados en ttnninos del sistema binario y del
sistema hexadecimal.

Ta bla 4.4

llas< IO llas<l6 llas< IO llas<l6


- 2 - 2
o 0000 o 8 1000 8
1 0001 1 9 1001 9
2 0010 2 10 1010 A
3 0011 3 11 1 O 11 B
4 0100 4 12 11 O O e
5 O 1 O1 5 13 1 1 O1 D
6 O1 1 O 6 14 1 11 O E
7 O1 1 1 7 15 1111 F
10 Capítulo 4 Propiedades de los enteros: Inducción matemática

Para convertir de base 10 a base 16, seguimos un procedimiento parecido al bosqueja-


do en el ejemplo 4.24. Aquí estamos interesados en los restos de las divisiones suces.i'va
entre 16. Por Jo tanto, si queremos representar el entero l 3,874.945 (base 10) en el sisu:a
hexadecimaJ, hacemos los siguientes cálculos:
Restos
16 113,874,945
16 (867,184 1 (ro)
16 (54,199 o (r,)
16 13,387 7 (r,)
16 l2ll 11 (=B) (r,)
16 1U 3 (r,)
o 13 (=D) (r,)
En consecuencia, 13,874,945 = (03B701)16•
Sin embargo, existe un método más sencillo para convertir de base 2 a base 16. Pcr
ejemplo, si queremos convertir el entero binario 01001101 (un byte) a base 16, descompo-
nemos el número en bloques de cuatro bits:
0100 1101
4 D
Despué s convertimos cada bloque de cuatro bits a su representación en base 16 (como se
muestra en la tabla 4.4) y tenemos que (01001101),= (4D)16• Si partimos del ntlmao
(A13f16 (de dos bytes) y queremos hacer la conversión en la otra dirección, reemplaza.
mos cada símbolo hexadecimal por su equivalente binario ( de cuatro bits, que también se
muestra en la tabla 4 .4):
A 3 F
1010 (XX}! 0011 1111
Esto implica que (A13F),. = (1010000100111111),.

Ejemplo 4.26 Necesitamos Jos enteros negativos para realizar Ja operación binaria de resta en términos
de la suma (es decir, (a-b) =a+ ( - b)]. Cuando trabajamos con la representación binaria
de los e nteros. podemos utilizar un método popular que nos permite realizar la suma. resta.
multiplicación y división (entera): el mttodo de complemento a dos. La popularidad del
método se basa en su implantación con sólo dos circuitos electrónicos, uno para negar y
otro para sumar.
En Ja tabla 4.5, los enteros del --8 al 7 se representan mediante el patrón de cuatro bits que
se muestra. Los enteros no negativos se representan como e n las tablas 4.3 y 4.4. Para
obtener los resultados para -8 S n s -1, consideremos primero la representación binaria
In
de j, el valor absoluto de n. Entonces hacemos lo siguie nte :
1) Reemplazamos cada 0(1) en la represen.ración binaria de !ni por 1(0); este resulta-
do se llama complemento a urws de Oa representación dada de) In!.
2) Sumamos 1 (= (XX}l en este caso) al resultado del paso l. Este resultado se llama
complemento a dos de n.
4.3 El algoritmo de la división: Números primos 221

Tabla 4.5
Notad6n del complemento a dos
Valor represmtado Patrón de cuatro bit!
7 o 1 1 1-
6 o 1 1 o
5 o 1 o 1<-
4 o 1 o o
3 o o 1 1
2 o o 1

~
1 o oo
o o o o
-1 1 1 1
-2 1 1 1
-3 1 1 o
-4 1 1 o o
-5 1 o 1 1
-6 1 o 1 O+-
-7 1 o o 1
-8 1 o o o-

Por ejemplo. para obtener la representación del complemento a dos de -6, hacemos lo
siguiente.
6
1) Empezamos con la representación l
binaria de 6. 0110
2) Intercambiamos los ceros y los unos; este l
resu1tado es el complemento a unos de 0110. 1001
3) Sumamos uno al resultado anterior. l
1001 + 0001 = 1010
También podemos obtener los patrones de cuatro bits de los valores -8 :s n :s -1
usando los patrones de cuatro bits de los enteros Oa 7 y complementando (intercambiando
los ceros y los unos) estos patrones como se muescra. en la tabla 4.5 mediante cuatro de
estas parejas. Observe, en esta tabla. que los patrones de cuatro bits de los enteros no
ne gativos comienzan en O. mientras que 1 es el primer bit de los enteros negativos de la
tabla.

¿Cómo realiz.amos Ja resta 33 - 15, en base 2, mediante el método de complemento a dos


con patrones de ocho bits(= un byte)?
Queremos determinar 33 - IS = 33 + (-15). Tenemos que 33 = (00100001)1 y 15 =
(00001111) • Por Jo tanto, represcntamos-15 como
2

11110000 + IXXXXXlOl = 11110001.

La suma de enteros representados en la notación del complemento a dos es igual a la


222 Capítulo 4 Propiedades de los enteros: Inducción matemática

suma binaria común, excepto que todos los resultados deben tener patrones deJ misa
tamaño. Esto significa que cuando se suman dos enteros por el método de complemelJIOa
dos, cualquier bit adicional que aparece en e l lado izquierdo de la respuesta (por un a-
rreo final) se debe descartar. Ilustramos esto en los siguientes cálculos.

3 3 - - + 00100001
- 15 + 11110001
- ¡--------1~
Este bit Sf" Respuesta• (CXXJ10010'2• 18
d escart a. -t..Este bit indica que la
rHpuesta es no negativa.

Para determinar 15 -33, utilizamos 15 = (00001111), y 33 = (00100001),. Entonces,


para calcular 15 - 33 como 15 + (-33), representamos - 33 corno 11011110 + 00000001:
11Ol ll 11. Esto nos da e l resultado
15 00001111
- 33 - + 11011111
-- 11101110
LEste bit indiu que la
respc,¡tsta u negativa.

Para obtener la forma positiva de la respuesta, hacemos lo siguiente:

11101110
1) Tomamos el complemento ¡
a unos. 00010001
2) Sumarnos 1 al resultado ¡
anterior. 00010010
Como (00010010),= 18, la respuesta es - 18.

Un problema que hemos evitado en los dos cálculos anteriores implica el tamaño de los
enteros que podemos representar con patrones de ocho bits. Independientemente del ta-
maño de patrón que usemos, el tamaño de los enteros que podemos representar es limita•
do. Cuando sobrepasamos ese tamaño, ocurre un en-orde desbordamiento. Por ejemplo, si
trabajamos con patrones de ocho bits e intentamos sumar 117 y 88, obtenemos
1 1 7 - - > 01110101
+ 88 + 01011000
-- 11001101
LEste bit indica que la
,respunta es negativa.

Este resultado muestra cómo podemos delectar un error de desbordamiento cuando suma.
mos dos números. En este e&o se indica un error de este tipo: Ja suma de los patrones de
ocho bits para dos enteros positivos produce el patrón de ocho bits de un entero negativo.
En forma análoga. cuando la suma de (]os patrones de ocho bits de) dos enteros negativos
produce un patrón de ocho bits de un entero positivo, se detecta un error de desbordamiento.

Para ver por qu6 funciona en general el procedimiento del ejemplo 4.27, sean x,y E z•
conx>y.
4.3 El algoritmo de la división: Números primos 223

Sea 211 - 1 :S x < 2•. E ntonces, la representación binaria dex está formada porn bits (con
un 1 e n el lugar más significativo). La representación binaria de 2• consta den+ 1 bits: un
1 en el lugar más significativo seguido de n ceros. La representación binaria de 2• - 1
consta de n u.nos.
Al restar y de 2•- 1, tenemos
(2" - l} - y =~ - y, el complemento a unos de y.

Entonces, (2• - 1)-y + 1 nos da el complemento a dos de y. y


x - y = x +[(2" -1) - y + 1)- 2",
donde el término final, -2•, da como resultado la eliminación del bit adicional que surge en
el 1ado izquierdo de la respuesta.
Cerraremos esta sección con un último resultado acerca de los enteros compuestos. (Este
resultado se utilizará posteriormente en el programa en Pascal que se muestra en la figura 8.3.)

Si n E z • y n es compuesto, entonces existe un primo p tal que p I n y p S J;.


Demostración: Como n es compuesto, podemos escribir n. = n 1 "-2, donde 1< n1 < n y 1< "2
< n. Afirmamos que uno de los enteros n1, n., debe ser menor o igual que J;. En caso
contrario, tendríamos que n1 > J; y ni> J; dan lugar a la contradicción n = n 1 n 2 >
( JnXJ;) = n. Sin ~rdida de generalidad, suponemos que n 1 s./;. Si n 1 es primo, se
sigue el resultado. Sin1 no es primo, entonces por el lema4.l existe un primop < n 1 tal que
p ln,. Así, p ln y p S -.Ín.

EJERCICIOS 4.3 1. Verifique las partes restantes del teorema 4.3.


2. S.ana, b. c. d E z·. Demuestre que (a)[(a l b) /1 (c i d)] => ac l bd; (b) a Ib => ac l bc; y (e)
= lbc=> a lb-
I
3. Si p, q son primos, demuestre que p q si y sólo si p = q.
4. Sia, b, ce z• ya l bc, ¿implicaestoquea jb o a jc?
5. Para cualesquiera enteros a, by e, demuestre que si a fbc, entonces afb y ate.
6. Sean E z ·, donden ~ 2. Demuestre que si a¡, a2, . .• ·ª•· bi, b 1, •.. , b,.E z • ya, I b,paratodo
1 s i s n, entonces (a1 a.i · • · aJ 1(b1 b1 • • • bJ.
7. a) F..ncuentretresentcrospositivosa.b. ctales que31 1 (Sa+7b+llc).
b) Si a. b. e E Z y 31 l<Sa + 7b+ 11c), demuestre que (i) 31 l(21a + 17b + 9c) y(ü) 31 l (6a +
21b + 1c).
8 . Una tienda de comestibles realiza un concurso semanal para promover sus ventas. Cada cliente
que adquiera más de $20 en mercancía recibe una tarjeta de juego con 12 m1meros en ella; si
cualquiera de estos números suman exactamente 500, entonces el cliente recibe un bono de
compra de $500 {en la tienda de comestibles). De.spués de comprar artículos por $22.83 en la
tienda, Leonor recibe su tarjeta dejuego en la que están impresos los siguientes 12 números: 144,
336, 30,66. 138,162,318, 54, 84, 288, 126 y 456. ¿Gana Lccoo,un bono deSSOO de compra?
9. S<ana. b E z·.Si b la yb l<a + 2). demuestre que b = 1 o b= 2.
1 O. Si n E z• y n es impar, demuestre que 8 I(n1 - 1).
11. Si a. b E z· y ambos son impaces. demuestre que 2 l(a'+ b') pero que 4((a'+ b').
24 Capítulo 4 Propiedades de los enteros: Inducción matemática

12. Determine el cociente q y el resto r para cada uno de los siguientes casos, donde a es el
dividendo y bel divisor.
a) •=23, b=7 b) a=-11S, b=l2
e) a=O, b=42 d) •=37, b=l
e) •=434, b = 31 f) a=-644, b = SS
13. Sin EN, demuestre que 3 1(7•-41.
14. Escriba cada uno de los siguientes números (dados en base 10) en base 2, base 4 y base 8.
a) 137 b) 6243 e) 12,34S
15. Escriba cada uno de los siguientes enteros (dados en base 10) en base 2 y base 16.
a) 22 b) 527 e) 1234 d) 6923
16. Convierta cada uno de los siguientes números hexadecimales a base 2 y a base JO.
a) A7 b) 4C2 e) 1C2B d) A2DFE
17. Convierta cada uno de los siguientes números binarios a base 10 y a base 16.
a) 11001110 b) 00110001 e) 11110000 d) 01010111
18. Escriba los siguientes enteros en la representación de complemento a dos. En este caso, los
resultados son patrones de ocho bits.
a) IS b) -IS e) 100
d) -65 e) 127 f) -128
19. Si una máquina guarda los enteros mediante el Imtodo de complemento a dos, ¿cuáles son los
enteros m.á.ximo y mínimo que puede guardar si utiliza patrones de bits de {a) 4 bits? (b) &bits?
(e) 16 bits? (d) 32 bits? (e) 2" bits, n E Z"7
20. En cada u.no de los siguientes problemas estamos usando los patrones de cualro bits para las
representaciones de complemento a dos de los enteros -8 a 7. Resuelva cada problema (de ser
posible) y despu~ convierta los resultados a base 10 para verificar sus respuestas. Tenga cui.
dado con los errores de desbordamiento.
a) 0101 b) 1101 e) 0111
+ 0001 + 1110 +1000
d) 1101 •l 1011 f) 0101
+ 1010 + 0101 +0100
21. Si a. x. y E Z ya'#- O, demuestre que a.r= ay ⇒ x = y.
22. Escriba un programa (odesarrolleun algoritmo) para convenir un encero positivo de base 10a
baseb,don<le2 s b s 9, .
23. E1 algoritmo de la división se puede generalizar de la manera siguiente: para a, b E Z y b -j O,
existen q, r E Z con a = qb + r, O s r < [b 1- Use e l teorema 4.5 para verificar esta forma
gene ralizada del algoritmo para b < O.
24. Escriba un programa (o desarrolle un algoritmo) para convenir un entero positivo de base IOa
base 16.
25. Paran E z·, escriba un programa (ode$11TOlle un algoritmo) que imprima todos los divisores
positivos den.
26. Defina el conjunto X~ z• en fonna recursiva, como sigue:
! ) 3 E X; y
2) Si a, b E X, entonces a +b EX.
Demuestre que X= {3A: l lc E z·},d conjuntodetOOOS los enteros positivos divisibles entre 3.
27. Sean E z· con n = r,• 10'+ · · · + r1 • 102+ r, • JO+ r0 0a representación en base IOde n).
Demucstrcquc(a)2 j nsiysólosi2 I r0 ;(b)4 I nsiysó1osi4 I (r1 •10+r0 )~y(c)8 J n
si y sólo si 8 1 (r 1·102 + r,·10 + ro).
Establezca un teorema general sugerido por estos resultados.
4 .4 El máximo común divisor: El algoritmo de Euclides 225

4.4
El máximo común divisor:
El algoritmo de Euclides
Siguiendo con la operación de división desarrollada en Ja sección 4.3 analizaremos los
divisores de una pareja de ente ros. ·

Para a,. b E Z, un e ntero positivo e es un divisor común de a y b si e Ia y e Ib.

Los divisores comunes de 42 y 70 son 1, 2, 7 y 14; 14 es el mayor de los divisores comunes.

Definición 4.3 Sean a, b E Z, donde a ~ O o b ~ O. Entonces e E z• es el máximo común di\li.sort de a,


b sí
a) c ja , c l b (es decir, ces un divisor común de a,. b), ·y
b) para cualquier común divisor d de a y b, tenemos que d i c.

El resultado del ejemplo 4.29 satisface estas condiciones. Sin embargo, este ej emplo
trabaja con dos enteros pe.queños. ¿Qué haríamos con dos enteros de 20 dígitos cada uno?
Consideremos las siguientes preguntas:

1) Dados a,. b E Z, donde al menos uno de ellos es distinto de O, ¿existe siempre el


máximo común divisor de a y b? Si existe, ¿cómo podemos encontrarlo?
2) ¿Cuántos máximos comunes divisores puede tener un par de enteros?

Al trabajar con estas preguntas, nos concentrarnos en a,. b E z•.

TEOREMA4.6 Para cualesquiera a, bE z•, e:{iSte un l1nicoc E Z• queeselmá:x.imocomún divisor de a. b.


Demostración: Dados a, bE z•, seaS; (as+br ls, r E Z ,as+ br>O}. ComoS4'0, por
el principio del buen orde n, S tiene un elemento mínimo e. Afirmamos que e es el máximo
común divisor de a, b.
Como e E S, e :;;; ax+ by, para algunos x, y E Z. En consecuencia, si d E Z y d a y I
d Ib, entonces por el teorema 4.3(f) d 1(ax+ by), por lo que d Ic.
Si eta, podemos usar el algoritmo de la división para escribir a:;;; qc + r, con q, r E z•
y O< r< c. Entonces r ; a - qc ; a - q(ax +by) ; (1-qx)a + ( -qy)b, por lo que r ES, lo
que contradice la elección de e como el elemento mínimo de S. En consecuencia., e a, y I
por un argumento similar, e b. I
Así, cualesquiera a, b E z• tienen un máximo común divisor. Si c1 , c2 satisfacen las dos
condiciones de la definición 4.3, e ntonces, con c 1 como el máximo com6n divisor y c2

t Grtakst Ccmnu>n di11üor (gcd). En español mcd. (N. thl E.)


Capítulo 4 Propiedades de los enteros: Inducción matemática

como un divisor común, se siguequec1 1~- Si invertimos los papeles, vemos qucc 1 lci;y,
por el teorema 4.3(b), tenemos que c 1 ;:;; ei, ya que c1, c2 E z•.

Ahora sabemos que para cualesquiera a, b E Z♦• el máximo común divisor de a, b


existe y es único. Denotamos este m1mero con mcd(a, b). Aquí mcd(a, b) = mcd(b, a); y
para cualquier a E Z , si a ,. O, entonces mcd(a, O) = 1a¡. También, cuando a, b E z•,
tenemos que mcd(-a, b) = mcd(a,-b) = mcd(-a,-b) = mcd(a, b). Finalmente, mcd(0,0)
no está definido y no es de interés para nosotros.
Del teorema 4.6, vemos que mcd(a, b) no sólo e~ste sino que mcd(a, b) también esel
entero positivo más pequeño que podemos escribir como una combinaci6n lineal de a y b.
Sin embargo, debemos darnos cuenta de que si a, b. e E z+y e=- ax+ by para algunosx,
y E Z, entonces no necesariamente debemos concluir que e sea mcd(a, b); a menos que
también sepamos de alguna manera que e es el entero positivo más pequeño que puede ser
escrito como una combinación lineal de a y b.
Los enteros a y b son primos relativos si mcd(a, b) =- l; es decir, cuandoexiste x, y E Z
conax+by = l.

Comomcd(42. 70) = 14, podemoseoconttar x, y E Z tales que42.t+ 70y= 14 o 3x + 5y= l.


Porinspección,x=2,y =-I es una solución; 3(2)+ 5( - 1)= l. Peroparak E Z, 1 = 3(2-5.t)
+ 5( -1 +3k),entonces 14 = 42(2-5k)+ 70(-1 +3k)y las soluciones parax, y no son únicas.
En general, si mcd(a, bí=d,entoncesmcd((a/d~(b/d))= l. (¡Verifiqueesto!)Si (a/d).r.+
(bld)y,= 1, entonces 1 = (ald)(x,,-(bld)k) + (bld)(y,+ (ald)k), para cualquier k E Z. Así,
d= a(x,, - (bld)k) + b(y0 + ( a/d)k), lo que produce un número infinito de soluciones para L,
ecuación ax + by =d.

El ejemplo y las observaciones anteriores funcionan bastante bien si a, b son pequeños.


I
¿Pero cómo se puede encontrar mcd(a. b) para a, b E z+arbitrarios? Si a b, entonces
I
mcd(a, b) = a; y si b a, entonces mcd(a. b) = b; en los demás casos, recurrimos al siguien-.
te resultado, que debemos a Euclides.

'0REMA4.7 Algoritmo de Euclides. Si a, b E Z♦• aplicamos el algorinno de la división como sigue:

a=q1b+ r1, 0<r, < b


b = q2r1 +r2, O< r 2<r1
Tt =q 3T2 + T3, O< r 3 < r 2

T¡ = Q;.+2T;+1 + T1+2,

Tk-3 = qt-1 Tt-2 + Tt- lt 0 < T.t-1 < Tk- 2


Tt-2 = qt Tt-1 + TJ:, O< r t<T.t--1

Tt-1 = t.jhl Tt•

Entonces, Tt, el último resto distinto de cero, es igual a mcd(a, b).


4.4 El máximo común divisor: El algoritmo de Eudides 227

De mostración; Para verificar que r1 = mcd(a, b), establecemos las dos condiciones de la
definición 4.3.
Comenza,emos con el primer proceso de división enumerado arriba. Si e a y e h. I I
I
entonccscomoa=q1b+ r,, se sigue qucc r1• Después, [(el b) A (c!ri)J => clr,, ya que
b = q1r1 + r1• Continuando hacia abajo por los procesos de división, llegamos hasta donde
el I I
r1 _ 1 y e r1 _ 1• Por la pcnúJtima ecuación, concluimos que e rt> !oque verifica la condi-
ción (b) en la definición 4.3.
Para establecer la condición (a), seguimos un orden inverso. De la última ecuación. r1 1
r._ 1por Jo que r. 1Tt-1, ya que r••1= qtrt- i + rt. Continuando hacia arriba por las ecuaciones,
llegamos basta donde r, 1r, y r, 1r,, por lo.que r, 1r,. Entonces [(r, 1r,) A (r, 1r1)] => r, 1b
y finalmente [(r,I r,) A (r, 1b)] => r,¡o. De aquí que r,= mcd(a, b).

Hemos usado ahora la palabra a/gorirmo para describir las proposiciones establecidas
en los teoremas 4.S y 4.7. Este támino aparecerá con frecuencia en los demás capítulos de
este texto, por lo que sería buena idea mencionar Jo que denota.
En primer lugar, un a/goritmq es una lista de instrucciones p~cisas diseñadas para
resolver un tipo de problema particular, no solamente un caso especial. En general. espe-
ramos que todos nuestros algoritmos reciban una entrada)' nos proporcionen el resultado
(o resultados) necesario como solido. De igual modo, un algoritmo debe proporcionar el
mismo resultado si repetimos el valor (o valores) para 1a entrada. Esto sucede cuando 1a
lista de instrucciones es tal que cada resultado intermedio proveniente de la ejecución de
cada instrucción es ll.nico y sólo depende de la entrada (inicial) y de cualquier resultado
que se pudiera haber obtenido en cualquiera de las intrucciones precedentes. Para lograr
esto hay que eliminar toda vaguedad del algoritmo~ las instrucciones deben describirse de
forma simple, pero no ambigua. de modo que pueda ser ejecutada por una máquina. Por
úJtimo. nuestro algoritmo no puede continuar por siempre. Debe terminar despu~s de la
ejecución de un m1mcro finito de insttuc:ciones.
En el teorema 4.7 determinamos el máximo com6n divisor de dos enteros positivos
cualesquiera. Por lo tanto, este algoritmo recibe los dos enteros positivos a, b como entra-
da y genera su máximo comWI divisor como salida
El uso de la palabra algoritmo en el teorema 4.5 se basa en la tradición. Tal como está
establecido, no proporciona las intruccioncs precisas para determinar la salida deseada.
(Mencionamos este hecho antes del ejemplo 4.23.) Para eliminar esta desventaja del teore-
ma 4.S, indicaremos las instrucciones en el programa en Pascal de la figura 4 .8.

Ahora aplicaremos el algoritmo M: Euclides (Teorema 4.7) en los siguientes cinco


ejemplos.

Determinaremos el máximo comlln divisor de 250 y 111. y expresaremos el resultado


como una combinación lineal de estos enteros.
250-2(lll)+28, 0<28<111
lll • 3(28) +27, 0<27<28
28 ~ 1(27) + !, O<! <27
27 = 27(1) + O.
Capítulo 4 Propiedades de los enteros: Inducción matemática

Así, 1 es el último resto distinto de cero. Porlo tanto, mcd(250, 111) = 1, y así, 250 J
11 1 son primos relativos. Si trabajamos hacia atrás en la tercera ecuación, tendremos 1=
28 - 1(27) = 28 - 1[111 - 3(28)] = ( -! XIII)+ 4(28) = ( -IXlll) + 4[250- 2(111)1 =
4(250) -9(111) = 250(4) + 111( - 9), una combinación lineal de 250 y 111.
Esta expresión de 1 como combinación lineal de 250 y 111 no es llnica, ya que 1 =
250[4- lllk] + 111[ - 9 +250k], paracualquíerkE Z.
También tenemos que rncd(- 250, 111) = mcd(250, - l l l) = mcd(- 250, -lll):
mcd(250, ll l) = l.

Nuestro siguiente ejemplo es un poco más general, ya que se trata del máximo comÚI
divisor de un número infinito de parejas de e nteros.

Para cualquier n E Z+, demostraremos que los enteros positivos 8n + 3 y 5n + 2 soa


primos relativos.
Aquí tenemos que 8n + 3 > Sn + 2, y como en el ejemplo anterior, podemos.escribir lo
siguiente:

8n +3= 1(5n +2)+ (3n + 1), 0<3n+1<5n +2


5n +2= 1(3n + 1)+ (2n + 1), 0<2n+1<3n+l
3n + 1 = 1(2n + 1) + n, 0<n<2n+l
2n + 1 =2(n)+ 1, 0<l<n
n = n(l) + O.

Por lo tanto, el último resto distinto de cero es 1, por lo que mcd(Sn + 3, 5n + 2); l. Pero
también podríamos haber llegado a esta conclusión si observamos que

(8n +3)(-5) + (5n + 2)(8) = -15 + 16 = l.

Y como 1 está expresado como una combinación lineal de 8n + 3 y Sn + 2. y ningún otro


número positivo más pequeño puede tener esta propiedad, se sigue que el máximo común
divisor de 8n + 3 y Sn + 2 es l. para cualquier entero positivo n.

Una vez determinado et máximo común divisor del ejemplo 4.31. usaremos ahora e]
algoritmo de Euclides para escribir un programa que determinará mcd(a. b) para a, b E
z·. La salida de este programa se muestra mediante la evaluación de mcd(456, 624) y
mcd(l 16,641).
El programa en Pascal de la figura 4.9 utiliza la operación binaria Mod; parax. y E z•,
x Mod y es el resto después de dividir x entre y. Por ejemplo, 7 Mod 3 es 1 y 18 Mod Ses
3. (En el capítulo 14 trabajaremos con más detalle '1a aritmética de los restos".)
4.4 El máximo común divisor: El algoritmo de Euclides 229

Program. EuclideanAlgori thm (input,output);


Var
a,b.c,d,r: integer;
Begin
Wri tel.n ( 1 Que.remos determinar el máximo común ' ) ;
lriteln ( 'divisor de dos enteros po•itivos a,b. ' );
Write ('a - ' );
Road ( a );
lrite('b•');
Read ( b );
r:•allodb;
d : - b;
Whilo r > O do
Becin
e := d;
d := r;
r:=-cllodd
End;
Writaln ( 'El máximo común divisor de' , a:O,' y');
Write (b :O . ' 1s ',d:0)
End.

Queremo• determinar el máximo común


divisor de dos enteros positivos a, b.
a = 456
b = 624
El m6ximo común divisor de 456 y
624 is 24

Queremos det erminar el máximo común


divisor de dos enteros positivos a, b.
a = 116
b = 641
El máximo común divisor de 116 y
641 1s 1

Figura 4 .9

Jorge tieoe dos recipientes no marcados. Un recipiente contiene 17 onzas y el otro SS


onzas. Explique cómo puede usar Jorge los dos recipientes para medir exactamente una
onza.
Por el algoritmo de Euclides tenemos que

SS= 3(17) + 4, 0<4< 17


17 =4(4) + 1, 0<1 < 4.

= = =
Por lo tanto, 1 17 -4(4) 17 -4[55 -3(17)) 13(17)-4(55). Por lo tanto, Jorge debe
llenar el recipiente pequelio (de 17 onzas) 13 veces y vaciar el contenido (las primeras
doce veces) en el recipiente mayor. (Jorge vacía el recipiente mayor cada vez que est~
230 Capítulo 4 Propiedades de los enteros: Inducción matemática

Deno.) Antes de llenar el recipiente pequeño la 131 • vez. Jorge tiene 12(17) - 3(55) =
204 - 165 = 39 onzas de agua en el recipiente mayor (de 55 onzas). Después de llenar el
recipiente pequeño por 131 • vez.. éJ vaciará 16(= 55 -39) de este recipiente, y llenará el
recipíente grande. Quedará exactamente una onza en el recipiente pequeño.

Al ayudar a los estudiantes en sus cursos de programación, Juan observa que en promedio TI
puede ayudar a un estudiante a depurar un programa en Pascal en 6 minutos, pero tarda 10
minutos e n depurar un programa escrito en APL. Si trabajó en forma continua durante
104 minutos y no desperdició tiempo, ¿cuántos programas depuró en cada lenguaje?
En este caso buscamos enteros x, y 2: O tales que 6x + lOy = 104, o 3x + 5y = 52. Como
mcd(3, 5) = 1, podemos escribir 1 = 3(2) + 5( - 1), por lo que 52 = 3(104) + 5( - 52) =
3(104 -5k) + 5( - 52 + 3k), k E Z. Para obtener O S x = 104 - 5k y O S y= - 52 + 3.t,
debemos tener (52/3) s k s (104/5). Así, k = 18, 19, 20 y cl<Ísten tres posibles soluciones:

a) (k=!8): x = l4, y=2 b) (k = l9): x=9, y=S


e) (k = 20): x = 4, y =8

La ecuación del ejemplo 4.35 es un ejemplo de una ecuaci6n diofántica: uaa ecuacióo
lineal que requiere soluciones enteras. Este tipo de ecuación fue estudiada por primera vez
por e1 algebrista griego Diofanto. que vivió en el siglo m o.e.
Una vez resuelta una de estas ecuaciones, veamos ahora si podemos determinar cuándo
tiene solución una ecuación diofántica La demostración se deja al lector.

"EOREMA4.8 Sia, b, e E Z♦• la ecuación diofántica ax+ by=c tiene una solución enterax =Xo,,Y ~ y0 si
y sólo si mcd(a, b) divide a c.

Cerramos esta sección con un concepto relacionado con el máximo común divisor.

>efinición 4 .4 Sia. b, e E Z♦• e es unmtiltiplo común de a, b sic es un múltiplo de a y deb.Además, ces


el mínimo común múltiplo de a, b si es el más pequeño de los enteros positivos que son
múltiplos comunes de a. b. Denotamos e con mcm(a, b).

a) Como 12 = 3 • 4 y ningún otro entero positivo mínimo es un móltiplo de 3 y 4,


tenemos que mcm(3, 4) = 12 = mcm(4, 3). Sin embargo, mcm(3, 2) f.12, ya que
aunque 12 es un múltiplo de 3 y 2, existe un múltiplo más pequeño. a saber, 6. Y
como ningún otto múltiplo común de 3 y 2 es menor que 6, se sigue que mcm
(3,2)=6.
4.4 El máximo común divisor: El algoritmo de Eudides 231

b) Para cualquier n E Z♦• tenemos que mcm(l. n) • mcm(n, 1) • n.


e) Si a, n E Z♦• tenemos que mcm(a. na)= na. [Esta proposición es una generaliza-
ción de la parte (b). La proposición anterior se sigue de ésta cuando a• l.]
d) Si a, m, n E z• con m :s n, entonces mcm(a'", a") =a". [Y mcd(a'", a") =a'".]

TEOREMA4.9 Sean a, b, e E z• con e= mcm(a, b). Si des un ml'.i1tiplo coml1n de a y b, entonces e Id.
Demostración: En caso contrario, usarnos el algoritmo de la división para escribir d = qc +
r, donde O< r< c. Como e = mcm(a, b), se sigue que e= ma para a1gl1n m E z•. Además,
d - na para algún n E z•, ya quedes un múltiplo de a. En consecuencia, na , , _ qma + r =>
(n - qm)a = r > O, y res un múltiplo de a. De forma análoga, se puede ver que res un
múltiplo deb, por lo que res un múltiplo común dea, b. Pero como O< r< e, contradeci-
mos la afirmación de que -e es el mínimo común múltiplo. Por lo tanto, e Id.
El último resultado de esta sección une Jos conceptos de máximo común divisor y el
mínimo común múltiplo. Además, nos proporciona una forma de calcular mcm(a, b) para
cualquier a, b E z+. La demostración de este resu1tado se deja al lector.

TEOREMA 4.1 O Para a, be z•, áb = mcm(a, b). mcd(a, b).

Por el teorema 4.10 tenemos lo siguiente:


a ) En el ejemplo4.31 vimosqucmcd(250, 111) = l. En consecuencia, mcm(250, 111)=
(250Xl 11) = 27,750.
b ) El ejemplo de la parte (a) es un caso panicular de un resultado más general: para
cualesquiera a. b E z·, si a. b son primos relativos, entonces mcm(a. b) = ab.
e) El primer cálculo (después del programa) de la figura 4.9 establece el hecho deque
mcd(456, 624) = 24. Como resultado tenemos que

mcm(456,624) = (456)(624)/24 = 11,856.

EJERCICIOS 4.4 1. Para cada uno de los pares siguientes a, b E Z♦• detcnnine mcd{a, b) y cxprbclo como una
combinación lineal de a, b.
•> 231, 1820 b) 1369, 2597
e) 2689, 4001 d) 798:2, 7983

2. Para a, b E Z•y s, t E Z, ¿qutpodemos decir de mcd(a,b) si


a) as+bt = 2? b) as+bl=3?
e) as+bt=4? d) as+b1 = 6?

• 3. Para a, b E z• y d =mcd(a, b), demuestte que mcd(ald. bid)= l.


4. Para a, b E z·. demuestre que mcd(na, nb) =n mcd(a, b).
Capítulo 4 Prop iedades de los e nteros: Inducción matemitica

5. Para a, b, e e z·, demuestre que si d = a+ be, entooces mcd(b. d) =mc.d(a. b).


6 . Scana, h, e e z·
con mcd(a. b)= l. Si a le yb lc, demuestre que ah!c. ¿Valeesteresultadosi
mcd(a, b)" I?
7. Sean a, b e z. donde al menos uno de ellos es distinto de cero.
a) Utilice cuantificadores para restablecer la defin.jción: de e• mcd(a, b), donde e e z · .
b ) Use el re.sultado de la parte (a) para decidir cuando e~ mcd(a, b) para alg\111 e e z•.
8. Si a, b son primos re lativos y a> b, demuestre que mcd{a - b, a+ b) = 1 o 2.
9. Seana, b. e E z·con mcd(a, b) s l. Si a Jbc, demuestroquea Jc.
10. Scana, be z· con a par y b impar. Demuestre que mcd(a. b) • mcd(a/2, b).
11. Sean a, b e z· donde a ;,, b. O.muestre que mod(a, b) = mod(a - b, b).
12. Sean a, b, e, dentcros positivos fijos. Si (ad- bc) ja y (ad- be) lc, dcmucstrcque mcd(an+b,
C1t + d)=I para cualquier ne z·.

13. Demuestre que pararualqu.ier n E z·. mcd(Sn + 3. 7n + 4)= l.


14. Una ejecutiva compra $249.00 de j uguetes para los niños de sus empicados. Pan cada niña,
compra una mufteca. de $3.30; cada niño recibe un conjunto de soldados que cuestan S2.90.
¿Cuántos juguetes de cada tipo compró?
15. Dccenninc los valores de e e z·. JO< e < 20. para los que la ecuacióo diofántica 84x + 990y =t
no tiene solución. Dctennine las soluciones para los valores restantes de c.
16. a ) Determine los e nteros w, x. y E Z que satisfagan el siguiente siSlema de ecuaciones
diofánticas:
w+ x +y • SO
w + l3z + 31y • 116
b) ¿ Existe u.na solución en la pane (a) taJ que~ .t. y > O?
e) ¿Existe una solución en la pane(a) tal que w> 10. x> 28 y y >-15?
1 7. Verifique el teorema 4.8.
18. Verifique el teorema 4.1 O.
1 9. Si a. b e z · con a = 630, mcd(a, b) = 105 y mcm(a, b)=242.SSO. ¿c:uíl es el V1lor de b?
20. Para cada pareja a, b deJ ejercicio 1, determine mcm(a, b).
21 . Para cualquier n E z·, ¿cuánto valen mcd(n, n + 1) y mcm(n. n + l )?
22. Demuestre que mcm(na,. nb) = n mcm(a, b) paran. a, be z·.

4 .5
El t eorema funda mental d e la
aritmét ica

En esta sección extenderemos el lema4.l y mostraremos que paracualquiern E z•,n> 1,


n es primo o n puede escribirse como producto de. primos, donde la representación como
producto de primos es única, e xcepto por c1 orden de los factores. Este resultado, conocido
como el teorema fundamemal dt la aritmética. puede encontrarse e n una forma equiva-
lente e n e l libro IX de los Elt mtnros de Euclides.
Los siguientes dos le mas nos ayudaran a lograr nuestro objetivo.

,MA4.2 Si a. b E z• y pes un primo, entoncesplab => p¡aop lb.


I
Demostración: Si p a, entonces hemos tenninado. Si no. como pes primo, se sigue que
4.5 El teorema fundamental de la aritmética 233

mcd{p, a)= 1 y, por Jo tanto, existen enteros x, y con px + ay = l. Entonces b = p(bx) +


I I
(ab)y, donde p p y p ab. Así, se sigue de las panes (d) y (e) del teorema 4.3 que p b. I

lema 4.3 I I
Sea a; E z • para todo 1 :s i :s n. Si pes primo y p a 1a 2 • • • a.,, entonces p a, para algún
1 Si S n.
Demostración: Dejamos la demostración de este resultado al lector.

Con el lema 4.2, tenemos otra oportunidad para establecer un resultado mediante el
método de la demostración por contradicción.

Queremos mostrar que Jies irracional.


En caso contrario, podemos escribir.fi. = alb, donde a. b E z• y mcd(a, b) = 1. Enton-
ces-ti.= alb ⇒ 2 =a'lb' ⇒ W=a' ⇒ 21 a' ⇒ 2 ja. (¿Por qué?) Además, 2 l a ⇒ a= 2c
21
para algún e E z·, por lo que 2b'= a'= (2c)'= 4c' y Ir= 2c'. Pero entonces Ir ""> 21
b.
Como 2 divide a a y a b, esto implica que mcd(a, b) ~ 2, pero esto contradice nuestra
hipótesis anterior en e] sentido de que mcd(a, b) = l. [Nota: esta demostración de la irra-
cionalidad de.Ji era conocida por Aristóteles (384 - 322 A.c.)y es similar a la dada en el
libro X de los Elememos de Euclides.)

Antes de pasar al resultado principal de esta sección, observemos que el entero 2 del
ejemplo anterior no tiene un papel especial. Pediremos aJ lector que, en la sección de
ejercicios, demuestre el hecho de que.,{p es irracional para cada primo p. Ahora que he-
mos mencionado este hecho, es hora de presentar el teorema fundamental de la aritmética.

leorema 4.11 Cada entero n > 1 puede escribirse como un producto de primos de fonna única, excepto
por el orden de éstos. (En este caso, un solo primo se considera un producto de un factor.)
Demostración: La demostración consta de dos partes: la primera demuestra la existencia
de una factorización con primos y la segunda se refiere a la unicidad.
Si la primera parte no fuera verdadera. sea m > 1 el entero más pequeño que no puede
expresarse como un producto de primos. Como m no es primo. pode mos escribir m =
m1m.i, donde l< m1 < m. l < mi< m. Pero entonces m1, "½ pueden escribirse como produc•
tos de primos, ya que son menores que m. En consecuencia, usamos la representación m =
m 1f1½ para obtener una factorización de m con primos.
Para establecer la unicidad de una factoriz.ación como producto de primos. usaremos la
forma alternativa de la inducción matemática (feorema4.2). Para el entero 2, tenemos una
única factorizaci6n como producto de primos y, suponiendo verdadera la unicidad de la
representación para 3, 4, 5, . .. , n - 1, suponemos que n = p~• p;
1
p:• q;• q¡ · ·· q:·,
• • • =
donde cadap;, 1 :s; is ky cadaq¡, l Sj s r, es un primo. También suponemos que p 1 < P.l<
· · · <p1; yq 1<q2 <· · • <q, y s;>Opara 1 :S: is k, t¡ >O para lS j Sr.
Comop1 1n, tenemos quep1 1 1 q; qi ··· q;·. Por el lema4.3,p1 lt¿ para alg"ún 1 :S:j sr.
Como p 1 y q¡son primos, tenemos que p1 = q¡. De hecho,j = 1, ya que de lo contrario, q 1 1 n
=> q 1 = p, para algún l< e S k y p 1< p, = q 1< q¡ = p 1• Usamos p 1 = qi, para ver que n 1 =
34 Capítulo 4 Propiedades de los enteros: Inducción matemática

nlp1 = p~- 1 Pi •••p;• = q:•- 1 q{ •• • q~. Como n 1 < n, se sigue de la hipótesis de inducciól
que k = r,p¡= q; para l S; S: k, s1 - 1 = t 1 - l (por lo que s 1 =t 1) y S; = t 1 para 2 Si :S .t.
Obtenemos de aquí que la factorización como producto de primos de n es llnica.

Usaremos ahora este resultado en los siguientes cinco ejemplos.

Para el entero 980,220, podemos determinar la factorización como producto de primos


como sigue:
980,220 = 21(490,110) = 21(245,055) = 2'31(81,685) = 22 31 51(16,337)
= 21 31 51 171(961) = 2'· 3 · 5 · 17 · 312

[~ffl Sean E z + y supongamos que


(') 10·9 · 8·7 -6- 5-4· 3 · 2·n = 21 ·20· 19· 18· 17 · 16· 15· 14.
Como 17 es un factor primo de l entero que está en el lado derecho de(•), wnbitn debe ser
un factor del lado izquierdo (por la parte de unicidad del teorema fundamental de la añt-
mética). Pero 17 no divide a ninguno de los factores 10, 9, 8, ... , 3, o 2, por lo que 17 1 n.
(Un argumento similar muestra que 19 In.)

Para n. E Z·, queremos contar el nümero de divisores positivos de n. Por ejemplo, el


número 2 tiene dos divisores positivos: 1 y él mismo. De la misma forma. 1 y 3 son los
llnicos divisores positivos de 3. En el caso de 4, tenemos ues divisores positivos: 1, 2 y 4.
Para determinar el resultado en el caso de cualquier n E z•, n > 1 , usamos el teorema
4.11 y escribirnos n = P? p;• · · · p;•, donde para cada l s i s k, p, es un primo y e;> O. Si
I
m n, entonces m = p(' p{• · · · p{•donde O :S /; :S e; para todo I s i :S k. Así, por la regla
del producto, el número de divisores positivos den es

(e1 + l)(e, + 1) ···(e,+ 1).

Por ejemplo, como 29,338,848,000 = 2' 3' 5' 7' 11, vemos que 29,338,848,000 tiene
= =
(8 + I X5 + I X3 + 1)(3 + IXI + 1) (9X6X4X4X2) 1728 divisores positivos.
Si queremos saber cuántos de estos 1728 divisores son mdltiplos de 360= 23 3 1 5, enton-
cesdebemos darnoscuenta deque querernoscontarlosenterosdelafonna 2 11 3r: 5•, 7,.111'
donde
2St1 :S5, O:s ,. :s 3, 0:St,:Sl.
En consecuencia, el número de divisores positivos de 29,338,848,000 que son divisibles
entre 360 es
[(8 - 3) + 11(5 - 2) + 11 (3-1) + 11 (3-0) + ll(l -0) + l] = (6)(4)(3)(4)(2) = 576.
Para deternúnar cuántos de los 1728 divisores positivos de 29,338,848,000 son cuadrados
perfectos, necesitamos considerar todos los divisores de la forma 2'' 3~ 5" 7'• 11", donde
cada s., s1 , s3 , s,, s5 es un e ntero par no negativo. En consecuencia. tenemos en este caso
4.5 El teorema fundamental de la aritmética 235

5 opciones para s 1; a saber, O, 2, 4, 6, 8;


3 opciones para Sú a saber, O, 2, 4;
2 opciones para s3 y s..; a saber, O, 2; y
1 opción para s5 ; a saber, O.

Se sigue entonces que el número de divisores positivos de 29,338,848,000 que son cua-
drados perfectos es (5)(3)(2X2)(1) = 60. [De manera análoga, tenemos que existen
(3X2)(2X2Xl ) = 24 divisores positivos de 29,338,848,000 que son cubos perfectos y
(3X2XlX1Xl) = 6 que son potencias cuartas perfectas.]

Para nuestro siguiente ejemplo necesitamos el equivalente multiplicativo de la noca-


ción sigma (para la suma) ya comentada en la sección 1.3. Aquí usaremos la letra griega
mayúscula Il para la notación pi.
Podemos usar la notación pi para expresar el productox1 x2 x 3 .t.x5 x6, por ejemplo, como
íl :. 1
x;· En general, es posible expresar el producto de losn - m + 1 términosx_ x_i, x_2,
....x,., donde m, n E Z y m s n, como n;_x,.Como en el caso de la notación sigma, la
letra i es el índice del producto y este índice representa los n - m + 1 e nteros a partir del
llmite inferior m y hasta llegar a1 límite superior n (e incluirlo).
Esta notación se demuestra en los siguientes casos:

1) II7-3x¡ =x3x,.xsx,x, = IlJ.1xj, ya que la letra i no tiene nada de particular;


2) n t. , i=3·4-s-6=6!12!:
3) fi /•• i = m(m + l )(m + 2) · · · (n - l)(n) = n lJ(m -1)!, para cualesquiera m,
nez•conm sn;
4) TT/.2 3i = (3 · 2)(3 ·3)(3 · 4)(3 · 5) = 3'(5!/1!) = 3<•- 2>"(5!/l!), y para cualesquiera
m,n E z• con m s n, ycualquier e E R, distinto de cero. tenemos que

..
,íl ci = c"-*• (n IJ(m -
1
!)!); y,

Sim. n E z•.sean m = p~ p;1 • •• p;• y n = pf pf ··· p[, , dondecadap;es primoyOS e;


y O sJ; para todo 1 si s. t. Entonces.si a¡= mín{ e¡• J;}. el mínimo (o más pequeño) de e1 y
f, y b, = máx(e, J;}, el máximo (o más grande) de,, yf., para todo 1 :S i :Sr, tenemos que

mcd(m, n) = pf1 Pl- : • • • p~ = Óp:"


f• l

mcm(m,n) = pt1 JJ:"2. • • pf1 =ílp~.


i• I

Por ejemplo, sea m =491,891,400 = 2 3 S' 7' 11 13' ysea n = 1,138,845,708 =2' 3' 71
3 3 1

• 1l2 131 171 • Entonces. conp1 = 2, (>2 = 3, p 3 = 5,p4= 7,ps = 1I, p6= 13,p,;;;; 17, vemos que
a 1= 2, a 2 = 2, a 3 = O (el exponente de 5 en la factorización de n debe ser O, ya que 5 no
Capítulo 4 Propiedades de los enteros: Inducción matemática
-
aparece en la factorización como producto de primos),ª•= 1, a,= 1, ~-= 2, y a,= O. A!;í.
mcd(m, n) = 2' 3' 5" 7' l1 ' 13' 17' = 468,468.
Tambiin tenemos que
mcm(m, n) =2 3 33 5 1 7' 11' 13' 17' =l ,195,787,993,400.
Nuestro óltimo resultado de la sección relaciona el teorema fundamental de la aritméci-
ca con el hecho de que cua1esqtúera dos enteros consecutivos son primos relativos (como
se observó en el ejercicio 21 de la sección 4.4).

Aquí buscamos una respuesta para la siguiente pregunta. ¿Podemos encontrar tres enteros
positivos consecutivos cuyo producto sea un cuadrado perfecto? Es decir, ¿existen m. n E
z• tales que m(m + 1Xm + 2) = 1r?
Supongamos que existen dichos enteros m, n. Rccoroemos que mcd(m, m + 1) = 1 =
mcd(m+ 1, m+2). pcrloque,par:acualquiecprimop,si p J(m + 1), entoncesp{m y p{(m+
2). Además, si p 1(m + 1), se sigue que p J ,r. Y como n' es un cuadrado perfecto, por el
teorema fundamental de )a arinni tica. vemos que los exponentes dep en 1as factorizaciooc:s
como producto de primos de m + 1 y fil deben ser el mismo ebtero par. Esto es cierto pan
cualquíeT divisor primo de m + 1, pcr lo que m + 1 es un cuadrado perfecto. Como ,r y m +
l son cuadrados perfectos, concluimos que el pnxlucto m(m + 2) tambil!n es un cuadrado
perfect0. Sin embaI¡¡o, el producto m(m + 2) es tal que,,;<,,; + 2m =m(m + 2) <,,; + 2m +
1 = (m + 1)1 . En con.secuencia. vemos que m(m + 2) está entre dos cuadrados perfeccos
consecutivos y no es igual a ninguno de ellos.Así, m(m + 2) no puede ser un cuadrado perf«IO,
y no existen tres enteros positivos consecutivos cuyo producto sea un cuadrado perfecto.

ERCICIOS 4.5 1. Escriba cada uno de los siguientes nómcros como un producto de primos

•l 148,500 b) 7,114,800 e) 7,882,875


2. Dctermined máximo eomtindivisor y eJ mínimo comiin mdltiplo para cada pareja.de nllmm,s
deJ ejercicio anterior.
3 . Sean, e z· arbítrarió y pi,/Jh "1, . .. ,p, prilD05 distintos. Si me z· y la. fa.c:torización como
producto de primos de m es A"' Pi
¡,," ··· Pi,
¿cu'1 es la factoriu.ción como producto de pri-
mos (a) de m'? (b)de m'?
4. Detennioe la factoriUlción como producto de primos de lo siguiente:
a) 8! b) 10! e) 12! d) 15!
5. Encuentre el valor del entero positivo mú grande ta1 que 2" divida a 22!.
6. Vcrif,que el lema 4 .3.
7. Demuestre que ./p es irracional para cualquier primo p.
8. Si p es cualquier primo. demuestre que: fp es irracional.
9 . a) Demuestre que log,o2 es irracional.
b) Para cualquier primo p, demuestre que log 10 pes irracional .
10 . Enwentre el número de divisores positivos para ca.da uno de los siguientes enteros.
4.5 El teorema fundamental de la a ritmética 237

a) 2·3 · 5 · 7·ll·l3•17-19·23 b) 2·3'·5'·7'·11'·13"·17' e) 17!


11. F.ncuentre el nómero de divisores positivos para cada entero del ejercicio J.
12. a) ¿OJántos divisores positivos tienen= 2w39 5•7io J 13 135 371°?
b ) Para 1os divisores de la parte (a). ¿cuántos son
i) divisibles entre 23 3'5' 11 2 3'P?
ü) divisibles entre 1,166,400,000?
iii) cuadrados perfectos?
iv) cuadrados perfectos divisibles entre 22 J' 5'211 2?
v) cuadrados perfectos divisibles entre 21 3 '5'7'?
vi) cubos perfectos'?
vil) cubos perfectos que son mLHtiplos de 2 10 39 5'2-P 11 1 132 372?
vili) cuartas potencias perfectas?
ix) quintas potencias perfectas?
x) cuadrados perfectos y cubos perfectos?
13. Seanm. n E z· talesquemn = 2"3' 971 113 131. Si mcm(m, n) = 2 2 33 5 2 7 1 112 131, ¿cuál esel
mcd(m,n)?
14. Amplíe los resultados dc1 ejemplo 4.42 y encuentre el máximo común divisor y el núnimo
común múltiplo para los tres números del ejercicio 1.
~ 15. Determine el cuadrado perfecto más pequeño que es divisible entre 7!.
16. Para cualquier n E z•, demuestre que n es un cuadrado perfecto si y sólo sin tiene un m1mero
impar de divisores positivos.
17. F.ncuentre el entero positivo más pequeño n para el cual el producto 1260 x n es un cubo pcrfecto.
18. Doscientas monedas muneradas del l al 200 se colocan eo hileras a lo taigo de la barra de m:ia
cafc«ña Se ,wgnan números (del I al 200) a doscientos estudiantes y se les pide dar la vuelta a
algunas monedas. flestudiantecon d mlmero 1voltea todas tas monedas. El eswdianteoondnúme-
ro 2 da la vuelta a las monedas"' fc,maaltemada, empezando porl a ~ En gencrnl, el csrudian-
te con el número n, para cada 1 :S n .s 200, voltea una moneda cada n, comenzando con l a ~
a) ¿Cuántas veces será volteada la moneda número 200?
b) ¿Habrá otra moneda que sea volteada tantas veces como la 200?
e) ¿Habrá alguna moneda que sea volteada más veces que la 200?
19. ¿Cuántos productos diferentes pueden obtenerse al multiplicar cualesquiera dos enteros (dis-
tintos} del conjunto
a) {4, 8,16,32}? b) {4,8,16,32,64}?
e) {4,8,9,16,27,32,64,81,243}?
d) {4,8, 9, 16,25,27, 32,64,81,125,243,625, 729,3125}?
e) {p2,p3,p4,p',p6, q 1, q3, q', q', q6, ,-2, r,
4
T , r}, donde p, q, y rSOD primos distintos'!

20. Escriba u n programa (o desarrolle un algoritmo) para encontrar la factorización como produc-
to de primos de un entero n > l.
21 . En un triánguloABC, la longjrud del lado BC es 293. Si la longjtud del ladoAB es un cuadrado
perfecto, la longitud del lado AC es una potencia de 2, y la longirud del lado AC es el doble de
la longitud del lado AlJ, determine el perímetro del triángulo.
22. Exprese cada uno de los siguientes ejemplos en una forma más simple.

a)
...ñ(-lt
,
!]
b) D,<-lY,dondenEZ-

f¡(i + l)(i + 2)
d ) •;,,' (i ~ l)(i)
e) (-1)', donde n EZ-

•>íl--i-
.-,10-i+l
r) !!2n-\+ 1 ,donde nEZ-
Capítulo 4 Propiedades de los enteros: Inducción matemática

23. a ) Evalúe ú2' y [V·


b) Sia E R y n E z·, encuentre una fórmula para :Q:t1 y Qa(I_,>_
24. Use la notación de producto para escribir lo siguiente.
a) (!' + 1)(2' + 2)(3' + 3)(4' + 4)(5' + 5)
b) (l+x)(l+x')(i+x')(l+x')(l+x')
e) (1 + x)(l + x'Xl + x')(l + x')(l + x')(l + x")
d) (6)(24)(60)(120)

25. Demuestre que sin Ez· y n 2: 2,entonces fI (1-{) = (n + 1)/(2n).


1-2 J

26. ¿Cuándo tiene un entero positivo n exactamente


a) dos divisores positivos? b) tres divisores positivos?
e) cuatro divisores positivos? d ) cinco divisores positivos?
27. Sean E z·. Decimosque nes un enteroperfecto si 2nesigual a la suma de todos sus d i ~
positivos. Por ejemplo, como 2(6) = 12 = l + 2 + 3 + 6. se sigue que 6 es un entero perfecto.
a) Verifique que 28 y 496 son enteros perfectos.
b ) Si m E z·y Z-- l es primo, demuestre que i-:· 1(2• - t) es un entero perfecto. [El resultado
de Ja parte (d ) de l ejercicio 1 de la sección 4.1 puede ser de utilidad.)
28. Se puede utilizar el principio del buen orden para dar otra demostración del hecho de que.fi.
es irracional. Supongamos ~e .fi es raci~nal y consideremos el conjunto (distinto del vado)
S ~ z·dado por S = {a la42 E z·}.
Utihcecste conjunto S para obtener una contradiccióo.

4.6
Resumen y repaso hist órico

Según el matemático prusiano Leopold Kronecker (1823-1891 )...Dios creó los enteros, y
lo demás es obra del hombre... Todos los resu]tados de la investigación matemática más
profunda deben poderse expresar en última instancia en la forma simple de propiedades de
los enteros". En el espíritu de esta cita,. vimos en este capítulo cómo la obra del Todopode,-
roso ha sido desarro1lada ampliamente por el hombre en los últimos 24 siglos.
Si e mpezamos en el siglo cuarto [A.C.}, en los Elementos de Euclides no sólo encontra-
mos la geometría que aprendimos en el bachillerato, sino también las ideas fundamentales
de la teoría de números. Las proposiciones l y 2 del libro VII de Euclides incluyen un
ejemplo de algoritmo para determinar el máximo comtln divisor de dos enteros positivos
utilizando una técnica eficiente para resolver, en un número finiw de pasos, un tipo espe-
cífico de problema.
El término algoritmo, como su predecesoralgorism, era desconocido para Euclides. De
hecho, este término figuró en el vocabulario de la mayoría de la gente sólo a finales de
la década de 1950, cuando la revolución de la computación comenzó a tener efectos en la
sociedad. La palabra viene del nombre del famoso matemático, astrónomo y escritor de
libros de texto islámico Abu Ja'far Mohammed ibn MU~ al-KhowáriZIIU (cerca de 780 -
850). La última parte de su nombre, al-Khowarizm.i, que se traduce como "un hombre
de Ja ciudad de Khow§rizm", dio lugar al término algorism. La palabra álgebra viene de
4.6 Resumen y repaso histórico 239

Euclides (a,prox. 400 A.C.)

al- jabr, que está contenida en el título del libro de texto de al-Khowarizmt Kitab al-jabr
w'al muquabala. Traducido al latín durante el siglo trece, este libro tuvo un efecto profun•
do en los matemáticos de la época del renacimiento europeo.
Como lo mencionamos en la sección 4.4, el uso que damos a la palabra algoritmo tiene
la connotación de un método preciso, que se sigue paso a paso, para resolver un problema
en un número finito de pasos. La primera persona a quien se atribuye el mérito de haber
desarrollado el concepto de un algoritmo de computador fue Augusta Ada Byron ( 1815-
1852), condesa de Lovelace. Hija única del famoso poeta Lord Byron y Annabella
Millbanke,AugustaAda fue criada por su madre, quien estimuló sus talentos inteleccuales.
Con una enseñanza en matemáticas impartida por personajes de la talla de Augustus De
Margan (1806-1871), continuó sus estudios ayudando al talentoso matemático inglés
Charles Babbage (1792- 1871) en el desarrollo del diseño de una de las primeras máquinas
de cálculo, 1a ..máquina analítica". Los apuntes más completos sobre esta máquina se
encontraron en sus escritos, donde también puede encontrarse un gran talento literario y la
esencia de los algoritmos modernos para los computadores. En el capítulo 2 del libro de
S. Augarten [l] se pueden encontrar más detalles acerca de la obra de Charles Babbage y
Augusta Ada Byron Lovelace.
En el siglo posterior a Euclides, encontramos algo de teoría de números en la obra de
Eratóstenes. Sin embargo, fue cinco siglos después cuando Diofanto de Alejandría obtuvo
los primeros grandes logros en este campo. En su obraArithmetica, sus soluciones enteras
de ecuaciones lineales (y de orden superior) fueron como un faro matemático que guiaba
hacia la teoría de números, hasta que el matemático francés Pierre de Fermat ( 1601-1665)
entró en escena.
El problema que presentamos en el teorema 4.8 fue investigado por Diofanto y analiza.
do posteriormente, durante el siglo vn, por los matemáticos de la India, pero sólo hasta la
década de I 860 fue resuelto completamente por Henry John Stephen Smith ( 1826-1883).

• Si desea conocer más detalles acerca de la obra de éstos y otros matemáticos que se han
dedicado al árcade la teorfade números, consulte L. Dickson [4]. El capítulo 5 de l. Ni ven
240 Capítulo 4 Propiedades de los enteros: Inducción matemática

Augusta Ada Byron, condesa de Lovelace (1815-1852)

y H. Zuckerman [10) trata de las soluciones de las ecuaciones diofánticas y de sus aplica.
cienes.
En la obra Formulario Matematico, publicada en 1889, Giuseppe Peano (185&---1932)
formuló el conjunto de 1os enteros no negativos con base en tres términos indefinidos:
cero, número y sucesor. Su formulación es la siguiente:

a) Cero es un número.
b) Para cualquier número n, su sucesor es un número.
e) Ningún número tiene a cero como su sucesor.
d ) Si dos números m, n tienen el mismo sucesor, entonces m ~ n.
e) Si Tes un conjunto de números donde O E T, y donde el sucesor de n está en 1
siempre que n esté en T, entonces Tes el conjunto de todos los números.

En estos axiomas se ve la estrecha relación entre el concepto de orden (sucesor), la técnica


de inducción matemática y la idea de nllmero (es decir, Jos enteros no negativos). Peano
atribuyó la formulación a Richard Dedekind (1831- 1916), quien fue el primero en desa-
rrollar estas ideas; sin e mbargo, estos axiomas se conocen en general como los ..axiomas
de Peano".
El primer europeo que aplicó el principio de inducción en las demostraciones matemá-
ticas fue el científico veneciano Francesco Maurocylus (1491-1575), cuyo libro de arit-
mética, publicado en 1575, contenía dicho trabajo. En el siglo siguiente, Pierre de Fermat
hizo algunas mejoras a la técnica de su obra mediante el ..método de descenso infinito".
Blaise Pascal (cerca de 1653), al demostrar resultados de combinatoria como C(n, lc.')IC
(n, k + l) = (k + 1)/(n - k), O :s k :s n - 1, utilizó la inducción y se re firió a esta técnica
como el crabajo de Maurocylus. El término actual inducción maumática no fue utilizado
4.6 Resumen repaso histórico 241

hasta el siglo XIX, cuando apatt,ció co la obra de Augustus De Morgan ( 1806-1871 ). En


1838, El describió el proceso con gran cuidado y le dio el nombre de inducci6n matemáti-
ca. (Un interesante examen de este tema aparece en el anfc~o de W. H. Bussey (2].)
El texto de B. K. Youse (13] ilusa,, mllltiples y variadas aplicaciones del principio de
inducción matemática en álgebra, geometría y trigonometría Si desea más información
acerca de la pertinencia de este método de demostración en los problemas de programa-
ción y el desarrollo de algoritmos, el texto de M. Waod ( 12] (particularmente el capítulo 2)
proporciona una base amplia y ejemplos.
Es posible encontrar más detalles acerca de la teoría de nWlleros en los textos de
G. Hardy y E. Wright (5], W. J. LeVeque (7, 8Je l. Niveo y H. Zuckcnnan (10). En uo nivel
comparable al de este capítulo, el capítulo 3 de V. H. Lamey [6] proporciona una agrada-
ble introducción a este material. El texto de K. H. Rosen [11) integra aplicaciones en
criptografía y ciencias de la computación en su desarrollo del tema. El anícuJo de M. J.
CoUison [3] examina la historia del teorema fundamental de la aricmtcica. Los artículos
que aparecen en [9] hacen un recuento de algunos desarrollos interesantes en la teoría de
números.

BIBLIOGRAFÍA

l. Auganen. Stan. Brr by Bn; An /Uwsrraud Hisrory o/ ú,mpuren, Nueva York. 1ick.nor &
Fields, 1984.
2. Bussey, W. H. ...Origins of Mathema1ical lnduction", Amuican MOIMmatical Monthly 24.
1917, pp. 199---207.
3. CoUison. Mary Joan, ·-nte Uoique Factoriz.atioa'lbcorem: From Euclid toGauss... MalhemaLics
Magatin< 53, 1980, pp. 96-100.
4. Dickson, L., Hi.rtory o/ the Theory o/ Nwnlurs, Washington, O.C., Came¡ie lnstitulion of
Washingt00, 1919. ReimprC$0 por O>elsea, en Nueva York, 1950.
5. Hardy, Godfrcy Harold, y Edward MaitlandWrighc, An lntroducrion io the TMoryo/Numbers,
s•. cd., OxJord, Oxford University Press. 1979.
6. Lamey, Violet Hachmeister, Absrmct Algd>ra: A First Cowrse, Boston. Prindle. Weber &
Schmidt, 1975.
7. LeVeque, Wil1iam J.• Ekmenrary Theory ofNumbtrs, Reading. Mass.. Addison-Wesley, 1962.
8. LtVeque, WiUiam J•• Topics in Number Tkory, \."Ols. 1 y 11. Reading, Mass.,Addison-Wesley,
1956.
9. LeVeque, William J. (editor),Srudies in Number TMory, MAA Studies in Mathematics, vol. 6,
En¡lewoods Cliffs, NJ., Prentice-Hall, 1969. Publicado por la Matbemacic.al Association of
America.
10. Niven. lvan y Herbert S .• Zuckerman. An lnrroduction to the Thory o/ Numben, 4•_ ed.,
Nueva Yort. Wtley, 1980.
1 J. Rosen. Kenneth H., ~nrary Numbtr Thtory, 3•. ed., Reading, Mass., Addison-Wesley.
1993.
12 Wand, Mitchell.lnduction. Rtcwsion, and Programmiltg, Nueva York. Elsevler Non.h Holland,
1980.
13. Youse. Bevan K.• MatMmalical lnduction. Englewoods Qiffs, N.J., Prentice-Hall, 1964.
Capítulo 4 Propiedades de los enteros: Inducción matemática

d ) Verifique la conjetura de la parte (c) para todo11E


EJERCICIOS z·mediante el principio de inducción matemática.
COMPLEMENTARIOS
7. Paran E z·. use la inducción matemática para mostm: 18.

. Sean a, d enteros fijos. Detennine una fórmula para la


que
i i' n(n + 1)(2n + 1)(3n' + 3n -1).
""y
nade a+ (a + d) +(a+ 2d) + · · · +(a+ (n- l)d). para
: z•. Verifique su resultado mediante inducción matemá- f- l 30
19.
8. Sean E Z•, n impar y no divisible entre 5. Detnues'R
En el síguiente segmento de programa en Pascal, las que existe una potencia den cuya cifra de las unidades es l.
iablcs n y sum son variables enteras. Desputs de ejecu- 9. Para cualquier n E Z, n ~ O, demuestre que
.c este segmento de programa, ¿cuál es el valor de n im- a) 211o+1 + 1 es divisible entre 3.
,so por la proposición Writeln? b) n3+(n+ IY+(n + 2)1 esdivisible entrc9.
n 1 n' lln
: "' 3; e) 7 + 3 + 21 es un entero.
a : = O;
ile SUIII < 1 0 ,000 do
Begin
10. Verifique que paracualquier n E z·. tal que sene -o.
entonces
= n + 7;
2
sen8+sen38+sen58+ ••+sen(2n -1)8-= seo n0.
iteln ('El valor den e s · , n:0, · .•);

11. Sin E Z• y n 2:. 2, dc~uestre que 1!'


""ª
<(2:) <4•.
Consideremos las siguientes seis ecuaciones.

Q 1 -1 12. Sin E z·, demuestre que 57 divide a 7...1 + s>1.


l) 1-4• - (1+2) 13. Para cualquier n E z·. muestre que
• 1-4+9 = 1 + 2+3 a) si n 2: 64, entonces n puede escribirse como una
4) 1-4+9-16=-(1+2+3+4) suma de milltiplos de 5, de 17 o de ambos.
5) 1-4+9-16+25= 1+2+3+4 +5 b) sin 2: 108,entoncesnpuedeexpresarsecomouna
6) 1 -4+9-16+25-36 • - (1+2+3+4+5+6) suma de mlmeros 10 y 13.

njeture la fórmula general suge rida por e stas seis 14. s; n E z·, demuestre que ±-2..., = f (- l Y-' (!).
ociooes y demuestre su conjetura. +,
1-1 n ;-1 r

Para n E z·, demuestre lo siguiente por inducción 15. Dadon E Z•escribamos r =r0 +r1 • IO+r2 • 1()1 + · · ·+
temática: r.-10",dondcO :S r, :s 9para 1 :s i :Sn-1 y0<r.:s9.
a) Demuestreque9 l rsiys61osi9!(r. +r_1 + •·+
a) 5 l(n'-n) b) 6 l(n'+5n) ri+r1 +ro).
b) Demuestre que 3 I rsi y sólo si 3 !(r. + r...1 + · • • +
Para todo n E z•. sea S(n) la proposición abierta: n + 2
r 1 +r1 + ro>.
41 es primo.
e) Si , = 137486.¡225, donde x es un solo dígito, de,.
a) Verifique que S(n) es v«<ladera para todo 1 S • S 9.
temtinc el valor o valores de x de modo que 3 It.
b) Si S(k) C$ verdadera. ¿implica esto que S(k + J) es
¿Qut valores de x hacen a t divisible entre 9?
verdadera para todo k E z•?
16. Francisco gasta $6.20 en dulces para darlos como pre-
Paran E z·, defina la suma s. mediante la fórmula mio en un concurso. Si una caja de I0onzasdeestosdulces
cuesta SO.SO y una caja de 3 onzas cuesta $0.20, ¿cuántas
3
=.!.+ ! +!+· ·· + (n- l)+ _ n_ _ cajas compró de cada tamai'l.o?
" 2! 3! 4! n! (n + 1)!
17. a) ¿Cu:buos enteros positivos podemos expresar como
un pr<xlucto de nueve primos (se permiten repeti-
a) Verifique que s1 = .!. , s1 = 1 y s1 -= B. . ciones y oo importa el orden). si los primos pue-
2 6 24
b) Calcules_.. s, y s, . den elegirse del conjunto {2, 3, 5, 7, 11}?
e) Con base en los resultados de las partes (a) y (b). b) ¿Cuántos de 1os enteros positivos de la parte (a)
• conjeture una fórmula general para la suma de los tienen al menos una ocurrencia de cada uno de los
términos de s•. cinco primos?
Ejercicios complementarios 243

e) ¿Cuántos de los resultados de la parte (a) son 2 1. Evalúe las siguientes cantidades.
divisibles entre 4? ¿CUántos de los resultados de la
parte (b)?
Sea.na. b, e, d, m, n, s, t E z •conad - bc= 1.s =am +
•>D(ti) b);t(~(i+2)) c)}](D;)
yt = cm +dn. 22. ¿Olándo tiene un entero positivo n exactamente 15 di-
a) Determine m, n en términos de s, r. visores positivos?
b) Demuestre que mcd(s, t) = mcd(m. n).
23. Consideremos el conjunto {l. 2, 3}. Eoestecaso,pode-
. a} Diez estudiantes entran a unos vestuarios que con- moscscribir [I, 2, 3) = {l. 2) U {3), donde 1 + 2= 3. Para
tienen 1O gavetas. El primer estudiante abre todas el conjunto {l. 2, 3, 4 }, tenemos que { 1, 2, 3,4} = { 1, 4} U
las gavetas. El segundo estudiante cambia el esta- {2. 3}.dondc 1 + 4= 2 + 3. Sin embargo, lascosasc:ambian
do (de cerrado a abierto o viceversa) de cada gave- cuando analizamos el conjunto {l, 2. 3, 4, 5}. Si C ~ { l. 2,
ta en forma alternada, a partir de la segunda gave- 3, 4, 5} y se denota la suma de los elementos de C. vemos
ta. El tercer estudiante cambia entonces el estado que no hay forma de escribir { 1,2,3,4,5} = A U BconA n
de cada tercera gaveta, a partir de la tercera. En B= 6ys" =s,.
general. para l< k s 10, el k-ésimo estudiante a) ¿Para cuáles n E z•.
n ~ 3, podemos escribir
cambia el estado de cada k gavetas, a partir de la {l,2. 3, . .. ,n} =A U B,conA n B = f ys"=s,?
k--ésima. Oesputs de que el ~imo estudiante ha (Como antes, s" y s, denotan las sumas de los ele-
pas~ por todas las gavetas, ¿cuáJes de ellas con- mentos de A y B. respectivamente.)
tinllan abienas? b) Sta n E Z♦• n ~ 3. Si podemos escribir {l. 2, 3,
b) Responda la parte (a) si reemplazamos 10 por n E ... ,n} = A u B,conA n B = ey SA~ s,.
desaiba
z•.n ~ 2. cuántos de estos conjuntos A y B podemos deter-
z·.
lO. SeaA = {a1 , ara,, a... a,J !: Demuestre que A con- minar.
licne un subconjunto no vado S tal que la suma de los ele-
aentos deS es un múltiplo de 5. (Es posible tener una suma
~conste solamente de un sumando.)

5
Relaciones
y funciones

E o este capítulo extenderemos la tcoña de conjuntos del capítulo 3 para incluir los con-
ceptos de relación y función. Las funciones intervienen en el álgebra. la trigonometría y
el cálculo. Aqul, sin embargo, estudiaremos las funciones desde el punto de vista de la
teoría de conjuntos (incJuyendo las funciones finitas), y presentaremos algunas ideas nue-
vas que deberán tenerse en cuenta en el estudio de este tema. Además:, examinaremos el
concepto de complejidad de una función y su papel en el estudio del análisis de algoritmos.
Tomaremos una ruta a lo largo de la cual encontraremos las respuesw de los siguientes
seis problemas (muy relacionados entre sQ.

1) El Depanamento de Defensa tiene siete contratos distintos relacionados con un pro-


yecto de alta seguridad. Cuatro empresas pueden fabricar las distinw piezas a que
se refiere cada contrato; para maximizar la segwidad de todo el proyecto, es mejor
que cada una de las cuatro empresas se dedique a una pieza. ¿De cuántas maneras
pueden asignarse los contratos para que todas las empresas participen?
2) ¿En cuántas secuencias cuaternarias (O, 1, 2. 3) de siete dígitos hay al menos una
ocurrencia de cada uno de los digitos O, 1, 2, y 3?
3) Una matriz. m x n cuo-ww es una matriz A con m filas y n columnas, taJ que en la
filai, para todo 1 sis m.,yen lacolumnaj,paratodo 1 Sj s n, Jaeotradaaf que
aparece es O o l. ¿Cuántas matrices cero-uno de 7 x 4 tienen exactamente un 1 en
cada fila y al menos un 1 en cada columna? (La matriz. cero-uno es una estructura
de datos que se origina en ciencias de la computación.Aprenderemos más acerca de
ella en capftulos posteriores.)
4) Siete personas (no relacionadas entre sQ llegan a la recepción de un edificio que
tiene cuatro pisos más, y entran en el ascensor. ¿Cuál es la probabilidad de que el
ascensor se detenga en cada piso para dejar a los pasajeros?
5) Para los enteros positivos m., n con m < n, demuestre que

f (- 1)' ( n-n k )(n - k)"'•O.


a-o

6) Para cada entero positivo n, verifique que

245
246 Capitulo 5 Relaaones y funoones

¿Nota la relación entre los primeros cualrO problemas? Los primeros tres son el mism
problema pero en diferentes contextos. Sin embargo. no queda tan claro que los t1ltm111
dos problemas se reJacionen o que e.xista a1guna conexión enue elJos y Jos primeros cm-
tro. No obtante, estas identidades se establecerán mediante las mismas técnicas de coou,:
que desarrollaremos para resolver los primeros cuatro problemas.

5.1
Productos cartesianos
y relaciones

Definición 5.1 Para los conjuntos A. B ~ :\l. el producto cartesiano, de A y B se denota con A x By es
igual a ((a, b) 1a E A. b E 8).

Decimos que los elementos de A x B son pares ordenados. Para (a, b). (c. d) E Ax 8,
tenemos que (a. b) = (e, d) si y sólo si a= e y b = d.
Si A, B son finitos, se sigue de la regla del producto que I Ax B 1 : 1A 1· I B Auru¡,,c ¡:
generalmen1e no ocurre que A X B : B x A. tendremos que I Ax B 1 : 1B x A j. Así
mismo. aunque A, 8 ~ :iti., no es necesario que A X B !';'. lU, por lo que a diferencia de la
unión y la intersección, ~('.ill) no necesariamente es cerrado en esta operación binaria.
Podemos extender la definición del producto cartesiano para más de dos conjuntos. Si
nE z·. n~3 yA1tA:, ... ,A.~ \(.entonces el producto (deorden n) deAi,A2, ••• ,A.se
denota con A1 x A1 x · - - x A,. y es iguaJ a {(ai,Qz, ... , a.) 1a, EA,. J sis n}.t A los
elementos deA 1XA: X -- - x A. se les Uaman-up1as ordenadas, aunque generalmente usemos
el trnn.inoumaen lugar de 3-upla. Como en e1 caso de los pares ordenados, si (ah~• ... ,aJ.
(b,, b 1, ••• ,b,.) E A 1 xA 1 x- • • xA•. entonces (ai,a:•.... a.) :;;; (b 1• b 2, ••• ,b.) si y sólo si
a,:;;; b,, para todo I s i .s n.

Ejemplo5.1 j Sean ~:{1,2, 3, ... ,7},A= {2,3,4}, B= {4, 5}. En1onces

a) A x B : {(2 , 4), (2,5), (3, 4), (3,5), (4,4), (4, 5)}.


b) B XA •{(4,2),(4,3),(4,4),(5,2),(5,3),(5, 4)}.
e) B' : B x 8: {(4, 4), (4, 5), (5, 4), (5, 5)}.
d) B' : B x 8 x B : {(a,b,c)ja,b,c E B}; porejemplo, (4,5,5) E 8 3 •

t Cu-ando ullliccmos el produao cartestaOO de ucs o más conJuntos. debemo5 tener cuidado coa la falta dr
asoaat:1vldad. En el caso de tres coa1umos. por e,emplo, hay uu dtfCCCDcia enr.re c:oalesquícra dos de b
COCIJUDIO$A1 XA~ X A,. tAt X A::) X A, J A1 X (Al X A,) ya que $U$ respectivos elementos SOD temu ordenada
(a,. a1, o,) y los para ordell3dos diferentes ((a 1. oJ, aJ) y (a1• (a:_.o,)). Aunque tales diferencias son importan-
tes en c1crus situaciODCI, aquí no nos con.centraremos en ellas y u"1"Cmos siempre la forma sin paR:ntcsis A,
x A1 x Al- Tambitn coc~-c:ndttmos csio a1 tratar coa el producto cartesiano de cu.au-o o m5s ele.meatos.
S. 1 Productos cartesianos y relaciones 247

Si <l\l = R, R x R = ((x,y) 1 x, y E R) se reconoce como el plano -de lageometñacon


coordenadas y de) cáJculo de dos dimensiones. El subconjunto R•x R· es el interior del
primer cuadrante de este plano. De la misma forma. R 3 representa el espacio tridimensional
euclídeo, donde el interior tridimcnsionaJ de cualquier esfera (de radio positivo). los pla-
nos de dos dimensiones y las líneas de una dimensión son subconjuntos de importancia.

Se realiza un experimento i de la siguiente manera: Se tira un solo dado y se anota el


resultado; despu6s se lanza una moneda al aire y se anota el resultado. Determinaremos un
espacio muestra! ::f para %.
Sea %, la primera pane del experimento%, y sea::/,= { 1, 2, 3, 4, 5, 6} un espacio
muestral pan, %, .Asfmismo, sea::/,= {cara, cruz} un espacio muestra! para%,, la segun-
da parte del experimento. Entonces E/ "" E/1 x Y, es un espacio muestra) para i.
Este espacio muestral puede representarse gráficamente con un diagramad~ árbol que
presenta todos los resultados posibles del experimento !. En la figura 5.1 tenemos tal
diagrama de árbol, que procede de izquierda a derecha. Desde et punto del extremo iz-
quierdo, seis ramas originan los seis resultados de la primera etapa del experimento 1:.
Para cada uno de los puntos, numerados como 1, 2, . .. • 6, dos ramas indican los resulta-
dos subsecuentes del lanzamiento de la moneda. Los doce pares ordenados que se obser-
van en los extremos de la derecha constituyen el espacio muestral Y.

- - - (1, C.,a)
1
- - - - - - - - . (1. Cara)
_ _ _ (2,Ctuz)

2 - - - - - - - - . (2, C.ra)
- - - (3, Cruz)
3
- - - - - - - - . (3, Cruz)
- - - {4, Clra)
4
- - - - - - - - . (4. Ouz)
- - - (S, C.,a)
5
- - - - - - - - . (S. Cruz)
- - - (6, Cara)
6
- - - - - - - - . (6, Cruz} Figura 5.1

Ade más de su estrecha relación con los productos cartesianos. los diagramas de árbol
también aparecen en otras situaciones.
248 Capítulo 5 Relaciones y funciones

,~ ·, ;~ ,' En el campeonato de tenis de Wimbledon, las mujeres juegan como máximo tres setsenz
encuentro. La ganadora es la primera que gana dos sets. Si denotamos con N y E a cb-
jugadoras, el diagrama de árbol de la figura 5.2 indica las seis fonnas en que puedeg.anl!Se'
este encuentro. Por ejemplo, el segmento de línea con un asterisco (arista) indica qut"b..
jugadora E ganó el primer set. La arista con doble asterisco indica que N ha ganado el
juego al ganar el primer y el tercer set.

Primtr conjunto Segundo Tercer conjunto


conjunto (en caso necesario)
Figura 5.2

Los diagramas de árbol son ejemplos de una estructura genera] llamada árbol. Los
árboles y los grafos son estructuras importantes que aparecen en la ciencia de la computa-
ción y la teoría de optimización, que anaJizarcmos en capítulos posteriores.

Volvamos al producto cancsiano de dos conjuntos para ver que los subconjuntos de
esta estructura son de gran interés.

Defin ición 5 .2 Para los conjuntos A. B ~ SU, cualquier subconjunto A X Bes una relación de A en B.
Cualquier subconjunto de A X A es una relación binaria en A.

i:/~ :~'."~,. Con A, B, útl como en el ejemplo 5.1, las siguientes son relaciones de A e n B.

a) 0 b) {(2,4)}
e) {(2, 4), (2, 5)} d) {(2,4),(3,4),(4,4)}
e) {(2, 4), (3, 4), (4, 5)) f) Ax B

Como IAX B 1= 6, se sigue de la definición 5.2 que existen 26 posibles relaciones de A


en B.
5.1 Productos cartesianos y relaciones 249

En general, para conjuntos finitos A, B con I A 1 = m y \ B 1= n, existen 2- relaciones


de A e n B, incluyendo la relación vacía y la propia relacíón A x B.
También existen 2_...,( =2-) relaciones de Ben A , una de las cuales es también 0 y
otra e s B x A . La razón por la que obtenemos el mismo número de relaciones de B en A
que de relaciones de A en Bes que cualquier relación ~ 1 de Ben A puede obtenerse a
partir de una única relación ~ 2 de A en B, invirtiendo simplemente las componentes de
cada par ordenado en 9i:2 •

SeaB= ( l. 2 } ~ N ,"U = ~ (B)yA = "11 = {0, (1 ), (2 }, {1, 2}}.El siguiente es un ejemplo


de una relación b;naria en A: ffi = {(0, 0), (0, ( 1}), (0. (2 }), (0, { 1, 2}), ({ 1), {1 }), ({ 1).
{ l. 2}), ({ 2 ). {2 }), ({ 2}, { 1, 2 }), ({ 1, 2). ( 1, 2 })}. Podemos decir que la relaciónffi es la
relación inclus;ón, donde (C. D) E ffi si y sólo si C. D ~ By C ~ D.

Con A ;;:Gij. = Z· , podemos definir una relación binaria~ en el conjunto A como {(.:e, y) 1x
:S y} . Ésta es la conocida relación "es menor o igual que" para el conjunto de enteros
positivos. Puede representarse gráficamente como el conjunto de puntos, con componen-
tes enteras positivas, localizadas en o arriba de la línea )' = x en el plano enclídeo,t como
se muestra parcialmente en la figura 5.3.Aquí no podemos enumerar ta relación completa
como lo hicimos en el ejemplo 5.6, pero notamos, por ejemplo, que (7, 7), (7, 11) E ~.
pero (8, 2) ~ ffi. El hecho de que (7, 11) E ffi también puede ser denotado con 7 ffi 11;
(8, 2) ~ ffi se expresa como 8 ffi 2. En este caso, 7 ffi 11 y 8 ffi 2 son ejemplos de la
notación inftja para una relación.

2 3 4 x Figura 5.3

Nuestro último ejemplo nos ayudará a revisar la idea de un conjunto definido en forma
recursiva.

~ ' Sea@! el subconjunto de N x N donde ffi = ( (m, n) 1n = 7m}. En consecuencia, entre los
pares ordenados enffi se encuentran (O, O), (1, 7), ( 11, 77) y (15, l05). Esta relación ffi en
N tambi6n puede representarse en forma recursiva como
1) (0,0) E 9/!; y
2) Si.(s, r) E ie/l, entonces ( s + 1, r + 7) E ffi.

tTambifo se conoce como ··ptan6 euclidiano"' . (N. del E.)


250 capítulo 5 Relaciones y funciones

Usamos la definición recursivaparamostrarquecl par ordenado (3, 2l)(deNx N) estáe1


9t Nuestra forma de obtenerlo es la siguiente: Por la parte (1) de la definición recursiva.
partimos de (0, O) E 91. Entonces la parte (2) de la definición nos da

i) (0,0)E9t9(0+1,0 +7) = (1,7) E 9t;


ü) (l,7) E 9t9 (1+ 1,7+7) = (2,14) E9t; y,
iü) (2, 14) E 9t9 (2 + 1, 14+ 7) = (3,21) E 9t.

Cerraremos esta sección con a1gunas observaciones finales.

e.
1) Para cualquier conjunto A ~ '11, A x 0 = 0. (Si A x 0 -# sea (a, b) e A x ,.
Entonces a E A y b E 0. ¡Imposible!). De la misma forma, 0 x A = 0.
2) El producto cartesiano y las operaciones binarias de unión e intersección están rela-
cionadas como se muestra en el siguiente teorema.

TEOREMA 5.1 Para cualesquiera conjuntos A, B, C ~ 6\1:


a) A X (B n C) = (A X B) n (A X C)
b) A x (B U C) = (A X B) U (A x C)
e) (A n B) X e= (A X C) n (B X C)
d) (A U B) XC= (A XC) U (B x C)
Demostración: Demostraremos el teorema 5.J (a) y dejaremos las demás partes al lector.
Usaremos el mismo concepto de iguaJdad de conjuntos (como en Ja definición 3.2 de la
sección 3.1), aunque en este caso los elementos son pares ordenados. Para a, b E 6\1, (a, b)
EAx (B n C)<a>a E Ayb E Bn C<a>a E Ayb E B, C <,,> a EA, b EB,y aEA,b
E e ... (a, b) E A X By (a, b) E A X e.,,. (a, b) E (A X B) n (A XC).

EJERCICIOS 5.1 1. Sí "li = N, A = { 1, 2, 3, 4 ), 8 = {2, S} y C = {3, 4, 7). determine Ax 8; 8 x A; A U (8 xC);


(A U B) x C:(A x C) U (8x C).
2. Si "li = ( 1, 2. 3, 4, 5},A = {I, 2, 3)y 8 = (2. 4. 5}, dé ejemplos de
a) tres relaciones no vacías de A en B.
b) tres relaciones binarias no vacías en A.
3. SeaA = (1, 2,4, 8, 16} y 8= {I, 2, 3,4, 5, 6, 7). Si (2-x. S). {4,y -2) EA x8, ¿se cumple
que (2 - x, S) = (4, y - 2)?
4. SeaA, = (0, 1, 2, 3,4) ,A,= {I, 3, 7, 12},A,= {O, l. 2, 4, 8, 16, 32) yA.= {- 3,- 2, - 1, O. l. 2, 31.
a ) Sea9l:1 ~ A1 xA 2 x A3 xA, donde~1 = {(w, x, y. z) 1 w.xyz = O}. ¿Cuántas 4--uplas ordenadas
(o cuaternas) hay en una relación cuatemaria9?1 ?
b) Si91 2 ~ A,xA 1 xA,xA,es la relación cuaternaria donde (a., b, e, d) E 9i:2 si (y sólo sí) ob
cd < O, ¿cuánto vale ISt2 I?
S. Para. A, B, ~ como en el ejercicio 2, determine lo siguiente:(a) 1A x B 1: (b) el m'.imcro de
• relaciones de A en B: (e) el número de relaciones b inarias en A; (d) el nilmero de relaciones
de A en B que contienen (1, 2) y (1, S); (e) el número de relaciones de A en B que conticncn
5.2 Funciones: en eneral e in ivas 251

exactamente cinco pares ordenados; y (0 el mlmero de relaciones binarias en A que contienen


al menos siete elementos,
6. ¿Para cuj}cs conjuntos A. B ~ '.\l. es ,·Cfdadero que A x 8 = B x A?
7. Para un universo dado '\l. sean A, B. C. D subconjuntos no vacíos de!\l.
a) Demuestre que A xB !: CxDsi y sólo si A C: CyB CD.
b) ¿Qu~ le sucede al resultado en la pane (1) si cualquiera de los conjuntos A, B, C. Des
vacío?
8. La final masculina en Wimblcdon es ganada por el primer jugador que gane eres de cinco sets
en un juego. S1 C y M denotan a los jugadores, dibuje un diagrama de árbol que muestre todas
los formas posibles en que se puede decidir el Juego.
r
9 . Si ~ ,.. R , haga un esquema de la relación {(x. y) 1 + y2: 4 }. ¿Quésucedc si :U es R·?
10. Se sacan algunos microc1rcuitos lógicos de un contenedor. se prueban individualmente y se
etiquetan como dcfcctuosos o buenos. El proceso de prueba contimla hasta que se encuentran
dos defcccuosos o hasta haber probado cinco microcircunos en rotal. Utilice un diagrama de
árbol y presente un espacio muestra! para este proceso.
11. Complete la demostración del teorema 5.1.
12. Un rumor se difunde como sigue. El que lo origina llama a dos personas por tel~fono. Cada
una de estas personas telefonea a tres amigos. cada uno de los cuales a su vez llama a otros
cinco más. Si nadie rttibe más de una llamada y nadie llama al que k> originó, ¿cuá.nw pcrso--
nas conocen ahora el rumor7 ¿Cufntas llamadas telefónicas fueron hechas?
13. Para A, B, C ¡; 'U.demuestre que Ax (B-C) = (A xB)-(A x C).
14. Sean A, Bconjuntos con I B 1= 3. $1 existen4096 relac1onesdcA en B. ¿cufnto vaJe I Aj?
15 . ScaSJI: !;;;;; N x N donde (m, n) E 9t si (y sólosi)n = 5m + 2.
a) Dé una definición rccu.rsh·a para St
b) Utibcc la definición recursiva de Ja parte (a) para mostrar que (4. 22) e 9l
16. a) Dé una defirución tea1rsiva para la relación 911: !: z•x z• donde (m. n) E 9i si (y sólo si) m ~ n.
b) En la definición de la parte (a), verifique que (5, 2) y (4. 4) esttn en 9l.

5.2
Funciones: en g eneral
e inyectivas

En esta sección nos concentraremos en un tipo especial de relación l.lamadafanci6n. Uno


encuentra muchas funciones en el medio matemático y en el de las ciencias de la compu-
tación. Sin embargo, como en el caso de las relaciones, no hay razón para temer; no las
estamos abandonando, sino que reaparecerán en el capítulo 7, donde las examinaremos
con más cuidado.

Definición 5.3 Para los conjuntos no vacíos A. B, una/unción, o aplicaci6n.fde A en 8 , que se denota con
/:A ➔ B. es una relación deA en Ben laquecadaelemcn1odeA aparece ex.actamcnte una
vez: como la primera componente de un par ordenado en la relación.

Con frecuencia escribimos/(a) = b cuando (a, l,) es un par ordenado en la función/. Si


(a, b) E/, b se conoce como la imagtn de a mediante f. mientras que a es una preimagtn
de b. Además, la definición sugiere que/ es un método para asociar a cada a E A una
252 Capítulo 5 Relaciones y funciones

única b E B; denotamos este proceso con f (a) = b. En consecuencia. (a, b), (a, e) Ej
implica b = c.

[ ~ ' ,};! Si A= ( 1, 2, 3} y B = {w,x,y, z),f= ((1, w), (2,x), (3,x)} es una función, yen consecuen-
cia una relación, de A en B. !!11, = ( (1, w), (2, x)} y 91, = ( (1, w), (2, w), (2, x), (3, z)} soo
relaciones, pero no funciones, de A en B. (¿Por qué?)

Definición 5.4 Para la funciónf: A ➔ 8, A es el dominio defy Bes el codominio def. El subconjunto de
B formado por aquellos elementos que aparecen como segundas componentes de Jos pares
ordenados de/se conoce como la imagen de/y se denota también comof(A) ya queesd
conjunto de imágenes (de los elementos de A) mediante f.

En el ejemplo 5.9, el dominio dejes (1, 2, 3}, el codominiodefes (w,x,y, z) yla


imagen defesf(A) = {w, x).

Una representación gráfica de estas ideas aparecen en la figura 5.4. Este diagrama su-
giere que a puede verse como una entrada que es transformada por/en la salida corrcs-
pondiente,/(a). En este contexto, puede pensarse en un compilador de FORTRAN como
una función que transforma un programa fuente (la entrada) en su correspondiente progra-
ma objeto (la salida).

A Figura 5.4

En las ciencias de la computación aparecen muchas funciones interesantes.

a) Una de las funciones que aparece en el estudio de las estructuras de datos y el


análisis de algoritmos es la función parte entera, o función suelo. Esta función/:
R ➔ Z está dada por

/(x) = LxJ = el mayor entero menor o iguaJ que .t.

En consecuencia,f(x) = x, si x E Z; y si x E R - Z,/(x) es el entero inmediato a la


izquierda de x en la recta numérica real.
5.2 Funciones: en general e inyectivas 253

Para esta función tenemos que


l ) [3.8J =3, l3J = 3, l- 3.8J = - 4, l- 3J =· - 3;
2) [7.1 +8.2J =[l5.3J = l5=7+8 = [7.lj+[8.2J; y,
3) [7.7 + 8.4J = [16.lj = 16 # 15 = 7 + 8 = [7.7J + [8.4j.
En el lenguaje de programación BASIC, esta función se implanta mediante la fun-
ción predefinida INT. Con esta función vemos que

INT(S. l) = 5 = INT(S) INT(-7.8) = -8 = INT(-8).

b) Una segunda función, relacionada con la función suelo de la parte (a) y que cumple
un papel en el estudio de las ciencias de la computación es la función techo. Esta
función g: R ➔ Z está definida por

r
g (x) = Xl = el menor entero mayor o igual que X.
Así,g(x) =x cuandox E Z, pero cuandox E R -Z,g(x) es el entero inmediato a la
derecha de x en la recta numérica real. Al usar la función techo e ncontramos que
l) f3l = 3, [3.01] = f3.7] = 4 = f4l;
2) f- 31 = - 3, f- 3.0ll=f-3.71 = - 3;
3J f3.6 + 4.51 = f8.ll = 9 = 4 + s = f3.6l + f4.5]; y,
4) [3.3 + 4.2] = [7.51 = 8 # 9 = 4 + 5 = [3.31 + [4.21.
e) La función trunc (de truncar) aparece in Pascal y es una función de valores enteros
definida en R. Esta función elimina la parte fraccionaria de un número real. Por
ejemplo, trunc(3.78) = 3, trunc (5) = 5, trunc (- 7.22) = - 7.
d ) Al guardar una matriz en una tabla unidimensional. muchos lenguajes de programa-
ción lo hacen por filas, con el método de la fila principal. En este caso, si A -= (a,)..,.
es una matrizn x n, la primera fila de Ase guarda en los lugares 1, 2, 3, ... , n de la
tabla si comenzamos con a 11 en el lugar 1.

ª'' 1- --1
1 2 · · · n n+l n+2 · ··2n2n+l · ·· (i - l)n+j · ·· (n - l)n+n(=n')

El elementoa21 se encuentra entonces en la posición n + l, mientras que el a 34 ocupa


la posición 2n + 4 en la tabla. A fin de determinar el lugar de cualquier elemento a ;¡
de A. donde 1 :S i,j :S n. se define lafunci6n de acceso f de los elementos de A en
las posiciones 1, 2, 3, ... , ,t2 de la tabla. Una fórmula para la función de acceso e s
f(a,) = (i - l )n + j.

Podemos utilizar las funciones de suelo y techo de las panes (a) y (b), respectivamente, del
ejemplo 5.10 para formular de nuevo algunas de las ideas que examinamos en el capítulo 4.
a ) AJ estudiar el algoritmo de la división, aprendimos que para cualesquiera a, b E Z,
• con b > O, fue posible encontrar q, r E Z tlnicos tales que a -= qb + r y O s r < b.
Ahorapodemosanadirque q = LaibJ yr = a - LaibJ b.
254 Capítulo 5 R~aoones y funciones

b) En el ejemplo 4.41 encontramos que el entero positivo


29,338,848,000 = 213'5'1'11

tiene 60 = (SX3X2X2Xl) = rcs;1)1 rcs;l)1 r(3;1)1 r(3; l)l r(l; l)lli'1-


sorcs positivos que son cuadrados pcrfcc1os. También tiene 24 = (3X2X2X2Xl

(S3
r
= - + l)l res+ r(3+ r(3+ r(l
- - 3 l)l- - - l)l - - l)l - + -l)l divisores positivos que son cu-
3 3 3
bos perfectos. En general, sin E z~
con n > 1, sabemos que podemos escribir

dondek E z ·,p¡es primo para todo 1 :Si s. k,p, t-p,paratodo 1 s. i<j s k,yi
E z· para todo 1 :s i :s k. Esto se debe al teorema fundamental de la a r i -
Entonces si r E z•, vemos que el mlmero de divisores positivos den que son poteu-

cias r-ésirnas perfectas esfl:..,r<,; 'l- Cuando r = l obtcnemosf1 :,_,f,,+1] ,


íl!,1(e, + 1) que es el número de divisores positivos den.

Ejemplo 5.12 En la sección 4.2 presentamos el concepto de sucesión junto con nuestro análisis de m
definiciones recursivas. Nos daremos cuenta ahora de que una sucesión de números reales
r 1, , 2• 'J• .. . puede pensarse como una funciónf: z•-+ R donde/(n) =r., para todon E z·
De la misma forma, una sucesión de enterosaO" a 1, ª-:• •.. puede definirse por medio de una
función g: N ➔ Z donde g(n) =ª••para todo n E N.

En el ejemplo 5.9. existen 212 = 4096 relaciones de A en B. Hemos e xaminado una


función entre todas estas relaciones; ahora queremos contar el número total de funciones
de A en B.

Para el caso general, sean A, B coajuntos no vados con IA 1= m. l B 1= n. En conse-


cuencia. si A= {a ., a,, . ..• a..) y B = lb., b,,. .. , b.). entonces una funcióo típica/:
A ➔ B puede describirse como ((a1, z 1), (a,. z ,), ... ,(a_ z..)). Podemos sclecciou:
cualquiera de losn elementos deBcomoz, y después hacer lo mismo conz, . (Podemos
sclc<:cionar cualquier clcmento de B como-',; incluso podemos elegir el mJSDIO ele-
mento de B como .r1 y .r1 .) Continuamos este proceso de selección hasta que ímalmeare
sclcccioaamos uno de los n elementos de B como z.. De esta forma, utilizando la regla
del produao, existen n"= 18 ¡r• 1 funciones de A en 8.

Por lo tanto para A. B del ejemplo 5.9, existen 4 3 = 1B )l" I = 64 funciones de A en B, y


3'= IA 1' ' ' = 81 funcionesdcBcnA. Engcneral,nocspcrarnosque IA ¡r•r sea iguala
1 B I '"1. A diferencia de la situación para el caso de las relaciones, no siempre podemos
obtener una función de Ben A intercambiando simplemente los componentes de los pares
ordenados de una función de A en B.
5.2 Funciones: en general e inyectivas 255

Ahora que tenemos el concepto de función como un tipo especial de relación, centrare-
mos nuestra atención en un tipo especia] de función.

Definición 5.5 Una función/: A ➔ B se denomina uno a uno, o inyectiva. si cada elemento de B aparece
como máximo una vez como la imagen de un elemento de A.

Sif:A ➔ Bes uno a uno, conA, Bfinito. debemos tener que I A 1::5: 1B 1- Para conjuntos
arbitrarios A, B, síf: A ➔ Bes uno a uno, entonces para a.., a2 E A,/(a1 ) ; /(a2) ~ a 1 ; a2•

....,_~;~~ij Consideremos la función/: R ➔ R tal que/(x) = 3x + 7 para todox E R. Entonces para


cualesquiera x 1, x1 E R , tenemos que

f(x,) = f(x2)?3x, + 7 = 3x2 + 7=;,3x, = 3x,=;,x, = x 2,


por lo que la función dada/es uno a uno.
Por otro lado, supongamos queg: R ➔ Res la función definida comog(x) ;f-xpara
cada número real x. Entonces

g(O) = (O)' - 0= O g(l) = (!)'-(!) = 1 - 1 =O.


En consecuencia, g no es uno a uno, ya que g(O) = g(1) pero O i- 1, es decir, g no es uno a
uno ya que existen números reales Xi, x 2 tales que g(x 1} =g(x2) # x 1; .x2 •

~S,~~~j Sean A = (1, 2, 3} y B= (1, 2, 3,4, 5). La función

f = {(!, !), (2, 3), (3,4))


es una función uno a uno de A en B;

g = {(!, !), (2, 3), (3,3))


es una función de A en B. pero no es uno a uno, ya que g(2) ; g(3) pero 2 $ 3.

Para A, B como en el ejemplo 5.14 existen 2 1s relaciones de A en By 53 de éstas son funcio-


nes de A en B. Queremos saber ahora el número de funciones/: A ➔ B que son inyectivas
o uno a uno. De nuevo argumentaremos para el caso de conjuntos finitos generales.

1SiA= {a,.a,.a,, ... , a,.), B= (b,. b,, b, ...• b,), y m :s n, una ñmcióo inyedivaf:A ➔
Blieae la forma {(a,. x,), (a,,x,), (a,. x,) ... , (a.. x.)}. donde exislmn opciones para
x, (es decir, cualquier clcmeoto de B). n - 1 opciones para x, (es decir. cualquier•
- de B escepco el elegido como x,), n - 2 opciones para x,. y así sucai--.
......,. -... n - (•- 1) = n - m + 1 opciones para,_ Por la iqla del pmclacto,
el......,_...,___ illyedivas de A en 8 es

11(11- 1)(11-2) · • · (11-• + 1) = n!/(n - m)! =P(n, •) =P<I B 1, IA 1).


256 Capítulo S Relaciones y funciones

En consecuencia. para A, B como los del ejemplo 5.14, existen 5 • 4 • 3 = 60 funciones


inyectivas/: A ➔ B.

Definición 5.6 Sif: A ➔ By A 1 ~ A, entonces

f(A,) = {b E B lb = f(a), para algún a E AJ,


y /(A 1) se conoce como la imagen de A1 medianJe f

t Ejemplo _?:15 i Para A = { 1, 2, 3, 4, 5) y B = ( w, x, y, z), seaf: A ➔ B dada porf = {(l, w), (2.x), (3.x), (4,¡-),
(5. y)}. Entonces para A,= { 1}, A, = (1, 2 ), A,= { 1, 2, 3), A, = {2, 3), y A, = {2, 3, 4, 5),
encontramos las siguientes imágenes mediantef

f(A,) = {f(a) la EAJ = lf(a)la E {l}} = {f(a)la = l} = {f(l)} = {w};


f(A,) = {f(a)la E A,}= {f(a)ia E {1,2}} = lf(a) la = lo2} = {f(l ).f(2)} = {w,x};
f(A,) = {f(l), f(2), f(3)} = {w,x), Y f(A,) = f(A,) ya que f(2) = x = f(3);
f(A,) ={x); y.
f(A,) = {x,y}.

a) Sea g: R ➔ R dada por g(x) = =


x'. Entonces g(R) imagen de g = {O, + ~ ). La
imagen deZ medianteg esg(Z) = {O, 1, 4, 9, 16.... } y paraA 1 :;;; [- 2, 1Jobtenemos
g(A, ) = {O, 4).
b) Seah: Z x Z ➔ Z definida comoh(x. y)= 2x + 3y. En este caso. el dominio de hes
Z x Z (no Z). y el codominio es Z. Vemos, por ejemplo, que h(O, O)= 2(0) + 3(0) = O,
h(l, 3) = 2(1) + 3(3) = 11, y h(-3. 7) = 2(-3) + 3(7) = 15. Además, h(2. -1) = 2(2) +
3(-1) = 1, y para cada n E Z, h(2n, - n) = 2(2n) + 3(-n) = 4n - 3n = n. En conse-
cuencia,h(Z xZ) = irnagendeh= Z.ParaA,= {(0,n) 1n E z •} = {O} x z • C Z xZ.
la imagen de A, mediante hes h(A,) = {3. 6, 9 .. . . } = {3n In E Z'}.

TEOREMA 5.2 Sea/: A ➔ B con A 1• A2 !;;;; A. Entonces

a) f(A, UA,) = f(A,) Uf(A,). b) f(A, nA,)Cf(A,) nf(A,).


e) f(A, nA,) = f(A,) nf(A,) cuando fes inyectiva.

Demostración: Demosuaremos (b) y dejaremos las demás partes al lector.


Paracualquierb E B, b E f(A, nA,) ⇒ b =/(a) para algún a E A, ()A, =>(b = f(a)pan
algún a EA1) y (b =f(a)paraalgúna E A,) ⇒ b E/(A1) yb E/(A,J => b Ef(A 1) ()f(A,),
por loque/(A, n A,) C /(A,) ()/(A,).
5.2 Funoones: en general e inyectivai 257

)rfinici6n 5.7 S i/: A 4 By A 1 ~ A, e ntonces/L 1: A 1 ➔ 8 es lartstricción dt/a A1, donde/lA1(a) = }\a)


para todo a E A1•

Orfinición 5.8 Sea A , !:; A y/: A, ➔ B. Si g: A ➔ By g(a) = /ta) para todo a E A1, entonces g es una
tXltnsi6n dt fa A.

f(jemplo s.17. J Para A= ( 1, 2. 3, 4, 5), sea/: A ➔ R definida como/= {(l. 10), (2, 13). (3, 16), (4, 19),
(5. 22)). Seag: Q ➔ Rtal q ueg(q) = 3q+ 7 para todoq E Q . Por último, seah: R ➔ R tal
que h(r) = 3, + 7 para codo r E R. Entonces

i) ges una extensión de/(deA) a Q;


ü) /es la restricción de g (de Q) a A;
üi) hes una extensión de/(de A) a R:
iv) / es la restricción de h (de R) a A:
v) hes una extensión de g (de Q) a R: y
vi) ges la re stricción de h (de R) a Q.

, Ejemplo 5.ts l Sean A = (w, x, y, z), B = ( 1, 2, 3. 4, 5) y A, = (w, y. z). Sean/: A ➔ B. g 1: A 1 ➔ B las


funciones representadas por los diagramas de la figura S.5. Entonces g = /JA 1 y /es una
e xtensión de g de A 1 a A. Observemos que para 1a función dada g: A 1 ➔ B. existen cinco
formas de extender g de A 1 a A..

Figura 5.5
258 Capitulo 5 Relaciones y funciones

EJ ERCICIOS 5 . 2 1. Determine si cada una de las siguientes relaciones es una función. Si una relación es m
función. determine su imagen.
a) {(x. y) 1x. y E Z.y = :x?+7}, una relación de Zen z .
b) {(x, y) 1.r. y E R,f= x ). una relación de R en R.
e) {(x, y) 1.r. y E R,y = 3x + I}. una relación de R en R.
d) {(,,y)¡ ,, y E Q, x'+ -J= 1}, una relación de Q en Q.
e) ~es una relación de A en Btal que I A 1 = 5. 1B 1= 61 Y 1~ 1= 6.
2. ¿Define la regla/(x) = l/(x2- 2) una función/: R ➔ R? ¿Una función/: Z ➔ R?
3. Sean A = { 1, 2, 3. 4 ) y B = {x. y. z;}. (a} Enumere cinco funciones de A en B. (b) ¿Cuái:las
funciones/: A ➔ Bexisten? (c)¿CUántas funciones/: A ➔ B son uno a uno? (d)¿Oiántasfu.
cionesg: B ➔A existen?(e}¿CUántas funcionesg: B ➔ A son inyectivas?(f)¿Cuántas funcio.
nes/: A ➔ B satisfacen/(1) = x? (g) ¿Cuántas funcionesf: A ➔ B satisfacenf(I) = /(2) = .x?(kl
¿Cuántas funciones/: A ➔ B satisfaccn/(1) =x y /(2) =y?
4. Si existen2l87funcioncs/:A ➔ By JBI =3, ¿cuántovale JAI?
i i
S. Sean A. 8, C\;; R' . dnndeA = {(.q ) y = 2x + 1 }.B= {(x. y) [ y = 3x} Y C = {(.r, y) x - y = l)
Determine lo siguiente:
a) An B b) ene e) X uc d) iiul:
6 . SeanA, 8, ci;; Z'. donde A= {(x. y) 1y = 2x + l},8 = {(x. y) 1y = 3x} yC = {(-',)·) lx-y=n
a) Determine
¡¡ AnB ü) s nc ill),i" u c Iv> /fue
b) ¿Cómoafe.cta las respue$ta.S de (i) a (iv) si A, B, C ~ z·x z· ?
7. Determine lo siguiente:
•> [2.3-1.6] b) [2.3]-[l.6] e) [2.3] - [1.6]
d) [3.7] + f7.3l e) [3.4l[6.2J o t3.4Jf6.2l
g) [21'] b) 2(,rJ o 2(,r]
8. Determine si cada una de las proposiciones siguientes es verdadera o falsa. Si es fa.Isa. propor-
cione un contraejemplo.
a) l•J=[a]paratodoaEZ. b) l•J = [a]paratodo a E R.
e) l• J=f•l-1 para <odoaE R -Z. d) l• fbl = f• lbJparatodoa, bER.
e) - fal=f- • l paratodoaE R. f) - f•l = [-aJparatodo aER.

9 . Encuentre todos los números reales x tales que


a) 7[xJ = [7, J. b) [7xJ = 7.
e) [x+7J = x+7. d) [x + 7J•[xj+7.

10. Octerminetod-OslosxERtalesquc LxJ + Lx+-½J = L2:x.J-


11. a) Encuentretodoslosnúmeros reales x talesque í2zl = 2fxl-
b ) Encuentretodoslosnúmerosrealesxtalcsque f3xl =3fxl
e) Sean E z·
donde n > l . Determine todos losx E R tales que fnxl = nfxl.
12. Paran, k E z·. dcmuestrequeÍ ntkl =L(n - l )(kJ + l .
13. a} Sea a E R·, donde a;?: l. Demuestre que (i} lfal!aJ = 1 y que (ii)flaJ/a1= l.
b ) Si a E R· y O< a < 1, ¿Qu~ resultados de la parte (a) son verdaderos?
14. Sean a,, ai, a,, ... la sucesión de enteros definida en forma recursiva como
1) a¡= I; y
2) Para todo n E z·. tal que n ~ 2, a. = 2al .:i J·
a) Determine ª• para todo 2 :s n :s 8.
b) Demuestre que a. :S n para todo n E z·.
5.2 Funciones: en eneral e inyectivas 259

15. Para cada una de las siguientes funciones, determine si es uno a uno y determine también su
imagen.
a) f: Z-Z,f(x) = 2x•1 b) f: Q-Q.f(x) = 2x + 1
<) f: Z - Z ,f(x)=x' - x d) f: R-R,f(x)=é
e) f:[- ,,/2,,,/2]-R,f(x) = senx f) f:[O,,,J- R,f(x)=senx
16. Sea/: R ➔ R dondef(x) =x1. De1ermine/(A) para los siguientes subconjuntos A tomados del
dominio R.
a) A= {2,3} b) A= {- 3, - 2,2,3}
<) A=(-3,3) d) A = (-3,2]
e) A = [-7,2] f) A = (-4,-3]U[5,6)
17. Sean A= {l. 2. 3, 4, 5}. B = {w. x. >~ z}.A 1 = {2. 3, 5} C: A y g: A1 ➔ B. ¿De cuántas formas se
puede extender g a una función/: A ➔ B?
18. Déun ejemplo de una función/: A ➔ By Ai,A 1 S::: A para los cualesf(A 1 n AJ ~/(A¡) nJ(.A-;J.
[Así, la inclusión en el teorema 5.2(b) puede ser propia.]
19. Demuestre las partes (a) y (e) del teorema 5.2.
2 0. Si A= { 1, 2 . 3, 4, 5} y existen 6720 funciones inyectivas/: A ➔ 8, ¿cuánto vale J 817
21. Determine la función de acce.so /(a;¡) conforme lo descrito en el ejemplo 5 .lO(d). para una
matriz.A = (a v),.,... tal que (a) m = 12. n = 12: (b) m =1. n = 10: (c) m = 10. n = 1.
22. Para la función de acceso desarrollada en el ejemplo 5.10 (d). la matriz A = (a,,).... se guardó
en una tabla unidimensional mediante el método de la fila principal. También es posible guar-
dar esta maniz usando el método de la c:.otumna principal, por el que cada elemento a,1, t :S i :S m
de la primera columna de A se guarda en las posiciones 1. 2, 3, . . .. m. respectivamente. de la
tabla. dondea11 se guarda en la posición l. Entonces. los cle menlosa,2, l :S i :S m de la segunda
columna de A se guardan en las posiciones m + 1, m + 1, m + 3 . .. . . 2m, respectivamente , de
la tabla; etcétera. Encuentre una fórmula para la función de acceso g(a,,) en estas condiciones.
23. a) Sea A una matriimxn que se desea guardar (de forma adyacente) en una tabla unidimensionaJ
de r entradas. Determine una fórmula para la función de acceso si a 11 debe guardarse en la
posición~ 1) de la tabla {en vez de la posición l, como en el ejemplo 5.JO(d)) con (i} el
método de la fila principal; (ii) el método de la columna principal.
b) Establezca las conWciones que deben satisfacer m, n, r y k para que los resultados de la
parte (a) sigan siendo válidos.
24. El siguiente ejercicio proporciona una demostración combinatoria para la fónnuJa de la suma
que ya hemos visto en dos resultados an1eriores: (1) el ejercicio 26 de la sección 1.4; y (2) el
ejemplo 4.3.
Sean A= {a, b. c),B = {1.2. 3, . . . ,n.n + l}. S= {f:A ➔ B 1/(a) = /(c)yf(b) </(e)).
a) s; S,= {f: A ➔ B [/E Sy/(c) = 2.)¿cuámo vale IS, 11
b) s; S, = {f: A ➔ B [fE S y /(e) = 3.) ¿cuámo vale IS, 11
e) Para l :Si :S n.seaS,= lf: A ---.¡. 8 IJE Sy/{c) =i + t.¿cuánto vale IS, I?
d) Sea T, = {/:A ➔ B [ /ES y /(a) =f(b)}. ExpHca por qué [ T, [ =( ";').
• \. SeanT0 l(:A ➔ B l(E Sx((a) < ((b)l xT,= l(:A ➔ B l(E Sy [(al > [(b)l. Exol;quc
por qué[ T, I=1 1=(";').
T,
O ¿Qué podemos concluir acerca de los conjuntos S1 U S2 U S1 U .. . U s.
Y T , U T2 U T,?
L :. =
g) Use los resultados de las panes (c), (d), (e)y ( f) para verificar que 1i
2
n(n + 1)(2.n + l'J/6.
25. Una versión de lajWJción de Ackennann A(m,n) se define en forma recursiva para m. n EN
con las siguientes propiedades:
A(O,n)=n+J,na,0;
A(m,O) = A(m -1.1),m >O; y.
A(m,n) =A(m -1, A(m.n -1)), m,n >O.
260 Capítulo 5 Relaciones y funciones

[Tales funciones fueron definidas en la década de 1920 por el matemático y lógico alC!DZI
Wtlhelm Ackermann (1896--1962), quien fue a1umno de David Hilbcn (1 862-1943). Estas
funciones tienen un papel importante en las ciencias de la computación. en la teoría de fUJXio.
nes recursivas y en el análisis de algoritmos que implican la unión de conjuntos.]
a) CalculeA(l,3)yA= (2,3).
b) Demuestre que A(! , n) =n + 2 para todon EN.
e) Para todo n EN, muestre queA(2, n) = 3 + 2n.
d) VerifiquequeA(3,n) = 2"'"3 - 3 para todo n EN.

5.3
Funciones sobreyectivas:
Números de Stirling
del segundo tipo

Los resultados que desarrollamos en esta sección nos darán la respuesta a los primeros
cinco problemas planteados al principio del capítulo. Veremos que las funciones sobrt-ytctml
son la clave para todas las respuestas.

Definición 5.9 Una función/: A ➔ Bes sobre, o sobreyectiva, si /(A)::::: B; es decir, si para todo b E B
existe al menos un a E A con/(a) = b.

La función/: R ➔ R definida como /(x) = x3 es una función sobre, ya que en este caso
vemos que si res un número rea1 de] codominio de[. entonces el número real![; está en el
dominio de/y /(;{;,) = (~ )3 • Por to tanto, el codominio de/es R, que es igua1 a la imagen
de/y la función/resulta ser sobre.
La función g: R ➔ R. donde g(x) =,? para cada número rea] x, rw es una función sobre.
En este caso, ningún número rea] negativo aparece en la imagen de g. Por ejemplo, para
que -9 esté en la imagen de g, tendríamos que poder encontrar un número real r tal que
=r =-
g( r ) =
9. Por desgracia, r1- = - 9 :;:;;;, r 3i o r=-
3i, donde 3i, -3i E C, pero 3i, -3i
ff= R. Así, tenemos que la imagen (g) =g(R) = [O,+ oo) C R y 1a función g no es sobre. Sin
embargo, debemos observar que la funciónh: R ➔ {O, +oo) definida porh(x) =x2 síes una
función sobre.

Consideremos la función/: Z ➔ Z tal que/(x) =3x+ l para cualquier x E Z. En este caso,


la imagen de/ es{ . . , -8, - 5. - 2, 1, 4, 7, .. . } C Z y /ne es una función sobre. Si
analizamos con cuidado esta situación. veremos que, por ejemplo, el entero 8 no está en la
imagen de/ aunque la ecuación

3x+ 1=8
se pueda resolver con facfüdad para obtener x = 7/3. Pero ése es el problema, ya que el
número racional 7/3 no es entero; así, no existe x en el dominio Z tal que /(x) = 8.
5.3 funciones Sobroyectiva.: Números de Sb~ing del segundo tipo 261

Por otro lado, cada una de las funciones

1) g: Q ➔ Qdonde/(x): 3.r+ 1 panu E Q; y


2) h: R ➔ R donde/(x): 3x+ 1 parax E R,

es sobre. Además, 3x1+ 1 -; 3.ti + l ~ 3.r1= 3.ri ~ ·x1=Xi, sin i.mpona.r el hecho de que x1
y x.¡ sean nthneros enteros, racionales o reales. En consecuencia. las tres funcionesf. g y h
son uno a uno.

Si A:{!, 2, 3,4) y 8: {x, y, z}, entonces

/ 1• {(1, z), (2,y),(3,x), (4,y)} y /, = {(l,x}, (2,x), (3,y), (4,z)}


son, ambas, funciones de A sobre B. Sin embargo, la función g: {(1,x), (2,x), (3,y), (4,y)}
no es sobre, ya que g(A): {.r, y) e B.

Si A, B son con¡u.ntos finitos, entonces para que exista una función sobre/: A ➔ B
debemos tener que IA 1;e; 1B 1. Si el lector recuerda el desarrollo de !u dos primeras sec-
ciones de este capítulo, pensará que nuevamente ha,Y que usar la ree,ta del producto y
contare! nwnero de funciones sobre/: A ➔ B,dondelA l:m ;e; n: 1B I. Por desgracia. la
regla del producto no es adecuada en este caso. Obtendremos el resultado necesario para
algunos ejemplos específicos y despu6s conjeturaremos una fórmula generaJ. En el capítu-
lo 8 demosuaremos la conjetura por medio del prindpio de inclusión y exclusión.

5.22 Si A : {x, y, z) y B: 11, 2), entoncestodu lu funciones/: A ➔ B son sobre excepto/,:


{(x, 1), (y, 1). (z. 1)) y/,: {(x, 2), (y, 2), (z. 2)), lu funciones consranus. Por lo tanto,
existen IB P ' 1- 2 : 2' - 2 : 6 funciones sobre de A en B.

•==::._:.5=..i
.D Si A : {w. x, )' t) y B = {l, 2. 3), existen 3' funciones de A en B. Si consideramos los
subconjuntos de B de 2 elementos, existen 2' funciones de A. a { 1, 2J, 2' funciones de A a
{2, 3} y 2• funciones de A a { 1, 3}. Por lo tanto, tenemos 3(2') = (:) 2• funciones de II en
B que definitivamente no son sobre. Sin embargo, antes de afirmar que 3' - G)2' es la
respuesta ftna1, debemos observar que no todas estas (!) 2' funciones son distintas, ya que
si consideramos todas las funciones de A. en { 1, 2}, estamos eliminando, de entre estas
funciones, por ejemplo, la función {(w, 2), (x, 2), (y, 2), (<, 2)). Por otro lado, si considera-
mos las funciones de/1 en {2,3}, eliminamos la misma función: {(w, 2), (x, 2), (y, 2), (<, 2)).
Eo consecuencia., en el resultado 3' -(D2', hemos eliminado dos veces cada una de las
funciones constantes/: A ➔ B, doode /(A) es uno de los conjuntos {1), {2} o j3}. Si
~ustamos nuestro resuttado a este análisis. veremos que existen 3' -C)2' .... 3= G)3' -
ln 2' + G) 1':: 36 funciones sobre de A en B.
162 Capítulo 5 Relaciones y funciones

Si B sigue siendo el conjunto { 1, 2, 3), para cualquier conjunto A con IA 1 = m;,: 3,


existen G)3·-C)2• + (n 1· funcionesdeA sobre B. (¿Quéresu]tadodarfaesta fórmula
sim= 1?. ¿si m = 2?)

Los últimos dos ejemplos sugieren un patrón que estableceremos ahora, sin demostra-
ción, como fónnula general.

Sean A {1, 2, 3, 4, 5, 6., 7 ) y B {w, x. y. z). Si aplicamos la fórmula general con m - 1


y n = 4, vemos que existen

(:)4'-(;)3' + (~)2' - (;)1' =! (-1)'(4~ k)(4- k)'


= Í (-1)'(
4
~ k)(4-k)' = 8400 funciones de Á sobre B.

El resultado del ejemplo 5.24 es también la respuesta a las primeras tres preguntas del
principio del capítulo. Vna vez que eliminamos el vocabulario innecesario, reconocemos
que en estos tres casos queremos distribuir siete objetos diferentes en cuatro recipientes
distintos sin que quede ningún recipiente vacío. Podemos hacerlo en términos de las ÍtJD.
cienes sobre.
Para el problema 4 , tenemos un espacio muestra! Y que consta de las 4 7 = 16,384 formas
en que las siete personas pueden elegir alguno de los cuatro pisos. (Observe que 47 es
también el número total de funciones/:A ➔ Bdonde jA 1 = 7, 1BI = 4.) El suceso que nos
interesa contiene 8400 de estas selecciones, de modo que la probabilidad de que el ascen-
=
sor se detenga en cada piso es 8400/16834 0.5127, un poco más de la mitad del tiempo.
Por último, para el problema 5, como L :..0(-Ii(;. ~n-kY- es el número de funcio-
nes sobre/: A ➔ B para I A 1= m, 1B 1= n, para el caso en que m < n no existen tales
funciones y la suma es O.
Analizaremos el problema 6 en la se.cción 5.6.
No obstante, antes de pasar a algo nuevo, examinaremos un problema más.
5.3 Funciones Sobreyectivas: Números de Stirting del segundo tipo 263

En la empresa CH. Susana. la supervisora, tiene una secretaria, Teresa, y otras tres au;c;ilia-
res administrativos. Si hay que procesar siete cuentas, ¿de cuántas fonnas puede Susana
asignar las cuencas de modo que cada asistente trabaje al menos en una cuenta y que el
trabajo de Teresa incluya la cuenta más cara?
En primer lugar, la respuesta no es 8400 como e n el ejemplo 5.24. Aquí debemos con-
siderar dos subcasos disjuntos y después aplicar la regla de la suma.

a) Si Teresa, la secretaria, solamente trabaja con la cuenta más cara, entonces hay que
distribuir las otras seis cuentas entre las tre s asistentes administrati vos
de L ~(-lyt(¿t}(3-k)6 = 540 formas. (540 es el número de funciones sobre/:
A -4 B conlA J=6, 1B1= 3.)
b) Si Teresa trabaja con otras cuentas además de la más cara, la asignación de tareas
puede hacerse de ¿ :...,(-1)'{:,)<4-k)' = 1560 formas. (1560 es el número de
funcionesg: C-4D con! CI=6,IDI= 4.)
En consecuencia, la asignación de trabajo puede hacerse en las condiciones dadas de
540 + 1560 =2100 formas. [Ya hemos mencionado que la respuesta no sería 8400, sino (l/
4X8400) = WI BI X8400), donde 8400 es el número de funciones sobre f: A -4 B, con
IA !== 7, 1B 1= 4. Este hecho no es una coincidencia. co~o veremos al anaJiz.ar el teorema
5.3.]

Ahora continuaremos nuestro análisis con la distribución de objetos distintos en recipien-


tes sin que ninguno de éstos quede vacío, y para el caso en que los recipientes sean idénticos.

Si A ={a, b, e, d} y B ={
1, 2, 3}; entonces existen 36 funciones sobre de A en B o, en
forma equivalente. 36 formas de distribuir cuatro objetos diferentes en tres recipientes
distinguibles. sin que ninguno quede vacío (y sin tener en cuenta la localiz.ación de los
objetos e n un recipiente dado). Entre estas 36 distribuciones nos fijamos en Ja siguiente
colección de seis (una de las seis posibles colecciones de seis):

1) {a,bl, {el, {d), 2) {a, b), {dl, {el,


3) {el, {a,bl, {d}, 4) {el, {dl, {a,bl,
5) {d), {a,bl, {el, 6) {dl, {el, {a, bl,,
donde. por ejemplo, la notación {eh indica que e está en el segundo recipiente. Ahora
bien, si no hacemos distinción entre los recipientes, estas 6 = 3! distribuciones se vuelven
idénticas, de modo que hay 36/(3 !) = 6 formas de distribuir los objetos distintos a. b. c. d
entre tres recipientes idénticos, sin dejar ninguno vacío.


Capítulo 5 Relaciones y funciones

A,4.d-.,deformaseaqaopoclmMJs~los•olljoloodiltilloa•
pieales id6alicos, sinqueqaode~ndo,cs

-1 }:
n!~
. ( -1 >{,ªJ<
-
n-l'f".

Esta suma se-.como-Sl-. n)y sedeaomiaa-...deSlufifatld_,..,


Observemos que silAI=• .!: • =IBl,UÍIIIOlla! •,Sla.11)1-.-. . . . .

La tabla 5.1 enumera algunos números de Stirling del segundo tipo.

Tabla 5.1
S(a,a)

1~ 1 2 3 4 s 6 7 8
1 1
2 1 1
3 1 3 1
4 1 7 6 1
5 1 15 25 10 1
6 1 31 90 65 15 1
7 350 140 21 1
8 1 ~
1 ~ 1701 1050 266 28 1

Para m ~ n, L: 1s(m, i) e s el número de formas posibles de distribuir m objetos diferentes


en n recipientes idénticos sin que que.den recipientes vacíos. De la cuarta fila de la tabla
5.1 vemos que existen l + 7 + 6;;;;;; 14 fonnas de distribuir los objetos a, b, e, d entre ttes
recipientes idénticos, dejando tal vez algt:in recipiente vacío.

Continuamos ahora con la derivación de una identidad relacionada con los números de
Stirling. La demostración es de naturaleza combinatoria.

EOREMAS.3 Sean m. n enteros positivos tales que 1 < n :S m. Entonces

S(m + 1, n) =S(m,n -1) + nS(m,n).

Demostración: Sea A= {a1, t¾, ... , a,.. a_ .. 1} . Entonces S(m + l . n) cuenta el número de
formas en que los objetos de A pue.den distribuirse en n recipientes idénticos, sin que
quede ningún recipiente vacío.
Existen S(m. n - 1) formas de distribuir a i, 0,z, • .• , a., entre n - l recipientes idénticos.
sin que quede ninguno vacío. Entonces, al colocar a,. .. 1 en el recipiente vacío restante, se
tiene que S(m, n - 1) de las distribuciones contadas en S(m + 1, n); a saber. las distribucio-
nes en qu«; a.,...1 está solo dentro de un recipiente. Otra opción es distribuir a i, t¾, . . . , a.
5.3 Funciones Sobreyectivas: Números de Stirting del segundo tipo 265

entre los n recipientes idtnticos, sin que quede ningún recipiente vacío, con lo que obtene-
mos S(m, n) distribuciones. Sin embargo, para cada una de estas S(m. n) distribuciones, los
n recipientes quedan distinguidos por su contenido. Si seleccionamos uno de los n reci-
pientes idénticos para a .. + 1, tenemos nS(m, n) distribuciones del tota1 S(m + 1, n); a saber,
aquellas distribuciones para las que a• • , está en el mismo recipiente que otro objeto de A.
El resultado se sigue entonces de la regla de la suma

Para ilustrar el teorema 5.3, consideremos el triángulo que aparece en la tabla 5.1. En
este caso, el nllmero más grande corresponde con S(m + t , n), para m = 1 y n =3 y vemos
= = =
que S(7 + 1, 3) 966 63 + 3(301) S(7, 2) + 3S(7, 3). La identidad del teorema 5.3
puede usarse para extender la tabla 5.1 en caso necesario.
Si multiplicamos el resultado del teorema 5.3 por (n - 1)! tenemos

(1/n)[n!S(m + l,n)J = [(n - l)!S(m,n -1)) + [n!S(m,n)].

Esta nueva fonna de la ecuación relaciona a1gunas ideas del número de funciones sobre,
ya que si A= {ai, a 2 , ••• , a.,. a... 1 } y B ; {b1, bi, ... , b,. . i, b,.} con m ~n - 1, entonces

(1/nXEI número de funciones sobre h: A ➔ B)


= (El número de funciones sobre /: A - {a • • ,) ➔ B - { b. ))
+ (El número de funciones sobre g: A - {a.... 1 } ➔ 8 ).
Ésta es la razón por la que la relación observada a1 final del ejemplo 5.25 no es simplemen-
te una coincidencia.
Cerraremos esta sección con una aplicación que trata de un problema de conteo en e l
que los números de Stirling del segundo tipo se usan junto con el teorema fundamental de
la aritmé tica.

Consideremos el entero positivo 30,030 = 2 x 3 x 5 x 7 x 11 x 13. Algunas de las·


factorizaciones no ordenadas de este número son
Í) 30 X 1001 = (2 X 3 X 5)(7 X 11 X 13)
ii) 110x273=(2X5Xll)(3X7Xl3)
iii) 2310 X 13 = (2 X 3 X 5 X 7 X 11)(13)
iv) 14 X 33 X 65 = (2 X 7)(3 X 11)(5 X 13)
V) 22 X 35 X 39 = (2 X 11)(5 X 7)(3 X 13)
vi) 6 X 7 X 11 X 65 = (2 X 3)(7)(11)(5 X 13)
Los resultados dados en (i), (ii) y (iii) de muestran tres de las fonnas de distribuir los seis
objetos distintos 2, 3, 5, 7, 11, 13 en dos recipientes idénticos sin que quede un recipiente
vacío. Así, los primeros tres ejemplos son trcs de las S(6, 2) = 31 factorizaciones no orde-
nadas de 30,030 con dos factores; es decir, e xisten S(6, 2) formas de faclOrizar 30030
como mn donde m, n E z• para 1 < m, n < 30,030 y donde no importa el orden. De la
misma forma. los resultados de (iv) y (v) son dos de las S(6, 3) =90 formas no ordenadas
• de factorizar 30,030 con tres factores enteros, cada uno de los cuales es mayor que 1. Si
queremos a1 menos dos factores (mayores que 1) en cada una de estas factorizaciones no
~66 Capítulo 5 Relaciones y funciones

• ordenadas, entonces vemos que existen S(6 ,2) + S(6, 3) + S(6, 4) + S(6, 5) + S(6, 6) =
L ~S(6, i) = 202 de tales factoriz.aciones. Si queremos incluir la factorización con •
solo facU>r 30,030 (cuando distribuimos los seis objetos distintos 2, 3, 5, 7, 11, 13 en un
recipiente (idéntico)), tenemos un total de 203 factorizaciones.

:JERCICIOS 5.3 1. Dé un ejemplo de conjunios finiiosA y Bcon jA j, j Bj a< 4 y una función/: A ➔ Bt.al quc(a)
/no sea uno a uno ni sobre; (b)/sea uno a uno pero no sobre; (c)/sea sobre pero no uno a uno;
(d)/sea sobre y uno a uno.
2. De cada una de las siguientes funciones/: Z ➔ z. determine cuáles de ellas son uno a uno y
cuáJes son sobre. Si la función no es sobre, determine la imagen/(Z).
a) f(x)=x+7 b) f(x)=2x - 3 e) f (x)=-x+S
d) f(x) = x' e) f(x) = r'+x f) f (x)=r'
3. De cada una de las siguientes funciones g: R ➔ R, determine cuáles de eJlas son uno a uno y
cuáles son sobre. Si la función no es sobre, determine la imagen g (R).
a) g(x)=x+7 b) g(x)-2x - 3 e) g(x)= - x+S
dlgW=r' drW-r'+x fl 1W=r'
4. SeanA = {I, 2, 3, 4 ) y B = {I, 2, 3,4, S, 6).
a) ¿Cuántas funciones de A en B existen? ¿Cuántas de ellas son uno a uno? ¿Cumltas son
sobre?
b ) ¿Cuánw funciones de Ben A existen? ¿Cuántas de ellas son sobre? ¿CUántas son uno a
uno?
5. VerifiquequeL:-o<-It{.:,J{n-kY- = 0paran = 5ym=2, 3,4.
6. a) Verifique que 57 =¿ !.o{:~;!) S(7, i).
b) Proporcione un argumento combinatorio para demostrar que para todos m, n E Z\

m•= f ( ~ )(i!) S(n,i~


1-1 I

7. Sean A= {l, 2, 3. 4. 5. 6, 7} y B = {u, w; .x, y, t}. Determine el número de funciones/:A ➔ B


tal que (i)f(A) = (u, x): (ii) jf(A) 1=2; (iii)f(A) = (w, x, y ) ; (iv) lf{A) 1= 3; (v)/(A) = {v, x, ·
}; ,J. y (vi) lf(A) 1= 4.
b) Sean A, B conjuntos tales que I A 1= m ~ n = 1B 1· Si k E z· con 1 .s k .s n. ¿cuánw
funciones/: A ➔- B son tales que 1/(.A.) 1= k?
8. Un investigador del Instituto de Química tiene cinco ayudantes de laboratorio en un proyecto
de investigación en el que deben sintetizarse nueve compuestos. ¿De cuántas formas puede
asignar el investigador estas síntesis a los cinco ayudantes de modo que cada uno de ellos
trabaje al menos con una síntesis?
9. Use el hecho de que toda ecuación polinomial con coeficientes rea1es y grado impar tiene uoa
rafzreal para mostrar que la función/: R ➔ R detinidacomo/(x) = r'- 2r+xesuna función
sobre. ¿& uno a uno?
1 O. Supongamos que tenemos siete bolas de diferente color y cuatro recipientes numerados 1, ll, m
y IV. (a) ¿De cuántas formas podemos distribuir las bolas de modo que ningO.n recipiente quede
vacío? (b) En esta colección de siete bolas de colores diferentes. u.na de ellas es azul. ¿De aJánw
fonnas podemos distribuir las bolas de modo que ningiln recipiente que.de vacío y que la bola
azul quede en el recipiente 11? (e) Si eliminamos los mlmeros de los recipientes de modo que ya
no podamos distnguirlos, ¿de cuántas formas podemos distribuir las siete bolas de color entn:: los
cuatro recipientes idénticos, de modo que alguno o algunos de ellos puedan quedar vacíos?
5.4 Funciones especiales 267

11. Determine las dos filas siguientes (m= 9. lO) de la tabla 5.1 para los números de StirlingS(m.
n),donde 1 :S n S m.
12 . a) ¿De cw.ntas formas puede factorizarsc 31,100,905 en tres factores. cada uno mayor que J.
si no importa el orden de los factores?
b) Resuelva la parte (a) si el orden de los factores es significativo.
e) ¿De cw.ntas formas puede factorii.arsc 31,100,905 e n dos o mti factores, cada uno mayor
qy.e l. si no impona el orden de los factores?
d ) Resuelva la parte (e) teniendo en cuenta el orden de los factores.
13. a) ¿Cuántas factorizacionesnoordenadasdedos factores, cada uno mayor que l , tiene 156,009?
b) ¿Cu~tas factoriuciones no ordenadas de 156,009 tienen a1 menos tres factores, cada uno
mayor que l?
e) ¿De cuántas formas se puede factoriz.ar 156,009 en dos o más factores, cada uno mayor que
I , si no importa el orden de los factores?
d ) Sean p i, p 2, p 3, •••• p. n primos distintos. ¿De cuántas formas puede factorizarsc e l produc-
I1
to ;_1p¡ en dos o más factores, cada uno mayor que 1, si no importa el orden de los
factores?
14. Escriba un programa (o desarrolle un algoritmo) para calcular los m1meros de StirlíngS(m., n)
para l :S m :S 12 y 1 :S n :S m.

5.4
Funciones especiales

En la sección 2 del capítulo 3 mencionamos que la suma es una operación binaria cerrada
en el conjunto z•, mientras que n es una operación binaria cerrada en !l(GU) para cual-
quier universo~ dado. También observamos que ''tomar el negativo.. de un entero es una
operación unaria en Z. Ahora es tiempo de precisar estas nociones de operaciones binaria
y unaria (cerradas) en términos de funciones.

Definición 5.1 O Para cualesquiera conjuntos A, B no vacíos, cualquier función/: Ax A ➔ Bes una opera~
ción binaria en A. Si B ~ A, entonces se dice que la operación binaria es ce"ada (en A).
[Cuando B ~ A también podemos decir que A es cerrado medianlef]

Definición 5.11 Una función g: A ➔ A es una operación unaria, o mon.aria. en A .

a) La función/: Z x Z ➔ Z dada por/(a, b) = a-bes una operación binaria cerrada


en Z .
b) Si g: z+x z+➔ Z es la función tal queg(a. b) =a - b, entoncesg es una operación
binaria en z+. pero M es cerrada. Por ejemplo, vemos que 3, 7 E z• pero g(3, 1) =
3-7 =-4itz·.
e) La función h: R+ 4 R+ dada por h(a) = 1/a es una operación unaria en R·.
c.apítulo 5 Relaciones y funciones

Sca'U un universo y sean A, B ¡;; 'U. (a) Si/:"'('U)x~('U) -+ ~('U) estádadapor/(A,B)


=A U B, entonces/es una operación binaria cerrada en~('U). (b) La funcióng: ~~) ➔
~('U) dada por g(A) =Aes una operación unaria en ~('U).

,f inición 5.12 Sea/: Ax A ➔ B; es decir,/es una operación binaria en A.


a) /es conmutativa si/(a, b) =/(b, a) para todo (a, b) E Ax A.
b) CuandoB ~ A (es decir, cuando/es cerrada), decimos qucfesasociativasi pa.raa,
b, e E A,/(/(a, b), e)= /(a, /(b, e)).

La operación binaria del ejemplo 5.30 es conmutativa y asociativa, mientras que la opera-
ción binaria de la parte (a) del ejemplo 5.29 no cumple ninguna de estas condiciones.

a) Definamos la operación binaria cerrada/: Z x Z --> Z como /(a, b) = a + b - 3ab.


Como la suma y la multiplicación de enteros son operaciones binarias conmutativas,
se tiene que

f(a,b) =a+ b-3ab =b + a -3ba =f(b,a),


por lo que/ es c.onmutativa.
Para determinar si/es asociativa. consideremos a, b, e E Z. Entonces

f(a,b) = a+ b - 3ab f(f(a,b),c) = f(a,b) + c-3/(a,b)c


= (a + b - 3ab) + e - 3(a + b - 3ab)c
= a+b+c - 3ab-3ac-3bc+9abc,
mientras que
f(b,c) = b +c-3bc y
f(a,f(b, e))= a + f(b, e) - 3af(b, e)
=a+ (b +c - 3bc) - 3a(b + c - 3bc)
= a+ b + e - 3ab - 3ac - 3bc +9abc.
Como /(/(a, b), e)= /(a, f(b, e)) para todos a, b, e E Z, la operación binaria
cerrada/es asociativa y conmutativa.
b) Consideremos la operación binaria cerrada h: Z x Z ➔ Z donde h(a, b) = a Ib !-
= = = =
Entonces h(3, - 2) 3 1-2 I 3(2) 6, pero h(- 2, 3) - 2 l 3 I =-
6. Por lo tanto,
h no es conmutativa. Sin embargo, respecto de la propiedad asociativa, si a. b. e E
Z, vemos que

h(h(a,b),c) = h(a,b)lcl = albll cl Y


h(a, h(b, e))= alh(b, c)I = •lblcll = al bllcl,

de modo que la operación binaria cerrada hes asociativa.


5.4 Funciones ~ les 269

e) Seag: R x R ---+ Z laopenoción binaria cerrada dada por g(a, b) = ía +bl, el techo
de a + b. Como a + b • b + a para todos los números reales a. b (puesto <fue la
suma de mlmcrosrcalcs es conmutativa), tenemos queg(a, b) = ía +bl = íb +al =
g(b, a) y ges una operación binaria (cemda) asociativa.
Sin embargo, cuando con.sideramos la asociatividad, tenemos que, p:x- ejemplo,

=
g(g{3.2, 4.7),6.4) = g(T3.2 + 4.7],6.4) • g(T7.9], 6.4) g{S,6.4)
= rs + 6.41 = r14.4l =1s,
mientras que

g(3.2,g(4.7,6.4)) = g(3.2,í4.7 + 6.4"I) = g(3.2,fll.ll) = g(3.2, 12)


= í3.2 + 12] = íl5.2] • 16.
+
Como g(g(32, 4.7), 6.4) = 15 16 = g(3.2, g(4.7, 6.4)), este contraejemplo basta
para indicamos que g n.o es asociativa. Por lo tanto, para cienos m1meros reales a, b,
e, vemos que no ocurre que

[Ta+ b]+cl=ía + fb + <TI-

Si A= {a, b, e, d) , entonees IA xA 1= 16. Por lo tanto,existen4" funciones/: A xA --+A;


es decir, existen 4 16 operaciones binarias cerradas en A.
Para determinar el número de operaciones binarias cerradas conmutativas gen A, ve-
mosque existen cuatro opciones para cada una de lti asignaciones g(a. a). g(b, b), g(c, e)
y g(d, d). Entonces nos faltan otros 4 2- 4 = 16 - 4 = 12 pares ordenados (en Ax A) de la
forma (x, y), x "'y. Estos 12 pares ordenados deben considerarse e n conjuntos de dos para
garantizar la conmutatividad. Por ejemplo, necesitamos que g(a, b) = g(b, a); por lo que
podemos seleccionar cualquiera de los cuatro elementos de A como g(a. b), pero luego
tambii n debemos asjgnar esta selección ag(b, a). Por lo tanto, como existen cuatro opcio-
nes para cada uno de estos 12/2 = 6 conjuntos de dos pares ordenados, vemos que el
número de operaciones binarias cerradas conmutativas gen A es 4' • 4'• 410•

lfflnición 5.13 Sea/: A x A ➔ B una operación binaria en A. Un elemento x E A es un neutro (ele~nlO


neutro o eumento identidad) dc/si/(a, x) = f(x, a ) = a para lodo a E A.

a) Consideremos la operació n binaria (cerrada)/ : Zx Z---+ Z dadapor/(a, b) =a+ b.


En este caso, el entero Oe.s un elemento neutro, yaquc/(a, 0) =a+ O=O +a =/(O,
a)= a para cualquier entero a.
b) Vemos que no existe un neutro para la función de la parte (a) del ejemplo 5.29, ya
que si/tuviera un neutrox, entonces para cualquiera E Z,/ (a. x) •a~ a-x =a
~x=O. Peroentonces/(x.. a) =/ (O, a)= 0 - a -;1:. a, a menos que a= O.
• e) SeaA = { 1, 2, 3, 4, 5, 6, 7 ) y g: A x A --+A la operación binaria (cerrada)dada por
g(a, b) = m!n{a, b ); es decir, el mínimo (o más pequeño) de los números a, b.
Capitulo 5 Relaciones y funciones

Esta operación binaria es conmutativa y asociativa, y para cualquier a E A tenernos que


g(a. 7) = mín(a. 7) =a= mín(7,a} = g(I, a ). Así, 7 es un elemento neuuo parag.

En las partes (a) y (c) del ejemplo 5.34 anali:zamos dos operaciones binarias (cerradasi
cada una con un elemento neutro. La parte (b) de ese ejemplo mostraba que una opcracila
de ese tipo no necesariamente tiene un elemento neutro. ¿Podría tener una operación binaria
más de un elemento neutro? En el siguiente teorema veremos que la respuesta es no.

EOREMAS.4 Sea f: A x A ➔ B una operación binaria Si/ tiene un elemento neutro, entonces dicho
elemento es único.
Demostración: Si/tiene más de un elemento neutro, sean x 1• Xz E A , tales que

f(a,x,)=a=f(x1,a), para toda a EA, y


f(a,x,) =a= f(x,,a), para toda a EA.
Consideremos ax1 como e1e mento de A y a.tz como elemento neutro. Entonces/(xi, xJ =
x 1 • Ahora invirtamos los papeles dex 1 y .tz; es decir, consideremos ªXz como elemento de
A y ax1 como e leme nto neutro. Tenemos que/(x1, xJ ~ .xi. Por lo tanto,x1 ;:;x1 y/tiene a lo
sumo un elemento neutro.

Ahora que hemos aclarado el punto de la unicidad del elemento neutro, veamos cómo
interviene este tipo de e lemento en otro problema de conteo.

\!liftBJiJJJ1:r; I Si A ;:; {x, a, b, e, d }, ¿cuántas operaciones binarias cerradas en A tienen ax como elemento
neutro?
Sea/: A xA ➔A con/(x. y ) = y= f (y. x)para todo y EA. Entonces podemos representara
/ medfante una tabla como la que aparece en la tabla 5.2. En este caso, los nueve valores en los
que x es la primera componente [como en (x, e)] o donde es Ia segunda componente [como en
(d, x)] quedan determinados por el hecho de qucxesel elemento neutro. Cada una de las demás
entradas (vacías) de la tabla 5.2 pueden ocuparse con cualquiera de los cinco elementos de A.

Tabla 5.2
f X a b c d
X X a b e d
a a
b b
e e
d d

Por lo tanto, existe n 5 16 operaciones binarias cerradas en A tales que x es el elemento


neutro. De éstaS, 5 10 =5' • 5<4?--'V l son conmutativas. También vemos que.e x.isten 5 16 opera-
ciones binarias cerrad.as en A tales que b es el elemento neutro. Por lo tanto, existen 5 17 =
• (n 5 16 = U) (D
5 s ? -¡:2(S}-ll ;:; 51.s-W operaciones binarias cerradas en A que tienen un ele-
mento neutro, de las cuales 5 11 = (;) 5 1º;:; G) 54 • 5c,t...,y: = son conmutativas.
5.4 Funciones especiales 271

En general, si A= {ai, ~. ilJ, . . . , a. } paran E z·, entonces existen


i) nt-i) operaciones binarias cerradas/: A x A ➔ A;
ü ) n• • ~-m operaciones binarias cerradas conmutativas en A;
üi) n.i-noo-11 = n<►t'r operaciones binarias/: A XA ➔ A donde algún a, E A particular,
para 1 s i s n, es el elemento neutro;
iv) (;) (nl.. l)i - n<--01+ 1 operaciones binarias (cerradas) en A donde existe un elemento
neutro;
v) r 1 • n1t---1f-<..11Y2 operaciones binarias cerradas conmutativas en A; donde algún a,
E A particular, para 1 S i S n, es el elemento neutro; y
vi) (:) (n .....1) n 11..1'r-<---1>V2 = ,µi--m operaciones binarias cerradas conmutativas en A
que tienen un elemento neutro.

Después de ver varios ejemplos de funciones (en los ejemplos 5.29, 5.30, 5.32, 5.34 y
5.35) donde el dominio es un producto canesiano de conjuntos, analizaremos ahora fun-
ciones en las que el dominio es un subconjunto de un producto cartesiano.

lrefinidón 5 . 14 Para los conjuntos A y B, siD !;;;;; A x B, e ntonces JtA: D ➔ A dada pornA(a. b ) = a, se llama
la proyecci6n en la primera coordenada.

La función 1t8 se define de manera similar, y observamos que si D =-Ax B, e ntonces Jt.4
y 7ta son sobre.

Para A= 8 = Z , sea D = {(x, y) 1y = lx 1}. EntoncesD ~Ax 8 y D contiene el origen


(O, O) junto con todos los puntos de la rectay = x , dondex, y E Z y x > O, y todos los puntos
de la recta y = - x, dondex. y E Z y x < O. Unapanede la gráfica deD aparece en la parte
(a) de la figura 5.6. Aquí vemos que 1t,(D)= {.. . , -4, -3, - 2, -1, O, 1, 2, 3, 4, ... } = Z y
=
n,(D) {O, 1, 2, 3, 4, .. . ] N. =
A continuación, sea E = {(- 4, 4), (-2, 2), (0, O), (1 , 1), (2, 2 )). En este caso, E CD h.
Ax B; toda la gráfica de E se muestra en la figura 5.6(b). Ahora bien, vemos que 1t.4(E) =
{it,(-4, 4), rt,(-2, 2), rt,(0,0), 1t,(I, ! ), rt,(2, 2)) = {-4, -2, O, 1, 2) y rt,.(E ) = {O, 1, 2, 4 ).
l
Así, 11t,(E) 1 = 5, mientras que n,<E) 1 = 4 ya que it,.(- 2, 2) = 2 = n,.(2,2).

.. .. .
y y
4 - 4

. -.. 3

2 -
• 1
-
. .. 3
2
1
1 1 X ' 1 1 1 X
- 4-3-2- 1 1 2 3 4 - 4 - 3 -2 - 1 1 2 3 4

(a) (b)

/ Figur a 5.6
capítulo 5 Relaciones y funciones

Si A {w, x, y) y B = { 1, 2, 3, 4}, sea D = {(x, 1), (x, 2), (x, 3), (y, 1), (y, 4)}. Entoncesb
proyección"•' D ➔ A satisface 1t,(x, 1) = 1t, (x, 2) = 1t,(x. 3) = x y n,(y, l} = 1t,(y, 4) =¡.
Como 1tA(D) = {x, y} C A. esta función no es sobre.
Para el casode 1t,: D ➔ B, tenemos que 1t,(x, l ) =1t, (y, 1) = 1, x ,<x, 2) =2, xs(x, 3)=
3 y 1t.(y, 4) = 4, por Jo que 1t,{D) ::=By esta proyección es una función sobre.

SeanA=B = R, consideremos el conjunto O ~Ax Btal queD = {(x, y) 1y = r }. Entonces


D representa et subconjunto del plano eucüdeo que contiene los puntos de la parábola y = r.
Entre la infinidad de puntos de D tenemos el punto (3, 9). En este caso, 7tA(3 , 9) ::: 3, la
abscisa de (3, 9), mientras: que 1t,(3, 9) =9, la ordenada del punto.
Para este ejemplo, 1tA(D) = R =A, por lo que 1tA es sobre. (La proyección nA también es
uno a uno.) Sin embargo,1t,l.,D) = [O, + 00) C R, porlocual n.,noes sobre. [Tampoco es uno
a uno; por ejemplo, it,(2, 4) = 4 = lt,(-2, 4).

Ahora vamos a extender el concepto de proyección de fa forma siguiente. Sean A1, A~


... ,A. conjuntos e {i1,½, ... , i,..} !; {1, 2, . . . ,n},demodoque í 1 <ii< · ·<i.ym :S: n.
Si D ~ A 1 xA2 x · · · xA,.= X :_1.4,, entonces la función 1t: D -➔ A,, x A ;! x ···A;. dada por
1t(a1, a 2, ••• , a,.) ::: (A,,,ª':' . .. a,) es la proyección de Den las coordenadas i1, i2 , ••• , i..
Los elementos de D se 11arnan n-uplas (ordenadas); un elemento de Tt(D) es una m-upta
(ordenada).
Estas proyecciones surgen en forma natural en el estudio de las bases de datos
relaciona.les, una técnica estándar para la organización y descripción de grandes canticb-
des de datos mediante los modernos sistemas de cáJculo a gran escala. En situaciones
como las operaciones con tarjeta de crédito, no sólo deben organizarse los datos existentes
sino tambié n otros nuevos, como cuando se procesan las tarjetas de crédito para nuevos
clie ntes. Cuando se paga el saldo de las cue ntas e xistentes, o cuando se hacen compras coo
estas cuentas, hay que actualizar los datos. Otro ejemplo surge cuando se buscan registros
con fines especia1es. como cuando la oficina de admisión de una universidad analiza los
registros educativos en busca de estudiantes de bachillerato con cieno nivel de habilidad
matemática demostrado, para enviarles infonnación por correo.
El siguiente ejemplo muestra el uso de proyecciones en un método para organizar y des•
cribir los datos, a una escala más pequeña.

En ciena universidad se relacionan los siguientes conjuntos con fines de registro:

A1 ~ el conjunto de claves de los cursos ofrecidos en el área de matemáticas.


A2 = el conjunto de nombres de los cursos ofrecidos en el área de matemáticas.
A3 ::: el conjunto de profesores de matemáticas.
A.=-el conjunto de letras del a1fabeto.

Consideremos la tabla, o relación, D ~ A1 x A1 x A 3 x A, dada en la tabla 5.3.


5 .4 Funciones especiales 273

Tabl• 5.3

Cave de curso Nombre del cuno Profesor Letra de sección


MAlll Cálculo I P. Z. Cbinn A
MAlll Cálculo l v. Larney B
MA112 Cálculo n J. Kinney A
MA112 Cálculo n A. Sehmidt B
MA112 Cálculoll R. Mines c
MA113 Cálculo lll J . Kinney A
MA113 Olculo lll A . Sehmidt B

Los conjuntos A1, A2.A3 , A, son los campos dt. la bast. de datos rrlacion.al, y se dice que
la tabla D tiene grado 4. C.ada elemento de D se conoce como registro.
La proyección deD en A1 xA 2 x A1 x A, se presenta en la tabla S.4. La tabla 5.S muestra
los resultados de la proyección de D enA 1 x A:.
Las tablas S.4 y 5.5 son otra forma de representar los mismos datos que aparecen en la
tabla 5.3. Con las tablas S.4 y S.S podemos volver a formar la tabla 5.3.
Tabla 5.4 Tabla 5.5
Clave c1, cuno Proltsor L,tradeK<rl6n Nombro
Oa,-e de CW'SO del cuno
MAlll P. Z. Cbinn A
MAlll V. Larney B MAlll Cálculo I
MA lU J. Kinney A MA112 Cálculo n
MAlU A. Sehmidt B MA113 Cálculo Ill
MAlU R.Mines e
MA113 J. Kinney A
MA113 A . Sehmidt B

La teoría de las bases de datos relaciooales se ocupa de la rqnscnlación de los dalos de


diferentes formas y de las operaciones necesarias para tales rqnsenlaciooes, como es el caso
de las proyecciones. Tambi~n se estudia la implementación en computador de tales t6cnic:as:
En los ejercidos y la bibliografia del capítulo se menciooa este tema y algunos OlrOS aspectos.

EJERCIOOS 5.4 1. Para.A= {a, b. e}, sea/: Ax A ➔ A la operación binaria cerrada dada en la tabla 5.6. Dé un
ejemplo para mostrar que/no e.s asociativa.

. .. .
Tabla 5.6

f b e
b e
b
e e. e
b
b

2. Defina la operación binaria cerrada h: Q' x Q• ➔ Q• dada por h(a. b) = alb.


a) Muesuc que h no es conmutativa DJ asociativa.
b) ¿Ticoc la un elemento neutro?
3. Cada una de las siguienies funciones/: Z x Z ➔ Z e.s una operación binaria c:cmda en Z.
Dctemúne los casos en los que/es conmulltiva o asociativa.
74 Capítulo 5 Relaciones y funciones

a)/(x.y)=x+y - xy b) /(x,y) = máx{x, y}, el máximo (o más grande) de>, y


e) /(x.y)=i' d) /(x,y)=x+y-3
4. ¿Cuáles de las operaciones binarias cerradas del ejercicio 3 tienen elemento neutro?
S. Sea I A 1 = 5. (a) ¿Cuánto vale I Ax A I? (b) ¿Cuántas funciones/: A x A ➔ A existen? {e)
¿Cuántas operaciones binarias cerradas en A existen? ( d) ¿Cuántas de estas operaciones binarias
cerrada$ son conmutativas?
6. SeaA = (x. a, b, e, d).
a ) ¿Cuántas operaciones binarias cerradas f en A satisfacen f(a, b) = e?
b) ¿Cuántas de las funciones/de la parte (a) tienen ax como elemento neutro?
e) ¿Cuántas de las funciones/de la parte (a) tienen Un demento neutro?
d) ¿Cuántas de las funciones/de la parte (e) son conmutativas?
7. Sea/: z• x z· ➔ z· la operación binaria cerrada definida comof(a, b) = mcd(a. b).
a) ¿Es/conmutativa?
b) ¿Es/asociativa?
e) ¿Ticne/un elemento neutro?
8. Sea A= {2. 4, 8, 16, 32} y consideremos la operación binaria cerrada/: A xA ➔ Atal que
/(a, b) = mcd(a, b). ¿Tiene/un elemento neutro?
9. Parap, qprimosdistintos, sea A= {P-tf I O :S m :s: 31,0 :S n :S 37}.
a) ¿Cuánto vale I A I?
b) Si/: A xA -+ A es la operación binaria cerrada definida por /(a, b)= mcd(a, b), ¿tiene/un
elemento neutro?
1 O. Establezca un resultado que generalice los resultados de los dos ejercicios anteriores.
e
11. Para "FA~ z·,
sean f. g: Ax A-+ A las operaciones binarias cerradas dadas por /(a. b) =
mín{a, b}, y g(a, b) = máx{a, b ). ¿Tiene/un elemento neutro? ¿1iene g un elemento neutro?
12. a) Defina laopcraci6nbinariacerradaf: Zx Z-+ Z dada por/(a, b) =(a]+ [b]. (i)¿Es/
conmutativa'! (ü) ¿Es/asociativa? (ili) ¿Tienefun elemento neutro?
b) Considere la operación binaria cerrada g: R x R -+ R dada por g(r, s) = (r] + {s). (i) ¿& g
conmutativa'! (ü) ¿& g asociativa? (iii) ¿Tiene g un elemento neutro?
13. SeaA = R - Z. Considere la operación binaria h: A xA-+ Z dada por h(a, b) = laJ+fbl. ¿Es
h conmutativa?
14. Sean A = B = R. Determine xiD) y 1t.{D) para cada uno de los conjuntos siguientes D ~ A x B.
a) D={(x.y)lx = y') b) D={(x. y)ly= senx) e) D=((x.y)lr+f=I)
15. Sean.A.-. 1 :si :s 5, los campost de unatablaD ~ Xi.,A¡,dondcA1 = {U, V, W, X, Y. Z} (que
se utilizan como los nombreseD clave de varios cereales en una prueba), y A 2 =A3 =A..= A.3 =
Z ... La tabla Daparece en Ja tabla 5.7.

..
Tabla 5.7
Gramos de azúcar ~ de RD" de 4Jp deR.D de· ~de RD depn-
Nombre dan ,~ vitamina A por cada .-cpor'""' táDa po, ~
ddccnal radóo de 1 oma naóa de t oaza 1'1Ki6a de loaza rad6a de 1 ama

u 1 25 25 6
V 7 25 2 4
w 12 25 2 4
X o 60 40 20
y 3 25 40 10
z 2 25 40 10
•RD - ~ d i m a

• t El túmino domain también se puede traducir como ..dominio", pero en este contexto esti establecido
..campo... (N. del E-)
5.5 El principio d<I palomar 275

a ) ¿Cutl es el grado de la tabla?


b) Encuentre la proyección de D en A, x A.x As,
e) El campo de una tabla es una e ~ primaria de la tabla si su valor identifica en formad.n.ica
cada lista de D. Octennioc la (o las) elave(s) primaria(s) de esta tabla.
16 . Sean A,. 1 S j :S S. los campos de una tabla D ~ X ~1 J\, donde A.1 ={
1, 2} (que se utilizan
para identificar la producción diaria de cipsulas de vitamina de dos empresas farmac:6.lticas},
=
A 1 IA. D, E} y A3 = At~A.s = z•. La tabla Daparecccn la tabla S.8.

Tabla 5.8

.........
a.,...&

1
l
1
Vltaa.impnse:atc
fflladpsala

A
D
E
Camidaddo-
toladpmb.aaOI•

l0,000
400
30
~
_,,,
.....
1
1
1
- Niimm>dodplula
portr..co
100
100
100
2 A 4,000 1 250
2 D 400 l 250
2 E 15 1 250 1
•ut•Ullllid a d t s ~

a ) ¿Cuál es el ¡rado de la tabla?


b) ¿Cuál es la proyección deD coA 1 xA.1 ?; ¿cnA,xAtxA,?
e) Esta tabla no tiene clave$ primarias. (Vbsed ejercicio 15.) Sin embar¡o, podemos definir
una dave primaria compwsta como d producto cartesiano de un número mínimo de cam-
pos de la tabla. cuyas componentes, tomadas en conjunto. identifican en forma llnica cada
lista de D. Determine al¡unas claves primarias compuestas para esta tabla.

5.5
El principio del palomar

Cambiaremos nuestro ritmo para introducir un princípio de distribución interesante. Este


principio parece no tener nada en com!ln con lo que hemos estado haciendo antes; pero en
realidad nos será muy útil.
En matemáticas, a veces parece que una idea casi obvia, cuando se aplica de una mane-
ra un poco sutil, es la clave necesaria para resolver un problema difícil. Sin duda. en la lista
de estas ideas obvias puede colocarse la siguiente regla, conocida como el principio del
palomar.

Ahora bien, ¿qu~ tienen que ver las palomas que descansan en los nidos con las matc,-
máticas (discretas, combinatorias, etc.)? En realidad, este principio puede aplicarse en
varios problemas donde buscamos establecer si puede ocurrir ciena situac:::ión. Ilustrare,.
mos este principio en los siguientes ejemplos y veremos su uti)jdad en la sección S.6 y en
otros puntos del texto.
76 capitulo 5 Relaciones y funciones

Una oficina emplea a 13 oficinistas de archivo, por lo que al menos dos de ellos dela
cumplir años durante el mismo mes.Aquí tenemos 13 palomas Oos oficinistas de archiw)
y 12 nidos Oos meses del año).

Aquí hay una segunda aplicación más bien inmediata de nuestro principio.

Juan regresa de la lavandería con 12 pares de calcetines ( cada par de distinto cotOr) en una
bolsa A1 sacarlos de la bolsa aleatoriamente, tendrá que sacar cuando mucho 13 de ellos
para obtener un par.

A partir de este punto, la aplicación del principio del palomar puede ser más sutil

Vilma opera un computador con una unidad de cinta magnética. Un día le dan una cinta
que contiene 500,000 "palabras" de cuatro o menos letras minúsculas. (F.n la cinta, las pala-
bras consecutivas se separan con un carácter en blanco.) ¿Puede suceder que las 500,0Xl
palabras sean distintas e ntre sí?
A partir de las reglas de la suma y el producto, e l númCTO total de palabras posibles
diferentes, que utilizan cuatro o menos letras, es
26'+ 26' + 26'+ 26 = 475,254.
Estas 475,254 palabras son los nidos y las 500,000 palabras de la cinta son las palomas,
por lo que al menos una Í,alabra se repite en la cinta.

Sea S C z•, donde IS 1 = 37. Entonces S contiene dos elementos que tienen el mismo
resto al dividirse entre 36.
Aquí las palomas son los 37 enteros positivos de S. Por el algoritmo de la divisióo
(teorema 4.5), sabemos que al dividir cualquier entero positivo n entre 36, existen un
cociente único q y un resto único r, ta1es que
n =36q+ r,O :s; r<36.

Los 36 valores posibles de r son los nidos y el resultado se demuestra por el principio del
palomar.

Demostraremos que si se seleccionan 101 enteros del conjunto S = { 1, 2, 3, ... , 200},


entonces existen dos enteros tales que uno divide al otro.
Paracadax ES, podemos escribir x= 2'y, conk2:0, y rncd(2,y) = l. (Este resultado se
sigue de1 teorema fundame ntal de la aritmf tica.) Entonces y E T = { 1, 3, 5, . . . , 199},
I
donde T f ;: :; 100. Como se seleccionan 101 e nteros de S, por el principio del palomar
tenemos que existen al menos dos e nteros distintos de la forma a;:::; 2., y b = 2•ypara algón
I
(el mismo) y E T. Si m < n, entonces a b; en caso contrario, tenemos que m > n y
I
entonces b a.
5.5 El principio del palomar 277

Cualquier subconjunto de tamafto seis del conjuntoS = { 1, 2, 3, ..• , 9} debe contener dos
elementos cuya suma es 1O.
Aquí los números 1, 2, 3, ... , 9 son las palomas, y los nidos son los subcoojun10s { 1, 9),
{2, 8), {3, 7), {4, 6). {5). Cuando las seis palomas van a sus respectivos nidos, deben
ocupar al menos uno de los subconjuntos de dos elementos cuyos miembros suman 10.

El triángulo A CE es equilátero y AC = l. Si se seleccionan cinco puntos del interior del


triángulo, existen al menos dos puntos cuya distancia es menor que 1/2.
Para el triángulo de La figura 5.7, los cuatro triángulos más pequeOos son triángulos
equiláteros cong,uentes y AB = 112. Descomponemos el interior del triángulo ACE en las
cuatro regiones siguientes, disjuntas dos a dos:

R 1: el interior del triángulo BCD junto con los puntos del segmento BD. excluyendo
By D.
R1 : el interior del triánguloABF.
R,: el interior del triángulo BDF junto con los puntos de los segmentos BF y DF,
excluyendo B, D y F.
R,: el interior del triángulo FDE.

Ahora aplicamos el principio del palomar. Los cinco puntos del interior del triínglo
ACE deben ser tales que al menos dos de ellos estén en una de las cuatro regiones R¡, 1 s
i .s 4. donde cualesquiera dos puntos están separados por una distancia menor que tn.

A f Fig ura 5.7

5 .47 Sea S un conjunto de seis enteros positivos cuyo máximo es 14. Demostraremos que no
a:::....:::...._ __, todas las sumas de los elementos de todos los subconjuntos no vacíos de S pueden ser
distintas.
Para cualquiera de los subconjuntos no vacíos A de S, la suma de los elementos de A,
denotada comos., satisface 1 :S s. s 9 + 10+ · • • + 14=69, yexisten2'- 1 =63subconjuntos
no vacfos de S. Nos gustaría obtenu la conclusión del priocipio del palomar haciendo que
las sumas posibles, del l al 69, sean los nidos, y que los 63 subconjuntos no vacíos de S
sean las palomas, pero entonces tenemos muy pocas palomas.
Asf, en vez de considerar todos los subconjuntos no vacíos de S, examinaremos sola-
mente lossubcoojuntosnovadosA deStalesque I A 1 :S 5. Entooces, paracadasubcoojunlO
A se sigue que 1 .s s. . .s 1O + 11 + • • • + 14 =- 60. Existen 62 subconjuntos no vacfos A de
S con¡ A 1::s 5; a saber, todos los subconjuntosdeSexcepto0 y el mismo conjunto S. Con
78 Capítulo 5 Retaciones y funciones

62 palomas (los subconjuntos no vacíos A de S tales que I A 1 :s 5) y 60 nidos (las s . -


s.. posibles). se sigue del principio del palomar que los elementos de al menos dos de esa
62 subconjuntos deben dar la misma suma.

Scam E z+con m impar. Demuestre que existe un entero positivo n tal que m divide a 2•- l.
Considere los m + l enteros positivos 21 - l. 2 2 -1, i:~- 1, ... , 2•- I, 2-1 - l. Pord
principio del palomar y el algoritmo de la división. exjsten s. t E z· con I s s < t s m+ 1,
tales que 2' - 1 y 2' - 1 tienen el mismo resto al dividirlos entre m. De aquí tenemos que
2'-1 =q,m + ry 2' - 1 =q,m + r, paraq,, q, EN, y (2'- 1)-(2' - 1) = (q,m + r)-(q,m+r~
por lo que 2' - 2' = (c?l - q1)m. Pero 2' - zs =2' (2'- 1 - 1); y como m es impar, tenemos que
mcd(2r, m) = l. De aquí m 1(21- • - 1), y el resultado se sigue con n = t - s.

En sus próximas vacaciones de cuatro semanas, l:leberto jugará aJ menos un set de tenis
cada día. pero no más de 40 sets e n total durante este tiempo. Demuestre que independien-
temente de cómo distribuya sus sets durante las cuatro semanas, hay varios días consecu-
tivos durante los cuales jugará exactamente 15 sets.
Para 1 :s; i :s; 28, sea X; el número total de sets que Hebeno jugará desde el comienzo de
sus vacaciones hasta eJ final del i--ésimo día de las mismas. Entonces 1 :s; x 1 <.xi<• ••<.tu
s 40, y x 1 + 15 < • • • <-¼a+ 15 :s; 55. Ahora tenemos 28 nl1meros distintos x 1, Xi, . .. , Xn
y los 28 m1meros distintos x1 + 15, .xi+ 15, . .. , .tu+ 15. Estos 56 nl1meros sólo pueden
tomar 55 valores diferentes, por lo que al menos dos de eUos deben ser iguales; de aquí
concluimos que existe I Sj < i :s; 28 con X; == X¡+ 15. Asf, desde el comienzo del díaj + 1
hasta el final del día i, Hebeno jugará exactamente 15 sets de tenis.

IERCICIOS 5.5 1. En el ejemplo 5.41, ¿quiénes desempeñan el papel de las palomas y el de los nidos en nuestra
~licación del principio del palomar?
2. Demuestre que si ocho personas están en un cuarto, al menos dos de eJlas cumplen años d
mismo día de la semana.
'
3 . ¿Cuántas veces debemos tirar un solo dado para obtener el mismo resultado (a) al menos dos
veces? (b) al menos tres veces? (e) al menos n veces, paran~ 4?
4 . Dados 8 libros de Pascal, 17 de FORTRAN. 6 deAPL, 12 de COBOL y 20de BASIC,¿cuán-
tos de estos libros deben seleccionarse para asegurar que tendremos IO libros que tratan del
mismo lenguaje de programación?
5. Un auditorio tiene capacidad para 800 personas. ¿Cuántos asientos deben ocuparse para garan-
tizar que al menos dos personas sentadas en el auditorio tienen las mismas iniciales del nombre
y del primer apellido?

• 6. a) SeaS C z·.
¿Cuál esel valor mínimo de IS I paragarantizar laexistenciadedos elementos
x, y ES tales que x y y tengan el mismo resto al dividirlos entre 1000?
5.5 El principio del palomar 279

b) ¿CUál es el valor mínimo den tal que si S~ z• y IS


1=n.cntoncesexistentrcs elementos
x, y. z ES tales que los trcS tienen el mismo resto al divirlos entre 1000?
e) Escriba una proposición que generalice los resultados de las partes (a) y (b) y el ejemplo
5.43.
7. a) Demuestre que si se seleccionan 151 enteros de { l, 2, 3, ... , 300}. éntonccs la selección
I
debe incluir dos enteros x, y tales que x y o y x. I
b) Escriba una proposición que generalice los resultados de la parte (a) y el ejemplo S.44.
8 . Demuestre que si sdeccionamos 101 enteros del conjuntoS= {1, 2. 3, ... , 200}, existcnm. n
en la selección tales que mcd(m. n) l. =
9. a) Muestre que si se seleccionan cualesquiera 14 enteros del conjunto S ={l, 1, 3, ...• 25},
existen al menos dos enteros en esa selección cuya suma es 26.
b) Escriba una proposición que generalice los resultados de la parte (a) y el ejemplo S.4S.
10. SeaS= (3, 7, 11, 15, 19, ... , 95, 99,103}. ¿CuántoselementosdcSdcbemos seleccionar para
asegurar que existen al me nos dos cuya suma es 110'?
11. a) Si se seleccionan 11 enteros de {l, 2, 3, ... , 100}, demuestre que existen al menos dos, x
y y, ta1es que O<f.G- J,l<t. .
b) Escriba una proposición que generalice el resultado de la parte (a).
12. Sea ABC un triángulo equilátero, con AB = 1. Demuestre que si seleccionamos JO puntos en el
interior de este triángulo, existen al menos dos cuya distancia es menor de 1/3.
13. Sea ABCD un cuadrado con AB-= 1. Demuestre que si seleccionamos cinco puntos en el inte-
rior de este cuadrado, exis~ al menos dos cuya distancia entre sí es menor que 11..fi
14. Sea A i;; {1, 2, 3, .. . , 25) donde J A 1=9. Paracualquicr subconjuntoB de A, denocamoscon
s, la suma de los elementos en B. Demuestre que existen subconjuntos distintos C, D de A tales
quelCI = 1D l = Sys, =s,
1 5. Sea S un conjunto de cinco enteros positivos, tales que el máximo de ellos es 9. Demuestre que
no todas las sumas de los elementos de todos los subconjuntos no vacíos de S pueden ser
distintas entre sí.
16. Andrea tiene 46 piezas de papel rectangulares. Si l, w (medidos en centímetros) denotan el
largo y el ancho, respc,ctivamente, de cada pieza rectangular, entonces para esta situación tene-
mos que l, w son enteros positivos. donde 1 :S w s l s 90. De entre estos 46 rectángulos.
demuestre que Andrea puede seleccionar dos. digamos R1 y Rz. de tal forma que RJ cubqi
completamente a R, al colocarlo encima de R1,
17. ¿Cuántos enteros positivos entre I y 30 (inclusive) debemos seleccionar para garantizar que
dos de ellos, Wgamos x, y, son tales que su máximo comlln divisor es mayor que l '?
18. Durante las primeras seis semanas de su último afio de escuela. Braulio realiza al menos un
resumen Wario, pero no más de 60 resúmenes en total. Muestre que hay un periodo de días
consecutivos durante los cuales realiza exactamente 23 resúmenes.
19. a) Si S !: z• y IS l 2: 3, demuestre que existen x, y ES distintos tales quex + yes par.
b ) SeaS ~ z•xz~. Encuentre el valor mínimo de IS I que garantiza la existencia de distintos
pares ordenados (x1, xi), (y., yJ ES tales quex,+ y1 y .t2 + Yl son ambos pares.
y
e) Extendiendo las ideas de las partes (a) (b), consideremos Sr;;; z· x z·x z ·. ¿De que
tama1k> debe ser I SIpara garantizar la existencia de distintas temas ordenadas <x1o .ti,, x,),
(Y1, Yz, 11) E Stales quex1+ Ji.Xi + Yl, .t)+}'l, sean todos pares?
d) Generalice los resultados de las partes (a). (b) y (e).
~) Un puntoP(x, y) en el planocartesianoesunpunrotkl retículo six, y E Z. Dados distintos
puntos del retículo P(x¡, y1), P(x1, y.;J, ... , P.(x•• y.), determine el valor más pcquclto de n
qiie garantice la existencia de P/..z,, yJ, PJ.x¡. yiJ, 1 S i S j S n, tales que el punto medio del
segmento de línea que une PJ.x.;,y;) con PJ.x¡.11) sea cambién un punto del reúculo.
280 Capítulo 5 Relaciones y funciones

20. Seak E z•. Demuestre que existe un entero positivon tal que& y los únicos dígitos densca
cero y tres.

5 .6
Composición de funciones
y funciones inversas

AJ hacer cálculos con los elementos de Z, vemos que la operación {binaria cerrada) de la:
suma proporciona un método para combinar dos enteros, digamos a y b, en un tercc
entero, llamado a + b. Además, para cualquier entero e hay un segundo entero d ta1 tla!
e+ d = d +e = O; llamamos a del inverso aditivo de c. (También es cieno que e es el
inl'erso aditivo de d .)
En el caso de los elementos de R y la operación (binaria cerrada) de la multiplicacióa.
tenemos un método para combinar cualesquiera,; s E R mediante su productors. Además.,
para cualquier1 E R tal que1 '#- Oexiste un número real u tal queut= tu = l.AI número real
u se le llama el inverso multiplicativo de l. (El mímero real t es también el inverso
multiplicativo de u).
En esta sección estudiaremos primero un método para combinar dos funciones en uoa
sola función. Despuc!:s desarroUaremos el concepto del inverso (de una función) para fun..
ciones con ciertas propiedades. Para esto, necesitaremos las siguientes ideas preliminares.
Ya hemos analizado funciones que son uno a uno y funciones sobre; veremos ahora
funciones con ambas propiedades.

Definición 5.15 Si/: A ➔ B, entonces se dice que/ es biyectiva. o es una correspondencia biyectiva, si/es
inyectiva y sobre.

t'Éje,npló 5.~ · Si A = {l . 2, 3, 4) y B = {w, x, y, z), en,onces / = {(!, w), (2, x), (3, y), (4, z)) es una
correspondencia biyectiva de A (en) a B. y g = ((w, 1), (x, 2). (y, 3), (z. 4)} es una corres-
pondencia biyectiva de Ben (sobre) A.

Debemos señalar que cuando usamos el término correspondencia en el capítulo 1 y en


el ejemplo 3.10, el adjetivo uno a uno estaba implícito aun sin haberlo establecido.
Para cualquier conjunto A distinto del vacío, siempre e xiste una correspondencia
biyectiva muy simple pero importante. como se verá en la siguiente definición.

Definición 5 .16 La función IA: A ➔ A, definida como lA(a) ;;; a para todo a E A, es lafunci6n identidad
para A.
5.6 Composición de funciones y funciones inversas 281

Definición 5.17 Sif g: A ➔ B, decimos quefy g son iguales y escribimos!= g, sif(a) = g(a) para todo a
EA.

Un error común al l.rabajar con la igualdad de funciones sucede cuando/, g son funcio•
nes con un dominio común A yf(a) =g(a) para todo a E A. Podrfano ser el caso que/= g.
El error ocurre aJ no poner atención a los codominios de las funciones.

lrani,¡., 5,~1J Sean/: Z ➔ Z, g: Z ➔ Q donde/(x) = x = g(x), para todox E Z. Entonces/, g tienen como
dominio común Z, la misma imagen Z y actúan igual en cada elemento de Z . ¡Sin embar•
go,/ -:1:-g! En este caso.fes una correspondencia uno a uno, y ges inyectiva pero no sobre;
así, los codominios establecen una diferencia.

. li&en.,io ~,~2.] Consideremos las funciones/, g: R ➔ Z definidas como sigue:

six E Z
f(x)=tJ+l, six ER - Z g(x) = [x ], para todo x E R

Six E Z.entoncesf{x) =x = rxl = g{x).


Parax E R - Z .escribimosx = n +rdonden E Zy0< r < 1. (Porejemplo,six = 2.3.
escribimos 2.3 = 2 + 0.3, con n = 2 y r = 0.3; para x = - 7.3 tenemos - 7.3 = -8 + 0.7, con n
= -8 y r = 0.7.) Entonces

f(x) = LxJ + 1 = n+ 1 = íxl = g{x).


En consecuencia, aunque las funciones f. g están definidas por fórmulas difuemes. nos
damos cuenta de que son la misma función, ya que tienen el mismo dominio, el mismo
codominio y /(x) = g(x) para todo x del dominio R.

Ahora que contamos con los antecedentes necesarios, es tiempo de analizar una opera•
ción para combinar dos funciones apropiadas. Hemos visto operaciones que combinan
enteros y otras que operan en conjuntos. Ahora presentamos una forma de combinar dos
funciones apropiadas.

Definición 5.18 Si/: A ➔ B y g: B ➔ C. definimos lajunci6n compuesta, que se denota g o f: A ➔ C, como


(g o f)(a) = g(f{a)), para cada a E A.

Ejemplo 5.5~ 'j Sea A= { 1, 2, 3, 4).B= {a. b. e) y C = {w, x, y. zl conf:A ➔ B;f. B =+'\:dadas porf=
{{I, a), (2, a), (3, b), (4, e)) y g = {{a,x), (b. y), (c. z)}. Para cada elemento de A encontra-
mosque:

(g ,!)(!) = g(f(I)) = g(a) =x (g ,J)(3) = g(/(3)) = g(b) = y


• (g •/)(4) = g(/(4)) = g(c) = z
(g o/)(2) = g(/(2)) = g(a) = X
282 Capítulo 5 Relaciones y funciones

~~r lo que
g •/ = {(l ,x), (2,x), (3,y), (4, z)}.

; ~ ~ilj Sean/: R ➔ R,g: R ➔ R definidas porf(x)=x',g(x) = x+ 5.Enlonces


(g •/)(x) = g(f(x)) = g(x') = x ' + 5,
mientras que

(/•g)(x) = f(g(x)) = f(x + 5) = (x + 5)2 = x' + IOx + 25.

Aquí g of: R ➔ R y Jo g: R ➔ R, pero (g oj)(I) = 6 ,o 36 = (fo gXI): así, aunq,,


podemos formar ambas composiciones fo g y g o f, no ocurre que/ o g = g o f En conse-
cuencia, la composición de funciones no es, en general, una operación conmutativa

La definición y los ejemplos de la composición de funciones requieren que el codomirúo


de/= dominio deg. Si la imagen de/!: dominiodeg, en realidad esto será suficiente pan
obtener la composición g o f: A ➔ C. Así mismo, para cualquier/: A -+ B. observamos
que/o IA =/ = 1,of.
Una idea importante siempre presente en matemáticas es la de analizar si a1 combinar
dos entidades con una propiedad en común se obtiene un resultado con esta propiedad. Pa
ejemplo, si A y B son conjuntos finitos, entonces A n By A U B son también finitos. S111
embargo, para conjuntos infinitos A yB, tenemos que A U B es infinito pero A n B puede
ser finito. (Dé un ejemplo.)
Para la composición de funciones tenemos el siguiente resultado.

TEOREMAS.5 Seanf:A ➔ By g : B ➔ C.
a ) Si/, g son inyectivas, entonces g o fes inyectiva.
b) Sif. g son sobre, entonces g o /es sobre.
Demostración: a) Parademostrarquego/:A ➔ Ces inyectiva, sean ai, Gz E A con (go!J
(a1) = (g o /)(a,). Entonces (g o /)(a,) ; (g o fXa,) => g(f(a,)) a
g(f(o,)) ⇒ /(a1) = f(a,J, ya que ges inyectiva. También,/(a1) = f(a,)
=:) a 1=~. ya que/es uno a uno. En consecuencia, g o f es inyectiva.

b ) Parag of: A ➔ C, seaz E C. Ya que ges sobre. existe y E B cong(y)


= ,. Como fes sobre, e rule x E A con /(x) = y. De aquí, = g()I) ª
g(f(x)) = (g o /)(x), por lo que la imagen de g o f = C; al codominio
deg of. y g o fes sobre.

Aunque la composición de funciones no es conmutativa, si/: A ➔ B. g: B ➔ C. y h:


C ➔ D, ¿qué podemos decir acerca de las funciones (h o g) o /y h o (g o f)? Esto es, ¿es
asociativa la composición de funciones?
Antes de considerar el resultado genera1, primero analizaremos un ejemplo particular.
5.6 Composición de funciones y funciones inversas 283

- ~ ~S
~-..
5~5"" Sea/: R ➔ R, g: R ➔ R y h: R ➔ R, donde/(x) = x', g(x) = x + 5, y h(x) = ~ -
Entonces ((h o g) o[Xx) =(h o g)(f(x)) = (h o g)(x') = h(g(x')) =h(x'+ 5) =J<x' + 5)2 +2
= .,/x 4 + 10x2 +27.
Por otro lado, vemos que (h o (g o f))(x) = h((g o [Xx)) = h(g(f(x))) =
= = =
h(g(x')) h(x' + 5) J <x' + 5)' + 2 J x' + !Ox' + 27, como antes.
Así, en este caso particular, (h o g) o /y h o (g o f) son dos funciones con el mismo
dominio ycodominio, y para todox E R,((h o g) of)(x) = Jx' + IOx' +27 = (h o(g o/))(x).
En consecuencia. (h o g) o/= h o (g o f).

Al anaJizar nuestro siguiente teorema, veremos que eJ resultado del ejemplo 5.55 es
verdadero en general.

TEOREMA 5,6 Sí/: A ➔ B, g: B ➔ C, y h: C ➔ D, entonces (h o g) of= h o (g o f).


Demostración: Como las dos funciones tienen el mismo dominio.A. y el mismo coclominio,
D. el resultado se seguirá al mostrar que para todo x E A.((h o g) o fXx) = (h o (g o fXx).
(Véase el diagrama de la figura 5.8.)
Con la definición de la función compuesta encontramos que

((h •g)•/)(x) = (h•g)(f(x)) = h(g(f(x))),


mientras que

(h •(g •/))(x) = h((g •f)(x)) = h(g(f(x))),

En consecuencia, la composición de funciones es una operación asociativa.

Figura 5.8

Debido a la propiedad asociativa para la composición de funciones, podemos escribir


h o g o/, (h o g) o f o h o (g o/) sin problemas de ambigüedad.Además, esta propiedad nos
pcnnite definir las potencias de funciones, cuando corresponde.
284 Capítulo 5 Relaciones y funciones
r
Definición 5.19 Si/: A ➔ A, definimos/'=/. y paran E z·.J-' =f o(/').

Esta definición es otro ejemplo en que el resultado se define recursivamente. Enjr.+1:


fo (f'-). vemos quef"" 1 depende de una potencia previa,¡-.

;.~S~:d Si A = {l. 2, 3, 4 ), y/: A ➔ A está dada por/= {(!, 2), (2, 2), (3, !), (4, 3)), tenemos que
f' =fo f = {(!, 2), (2, 2), (3, 2), (4, !)} y f' =Jo f' =fofo/= {(!, 2), (2, 2), (3, 2), (4, 2)).
(¿Qué son¡• y/'?)

Pasemos ahora a la última idea de esta sección: 1a existencia de 1a función invertible y


algunas de sus propiedades.

Definición 5 .20 Para los conjuntos A, B ~ ótt, si 91: es una relación de A a B, entonces la inversa de 8t,
denotadag¡<, es la relación de B a A definida por g¡< = {(b, a) 1(a, b) E !1l ).

Para obtener~ de9t, simplemente intercambiamos las componentes de cada par orde-
nado en gt. Asf, si A = {!, 2, 3, 4),B = {w, x, y) ygt = {(! , w), (2, w), (3,x)), entonces91'
= {(w, !), (w, 2), (x, 3)), una relación de B aA.
Para los mismos conjuntos anterioresA, B, sea/:A ➔ Bdada por/= { !, w), (2.x), (3,y),
(4, x)) . Entonces/' = {(w, !), (x, 2), (y, 3), (x, 4)), una relación, pero no una función, deB
en A. Queremos analizar las condiciones en las que la inversa de una función da como
resultado una función; pero antes de pasar a un nivel demasiado abstracto, consideremos
el ejemplo siguiente.

ParaA = {I, 2, 3) y B = {w, x; y), sea/: A ➔ Bdada por/= {l, w), (2,x), (3,y)). Entonces
/'= ((w, !). (x, 2), (y, 3)) es una función deB en A y observarnos que/'o/ = 1, y/o/'= 1,.

Este ejemplo finito nos lleva a la siguiente definición.

Definición 5.21 Si/: A ➔ B. entonces se dice que/es invertible si existe una función g: B ➔ A tal queg of
1,..y/og ::= 18 .
::=

Nótese que la función gen la definición 5.21 también es invertible-.

Sean f. g: R ➔ R definidas por /(x) = 2x + 5, g(x) = (ll2Xx - 5). Entonces (g o fXx) =


g(j(x)) = g(2x + 5) = (l/2)[(2x + 5)- 5] = x, y (/o g)(x) = f(g(x)) = f((ll2Xx - 5)) = 2((1n)
(x- 5)) + 5 ::= x. por toque/o g ::= lay g of = JR. En consecuencia,/y g son funciones
invertibles.
5.6 Composición de funciones y funciones inversas 285

Habiendo visto algunos ejemplos de funciones invertibles, deseamos ahora mostrar


que la función g de la definición 5.21 es única. Por lo tanto, encontraremos los medios
para identificar una función invertible.

IEOREMA 5.7 Si una función/ : A ➔ Bes invertible y una funcióng: B ➔ A satisfaceg o/== lAyfo g ==
18 , entonces esta función g es única
Demostración: Sig noes única. entonces existe otra funciónh: B ➔ A con h of== 1A yf oh==
t.,. En consecuencia, h =h o 18 == h o (fo g) =(h o /)g = lA o g == g.

Como resultado de este teorema, llamaremos a la función g la inversa de Jy adoptare-


mos la notación g = ¡-1 • El teorema 5. 7 también implica que¡-•= f'.
u-
Podemos ver que cuandofes una función invenible, también Jo es la funciónJ-1, y 1)-1 ==
f. otra vez por la unicidad del teorema 5.7. Pero todavía no sabemos qu6 condiciones sobre
/garantizan que/sea u.na función invenible.
Antes de plantear nuestro siguiente teorema, notemos que todas las funciones invenibles
de los ejemplos 5.57 y 5.58 son biyectivas. En consecuencia, estos ejemplos dan sentido a1
siguiente resultado.

TEOREMA 5.8 Una función/ : A ➔ Bes invenible si y sólo si es inyectiva y sobre.


Demostración: Si f: A ➔ Bes invertible, tenemos una ónica funcióng: 8-+ A congo f == 1.-,
/ o g = 1,. Si a,, a, E A con/(a,) = /(a,). entoncesg(f(a,)) = g(f(a,)), o(g o /)(a,) = (g o/)
(a:J. Comog of= lA> se sigue quea1 =Gi, por lo que/es inyectiva. Para la propiedad de ser
sobre. sea b E B. Entoncesg(b) E A, por lo que podemos hablar de/(g(b)). Como/ o g =
1,. tenemos que b = 1,1.b) = (fo gXb) = f(g(b)). y /es sobre.
Recíprocamente, supongamos que f: A ➔ Bes biyectiva. Como/ es sobre, para cada
b E B existe una E A conf(a) = b. En consecuencia, definirnos la funcióng: B ➔ A como
g(b) ;- a, donde/(a) == b. Esta definición produce una ónica función. El único problema
que podría surgir es si g(b) = a, ;, a,= g(b) ya que /(a,) = b = /(a,). Sin embargo, esta
situación no puede suceder si/ es uno a uno. Nuestra definición de ges tal que g o/= l,4 y
fo g = 1. , por lo que encontramos que fes invenible, con g = J-1•

~~J Por el teorema 5.8 se sigue que la función/,: R ➔ R definida por/,(x) = x' no es invertible
(no es inyectiva ni sobre), pecrof,: [O,+ oo)➔[O. + oo) definida por/iC,x) = ..res invertible y
t,'(x) =,Íx.

El siguiente re sultado combina las ideas de composición de funciones y la inversa de


una función. La demostración se deja al lector.

Tq)REMAS.9 Si f: A -4 B , g: B ➔ C son funciones invertibles, entonces g o/: A ➔ Ces invertible y


(,g o/fl :::/ -1og•I.
286 Capítulo 5 Relaciones y funciones

Habiendo visto algunos ejemplos de funciones y sus inversas, nos preguntamos si exis•
te un método algebraico para determinar la inversa de una función invertible. Si la funcióo
es finita, simplemente intercambiamos las componentes de los pares ordenados dados.
¿Pero qué sucede si la función está definida por una fónnula, como en el ejemplo 5.59?
Afortunadamente, el álgebra demuestra ser un poco más que un análisis cuidadoso de
•'intercambio de componentes de pares ordenados... Esto se demuestra en los siguientes
ejemplos.

--~'~;60. Para m, b E R, m ;< O, la función!: R ---, R definida por f = {(x, y) 1y = mx + b) es una


función invertible, ya que es inyectiva y sobre.
Para obtener J-1 notamos que

¡-' = {(x ,y)IY = mx + b}' ={(y,x) jy =mx + b)


= {(x,y)lx = my + b} = {(x,y)IY = (1/m)(x - b)}.
Aquí ts donde qu~r;11mos interc.ambiar
las componentes di! los pares ordenados di! f .

Así.f: R---, R está definida porf(x) =mx+ b, yJ-': R ➔ R estádefinidapor ¡-'(x) =


(llmXx-b).

~!;_.6_t.' Seaf: R---, R-definida porf(x) = e", donde< = 2.7183, la base del logaritmo natural. De
la gráfica de la figura 5.9 vemos que/ es inyectiva y sobre, por lo que ¡-1 : R· ➔ R existe
y¡-'= {(x. y) I Y=e")' = {(x, y) 1x = <'} = {(x. y) 1 y = lnx). dedondeJ-'(x) = lnx.
Debemos notar que lo que sucede en la figura 5.9 sucede en general. Es decir, las
gráficas de/y¡-1 son simétricas respecto a la recta y = x. Por ejemplo, el segmento de recta
que conecta los puntos (1,e) y (e, l)es bisecado por la recta y :;; x. Esto vale para cualquier
par de puntos correspondientes (x,f(.r)) y (f(x),¡-'(f(x))).

,-,

Figura 5.9
5.6 Composición de funciones y funciones inversas 287

Este ejemplo también produce las siguientes fórmulas:

X= la(x) = (¡'•n(x) = ln(e'), para todo X E R.


x = la•(x) = (/•¡ 1)(x) = , • •, para todo x > O.

El resultado x = eta ~, para x > O, es bastante útil. En la implementación estándar de


Pascal, no existe la exponenciación. Para determinar 23, podemos aplicar la multiplicación
repetitiva, pero esto es ineficaz si se trabaja con un número como ( 5.73)'32 . Como exp y ln
son funciones definidas en Pascal, podemos determinar (5.73}°2 volviéndolo a escribrir
comoe".J:uo(,.ni, ya que 5.73 = ~ 573l de la fónnula anterior. Esto produceexp(4.32 • In(S.73))
en Pascal.

Aun cuando una función/: A ➔ B no sea invertible, el símbolo¡-• tiene uso en el


siguiente sentido.

o.finición 5 .22 Si/: A ➔ By B, <:;; B, entonces f"'(B,) = (x E A l f(x) E B,). Al conjuntof"'(B,) se le


conoce como la preimagen de B 1 mediante f

¡Cuidado! Aunque tengamos el concepto de una preimagen para cua1quier función, no


toda función tiene una función inversa. En consecuencia, no podemos suponer la existen-
cia de una inversa para una función/ sólo porque se usa el símbolo ¡-1• En este caso, se
necesita un poco de precaución.

l'&.nmra'!.,t:z,a SeaA,B <:;; donde A= ( 1, 2, 3, 4, 5, 6) y B = (6, 7, 8, 9, IO). Si/: A ➔ B conf = {{I, 7),

(2, 7), (3, 8), (4, 6), (5, 9), (6, 9)}, entonces se obtienen los siguientes resultados.

a) Para B, = (6. 8) <:;; B. tenemos quef"'(B,) = (3,4). ya que/(3) = 8 y/(4) = 6, y para


cualquier a E A,/(a) $. B 1 a menos que a= 3 o a::: 4. También aquf notamos que
lt·'(B,) 1=2 =1-B, l-
b) En el caso de B,= (7, 8) <:;; B, como/(!) = /(2) = 7 y /(3) = 8, encontramos que la
preimagendeB,mediante/es (1, 2, 3). Y aquí 1
IJ-
(B,) I = 3 >2 = 1B, l-
e) Ahora consideremos el subconjunto 8 3= {8, 9 } de B. Para e ste caso se sigue que
f"' (B,) = (3, 5, 6 ), ya que /(3) = 8 y /(5) = /(6) = 9. También encontramos que f"'I
(B,) 1=3 > 2 =1B, I-
d) Finalmente, si B, = { 8, 9, 10) <:;; B, entonces, con/(3) = 8 y/(5) =/(6) = 9, tenemos
= =
quef·'(B,) (3, 5, 6 ). Así,f"'(B, ) t·'(B,) aun cuando B,:, B,. Este resultado se
sigue del hecho de que no hay ningún elemento a en el dominio A tal que/(a) ::: 10,
es decirJ-'(( IO)) = 0.

Cuando f: A ➔ B, entonces para cualquier b E B escribiremos/·'(b) en vez def"'((b}).


Para la función del ejemplo 5.62, e ncontramos que

¡-1(6) = {4} ¡-1(7) ={1,2} ¡-'(8) = {3} ¡-'(9) ={5,6} ¡-1(10) = 0.


288 Capítulo 5 Relaciones y funciones

LEje,nplo S.ti j Sea f R ➔ R definida por


x>O
X :S O.

a) Detennine/(O),f(l). /(- l),!(5/3), /(- 5/3),!(2) y /(- 2).


b) Encuentref"'(O),/"'(l )f '(-l)./"'(2),¡-'(-2). /"'(3),/"' (-3), y ¡-'(-ó).
e) ¿Cuáles son los conjuntosf"'((-5, 51) y /"'([-ó, 51)?

a) /(0) = -3(0)+1 = 1 /(5/3) = 3(5/3) - 5 = O


/(!) = 3(1)-5 = - 2 /(-5/3) = -3(-5/3)+ 1 =6
/(-1)=-3(-1)+1=4 /(2) = 3(2)-5 = 1
/(-2)= -3(-2) + 1 =7
b) ¡-1(0) = {x ERl/(x) E {O}} = {x ERl/(x) = O}
= {x ERlx > OY3x -5 = O}U{x ERlx sO Y -3x + 1 = O}
= {x ERJx > Oy x =5/3}U{x E R Jx s oy x = 1/3)
= {5/3) U 0 = {5/3)
[Note que la línea horizontal y :;:; O(es decir, el eje _t) interseca la gráfica en la
figura 5.1 O únicamente en el punto (5/3, O).]

-5 (O. - 5)
-6
Figura 5.10

¡-'(l) = {x ERl/(x) E {l}} = {x ERJ/(x) = l}


= {x ERlx >O y 3x - 5 = l}U{x E R lx s o y - 3x + 1 = !}
= {x ERJx >O yx = 2}U{x E RJx s O yx = O}
= {2} U{0} = {0,2}

{Aquí observamos que la línea punteada y :;:; 1 inters.eca la gráfica de la figura


en los puntos (O, 1) y (2, l).J
5.6 Composición de funoones y funciones inversas 289

¡-1(-l) = {,-E Rjx>Oy:k-S= - l}U{,-E R jxs OY-3x + 1 = -1}


= {,-E R lx >O yx = 4/3}U{,-E R jx sO Yx =2/3)
= {4/3} u e- {4/3J
¡-1(2) • {-1/3, 7/3} ¡-'(- 2) = {!}
r'<-3> = 12131
¡-'(3) = {- 2/3,8/3}
¡- (-6)•{,- ERjx>Oy:k-S • -6}U{,- ER jx sOy -3.r + 1 = - 6}
1

• {x ERlx>O y x = -1/3}U{xER lxsOy x =1/3)


-0u0-0
e) ¡-1([-S.S] ={,- jf(x) E (-5,S] ={,-1-Ssf(x)sS).
(Casol)x>O: -5s:k-5s5
Os:kslO
Os x s 10/3-usamos O< x s 10/3.
(Caso 2) X S 0: -5 s-3x+ ls5
-6s-:k s4
2"x ac - 4/3-usamos - 4/3 sx sO.

Entonccs/-'([- 5. 51)= (x 1-4/3 s x ,e Oo O<x ,e 10/3) = [-4/3, 10/3). Como


no hay puntos (x, y) en la gráfica (de la figura 5.10) tales que y ,e -5. se sigue
de nuestros cálculos que¡-'([-ó, 51) =¡-'((-5. 51) =f"'((-5, 5]) =[-4/3. 10/3].

Ejemplo 5.64 a) Sea f; Z ➔ R definida por f(x) = x' + 5. La tabla 5.9 enumera /"'(B) para varios
subconjuntos B del codominio R.
b) Si g: R ➔ R se define como g(x) = x? + 5. los resultados de la tabla 5.10 muestran
cómo un cambio en el dominio (de Za R) afecta las preimágcnes (de la Tabla 5.9).

Tabla 5.9 Tabla 5.10

B r'<B> B
{6} {-1, 1) {6} {-1, 1}
(6, 7) {-1,1} (6, 7) [-Y2, -l)U[l, V2)
(6, 10) {-2, -1 , 1, 2) (6, 10) [-VS, -l)U[l, Y5)
(-4,S) e (-4,5) e
(-4,S) {O} (-4,S) {O}
[5,+=) z [5,+=) R

En nuestro siguiente resultado aparece el concepto de preimagen junto con las opera-
ciones de conjuntos (intersección. unión y complemento). El lector debe notar la diferen-
cia entre la parte (a) de este teorema y la pane (b) del~teorema 5.2.
290 Capítulo 5 Relaciones y funciones

TEOREMA 5.10 Si f: A ➔ 8 y 8 ,, 8, ~ 8 , entonces ( a ) ~ n 8,) = /·'(8,) n J-'(B1);


(b)J-'(8 , U 8,) =¡-' (8 ,) U J-'(8 2); y (c)f·'(8,) =f"'(B,~
Demostración: Demostraremos la parte (b) y dejamos las panes (a) y (e) para el lect<X.
Para a E A, a E f ·'(8, U 8,)<=>/(a) E 8 , U 8 , <=>f(a ) E 8 , of(a) E B, <=> a E ¡-'(B,)o
a E ¡-'(B,) <=> a E /·'(8,) U / · '(8,1.

Utilizando la notación de la preimagen, vemos que una función/: A ➔ Bes inyecti\"!


si y sólo si 1¡-'(b) 1 S I para cada b E 8.

Las matemáticas discretas estudian principalmente los conjuntos finitos, y el última


resultado de esta sección demuestra que la propiedad de ser finito puede llevar a resulta-
dos que no logran ser ciertos en general; además de ilustrar una aplkación del principio
del palomar.

TEOREMA 5.1 1 Sea f: A ➔ 8 una función para los conjuntos finitos A y 8 , donde I A 1= 18 1- Entonces
las siguientes propOsiciones son equivalentes: (a)/cs inyectiva; (b)/es sobre; y (c}/cs
invenible.
Demostración: Ya hemos demostrado en el teorema 5.8 que (e) => (a) y (b), y que (a),
(b) ~ (e). En consecuencia. este teorema quedará demostrado si vemos que para estas
condiciones sobre A, B, (a) <=> (b). Si suponemos (b) y f no es inyectiva, entonces existen
elementos a., a, E A, con a, ;, a,, tales que/(a,) = /(a,). Entonces I A 1> l/(A) 1= 1B 1,
lo que contradice el hecho de que I A 1: j B 1- Por e] contrario, si/ no es sobre, entonces
1/(A) 1 < 18 1- Si I A 1= 18 1tenemos que I A 1 > 1/(A) 1- y el principio del palomar
implica que/ no es inyectiva.

Utilizaremos el teorema 5.11 para verificar la identidad combinatoria del problema 6


que aparece aJ comienzo de este capítulo. Sin E z • y I A 1: 1B 1: n, hay n! funciones
uno a uno de A a B y í: :--0(- 1)1"(;.t)(n - kt' funcíones sobre de A a B. La igualdad
n! L :..0(- l)" <,,:,)(n - k)" es entonces el equivalente numérico de las panes (a) y (b) del
teorema 5. 11 . [Ésta es también la razón por la cual todos los elementos de la diagonal
S(n, n), 1 s n s 8. de la tabla 5.1 son iguales a uno.)

EJERCICIOS S.6 1. Sean/:A ➔ B,g: C ➔ D. Dcfinah:A x C ➔ B xDcomo


h(a,c) = (f(a),g(c)~

Demuestre que hes biyectiva si y sólo si /y g son biye,ctivas.


2. Seanf. g. h: Z ➔ Z definidas por/(x) =x - 1, g(x) =3x,
X par
h(x) ={~: x impar.

Determine (a)/o g,g of, g e h. h o g, /o(g oh), (Jo g) oh;


(b) J' ,f', g', g', h'. h', h"".
3 . Si<i\i. es un universo dado conS, T!'; 1U. (fijos}, definag:~(iU) ➔ :J>('iU.) por g(A} = Tn (SU
A) para A~ '\l . Demuestre quc g: = g.
5.6 CompoS1ci6n de funciones y funciones inversas 291

4. Sea1: N ➔ Ndcfinido por t<•l• 2". S1A • (l. 2, 3.4) y fe A ➔ N está dada por/= {(l. 2).
(2, 3). (3. 5), (4. 7)), encuentre g • f
S. =f. g: R ➔ R,dondegµ)= 1-x+.>'y/C.,) =a x+ b. Si(g oJXr)=9x'-9x+l.detenninea.b.
6. Seant 1: R ➔ R donde/(x) = ttt + by ,<.r) = cr + d para cualquier ;r E R. c:oo a. b, e, d
constantesreale.s.¿Qutrelación(es)debensatisfac:era. b, e, dsi (/• g)(x) = (g•~) pan.todo
xER?
7. a) Para A.= { 1, 2. 3.. .. , 7}, ¿cuántas funciones biycctivas/: A ➔ A sar:isfacen/(1) ¡{t 1"?
b) Rcspoodalap,ne(a)pa,aA=fxlxEZ·.1,;xs•J.
8 . a) Para A = {-2. 7) !;::; R, defina las funcionest g; A ➔ R por
2r2-8
/(x)•2x-4 g(x)•--.
, ... 2
Verifique que/• g.
b) ¿Se afecta el resultado de la parte (a) si cambiamos A por [-7. 2)"?
9. Si A. B s;; "U y 91,.91,s;; Ax B. demuestre que (a) (91, U 91,Y = !it¡U91j; (b) (9!f n 91,)' •
9!l n 911: y (e) (91\)' • 91,.
1O. Para cada una de las siguientes funciones/: R ➔ R. detemtine si/ es invertible, y si lo es,
determine J 1

•> f • {(x,y)l2x + 3y • 7) b) f•{(x,y)lax+by•c,b,'O}


e) /•{(x,y)iy•x'} d) /•{(x,y) (y . ,•.,.,¡
11. a) Encuentre la inversa de Ja función/: R ➔ R· definida por/(x) • ~:s.
b) Demuescrcque/•r = 1•• yr•/= la.
12. Dccerminer' para(a)/: R ➔ R.f(x) = -.r.(b)/: R' ➔ R1./(x, y)•(y, x); (e)/: R' ➔ (Rx R·J.
f(x, y) • (5.x; e").
13. Demuestre el teorema S.9.
14. Si A, 8 !;; z· conA • (1, 2, 3,4, 5, 6, 7) y B = {2,4,6.8, 10.12}. y/:A-+ Bdondc/=
{(l. 2), (2, 6), (3, 6), (4, 8), (5, 6). (6, 8), (7, 12)}, decermine la preimagen de81 mediante/en
cada uno de los siguientes casos.
a) B, •{2} b) B, ={6} <) B,={6,8}
d) B,•{6,8,10} t) B,• {6,8,10,12} f) B, • {10, 12}
1S. Sea/: R ~ R definida por

.r+7. .r:sO
f(x)- { - 2x+5, O<x<3
x-1, 3s.r
a) Encucnuej"(-10). /"(O). /"(4)./''(6)./"(7) y /"(8).
b) Determine la preimagen mediante/de cada uno de los intervalos (i) [- 5, -1): (ii) (- S. O);
(iii) (-2, 4]; (iv) (5, 10); y. (v) [11. 17).
16. Sea/: R ➔ R dcfiruda por /(x) = r. Para cada uno de los siguientes subconjuntos B de R.
encuenue j'(B).
a) B •/0,l) b) B•(-1,0,1) e) B•[0,1]
d ) B•[0, 1) •l B •[-1,1} 1) B ~[0,4]
g) B•[O,l]U[4, 9] b) B•(O, l]U(4,9)

Determine tres subconjuntos infinitos 8 de R para los cuales¡-1(8) = e.


17. Sea A, B s;; z· donde A= (l. 2. 3.4. 5} y B • (6. 7, 8. 9, 10, 11. 12}. ¿Cuántas funciones/:
A ➔ B son talC$que/"(16, 7. 8}) = (1, 2}1
292 Capitulo 5 Relaciones y funciones

18. Sea/: R ➔ R definida por /(x) =LxJ.


el mayor entero menor o igual quex. Encuentrer(B'
para cada uno de los siguientes subconjuntos B de R
a) B= {O,!} b) B= {-l,0, 1} e) B = [O,l)
d) B=[0,2) e) B=[-1, 2) f) B = [O,l]
g) B-[-1,2] h) B = [-1,0)U(l,3]
19. Sean/, g: z• ➔ z• donde para todox E z•,J(.x) = .x + 1 yg(x) = máx{J, x - 1), el máximo de
1 y X - J.
a) ¿Cuál es la imagen de.f? b) ¿Es /una función sobre? e) ¿Es la función/uno a uno?
d ) ¿Cuál es la imagen de g? e) ¿Es g una función sobre? O¿Es la función g uno a uno?
g) Pruebe que g o/= Ir. h) De1ermine(fo g)(x)parax = 2, 3, 4, 7, 12 y 25.
i) ¿Las respuestas de las partes (b), (g) y (h) contradicen los resultados del teorema 5.8?
20. Sean/, g, h, k: z ·x z · ➔ z· tas operaciones binarias cerradas definidas para todos (a, b) E
z·x z· por
f(a,b) = a + b, g(a,b)=ab,
h(a,b) = mfn{a,b}, k(a,b) =máx{a ,b}.

a) ¿Son estas funciones uno a uno?


b) ¿Sonf. g, h y k funciones sobre?
e) ¿Son estas fundones invertibles?
d) Deiennine¡-'(3). ¡·'(4). ¡-'(5). g'(4), g·'(6). g·'(7). g·'(8). y g·'(l6).
e) Paran E z·.
¿cuán10 vale IJ
1(n) I?
O ¿Oiántovale j g·'(p) 1- parap primo? ¿Oiánto vale I g·'(p') !? ¿Qiámovale I g·'(p") 1pan
m EZº?
g) Si p, q son primos distintos y m, n E z·, ¿Cuánto vale¡ g-1(¡,""q") I?
h) ¿Son infinitos a1gunos de los siguientes conjuntos?

i) DeJerminc el número de elementos de cada uno de los conjuntos finitos de la parte (h).
21 . Sean f. g, h las siguientes operaciones binarias cerradas en 3'(Zº) . Para A, B C: z·,/(A, B) =
A n B,g(A, B) = A U B, h(A, B) = A ll B.
a) ¿Son eslas funciones inyectivas?
b) ¿Son/. g y h funciones sobre?
c) ¿Es invertible alguna función?
d) ¿Es infinito alguno de los siguientes conjuntos?
(!) r'(9) (2) g''(8) (3) h''(8)
(4) /''({!}) (5) g''({2}) (6) h''({3})
(7) /''({4, 7)) (8) g' '({8, 12}) (9) h' '({5, 9})
e) Determine el número de elementos de cada uno de los conjuntos finitos de la parte (d).
22. Demuestre que/: A ➔ Bes inyectiva si y sólo si 1J-1(b) 1 :S 1 para todo b E B.
23. Demuestre las panes (a) y (c) del teorema 5. 10.
24. a ) Dé un ejemplo de 1,.::ia funciónf: Z ➔ Ztal que/sea(i)unoaunoperonosobre; y(ii)sobrc
pero no uno a uno.
b) ¿Contradicen los e~emplos de la pane (a) el teorema 5.ll?
25. Sea/: Z ➔ N definida por

si x> O
si x so.
a} Demuestre quejes inyectiva y sobre. b) Determiner1•
= =
26. Si J A 1 1B 1 5, ¿cuántas funciones/: A ➔ B son inv&tibles?
S. 7 Complejidad computacionaJ 293

27. Seanf. g. h, k: N __, N calesquef (n) = 3n,g(n) = ln!3J, h(n) = l(n + 1)13J, Yk(n) = L<• + 2)13j,
para cada n E N . a) Para cada n E N ¿cuánto va1en (g o JX.n), (h o /)(n), y (k o /)(n)'? b)
¿Contradicen los resultados de la parte {a) el teorema 5.7'?
28. Sean/:A ➔ B, g: B ➔ C. Demuestre que (a) g o/: A ➔ Csobre ~ g sobre; y, (b) g of:A ➔ C
inyectiva ⇒ /inyectiva.

5.7
Complejidad computacional'

En la sección 4.4 presentamos e1 concepto de algoritmo, siguiendo los ejemp]os estableci•


dos por el algoritmo de la división (de Ja Sec. 4.3) y el algoritmo de Euclides (de la Sec.
4.4). En esos ejemplos tratamos ciertas propiedades de un algoritmo general.

• La precisión de las instrucciones individuales paso por paso


• La entrada proporcionada al algoritmo y la salida que el algoritmo proporciona
• La capacidad del algoritmo para resolver ciertos tipos de problemas, no sólo casos
específicos del problema
• La unicidad de los resultados intermedios y finales, basados en la e ntrada
• La naturalez.a finita del algoritmo, en el sentido de que termina desputs de la ejecu•
cióo de un número finito de instrucciones

Cuando un algoritmo resuelve correctamente cierto tipo de problema y satisface estas


cinco condiciones, entonces es posible seguir analizándolo como sigue.

1) ¿Podemos medir de alguna manera lo que rarda el algoritmo en resolver un proble-


ma de cierto tamaño? Si logramos hacerlo, esto puede depender, por ejemplo, del
compilador uúlizado, por lo que queremos desarrollar una medida que no dependa
de aspectos como el tipo de compilador, la velocidad de ejecución u otras caracte-
rísticas de un computador dado.
Por ejemplo, si queremos calcular n! para n E z·, ¿con qué rapidez podemos
hacerlo? Si queremos calcular et' para a E R y n E z •, ¿existe alguna ..función den"
que nos describa ta rapidez con la que un algoritmo dado para ta1 exponenciación
lleva a cabo esta tarea?
2) Supongamos que podemos contestar preguntas como la planteada antes en e1 punto
l. Entonces, si tenemos dos (o más) algoritmos que resuelven un problema dado,
¿habrá tal vez una forma de determinar si un algoritmo es "mejor" que otro?

t El material de las secciones 5.7 y 5.8 puede dejarse por el momento. No se utitiz.ari mucho hasta d
capítulo 10 . El único lugar donde aparcccni este material antes <k-1 capítulo 10 es el ejemplo 7.13. pero é:ste
puede omitirse sin perder la continuidad.
294 Capítulo 5 Relaciones y funciones

En particular, supongamos que queremos analizar el problema de determinar si cierto


número real x e stá presente e n la lista den ntlme ros reales a 1, tzi, .. . , a ... Aquf tenemos 111
problema de tamaño n.
Si ex.iste un algoritmo que resuelve este problema, ¿cuánto puede tardar en hacerlo?
Para medir esto, buscamos una función /(n), l1arnadafundón dt complejidad en tiempo'
de l algoritmo. Esperamos que el valor de/(n) (tanto aquí como en general) se incremct11e
al incrementarse n. Además, nuestro interés principal aJ tratar cualquier algoritmo es la
forma en que se compona éste con valores grandes den.
Para analizar lo descrito hasta ahora de manera general, necesitamos presentar la si-
guiente idea fundamental

Definición 5.23 Sea n/, g: z + 4 R . Decimos que g dominaaf(of esdominada por g) si eXJsten constantes
m E R+ y k E z • tales que l/(n)I s m lg(nll paratodon E Z',dondenlek.

Observemos que si consideramos los valores de/(!), g(1),/(2), g(2), . . . , hay un punto
(a saber, k) después de] cual el tamaño de /(n) está acotado en la pane superior por 111
múltiplo positivo (m)del tamaño deg(n).Además, sig dolJ\IDa af, entonces IJ<nYg(n) 1 S 11
[es decir, e] tamaño del cociente /(n)/g(n) está acotado por m), para aquellosn E z• taJes
que n;, ky g(n) a< O.
Cuando fes dominada por g, utilizamos 1a notación Uamada "O mayúscula" para esto,
escribiendo/E O(g), donde O(g) se lee como ..de orden g .. o bien "O mayúscula deg".
Como Jo sugíerc la notación"/ E O(g)", O(g) representa el conjunto de todas las funciones
con dominio z• y codominio R dominadas por g. Estas ideas se demuestran en los siguien-
tes ejemplos.

Sean/, g: Z•➔ R dadas por/(n) = 5n,g(n) ;n', paran E z•.Si calculamos/(n) y g(n)para
1 s n ,s 4, encontramos que/(1) = 5, g(I) = l;/(2) ; 10, g(2); 4;/(3) = 15, g(3); 9; y,
/(4) = 20, g(4) ; 16. Sin embargo, n le 5 ,,. ri' le 5n, y tenemos que l/(n) 1= 5n Sri' =
1g(n) I• Así, si m; 1 y k; 5, tenemos que n le k, lfi'.n) 1 S m lg(n) 1- En consecuencia, g
domina a/y/ E ()(g). [Observe que IJ(n)lg(n) 1está acotado por 1 para todo n;, 5.]
También nos damos cuenta deque para cualquiern E z•, IJ(n) 1= 5n s 5n'; 5 lg(n)I-
Aquf se muestra el dominio deJ-por g, con k ;;;; l y m = 5. Esto es suficiente para demostrar
que las constantes k y m de la definición 5.23 no nec~itan ser únicas.
Además, podemos generalizar este resultado si consideramos ahora las funciones/i, g1:
z• ➔ R definidas por / 1(n) _,,,, an, g 1(n) = bn2 , donde a. b son números reales distintos de
cero. Si m E R• y m lbl le la l, entonces para todo n le 1 ( = k), l/,(n)I = lan l = la l n :s
m lbl n S m lbl n' = m lbn' 1= m I g,(n)I, y así.ti E ()(g,).

En é1 ejemplo 5.65, observamos que/E O(g). Si revisamos las funciones/y g, quere-


mos ahora mostrar que g e O(j).

t Podñamos 1ambién estudiar la función de complejida.d en espacio de un algorinno, que necesitamos


cuando intcniamos medir la cantidad de memoria requerida para la ejecución de un algoritmo en un problema
de tamailo .11_ Sin embargo, en we texto limitaremos nuewo estudio a la función de complejidad en tiempo.
5. 7 Complejidad computacional ~95

Una vez más, sean f. g: z• ➔ R definidas por/(n); 5n, g(n) ; n', paran E z•.
Si g E 0(/}, entonces en t6rminos de cuantificadores, tendríamos que
3m E R ' 3k EZ'Vn EZ' [(n a::k)~lg(n)lsml/(n)IJ.
En consecuencia. para mostrar que g <l. O(f), necesitamos verificar que
Vm E R'Vk EZ' 3n EZ' [(na:: k) /\(lg(n)I >ml/(n)I)].
Para esto, primero debemos darnos cuenta de que m y k son arbitrarios. por lo que no
tenemos control sobre sus valores. El único número sobre el cual tenemos control es el
entero positivo n que seleccionarnos. Ahora bien, independientemente de los valores de m y
k., podemos seleccionarn. E z· talcsquen >máx.{5m, k}. Entoncesn ~k(en realidad, n > k}
yn >5m ;> n'>5 mn, por lo que lg(n)I ; ,r>Smn ; m ISnl ;m l / (n) I Y g ~ O(f).
Para quienes que prefieren el método de demostración por contradicción, presentamos un
segundo punto de vista. Si g E O(j), entonces tendríamos que

n' ; lg(n )I smlf(n)I ;mn


para todo n ~ k, donde k es algún entero fijo positivo y m es una constante (real}. Pero
entonces, de n2 Se mn deducimos que n s m. Esto es imposible ya que n(E z·) es una
variable que puede crecer sin límite mientras que m es cons"tante.

a) Sean.f. g: Z• ➔ Rdadaspor/(n) ;5n'+3n+ 1 y g(n); n'. Entonces l/(n) I ; 1Sn'+


3n + 1 1; Sn'+ 3n + 1 s 5n2 + 3n'+ n'; 9n'; 9 lg(nl l- De aquí tenemos que para
todo n 2 1 ( ; k), lfl•l l s mlg(n)I para cualquier m;, 9, y/ E O(g). En este caso
también podemos escribir g E O(n').
Además, lg(n)I; n' :s; Sn' :s; Sn'+ 3n + 1; l/(n)I para tqdo n 2 l. Por lo que
1g(n) 1 s ml/Cn) I para cualquier m 2 1 y para todo n 2 k 2 l. En consecuencia, g
E 0(/). [De hecho, O(g) ; 0(/); es decir. cualquier función de z• a R dominada por
fo g está tambi¿n dominada por la otra función. Examinaremos este resultado para
el caso general en la sección de ejercicios.]
b) Consideremos ahora f. g: z· ➔ R. con /(n) = 3n3 + 7n2 - 4n + 2 y g(n) = n3 . Aquí
tenemos que l/(n)I ; l3n'+ 1n'- 4n+ 2 1 s l3n' 1+ 17n' 1 + l-4111 + 12 1 s 3n'+ 1n'
+ 4n3 + 2n1 = 16n3 = t6 lg(n)I, para todo n ~ l. Así, si m= 16 y k = l, tenemos que/
está dominada por g, y f E O(g), of E O(n') .
Como 7n-4 > Opara todo n ~ 1, podemos escribir n 3 s 3n3 s 3n 3 + (7n - 4)n +
2 sin 2 l. Entonces lg(n)I :s; l/(•l l para todo n 2 1, y g E O(fJ. [Como e n la parte
(a), también tenemos O(f) ; O(g) ; O(n') en este caso.]

Generalizaremos los r<Sultados del ejemplo 5.67 como sigue. Sea/: Z• ➔ R tal que/(n) ;
a,n'+~i,t-- 1 + • • • +a2n1 +a1n + ao., para a,. a,._1, . .. ,a,, a .. ao E R, a, :itO, t EN. Entonces

1/(n)I; la,n' + a,_,nH + · · · + a,n' +a,n +ao l


s la,n1+ laHn'-11+ · · · + la,n'I + la , n l + laol
; laJn ' + la,-1lnH + · · · + la,ln' + la ,ln + laol
:s IaJn' + 1a,_,¡,., + · · · + 1a,ln' + 1a,ln' + 1ao\n'
_ti _, , 1_ 1 , ..1.. 1,. l ..1. 1,. l ..1- l,,.1\n1
96 Capítulo 5 Relaciones y funciones

En ladefirución5.23, seam= 1a, 1 + 1 a,, I +· · ·+ 1a, 1 + 1 a, 1+ J a, 1 y k= l,yscag:


z · ➔ R dada por g(n) = n'. Entonces IJ(n) 1 ,s m lg(n) 1 para todo n ;, k, por lo que f está
dominada por g, o/ E O(n').
También es cierto que g E O(f) y que O(f) • O(g) = O(n').
Esta generalización proporciona los siguientes resultados acerca de algunas sumas.

a) Seaf: z · ➔ R dada porf(n) = 1 + 2 + 3 + • • • + n. Entonces (de los ejemplos 1.38 y


1.41),f(n) = (½)(n)(n+ 1) = (½)n' + (-j-)n, por lo quef(n) = I ;.,¡ E O(n 2 ).
b) Sig: Z· ➔ R con g(n) = l2 + 2 1 + 31 + • • • +n1 = (t)(n)(n + 1)(2n + l )(del ejemplo
4.3), entonces g( n) =(½)n3 + (½)n 2 + C-¼)n E O{n3 ).
e) Si t E z· y h : Z· ➔ R está dada por h(n) = L ;_/, entonces h(n) = l' + 21 + 3' + •
+ ,t ::s;: n' + ,t + n1 + · · · + n1 =n(n' ) = n ...1, por lo que h(n) E O(n,.1 ).

Ahora que hemos analizado varios ejemplos de dominio de funciones, cerraremos esta
sección con dos últimas observaciones. En la síguiente sección aplicaremos la idea de
dominio de una función en e l análisis de algoritmos.
1) Al analizar el concepto de dominio de una función, buscamos la mejor cota (o la
má.s fina) en el siguiente sentido. Supongamos que f. g, h: z· ➔ R son tres funcio-
nes tales que JE O(g) y g E O(h). Entonces también tenemos que JE O(h). (Pedi-
remos la demostración de este hecho en los ejercicios.) Sin embargo, si h (t. O(g), la
proposición f E O(g) es una ''mejor" cota sobre I f(n) 1 que la proposición f E
O(h). Por ejemplo, sif(n)=5,g(n)=5n yh(n) =n', paratodon E z ·,entoncesfE O(g),
g E O(h) y f E O(h) pero h <;: O(g). Por lo tanto, nos interesa más la proposición
JE O(g)quefE O(h).
2) Ciertos órdenes, como O(n) y O(n2 ) , aparecen con frecuencia cuando e studiamos el
dominio de funciones. Por lo tanto, se designan con nombres particulares. Algunos
de los más importantes de estos órdenes se enumeran e n la tabla 5.11.

Tabla 5.11
Forma O mayúscula Nombre

0(1) Constante
O(log,n) Logarítmica
O(n) Linea]
O(n log,n) n 1o& n
O(n') Cuadrática
O(n') Cúbica
O(n~),m =O, 1,2,3, ... Polinomial
O(c"),c > 1 Exponencial
O(n!) Factorial
5.8 Análisis de algoritmos 297

os 5.7 1. Use los resu1tados de la tabla 5.11 para determinar la mejor forma "O mayúscula" para cada
una de las siguientes funciones/: z• ➔ R.
a)/(n)=3n + 7
b)/(n) =3 +scn(l/n)
e)/(n) = n' -Sn' + 25n - 165
d)/(n) = Sn' + 3n log,n
e)/(n) = n' + (n - 1)'
r)/(n) = (n)(n + l )(n + 2)/(n + 3)
a)/(n)=2+4+6+ .. · + 2n
2. Sean f. g: z· ➔ R las funciones tales que /(n) = n y g(n) = n + (1/n), para n E z·. Use la
definición 5.23 para mostrar que/E O(g) y g E 0(/)
3. f.n los siguientes incisos,f, g: z·--+ R. Use la definición 5.23 para mostrar que g domina aj.
a) /(n) = 100 log,n, g(n) = (l)n
b) /(n) =Z', g(n) = 2"" - 1000
e) /(n)=3n', g(n)•Z'+2n
4. Sean/, g: Zº--+ R las funciones tales que/(n) =n + lOOy g(n) =n2• Use la definición 5.23 para
mostrar que/E O(g) pem g ~ O(j).
5. Sean/. g: z·➔ R las funciones/(n) = n 2 +n y g(n) =(½)n3, paran E z·. Use la definición 5.23
para mostrar que/E O(g) perog 'l. O(f). ·
6. Sean/, g: z• ➔ R dadas po,

f(n) = {n, paran impar


1, paran par

g(n) = ¡1, paran impar


n, paran par

Verifique que/~ O(g} y g ~ O</).


7. Scanf. g: z· ➔ R donde /(n) = ny g(n)= login, paran E z·. Mucstreque g E O(J) pero /E
O(g). (Suge~ncia: lím - " - = +oo. Para esto se requiere usare] cálculo.)
•➔• log1 n
8. Seanf. g, h: z ·--+ R donde/ E O(g) y g E (Xh). Demuestre que/ E O(h).
9. Si g: z•,R y e E R. definimos la funcióncg: z· ➔ R como (cgXn) = c(g(n)), para cadan E
z•. Demuestre que sif. g: z·--+ R son tales que/ e O(g), entonces/ E O(cg) para cualquier e
ER.c~o.
10. a) Demuestre que/ E O(j) para cualquier/: z·➔ R.
b) Seanf. g: Z•➔ R. Si/E O(g) y g E 0(/),demuestrequeO(/) = O(g). Es decir,demuestre
que si una función h: z· ➔ R está dominada por/, entonces h está dominada por g y
viceversa.
e) Sif. g: z · ➔ R, demuestre que si 0(/J = O(g), entonces J E O(g) y g E O(J).

5.8
Análisis de algoritmos
Ahora qu.e ya hemos presentado al lector el concepto de dominio de una función, es el
momento de ver la forma e n que se usa esta idea en el estudio de los algoritmos. En esta
sección prese ntaremos nuestros algoritmos como programas o segmentos de programas
Capitulo 5 Relaciones y funciones

en Pascal. (También presentaremos los algoritmos como listas de insuucciones. El lector


verá que éste será e l caso en capítulos posteriores.)
Comencemos con un programa que calculan! paran E z•.

En la figura 5.11 reproducimos un segmento de programa en Pascal que implementa un


algoritmo para el cálculo den! paran E z•. En este caso, el usuario introduce el valor de
n, que es el dato del programa. Las variables; y Factorial (ya declaradas con anterioridad
en el programa) son variables enteras.

Begin
i := l ; {Inicializa el contador }
Factorial : = 1: {Inicializa el valor de Factorial}

While i < • n do
Begin
Factorial : = 1 *Factorial;
í := i + l
End ;

Wr i teln ( 'El valor de n, ' factorial es Factorial, ' . ' )


End;
Figura 5.11

Nuestro objetivo es contar (medir) el número total de operaciones (como asignaciones,


sumas, multiplicaciones y comparaciones) implicadas en el cálculo den! e n este progra-
ma. Sea/(n) el número total de estas operaciones. (Entonces,/: z· ➔ R).
El programa comie nza con dos pl"Oposiciones de asignación, donde se inicializan los
valores de las variables enteras i y Factorial. Después se ejecuta el ciclo While n veces.
Cada una de estas ejecuciones implica las siguientes cinco operaciones:

1) Comparar el valor actual del contador i con n


2) Incrementar el valor de Factorial como i * Factorial; esto implica una multiplica-
ción y una asignación
3) lncl"ementar el valor del contador en 1; esto implica una suma y una asignación

Por 11ltimo, hay otra comparación. Ésta se realiza cuando; = n + l. de modo que el ciclo
While se termina y las otras cuatro operaciones (de los pasos 2 y 3 anteriores) no se JJevan
a cabo.
Por Jo tanto,/(n)= 2 + 5n + 1 = 5n + 3, por !oque/E O(n). En consecuencia. decimos
que este programa implementa un algoritmo O(n) o que el algoritmo tiene una compleji-
dad lin1tal m til!mpo. Si todas las operaciones implicadas tardan el mismo tiempo en eje-
cutarse, podemos ver que la función/"mide" el tiempo de ejecución. Sin embargo. si sólo
supiéramos que/ E O(n), entonces sabríamos que (1) el términodominanuen/erany (2)
el tiempo de ejecución es aproximadamente en, donde e es una constante que depende de
consideraciones como las características específicas del sistema de cálculo.
5.8 Análisis de algoritmos 299

Nuestra principal preocupación en este caso es que el ténnioo dominante sean, y que,
en consecuencia.JE O(n). Ya que sin es cada vez más grande. el ""orden de magnitud" de
Sn + 3 depende principalmente del valor n, el número de veces que se ejecuta el ciclo
While. Por lo tanto. podñamos haber obtenido/ E O(n) contando simplemente el número
de veces que se ejecutó el ciclo While. Usaremos este tipo de atajos en los cálculos de los
siguientes ejemplos.

~ En el ejemplo 4. 17 de la sección 4.2 presentamos los números de Fibonacci FO' F1, F!, F,•
. . , los cuales se definen en fonna recursiva como

1) fó-0, f; - 1; y,
2) F. - F._, + F.-,, paran .o 2.
El programa en Pascal de la figura 5.12 puede usar>< para obtener el valor de F. para un
entero no negativo n dado. En este algoriono iterativo calculamos F,,_ (en e l caso donde n ~
2) asignando o calculando primero todos los valores anteriores Fo. F1, F2, • •• , F_ 1• Aquí
definimos la función de complejidad/: N ➔ R de forma que /(n) cuente el número de

Program FibNuml ( input, ou tput) ;

Var
i . n. Fib, Last. NexttoLast, Temp : integer;
Begin
Wri teln( · ¿Para qué entero no negativo n'}
Wri teln ('desea encontrar el n\1.mero de P'ibonacci ?')
Write ( 'n=' ):
Readln(n);

It n•Othen
Fib : - O:
Ir n =- 1 then
Fib : • 1:
It n >- 2 tben
Begin
Last :• l;
NexttoLast O;

•, n, ' el n-ésimo minero de Fib:rl&cci es Fib, ' . ')

Figura 5.12
1() Capítulo 5 Relaciones y funciones

sumas realizadas en el ciclo For (sombreado) de la figura 5.12. Encontramos que/(0) =O,
/(1) = 0,/(2) = 1,/(3) = 2,/(4) = 3, . .. En general, para cualquier n E z• tenemos/(n) =
n - 1 y, en consecuencia.JE O(n).
[En la sección 2 del capítulo 1O presentaremos un segundo programa en P~al para el
cálculo de Jos números de Fibonacci Fo. F 1, F2 , •• • En ese momento compararemos la
eficiencia de ambos programas.]

Nuestro siguiente ejemplo presenta una situación en que se determinan tres tipos de
complejidad. A estas medidas se les llama complejidad del mejor caso, complejidad del
peor caso y complejidad del caso promedio.

En este ejemplo, examinamos un proceso típico de búsqueda. Buscaremos, en un conjunto


den~l) enterosA[IJ,A[2],A[3], . . ,A[n], un entero llamado Key (Clave). Si se encuen-
tra el e ntero, se imprime su primera posición en el conjunto; si no se encuentra, sedara un
mensaje apropiado.
No podemos suponer que los elementos se encuentran e n un orden particular. (Si lo
estuvieran, el problema serfa más fácil y podría desarrollarse un algoritmo más eficiente.)
La entrada para este algoritmo es el conjunto de enteros (que lee el usuario o se proporci<r
na, tal vez, como un archivo desde una fuente externa), junto con el número n de elemen-
tos del conjunto, y el valor del entero Key.
El algorittno se implementa en el segmento de programa en Pascal de la figura 5.13. En
este caso, la variable entera i se utiliza como un contador, y la variable booleana Found se
usa para regisuar (con el valor verdadero) la presencia de Key en el conjunto.

Begin

i := l ; {Inicializa el contador}
Found : = false; {Cambia a verdadero si se encuenera Key)

While (i <= n ) and (not Found) do


If Key = A[i] then
Found : = true
Else i : = i + 1; {Se increment.a el contador }
{sólo si se encuentra Key )

Ir Found = true then


lriteln ( 'El valor •, Key, · se l ocaliza en
la posición ·, i,'. ')
Else
Writeln ( ' El valor •, Key , • no aparece en
la lista.' )

End ;

Figura 5.13
5.8 Análisis de al oritmos 301

Oetinin,mos la función de complejidad /(n) para ..te algorilmo como el nwnero de


elementos del conjunto que se examinan hasta encontrar Key o haber terminado de anali•
zar el conjunto (es decir, el número de veces que: se ejecuta c:I ciclo While).
¿Qué es Jo mejor que puede pasaren nuestra búsqueda de Key? Si Key = A[l], tenemos
que Key es el primer elemento del conjunto, y solamente tenemos que compararlo con un
elemento del conjunto. En este caso encontramos que/(n) = 1 y decimos que la compkji-
dad ckl nujor caso de nuestro algoritmo es 0(1) (es decir, es constante e independiente del
tamaño del conjunto). Por desgracia. no podemos esperar que tal situación ocurra con
frecuencia.
De la mejor situación pasamos a la peor. En este caso, tenemos que examinar todos los
n elementos si (1) la primera aparición de Key es A[n], o (2) si Key no se encuentra en el
conjunto. En cualquier caso tenemos que/(n) =- n, y la compltj idad del peor caso es O(n).
(La complejidad del peor caso será la que consideraremos generalme nte durante el texto.)
Finalmente, deseamos obtener una estimación del promedio de elementos examinados.
Supondremos que los n elementos son distintos, que todos tienen la misma posibilidad
(con probabilidadp) de contener el valor Key, y que la probabilidad de que Key no es~ en
el conjunto es igual a q. En consecuencia, tenemos que np + q = l y p = (1 - q')ln.
Para cualquier 1 :S. is n, si Key estáenA[il, entonces han sido eXaminadosi elementos
del conjunto. Si Kcy no está en el conjunto, entonces examinamos losn elementos de tste.
Por lo tanto, la complejidad dtl caso promedio se determina mediante el número medio de
elementos del conjunto e xaminado, que es

f(n)= (l·p +2·p +3· p + · · · +n·p) +n·q =p(1+2+3 + · · · +n)+nq


=pn(n + 1)/l+nq.

= = =
Si q O, entonces Key está en el conjunto,p 1/n y.f(n) (n + 1)12 E O(n). Paraq 1n, =
tenemos adn una buena posibilidad de que Key se encuentre en el conjunto y/(n) = (l/(2n))
[n(n + 1)12) + (n/2) = (n + 1)/4 + (n/2) E O(n), [En general, para cualquier Os q s 1,
tenemos.f(n) E O(n).]

Al principio del análisis de la sección anterior. mencionamos cómo podríamos compa-


rar dos algoritmos que resolvieran correctamente un tipo de problema dado. Podemos
hace r esa comparación utilizando las funciones de complejidad en tiempo para los
algoritmos. Demostraremos esto en los siguientes dos ejemplos.

En algunos lenguajes de programación,. como Pascal estándar y C. no existe una función


integrada para la exponenciación. El algoritmo implementado en el segmento del progra-
ma e n Pascal de la figura S.14 tiene como salida los valores de a'-, donde a es un número
real y n es un entero positivo. En este caso, los valores de a y n están dados antes de la
ejecución del segmento de programa.. La variable realx se inicializa como 1.0 y después se
utiliza para almacenar los valores de-a, a1, a,, ... , a" durante la ejecución del ciclo For. La
función de complejidad en tiempo/(.n) para el algoritmo se determina mediante el número
de multiplicaciones que hay en el cic1o For. Por lo tanto,.f(n) = n E O(n).
02 Capítulo 5 Relaciones y funciones

Begin

X:~ 1.0;

For 1 : = 1 to n do
x : = x•a:
'll'ri teln (·El valor de ·, a, ' elevado a la potencia
n, ' es ', x ,' . ')

End;

Figura 5.14

En la figura 5.15 tenemos un segundo segmento de programa en PascaJ para evualuar d'
para cualquier a E R, n E z•. En este caso usamos la operación (predefinida) divísi6n
e ntre un entero distinto de cero Div. El valor de (e Div d) es simplemente la parte entera
(cociente) de cid donde e, d E Z, y d,¡,Q_Por ejemplo, 7 Div 3 es 2 (= l713J = Í713l- 1),
y 3 Div 4 es O ( =l314J= f314l- 1).

Begin

X:= 1.0;
i : = n;

While 1 > O do
Begin
If i <> 2 • (1 Div 2) tben {i es impar}
x : ª x•a;
1 : = 1 Div 2 ;
If i > o then

End;

Wri teln ( •El valor de a, · ele vado a l a potencia


n , • es ', x, ' . ' )

End;

Figura 5.15

Para este segmento de programa la salida es a•; la variable real a y la variable entera
(positiva) n reciben sus va1ores a1 principio del programa . La variable real x se inicializa
como 1.0 y después se utiliza para almacenar las potencias apropiadas de a hasta obtener
el valor de a-. El resultado de la figura 5.16 muestra Jo que sucede con x (y a ) para los
casos donde n = 7 y 8. Los números 1, 2, 3 y 4 indican la primera. segunda. te rcera y cuana
veces que se ejecutan l as proposiciones en el ciclo While (en particular, la proposición
5.8 Análisis de al oritmos 303

i : = i Div 2). Si n= 1, entonces, como 21 < 7 <2'. tenemos 2 < log1 7 < 3. En este caso, el
c iclo While se ejecuta tres veces y

3 - [log, 7J + 1 < log, 7 + 1,


J
donde Liog,7 designa el máximo entero en log,7, que es 2. Asl mismo, cuando n = 8, el
mlmero de veces que se ejecuta el ciclo While es

4 e [log,SJ + 1 - log,8 + 1,
ya que log,8 = 3.

n z 7 n • 8
X : • l.O X : • l.O
1 :• 7 1 8

t. x• a
l 1 ; •= 3
{x • a}
1{! a•a: • 4
;a
a : • a• a
: • .x*a
-
{x • a3} 2{! : -• 2 ...
2G : • 3{! :: -• a•a
l
a•a

:• x• a
3{; : • o
{x - &7}
4¡;
x•a {x = •ª}
o
[x • a7 2
a. a • a4] [x = ( ( ( a / /)
2]
Figura S.16

Definiremos la función de complejidad en tiempo g(n) para Oa implementación de)


este aJgoricmo de exponenciación como el nt1mero de veces que se ejecuta el c iclo While.
Éste es tam.biln el número de veces que se ejecuta la proposición i : =i Div 2 . (En este caso
hemos supuesto que cada operación Div se realiza durante un interva1o de tiempo constan-
te. Es decir, el intcrValo de tiempo para cada llamada a Div es independiente de la magni-
tud de i.) Con base en las dos observaciones anteriores, deseamos establecer que para todo
n 2: 1, g(n} .:S log2 n + 1 E O(log1'1). Estableceremos es10 por inducción matemática ()a
forma alternativa del teorema 4.2) sobre el valor den.
cuando n: l . vemos en la figura 5.15 que i es ímpar, x toma el valor de a= a' , y a• se
determina sólo después de 1 = log,l + 1 ejecuciooes del cicloWhile. Así, g(l ) = 1 :s log,1 + l.
Supongamos ahora que para todo 1 :s n :S k, g(n) :s log,n + 1. Entonces paran= k + 1.
durante el primer paso por el ciclo While, el valor de ; cambia por lk; J1
Como 1

lk+IJ
:s -
2
- .:S k,por lahipótesis
. . de .rnducc1ón
. eJecutarcmosel
. c 1.clo Wh1le
. g (lk+lj)
-
2
- m ás

veces, dondeg (lk; J)1


,; Jog, lk; J
1
+ l.
)4 Capítulo 5 R~aciones y f unciones

Po.-lotantog(k+ 1) :s 1 +[log, lk;l j+1}• 1 [10g, (k;1)+1]:


+ 1 +CTog,(k+ 1)-
log, 2 + IJ: log,(k+ 1) + 1
Para la función de complejidad e n tiempo del ejemplo 5.72, encontramos que /(n) E
O(n). Aquí tenemos queg(n) E O0og,n). Puede vcrifican;e que g está dominada por/pero
/no está dominada porg. Por lo tanto, paran grand,, consicletamoo que cscc segundo algoritmo
es más cficíente que el primero (del ejemplo S.72). (Sin embargo. observe que el código
del segmento de programa de la figura 5.14 es má.s sencillo que el de la figura S. l'S.)

Para cerrar esta sección, resumiremos lo aprendido haciendo las siguientes observaciones.
1) Los resultados que establecimos en los ejemplos 5.69 al 5.73 son ~tiles cuando
trabajamos con valores den de moderados a grandes. Para los valores pequcftos de
n, la.1cs consideraciones acerca de las funciones de complejidad en tiempo tienen
poca utilidad.
2) Suponga que los algoritmos A, y A2 tienen funciones de complejidad en tiempof(11)
y g(n) respectivamente, donde /(n) E O(n) y g(n) E ()(•'~ Aquí debemos tener
cuidado. Podríamos esperar que un algoritmo con C!)mplcjidad lineal sea ••ta1 vei
más eficiente" que uno con complejidad cuadrática. pero realmente necesitamos
más información. Si/(n) ;;;;; IOCX)n y g(n) •n2, entonces el algoritmo A 2 es mejor siel
tamaño del problema n no es superior a 1000. Si el tamaño del problema nunca es
mayor que 1000. entonces el algoriuno A1 es la mejor opción. Sin embargo, como
mencionamos en la observación l. sin crece, el algoriuno de complejidad lineal se
conviene en la mejor alternativa.
3) En la figura 5.17 hemos hecho una gráfica scmilogarftmica para las funciones aso-
ciadas con algunos de los órdenes dados en la tabla S.11. [En este caso hemos reem-

f (n)
l (n) • 2"
128

64 •• -• f(n) • n'
,,,,, ..., , '
32 __ __ _ _ ___ l (n) • n log,n

16
, .,,.,' ---
1
.,,,',,,,,,."
/
,...::::,,,,.,,,,.
---- --
,,,,.,,,,.,- -- -- - -

- - - -
((n),. n

- f(n)• log2n
:'/" __..---····
,,/"1 ,..,.,,.,,,,.
:,'/·....·
....
10 12

Fig u ra 5.17
5.8 Análisis de algoritmos 305

plazado la variable entera (discreta) n por la variable rea1 n (continua).] Esto debe-
ría ayudamos a desarrollar la intuición para sus tasas de crecimiento relativas (es•
pecialmente para valores grandes den).

Los dalos de la tabla5.12 proporcionan estimaciones de los tiempos de ejecución de los


algoritmos para ciertos órdenes de complejidad. En este caso tenemos los tamaños de
problema n = 2, 16 y 64 y suponemos que el computador puede realizar una operación
cada lo-' segundos= 1 microsegundo (en promedio). Entonces las entradas de la tabla
estiman los tiempos de ejecución en microsegundos. Por ejemplo. cuando el tamaño del
problema es 16 y el orden de complejidad es n log;t11, entonces el tiempo de ejecución es
muy breve (16 log,16; 16 · 4; 64 microse¡undos); para el orden de complejidad 2•, el
tiempo de ejecución es 6.5 x 10' microsc:gundos = 0.065 segundos. Ya que ambos interva-
los de tiempo son muy cortos, es difícil para un humano observar gran difere ncia entre los
tiempos de ejecución. Los resultados parecen ser instantáneos en cada caso.

Tabla 5.12

Orden de complejidad
Tamailo del problema n log,n n n loit,n n' r n!
2 1 2 2 4 4 2
16 4 16 64 256 6.5 X 10' 2.1 X 10'3
64 6 64 384 4096 J.84 X 10" > 10"

Sin embargo, tales estimaciones pueden crecer rápidamente, Por ejemplo, suponga que
utilizamos un programa cuya entrada es un conjunto A den enteros diferentes. Los resul-
tados de este programa se generan e n dos panes:

1) Primero el programa implementa un algoritmo que delermina los subconjuntos de


A de tamaño 1. Existen n de estos subconjuntos.
2) Despu~s se implementa un segundo algoritmo para determinar todos los subconjuntos
de A. Existen 2" de estos subconjuntos.

Supongamos que tenemos un supercomputador que puede determinar cada subconjunto


IA
de A en un microsegundo. Para el caso en que 1= 64, la primera parte de la salida es
ejecutada casi de manera instantánea. en apro~madamente 64 microsegundos. Sin embar-
go, para la segunda parte, la tabla S.12 indica que la cantidad de tiempo necesaria para
determinar todos los subconjuntos de A será aproximadamente de 1.84 x 101' microsegun-
dos. No podemos estar muy contentos con este resultado ya que

1.84 x 10" microsegundos= 2 . 14 x 10' días = 5845 siglos.

Ejercicios 5.8 1. En cada uno de los segmentos de programa en Pascal, las variables enteru i, j, n y sum se
declaran al principio del programa. El valor den (un entero positivo) es proporcionado por el
usuario antes de la ejecución del segmento. E.n cada caso definimos la función de complejidad
en tiempo/(n) como el ndmerode veces que se ejecuta la instrucción sum: • sum-+- l. Deter-
mine la mejor forma ''O mayúscula" de f
306 Capítulo 5 Relaciones y funciones

.a) Begin
: - O;
For i : .... 1 to n do
Forj : • l tondo
SU'..,. SUII + 1

End:
b) Begin
su.a : - O;

Por i :• l to. ndo


For j : • l to n • n do
su■ : • su ■ +l

End;
e) Begin
su11:•0;
For 1 : • 1 to n do
For j : • 1 to n do
su.a :"" su.a + l
End;
d) Be¡_in
sua: • O;
1 : z n;
While i > O do
Begln
sua: • sua+l ;
1:..,1Div2
End
End;
~) Begin
su■ : •O ;

For 1 : • l to n do
Begin
j : - n;
While J > O do
Begln
sua : = sua + l;
J : = J Oiv 2
End
End

End:

2 . El siguiente segmento de programa en Pascal implementa un algoritmo para detcn:nmar a:


valor máximo en un conjuntoA( l], A(2J, A[3], . . . , A[n] de enteros. El conjunto y c1 valor de
n( ~ 2) se dan al principio del programa; las variables enteras i y Max tambit!n se declaran a
principio del programa. (Los elementos del conjunto no son necesariamente distinto$.)
5.8 Análisis de algoritmos 307

Begin

For i :~ 2 to a do
It A(i] > llax t hea
Mu : • Ali]

End;

a) Si la función de complejidad del peor C3SO/(n) para este segmento estf determinada por el
m1mero de veces que se ejecuta la comparación A{,1 > Mu, encuentre la fonna apropiada
-O mayúscula" d<f
b) ¿Qut podemos decir acerca de tu complejidades del mejor caso y el ca.so promedio para
esta implementación?.
3. a) Escriba un programa (o desarrolle un algoritmo) que localice las primeras apariciones del
valor máximo en un conjuntoA(l ], A[2],A(3],. . , A{n) de enteros. (En eMe caso, n E z·:
los elementos del conjunto no tienen que ser disaintos.)
b) Oetcnnine la función de complejidad dcJ peor caso paca la implementación desarrollada en
la parte (a).
4. a) Escriba un programa (O deS,31ToUe un algoritmo) que determine los valores mínimo y máxi•
mo en un conjuntoA{1],/\(2).Al3), .. . ,A,[n) de enteros. (En este caso. n E z·
con n ~ 2;
los elementos del conjunto no tienen que ser distintos.)
b) Determine la funcióo de complejidad del peor caso para la implementación desarrollada en
la pane (a).
5 . La siguiente función y el siguiente segmento de programa en Pascal se utilizan para evaluar el
polinomio

8- !Ox +7z'-2r' + 3%' + 12r'.


La función Pow~r se utiliza para determinar el valor de ,'- pan la variable real r y la variable
entera A:, donde l > O. F.o eJ segmento de. progn.ma, Sum es una variable real mientras quejes
una variable enten; el valor de la variable real x (previamente definida) se asigna al comienzo
de la ejecución.Adcmjs, la variable a (previamente definida) tiene las siguientes componen1es
enteras.

a(OJ=8, a(l]- - 10, •12]-7, a(JJ--2, •14]= 3, y o(S)=l2.

Funetioo Power (r: real : k: integer ): rul ;


Var
1 : ioteger:
pi-oduct: real ;
Becin
produet :- 1 :
rori:•ltokdo
product : • product • r ;
Po• er : ,. product
End ;

Sua : = o(O}:
,or J : ,., l to 5 do
Su.e: - SU..+a(J] • Power (x, J):
08 Capítulo 5 Relaciones y funciones

a) ¿Cuántas sumas se realizan para evaluar et polinomio dado'? ¿Cuántas multiplicaciones?


b) ¡,Cuántas sumas y cuántas multiplicaciones se realizan si ajustamos laenuada y el segmeo.
to de programa para trabajar con e l polinomio

donde co. c 1• c 2 , e,• ... , e,._., c. son enteros y n es un entero positivo?

6. Primero observemos que el polinomio del ejercicio anterior puede escribirse mediante el mito-
do de multiplicación anidada:

8 +z(-10+ x(7 +x(-2 +x(3 + 12.r))))

Usando esta representación, el siguiente segmento de programa en Pascal {que implementad


mirodo de Homu) puede utilizarse para evaluar el polinomio. Aquí no necesitamos la función
en Puca1 Power. Sin embargo. como en el ejercicio 5, Sum es una variable rea] y j es una
variable entera, y las mísmas c.ondiciones se dan para la variable real x y la variable a.

Sw, : - a(5];
F'or J:- 4 downto o do
su :• a{J]+x • sua;

Responda las preguntas de las partes (a) y (b) del ejercicio S para este nuevo segmento de
programa.
7. Sea aa, a 2, a 1 , ••• una sucesión de enteros definida recursivamente como
1) a 1 =0;y.
2) Paran> l. a.= 1 + at,,,21•
Demuestre que a. = Llo~Jpara todo n E z•.
8. Sea a1, a1, a,• . . . una sucesión de enteros definida recursivamente por
l )a1=0;y,
2) Paran> 1, a . = 1 + ar....,1.
Encuentre una fórmula explícita para a. y demuestre que la fórmula es correcta.

5.9
Resumen y repaso histórico

En este capítulo desarrollamos el concepto de función, que es de gran imponancia en


todas las Meas de las matemáticas. Aunque nos interesamos principalmente por las funci~
nes finitas, la definición tambifo se aplica a los conjuntos infinitos e incluye las funciones
de la trigonometría y el cálculo. Sin embargo, pusimos énfasis en el papel de una función
finita cuando transformamos un conj unto finito en otro conjunto finito. En este marco, la
salida del computador puede pensarse como una función de la e ntrada, y un compilador
puede considerarse como una función que transforma un programa (fuente) en un conjun-
to de instrucciones del lenguaje de máquina (programa objeto).
La palabrafunci6n, e n su forma latina, fue introducida en 1694 por Gonfried Wilhelm
Leibniz (1646---1716) para designar una cantidad asociada con una curva (como·ta pe&
djente de la curva o las coordenadas de un punto sobre la misma). En el año 1718. bajo ta
dirección de Johann Bernoulli (1667-1 748). una funcíón era considerada como ~na ex-
5.9 Resumt:n y repaso histórico 309

Gottfriod Wilhelm Leibniz (1646-1716)

presión algebraica formada por constantes y una variable. Las ecuaciones o fórmuJas con
constantes y variables surgieron posteriormente con Leonhard Euler (1707-t 783). Su de--
finición de "función" es la que generalmente se encuentra en los libros de matemáticas
de nivel bachillerato. Además. hacia 1734, encontramos en el trabajo de Eulcr y Alexis
Clairaut ( 1713-1765) la notación f(z), que sigue en uso actualmente.
La idea de Euler permaneció intacta hasta la tpoca de Jean Baptistc Joseph Fourier
(1768-1830), quien encontró la necesidad de un tipo más general de función en su estudio
de las series trigonomttricas. En 1837, Peter Oustav Lejeune Dirichlet (1805-1859) esta-
bleció una formulación más rigurosa de los conceptos de variable. función y la corrcspon•
dencia entre la vañable independiente x y la variable dependiente y, cuando y= f(x). El
trabajo de Dirichlet enfatiuba la relación entre dos conjuntos de n11mcros y no pedía la
existencia de una fórmula o expresión que relacionara los dos conjuntos. Con los desarro-
llos de la teoría de conjuntos ocurridos durante los siglos XIX y XX se llegó a una genera.Ji•
ución de la función corno un tipo particular de relación.
Además de su trabajo fundamental respecto a la definición de una función. Dirichlet
tambi~n hizo mucho en las matemáticas aplicadas y la teoría de números. donde le pareció
necesario, y formuló por primera vez de manera formal el principio del palomar. En
consecuencia, este principio se conoce a veces como el principio de distribución de Dirichlct
o el principio de la caja de Dirichlet
Los siglos XIX y XX vieron el u.so de la función especial. la correspondencia uno a uno,
en el estudio del infinito. Cen:a de 1888, Richard Dedekind (1831-1916) definió un con-
junto infinito como aquel que podía ponerse en correspondencia uno a uno con un
subconjunto propio de sí mismo. [Galileo (1564--1642) observó esto para el conjunto z·.J
Si dos conjuntos infinitos pueden ponerse en correspondencia uno a uno entre sí, se dice
que tienen el mismo númuo cardi.nal lraJUfiniu,. En una serie de artículos, Georg Cantor
( 1845-1918) desarrolló la idea de los niveles de infinito y mostró que I Z 1= 1Q I pero
¡.
1Z 1 < 1R unconjuntoAcon I A 1= 1 Z I esconzableonunurob/eyescribimos I Z 1=
N0 como lo hizo Cantor, usando la letra hebrea áleph, con el subíndice O, para designar el
10 Capítulo S Relacione-; y funciones

Peter Gustav Lejeune Dirichlet (1085-1859)

primer nivel de infinito. A fin de mostrar que I Z 1< 1R l. o que los números reales eran
no numerables, Cantor disefió una técnica que ahora se conoce como el método diagonal
de Cantor. (En el apéndice 3 se puede encontrar más información acerca de la teoóa de los
conjuntos numerables y no numerables.)
Los números de Stirling de la sección 5.3 llevan el nombre de James Stirling (1692-
1770), pionero en el desarrollo de las funciones generatrices, tema que analizaremos pos-
teriormente. Estos números aparecen en su obra Me.thodus Differentialis, publicada en
Londres en 1730. Stirling era ayudante de Sir Issac Newton (1642- 1727) y utilizaba la
serie de Maclaurin en su trabajo 25 años antes de Col.in Maclaurin ( 1698- 1746). Sin em-
bargo, aunque su nombre no aparece ligado a esta serie, sí aparece en la aproximación
conocida como la fórmula de Stirling: n! ,; (2ttn) 112 e-41 n", la cual, es justo decirlo, fue
desarrollada realmente por Abraham Oe Moivre ( 1667-1754).
Con los principios de conteo desarrollados en la sección 5.3, los resultados de la tabla
5.13 extiende n las ideas resumidas en la tabla 1.8. En este caso contamos el nwnero de
fonnas en que podemos distribuir m objetos en n recipientes, en las condiciones prescritas
en las tres primeras columnas de la tabla. (Los casos en que ni los objetos ni los recipientes
son distintos serán analizados en el capítulo 9.)
Por último, la notación ·•o mayúscula" de la sección 5.7 fue introducida por Paul Gustav
Heinrich Bachmann ( 1837-1920) en su libro Analyri5che Zahlenrheorie, una obra impor·
tante en la teoría de números, publicada en 1892. Esta notación se ha vuelto muy impor·
tante en _la teoría de aproximación, en áreas como e l análisis numérico y el análisis de
algoritmos. En general, la notación f E O(g) indica que no conocemos la función/ de
manera explícita pero sí una cota sobre su orden de magnitud. El símbolo "O mayúscula"
se conoce a veces como el símbolo de Landau, en honor de Edmund Landau (1877-1938),
quien usq esta notación a lo largo de su obra.
En el capítulo 4 de D. l. A. Cohen [3] y en el capítulo 6 del texto de R. L. Graham, D. E.
Knuth y O. Patashnik [71 aparecen más propiedades de los números de Stirling de l seguo•
5.9 Análisis de algoritmos 311

Los objttM
.....
distintos

sr
Los rtelpie.ntts
"""
distintos
sr
sr
,.,-
AJcuDOSttd·
..
,....... podña,,

sr
No
Númtro
d•
clbtribodona
n~
n! S(m,n)
Sí No Sí S(m, 1) +S(m,2)+ · .. +S(m,n)
Sí No No S(m,n)

No Sí sr {n + ;:-1)
No sr No (n + (m
(m - n)-1} • (m - 1}
- n) m-n

= (m - 1}
n- 1

do tipo. Para más infonnación acerca de los conjuntos infinitos y el trabajo de Geor¡
Cantor, consulte el capítulo 8 de H. Eves y C. V. Newsom (6), o el capítulo IV de R. L.
Wilder (10). La obra de J. W. Dauben [SJ analiza la controversia de principios de siglo en
torno a la teoría de conjuntos y muestra cómo aJgunos aspectos de la vida penonaJ de
Cantor cumplieron un papel esencial en su comprensión y defensa de la teoría de con-
juntos.
En el artículo deA. Soifer y E. Lozansky (9) se incluyen más ejemplos que demuestran
la forma de aplicar el principio del palomar. En el artículo de D. S. Clark. y J. T. Lewis [2)
se analizan otros resultados y extensiones de problemas que surgen de este principio. Du·
rante el siglo xx se han realizado muchas investigaciones declicadas a las generalizaciones
del principio del palomar, que culminan en el tema de la teoría de Ramsey, la cual recibe su
nombK de Frank PlumptOD Ramsey (1903-1930). En el capítulo S de D. I.A. Coben [3)
aparece una interesante introducción a la teoría de Ramsey. El texto de R. L. Graham, B. L.
Rothscbild y J. H. Spencer [8] proporciona más información importante.
En el libro de C. J. Date [4) se estudia en forma amplia el tema de las bases de datos
relacionaJes. Por último, el texto de S. Baasc (1) es una excelente obra para continuar el
estudio del análisis de algorilmos.

BIBLIOGRAFÍA

l. Baase, Sara, Compw,r Algorúhms: /nmxi,,cÚQn to D,sign and ,w,lysi,. 2'. ed., Readin¡,
Mas.s.• Addiwn-Weslcy, 1988.
2. Clark. Dean s.• y James T. Lewis, "Herbert and the Hungarian Matbematician: Avoidin¡
Certain Subsequencc Sums.., 7M Colkge M01hanatics JoJUn4l, 21, no. 1, Mano 1990,
págs. 100-104.
12 Capítulo 5 Relaciones y funciones

3. Cohcn, Daniel l.A., Basic Techniques ofCombinatorial TMory, Nueva York: Wiley, 1978.
4. Date, CJ., An lntroducrion to Databa.se Systems, 3a. ed, Rcading, Mass .• Addison-Wcslcy,
1982.
.5. Dauben, Joseph Warrcn, Gebrge Carnor: His MatMnt(ltics and Phik,sophy of the /nfinitt,
LawrenceviUc, N.J., Princcton University Press, 1990.
6. Eves, Howard, y Carroll V. Ncwson, An lnJroduclion to tM Foundations and F~mol
Concepts of Mathe.matics, edición revisada, Nueva York, Holt, 1965.
7. Graham. Ronald L .• Dona1d E., Knuth, y Oren Patashnik. Concrete Mathmuuics. Readiug.
Mass.. Addison-Weslcy, 1989.
8. Graham. Ronald L., Bruce L. R01schild, y Jocl H. Spenccr, Ramsey Theory. 2a. ed.• Nueva
York. WUey, 1980.
9 . Soifer, Alcxander y EdwardLozansky,"Pigeons in Every Pigeonhole'',Quantum (Enero 1990).
págs. 25--26. 32.
10. Wíldcr, Raymond L., /ntr<>duction to tite Foundr.m·ons of Mathematics, 2a. cd., Nueva Yod.
Wiley, 1965.

EJERCICIOS
4. Sea'll =N yseanA. B ~'U con 1 < IA 1 < IBl.s;
e xisten 262.144 relaciones de A a B. determine todas las
COMPLEMENTARIOS posibilidades para I A I y [ B I-
S . Si UUI' <i\1.2 son conjuntos universales con A, B ~ '\1.1, y
1. Sean A, B conjuntos cualesquiera. c. D !;: G\.1.
2
, demuestre que •
a) Demuestre que a) {A n B) x(C n D)=(A XC) n (BxD).
;¡ (A X B) n (B xA) ={A n B )x (A nB); y b) {A UB)x(CUD)=(AxC) U(B x D)U{AxD)U
ü) {A xB) U (B xA) C (A U B )x(A U B). (B XC) (podo que, en general, (A U 8) x (CU D);)
b) Dé un ejemplo que muestre que (A U B) x (A U 8) {Ax C) U (Bx D)].
n.o necesariamente es unsubconjuntode(A xB) U 6. Sea A= {I, 2, 3. 4. 5) y B = {I, 2, 3, 4, 5, 6 ). ¿CuAntas
(BxA). funciones inyectivas f: A ➔ B satisfacen (a) /( 1) = 3? (b)
Determine si cada una de las siguientes proposiciones f{I) =
3./(2) 6? =
verdadera o falsa. Para cada proposición falsa d~ un 7 . Sea 9't ~ z· x Zp la relación definida recursivameme
mU"aejemplo.
a) Si/: A ➔ By (a, b), (a, e) Ef. entoncesb = c. 1) (l, I) E9l; y,
b) Si/: A ➔ 8 es una correspondencia biyectiva y A, 2) Paracualquier{a. b) E9! tenemosque (a + 1.b)E
B son finitos. entonces A = 8. 91 y (a, b + 1) E9l.
e) Si/: A ➔ 8 y es inyectiva, entonces/es invertible. Demuestre que R = z· X z·.
d ) Si/: A ➔ Bes invenible, entonces/es inyectiva.
e) Si/:A ➔ Bes inyectiva y g, h: 8 ➔ C congºf=
8. Sea9i C z•xz• la relación dada por lasiguientedcfi..
hº/. entonces g = h. nición recursiva.
1) (1, 1) E9l; y,
f) Si/: A ➔ By A 1 , A 1 ~ A. entonces /(A 1 n A1) =
2) Para cualquier (a. b) eg¡, los tres pares ordenados
/{A,) n /(A,).
e) Sif: A ➔ By B 1, B1 ~ B, entonces/~1 (81 n B:) = (a + I, b), (a+ l, b + 1). y (a+ 1, b + 2) tambiá:I
están en 91..
J-'(B,) nf'(B,).
Demuestre que 2a ~ b para todo {o. b) E 9t
:l. Con~= Z. sean A. B ~qi donde A= {2, 3, 5} y B=
9. Detennine todos los números reales .:e para los cuales
1, 2, 5, 6}.
a) Haga un diagrama de A x 8 como un subconjunto x' - [xJ = 1/2.
del plano euclídeo. 10. SeanA,,A y B conjuntos con { !, 2, 3, 4, 5 } =A1 CA. B
b) Si 91: es la relación de A a B definida como {(a. = {s, t, u, v. w, .x}, yf:A 1 ➔ B. Si/puede ser extendida a A
b) /a + bes impar}, haga un diagrama de2:ii como en 216 fonnas. ¿cuánto vale IA
1?
un subconjunto del plano euclídeo.
e) ¿Cuántas relaciones de A a B no son funciones de 1 1. Sean A= {l. 2, 3,4,5} y B= (,, u, v. w,,. y.,}. (a)Si
AaB? generamos una runción/: A ➔ B en fonna aleatoria, ¿cuál
EJERCICIOS COMPLEMENTARIOS 313

s la probabilidad d e que sea uno a uno? (b) Escriba un 20. Si n E z· con n 2: 4, verifique que S(n. n - 2) =
~ (o desarrolle un algoritmo) para generar funcio- (;)+3(;).
a aleatorias/: A ➔ By haga que el programa imprima el
21. Si f: A ➔ A es cualquier función. demuestre que para
limero de funciones generadas hasta obtener una función
II03Wl0.
todos nr. n E Z♦.J- o r=
ro J-. (Primero, tome m = l y
realice la inducción sobre n. Después realice la induc.ción
D.. Sea S un conjun10 de siete enteros positivos, donde el sobre m. Esta t~nica se conoce como inducci6n dobk .)
lhimo es 24. Demuestre que las sumas de los elementos
22. Sean/: X➔ Y. ypancadai E /. seaA, !;; X. Demuestre que
:1todos los subconjuntos no vacíos de S no pueden ser dis-
iltas. a) /(U,.,A,) - U..,f(A,).
13. En un periodo de diez d.fas, una secretaria escribe a
b) t(n.., A,) ~ n..,/(A,).
e) /(n..,A,) = n.,,/(A,), para/ inyectiva
li:p.rina 84 cartas a diferentes dientes. Flla mecanografía
12 de estas cartas el primer día, siete el segundo, tres el 23. Si/: R ➔ R con/(x) =X",paran E z·. ¿es/invertible?
iovcoo, y termina las \Jltimas ocho el décimo día. Muestre 24. Sea A un conjunto con IA 1= n
IIC> para un periodo de tres días consecutivos, ella escribe
1máquina a1 menos 25 cartas. a) ¿CUántas operaciones binarias cerradas existen so-
bre A?
1'- Si {x1, x 1 , ••• , .x,) C Z · , muestre que para alglln i 'F j .
b ) Una operación ternaria cerrada (3-aria) sobre A es
¡+.x ox,- x, esdivisiblecntrc 10.
1 una función/: A x A x A ➔ A. ¿Cuántas opera•
15. Seann E Z♦,n impar. Sii1, i1, ... ,i,.esunapcnnutación ciones ternarias existen sobre A 7
lelos enteros l, 2•... , n, demuestrequc(l -i1)(2 - ii) · ·
e) Una operación k-:aria cerrada en A es una función
1 - i.) es un encero par. (¿Qué principio de conteo se usa
/:A 1 xA 2 x · ·· xA.➔A,dondeA1=A,paratodo
qui?)
1 ~ i :s k. ¿Cuántas operaciones k-arias cenadas
·'- Como sus dos padres trabajan, Tomás, Eduardo y Car- existen en A?
3S deben realizar diez quehaceres semanales entre ellos. (a)
d) Una operación k-aria cerrada paraA es conmutativa si
Decwintas formas pueden Wvidirse las tareas de tal mane-
aque cada uno sea responsable de al menos una de ellas?
f(a., a,, . .. ,a,) = /(,<(a,), "(a,), ... , ,<(a,)),
b} ¿De cuántas formas pueden asignarse las tareas si To-
lis.. por ser e l mayor, debe cortar el ctsped (una de las diez ª•
donde a1, ª:• . ... E A (repeticiones pennitidas),
l'C3S semanales) y a ninguno se le permite estar ocioso?
y 1t(a), 1t(a1), .. . • Jt(a_J es cualquier redisposición
7. Sean EN. n ~ 2. Muestre que S(n, 2) = 2""1 -J. de a 1, a,, .. . .ª•· ¿Cuántas operaciones k-arias
8. La señora Báez tiene cinco hijos (Migud, Ricardo, cerradas en A son conmutativas?
lavid. Enrique y Donaldo} a quienes les gusta leer libros de 25. Una función/: R ➔ R es creciente si para n\1.mcros
cpor1C$. Como se acerca la Navidad, ella visita una librería reaJesx, y, tcnemosx < y =) j(x) </(y). Demuestre que sit
onde encuentra 12 libros diferente$ de deportes. g: R ➔ R son funciones crecientes, entonces g o/: R ➔ R
a) ¿De cuántas formas puede seleccionar nueve de es creciente.
estos libros? 26. Si <i\.l. es un universo y A~ G\I., definimos lafa.nción
b) Dcspu6i de hacer su compra, ¿de cuántas formas C(Uacteristica de A comox..,: G\1.-➔ {O, 1}, donde
puede distribuir los libros entre sus hijos de modo
que cada uno de ellos reciba al menos un libro?
e) Dos de los nueve libros que la señora Báez eligió x,x()={!,O, xEA
x<tA
son de baloncesto, el deporte favorito de Donaldo.
¿De cuántas formas puede ella distribuir los libros Para los conjuntos A, B ~ UU., demuestre cada uno de los
entre sus hijos para que Donaldo obtenga a1 menos siguientes casos:
ambos libros de baloncesto? a) XA= - x, · X• donde (x, · x,)(x) = x,(x) · x.(x)
9. Sean m, n e z·
con ni?:m. (a) ¿De cuántas formas po- b ) X,,ivs=XA + X• - )(,m.s
emas distribuir n objetos distintos en m diferentes rcci- e) x;.- 1 - x,, donde (1 - x,)(x) = l(x) - x,(x) =·
ientcs sin dejar ninguno vacío? (b) En el desarrollo de (x1+ 1-x,(x)
1
+ • • • + x)". ¿cuál es la suma de todos los coeficientes (Para <i\.l. finito, la colocación de 1os elementos deGU. en
un orden fijo produce una correspondencia biyectiva entre
mllinomiales ( 11\,11::t, ~- .. " .. ) . donde n, + n! + · · · + n. = n y
los subconjuntos A deG\l y las disposiciones de ceros y unos
,>O para Is is m? obtenidas como imagen de (\1, mediante XA· Estas disposi•
14 Capítulo 5 Relaciones y funciones

:s pueden usarse después para el almacenamiento y con- a) Sean= P~P?PJ~ · ·· p;1 , donde P1, P 2, P 3, •. ·, Pt
)) de cienos subconjuntos de GU.} son primos distintos y e 1 cs un entero positivo para
todo I S i s k. ¿Cuánto vale t (n)?
'· Si A = {.r, y, t}, sean.f. g: A ➔ A dadas po,f= {(x, y),
b) Determine los tres valores más pcquefios den E
, ,), (,. x)}, g = {(x, y). (y, x), (,.,)).Determine lo sí-
ü.ente:/o g, g oJ,J-•, g-1, (g OJrl,J-1 0 g-1, y g-1 0 ¡-1.
z• para los cuales t(n) = k, donde k = 2. 3, 4, S, 6.
e) Para cuaJquierk E Z•, k> l, dcmuC$trcquet-1(l)
l. a) Si/: R ➔ R sedefinccomo/(x)= 5.t+ 3,encuen- es infinito.
:¡-'(8). d) Si a, b E z·
con mcd(a. b) = 1, demuestre que
b) Sig: R ➔ R, dondeg(.x) = ¡.r+ 3.x+ 1\, encuentre t(ab) = t(a)t(b).
,r'(l).
<) Para h: R ➔ R. dada po, 34. Sean a. b, e E R con a2 +be * O. Si /(x);: (ax+ b'JI
(c.x- a), verifique que (f•f)(x) =x para todox E R,xi:-alc.
35. a) ¿Cuántos subconjuntos A = {a, b, e, d} ~ z· . don-
de a, b, e, d> l, satisfacen la propiodada • b- e -d= 2- 3-
h(x)=1 ~.
~ I 5 · 7 · 11 · 13 · 17 · 19?
b) ¿Cuántos subconjuntos A= {a1, a7 . •• , a.} C z•.
1
encuentre h" (4). donde o; > 1, 1 s i s m, satisfacen la propiedad
J. Si A= {l. 2, 3, .. . . 10}. ¿cuántas funciones/: A ➔ A n :1a; = íl j..1Pp donde los pl.. l :S j S n, S0D
primos distintos y n ~ m?
tisfaccn (simultáneamente)/·'({ l, 2, 3)}= 0, /·'({4, S)l =
, 3, 7} y J-'({8, IO}) = {8. 10}? 36. ™un ejemplo de una función/: z• ➔ R tal que/E
). Sea b 1 • b 1, b, • .. . la sucesión entera definida 0(1 ) y quefsea uno auno: (Por lo tanto,/no es constante.)
cursivamente por 37. Sean/. g: Z· ➔ R, donde
1) b,= 1; y
2) Paran> 1, b.=b¡_.,,, + b¡_,, f(n) ={~. paran P"'
paran impar
g(n) ={3,
4,
paran par
paran impar
1CUentre una fórmula explícita para b. y demuestre que su
rmula es correcta. Verifique si se cumple Jo siguiente: (a) /E O(g), (b) ge
1. En el lenguaje de programación Pascal, las funciones 0(/).
cd y su.ce ( predecesor y suc:esor, respectivamente) son fun- 38. Para/, g: Z·--+R,definimos/+'g: Z·➔ R como(f+g)
ones de Za Z tales que pred(x) = 1t(;r) =.x - 1 y succ(.x) = (n) =/(n) + g(n), paran E z·.
[Nota: El signo+cn/+ges la
·x)=x + l. suma de las funciones/y g. mientras que el signo+ en/(n) ♦
a) Determine (1t o O')(.x),(O' o 1tX.x). g(n) se usa para la suma de los números reales/(n) y g(n).J
a) S=t fr g,: Z' ➔ R, oon f E O<f,).g E O(g,). Sí
b) Determine 1t2, 1t3. 1t"(n 2 2), <r. &. 0"{11 ~ 2).
f,(n) ~ O. g ,(n) 2 O para todo n E z•,
demuestre
e) Determinex · 2, n·1 , n....,(n 2: 2),0'-2, o · 3,o""(n ;?2). que/+ g E ()(JI+ t 1>•
donde, por ejemplo. c,~ 2 = c,- 1 oa- 1 • (ao ar 1 = b) Si las condiciones de la pan.e (a),f.,(n) ~ O, g 1(n) ~
(0 --2)-1• (Véase el ejercicio complementario 32.) O para todo n E z·.
no se saúsfacen, proporcione
l. Sea/: A ➔ A una función invenible. Paran E z•, de- un contraejemplo para mostrar que/ E 0(/1), g E
.ucstre que (J-T1 = (f-1) • . (Este resultado puede usarse para O(g,) ~ í/ + g) E O<f, + g,) .
:finir1-- así como (J-)-1 o (f- 1r .) 39. Seana, b E ~ .con a, b> l. Sean/. g: Z•--+ Rdadas
3. Paran E Z•, defina t: Z• --+ z• como t(n) = el número por /(n) = log. n, g(n) = log.n. Demuestre que/ e O(g) y
: divisores enteros positivos den. g E 0(/). [Por lo tanto. O(log.n) = OQog,n).}
6
Lenguajes:
Máquinas
de estados finitos

E n esta era de computadores y telecomunicaciones, nos enfrentamos día con día a situa-
ciones de entrada y salida. Por ejemplo, al comprar un refresco en una máquina expen-
dedora. damos como entrtUUl ciertas monedas y después oprimimos un botón para obtener
la salida esperada. es decir, el r efresco. La primera moneda que damos como entrada pone
la máquina en movimiento. Aunque generalmente no nos preocupamos acerca de lo que
ocun e dentro de la máquina (a menos que se descomponga y tengamos una pérdida).
conve ndría notar que, de alguna forma, la máquina neva un re gistro de las monedas depo-
sitadas, hasta que se introduce el impone correcto. Sólo entonces, y no antes, la máquina
deja salir el refresco esperado. En consecuencia, para que el vendedor tenga la ganancia
esperada por cada refresco, la máquina debe recordar inu mamenu, conforme se va inser-
tando cada moneda, la suma de dinero depositado.
Un computador es otro ejemplo de un dispositivo de entrada y salida. En este caso, la
entrada es por lo general algún tipo de información y la salida e s el resultado obtenido
después de procesar esta información. La forma en que se procesa la entrada depende de la
labor interna de l computador. el cual debe tener la capacidad de recordar la información
anterior cuando trabaja con nueva información.
Con los conceptos desarrollados antes acerca de los conjuntos y las funciones, en este
capítulo estudiaremos un modelo abstracto llamado máquinc, de estados finitos, o circuito
secuencial. Estos circuitos son uno de los dos tipos básicos de circuitos de control que se
encuentran en los computadores digitales. ( El otro tipo es el circuiro combinaiorio o red
de puertas, que veremos en el capítulo 15.) También se e ncuentran en otros sistemas,
como la máquina expendedora, o en los controles de los ascensores o los sistemas de
semáforos.
Como su nombre lo indica, una máquina de estados finitos tiene un m:irnero finito de
estados internos, en que la máquina recuerda cierta información cuando está en un estado
particular. Sin embargo, antes de pasar a este concepto necesitamos materia] de la teoria de
conjuntos para hablar de la entrada válida para dicha máquina.

315
6 Capftulo 6 Lenguajes: Máquinas de ~dos finitos

6.1
.enguaje: La teoría de conjuntos
de las cadenas

Las sucesiones de símbolos, o caracteres, tienen un papel clave en el procesamiento de


información por parte de un computador. Como los programas de computación pueden
representarse en tirminos de sucesiones finitas de caracteres, se necesita a1guna forma
algebraica para manejar tales sucesiones finitas, o cadena.t.
En esta sección utilizaremos I para designar un conjunto de símbolos finito distinto del
vacfo. llamado cole<:tivamente alfab,w. Por ejemplo, l: = (O, t) o l: = (a, b, e, d, ,¡.
En cualquier alfabeto l:, no enumeramos los elementos que puedan formarse mediante
otros elementos de l: por yuxtaposición (es decir, si a. b E I , entonces la cadena abes la
yu.xtaposición de los símbolos a y b). Como resultado de esta convención. haremos a un
lado los alfabetos como l:= (O, 1, 2, 11, 12} y l: = {a, b, e, ba, aa). (AdemJs, este conve-
nio nos ayudará en la definición 6.5, cuando hablemos de la longitud de una cadena.)
Usando un alfabeto I como punto de partida podemos construir cadenas con base en
los símbolos de ~ mediante la siguiente idea.

~nición 6.1 Si :t es un alfabeto y n E z •, definimos laspottnciastk :t recursivamente de la siguiente


manera:
1) l:' =l:,y
2) Z:-' = (-')' lx El:, y E .E"}, donde -'Y denota la yuxtaposición dex y y.

Sea I cualquier alfabeto.


Sin= 2, entonces l:' = (-'YI x. y E l:}. Por ejemplo, si l: = {O, 1), encontraremos que
l:' = {00, 01, 10, ti }.
Cuandon = 3, loselementos deV tienen la fonnall\l, donde u E :t e u E 1:2. Pero como
conocemos la forma de los elementos de lr. tambitn podemos considerar que las cadenas
en I:' son sucesiones de la forma &U)', donde u, .x, y E :t. Como un ejemplo para este caso.,
supongamos que l: = {a. b, c. d, e}. Entonces l:, contiene 5, ~ 125 cadenas de tres símbo-
los, entre ellos aaa. acb, ace, cdd y eda.
En general, para todon E z • encontramos que ll:•I = ll:~. ya que estamos trabajando
con disposiciones (de tamaño n) en las que está permitido repetir cualquiera de los Jl:I
objetos.

Ahora que hemos examinado l:• para n E z·, veR:mos una potencia más de I .

:>efinición 6. 2 Para cualquier alfabeto l: definimos ri' = (l.}, donde A. denota la cadena vacía, es dec:ir, la
cadena que no consta de ningún símbolo tomado de :t.

El símbolo A nunca es un elemento de nuestro alfabeto I , y no debemos confundirlo


con el espacio en blanco que se encuentra en muchos alfabetos.
6. 1 Lengua·es: La teoría de conjuntos de las cadenas 317

Sin embargo, aunque :le :t, ocurre que fJ S: l:, por lo que e n este c:aso es necesario ser
cuidadosos. Observarnos que
1) (A.)!Z l: yaqueA.E:t,y
2¡ !"-l" fyaque IP-ll = 1.. 0= 181.
Para hablar colectivamente de los conjuntos IJ, l:1, l:2, ... , presentaremos la siguiente
notación de las uniones de tales conjuntos.

Dtfinidón 6.3 Si :tes cualquier alfabeto, entonces


a) l:' - u :.,l:ª= u ...- 1:•, y
b) l:' • U:-ol:ª.

Vemos que la única diferencia entre los conjuntos :t+ y :t• es la presencia del elemento
)., ya que"- E I:' sólo cuando n = O. AdemAs, l:• =1:• U 11'.
Además de usar el término cadena, tam.bi~n nos referiremos a los elementos de l:"' o :t•
como palabras y a veces como enunciados. Para I = {O, l. 2}, e ncontramos palabras
como0,01, 102y 1112tantoenl:"'comoen V .
Por último, observemos que aunque los conjuntos r yr son infinitos, los e lementos
de estos conjuntos son cadenas finitas de símbolos.

Para:t= {O, 1}, el conjuntor consta de todas las cadenas finitas de e.eros y unos junto con
la cadena vacía. Para un n razonablemente pequeño, sf podrfamos enumerar todas las ca-
denas de :t•.
Si l: = ( p, O, 1, 2, ... , 9, +, - , x, /, (, )) , donde P denota el espacio en blanco, es más
difícil describir :t•; paran > 2. :t- tiene demasiadas cadenas para enumerarlas. En :t• en-
contramos expresiones aritmiticas que nos son familiares, como (7 + 5)1(2 x (3 - 10)), así
como disparates de tipo +X(llx + 3/(.

Ahora nos enfrentamos a una situación conocida. Como en el caso de las proposiciones
(Cap. 2), los conjuntos (Cap. 3) y las funciones (Cap. S), una vez mAs necesitamos poder
decidir cuándo debe considerarse que dos objetos bajo estudio. en este caso cadenas. son
iguales. Analiu.remos este asunto a continuación.

Definición 6.4 Si wl' wl E :t•, entonces podemos escribirw1 sx1,;t1 • • • x. y w1 == y 1,y1 •• ·Y., param, n E
z·, y x1, x1 • • • x., yl' y1 ••• Y. E l:. Decimos que las cadenas w1 y w1 son igual.es, y
escribimos w1 = w 2, si m ~ n, y x, = y1 para todo 1 ~ i ~ m.

Se sigue de esta definición que dos cadenas de.E+ son iguales sólo cuando cada una está
formada por el mismo nllrnero de símbolos de l: y los símbolos correspondientes en las
dos cadenas coinciden i<Mnticamente.
Tambii!:n necesitamos e1 mlmero de símbolos de una cadena para definir otra de sus
propiedades.
18 Capítulo 6 Lenguajes: Máquinas de estados finitos

iefinición 6.5 Si w E z..,


entonces w =x1 x 2 ••• x•• dondex; E I para todo 1 $ i Sn. Definimos la longitud
de w, que se deno,a con II wll, como el valor n, Para el caso de),,, 1enemos que 11 ),,11 = O,

Como resultado de la definición 6.5, encontramos que paracua1quier alfabeto 1:. si w E


I." y llwll .e 1, entonces w E I:, y viceversa, Además, para cualquier y El:*, IIYII = 1 si
y sólo si y E I . Si :tcontiene el símbolo p (para el espacio en blanco). tambi~n ocurre que
11~11 = 1.
Si utiliz.amos un alfabeto particular, como :t = {O, 1, 2}, y examinamos los e lementos
x = 01, y = 212 y z = 01212 (de l:*), encontramos que

Hzl = 10121211 = s = 2 + 3 = 1m n + 1121211= lxl + HYII-


Para continuar nuestro estudio de las propiedades de las cadenas y los alfabetos, nece-
sitarnos ampliar la idea de yux.taposición.

>efinición 6.6 Seanx. y El:°" conx =x 1 x 2 ••• x. yy =y1 y1 •.• y_., tales que cadaxi' para l ~ i S m, y cada
Yp para 1 Sj S n, está en I . La concatenación dex y y, que ~ibimos como.xy, es la cadena
x 1 x1 • •• x.Y 1Y2 • • •Y.·
La concatenaci6n de x y A es

y la concatenaci6n de A- y x es

Finalmente, la concatenación de A y A es U= A.

Arriba hemos definido una operación binaria cerrada en I.* (y ~). Esta operación es
asociativa pero no conmutativa (a menos que ll: I = l), y comoxA= AX = x para cualquier
x E I.*, el elemento A El:* es el neutro para la operación de concatenación. Las ideas de
las últimas dos definiciones (la longitud de una cadena y la operación de concatenación)
están relacionadas con el resultado
Uxylt =l xll + IIYD, para cualesquiera .i:,y E :t•,
de donde obtenemos el caso especial
llxU= UxU+ O= Uxll + Ukl = Uúll (= 11>-xll).
Por último,paracualquierzE S,tenemosque 11•11 = ll<A.11 = llkll = 1,mientrasque 11«11 = 2.
La operación binaria cerrada de concatenación nos permite plantear ahora otra defini~
ción recursiva. Ya definimos las potencias de un alfabeto l:. Ahora examinaremos las po-
tencias de las cadenas.

Definición 6 . 7 Para cualquier x E l:*, definimos laspouncias dexcomoxO = A. x 1 • x, x:2 = .u, xl =d,
=XX", . . • ,donden EN.
• • • • ~1
6. 1 uajes: La teoría de conjuntos de las cadenas 319

Esta definición es otro ejemplo de definición recursiva de una entidad matemática: la


entidad matemática que buscamos en este momento se obtiene a partir de entidades (com•
parablc.s) previamente obtenidas. Además. la definición nos proporciona un camino para
trabajar con una concatenación de n-partes (como una potencia (n + 1)) de una cadena
consigo misma e incluye el caso especia) en que la cadena es sólo un símbolo.

Si l:= {O, 11 y x = 01, cn1onces .r' = )., x' = 01, x' = 0101 y x' = 010101. Para cualquier
n > O, X- consta de una cadena de n ceros y n unos, donde el primer símbolo es O y los
símbolos posteriores se alternan. En este caso, 11-" U= 4 = 2llxll, 11-"II = 6 = 3 llxll, y para
todon EN, lix" II =nllxU.

Ahora estamos listos para enfrentar el tema principal de esta sección, el concepto de
lenguaje. Sin embargo, antes de hacerlo necesitamos saber más acerca de otras tres ideu.
Estas ideas se refieren a subsccciones especiales de cadenas.

Dofini<ión 6.8 Si x. y E i:• y w = x.y, entonces la cadenax es un prefijo de w, y si y ~ A. entonces x es un


p,r.fijo propio. De manera similar, la cadena y es un sufijo de w y es un sufijo propio
cuandox•~

Sea :t :s. { a. b, e}, y consideremos la cadena w = abbcc. Entonces cada una de las cadenas
A, a. ah, abb, abbc, y abbcc es un prefijo de w, y, excepto la misma abbcc, cada una es un
prefijo propio. Por otro lado, cada una de las cadenas A., e, ce, bcc, bbcc y abbcc es un
sufijo de w, y las primeras cinco cadenas son sufijos propios.
En general, para cualquier alfabeto ~ sin E z • y x 1 E :t para todo 1 Si S n, entonces
cada una de las cadenas A., x1, X1X1, .:r1:c2 .x,, ... , y X1X1x, • •• :e,. es un prefijo de la cadena
x=x1Xix, .. . x•. Por otro lado, l..,:c.,x.,..1 x••x.wx- 1..r.. ... , y x 1.:r2 x, ...x.son sufijos de.esta
cadena.Asf, x tienen+ 1 prefijos, n de los cuales son propios; la situación es la misma para
los sufijos.

Si lxll • S, U,11 =4 yw =.cy,entonceswúeneaxcomounprefijo propioyaycomoun


sufijo propio. En total, w tiene nueve prefijos propios y nueve sufijos propios, puesto que
A es un prefijo propio y un sufijo propio para cada cadena de :E... En este caso, ry es un
prefijo y un sufijo, pero en níoglln caso es propio.

Para un alfabeto dado l:, sean w, a, b, e, d E :E•. Si w ~ ab = cd, entonces


i) a es un prefijo de e, o e es un prefijo de a; y,
ii) bes un sufijo de d, o d es un sufijo de b.
20 Capítulo 6 lenguajes: Máquinas de estados finrtos

tefinición 6.9 Si x. y, z E :t• y w = xyz, entonces y es una subcadeTUJ de w. Cuando x o z es diferente de


A. (de modo que y es diferente de w), y es una subcadena propia.

Para I: = {O, 1 }, sea w = 001 O111 O E :t•. Las siguientes son subcadcnas de w:
1) l O11: Esto surge solamente de una forma: cuando w = xyz, con x = 00, y = l O11 y
z: 10.
2) 10: Este ejemplo surge de dos fonnas:
a) w =.ryz donde x= 00, y= 10 yz = 1110, y
b) w=.ryzpara,=001011,y = lOy z =)...
En el caso (b) la subcadena también es un sufijo (propio) de w.

Ahora que estamos familiarizados con las definiciones necesarias, es hora de pensar en
el concepto de lenguaje.
Cuando consideramos el alfabeto comlln, incluyendo e l espacio en blanco, muchas
cadenas como qxio, el wxxy rojo atzl, y aeytl no representañ palabras o partes de enuncia-
dos del español, aunque sean elementos dc:t•. En consecuencia, para considerar solamen-
te las palabras y expresiones que tienen sentido en el idioma español, nos concentraremos
en un subconjunto del:.*. Esto nos lleva a la siguiente generaJización.

~finición 6.1 O Para un alfabeto dado~ cualquier subconjunto del:• es un lenguaje sobre I.. Esto incluye
el subconjunto 0, al que llamaremos lenguaje vacío.

Si :E= {O, l }. los conjuntos A= (O, 0 1, 001} y B = {O, 0 1, 001, 0001, ... } son ejemplos de
lenguajes sobre l:.

Si l: es el alfabeto de 26 letras, 10 dígitos y los símbolos especiales utilizados en una


implementación dada de Pascal, la colección de programas ejecutables para esa imple-
mentación constituye un lenguaje. En la misma situación, cada programa eje.cutable ~
dría ser considerado un lenguaje, como podría serlo también un conjunto finito panicular
de tales programas.

Ya que los lenguajes son conjuntos, pOdemos formar la unión, intersección y diferencia
simétrica de dos lenguajes. Sin embargo, para poder trabajar, es más útil una extensión de
la operación binaria cerrada definida (en la Def. 6.6) para cadenas.

>efinición 6.11 Para un alfabeto 1: y los lenguajes A. B ~ l:.* , laconcatenaci6n de A y 8 , denotadaconAB,


es {abla E A, b E BJ.
6.1 Lenguajes: La teoría de conjuntos de las cadenas 321

Podríamos comparar la concatenación con el producto cartesiano; veríamos que, aJ igua)


que A xB -:t.BxA e n general, tambi6nAB-:;:. BA en general. Para A, B finitos, teníamos que
IAxB I = IB x Al,peroaquípuede ocurrirque IABI,. IBAI incluso para lenguajes finitos.

Se an :E= {x, y, z }, y sean A, 8 los lenguajes finitos A• {x, ;r;y• .t}, B= { A,y}. Entonces
AB = {x, X)', i. xyy, zy) y BA = {x. xy. z. yx, yxy, yz), por lo que
l) IABl =H6 = IBAl, Y
2) IABl =546 = 3 · 2= IA IIBI.
Las diferencias surgen debido a que hay dos formas de representar xy: ( 1) xy para x E
A,y E By (2)xyl,.dondexy E A y 1,. E B. (El concepto de unicidad de la representación es
algo que no podemos dar por hecho. Aunque no se cumple en este caso. es la clave del
éxito de muchas ideas matemáticas, como lo vimos en el teorema fundamental de la arit-
mética (Teorema 4.11 ) y como lo veremos de nuevo en el capítulo 15.)

El ejemplo anterior sugiere que para lenguajes finitos A y 8 , IAB I ~ IA 11 B!. Podemos
mostrar que esto es cierto en general.

El siguiente teorema analiza algunas propiedades satisfechas por la concatenación de


lenguajes.

TEOREMA6.1 Para un alfabelto l:, sean A, B, C C :I.*. Entonces


a) A{>.} = {>.}A =A b) (AB)C = A(BC)
c) A(B U C)=AB U AC d) (B UC)A = BA UCA
e) A(Bí1C) \;; AB n AC f) (B í1 C)A\;;BAnCA
Demostración: Nos ocuparemos de las partes (d) y (t) y dejaremos las demás para el
lector.
(d) Como tratamos de demostrar que dos conjuntos son iguales, de nuevo utilizaremos
la idea de igualdad de conjuntos de la definición 3.2. Si partimos de un elemento x El:*,
tenemosque x E (BU C)A :=;x=yzparayE BU Cy z E A=> (x;;;;: yzpara y E B, z E
A) o (x=yz para y E C, z E A) =>xE BA ox E CA, por lo cual (BU C)A \;; 8A U CA.
Recíprocamente, si x E BA U CA => x E BA o x E CA => x - ba1 donde b E By a 1 E A,
ox= ca2 donde c E Cy a.:i E A. Supongamosquex=ba 1 para b E B, a, E A. ComoB C
BU C, tenemosquex=bai, donde b E BU C y a 1 EA. Entoncesx E (BU C)A, por1oque
BA U CA ~(BU C)A. (El argumento es similar si x = caz.) Una vez establecidas ambas
inclusiones, se sigue que (BU C)A • BA U CA.
(f) Parax E :i:•, vemosquex E (B n C)A =>x= yz donde y E B n Cyz E A=> (x=
yzparay E By z E A)y (x= yzparay E Cy z EA) =>x E BA yx E CA =>x E BA (1 CA,
por lo que (B n C)A \;; BA n CA .
Sil:= {x, y,<), sean B = {x, xx, y ), C= {y. xy ) y A= {y, }J'). Entonces xyy E BA n CA,
pero xyy ~ (B n C)A. En consecuencia. (B n C) A e BA n CA.

Análogamente a los conceptos del:~. :i:•, :E+, damos las siguientes definiciones para un
le nguaje arbitrario A C l:"'.
Z2 capítulo 6 Lenguajes: M.jquinas de estados finitos

efinición 6.12 Para cualquier lenguajeAS:: :E* podemos construir otros lenguajes de la manera siguiente:

a) A' = {'-),A' =A y para cµalquiern E Z.,A-' = {abl a E A, b E A" )


b ) A•=U,.ez•Aª ,lacl.au.surapositivadeA
e) A* = A• U {A}. El lenguaje A* es la clausura <k Kleene de A, en honor del lógico
norteamericano Stephen Cole K.leene (1909- ).

- Sí :E= {x, y, <) y A= {x}, entonces (!) A' ={,.}; (2) A•= {:t"} paran EN; (3) A•= {x'I
n;, I} y(4)A•= {.x"ln;, o¡.

- Sea:E= {x.y) .

a) Si A= {.:u,.:cy, yx, yy} =l:2, entonces A* es el lenguaje de tcxlas las cadenas w E :t•
tales que la longitud de w es par. .
b) Con A como en la parte (a) y B = {x, y}, el lenguaje BA* contiene todas las cadenas
de:E• de longitud impar y también vemos que, en este caso, BA* =A*B y que:E• =
A*UBA*.
e) El lenguaje {x}{.r, y}* (la concatenación de los lenguajes {x} y {x, y }*) contiene
cada cadena del:* para la cual .res un prefijo.
El lenguaje que contiene todas las cadenas de l:* para las cuale s yy es un sufijo
puede definirse como {.r, y}* {yy}.
Cada cade na del lenguaje {.r. y}*{x.:cy}{.r, y }* tiene ax.:cy como una subcadena.
d) Cualquier cade na del lenguaje {.r}*{y}* consta de un número finito (tal vez igual a
O) de .r seguido por un número finito (que también puede ser cero) de y. Y aunque
{x}•(y}* ~ {x, y}*, la cade na w = xyxestá en {x, y)• pero no en {x)*{y)•. Por lo
tanto, {x)*{y)• e {x, y }•.

En el '1.gebra de los números reales, si a, b E R y a. b > O, entonces a'2 =Ir:::>- a= b. Sin


embargo, en el caso de los lenguajes, •i:E= {x, y),A = {1-,x, x',,4, .. . ) = {.x"ln ~0)-{x')
y B = (-"'In~ O), entonces A' =B'(=E), pero A -t B. (Nota: Nunca ocurre que A E :t, pero
es posible que A E A !;;; :i:•.)

Continuaremos esta sección con un lema y un segundo teorema que tratan acerca de las
propiedades de los lenguajes. El lema proporcíona otra situación en que utilizamos el
principio de inducción matemática.

:MA6.1 Sea l: un alfabeto, con lenguajes A, B ~ l:*. Si A ~ B, entonces A" ~ 8" para todo
nez•.
6.1 Len uajes: La teoría de conjuntos de las cadenas 323

Demostración: Consideremos la proposición abiena S(n): A S: B ~A'~ 8". Como A 1 . .


A e:; B = 8 ', vemos que S(1) es la proposición: A e:; B ⇒ A e:; 8 , por lo que el resultado es
cieno en este primer caso. Si suponemos que S(k ) es verdadero, tenemos que A , B ==>
A'c:; B'. De la hipótesisdeS(k + 1) sabemos queA e:; 8 y deS, tenemosA' e:; B' (por Modus
PoMns). Consideremos ahora una cadena x de A.•· 1• Entonces, de la definición 6.12, sabe-
mosque x = x 1.x., donde x 1 E A, Xt E A•. Como A S: By Aª ~ H (por la hipótesis de
inducción), tenemos que x 1 E B y .ra E H . En consecuencia, .r::. XtXa E Blr = 8"1• Por Jo
tanto, S(k + 1) es verdadera y, por el principio de inducción finita, se sigue que si A ~ B,
entonces A• ~ Ir para todo n e Z...

TEOREMA6.2 Para un alfabeto :E y los lenguajes A. 8 e:; :i:•,


a) Ac:;A8' b) A!;;B'A
e) Ac:; B ~ A •i;; s • d) Ac:;B~A• c:; s•
e) AA'•A' A • A ' f) A'A" = A' = (A')" = (A ' j< = (A')"
g) (A U B)' = (A'UB')' ~ (A ' B ' )'

Demostración: Haremos las demostraciones de las panes (c) y (g).


(e) SeaA i;; 8 yx E A'. Entonccsx E A" ⇒ x EA•, para algún n E z·. Del lema 6.1 se
sigue entonces quex E Ir S: Ir con lo que hemos mostrado que A•~ Ir.
(g) [(A U B)•= (A• U 8•)• ]. Sabemos que A c:;A•, 8c:; 8 • ⇒ (A U 8) !; (A ' U B' )
⇒ (A U B)' e:; (A• U 8')' (por la parte d). A la inversa, tambifo vemos que A. 8 !:: A U
8 ⇒ A•, B • e:; (A U 8)' (por lapane d) ⇒ (A'U 8') !:: (A U B)' ⇒ (A ' U 8•)• !:: (A U
B)' (por las partes d y f). De ambas inclusiones se sigue que (A U 8 )' = (A • U 8')' .
[(A• U B•) • = (A• B•)•]. Primero vemos que A•, B• ~A• B* (por las partes a y b) ::>
(A • U 8') e:; A• 8' ⇒ (A• U 8')' e:; (A• 8')' (por la pane d). Recíprocamente, si .ty E
A* B•, dondex E A • y y E B*, entoncesx, y E A• U B• , por lo quexy E (A • U Bo/ y
A *B* !; (A• U B•)• . Usando nuevamente las panes (d) y (f) para moslrar que (A• B•)• !:
(A• U B*)*, de donde se sigue el resultado.

Para cerrar esta primera sección analizaremos la idea de un conjunto definido en forma
recuniva (dada en la sección 4.2), como lo demuestran los siguientes cuatro ejemplos.

Para el alfabeto I: = {O, 1} consideremos el lenguaje A ~ :t• , donde cada palabra de A


contieoesolamente una aparición del símbolo O. Entonces A es un conjunto infinito y entre
las palabras de A podemos encontrar O, 01, 10, 0 111 1, 11 11O111 y 111 1111 111O. Tam-
bién existe una infinidad de palabras de :E•que no están en A (como 1, 11, 00, 000, 010 y
O1111 111111O). Podemos definir este lenguaje A de manera recursiva. como sigue:

1) Nuestro paso inicial indica que O E A ; y


2) Para el proceso recursivo queremos incluir en A las palabras 1x y xl, para cualquier
palabrax E A.

Con esta definición, el siguiente análisis muestra que la palabra 1011 está en A.
Capítvlo 6 Lenguajes: Máquinas de estados finitos

De la parte (1) de nuestra definición. sabemos que O E A . Entonces, si aplicamos la


parte (2) de nuestra definición tres veces, vemos que :

i) 01 E A, puesto que O E A;
ü) 011 EA,yaque OI E A;y
iii) como 0 11 E A, tenemos que 1011 E A.

Para :E= {(,)}. el alfabeto que contiene a los paréntesis izquierdo y derecho, queremos
examinar el lenguaje A ~ ~ • que consta de todas las cadenas no vacías de pmintesis que
son gramaticalmente correctas como expresiones algebraicas. Entonces tenemos, por ejem-
plo, las tres cadenas (( )), ((() ( ))); y ( ) () () de este lenguaje, pero no tenemos cadenas
como (( ) ( ); ()) (( ); o) () (())). Vemos que para que una cadenax (<listinta de A) esté en
A , entonces
i) debemos tener el mismo número de paréntesis izquierdos que derechos; y
ii) el número de paréntesis izquierdos (siempre) debe ser mayor o igual que el número
de paréntesis derechos, al examinar cada uno de los paréntesis en :e, leyéndolos de
izquierda a derecha.
El lenguaje A puede definirse de manera recursiva como sigue:
1) () está en A; y
2) Para cualesquiera x, y E A, tenemos (i) xy E A y (ü)(x) E A.

[Como mencionamos antes del ejemplo 4 .19, también tenemos una restricción impllcita.
que ninguna otra cadena de paréntesis está en A, a menos que se obtenga mediante los
pasos ( 1) y (2) anteriores.]
Con esta definición recursiva mostraremos la forma de establecer que la cadena (( )
()) E :i:• está en el lenguaje A.

Pasos Razones
! ) () está en A. Parte (1) de la definición recursiva
2) () () está en A. Paso(!) y parte (2i) de la de finición
3) (() ()) está en A. Paso (2) y parte (2ii) de la definición

Paral: = {O, l }, la siguiente es una definición recursiva del lenguaje A~ I.* tal que:c E A
si el número de ceros de :e es igual al número de unos.
1) AEA; y
2) si x E A, entonces cada uno de los siguientes también está en A:

1) 0lx U) x0l W) Oxl


lv) lOx v) xtO Yi) lx0
(Y ninguna atta cadena de ceros y unos está en A.)
uajes: La t~óa de con·untos de las cadenas 325

Dado un alfabetoI.. consideremos la cadena.z :: .z1 ~.z3 ••• x_1 .z. E :t•. dondex,E :tpara
cada 1 Si S n y n E z•. El inverso de .z, que se denota~, es la cadena que se obtiene de.z
leyendo los símbolos (en x) de derecha a izquierda; es decir, r:: x• .z_1 ••• x1 x2 xi" Por
ejemplo, si I =
{O, 1} y x = 01101, entonces r=
10110 y para w • 101101 tenemos que
w' = 101101 =w. En general, podemos definir el invcnode unacadena (de I•) de manera
recursiva como sigue:
1) l.'=A;y
2) Para cualquiern EN, si x E r" 1, entoncesescribimos.z = r;y doadcz E I y y e .r,
r
y definimos = (zy)' = {y')t.
Con esta definición recursiva podemos demostrar que si :t es cualquier alfabeto y X¡,
~ Et•, entonces (x1 x;f = x,fxf.
Demostnción : La demostración es por inducción matemá.tica sobre el valor de llx1 II. Si
Bx, 11 =0, entonccs, x, = A.y (x.xi/=(},.x;)' =xf =xfA. =xfA.• =xfxf ya que;.•= A.S<:gt!n la
parte ( 1) de la definición recursiva. En consecuencia, el resultado es cieno en este primer
caso, el cual constituye el paso inicial. Para el paso inductivo supondremos que el resulta-
do es verdadero para cualesquiera y, x, E I•, donde b 11 • k para algún k E N. Conside-
remos ahora lo que ocurre con x,, x, E I•, doodex, = zy., con lltll = 1 y IIY, 1= k. Aquf
vemos que (X¡X2f = (zy1x1f:: (y,x0"z [de la pane (2) de la definición recursiva] (de=mrz
la hipótesis de inducción) =xf<zy,)' [de nuevo por la parte (2) de la definición recursiva] =
xfxf. Por lo tanto, el resultado es cierto para todo xi, x 2 E :t• por el principio de inducción
matemática.

EJERCICIOS 6.1 l . Sea :t • (a, b, e, d.,). (a) ¿Cuánto valen l:t'I y l:t'I? (b) ¿O,áotas cadenas de :t• tienen una
longitud de al menos cinoo?
2. Paral:• {w, x. y, 4}, determ.it1ed m1merodeeadenas en J:- de lon¡itud cinco(a)quccomien-
zan con w; (b) con precisamente dos w; (e) sin""; (d) con un número par de w.
3. Si x e i;• y 8,'11 =36, ¿cuánto vale lxll1
4, Sca:t= (P, -< JI t). doodcP denou un espacio en blanco, de modo que,j! •x.llll• py ,j!y•
xy, pcro xAy ='X'Y· Calcule lo siguiente:
•l N bl IUI <l 1111 dl 1W1111 •l 11!'1 !) lx~M al 111>-1 ~l P.''I
5. Seal: • {v, w. x. y. d y A =l.1'-i J:•. ¿Cutntas cadenas en A tienen arycomo prefijo propio?
6. Sea :t un alfabeto. Sea .r,E l: para I S i S IOO(doode .r,•.r, para cualquier IS i <jS 100).
¿Cutntas subcadenas no VICfas e:ósten para la cadenas • x, Xi •• • i,.?
7. Para el alfabeto t = {O. 1 J, sean A, 8, C C J:• los siguientes lenguajes:

A • {O, 1, 00, 11,000, lll, <XXXJ, 1111},


B •{wE:t"l2sl wl},
C•{wE :tºl2.e Uwl}.
Determine los siguientes subconjuntos (lenguajes) de :t•.

a) AnB b) A-8 <) At.B


d) ;1 n c •> snc !) B U C
al {Añl'.')' b) XnC' 1) XnB
l26 Capítulo 6 Lenguaj~: Máquinas de estados finitos

=
8. Sean A= { 10, 11}, B {00, 1) lenguajes para el alfabeto l: = {O, 1 }. Determine lo siguiente:
(a)AB; (b) BA; (c)A'; (d) B'.
9. Si A, 8, C y D son lenguajes sobre l:, demuestre que (a) (A C B A C C D) ~ AC C BD; y
(b)A 0 = M= 0.
10. Paral: = {,;y, z). sean A, B ¡;; I:• dadas por A= {xy} y B = {1-,x). Determine (a)AB; (b) &<;
(e) 8 3; (d) S-; (e) A • .
11. Dado un alfabeto 1:,.·¿existe algún lenguaje A~ V tal que A• =A?
12. Para l: = {O, 1}, determine si la cadena 00010 esti en cada uno de los siguientes lenguajes
(tomados de l:*).
a) {O, l}" b) {000, 101}{10, 11) e) {00){0}"{10}
el) (000) ' {1}"{0) e) {00)' {10}' f) {0}' {1}'{0}"
13. Para J:; {O,
I }, describa las cadenas en A" para cada uno de los siguientes lenguajes A C i::•:
a) (01} b) {000} e) {0,010) el) {l, 10)
14. Para l: = {O, 1 J. determine todos los posibles lenguajes A. B ~ 1:• tales que AB = {01. 000.
0101, 0111, 01000, 01011 I}.
lS. Dado un lenguaje no vacío A!: l:•, demuestre que si A 1 =A. e ntone.es A E A.
16. Para un alfabeto dado l:. sea a E l:, con a fijo. Defina las funciones p.,. s., r: i:• ➔ :E• y la
función d: l:'" ➔ J:• como sigue:
i) La función prefijo (con a): pJ..x) = ax, x E :t•.
ii) La función sufijo _(con a): sJ.x) = xa, x E :t•.
iü) La función de inveí$ión: ,Q.) = l; para x e :t·, si x = x 1 x2 • • • x ...1 x., donde~ El:, para
todo 1 Si S n.entonccs r(x) = x., x...1 • •• x2 x1 =:t! (como se definió en el ejcmplo6.17).
ÍY) La función de e]iminación fmnta1: para cada x E :E", si x = x 1x 2x 1 •• • x., entooccs d(_x) =
X2~- •• X• •

a)
¿Cuáles de estas cuatro funciones son uno a uno?
b ) ¿Oláles de esw cuatro funciones son sobre? Si una función no es sobre. dctcnnine su imagen.
e) ¿Es alguna de estas funciones ínvertiblc? F.n tal caso, determi~su función inversa.
d) Sea :t = {a, e, i, o, u}. ¿Cuántas pa1abras x E I:' satisfacen r(x) = x? ¿Cuántas en V'!
¿Cuántas en r", donde n EN?
e) Parax E :t•, determine (dopJ(x) y (r 0 d 0 r 0 s.J(x).
=
f) Si J: {a. e, i, u} y 8 º· =
{&. ai, ao, 00, eio, eiouw} C :t•, determine r-1(8 ), p;1(B).
s;'(B), y W'(B)I.
17. Si A(~ 8) es un lenguaje y A2 = A, demuestre que A = A• .
18. Proporcione las demostraciones de las partes restantes de los teoremas 6.1 y 6.2.
19. Demuestre que para cualesquiera lenguajes finitos A, B C. :t• IABI S IA J IBI .
20. Si A. B son lenguajes sobre:t y A S: B• , demuestre que A•~ B•.
21. Para un alfabeto dado :t, sea / un conjunto de índices, donde para cada i e / • B1 S: :t•. Si A S:
:t•, demuestre que (a) A(UiE, BJ = U.v AB; y (b) (Uu B;} A = UEJ B;A. [Estos resultados
generalizan las partes (e) y (d) del teorema 6.1 .]
21. Para I: = {;e; y}, use los lenguajes finitos de :t• (como en el ejemplo 6.12), junto con las
operaciones d e conjuntos, para describir el conjunto de cadenas de E• ta1cs que (a) contiCDC11
cx·actamcnte una ocurrencia dex; (b) contienen exactamente dos ocurrencias de ,r. (c) comim-
zan con x ; (d) terminan conyxy. (c)comienzan conxoterminan con yxyo ambas; (O comien-
zan con x o terminan con yxy pero no ambas.
23. Sea:tel alfabeto {O, I} yseaA ~E• el lenguajedefinidorecursivamcntedelamancrasiguiezu:
1) Los símbolos O, 1 csl!n en A: ésta es la base de nuestra definición; y
2) Para cualquier palabrax e A. la palabra Oxl tambitn está en A; esto constituye el proceso
recursivo.
6.2 Máquinas de estado finito: Un primer encuentro 327

(a) Determine cuatro palabras diferentes de A; dos de lon¡:itud 3 y dos de longitud S.


(b) Use la definición recursiva dada para mostrar queOOOl 111 está en A.
(e) Explique po< qu<! 00001111 oo est-' en A.
24. Para el alfabeto ~ = {O, 1) , sea A ~ t• el lenguaje fonnado por todas las palabras que no
cootienen la subcadena 10. Entonces, las palabras como l.. O, OO. 000. 0000, 01. 001, 0011,
0111, l. 11 , 111 csún en A, pero ninguna de lu palabras 10, 101, 100. 0 10, 0010 o 11110
est.in en este Len¡uaje.
a) Di una definición recursiva del lenguaje A.
b) Use la definición de la parte (a) para determinar si 00111 estf en A.
15. Use la definición recursiva del ejemplo 6.15 para verificar que cada una de las siguientes
cadenas esti en el lenguaje A de ese ejemplo.
•> (())() b) (())()() e) () (()())
26. Proporcione una definición recursiva de cada uno de los siguientes lenguajes A ~ :t•. donde
l: = {O. l}.
a) z E A si (y sólo si) el nilmero de ccros de z es par.
b) z E A si (y sólo si) sólo aparece un cero en x.
e) z E A si (y sólo si) todos los UDOS de x preceden a todos los ceros.
27. Para cualquier alfabeto :t. una cadenax de V es unpalfndromo si x =x! (es decir. z es igua] a
su inverso). Si A e :t•. donde A - {x e :V lx • .xA), ¿cómo p()demos definir el lenguaje A de
manen. tt:CUrsiva?

6.2
Máquinas de estados finitos:
Un primer encuentro

Regresemos ahora a la máquina expendedora mencionada al comienzo de este capítulo y


anali~mosla en las siguientes circunstancias.
En cierta oficina, una máquina expendedora tiene dos tipos de refrescos en lata: cola
(C) y cerveza de raíz (RB). El costo de una lata es de 20 centavos. La máquina ac.epta
monedas de cinco, diez y veinticinco centavos y devuelve et cambio necesario. Un día.
María decidió comprar una lata de cerveza de raíz. Fue a la máquina expendedora, insertó
dos monedas de cinco centavos y una de diez centavos, en ese orden, y oprimió el botón
blanco, marcado con W. El resultado: salió su lata de cerveza de raíz. (Para obtener una
lata de cola se presiona el botón negro, designado por B.)
Podemos representar lo que hiro María para hacer su compra, como aparece en la tabla
6.1; to es el instante inicial, cuando ella inserta su primera moneda de cinco centavos, y ti,
t2 , 11 , 14 son los instantes posteriores, con 11 < l : < t 3 < 1••
T>bla 6 .1
,. ,, r, ,, ,.
Estado (1) ... (4)s1 (S~ (7)s,(I~ (IO)s,(2~ (13)s,
Entrada (2) 5~ es>s~ (8) 1~ ( ll )W
Salida (3)Nada (6) Nada (9) Nada (12)RB
Los a.d.mcros (1). (2), ... , (12). (13) de ese.a tabla indican d orden de sucesos en la compni de la cerveza de
raíz de Maña. Pan~ enuada. en d ins~ t., O :S: i S 3, u.isle en e.se i,u1w1u una salida ~poadien1t y
IIICgO un cambio de estado. El si¡uiente estado ti~ 1 depe,ide de la entrada y el ~ (presente) ai d instante , ..
28 Capítulo 6 lenguajes: Máquinas de esta dos finitos

La máquina está en un estado de espera en el estado s0 • Espera que un cliente comience


a insertar monedas con un total de 20 centavos o más y oprima un botón para obtener un
refresco. Si en cualquier momento el total de monedas insertadas excede los 20 centavos,
la máquina proporciona e] cambio necesario (antes de que el cliente oprima el botón para
obtener su refresco).
En el instante to, María inserta en la máquina su primera entrada, 5 centavos. No recibe
nada e n ese momento, pero en e] tiempo siguie nte e" la máquina está en el estados1, donde
ésta recuerda su total de 5 centavos y espera una segunda entrada (de 5 centavos en el
instante r1). Nuevamente, la máquina (en el instante 1 1) sigue sin proporcionar una salida,
pero al siguiente instante, t2 , está en el estados2, recordando un total de diez centavos, que
es igual a S centavos (recordados en el estado s 1) más cinco centavos (insertados en el
instante t 1). Al introducir la moneda de diez centavos (en el instante ri) como siguiente
entrada a la máquina, María todavía no recibe el refresco ya que la máquina no "sabe" el
tipo que ella prefiere, pero ahora ·'sabe., (t3) que ha insertado el total necesario de 20
centavos = 10 centavos (recordados en el estado si) más 10 centavos (insertados en el
instante Ji). Por último. María oprime e l botón blanco y en el instante t 3 la máquina entrega
la salida (su lata de cerveza de raíz) y de spu6s regresa. en el instante t,, al punto de partida
So, justo a tiempo para que Rodrigo, el amigo de María, deposite una moneda de 25 centa-
vos, reciba una moneda de S centavos como cambio, oprin:ia el botón negro y obtenga la
lata de re fresco de cola que desea. La compra hecha por Rodrigo se analiza e n la tabla 62.
Tabla 6.2
.. ,. ,,
Estado (!) So (4) s, (20~) (7) So
Entrada (2) 25e (5) B
Salida (3) cambio de 51 (6) e
Lo que ha ocurrido en el caso de esta máquina puede abstraerse para ayudar en el análisis
de ciertos aspectos de los computadores digitales y los sistemas de comunicación telefónica.
Las principales características de esta máquina son las siguientes:
1) La máquina sólo puede estar en uno de una cantidadfin ira de estados en un instante
dado. Estos estados son los estados internos de la máquina y, en un instante dado, la
memoria total disponible de la máquina es el conocimiento del estado interno en el
que se encuentra e n e se instante.
2) La máquina aceptará como entrada sólo un número finito de símbolos, que se conocen
en conjunto como el alfabeu, de entrada !>. En el ejemplo de la máquina e xpende.do,.
ra, el alfabeto de entrada es {moneda de 5 centavos, moneda de 1O centavos, moneda
de 25 centavos, blanco. negro}, y cada elemento se reconoce por su estado interno.
3) Mediante cada combinación de entradas y estados internos se determina una salida
y un estado siguiente. El conjunto finito de todas las salidas posibles constituye el
alfabeto de salida O de la máquina.
4) Suponemos que los procesos secuenciales de la máquina están sincronizados por
pulsos de reloj separados y distintos y que la máquina opera de manera determinista,
ya que la salida queda completamente determinada por la entrada total proporcio-
nada y el estado inicial de la máquina.

Estas observaciones conducen a la siguiente definición.


6.2 Máquinas de estado finito: Un primer encuentro 329

· ºón 6 .13 Unamáquinadt tstadosfinicoses una 5-uplaM ~ (S, §. O. v. ro). donde S esel conjunto de
estados internos de M; §es el alfabeto de entrada de M; () es el alfabeto de salida de M; v:
S X§ ➔ Ses lafunci6n siguiente uu:uío,· y w: S x§➔ t) es lafuncidn dt salida.

Según la notación de esta definición, si la máquina está en el estados en el instante r, e


introducimosx en ese momento, entonces la salida en el instante!; es w(s, x). Esta salida es
seguida por una transición de la máquina, en el instante t;..1, al siguiente estado interno.
dado por v(s, x).
Suponemos que cuando una máquina de estados finitos recibe su primera entrada, esta-
mos en el instante10 :;::: O y la máquina está en un estado de inicio ya detenninado, denotado
con Se>- Nuesuo desarrollo se concentrará principalmente en la salida y las transiciones de
estado que ocurren de manera secuencial, con poca o ninguna referencia a la sucesión de
pulsos de reloj en los instantes 10, , ., 12, •••
Puesto que los conjuntos S, ~y O son finitos, es posible representar v y oo, para una
máquina de estados finitos dada, por medio de una tabla que enumera v(s, x) y Ca>(s, x) para
todos ES y todox E §. y que se conoce como latab/atk estados o tablad~ traruici.ón de
la máquina dada. Una segunda representación de la máquina se hace por medio de un
diagrama de tstados.
Mostraremos la tabla de estados y el diagrama de estados en los siguientes ejemplos.

Consideremos la máquina de estados finitos M = (S,§. O, v. w), donde S:;::: {s0 , s 1, s 2), ~ =
{O} :;::: {O, l} y v, (J) aparecen en la rabia dt tsuulos de la tabla 6.3. La primera columna de
la tabla enumera lostsrados (pnstntts) de la máquina. Las entradas de la segunda fila son
los elementos del alfabeto de entrada,. que se enumeran una vez bajov y otra vezbajo(J).
Los seis números de las dhimas dos columnas (y las últimas tres filas) son elementos del
alfabeto de salida().
Tabla 6.3

o
V

1
..
o 1
So So S¡ o o
S¡ s, S¡ o o
s, So S¡ o 1
Por ejemplo, para calcular v(s¡, 1), encontramoss1 e n la columna de estados presentes y
procedemos en horizontal sobre s., hasta estar debajo de la entrada 1 en la sección de las
tablas de v. Esta entrada da v<s1o I ):;::: s 1• De la misma forma vemos que m(s., 1):;::: O.
Si s0 indica el estado inicial y la entrada proporcionada a Mes la cadena 1OI O, entonces
la saJjda es 00 IO. como lo demuestra la tabla 6.4 . En este caso, la máqui na se queda en el

Tabla 6.4

E,tado So v(so, 1) = S¡ v(s1, O) = s, v(s2, 1) =s1 v(s .. O) ~,,


Entrada o 1 o o
Salida w(so, 1) = O w(s1 , O) = O w(s,. 1) = 1 w(si, 0)-0
30 Capitulo 6 lenguajes: Máquinas de estados finitos

estados1 , de modo que si tuviéramos otra cadena de entrada, proporcionaríamos el primer


carácter de esa cadena, O en ese caso, en el estado s2, a menos que se vuelva a ª"anear la
máquina para iniciar de nuevo en s0 •

Puesto que estamos interesados principalmente en la salida y no en la sucesión de esta-


dos de transición, la misma máquina puede representarse por medio de un diagrama dt
estados. En este caso podemos obtener la cade na de salida sin enumerar los estados de
transición. En tal diagrama, cada estado internos se representa mediante un círculo con s
dentro de é l. Para los estados S; y s;, si v(s;, x ) = Sj para x E !J, y co(s;. x) = y para y E (),
representamos esto en el diagrama de estados trazando una arista dirigida (o an-o) del
círculo paras; al círculo para sJ y rotulamos el arco con la entrad.ax y la salida y, como se
muestra en la figura 6.1.

Con estos convenios, el diagrama de estados de la máquinaM de la tabla 6.3 aparece en


la figura 6.2. Aunque la tabla es más compacta, el diagrama nos permite seguir una cadena
de e ntrada a través de cada uno de los estados de transición que determina. eligiendo cada
uno de los símbolos correspondientes de salida antes de cada transición. Si la cadena de
entrada es 0011 Ol O1, entonces, partiendo del estado So, la primera entrada de O produce
una salida de O y nos lleva de nuevo a so- La siguiente entrada de O produce el mismo
resultado, pero para la tercera entrada. l , la salida es O y estamos ahora en el estado.J1• Si
seguimos de esta fonna. llegamos a la cadena de salida ()()(X)()lOI y terminamos en el estado
s 1• (Observemos que la cadena de entrada 00110101 es un elemento de §*, la clausura de
Kleene de 9- y que la cadena de salida está en O*, la clausura de Kle.ene de O.)
Si partimos de so. ¿cuál es la salida para la cade na de entrada 1100101101?

Figur,1 6.2

Para la máquina expendedora ya descrita en esta sección, tenemos la tabla de estados de la


tabla 6.5, donde
1) S ~ {so. si, s2, s 3, s 4 } , de modo que en el estado St-o O~ le~ 4, la máquina recuerda
retener Sic centavos.
6.2 Máquinas de estado finito: Un primer encuentro 331
Tabla 6.5

5c !Oe
y

25e B w Se J()e
..
25,t B w
,,
,,•• ,, s,
s,
..
s,
So
s, ,,•• n
n
n
n
Se
!Oe
n
n
n
n
s, ,, s, s, s, s, n n !Set n n
,, ••
s, .. ••
•• ..
s, s,
So
s,
So
n
se
Set
!Oe
20,t
25,t
n
e
n
RB

2) §= (5 centavos, 10 centavos, 25 centavos, B, W}, donde B denota el botón negro


que se presiona para obtener el refresco de cola. yWcl botón blanco para la cerveza
de ruz.
3) (1 = (n(nada), RB(ccrveza de raíz), qcola), 5 centavos, 10 centavos, 15 centavos,
20 centavos, 2S centavos}.

Como observamos en el análisis anterior al ejemplo 6.19, para una máquina de estados
finitos general M = (S, § , ll, v, w) , la enlr.\da puede pensarse como un elemento de §•, y
la salida como uno de ()•. En consecuencia. podemos extender los dominios de" y ro de S
x §a S >< f*. Panco ampliamos el codominio a o •, recordando que, de ser necesario.
tanto l• como O* contiene n la cadena vacía A.. Con estas extensiones, six1 x2 •• • i,t E ~•.
para k E z•, y si partimos de cualquier estado s 1 ES, tenemos
v(s,.x,) =s,t
v(s,..r 1.r1) = v(v(s,..r1),.r,) a v(s1 ,.r,) =s,
v(s,,.r1.r1 .r3) =v(v(v(s,,x,),x,),x,)
______, =v(,,,.r,) =••
s, )
v(s,,x,) =s,

v(S1,.t1.t2 ... .t4) =v(s1,.t1)"""S.t1'1, ~


w(s,.x,) = y,
w(s,.x,x,) = o,(s,.x,)w(v(,,.x,),x,) = w(s,.x,)w(s,,x,) - y, y,
w(s1,.t1.t2XJ) = w(s1 ,x,)w<s2 ,xi)w{s1 , XJ) ""'Y1Y1Y1

w(s1 , x1x2 . . . xJ = w(s1 , z1)w(s2,xi) ..• w(s1,zt) = Y1Y2•. •YtEO•


Además, v(s 1, A)= s 1 pan cualquier s 1 E S.
(Usaremos estas extensiones de nuevo en el capítulo 7 .)
Cerraremos esta sección con un ejemplo pertinente en las ciencias de la computación.

Sean.t =x, x4 .t1 .t1 .t1 sOOl 11 YYª y,y.y3 y1 y1 =01101 m1meros binarios, doodcx1 y y1 son
los bits menos significativos. Los ceros restantes en x y y aparecen para que las dos cade-
nas x y y sean de igual longitud y garanticen el espacio suficiente para completar la suma.

.........
t El estado s1 csi dderminado poc s, y i 1• No es solamca1e el se¡uGdo de una lisa f!"determiftada 6e
Capítulo 6 lenguajes: Máquinas de estados finitos

Figura 6.3

Un sumador binario en serie es una máquina de estados finitos que podemos usar pan
obtener x +y. El diagrama de la figura 6.3 ilustra esto, donde t =~z.z.,;z1 tiene el digito
menos significativo e n el bit z1 •
En la suma t = x + y, tenemos
x= O O 1 1
+y=+0 1 O 1
z= 1 o
,,. o o
tercera '
.....
primer.i

Observamos que para la primera sumax1 :;;:; y 1 :;;:; 1 y z1 :;;:; O. mientras que para Ja tercera
suma tenemosx1 :;;:;y3 = 1 y z 3 = 1 por acarreo de la sumadex2 y y2 (y el acarreo dex 1 + y 1).
En consecuencia, cada salida depende de la suma de dos entradas y de la habilidad para
recordar un acarreo de O o 1, lo que es crucial cuando el ~ eo es l .
El sumador binario en serie se modela mediante una máquina de estados finitos M =
(S. §, O, v, ro) como sigue. El conjunto S:;;:; {s0 , s 1 }, donde s1 indica un acarreo de i; § =
{OO. 01, 10, 11}; así, existe un par de entradas, dependiendo de si buscamos O+ O, O+ 1,
1 + O o 1 + l. respectivamente; y O= {O, 1}. Las funciones v y ooestán dadas en la tabla de
estados (Tabla 6.6) y el diagrama de estados (Fig. 6.4).

Tabla 6.6

00 01
.10 11 00 01
.. 10 11
So So So ~ S¡ ó 1 1 o
s, So s, S¡ s, 1 o o 1

En la tabla 6.6 encontramos, por ejemplo, que v(s1, 01 ) = s 1 y ro(.s1, 01) = O.ya que r 1
indica un acarreo de 1 desde la suma de los bits anteriores. La entrada 01 indica que
estarnos sumando O y 1 (y acarreando un 1). De aquí que la suma sea 10 y c.o(s 1, 01) = O
para el O de 10 . El acarreo se recadena otra vez en s 1 = v(s1• 0 1).
Del diagrama de estados (Fig . 6.4) vemos que el estado inicia) debe ser So ya que no hay
acarreo anterior a la suma de los bits menos significativos.

Figura 6.4
6.2 Máquinas de estado finito: Un primer encuentro 333

Los diagramas de estados de las figuras 6.2 y 6.4 son ejemplos de grafos dirigidos
etiquetadoJ. Veremos más de la teoría de grafos en este libro. ya que esta teoría no sólo
tiene aplicaciones e n las ciencias de la computación y la ingeniería eléctrica. sino también
en la teoría de códigos (códigos de prefijo) y optimización (redes de transporte).

EIEROCIOS 6.2 1. Con la m~uina de estados finitos del ejemplo 6. 18. detennine la salida para cada una de las
siguientes entradasx E §,•, asf como el último escado interno en el proceso de transición. (Suponga
que siempre partimos del estado so-)
a) x = 1010101 b) x = 1001001 e) x = 101001000
2. Para la máquina de estados finitos del ejemplo 6. 18 . una cadena de entradax produce la cadena
de salida 00101, si panimos del estado si>' Determinex.
3. Se.a M = (S• .9-,0, v, ro) una máquina de estados finitos donde S = {So.si, s 1 , s 1 }, § = {a, b, e}.
O = {O, I}, y vy mestándetennjnados por la tabla6.7.

Tabla 6.7

. ,,. ,, • ..
b e
o 1.
b e
1
,,'• ,,So ,, o o
,,s, ,, o
1

,,s, ,,s, ,, 'º 1


1
1
o 1

a) Si partimos de so, ¿cuál es la salida para la cadena de entrada abbccc?


b) Trace el diagrama de estados para esta máquina de estados finitos.
4. Proporcione la tabla de estados para la máquina expendedora del ejemplo 6.19, si el costo de
cada lata de cola o de cerveza de raíz se incrementa a 25 centavos.
S. Una máquina de estados finitosM= (S. §. C. v.ID) tiene.§ = O= (O, 1} y scdetemlinamcdiante
el diagrama de estados que se muestra en la figura 6.5.

Inicio

Figura 6.5

a) Determine la cadena de salida para la cadena de entrada 11O111, comenzando en si>' ¿CuáJ
es el último estado de transición'?
b) Responda la parte (a) para la misma cadena pero tomando s 1 como el estado inicial. ¿Qué
sucede cuando s 1 y s, son los estados iniciales?
34 capitulo 6 lenguajes: Máquinas de Ht.ados finitos

e) Encuentre la tabla de estados para esta máquina.


d ) ¿Desde qu~ escado debemos comenzar para que la cadena de entrada 10010 produzca la
saJida 10000?
e) Determine una cadena de entrada .x E ,-. de longjrud mínima, tal que v(s., x) = .r1• ¿Es x
tlnico?
6. La miquina M tiene § = {O. 1 J • () y se determina mediante el diagrama de estados de la
figura 6.6.

Figura 6.6

a ) Describa con palabras lo que hace esta m.iquina de estados finitos.


b) ¿Qu~ dcbc recordarclestados1?
e) Encuentre dos lenguajes A, 8 !::§• tales que para c ada :x EAB, co(Jo, x) tiene al como un
sufijo.
7. a ) Si S, §yo son conjunt0$ finitos, con (SI • 3, lll = S. y IOI = 2. determine
1) ISx~I:
ii) el número de funciones v: S x § ➔ S; y,
iü) el número de funciones co: S x () -+ ().
b) Para s. §, y €l de la panc (a). ¿cutntas JMqUin.as de estados finitos determinan'?
8. SeaM = (S.§.(), ", C>) una?MqUinadccstados finitos con §=O = {O, 1} yS, vy rodctermina-
dos por el diagrama de estados de ta figura 6.7.

Figura 6.7
a) Encuemre la salida para la cadena de entrada x = 01101 11011.
b) Dt la tabla de transición para esta núquina de estado$ finitos.
e) Si partimos del estado.fo Yla salida para la cadena dccntrad,u cs 0000001. determine todas
las posibilidadcs para x.
d) Describa con paJabras lo que hace esta máquina de estados finitos.
6.3 Máquinas de estado finito : Un segundo encuentro 335

9. a) Encuentre la tabla de estados para la máquina de estados finitos de la figura 6.8, donde J=
0 = {0, I}.

º· o

'· o

Figura 6.8

b) Seax E J• con l\.rlJ =4. Si 1 es un sufijo deO>(So,.r), ¿cuáles son l~ posibilidades para la
cadenax?
e) Sea A C {O, l}* el lenguaje tal que ©(so. x) tiene l pomo un sufijo para todo x de A.
Determine A.
d ) Encuentre el lenguajeA C {O, I ) • tal que 0>(.1o,.r)tiene a 111 como un sufijo para todoxde
A.

6.3
Máquinas de estados finitos:
Un segundo encuentro

Habiendo visto a1gunos ejemplos de máquinas de estados finitos, pasemos al e studio de


otras máquinas que son importantes para el diseño del hardware de computadores. Un tipo
importante de máquina es e] reconoa dor secuencial.

En este caso, , = () = { O, 1} y queremos construir una máquina que reconozca cada apari-
ción de la secuenda 111 al encontrarla e n cualquier cadena de e ntradax E j • . Por ejemplo,
si x= 1110101111, entonces la salida correspondiente debe ser OOIOOCXX>l 1, donde un 1 en
la posición i-ésima de la salida indiea que aparece un 1 en las posiciones i, i - l , e i - 2 de x.
En este caso, las secuencias 111 pueden solaparse. por lo que podemos pensar que algunos
caracteres en la cadena de entrada son caracteres de más de una tema de unos.
Si s0 denota el estado inicial, vemos que debemos tener un estado que recuerde 1 (el
posible comienzo de 111) y un estado que recuerde 11. Además, cada vez que nuestro
símboÍo de entrada sea O, regresamos a s0 para comenzar de nuevo la bósqueda de tres
unos consecutivos.
En la figura 6.9, .s1 recuerda un único 1, y Sz recuerda la cadena 11. Si se llega a s2,
entonces un tercer .. 1.. indica la presencia de la tema en la cadena de e ntrada, y la salida 1
reconoce esta presencia. Pero también este tercer 'T' significa que tenemos los primeros
dos unos de otra posible tema que sigue dentro de la cadena (como sucede en 11101011
:36 Capítulo 6 Lenguajes: Máquinas de estados finitos

o.o ,.,

Figura 6.9

" 1•• 1). Así, después de reconocer la presencia de 111 con una salida de 1, volvemos a1
estado s2 para recordar las dos entradas de 1 " l ".
Si nos interesa reconocer tcxias las cadenas que terminen en 111, entonces para cual-
quier x E§•, la máquina reconocerá tal se.cuenciacon una salida final de l. Esta máquina
e s entonces un reconocedor del lenguaje A = {O, l }•{ 111 }.

En la figura 6.10 aparece otra máquina de estados finitos que reconoce la misma terna
111. Las máquinas de estados finitos representadas por los diagramas de estados de las
figuras 6.9 y 6.10 realizan la misma tarea y se dice que so~ equivalentes. El diagrama de
estados de la figura 6.10 tiene un estado más que el de la figura 6.9, pero por el momento
no nos intere sa obte ner una máquina de estados finitos con un número mínimo de estados.
En el capítulo 7 desarrollaremos una técnica que toma una máquina de e stados finitos dada
M y encuentra otra máquina equivalente a ésta, con el número más pequeño de estados
internos necesarios.

o.o ,.,

Inicio

Figura 6.10

El siguiente ejemplo es un poco más selectivo.

Ahora no sólo queremos reconocer la presencia de 111 sino también queremos re.conocer
únicamente aquellas ocurrencias que terminen en una posición mú1tiplo de tres. En conse-
cuencia. si~ = O = {O, l} y x e~•. dondex = 1110111, entonces queremos que Cü(s0 , .r) =
0010000, y no 0010001. Además, para x E~•. donde x = 111100111, la salida Cll(s~ x)es
001000001, y no 001100001, ya que, considerando la longitud, no se permite el
solapamiento de las sucesiones 111.
Otra vez comenz.amos en s0 (Fig. 6.11), pero ahoras1 debe recordar un primer 1 sólo si
éste ocurre enx en la posición 1, 4, 7 , . . Si laentradaens(lesO, simplemente no podemos
regresar a s0 como en el ejemplo 6.21. Debemos recordar que este O es el primuo de los
tres símbolos que no nos interesan. De aquí que de s0 vamos a s3 y luego a S,4, procesando
6.3 Máquinas dt estado finito: Un undo encuentro 337

Inicio

Rgura 6 .11
'·º
cualquier tema de la forma Oyz donde O aparece en x en la posición 3k + 1, k ~ O. El mismo
tipo de situación sucede en J 1 cuando la entrada es O. Por 111timo, en s2, la sucesión de 111
se reconoce con una salida de 1, si ésta ocurre. Entonces la máquina regresa a s0 para dar
paso al siguiente símbolo de entrada de la cadena.

La figura 6.12 muestra los diagramas de estados para una máquina de estados finitos que
reconocerá la ocurrencia de una secuencia de 0 101 en una cadena de entrada x e §•,
donde § =C ; {O, 1 ). La nw¡uina de la figura 6.12(a) reconoce con una salida de 1 cada
ocurrenc ia de 0101 en una cadena de entrada, sin importar dónde ocurra. En la figura
6.12(b) la máquina reconoce con una salida de 1 solamente aquellos prefijos de x cuya
longitud es un mllltiplo de cuatro y cuyo final es 0 101. (Por lo tanto, en este caso no se
permite el solapamiento.) En consecuencia, parax; OIOIOIOOJOl ,6>(s,.x) = 00010100001
para (a), mientras que para (b), ro(s,, x) ; 00010000000.

(a) lb)

Figu ra 6.12
338 Capítulo 6 Lenguajes: Máquinas de estados finitos

Ahora que ya hemos analizado algunas máquinas de estados finitos que reconocen
sucesiones, es el momento de examinar un conjunto de sucesiones que no pueden ser
reconocidas por cualquier máquina de estados finitos. Este ejemplo nos da otra oportuni-
dad para aplicar el principio del palomar.

Sean § :;:: O ;:; { O, 1}. ¿Podremos construir una máquina de estados finitos que reconozca
con precisión las cadenas delle nguajeA: {01, 0011, 000111, .. . } : (O'PI i E Z•}? Si lo
logramos y s0 denota el estado inicial, esperaríamos que co(s0 , 01) ~ 01, ro(s0 , 0011) :-
0011, y, en general, Ol(s., 0'1~: O'l', para cualquier i E z•. (Nora : En este caso, por
ejemplo, queremos Cll(s0 , 0011) = 0011, donde el primer 1 es la salida para reconocer la
subcadena 01 y el segundo t es para reconocer la cadena 0011.)
Suponga que existe una máquina de estados finitos M - (S, §, O, v, co) que puede reco-
nocer precisamente las cadenas de A. Seas0 E S, dondes0 es el estado inicial, y sea ISJ =
n ~ l. Consideremos ahora la cadena ~ 11,...1 del lenguaje A. Si nuestra máquina de estado
M opera com:ctamente, entonces Cil(s~ 0""'1 1'"'1) = 0"'"'1 1...1• Por lo tanto, en la tabla 6.8
vemos la forma en que esta máquina de estados finitos procesará los n + 1 ceros, comen--
zando en el estados~ para después continuar con losnestados s1 =v(.so. O).s1 =v(s1, O), ... , y
s,. = v(s,._1, O). Como ISI =n, aplicando el principio del palomar a los n + 1 estados so, s.,
s1 , • •• , s,._., s,., nos damos cuenta de que hay dos estados .f; y s¡ donde i <j pero s1 = s,-

Tabla 6 .8

Estado s, S¡ s, S111-l s, s,...1 s,. s2111..1

Eotnada o o o o o 1 1 1
Salida o o o o o 1 1 1

En la tabla 6.9 vemos cómo la eliminación de lasj-i columnas (para los estados.t;.1,
... , si) da como resultado la tabla 6.10. Esta tabla nos muestra que la máquina de estados
finitosMreconoce Jacadenax=()('"• 1) - U-il 1,.. ,, donden + l - U - i) <n + l. Por desgracia,
x E A, de modo que M reconoce una cadena que supuestamente no debe reconocerse. Esto

Tabla 6.9

Estado ., S¡ s, s, S1♦ 1 s,,, .S2,,,+1

Entrada o o o o o ... ~:
O 1 ... 1 1
Salida o o o o o ...
... O 1 ...
. ..
1 1

Tabla 6.10

Estado s, s, s, :::Hffi; :::Jilii


, s,., :::
S;• 1 s,,, S2,,.+1

Entrada o o o ... O O ... O 1 .•. 1 1


Salida o o o ...
. ..
O O ...
...
O 1 ...
... 1 1
5_3 Mhquinas de estado finito: Un segundo encuentro 339

demuestra que no podemos construir una máquina de estados finitos que reconozca preci-
samente las cadenas del lenguaje A = {Qil 1f i E Z•}.

Una clase de máquinas de estados finitos importante en e l diseño de dispositivos digitales


es la de las mdq•inas ,ú r<rraso ,ú k-unúla,ús, donde k E z·. Para k; 1, quemnos
construir una máquina M tal que si x = x, ~ ... z..,x., entonces para el estado inicial so.
oo(so, x) = 0x1 x2 • • • x_1i de tal manera que la salida es la entrada retardada una unidad de
tiempo (pulso de reloj). [El uso de O como un primer símbolo en oo(so. x) es convencional.)

5 Sean ~ O - {O, 1}. Con el estado inicial s,, Ol(s,. , ) O paru - O o I ya que la primera
safüia es O; los estados s1 y s2 (en la Fig.·6.13) recuerdan una e ntrada anterior de O y I.
respectivamente. En la figura. podem0$ etiquetar, por ejemplo, el arco que va des1 a s2 con
l. O, ya que con una entrada de l necesitamos ir a s2• donde se recuerdan las entradas de 1
en el momentot1 para que se puedan convertir en salidas de 1 en el momentor;,.,· El Oen la
etiqueta 1, O es la salida. ya que el inicio en s 1 indica que la entrada anterior fue O, que se
convierte en la salida actual. Las etiquetas de los ouos arcos se obtienen con el mismo tipo
de razonamiento.

Figura 6.13

~;26 Si observamos la estructura para el retardo de una unidad, extendemos nuestras ideas a la
máquioade retraso dedos unidades de la figura 6.14. Six E §•. seax=x1~ .. . x. donde
m > 2; si s0 es el estado inicia), entonces Ul(s0, x):: ()()x1 •• • x...z. Para los estados s0 , s,, s2 la
salida es O para todas las entradas posibles. Los estados s3, s4 , s, y s, deben recordar las dos
entradas anteriores 00, 01, 10 y 11, respectivamente. Para obtener los otros arcos del
diagrama, debemos considerar uno de ellos y después utilizar razonamientos similares
para los demás. En el arco des, as, de la figura 6.14{a), consideremos la entrada O. Como
la enlrada anterior de s, de s2 es O, debemos ir al estado que recuerda las dos entradas
anteriores OO. &te es el estado s 3• Si regresamos dos estados, des, a s2 a s0, vemos que la
entrada es 1 (des0 as2). Esto se conviene entonces en la salida (retardada dos unidades) del
arco des, as,. La máqui na completa se muestra en la parte (b) de la figura 6.14.
340 Capítulo 6 lenguajes: Máquinas de estados finitos

(a)

Figura 6.14

Regresemos ahora a algunas propiedades adicionales que surgen en el estudio de las


máquinas de estados finitos. Usaremos 1a máquina de la figura 6.15 para los ejempJ05 de
los términos definidos.

Definición 6 .14 Sea M: (S, j, l), v, oo) una máquina de estados finitos.
a) Para S¡, s¡ ES, decimos que si se puede alcanzar desde s1 si s1 = S¡ o si existe una
cadena de entrada x E j• tal que v(s1, x ) = s¡, (En la figura 6.15, podemos a1canzar
el estado s3 desde s0, s1, s2 y s 3 pero no desde s,, s5, S&> o s1• No podemos akanzar
ningún estado desde S3> a excepción del propio s 3.)
b) Un estados E Ses tronsitoriosi v(s,x)=sparaxE §• implicax= l;esdecir, no existe
x E §• tal quev(s, x)=s. (Para la máquina dela figura6.15,s, ese! llnicoestado transitcrio.)

1,1
o.o

Figura 6.15
6.3 Máquinas de estado finito: Un segundo encuentro 341

e) Un estados ES es un sumukro, o estado tú sumukro, si v(s, x):: s, para todo x E


j•. (s, es el wúco •wnidero de la figura 6. 15.)
d) SeaS1 í:S, § 1 C § . Siv1 ::vl,r111.-1: S1 x l1 ➔ S1 (esdecir,larestticcióode vaS1 X
§ 1 C S x § ) tiene su imagen dentro de S., entonces con ~:: Ct1lsrt,,• M 1 :: (S.,
, ,. IJ, v 1, w 1) . . una subm4qu.ina de M. (Con S, = (s., s,, ,,, ,,¡, e , , = (O, ¡ }.
obtene!D05 una subnáquina M 1 de la rmquina M de la figura 6.15.)
e) Una máquina C$ faenemenle conexa si para cualesquiera estados S;, s1 ES, pode--
mos alcanzar s1 desde S¡. (La máquina de la figura 6. 15 no es fuertemente conexa,
pero la submáquina M1 de la parte (d) tiene esta propiedad.)

Concluímos C$ta sección con un resultado que utiliza un diagrama de árbol.

,etinición 6.15 Para una máquina de estados finitos M, sean s,.. s dos estados distintos en S. La cadena de
1
entrada más cona x E § • es una sec~ncia tú rransftrtnda (o tran.sicWn) desdes, a s si
1

a) v(s,. .r) • s1, y


b) y e1• con i,(,,,y) • s, ~ lyl a,:l.rl,

Puede haber mú de una secuencia de este tipo para dos estados S;. s,r

Encontraremos una secuencia de transferencia desde el estado s 0 hasta el estado s2 para la


náquioa de e5tados finitos M dada por la tabla de estados de la tabla 6.11, donde j = IJ =
(O, 1).

-
r.,bla 6.11

o o
.. .,... ..,,

1
o
o
1
1
1

...
s,
..
s,
s,
s, .. o
o
o
1
1
,,•• s, 1
s,
s, ., s,
••
1
1
1
1

Al construir el diagrama de árbol de la figura 6. 16. partimos del e5tado s0 y encontra·


mos aquellos estados que podemos alcanzar desde So utilizando cadenas de longitud uno.
En este ca.so encontramos s 1 y s.. Luego hacemos lo mismo con s1 y s.. para encontrar.
como resultado, los estados que podemos alcanzar desde So mediante cadenas de entrada
de lon¡itud dos. Si seguimos desarrollando el árbol de i74uierda a derecha, llegamos a un
vtrtice etiquetado coo el estado deseado, s1• Cada vez que alcanzamos un vbtice etiquetado
con un estado utilizado anteriormente, terminamos esa parte del desanolJo, pues ya no pode-
mos alcanzar más estados nuevos. Dcsputs de Ucear al estado que deseamos, regresamos a.so
42 C.apítulo 6 Lenguajes: Máquinas de estados finitos

y utilizamos la tabla de estados para etiquetar las ramas, como se muestra en la figura 6.16.
Por lo tanto, para x ""- ()()(X), v(so,. x);;; ~ con oo{.fo, x) ""- 0100. (En este caso, ..tes único.)

JERCICIOS 6.3 l. Sean l = C = (O. 1}. (a)Consuuya un diagrama de estados para una máquina de estados fini·
tos que reconozca cada ocurrencia de 0000 en una cadenax E J•. (Se permite el solapamiento.) (b)
Construya un diagrama de estados para una máquina de estados finitos que reconozca cada cadem
x E ~ que termine en 0000 y que tenga una longitud de 4k. k E Z•. ( No se permite el solapamiento.)
2. Resuelva el ejercicio l para cada una de las sucesiones 0110 y 1010.
3 . Construya un diagrama de estados para una máquina d e estados finitos con ~ =O = {O, l} que
reconozca todas las cadenas del lenguaje {O, l}• {QO} U {O, l )•{11 }.
4. Para ~ = O= ( O. l} , una cadena x E g.• tiene paridad par s i contiene un mlmero par de unos.
Construya un diagrama de estados para una máquina de estados finitos que reconozca todas las
cadenas no vacías d e paridad par.
S. La tabla 6.12 define v y copara una m~uina de estados finitos M donde§ =O= {O. 1 }.
Tabla 6.12
. ..
o 1 o 1
...s, ... s, o o
... s, 1 "¡

a) Trace el diagrama de estados para M.


b) Determine la salida para las siguientes secuencias de entrada, comenzando en s0 en cada
caso:
(l)x = 111; (il)x = 1010; (iii)x = 00011.
e) Describa con palabras lo que hace la máquina M.
d) ¿Cómo se relaciona esta máquina con la de la figura 6.13'!
6. Muestre que no es posible consuuir una máquina de estados finitos que reconozca precisamen-
te aquellas secuencias del lenguaje A = ((i, l' I i,j E z~, i> j }. (El alfabeto de A, en este caso,
es :!:= (0.1}.)
7. Para cada una de las máquinas de la tabla 6.13, determine los estados transitorios, los estados
de sumidero, las submáquinas (donde § 1 = {O. 1}) y las submáquinas fuertemente conexas
(donde § , = (0. 1 }).
Tabla6. U
. - . .. . ..
o 1 o 1 o 1 o 1 o 1 o 1
,. ...s, ,, ,,s,
,, ,. ,,s,
So o o
o 1 ,,... ...... ,, o 1
s, 1 o o
1
1
1
s, ,, ,, o o
So
s, s, s, o o ,, s, s, 1 1
s, s, s, 1 o s, ...s, ,.s. o o s, s, ,. o o
,. ,. ,. ,.,, s, 1 o
s, s, s,
1
o
1
1
s, 1 1
s, ,.s, 1 o
... s, s, o o
(a) (b) (e)
6.4 Resumen re?aSO histórico 343

8. Determine u.na $CCUCDCÍl de transferencia del estado s2 al estado s, en la máquina de estados


finitos (e) del ejercicio 7. ¿Es esta secuencia tinic:a1

6.4
Resumen y repaso histórico

En este capítulo vimos una introducción a la teoría de lenguajes y a una estructura discreta
llamada m4qu.ina ch estadosfinit<>s. Con nuestro désarrollo anterior de la teoña elemental
de los conjuntos y las funciones finitas, pudimos conjuntar algunas nociones abstractas y
modelar dispositivos digitales como los reconocedores y retardadores de secuencias. Un
tralamiento similar de este material aparece e n el capítulo I de L. Dornhoff y F. Hobn [3),
así como en el capítulo 2 de D. F. Stanat y D. F. McAllister (14).
La máquina de estados finitos que hemos desarrollado se basa e n el modelo planteado
en 1955 por O . H. Mcaly en [11). por lo que se conoce como la "máquina de Mealy". El
modelo se basa en conceptos anteriores que aparecen en la obra de D. A. Huffman [8) y E.
F. Moore [12]. Para profundizar en el estudio de los primeros trabajos relacionados con
diferentes aspectos y aplicaciones de la máquina de estados finitos. consulte el material
editado por E. F. Mocn [13]. Enloscapítulos9al 15deZ. Kohavi [9) puedcenconlnlrsemás
información acerca de la síntesis actual de taJes máquinas y la.s consideraciones ~latívas a
ellas en hardware, junto con un análisis amplio de muchas ideas relacionadas con el tema.
Para más detalles acerca de los lenguajes y su relación con las máquinas de estados
finitos, hay que analizar el módulo UMAP de Williarn J. Barnier [l], los capítulos 7 al 10
de]. L. Ocrsting [4] y los capítulos 7 y 8 de A. Gil! [5]. Una amplia cobertura de estos
temas (y otros relacionados con ellos) aparece en los textos de J. G. Brookshcar [2], J. E.
Hopcroft y 1. D. Ullman [7], H. R. Lewis y C. H. Papadimitriou [10], y D. Wood [15].
Uno podría sorprenderse al saber que las ideas b1sicas de la teoría de autómaw se
desarrollaron para resolver cuestiones mas bien teóricas de los fundamentos de las mate•
máticas, como lo estableció en 1900 el matemático alemán David Hilbcrt ( 1862-1943).
En 1935, e l matemático y lógico ingl~s Alan Mathison Turing (1912-1954) se interesó en

David Hilbert (1862-1943)


Alan Mathison Turing (19 12-1954)
Cortda de TheGra~Coll«tion. Nueva 'it)r1(

el problema de decisión de Hilbert, que preguntaba si podría haber un método general


aplicable a cualquier enunciado para determinar si éste era verdadero. El enfoque de Turing
para la solución de este problema lo llevó a desarrollar lo que ahora se conoce como la
máquina de Turing, el modelo más general de una máquina de cómputo. Con este modelo
pudo establecer resultados teóricos muy profundos acerca de la forma en que tendrían que
funcionar los computadores, antes de que fueran construidos realmente. Durante la Se-
gunda Guerra Mundial, Turing trabajó en la oficina para asuntos externos de Bletchley
Park., donde hizo un amplio uso del criptoanálisis de los mensajes nazis. Sus esfuerzos
produjeron una máquina descifradora mecánica, Enigma, elemento muy importante que
contribuyó a la caída del Tercer Reich. Después de la guerra y hasta su muerte, el interés de
Turing acerca de la capacidad de las máquinas para pensar lo llevó a desempeñar un
papel preponderante en el desarrollo de los computadores reales (no sólo teóricos). Para
más detalles acerca de Ja vida de este importante académico, consúltese la bibliografía de
A. Hodges [6].

BIBLIOGRAFÍA

1. Bamier, William J.• "Finite-State Machines as R~og.nizers.. (Módulo UMAP67 I ), TM UMAP


Joumal 1, núm. 3 (1986), págs. 209-232.
2. Brookshear, J. Glenn, TheoryofCompUlalion: Fonnal languages,Automata, and Complaity.
Reading, Mass., Benjamin/Cummings, 1989.
3. Do[Jlhoff, Lany L y FranzE. Hohn,Applied ModernAJgebra, Nueva York, Macmillan, 1973.
4. Gcrsting, Judith L., Mathemalical S1ructuru for Compuu r Science, San Francisco, W. H_
Frceman, 1982.
5. Gill, Arthur, Applwi Algebra for the Computer Sciences, Prentice-Hall Series in Autornatic
Computation, Englewood Oiffs, N.J., Prentice-Hall, 1976.
6. Hodges, Andcew, Alan Turing: The Enigma, Nueva York, Simon and Schustcr, 1983.
7. Hopcroft. John E. y Jeffrey D. Ullman, lntroducricn to Awomara TMory, Languages and
CompUlalior,. Reading. Mass., Addison-Wesley, 1979.
Ejercicios complementarios 345

8. Huffrnan, D. A., '"Toe Synthesis of Sequential Switching Circuits", Joumal of 1M Franklin


/ns1itute 251 (Marzo de 1954), pigs. 161-190; (Abril de 1954), pigs. 275-303. Reimpreso en
Moore[l3].
9. Kohavi. Zvi, Swilching and FiniteAutomata Thtory, ~ ed.. Nueva York, Me Graw-Hill, 1978.
1O. ~wis. Hany R. y Christos H. Papadimílriou.El<m<nts ofthe TheoryofCompuratwn, Englewood
Qi.ffs. N.J.• Prentice-Hall, 1981.
11. Mealy, G . H ., ..AMethod for Synthes.izingSequential Circuits",Bell System Technicalloumal
34 (Septiembre de 1955). págs. 1045-1079.
12. Moore. E. F., "Gcdanken-experiments on Scquential Machines". Automaza Studus, .Annals of
MatMma1ical Studits. ntlm. 34, págs. 129-153, Princeton, N.J.. Princeton University Press,
1956.
13. Moore, E. F.. editor. Sequen.tial Machines. Selected Papers, Rcading, Mas.s., Addison-Weslcy,
1964.
14. Stanat. Donald F. y David F. McAllister, Discrrte Mathemalics in. Compuur Scien.ce, F.nglewood
Qiffs, NJ., Prentice-Hall, 1977.
15. Wood., Derick, Theoryo/Q»nputation, Nueva York, Wiley, 1987.

EJERCICIOS
COMPLEMENTARIOS
1,1

l. Sean l:1 = {w. x. yJ y l:z = {X. )l. z} dos alfabetos. Si


A1 ={.tfli. j e z•.j>i~ 1}.A2 = twYli,J e z·. i>J'?.
l},A, = {w'.ijtJ ,j E z•. j > i ~ I}. y A,: {z'(wzYw' I
tjE z·. i~ l .j~ 2}, determine si cada una de lassiguien-
lCS proposiciones son verdaderas o falsas.
a) A 1 es un lenguaje sobre l:1•
Fi gura 6.17
b) A1 es un lenguaje sobre 1:..z.
e) A.1 es un lenguaje sobre l:1• 6. Sea M la máquina de estados finitos que se muestra en
d) A1 es un lenguaje sobre l:r la figura 6.18.
e) A1 es un lenguaje sobre l:1 U~. a) Encuentre la tabla de estados para esta má:¡uina.
f) A 1 es un lenguaje sobre l:1 n ~. b) Explique lo que hace esta m~uina.
e) A,. es un lenguaje sobre l:1 .6 ~. e) ¿Cuántas cadenas de entradadistinwx cumplenquc
b) A1U A1 es un lenguaje sobre 1:1• llxll=8y,<s0,x):s0?¿CUántas"1Jnplcn llxll = 12'?
2,. Si A, B ~ l:", ¿se cumple que A•~ s• =>A , B?
3. Dé un ejemplo de lenguaje A sobre un alfabeto l: tal º·o
cp,c(A~••(A•)'.
4. Para un alfabeto dado l: y un conjunto de índices l.
sea B. , l:* para cada i e/. Si A C 1:• , demuestre que (a)
A~B) \;; íl,.,AB,; y (b) (íl.., B)A ¡;; f'\., B;A. [En este
caso. por ejemplo, A(íl;,¡, B) denota la concatenación de
los lenguajes A y íl,4!1 B,.)
S. SeaM la máquina de estados finitos que se muestra en
lafigura6.17. Para los estados s,.. s1 0 s; i, j S2, seao,cl
conjunto de todas las cadenas de salida no vacías que M o.o
=
put.de producir al ir del estados; al estado si. Si; = 2,j O,
por ejemplo.o,. = {0111, 001•. Figura 6.18
346 capítulo 6 Lenguajes: Máquinas de estados finitos

7. ~ = (S. § ,O,v, co) unamáquinadeestadosfinitos Tabla 6.15


con ISI = n,yseaO E§.
a) Mucsn-e que para cada cadena de entradaCXXX}.
la salida es finalmente periódica.
. .
o 1 o 1
b) ¿Cuál es el máximo ndmcro de ceros que pode-
mos dar como entrada antes de que comience la So s, s, 1 o
salida periódica? o
e) ¿Cu.il es la longitud del máximo periodo posible? ,,s, s, s,
o
1

s, ,,s, ...s, 1
1
o
8. Trace el diagrama de estados para una máquina de es-
tados finitos M= (S, §,e,, v. w), donde§, = ll = {O, 1 }, sí
paracua1quierxE J •,M poneunprimer 1 cuandorecono-- 13. Sean § = O= {O, 1} para las dos máquinas de estados
ce la subc.adena 1111 y después pone un segundo l cuando finitos M 1 y M 2 dadas en las tablas 6.16 y 6.17, respectiva-
moonocc la subcadena 0000, después de lo cual su salida mente. El estado inicial para M 1 es s,, mient:rM que sJ es d
es constante e igual a O. estado inicial para M2•
9. Sea j = O= {O, 1 }. Construya un diagrama de estados
para una máquina de estados finitos que invierta (de Oal o Tabla 6.16 Tabla 6.17
de 1 a O) los símbolos que aparecen en la 4•, en la &•, en la
tr... , posiciones de una cadena de cntradax E §~. Por
ejemplo, si s 0 es el estado inicial, entonces Ol(s~ 0000) =
•. .,, ., ..,
=
0001, ro(s~ 000111) 000011, y ro(,0 , 00000011 1) = o 1 o 1 o 1 o 1

,.,,
,,... ,,So ,,
000100101. s, 1 o s, s, 1 1
10. Para J =O= {O, l} , sea M la máquina de estados fini- o o ,. ,. 1 o
tos dada en la tabla 6.14. Si et estado inicial de M n.o es s1, ,, ,, ... o 1
encuencre una cadenadc c ntradax (de longitud núnima) tal
que v(sl' x ) = s1, para todo i = 2. 3, 4. (Por lo tanto, x lleva la
máquina Mal estado s 1 independie ntemente del estado ini- Conectamos estas m'4uinas como se muestra en la fi.
cial.) gura 6.19. Cadasímbolodcsalidadc M1 se convierte en un
símbolo de entrada para Mr Po r ejemplo, si damos Ocomo
entrada de M 1, entonces w1(so> O)= 1 y v1(s,, O) = s0 • Como
Tablo 6.14

o
. 1 o 1
.,
resultado, damo5 coroo entrada l(= W 1(s,, 0)) a M1 para
obtener ~ (sJ, 1) = 1 y v1(s1, 1) := s4 •

s,
s, ,,,. s,s, o o
o 1
s, s, s, 1 o
,. ,, ,. 1 1 Figura 6.19

Construimos una miqu.inaM : (S, §,(>, v, w)que re-


11. Muestre que no podemos construir una máquina de presenta esta conexión de M 1 y M 1 como sigue:
estados finitos que reconozca con precisión aquellas secuen- § = O= {O, l }.
cias del lenguaje A = {0'11) i.,j E z ._ i <j}. (El alfabeto para
Aes :!: = {O, l }.) S = S 1 x s1 • donde s, es el conjunto de estados
internos para M1, i = 1, 2.
12. Si § = C= {0.1 }. sea M la máquina de estados finitos
dada en la tabla 6.1 S. En este caso, s0 es el estado inicial. v:Sx .f ➔ S,donde
Sea A ~ §• el conjunto tal que ..r E A si y sólo $i el último v((s, l). X) = (v1(S, ..r), v2(1, co1(s, ..r)}), para S E $1, l E S2,
símbolo en ©(sr, x) es 1. [Puede haber más de un I en la yxE § .
cadena de salida co(sO' ..r).] Construya una máquina de esta-
ro: Sx~ ➔D, donde
dos finitos donde el último símbolo d e la cadena de salida
sea I para todo y E §• -A. Cll((s, t), x) = ro,_(t, m 1(s. x), paras E S1, t E S2, y x E §.
Ejercicios complementarios 347

a) Encuentre una tabla de estados para la máquina cribir un programa (o desarrolle un algoritmo) que simule
M. la máquina de la tabla 6.18.
b) Det=nine la cadena de salida para la cadena de entra-
da llOl. ¿En qué estado está cada una de las IM-
quihas M 1 y M2 después de proce$M esta cadena?
l.C.. Aunque el diagrama de estados parece más oonveniente
que la tabla de estados cuando trabajamos con una máqui-
Tabla 6.18

o
. 1 o 1
..
ude estados finitos M = (S, ~. CJ, v, w), si las cadenas de
cmrada son cada vez más largas y los tamaftos de S. §. y CJ ,, ,, ,, o o
,e iocrementan, la cabla de estados es dtH para simular la ,, ,, ,, o o
mquina en un computador. La fonna de bloque de la tabla ,, ,, ,, 1 1
mgiefe el uso de una matriz o una tabla de dos dimensio-
aes para almac.enar v, w. Utilice esta observación para es-
7
Relaciones:
La segunda welta

E n el capítulo S presentamos el cooccpto de relación y despuis nos concentramos en


algunos tipos especiales de relaciones llamadas funciones. Regresaremos a las relacio-
nes en este capítulo para enfatizar el estudio de las relaciones sobre un conjunto A. es
decir, subconjun1os de A X A. Dentro de la teorfa de lenguajes y las máquinas de estados
finitos del capítulo 6, encontramos muchos ejemplos de relaciones sobre un conjunto A,
donde A representa un conjunto de cadenas de un alfabeta. dado o un conjunto de estados
internos de una máquina de estados finitos. Desarrollaremos distintas propiedades de las
relaciones, junto con las formas de representar relaciones fin.itas para su uso en un compu-
tador. Los grafos dirigidos reaparecen como una segunda manera de representar tales rela-
ciones. Por último, entre las muchas posibles relaciones sobre un conjuntoA, hay dos tipos
especialmente importantes: las relaciones de equivalencia y los órdenes parciales. En par-
ticular, las relaciones de equivalencia surgen en muchas áreas de las matemáticas. Por el
momento usaremos una relación de equivalencia sobre un conjunto de estados internos en
una máquina de estados finitosM, a fin de encontrar una máquina M 1• con el menor núme-
ro de estados internos posible, que realice las tareas que Mes capaz de realiz.ar. El proce-
dimíento necesario para lograrlo se conoce como proceso de minimización.

7. 1
Repaso de re laciones:
Propiedades d e las relaciones

Comencemos recordando algunas ideas fundamentales consideradas anteriormente.

Definición 7 .1 Si A.. B son conjuntos. una rrlaci6n dt A tn B C$ cualquier subconjunto de A X B. A los


subconjuntos de A X A se les llama rtlaóones sobrr A.

a) Definimos la relaci6n9l sobre el conjuntoZ comoa9tb,o(a. b) E 9t sia s b. Este


subconjunto de Z x Z es la relación ordinaria "menor o i¡ual que" sobre el conjun-
to Z, y también puede definirse sobre Q o R. pero no sobre C.
349
:so capltulo 7 Relaciones: La segunda vuelta

b) Sean E z·. Parax.y E Z, la relación módulo n!;ft. está definida comox~y si x - y


es un ml'.iltiploden. Por ejemplo, sin= 7, encontramos que 9~2. -39l: l l, (14.0) E
91, pero 3 gJ. 7 (es decir, 3 no está relacionado con 7).
e) Para el universo e.u = { 1, 2, 3, 4, 5, 6, 7 }, consideremos el conjunto (fijo) C !;; ~
donde C ; { 1, 2, 3, 6 }. Definimos la relací6n91 sobreif (<t) comoA9lB si A fl C ;
B n C. Entonces los conjuntos {1, 2, 4. 5} y { I, 2, 5, 7} están relacionados, ya que
( I, 2, 4, 5} fl C; ( I, 2 } ; ( I , 2, 5, 7) fl C. Igualmente encontramos que X;
(4, 5}y Y; (7} también se relacionan, ya que X n C ;¡j; Y n C. Sin embargo, los
conjuntosS= ( 1, 2, 3, 4, 5) yT= ( 1, 2, 3, 6, 7) no eslán relacionados;es decir, S!JÍT,
ya ques ne; {I, 2, 3) ,' (I, 2, 3, 6}; rn c.

Sea I un alfabeto, con lenguaje A ~ I.*. Parax, y E A, definimosx~y si x es un prefijo de


y. Pode mos definfr otras relaciones sobre A reemplazando ..prefijo" con un "sufijo.. o una
..subcadena."

Consideremos una máquina de estados finitos M = (S, §,, (), v, oo).


a) Para s1, s 2 E S. definimos s 1 9l s 2 si v(s 1• x) ; s 2, para algún x E !}. La relación !11
establece el primer nivel de accesibilidad.
b) También podemos dar Ja relación del segundo nivel de accesibilidad para S. En este
caso, s 1 9J? s2 si l-(s 1• x 1 x 2)-= s2, para algunas x 1 x1 E §2. Esto puede extenderse a
niveles superiores si surge la necesidad. Para la relación general de accesibilidad.
tenemos que v(s1 • y) -= si, para algún y E §•.
e) Dadoss 1, s 2 ES, definimos la relación de /-equivalencia, que se denota cons 1 E 1 s:
y se lee "s1 es 1-equivalente a sz", cuando <ó(si, x) = ffi(s:,..x ) para todo x E J. En
consecuencia. s 1 E 1 s2 indica que si la máquinaM pane de] estado s1 o s2 , la saJidaes
la misma para cada e1emento de ~ Esta idea puede e xtenderse a los estados k-
equivalentes. donde escribimos s 1 Et s1 si ro(si, y) = Ctl(sz, y) para todo y E SJ. En
este caso, obtenemos la misma cadena de salida para cuaJquier cade na de entrada
en 9t si empezamos en s 1 o s1•
Si dos estados son k-cquiva1entes para todo k. E Z\ entonces decimos que son
equivalentes. Analizaremos esta idea con más detalle en secciones posteriores de
este capítulo.

Ahora analizaremos algunas de las propiedades que puede satisfacer una re lación.

~nición 7.2 Una relación g¡ sobre un conjunto A es reflexiva si para todo x E A (x, x) E Sit.

Decir que la relación 9l: e s reflexiva significa simplemente que cada elemento x de A
se relaciona consigo mismo. Todas las relaciones de los ejemplos 7 .1 y 7 .2 son reflexivas.
La relación general de accesibilidad del ejemplo 7.3(b) y todas las relaciones mencionadas
7.1 Repa$0 de relaciones: Propiedades de las relaciones 351

en la pane (c) de ese ejemplo también son reflexivas. [¿Qué tic:nen de malo las relaciones
del primer y segundo niveles de accesibilidad dadas en las partes (a) y (b) del ejemplo 7.3?]

Para A = {l. 2, 3, 4), una relación91 ~ A x A es reflexiva si y sólo si 912 {(l. 1), (2, 2),
(3, 3), (4, 4)}. En consecuencia, 91, = ((1. 1), (2, 2), (3, 3)) no es una relación reflexiva
sobre A, mientras que9l2 = {(X.y) lx.
y EA, .r~y } es reflexiva sobre A.

Dado un conjunto finito A con IA 1 = n, tenemos que I A X A 1 = n2, por lo que hay 2'
relaciones sobre A. ¿Cuántas de ellas son reflexivas?
Si A = {a1,a2 , . , , a.,} unarelación9i sobre A es reflexiva si y sólo si {(a;,a,)l t :Si
s n} ~9't. Si consideramos losdemásn2 - n pares ordenados deA X A [los de la fonna(a;,
a;), donde i -1:-j para 1 s i,j s n). conforme construimos una relación reflexiva 91: sobre A
podemos incluir o excluir cada uno de estos pares ordenados, así, por la regla del produc-
to, existen 2<.,.i...,., relaciones reflexivas sobre A .

Definic ión 7 .3 La relación 9l sobre el conjunto A es simi.trica si (x, y) E 9l ~y. x ) E 9'l: para todosx, y EA.

E!lllllliJI Si A= {l. 2, 3}, tenemos:


a) 91, = {(l. 2), (2, 1), ( 1, 3), (3. 1)) es una relación simé1rica (pero no reflexiva) sobre
A;
b) 91, = ((1. I). (2 , 2). (3, 3), (2, 3)] es una relación reflexiva (pero no simélrica) sobre
A;
e) :'ll, = {(l. 1), (2, 2), (3. 3)} y 911, = ( ( 1, 1), (2, 2), (3, 3), (2. 3),(3, 2)} son dos
relaciones sobre A que son re flexivas y simétricas; y
d ) 8R, = {(l. 1), (2, 3),(3, 3)} es una relación sobre A que no es simétrica ni reflexiva.

Para contarlas relaciones simétricassobreA = {a1, a 2, ••• ,a.) escribimosA x A comoA,


UA 2, donde A,= {(a;,a,) I I Si S n} yA , = {a,. a,} l t S i,j S n,y i ¡I, j], por lo que cada
(>M ordenado de A x A está exactamente en uno de los conjuntos A., A 2 • Para A?, \ A1 1 ~
IA x A I- IA1 1= n: - n = n(n - 1), un entero par. Et conjunto A 2 contiene (l/2)(n?-n)
subconjuntos S¡¡ de la forma {(a;. a¡), (a¡, a;)} donde 1 s i < j s n. Para construir una
relación simétricasit sobre A, para cada par ordenado deA 1 tenemos nuestra elección usual
de exclusión o inclusión. Para cada uno de tos (l/2}(n 2 - n}subconjuntos S;¡ (1 Si <j :S n)
de A: tenemos las mismas dos opciones. Así, por la regla del producto, existen zt • 211 n»-~
• 1 ~ 2nnlúo?.+..) relaciones simétricas sobre A.
Para contar las relaciones sobre A que son reflexivas y simétricas, tenemos sólo una
opción para cada par ordenado de A1, por Jo que tenemos 21 1n l(_,_,.., relaciones sobre A que
son reflexivas y simétricas.
152 capitulo 7 Relaciones: La segunda welta

>efinición 7.4 Para un conjunto A, una relación 9i sobre A es transitiva si para todos x. y, z E A, (x, y).
(y. z) E 91: ~ (x, z) E 9l (Así, si x "está relacionado con" y, y y «está relacionado con" l.
queremos que x ..esté relacionado con" z, donde)' tiene el papel de ..intermediario".)

Todas las relaciones de los ejemplos 7.1 y 7.2 son transitivas, al igual que las del ejemplo
7.3(c).

Definimos la relación g:¡ sobre el conjunto z-- como a:3i b si a divide (e xactamente) a b~ es
decir, b = capara algún e E z·. Ahora bien, si xg:¡y yy9tz, ¿tenemosx9tz? Sabemos que
x9i y ⇒ y = sx para algún s E Z ... y y!Ji z ⇒ z = ry donde 1 E z+. En consecuencia. z = ty =
t(sx) = (ts)x para rs E Z+, por lo que obtenemos xell:z y9les transitiva. Ad'!más, ~ es
reflexiva, pero no simétrica, ya que, por ejemplo, 2 91: 6 pero f:i.Ít.2.

Consideremos la relación g:¡ sobre el conjunto Z donde definimos a ~b si ab:? O. Para


cualquierenterox tenemos que .a= i2 :? O, por lo que x9?x y 91: es reflexiva. También, si
x. y E Z y x~y. entonces
rlty ~xy .eO~yx .eO~_ylt!x,
por Jo que la relación 91 también es simétrica. Sin embargo, aquí encontramos que (3, O),
(O, - 7) E 91, ya que (3XO) 2' O y (0)(- 7) 2' O, pero (3, -7) ~ 9l ya q ue (3)(- 7) <O.En
consecuencia, esta relación no es transitiva.

Si A ; ( l. 2, 3, 4}. entonces 911 ; ( (1, 1). (2, 3). (3, 4). (2, 4)} es una relación tránsitiva
sobn: A. mientras que9l2 ; ( (l. 3), (3, 2)} no es transitiva, ya que (1, 3), (3. 2) E 91, pero
( l.2)~91,.

En este momento, es probable que el lector esté listo para contar el número de relaci~
ncs transitivas sobre un conjunto finito. Pero esto no es posible ya que, a diferencia de los
casos de las propiedades reflexivas y simétricas. no existe una fónnula general conocida
para el número total de relaciones transitivas sobre un conjunto finito. Sin embargo, en
una sección posterior de este capítulo, tendremos los conceptos necesarios para contar las
relaciones~ sobre un conjunto finito que son (al mismo tiempo) reflexivas, simétricas y
transitivas.
Examinemos una (iltima propiedad de las relaciones.

~finición 7.5 Dada una relación~ sobre un conjunto A, ~ es antisimitrica si para todos a. b E A,
(a91:b y b~a) :::.:>a= b. (En este caso, la única forma en que podríamos tener a "relacio-
nado con" b y b "relacionado con" a es cuando a y b son el mismo eleme nto de A.)
7 .1 Repaso de rela<iones: Propiedades de las relaciones 353

~ .11· . Para un universo dado"U. definimos la relación!!ll sobre~('U)como (A. 8 ) E 9i si A S: B,


para A. B ¡;; "U. As/, 9i es la relación de subconjunto del capítulo 3 y si A91 B y B!!ll A.
entonces A ~ By B ~ A.. de lo que obtenemos A. ,,,,_ B. En consecuencia. esta relación es
antisi~trica. asf como reflexiva y transitiva. pero no sim~trica

Antes de comenzar a pensar que ..no simétrico" es sinónimo de "antisimétrico", consi-


deremos lo siguiente.

ParaA =( =
!, 2, 3}, larelaciónS'lsobreA dada por91 ( (!, 2), (2, 1), (2, 3)} noes simtlrica,
pues (3, 2) ,t 91; y tampoco es antisimélrica, ya que (!, 2) (2, 1) E 91 pero 1 # 2. La
relación 91, = ((l. !), (2, 2)} es simtlrica y antisimtlrica.
¿Cuántas relaciones sobre A son antisimétricas? Si escribimos
A X A - {(!, !), (2, 2), (3,3)} U {(1,2),(2, !), (!, 3), (3, !), (2, 3), (3, 2)},
debemos hacer dos observaciones mienuas tratamos de construir una relación antisiim:trica
91 sobre A.
1) Podemos incluiroexcluircualqu.ierelemento (x, X, E A x A, sin preocupamos por
el hecho de que 9l sea o no antisimétrica.
2) Para un elemento de la forma (X. y),x ;, y, debemos analizar (x. y) y(), x) y observar
que, para que$/: sea antisimétrica, tenemos tres opciones: (a) colocar (x, y) en91:; (b)
colocar (y, x) en 91; o (c) no colocar (x. y) ni (y, x) en 91. (¿Qut ocurre si colocarnos
(x. y) y (y. x) en 91?)
Así, por la rcila del producto. el número de relaciones antisimétricas sobre A es (23)(3'} =
(2 3XJº'-l112). Si I A 1 =n > O, entonces existen (2~X3{.a~Y2) relaciones antisimétricas sobreA.

Para nuestro siguiente ejemplo regresaremos al concep10 de dominio de una función,


que definimos por primera vez en la sección 5.7.

~ 7.13 Sea 'f/ el conjunto de todas las funciones con dominio z· y codominio R; es decir, f!l =
lflf: z• ➔ R ). Paraf. g E ',F, definimos la relación9! sobre',F como/9!g si/es dominada
por g (o/ E O(g)). Entonces 9t es reflexiva y transitiva.
Si/. g : z • ➔ R se definen como/(•) = n y g(n) = n + 5, entonces/ 9i g y g91 /pero/
#-g. por lo que 9't no ~s an1isimltrica. Además, si h: z· ➔ R está dada por h(n) ; rr,
entonces(/. h), (g. h) E 91 pero ni (h, /) ni (h. g) están en 9!. En consecuencia, la relación
.9t tamp<Ko u simltrica.

En este momento hemos vis10 las cuatro propiedades pñncipales que surgen en el estu•
dio de las relaciones.Antes de temlinar esta sección, definiremos dos conceptos más, cada
uno de los cuales utiliza tres de estas cuatro propiedades.

Definición 7 .6 Una relación 9t sobre un conjunto A es un ordtn parcial, o una nlación dt orden parcial,
si 9i: es reflexiva, antisimétrica y transitiva.
Capítulo 7 RelaciOnes: La :,egunda vuelta

La relación del ejemplo 7.l(a) es un orden parcial, pero la relación de la panc (b) de ese
ejemplo no lo es, ya que no es antisimétrica. Todas las relaciones del ejemplo 7.2 son
órdenes parciales, al igual que la relación de subconjunto del ejemplo 7.11.

Nuestro siguiente ejemplo nos permite relacionar esta nueva idea de orden parcial con
resultados que estudiamos en los capítulos 1 y 4.

Comenzamos con el conjunto A= { J, 2, 3, 4, 6, 12}, el conjunto de divisores enteros


positivos de 12, y definimos la relación 9l sobre A como x 91:y si x divide (exactamente) a
y. Como en el ejemplo 7.8, encontrarnos que 91: es reflexiva y transitiva.Además, six, y E
A son tale s que x ~ y y y 91: x, entonces
x9ly ⇒ y= ax, para algún a E z•, y
yell X=> X= by, para algún b E z·.
En consecuencia. se sigue que y= ax= a(by) = (ab)y, y como y f. O, tenemosab = l. Como
a, b E Z", ab = 1 ~a= b;:;1. por lo que y;::; x y9l: es antisimétrica y define un orden parcia]
para el conjunto A.
Supongamos ahora que queremos contar el número de pares ordenados que apartcen
c:n esta relación 9t Podemos simplemente enumerar los pares ordenados de A x A que
forman91::

~ ={( 1, 1), (1,2), (1,3), (1 , 4), (1, 6), (1, 12), (2, 2), (2, 4), (2,6),
(2, 12), (3, 3), (3, 6), (3, U), (4, 4), (4, 12), (6,6), (6, 12), (12, 12))
De esta manera, vemos que hay 18 pares orden3dos en la relación. Pero si quisiéramos
analizar el mismo tipo de orden parcial para el conjunto de divisores enteros positivos de
1800, definitivamente nos desanimaría el método de simplemente enumerar todos los pa-
res ordenados. Así que analicemos la relación 9i con más detaJle. Por el teorema funda-
mental de la aritmética, podemos escribir 12 :;; 22 • 3; vemos entonces que si (c. d) E Q,
entonces

d = 2'·3".
donde m, n, p, q EN con O :s m :S p :S 2 y O :S n :S q ::s l.
Si tenemos en cuenta el hecho de que O :s m :s p s 2, ve mos que cada opción para m, p
es simplemente una selección de tama1ío 2 de un conjunto de tamaño 3 (el conjunto {O, 1,
2}) dende se permiten las repeticiones. (En cualquiera de este tipo de selecciones, si cx.iste
un entero no negativo más pequeño, entonces se le asigna a m.) En el capítulo 1 aprendimos
quedicluselección puede hacersede {3 " 1j i- :;;W=6 formas. Y.de manera similar, podemos
seleccionar n y q de p•¡- 1j :;; rn :;;
3 formas. Así. por la regla del pnxlucto. deben existir
(6X3) = 18 pares ordenados en~. como ya vimos antes cuando las enumeramos.
Supongamos ahora que analizamos una situación similar, el conjunto de divisores e nte-
ros positivos de 1800:;; V• 32 • 52 • Aquí trabajamos con (4)(3)(3) = 36 divisores y un par
ordenado usual para este orden parcial (dado por la división) se ve como (2' • 3$ • 5:, 2"• l "
• 5"), donde r. s, t, u, u, w E N, con O ::s r :s u :s 3, O :s s :S u :S 2 y O :s t s w :s 2. Así,
el número de pares ordenados de la relación es
7. 1 Repaso de r~aciones: Propiedades de las retaciones 355

y definitivamente no queremos enumerar todos los pares ordenados de la relación para


obtener este resullado.
En general, paran E z• conn > 1. usamos el teorema fundamental de la aritmética para
escribir

donde k E z·. y
p 1 < p2 < P1 < · · · < p, p, es primo. e, E z·
para cada l i s s
k. Entonces
n tiene íl~1(e, + l) divisores enteros positivos. Y cuando consideramos el mismo tipo de
orden parcial para este conjunto (de divisores enteros positivos den). \'emos que el núme-
ro de pares ordenados en la relación es

Ii(<•;+ 1)2+ 2-1) = Ii(•; +2 2).


i•l i•I

Definición 7. 7 Una relaci6n de equivalencia 9'l sobre un conjunto A es una relación que es reflexiva.
simttrica y tr2nsitiva .

a) La relación del ejemplo 7. l (b) y todas las del ejemplo 7.3(c) son relaciones de
equivalencia.
b) SiA = ll,2,3},cn1onces

9l, • {(1, 1), (2, 2), (3, 3)),


9l¡ ª {(1, 1), (2, 2), (2, 3), (3, 2), (3, 3)},
9l, = {(1, 1), (1, 3), (2, 2), (3, 1), (3, 3)), y
91, = {(l , 1), (1, 2), (1 , 3),(2, 1), (2, 2), (2, 3), (3, 1),(3, 2), (3, 3)} = A X A
son relaciones de equivalencia sobre A .
e) Para cualquier conjunto A. A x A es una relación de equivalencia sobre A. y si A =
{a 1,a:.- .. , a.}. entonces la relación de igualdad Si= {(a,.a,)l 1 :Si :s: 11 } es la
mínima relación de equivalencia sobre A .
d) Sean A= {l. 2,3.4,5,6. 7}. 8 = {x.y.:} y/: A ➔ Bla función sobre
f = {(1,x), (2, z ), (3, x ), (4,y), (5,z), (6,y), (7, x)}.
Definimos la relación 9? sobre A como af) b si/(a) = /(b). Entonces. por eje m-
plo, vemos que 1g¡ l. 1g¡3_ 2g¡5, 3g¡ 1 y 4gi6.
Para cualquier a E A.j{a) :;:: /(a) ya que/es una función: entonces. aJia y 5? es
reflexiva. Supongamos ahora que a, b E A y a 5t b. Entonces a 9l b ⇒/(a) :;:: ft.b) ⇒
/(b) •/(a) ⇒ b9. a. por lo que~ es simltrica. Por último. si a. b, e E A y ar,t b. b
::Jlc, en1onccs/(a) = /(b) y /(b) = /(e). En consecucncia./(a) =/(e) y (a:Jlb A b9l
e) ⇒ a9tc. Así, 9t es transitiva. ComoSt es reflexiva. simétrica y uansitiva. es una
relación de equivalencia.
En cSlc caso. f'/1 ~ ( ( l. 1). ( 1. 3). ( l. 7), (2. 2). (2. 5), (3. 1). (3. 3). (3. 7). (4. 4),
(4, 6 ). (5, 2), (5. 5). (6. 4), (6, 6) , (7, 1). (7. 3) . (7. 7)}.
56 Capltulo 7 Relaciones: La segunda vuelta

e) Si gj: es una relación sobre un conjunto A, entonces !!11: es al mismo tie mpo una
relación de equivalencia y un orden parcial e n A si y sólo si g¡ es una relación de
igualdad sobre A.

JERCICIOS 7.1 1. Si A = { l, 2, 3, 4}, dé un ejemplo de una relación 9t sobre A que sea


a ) reflexiva y simétrica. pero no transitiva
b) reflexiva y transiliva, pero no simétrica
e) simétrica y transitiva. pero no reflexiva
2. Para la relación (b} del ejemplo 7.1. determine cinco valores de x para los cuaJes (.r. 5) E 9t.
3. Para la relac1ón97t del ejemplo 7. 13. sea/: z·➔ R dondeftn) = n.
a ) Determine tres elementosfi,.h,/3 E ';1 tales qucJi9i:fy / 91:Jjpara todo I Si S 3.
b) Encuentre tres e lementos g 1• 8?, g 3 E CJ tales que g,9'1/perofli:g, para todo J Si S 3.
4 . a) Formule de nuevo las definiciones de las propiedade s reflexiva, simétrica. transitiva y
antisimétrica de una relación 9i (sobre un conjunto A), usando cuantificadores.
b) Use los res uliados de la parte (a) para especificar cuándo una relación ~ (sobre un con-
junto A) (i) no es reflexiva: {ii) no es simétrica; (iii) no es transitiva; y (iv) no es
anti simétrica.
5. Para cada una de las siguientes relaciones. determine si 1a·retación es reflexiva , simétrica.
antisimétrica o transitiva.
a ) ;Ji C z· z·.
x al
dondea~b si b (se lee ..a divide a b-, como se definió en la sección
4.3).
b ) ~ es la relación sobre Z tal quea9i':b si alb.
e) Para un unh•erso dado 'l.l y un subconjunto fijo C de '\l. d efin imos 9'I sobre 9'('\l )
como sigue: Para cualesquiera A. B C "1 tene mos A 9tB si A n C B n C. =
d ) En el conjunto A de todas las rectas de R 2, definimos la relación 9'I: para dos rectas' " t1
como f1 9i e1 si f1 es perpendicular a f1 •
e) 9'1 es la relación sobre Z tal que x5lly si x + y es par (impar).
f) 91 es la relación sobre Z tal que xsity si x - y e s par (impar).
g) 9t es la relación sobre z·
tal que .x9'1: y si mcd(a. b) = l; es decir. si a y b son primos
relativos.
h) Sea Tel conjunto de todos los triángulos de R 2• Definimos9't sobre Tcomo t 19'tl? si t1
y lz tienen un ángulo con la misma medida.
i) 91 es la relación sobre Z x Z tal que (a. b):,t (c. d) si a s c. [Nota: 9't C ( Z x Z) x
( Z X Z).)
j ) 8/t es la relación sobre z· dada por x~ y si ,x' + / es par.
6. ¿CuáJes de las relaciones del ejercicio 5 son órdenes parciales? ¿ Cuáles son relaciones de
equivalencia?
7. a ) Sean 9t 1• 91: relaciones sobre un conjunto A. Demuestre o pruebe que es falso que 311, 9.!
reflexivas ~ 91 1 n 9ii reflexiva.
b ) Resuelva la parte (a). sus1i1uyendo cada ocurrencia de ..reflexiva- por (i) simétrica; (ii)
antisimélrica y (iii) 1ransi1iva.
8 . Resuelva el ejercicio 7 reemplazando cada ocurre ncia den por U.
9. Para cada una de las siguie ntes proposiciones acerca de las relaciones sobre un conjumo A.
1A 1 = n, determine si la proposición e s verdadera o falsa. Si es falsa, dé un co ntraeje mplo.
a) Si 91: es una relación reflexiva sobre A, entonces 1 91: 1 2::: n.
b ) Si ~ es una relación sobre A y [ St 1 2::: n, entonces 91: es reflei:.iva.
e) Si 9'(1, 9'!: son relaciones sobre A y El!: ~ 9t1, entonces9i 1 re flexiva (simétrica, anti simétrica.
transitiva) ~ !11:i re flexiva (simétrica, antisimétrica, transitiva).
7 .2 Reconocimiento por computador: Matrices cero-uno y rafos dirigidos 3 S7

d) Si 3t1, Sll:1 son relaciones sobre A y9t2 :2 91:i, cntonces9ti reflexiva (silmtrica. antisimttrica,
transitiva) ⇒ 911 rcflexi~•a (simi:trica. antisi~ca. transitiva).
t ) Si 9t C$ una relación de equivalencia sobre A, entonces n :si l9t IS n 2•
10. Si A= (w. x, y, z). determine el número de relaciones sobre A que son (a) reflexivas; {b)
simétricas: (e) reflexivas y simttricas: (d) reflexivas y contienen a (x. y): (e) simfflicas y con•
tienen a (x. y); (0 antisimitricas: (&) a n t i s i ~ y contienen a (.r. y); (h) sirnwicas y
antisimwicas; e (i) reflexivas. si~cas y antisimwicas.
11. Sea n E z· con n > 1 y sea A el conjunto de los divisores enteros positivos de n. Defina la
relación 9t sobre A como x9ty si x divide (exactamente) a y. Detcnninc la c.anlidad de pares
o«len>dos que hay en la «ladón 91 cuando n C$ (a) 10; (b) 20; (e) 40: (d) 200; (<) 210; y (f}
13860.
12. Suponga que p,. p 1• p, son primos distintos y que n, k e Z♦• con pf~p;. Tome A como el
conjunto de los divisores enteros positivos den y defina la rclación9t sobre A cornox!ly si x
divide (euctamcnte} ay. Si cxiMen 5880 pares ordenados enSt. determine k y IA1.
13. ¿Qut tiene de incorrecto el siguiente argumento?
Sea A un conjunto y g¡ una !"Clación sobre A. Si 9t es simwic.a y transitiva. entonces !tes
reflexiva.
Demostración: Sea (x. )') E 91:. Por la propiedad de simetría. (y, x) E 9t. En1onccs, como
(x. y). {y, x) E f1t. se si¡ue de la propiedad tran$itiva que (x. x) E 9t En consecuencia, 91 e$
reflexiva.
14. Sea A un conjunto tal que· IA1= n y sea 9t una relación sobre A anrisimttrica. ¿Cuál cs el
múimo valor de l9t I? ¿Cuántas relaciones anlisi~ricas pueden tener ese tamafto?
15. Sea A un conjunto tal que !A 1 = n y sea91 una relación de equivalencia sobreA tal que I.St: I =
r. ¿Por qul r - n siempre es par?

16. Una relación~ sobre un conjun10.4 es ir~jlexivo si para todo o E A. (o. a)E 9t.
a) Dé un ejemplo de una relación ~ sobre Z 1al que ~ sea irreflexiva y transitiva pero no
simétrica.
b) Sea911 una relación no vacía sobre un conjunto A. Demuestre que s1 ~( satisface dos cuales-
quiera de las siguientes propiedades (irreflexiva, siffll!trica y transitiva) entonces no puede
satisfacer la tercera.
e) Si IA1• n =:! l. ¡,cutnt:as relaciones diferentes sobre A son irreflexivas? ¿Cuintas no son
reflexivas ni irreflexivas?

7.2
Reconocimiento por computador:
Matrices cero-uno y grafos
dirigidos

Puesto que nuestro intcr~s se centra en las relaciones sobre conjuntos finitos, dirigiremos
nuestra atención a las fonnas de representarlas de modo que podamos verificar fácilmente
las propiedades de la sección 7 .1. Por esta razón, desarrollaremos ahora las herramientas
necesarias: la composición de relaciones, las matrices cero-uno y los grafos dirigidos.

De manera similar a la composición de func iones. las relaciones pueden combinarse en


las siguientes circunstancias.
58 Capitulo 7 Relaóooes, La segunda vuelta

~finición 7 .8 e e
Si A, B Y e son conjuntos Y~ 1 A X B Y2li2 B X e, entonces la relación compuesta g¡I
of;l2 es una relación de A en Cdefinida comoSJl:1 o~2 ;;;; { (x. z:) lx E A, z E Cy e xiste y E
B tal que (x. y) E 91,. (y. z) E 91, }.

¡Cuidado! La composición de dos relaciones se escribe en orden inverso al de la com-


posición de funciones. En breve veremos por qu¿.

Sean A; [ l. 2. 3, 4 }, B ; [w. x. y. z) y C ; [5, 6. 7}. Consideremos 91, ; {(I, x), (2,x),


(3, y), (3, z)}, una relación de A e n By 91, ; {(w, 5), (x, 6) ) una relación de Ben C.
Entonces91,•91,; {(I, 6), (2, 6)} es una relación de A en C. Sie!i, ; ((w, 5), (w, 6)} esotra
relación de B en C, entonces 91: 1o8i1 = 0.

Sean A el conjunto de los empleados de un centro de cálculo. B un conjunto de lenguajes


de programación de alto nive l y Cuna lista de proyectos f P,,Pl, ... • p. } de los cuales los
administradores deben asignar trabajo a las personas de A. Consideremos 911 ~ A x B
donde un par ordenado de la forma (L. Pérez, Pascal) indica que el empleado L. Pérez
utiliza con eficiencia Pascal (y posiblemente otros lenguajes de programación). La rela-
ción 9l2 ~ B x C consta de los pares orde nados del tipo (Pascal, p1 ), lo que indica que el
lenguaje Pasea] se considera e sencial para las personas que trabajan en el proyecto p 1•
En la relación compuesta91:1 o,0i:1 encontramos e] par (L. Pérez, p 1) . Si no existe otro par
en 91:1 que tenga a p1 como segunda componente, sabemos entonces que si L. Pérez e s
asignado a Pi esto se de be exclusivamente a su uso eficiente de Pasea]. (En este caso,
9i1 o 9i1 se utiliza para establecer una concordancia e ntre los empleados y los proyectos
con base en el conocimiento de los lenguajes específicos de programación por parte del
empleado.)

El siguie nte resultado, parecido a la propiedad asociativa de la composición de funcio-


nes, es válido para las relaciones.

EOREMA 7.1 Sean A, B, e, Dconjuntos y St1~ A X B,Sl:1 ~ B X C y 911!; e X D. Entonces 91:1 0 (~10 81:3)
= (.9ft10912) 0 9A3.
Demostración: Como 911 o (9A1 o !?AJ) y (~1 o ~ 1 ) o 9?3 son relaciones de A a D, e xiste una
razón para pensar que son iguales. Si (a, d) E ~ 1 o (9i:7 o 9l:1) , entonces existe un elemento
b E B tal que (a, b) E 91, y ( b, d ) E (91,•91,). Además, (b, d ) E (91,o S'I,) => (b, e) E 91,
y(c, d ) E9J: 3 paraalgúnc E C.Enconces, (a, b) E 91 1 y (b, e) E S'l:2 ::::;.(a , e) E 9l: 1 o912 • Por
último, (a, e) E 91, • % y (e, d) E 91,=>(a, d ) E (91, 091,) o 5'!, y e!i, o (91,•91, ) !:;; (e!I, • 811,)
o 91: 3• La inclusión opuesta se obtiene mediante un razonamiento similar.

Como re sultado de este teorema. no existe ambigüedad alguna c uando escribimos


!¾1 o 9! 2 ° 91:3 para cualesquiera de las relaciones del teorema 7 .1. Además, ahora podemos
definir las potencias de una r elación 9l sobre un conj unto.
7.2 Reconocimiento por computador: Matrices cero.uno y arafos dirigidos 359

Dtfinición 7.9 Dado un conjunto A y una relación 9t sobre A, definimos las potencias de ~ en forma
recursiva como (a)91: 1 = 91:; y (b) paran E z•. ~•1 ::;; ~ o9't".

Observemos que paran E z•, ~es una relación sobre A .

Si A ; { 1, 2, 3, 4) y91; {(l, 2), (1, 3), (2, 4), (3, 2)). e ntonces 912 ; {(l. 4), ( 1, 2), (3, 4)),
91' ; {(1, 4)} y paran a: 4, 91"; 0.

Conforme el conjuntoA y la relaci6n91: crecen, los cálculos parecidos a los del ejemplo
7.19 se vuelven tediosos. Para evitarlo. la herramienta que necesitamos es el computador.
una vez que enconcremos la forma de indicar a la máquina la información relativa al con-
j unto A y ta relación 91: sobre A.

~nidón 7.10 Una ma1ri1. etro-uno m x n E = (e,).• . es una disposición rectangular de números en m
filas y n columnas, donde cada ' •• para 1 S i s m y 1 s j :S n, denota la entrada de la i-
lsima fila y la j-ésima columna de E y cada una de dichas entradas es O o 1. [fambiln
podemos escribir matriz (0,1) para este tipo de matriz.}

La matriz

E; 0
1 O O l]
[l Ol OO Ol
es una matriz (O, 1) de 3 x 4, donde, pOr ejemplo, t 11 -= 1, tu :: O y e,1 = 1.

Al trabajar con estas matrices usamos las operaciones comunes de suma y multiplíca-
ción de matrices suponiendo que l + 1 :: 1. (Por lo tanto, Ja suma es booleana.)

Consideremos los conjuntos A . 8 y C y las relaciones 911, 9ti del ejemplo 7.17. Si el orden
de los elementos de cada uno de estos conjuntos se fija como e n ese ejemplo, definimos las
mairi.ces dt rdaciQn de 9't1o 911 como sigue:

(1)
(2) O
M(~,) • (3) O
(4)
(w)


O
(x)

O
1
l

o o
( y)
O
O
1
M(91
l
)J;/
(y) 0
(z) O
¡~ .
(5) (6)

~
0
O
360 Capitulo 7 Relac~: La segunda vuelta

Al construir M('3t.1) . trabajamos con una relación de A en B, de modo que los elementos
de A se usan para señalar las filas de M(f/l. 1) y los elementos de B indican las columnas. Así,
por ejemplo, para indicar el hecho de que (2, x ) E ~ 1• colocamos un I en la fila marcada
con (2) y la columna marcada con (x). Cada O de esta matriz indica un par ordenado e n
A x B que se omite en ~ ,- Por ejemplo, como (3, w) ~ 9i1, hay un O en la entrada de la fila
(3) y la columna (w) de la macriz M(911). El mismo proceso se usa para obtener M(PR.2) .
Si multiplicamos las matrices¡, vemos que

M(g¡1)·M(g¡2)=[~ ¡ ~ ~1[~ ! ¡
0000000
~I=[~
000
~l =M(g¡,.(it,),

y en gene ra] tenemos que si 9i1 es una relación de A en By 91:2 es una relación de 8 en C,
entonces M(9'. 1) • M(f/i.2 ) • M(9. 1 o eA:2) . fu decir, el producto de las matrices de relación para
911, 9h, en ese orden , es igual a la matriz de relación de la relación compuestae7t1 0912• (Ésta
es la razón por la que la composición de dos relaciones se escribió e n el orden dado en la
definición 7.8.)

En los ejercicios 9 y 10 (que aparecen al final de esta sección) se pedirá al lector que
demuestre el resultado general del eje mplo 7.21, junto con algunos resultados de nuestro
siguiente ejemplo. en el que se: muestran más propiedades de las matrices de relación.

Sean A= { 1, 2, 3, 4} y 9l = {(I, 2), (1 , 3), (2, 4), (3, 2) }, como en el ejemplo 7.19. Si
conservamos fijo e l orden de los elementos de A, definimos la matriz d~ relación para 9t
como sigue: M(~) es ia matriz (O, 1) 4 x 4 cuyos elementos m ,¡, para 1 ::S i, j ::S 4, están
dados por
(i,j)E(it,
m,;=1~: sí
en caso contrario
En este caso tenemos que

0O O
1101
O 1
M((lt)= [ O 1 O O.
o o o o
Ahora bien, ¿cómo podemos utllizar esto? Si calculamos (M(9A)}2 con el convenio de
que 1 + 1 = 1. entonces tenemos que

01 01]
(M(g¡))' = O O O O
[O O O 1 '
o o o o
t El lector no familiarizado con la muhiplicadón de nw.riccs o que sólo desee hacer un breve repaso
deberá consu:1ar el Apendice 2.
7.2 Reconocimiento por computador: Matrices cero-uno y rafos dirigidos 361

que es la matriz de relación de 91: o 9l = !Jt2. (Verifique el ejemplo 7. 19 .) Además,

oo o
o o• º]
o o
la matriz de relación de9'l"= e.

~-A.-~•-
Lo que ocurre aquí puede aplicarse a la s ituación general. Ahora estableceremos algu•
nos resultados acerca de las matrices de relación y su uso en el estudio de las relaciones.

. r,'<"'"" .... - • -. --"~_,.:::r,;,'ft:.:


lLl l i - -• la.-izfe
-·:.~ ;:_:~::; ~~t:,¿~:::,
--:~-" .._..,..,,••=•
iaii)IÍJ lillDoi9t =A X A
.. '._-'~• •. ~~~ -¿..6 ;.- : .. ;~~

Usaremos la matriz (O, 1) de una relación para reconocer las propiedades refle xiva,
s imétrica. antisi~trica y transiliva. Para esto, necesitamos los conceptos de las siguientes
tres definicio nes.

Definició n 7 .11 Sean E= (e,)_ . .. F = (1,). •• dos matrices (O, 1) m x n . Decimos que E pnade. o u
menor que, F. y escribimos E .S F. si ~. =-f. para todos l :S i S m, 1 :S j S n.

~-~ SiE= [ó gl] yF= [ó? iJ. tenemos que E :s F. De hccho.existen ocho matrices(O. 1)
G para las cuales E :s G.

Definición 7 .12 Paran E Z♦• ' • = (64).,. • es la matriz (O, 1) n X n tal que

~,={~: si i = j.
si i /j.

Definición 7 . 13 Sea A= (a.).. x . una matriz (O, 1). La traspuesta de A. que se escribcA•. es la matriz (a;). JI •
tal que a¡ :a11• para todos I Sj s r., 1 :S i s m.
362 Capítulo 7 Relaciones: la segunda vuelta

- SiA =[~ H•ntonceoA"=[~ o ll


O 1 .
Como muestra este ejemplo, la i-ésima fila(i---ésima columna) de A es igual a lai-ésima
columna (i-ésima fi1a) de Au. Esto indica un método que podemos utilizar para obtener la
matriz Aª de la matriz A.

íEOREMA 7.2 Dado un conjunto A con IA 1 = n y una relación 91: sobre A. sea M la matriz de relación
para 9t Entonces

a) 91 es reflexiva si y sólo si /~ S M.
b) ~ es simétrica si y sólo si M = M ''.
e) 9! es transiliva si y sólo si M·M = Mi S M .
d) 9! es antisimétrica si y sólo si M n Mu: s , •. (la matriz M n M 11 se forma operando
en los elementos correspondientes de M y Mª de acuerdo con las reglas O () O= O n
l = 1 n O= O y 1 n I = 1; es decir, el producto us~I de ceros y unos.)

Demostración: Los resultados se siguen de las definiciones de las propiedades de una rela-
ción y de la matriz (0, 1). Demostraremos esto para la pane (c), usando los elementos dcA
para designar las filas y columnas de M , como en los ejemplos 7 .21 y 7.22.
Sea M 2 s M. Si (x. y). (y, z) E~. entonces existen unos en la fila (x), columna (y) y en
la fila (y). columna(.?) de M. En consecuencia, en la fila (.r), columna (t) de M 1 hay un l.
Este I también debe aparecer en la fila (x), columna (z) de M ya que M 2 s M. Por lo tanto,
(x. z) E f7t y ~ es transitiva.
Recíprocamente, si~ es transitiva y M es la matriz. de relación de9l, sea si: el elemento
de la fila (x), columna (z) de M 2 , con s..:~ l. Para que s_.:sea igual a J en M', debe existir
al menos un y E A tal que m .r:r ~ m, • ~ 1 en M. Esto ocurre sólo si x g¡y y y91 z.. Si S1t es
transitiva, entonces se sigue que x 9i z_Así, m.r::;:;: 1 y M 1 .s M .
La demostración de las partes restantes se deja al lector.

La matriz de relación es una herramienta útil para el reconocimiento de cienas p~


piedades de las relaciones por parte del computador. Almacenando infonnación como
ya lo hemos descrito, esta matriz es un ejemplo de una estructura de datos. También es
interesante la forma en que se usa la matriz de relación en el estudio de la teoría de
grafost y la manera en que esrn teoría se usa para reconocer cienas propiedades de las
relaciones.
En este momento presentaremos algunos conceptos fundamentales de la teoría de grafos.
Con frecuencia daremos estos conceptos de ntro de los ejemplos y no en ténninos de defi-
niciones fonnales. Sin embargo, en el capítulo 11. la exposición no dará por hecho lo dado
aquí y será más rigurosa y amplia.

t Puesto que la terminologfa de la 1eoria de grafos no es estándar, el lector podría cnconrr:u aJguU5
diferencias entre nuestras definicione:S y las de otros 1cxtos.
7.2 Reconocimtento por computador: Matrices cero-uno y grafos dirigidos 363

>dinición 7 .14 Sea V un conjunto finito no vacío. Un grafo dirigido (o digrafo) G sobre V está
formado por los elementos de V, llamados virrius o nodos de G. y un subconj unto E
de V x V, conocido como las aristas (dirigidas) o arcos de G. Si a, b E Vy (a, b) E
Et. entonces existe una arista de a a b. El vlrtice a es el origen o fuente de la ar-isla. y
b es el término. o vértice terminal, y decimos que bes adyacente desde a y que a es
adyacente hacia b. Además, si a'/= b, entonces (a, b) f! (b, a). Una aris1a de la forma
(a, a) es un la,o (en a).

Para V= [ 1, 2, 3, 4, 5), el diagrama de la figura 7. 1 es un grafo dirigido G sobre V con el


conjunto de aristas [ (1, 1), (1, 2), ( 1, 4). (3. 2)). El vértice Ses pane del grafo aunque no
sea el origen o el final de una arista; y se le conoce como v~niceaislodo. Como vemos, las
aristas no tienen que ser segmentos de recta. ni impona su longitud.

b b

<•>
tJ/

(b)

Figura 7 .1 Figura 7.2

Cuando desarrollamos un diagrama de flujo para analiz.ar un programa o un algoritmo,


trabajamos con un tipo especial de grafo dirigido donde la fonna de los v~rticcs puede
tener importancia en el análisis. Los mapas de carreteras son grafos dirigidos: en ellos. las
ci udades y los pueblos están representados por los véniccs y las carreteras que unen dos
localidades están dadas por aristas. En estos mapas, una arista está dirigida con frecuencia
e n ambos sentidos. En consecuencia, si Ges un grafo dirigido y a. b E V, con a#=. b. y
(a. b),(b, a) E E,entoncesse usa la arista única no dirigida [a. b) = [b, a) de la figura 7.2 (b)
para representar las dos aristas dirigidas de la figura 7.2(a). En este caso.a y b son vénices
adyacentes. (Tampoco impona la dirección en los lazos.)
Los grafos dirigidos tiene n un papel imponante en muchas situaciones dentro de las
ciencias de la computación. El siguiente ejemplo muestra una de ellas.

Elefflplo_7~i_~.. Los programas pueden procesarse más rápidamente si cienas instrucciones del programa
se ejecutan en forma concurrente. Pero, para esto. debemos estar conscientes de que algu•
nas instrucciones dependen de instrucciones anteriores del programa. puesto que no pode-

t En csle capítulo sólo pcnniti~mos la existencia de una arista de a a b. Las situaciona en que .apare«n
varias aristas se llaman mwltigrafos 5C analitaran en el capí1ulo 11.
!64 Capitulo 7 Relaciones: La,egunda vuelta

mos ejecutar una instrucción que necesita resultados de otras proposiciones que no hait
sido ejecutadas todavía.
En Ja figura 7.3(a) tenemos ocho instrucciones de asignación que conforman el princi-
pio de un programa. Representamos estas instrucciones mediante los ocho vértices co-
rrespondientessi, S2-s3• • • . ,s3 ele la pane (b)de la figura.donde una aristadirigidacomo(s i,ss)
indica que la instrucción s5 no puede ejecutarse antes de que se ejecute la instrucción s 1 • El
grafo dirigido resultante es el grafo de precedencia de las líneas dadas del programa.
Observe cómo este grafo indica, por ejemplo, que la instrucción s1 se ejecuta después que
las instrucciones s 1, s 2, s 3 y s,.. Así mismo, vemos que una instrucción como s1 debe ejecu-
tarse antes que cuaJquiera de las instrucciones s 2, s,, ss, 5-J o s,. En general, si un vértice
(instrucción) ses adyacente de otros m vénices (y sólo de e llos), entonces las instruccio-
nes correspondientes para estos m vértices deben ejecutarse antes de que pueda ejecutarse
la instrucción s. En forma análoga, si un vértice (instrucción) s es adyacente a otros n
vértices, entonces cada una de las instrucciones correspondientes de estos vénices necesi-
ta la ejecución de la instruccións antes de poder ejecutarse. Por último, del grafo de prece-
dencia vemos que las instrucciones s 1 , si y s6 pueden procesarse en forma concurrente. De
acuerdo con el lo, las instrucciones s2 , s, y s1 pueden ejecutarse al mismo tiempo, para
después ejecutar las instrucciones s5 y s1 • (0 bien, podríamos procesar las instrucciones s2
y s, en fonna concurrente, y después Jas instrucciones s3, s,. y s,.)

s, s,
(s,) b 3:
(s, ) e b+ 2 :
(s,) a 1.
(s,) d a • b +5; s,
(s,) e d-1;
(s,) f 7·
(s,) e = e + d:
(s,) g b • f; s., s, s,
(al (b)

Figura 7 .3

Ahora queremos cons iderar la forma en que las relaciones y los grafos dirigidos se
relacionan entre sí. Para comenzar, dado un conjunto A y una relación ~I:: sobre A, podemos
construir un grafo dirigido G con el conjunto de vértices A y conjunto de aristas E!:; A X
A, donde (a. b) E E si a. b E A y agj: b. Esto se demuestra en el siguiente ejemplo.

Para A = {1, 2, 3. 4 }. sea !1l = {(1, 1). ( l. 2), (2. 3), (3. 2), (3. 3). (3. 4). (4. 2)} una relación
sobre A. El grafo dirigido asociado con 91 aparece en la figura 7.4(a)_ Si no se tienen en
cuenta las direcciones. obtenemos el grafo no dirigido asociado que se muestra e n la pane
(b) de la figura.Aquí vemos que el grafo es conexo, en el sentido de que, para cualesquiera
dos vértices x, y con x -4=- y, existe un camino simple que comienza en ..r y tennina en y. Tal
7 .2 Reconocimiento por computador: Matrices cero-uno rafos di( idos 365

camino simple consiste en una sucesión finita de aristas no dirigidas, de modo que las
aristas { l, 2), {2,4} proporcionan un camino simple de l a4, y las aristas (3,4 }, (4, 2 } y
{2,1} ofrece n un camino simplcdc3 a !.La sucesión de aristas{3,4}, (4,2} y (2, 3} propor•
ciooan un camino simple de 3 a 3. Este camino simple arrodo se conocecomocic/Q. Éste es
un ejemplo de un ciclo no dirigido de longi1ud tres, ya que tiene tres aristas en él.
Cuando trabajamos con caminos simples (en grafos dirigidos y no dirigidos), no deben
repetirse los véniccs. Por lo tanto, la sucesión de aristas (a, b }, (b, <), (<./}. (/. b), {b, d}
de la figura 7.4(c) no se considera un camino simple (de a ad) ya que pasa por el vértice
b más de una vez. En el caso de los ciclos, el camino simple comienza y tennina en e l
mismo vénicc y tiene al menos tr,sarisras. La sucesión de aristas (b,f), (/. <), (<, d), (d, e),
(c, b) presenta un ciclo dirigido de longitud cinco en la figura 7.4(d). Ninguna de las seis
aristas (b.f), (/.<),(e, b), (b, d), (d, <), (<, b) produce un ciclo dirigido en la figura, debido
a la repetición del vértice b. Si hacemos caso omiso de sus direcciones, las seis aristaS
correspondientes, en la parte (e) de la figura, pasan igualmente por el vértice b más de una
vez. Eo consecuencia, no se considera que estaS aristas formen un ciclo para el grafo no
dirigido de la figun, 7.4{c).
Ahora bien. puesto que hemos pedido que un ciclo tenga una longitud de al menos tres,
no consideraremos a los lazos como ciclos. También observamos que los lazos no tienen
nada que ver con la conexión de los grafos.

°4 °4 w.w.
(a)

figura 7.4
(b)
1
(e) (d)

Optamos por definir fonnaJmente la siguiente idea. debido a su imponancia para lo


he.cho e n la sección 6.3.

Definición 7 .15 Un grafo dirigido G sobre V csfuuumentt conuo si para todos x, y E V. tales que x I= y.
existe un camino simple (en G ) de arisw dirigidas de .x. a y; es decir, o bien la arista
dirigida (.x.. y) está en G o, para algún n E z · y vértices distintos U1, l>z, •••• u . E V, las
aristas dirigidas (z, u,) (u ,. u,), .. .• (ub y) eswi en G.

En este sentido hablamos de las máquinas fuertemente conexas e n el capítulo 6. El grafo


de la figura 7.4(a) es conexo pero no fuencmente coocxo. Por ejemplo, no existe un camino
simple dirigido de 3 a l. En la figura 7.5.el grafo dirigido sobre v~ (1, 2, 3,4} es fuertemcn•
te conexo y sin laws. Es.to también es cierto en cuanto al grafo dirigido de la figura 7.4(d).
66 Capítulo 7 Refaciones: La segunda vuelta

Q---0 '

0 4
figura 7.5
3
0---0
(9!,)

Figura 7.6
4
(31.J

Para el conjunto A= { l, 2, 3, 4}, consideremos las relaciones9i1 = {(1 , J). (1 , 2), (2, 1),
(2, 2), (3, 3), (3, 4), (4, 3), (4, 4)} y g¡, = {(2, 4), (2. 3), (3, 2), (3, 3). (3. 4)}. Como lo
muestra la figura 7 .6, los grafos de estas relaciones son disconexos. Sin embargo, cada
grafo es la unión de dos grafos cone :<os llamados componentes del grafo. Para9l., el grafo
está fonnado por dos componentes fuertemente conexas. Para9l: 2, una componente consta
de un vértice aislado y la otra componente es conexa pero no fuertemente conexa.

Los grafos de la figura 7.7 son ejemplos de grafos no dirigidos sin lazos y que tienen una
arista por cada par de vénices distintos. Estos grafos ilustran los grafos completos de n
vértices que se designan por K,.. En la figura 7 ,7 tenemos ejemplos de los grafos completos
con tres, cuatro y cinco vértices, respectivamente. El grafo completo K 1 consta de dos
vértices x, y y una arista que los une. mientras que K, consta solamente de un vértice, sin
aristas, debido a que no se permiten los !aros.
EnK3, se cruzan dos aristas, {3. 5} y { 1, 4}. Sin embargo, ningún punto de intersección
crea un nuevo vértice:. Si íntc ntamOS evitar el cruce de las aristas mediante: otro trazo del
grafo, volveremos a tene r el mismo problema una y otra vez. Esta dificultad será analizada
en el capítulo 11 cuando hablemos de la planarid.ad de los grafos.

Figura 7.7
7 .2 Reconocimiento por computador: Matrices cero-uno y grafos dirigidos 367

Para un grafoG sobre un conjunto de vértices V, el grafo da lugar a una re1ación9l sobre
V tal que .:c91y si (x, y) es una arista de G. En consecuencia, existe una matriz (O, 1) para G,
y como esta matriz de relación proviene de las adyacencias de los pares de vértices, se le
conoce como ma.trit de adyauncia de G y como matriz de relación de 91'.

En este momento ligaremos las propiedades de las relaciones y la estructura de los


grafos dirigidos.

Si.4 = {l. 2 , 3) y91 = ((!, 1), (l. 2), (2,2),(3, 3), (3, 1)1, entonccs91es una relación reflexiva
y antisimétrica sobre A, pero no es simétrica ni transitiva Et grafo dirigido asociado a 9l:
consta de cinco aristas, tres de tas cuales son luos que resultan de la propiedad reflexiva de
srt. (Vtasc la Fig. 7.8.) En general. si !11 es una relación sobre un conjunto finito A, entonces
9t es reflexiva si y sólo si su grafo dirigido posee un lazo en cada vértice (elemento de A).

=
La rclaci6n91 ( (l. 1), (!, 2), (2, !), (2, 3), (3. 2) ) es simétrica sobre.4={ l . 2. 3), pero no
es reflexiva, antisiméuica ni transitiva. El grafo dirigido de 91: aparece en la figura 7.9. En
general, una relación 91: sobre un conjunto finito A es simétrica si y sólo si su grafo dirigido
contiene solamente lazos y arisw no dírigidas.

Para.4 = {l. 2, 3). considercmos 91 = ((! , !), (l. 2). (2, 3). (l. 3)). El grafo dirigido de91
aparece en la figura 7.10. En este caso.~ es transitiva y antisimétrica, pero no refle xiva ni
sim~trica. El grafo dirigido indica que una relación sobre un conjunto A es transitiva si y
sólo si su grafo dirigido satisface Jo siguiente: para cualesquiera .:e, y E A , si existe un
camino simp1e (dirigido) de x a y en el grafo asociado, entonces también existe una arista
(x, y). [En este caso, (1, 2). (2. 3) es un camino simple (dirigido) de J a 3. por lo que
también tenemos, por transitividad. la arista (1, 3).) Observe que el grafo dirigido de la
figura 7.3 del ejemplo 7.26 tambitn tiene esta propiedad.
La relación 9lt es antisim~trica debido a que no existen pares ordenados en 9lt de la
forma (x. y) y (y, x) con .:e t y. Para usar el grafo dirigido de la figura 7.10 en la caracteri-
zación de la simelrfa, debemos notar que para cualesquiera dos v~rticcs x, y con x -/:- y, el
grafo contiene como mnim.o una de las aristas (x, y) o ()', x). Por lo tanto, no existen
arisw no dirigidas distintas de los lazos.

Figur• 7.8 Figura 7.9 Figura 7.10

Nuestro úhimo ejemplo trata de las relaciones de equivalencia.


368 Capítulo 7 Relaciones: la segunda vuelta

Q-0
Q--0
Q
Figura 7.11

Para A { t, 2, 3, 4, 5 ~- las siguientes son relaciones de equivalencia sobre A :

9!, = {(!, !), (1,2), (2, !), (2, 2), (3,3), (3, 4), (4, 3), (4,4), (5,5)),
=
9!, {(!, !), (!, 2), (!, 3), (2, 1), (2, 2), (2, 3), (3, ! ), (3, 2), (3, 3),
(4, 4), (4, 5), (5, 4), (5, 5)).

Sus grafos asociados aparecen en la figura 7 .11. Si pasamos por alto los lazos de cada
grafo, vemos que el grafo se descompone en componentes, comoKh K 2 y K 3• En general,
una relación sobre un conjunto finito A es una relación de equivalencia si y sólo si su grafo
asociado es un grafo completo aumentado con tos lazos en cada vénice o si consta de la
unión disjunta de grafos completos aumentada con lazos en cada vénice.

EJERCICIOS 7 .2 1. Para A = { 1, 2, 3. 4}. sean Si: yY las relaciones sobre A definidas como 91: = ((1. 2). ( l. 3). (2,
4 ). (4. 4)) y !:f = {(I. 1). (1, 2). (l. 3). (2. 3). (2. 4)). Determ;ne Sl•.'I'• .'/'• Sl.Sl'.91'·!:f' y !:f'.
2. Si 59: es una relación reflexiva sobre un conjunto A. demuestre que5t1 también es reflexiva sob<eA.
3 . Proporcione la demostración de la inclusión opuesta del teorema 7.1.
4. Para los conjuntosA. 8 y C. consideremos las relacioncsSi1 C A x B,!}iz C 8 X Cy 2/h C B X
C. Demuestre que (a)9'11 ° (9'1z U 31,) = (~ 1 °911J U (g¡1 °21t,); y (b) 21t 1 °(2,t2 il&Jt.i) C (:1t1 °stJ
n (~1°91,.). (Dé un ejemplo para mostrar que la inclusión propia puede <X'Unir en la parte ( b).)
S. Para una relación ~ sobre un conjunto A. defina~º = ((a. a) la E A}- Si 1
[ A =n. demuestre
que existen s. t EN, con O 5 s < t 5 '!"'-, tales que91:' =31!.
6. s; A= { I, 2. 3. 4). sea '31 = 1(1. 1), (l. 2). (2. 3), (3. 3). (3. 4). (4. 4)1 una reladón sobre A.
Encuentre dos relaciones~. B'sobreA talesque::f' cf; B"pero~ 0 .:f.=91°'8'= ((l. 1).(1. 2). (1.4)1.

:::~:~~[•Ctt:·ili,::::,: : ~~(:u:
1 O OO
6

F=,:;::facen E ~ F'

9. Considere los conjuntos A= {ai.a2••. . • a .. }.B = lb,.b:.. . . . b. } y Cz (e,, c 2, •• • • c,} .donde


los elementos de cada conjumo permanecen fijos en el orden dado. Sea9'! 1 una relación de A a
By seag¡! una relacióndeB a C. La matriz de relación de9'!, esM(EJt;). donde i= l. 2. Las filas
y columnas de estas matrices están indexadas por los elementos de los conjuntos adecuados A..
7 .2 Reconocimiento por computador: Matrices Cffo-uno rafes dirtgidos 369

By C. de acuerdo con los órdenes ya dados. La matriz dc9t1 • $12 es la matrizm X pM(511, 0 9t¡),
tal que los elementos de A {en el orden dado) constituyen un índice de las filas, y los elementos
de C (tambifn en el orden dado) son un índice para las columnas.
Muestre que para todo 1 S ; :S m y I S j s p. los eJemen1os de la i..fsjma fila y la j--ésima
columna de M(fll,) . M~J y M~ •• f/lJ ,00 iguales. [Por lo WIIO. M(f/1,) . M(f/lJ. M(fll, . f/lJ.)
10. Sea A un conjumotalque IA 1 =n, y consideremos que el orden dela lista de suselemcncoses
fijo. Para 9t CA x A, sea M(9t) la matriz de relación correspondiente.
a) Demuestre que M(l!A.) • O(la matriz n x n con todos sus e lementos iguales a 0) si y sólo si
fll• e.
b) Demues1requeM(9l)-= l (la matrizn x n con todos sus elementos iguales a 1) si y sólo si
91: -=A X A.
c) Use el resultado del ejercicio 9,junto con el principio de inducción matelmtica. para de·
mosirar que M(~) :a [M(9t)r para todo m E z•.
1 1. PropOl"cione las demostraciones del teorema 7.2(a). (b) y (d).
12. U5C el teorema 7.2 para escribir un programa (o para desarrollar un algoritmo) que reconozca
las relaciones de equívalencia sobre un conjunto finito.
13. a ) Trace el digraío G,-= (V1.E1). donde V,-= (a. b, e, d. e,/} y E,= {(a, b). (a, d ), (b, e), (b, e).
(d. b). (d. ,). (<, e),(,.[).(/. d)).
b) Trace el grafo nodiri¡idoG1 =(V:,E1), talque V1 = {s. t, u, u. w. x,y. l} yE1 z ((s.1},
fs. •J. fs, •J. f• •J. ft, wJ, fu. w). fu. •l. fu, w), fu. •J. fu. y), {w. , J. {.r. y) ).
14. Para el grafo dirigidoG =(V. E ) de la figura 7.12. clasifique cada una de las sl¡uicntes propo--
siciones como verdadera o falsa
a) El vértice e es el origen de dos aristas en G.
b) El vfnice ges adyacente al vértice h.
e) Existe en G un camino simple dirigido de da b.
d ) Exiuen dos ciclos dirisidos en G.

Figura 7.12
1 5 . Para A• {a, b, c. d. e,/}. cada grafo o digrafo de la figura 7.13 representa una relación 9't sobre
A. Determine larelación9l CA x A cncadacaso.asícomo su matriz de relación asociadaM(9t).
16. Para A= (U. w, x.,; z}.cada una de las si¡uienccs es la matriz{O, l)deuna relación9l sobre A.
En este caso. las filas (de arriba hacia abajo) y las columnas (de izquierda a dcrttha) C$tán
indexadas en el orden u, w, x. ~ z_ Determine la relación 9' C A x A en cada caso y crace el

: ~i : ~i
grafo dirigido C asociado con 9t.

~ ~
o 1 o1
1 0 1 O
a) M(9t) - O O O O 1 b) M (9t)- 1 1 O O 1
[o o O O 1 [1 O O O 1
o o o o o oo 1 1 O
370

b b b b

~ ti :d.

(i)

Figur.i 7.13
1) di

(i) (,i)

(1v)
•1

17. Sea G • (V, E) un grafo dirigido con matriz de adyacenciaM. ¿Cómo podemos identificar un
vlnicc aislado de G a partir de la malriz M?
18 . a) SuG = (V. E)el grafo diri¡idotal que V• {l. 2. 3. 4, S.6. 71 yE= {(y.J)I 1 s i <j s 71.
l) ¿Cuántas aristas existen para este gnfo?
U) Cuatro de los posibles caminos s imples dirigidos en G de I a 7 serían:
1) (l. 7); 2) (1, 3), (3. S), (5, 6). (6, 7);
3) ( 1.2), (2. 3), (3, 7); y 4) (l. 4), (4. 7)
¿Cuántos caminos simples diri¡idos e xisten (en total) en G de l a 7?
b) Ahora. sean e z·. n ~ 2 yconsidere el grafo dirigido G = (V. E )con v. {1. 2. 3.... ,ni
yE•{(i,J) i l Si<jsn}.
i) Detenninc El.I U) ¿Cuántos caminos simples dirigidos existen cnG de tan?
lli) Si a. b E z•con I s a< b s n. ¿cuántos caminos simples d irigidos existen en G dea
ab?
(El lector puede consultar el ejercicio 20 de la sección 3. 1.)
= =
19 . Para A 11.2. 3. 4 }, seaéil {(1. 1). (1. 2). (2. 3). (3, 3). (3, 4)) una relación sobre A. Trace el
gr2fo dirigido G sobre A asociado con~- Haga to mismo con~:. 91:, y 9!~.
2 0. Para IA1=5,¿cuintasrclaciones!,I sobreA existen? ¿Ctwlwdces1asrclacioncsson simmi-
cas?
21 . Sea IA1 = 5. (a) ¿Cuántos grafos dirigidos pueden construirse sobre A? (b) ¿Cuántos de wos
grafos de la parte (a) son en realidad no dirigidos?
22. ¿Cul.nlU arista.S (no dirigidas) existen en los ¡rafos completos K.. K1 y K.. donde n E Z "'?
23_ a) Manteniendo fijo el orden de los elementos como J, 2. 3, 4, 5, determine las malri«s de
relación (0, )) para las relaciones de equivalern;ia del ejemplo 7.33.
b ) ¿Conducen los resultados de la parte (a) a alguna generalización?
24. a) Sea 91: la relación sobre A =- ( 1. 2. 3. 4. 5. 6. 7}. ta1 que el &rafo dirigido asociado con~
consta de las componentes que se muestran en la figura 7 .14. cada una de las cuales es un
ciclo dirigido. Encuentre el entero n > l mú pcqueOO. tal que~= 91, ¿Cuál es el valor
mínimo de n > J para el que el grafo de !:,lº contiene algunos lazos? ¿Ocurre en algún
momento que el grafo de !jlA conste únicamente de lazos?
b) Responda las mismas prqunw de la parte (a) para la rclación91: sobre A= {l. 2. 3, _.. • 9.
10}. si el grafo dirigido asociado con 9Q: se ffllle$tra en la figura 7.15.
e) ¿Indican algún hecho general los resultados de las panes (a) y (b)?
7.3 órdenes parciales: Dia ramas de Hasse 371

06 Vó
Figura 7.14 Figura 7.15
S 9

25. Trace un ¡ralo de precedencia pan el siguiente segmento de programa de cómputo:


8

(s1) a : • I;
(s,) b ;• 2;
(s1) a :• a.+3;
(s,) e :• b;
(ss) a :• 2•• -1:
(s,) b : • a • e;
(s,) : • 7:
(s,) :• c+2:

7.3
Órdenes parciales:
Diagramas de Hasse

Si pedimos a algunos niños que reciten los números que conocen. oiremos una respuesta
uniforme de ··1, 2, 3, .. . ". Sin prestar atención al hecho, dan una lista de los números en
orden creciente. En esta sección analizaremos más de cerca esta idea de orden, algo que tal
vez hemos dado por sentado. Comenzaremos por señalar algo acerca de los conjuntos N 9
Z,Q,R y C.
El conjuntoN es cerrado en las operaciones binarias de suma y multiplicación (ordina-
rias). pero si buscarnos la respuesta a la ecuación x + 5 = 2, vemos que ningún elemento de
N es una solución. Así, extendemos N a Z. donde podemos realizar la resta. así como la
s uma y la mul1iplicación. Sin embargo, pronto encontramos problemas al tratar de resol-
ver la ecuación 2r + 3 = 4. Si nos extendemos a Q, podemos realizar la división entre
números distintos de cero, además de las otras operaciones. Pero esto tambiin demuestra
ser inadecuado: la ecuación zl- 2 = O necesita que introduzcamos los números reales, pero
i rracionales :✓-f.. Incluso después de ampliar Q a R , surgen más problemas al intentar
resolve r x2+ 1 =O. Por último, llegamos a C, el sistema de los números complejos, donde
podemos resolver cualquier ecuación polinomial de la forma e.X-+ c._i-.r"""1 + · · · + c,xl +
c 1x + c0 =O.donde e; E C para O s i s n, n > O y c. '# O. (Este resultado se conoce como
el teorema fundamental del álgebra. Su demostración requiere material relativo a las fun-
ciones de variable compleja. por lo que no la incluiremos aquí.) Durante este proceso de
construcción de N a C. para tener una mayor capacidad de resolver ecuaciones polinomiales,
algo se perdió al pasar de R a C. En R. dados dos números r., r 2, con r1 +. r 2, sabemos que
+
r 1 < r 2 o r 1 < r 1• Sin embargo. en C tenemos que (2 + 1) (1 + 21), pero ¿qué significado
podemos dar a una proposición como ..(2 + i) < ( 1 + 2i)'"? ¡Hemos perdido la capacidad de
--ordenar'" los elementos en este sistema numérico!
372 c,pitulo 7 Relaóones: La segunda vuelta

Para comenzar a analizar más de cerca el concepto de orden procederemos como en la


sección 7.1. Sea A un conjunto y 9Ji una relación sobre A. B par (A. 9i) es un conjunto parr:iol-
men1e ordenado. si la relación 91: sobre A es un orden pardal, o una relación de ordenamiento
parcial (como 1aestab1ecidaen ladefitúción 7.6). Si decimos que A es un conjunto parcialmen-
te ordeflOOo, entendemos que existe un orden parcial3t que hace de A un conjunto parcialmen-
te ordenado. Los ejemplos 7.1 (a), 7.2, 7.11 y 7.15 son conjuntos parcialmente ordenados.

Sea A el conjunto de cursos ofrecidos en una escuela. Definimos la relación 9l sobre A


comox91iy si x, y son el mismo curso o si.r es un prerrequisito para y. Entonces9!: hace de
A un conjunto parcialmente ordenado.

- Definimos 91 sobre A ; ; ; {1, 2, 3, 4} como x !1?:y si x Iy; es decir, x divide (exactamente) ay.
EntoncesSl; ((1, l), (2, 2), (3, 3), (4,4), (1, 2), (1, 3), (1, 4), (2, 4)) es un orden parcialy (A,
!:Yl:) es un conjunto parcialmente ordenado. (Esto es sintilar a lo que aprendimos en el ejemplo
7.15.)

En la construcción de una casa. cíertas tareas, como la COnstrucción de los cimientos,


deben realizarse antes de emprender otras fases de la construcción. Si A es un conjunto de
tareas que deben realizarse para construir una casa, podemos definir una relación 9i sobre
A comox~y six, )'denotan la misma tarea o si la tareaxdebe realizarse antes deque inicie
la tarea y. De esta forma imponemos un orden sobre los elementos de A, lo que lo convierte
en un conjunto parcialmente ordenado, también conocido como la red PER'.T (siglas en
inglés de técnica de evaluación y revisión de programas). (Estas redes comenzaron a ad-
quirir importancia durante la década de 1950 para el control de la complejidad que surgió
en la organización de las múltiples y distintas actividades necesarias para lle var a cabo
proyectos a muy gran escala. Esta técnica fue desarrollada realme nte y utilizada en primer
lugar por la Marina de Estados Unidos para coordinar los diversos proyectos necesarios
para la construcción del s ubmarino Polaris.)

Consideremos los diagramas de la figura 7. 16. Si (a) fuera parte del grafo dirigido
asociado con una relación Si, entonces, como (1, 2), (2, l) E91. con 1 #- 2,9l no podría ser

(a) (b)
Figur¡a 7.16
7.3 Órdenes parciales: o· ramas de Hasse 373

antisitmtrica Para (b), si el dia¡rama fuera parte del grafo de una relación transitiva !l.
e ntonces (1, 2), (2, 3) E 9'1_⇒ ( 1, 3) E 9'1. Como (3, 1) E 9'I y 1 f. 3, 9'I no es antisimttrica,
por lo que no puede ser un orden parcial.
A partir de estas observaciones, si tenemos unarclación9t sobre un conjunto A, y Ges
el grafo dirigido asociado con 9t, veremos que:

i) Si G contiene un par de aristas de la fonna (a. b), (b, a), para a. b E A cona" b, o
ü) Si 91 es transitiva y G contiene un ciclo dirigido (de longitud mayor o igual que
tres).

entonces la relación 91: no puede ser antisi~trica, por lo que (A, 9t) no es un orden
parcial.

Consideremos el grafo dirigido para el orden parcial del ejemplo 7.35. La figura 7. l 7(a) es
la representación gráfica de 9l En la pane (b) de la figura. tenemos un diagrama un poco
más sencillo, que se llama diagrama <Ú Ha.u~de91:.
Cuando sabemos que una relación 91: es un orden parcia) sobre un conjunto A, podemos
eliminar los lazos de los vértices de su grafo dirigido. Puesto que 9'i tambi~n es transitiva,
basta onn tener las aristas (l. 2) y(2, 4) para garantizar la existencia de la arista(], 4), así que
no necesitamos incluir dicha arista. De esta forma obtenemos el diagrama de la figura 7. l 7(b).

(a) (b)
Figura 7.17

~ ,38 En la figura 7.18 tenemos los diagramas de Hasse de los c uatro conjun1os parcialmente
ordenados siguien1es. (a) Si"1 = ( 1, 2, 3) y~ =:f("1),91es la relación de inclusión sob<e
Á. (b) En este caso,9'1 es la relación -divide exactamente" aplicada a A• ( l. 2, 4, 8). (c) y
(d) AquJ tenemos la misma relación que en la parte (b), aplicada a {2, 3, .5, 7} en la parte
74 Capftulo 7 Relaciones: la segunda weha

__________ 1, r 1----------
11.21 11.3} 12,31 12 385

\;< XI
11! [21 [31

1~ .... AJ\\
(a)

Figura 7.18
""''/ 1

(bl
2

(e)
3 5 7 2

(d)
3 5 7 11

(e) y a (2, 3, 5, 6, 7, 11, 12, 35, 385} en la parte (d). En la parte (e) observamos que un
diagrama de Hassc puede tener todos sus vértices aislados; también puede tener dos (o
más) componentes conexas, como se muestra en la pane (d).

Sea A = { 1, 2, 3, 4, 5 }. La relación 9i sobre A, definida como xSliy si x :S y, es un orden


parcial. Esto hace de A un conjunto parcialmente ordenado que podemos designar por
(A.$). Si B; { l . 2, 4} e A, en1onces el conjunto (B x B) n g¡; {(l. 1), (2, 2), (4, 4),
(1, 2), (1, 4), (2, 4)} es un orden parcial sobre B.
En general, si 9i es un orden parcial sobre A, entonces para cualquier subconjunto B de
A, (8 x B) n 91: hace de B un conjunto parcialmente ordenado, si el orden parcial sobrcB
se induce de 91:.

Ahora analizaremos un tipo especial de orden parcial

e'1inici6 n 7 . 16 Si (A,~) es un conjunto parcialmente ordenado, decimos que A es totalmente orde1Ull:W si


para tOOos .r, y E A ocurre que .r!?l:J o yffi .r.
En este caso. decimos que .9l es un orden total.

ili!i :,. a) Sobre el conjunto N, la relación~ dada por .r ~ y si .r .s y es un orden total.


b) La relación de inclusión aplicada a A; J'("U.), donde 'U. ; {1, 2, 3) es un orden
parcial, pero no total: { 1, 2 }, {I, 3} EA pero no ocurre que { 1, 2) ~ { 1, 3} ni {1, 3) ~
{I; 2}.
e) El diagrama de Hasse de la pane (b) de la figura 7.18 es un orden total.

¿Podrían surgir estos conceptos de orden parcial o total en un problema de la industria?


Pensemos que un fabricante de juguetes está a punto de lanzar al mercado un nuevo
producto y debe incluir un conjunto de instrucciones para su montaje. Para ensamblar el
7.3 Órd<!nes parciales: Diagramas de Ha»e 3 75

l
e f",gura 7.19

nuevojuguete hay que rea1izar siete tareas.A, 8 , C, ...• G, que deben ejecutarse en el orden
parcial dado por el diagrama de Hassc de la figura 7. 19. Aquí vemos, por ejemplo, que B,A
y E deben tenninarse antes de realizar la tarea C. Puesto que el conjunto de instrucciones
debe serunalistadeestas tareas, numeradas 1, 2. 3, ...• 7, ¿cómo puede escribir esa lisia el
fabricante y asegurarse de conservar el orden parcial del diagrama de Hassc?
Lo que realmente nos estamos preguntando en este caso es si podemos tomar el orden
parcial 9l: dado por el diagrama de Hllsc y encontrar un orden total 9' sobre estas tareas,
para el cual ~ C 9'. La respuesta es sí, y la t6cnica que usaremos se llama ordenación
topológica.

En este caso, hemos presentado nuestro algoritmo como una lista precisa de instruccio-
nes. sin hacer referencia a su implementación en un lenguaje de programación particular.
Antes de aplicar este algoritmo a nuestro problema, debemos observar el uso dclibc•
rado de la palabra ..un .. antes de la palabra .. vértice" en el paso 2. Esto implica que la
selección no tiene que ser única y que podemos obtener varios órdenes totales <J que
contengan a 9t. Así mismo, en el paso 3, para los vértices u,..1 tales que 2 ::s; ; :S n. usamos
la notación u,< U;..1 ya que sugíere mejor la idea ..u, antes de u,..1.. que la notac ión u, <J
\1,-1•
En la figura 7.20. mostramos los diagramas de Hasse que surgen al aplicar el algoritmo
de ordenación 1opológica al orden parcial de la figura 7.19. Debajo de cada diagrama
mostramos cómo se va obteniendo el orden total.
176 Capitulo 7 Relaciones: La segunda vuelta

(k = 1) Hl (k= 2) H1 (k = 3) H3 (k • 4 ) H, (k - 5) Hs (k - 6) H6 (J: - 7) H1
G F o G F G

XAÁA
B

D
E B

f<O
E B

G<F< D
E B

C<G
< F< 0
E
/\ . .
B
A

A< C< G
<F<O
E B

B<A<C
< G<F<D
E
E<B<A<C
<G<F<D
.
E

Figura 7.20

Si el fabricante de juguetes escribe las instrucciones en una lista, como 1-E. 2-B, 3-A.
4-C, 5-G, 6-F, 7-D, tendrá un orden totaJ que conserva el orden parcial necesario para el
montaje correcto. Este orden total es una de 12 respuestas posibles.
Como es usual e n las matemáticas discretas y combinatOrias, este algoritmo ofrece un
procedimiento que reduce el tamaño del problema con cada aplicación sucesiva.
En el algoritmo de ordenación topológica, vimos la forma en que se utilizó el diagrama
de Hasse para determinar un orden total que contuviera un conjunto parcialmente ordena.·
do dado (A, ~). Este algoritmo nos pide ahora que analicemos más propiedades de un
orden parcial. Para empez.ar, subrayaremos especialmente el papel de] véniceu,, en el paso
2 del algoritmo. ¿Cuál es la relación entre el orden parcial (A,91) y su diagrama de Hasse,
de modo que podamos describir un vértice corno Ut en términos de 91? Esta pregunta nos
lleva a los conceptos siguientes.

>efinición 7 .1 7 Si (A, 9/t) es un conjunto parcialmente ordenado, entonces un elemento x E A es un ele--


mentoma.xima/ deA si para todo a E A, a 'F- x ⇒ xf/t,a. Un elemento y E A es un elemento
minimal de A si para todo b E A. b -:/= y, entonces b ~ y.

Si usamos 1a contrapositiva de la primera proposición de la definición 7.17, entonces


podemos establecer que x(E A) es un elemento maximal si para todo a E A, xffia ⇒ x =
a. De manera similar.y E A es un elemento minimal si paracualquierb E A, b9!ly=> b = y.

~ Sean CU= {l. 2, 3) y A=.f('U).


a) S~a91 la relación de inclusión sobre A. Entonces íll es maximal y0es minimal para
el conjunto parcialmente ordenado (A, ~ ).
b) Para B. la colección de subconjuntos propios de { 1, 2. 3}. sea 9f la relación de
inclusión sobre B. En el conjunto parcialme nte ordenado (B. ~). los conjuntos
{ 1, 2}, { 1, 3} y {2. 3) son elementos maximales; 0 sigue siendo el único elemento
minimal.
7.3 Órdenes parciales: Diagramas de Hasse 377

Si 91 es la rel!iCión ..menor o igual que" sobre el conjunto Z, tenemos que (Z, S) es un


conjunto parcialmente ordenado sin elementos maximales ni minimales. Sin embalgo, el con•
junto parcialmente ordenado (N, S:) tiene el elemento minimal O pero no tiene elementos
maximales.

Después de volvér a analizar los órdenes parciales de las panes (b), (c) y (d) del ejemplo
7 .38, podemos hacer las observaciones siguientes.
1) El orden parcial de la parte (b) tiene el elemento maximal único 8 y el elemento
minimal único 1.
2) Cada uno de los cuatro elementos (2, 3, S y 7) es un elemento maximal y un ciernen•
to minimal para el conjunto parcialmente ordenado de la pan.e (c) del ejemplo 7.38.
3) En la parte (d), los elementos 12 y 385 son maximales. Cada uno de los elementos
2, 3, 5, 7 y 11 es un elemento minimal para este orden parcial.

¿Existen condiciones que nos indiquen cuándo un conjunto parcialmente ordenado debe
tener un elemento maximal o minimal? •

,OREMA 7.3 Si (A,9.:) es un conjunto parcialmente ordenado y A es finito, entonces A tiene un elemento
maximal y uno minimal.
Demostración: Sea a, E A. Si no existe un e1ementoa E A tal que a #a, y a ,~a. entonces
a 1 es maximal. De otra forma, existe un elementoa2 . ~ A tal que a1 # 0 1 y a 1~a2• Si ningún
elemento a E A, a 'Fai satisface ai~a. entonces a2 es maximal. En caso contrario, pode•
mos encontrara, E A tal quea, 'F Oz, a3 'F a1 (¿por qué?), mientras que a19la2 y a 29ia3• Si
continuamos de esta forma, como A es finito, llegaremos a un elemento a,. E A tal que a.
~apara todo a E A, a /=a,., de modo que a,. es maximal.
La demostración de la e xistencia de un elemento minimal es similar.

Si regresamos al algoritmo de ordenación topológica, vemos que en cada iteración del


paso 2 del algoritmo estamos seleccionando un elemento maximal del conjunto parcialmen•
te ordenado original (A.SO, o un conjunto parcialmente ordenado de la fonna (B. ~ '), donde
0 :/=-B CA y9i' =(B x B) fl9!. Al menos existe un elemento de este tipo (en cada iteración)
debido al teorema 7.3. Después, en la segunda parte del paso 3, six es el elemento maximal
seleccionado (en el paso 2), eliminamos del conjunto parcialmente ordenado actual todos los
elementos de la forma (a, x). Esto produce un conjunto parcialmente ordenado más pequeño.
Ahora pasemos al estudio de algunos otros conceptos relativos a los conjuntos parcial•
mente orQenados.

mnidón 7 .18 Si (A,!11) es un conjunto parcialmente ordenado, entonces decimos que x E A es un e le-
mento mínimo si x 9t a para todo a E A. El elemento y E A es un e lemento máximo si a g¡
y para todo a E A.
378 capitulo 7 R,laoones: La,egundawelta

~ - Sea6U. = { 1, 2, 3} y9't la relación de inclusión.


a ) Si A = ~(<\! ), el conjunto parcialmente ordenado (A, ~) tiene a 0 como elemento
mínimo y <\L como elemento máximo.
b) Si Bes la colección de los subconjuntos no vacíos de qJ,, el conjunto parcialmente
ordenado (B, S:) tiene a CU. como elemento máximo. En este caso no existe et ele~
mento mínimo, pero hay tres elementos minimales.

Para los órdenes parciales del ejemplo 7.38, vemos que


1) El orden parcial de la parte (b) tiene un elemento máximo 8 y un e lemento mínimo l.
2) No existen elementos máximo ni mínimo para el conjunto parcialmente ordenado
de la parte (e).
3) No existen elementos máximo ni mínimo para el orden parcial de la pane (d).

Hemos visto que es posible que un conjunto parcialmente ordenado tenga varios elemen•
tos maximales y minimales. 4Qué podemos decir de los elementos mínimo y máximo?

íEOREMA 7.4 Si el conjunto parcialmente ordenado (A, Si) tiene un elemento máximo (mínimo), enton-
ces ese elemento es único.
Demostración: Supongamos que x, y E A y que ambos son elementos máximos. Comox es
un elemento máximo,y~x. De la misma forma. x9l: y, puesto que y es un elemento máxi-
mo. Como9t es antisimétrica, se sigue que x: y .
La demostración para el caso del elemento mínimo es análoga.

)efinici6n 7.19 Sea (A, 90 un conjunto parcialmente ordenado con B ~A.Un elementox E A es una cota
inferior de B si x ~ b para todo b E B. De manera similar, un elemento y E A es una cota
supe rior de B si b ~ y para todo b E B.
Un elemento x ' E A es una máxima cota infuior o ínfimo (fnf) de B si es una cota
inferior de By si para todas las demás cotas inferiores x' de B tenemos que x'Sil: x' . En
forma análoga, y' E A es una mínima cota superior o suprem0 (s up) de B si es una cota
superior de By si y ' 9i'y" para todas las demás cotas superiores y" de B.

Sea~= {1, 2, 3, 4}, conA = ,1'(~) y sea9lla relación de inclusión sobre A. Si B= { { 1},
{2}, { 1, 2} }, entonces {1, 21, {I, 2, 3}, {I, 2, 4} y {I, 2, 3, 4} son cotas superiores paraB
(e n (A, 9"1)), mientras que { l, 2 ) es una mínima cota superior (y está en B). Por otro lado,
una máxima cota inferior para Bes 0, que no está en B.
7.3 Órdenes parciales: Diagramas de Hasse 379

Sea!:1t la relación "menor o igual que" para el conjunto parcialmente ordenado (A. 91).
a) Si A= R y B = [O, l], entonces B tiene ínfimo O y supremo l. Observemos que O, l
E B. Para el conjunto e - (O, 11, Ctiene ínfimo O y supremo 1, 1 E C pero 0 (J. C.
=
b) Sea A R de nuevo, y B =(
q E Q \ ef' < 2). Entonces B tiene a ✓ 2 como supremo
y -fi. como ínfimo; ninguno de estos números reales está en B.
e) Ahora, sea A = Q, con B como en la parte (b). Entonces B no tiene ínfimo ni supremo.

Estos ejempl os nos llevan al siguiente resultado.

TEOREMA 7.5 Si (A, 91:) es un conjunto parcialmente ordenado y B ~ A, entonces B tiene a lo sumo un
ínfimo (supremo).
Demostración: Se deja al lector.

Cerraremos esta sección con una última estructura ordenada.

Definición 7 .20 El conjunto parcialmente ordenado (A, 91) es un retículo si para cualesquiera x, y E A, los
elementos sup{x, y } e ínf{x, y} e xisten en A.

Para A= N y .x, y EN, definimos xSty como x s y. Entonces sup{x, y} = máx{.x, y},
ínf{x, y} = mín{x, y} y (N. S) es un retículo.

Para el conjunto parcial.mente ordenado del ejemplo 7.44(a), si S, T ~~. con sup{S, T}•
SU Te ínf(S, n = S n T, entonccs (il'("tl. ). ~ ) es un retículo.

Consideremos el conjunto parcialmente ordenado del ejemplo 738(d). En este caso ve-
mos, por ejemplo, que
sup(2, 3 ) =6, sup{3, 6) =6, sup{5, 7}; 35, sup{7, 11}; 385, sup( 1! , 35} ; 385 e

ínf( 3. 6 ) = 3, ínf{2, 12} ; 2, ínf(35, 385} ; 35.

Sin embargo. aunque sup{2. 3} e xista, no existe un ínfimo para los elementos 2 y 3.
Además, tampOCO tenemos (entre otras cosas) ínf(5. 7}, fnf( 11, 35}, ínf(3, 35 } y sup{3,
35}. En consecuencia. este orden parcia] no e s un retículo.

EJERCICIOS 7.3 1. Trace el diagrama de Hassepara el conjumo parcialmenteordcnado (.f(GU). ~ ).talque <l\l. = (l .
2. 3.4}.
2 . Sea.A= { 1, 2, 3. 6. 9, 18} y defina 91: sobreA por x~y sixly. Trace el diagrama de Hasse para
el conjunto parcialmente ordenado (A. 911:) .
380 Gipitulo 7 Re!aciooes: Li! segunda vuelta

3. Sean (A, 91:1 ), (B, 91:J dos conjuntos parcialmente ordenados. En A x B. ~fina la relación~
como (a, b).CY!:(x, y) si a 91 1x y b 9izy. Demuestre que 9l: es un orden parcial.
4. Si las relaciones~" 912 del ejercicio 3 son órdenes totales, ¿es 511 un orden total?
S. Ordene topológicamente el diagrama de Hassc de la parte (a) del ejemplo 7 .38.
6. Para A = {a. b. e, d. ~}. el diagrama de Hasse del conjunto parcialmente ordenado (A. 91:)
aparece en la figura 7.2L
a) Determine la matriz de relación de9'.
b) Construya el grafo dirigido G (sobre A) asociado a9!:.
e) Ordene topológicamente el conjunto parcialmente ordenado (A. !:!11:).
7. El ¡tafo dirigido G de una relación ~ sobre el conjunto A = { 1, 2, 3, 4} aparece en ta figura
7.22. (a) Verifique que (A, 91) es un conjunto parcialmente ordenado y encuentre su diagrama
de Hasse. (b) Ordene topológicamcnte (A, 91). (e) ¿Cuántas aristas dirigidas más se necesitan
en la figura 7.22 para extcndcc (A, 9l) a un orden total?
8. Sea ~ un.a relación transitiva sobre un conjunto A. Demuestre que 91: es un orden parcial sobre
A si y sólosi9! n g-f = {(a. a) la EA }.
9 . Demuestre que un conjunto parcialmente ordenado finito (A. 91) tiene un elemento minimaL
1 O. Demuestre que si un conjunto parcialmente ordenado (A. 31:) tiene un elemento mínimo. éste es
único.
11. Demuestre el teorema 7.5.
1 2. Dé un ejemplo de un conjunto parcialmente ordenado con cuatro elementos maximales pero
que no tenga elemento máximo.
13. Si (A,9lf)es un conjunto parcialmente ordenado pero no es un orden total y 0 'F B. ¿implica esto
que (B x 8) n~c:onvicrte a Ben un conjunto parcialmente ordenado pero ooenunordentotal?
14. Si 911 es una relación sobre A, y G es el grafo dirigido asociado, ¿cómo podemos reconocer a
panir de G que (A,~) es un orden total'l
15 . Si Ges el grafo dirigido de una relación 91: sobre A, con IA 1 = n, y (A, 50 es un orden cotal,
¿cuántas aristas (incluyendo los lazos) hay en G?
16. Sea M(9i:) la matriz de relación de la relación 91: sobre A, con IA 1 = n. Si (A, 9i) es un orden
total, ¿cuántos unos aparecen en M(91:)?
17. a) Describa la estructura del diagrama de Hasse de un conjunco totalmente ordenado (A, 91),
donde IAI=n >e l.
b) Para un conjunto A tal que IA 1 ~n ~ t , ¿cuántas relaciones sobre A son órdenes totales?
18. a) Para A= {ai. a 1• .• •• a . }. sea (A. 9!) un conjunto parcialmente ordenado. Si M(!:Jt.) es la
matriz de relación correspondiente, ¿cómo podemos reconocer un elemento minimal del
conjunco parcialmente ordenado a partir de M(?Jt.)?
b) Responda la pregunta de la parte (a) reemplazando el adjetivo ..minimal" por el adjetivo
··maximal".
c) ¿Cómo podemos reconocer la e~istencia de un elemento máximo o mínimo en (A. 9i) a
partir de la matriz de relación M(2/l.)?

Fiaura 7 .21
7.3 Órdenes parciales: Diagrama,; de Hasse 381

19. Sea CU ={ =
1, 2, 3. 4), con A ~(CU) y seagfj: la relación de inclusión sobre A. Para cada uno de
los siguientes subconjuntos B (de A), determine el ínfimo y el supremo de B.
a) B •({l},{2}} b) B~ {{l},{2},{3},{1,2}} e) B~{f,{l},{2},{1,2}}
d) B ~ {{l},{l,2},{l,3},{1,2,3}} e) B ~ {{l},{2},{3},{1,2),{l, 3}, {2, 3}}
1) B ~ {{l},{2},{3},{l,2},{l,3},{2, 3},{1,2,3}}
20. Sea C\I. = {I. 2. 3.4. 5, 6, 7}, con A = ~(C\I.) y sea91: la relación de inclusión sobre A. ParaB=
{{l ). {2}. {2, 3)} 1: A , detennine lo siguiente:
a) El mlmero de cotas superiores de B que contienen (i) tres elementos de C\I.; (ii) cuatro
elementos de l\L; (iii) cinco ele mentos de ~ -
b) El número de cotas superiores para B e) El supremo de 8
d) El número de cotas inferiores para B e-) El ínfimo de B
21 . Defina la relación 91: sobre el conjunto Z de la forma siguiente: a9'tb si a - b es un entero par
no negativo. Verifique que St: define un orden parcial en z. ¿Es este orden parcial un orden
total?
22. Pan.A==- {2. 3. 4, . . . 1998. 1999. 2000}, defina la relación9i sobre A comox9ly si .rd.ivide
(exactamente) a y. ¿Cuántos elementos maximales existen para el orden parcial (A, 90?
23. a) Si A z {x. y}, ¡,cuántos órdenes parciales sobre A tienen a .rcomo ele mento rrúnimal?
b) Si B = {x. y, z}. ¡,cuántos órdenes parciales sobre B tienen a .r como elemento minirnal?
24, Para X = {O. 11 , sea A =X x X . Defina la relación9t sobre A como (a, b)~( e, d) si (i)a < e; o
bien (ii)a -;::: e y b s d.
a) Demuestre que ~ es un orden parcial para A.
b ) De1crmifl(: todos los elementos maximales y minimalcs para este orden parcial.
e) ¡,Existe un elemento mínimo? ¿Existe un e lemento máximo?
d ) ¡,Es este orden parcial un orden total?
25. SeaX = {O. l . 2) yA =X x X. Defina la relación~ sobreA como en el ejercicio 24. Responda
las mismas preguntas de dicho ejercicio para esca relación 91 y el conjunto A.
26. Paran E z·, sea X= {O. t. 2, . . . , n - 1.n} y A= X x X. Defina la relaci6n91 sobre A como
en el ejercicio 24. Recuerde que cada elemento de este orden tola19l es un par ordenado cuyas
componentes son a su vez pares ordenados. ¿C uántos elementos tiene 9??
27. Sea (A, 91) un conjunto pattialmente ordenado. Demuestre o refute las siguientes proposiciones.
a) Si (A, 91) es un retículo, en1onces es un orden total.
b ) Si (A, 91) es un orden tOlal, entonces es un retículo.
28. Si (A, 9'1) es un retículo y A es finito, demuestre que (A. 91) tiene un e lemento máximo y un
e le mento mínimo.
29. Si A==- {a. b, c. d. e, u, w. .r. y, z}, considere el conjunto parcialmente ordenado (A. :11:) cuyo
diagrama de Hasse se muestra en la figura 7.23. Encuentre
a) ;nf {b, e) b) ;nf{b, w} e) ínf{e, ,} d) sup{c, b)
e) sup{d, x} f) sup{c, e} g) sup{a, u}

Figura 7.23
82 Capítulo 7 Reladones: La segunda vuelta

¿Es (A.91) un retículo? ¿Existe un elemento maximal? ¿Un elemento minimal? ¿Un elemenco
máximo? ¿Un clememo mínimo?
30. Sea (A. 91) un conjunto 1otalmente ordenado. Si para todo 01- B ~ A e l conjunto totalmente
ordenado (8, (B x B) n 9l) tiene un elemento mínimo, entonces se dice que (A. :3t) estt bwr
ordenado. (Vimos esta idea en la sección 4.1. donde usamos e l buen orden de (Z·. S) pan
establecer el principio de inducción matemática.)
Para cada uno de los siguientes conjuntos totalmente ordenados. determine si el conjunto
está bien ordenado.
a) (N, ,s) b) ( Z, ,s) e) (Q, ,s)
d) .(Q*, :S) ~) (P, ::s), donde Pes el conjunto de todos los primos.
O (A. S). donde A es un subconjunto no \"acío de z·
g) (A. S). donde 0 'FA e Z y A es finito

7 .4
Relaciones de equivalencia
y particiones

Como ya observamos en la definición 7.7, una relación ~ sobre un conjunto A es una


re1ación de equivalencía si es reflexiva, simétrica y transitiva. Para cualquier conjunto A
?= 0. la relación de igualdad es una relación de equivalencia sobre A, donde dos elementos
de A están relacionados si son idénticos; 1a igualdad establece así la propiedad de ..ser lo
mismo" entre los elementos de A.
Si consideramos la relación 9't sobre Z definída por .r Sii y si .r - y es un múltiplo de 2,
entonces 91 es una relación de equivalencia sobre Z donde todos los enteros pares están
relacionados entre sí, aJ igual que todos los enteros impares. En este caso, por ejemplo, no
tenemos que 4 = 8, pero sí que 4 91: 8, pues aquí ya no nos preocupamos por el tamaño de
un número sino solamente por dos propiedades: '"paridad" o ..disparidad". Esta relación
separa a Z en dos subconjuntos fonnados por los enteros impares y los pares: Z = {.. . , - 3,
- 1, 1, 3, .. . ) U{ .. . , -4, -2, O. 2. 4, ... ). Esta s,paración de Z es un ejemplo de una
partición, concepto íntimamente ligado al de relación de equivalencia. En esta sección
analizaremos esa relación y veremos cómo nos ayuda a contar el número de relaciones de
equivalencia sobre un conjunto finito.

~ finición 7 .21 Dado un conjunto A y un conjunto de índices/, sea 0-¡. A, ~ A para cada i E / . Entonces
{A,}ifi, es una partición de A si
a)A - UA; b) A¡nA;=0, paratodosi,jE/talesquei-=/:-j.
;,,1

Cada subconjunto A, es una celda o bloque de la partición.

Si A= { 1, 2, 3, .... 10}, entonces en cada uno de los siguientes casos se detennina una
panición de A:
a) A 1 ={1, 2, 3, 4,5},A2 = {6,7,8,9,10}
b) A,={1,3,5,7, 9},A,={2,4,6, 8,10}
e) A,={!, 2, 3},A, = {4,6, 7, 9},A, = {5, 8, 10}
d) A;={í,í+5},1,; ¡,;5
7.4 Relaciones de equivalencia y particiones 383

Sea A= R y para cada i E z. sea A;;;;;; [i, i + 1). Entonces {A;},e:z es una panición de R.

¿Cómo se relacionan las particiones con las relaciones de equivalencia?

Sea9'1: una relación de equivalencia sobre un conjunto A. Para cualquier x E A, la clase de


equivalencia de x, que se denota con (x], se define como (.:r] = {y E A \ysrtx}.

Definimos la relación~ sobre Z como x~y si 41(x-y). Para esta relación de equivalen-
cia, tenemos que

[O] = {. . . , - 8, -4,0,4,8, 12, . ..} = {4klk E Z}


[1] ={... , -7, -3, 1,5, 9, 13, . ..} ={4k + l lk E Z}
[2] = {. . . , - 6, -2,2,6, 10, 14, .. .} = {4k + 2lk E Z}
[3) = {.. . , - 5, -1, 3, 7, 11, 15, ...} ={4k + J lk E Z}.
Así mismo, tenemos, por ejemplo, que [6] ; [2]; [- 2], [51] ; [3] y [17] = [IJ. Lo que es
más imponante, {[O], [1 ], [2], [3]} es una partición de Z.
[Nota: En este caso, el conjunto de índices de la partición está implícito. Si, por ejemplo,
A0 = (OJ, A1 ;;;;; [1], A2 =[2] y A1 ;;;;- [3]. entonces un conjunto posible de índices/ (como en la
definición 7.21 ) es {O, 1, 2, 3}. Cuando a una colección de conjuntos se le 11arna panición (de
un conjunto dado) sin que se especifique un conjunto de índices, el lector deberá entender
que esa situación se parece a ésl.a, en la que el conjunto de índices está implícito.]

Definimos la relación 91 sobre el conjunto Z como a 91: b si al= b2 (o a= ±b). Para todo
a E Z, tenemos que a2 ; a1, por lo que a 9. a. y 211 es reflexiva. Si a, b E Z con a !?11.b ,
entonces tr = Ir, lo que implica que b 2 = a2 o b ~ a. En consecuencia. la relación 9'l: es
simétrica. Por último. sía. b. e E Z , cona87tb y b 9i c. entoncesci= Ir y lr=-c2. por lo que
tr = c1 y a 9Jt c. Esto hace que la relación sea transitiva. Una vez establecidas las tres
propiedades necesarias, ahora sabemos que 9? es una relación de equivalencia.
¿Qué podemos decir de la partición correspondiente de Z'!
En este caso tenemos que [O] ; {O}. [!] ; [-1]; {- 1. 1}, [2] = [-2] ; {-2. 2} y, en
general. para cualquier n E Z+, [n] =- [- n ] =- {-n, n}. Además, tenemos lapartició11

Estos ejemplos nos llevan a la siguiente situación general.

TEOREMA 7.6 Si!:i!: es una relación de equivalencia sobre un conj unto A. y x, y E A, entonces (a)x E [x],
(b) x9ly si y sólo si (x} ; [y]; y (c) [x] = [y) o (x) n [y]; 0.
384 Capítulo 7 Relaciones: La segunda vuelta

Oemostr~ción:

a) Este resultado se sigue de la propiedad reflexiva de Slt.


b) Six9ly, sea w E [x]. Entoncesw9tr, y, como911 es transitiva, w9ly. Por lo tanto, wE
[y) y [xi s; [y). Como91 es simétrica. x9iy => y9ix. Así, si t E [y]. entoncest91y
y, porlapropiedadtransitiva.r9lx. Porlotanto,r E [x] y [y]!;;; [x]. Enconsecuencia.[x]
: [y].
Recíprocame nte, sea [x]: [y]. Como x E [x] por la parte (a), e ntonces.,: E (y] o
x 9ly.
e) Esta propiedad indica que dos clases de equivalencia sólo pueden relacionarse en
una de dos formas: o son idénticas o son disjuntas.
Supongamos que [x] i=- (y]; mostrare mos que esto implica que (x] n (y] = e. Si
(xi n ( y) ¡/, 0. entonces existe u E A tal que u E (x] y u E (y]. Entonces u9lx, u91
y y, como 9lJ: es simétrica. x ~ u. Ahora bíén. (x ffi u y u 9R y} ~ x 91: y, por la
propiedad transitiva. Además.x :91:y ~ [x] = [y] por la parte (b). Esto contradice la
hipótesis de que [x) -/:- [y]. por lo que debemos rechazar la hipótesis de que (xJ n (y)
0, de donde se sigue el resultado.
7-=-

Observe que si 9lJ: es una relación de equivalencia sobre A, entonces, por las partes (a) y
(e) del teorema 7 .6, las distint~s clases de equivalencia determinadas por9'i nos proporcio-
nan una panición de A.

a) Si A= {1, 2, 3, 4, 5 1 y91: {(I, 1), (2, 2), (2, 3), (3, 2), (3. 3), (4. 4), (4, 5). (5,4),
(5. 5)}, entonces~ es una relación de equivalencia sobre A. En este caso. [l] • {1}.
(21 = (2. 3) = {3]. [4] = {4.51 = (51 yA =fi l U [2] U [4 ] con [11 U 121: 0. (1) n
{4): 0 y (2] n {4) = 0. Así, (1 1], [2], [41) determina una panición de A.
b) Consideremos de nuevo la parte (d) del ejemplo 7. 16. Tenemos que A• { l. 2, 3, 4,
5, 6 , 7). 8: (x. y, ,J. y/: A-+ 8 es la función sobre

f = {(l,x), (2, z), (3,x), (4,y), (5, z), (6,y), (7, x )).
Ya hemos mostrado que la relación 9t definida sobre A comoa !1! b sif(a) = J(b) es
una relación de equivalencia. En este c aso,

¡-'(x) = {l,3,7}=[1) ( = [3) =[71),


¡-'(y) = {4,6} = [4) (= [61), y
¡-'(z) = {2, 5}=[2) (= [51).

Con A : (1) U (4) U (2] :f-'(x ) U/·'(y) U¡-'(,), vemos que IJ-'(x)f'(y),J-'(,))
determina una partición de A.
De hecho. para cualesquiera conjuntos no vacíos A. B. si/: A - 8 es una función
lb
sobre, entonces A = Ui.E 11/ - 1(b) y f/·'(b) E B} nos proporcionan una partición de
A.

En ANSI FORTRAN. una sentencia no ejecutable 11amada EQUIVALENCE nos permite


- · · - ..1 • • - - ~ - • • • _ : .LI. • .J.··--------- ..1 • ..1 • • • • • l-. • • • • • l . •
7.4 Re,l~ciones de equN'alencia y particiones 385

Por ejemplo, dentro de un programa. la instrucción

EQUIVALENCE ( A. C, P). (UP. DOWN )

informa a1 compilador que las variablesA. C y P companirán la misma posición de memo-


ria. mientras que UP y OOWN compartirán otra. En este caso, el conjunto de todas las
variables del programa se divide mediante la relación de equivalencia Slt. donde V19t Vi s i
V1 y Vi son variables del programa que comparte n la misma posición de memoña.

En el lenguaje de programación PascaJ, la colecc:ión de todas las instrucciones (válidas)


puede dividirse en las siguientes 11 celdas.

1) lnstrucciones de asignación 6) Instrucciones goto


l ) Instrucciones case 7) Instrucciones if
3) lnstrucciones compuestas 8) Instrucciones de llamada a procedimiento
4) Instrucciones vacías 9) Instrucciones ~peat-until
5) Instrucciones for 10) Instrucciones while
11) Instrucciones with

(¿Cuál es una relación de e.quivalencia adecuada para esta panición?)


Por medio de esta partición, podemos considerar una de las tareas principales de un
compilador. a saber, la capacidad para reconocer la celda de esta partición en la que puede
encontrarse una insuucción dada. Con esto podemos prever la forma en que procederá el
compilador cuando decida, Por ejemplo, si una instrucción s cae en la celda (3). Cuando
esto ocurra, el compilador debe detenninar si la instrucción comienza con bqin para
entonces llamar al procedimiento que controla este tipo de instrucción. En este caso, el
procedimiento Olamado por el compilador) debe procesar cada instrucción del progra♦
ma hasta llegar a un ead que no concuerde. Si nuestra instrucción se encuentra en la
celda (9) de esta partición, entonces, en estas circunstanc ias, el compilador debe decidir
que s comienza con repeat de modo que pueda llamar e ntonces al procedimiento co-
rrecto para controlar dicha instrucción. En este caso, el procedimiento (llamado) proce-
sara cada instrucción del programa hasta llegar a un until no concordante. Al hacerlo. el
compilador procesa la expresión que sigue a u.otil y así genera el código necesario para ·
decidir cuándo (el compilador) debe term inar la generación de código para la insuuc-
•ción repea t-until.

L:!=?Eem1:.::.l
S::..J Una vez que hemos visto algunos ejemplos de la forma en que una relación de equivalen-
cia induce una partición de un conjunto, podemos volver atrás. Si una relación de equiva-
lencia SI sobre A= ( l. 2. 3. 4, 5, 6, 7} induce la partición A= ( l. 2) U (3) U ( 4. 5, 7) U
(6 },¿qut.s91?
Consideremos el subconjunto { 1, 2} de la panición. Este subconjunto implica que ( 1) =
( 1, 2) = [2). por lo que ( l . 1). (2. 2), ( l. 2). (2. 1) ES!. (Los primeros dos pares ordenados
son necesarios debido a la propiedad n:ílexiva deSl: los otros preservan la simetría.)
= = =
De manera similar, el subconjunto (4, 5. 7} implica que mediante 91 . [4) [5) (7)
{4, S. 7 } y que, como relación de equivalenc:ia,.91 debe contener a {4, S, 7} x {4, 5, 7}.
86 C.piwlo 7 ~ , La segunda vuelta

De hecho,9'1 = ((l. 2} x (l. 2}) U ({3} x {3}) U ((4,5. 7} x {4. 5. 71) U ({6} x (6})
y 19'1 1=2'+ 1'+3'+ I' = 15.

Los resultados de los ejemplos 7.53, 7.54, 7.55 y 7.58 nos llevan a lo siguiente.

EOREM A 7 .7 Si A es un conjunto, entonces


a) cuaJquier relación de cquivaJcncia ~ sobre A induce una partición de A; y
b) cualquler particíón de A da lugar a una relación de equivalencia 91: sobre A .
Demostración: La parte (a) se sigue de las partes (a) y (c)del teorema 7.6. Para La parte (b),
dada una partición {A, }rio:1 deA, definimos la relacióng)i: sobre A comox~y. six y y están
en la misma celda de la partición. Dejaremos a1 lector los detalles de la verificación de que
ffi es una relación de equivalencia.

Con base en este teorema y los ejemplos analizados. enunciamos el siguiente resultado,
cuya demostración se esboza en el ejercicio 18 del final de e sta sección.

:OREMA 7.8 Para cualquier conjunto A, existe una correspondencia uno a uno entre el conjunto de
re1aciones de equivalencia sobre A y el conjunto de paniciones de A .

Nos interesa principalmente eJ uso de este resultado para conjuntos finitos.

a) Si A= { 1, 2, 3, 4, 5, 6}, ¿cuántas rel.iciones sobreA son relaciones de equivalencia?


Resolveremos este problema contando las particiones de A, observando que una
partición de A es una distribución de los elementos (distintos) de A en recipientes
idénticos, sin que quede ninguno vacío. De la se.cción 5.3 sabemos. por ejemplo,
que ex.isten S(6, 2) particiones de A en dos recipientes idénticos no vacíos. Si usa.

de l a 6, tenemos L:.
mos los números de Stirling del segundo tipo, como el número de recipientes varía
1
S(6, 1j = 203 particiones diferentes de A. En consecuencia.
existen 203 relaciones de equivalencia sobre A.
b) ¿Cuántas de las relaciones de equivalencia de la parte (a) satisface n que 1, 2 E (4)?
Si identificamos 1, 2 y 4 como el "mismo" elemento en esta relación de equiva•
lencia, contamos, como e n la parte (a). para el conjunto B = { l. 3, 5, 6} y tenemos
que existen
(2] = (4].
L:1
5(4, ,j = 15 relaciones de equivalencia sobre A para las que [I] =

Concluimos con la observación de: que: si A es un conjunto finito con IA1 = n,entonces
para cualquier n :S r S Tr. existe una relación de equivalencia!,( sobre A tal q ue l.911: J = ,-
=
si y sólo si existen n., n2, . . . 'nt E z· tale s que r :zln, n yr:]n; =,.
7.4 Relaciones de equfvalenda y particiones 387

EJERCICIOS 7 .4 1 . Determine si cada una de las siguientes colecciones de conjuntos es una panición para el
conjunto dado A. Si la colección no es una partición, indique por qué.
o) A • {1,2,3,4,5,6, 7.8): A ,• {4,5,6), A,• {1,8), A, • {2,3, 7).
b) A = {o,b,c, d,e,f,g,h}: A, = {d,e}, A,•{a.c,d}, A, •{f,h}, A, •{b,g).
e) A • {1,2,3,4,5,6, 7,8): A,= {1,3,4, 7}, A, = {2,6}, A,= {5,8}.
2 . Sea A = ( l. 2. 3, 4, 5. 6.1, 8}. ¡,Dc cuámu ÍOt'R'la$ podemos dividir aA comoA 1 U A2 U A,, de
modo que
a) l,2EAi. 3,4EA2, y 5,6,7EA.,?
b) 1,2 EA,, 3,4EA,, 5,6EA,, y IA,I • 3?
e) l,2EA., 3,4EA 2 , y S,6EA:,1
3. Si A = { 1, 2. 3, 4, S} y 9t es la relación de equivalencia sobre A que induce la partición A •
{1.2) U (3.4} U (5}.¿qules1)!?
4 . ParaA • {l. 2, 3. 4. 5. 6),91= ((1 , !), (!, 2),(2. 1),(2, 2), (3, 3), (4. 4), (4, 5), (5, 4), (5. 5), (6.
6)) es una relación de equivalencia sobre A.
•> ¿Qut son (1 ).{21 y [3] en CSl3 relación de equivalencia?
b) ¿Qui!! partición de A induce 9t?
5 . Si A =A 1 U Az U A1, dondeA 1 • { l. 2). A: • {2, 3, 4} yA, • (5 ), defina la relación~ sobre A
romo x 9t )' si x y y están en el mismo subconjunto A,. para l !i i ~ 3. ¿Es 5t una relación de
equivalencia?
6. Para A = R1• defina 91 sobre A como Cx1, y 1)91 (xz. yi) s i .r1 a Xz.
a) Verifique que 91 es una relación de equivaJencia sobre A.
b) Describa ¡eométricamente las clases de equivalencia y la partición de A inducida por 9t
7 . Tome A• ( l. 2. 3, 4, S} x {1, 2, 3, 4, 5} y dcftna91 sobre A como(x., y 1)9t(.x2,yJ six1 + y 1 •
X1+ Y!•
a) Verifique que~ es una relación de equivalencia sobre A.
b) Dctcnninc las clases de equivalencia [(l. 3)}. ((2. 4)] y [(l .!}).
c) Determine la partición de A inducida por 911.
8. Si A"" (l. 2. 3. 4. S.6. 71,defina.S?sobreA como (x. y) €9'1: si .r - yes un múltiplo de 3.
a) Mue$lre que 91: es una relación de equivalencia sobre A .
b) Dctenninc las clases de equivalencia y partición de A inducida por St.
9. ParaA • ((-4.-20), (-3,-9). (-2.-4),(-1.-11) . (-1,-3). ( l. 2).(1,5), (2, 10). (2. 14), (3,6),
(4, 8). (4, 12)}. defina la relación Si sobre A como (a. b)9t(c, d) si ad= be.
a) Verifique que 91 es una relación de equivalencia sobre A,
b) Encuentre las clases de equivalencia [(2, 14)]. [(-3, 9)] y [(4. 8)].
e) ¿Cuinw celdas tiene la partición de A inducid.a por 9'?

10. Defina la relación~ sobre como.r9l:y si .Vy = 'Z' para aJ¡,ln n E Z.
a) Verifique que 9i es una relación de equivalencia sobre z·.
b) ¿Cuintas clases de equivalencia distintas encontramos entre (1 J, (2). {31 y (4]?
e) ¿Cuinw clases de equivalencia dis1intas encontramos entre 16). [7). (21 }. [24). [28), (35),
(42] y (48]?
11. ScaA un conjunto no vado y B un conjunto fijo tal que B ~A. Defina la relación9t sobrc.1(A,) ·
como X 91: Y. para X. y~ A si B n X=B n Y.
a) Verifique que Gll es una relación de equivalencia sobre 3'(.A).
b) Si A• { 1, 2, 3} y B • { l. 2}. encuentre la panición de.Jl(A) inducida por 91.
e) Si A• (l. 2. 3.4. 5 ) y B• {I, 2. 3},encuenu-e [X] si X• (1, 3, 5}.
d ) ParaA, • { 1, 2, 3,4, 5} y B = {I, 2. 31, ¿cuintascWCS de equivalencia hay en la partición
inducida por ~?
12. Definimos la rclación9t E z·
como a9tb si mcm(a. 16) = mcm(b. 16).
a ) Verifique quc.:11 es una relación de equivalencia sobre z·.
l8 Capítulo 7 Relaciones: La segunda vuelta

b) Determine cada una de las siguientes clases de equivalencia: [ t ]. (2J. [3), [ 10], [161. [25J.
(32J. (33], (48] y (64J.
13. ¿Cuántas de las rdac:iones de equivalencia sobre-A= {a. b. c. d, e.f} tienen (a) exactamente
dos clases de equivalencia de tamaño 3? (b) exactamente una clase de equivalencia de tamaño
3? (e) una clase de equivalencia de tamai'io 4? (d) a1 menos una clase de equivalencia con tres
o más elementos?
14. Sea A= {u, w. x. y, z} . Determine el número de relaciones sobre A que son (a) reflexivas y
simétric as; (b) relaciones de equivalencia; (e) reflexivas y simétricas pero no transitivas; (d)
relaciones de equivalencia que determinan e xactamente dos clases de equivalencia; (e) relacio-
nes de equivalencia tales que w E [xJ; (f) relaciones de equivalencia en las que u, w e (x}; (g)
relaciones de equivalencia en las que w E (.r] e y E h:]: y ( h) relaciones de equivalencia en las
que w E (x]. y E [zl y (x] , (zJ.
1 S. Si I A 1 = 30 y la relación de equivalencia 9'I sobre A divide a A en las clases de equivalencia
(disjuntas)A 1,A1,y A 3,demodoque !A 1 I = IA?l = IAJl , ¿cuántovale 191:I?
16. Sea A= { 1. 2. 3. 4. 5. 6, 7 }. Para cada uno de los siguientes valores der. de1ermine una relación
de equivalencia ~ sobre A tal que 191:] = r , o explique porqué no existe dicha relación. (a) r=
6; (b) ,=
7 ; (e) , = 8; (d ) ,=9; (e) , - JI; (1) 22; (g) ,=
23; (h) , : JO; (i) 31. ,= ,=
17. Proporcione los detalles de la demostración de la pane (b) del teorema 7.7.
18 . Para cualquier conjunto A 'F 0. sea P(A) el conjunto de todas las particiones de A. y E(A) d
conjunto de todas las relaciones de equivalencia sobre A . Definimos la funcíón/: E(A ) ➔ P(A)
como sigue: Si 9i es una relación de equivalencia sobre A. entonces.ft$) es la partición de A
inducida por at. Demu~tre que/es uno a uno y sobre. como establece el 1eorema 7.8.

7.5
Máquinas de estados finitos:
El proceso de minimización

En la sección 6.3 vimos dos máquinas de estados finitos que realizaban la misma tarea
pero con diferente número de estados internos. ( Véanse las Figs. 6.9 y 6.1 O.) La máquina
con el mayor número de estados finitos contiene estados redundantes, e sto es , estados que
pueden eliminarse debido a que oLros estados realizarán sus funciones. Puesto que la
minimización del número de estados en una máquina reduce su complejidad y su costo.
buscamos un proceso para transfonnar una máquina dada en otra que no tenga estados
internos redundantes. Este proceso se conoce como proceso de minimiz.aci6n, y su desa-
rrollo se basa en tos conceptos de relación de equi,·alencia y partición.
Si partimos de una máquina de estados finitosM = (S, !}, e, v, ro), definimos la relación
E 1 sobre S como s 1 E 1 s2 si CO(s1• x) = W(s2 , x) para todo x E !}_ Esta relación E 1 es una
relación de equivalencia sobre S, y divide a Sen subconj untos tales que dos estados están
en e l mismo subconjunto si producen la misma salida para cada x E !J. En este caso, los
estados s 1 y s 2 son / -equivalentes.
Para cualquier k E z·. decimos que los estados si, s2 son k -equivalentes si co(s 1, x) =
W(s2 , .r) para iodo .r E §i . En este caso. <O es la extensión de la función de salida dada a
S x j•_ La relación de k-equivalencia también es una relación de equivalencia sobre S:
divide a Sen subconjuntos de estados k-equivalentes. Escribimos s1 Ets 2 para denotar que
s 1 y s2 son k-equivalentes.
Por último. si s., s1 E S y s,, s1 son k-cquivalentes para todo k ~ l , decimos que s 1 y s:
....... ,.,,,,.· .,.,,. . . ~,.,.., ,.,.,. ·1-,,· .......... r en ("',._ _ ..,_ --·- --- · -- ··---- - ··· _.
7 .4 Relaciones de equivalencia y particiones 389

nuestra máquina., entonces s 2 será redundante y puede eliminarse. A.sí, nuestto objetivo es
determinar la partición de S inducida p0r E y seleccionar un estado de cada clase de equi-
valencia. Eo1onccs tendremos una realización mínima de la máquina dada.
Para esto, comenzaremos con las siguientes observaciones.
a) Si dos estados de una máquina no son 2-equivale ntes, ¿podrían ser ~uivalentes?
(o k-equivalentes, para k 2: 4?)
t
La respuesta es no. Si s1, s1 ES y s 1 1 s1 (es decir, s1 y s1 no soo 2-equiva.Jentes).
entonces existe al me nos una cadena xy E §Z tal que CD(si, xy) u 1 u, t- w 1 w2 = =
<O(s:z. .iy). donde u 1• U:i:, wi, w, E fJ. Asf. respecto de E,, 1enemos que s1 'j.1 5,: ya que
=
para cualquier Z: E §. O(S¡. ~) U1 U:i:U,t- W1W1WJ ro(s,,xyz:). =
En general. para encontrar cslados que son (k + 1)-equivalentes, buscaremos
estados que sean k-equivalentes.
b) Supongamos ahoraques1, s1 E S y J 1 ~s1 • Queremos determinar sis, E,.s2• Es decir,
¿se cumple ct(s1, x.x,.t,): <O(s1, x.x,,x-1) para todas las cadenas x1.tiX1 E §31 Exami-
ne mos lo que ocurre. Primero obtenemos Ol(s., x 1) ::: Ol(.r2• x 1), ya que .s1 E,s 2 s 1 E 1 =
s1• Entonces existe una transición a los cstadosv(s1, x 1) y v(s1:tx,). En consecuencia.
Ol(J't, x 1x,x3 ) = (l)(s2.x1x,x,) si w(v(si, x 1),x,x1) ;;Ol(V(s2 , x 1),..rzx,) [es decir, si v(s .,x 1)
E,v(s,,x,)J.
En general, para .r1, s2 E S tenemos que s1 Et. 1 s2 si (y sóJo si) (i) s 1 Ets1 y (i i)
>(s., x) E, v(s,. x) para todo x E ~ -
Con estas observaciones como guía. presentamos ahora un algorinno para minimizar una
~uina de estados finitos M.

Du.stramos el algoritmo en el siguiente ejemplo.

Con ~ = () = {O, 1 J. sea M dada por la tabla de estados que se muestra en la tabla 7 . l. Si
observamos las filas de salida. vemos que s 3 y s, son 1-cquivalentes. al igual que s1 , s, y s..
En este caso, E 1 divide a S de la forma siguiente:

P1: {s1} , {,,,s,, s.), {,,,s,).


Para cualquier s E S y c ualquier k E z •• .r Ets. por lo que continuamos este proceso para
determinar P2 , no nos preocupamos por las clases de equ.ivaJencia que tienen un ónico
estado.
l90 Capítulo 7 Re@ciones: La segunda vuelta

Tabla 7.1

V .,
o 1 o 1
S¡ s, s, o 1
s, s, s, 1 o
s, s, s, o o
s, s, s, o o
s, s, s, 1 o
s, S¡ s, 1 o

Comos~1s 4, existe cierta probabilidad de que tengamos s 3 ~ s,. En este caso. v(s,, O)=
s1 , v(s,, O) = s, con s 2 E 1 s, y v(s3 , 1) = s4, v(s,., 1) = sJ con s4 E 1 s3• Por lo tanto, v(s3, x)E.
v (s,, x) para todox E ~ y s 3 E,s4 • En forma similar, v(s1 , O) = s5, v(s5, O) =s2 cons, E,S:?Y
V(s1, 1) = s2, v(s:i, 1) ;:;:s5 con s2 E 1 ss. Así, s 2 ~s,. Por último. v(s,, O) =s1 y v(s6 , O) :;;;,s1 pero
s 2 l2'1 s 1, por lo que s5 1l2 s6• (¿Por qué no analiz.amos la posibilidad de que s 2 E?s6 ?) La
relación de equivalencia E:i divide a S como sigue:
P,: {s,}, {s,,s,}, {s,,s,}, {sJ.
Como P 2 t- P1• continuamos este proceso para obtener P3• Para detenninar si s2 E1 ss.
vemos que v(s2, O);;;; s!i, v(s5, O) = s 2 y SsE::is?. Además, v(s2 , 1) = s2 , v(s,. 1) ;;;; s, y s?E:zss.
Como s2 E,s, y v(si, x) Ei v(s5, x) para todo x E §, tenemos s 2 E,ss. Para s 3, s_., (v(s3, O)=
s1 ) Et(s$:;;; v(s,, 0)) y (v(s 3, l) = s.-) Ez(s3 = v(s.-, 1)), por lo que s 3 E3 s, y E 3 induce la
partición P3 : {s¡}, {s2 , s~}, {s3 ,s.. }, {s6 }.
Ahora tenemos que P 3 = P 2 y el proceso ha terminado, como lo indica el paso 3 del
algoritmo. Vemos que s, y s6 pueden considerarse estados redundantes; si los eliminamos
de la tabla y reemplazamos sus ocurrencias posteriores con s2 y s3 , respectivamente. llega•
mas a la tabla 7 .2. Ésta es una máquina minimal que realiza las mismas tareas que la
máquina dada en la rabia 7.1.
Si no queremos que los estados salten subíndices. podemos cambiar el nombre de los
estados de esta máquina minimal. En este caso, tendríamoss 1, s!, s3, s,.( =s6 ), peros, noes
la mismas, con la que comenzamos en la tabla 7.1.

Tabla 7.2

o
V ..
o 1 1
S¡ s, s, o 1
s, s, s, 1 o
s, s, s, o o
s, S¡ s, 1 o

Tal vez piense ¿cómo sabemos que podemos detener el proceso cuando P 3 ~ P2? Des~
pués de todo, ¿no podría ocurrir. por ejemplo, que P4 /:- P3o que P, = P.1peroP, "F P4 ? Para
demostrar que esto no ocurre, definimos la siguiente idea.
7.5 Máquinas de estados finftos: El proceso de minimización 391

Dtfinición 7 .23 Si P1, P1 son dos paniciones cualesquiera de un conjunto A. entoncesP2 es un refin.amitnto
de P 1, que escribimos como P1 S P,. sí cada celda de P2 está contenida en una celda de P 1•
Cuando P1 .s P1 y P2 t- P h escribimos P, < P1• Esto ocurre cuando al menos una celda de
P? está contenida propiamente en una celda de P ,.

En el proceso de minimización del ejemplo 7.60. tenemos que P, = P1 < P 1• Siempre


que apliquemos el algoritmo, al obtener Pt--1 de P, tene mos que Pl+I s Pt, puesto que la
(k + 1}-equivalencia implica la k-equivalencia. Así, cada partición sucesiva refina la parti-
ción anterior.

Al aplicar el proceso de minimización, si k 2: 1 y P1 y P1♦1 son particiones tales que P,., =


Pt. entonces P,..1 • P, para cualquier r ~ k + l.
Demostración: En caso contrario. sea r( ~ k. + 1) el mínimo subíndice 1al que P ,..1 #- P,.
Entonces P,..1 < P,. de m<Xlo que existen si. sl ES tales que s 1 E,s2 peros1 ~,.1s,. Pero s 1
=
E,s1 v(si, x ) E,_1 v(s 2, x). para todox E 5 y con P,• P,.1, tenemos que v(s., x) E, v(s2, x).
para todox E §, por Jo que s 1 E,.1 s2• En consecuencia. P°,.1 = P,..

Concluiremos esta sección con la siguiente idea relacionada con lo anterior. Sea Muna
máquina de estados finitos con s1• s1 E S y s1, s1 no cquivaJentcs. Entonces debe existir un
t.
mínimo enterok. 2: Otal que s 1 Etsl peros1 1s1. Para k = 0, le nemos que s 1 y s2 producen
diferentes filas de salida en la tabla de estados de M . En este caso, es fácil determinar un
+
x e § tal que Cll(s., x) C!>(s2, x), e l cual distingue los estados no equivalentes. Para k. i!: l.
s1 y s? producen las mismas filas de salida en la tabla Ahora bien, si queremos distinguir
estos estados. necesitamos determinar una cadcna..r _,, , ..r1 x: . .. xlxt.i E §,t--• tal que (1)(s1• x)
t- (1)(s1 , x) aunque 6.>(s1• ..r1 x:. . . Xt) _,,,, 0>(s2 , x 1 x?. • x 1). Dicha cadena x es una cadtna
característica para los estados s1 y s~. Pueden existir más de una de estas cadenas. pero
cada una tiene longitud ( minimaJ) k. + 1.
Ante$ de intentar encontrar una cadena caracteristica para dos estados no equivalentes
en una máquina de estados finitos específica. vamos a analizar la idea principal en este
caso. Así. supongamos que s., s2 ES yque para algún k. E z· (fijo) tenemos que s 1 E,s?
pero s1 2-:: 1 s,. ¿Qué podemos concluir?
Tenemos que

s, t ,., s, :> 3x, EJ [v(s.,x,)t,v(s,,x,)]


=> 3x,E 1 [v(v(s.,x,),x,) t,., v(v(s,, x,),x,l),
o 3 x,E .t [v(s, ,x,x,) t,.,v(s,,x,x,)],
=> 3 x1 E .t [v(s.,x,x,x1) t ,.,v(s,,x, x, x,)]
=> ..
=> 3 x, E 1 [v(s.,x,x, ... x,) t ,.,.,v(s,,x,x, . . . x,)I
=> ..
::;> 3 .r.i: E ' (v(si. Xt Xl ... X.t) t ,v{S2,X1Xz ••• x,)).
92 Capítulo 7 Retadones: la segunda vuelta

Esta última proposición acerca de los estados v(s 1, x 1 xi··· x1 ) y v(s2, X 1X:! •. x.), que no
son 1-equivalentes, implica que podemos e ncontrar xM E ,9, tal que

w(v(si,X1X2 ... x.),Xu 1) :f <a>(v(s2,.r1x 2 . . -X.t), x.t•1)- (1)


Es decir, estos únicos símbolos de salida de() son diferentes.
El resultado de notado con la ecuación ( 1) también implica que
w(si, x) = w{s1, x 1 x 2 • • • XtXu 1) +w(si, x 1 x2 .. . X.1::X.1:+ 1) = w(s:.x).
En este caso tenemos dos cadenas de salida de longitud k + 1 que coinciden en los prime-
ros k símbolos y difieren en el símbolo k + l.

Usaremos las observaciones anteriores.junto con las particiones P,. P2 , •• •• P1:, Pt+i del
proceso de minimización, para trabajar con el siguiente ejemplo.

Las particiones que se muestran abajo se tornaron del ejemplo 7.f:IJ. En este caso, s2 E,s"
peros2 É 2 s 6• Así, buscamos unacadenadeentndaxde longitud dos tal queOJ(s2,x)t- OJ(s6 ,x).
1) Comenzamos con P2 , donde para s2 , s6 tenemos que co(s2, O) :. l oo(s6 , O), pero =
v(s2• 0) = ss y v(s6 , 0) = s 1 están en diferentes celdas· de P 1; es decir.

s, = v(s2 ,0) t, v(s,,0) = s,.


(La entrada O y la salida l nos proporcionan las etiquetas para las flechas que salen
de las celdas de Pi hacia las de P 1.)
2) Sí trabajamos con s 1 y s5 en la partición P 1 vemos que
w(v(s,,0),0) = <ú(s,,O) = 1 ;la O= w(s.,O) = w(v(s,,0),0).
3) Por lo tanto, x = 00 es una cadena característica minimal para s1 y s 6 , puesto que
Ol(s,, 00) = 11 f. 10 = Ol(s,, 00).

P,: {s1}, {s2 , s,), {s,. s,}, {s,}

o~
P1: {sJ, {s2 , s, , s,J, {s,, sJ

o.o 1 º· l 1

Si aplicamos el proceso de minimización a la máquina dada por la tabla de estados de la


parte (a) de la tabla 7.3, obtenemos las particiones de la parte (b) de la tabla. (En este caso,
P, = P 3 . ) Encontramos que los estados s 1 y s, son 2-equivalentes pero no 3-equivalentes.
Para construir una cadena característica (de longitud 3) para estos dos estados. procede-
mos como sigue :

1) Comos 1 lf s,. usamospan.icionesP y P


3 3 2 paraencontrarx1 E 9-(asaber.x1= J) tales que

(v(s., 1) = s,)t,(s, = v(s,, 1)).


7.5 Máquinas de Htados finitos: E.I proceso de minimización 393

2) Entonces v(s., 1) "t, v(s,, 1) => 3 x,


E j (en este caso, 1) con (v(s,. 1), 1) x,= t,
(v(s,. 1), l)o v(s 1• 11) f, 1 v(s,, 11). Usamos Jas particiones P2 y P1 para obtener que
Xi= l.
3) Ahora U$3JTlOS la partición P 1 donde vemos que para x3 = 1 E §.

w(v(s., 11), 1) = O,1 1 - w(v(s,, 11), 1), o


w(s., 111) = 100 ,l 101 = w(s,, 111).
En la pane (b) de la tabla 7.3, vemos cómo hemos llegado a la cadena caracterís-
tica mini mal x ~ 11 1 de estos estados. (Observe tambiln cómo esta parte de la tabla
indica que 11 es una cadena característica minima1 para los eslados s 2 y s,. que son
l ..equivaJentcs pero no 2-equivalentes.)

Tabla 7.3

o
y

1
.
o 1
P,:{s1,s,J, {s,J, {sJ, {s,J
l~~,1
s, s, s, o 1 P,:{s1,s,~,¡
s, s, s, o o 1, 0 1,0
s, s, s, o 1
s, s, s, o 1 P1: {s1, s,, s,}, {s2, S5}
s, s, s, o o 1,d !i,o
(a) (b)

Se puede seguir trabajando con las máquinas de estados finitos. Entre ouas omisiones.
hemos evitado dar una explicación o demostración rigurosa de la razón por la que funcio-
na el proceso de minimización. El lector interesado deberá consultar las referencias del
capítulo para conocer más detalles acerca de este tema.

EJERCICIOS 7.5 1. Aplique el proceso de minimización a cada una de las m"Juinas de la tabla 7.4.

lobla 7 4

o
V

o
.. V .. V ..
1 1 o 1 o 1 o 1 o 1
s, s, ,,,. o 1 s,
'•s, ,.s, o o ,, s, o o
s, s, 1 o s, o 1 ,, '•s, s, o o
s, s, ,. o s, ,, ,, o o
,.'• s,s, s.
1
,. s, s, o s.
1
1 o
1
s. s, o o
s, s, s, 1
1
o s,
,.s, ,.,. o 1 s, ' • ,,s. o o
o o '• ' '· s, 1 o
'• ,, ,. s, o o
(a) (b) (c)

2. Para la miquina de la rabia 7.4(c). encuenue una cadena característica para cada uno de los
siguientes pares de estados:
(a) sh sl; (b) s2• s,; (e) s,. s,.
¡4 Capitulo 7 P.elacionE5: La segunda vuelta

Figul'a 7.24

3. Sea M la máquina de estados finitos dada en el diagrama de estados que se muestra en la figura
7.24.
a) Minimice la máquina M. .
b) Encuentre una cadena característica para cada uno de los siguientes pares de estados: (i)sJ.
~: (ii) S), s.; y (íii) J"1, h

7.6
Resumen y repaso histórico

El concepto de relación surge de nuevo. En el capítulo S presentamos esta idea como una _
generalización del concepto de función. En eJ capítulo 7 nos concentramos en las relaciones
y las propiedades especiales: refle;.;:iva, simétrica, anti simétrica y transit iva. Como resulta-
do, nos centramos en dos tipos particulares de relaciones: los órdenes parciales y las rela-
ciones de equivalencia.
Una relación 9i sobre un conjunto A es un orden parcial, lo que hace de A un conjunto
parcialme nte ordenado, si9i es reflexiva, antisimétrica y transitiva. Tal relación generaliz.a
Ja conocida relación "menor o igual que" en los números reale s. ¡Trate de imaginar el
cálculo, o incluso el álgebra elemental, sin ella! O bien, tome un programa sencillo y vea
lo que ocurre cuando el programa entra al computador en fonna azarosa. pennutando el
orden de las proposiciones. El orden está siempre cerca de nosotros. H emos crecido tan
acostumbrados a él que a veces lo damos como un hecho. Para un conjunto parcialmente
ordenado finito, el diagrama de Hasse, un tipo especial de grafo dirigido. proporciona una
representación gráfica del orden definido en el conjunto: también es ótil cuando se necesi-
ta un orden total, que incluya el orden parcial dado. Estos diagramas reciben el nombre del
especialista alemán en teoría de números Helmut Hasse ( 1898-1979}, quien los introdu•
jo en su libro de texto HOhere Algebra (publicado en 1926) como una ayuda para el estu-
dio de las soluciones de ecuaciones polinomiales. El método utilizado para obte ner un
orden total a panir de un orden parcial es el algoritmo de ordenación topológica adecuado
en la solución de redes PERT (siglas en inglés de técnica de evaluación y R:visión de
programas). Como ya hemos mencionado, e ste método fue desarrollado y utilizado por
vez primera por la Marina de Estados Unidos.
7 .6 ~umen y repaso histórico 395

Aunque 1a relación de equiva1encia difiere del orden parcial sólo en una propiedad, es
bastante diferenle en escructura y aplicación. No intentaremos bosquejar el origen de la
relación de equivalencia. pero las ideas que subyacen en las p.:;piedades reflexiva. simé·
trica y transitiva pueden encontrarse en/ Principii di Gtomttria (1889). obra del mate-
mático italiano Giuseppc Peano ( I 858-1932). La obra de Car! Friedrich Gauss (1 m-I 855)
relacionada con la congruencia. desarrollada por il en la d«ada de 1790, wnbiin utiliza
estas ideas en esencia, aunque no en nombre.
Básicamente, una relación de equivalencia ~ sobre un conjunto A generaliza la igual-
dad: induce una característica de ..ser lo mismo" entre los elementos de A. Esta noción de
..ser lo mismo" hace entonces que el conjunto se separe en subconjuntos llamados clases
d t equivalencia. Recíprocamente, vemos que una partición de un conjunto A induce una
relación de equivalencia sobre A. La partición de un conjunto surge en muchas panes de
las matemáticas y las ciencias de la computación. En éstas últimas, muchos algoritmos de
bllsqueda se basan en una lknica que reduce en forma sucesiva el tamaño de un conjunto
dado A cuya búsqueda se realiza. Al dividir A en subconjuntos cada vez más pequeños,
aplicamos el procedimiento de búsqueda en fonna más eficiente. Cada partición sucesiva
refina la anterior, lo que es fundamental, por ejemplo, en el proceso de minimización para
las máquinas de estados finitos.
En todo e] capítulo hemos hecho hincapié en la intercone;itión entre las relaciones, los
grafos dirigidos y las matrices (O, 1). Estas matrices proporCionan una disposición rectan•
guiar de infonnación acerca de una relación, o grafo, y son útiles en ciertos cálculos. El
almacenamiento de la información de esta forma. en disposiciones rectangulares y en po--
siciones consecutivas de la memoria. ha sido practicado en las ciencias de la compulación
desde finales de la década de 1940 y principios de la década de 1950. Para más delalles
acerca del fundamento histórico de estas consideraciones, consulte las páginas 456 a 462
de D. E. Knuth [3]. Otra forma de almacenar la información acerca de un grafo es la
rtprtsuataci6n de lista de adyacencia. (Véase e] ejercicio complementario 13.) En el estu-
dio de las esuucturas de datos, las listas enlazodas y las listas doblemtme enlazadas son
muy impoI1antes para la implementación de tales representaciones. Para más detalles, con-
sulte e l texto de A. V. Aho, /. E. Hopcroft y J. D . llllman (I J.

Ciluseppe Peano (1858-1932) Carl Friedrkh Ciauss (1777-1155)


Capitulo 7 Relaciones: La segunda vuelta

Respecto a la teoría de grafos. estamos en un área de las matemáticas que data de 1736,
cuando el matemático suizo Leonhard Euler (1707-1783) resolvió el problema de los siete
puentes de KOnigsberg. Desde entonces, esta área ha evolucionado en forma considerable,
panicularmente junto con las estructuras de datos en la ciencia de la computación.
Para una exposición similar de algunos de los temas de este capítulo, véase el capítulo
3 de D. F. Stanat y D. F. McAllister [6]. Una presentación interesante del ''problema de
equivalencia" aparece en las páginas 353--355 de D. E. Knuth (3), para los que deseen más
información acerca del papel del computador junto con el concepto de relación de eq_uiva.
!encía.
Los primeros trabajos acerca del desarrollo del proceso de minimización aparecen en el
artículo de E. F. Moore (5], que se basa en ideas anteriores de D. A. Huffman [2]. El
capítulo 10 de Z. Kohavi (4] abarca el proceso de minimización para diferentes tipos de
máquinas de estados finitos e·incluye algunas consideraciones de hardware en su diseño.

BIBLIOGRAFÍA

1. Aho. Alfred V.. John E. Hopcroft y Jeffrey D. Ullman. Da1a Stru.ctures and Algorilltms. Rca--
ding. Mass.. Addison-Wesley. 1983.
2. Huffman. D. A .. "The Syn1hesis of Scquen1ial Switching Circuits... Jouma/ of the Franklin
/,utilute 257. núm 3 . págs. 161-190; núm. 4. págs. 27S-303. 1954.
3. Knuth. Donald E., TheArt. ofComputer Programming. 2•. ed.. YOI. 1. Fundamental algorithnu.
Reading. Mass .. Addison-Wesley. 1973.
4. Kohavi. Zvi, Switching and Finite Automata Theory. T. ed.. Nueva York. Me Graw-Hill. 1978.
5. Moore. E. F.. "Gedankcn-experimcnts on Scquen1ial Machines", Automata Studies. Anna/s of
Mathem.aticaf Srudies, núm. 34. págs. 129-153. Princcton, N.J.. Princcton UniYcrsity Prcss.
1956.
6. Stanat, Oonald F. y David F. McAllis1er. Discrete Ma1hematícs in Computer Sciena. Englewood
Cliffs, N.J., Prentice-Hall. 1977.

4. Si al es una relación sobre un conjumo A. demuestre o


EJERCICIOS
mues1re que es falso que 9'12 reflexiva ⇒ 9! reflexiva.
COMPLEMENTARIOS
5. Para losconjumosA. By Ccon relaciones 9l:1 ~Ax B
y 91: ~ B x C. demucs1re o pruebe que es falso que (:?!1 °
Sea A un conjunto e / un conjunto de índices donde. %Y- =21f,ogt,.
ira cada i E / . $,es una relación sobre A. Demuesire o
6. Para cada una de las siguientes relaciones sobre el con-
uebc que es falso lo siguiente.
j unto dado. delennine si la relación es refle xiva, simétrica,
a) l},9...- es reflexiva sobre A si y sólo si cada al, es anti simétrica o lransitiva. También determine si es un orden
reflexiva sobre A. parcial o una relación de equivalencia y. si ocurre esto úhi-
mo. describa la partición inducida por la relación.
b) ~ es reflexiva sobre A si y sólo si cada ~ . es a) S?eslarelaciónsobre Qtalquea3l bsi l a- b] <l.
reflexiva sobre A. b) Sea T el conjunto de todos los triángulos en el pla-
no. Para t 1• t? E T. definimos r13U1 si ! 1• ti lienen la
1 Repita el ejercicio I, reemplazando ··reflexiva" por {i) misma área.
métrica; (ii) antisimétrica; (iii) transitiYa. e) Para T como en la parte (b), definimos ~ como t 1
l. Para un conjunto A. sean at 1 y ~ 2 relaciones simé1ricas 9l' t? si al menos dos lados de 11 están contenidos
breA. Si ~ 1 º~? !í;;; 9l'1 °;1l1• demues1requeg¡ 1°91: =Eih 0 9l' 1• dentro del períme1ro de lz.
Ejercicios complementario$ 397

d ) Sea A= (l. 2, 3,4,5.6. 7}. Definamos9t sobre A


comox9t:ysixy 2: 10.
e) Definimos 91: sobre Z como a 91 b si 71 (a - b).
f) ParaA = (l,2,3,4) X {1,2,3.4).definamos@l
sobre A como (x1. Y1) 9i: (xz, Yi) si ()'1 - x,) =
: (yz - X¡).
7. Dé un ejemplo de un conjunto parcialmente ordenado
con 5 elementos minimales (maximales) pero sin elemento
mínimo (máximo).
8. Para un conjunto A. sea C = (P; 1P; es una partición de
A}. Definamos la rclacióngJ¡. sobreC como P,9'.P1 si P, :s P¡:
es decir. P; es un refinamiento de Pr
a) Verifique que 31 es un orden parcial sobre C.
b ) Para A= { l. 2, 3. 4, 5}, sean P;. 1 :S i :S 4 las Figura 7.25
paniciones siguientes: P 1: (1. 2}. {3. 4, 5}: Pi:
11. 2). {3. 4 ). 15); P,, 11 ). {2}. {3. 4. 5}; P.; {l.
2), {3}, {4}, {5 }. Trace el diagrama de Hasse para 13. Hemos vi sto que podemos utilizar la matriz. de
C = {P,l l :Si :S 4 ), donde C está parcial~nte ady~encia para representar un grafo; esto es ineficiente
ordenado por el refinamiento. cuando hay muchos ceros presentes. Un mejor método para
9. Si el grafo completo K. tiene 45 aristas, ¿cuál es el va- el almacenamiento de dicho grafo se logra mediante la n -
lor de n? prtst,uación de lista dt'adyacencia. q ue se forma con una
lista dt adyacencia para cada vértice u y una lista de iruii-
10. Sea A = {l. 2, 3, 4. 5. 6} x (l. 2. 3. 4. 5. 6}. Defina-
ces. Para el grafo que se muestra en la figura 7.26, la repre-
mos g¡ sobre A como (xi. Ji )~ (xz, yJ si X1Y1 = X2Yz
a) Verifique que 91 es una rclaC'ión de equivalencia
sentación aparece en las dos listas de la tabla 7.5.
sobre A.
b) Determine las clases de equivalencia [( l. 1)].
1(2. 2)). 1(3. 2)) y 1(4, 3)).
11. Sea g = 1/: z·, R }: es decir. 'ttes el conjunto de Tabla 7.5
todas las funciones con dominio z· y codominio R. Lista de índica
Lista de adyacencia
a) Definimos la relación91 sobreScomo g9i:h, para
g. h E 'fJ, si ges dominada por h y hes dominada 1 1 1 1
por g. Demuestre que:,t es una relación de equiva- 2 2 2 4
lencia sobrer:;J. 3 3 3 5
b) Para/ E 'tt. sea [/1 la clase de equiva1encia de f 4 6 4 7
para la relación ~ de la parte (a). Sea-;T' el conjun- 5 1 5 9
to de clases de equivalencia inducidas por 91. Defi- 6 6 6 9
namos la relación Y sobrc9' como {g].:.f(hJ, para 7 3 7 11
{g), (hl E S", si ges dominada por h. Verifique 11
8 5 8
que .(j' es un orck.n parcial. 2
9
e) Para Ji: en la pane(a), sca~/./i,/! E 'gcon/1, /2 E
10 7
l/l Si/ 1+ h: z · ---t R se define como if1+ liXn> =
J1(n) + / 1{n) , paran E z-.demuestre o muestre que
es falso que/1 + h E l/1.
12. Sí A ~ { 2. 4. 8. 20, 28. 56, 112. 224. 336. 672, 1344. 3. Para cada vértice u del grafo, enumeramos (de pre ferencia.
9, 15. 45. 135. 405. 675). la relación~ sobre A dada por x en orden numérico) cada vé niee w adyacente desde u. Así.
9ty si x 1}' es un orden parcial cuyo diagrama de Hasse está para 1, enumeramos 1. 2, 3 como las prime ras tres
estrucluradocomo se muestra en la figura 7 .25. Etiquete los adyacencias en nuestra lista de adyacencia. A continuación
vértices del diagrama para mostrar el orden parcial. ¿De del 2 de la lista de índices colocamos un 4. lo que indica
cuántas formas diferentes podemos ordenar 1opológicamcn1e dónde comenzar a buscar las adyacencias de 2 e:. la lista de
:'~ para obtenér un orden total 'atal que ~ C V ! (A veces se adyxencia. Puesro que hay un 5 a 13 dere.:ha de 3 e n la lista
Wce que ~'11. está inmerso en ff.) de índices. sabemos que la única adyacencia desd~ 2 es 6.
398 Capitulo 7 Reladones, la segunda ,,,.,iu,

Represente cada uno de los grafos de la figura 7.27 de


esta forma.
14. Una representación de lista de adyacencia de un grafo
dirigido G está dada por las listas de la tabla 7 .6. Construya
G a partir de esta representación.

Tabla 7.6
Lista de adya«oda Lista de indices

Figura 7.26 1 2 1 1
2 3 2 4
3 6 3 5
4 3 4 5
De la misma forma, el 7 a la derecha del 4 en la lista de 5 3 5 8
índices nos indica que vayamos a la séplima en1rada de la 6 4 6 10
lista de adyacencia (a saber. 3), para ver que el vértice 4 es 7 5 7 10
adyacente a los "értices 3 (el séptimo vértice en ta lisia de 8 3 8 10
adyacencia) y 5 (el oc1avo vértice en la lista de adyacencia). 9 6
Nos de1enemos en e l vértice 5 debido al 9 a la derecha del 5
en la lisia de índices. Los nueves de la lista de índices que
aparecen junto al 5 y al 6 indican que no hay vértices adya-
centes desde el vértice S. De forma aná1oga, los onces próxi- 15. Sea G un grafo no dirigido con conjunto de vértices V.
mos a 7 y 8 en la lista de índices indican que el vértice 7 no Defina la rclación9i sobre Vcomou ~ wsiu = wo si existe
es adyacente a ningún vértice en el grafo dirigido dado. un camino simple de u a w (o de w a u. puesto queG DOCSÚ
En general, este método nos permite determinar con fa. dirigido). (a) Demuestre que 911: es una relación de equiva-
cilidad los vé:nices adyacentes desde un vértice u. Éstos apa~
lencia sobre V. (b) ¿Qué se puede decir acerca de la parti-
recen en las posiciones índice(u ). índice(u)+ J• •• •• índicc(u ción asociada?
+ 1) - 1, de la lista de adyacencia. 16. a ) Para la máquina de estados finitos dada en la tabla
Fínalmen1e. la última pareja de e lementos de la lista de 7.7. determine una máquina mini mal equivalente a
índices. 8 y I J. es un "fantasma" que indica de dónde arran- e lla.
caría la lisia de adyacencia.si hubiera un octavo vértice en el b) Encuentre una cadena minimal que distinga los
grafo. estados s_. y s(o,

(b) IC)

Figura 7 27
Ejercicios complementarios 399

llbla7.7
V .. e) ¿Cuántas clases de equivalencia existen en la par-
1ición de A inducida por 9t? ¿Cuántos elementos
de A hay en cada clase de equivalencia?
o 1 o 1 d) Gcn«alíce los resultados de las panes (a). (b) y (e).
,, s,
'• l o 20. a ) Para"lt ={l,2.3},sea A ; ~("lt). Definalarela•
,, ,, s, o o ci6n Slt sobt< A corno B Slt Csi 8 s;; C. ¿Cu4ruos
,, o pares ordenados hay en la relación ~?
,. ,,s, s,
,, 1
o o b) RC$ponda la parte (a) para "lt ; {l. 2. 3, 4).
,, ,, s, o o <) Generalice los resullados de las panes (a) y (b).
s, s, ,, o o 21 . Para II E z •. sea ~ = {l. 2. 3 • . ... nJ. Defina la
s, s, s, 1 o relación Slt sobe ~('11) corno A Slt B si A r:l By 8 r:l A.
s, s, s, o o ¿C'\ántos pares ordenados hay en es1a relación?
22. Sea A unconjuntofini10 novacíotalqucB ~A (Bfijo)
17. Ea el centro de c.i.lcuJo, María tiene que ejecutar 10 y IAI ;n. lBI ;m. DefinalarelaciónSltsobr<~(A) como
psogramas que, debido a las prioridades, esttn restringidos X9t Y, pan. X, Y~ A. si X n B = r n B. Entonces9t es una
alas siguientes coodicioocs: (a) 10 > 8, 3; (b) 8 > 7; (e) 7 > relación de equivalencia. como se verificó en el ejercicio 11
l;(d)3>9.6; (e) 6>4, l : (09>4.S: (g) 4,l, l > 2:donde. dclaseccióo7.4.
por ejemplo, 1O> 8, 3 significa que el pro¡rama 1Ose debe a) ¿Cuántas clases de equivalencia hay en la parti-
cjccutarse antes que los programas 8 y 3. Determine un or- ci<ln de 3'(A) inducida por SIi?
den para la ejecución de estos programas de modo que se b ) ¿Cutntos subconjuntos de A hay en cada clase de
IMisfagan las prioridades. equivalencia de ia partición inducida por 9t?

11. a ) Trace el diagrama de Hasse para el conjunto de 23. Para A -1,. 9. sea (A, 8t) un conjunto pard almcntc orde-
divisores enteros positivos del entero n donde n es nado y 8 ,<B !:A 1al que91' ;(8 x 8 ) n Slt. Si (8,Slt') esú
(i) 2: (ii) 4; (iii) 6: (iv) 8; (v) 12; (vi) 16: ( YÜ) 24; totalmente ordenado. decimos que (B. 91:') es una cadena en
(viii) 30; ( ix) 32. CA, 91:). En el caso en que Bes finito . podemos ordcnu los
b) Paracu:alqu.ierl $ns 35, muestre que el diagrama elementos de B como b1 9t' i,z91:' b,91:'.. . gr b...., 9- b. y
de Has.se para el conjunto de los divisores enteros decir que la cadena tiene long irud n. Una cadena (de longi-
positivos de n u: ve como uno de los nueve tud 11) sedcnominama.xima/si nocxistcunelcmentoa EA
diagramas de la parte (a). (no haga caso de los nú- tal que a E lb,. bi,b,. . .. . b.} ya91:bi, b0 9l aob,91:a $tb;. 1,
meros de los v&tioes y coootntrese en la esuuau- para algún 1 :s; ~ n - l.
ra dada por los vtr,iccs y las aristas.) ¿Qut ocurre a) Encucnuc dos cadenas de lon¡itud tres para el 000-
para n • 36? jun10 parcialmente ordenado dado por el dia¡ rama
e) Paran E z.. sea t (n) = el número de divis«cs en- de Hasse de la figura 7. 19. Encucrure una cadena
teros po$itivos de n . (V~ el ejercicio comple- ma.ximal para este conj unto parcialmente ordena--
mentario 33 del capítulo S.) Sean m. ne z· y S, T do. ¿Cuántas cadenas maximales liene?
los OOfljuntos de todos los divisores enteros positi- b) Para el conjunto parcia]men1e ordenado dado pcx
vos de m, n, respectivamente. Los resultados de el diagrama de Hassedc la fi¡u.ra 7.18(d).encucn-
las partes (a) y (b) implican que. si los dia¡ramas tre dos cadenas muimales de dife rente lon¡ itud.
de Hassc de S. T son esuuctura!mente iguales.. cn- ¿Cutl es la longirudde una cadena ma.ximal de lon-
1onces t (m) = t (n). ¿Es cierto el recíproco? girud máxima para este coajunto parcialmente or-
d) Muestre que cualquier diagrama de Hassc de la denado?
pane (a) es un retículo si definimos inffx, y } =- <) Sean'll;{l.2. 3, 4J yA;~('ll). Para el conj un10
mcd(x. y) y sup{,. yJ; m<m(x; y). parcialmente ordenado (A, C ), encuentre dos ca-
19. Para§; (O, 1J.sea,h {x,x,x,x.x,I_. E .f, l s; s lJ.
denas maximales. ¿Cuántas cadenas maximales tie-
a) ¿CuJJ\10 vale L<1? ne este conj unto parciahnen1e ordenado?
d ) Si CU. = {1 , 2. 3 • .. . , n) , ¿cuántas cade nas
b) Para x E A. definimos el ~ de x. que se denota
muimales hay en el conjunto parcialmente orde-
con wt(x). c:omowt(x) =-I.,'.,.x•. [Por lo tanto, wt(x)
es eJ número de unos que aparecen enx.J Defina la
nado (~('11). S:)'
relación 9t sobre A romo x 91: y si wt(x) • wt(y). 24. Para 9f/lC C A,sea(C.~')unacadenamaximalcnel
Verifique que 91: es una relación de equivalencia cooj unto parcialmcn1<onlenado ~ . 9t),donde9t' ; ( C x C)
sobre.A. n si. Si k>selement0$ de Cscordenan c:omoc19t' ci9!' . .. §t'
400 capítulo 7 Relaáones: La segunda VUEita

c., demuestre que e, es un elemento minimaJ en (A, 9t) y que el elemento 6. Determine una anticadena de longi-
c. es maximal en (A, 91). tud máxima para este conjunto parcialmente orde-
nado.
25. Sea (A. 9'1) un conjunto parcialmente ordenado en el
que la longitud de una cadena maximal de longitud máxima b) Si "11 = {I, 2, 3,4), sea A = ~('11). Encuentre dos
es n 2 2. Sea M el conjunto de todos los elementos anticadenas diferentes para el conjunto parcialmen-
= =
maximales de (A,$) y B A - M. Si g¡• (8 x 8) n 9l, te ordenado (A.~). ¿Cuántos demcntos aparecen
demuestre que la longitud de la cadena de longitud máxima en una anticadena de longitud máxima para este:
en (B .:Ji:') es n- l. conjunto parcialmente ordenado?
e) Demuestre que en cualquier conjunto parcialmen-
26. Sea Vl. 91) un conjunto parcialmente ordenado y 0 -# C
te ordenado (A, 91), el conjunto de todos los elemen-
CA. Si (C x C} ngr¡ = 0, e ntonces paracualesquiera:c, y
tos max.imales y el conjunto de todos los elementos
E Cdistintos, tenemos quex~ y y y'fl .x. Decimos que los
minimales son anticadenas.
elementos de C forman una anticaden.a en e l conjunto par-
cialmente ordenado (A, 9t). 27. Sea (A, 91) un conjunto parcia1mente ordenado en d
a ) Encuentre una anticadena con tres elementos en el que la longitud de una cadena de longitud máxima es n.
conjunt o parcia lmente ordenado dado e n el Utilice la inducción matemática para demostrar que los ele-
ctiagramadc Hassedela figura 7.IS(d). Determine mentos de A pueden separarse en n anticadenas C1, C1,
una anticadcna de longitud máxima que contenga C. (tales que C; n C1= 6 para l s i< i :S n).
PARTE

2
TEMAS
ADICIONALES
DE CONTEO
8
El principio
de inclusión
y exclusión

A hora regres;aremos al tema de conteo, analizando el principio de inclusión y exclusión.


Extendiendo las ideas de los problemas de conteo sobre los diagramas de Vcnn del
capítulo 3, este principio nos ayudará a establecer la fórmula que conjeturamos en la sec-
ción 5.3 para el número de funciones sobreyectivasf: A ➔ B, donde A, B son conjuntos
finitos (no vacíos). Otras aplicaciones de este principio demostrarán su naturaleza versátil
en la matemática combinatoria como un método indirecto para los problemas de conteo
que surgen e n muchas situaciones muy diferentes.

8 .1
El principio de inclusión
y exclusión

En e sta sección desarrollaremos algo de notación para enunciar nuestro nuevo principio
de conteo. Después estableceremos el principio mediante un argumento combinatorio.
Los ejemplos mostrarán la forma en que se aplica dicho principio.
SeaS un conjunto tal que IS 1 • Ny sean c 1, c 2, • ••• e, una colección de condiciones o
propiedades satisfechas por algunos, o todos, los elementos de S. Algunos elementos des·
pOdrían satisfacer más de una de las condiciones, mientras que otros podrían no satisfacer
ninguna. Para todo 1 ::s i s r, N(c,) denota el número de elementos de S que satisfacen la
condición e,. (Los elementos de S se cuentan cuando satisfacen solamente Ja condición C; y
tambi6n cuando satisfacen e; y otras condiciones C¡, paraj /; i.) Para cualesquiera i,j E{ 1,
2, 3• . ·.. • t}, tales que i I=- j , N(c, e¡) denotará el número de elementos de S que satisfacen
ambas condiciones e;, c1 y tal vez otras más. [N( c; C¡) no cuenta los elementos de S ques6lo
satisfacen C;, e¡.] Continuando de esta forma. si I s i,j, k :s z son tres enteros distintos,
entonces N(c; c; cJ denota el número de elementos de S que satisfacen, tal vez entre otras,
cada una de las condiciones e, e, y C,:.
Para cada 1 ::s i :s z, N(~) = N - N(c,) denota el número de elementos de S que no
satisfacen la condición e,. Si 1 ::s i ,j s z,coni f.j,N(C¡C¡) denota el núme ro de elementos
deS que no satisfacen alguna de las condicionesc;o e,. [Esto no es lo mismo queN(C;CJ).]
Del diagrama de Venn de la figura 8.1, vemos que si N(c;) denota el número de elemen-
tos del círculo de l lado izquierdo y N(c¡) denota el número de elementos del círculo del
403
404 Capítulo 8 El principio de inclusión y exdusión

lado derecho, e ntonces N(c, c1) es el número de e lementos en el solapamiento. mientras


que N('l;'l¡) cuenta los elementos que quedan fuera de la unión de estos círculos.
En consecuencia, de la figura 8.1,N(l;l1) = N -[N(c,) + N(c;)] +N(c, e;), donde se suma
el último término debido a que fue eliminado dos ve.ces en el t~nnino [N(cJ + N(c1)].
De manera análoga. de la figura 8.2 obtenemos que

N(c.c;c,) = N - [N(c,) + N(c;) + N(c, )]


+ (N(c1c1) + N(c,c,) + N(c;c,))- N(c,c1c,).
A partir del patrón sugerido en estos dos casos, establecemos el siguiente teorema.

TEOREMAS. ! El principio de inclusi6n y exclusión. Consideremos un conjunto S tal que 1 = N y las IS


condiciones e;, 1 :s: i s t satisfechas por algutlos de los elementos de S. El número de
elementos de S que no satisfacen ninguna de las condiciones e,. 1 s i s t, se denota coa
N = N (C1C1 7:1 •• • C, ), donde

Ñ =N - (N(c1) + N(c,) + N(c,) + · · · + N(c,))


+ (N(c, e,)+ N(c,c,) + • • • + N(c,c,) + N(c,c,) + · •· + N(c~, e,)]
- (N(c, c,c,) + N(c,c,c,) + • • • + N(c,c,c,) + N(c,c,c,) + · · ·
+ N(c,c,cJ + · · · + N(c,_,c,_, c,)] + • • · + (-1)'N(c,c,c, .. . e,), (I)
o
Ñ =N - L N(c,) + L N(c,c;) - L N(c,c1c,) + ···
1~ • ISi<fS' 1SK;<.tst
+ (-1)' N(c, c,c, ... cJ. (2)
Demostración: Aunque este resultado puede establecerse mediante inducción en t, dare-
mos aquí un argumento combinatorio.
Para cada x E S, mostramos que .x contribuye con el mismo número, O o 1, a cada ladc
de la ecuación (2).

N(c,c)

Figuni 8.1 Figura 8.2


8. 1 El prinópio de inclusión e,rdusión 405

Si x no satisface ninguna de las condiciones. entonces x se cuenta una vez en N y una


vez en N. pero no se cuenta en los delMS términos de la ecuación (2). En consecuencia. x
contribuye con 1 a cada lado de la ecuación.
La otra posibilidad es quex satisfaga exactamente r de las condiciones, donde 1 s r s
t. En este caso. x no contribuye a N . Pero, en el lado derecho de la ecuación (2). x se
cuenta

1) una vez en N.
2) r veces en ¿ N(c,). (Una vez por cada una de las r condiciones.)
,_,
3) (j veces ,Lcn N(c,c1). (Una,vevpor cada par de condiciones:clegidas

de las rque satisface.)

4) ( ') veces cn ¿ N(c1 c1c,). (¿Por qué?)


3 IM<./<.bl

(r + 1) t) = 1 vez en ¿ N(c,, c,1 ••• e,.), donde la suma se toma sobre todas ;as

selecciones de tamaño r de las t condiciones.


En consecuencia, en el lado derecho de la ecuación (2), x se cuenta

1-r + (j-(;)+ · · · ~ (-l)t)-(1 + (-l)f=CY-Ov~


por el teorema del binomio. Por lo tanto, los dos lados de la ecuación (2) cuentan los
mismos elementos de S y se verifica la igualdad.

Un corolario inmcdfato de este princ:ipio es el siguiente:

COROLARIO 8.1 Según las hipótesis del teorema 8.1, el número de elementos dcS que satisfacen al menos
una de las condiciones e;. donde l :s i s t, csú. dado por N(c I o c 2 o .. . o e,) = N - "R.

Antes de resolver aJgunos ejemplos, analizaremos una notació n adicional para simpli•
ficar el enunciado del teorema 8.1.
Escribimos

S. = N,
S1 • {N(ci) + N(c,) + · · · + N(c,)],
s,- [N(c,c,) + N(c, c,) + •••+ N(c1 c,) + N(c,c,) + · · · + N(c~,<;)J,
y, en genera),

S, =¿ N(c,, c,, .. . e,.), 1 :sk :s1,


06 (apítulo 8 El principio de indusión y exclusión

donde la suma se toma sobre todas las selecciones de tamaño k de la colección del condi-
ciones. Por lo tanto, S1 tiene ÜJ sumandos en ella.

Ahora veremos cómo se aplica este principio para resolver algunos problemas de conteo.

Detenninaremos el número de enteros positivosn tales que 1 < n < 100 y n nocs divisible
entre 2, 3 o 5.
En estecaso,S= {I, 2, 3, ... , 100) y N= 100. Paran ES, nsatisfacc

a) la condición c 1 sin es divisible entre 2,


b) la condición c2 si n es divisible entre 3, y
e) la condición e, sin es divisible entre 5.

Entonces, la respuesta de este problema es N(C 1C2 C3 ).


Como en la sección 5.2, usamos la notación LrJ para denotar el máximo entero menor
o igual que r, para cualqlller número real r. Esta función es útil en este problema. ya que
vemos que
N(c1) = lt00/2J = 50 [puesto que los 50(= lt00/2J) enteros positivos 2, 4, 6, 8, ... ,
98(= 2 • 49), 100(= 2 • 50) son divisibles entre 2);
N(c,) = LI00/3J =L33 J/3 J = 33 [puesto que los 33(= lt00/3J) enteros positivos 3, 6, 9,
12, ... , 96(= 3 • 32), 99(= 3 • 33) son divisibles entre 3);
N(c,) = ll00/5J = 20;
N(c1c,) =LI00/6J= 16[puestoquehay 16(=LHJ0/6J)elementos enSque son divisibles
entre 2 y 3, y por tanto divisibles entre mcm(2, 3) = 2 · 3 = 6);

N(c,c,) = LHJO/lOJ = 10;


N(c,c,) = LHJO/lSJ = 6;
y, N(c1 c,c,) = l l00/30J = 3.

Si aplicamos e l principio de inclusión y exclusión, tenemos que

N(c1 c,c,) = s0 -s1 + S, - S,=N - [N(c1) + N(c,) + N(c,))


+ [N(c1 c,) + N(c1 c,) + N(c,c,)]-N(c,c,c,)
= 100 - [50 + 33 + 20] + [16 + 10+ 6]-3 =26.

(Estos 26 nGmeros son 1, 7. 11, 13, 17, 19, 23, 29, 31, 37, 41, 43, 47, 49, 53, 59, 61, 67, 71,
73, 77, 79, 83, 89, 91 y 97.)

En el capítulo I encontramos el número de soluciones enteras no negativas de la ecuación


x1 + Xz + x 3+ x, = 18. Respondere mos ahora la misma pregunta con la restricción adicional
x; .s 7,paratodo 1 :s; is 4.
Ses el conjunto de soluciones de x 1 + x 1 + x3 + x, = 18, con O :S x, para todo 1 :S i :S 4.
Así, lsl = N= ( •~~- •) = G!).
8 .1 El principio de inclusión y exdusión 407

Decimos que una solución x .. x2• x1 • ~ satisface la condición C;, donde 1 :S i :S 4, si


x, > 7 (o x. 2 8). La respuesta al prob1ema es entonces N(C1C2C1CA).
Por simetría, N(c,) = N(c,) = N(c,) = N(c,). Para calcular N(c,), consideramos las solu-
ciones enteras de x 1 + ~ + .r1 + .r,. -= 1O, donde cada .r; .!: Opara toda 1 :S i s 4. Eotooces
sumamos 8 al valordex 1 y obtenemos las solucionesde.r1 +~ +.r3 +x.,. :;:; IS que satisfacen
la condición c 1• Por lo tanto, N(c¡) :;:; ( 0 : • 1):;:; (:!), para todo l :Si :S 4, y S 1 = (~){:!).
Del mismo modo, N(c1 c1) es el nlUilero de soluciones enteras de Xi + ..t2 + .r1 + .r.. = ~
donde z, ;,, para
O, todo 1 :S i :S 4. Así, N(c, e,) =(º/•')= (D y S, (1X\).
=
Como N(c; ci cJ = Opara cualquier selección de las tres condiciones y N(c1 c 2 c,cJ = O,
tenemos que

N(c,c,c,c,) = N -s, +s,-s, + s,= (ll)-(:Xg) + (lXl)-o +0=246.


Asf. de ]as 1330 soluciones enteras no negativas dex1 +.xi +.r3 +.r,. = 18, s6lo246de ellas
satisfacen ..t1 s 7 para todo 1 :s: i s 4.

Nuestro siguiente ejemplo establece la fórmula que conjeturamos en la sección S.3 para
contar las funciones sobreyectivas.

= =
Para conjuntos finitos A. B, tales que IA1 m 2: n 1B 1, sean A= [a., a,, . .. ,a,,}, B =
[b., b,,..., b.) yS el conjunto de todas las funciones/: A-> B. Entonces N lsl rf". = =
Para todo 1 s i s n, sea e; la condición sobre S tal que una función/: A ➔ B satisface
e; si b; no está en la image n de f (Observe la diferencia entre e; en este caso y c1 en los
ejemplos 8.1 y 8.2.) Entonces N( Q) es el número de funciones en S que tienen b; en su
imagen y N(C 1C2 • • • c.) cuenta el número de funciones sobre/: A ➔ B .
Para cualquier 1 sis n, N(c1) = (n- I Y-, ya que cualquier elemento de B. excepto b;.,
puede usarse como segunda componente de un par orde nado para una función/: A ➔ B,
cuya imagen no incluye b,- De la misma forma. para cualquier 1 s i < j s n. existen (n - 2)-
func iones/: A ➔ B cuya imagen no contiene b1 ni b-. De estas observaciones obtenemos
= = =
queS, {N(c,) +N(c,l + · - · +N(c.)] n(n - 1)" (;)(n-1)" y S, = [N(c,c,) +N(c,c,) +
· · · +N(c,c.) +N(c,c,) + · -- + N(c,c,) + · · · + N (c,_ ,c.)] = (;)C
n-2)".En general,para
todol sksn,

s, = L
l"i1<lz<·· · ~
N(c,, e,, -.. c,J = (nk )en -k)"
El principio de ínclusión y exclusión implica que el número de funciones sobre de A a Bes

N(c,c,c, ... e,) = N -s, +s,-s,+ -• •+ (-l)"S.

=~-t~-~+~~-~-~~-~
+ · · · + (-l)"(n -n)"= ±(-1y(n)(n -i)"
1

=ic-1/
,-o \n n- ,.)cn -i)".,..
I08 capítulo 8 El principio de indusión y exdusión

Antes de terminar de analizar este ejemplo, notemos que

±c-il\n n- ,.)<n-i)'"
,-0

también puede evaluarse en el caso m < n. Además, para m < n, la expresi6n


N(c, ;;,e, ... c.)

sigue conrando el nwnero de fuaciones / : A ➔ B, rales que IA 1 = m, 1B 1 = n y cada


elemento de B está en la imagen de f. Pero ahora este nWJlero es O.
Por ejemplo, supongamos que m = 3 < 7 = n. Entonces N(C1C2 C3 ••• C1 ) cuenta el nll-
mero de funciones sobre/: A ➔ B para IA 1 = 3 y I B 1 = 7. Sabemos que este nómero es O,
y también encontramos que

±
,.. c-1Y (l,)(1 - í)' = G)7' - (l)6' + (l)s' - G}4' + (l)3'- (l)2' +-(D 1'- (!)o'
= 343-1512 + 2625 - 2240+ 945 - 168+ 7-0=0.
Por lo tanto, para cualquier m,. n E z•, si m < n, enton~

±c-il\n n- ,.)(n -i)'"=0.


,-0

Resolveremos ahora un problema similar a los del capítulo 3, rel3e:ionado con los
diagramas de Venn.

¿De cuántas formas pueden permutarse las 26 letras del alfabeto de tal manera que ningu-
no de los patrones sea car, dog, pun, o byte?
Sea Sel conjunto de todas las permutaciones de 26 letras. Entonces IS 1 = 26!. Para
texto l s i s 4 , una permutación ~ S satisface la condición e; si la pennutación contiene·
ca,; dog, pun o byte, respectivamente.
Para calcular N(c,), por eje mplo, contamos el número de formas en que pueden
permutarse los 24 símboloscar, b, d, e,f,. ,p, q, s, t... . , x. y. z. Asf,N(c1)::;;:24! yde una
manera similar obtenemos

N(c,) = N(c3) = 24!, mientras que N(c,) = 23!

ParaN(c1 c2), trabajamos con los 22 símbolos ca,; dog, b, e, f, h, i, . . . , m. n. p. q, s. t, .


x, y, z, que pueden pennutarse de 22! fonnas. De aquí obtenemos que N(c1 c0 = 22! y con
otros cálculos semejantes obtenemos
N(c,c,) = N(c,c,) = 22!, N(c,c,) =21!, i +4.
Además,
N(c,c,c,) = 20!, N(c,c;c,)= 19!, lsí<js3,
N(c, c,c,c,) = 17!
8.1 El principio de indusión y exclusión 409

Así, el número de permutaciones de S que no contienen ninguno de los patrones dados


es N(c,c,c,c,) = 26! - [3(24!) + 23!) + [3(22!) + 3(21)!)- [20!) + 3(19)!1 + 17.

Nuestro siguiente ejemplo trata un problema de teoría de números.

Para n E Z♦• n > 2, sea 4>(n) el número de enteros positivos m. tales que I < m < n y
mcd(m, n) = 1, esto es, m, n son primos relativos. Esta función es lafunciónphi ck Eulery
surge en varias situaciones del álgebra abstracta relacionadas con la enumeración. Encon-
= = = = =
tramos que«2) l , ~(3) 2, ip(4) 2, ip(5) 4 y ip(6) 2. Para cualquier primop, +(p)=
p- l. Queremos obtener una fórmula para«n) relacionada con n para DO tener que hacer
una comparación caso por caso para todo m, 1 ::s m < n, con el entero n.
"- La deducción de nuestra fórmula usará el principio de inclusión y exclusión, como CD
el ejemplo 8.1. Procedemos como sigue: paran 2 2, usamos el teorema fundamental de la
aritmética y escribimos P1 p'l ·· · p': , donde Pi, pz, . . . , Pi son primos distintos y t 1 ~ 1,
para tcxio 1 ::si :s t. Consideremos el caso en quet= 4. Esto será suficiente para demostrar
la idea general
Si S= {!, 2, 3, ... , n), tenemos N= lsl y para tod~ 1 :si :s 4,decimos quek e S
satisface la condición c1 si k. es divisible entre p,. Para 1 ::s k < n. mcd(k. n) = 1 si k no es
divisible entre cualquiera de los primosp1, donde 1 ::si :s 4. Por lo tanto«n) = MC1C2C3C.- ).
Para todo 1 :s i :s 4, tenemos N(cj = nlp;; N(c;C¡) = nl(p;p;). para todo 1 :s i <j :s 4.
También,N(c;c1 cJ =nl(p;p1pJ, para todos 1 :si <j < 1 :s 4, y N(c, c,c,c,) = nl(p,p.,p,pJ.
Así,

«n) =S.-S, +S, -S, +S,

=• - [;.+ ···+;} [ p~p, +p:, +···+ p:j


n n _l n
- [ p,p,p, + ... +p,p,pJ +p ,p,p,p,
=n[1-(2.+ . ·
Pi
+.!..) + (_.!.._+-
P◄ P1Pl
1
PiPJ
1
-+ •· · +--)
PlP◄

- ( p ,~p, + . .. + p,~p.)+ p¡p,~,p.]


= __n__ [p,p,p,p, - (p,p,p, + p,p, p, + P•PiP• + p ,p,p,)
p,p,p, p,
+ (p,p, +p,p, +p,p, + p,p, + p,p, +p,p,)
- (p, + p, +Pi+ p,) + !)
=--•-[(p1 - !)(p, - l)(p, - l)(p, - !))
P1P2PJP,
=n [P•- 1 . p, - 1 . p, - 1 . P• - !]
= n TI ( 1 - . !. ).
• Pi Pi Pl P• 1- 1 p¡
110 úpítulo 8 El principio de indusión y exdusión

Prograa EulerPhiPunction ( input, output);


Var
i. j . k. n , phi, originalvalue: integer;
Begin
Wri te ( • El valor de n es ' ) ;
Read ( n):
phi := n;
originalvalue := n ;
Ir n llod 2 = O then
Begin
phi : = phi Div 2;
Wbile n Mod 2 = O do
n:=n0iv2
End;
I t n llod 3 • O then
Begin
phi := ( phi• 2 ) Div 3;
llhile n llod 3 = O do
n : - n0iv3
End;
i := 5;
While n >= 5 do
Begin
j : = l;
Repeat
j: - j+l;
k : =illodj
Until ( k =
O) or (j - trunc(sqrt(i)));
If (k <> O) and ( n llod i =
O) then
Begi n
phi := (phi• (i - 1 )) Div i;
llbile n llod i - O do
n : = n ~iv 1
End;
1 := 1 + 2
End ;
Write('Para n= ' , o riginalvalue : O, · existen ', phi: O);
writeln( • números menores que •, originalvalue : O);
writeln( · y primos relativos a él. •)
End.

El v alor de n es 131
Para n :::: 131 existen 130 númer os menores que 131
y primos relativos a él.

El valor de n es 31500
Para n = 31500 existen 7200 números menores que 31500
y primos relativos a él.

El valor de n es 1 98000
Pa ra n = 1 98000 existen 48000 númer os menores que 198000
Figur,1 &.3 L-Y_ P_r _im
_o_s_re_l_a_t_i_vo_s_a_é_1 ._ _ __ _ __ __ _ _ _ _ _ _ __,
8.1 El principio de indu"6n y exdu<ión 4 11

En generaJ, «n) = n íl,1. (1-(1/p)), donde el producto se toma sobre todos los primos
p que dividen a n. CUando n = p, un primo, «•>
= «P) = p(l - (1/p)] = p - 1, como
observamos antes. Paran= 23,100 encontramos que

«23,100) • «2' · 3 · S' · 7 · 11)


• (23,100)(1- (1/2))(1 - (1/3))(1- (1/5))(1- (1/7))(1 - (1/11))
• 4800.

El programa en Pascal de la figura 8.3 evalúa «n) paran ;, 3. En este caso, usamos la
esuuccura de control Repeat~Until para determinar si el entero i es un primo. La condición

j :: trunc (aqrt ( i ))

para terminar la ejecución de este ciclo se sigue del resultado del ejemplo 4.28.
Usamos este programa para encontrar ~n) en los casos n = 131, n = 31,500 y n =
198,000.

La función phi de Euler tiene muchas propiedades interesantes, algunas de las cuales
serán analizadas en los ejercicios de esta sección y en los ejercicios complementarios.

Nuestro ejemplo final retoma la teoría de grafos que estudiamos en el capítulo 7.

En ciena región hay cinco villas. Un ingeniero tiene que diseñar un sistema de carreteras
de dos sentidos de modo que al terminar el sistema, ninguna villa quede aislada. ¿De
cuántas formas puede hacerlo?
Si llamamos a las villas a. b, e, d y e, buscamos el mlmcro de grafos no dirigidos sin
lazos sobre estos v6rticcs. de modo que no haya v6rtices aislados. En consecuencia, quere..
mos contar situaciones como las que se ilustran en las panes (a) y (b) de la figura 8.4. pero
no como la que se muestra en la parte (c).

..
·i. :
a..________ b

(a)

Figura 8.4
d~ e

(b)
<1:
(e)
12 (.apitulo 8 El principio de indusión y exdusión

Sea Sel conjunto de grafos no dirigidos sin lazos G sobre V= (a, b, e, d.,¡. Entonces
1S 1 = 2 10 pues hay (~)= 10 carreteras posibles dedos sentidos entre estas cinco villas y
cada carretera puede incluirse o excluirse.
Para cada 1 s i :s 5, sea e, la condición de que un sistema de estas carreteras aísle las
villas a, b, e, d y•·
respectivamente. Entonces la respuesta al problema es N(r,r,r,r,r,).
Para la condición ci. la villa a está aislada, por lo que tenemos que analizar las seis
aristas (carreteras) {b, e}, {b, d}, {b, •J. ( c,d}, (e,,}, {d, •J. Tenemos dos opciones para
cada arista (a saber, poner la arista en el grafo o dejarla fuera), po< lo que N(c 1) = 2'. Pe<
simetría, N(c.) = 2° para todo 2 s i :s 5, y S1 = (:}2°.
CUando se aíslan las villas a y b, cada una de las aristas (e, d), {d, ,¡, {e, •J puede
poncru o no en el grafo. &to produce 23 posibilidades, po< lo qucN(c1 e,)= 2' y S, = (02'.
Usamos argumentos similares paraobtenerN(c, c1c,) =21 y S, =- (n21; N(C1Clc)cJ='l4
y S, = (!)2'; y N(c1 c,c3 c,c,) = 2°y S, = {:)2°.
En consecuencia. N(r1r 2 l ,l,l,) =2 1º- (D:26+ C)21 - (:)21 + (!)2°- {!}2°= 768.

JERCICIOS 8.1 1. Determine el mlmcro de enteros positivos n, l s n :s 2000, tales que


a) DO son divisibles entre 2. 3 ni S. b) no son divisibles entre 2. 3, S ni 7.
e) no son djvisibles entre 2,.3 ni s. pero sf soo divisibles entre 7.
2. Dctermi.ne el n11mcro de solucioocs enteras de x1 + ..r2 + x, + .r.. • 19, si
a) O .s x.; para todo 1 s; s 4 b) O :s Zi< 8 para todo J si :s 4
tj0:S~ S ~O:s~s6.3:s~ :s7, 3 :s ~ :s8.
3. ¿De cuánw formas se pueden colocar todas las letras de la palabra INFORMATION de tal
maneraquelUngdn par de lcuas consecutivas aparezca mis de una vez? [Aquí ~ ~
tar disposiciones como IINNOOFRMTAy FORTMAIINON pero no INFORINMOTA (donde
"'IN" aparece dos veces) o NOR'IFNOIAMI (donde " NO" aparece dos veces).]
4. Determine el mlmerode solucionesenterasparax1 +x1 +x,+x.= 19donde -5 S ~s JO para
todol s is 4.
5. Encuentre el mlmerodceoteros positivout&Jesqucx :S 9,999.999 y lasumadelosd:fgitosdc
xsca igual a 31.
6 . El profesor Ballesteros acaba de escribir el examen final para su curso de matemtticas avanza-
das para ingeniería. Este examen tiene 12 pre¡untas, que en total valen 200 puntos. ¿De cuw:'.1-
tas maneras puede asignar el profesor Ballesteros los 200 puntos si (a) cada pregunta debe
valer al DlCDO$ 10, pero no mis de 2S puntos?,(b) cada pregunta debe valer a1 menos 10, pero
no más de 25 puntos y el valor de los p.mtos para cada pregunta debe ser mdltiplo de 5?
7. En su tienda de flores, Margarita desea colocar 1S planw diferentes en cinco anaqueles dcJ
escaparate. ¿De cuántas formas puede colocarlas de ta1 manera que cada anaquel tenga al
menos una planta. pero no m'5 de aiatro?
8. ¿De aiinW formas puede seleccionar Camilo nueve canicas de una boba coo doce (idmticas
cxccpco Por el color), donde tres son rojas, tres azules. tres bbncas y tJ'C$ verde$?
9. Encuentre el m.1mcro de pennutaciones de a. b, e, .. . . x. y. t. de modo que no aparezcan los
pacrones spin, gam.e, path. o n~t.t
1O. Responda la pre¡unta del ejemplo 8.6 para el caso de seis villas.
11. Si se tiran ocho dados distintos, ¿cuQ es la probabilidad deque aparucan seis m1meros distintos?

t Se C011Sidcran aquf las 26 leuu del alfabeto ing)& (N. del R.T.)
8.2 Generalizaciones del principio 413

12. ¿CUántos DWnCn>$ de la seguridad socw (SCCUCllcias de nueve dígitos) tienen al menos una
vez cada uno de los dígil0$ 1, 3 y 7?
13. ¿De cu.intu formas se pueden oolocar tres .x. tres y y tres z de modo que no aparezca la misma
letra tres veces oonsec:utivas?
14. El municipio de Frostburg tieoe a su cargo cuatro tropaS de scows, cada una con 20 ni6os. Si
el jefe de tropa selecciona SO de esu>s nittos para repcesentar a su municipio en la reunión de
scocas del estado, ¿c:u.'1 es la probabilidad de que su sdccci6o incluya al menos un nifio de
cada una de las cuatro tropas?
15. Si Zacarías tira un dado cinco vca:s, ¿cuál es la probabilidad de que la suma de sus cinco
tiradas sea 20?
16. En un curso de matem1ticas de 12 semanas, Soledad encontró a siete de sus amigas del co1e--
¡io. Durante el curso, dla encontró a cada amip 3S veces dunnte la comida. a cada par de
ellas 16 voces, a cada tema ocho veces, a cada cu.ate:ma c:u.atro veces,. a cada ooojunto de cinco
dos ve.ces, y a cada conjunto de seis una vez.. pero nunca a las siete aJ mismo lietr'.po. Si comió
durante los 84 días del curso. ¿comió alguna vez sola?
17. Calcule 9(n) paran ;gua! a (a)5 1; (b)420; (c)l2300.
18. Calcule 9(n) paran ;gua1 a (a) 5186; (b) 5187; (e) 5188.
19. Sean E z•. (a) Dctcmrine .in. (b) Dctcmrine+(l"p), doodep es un primo ;mpar.
20. ¿Para cuiles n E z· es +(n) impar?
21. ¿CU.á.otos enteros po$itivosn menores que 6000 (a) satisfacen mcd(n, 6000) a I? (b)compat•
ten un divisor primo con 6000?
22. Si m. n E z•, demuestre que+(n-) • ,;-- 1+(:n).
23. Encuentre ues valo= paran E z· tales que .in)= 16.
24. ¿Para~ enteros positivos n ocurre que t(n) es una potencia de 2?
25. ¿Para~ enteros positivos n ocurre que 4 divide a ♦(n)?
26. Extie.nda el programa de la figura 8.3 de tal manera que funcione para el caso n • 2 y que
imprima UD resultado gramaticalmente corrt.eto.
27. Para el programa de la figura 8.3, ¿cu'1 es d propósito de los aes ciclos Wbile de dos Uneas?
¿Por qu~ i se iocrcmenta en 2 (y no en 1) en la ICoea i: = i + 2? (¿Se observa alguna diferencia
si i se incrementa eo 1?)

8.2
Generalizaciones del principio

Consideremos un conjunto S coo IS1 = N y condiciones e,. e,, .. .• e, satisfechas por


alguno de los e1ementos de S. En la sección 8.1 vimos cómo la inclusión y exclusión nos
proporcionan una manera de determinar N(c1C1 ••. C,). el número de elementos de S que
no satisfacen ninguna de las r condiciones. Si m E Z· y 1 s m s t, queremos detenninar
ahora E•• el número de elementos de S que satisfacen exactamente m de las t condiciones.
(Ahora podemos ob!encr E;,.)
Podemos escribír ecuaciones como

E 1 = N(c, -e,"lf, . .. "l!,) + N("l!1 c,c3 •• • ~) + · · · + N("/!1 "1!,c, ... ~-,<,),


y
E,= N(c,c,"l!, .. . "/!.) + N (ct"lf,c, .. . "/!.) + · · · + N(c,"lf,"l!, .. . "/!Hc,..,c,),
14 Capitulo 8 El principio de inclusión y exdUSK>n

y aunque estos resultados no nos ayudan tanto como quisi~ramos, serán un punto de par•
tida útil cuando examinemos los diagramas de Venn para los casos t = 3 y 4.
En la figura 8.S, dondet= 3. colocamos una condición numerada a1 lado del círculo que
representa los elementos deS que satisfacen una condición particular. Entonces E, es igual
al número de elementos en las regiones 2, 3 y 4. Pero tambi~n podemos escribir

E, = N(c1} + N(c,} + N(c,}-2[N(c1 c,} + N(c1 c3) + N(c,c,}] + 3N(c1 c,c,).

En N(c1) + N(c1) + N(c,i) contamos los elementos de las regiones 5, 6 y 7 dos veces y los de
la región 8 rres veces. En el siguiente tlrmino, los elementos de las regiones S, 6 y 7 se
eliminan dos veces. Quitamos los elementos de la región 8 seis ve<:es en 2[N(c1 c1 ) +
N(c1 c3 ) + N(c1 c1)], y desputs los añadimos en el l6rmino 3N(c1 c2 c1); por llltimo, no con-
tarnos ninguno de los elementos de la región 8. Por lo tanto, tenemos E1 = S1 - ~ + ~;:;
s, -(l~ + ilfS,.

Cr• 3)
Figura 8.5

En el caso ~ E:. nuestra ecuación anterior indica que queremos contar los elementos
de Sen las regiones 5, 6 y 7. Del diagrama de Venn.

E,=N(c1c,} + N(c1 c3} + N(c,c,}- 3N(c1 c,c,} =S,-3S, = S,- (l)s,,


y
E,= N(c,c,c,) =S,.

En la figura 8.6, las condiciones c.,~ e, están asociadas con subconjuntos circulares de
S. mienuas que e, está asociada con el área de forma bastante irregular constituida por las
rcgiones4,8, 9, 11, 12, 13, 14y 16. Paracad.a Is i :s 4, E, c:stádcterminadacomosigue:
E, [regiones 2, 3, 4, 5]:
E,= [N(c1) + N(c,) + N(c,) + N(c,)]
-2[N(c1 c,) + N(c 1 e,}+ N(c1 c,} + N(c,c,) + N(c, c,} + N(c,c.)]
+ 3(N(c1c1 c,} + N(c1 c,c.} + N(c1 c,c.) + N(c,c, c,}]
• -4N(c,ezc,c,)
= S1 -2$, +3S, - 4S, ~ S, -(l)s, + (!)s,-(l)s,.
8.2 Generafaaóones del principio 415

(t• 4) e,
Figura 8.6

[Nota: Si tomamos un elemento de la región 3. encontramos que se cuenta una vez en E1 y


una vez. en S1 [en N(c,.)). Si tomamos un elemento de la región 6, vemos que no se cuenta
en E,; se cuenla dos veces en S 1 (en N(c,) y en N(c,)] pero.se elimina dos veces en 2S, [ya
que se cuenla Wlll vez en S, en N(c,c,)J. por lo que al final no cuenta. El lectoc debería
examinar ahora un elemento de la región 12 y uno de la región 16 y mostrar que cada uno
conttibuye en Oa ambos lados de la fórmula para E,.]
E, (regiones 6-11):
De la figura 8.6, E,= S, - 3S, + 6S, = S, - füS, + S)S,. Para más detalles sobre esu
fórmula. examinemos los resultados que aparecen en la tabla 8.1. donde junto a cada su-
mando de Sz. S3 y S4 enumeramos las regiones cuyos elementos se cuentan para detenninar
ese sumando en particular. Para calcular S, - 3S> + 6S,, encontramos los elementos de las
re¡jones 6 a 11, que son pr,cisameute los que se cuentan para E,.

Tobla 8.1
s. s, s.
N(c1 c,): 1, 13, 15, 16 N(c1c,c,): 15, 16 N(c1 c,c,c.): 16
N(c1 e,): 10, 14, 15, 16 N(c,c,c.): 13, 16
N(c1 c,): 11, 13, 14, 16 N(c,c,c.): 14, 16
N(c,c,): 6, 12, 15, 16 N(c,c,c.): 12, 16
N(c,c.): 8, 12, 13, 16
N(c,c.): 9, 12, 14, 16

Por último, los elementos de E, sceocuentran en las regioocs 12a 15 y E1 = Si -4S4 =


S,-r,>5,; los elementos para E. son los de la región 16 y E,• S,.

Estos resultados nos llevan al siguiente teorema


416 Capitulo 8 El princip.O de indusión y exclusión

IEOREMA8.2 De acuerdo coa las hipótesis del teorema8.l, para cada 1 ::s m ::s r, el número de elementos
de S que satisfacen exactamente m condiciones c 1, c2 , ••• , e, está dado por
ER = SR - {Rt'lS~+t + {R!')s~.,- · · · + (-l)~i.!~)s,. (1)
(Si m = O, obtenemos el teorema 8. 1.)
Demostración: Argumentando como en el teorema 8.1, sea x ES y examinemos los si-
guientes tres casos.
a) Cuando .x satisface menos de m condiciones, contribuye con O a cada uno de los
términos E,.,., S,., s... 1, ••• , S, por lo que no se cuenta en cada lado de la ecuación.
b) Cuandox satisface exactamente m de las condiciones, se cuenta una vez en E,. y t1111
vez en S.,, pero no en s.... 1, ••• , Sr En consecuencia. se incluye una vez en la cuenra
para cada lado de la ecuación.
e) Supongamos que .r s_atisface r de las condiciones, donde m < r :5 t. Entonces x DO
contribuye a E•. Se cuenta(;) veces en S., (,.:1) vecesenS,..i, ... , y(;) veces ea
S,. pero Oveces en todos los túminos posteriores aS,. Por lo tanto, del lado dcrtchode
laecuación,xsecuenta (~)- ( Rt'}(.:,)+ (R;')(.:,) - ··· +(-ly-•(...'..) (;) v-.
ParaOsksr-m,
m+k)( r ) (m+k)! - r!
( k +k k!m! +k)!(r-m-k)!
m (m
r! 1 r! (r-m)!
m! k !(r - m - k)! =-m!(r-m)! k!(r-m - k) !
- - - . --'---'--

=(:)('~m).
En consecuencia, del lado derecho de la ecuación (1 ), x se cuenta

(:)('~m)-(:)('~m) +(:)('~m)-... +(-1r(:t=:)


=(:)[('~m)-('~m)+('~m)- ___ +(-ir (;=:)]
: (:)11- 1rR: ( : ) " o: o VOCCS,

con lo que se verifica la fórmula.

Con base en este resultado, si L. denota el número de elementos de S (segt1n las hipóte-
sis del teorema 8.1) que satisfacen al TMnos m de las t condiciones, obtenemos la ·fórmula
siguiente.

:OROLARIO 8.2 LR= s. - (.~,)s••,+ {:!l)sR., - · · · + ( - 1r"(!:1,)s,.


Demostración: La demostración se bosqueja en los ejercicios del final de esta sección.
8.2 Generalizaciones del principio 417

Si m = 1, el resultado del corolario 8.2 se convierte en

L, • S, -(~}s.+@s,-... +c- 1y-'(' ~ 1)s,


- s, - s,+s,- ... + (-1y- s,. 1

Si comparamos esto con el resultado del teorema 8.1, encontramos que


L, = N-N•jSj-Ñ.
Este resultado no nos sorprende, ya que un elemento :r de S se cuenta en L 1 si satisface al
menos u.na de las condiciones Ci. c1 , e,, ...• e" es decir, si x E S y x no se cuenta en N =
N(C1C2 C, . .. C,).

Si regresamos al ejemplo 8.6, encontraremos las cantidades de sistemas de carreteras de


dos sentidos de tal fonna que exactamente (Ei) y al menos (LJ dos de las villas permanez-
can aisladas.
Los result.ados calculados previamente para este ejemplo muestran que

E, = s,- (l)s, + (l)s, - (l)s,- 80- 3(20) + 6(5) -10(1) • 40,


r,, = s,- (l)s, + (l)s, - (l)s, = 80 - 2(20) +"3(5)- 4(1) = 51.

EJERCICIOS 8. 2 1. ...,. 1a situación de tos ejemplos 8.6 y s.1. ca1cuJeE. - os ; s sy muestre que I.'..e.. ". 1sI.
2. •l ¿De awiw formas se pueden colocar las leu.s de ARRANGEMENT de modo que bayo
exactamente dos pares de lct:ru id&tticas consecutivas? ¿al menos dos pares de letras idm-
ticas consecutivas?
b) Respooda la parte (a). reemplazando dos con t =
3 . ¿De cuinW formas podemos colocar la, lelras de CORRESPONDENrS de modo que (o) no
haya un par de letras id!:nticas consecutivas? (b) baya exacwneotc dos pares de letras idmticas
consecutivas? (c) baya al menos tres pares de letras idmtieas consccutivas?
4. SeanA= {1.2, 3, ., . , 10) yB• {l . 2,3, ... , 7}.¿0linlasfunciones/:A ➔ B satisf-
1.flA)i =4? ¿Olinw satisfacen 1./lAll s 4?
S. ¿De cuántas formas se pueden distribuir diez premios distintos entre cuatro estudiante5 de
modo que exacwncntc dos escudiantes no reciban ninguno? ¿De cuánW fonnas puede hacene
esto de modo que al menos dos estudiantes DO reciban premio?
6 . Silvia organiza un almuerzo para ella y nueve de las mujeres de su dub de tenis. En la maftana
del almueno, coloca las tarjetaS 000 los oombres en los diez lugares de su mesa y sale sólo por-
diez minutos. Su esposo Heberto llega a casa desputs de su juego de tenis y, por des&racia.
deja la puerta abierta. Un viento repentino hace volar las diez. tarjetas. ¿De cuánw íonnas
puede Hebcrto volver a colocar las diez tarjcw en los lugares de la mesa de modo que exacta•
mente cuatro delas mujeres se sienten donde Silvia qucria? ¿De ClWlW formas oc:urririque al
menos cuatro de ellas se sienten donde deberfan?
7 . Si se extraen 13 cartas de una baraja com11n de 52, ¿cuál es la probabilidad de que esw 13
urtas (a) incluyan al menos una cana de cada palo? (b) excluyan exactamente un palo (por
ejemplo, trtboles) (c) excluyan exactamente dos palos?
8. El siguiente es un bosquejo de la cletoo>Uación del corolario 8 .2. Es=l>a los detalles necesarios.
a) Primero observe que E.• L.= S,.
118 capítulo 8 El principio de indusión y exdusión

b) ¿Cumto vale E. . 1? y ¿cómo se relacionan L, y L,.1?


e) Muestreque~_,:::S,.i(!:n-s,.
d) Para 1 :s m :s t - 1, ¿cómo se relacionan 4., 4.. 1 y E.1
e) Use los resultados de los pasos (a) a (d) para cstable.ccr el corolario mediante un tipo de
inducción hacia atris.

8 .3
Desórdenes: Nada está
en el lugar correcto

En el cálculo elemental encontramos que la serie de Maclaurin de la función exponencial


eslá dada por

de modo que

,-•=i"-o (-n!1)" =1 -1 + .!.2! _.!.+


3!.
· ··

Hasta cinco cifras decimales, ,,-, = 0.36788 y 1 - 1 + (1/2!) - (1/3!) + · · · - (1./7!) ~


0.36786. En consecuencia. paracualquierk. E Z .., si k :! 7 , entonces e-1 es una muy buena
aproximación a r :..((- 1)')/n!.
Estas ideas nos serán de utilidad en los siguientes ejemplos.

En un hipódromo, Rafael apuesta que cada uno de los diez cabalJos de una carrera llegará
a 1a meta precisamente como dicen los pronósticos. ¿De cuántas formas podrán llegar a la
meta los caballos de modo que pierda todas sus apuestas?
Si quitamos las palabras caballos y ca"era del problema. lo que realmente queremos
saber es el número de formas en las que podemos colocar los números 1, 2, 3, ... , 10 de
modo que el l no esté en el primer lugar (su Jugar natural), 2 no esté en el segundo (su
posición naturiJ), .. . y 10 no esté en el décimo lugar (su posición natural). Estas disposi-
ciones son losdes6rdenu de 1, 2, 3, ... , 10.
El principio de inclusión y exclusión es la clave para calcular el número de desórdenes.
Para cada 1 s i s 10, un orden de l. 2, 3, .. . , l 0satisface lacondici6nc,siel enteroiesü
en el lugar i-ésimo. Obtenemos el número de desórdenes, d1o,, como sigue:
d,. =N(c, 1:,1:, ... 1:,.) = 101 - ('/')9! + ('l')8! - ('1')7! + • • • + (:S)o!
= 10![1 - ('/')(9!/10!) + ('J')(S!/ 10!) - ('¡')(7!/10!) + · · · + (J::){0!/ 10!))
= 10![1 - 1 + (1/2!) - (1/3!) + · · · + (1/10!)) ., (10!)(,-1).
El espacio muestra! consta en este caso de las 10! formas en que pueden terminar los
caballos la canera. Así, la probabilidad de que Rafael pierda todas sus apuestas es aproxi-
madamente igual a (IO!Xc1)/(10!)=r1• Esta probabilidad es (casi) la misma si el nllmao
de caballos en la carrera es 11, 12, . . . Por otro lado, paran caballos, donde n i?: 10, la
probabilidad de que nuestro apostador gane al menos una apuesta es, aproximadamente,

1 - ,.., 0.63212.
8.3 Desordenes: Nada está en el lugar correcto 419

8í!i.8 El número de desórdene s de 1, 2, 3, 4 es

d. = 4![1 -1 + (1/2!) - (1/3!) + (1/4!)]


= 4![(1/2!)- (1/3!) + (1/4!)] = (4)(3) -4 + 1 = 9.
Estos nueve desórdenes son

2143 3142 4123


2341 3412 4312
2413 3421 4321.

Entre las 24 - 9:; 15 permutaciones de 1, 2, 3, 4 que no son desórdenes están

1234 2314 3241


1342 2431 2314.

Patricia tiene que revisar siete libros para la empresa C-H, por Jo que contrata a siete
personas para hacerlo. Desea dos revisores por cada libro, asf que, la primera semana, da
a cada persona un libro para que lo lea y después los redistribuye al inicio de la segunda
semana. ¿De cuántas formas puede hacer las dos distribuciones de modo que tenga dos
revisiones (por personas diferentes) de cada libro?
Ella puede distribuir los libros de 7! formas la primera semana. Si numeramos los libros
y tos revisores (de la primera semana) como 1, 2 , . . . , 7, para la segunda distribución ella
de be ordenar estos números de modo que ninguno de ellos esté en su posición natural, lo
que puede hacer de d1 formas. Por la regla del producto, puede hacer las dos distribuciones
de (7!)d., =(7!)'(e·') fonnas.

EJERCICIOS 8.3 t . ¿Oc cuántas formas se pueden colocar los enteros 1, 2, 3, ...• 10 en una línea de modo que
ningún emero par quede en su posición natural?
2. a ) Enumere los desórdenes de 1, 2, 3, 4, S de modo que los ttes primeros mlmeros sean I , 2 y
3, en algún orden.
b ) Enumere los desórdenes de 1, 2, 3, 4 , 5, 6 de modo que los tres primeros números sean 1, 2
y 3, en algún orden.
3. ¿Cuántos desórdenes de 1, 2, 3, 4, S existen?
4. ¿Cuántas permutaciones de 1, 2, 3, 4, 5, 6, 1 no son desórdenes?
S. a) Sea A = { 1. 2, 3, ... , 7}. Una función/: A ➔ A tiene un punto fijo si para algún ..r E A,
/(..r) = x . ¿Cuán1as funciones inyectivas/: A ➔ A tienen al menos un punto fijo?
b) ¿De cuántas formas podemos diseñar un código secrc10 asignando a cada letra del alfabeto
una letra diferente que la represente?
6. ¿Cuántos desórdenes de 1, 2. 3. 4 , S. 6, 1, 8 comienzan con (a) l . 2. 3 y 4, en algún orden? (b)
S, 6, 7 y 8, en algtln orden?
7 . Paca los enteros positivos 1, 2. 3, .. . , n - 1, n, existen 11.660 desórdenes tales que l , 2, 3, 4
y 5 aparecen en las primeras cinco posiciones. ¿CuáJ es el valor den?
o Capítulo 8 El principo de incius;6n y exclusión

8. Cuatro aspirantes a un puesto tendrán entrevistas de 30 minutos cada uno: 15 minutos con cada
una de las supervisoras, Nancy y Yolanda. (Las entrevistas son en oficinas independientes y
comienzan a las 9:00 A.M.) (a) ¿De cuántas formas $C pueden propamar estas entrevistas W-
rante un periodo de una hora? (b) Um. aspirante. Josefina. Ucp. a las 9:00 A.M. ¿Cuil es la
probabilidad de que tenga sus dos entrcvisw consecutivas? (e) Regina, otn. aspirante. llc¡:a a
las 9:00 A.M. y espera terminar a tiempo para s.alir a las 9:50 A.M. a otra cita. ¿Cuál es la
probabilidad de que Rcgi.na salga a tiempo?
9 . ¿De cuintas formas puede la seoora Femández distribuir diez libros entre sus diez nifios (uno
por cada niOO) y dcspu~ recogerlos y redistribuirlos de modo que cada nitk> tenga la oportuni-
dad de c,caminar dos libros diferentes?
10. a) Cuaodon bolu.. numeradas l. 2. 3, ... , n ,sesacandeunrccipiente.seproduce un mcuentro
casual si lam-tsimabolaquese'3C3tieneelo0merom,paraalglln 1 s m s n.F.ncueotrc
la probabilidad de que (i} no haya cncucntrOS casuales; \ti) haya ex.acwnente un cocuentro
casual; (iii) al menos un encuentro casual; y (iv) r encucntros casua1es, para 1 :s r :S n.
b) Aproxime las respuestas a las preguntas de la parte (a).
11 . Diez mujeres asisten a un desayuno de negocios. Cada una deja en d guardarropa su abri¡o y
su portafolios.Al salir, cada una recibe un abri¡o y una bolsa al azar. (a) ¿Dc0.Wltas formas se
pueden distribuir los abrigos y 105 portafolios de modo que nin¡una de eUas reciba alguna de
sus pertenencias? (b) ¿De cuántas formas se pueden distribuir de modo que ninguna reciba sw
dos pertenencias?

12. La seftora Pozos ensefia geometría y biología a un grupo de 12 eszudiances avanzados en un


aula que sólo tiene 12 pupitres. ¿De c:uinlas formas puede asignar los pupitres a los estudiantes
de modo que (a) ninguno de ellos se siente en el mismo pupitre en ambas clases? (b) exacta•
mente seis esrudiantes se sienten en el mismo lugar en ambas clases?
13. Dt un argumento combinatorio pan verificar que par3 cada n E z•,

(Para cada I s k!< •· d, es el n~mero de de$ónlenes de l. 2, 3..... .t; d,• l.)


14. a) ¿Decuffltas formas se pueden colocarlos enteros 1, 2. 3, ... ,nen una línea de modo que
no aparezcan los patrones 12, 23, 34. ... , (n- l)n?
b) Muestre que el resultado de la parte (a) es igual a d ••1 + d. (d. es el nllmero de dcsórdenes
de l. 2. 3..... n}.
1 S. Responda la parte (a) del ejercicio 14 si los ndmeros se colocan en un círculo y, al contar en el
sentido de las manecillas del reloj alrededor del círrulo, no aparecen los patrones 12. 23, 34, . ...
(n - l)n. ni.
16. ¿Cuil es la pn>babilidad de que el jugadol' del ejemplo &.8 pne (a) exactamente cinco de sus
apuestas? (b) al menos cinco de sus apuesw?

8.4
Polinomios de torre

Consideremos el '•tablero de ajedrez" de seis cuadros de la figura 8.7. En el ajedrez, la pieza


llamada 'º"tpuede moverse, en cada jugada. en forma horizontal o verticaJ sobre el nllmero
de espacios desocupados que desee. En este caso, una tone que esté en el cuadro 3 de la
figura puede moverse en una jugada a los cuadros 1. 2 o 4. Una torre situada en el cuadro 5
puede moverse al cuadro 6 o al cuadro 2 (aunque no haya un cuadro entre los cuadros 5 y 2).
8.4 Polinomios de torre 421

Para k E z•, queremos determinar el número de formas en que k torres pueden colocar-
se en este tablero de ajedrez de tal manera que ningún par de ellas pueda capturarse entre
sí; es decir, no puede haber dos de ellas en la misma fila o columna del tablero. Este
número se denota con r ... o con ri( C ) si deseamos especificar que estamos trabajando en un
tablero C en particular.
Para cualquier tablero de ajedrez. r1 es el número de cuadrados que hay e n el tablero.
En este caso, r 1 = 6. Podemos colocar dos torres sin capturarse en los siguientes pares de
r,=
posiciones: (!, 4), ( 1, 5), (2, 4), (2, 6), (3, 5), (3, 6), (4, 5) y (4, 6), por lo que 8. Si
continuamos, vemos que r3 -= 2, usando Los lugares (1, 4, 5) y (2, 4, 6); r1 = O. para k ~ 4.
Si r0 = 1, el polinomio torre, r(C, x), para e l tablero de la figura 8.7 se define como
r (C,x) =- l + 6.x+ Sr+ 2.r. Paracualquierk ~ 0, el coeficiente de:1! esel número de formas
en que podemos colocar k torres en et tablero de ajedrez C sin que se capturen entre sí.
Lo que hemos hecho aquí (usando un anáJjsis caso por caso) pronto se hará tedioso.
Conforme crezca el tamaño del tablero, tendremos que considerar casos en que números
como r, y r5 sean distintos de cero. En consecue ncia, haremos algunas observaciones que
nos permitirán hacer uso de tableros pequeños y de alguna manera descomponer un table-
ro grande en subtableros más pequeños.

El tablero de ajedrez C de la figura 8.8 está fonnadQ por 11 cuadrados sin sombrear.
Observemos que C consta de un subtablero de 2 X 2 C1 y de un subtablero de siete cuadra-
dos e; localizado en la esquina inferior derecha. Los subtableros son di$juntos, ya que no
tienen cuadrados en la misma fila o columna de C.
Al hacer los cálculos como lo hicimos en nuestro primer tablero de ajedrez, encontra-
mos que

r(Ci,x) = 1 +4x + 2x', r(C,,x) = 1 + 7x + 10.r'+ 2x',


r(C,x) = 1 + llx + 40.r2 + 5&'+28x' +4.i'= r(C1,x) · r(C,,x).
Por Jo tantor(C. x) = r (C1,x) · r(C:, x). ¿Pero fue esto suerte o hay algo que deberíamos
examinar con más cuidado? Por ejemplo, para obtener r3 para C, necesitamos saber de
cuántas formas pueden colocarse en el tablero C tres torres sin que se capturen entre sí.
Hay tres casos:

a) Las tres torres están en el subtablero C1; (y ninguna en e l C1): (2Xl ) = 2 formas.
b) Dos torres están en el subtablero C2 y una e n C1: (10)(4) :.:: 40 formas.
e ) Una torre está en el subtablero C 2 y dos en C1: (7X2):.:: 14 formas.

Fig ur.11 8.7


g
Figu ra 8.8
22 Capitulo 8 El principio de inclusión y e><dU>ión

En consecuencia, se pueden colocar en el tablero tres torres sin que se capturen entre sí
de (2Xl) + (IOX4) + (7X2) = 56 formas. Aquí vemos que obtenemos 56 justo como el
cocficienre dct' en el producto r(C1.A") • r((;, .t).

El último resultado de esta sección demuestra el tipo de principio que vimos en otros
resultados de matemáticas combinatorias y discretas: dado un gran tablero de ajedrez. hay
que separarlo e n pequetios subtableros cuyos polinomios de t ~ puedan detc:nninarse
por inspección.
Consideremos el tablero de ajedrez. C de la figura 8.9. Seaki? 1. Para cuaJquiercuadra-
do de C, como el designado por(*), debemos analizar dos posibilidades.
a) Coloque una torre en el cuadrado elegido. Luego debemos eliminar, como lugares
posibles para las demás k - 1 tones, los demás cuadndos de C de la misma fila o
columna del cuadrado elegido. Usamos C, para designar el subtablero restante más
pequeño.
b) No usamos para nada el cuadrado elegido. Las k torres se colocan en el subtablero
c. (C con el cuadrado elegido eliminado).

FigurA 8.9

Puesto que estos dos casos incluyen todos los posibles y son disjuntos.

r.(C) = ,,_1(C,) + r.(C,).

De aquí vemos que

r,(C)r' = ,...,(C,).r' + r,(C,).r'. (1)

Sí n es el nlimero de cuadrados del tablero (en este caso, n = 8). entonces la ecuación (1) es
válida para todo 1 .:S k s n. y escribirnos

}: r.(C).r' • }: ,,_1(C,).r' +}: r,(C,).r'. (2)


.t•I l:•l l•J

En la ecuación (2), nos damos cuenta de que la suma puede detenerse antes de k = n.
Hemos visto casos. como en la figura 8.7. donde r. y algunos rt anteriores son O. La suma
comienza e n k ;: l , ya que de otro modo tendríamos el t~rmino r_1(C~).r" en el primer
sumando del lado derecho de la ecuación (2).
8.4 PolinomtOS de torre 423

La ecuación (2) puede escnbine nuevamente como

1 + }: r..(C).t'=x · r(C.,x) +
t• J
±r,(C,)r' + 1,
•-1
de lo que se sigue que

r(C,x) = x · r(C.,x) + r(C.,x). (4)


Usamos ahora esta última ecuación para determinar el polinomio torre para el tablero
de ajedrez que se muestra en la figw-a 8.9. Cada vez que se usa la idea de la ecuación (4).
marcamos el cuadrado especial que estamos usando con un (• ). Los par~ntesis se colocan
cerca de cada tablero para denotar su polinomio torre.

(@) .. (83H@) ·
·[. (E3) .(EE)l·[. (EE).(fF)l
-r(a) (EE) .¡. (~) ·(83)]
•h

• x 2(1 + 2x) + 2x(J + 4x + 2x 2) + x(J + 3x + x 2)

{(B)·(EE)]-
,- 3x + 12x2 + 7x3 + x(I + 2x) + (1 + 4x + ZX') • 1 + 8x + 16x' + 7x'.
capítulo 8 El principio de inclusión y exclusión

8 .5
Disposiciones con posiciones
prohibidas

Si bien los polinomios torre de la sección anterior parecen interesantes por sí solos, ahora
mostrarán su utilidad para resolver los siguientes problemas.

~Í~
Í1:: Al hacer las disposiciones de asientos para la recepción de la boda de su hijo, Grac:iela y
Nicolás están colocando a cuatro parientes, que denotamos con R,. para l s i s 4, que no
se llevan bien entre sí. Hay un único asiento desocupado en cada una de las cinco mesas Tr
donde 1 s j s 5. Debido a las diferencias familiares,
a) R 1 no se sentará en T 1 o 72 b) R1 no se sentará en 7 2•
e) R3 no se sentará en T3 o T,. d) R. no se sentará en 7,. o Ts.
Esta situación se representa en la figura 8.1 O. El número de formas e n que podemos
sentar a estas cuatro personas en mesas diferentes, y que satisfagan las condiciones (a) a
(d), es el número de formas en que podemos colocar cuatro torres, sin que se capturen en
e l tablero de ajedrez formado por los cuadrados sin sombrear: Sin embargo, como sólo hay
siete cuadrados sombreados, mientras que hay y trece sin sombrear, seña más fácil traba-
jar con el tablero sombreado.
Comenzare mos con las condiciones que se requieren para aplicar la inclusión y la ex-
clusión: para todo I s i s 4, sea C; la condición donde una asignación de asiento de estas
cuatro personas (en diferentes mesas) se realiza de modo que un pariente R. quede en una
posición sombreada prohibida. Como es usual, ISI denota el número total de fonnas en
que pueden colocarse los cuatro parientes, uno en cada mesa. Entonces ISI =- N=- So=-5!.

R, T, T, T, T, T,

.,
.,
.. 9
Para determinar S1 consideremos lo siguiente:
Figura 8.10

• N(c1) =- 4! + 4. ya que hay 4! formas de sentar a R 2, R 3 y R,. si R1 está en la posición


prohibida 7 1 y otras 4! formas si R1 está en la mesa 72, su otra posición prohibida.
• N(c,) = 4!, ya que después de colocar aR2 en una mesa prohibida T2, debemos colocar
a R 1• R>y R~ en T¡, T3 , T,. y Ts, una persona por mesa.
= 4! + 4 !, donde un sumando se debe a que R1 se encuentra en la posición
• N(c 3 )
prohibida 7 3 y el otro sumando para R3 que se encuentra en la posición prohibida T,.
• N(c.c) _,, _ 4 ! + 4!, cada uno de los dos sumandos surge cuando R,. se coloca en cualquie-
ra de Jas dos posiciones prohibidas T, y T5•
Por lo lanto S1 = 7(4!).
8.5 Disposióones con posiciones prohibidas 425

Regresando a S2 tenemos estas consideraciones:


• N(c1c-J = 3!, ya que d e ~ de colocar a R1 en 7 1 y a R2 en Ti. hay tres mesas (T,. T4
y T,) donde pueden sentarse R, y R,.
• N(c1c1) = 3 ! + 3! + 3! + 3!. ya que hay cuatro casos en que R1 y R) se colocan en
posiciones prohibidas:
1) R1enT1; R3enTJ ii) R,enT2; R3coT3
W) R1cnT1 ; R 1 cnT, iv) R1 cnT1;R3 coT4•
De una manera similar encontramos que N(c,c,) = 4(31), N(c,c,)=2(31), N(c,c,) = 2(31)
y N(c,c, ) = 3(31). En consecuencia, S.= 16(31).
Antes de continuar. haremos algunas observaciones acerca de S1 y Si- Para S1 tenemos
7(41) = 7(5 -1)1, donde 7 es el n6mero de cuadrados sombreados de la figura 8.10. Así
mismo, S1 = 16(3!)= 16(5-2)!, donde 16 esel número de formas en quedos tones pueden
colocarse en el lab)ero sombreado sin que se capturen entre sí.
En genera]. para todo O ::S is 4, S1= r,{5 - i)!, donde,, es el ntlmero de formas en que
es posible colocar i torres sin que se capturen en e1 tablero sombreado de la figura 8.1 O.
En coosecueocia, para facilitar La solución de este problema, n:g¡-esamos a r(C, x), el
polinomio torre de este tablero sombreado. Si usamos la descomposición de C en los
subtableros disjuntos de las esquinas superior izquierda e ihferior derecha, encontramos que

r(C,x) = (! + 3x + r')(l + 4x + 3x') =1 + 7x + 16%2 + 13.r + 3x',


así

.
N(c, c,c,c,) =s.-s, + s,- s, + s, = 5! - 7(4!) + 16(3!)- 13(21) + 3(1!)

..
=,L (-1¡1,,(5-i)! = 25.
Graciela y Nicolás pueden respirar tranquilos. Hay 25 formas en que pueden sentar a
estos áltimos cuatro parie ntes en la recepción y evitar cualquier aJtercado.

Nuestro último ejemplo demuestra cómo un pequeño rcordenamiento de nuestro table-


ro de ajedrez puede ayudarnos en nuestros cálculos.

Tenemos un par de dados; uno es rojo, el otro verde. Tiramos estos dados seis veces. ¿Cuál
es la probabilidad de obtener los seis valor-es distintos del dado rojo y del verde si sabemos
que no aparecieron los pares Ofdenados (1.2), (2,1), (2,5), (3,4), (4 ,1), (4,5) y (6,6)? [Un
par ordenado (x, y) indica x en el dado rojo y y en el verde.]
Si reconocemos que este problema se refiere a permutaciones y posiciones prohibidas,
construimos el tablero de ajedrez que se muestra en la figura 8.11 (a), donde los cuadrados
sombreados constituyen las posiciones prohibidas. En esta figura. los cuadrados sombreados
se encuentran dispersos. Si nuevamente etiquetamos las filas y las columnas, podemos
volver a dibujar el tablero de ajedrei como se muestra en la figura 8.ll(b), donde hemos
tomado los cuadrados sombreados de la misma fila (o columna) del tablero que se muestra
en la panc (a) y los hemos hecho adyacentes. En la figura 8.ll(b), el tablero de ajedrez C
(de siete cuadrados sombreados) es la unión de cuatro subtableros disjuntos entre sí, y así

r(C,x) = (1 + 4x + 2r')(l + x)' ~ 1 + 7x + 17x' + 19.r' + !Ox' + 2x'.


capitulo 8 El principio de inclusión y exclusión

4 5 6 5 3 4 2 6

(a) (b)

Figura 8 .11

Para todo l s i s 6, definimos c1 como la condición tal que. habiendo tirado los dados
seis veces. los seis valores se presentaron tanto en e] dado rojo como en el verde. pero i en
el dado rojo se empareja con uno de los números prohibidos en el dado verde. [Nótese que
N(cs) :: O.] Entonces el número de sucesiones (ordenadas) de las seis tiradas de los dados
para el suceso en cuestión es

• •
(6!)N(c,c,c, c,c, ~ = (6!)¿ (-lYS,= (6!) ¿ (-1y,, . (6-i)!
,.. 1-0

= 6![6! - 7(5!) + 17(4!) - 19(3!) + 10(2!) - 2(1!) + 0(0!)]


= 6![192] = 138,240.
Puesto que el espacio mucsttal consta de lOdas las sucesiones de seis pares ordenados
seleccionados con repetición de los 29 cuadrados sin sombrear del tablero de ajedrez, la
=
probabilidad de este suceso es 138,240,/(29)6 0.00023.

cJERCIOOS 8.4 1. Verifique directamente los polinomios torre pan¡ (a) el tablero de ajedrez sin 50fflbrcac de las
,s.s figuras 8.8 y 8.9. y (b) el ubl<ro de ajcdrel - d e las fi¡¡un> 8.10 y 8.ll(b).
2 . Construya o describa un tablero de ajedrez mínimo (cantidad mínima de cuadrados) para el
cualrio• O.
3. a) E.ncuentre el polinomio torre para el tablero de ajedrez csttndu de 8 x 8.
b) Rcspooda la panc (a) reemplazando 8 con n, para n E z·.
8.6 Resumen y repaso histórico 427

(;)
# ~
(ji)

1
(iii)
1
(1 11
1 1

Ea:a
M
Figura 8 .13

4. Encuentre los polinomios torre para el tablero de ajedrez som_breado de la figura 8.12.
S. a) F.ncuentre los polinomios torre para los tableros sombreados de la figura 8.13.
b) Generalice el tablero de ajedrez (y el polinomio torre) para la figura 8.13(i).
6 . Sea C un tablero de ajedrez que tiene m filas y n columnas, con m s n (para un total de mn
cu.adrados). Para O :s: k s m, ¿de cuántas formas pueden colocarsck torres (idénticas) en C sin
que se capturen entre sí?
7 . La profesora Ruth tiene cinco ayudantes para conegir programasen su cursos deAPL. BASIC,
FORTRAN, Pascal y PUi. A los ayudantes Juana y Carlos les disgusta FORTRAN. Sandra
desea evita.e BASIC y PUi. Pablo detesta APL y BASIC y Teodoro se rehúsa a trabajar c.on
FORTRAN y Pascal. ¿De cuántas formas puede asignar la profesora Ruth la corrección de
programas en un lenguaje a cada ayudante, abarcar los cinco lenguajes y mantener a todos
contentos?
8. ¿Por qué tenemos 6! en el t&mino (C1C1 . . . C6 ) para la solución del ejemplo 8. 12?
9 . Cinco profesores llamados Alberto, Violeta. Lorenza, Jaime y Maria Luisa scran asignados ·
para impartir un curso a escoger entre cálculo 1, cálculo U, cálculom, estadística y combinatoria
Alberto no impartirá cálculo ll ni combinatoria, Lorenza no soporta la estadística. Violeta y
Maria Luisa se rehásan a cnsclíar cálculo I o cá1culo m y Jaime detesta cálculo 11.
a) ¿ De cuántas formas puede decidir el coordinador del depanamcnto de matemáticas la asig-
nación a cada uno de estos profesores de uno de estos cinco cursos y mantener la paz en el
departamento?
b) Para las asignaciones de la parte(a). ¿cuál es la probabilidad deque a Violeta se le asigne el
curso de combinatoria?
1 O. Un par de dados, uno rojo y el otro verde. se tira seis veces. Sabernos que los siguientes pares
o,denados no apanrie<on en el ,csu!tadoc (l. !). (l. 5), (2, 4), (3. 6), (4 , 2). (4, 4). (5. ! ) y (5. 5).
¿Cuál es la probabilidad de que cada valor haya caído en el dado ro~ y el verde?
11. Un servicio de citas por computador quiere reunir a una de cuatro mujeres con uno de seis
hombres. De acuerdo con la información que proporcionaron al unirse al servicio. podemos
sacar las siguientes conc lusiones.
• La mujer 1 no serla compatible con los hombres 1. 3 o 6.
• La mujer 2 no seria compatible con los hombres 2 o 4.
428 Capítulo 8 El principio de indusión y exdusi6n

• La mujer 3 no scóa compatible con los hombres 3 o 6.


• La mujer 4 no sería compatible con los hombres 4 o S.
¿De cuántas formas puede el servicio reunir con éxito a cada una de las cuatro mujeres con un
compafiero compatible?

8.6
Resumen y repaso histórico

En el primero y tercer capítulos de este texto estudiamos problemas de enumeración en


que debíamos tener cuidado con las situaciones donde las disposiciones o selecciones se
contaran en e xceso. Esta situación llegó a complicarse en el capítulo 5, cuando tratamos
de contar el m1mero de funciones sobre entre dos conjuntos finitos.
Con los diagramas de Vcnn como guía, en este capítulo obtuvimos un modelo llamado
principio de inclusión y exclusión. Usando este principio, volvimos a plantear cada pro-
blema en términos de condiciones y subconjuntos. Con las fórmulas de conteo de
permutaciones y combinaciones que desarrollamos anteriormente, resolvimos algunos
subproblemas más sencillos y dejamos que el principio manejara nuestra preocupación
por el recuento excesivo. Como resultado, fuimos capaces de resolver una variedad de
problemas, algunos de teoría de nWI'leros y uno de teoóa de grafos. También proporciona.
mos la fórmula conjeturada en la sección S.3 para el m1mero de funciones sobre para dos
conjuntos finitos.
Este principio tiene una historia interesante, la cual se encuentra en distintos manuscri•
tos con nombres como "El mttodo de la criba" o el "Principio de clasificación cruzada".
Una versión de este principio desde el punto de vista de la teoría de conjuntos, el cual tiene
que ver con uníones e intersecciones de conjuntos. se encuentra en Doctrine o/ Chances
(171 8), un texto de teoría de probabilidad deAbrabam DeMoivre (1667-1754). Un poco
antes, en 1708, Pierre Rtmond de Montmort (1678-1719) usó la idea subyacente en el
principio en su solución del problema que por lo general se conoce como~ problinu des
rencontres o emparejamientos. (En este antiguo juego de cartas francés, las 52 cartas de
una primera baraja se colocan hacia arriba en una fila, tal vez. sobre una mesa. Luego se
reparten las S2 cartas de una segunda baraja, y cada carta nueva se coloca sobre cada una
de las 52 cartas previamente colocadas sobre la mesa. La puntuación del juego se determi•
na contando los emparejamientos resultantes: deben coincidir el valor de la carta hacia
arriba y el de la repartida.)
El crédito para la forma en que desarrollamos y trabajamos con el principio de inclu•
sión y e xclusión es para James Joseph Sylvester (1814-1897). (Este brillante matemático
inglés también hizo grandes contribuciones a la teoña de ecuaciones; la teoría de matrices
y detennÍJJantes; y la teoría de invariantes que fundó junto con Arthur Cayley (1821- 1895).
Además, Sylvester fundó la.American Journal of Ma1hematics, la primera revista estado-
unidense de.dicada a la investigación en matemáticas.) La importancia de la t6cnica de
inclusión y exclusión no fue apreciada en general; fue más adelante cuando la publicación
de W. A Wbitworth [10] hizo que los matemáticos tuvieran más conciencia de su poten•
cial y uso.
Para más detalles acerca del uso de este principio. consulte el capítulo 4 de C. L Liu
(4], el capítulo 2 de H. J. Ryser [8] o el capítulo 8 de A. Tucker [9]. Otros resultados sobre
8.6 Resumen y repaso histórico 429

Jomes Joseph Sytvestor (1814-1897)

teoría de números relacionados con el principio, incluyendo la fórmula de inversión de


Mobius, apan:cen en el capítulo 2 de M. Hall [I], el capítulo X de C. L. Liu [5] y el
capítulo 16 de G. H . Hardy y E.M. Wright [3]. En el artículo de G. C. Rota [7), se pmenta
una extensión de esta fórmula.
El artículo de D. Hanson, K.. Scyffarth y J. H. Westoo [2] proporciona una genmiliza-
cióo importante del problema de desorrlen analizado en la sección 8.3. Las ideas subya-
centes en los polinomios torre y sus aplicaciones fueron desarrolladas a fines de l a ~
de 1930y durante las de 1940 y 1950. E n los capítulos 7 y 8 de J. Riordan [6] aparu:e más
materia) acerca de este tema.

BIBLIOGRAFÍA

t. Hall, Marshall, Jr., Comhinalorial ThLory, Waltham. Mass.• Blaisdell, 1967.


2. Hamon, Den.is, Karen Scyffanh, y J. Harlcy Weston, "'Matchings, Derangements, Rencontres"',
Mow:malics Mato,line S6. nwn. 4 , septiembre de 1983, págs. 224-229. ·
3. Hardy. Godfrey Harold y Edward Maitland Wright, An lnrroducrion ro 1M Thtory ofNumbtn.
5" od., Oxford, Oxford Universi1y Press, 1979.
4. Liu, C. L., lnrroduccwn 10 Combinawrial Marhtmarics, Nueva York, McGraw-Hill. 1968.
5. Uu. C. L., Topics in Combinau>ri.al Marhematics. MathcmacicaIAssociation ofAmerica. J972.
6. Riordan, John,An inlrOduction ro Combin.atoriaUnalysi.r, Princeton, NJ.• Princecoo Univcrsíty
Press. 1980. {Pllblicado originalmente en 1958 por Jobo Wiley & Sons.)
7. R0ta, Gian Cario, "On thc Foundations of Combinatorial Theory, l Theory of MObius
Functions.., ü:itschriftfti.r WahrscMinüchluiu TMorü 2, págs. 340-368. 1964.
8. Ryser, Hcrbett J., Combinazorial MarMmarics, Caro! Mathcmatical Monognph, Nd.m. 14.
Publicado por The Mathcmatical Association of America, distribuido por John Wiley &. Sons,
Nueva Yort, 1963.
9. Tucker, Alan. Applied CombinauJrics, 2• ed., Nueva York, Wtley. 1985.
• 10. Wbitworth, William ADeo. Choiu and Chl:lnu. (Publicado ori¡i.nalmente en Cambridge en
1867.) Reimpresión de las• ed., 1901, Hafncr, Nueva Yorfc, 1965.
o Capitulo 8 El principio de indusión yexdusión

ejemplo 1.16, no hacemos distinción entre dos <lisposicio-


EJEROOOS oes. li la primera se puede obtener de la segunda mediante
COMPLEMEITTARIOS la rotación de las posiciones de las diez personas.)
8. Coa el re,wlado del teo<ema 8.2, demucsue que el al-
Detcrminecuán1osn E z -satisfacenquen :S SOOy no mero de formas en que podemos colocar , objetos diferen-
1 divisibles entre 2, 3, S, 6. 8 o 10. tes en n recipientes distintos de modo que m recipientes con-
tengan a.actamcote r objetos es
• ¿Cuántos coterosncxisten tal que OS n < 1,000.000y
e la suma de los dígitos de n sea menor o igual que 37?
_ Para el bazar de la iglesia de la próxima semana, Jost y (-trn!s! ± (-l)'(n-i)•-•
sobrino menor Jerenúas deben colocar seis pelotas de m! ,-.(i - m)!(n -i)!(,-ir)!(r!Y
isbol, seis balones de baloncesto, seis de fútbol y seis de
lcibol en las cuatro repisas del puesto de deportes patro- 9. Si se elige al azar una disposición de las letras de
,ado por su tropa de scouu. ¿De cuánw formas pueden SURREPITl10US, ¿cufl es la probabilidad de que coorm-
ccr esto de modo que haya a1 menos dos, pero no más de ga (a) exactamente tres pares de letras idffltic&.s COftS.CQlb.
~ balones en cada rcpi$3.'? (En este caso, las seis pelotas vas? (b) a lo mú eres pares de letras idénticas consecutivas?
cuaJqwcra de los cuaU'O depones son idmticas.)
1 o. ¿Oc cuántas formas podemos ordenar cuatro w, cuatro
.. Encuentre t! número de enteros positivos n tales que .r, cuatro y y cuatro t de modo que no haya cuatro letras
a) 1 :S n :S 1000 y n no es un cuadrado perfecto. cubo idEnticas consecutivas?
o cuana potencia.
11. a) Dados n objetos distintos, ¿de cuintas formas po-
b) 1 :s n :S 3000 ynnoesun cuadradoperfcc:::to,cubo
demos seleccionar r de ellos de modo que cada se-
o quinta potencia.
lección incluya m objetos particulan":$ de los n?
f. ¿Oc eumw formas podemos colocar los enteros 1, 2, (Aquí m :S r :S n).
, . , , 8 en una recta de modo que no aparezcan los patro- b ) Mediante el principio de inclusión y a.clusión, de-
,s 12. 23 •. .. . 78, 81? muestre que para m S r :S n,
;. a) Si disponemos de k colores diferentes, ¿de cuántas
formas podemos pintar las paredes de un cuano
pentagonal si las paredes adyacentes deben pinar-
se con colores diferentes?
(nn -m)
-r
• f (- l)'(":)(•-i).
1-0 rl

b) ¿CuáJ es el valor mínimo de k para c1 cuaJ existe


dicha coloración? 12. a) Sea A e z•. Si tenemosA.coloresdiferentcsdispo-
e) Responda las panes (a) y (b) para un cuarto nibles, ¿de c:ujnw formas podemos c:olorcM los
hexagonaJ. v&ti<:es del grafo que se muestra en la figura 8.14(a)
7. ¿Oc cuinw formas se pueden sentar cinco parejas al- de modo que dos vtn:iccs adyacentes cualesquien
!dedor' de una mesa redonda de modo que ningán miembro no compartan el mismo color'? Este resultado aocr-
e la pareja se siente junto a1 otro? (En este caso, como en el ca de A se llama polinomio cromático del grafo. y

.,
..
'
d
(a) (b)

• Figura 8.14
431

d nlor mínimo de ).. para el cual el valor de este tos con una puerta común D¡. 1 $ j s S. se pinten
polinomio es positivo es el nt,ÍIMro cromálico del con colores diferente$?
pfo.¿Ow,ovalcd mlmc,oao,njócodccstepo? 13. Sin E z•, dem1Je$Ue que
(Amlizarcmos con nw detalle esta idea en el ca--
a) +(2,,) = 2+<•) si• es par, y.
pltulo 11.)
b) +(211) = +(n) sin es impar.
rt) Si existen seis colores disponibles. ¿dec::u-'ntas for•
mas se pueden pintar los c:ua.rtos R,. 1 S i S S, que 14. Sean a. b,c E Z♦• coo e= mcd(a, b). OcmUCSb"C que
aparecen en la figura 8.14(b) de modo que los cuar- +(ab)+(c) • +(a)+(b)c.
9
Funciones
generatrices

E n este capítulo y el siguiente continuaremos nuestro estudio de la enumeración con el


importante concepto de la función generatriz,.
E1 problema de hacer selecciones. con repeticiones permitidas, fue estudiado en el ca-
pítulo 1. Ahí tratamos de obtener, por ejemplo, el número de soluciones enteras de la
ecuación c 1 + c1 +e, + e,= 25 donde c1 ~ O para todo 1 :S i s 4. En el capítulo 8, con el
principio de inclusión y exdusión, fuimos capaces de resolver una versión más restringida
del problema, como c 1 + c2 + c 3 +e,= 25 con O :S e, < 10 para todo 1 s i :S 4. Si, además,
quisi~ramos que e, fuera par y c 1 un múltiplo de 3, podríamos aplicar los resultados de los
capítulos 1 y 8 en varios subc:asos.
La fuerza de la función generatriz descansa en que no sólo permite resolver los tipos de
problemas ya analizados anteriormente sino que tambiin ayuda en situaciones nuevas
donde pueden aparecer más restric.ciones.

9.1
Ejemplos introductorios

En lugar de definir en este punto una función generatriz. examinaremos algunos ejemplos
para derivar la idea a partir de eUos. Veremos que ya hemos tratado este concepto e_n·
situaciones anteriores.

9.1 En sus compras del sábado, Mónica compró 12 naranjas para sus hijos, Graciela. María y
Francisco. ¿De cuántas formas puede elJa distribuir las naranjas de tal forma que Graciela
obtenga al menos cuatro y Maria y Francisco obtengan al menos dos, sin que Francisco no
obtenga más de cinco? La tabla 9.1 enumera todas las distribuciones posibles. Vemos que
tenemos todas las soluciones enteras de la ecuación e,+ c1 + CJ = 12 tal que 4 ::s C1, 2 ::s c1
y2 se, ::s 5.
Al considerar los primeros dos casos en esta tabla. encontramos las soluciones 4 + 3 + 5:.:
12 y 4 + 4 + 4:.: 12. ¿Sucedió algo así en nuestras experiencias algebraicas anteriores? Al
multiplicar polinomios añadimos las potencias de la variable, y aqui. al multiplicar los tres
polinomios.
(x" + x.s + x6 +x1 +x1)(x1 +x1 +x" + x 5 + x6)(r2+.r +x"+ x' ).
dos de los caminos para obtener zl 2 son como sigue:
433
434 Capítulo 9 Funciones generatrices

Tabla 9.1

G M F G M F
4 3 5 6 2 4
4 · 4 4 6 3 3
4 5 3 6 4 2
4 6 2 7 2 3
5 2 5 7 3 2
5 3 4 8 2 2
5 4 3
5 5 2

l) Del producto x' x' x', donde x' se toma de (x' + x' + · · · + x'), x' de (x' + x' + · · · +
.t") y x' de (r +x' + .. . +x').
2) Del producto x' x' x', donde el primer x' se encuentra en el primer polinomio, el
segundo x' en el segundo polinomio y el tercer x' en el tercer polinomio.
Al examinar el producto

(.r' + .r' + .r' + x' + x')(x' + x' + .r' + .r' + x')(x' + x' + .r' + .r')

con más cuidado. nos damos cuenta de que obtenemos el producto :ixl,! para toda tema
(i. j, k) que aparece en la tabla 9.1. En consecuencia, el coeficiente de 2 en r
f(x) =(x'+.r'+ · · · +x')(.r'+x3+ · · · +x6)(x'+x3 + ··· +.r')
cuenta el número buscado de distribuciones, 14. La funciónj(x) es lafunci6n generatriz
para las distribuciones.
Pero, ¿de dónde salieron los factores en este producto?
Por ejemplo, el factor x' + x5 + x' + x1 + x', indica que podemos dar a Graciela 4 o So 6
o 1 u 8 naranjas. Una vez. más hicimos uso de la interacción entre la o exclusiva y la suma
ordinaria. El coeficiente de cada potencia dc.::c es 1 ya que, considerando las naranjas como
objetos idénticos, sólo hay una forma de darle a Graciela cuatro naranjas, una forma de
darle cinco naranjas y así sucesivame nte. Como María y Francisco deben recibir cada uno
al menos dos naranjas. los otros términos (x2 + ..r3 + · · · + .x6) y (x2 + x3 + • • · + x5) comien-
r
zan con x2 y, en el caso de Francisco, nos detenemos en para que no reciba más de cinco
naranjas. (e.Por qué detenemos en x6 el ténnino de María?)
La mayoña de nosotros nos hemos convencido razonablemente de que el coeficiente
dc.x11 cnf{.x) nos da la respuesta Sin embargo, algunos serán un poco escépticos acerca de
esta nueva idea. Parece que serfa más rápido enumerar los casos de la tabla 9.l que multi-
plicar los tres factores de /{x) o calcular el coeficiente de x 12 en/(x). Por ahora esto podría
ser verdad, pero al pasar a problemas con más incógnítas y mayores cantidades que distri-
buir, la función generatriz será de gran utilidad. (El lector puede darse.cuenta de que los
polinomios torre del capítulo 8 son un ejemplo de funciones generatrices.) Por ahora exa-
minemos dos ejemplos más.
9.1 Ejemplos introductorios· 435

u Si existe un m1mero ilimitado (o al menos 24 de cada color) de dulces de jatea de color


rojo. verde, blanco y negro, ¿de cuántas fonnas puede seleccionar un niño 24 de estos
dulces de tal manera que tenga un número par de dulces blancos y al menos seis dulces
negros?
Los polinomios asociados con los colores de los dulces de jalea son los siguientes:
• rojo (verde): 1 +x +i' + · · · +x'-', donde el 1 restante es por Ir', ya que un.a posibi-
lidad es que ninguno de los colores rojo (y verde) sea seleccionado.
• blanco: (1 +x'+x'+x'+ · • • +x"')
• negro: (x'+ x' +x' +-·· +x"')
Asf, la respuesta es el coeficiente de x1-4 en la función generatriz

f(x) = (l +x +x'+ ·· · +x"')'(l +x'+x'+ · · · +x"')(x'+x1 + · · · + x"').


Una de tales selecciones está formada por cinco dulces de jalea rojos, tres verdes, ocho
blancos y ocho negros; esta selección surge de r en el primer factor, t' en el segundo
factor y x' en los últimos dos factores.

¡Un ejemplo más antes de cerrar esta sección!

¿CUántas soluciones enteras tiene la ecuaciónc 1 + c1 +e,+ e, ; 25 si Os e, para iodo 1 :S


i :S 4? .
Otra alternativa es preguntar de cuántas fonnas se pueden distribuir 25 monedas de un
centavo (i~nticas) entre cuatro ni6os?
Podemos describir las posibilidades para cada niño mediante el polinomio 1 + x + ,:1 +
x1 + · · · + .rJ. Entonces la respuesta a este problema es el coeficiente de~ en la función
generatriz
f(x) = (l + x +x'+ • - - + x")'.
También podemos obtener la respuesta mediante el coeficienterS en la función generatriz
g(x) = (!+X +x' +x' + · · · +x"+x"+ · · •)',
si expresamos la pregunta en tt!:rminos de una distribución de 25 monedas entre cuatro
nil\os, tomadas de un níimero grande (o ilimitado) de monedas de un centavo. [Mientras
que/(.x) es un poJjnomio, g(x) es una serie de potencias en .t.] Observe que nunca se usan
los t6rminos x!, para toda k ~ 26. Entonces ¿por qut nos preocupamos por ellos? Porque
habrá veces que será más fácil calcular con una serie de potencias que con un polinomio.

EJERCICIOS 9.1 1 . Pata los si¡uicntcs ejercicios, determine una función generalliz e indique el coeficiente de la
función noccsaria para resolver el problema. (Proporcione las formas polinomial y de serie de
poteocias de la función generauiz. cuando sea apropiado.)
Enruentre el número de soluciones enteras para las siguientes ecuaciones:
a) C1 + Ci + Ci +e, • 20, o :S C¡ ::s 7 para todo I s i :s: 4.
b) e, +c 1 +c, +c, • 20,0 s c,paratodo 1 ::Si ::s: 4,conc1 yc,parcs
e) e, + c1 + c, + c, +cs=30, 2 Sc1 .S4, y3 :s e; :S: 8paratodo2 ::S: i s S
d) e1 + c1 +e,+ e, +es= 30. OS c, para todo 1 :s: i ::S S, con Cz par y e, impar.
36 capítulo 9 Funciones generatrices

2. Determine la función generatriz para el número de fonnas de distribuir 35 monedas de un


centavo (que se obtieoen de un fondo ilimitado) entre cinco niños, si (a) no hay rcstr:icciooes;
(b) cada niño obtiene al menos un centavo: (e) cada nii'\o obtiene al menos dos centavos; (d) el
niño más grande obtiene al menos 10 centavos; y, (e) los dos niños más pcquCOOs deben obte-
ner al menos 10 centavos.
3 . a) F.ncuentre la función generatriz para el número de formas de seleccionar 10 barras de dulce
de un suministro grande de seis diferentes tipos.
b) Encuentre la función generatriz para el número de formas de seleccionar r objetos de una
colección de n objetos distintos, si se permite la repetición.
4 . a) Ex.plique por qu~ la función generatriz para el nómero de formas de tener n centavos eo
monedas de uno y cinco centavos es (1 +x +r +.t' + · · · ){1 + .x5 +r0 + • • · ).
b) Encuentre la función generatriz para el número de formas de tener n centavos en monedas
de uno. cinc.o y diez centavos.
5. Encuentre la función generatriz para el número de soluciones enteras para la ecuación c 1 + c1 +
e,+ e, = 20dondc - 3 :S Ca,-3 :S Ci,-S :S c3 :S S yO :S e,.
6. Para S = {a, b, c},eonsiderc lafunción/(..r)=(l +ax}(I +bx)(l +ex)= 1 +a.:c+bx+cr+abi
+acr +bcx1+. abd. En este caso, en/{x),
• El coeficiente de ,fl es l . para el subconjunto S de S.
• El coeficiente de .x1 es a+ b +e, para los subconjuntos {a}, {b} y fe} de S.
• El coeficiente de x1 esab + ac + be, para los subconjuntos {a.,b}, {a.e} y {b,c} de S.
• El coeficiente dex:1 es abe, para el subconjunto {a, b, e} = S.
En consecuencia. /(.x) es la función generatriz para los subconjuntos de S. ya que cuando
calcu.lamos/(1), obtenemos una suma en que cada u.no de los ocho sumandos corresponde a un
subconjunto de S; el sumando I C011'CSponde al fJ. (Si vamos un paso más allá y establecemos
a = b = e = 1 en/(.x), entonces/(}) =- 8. el nOmero de subconjuntos de S.)
a) Dt la función generatriz para los subconjuntos de S = {a. b, e, ... , r, s, t}.
b) Responda la parte (a) para el caso de las selec.ciones en que cada uno de los elementos
puede ser rechazado o selec.cionado basta ues veces.

9.2
Definiciones y ejemplos:
Técnicas de cálculo

En esta sección examinaremos varias fórmulas y ejemplos relacionados con las series de
potencias que usaremos para obtener los coeficientes de términos particulares en una fun.
ción generatriz.
Comenzaremos con el siguiente concepto.

efinici6n 9.1 Sea ao, a 1, ~ •••• una sucesión de m:irneros reales. La función

f(x) =.. + a,x + a, x' + · · · = L a;x'


es la función generalri,z de la sucesión dada.
..
,

¿De donde viene esta idea?


9.2 DefK1ióones y ejem~: T~icas de cálculo 437

. . .~ f Para cualqwu n E z•

(1 +x)•-(~) + (~► + (; )x2 + ·· · + (:►,


de modo que (1 + x)" es la función generatriz para la sucesión

(~). (~).(;).... .(:),o.o,o,...


a) Paran E Z+,

(1 -x"• 1) = (1 - x)(l + x + x2 + x' + · · · + x").


Asf (I -X"" 1)/(1 -x) = (1 + x+ x' + • • • +X")y (1 -X"" ')/(1 - x)es la función
generatriz para la sucesión 1, 1, 1, ...• 1, O, O, O, .. . , donde los primeros n + 1
ttnninos son 1.
b) Si extendemos la idea de la parte (a), encontramOS que

1 •{1 .-x)(l + x +x2 +x'+x'+ • · ·),

as{ 1/( 1 - x) es la función genuatriz para la sucesión 1, 1, 1, 1, ... (1/( 1 - x) = 1 +


r + · · · es válida para todos los realesx tales que I x 1 < 1; en este rango de
.x + ,:2 +
valores, la serie gtomitrica l + z + x.1 + • • • converge. Sin embargo, en n.uestro
trabajo con funciones generatrices nos ocuparemos mi.s de los coeficientes de las
potencias de .r que de la convergencia. Esto no quiere decir que el concepto de
convergencia no sea imponante, sino que no lo necesitamos para el material que
estudiaremos en este capítulo.]
e) Coo 1/(1 - x) = 1 + x + x' + x' + · · · = L:.x', tomando la derivada de cada lado
obtenemos

.!.__l_= .!._ (l - x)· '= (-1)(1 - x)·'(-1) • -1-


dx l -x dx (1-x )'
•.!._(l +x +r+x'+ · · ·) = 1+2.r + 3r'+ 4x'+ · · ·
dx
En consecuencia, 1/(1 -x)'es la función genentriz para la sucesión 1, 2, 3, 4, ... ,
mientras que x/(1 -x)' =O + lx+ 2x' + 3-<' + 4x' +•••es la función generatriz para
la sucesión O, 1, 2, 3 •• ..
d) A partirde(c), (d/dx)[x/(1 - x)'] =(d/dx)[O + x+Zx'+ • · ·], o(x + 1)1(1 -x)'= 1 +
2'x + 3'x' + 4'x' + • • •. Por lotanto,(x+ 1)/(1 - x)' genera I', 2'. 3', ... y x(x + 1)1
(1 -x)' genera O'. 12, 2'. 3', ...

a) De la pane (b) del ejemplo 9.5 sabemos que la función generatriz para la sucesión
J, J, J, l ... . es/(x) = 1/() -x). Por lo tanto, la función
438 Capítulo 9 Funciones generatrices

g(x) = f(x)-:i' = 1/(1- x) -:i'


es la función generatriz para la sucesión 1, 1, O, l, l, 1, ..
mientras que la función

h(x) =f(x) + 2x' = ! /(! - x) + 2x'


genera la sucesión 1, l, 1, 3, 1, 1, ...
b ) ¿Podemos usar los resultados del ejemplo 9 .5 para encontrar una función generatriz
de la sucesión O, 2. 6, 12, 20, 30, 42, .. .?
Aquí observamos que

ao=O= O' + O, a 1 =2=12 + 1,


ª' = 6 = 22 + 2, a,= 12 =32 + 3,
a.= 20= 4 + 4, ....
2

En genera], tenemos a,. = n 2 + n, para todo n ~ O.


Usando los resultados de las partes (e) y (d) del ejemplo 9.5 vemos que
x(x + !) +- x - x(x + l) +x(l-x) 2x
(! - x)' (! - x)' (! - x)'_ (! - x)'
es la función generatriz de la sucesión dada. (Aquí la solución depende de nuestra
habilidad para reconocer cada a11 como la suma de n 2 y n. Si no vemos esto, no
podremos responder la pregunta dada. En consecuencia, en el ejemplo 10.5 del
siguiente capítulo analizaremos otra técnica que nos ayude a reconocer la fónnula
para a,.)

Para todon E z·, el teorema del binomio señala que ( l +x)- = (;) + (í)x + (;)r + · · · +
(:)x". Queremos extender esta idea a los casos en que (a)n < O y (b) n noes necesariamen-
te un entero.
9.2 Definiciones y ejemplos: Tém<M de cálculo 439

=-'-'"'t:I......· Paran E z•, el desarrollo en serie de Maclaurin para (1 + x)~ es!! dado por
(1 + x)-• = 1 + ( -n)x + (-n)(-n - l)r'/2!
+ (-n)(-n - 1)(- n - 2)r'/3! + · · ·
i;
_ 1+ (-n)(-n -1)(-n -2)-- •(- n- r + 1) x'

+;-Ir·
,....1 r!
-i ( -1)' ( n

Por lo tanto (1 + xr= (,) + (7)x + (-;)x' + · · · = r:. (-;-)r. Esto generaliza el
teorema del binomio del capírulo I y muestra que es ( 1 + xr« la función generatriz de la
sucesión (,), (7), (-;), (,), ...

Encuentre el coeficiente de x' en (1 - 2.r)1.


Con y;; - 2.r. usamos el resultado del ejemplo 9.7 para escribir (1 - 2x)-7 = (1 + y)-7 ::
-r::.. (;'}y'= r::..(;')(-2xy. En consccucncia, el coeficiente de x' es ( ,')(-2)'= (-1)'
('·:-')(-32)=(32)('1)= 14,784.

u Para cualquier número real n. el desarrollo en serie de Maclaurin para (1 + x"f' es

1 + nx + n(n - l )x2/2! + (n)(n - l )(n -2)r'/3! + • • •


i;
_ + n(n - l)(n - 2) · • • (n - r + 1) r.
1
,-1 r!
Por lo tanto
(I +lxr"'- l + i: (-1/3)(-4/3)(-7/ 3)-- · ((- 3r + 2)/ 3) (lx)'
• ,-1 r!

~ (-1)(-4)(-7} · · · (-3r + 2)
=1 + ""'~~~
t-1
~ ~ -~ -~..,
r! •
y (1 + 3x)''n genera la sucesión 1,-1,(-IX-4)12!, (- IX-4X- 7)13!, ... , (-IX-4X-7) · · ·
(-3, + 2y,1... -.

9.1t Determine el coeficiente dex" enft.x) =(x2 + .rJ + x' + ···"f.


Como (x' + x' + x' + • • •)=x'(I + x ·+ x' + • • •) =.r/(1-x), el cocficicntede.r'' en/(x)
es el coeficiente ele zu en (.x2/(l -x))' =xSl(l -x)'. Por lo tanto. el coeficiente buscado es
el de x1 en (1 - xt', es decir, (7')(- 1)7 = (-I)'('+J- 1)(-I)'= (\º) = 120.
En general, paran E z·. el cocficientedex"en/(x)csO, si Os II s 7. Pan todon 2' 8,
el cocficicntedcX"'cn/(x)cs el cocficicntedcX"'-1 en (1-.x)--', que es (~}(-I )--1 = (::n
Capítulo 9 Funciones geoeratóces

Antes de continuar reuniremos las identidades que se muestran en la tabla 9.2 para
referencias posteriores.

¿De cuántas formas se pueden seleccionar , objetos de n distintos objetos si se permite la


repetición?
Para cada uno de Jos n distintos objetos. la serie geométrica 1 + x + x2 + xl + .
representa Jas posibles elecciones de ese objeto (es decir ninguno, uno, dos, ...). Conside-
rando todos los n djstiatos objetos. la función generatriz es
f(x) = (1 + x + x' + x3 + • • •)",
y la respuesta requerida es el coeficiente dex' enf(x). Ahora bien, por la identidad 7 de la
tabla 9.2 obtenemos

(J+x+x'+x3 +···)"= (-1-)" =--=L


1 •
(1- x) (1-x)" ,..
(n+ i-1)
. x',
'
de modo que el ·c oeficiente de x' es

el resultado que enconuamos en el capítulo 1. .


9.2 Definiciones y ejemplos: Técnicas de cálculo 441

¿De cuántas formas puede un capitán de policía distribuir 24 cargas de rifle a .cuatro ofi-
ciales de forma que cada oficial reciba a1 menos tres cargas. pero no más de ocho?
Las opciones para el número de cargas que recibe cada oficial eslán dadas po< x' + X' + · · ·
+,'_Hay cuauo oficiales, así que la función generatriz resultante esf(x) = (.r' +X'+ · · · + x'"f.
Buscamoselcoeficientede:i2Aen/(x).Con(.x1 +x'+ · · · +r)4 =x12(1 +x+x1+ • • •
+ .t5)' =x 11(( 1 -x6Y(l - x))", la respuesta es el coeficiente de x12 en
, .(1 - x')'(l - xf' = [1- (l)x'+ (l)x12 - (l)x" + ? ][(,') + (-;')(-x) + (;')(-x)2 + · · ·].
que es [(it)(-1)12 - (1)(,')(-1)' + (l)(,')l = l(ID- (:)(:) + (l)] = 125.

Verificaremos que para todo n E V , ('."} = I;..,(;t


Como (l + x'y-A = [(1 + x)"] 2, al compararlos coeficientes (de potencias similaresdex),
elcoeficientedex"cn(J +x)2",guees (2:idebeserigualalcoeficientedcX'en[{;) + (nx
+ (;)x' + · · · + (;)x-J', y esto es (o)(;)+ (;)(_:,) + (;)(_:,) + · · · + (: ){.).Como (:) =
i
(..:r para todo O S r S n, obtenemos el resultado.

Detcnnine el coeficiente dex' en 1/(x 3Xx 2)2 •


Como 1/(x a)-(-l/aXl/l - (;,:/a)))-(-1/a)[l + (x/a) + (xla)' + • • •) paracualquie,a"' O,
podemos resolver este problema encontrando el coeficiente de x' en 1/(x- 3Xx - 2)2 expre-
sado como (-1/3)[1 + (x/3) + (x/3)' + · · · ](1/4)( (-o') + (,') (- x/2) + (,') (-x/2)' + - - - J.
Una técnica alternativa usa la descomposición en fracciones simples.
1 A B C
(x-3)(x-2)2 x-3 + x-2 + (r -2)2
Esta descomposición implica que
1 =A(x -2)2 + B(x - 2)(x-3) + C(x - 3),
o
0·x' +0 ·x + 1 =1 =(A+ B)r2 + (-4A -58 +C)r + (4A +6B-3C).
Si comparamos los coeficientes (de.x2, x y 1, respectivamente). vemos que A + B =o:
-4A + SB + C =
O y 4A + 6B - 3C =1. Al resolver estas ecuaciones obtenemos A = 1. B =
- 1 y C=-1. Por lo tanto,
capítulo 9 Funoones generatrices

El coeficiente de .r' es (-1/3Xl/3)1 + (lnX Jn)1 + (-l/4)¡¡')(-ln)1 ; -{(113)' +


7(Jn)"'].

Usaremos las funciones generatrices para determinar la cantidad de subconjuntos de S;


{ l, 2. 3, ... , 15} de cuatro elementos que no contienen enteros consecutivos.
a) Consideremos uno de taJes subconjuntos, por ejemplo, { 1, 3, 7, 1O} y escribamos
1 s 1 < 3 < 7 < 10 s IS. Yembs que este conjunto de desigualdades determina la$
difcsencias 1-1; O, 3-1; 2, 7-3 ;4, 10-7; 3 y IS- JO; S, y estas difc,encias
suman 14. Si consideramos otto de estos subconjuntos (2, S, 11, IS}, escribimos 1
s 2 < S < 11 < IS s IS; estas desigualdades producen las diferencias 1, 3, 6, 4 y O,
que también suman 14.
Veamos las cosas al rev~: los enteros no negativos O, :z. 3, 2 y 7 suman 14 y soo
las diferencias que surgen de las desigualdades I s 1 < 3 < 6 < 8 s IS (pan el
subconjunto (1, 3, 6, 81).
Estos ejemplos indican que hay una correspondencia uno a uno entte los
subconjuntos de cuatro elementos que queremos contar y las soluciooes enteras de
c 1 + c1 + c3 + e, +es= 14, donde O s ci, e, y 2 :S CJ, c3, e,. (Nota: las desigualdades
ei, e,. e, ~ 2 garantizan que no hay enteros consecutivos en el subconjunto.) La
respuesta es el coeficiente de xl' en

r
f(x) = (1 +X+ .r + + · · ·)(.r + t' + x' + · · •)3 •
(1 + x +x'+x'+ · · ·)-x'(l-x)-'.
Éste es entonces el coeficiente de .r' en (1 - xt', que es igual a (i')H)' ;(":-');
('¡') :495
b) Otra forma de analizar el problema es la siguiente.
Para el subconjunto ( 1, 3, 7, 1O} analizamos las desigualdades estrictas O < 1 <
3 < 7 < 10 < 16 y nos fijamos en el número de enteros que se encuentran estriaa-
mente entre los pares sucesivos de estos n!lmcros. Obtenerl'los O, 1, 3, 2 y S; el Ose
de.be a que no hay enteros entre Oy 1, el 1 se debe al entero 2 entre 1 y 3; 3 para los
enteros 4, 5, 6 entre 3 y 7, etcétera. Estos cinco enteros suman 11. Cuando hacemos
lo mismo para el subconjunto (2, S, 11, IS} , las desigualdades estrictas O< 2 < S <
11 < 1S < 16 producen los resultados 1, 2, S, 3 y O, que tambi~n suman 11.
Por otto lado, vemos que los enteros no negativos O, 1, 2, 1 y 7 suman ll y
aparecen como los nlltneros de enteros distintos entre los enteros que aparecen en
las desigualdades estrictas sucesivas O< 1 < 3 < 6 < 8 < 16, que corresponden al
subconjunto { 1, 3, 6, 8}.
Estos resultados indican que hay una correspondencia uno a uno entre los
subconjuntos deseados y las soluciones enteras de b1 + b:t + b, + b,. + b, = 11, donde
O :S b1, b, y 1 ~ bi. b,, b,.. (Nota: en este caso, b,,b2, b,, ~ l garantizan que no hay
enteros consecutivos en el subconjunto.) El número de soluciones es el coeficiente
dex11 en

g(x) =(l +x +x'+ · · ·)(x+x'+x'+ ···)'(l +x+x'+ ·· ·)


~i'(l-x¡-•.
9.2 Definiciones y ejemplos: Técnicas de «lk,.,lo 443

La respuesta es (-/)(-1)' = 495. como antes. (El lector debería revisar el ejercicio
complementario 17 del capítulo 3.)

En nuestro siguiente ejemplo utilizaremos la última identidad de la tabla 9.2. (Esta


identidad se usó en los ejemplos 9.12 y 9.13, aunque de manera impUcita.)

9.16 Sea/{x) = xl(l - x'j'. Ésia es la función generatriz de la sucesión 0o, a., a,, ...• donde a,=
""'====e, kpara todok EN. La funcióng(x}=x(x + 1)1(1-x'y genera lasueesiónb.,b.,b,, ...• tal
queb, =k', k EN.
La función h(x):;;: f(x)g(x) da. en consecuencia ao b0 + (ao b1 + a 1 b0 )x + (ao b1 + a 1 b1 +
a: b0)r + ••••de modo que h(x) es la función gcocratriz de la sucesión Co, c 1, c1, • •• tal que
para cada k EN,

Por ejemplo. tenemos que


G,- 0 -0' = 0
c,-0·1'+ 1-0'=0
c,-o-2'+ 1-12 +2-0'-1
c,-0-32 + l ·2' + 2· 12 +3 -0' = 6
c,=0-42 + 1-32 +2-22 + 3· 12 +4-0'=20
y en general, ci = L:.i<k-ff_ (Simplificaremos esta fórmula en los ejercicios de la sec-
ción.)
Cuando una sucesión co, e 1, c1, ••. se obtiene de dos funcíones generatrices /(x) [para
ao, ah~•...] y g(x) [parabo. btt "2, .. .], como en este ejemplo, la sucesión c.. c1, e~ ... es
la convoluci6n de las sucesiones ao, ª" ~ ... y bo. bi, Di, ...

Nuestro último ejemplo presenta dos situaciones particulares de la convolución de su-


cesiones.

q.,plo9.17 a) Si/(x)= 1/(1-x)= 1 +x +r +.r'+ ··•y g(x)= 1/(1 +x)= 1-x+r-.r'+ · · ·,


tenemos que
f(x)g(x) - 1/(1-x)(l +x)= 1/(1 -.r') • 1 +x'+x'+x'+ · · ·.
En consecuencia. la sucesión 1,0, 1,0, 1,0•... es la convolución de las sucesiones 1,
1, 1, 1, 1, 1, ... y 1,-1. 1,-1, 1,-1, ...
r
b) Seah(x) = 1 + x + + .r'; es decir, h(x)es la función generatriz de la sucesión 1, 1,
1, 1, O, O, O, ... Sea k(x) = 1/(1 - 3x) = 1 + 3x + (3x'j' + (3x}' + • • •. la función
generatriz de la sucesión 1, 3, 32, 33 , ••• , entonces h(x)k(.x) = I.;:cf•x' es la función
generatriz de la convolución de estas dos sucesiones y

G, 3 1, c1 = 1 + 3 = 4, e, = 1 + 3 + 9 = 13, y
e,. • 311-1 + 311-2 + 311-1+ 3"' n ti!:: 3.
14 Capitulo 9 Funciones generatrices

IERCICIOS 9.2 1. Encuentre las funciones geoeratrices para las siguientes sucesiones. (Por ejemplo, en el caso
dela sucesión O, 1, 3, 9, 27, , 1arespucstapcdidaesx/(1-3x), no I.:3;.t ..1 osimplememe
0+x+lr+!l.r+· ·· .]
•> (:). (l). (:)•...• (!) b) (l), 2(l), 3(!), ... ,8(1)
e) 1,-1,1, - 1,1,-1, .. . dl 0.0.0, 1,1,1, 1, 1,1, . ..
•> o,o.o,6, -6,6, - 6,6, . . . f) 1,0,1.0,1. 0.1 •. . .
g) 1,2,4,8,16, . .. b) 0,0,1,a,a',a', . . . ,af0
2. Determine la sucesión generada por cada una de las siguientes funciones generatrices.
a) f(x)•(2x-3)' b) f(x)=x'!(l - x) e) f(z)=x'/(1-r)
d) f(x) = 1/(1 + 3x) e) f(x) = 1/(3 - x) f ) f(x) = 1/(1-z) + 3x' -11
3. F.n cada uno de los siguientes ejercicios,/(x) es la función gcnera1riz de la sucesión ao, 01,02o •.. ,
mientras que la sucesión br,. b 1 , h¡• ... es generada por la función g(x). Exprese g(..t) en ténni-
nos def(z).
a) b,=3 b) b,=3
b,.=a,., nE N, n+3 1>,= 7
b.=a,,, nEN, nf3,1
e) b, = 1 d) b,=1
b,=3 b,=3
b. =2a,., n EN, n +l,3 l>,=7
b.=2a. +5, nE N, nfl, 3,7
4. Determine la constante (es decir, el coeficiente de,!') en (3r - (2Jx))1'.
s. r
a) Encuentre el coeficiente de :i' en() +X+ + x' + . .• )15•
r
b) Encuentre el coeficiente de i1 en (l + ..t + x2 + x! + · · • paran E z•.
6. Encuentre el coeficiente de r> en (x' + .x' + r + · · · )6.
7. Encuentre el coeficiente de rO en (x2 + x' + ;,! + r + X',1.
8. Paran E Z♦,eocuentre en (1 +x + x2)(1 + x)" el cocficientede(a)z'; (b).r' y, (c)x' para O :Sr
S n+2,rE Z.
9. Encuentre el coeficiente de .x15 en los s iguienles ejercicios.
a) x'(l -2x)" b) (x' - 5x)/(l - z)' e) (1 + x)'/(1- x)'
1 O. ¿De cuántas formas se pueden asignar dos docenas de robots idénticos a cuatro líneas de en-
samble de modo que (a) al menos u-es robots se asignen a cada línea? (b) a1 menos tres, pero no
más de nueve robots se asignen a cada línea?
1 1. ¿De cuántas formas pueden separarse 300 sobres idmticos, en paquetes de 25, entre cuatro gru-
pos de estudiantes, de modo que cada gnipo tenga al menos 150, pero no más de 1000 sobres?
12. Se distribuyen dos cajas de refresco, con 24 botellas de un tipo y 24 de otro, entre cinco
inspectores que realizan pruebas de calidad. ¿De cuántas formas pueden distribuirse las 48
botellas de modo que cada inspector reciba (a) al menos dos botellas de cada tipo? (b) al menos
dos botellas de un tipo particular y al menos treS de la otra?
13. Si se tira un dado 12 veces, ¿cuil es la probabilidad de que la suma de las tiradas sea 30?
14. Carolina red.ne el dinero de sus primos para hacer una fiesta en honor de su tía Si ocho primos
prometen darle S2, $3, $4 o SS cada uno y otros dos le dan $5 o Sto, ¿cuál es la pr-obabilidad
de que Carolina reúna exactamente $40?
15. ¿De cuántas formas puede Tobías seleccionar canicas de una colección grande de canicas azu-
les , rojas y amarillas (todas del mismo tamaño) si la selección debe incluir un mlmero par de
canicas azules?
16. ¿Cómo puede repartir María 12 hamburguesas y 16 hotdogs. entre sus hijos Ricardo, Pedro,
Cristóbal y Jaime, de modo que Jaime reciba al menos una hamburguesa y tres hotdogs, y que
cada uno de sus hermanos reciba al menos dos hamburguesas pero a lo m1s cinco hotdogs?
9.3 Particiones de enteros 445

17 . Verifique que (1 - x - x2-x'-x' - :x!- M es la función generatriz del mlmcro de formas en


que podemos ob<e:oer la suman. con n E N. cuando tiramos un dado un mlmero arbitrario de
veces.
18. Muestre que (1 -4.xtv.t genera la sucesión(~11 ) , n EN.
1 9 . Considere la panc (al cid ejemplo 9.15.
a) Dctcnninc las diferencias en las desigualdades que se obticoco del subconjunto {3, 6. 8,
lS} de S y verifique que esas diferencias se suman corrcaamcntc.
b ) Encuentre el subconjunto de S que determina las diferencias 2, 2. 3. 7 y O.
e) F.ncueotrc el $ubcoojunto de S que deternüna las difereocias a, b, e, d y~. doode O :s a, ~ y
2sb,c, d.
2 0 . ¿De cuánw formas podemos scleccionar siete enteros no consecutivos de ( 1, 2 , 3, . ..• SO)?
21 . Use Las siguientes fórmulas para sumas (que apan::ccn en d material y ejercicios de la scccióo
4.1) pan. simplificarla apraióo dccten el ejemplo 9.16:

íi• ,t.i • k(k + 1)/2, f ;• - ±i'•k(k + 1)(2k + 1)/6,


.-0 ,_1

±e=± i'• k'(k +


,-0 i• l
1)'/4.

22. a) Enrucntrc los primeros cuatro t.Enninos e.o. ci, c1 y c1 de las convoluciones de los siguientes
pares de sucesiones.
i) a. al, b.:-1,paratodon EN.
ii) a.• 1. b.=2", para.todon E N.
iii) ao• 01 =a1 • a, = l;a.•O. n E N,n _, O, l, 2, 3; b.= 1, para todon EN.
b) En<uen~ una fórmula gcncr>I para c. en cada uno de los resultado, de la panc (•~

/:;~=:~~/:.:\~¡~;!'::,':'°~."'
23. Encuentre una fórmula para la convolución de cada par de sucesiones:

e) a.= l. O s n ~
S; b, = n. para todo• e N.
3.a. = 0, para todon it: 4; b0 =0, ó1 = l,ó2 =2, b1 =3,ó.=0, para
todon e'! 4.

9.3
Particiones de enteros

En el estudio de la teoría de números nos enfrentamos al problema de descompoDCC un


entero pOSitivo n en sumandos positivos y buscar el mlmcro de estas descomposiciones,
sin tener en cuenta el orden. Este m1mero se denota con p(_n). Por ejemplo,
p(l) = 1:
p (2) = 2: 2= 1+ 1
p(3) = 3 : 3 =2 + 1 =1 + 1 + 1
p(4) = 5: 4 = 3 + 1• 2+ 2 • 2+ 1+ 1 • 1+ 1+ 1+ 1
p ~ )= ~ 5 =4 +1• 3+2 • 3 + 1 + 1 • 2 + 2 + 1
= 2 + 1 + 1+1 •1+1+ 1+1 + 1
Qusitramos obtener p(.n) para un n dado sin tener que enumerar todas las particiones.
Necesitamos una herramienta para llevar un registro de los m1mcros 1, 2 •... , n que se
usan como sumandos de n.
446 Capitulo 9 Funciones generatrices

Si n E z+. el número de unos q ue podemos usar es O o l o 2 o... La serie de potencias


1 + x + x1 + x1 + x' + · · · lleva un registro de ello. De manera similar, I + xl + x' + x> + · · ·
lleva el registro del número de doses que hay en la partición den, mientras que l + x3 +~
+ x9 + · · · Ueva el registro del número de treses. Por lo tanto, para determinar p( 10), por
ejemplo, queremos obtener el coeficientedex1° enf(x) = (1 + x +xl +r + • • •) • (1 +r+
x'+,6+ -· -)(1 + x'+,6+,6+- • ·)· · •(l + x"+x"'+ • • •)oeng(x) = (l +x+x' +x' + · •·+
x")(l + r +,6 + · · · +x")(l + x' +,6 +x') • · · (1 +x1').
Preferimos trabajar con /(x), puesto que se puede escribir en la forma más compacta
10
1 1 1 1 1
/ (x ) = - - - -- - · · · --=íl- -
(1-x) (1 - x') (1-.r') (1 -xui¡ ;. ,(1-x')"

Si extendemos este producto más allá de i = 10, obtenemos P(x) ;;;; fL:1[1/(1- X')], que
genera la sucesiónp(O), p(l), p(2), p(3), ... , donde definimos p(O) = l.
Por desgracia. es imposible calcular realmente el número infinito de ttrminos en el
producto P(x). Si solamente consideramos II;.,¡[ 1/(1-x')J para un r fijo, entonces el coefi-
ciente de.r4 es el número de particiones den en sumandos que no son superiores ar.
A pesar de la dificultad en el cálculo de p(_n) a partir de P(:x) para valores grandes den,
la idea de la función generatriz es útil para el estudio de ciertas descomposiciones partiai•
lares.

Encuentre la función g~neratriz del níimero de formas en que un agente de publicidad


puede adquirir n minutos (n E Z ..) de tiempo en la radio si los espacios de tiempo para los
comerciales se venden en bloques de 30. 60 o I 20 segundos.
Sea 30 segundos una unidad de tiempo. Entonces la respuesta es el número de soluci~
nes enteras de la ecuación a+ 2b + 4c =2n, con O :s. a, b, c.
La función generatriz asociada es
/(x)= (1 + x +x'+ · · ·)(l +x'+x'+ · · ·)(1 + x'+x'+ • • •)
1 1 1
=l - xl-xll-x4 '
y el coeficiente dexl" es el número de descomposiciones de 2n en unos, doses y cuatros, la
respuesta del problema.

Encuentre la función generatriz de pJ...n), el número de descomposiciones de un entero


positivo nen sumandos distintos.
Antes de comenzar. consideremos las 11 descomposiciones de 6:
1) 1 + 1 + 1 + 1 + 1 + 1 2) 1 +1+1+1+2
3) 1+1+1+3 4) 1 +1+4
5) 1+1+ 2 +2 6) 1 + 5
7) 1+2+ 3 8) 2 +2+ 2
9) 2+ 4 10) 3 + 3
11) 6
Las descomposiciones (6), (/), (9) y ( 11) tienen sumandos distintos, por lo que p,(6) = 4.
9.3 Particiones de enteros 447

AJ calcular p/...n), para cuaJquier k E z• existen dos opciones: O bien k no se usa como
uno de los sumandos den, o sí se usa. Esto se puede contar mediante el polinomio 1 + x1
y, en consecuencia, la función generatriz de estas descomposiciones es

P.(z) = (1 + xXl +.r)(l +.r) · · · = Il(l +r).


,.,
Para cualquier n E z•.
pJ.n) es el coeficiente de 1t' en (1 + xXI + r)· ·
·(l + Jt').
[DefinimospJ.0) = l.] Cuaodon = 6, el coeficientedex'en (1 + x)(I +.r)· · ·(I + x') es 4.

Si consideramos las disposiciones del ejemplo 9 .19 veremos que hay cuatro descomposi-
ciones de 6 con sumandos impares: ( 1), (3), (6) y ( 10). Tambifn tenemosp,(6) = 4. ¿Es una
coincidencia?
Sea prJ..n) el número de descomposiciones de n en sumandos impares. cuando n ~ l.
Definimospo(O) = l. La función gcocratri2de lasucesiónpo(O),po(l),p,(2), ... estidada por

P,(x)=(l +x +x'+.t'+ ···)(l +.t'+.r'+ · · ·)(l +.t'+x10 + · · ·)·


(l+x'+ .. ·)· .. =-1_ __1_ __ 1___ 1____ _
1-x 1-.t' 1-.t' 1 - x'

Ahora bien, como


1 - x' 1 - x' 1-.r'
l+x=-¡-:-;, 1 +x'= 1-x'' 1+.t' = 1 - .t''
tenemos

P.(z) • (1 + x)(l + .r)(l + .r)(l + x') · · ·


1-x' l-x'l-.r'l -i' 1 1
= -¡-:-; 1-x' l - .t' 1-x•-- · • 1-x 1 - x• · · · • P.(x).
De la igualdad de las funcionesgene,alrices, tenemosquepJ.n) = A,(n), para todon ;o O.

De nuevo. solamente permitiremos el uso de sumandos impares pero. en este ejemplo,


cada uno de dichos sumandos (impares) debe aparecer una cantidad impar de veces, o no
aparecer en la suma. Por ejemplo. existe una de tales descomposiciones del entero 1 (a
saber, 1) pero ,w existe una descomposición de este tipo para el entero 2. Para el entero
3 tenemos dos de estas descomposiciones: 3 y 1 + 1 + 1. Cuando analizamos las posibili-
dades del entero 4, encontramos la descomposición 3 + 1.
La función generatriz de las descomposiciones descritas es

/(x)=(l + x+.t'+.t' + · ··)(l+.t'+.:t'+x"+ ·· ·)(l +x'+.:t"+x"+ ···)·· ·


= ñ(1+ f p•1)(2ó+1)) .
.l:•O i• O
448 (.apítulo 9 Funciones generatrices

La función generatriz ne está dada por

(*) (x+x'+x'+ · ··)(x'+x"+x"+ •· ·)(x'+x"+x"+ •··) ·· ·


ya que si así fuera, el producto no podría tener términos donde.x apareciera con una potcD-
cia finita. La situación dada en la ecuación (•) ocurriría si pensáramos que todo entero
positivo impar debería aparecer como sumando al menos una vez.. En tal "descomposi-
ción", el número de sumandos y la propia suma serían infinitos. En consecuencia, aunque
no se enuncie en fonna explicita. debemos observar que cada sumando impar podría no
aparecer, situación tenida en cuenta por el (primer) sumando 1. que aparece en cada factor
de/(x). De hecho, éste es el caso salvo cuando se trata de un número finito de sumandos
impares. Por supuesto, cuando un sumando impar aparece en una descomposición, lo hace
una cantidad impar de veces.

Cerraremos esta sección con la idea llamada grafo de Furer. Este grafo usa filas de
puntos para representar la descomposición de un e ntero donde el número de puntos por
fila no aumenta al pasar de cuaJquier fila a la ínferior.
En la figura 9.1 vemos los grafos deFerrerde dos descomposiciones de 14: (a) 4 + 3 +
3 + 2 + I + l y (b) 6 +4 + 3 + l . El grafo de la pane (b) es unazrasposición del grafo de la
pane (a), y viceversa, pues podemos obtener un grafo a partir del oteo mediante un inter-
cambio de filas y columnas.

(b)

(a)
Figura 9.1

A menudo estos grafos indican algunos resultados acerca de descomposiciones. Aquí


vemos una descomposición de 14 en sumandos tales que 4 es el mayor y una segunda
descomposición en exactamente cuatro sumandos. Existe una correspondencia uno a uno
entre el grafo de Ferrer y su trasposición, por lo que este ejemplo es un caso particular del
resultado genera]: el número de descomposiciones de un entero nen m sumandos es igual
aJ número de descomposiciones de n en sumandos tales que m es el sumando más grande.

:JERCICIOS 9.3 1. Encuentre todas las descomposiciones de 7.


2. Determine la función generatriz de la sucesión ao, 41, 4 2 , ••• , donde ~ es el número de
descomposiciones del entero n en (a) sumandos pares: (b) sumandos pares distintos; y (e)
sumandos impares distintos.
3. Enftx) = [1/(1 -x)][l/(1 -x')][l/(1 -.r'JJ, el coeficiente de .t' es 7. lnterpreteeste resultado en
términos de las descomposiciones de 6.
9.4 La funóón generatriz exponencial 449

4. Encuentre la función generatriz del mlmcro de soluciones enteras de


=
a) 2w + lr + Sy + 1z n, Os w,x.y.z
b) 2w+lr+Sy+1z=n, Osw, 4sz,y, Ssz
e) 2w+lr+Sy+1z•n, 2sw::s4:sx:s7sy:sl0sz

S. Encuentre la función generatriz del número de descomposiciones del entero n no negativo en


sumandos tales que (a) cada sumando debe aparcccr una cantidad par de veces; y (b) cada suman-
/ do debe ser par.
6 . ¿CUil es la función generatriz del número de desc:omposicicoes den E Nen sumandos tales
que (a) no pueden aparecer mts de cinco veces; y (b) no pueden ser mayores que 12 ni aparecer
mú de cinco veces'!
7. Muestre que el nllmcro de descomposiciones de un entero positivon ta.les que ningún sumando
aparece mú de dos vcoes es l&tl&l al n11mero de descomposiciones den donde ningún sumando
es divisible entre 3.
8. Muestre que el mlmero de descomposiciones den E z· tales que ningún sumando es divisible
entre 4 es igual al número de descomposiciones de n donde ningún sumando par se repite
(aunque los sumandos impares podñan repetirse).
9 . Use un grafo de Fcrrer para mostrar que el nwnero de descomposiciones de un entero n en
sumandos que no sean mayores quemes i¡ual al número de descomposiciones den en cuando
mucho m sumandos.
1O. Por medio de un grafo de Fem:r, muestre que el nllmero de descomposiciones den es l¡u.a) al
mlmero de descomp>siciooes de 2n en n sumandos.

9.4
La función generatriz
exponencial

El tipo de función generatriz con el que hemos trabajado se conoce como la función
gcneratri:z. ordinaria de una sucesión dada. Esta función surgió en los problemas de selec-
ción, donde el orden no era importante. Sin embargo, ahora que pasarnos a los problemas
de disposiciones donde el orden es crucial, buscamos una herramienta semejante. Para
cnconuarta. regresaremos al teorema del binomio.
Paracualquiern E z·, (1 +i)'= (:)+ (;}x + (;},' + · • • +(:}r'. por lo que (1 u)'co la
función generatriz (ordinaria) de la sucesión (;H;J. (;). ...
,(;). O. O, ... Al analizar esta
idea en el capítulo 1, tamb~n escribimos(:)= C(n. r) cuando quisimos enfatizar que(:)
representaba el número de combinaciones den objetos tomados de r en r, con O :S r :S n. En
consecuencia, (1 + x)' genera la sucesión C(n, O), C(n, 1), C(n, 2), ... , C(n. n), O, O, ...
Para cada O :S r s n,

C(n, r) = - •-!- = ( 2.) P(n,r),


r !(n - r)! r!
donde P(n, r) es el número de pen:nutaciones den objetos tomados de r en , . Así,

(1 + x)" = C(n, O) + C(n, l)x + C(n. 2).r' + C(n, 3).r' + · · · + C(n, n)x"
:i' x' x"
• P(n, O) + P(n, l)x + P(n,2)2! + P(n, 3)
31 + · · · + P(n,n);;¡.
150 capítulo 9 Funciones generatrices

Por lo tanto, si nos fijamos en e1 coeficiente deX/r! que aparee.e en (1 + i'f-, con Os r :S
n, obtenemos P(n, r). Con base en esta observación, tenemos la siguiente definición.

,efinición 9.2 Para una sucesión de números reales ao, ai, az, a3, ••• ,

x' x' • x'


f(x)=ao+a ,x + a,- +a,-+ · · · = Iar:-
2! 3! i-0 1!
es la ftmci6n genera1riz exponencial de 1~ sucesión dada.

Si analizamos el desarrollo en serie de Maclaurin para e'", tenemos que

e'=l+x+==+~+~+---=:Í: :'.
2! 3! 4! i-0 i!'

de modo que r es la función generatriz exponencial de la sucesión 1, l. 1, ... (e'" es la


función generatriz ordinaria de la sucesión 1, 1, 1/2!, 1/3!, 1/4!, ...)

Nuestro siguiente ejemplo muestra que esta idea nos puede servir para contar cienos
tipos de ordenaciones.

¿De cuántas formas podemos orde nar cuatro letras de ENGINE?

Tabla 9.3

EEN N 4!/(2!2!) EGNN 4!/2!


EEGN 4!/2! E l NN 4!/2!
EE IN 4!/2! GI NN 4!/2!
EEG I 4!/2! El GN 4!

En la tabla 9.3 enumeramos las selecciones posibles de tamaño 4 de las letras E. N, G,


I, N, E, junto con el mlmero de ordenaciones que dete rminan estas cuatro le tras.
Obtenemos entonces la respuesta por medio de una función generatriz exponencial.
Para.la letra E utilizamos [ l + .x+ (.t'/2!)] ya que hay que ordenar 0, 1 o 2 letrasE. Observe
que el coeficiente de x212! es 1, el número de formas distintas de ordenar (solamente) dos
E. De manera similar, tenemos [l + .x + (.xln!)] para la disposición de O, 1 o2 letras N. Las
orde naciones de las letras Ge I se representan mediante (1 + .x).
En consecuencia, encontramos que Ja función generatriz exponencial es/(.x) =[l + x+
(r/2!)]2(1 + x )'; afirmamos que la respuesta pedida es el coeficiente de x'/4! en_/!.x).
Para dar sentido a nuestra afirmación, consideremos dos de las ocho formas en que
aparece e1 término x'/4 ! en el desarrolJo de

f(x) = [l + x + (.r/2!)](1 +X+ (x'/2!))(1 + x)(l + .t).


9.4 La función generatriz exponencial 451

1) El tmrúno aparece a partir del producto (r/2!)(,'/2!)(1)(1), donde (r/2!) se toma


de cada uno de los dos primeros facton:s (a saber, [ 1 + x + (r/2!)1) y 1 se tolllll de
cada uno de los dos últimos factores [a saber, (1 + x)]. Entonces, (r/2!)(.t'/2!)(1)(1)
= .r'/(2!2!) = (4!/(2!2!))(.r'/ 4!) y el coeficiente de .r'/4! es 4!/(2!2!), el número de
formas en que podemos ordenar las cuatro letras E, E, N, N.
2) El término también aparece a partir del producto (,'/2!)(1)(x)(x), donde (r/2!) se
toma del primer factor(a saber, (1 + x + (r/2!)]), 1 se toma del segundo factor (de
nuevo, (1 + x + (r/2!)]), y x se toma de cada uno de los dos últimos facto= [a
saber, ( 1 + x)]. En este caso, (r/2!)(1)(x)(x) =.r'/2! = (4!/2!)(.r'/4!), de modo que el
coeficiente de (.r'/4!) es 4!12!, el nllln.ero de formas en que podemos ordenar las
cuatro letras E, E, G, l.

En el desarrollo completo de j(x), el téllTUno relativo a .r'/4! es

x' x' x' x' x" x' x' )


- +-+-+-+-+-+-+~
~ !2! 2! 2! 2! 2! 2! 2!

= fü~!) + ffi) + ffi) + ffi)+ ffi)+ ffi) + ffi) + ](f).


41

y el coeficiente de x'/4! es la respuesta ( 102 disposiciones) producidas por los ocho resul•
tados de la tabla.

x' x' ~ x2 x3 x'


e'= 1 +x +-+-+-+ ··· e-• = 1-x +---+-- • • •
2! 3! 4! 2! 3! 4!
Si sumamos estas series, obtenemos

x' x' ...)


t "+ t-•=2 ( 1+-+-+
2! 4! '
o
e•+e-" r
- - - ! + - + - +· ·
x'
2 2! 4!
Si restamos e~ de e. tenemos que

e'- - e-• x> x 5


- - -=x +-+-+···.
2 3! 5!

~ 9~5. Un barco lleva 48 banderas, 12 de cada uno de los colores rojo. blanco, azul y negro. Se
colocan doce de estas banderas en un mástil para comunicar una sefial a otros barcos.
a) ¿Cuántas de estas señales utilizan un número par de banderas azules y un nllmero
impar de banderas negras?
52 Capítulo 9 Funciones generatrices

La función generatriz exponencial

r' x'
f(x)= ( l+x+-+-+··· r' z'
)' ( !+-+-+··· x' x'
)(x+-+-+···)
2! 3! 2! 4! 3! 5!
tiene en cuenta todas las señales de este tipo formadas por n banderas, n ~ 1. Los
últimos dos factores de /{x) restringen las señales a un número par de banderas
azules y un número impar de banderas negras, respectivamente.
Como

f(x) =<•"l,fe'+,-'
~-2-· - )(•'- --,-·)
2
- =(1
¡)(t2')(<"- ,-"'J =¡1C•"-1)
=!f:i<4xY
4~
_1)=(!)i<4xY,
1! 4 i• l 1!

el coeficiente de x''/12! en ft.x) produce (l/4X4") = 4" señales formadas por 12


banderas con un número par de banderas- azules y un número impar de banderas
negras.
b) ¿Cuántas de las señales tienen al menos tres banderas blancas o no tienen ninguna?
En este caso. usamos la función generatriz exponencial

x' x' ···)(! +-+


g(x)= ( l+x+-+-+ x' -+· x' x' +· · ·)'
z' · ·)(l+x+-+-
2! 3! 3! 4! 2! 3!

=•{··-x -f)<•"l'
=•+•- -f) e" - (1/2)x',"
X = Xr" -

= i ~ - Xi~_)'._ (x /2)(i º=2'.l_ 2


;-o 1! i-0 ,! )
,! ·-o

Aquí, el faccor (1 + + fi i
+ · · ·) :;:; e' - x- "F.- de g(x) restringe las señales a
aquellas que tienen tres o mú de las 12 banderas blancas o ninguna de ellas. La
respuest.a para el número de señales buscadas es el coeficiente dex11/12! eng(x). Si
nos fijamos en cada sumando (con una suma infinita), tenemos que:

Í, < ~i; e n este caso tenemos el t6rmino ~ = 4


4 12
i) (-fri), de modo que eJ
i-0 ,!
coeficiente de x 11/ 12! es 4 11;

\J!;..o < ,!~Y ); ahora vemos que para obtener .x /12! debemos fijarnos en el tér-
3 12
ü)
mino.x[(3.x)11/ l 1!] = 3 11 (.x12/ l l !) = (l2)(311)x12/12!, y el coeficiente dex12/ 12! es
(12)(311 ) ; y

iü) (x'/2J f_ (Jx)');


para este último sumando, observamos que (x'/2)[(3x)'°/I0!J
\-o ,!
= (1/2X3'')(x12/10!) = (l/2X12Xl 1X3'')(x"/12!), y esta vez el coeficiente de
x"/12! es (112Xl2X11X3'').
9.4 la función geoe<atriz exponencial 453

En consecue ncia. el m1mero de señales con 12 banderas y al menos tres banderas


blancas o ninguna de ellas, es

4 12 - 12(311) - (1/2)(12)(11)(3"') = 10,754,218.

Nuestro último ejemplo nos recuerda algunos resultados pasados.

Una empresa contrata a 1 t nuevos empleados. cada uno de los cuales es asignado a uria de
cuatro subdivisiones. Cada subdivisión recibe aJ menos a uno de ellos. ¿De cuántas formas
se pueden hacer las asignaciones?
Si llamamos a las subdivisiones A. B, C y O, podemos contar. en forma equivalente, e l
número de sucesiones de l l letras en las que al menos aparece una vez cada una de las
letras A, B, C y D. La función generatriz exponencial de estas disposiciones es

f(:t) = (:t + -x+-+::.+


' x ' ' · · ·)' = (,'-1)'= •" -4e" +6e" -4e' + l.
2! 3! 4!
La respuesta es entonces el coeficiente de x 11/ l 1! en/(.x):

411 -4(311) + 6(211) - 4(111) ±c-1y(4)(4-


= ,.. ¡
i)11,

La forma de esta respuesta nos debe recordar algunos de los problemas de conteo del
capítulo 5. Si dejamos de lado el vocabulario. estamos contando el número de funciones
sobreyectivasg: X-+ Y donde lxl
= 11, =4.1rl

EJERCICIOS 9.4 1 . Encuentre la función generatriz exponencial de cada una de las sucesiones siguientes.
a) 1,-1,1, -1,1, -1,.. . b) 1,2,2',2', 2', . . .
e) 1,-a,a2,-a3,a', ...• aER d) 1,a2,a'.a\ . . . ,aER
e) a.•',a',•'• ... •• E R r) O, 1,2(2),3(2'),4(2'), ...
2. Determine Ja sucesión generada por cada una de las siguientes funciones generatrices
exponenciales.
a) /(x) = 3i" b) /(x)=6e"'-3e"'
e) /(x)-,'+r d) "f(x) =?-lr'+Sx'+ 7x
e) /(x) =1/(1- x) f) /(x) = 3/(1 - 2x) + ,'
3. En los siguientes ejercicios. la función/(x) es la función generatriz exponencial de la sucesión
ao, O¡, a1, ••• , mientras que la función g(x) es la función generatriz e::itponcncial de la suce-
sión bo, b1o b:. ... Exprese g(x) en t&mi00$ de /(x) si
a) b,=3 b) •. =S", nEN
b,.aa,., nEN,n+.3 b,=-1
b,..::.a,., nEN,n+3
e) b1 =2 d) b,~2
b,=4 b,=4
b.=24,,, nEN,n+l,2 b,=8
b.=:Z-..+3, nEN,n+l,2,3
capítulo 9 Funciones generatrices

4. a) Para el barco del ejemplo 9.25, ¿cu'1ltas scña1es usan al menos una bandera de cada color?
(Resuelva esto mediante una función generatriz exponencial.)
b) Enuncie la parte (a) en una forma alternativa que use el concep<o de función sobrcyectiva.
e) ¿Cuíntas sei\ales hay en el ejemplo 9.25, si el total de banderas azules y negras es par?
S. Encuentre la función generatriz exponencial del número de formas en que podemos ordenar n
letras. n ~ O. seleccionadas de cada una de las siguientes palabras.
a) HAWAil b) MISSISSIPPI <) ISOMORPHISM
6. Para ta pane (b) del ejercicio S. ¿cuál es la función generatriz exponencial si la disposición
debe contener al menos dos letras I?
7. Supongamos que la empresadclcjemplo9.26contrataa 25 empleados. 06 la función generatriz
exponencial para el número de formas de asignar estas personas a las cuatro subdivisiones, de
modo que cada subdivisión reciba al menos 3 personas, pero no más de 10.
8. Dadas las sucesiones ao, a 1• a 2, • •• y bo. b1, b2, • •• , con funciones generatrices exponenciales
/(x), y g(x) respectivamente, muestre que si h(x) ;/(x)g(_x), entoncesh(x) es la función generatriz.
exponencial de la sucesión co, Ci, c1, ... , donde e,. = 4 -o~)a,,b._, para cada n ~ O.
9. Si generarnos una sucesión ternaria (0, 1, 2) de 20 dígitos en fonna aleatoria, ¿cuál es la
probabilidad de que (a) tenga un número par de unos? (b) tenga un número par de unos y un
número par de doses? (c) tenga un número impac de ceros? (d) el total de ceros y unos sea
impac? (e) el número total de ceros y unos sea par?
10. ¿CUántas sucesiones cuaternarias (O, l , 2, 3) de 20 dígitos hay cuando (a) se tiene al menos un
2 y un número impar de ceros? (b) ningún símbolo aparece exactamente dos veces? (e) ningtln
símbolo apare.ce exactamente tres veces? (d) hay exactamente dos treses o ninguno?
1 1. Se van a distribuir 2S contratos (para 25 componentes diferentes del transbordador espacial)
entre cinco compaMas: c 1, c1, •• • , cs. Encuentre la función generatriz exponencial para el
número de formas en que es posible otorgac estos contratos de modo que (a) La compai!iía e,
obtenga al menos cinco contratos y las demás al menos dos; (b) cada compaAfa oblenga al
menos un contrato, la compaiHa c1 obtenga más contratos que la c1 y que la compaAfa c 2 obtco-
ga a lo más cinco contratos.

9.5
El operador de suma

Esta última sección presentará una técnica que nos ayudará a pasar de 1a función generatriz
(ordinaria) para la sucesiónao, a 1• a_z, • • . a la función generatriz de la sucesión ao- ao+a1,
ao+ a¡+ Clz, .•.
Para/(x) = ao. a,x + a,x' + a,z'+· • •.consideremos la función/(x)/(1 - x),

f(x)/(1-x) = [ao+ a,x + a,x' +a,x3+ · · ·)[I + x +x' +r' + • • •]


= ao + (ao + a,)z + (ao +a,+ a,)x'
+ (ao+a, +a,+a,)x'+ • •·,
de modo que/(x)/(1 - x) genera la sucesión de sumasao, ao + a 1• ao + a1+a1, ao +a1 +a,+
a 3, ••• Ésta es la convolución de la sucesión ao. a., a 2 , • • • y la sucesión bo, b1, ~•..• donde
b.= 1 para todo n EN.
Esta técnica será útil en el ejemplo siguiente.
9.5 El operador de suma 455

Encuentre una fórmula para 02 + 12 + 21 + · · · + n 1 como función den.


Como e n la sección 9.2, comenzamos con g(x) = 1/(1 - x) = l + x + x2 + · · ·. Entonces

1
(-1)(1 - xr'(-1) = -- = dg(x) = 1 +2x + 3.r' + 4x'+ · .. ,
(1 -x)' dx
de modo que x(l - x)2 es la función generatriz para O, 1, 2, 3, 4, ... Si repetimos esta
técnica, tenemos que

X!_ [x(dg(x>)] = x(l +x) =x + 2'x2 + 3'x'+ . ..


dx dx u-~ ·
por lo que .r(l + x)/(1 - xY genera 02, 11, 21, 32, . .. Como consecuencia de nuestra obser-
vación anterior,

x(l + x) 1 x(l + x)
(1-x)' (1-x) (1-x)'

es la función generatriz de 02, 02+ 12, 02+ 12+22, 0 2 + 12+22 + 32, ... Por lo tanto, el
coeficientedex" en.x(l + x)/(I -x'f es L:':.i
2
• Pero el coeficiente dcx• cn x(J + x)/(1-x'f
también puede calcularse como sigue:

x(l +-
- x) = (x +r)(l-x)-'= (x +x2)[(-4) + (-4) (-x) + ( -4) (-x)'+ · · · ]
(1 - x)' O 1 2 '
donde el coeficiente de x" es

(n~\)<-1i-- + (n~\ )<-1i•-•


1

= (-1)"_,(4 + (n -1)-1)(-1)"- '


n-1
+ ( - 1)"- ' (4+ (n-2) - 1) (- 1)•-•
n-2
n+2) (n+l) (n + 2)! (n+l)!
= ( n - 1 + n -2 = 3!(n - 1)! + 3!(n - 2)!
= ¡ [(n + 2)(n + l)(n) + (n + l)(n)(n -1)]
= ¡(n)(n + l)[(n + 2) + (n - 1)] = (n)(n + 1)(2n + 1)/6.

EJERCICIOS 9 .5 1. Contim1e el desarrollo de las ideas planteadas en el ejemplo 9.27 y obtenga la fórmula
4,.,/=[n(n + 1)/2) 2

2. a) Encuentre la función generatriz de la sucesión de productos O• ( - 1), 1 • O, 2 • 1, 3 • 2,


4. 3•... , ¡. (i - 1)....
b) Use el resultado de la parte (a) y obtenga una fórmula para 2 ¡.oi(i - 1).
56 Capítulo 9 Funóones generatrices

3. Sea/(x) la función gcnecatriz de la sucesión ag. a., a2, . . . ¿De qué sucesión es fuDCión genenrriz
(1-x)/(x)?
4 . Si/(x) =k -oa.x-, ¿cuál es la función generatriz de la sucesión ao, Do+ ai, Do + a 1 +ai,a.,+a1
+a2 +a¡. ...? ¿Cuál es la función generatriz de la sucesión ao, ao+ai.ao+ a1 +a2,a1+a:+a,,
a1 +aJ+a., ...?
5. Sí/(x) =k-o a,.:<', muestre que ¿::., {L:;a,) x' =x/(xY(I - x).
6 . Sea/(x) = ¿°_0 a;~. con/(}) -=- I :O a;, un número finito. Verifique que el cociente V(.r)-
j(l)l/(x - I)es la función generatriz de la sucesión So, 1 1• s2• . dondes. = 4 __..a,.
• ••

7 . F.ncuentrc la función generatriz de La sucesión ao. a1, a1, ••• , donde a. = }:_0 (1/i!), n EN.

9 .6
Resumen y repaso histórico

A principios de1 siglo xm, el matemático italiano Leonardo de Pisa (c. 1175-1250), en su
Liber Abad, presentó al mundo europeo Ja notación aribiga de los numerales y los
algoritmos para la aritmética. En este texto también surgió el estudio de la sucesión O, 1, 1,
2, 3, 5, 8, 13, 21, . . . , la cual puede de finirse en forma recursiva como F 0 O, F 1 I y= =
F • • 2 = F • • 1 + F~, n 2:: O. Como Leonardo era hijo de Bonaccio, la sucesión recibió el
nombre de números de Fibonacci. (Filius Bonacci es la forma en lalÍD de "hijo de Bonaccio".)
Si consideramos la fórmula

vemos que F0 = O, F 1 = 1, F 2 = 1, F, = 2, F,. = 3, ... Es decir, esta fórmula determina cada


número de Fibonacci en función de n. (Tenemos entonces la solución general para la rela-
ción recursiva de Fibonacci. Veremos más detalles de esto en el siguiente capítulo.) Sin
embargo, esia fórmula no fueobteruda hasia 1718, cuandoAbraham DeMoivrc (1667-1754)
obtuvo el resultado a partir de la función generatriz

l [ 1 1 ]
f(x)=1-:-x'=V5 1-(1\V5~-1-(1-2 ·
V5)x
Leonhard Eu1er ( l 7C17-l 783) extendió las técnicas de la función generatriz y avanzó en
el estudio de las descomposiciones de enteros en su obra de dos volúme nes /nzroductio in
Analysin lnfinitorum (1748). Con

P(x) =-1- __1___l__,, =;.tl


1 - xl-x'l-x'
Ji-1-.,
- x'

1.enemos la función generatrizdep(_O),p(l ), p(2), . . . , dondep(_n) es el número de descom-


posiciones den en sumandos positivos y se define p(O) = 1.
9.6 Resumen y repaso histórico 457

Loonhard Euler (1707-1783)

En la i!ltima parte del siglo xvm surgieron más desarrollos acerca de las funciooes
generatrices junto con las ideas de teoría de la probabilidad, especialmente con lo que
ahora se llama "función generatriz de momentos... Estos conceptos relacionados entre sí
fueron presentados en forma completa y por primera vez por el gran estudioso Pierre--
Simon de Laplace ( 1749-1827) en su obra de 1812 ThLorit Analyriqut dts Prolxzbiliris.
Por 1lltimo, mencionamos a Norman Mac1eod F-errers (1829-1903), en cuyo honor re-
ciben su nombre los grafos de Ferrer.
Desde nuestro punto de vista. el estudio de las funciones generatrices ordinaria y
exponencial proporciona una tknica poderosa que unifica las ideas de los capítulos 1, 5
y 8. Al extender nuestra experiencia anterior con los polinomios al caso de las series de
pote ncias y al extender el teorema del binomio a ( 1 + x'f a los casos en que n no tiene que
ser positivo ni entero, encontramos las herramientas ncc:esarias para calcular los coefi•
cientes de esca.s funciones generatrices. Esto valió la pena, pues los cálculos algebraicos
que realizamos han tenido er, cuenta todos los procesos de selección que intentábamos
analizar. Tambifn observamos que ya habíamos estudiado las funciones generatrices en un
capftuJo anterior y cómo surgieron en el estudio de las descomposiciones.
El concepto de descomposición de un entero positivo nos permite ahora completar los
resúmenes de nuestros análisis anteriores acerca de las distribuciones. dados en las tablas
1.8 y S. 13. Ahora podemos trabajar con las distribuciones de m objetos en n( s m) reci-
pientes para los casos en que ni los objetos ni los recipientes son distintos. Estos casos son
analizados en las entradas de las filas segunda y cuarta de la tabla 9.4. La notaciónp(m, n),
que aparece e n la úJtimacolumnadcesta tablL se usa para designare) númerodedcscom•
posiciones del entero positivo m en exactamente n sumandos (positivos). (Esta idea será
analizada con más detalle en e l ejercicio complementario 3 del siguiente capítulo.) Los
458 capítulo 9 Funcione5 generatrices

tipos de distribuciones de las filas primera y tercera de esta tabla también se enumeran en
]a tabla 5.13. Los incluimos de nuevo para comparar y tener una tabla completa.

-
Tabla 9.4
Los objdm son Los rteipientesson Alguno$ ,..;pie,,ta, N....... de

..: -•)
d.istint06 poedm estar vados distribadoDtS

No Sí Sí (
No No Sí (1) p(m), paran = m
(2) p(m, 1) + p(m,2) + · · · +
p(m, n), para n < m
No Sí No (..::=:-•¡ = (::!) = (~:/)
No No No p(m,n)

El lector interesado en un análisis similar aJ del material de este capítulo debe consultar
el capítulo 2 de C. L. Liu [3] y el capítulo 6 deA. Tucker [7]. El texto de J. Riordan [5] trata
ampliamente las funciones generatrices ordinaria y exponencial. Un interesante artículo
de referencia acerca de las funciones generatrices, escrito por Richard P. Stanley. aparece
en el texto editado por G-C. Rota [6]. El texto de H. S. Wtlf [8] trata las funciones generatrices
y algunas formas de aplicarlas en las matemáticas discretas. F.ste trabajo muestra tambi~n
cómo estas funciones proporcionan un puente entre las matemáticas discretas y el análisis
continuo (en particular, la teoría de funciones de variable compleja).
El lector interesado en aprender más acerca de la teoría de descomposiciones debe
consultar el capítulo 10 de l. Niven y H. Zuckerman [4].
Por último, un extenso análisis de la función generatriz de momentos y su uso en la
teoría de probabilidades aparece en el capítulo 3 de H. J. Larson [2] y en el capítulo Xl de
la extensa obra de W. Feller [I].

BIBLIOGRAFÍA

1. Feller, William, A n lmroduction to Prebability Theory and lis Applications, Vol. 1, 3• cd.,
Nueva York, Wiley, 1968.
2. Larson, Harold J., lruroduction lo Probability TMory and Szatistical lnfe~nce, 2• ed .• Nueva
York, Wilcy, 1969.
3. Llu, C. L., lntr-t><.WCrion to Combinatorial Ma1lwnatics, Nueva York, McGraw•Híll, 1968.
4. Ni ven. Jvan, y Herbert Zuckennan, An ln.troduction to W Theory of NUltW<rs, 4• ed.• Nueva
York. Wiley. 1980.
S. Riordan, John,An lntroduction to Combi.naU>TUJIAnalytis, Princeton, NJ., Princcton University
Press, 1980. (Publicado originalmente en J958 por Jobn Wiley & Sons.)
6. Rota. Gian-Carlo, editor, Studies in Combinatorics. Studics in Mathcmalics, Vol. 17, War
hington, D.C., Thc Mathcmatical Association oí Amcrica. 1978.
7. Tuckcr, Alan, Applied Combinatorics, 2• cd., Nueva York, Wiley, 1984.
8. Wilf, Hcrbcrt S., GeMraJing functionology, San Diego, Calif., Acadcrnic Prcss, 1990.
Ejefcicios complementarios 459

EJERCICIOS
S. Para los enteros n, k z O, sean
COMPLEMENTARIOS
• P1 el nwneto de particiones de n.
• P1 el nwnero de particiot'le$ de 2n + k, donde n + k
es el sumando más grande.
1. Encuentre la función generatriz. para cada una de las
• P, el ndmero de particiones de DI + k en precisa-
iucesiooes siguicnces.
mente n + k. sumandos.
1) 7,8,9,10,... b) 1,a,a2 ,a\a♦, • • • • aE R
e) l,(l+a),(l+a)',(l+a)', ... , ae R Usando el coooepto de grafo de Fcrrcr, demuestre que
d) 2,l+a,l+a\l+a\ ... , aE R P1 = P2 yP2 -=P~yconcluyaqueel mlmcrodedesa>mposi-
ciones de 2n + k en precisamente n + k sumandos es el
2. Encuentre el coeficiente de .r'3 en
mismo para todo k.
/(z)-= (r' + x• +x 11 + xu. + .r1.,.)'°. 9. Simplifü¡uelasumasiguicntc,paranEZ*:{~) +2{;)
+ 3 (;) + · · · + n(:). (Sugerencia: tal vez. deba comenzar
3. FJ sar¡:ento Pércz. debe distribuir 40 municiones (20 con el teorema del binomio.)
para fusiles y 20 para pistolas) enuc cuatro oficia.les de po-
licfa, de modo que cada oficia] reciba aJ menos dos. pero no 1 O. Determine la función generatriz para el nCmcro de dcs-
.más de siete municiones de cada típo, ¿De cutntas fonnas composiciones den EN,donde 1 aparece a lo~ una vez.
puede hacerlo? 2 aparece a lo sumo dos veces. 3 un máximo de tres veces y,
en ¡encral, k. aparece cuando mucho k veces. para cada k E
4. Encuencre una función generatriz para et ndmcro de z·.
íocmas de descomponer un entero positivo n en sumandos
cteros positivos, de modo que cada sumando aparezca un 11. En la tienda de un área rural hay doce buzones.
m1mcro impar de veces o ninguna. a) Si una repartidota tiene 20 periódicos idmtk:os,
¿de cuántas fonnas puodc:distribuirlos de modo que
5. Paran E z•, rooestrequeel ndmerodedescomposiciones en cada buz.ón haya a! menos uno?
de,11 en~ que ningún sunwd:> par se rc:pite(un sumando impar
b ) Si los buzones estin en dos filas con seis cada
pxde ooo rq,ctir,e) es igual al n6mcn>ded<sc:ornpo.<ici de una, ¿cuil es la probabilidad de que una distribu-
11 tales que ningún sumanck> aparece mú de tres veces.
ción de la parte (a) ten¡a 10 periódicos distribui-
6. ¿Cuintos nwncros telefónicos de lOd!gitos utilizan so- dos en los seis buzones de arriba y 10 en los seis
lamente los df¡itos I, 3, 5 y 7. si cada dígito aparece a1 de abajo?
menos dos veces o ninguna?
12. Sea S un conjunto con n. objetos distintos. Verifique
7. a ) ¿De quE sucesión de mlmeros esg(x) • (l -2x)4"'1 que e~/(1 - .r)• es la función generatriz exponencial para el
la función genecaaiz exponencial? s.
mlmero de formas de elegir 171de los objetos de para OS m :!':i n,
b) Encuenuc a y b taJes que() - axf sea Ja función y diStribuya estos objetos en k recipientes distintos, donde
generalril exponencial de la sucesión 1, 7, 7 · 11, el orden de los objetos en cualquier recipiente es significa-
7 · 11 · 15. .. tivo en la distribución.
10
Relaciones
de recurrencia

n algunas secciones anteriores del texto vimos algunas definiciones y construcciones rc-
E cur,ivas. En las definiciones 5.19,6.7, 6. 12 y7.9 obtuvimos los conceptos del niveln + 1
(o de tamañon + 1) a partir de conceptos similai:es en el niveln (o de tamañon), despufs
de definir el concepto en un primer valor de n, como O o l. Cuando trabajamos con los
números de Fjbonacci o de Lucas e n la sección 4.2, vimos que los resultados en el nivel
n + 1 dependían de los de los niveles n y n - l ; para cada una de estas sucesiones de enteros,
la base constaba de los dos primeros enteros (de la sucesiOn). Ahora nos encontramos en
una situación similar. Analizaremos las funcionesa(n), q ue de preferencia escribiremos a.
(paran~ 0), donde a. depende de algunos de los términos anteriorcsa. _1,a,._2, • • • ,a.. ao-
Este estudio de las llamadas reladonu de ruurrencia o ecuaciones en diferencias es la
contrapartida discreta de las ideas que se aplican en ecuaciones diferenciales ordinarias.
Nuestro desarrollo no usará ideas de las ecuaciones diferenciales pero comenzará con
la noción de progresión geométrica. A medida que vayamos desarrollando más ideas, ve.
remos muchas de las aplicaciones que hacen este tema tan importante.

10.1
La relación de recurrencia lineal
de primer orden
Una prog,uión geométrica es una sucesión infinita de números, como s.:15, 45, 135, .•. ,
donde el cociente de cualquier término (distinto del primero) entre su predecesor es una
constante, Üamada razón común. Para nuestra sucesión, esta razón común es 3: 15 = 3(5),
45 = 3(15), etcétera. Si ao, ai, a 2• • •• es una progresión ge.ométrica, entonces a¡/ao = az/a1 =
· · · =a•• 1/a,. = · · · = r , la razón común. En esta progresión geométrica particular, teoe-
mosque a•• 1 = 3a.. n 2: 0. ·
La relación de recurrencia a,.. 1 = 3a., n 2: O, no define una única progresión geométtica.
La sucesión 7, 21, 63, 189, ... tambifn satisface la relación. Para distinguir la sucesión
particular desctjta por a,. .. 1 ;; 3a., necesitamos conocer uno de los términos de la sucesión.
Por lo tanto,

naeO, ao=S,
461
462 Capitulo t O Relaciones de recurrencia

define en forma única la sucesión 5, 15. 45, ... , mientras que ·


n ~o. O¡ = 21,
identifica a 7, 21, 63, •.. , como la progresión geomttrica en cuestión.
La ecuación a.+1 = 3a., n ?: O es una relación de recurrencia, ya que el valor de a. +1
(considerando actual) depende de ª• (considerando anterior). Como a. +1 sólo depende de
su predecesor inmediato, decimos que la relación es de primer onlen. En panicular, ésta es
una relación de recurrencia homógenea, linea~ de primer orden. con coeficienzes constan-
tes. (Hablaremos más de esto posteriormente.) La forma general de esa ecuación esa.. 1 =
da., n ?: O, donde d es una constante.
Los valores como ao o ah que se dan además de la relación de recurrencia. se conocen
como condiciones de frontera. La expresión ao =A, donde A es una constante, también se
conoce como condición inicial . Nuestros ejemplos muestran la imponancia de la condi-
ción de frontera para determinar una única solución.
Regresemos a la relación de recurrencia
a,,+1 = 3a.,,, n ~o. a,= 5.
Los primeros cuatro términos de esta sucesión son
a. = 5,
a, = 3<>o = 3(5),
ª' = 3a, = 3(3"") = 32(5), y
ª' 2 3
= 3a, = 3(3 (5)) = 3 (5).
Estos resultados sugieren que para cualquier n ~O,ª• = 5(3"). Ésta es la solución general
de la relación de recurrencia dada En la solución general, el valor dea. es una función de
n y ya no depende de los términos anteriores de la sucesión, una vez definido ao, Por
ejemplo, para calcular a 10 , basta calcular 5(3 1°} = 295,245; ya no hay necesidad de comen-
zar en ao e ir agregando hasta llegar a a10•
El ejemplo anterior nos conduce a lo siguiente. (Este resultado puede demostrarse por
inducción matemática.)

Lasolucióogeaeraldela-de.-.
a,,. , =da.. doade. ~o.
.. lmica y eslá ciada pe,
a,,= Ad", 112:8.

Así, 1a solución a. =Ad•, n ~ Odefine una función discreta cuyo dominio es el conjunto
N de los enteros no negativos.

Resuelva la relación de recurrencia ª • =?a. _i , donde n ~ J y ai = 98.


Ésta es sólo una forma alternativa de la relación a,. .. 1 = 7a,.. paran~ Oy a2 = 98. Por lo
tanto, la solución general tiene la forma a,. = a<l,_7•). Como a 2 = 98 = aJ,,.7'2), esto implica
que ao = 2 y a,. = 2(7•). n ~ O es la única solución.
10.1 La relación de recurrencia lineal de primer orden 463

Un banco paga un interfs (anual) del 6% para cuentas de ahorros, con un inten!s compues-
to mensual. Si Patricia deposita $1000 el primero de mayo, ¿cuinto dinero tendrá deposi-
tado un al!o despub?
La tasa de intettS anual es del 6%, de modo que la tasa mensual es 6%/12 0.5% = =
0.005. Para OS n S 12. sea p. el valor del depósito de Patricia al final den meses. Entonces
p•• 1 s p. + 0.00Sp~ doode O.OOSp. es el intettS obtenido sobre p. durante el mes n + 1, para
OS n,; 11 y p0 = $1000.
La relaciónp•• , =(l.OOS)pe p. = $1000tiene la solución p. =p,(1.005)' = $1000(1.005)'.
Por lo tanto, al final del año, el depósito de Pauicia tiene un monto de $1000(1.005)11 =
$1061.68.

La relación de recurrencia a.. , - da. = O es /wal debido a que cada tmnino con
subíndice aparece elevado a la primera potencia (comox y y en la ecuación de una recta en
el plano). A veces, podemos transformar una relación de recuaencia no lineal en una
lineal mediante una sustitución algebraica

Determine a,2 si.a!.1 = SD;, dondeª•> Oparan :i?: Oy Do = ·2.


Aunque esta relación de rccwrencia no es lineal respecto de a.. si b. = O: , entonces la
=
nueva relación b•• 1 Sb. paran 2: O y b0 = 4 es una relación lineal cuya solución es b. =
4 • s•. Podo tanto, a.= 2(,ÍS)• paran~ O y an = 31,250.

La relación de rccurrencia lineal general de primer orden con coeficientes constantes


tiene la formaa•• 1 + ca. =J(n), n ~ O, donde e es una constante y /(n) es una función en
el conjunto N de los enteros no negativos.
Cuando /(n) = O para todo n E N, la relación es homoglnea; en caso contrario, la
relación es no homoginea. Hasta el momento hemos tratado solamente con relaciones
homogéneas. Ahora resolveremos una relación no homogénea. Desarrollaremos técnicas
cspcct'ficas que funcionan para todas las relaciones de recum,ocia liocales bomogtncas
con coeficientes constantes. Si bien hay muchas técnicas diferentes que son O:tiles en los
problemas no bomogtncos, ninguna de ellas sirve para todos los casos; pero si podemos
reconocer un patrón de nuesrras experiencias anteriores, es más probable que tengamos
bito.

==:.;1M=-".J Tal vez el más popular, aunque no el más eficiente, de los métodos para ordenar datos es
la t~cnica llamada ordenación por d mttodo de la burb11ja. En este caso, la entrada es
una lista A[!], A[2], . . . , A[n] den owneros reales que debe ordenane en forma ascen-
dente.
El segmento de programa en Pascal que aparece en la figura 10. 1 proporciona una
implementación para este tipo de algoritmo. En este caso, la variable entera i es el conta-
dor para el ciclo For exterior. mientras que la variable entera j es el contador para el ciclo
For interior. Por 111timo. la variable real temp se usa para guardar lo necesario al hacer un
intercambio.
Capitulo 1O Retaciones de recurrencia

Begin
For i : • l to n - 1 do
Fo r j : = n downto i + l do
If A(j] < A(j -1] then
Begin
temp : * A(j - 1] :
A(j -1] := A(j]:
A(j] : = temp
End
End:
Figur.a 10.1

Comparamos el último elemento de la lista dada, A(n], con su predecesor, A(n - l]. Si
A(n] < A[n - l], intemunbiamos los valores guardados en A[n- l] y A(n]. En todo caso,
ahora tendremos que A[n - 1] S A(n]. Después comparamos A[n - I] con su predecesor
inmediato,A(n - 2]. Si A(n- 1] < A[n -2], los intercambiamos y continuamos el proceso.
Después de n - 1 comparaciones de este tipo, el número más pequeño de la lista está en
A[l]. Después repetimos este proceso para Jos n - 1 números restantes guardados en la
lista (más pe<¡ue ña)A[2],A[3], ... ,A[n]. De esta manera. caila vez (contada por,) que se
realiza este proceso, el número más pequeño de la sublista restante «sube» (como una
burbuja) hasta el frente de esa sublista.
En la figura 10.2 aparece un pe<¡ueiio ejemplo e n quen = 5 y A(l ] = 7 ,A[2] = 9,A[3] =
2,A[4] = 5 yA[SJ = 8 para mostrar la forma en queel ordenamiento de burbuja de la figura
10.1 ordena en forma ascende nte una sucesión dada. En esta figura. cada comparación que
provoque un intercambio se d~nota mediante el símbolo J;
el símbolo } indica una compa•
ración que no produce intercambios.
Para determinar la función de complejidad en tiempo/(n) cuando usamos este algoritmo
en una entrada (lista) de tamaño n ~ 1, contamos el total de comparaciones fC-31izadas para
ordenar los n números dados en forma ascendente. L
Si a. denota el número de comparaciones necesarias para ordenar n números de esta
forma, entonces tenemos la siguiente relación de recurrencia:

a" = an-1 + (n -1), n ~ 2, a1= 0.

Esto surge de lo siguiente: dada una lista de n números. hacemos n - 1 compar3ciones


para subir el número más pequeño hasta el principio de la lista. La sublista restante den - l
números requiere entonces ª• - 1 comparaciones para ordenarse completamente.
Ésta es una relación lineal de primer orden con coeficientes constantes. pero el término
n - l la hace no homogénea. Puesto que no tenemos una técnica para resolver esta rela·
ción, enumeraremos algunos términos para ver si hay un patrón que podamos reconocer.

a1 = O
a, = a, + (2 - I) = 1
a3 =a, + (3 - ! ) = 1 +2
a, = a,+ (4- !) = 1 + 2 + 3

En general, a.= 1 + 2 + · · · + (n - 1) = [(n - l)n]/2 = (n2 - n)/2.


465

i= 1 A(1] 7 7 7 2
)i- 2
A(2] 9 9 9 2
)i= 3
A(3] 2 2 9 9
} =4
A(4)

A(S]
:l j=5
8 8 8
s
8

Cuatro comparaciones y dos intercambios.

i =2 A(1] 2 2 2

A(2] 7 s
)=3
A(3] 9 9 s 7
)i=4
A(4] 5 9 9
: i j=S
A(S] 8 8 8

Tres comparaciones y dos intercambios.

i=3 A(1] 2 2

A(2] 5

A(3] 7
: ) j =4
A(4] 9 8
)i =s
A(S] 8 9 9

Dos comparaciones y un intercambio.

i•4 A(1]

A(2] s
A(3] 7

A(4)

A(S)
:l j =s

Una comparación y ningún intercambio.


Figura 10.2
i66 capítulo 1O Relaciones de recurrencia

Como resultado, el algoribno de la burbuja detennina la función de complejidad ca


úempof: z• ➔ R dadapor/(n)=a, = (n'-n)/2. Eo coosecueocia, como medida del úem-
pode ejecuci6o de este algoribno, escribimos/ E O /n'). Eotooces decimos que el algoritmo
de la burbuja requiere 0/n') comparaciones.

Eo la parte (b) del ejemplo 9.6, buscábamos la función generatriz de la sucesión O, 2, 6, 12,
20, 30, 42, . .. y lasolucióo se basó en ouestracapacidadparareconocerquea. =n'+ npara
cada n E N. Si no vemos esto, tal vez podamos analizar la sucesión dada y determinar si
existe otro patrón que pueda ayudamos.
En este caso, a.= O, a,= 2, a,= 12, a.= 20, a,= 30, a,= 42 y
a1 - ao=2 a.-a,=8
ai-a, =4 a,-a. =IO
a3-a2=6 a.-a,= 12.
Estos cálculos sugieren la relación de recurrencia

ª• - a,.-1 = 2n, ao=0.


Para resolver esta relación. procederemos en forma diferente del método utilizado en el
ejemplo 10.4. Consideremos las siguientes n ecuaciones:
a,-a.=2
a2-a1 =4
a, - a,=6

a,.-"1.- 1= 2n.
Al sumar estas ecuaciones, la suma del lado izquierdo contiene a; y - a; para todo 1 S i S
n - 1. Así, obtenemos
a,-ao=2+4+6+ · · · +2n=2(1+2+3+ · - - +n)
= 2[n(n + 1)/ 2] = n' + n.
Como ac, = O, tenemos que a. = n2 + n para n E N, como lo habíamos determinado en la
parte (b) del ejemplo 9.6.

En este momento analizaremos una relación de recurrencia con un coeficiente variable.

Resolveremos la relación a. = n • a._ 1, donde n ~ 1 y ao = 1.


Si escribimos los primeros cinco ténninos de la relación, tenemos:

ao= 1 a,=3-a,=3 -2-1


a 1 =1·ao=l a,=4 -a,=4 -3-2·1
a2=2•a1= 2·1
1O.1 La relaóón de recurrencia lineal de primer orden 467

Por lo tanto. a,. = n! y la solución es la función discreta a .. n ~ O. que cuenta las


permutaciones de n objetos.

Ahora que estamos en el tema de las permutaciones, analizaremos un algoritmo recursivo


para generar las permutaciones de ( 1, 2, 3, .• • , n - 1, n} a panir de las permutaciones de
{l,2,3,. .. ,n - 1}.t
Sólo existe una pennutación de{!}. Si analizamos las permutaciones de ( 1, 2}.

2
2

vemos que despuhi de escribir la permutación 1dos veces, pasamos el 2 de un lado al otro
del 1 para obtener las permutaciones enumeradas. Si escribimos cada una de estas dos
permutaciones tres veces, intercalamos el número 3 y obtenemos
2
2
3 1 2
) 3 2
2 3
2

Vemos aqul que la primera permutación es 123 y que obtenemos cada una de las si-
guientes dos permutaciones a partir de su predecesor inmedjato, intercambiando dos mt-
meros: 3 y el entero a su izquierda. Cuando e) 3 llega al lado izquierdo de la permutación,
examinamos los números restantes y los permutamos de acuerdo con la lista de
permutaciones generadas para ( 1, 2}. (Esto hace que el procedimiento sea recursivo.)
De~s de esto, intercambiamos el 3 con el entero a su derecha. hasta que el 3 está en el
lado derecho de la permutación. Observemos que si intercambiamos 1 y 2 en la última
permutación. obtenemos 123, la primera permutación enumerada.
Para el caso S = (1, 2, 3, 4}, enumeramos primero las seis permutaciones de ( 1, 2, ~}
cuatro veces. Si comenzamos con la permutación 1234, intercalamos el 4 en las restantes
23 permutaciones, como se indica en la tabla 10.1. La lloica idea nueva aqul es la siguien-
te: al pasar de la permutación (5) a la (6) a la (7) a la (8), interumbiamos 4 con el entero
a su derecha. En la permutación (8), cuando 4 ha alcanzado el lado derecho, obtenemos la
pennutación (9) manteniendo fija la posición de 4 y reemplazando la permutación 132 por
3 12 de la lis1a de permutaciones de ( 1, 2, 3}. Despu6s de esto, continuamos con las prime-
ras ocho permutaciones, hasta llegar a la permutación (16), donde 4 está de nuevo a la
derecha. Oespu~s permu1amos 321 para obtener 231 y continuamos intercalando 4 hasta
generar las 24 permutaciones. De nuevo, si intercambiamos l y 2 en la última pennuta.ción,
ob<enemos la primera permutación de nuestra lista.
La bibliografía del capítulo proporciona más informacíón acerca de los procedimientos
J'CCW'Sivos para la generación de permutaciones y combinaciones.

t El mMelial qoe se ptCSCllta de aqaf al fu&aJ de la scc:cióo es aa dig:rcsi6a que asa la idea de rccursi6a.
No trata los ~todos para resolver rebciooes de rec\l.JTCDCia y pllCde omitirse sin perder la continuidad.
68 (apítulo 1O Relaóooes de recurrencia

Toblo 10.1
(!) 1 2 3 4
(2) 1 2 4 3
(3) 1 4 2 3
(4) 4 1 2 3
(5) 4 1 3 2
(6) 1 4 3 2
(7) 1 3 4 2
(8) 1 3 2 4
(9) 3 1 2 4
(10) 3 1 4 2
(ll) 3 4 1 2
... ... ... ...
(IS) 3 2 4 1
(16) 3 2 1 4
(17) 2 3 1 4
... ... ... .. .
(22) 2 4 1 3
(23) 2 1 4 3
(24) 2 1 3 4

Cerraremos esta primera sección regresando a una idea anterior: el máximo com11n
divisor de dos enteros positivos.

===;..ª'
..--.",..,"--~.., Los mitodos recursivos son fundamcnta1es en el área de las matemáticas discretas y en el
anilisis de aJgoriunos. Estos OK:todos surgen cuando queremos resolver un problema dado
al descomponerlo o referirlo a problemas similares más pequeftos. En el lenguaje de pro-
¡ram,icióo Pascal podemos implementarlos mediante el uso de funciones y procedimien-
tos recursivos, que permiten llamarse a sí mismos. Este ejemplo proporciona una función
de este tipo.
Al calcular mcd(333, 84), obteocmos lo siguiente mediante el algoriano de Euclides
(presentado en la sección 4.4).

333 • 3(84) + 81 0<81 <84 (! )


84 • 1(81) + 3 0<3<81 (2)
81 • 27(3) + o. (3)
Como 3 es el tlltimo resto distinto de cero, el algoriano de Euclides indica que mcd
(333, 84) = 3. Sin embargo, si solamente usamos los cálculos de las ecuaciones (2) y (3),
=
entonces tenemos que mcd(84, 81) 3. La sola ecuación (3) implica que mcd(81, 3) 3, =
puesto que 3 divide a 81. Por lo tanto,

mcd(333, 84) = mcd(84, 81) =mcd(81, 3) =3,


donde los enteros que aparecen en los cálculos sucesivos son cada vez más pequeños,
conforme pasamos de la ecuación (1) a la (2) a la (3).
1O. 1 La relación de recurrencia lineal de primer orden 469

Tambii n observamos que


81=333Mod84 y que3 = 84 Mod81.
Por lo tanto, obtenemos que
mcd(333, 84) = mcd(84, 333 Mod 84) = mcd(333 Mod 84, 84 Mod(333 Mod 84)).
Estos resultados proporcionan el siguiente método recursivo para el cálculo de mcd(a,
b), con a, b E z·.
Supongamos que la entrada está dada por a, b E z·.
Paso l . Si b Ia (o a M od b = 0), entonces mcd(a, b) = b.
Paso 2. Si b .Ya. entonces hay que realizar las siguientes tareas en el orden dado.
i) Seaa=b.
ü) Hagamos b = a Mod b, donde el valor de a para esta asigoac.ión es el valor
anterior de a.
ül) Regresamos al paso l.

Utilizamos estas ideas en el programa eo Pascal de la figura l0.3, donde el segmento


sombreado es una fanci6n recursiva que determina mcd(a.b). (Tal vez el lector quiera
comparar este programa con el que apa,ece en la figura 4.9.)

Program EuclideanAlgori thm2 {input, outpu t);

Var
p,q: integer;

Begin
Writeln ("Este proqra::i.a •stá dis♦~c!o p.ara
encontrar ·};
Writeln ('el mcd d e dos enteros positivos') ;
Writeln ('pyq.');
~"rite ( 'El priJNr e ntero positivo p es ·);
Readln (p);
Writeln ( 'El segundo ent•ro positivo q H • >;
Rieadln (q);

Writeln t 'El ='xi.0o común divisor de ·);


h"riteln (p.· y •.q.• H ' , mcd (p,q),'.' )
End.
Figura 10.3
470 Capítulo 10 Refaciones de recurrencia

EJERCICIOS 1 O.1 1. Encuentre una relación de recurrencia, con una condición i.nJcial. que determine de manera
única cada una de las siguientes progresiones gco~cas.
a) 2, 10, 50,250,... b) 6, -IS, 54, -162, ..
e) !, 1/3, 1/9, 1/2:1,.. . d) 7, 14/ 5, '28/15,56/125, ...
2. Encuentre la solución general para cada wta de las siguientes progresiones geométricas.
a) a,..1-l.Sa,.=O,n2:0 b) 4a,.-Sa.,_1.:0,n.?!.l
e) 3a,..1 -4a.. =O,n2:0,a1""'5 d) 2a,.-3a,._1 =0,nc:l,(4=81
3. Si a., n ~ O, es una solución de la relación de rccunencia ª• • 1 - da.: O y a,= 1S3149. ~ =
1377/2401, ¿cuánto valed?
4. El número de bacterias en un cultivo es de 1000 (aproximadamente)y estent1mero aumenta un
250% cada dos horas. Use una relación de rccurrencia para detenninar el ndmero de bacterias
presentes después de un día.
S. Si Laura invierte $100 con un interés compuesto trimestraJ de 6%. ¿cuántos meses debe espe-
rar para que su dinero se duplique? (Ella no puede retirar el dinero antes que se cumpla e!
trimestre.)
6. Hace 15 años Pablo invirtió sus ganancias de la bolsa, en una cuenta que pagaba U1l 8% de
inter6s trimestral compuesto. Si ahora tiene $7218.27 en su cuenta. ¿cuál fue la inversión
inicial?
7. Seaxi. x21 ••• , .:cw una lista de números reales distintos que debe ordenarse mediante la tttnica
del Jm.todo de la burbuja del ejemplo 10.4. (a) ¿Desph: de cuántas comparaciones estarán
ordenados en forma ascendente los 10 números más pcquciios de esta lista? (b) ¿Cuántas com-
paraciones más se necesitan para terminar ta ordenación?
8 . Para la implementación del método de la burbuja de la figura 10.1, el ciclo For e~terior se
ejecuta n - 1 veces. Esto sucede en fonna independiente de los intercambios que ocurran
durante la ejecución del ciclo For interior. F.n consecuencia. para i = k, 1 S k s; n - 2, si ta
ejecución del ciclo For interior no produce intercambios. la lista estará ya en orden ascendente.
por lo que la ejecución del ciclo For exterior para k + 1 S i S n - 1 ya no es necesaria
a) Parala situación aquí descrita. ¿cuántas comparaciones inncccsarias se realizan si la ejecu-
ción del ciclo For interior para i = k (l S k. S: n -2) oo produce intercambios?
b) Escriba una versión mejorada del ~todo de la bwbujaque aparece enla figura 10.1. (Su
resultado debe eliminar las comparaciones innecesarias analizadas al inicio de este ejerci•
cio.)
e) Use el número de comparaciones como una medida del tiempo de ejecución para determj.
nar la complejidad en tiempo del mejor y el peor caso para el algoritmo implementado en la
parte (b). .
9 . Supongamos que las pennutaciones de { l. 2. 3, 4, 5} se generan mediante el procedimiento
desarrollado después del ejemplo 10.6. (a) ¿CUál es ta 111tima permutación de la lista? (b)
¿Cuáles son las dos permutaciones que preceden a 2S134? (e) ¿CUáles son las tres permutaciones
que siguen a 25134? (d) ¿Qué posición ocupan las permutaciones 2 1543, 35421 y 31524?
10. Seanx1,.t2,XJ. ... ,x,,n nt1mcrosrealcs,conx1 Sx2 Sx,S · · · sx.. Lanudianadce.steconjunto
de n números se define como
xt-. 1)1'2, para n impar
(l/2)[x.n:+~.J.para n par.
a) Supongamos queA{l],A[2J.A(3), ... ,A[nJ es una lista den nt1meros reales,n.oneccsaria•
mente en orden ascendente o descendente. Escriba un programa (o desarrolle un algoritmo)
para determinar la mediana del conjunto de mlmeros reales enumerados en la lista.
b) Analice la complejidad en tiempo del peor caso para el programa escrito en la parte (a).
10.2 La relación de reamenáa lineal homogéneo de segundo orden con..• 471

10.2
t.. relación de recurrencia lineal
homogénea de segundo orden
con coeficientes constantes

Seak E Z• y C,(:F O),C,.,.C••,, . .. ,C,.-.( J O) números reales. Sia., n ~O.es una función
discreta, entonces
C.a. + C,.1 0,.1 + C,., a,_, + · · · + C..,a,.-,. =/(n), n _. k,
es una ,elación de ,ccurrencia lineal (con coeficientes constantes) de orden k. Cuando/(n)
= Opara todo n ~ O, decimos que la relación es Jwrnoginta; en otro caso, es no homogénea.
EnCSla seccióo ooscemraremoseo la,clación bomogffleade orden dos: e.a.+ c•. ,a..,
+
c•. ,a..,= o.n~2. Coo baseeo nuestrotnbajode la sección 10.1, buscamos uoasolucióa
de la fonnaa,.= cr", doodc e,/, Oy r,/, O.
Si sustituimosª·= cr" CD CA+ c._1ª-- 1+ c. -2'2.-2: • O, obtenemos
C.cr•+ C.-1 0--- 1 + C.-2 cr•- 2 = 0.
Si e, r# O,esto se convierte en c,r + C._ 1r+ C•. 2 = O, una ecuación cuadrática llamada la
ecuación característica. Las raíces ra. r2 de esta ecuacióo-están en al¡uno de los tres casos
siguientes: (a) r,. r, soo oúmeros reales distintos; (b) r,. r, soo complejos conju¡ados; (c)
ri. r1 son reales, pero r1 =r2 . En todos los casos, r1 y r1 son las ralees caracterlsticas.

Cuo(A): (Rakes rHles distintas)

10,& =O, doodc n ~2 yao= 1, a,= 2.


Resolveremos larelacióa derecurrenciaa. +a,.., -6a,. _,
===......:.....:.'1 Sia.=c,..conc, r+. 0,obtencmoscr4 +c~- 1 - 6cr•- 2 =0dedondese sigue la ecuación
característica r1 + r- 6 == O:
O• r'+ r-6E (r + 3)(r -2)~r =2, -3.
Puesto que teoemos dos raíces reales distintas, a.= 2• y a,,= (- 3)- son soluciones [al
igual que b(2") y d(-3)" con b,d constantes arbitrarias]. Son soluci<>Ms liMalm,nt< úuk-
pendkntes pues una no es múltiplo de la otra; es decir. no existe una constante real k tal
que (-3)" = 1(2') para todon E N.t Escribimos a,. = c,(2") +c,<-3)"como solucióo gene-
ral, donde Ci, c1 son constantes arbitrari.u.
Si ao = 1 y a1 = 2. entonces c1 y c2 quedan determinadas de la forma siguiente:

1 =ao =c1(2)º+c,(-3)º = e, +e,


2 = a, - c1(2)1 + c,(-3)1 = 2c1 - 3c1 •

Al resolver este sistema de ecuaciones, vemos quec1 = l, c1 =O.Por lo tanto, a..,= 2•, n 2:
O es la únka solucióo de la relación de recurreocia dada.

t Taniri6:I podemos decir cp: las sohtcioaes a. • 2• y a,, = (-3)" sa:,, liMOUM• ~ s i s e
cumple lasiguieare condición: Pa'al¡, .ti E R.sit1(2")+.ti(-3)" • 0i-al0don e N,cuonca: t. =.G •O.
472 Capitulo 1O Relaciones de recurrencia

Una relación de recurrencia homogénea de segundo orden interesante es la relación de


Fibonacci. (Ya mencionamos esto anteriormente, en las secciones 4.2. 5.8 y 9.6.)

Resolveremos la relación de rccurrencia F,. ... 2 :;;:. F,..1 + F._. donde n ~ Oy F0 = O. F1 = l.


Comoenel ejemploanterior. seaF• .;cr\ parac, rf:-0, n~O. Al sustituir obtenemos
cr•• 2 = cr"• 1 + cr";dc aquí tenemos la ecuación característica r2- r- l = O. Lasrafoes
características son r= ( I % ../sy'i, de modo que la solución general es F. =<,((l + ../s)/2]"
+ c2(L-../s)l'2]•.
Para encontrare.. c2, usamos los valores iniciales dados y escribimos O= F,. = c1 + c21
1 = F, =c,((l +../s)/2] +c2[(1 - ../s)l'2]. Como-<, =e,, tenemos que 2 = c,(1 + ..fs) -
c,(1- ..fs) y e, = 11 ../s. La solución general está dada por

Paran;eo, seaS= {I, 2, 3, ... , n} (cuandon=O,S = S)y seaa,,el número de subconjuntos


deS que no contienen enteros consecutivos. Encuentre y resuelva una relación de recurrencia
para a•.
Para OS:n S:4, tenemosezo= l ,a1 • 2,az = 3,a¡= 5 y a.. =8. [Porejemplo,a,= 5 ya que
S= {l. 2, 3} tieoc a e, (1 }, (2}, {3} y { 1, 3} como subconjuntos sin enteros coosecutivos
(y ningún otro subconjunto). Estos primeros cinco términos nos recuerdan la sucesión de
Fibonacci. Pero ¿cambiarán las cosas a medida que avancemos?
Sean ;,2y S= {I, 2, 3, ... , n-2, n-1, n}. Si 11 ~ Sy II se cuenta en a,,, hay dos
posibilidades:
a) n EII: Cuando esto ocurre, (n- 1) 'le. 11 y 11- (n} se contaría en a,,_,.
b) n '/! A: En este caso, A se contaña en a •• 1•
Estos casos abarcan todas las posibilidades y son mutuamente disjuntos, por lo que
podemos concluir que la relación de rccum:ncia de este problema es a,. = ª•- i + a.- 2:, donde
n 2::2 yao= l,a1 =2. Ahora podríamos despejara., perosiobservamosquea,. =F. +2, n 2:0,
entonces el resultado del ejemplo 10.9 implica que

Analic.emos una relación análoga en una aplicación para las ciencias de la computación.

En muchos lenguajes de programación podemos considerar que las expresiones aritméti•


cas válidas, sin parlnusis, estrut formadas por los dígitos O, 1, 2, ... , 9 y los símbolos de
las operaciones binarias+, • ,/.Por ejemplo, 3 + 4 y 2 + 3 • 5 son expresiones aritm&icas
válidas; 8 + • 9 no lo es. En e ste caso, 2 + 3 • 5 = 17, ya que hay una jerarquía de
10.2 La relación de recurrencia lineal homogénea de segundo orden con... 473

operaciones: la multipl icación y la división se realizan antes que la suma. Las operaciones
del mismo nivel se realizan en su orden de aparición, al analizar la expresión de izquierda
a derecha.
Paran E Z♦• sea a. el número de estas expresiones aritméticas (válidas) formadas por n
símbolos. Entoncesa1 = 10, puesto que las expresiones aritméticas con un símbolo son los
10 dígitos. A continuación, a, = 100, con las expresiones 00, 01, ... , 09, 10, 11, . .. , 99.
(No hay necesidad de un signo más antes del número.) Si n ~ 3, consideramos dos casos
para obtener una relación de recurrencia para a.:
1) Si x es una expresión aritmética de n - 1 símbolos, el último de éstos debe ser un
dígito. Si agregamos otro dígito a la der echa de x, obtenemos lOa. _ 1 expresiones
aritméticas de n símbolos, donde los últimos dos símbolos son dígitos.
2) Sea y una expresión de n - 2 símbolos. Para obtener una expresión con n símbolos
(que no se contaron en el caso 1), agregamos a la derecha de y una de las 29 expre-
siones con dos símboJos + 1, .. . , + 9, + O, *1, ...• *9, *O, /1, . . . , /9.
Deestosdoscasosobtenemosa. = 1~_1 +29a._2,dondcn :2: 3 ya1 = 10,az = 100.
En este caso, las ralees características son 5 :t 3../6 y la solución esa,= (5/(3../6))
[(5 + 3../6)'- (5 -3../6)'] paran?; l. (Verifique este resultado.)
Otra fonna (tal vez más sencilla) de obtener la solución de este problema es la de
usar la relación de recurrencia a,. = IOa._ 1 + 29ti._ 2, con a 2 = 100 y a 1 = 10, para
calcular un valor dea,, (a saber, a. = (a,- IOa,)/29 = O). La solución para la relación
de recurrencia

a. = l Oa11-1 + 29a,.-2, n ~ 2, ao = O, ª1 = 10
es
ª· = (5/(3V6))[(5 + 3v'6r- (5-3v'6rJ. n 2:0.

Ahora pasaremos a una aplicación e n las ciencias físicas.

Consideremos la red line.al de la figura 10.4, donde hay k resistencias de un ohmio y k


resistencias de tres ohmios conectadas por medio de cables a un ge nerador que proporcic:r
na un voltaje constante V. Encuentre una fórmula para u., O S: n ~ k. que dé el voltaje e n
cada punto de unión como función de n. (Éste es ·igual a la diferencia de potencial a través
de la resistencia de un ohmio por debajo de ese punto de unión.)

( l~l
Figura 10.4
_v.._,_ , _,_~ ___,_ ,,_ _.v•_• ID•
74 Capitulo I O Relaciones de recurrencia

Sabemos que "• = V, el voltaje proporcionado por el generador. Para encontrar una
relación de rccwreocia., aplicamos los siguientes principios.
1) Ley de Kirr:hhoff: En cualquia punto de unión, lasuma de las intensidades que fluya,
en el punto de unión es igual a la suma de las intensidades que saleo de ese punto.
2) Ley de Ohm: Si la düeroncia de potencial a travts de una rosistencia de R ohmios es
V2 - V1, entonces la intensidad en esa resistencia es (V2 - VJIR. ·
Consideremos la intensidad en el punto de unión etiquetado como v•• 1• Por la ley de
Kirchhoff. l • 11 + /2 • Pcx- la ley de Ohm. /::: (v•• 2 - v•• 1Y3. /1 = v • • 1 e 12 = v. .. 2- E.n
consecuencia.

-u,..2-v••1 "••1 v,..1-v.


- ---+ - -- Osn s k - 2,
3 1 3 '
o u•• 2-Su..:+1 +u,. = O.
Seav. ; e,.., eac, r'i= Oyo S n S: t. La ecuación característica es r - Sr+ 1 = COD
rafees a= (5 + ,/21 Yl, b = (5 - .ffiYl. De esto obtenemos que v, = c,d' + c,b'.
º·
Independientemente del valor de Vo. portas dos leyes tenemos (v 1 - v 0)13 = v 0/l, o v 1 =
4vo- Demodoquev, =c1 +c2 y 4v0 =v1 = cia+c,,J,. Deestosesiguequec1 =vtl.b- 4)/(b -a).
y e,=v,J.a -4'¡/(a-b), y
(b-4) (a -4)b]
u,. = Uo[- - d ' + - - • .
(b -a) (a-b)
Como
(b - 4)a" - (a - 4)b']
V = v.,•~[ (b -a) ,

tenemos que v 0 = (b- a)V/((b - 4)a' - (a - 4)b'J, de donde obtenemos

(b -4)a"- (a -4)b"]
u. • V [ ~(b- --4~)a"
..,.....--(~a---4~)-,b' .

Plantearemos la relación de recurrencia del siguiente ejemplo en dos Connas. En la


primera parte veremos la utilidad de las variables auxiliares.

. . . . - 10.U Encucotre u.na relación de rccurrcacia para el o11rncro de sucesiones binarias de longitud n
que no tienen ceros consecutivos.
a) Paran~ l, scaa,.cl nlllnero de tales succsiooes de longitudn. Sean al0> la cantidad
1
de ellas que terminan en O y d,, ' la cantidad de ellas que terminan en 1. Entonces.
a.= a!_º)+ o!''.
Comenzamos por obtener una relación de recurrencia para a.. n '2: l , calculando
a 1 ::: 2 y teniendo en cuenta cualquier sucesiónxde Joogitudn-1( >0), tal que.roo
contiene ceros consecutivos. Si x termina en 1, podemos afiad.irle un Oo un t, lo que
nos da 2e de las sucesiones contadas porª·· Si la sucesión x termina enº· enton-
10.2 la relación de reamencia lineal homogénea de segundo orden con... 475

ces solamente podemos añadir l. lo que produce a!{ sucesiones contadas por a..
Puesto que estos dos casos abarcan todas las posibilidades y no tienen nada en
común. tenemos

a.= 2·ai~1+ 1-a!°J,


i
La /t-t$il'l'la poski6n "'
La n-bi!N posici6n
puede ser O o 1. solamente puede ser 1

Si consideramos cualquier sucesión y contada en ª•-l• tenemos que la sucesión


yl se cuenta coa~ . Dela misma forma. si lasucesiónzl se cuentacna!~1.entonccs
z se cuenta en a,._ 2• En consecuencia. a,._ 1 : ~ y

Por lo tanto. la relación de recurrencia de este problema es a. = ª• _1 + a,._ 2 , donde


n 2: 3 y a, = 2, a,= 3. (Dejamos los detalles de la solución al lector.)
b) Otra alternativa es )a siguiente: sin~ 1 ya,.cuentael nt1merodesucesiones binarias
sin ceros consecutivos, entonces a, = 2 y ai = 3 y para cada n ~ 3 consideramos las
sucesiones binarias contadas por a,.. Estas sucesiones tienen dos posibilidades:
(Caso 1: El n-tsimo sfmbolo es 1) Aquí vemos que los n - 1 símbolos anteriores
forman una sucesión binaria sin ceros con.secutivos. Existen a •• 1 de esas sucesiones.
(Caso 2: El n-ésimo símbolo es O) En este caso, cada sucesión termina realmente en
1O y los primeros n - 2 símbolos forman una sucesión binaria sin ceros consecuti•
vos. En este caso, existen a._ 2 de esas sucesiones.
Puesto que estos dos casos abarcan todas las posibilidades y no tienen sucesiones
en común, podemos escribir

a,. = a..- 1 + ª•-2, 01 = 2, a2 = 3, n~3.

como vimos en la pan.e (a).


Tanto en la parte (a)como en la parte (b) podemos usar la relación de rocummcia y a,=2,
az= 3 para regresar y determinar un vaJorparaao; a saber,ao=az-a1 = 3 - 2; l. Entonces
podemos resolver la relación de recurrencia

a,. =ª•-1+ ª•-h ac, = 1, a1 = 2, n~2.

Antes ele continuar queremos aseguramos de que el lector comprende la razón por la
que necesitamos un argumento general cuando desarrollamos nuestras relaciones de
recum:ncia. Cuando estamos demostrando un teorema no extraemos conclusiones ge_nera•
les de unos cuantos (o incluso muchos) casos paniculares. Aquí ocurre lo mismo. El si•
guientc ejemplo debe servir como guía.

....... 10.14 Comenzamos con n monedas de l centavo cada una. idénticas. y sea a. el nl1tnero de
formas en que podemos ordenar estas monedas, de manera contigua en cada fila, de modo
que cada moneda que quede sobre la fila inferior toque dos monedas de dicha fi.Ja. (En esta
476 capitulo 1O Relaciones de recurrencia

@
(n- 1)
~
(n = 3)
& ~

~ ~ ~
®1)

(n - 2)

~~~~
~
(n = 4)
• d@
(n - 5)
a

(n - 6)
&.... ~ ~ ~
Figura 10.S

disposición no impona si la moneda está hacia arriba o hacia abajo.) En la figura 10.5
tenemos las posibles disposiciones para 1 S n S 6. De esto se sigue que

a1 = 1, a. =3, a5 = 5, a, = 8.

En consecuencia, estos resultados podrían sugerir que. en general, a.= F., el n~ simo
número de Fibonacci. Por desgracia, esto es incorrecto, ya que vemos, por ejemplo, que
a, = 12,; 13 =F1 , a,= ¡g,;2¡ =·F,, a,= 25,; 34 = F,.
(Las disposiciones de este ejemplo fueron analizadas por F. C . Auluck en la obra referida
[2].)

ij-~ El último ejemplo del caso (A) muestra cómo extender los resultados de las relaciones
de recurrcncia de segundo orden a órdenes superiores.

Resuelva la relación de recurrencia

2a11•3 =a,.., +2a....1 - ª"' n 2=: O, Do= O, a, = 1, a2= 2.


Sea a.= cr-, para c. r t, O y n ~ O; obtenemos la ecuación característica 2r 3 - r 2 - 2r +
1 = O = (2r - 1Xr - l)(r + 1). Las raíces características son 1/2, 1 y - 1, de modo que la
10.2 La re!ación de recurrencia lineol homogéneo de segundo Otden con... 477

solución esa,, = c1(1)' + c,(-1)' + c,(1/2)' = e, + c,(-1)' +c,(lf2)'. (Las soluciones 1, (-1)'
y (1/2)' son linealmente independientes pues es imposible expruar cualquiera de ellas
como una combinación lineal de las otras dos.t De O:: ao, 1 =a 1 y 2 =~. obtenemos que
C¡ = 5/2, Cz = 1/6 y C¡ = -8/3.

Cuo (B): (Ralees complejas)

Antes de pasar al coso de los ralees complejos, recordemos el teorema de DeMoivre: (cos
8 + i sen8)• = cos ne+ i sen ne, n;,; O.(Ésta es la parte (b) del ejercicio 8 de la sección 4.1.)
Si z=.<+ iy E C.ziffi, podemos escribir z= r(cos 8 + isen 9), donde r= ~x' +y' y
(y/x) = tan 9, para x ;/ O. Si x = O entonces para y > O, z = yi = yi sen(x/2) = }1cos(Jt/2)
= =
+ i sen(x/2)) ypara y < O,z =yi jy Ii sen(3"'2) IY l(cos(3lt/2) +i sen(3lt/2). En todos
los casos, z" = r'(cos ne + i sen ne ), paran ;,; O, por el teocema de DeMoivre.

.;.1,j Determine (!+ ./3,)" .


La figura 10.6 muestra una forma geométrica de r e p r e s e n ~ complejo 1 +
./)¡ como el punto ( 1, ./3) en el planoxy. En este coso, r= .Ji' + (./3)' y 8 = "'3.
y

(1,V3)

Figura 10.6

As[ 1 + V3i • 2(005(1'/ 3) + isen('ff/ 3)), y (1 + V3i)'º= 210(cos(!0.,,/ 3) +


isen(I0,,/3)) = 21º (cos(41r/3) + isen(41r/ 3))- 210 ((-1/2)- (:v'3/2)i) =(- 2')(1 + V31).

Usaremos estos resultados en los siguientes ejemplos.

E:!===~1..:;¡;1¡:':1.3 Resolveremos la relación de recurrenciaa, = 2(a,._1 - a._,), donde n;, 2 y a. = 1, a, = 2.


0.
Si a.= e~. e, r#, Oy obtenemos la ecuación caractcñsticar2 - 2r+ 2= O, cuyas rafees
son 1 ± i. En consecuencia, la solución general tiene la forma c1(1 + ff + c2(1 - ,--,., donde

t Otra forma de decir e5lO es~ las soJoc::iooes l. (-1)" y (tny soa liocalmentc indepaidic:mcs poes si
.ti, .t1, .t, S0D 1111.mcros reales y k1\l) + k,(-1)" + k,(ln)" para toda II EN, ClltODCCS A:1 • *2 =A:, • o.
478 Capítulo 1O Relaciones de recurren.da

c 1 y c2 son constantes complejas arbitrarias. {Como en el caso (A), hay dos soluciones
linealmente indepcndicnu:s: (1 + i)' y(l - ,)'.]

1 + i = V2(cos(,r/4) + i sen(,r/4))
y
1 -i = V2(cos(-,r/ 4) + isen(- ,r/4)) = V2(cos(,r/4)- isen(,r/4)).
De aquí obtenemos

a. = c 1(V2)"(cos(n.,./ 4) + i sen(n-,,/ 4)) + c,(V2)"(cos(n-r./ 4)-i sen(n,r/4))


= (V2)'(k1 cos(n,r/4) + k2 sen(n,r/4)],

donde k1 ;: c1 + c2 y k,:; (c1 - c;Ji.

1 = ao= [k, cos0+k2 sen0] =k,


2=a1 =V2(l •cos(,r/4)+k2 sen(ir/4)],o 2=1+k2, y k2 =1.
La solución para las condiciones originales dadas esa.= (,Í2)' · [cos(n,r/4) +sen(n,r/4)],
n ~ O. [Nora: Esta solución no tiene números complejos. Tal vez esto confunda un poco al
lector. ¿Cómo partimos de ci, c 2 complejos y terminamos Con k1 ~ c 1 + '1 y k-: = (c1 - cJi
reales? Esto ocUIJe si c., c2 son complejos conjugados.

Analizaremos ahora una aplicación del álgebra lineal.

Para b E R+, consideremos el detemtlnantet D,. den -X n dado por

b b o o o o o o o o o
b b b o o o o o o o o
o b b b o o o o o o o
o o b b b o o o o o o
o o o b b b o o o o o
o o o o o o b b b o o
o o o o o o o b b b o
o o o o o o o o b b b
o o o o o o o o o b b

Encuentre el valor de D,. como función den.


Sea ª•• n ~ 1, el valor del determinante D. den X n. Entonces

a,= lbl =b y a,= 1: :1 =O.

t El dcsam>Uo de los detcnn.iD&lllcs se analiza en el Apfodice 2.


10.2 La relación de recurrencia lineal homogénea de segundo orden con... 479

Si desarrolliunos D, por la primera fila. tenemos que D.=

b b o o o o o o o
b b b o o o o o o
o b b b o o o o o
b o o b b b o o o o
o o o o o b b b o
o o o o o o b b b
o o o o o o o b b
(tste esO., . ,.)
b b o o o o o o o
o b b o o o o o o
o b b b o o o o o
-b o o b b b o o o o
o o o o o b b b o
o o o o o o b b b
o o o o o o o b b

Cuando desarrollamos el segundo detenninante por su primera columna, vemos que D. =


bD. _1 -(bXb)D._1 = bD._ 1 - b2D,._2• Esto se traduce en la relación a. = ba. _1 -lra. _2 para
n~ 3, a, =b, a, = O.
Sia, =cr",c, r ,j,. Oy n ~ 1, la ecuación caracterfstica prnduce lasralcesb{(l/2) :t i,ÍJ/2].
Por lo tan.to,
a,= c 1 [b((l/ 2) + iV3/ 2)r + c,[b((l/ 2) -iV3/ 2)]•
= b"(c1 (cos(,r/ 3) + isen(,r/ 3))' + e, (cos(,r/ 3) - isen(,r/ 3))']
= b"(k, cos(n,r/ 3) + k,sen(n,r/ 3)].
b = a, = b[k, cos(,r/ 3) + k, sen(,r/ 3)]. y 1 = k 1(1/2) + k,(V'.l/ 2), o k, + V3 k, = 2.
O= a, = b2(k 1 cos(21r/ 3) + k2 sen(21r/3)], y O= (k,)(-1/ 2) + k2(V3/2), o
k 1 = V3k2 •
Por lo tanto, k, =1, k, =1/,ÍJ y el valor de D, es
b' [cos(n,r/ 3) + (l/V3)sen(n,r/ 3)].

Pasemos al problema de las ralees múltiples.

caso (C): (Raíces reales repetidas)

Resuelva la relación de recurrenciaa,. .. 1 = 4a,.. 1 -4a.,. donde n ~ O y ao = 1, a 1 =3.


Como en los otros dos casos, sea a,. = cr•, e, r-/; O y n ~ O. Entonces la ecuación
característica es r 2 - 4r + 4 = Oy las raíces características son ambas r = 2. (Así, r = 2 es
480 Capitulo 1O Relaciones de recurrenda

una "rafz de multiplicidad 2".) Por desgracia. no tenemos dos soluciones linealmente in-
dependientes:? y 2• son definitivamente múltiplos una de la otra Necesitamos otra solu-
ción (independiente) . Intentemos con n(2").
Si sustituimos ª• = n(2") en la relación dada. obtenemos 4a_. . 1 - 4a,. = 4(n + l )2• • 1 -
4n2' = 2(n + 1¡2• ••-n2.. , = [2n + 2-n]2.. ' '= (n + 2)2..' = a • • ,, por loquen(2•) es la
segunda solución independiente. (Es independiente puesto que no se tiene que n2" = J:2•
para todo n ;;?: O si k es una cons1ante.)
La solución general es de la forma a,.= c 1(2") + c2 n(2"). Si ao = 1, a 1 = 3 1 tenemos que
ª• =2• + (l/2)n(2') =2• + n(2• - 1¡, n;, O.

Nuestro último ejemplo se rclacíona con algo de probabilidad.

Si se registra un primer caso de sarampión en un sistema escolar. sea p. la probabilidad de


que se informe de al menos un caso durante la n-ésima semana posterior al primer caso
reportado. Los registros escolares muestran una evidencia de que p,,, = p,._1 - (0.25)p~ -l•
= =
donden e?: 2. Comop0 O y p 1 l, si el primer caso (o un nuevo brote) se registra el lunes
primero de marzo de 1993, ¿cuándo se redujo 1a probabilidad de la aparición de un nuevo
caso a menos de 0 .01 por vez primera?
Si p~ = cr•, e, r=I O, la ecuación característica de la relación de recwrencia es r 2 - r +
(1/4) = O= (r-(1/2))'. La solución general tiene la fonnap, = (c1 + c,nXl/2)', n ;,O. Para
Po = O y p 1 = 1, obtenemos c 1 = O, c2 = 2, por lo que P~ =nl-""+ 1, n ~ O.
El primer entero n para el que P~ < 0.01 es 12. Por lo tanto, no fue sino hasta la semana
del 16 de mayo de 1993 cuando la probabilidad de que se presentara otro caso nuevo fue
menor que 0.01.

EJERCICIOS 10.2 1. Resuelva las siguientes relaciones de recurrencia. (Ninguna de las respuestas fin.al.es contiene
números complejos.)
a) a., =Sa..-1 +6a,.- 2, n2:2, ao=l, a,=3
b) 2a... - 2- lla.,-1+5a., = 0. n2: 0, ao=2, a,=-8
e) 3a....1=2.a,.+a,._1, n2:l, ao = 7, a1= 3
d) a...1 + a.. -0, n :c=:O. ao = O. a1 = 3
o.
e) a~... 2 +4a.. ... n 2!'.Q, ao = a 1 = l
f) a,. - 6a,,-1+9a,,_z= 0. n2!.2, ao=5. a 1 = 12
g) a,.+:z.a,._, + 2.a,.-z=0. n2!.2. ao=l, a , = 3
10.2 La relación de recurrencia lineal ho undo orden con ... 481

2. a) Verifique las soluciones fina1es de los ejemplos 10. 11 y 10.19.


b) Resuelva la relación de recuneaciadelejemplo 10.13.
3 . Si ao •O,ª•• I, ai •4 y a 3 = 37 satisfaoen la relación de rccum::ncia a,,. 2 +ba... 1 + ca,, s:: O,
doodc n ~ O y b,c son COOStantC$, encuentre a_.
4. Encuentre y reSÜelva una relación de recurrenc:ia para el mlmero de formas de estacionar las
motocicletas y los autos peqUCOOS en una fila de n espacios si cada motocicleta requiere un
espacio y un auto dos. (rodas las motocicletas tieocn aparieoc:ia idmtica. lo mismo que los
autos, y queremos utilizar todo$ los espacios.)
5. En el ejercicio 14 delaseccióo 4.2 vi.mosque Ft + F1 + F1 + ···+F.= L~=
F•• rl. tsla.
es una de las muchas propiedades de los números de Fibonacci, dcscubienas por el matemático
ftllDC& ~ Loc:as (1842-1891~ Auncp,e establecimos el l<SUltado medianu: la inducci6o
matermtica. vemos que es fácildesarrollarcstafórmulasi sumamosclsistemaden+ 1 ecuaciones
F,=F, - F,
F-. = F, - F,

F.-1 = F. •• -F.
F,,'""' F,..1 - F...1.
Desarrolle fónnulas para las sumas siguientes y verifique desputs el resultado general por
inducción matemática.
a) F1 + F3 + F,+ ··· +F1,,.1,doodene z•
b) Fo+ F1 + F,+··· +F:..doodenEz•
6. a) Demuestre que
F.., l+VS
lfm-=--.
•-• F. 2
(Este límite se conoce como la razón 6urta y se dcno(a oon frecuencia como a..)
b) Consjdere un pcnú.gono regular ABCDE inscrito en una circunferencia, como se muestra
en la figun 10.7.
i) Use la ley de los senos y ta fórmula del seno del doble de un tnguto para mostrar que
,4.CIAX = 2 cos 36º.
ü) Comocos ts• = sen 12• = 4 sen 18° cos 18°(1 - 2 scn 2 18°} (¿Porqu~?). muestre que
sen 18° es una raíz de la ecuación polinomial 8r-4x+ 1 = Oy deduzca que seo 18º =
(./s - 1)14.
<) Verifique que ACIAX =(1 + ./s)12.
8

·(lfi)'
f D Figura 10.7

7. Para n ~ o. sea º• el número de formas tn que una sucesión de unos y doses suman n. Poc
ejemplo,ai• 3. pues( I) 1, 1, l; (2) 1, 2; y (3) 2, 1 suman 3. Encuentre y resuelva una relación
de ICCUJTCncia pan a,..
8. Paran EN. demuestre que a) L:.~l••· • 4F•.• y b) r :'"°"F.., = t IF••,.
482 (.apitulo 1O Relaciones de recurrencia

9. Encuentre y resuelva una relación de recuncncia para el 1111mero de formas de apilar n fichas
de póquer de color rojo, blanco, verde y azul, de modo que no baya fichas azules consecutiv~.
1O. En un casino, se tienen fichas de póquer de~) colores, uno de los cuaJes es el azul. Encuen-
tre y resuelva una relación de: recum:ncia para el ntl.mero de formas en que José puede apilar n
de estas fichas sin que haya fichas azules consecutivas.
11. Un alfabeto:I consta de los cu.atto caracteres numéricos l, 2. 3, 4 y los siete caracteres alfab6ticos
a. b, e, d, e, f, g. Encuentre y resuelva una relación de rccurrencia para el nómero de palabras
de longitud n (en :I• ) tales que oo aparez.c:an caracteres alfaMticos (idénticos o distintos) co
forma consecutiva.
12. Un alfabeto l: consta de siete caracteres numáicos y k caracteres alfabéticos. Para n ~ O, a~
cuenta el número de cadenas (en 1:-) de longitud n que no contienen caracteres alfabtticos
(idénticos o distintos) consecutivo.s. Si a,.. 2 =7a•• 1 + 63a., n~ O, ¿cuál es el valor de k?
13. Resuelva la relación de recurrencia ª•. 2 = (a,.. 1)(aJ n ~ O, ao;:; 1, a 1 : 2.
14. Paran;?: 1, sea a,. el nllmero de formas de escribir n como una suma ordenada de enteros
positivos, de modo que cada sumando sea mayo..- o igual que 2. (Por ejemplo, a,= 3, puesto
que podemos representar S como 5, 2 + 3 y 3 + 2.) Encuentre y resuelva u.na relación de
rccum:ncia para a•.
15. Una partfcula se mueve en forma horizontal hacia la derecha. Paran E z--, la distancia que re-
corre la partícula en el (n + l~mo segundo es igual al doble de la distancia que recorre duran-
te el n~imo segundo. Six•• n 2: O. denota la posición de la partícula al inicio del (n + 1)-ésimo
segundo, encuentre y resuelva una relación de recurrencia para x. , donde .ro= l y x 1 = S.
16. Paran ~ l, sea D. el siguiente determinante de n x n.
1 o o o o o o
1 2 1 o o o o o
o 1 2 1 o o o o
o o o o o 2 1 o
o o o o o 1 2 1
o o o o o o 1 2
Encuentre y resuelva uná relación de rccum:ncia para el valor de D,...
17. Resuelva la relación de recurrenciaa!.2 + Sa!.1 + 4aJ =O.donde n ~O y ao=4, a 1 = 13.
18. Detemtine las constantes b y e si a. = c 1 + c:z(7•), n 2: O, es la solución ge neral de la relación
a• • 2 +ba• • 1 +ca,,=0, n2:0.
19. Demuestre que dos números de Fibonacci consecutivos son primos relativos.
20. Escriba un programa (o desarrolle un aJgoritmo) paradetcnninar si un entero no negativo dado
es un mlmcro de fibonacci.

10.3
La relación de recurrencia
no homogénea

Estudiaremos ahora las relaciones de recurrencia

a. + C.-,••-• =f(n), n '< 1, (1)


ª· + c._,.__, + c._,.__,= f(n) , n "'2, (2)
10.3 la relación de rerurrenda no homogénea 483

doodeC._, y C._1 son constantes. C,._1 'F Oen ta ecuación (1), C,._1 -/; 0,y/(n) noes idéntico a
cero. Aunque no existe un método general para resolver todas Jas relaciones no homogé-
neas, existe u.na t~nica útil cu.ando la función/(n) tiene una cierta forma.
Comenzamos con el caso panicular de la ecuación (1) cuando C,. _1 =- -1. Para la rela-
ción no homogénea a. -a._ 1 ;;;;/(n), tenemos

a, =a,, + /(1)
a, =a, + /(2) =a,,+ /(1)+ /(2)
ª' ª'
= + / (3) = ao + /(1) + /(2) + /(3)

a, = ao+/(!) + · · · + /(n) = ao+ ±t(i).


i• I

Podemos resolver este tipo de relación en términos de I,;_J(l), si podemos encontrar


u.na fórmula adecuada mediante nuestro trabajo anterior.

Resuelva la relación de recurrenciaa,.-°"_1 = 3n2, donde n ~ 1 y ao = 7.


En este caso,/(n) = 3n', por lo que la solución general es

• • 1
a,=ao+ L/(i) = 7+ 3¿ i1 = 7+-(n)(n + 1)(2n + 1).
1•1 i• I 2

Cuando no se conoce una fónnula para la suma. el siguiente procedimiento nos permi-
tirá trabajar la ecuación (1 ) para cienas funciones/(n), independientemente del vaJor de
C,. • 1 (1- O); también funciona para la relación no homogénea de segundo orden de la ecua-
ción (2); de nuevo, para cienas funcionesf (n). Este método se c.onocc como el mirodo.<k
coeficie.nus indeterminados y se basa en la relación homogénea asociada que se obtiene al
reemplazar /(n) con O.
Para cualquiera de las ecuaciones (1) o (2), sea a!" la solución general de la relación
homoginea asociada. y sea a~>una solución de la relación no homog~nea dada. El térmi-
no a~, >es una solución particular. Entonces a11 = a~•> + a~,> es la solución general de la
relación dada. Para obtener a~> usamos la forma def(n) para sugerir una forma de a~').

:..;:;:.,< Resuelva la relación de rccwrencia a., - 3a.,_ 1 = 5(7•). donde n ~ 1 y ao = 2.


=== - 1_0.U
La solución de la relación bomogtnea asociada es a'," =c(3"). Puesto que/(n) = 5(7•),
buscamos una solución particular a!'' de la forma A(?"). Como a!'> dc:be ser una solución
de la relación no homogénea dada. sustituimos a~,>= A(7") en la relación dada y vemos
queA(7•)-3A(7•· 1) = 5(7"), n 2 l. Si dívídimosentre 7•• 1, vemos que 7A -3A = 5(7), por
lo que A = 35/4 y•!'' =(35/4)7• = (5/4)7.. 1, n 2 O. La solución general es a,= c(3") +
(5/4)7•· 1• Si 2 =a,= e+ (5/4)7, entonces e= -27/4 y a.= (5/4)(7•· 1) -(1/4)(3'"'), n 2 O.
84 capitulo 1O Relaciones de recurrencia

Resuelva la relación de rccurrenciaa._- 3a._ 1 = 5(3•). donde n 2: 1 y ao = 2.


Como en el ejemplo 10.22, a~">::.. c(3"), pero en este caso d,."' y/(n) no son linealmente
independientes. Como resultado, buscamos una solución particular a;,> de la forma Bn(3•).
(¿Qué ocurre si sustituimos a:''= 8(3•), en la relación dada?)
Al sustituir a~,> = Bn3•. en la relación dada obtenemos

Bn(3") - 3B(n -1)(3"-1) = 5(3"), o Bn - B(n -1) = 5, entonces B = 5.

Por lo tanto, a,. = a!'>+ al'l= (e + 5n)3•. n 2: O. Si tlo = 2, la solución general es a. =


(2 + 5nX3").

Los dos ejemplos anteriores nos llevan a la siguiente generalización.

dandetes-..--y•ez+.Si.-...• -
asociada

__,.. tlf'= Ar",dande A e s - - S i


.......,._aoociada._d!'=...... - .
CCIISiclelaDosabcndcasodolanlat:idllllO
-.+c....-....+c:..-
dandetes ... - . E a ... c:aso,
a) 0::'=Ar",A-sir"DOCS_. . . . .
b) d!' = &tr", B - . si<" =c.,-+cr,.
e) d!'=Orr",C-.-.e'=(c1 +

~ _¡,!~ 4 Las ton-es de Hanoi. Consideremosn discos circulares (con diferentes diámetros) y aguj,e.
ros en su centro. Estos ruscos pueden apilarse en cualquiera de las espigas que se muestran
en la figura 10.8. En la f i ~ n = 5 y los discos se apilan en la espiga I. sin que ningl1n
disco quede sobre otro más pequeño. El objetivo es pasar los discos, de uno en uno. de
modo que la pila original termine en la espiga 3. Cada una de las espigas 1, 2 y 3 puede
usarse para ubicar en forma temporal los discos, pero no se permite que un disco más
grande quede sobre otro más pequeño. ¿Cuál es el número mínimo de movimientos nece-
sarios para hacer esto con n discos?
Paran~ O, seaª" eJ número mínimo de movimientos necesarios para pasar los n discos
de la espiga 1 a la espiga 3 de la forma descrita. Entonces. para n + 1 discos. hacemos lo
siguiente:
a) Pasamos los n discos de arriba, de la espiga 1 a la espiga 2, con las indicaciones
dadas. Esto se realiza en ª• pasos.
10.3 La relación de recurrencia no homogén~ 485

• •

F'tgurJi 10.8

b) Pasamos el disco más grande de la espiga I a la 3. Esto se hace en un paso.


e) Por óltimo, pasamos los n discos de la espiga 2 sobre el disco mayor que ahora está
en la espiga 3. de ouevo. coo las instrucciones dadas. Esto requiere otrosª• movi•
mientos.
Esto produce la relación a•• 1 = 2a,, + l, donde n ~ Oy Do= O.
Paraª•• 1 - 2a. = 1, a!" =c(2"). Como f (n) = 1 no es solución de a•• , - 2a. = O, sea
a';.'> = A(l )• = A; de la relación anterior vemos que A =2A + l. por lo que A =- 1 y ª• =
c(2•)- l. Dea., = e- 1, concluimos que e= 1 y que a.= 2• - 1, n 2: O.

El siguiente ejemplo proviene de las matemáticas financieras.

Paulina recibe un préstamo de S dól=s que debe pagar en T periodos. Si i es el tipo de


interés po< periodo del préstamo, ¿cuál es el pago (constante) P que debe hacer al final de
cada periodo?
Sea a. la cantidad debida en el prestamo al final del n~imo periodo (desputs del n-tsimo
pago). Entonces, al final del (n + l~imo periodo, la cantidad que Paulina debe al banco
es a. (la cantidad que debía al final del n-ésuno periodo) + ia. (el interés acumulado du-
rante el (n + 1)-ésimo periodo)- P (el pago que hizo al final del (n + 1)-ésuno periodo).
Esto nos da la relación de recurrencia

a,.+ 1 =a,. + ia,. - P, OsnsT-1, a.,=S, ar=O.

Para esta relación, i :> = c(l + ,y. y a~,> = A. ya que ninguna constante, como -P, es
solución de la relación homogénea asociada. Sia~') =A, tenemos que A -(1 + i)A =-P, de
modo que A = Pli. De a.= S, obtenemos a. = (S - (Ph))(I + if + ( Ph), O,; n ,; Xs
Como O= ar= (S - ( P/,))(1 + fi' + (Ph), se sigue que

(P / i)=((P/i)-S)(l + i)r y P = (Si)[i-(l+i)"']"1•

Consideremos ahora un problema en el análisis de algoritmos.


486 capítulo 1O Relaciones de recurrencia

Paran~ l. sea S un conjunto con 2• números reales.


El siguiente procedimiento se usa para de~ rm.inar los elementos máximo y mínimo de
S. Queremos determinar el número de comparaciones realizadas entre los pares de ele-
mentos de S durante la ejecución de este procedimiento.
Si a. designa el número de comparaciones necesarias, entonces a, = 1. Cuando n = 2.
lsl = 2' = 4, porloqueS= {x,.z,,y,.y,) =S, U S,, dondes,= {x ,.z,),S,= (y,, y,). Como
a 1 ;;:; l, se usa una comparación para determinar los elementos máximo y mínimo de cada
S 1, Si. Si compararnos los elementos mínimos de S1 y S2 y después comparamos sus ele-
mentos máximos, encontramos los elementos máximo y mínimo de S r vemos que G:2 = 4
=2a,+ 2.Engeneral,si lsl = 2.. ',escribimosS = S, US,,donde ls,1 = ls, I =2•. Para
de terminar los elementos máximo y mínimo de S 1 y S2 necesitamos a. comparaciones. Si
comparamos los elementos máximos (mínimos) de S 1 y S2 necesitamos una comparación
más; en consecuencia. a..,. 1 = 2a" + 2, n ~ l.
11
En este caso. a<._ >;; c(2")y a!' >= A, una constante. Si sustituimos a~,> en la relación,
vemos que A = 2A + 2 oA :-2. Así.a,. = c'.2--2 y comoa1 : 1 = 2c- 2, obtenemos e== 3ll.
Por lo tanto, a. = (312X2') - 2.
¡Cuidado! La existencia de este procedimiento. que necesita (3/2)(211) - 2 comparacio-
ne s, no e xcluye la posibilidad de lograr los mismos resultados mediante otto métcxio nota-
blemente mejor que requiera menos comparaciones.

En 1904, el matemático sueco Helge von Koch (1870-1924) creó la interesante curva
conocida ahora como curva ..copo de nieve" de Koch. La construcción de esta curva se
inicia con un triángulo equilátero, como se muestra en la parte (a) de la figura 10.9, donde
el triángulo tiene lado 1, perímetro 3 y área ../314. (Recuerde que un triángulo equilátero
de lado , tie ne perímetro 3s y área s'../314.) El triángulo se transforma entonces en la
estn:lla de David de la figura 10.9(b) quitando el tercio medio de cada lado (del triángulo
equilátero original) y añadiendo un nuevo triángulo equilátero cuyo lado tiene longitud
de 1/3. Así, cuando pasarnos de la parte (a) a la parte (b) de esta figura, cada lado de
longitud 1 se transforma en 4 lados de longitud l/3 y obtenemos un polígono de 12 lados
de área ( ../3/4) + (3X ../314Xl/3'}' = ../313. Si continuamos este proceso, transformamos la

6
(a)
o o
(b) (e)

Figura 10.9
10.3 La relación de recurrencia no homogénea 487

figura de la parte (b) en la de la pane (c) quitando el ten:io medio de cada uno de los 12
lados de la estrella de David y añadiendo un triángulo equilátero de lado 1/9(= (1/3)').
Ahora tenemos [en la Fíg. 10.9(c)] un polígono de 4'(3) lados cuya área es
(VJ/3) + (4)(3)(V3/4)[(1/3)~2 E IOVJ/27.
Para n ~ O, sea a., el área de] poügono P,. obtenida a panir del triángulo equilátero
original despuá de aplicarn uansformaciones del tipo descrito arriba [la primera, de P0 en
la figura I0.9(a) aP, en la figura 10.9(b) y la segunda de P, en la figura I0.9(b) a P, en la
figura 10.9(c)]. Al ir de P. (con 4'(3) lados) a P,., (con 4.. '(3) lados), vemos que
ª··· ª· +
=
1
(4"(3))(V3/4)(1/3.. )' = ª· + (l/(4V3))(4/9)'
puesto que al uansformar P. en P•• 1 eliminamos el tercio medio de cada uno de los 4"(3)
lados de P,y agregamos un triángulo equilátero de lado (1/3.. ').
La parte homogtoea de la solución de esta relación de recwrencia no homog6nea de
primer orden es a';'> =A(l'f =A (que se obtiene de la relación de recu.rrcncia homogénea
asociadaa.,. 1 =a.). Puesto que (419)- no es una solución de la relación homogtnea asocia-
da. la componente particular de la solución está dada por a~") = 8(419'1'. donde Bes una
constante. AJ sustituir esto en la relación de recurrencia
a,., = a.+ (l/(4V3))(4/9)',
vernos que
8(4/9)"" = 8(4/9)' + (l/(4V3))(4/9)",
por lo que
8(4/9) = 8 + (l/(4V3)) 8 ª (-9/5)(1/(4V3)).
En consecuencia.
a.= A + (-9/5)(1/(4V3))(4/9)' = A - (l/(5V3))(4/9)'-', n 2: O.
Como V3/4-ao=A-(l/(5V3))(4/9t'=A-(l/ (5V3))(9/4), se sigue que A=
(VJ/4) + (9/(20V3)) = (15 + 9)/(20V3) = 24/(20V3) = 6/(5v'3) y
ª· = (6/(5VJJ)- (l/(5VJ)X4/9r' = (l/(5V3))[6 - (419)•-•¡, n "o.
[Cuandoncrece, vemos que (419)-- 1 tiende a Oya. tiende al valor finito 6/(543). También
podemos obtener este valor si continuamos realizando los cálculos que hicimos antes de
introducir la relación de recurrencia, y entonces observaremos que el área límite está dada
tambifo por

.
(VJ/4) + (VJ/4)(3)(1/3)2 + (VJ/4)(4)(3)(1/32) 2 + (VJ/4)(4')(3)(1/33) 2 + · · ·
.
.... -~
= (VJ/4) + (VJ/4)(3) ¿ 4"(1/3.. ')' • (VJ/4) + (l/(4V3)) ¿ (4/9)"
= (VJ/4) + (l/(4V3))[1/(I - 4/9))- (VJ/4) + (l/(4V3))(9/5) = 6/(5V3),
utilizando el resultado para la suma de una serie geom~trica de la pane (b) del ejemplo
9.5.)

Cada uno de los dos ejemplos siguientes trata una relación de segundo orden.
J88 capítulo 1O Relaciones de recurrenda

~~J Resuelva la relación de recurrencia

a 11... 2-4a11... 1 +3a,. = -200, n ~ O, ao = 30CN), a1 = 3300.


En este caso, a;" = c 1(3') + c,(I' ) = c1(3') + e,. Como /(n) = -200 = -200(1') es una
solución de la relación homogénea asociada, at'= An para alguna constante A. Esto nos
lleva a
A(n + 2) - 4A(n + !) + 3An = -200, -2A = - 200, A= 100.
Por lo tanto, a,,; c 1 (3") + c2 + 100n. Con ao =3(XX) y a 1 =3300, tenemos que a11 =
100(3') + 2900 + 100n, n ;;e O.

El siguiente ejemplo proporciona otra oportunidad de analizar la función de compleji-


dad en tiempo para un algoritmo.

En la figura 10.1Otenemos un programa en Pasea] que puede usarse para calcular el n-ésimo
número de Fibonacci F., para n 2: O. Debido a las dos líneas (sombreadas) al final de la
función Fib, este programa (o algoritmo) es recursivo. (En Cl ejemplo 5.70 de la sec:ción
5.8 teníamos un programa iterativo para calcular F. para n e N.) Definiremos Ja fun-
ción complejidad en tiempo/para este programa haciendo/(n);; a,.;; el número de sumas
realizadas para calcular F.," ;;e O. Entonces/(0) =ao =O.f(I) =a1 =Oyen la línea del
programa

(' ) Fib := Fib(n-1 ) + Fib(n - 2)

obtenemos la relación de recurrencia no homogénea


a,, = a,._1 +a.,-2+ 1, n ~2.
donde el sumando 1 se debe a la suma en (*).
Aquí vemos que ~"'l : c1(.tp..)'" + cJl=f-)" y que a~,> =A, una constante. Al sustituir a~'' en
la relación de recurrencia no homogénea, tenemos que
A=A+A+l,
por lo que A = -1 y a, =e,(!!#-)' + c,(!=p.)' - l.
Como ao : O y a, : O se sigue que

O=c1 + C1 - 1, o c1 + c2 = l. y

1+ v's\ /1 -v's\
O= c,(- -:-J+c'\--~
¡+ v's\ {1 -v's\
2 2
- ) -1, C¡( - 2-~-) + C'\-2-~-) = l.
A panir de estas ecuaciones vemos que c 1 = (1 + ,Í5')1(2/,ÍS), e,= (,ÍS - 1')1(2,ÍS). En
consecue ncia.

f(n) =a, =(
¡2vs)
+ v's\(l + v's\• (¡ - \/5\(1- \/5\•
_2_~-1 - 2VS) _ 2_:_) -l
= (l/V5)[(1 + '✓5)/2]"'' - (l/V5)[(1 - '✓5)/2]"' ' - l.
10.3 La relación de rocurrencia no homogénea 489

Pro¡ram FibNum21input,output ):
Var
n : integer:

Function Fib( n: intege r ) : inte¡er;

Begin
If º"'º then
fib :• O:
Ir n=l then

Begin
Writeln( 'Proporciona el entero no negativo n . • ) ;
Readln(n);
Writeln ( 'El número de Pibonacci para el subindice n .. ', n: O);
llrite( ' es '. Fib(n ) : O. '. ' )
E:nd .

F"tgura 10.10

Conformen crece, vemos que ((1 - ,ÍS)l2]"' tiende a O, pues 1(1 - ,ÍS)/21 < l.
=
En consecuenciaJ(n) (1/,ÍS)(( 1+ ,ÍS)/'2)--' = (( 1 + ,ÍS')/(2,ÍS)X( 1 + ,ÍS)12'f y pode-
mos escribir/ E O [~)7, Jo cual indica que este enfoque para el cálculo de F. es de
compf.ejidad e:rponLncial y es bastante ineficiente en comparación con el método del ejemplo
5.70 de la sección 5.8.

Ahora resumiremos y extenderemos las t6cnicas de solución ya analiz.adas en los ejem•


plos 10.22 a 10.29.
Dada una relación de rccurrencia lineal no bomog~nea (con coeficientes constantes) de
la forma C.a. + C•• 1a.,_ 1 + · • · + C.-ta.-a = j(n). donde C•• C•. .t '# O, sea a!' >la pane
homogénea de la solución ª•·
1) Si/(n) es un múltiplo constante de una de las formas de la primera columna de la
tabla 10.2 y no es solución de la relación homogénea asociada. entonces a~,> tiene
la forma que se muestra en la segunda columna de la tabla 10.2. (En este caso.A, B.
A.o. Ai. A:, . .. , A,_i. A, son constantes detenninadas mediante la sustitución de al'1
en la relación dada; ,, r y ex. también son constantes.)
2) Cuando/(n) es una suma de múltiplos constantes de ttnninos como los de la prime-
ra columna de la tabla para el punto ( 1), y ninguno de estos ténninos es una solu-
ción de la relación homogénea asociada, entonces a!'> se fonna como la suma de
_los términos correspondientes en la columna encabezada por tf.'>. Por ejemplo, si
f(n) = n' + 3 sen 2n y ninguno de los sumandos de / (n) es solución de la relación
homogénea asociada, entonces a!'> ~ (A~ + A1n + Ao) + (A sen 2n + B cos 2n).
490 capitulo 1O Relaciones de recurrencia

Tabla 10.2
a'"
e, una constante A, una constante
n A1n +Ao
n' A 2 n 2 +A1n+Ao
n 1, tE Z+ A,n'+A,-1 nt-1 + •·· +A1 n + A 0
r",rE R Ar"
senan A senan + B cosan
cosan A senan + B cosan
n' r" r"(A,n' + Ar-1 n'- 1 + · ·· + A 1 n +A0)
r" senan A r'" scnan + Br" cosan
r" cosan Ar" sen an + Br" cosan

3) Las cosas se complican si un sumando.fi(n) de/(n) es un múltiplo constante de una


solución de la relación homogénea asociada. Esto ócurrc, por ejemplo, cuando/(n)
contiene sumandos como cr• o (c 1 + c-in)r" y , es u.na raíz característica. Si fi(n)
causa este problema.. multiplicamos la solución particular a!f, correspondiente a
/i(n) por la mínima potencia den, rt, para la que ningún sumando de ,tf1(n) es una
solución de la relación homogénea asociada. Entonces n•a~t es la parte correspon-
diente de a~' >.
Para verificar aJgunas de nuestras observaciones acerca de las soluciones particulares
para las relaciones de recurrcncia no homogéneas, la siguiente aplicación nos proporciona
una situación que puede resolverse de más de una manera.

r ~~~jo¡ Paran~ 2, supongamos que hay n personas en una fiesta y que cada una de ellas da ]ama-
no (exactame nte una vez) a todas las demás personas (y nadie estreeha su propia mano). Si
a11 cuenta e] total de saludos de mano. e ntonces

ª""' 1 =a,..+ n, (3)

puesto que cuando la (n + 1)-ésima persona llegue, estrechará la mano de las n personas
que ya habían llegado.
De acuerdo con los resull>dos de la tabla 10.2, podóamos pensar que la solución parti-
cular de la ecuación (3) esA,n + A.o. paraA 0 y A 1 constantes. Pero en este caso, la relación
homogéneaasociadaesa,. .. 1 ~ a. oa... 1 -a. ~o. para lo cual a!,"'>= c(l'f =e, donde ces una
constante arbitraria. Por lo tanto, el sumando A0 de (A1n + A.o) es una solución de la rela-
ción homogénea asociada. En consecuencia, la tercera observación anterior indica que
debemos multiplicar A 1n + A0 por la mínima potencia de n para la que ya no existe un
sumando constante. Hacemos esto multiplicando A 1n + Ao por n 1 y vemos que
10.3 la relación de recurrencia no homogénea 491

Cuando sustituimos este resultado en la ecuaci6o (3) obtenemos


A 1(n + !)' + Ao(n + ! ) = A,n' + Aon + n,
o A ,n' + (2A 1 + A 0 )n + (A,+ Ao) = A ,n' + (Ao + l)n.
Al comparar los coeficientes de las potencias semejantes den tenemos que
(n'): A 1 = A 1;
(n): 2A ,+ Ao=Ao+ l; y
(n°): A , + Ao = O.

Por lo tanto,A 1 = 1/2 y Ao= -112. por lo que a!,"=(112),,' + (-lf2)n y a,,= a';." + a!," = e+
(lf2XnXn - 1). Como a, = 1, se sigue del hecho 1 =a,= e+ ( lf2X2XJ) que e= O y a, =
(lf2XnXn - 1). paran;, 2.
Tambitn podemos obtener este resultado teniendo en cuenta lasn personas que están en
la habitación y pensando que cada apretón de manos corresponde a una selección de tama-
~o 2 de este conjunto de lalllañon, y que hay (;) = (n!)/(2!(n-2)!) = (lf2XnXn- 1) de tales
selecciones. [O bien considerar a las n personas como los vtnices de un grafo no dirigido
(sin lazos) donde cada arista corresponde a un apretón. Nuestra respuesta es entonces el
nwncro de aristas que hay en el grafo completo K~ y hay (l l = (lf2XnXn - 1) de tales
aristas.]

Nuestro último ejemplo muestra la forma en que podemos usar los resultados de la
tabla 10.2.

,~,0.11.f a) Consideremos la relación de recurreocia no homogénea

a,., - !Oa,, 1 + 21a, =/(n), n "' O.


En este caso, la pane homogénea de 1a solución es
al" - c1(3') + c2(7"),
para las constantes arbitrarias Ci, C2.
En la tabla 10.3 enumeramos la foona de la solución particular para ciertas opcio-
nes dc/(n). En este caso, las 11 coostantes A0 para OSiS 10, son constantes particu-
lares determinadas aJ sustituir a;,> en la relación de rccuncncia no bomogtnea dada

1:lbla 10.3
/{n)

5 Ao
3n' - 2 A)ft2 + Azn+ A 1
7(11") A,(11")
3 J(r ' ), r ,; 3. 7 A,(r")
6(3) A.,n3
2(3")-8(9") A,n3' + A.(9")
4(3") + 3(7") A,n3" + A io117"
192 capítulo 1O Relaciones de recurrencia

b) La componente homogénea de la solución de

ª• + 4a.-, + 4a._, =f(n), n 2 2,


es
a:•>= c,(-2}" + c,n(-2}",
donde ci, c2 son constantes arbitrarias. En consecuencia.
1) si/(n) = 5(-2)", entonces a."' =
An'(-2)";
2) si/(n) = 7n(-2)", entonces a!' ' = Bn'(-2)"; pero
3) si/(n) = -lln'(-2)", entooces a!' ' = Cn'(-2)•.
(Aquí, A. B. C se determinan al sustituir a!:) en la relación de recurrencia no homo-
génea dada.)
e) Cuando analizamos la relación de recurrencia no homogénea

ª••• + ª• = f(n), n 2'0,


vemos que a!•l = c 1cos(rm/2) + c 2 sen(nx/2), parac1 y c1 constantes arbitrarias. Por
lo tanto, si

f(n) = 5 cos(n1r/3)-7sen(n1r/4),
entonces
a(•>= A cos(n,r/3) + Bsen(n,r/J) + C cos(n,r/4) + D sen(n1r/4).
(En este caso, las cuatro constantes A, B, C y D pueden ca1cularse al sustituir a~, >en
la relación de recurrencia no homogénea dada.)

jercicios 10.3 1. Resuelva las siguientes relaciones de rectmcncia.


a) a,....1 - a,.=2n+3, na:: O, ao=l e) a,. ... ,-2a,. =5, n~O, ao = l
b) a,..1 -a,. =3n2 -n, ni:!::0, ao =3 d) a,..,-2a,.=2", na::O, ao= l

2. Use una relación de recurn:ncia para obtener la fórmula para I,;"/1.


3. a) Tracemos n líneas en el plano de modo que cada una de ellas intersequc a las demás pero
que cualesquiera tres líneas no coincidan en ningt1n un punto. Paran ~ O. sea a,, eJ número
de regiones en las que se separa el plano mediante estasn líneas. Encuenttt. y resuelva una
relación de recurrencia para a..
b) Para la situación de la parte (a), sea b,. el número de n:gioncs infinitas obtenidas de esa
forma. Encuentre y resuelva una relación de rccurrcnc:ia para b...
4. En el primer día de un nuevo a.fto. Jo~ deposita $ 1000 en una cuenta que paga un interés
compuesto mensual del 6%. Al principio de cada mes, él agrega $200 a esa cuenta Si continúa
haciendo esto durante los cuatro años siguientes (de modo que realice 47 depósitos más de
S200). ¿cuánto dinero tendrá en su cuenta después de esos cuatro aik>s?
5. Resuelva las siguientes relaciones de rccurrcncia.

a) a,..2+3a,,- 1 +2a,,=3", n2:0, ao=O, a1=l


b) a.,.. 2+4a,. ..1 +4a,. = 7 , n.?: 0, ao=l, a1=2
e) a,, .. 2-a.. = scn(n-rr/2), n.?: 0, ao= l, a1=l

6. Resuelva la relación de recurrenciaa... 2 - 6a,,.1 + 9a. = 3(2"} + 7(3'"), donden ~ O y ao = 1,


D1= 4.
10.4 El uso de las funciones gffleratrices 493

7. Resuelva la relación de recurrencia a•• ,- 3a,., 1 + 3a,.. 1 - a.• 3 + Sn, n~ O.


l . Detemtine el número de sucesiones cuaternarias (0,1. 2, 3) de n dígitos en las que nunca
aparece un 3 en cualquier lugar a la derecha de un O.
9. Mercedes pide prcsudos S2500, con un interts compucslo mensual del 12,._, para adqwrir un
comp.itadoc. Si la deuda debe pagarse co. dos aftas. ¿cuil es el pago mensual?
11. La solución general de la relación de rccurrcncia a•• 1 + bia,.. 1 + i>za.. =b,n + ó,, n ~ O, con ó1
constante para l :s; i S 4, es c-12" + e~" + n - 1. Encuentre b, para cada 1 S iS 4.
11. R~clva las siguientes relaciones de recu.rrcncia
a) a;..2 - Sa!.., +6a! = 7n, nzO, 0o•o1 =1
b) a. + na..-1 =n!, n.t:l, 0o=l
e) a!-2a..- 1= 0, n.t:l , ac,•2 (Seab,. • logzo,.,n c: O.)
12. a) Paran~ 1, el n--áimo nómerotriangu!ar ,,. se define como r,.= 1 + 2 + • · • + n=.r(n+ IY2.
Encuentre y resuelva una relación derccurrencia paraJ,,. n~ l. dondeJ.• t 1 + t: + • • · + 1,,.
la suma de los primeros n m1mcros lriangu!ares.
b) Enunllborarorioorgfnioo, Arturosín<eúzaunacsauauncristalinaformadapor 10.000,000
de capas triangulares de 4tomos. La primera capa de la csuuctura tiene un ilomo. la segunda
tres átomos. la tercera tiene seis y, en general. la n-tsima capa tiene 1 + 2 + · · · + n = r,.
átomos. (Podríamos considerar cada capa. distinta de la ólti.ma. como si se colocara sobre
los espacios que quedan entre los Wmos vecinos~ la s i¡uientc capa. Vúse la fi¡ura
10.11.)
1) ¿CU.Antos átomos hay en una de estas estructuras aistalinas?
H) ¿Cuántos átomos están empaquetados (estrictamente) entre las capas 10,000 y la
100,000?

C,, •1)
o
Figura 10.11
~
(n - 2)
A
(n - 3)
A
(n - 4 )

13. Escriba un pto&J'Wl3 (o desarrolle un algoritmo) para resoh'cr el problema de las torres de
Hanoi. Paran E z·, el pro¡rama debe proporcionar los pasos necesarios para pasar losndiscos
de la espiga I a la 3 con las restricciones especificadas en el ejemplo l 0.24.

10.4
El método de las funciones
generatrices

Con tantos casos por considerar para la relación de recurrcocia lineal no homogénea. aher
ra recibiremos la ayuda de la función generatriz. Esta técnica encontrará las soluciones
194 (.apítulo 1O Relaciones de recurrencia

particular y homogénea de a,., y también incorporará las conWciones iniciales dadas; ade-
más, podremos hacer más cosas con este método.
Demostraremos este método e n los ejemplos siguientes.

Resuelva la relaciónª• - 3a,._1 = n, n ~ 1, ao = l.


Esta relación representa un conjunto infinito de ecuaciones:
(n = 1) a 1 -:lao= 1
(n =2) a2-3ai=2
(n =3) a3 -3a, = 3

Al multiplicar la primera de estas ecuaciones por .x, la segunda por x2, la tercera por x3,
etcétera, obtenemos
(n = !) 1
01X - 3aox1 = lx1
(n = 2) a2x2 -3a1r2 =2x2
(n=3) a3x3- 3a2xl=lx3 .

Si sumamos este segundo conjunto de ecuaciones, vemos que

. . a.-,x"= L'.. nx".


..L'.,a.x" - 3 L'. (1)

Queremos despejar a, en términos de n. Para esto, sea /(x) = I,:.,a.... la función


generatriz (ordinaria) para la sucesión ao. a 1, az. ... Entonces podemos volver a escribir la
ecuación (1) como

(f(x) - ao)- 3x j; a._,x"- = j; nx"(=.j;-onx•).


11• 1
1

... ,
(2)

Como L:. P•- 1r - 1 = L :.Oa,. x" - J(x), el lado izquierdo de la ecuación (2) se convierte
en (f(x ) - l ) -3x/(x).
Antes de continuar, necesitamos la función generatriz de la sucesión O. 1, 2, 3, .
Recordemos, de la parte (e) del ej emplo 9.5, que

_1_ = .!._(_1_) =1+2x+3x' + · · ·


(1 - x)' dx 1-x

Porlotanto,x/(l -x)' =x+2.x'+3x'+ · · · ,y

¡ X
(f(x)- l) -3x/(x) = _ x - , y
(1 - x )' /(x) = (1 - 3x) + (! - x)1(1 - 3x)
10.4 El uso de la> funciones generatrices 495

Si usamos una descomposicióo en fracciones simples veremos que

x A B C
(1-x)'(l-3x) = (1-x) + (1-x)' + (1-3x) '
o
x =A(! - x)(l - 3x) + B(l - 3x) + C(l - x)'.

Con las siguientes asignaciooes para x, obtenemos

1
(x=l): l - B(-2), B=--.
2

(x = 1)=1-e@)'. C= ~-
4
(x=O): O-A+ B + C, A=-(B+C)=-!.
4

Por lo tanto,
1 (-1/4) (-1/2) (3/4)
f(x) - - - + -- +--+- -
1- 3x (1 - x) (! - x)' (1 - 3x)
(7/4) (- 1/4) (-1 /2)
= ---+- - + - -.
(1- 3x) (1 - x) (1- x)'
Encontramos a. aJ determinar el coeficiente de x• en cada uno de los tres sumandos.
a) (7/4)1(1 - 3x) = (7/4)(1/(1 -3x)] = (7/4)11 + (3x) + (3x)' + (3x)' + · · · J y el coefi-
ciente de x• es (7/4)3•.
b) (-1/4)1(1 - x) = (-1/4)(1 + x +x' + .. · J y el coeficiente dex• es (-1/4).
e) (-112)1(1 -x)' =(-112)1(1-x¡-• =(-1/2)(1.') + (7 )(-x) + (1)(-xJ' + ('j2)(-x)' + · · · J
y el coeficiente de x• está dado por (-1/2)(-;')(-l)• = (-l/2X-IJ'('-;-'J • (-IJ• =
(-112X• + IJ.
Por lo laJlto, a.= (7/4)3• - (l/2)n - (3/4), n ~ O. (Observe que a!' ' no tiene nada de
particular. Además, se obtiene la misma respuesta con las ~nicas de la sección 10.3.)

En nuestro siguiente ejemplo extenderemos lo aprendido en el ejemplo I0.32 a una


relación de segundo orden. Esta vez. presentamos la solución dentro de una lista de instruc-
ciones que pueden seguirse para aplicar el m~todo de la función generatriz.

, . . . . . t&.31 j Consideromos la rolación de rocumncia

ª"•2- Sa,. ..1 + 6a,. = 2, n.:O, ao•3, a1=1.

1) Primero multiplicarnos esta relación porr• 2, puestoqucn + 2cs el máximo subíndice


que aparece en ella. Esto nos da
Capítulo 1O Refac:iones de recurrencia

2) Después sumamos todas las ecuaciones representadas por el resultado del paso (1)
y obtenemos

:Í: a,.+2.rº
..-o
2- S f a,....
... o
1x" + 2 +6 f a,.x".. 2 fo.r...2.
..-o
1

...
3) Para que todos los subíndices de a concuerden con e] exponente correspondiente en
x, volveremos a escribir la ecuación del paso (2) como
.
L a•• ,x..2 -
-
5x L a,. 1 x•• 1 + 6r' L a, x• =2x'
. ""
L x•.
..-o ""'º ""'º ..-o
En este caso, también escribimos de nuevo la serie de potencias del lado de=bo de la
ecuación en una forma que oos permita usar Jo aprendido en la sección 2 del capítulo 9.
4) Sea/(x) = 2,~..oª•x• la función generatriz de la solución. La ecuación del paso (3)
toma entonces la forma
2x'
(f(x) - ao - a, x) - 5x(f(x) - ao) + 6x 2/(x) = _ x ,
1

o
(f(x) - 3 - 7x) - 5x(/(x) - 3) + 6x2 f(x) =~ .
1- x
S) Despejamos /(x) para obtener
. 2x'
(1 - 5x + 6x 2)/(x) = 3 - 8x +¡:=-;,

de lo cual se sigue que


3 - Sx 2x'
f(x)
(1 - 5x + 6x') + (! - 5x + 6x')(I - x)
(3 - Sx)(I - x) + 2x2 !0.2 - !lx +3
(! - x)( l - 2x)(l - 3x) (1 - x)(I - 2x)(l - 3x) ·
Hacemos una descomposición en fracciones simples (a mano o por medio de un
programa de cá1cu1o simbólico) para obtener

2 1
f(x)= - - +-- = 2± (3x)" +
1-3.r 1-.r ..--o ... o
±x-.
Por Jo tanto, a,. = 2(3•) + 1, n ~ O.

Consideremos un tercer ejemplo, que tiene un resultado conocido.

Sea n E N . Para r ~ O, sea a(n,r) el número de formas en que podemos seleccionar r


objetos de un conjunto den objetos distintos, permitiendo las repeticiones.
Paran~ 1, sea {b¡, b 2, ••• , b,.} el conjunto de estos objetos. Consideremos el objetob,.
Pueden ocurrir dos casos.
10.4 El uso de las funciones generatrices 497

a) El objeto b1 nunca es seleccionado. Por Jo tanto, los r objetos se seleccionan de


{b,, . .. , b.). lo que podemos hacer de a(n-1, r) formas.
b) El objeto b1 se selecciona al menos una vez. Entonces debemos seleccionar r- 1
objetos de {b., b,, ... , b.), de modo que podamos seguir seleccionando b, además
de la selección ya hecha Hay a(n, r-1 ) formas de hacer esto.
Entonces a(n, r) = a(n - l. r) + a(n, r- 1) ya que estos dos casos aban:an todas las
posibilidades y son disjuntos.
Sea/, = I.:.a(n, r).t' la función generatriz de la sucesióna(n, 0),a(n, l),a(n, 2), ... [En
este caso,/. es una abreviatura def.(x).} De la ecuación a(n, r) = a(n-1 , r) + a(n, r- 1),
donde n .?: l y r.?: l se sigue que

. a(n, r).t' • a(n-l


.
, r).t' + a(n,r- 1).t'
.
¿a(n,r).t' • ¿a(n - 1,r).t' + ¿ a(n,r - 1).t'.
y

,...¡ ,.., ,...,

Si observamos que a(n, O)= 1 paran;, O y a(O, r) = O para r > O, podemos escribir

f. -a(n, 0)-J.-, - a(n -1,0) + x ¿a(n, r - 1).r',


~,
de donde f. - 1 =/.-, - 1 + xf.. Podo w,to, f.-xf. =f.., of. =f. _,f(I - x).
Por ejemplo, sin = S, entonces

!, = _l!_ = _i_. _L= _ L _ _ L = _ ¡,_,-


(1- x) (1- x) (1 - x) (1- x}' (1 - x)' (1 - x)'
fo 1
= (1 - x)' = (1 - x)'

ya que / 0 =a(O, O)+a(0,1).r + a(0, 2).r'+ · ·· -1 +0+0+ ·· ·.


En general,/.= 11(1 - x'f = (1 - x)"", de modo que a(n, r ) es el coeficiente de r en (1- x)"",
igual a (:•)( - l )' = (~;-•).
(Aquí tratamos una relación de recurrcncia para a(n, r), una función discreta de dos
variables (enteras), n, r;, O.)

Nuestro último ejemplo muestra cómo utilizar las funciones generatrices para resolver
un sistema de relaciones de rccwrcncia. ·

=====J
1~ Este ejemplo proporciona un modelo aproximado de la propagación de los neutrones de
a1ta y baja energía cuando chocan con los núcleos de material fisionable (como et uranio)
y son absorbidos.Aquí trabajamos con un reactor rápido que no tiene un moderador (como
el agua). (En realidad, todos los neuuooes tienen una energía bastante alta y no sólo dos
niveles de energía nenen un espectro continuo de niveles de energía, y los neuuones del
exb"emo superior del espectro son los oeuuones de alta energía. Los neutrones con mayor
energía tienden a producir más neutrones que los de menor energía.)
498 capitulo 1O Relaciones de recurrencia

Consideremos el reactor en el instante O y supongamos que se inyecta un neutrón de


alta energía a1 sistema. Dwante cada intervalo de tiempo posterior (cerca de 1 microsegundo,
o 101 segundos) ocurre Jo siguiente.
a) Cuando un neutrón de alta energía interactúa con un núcleo (de material fisionable),
e l resultado de la absorción (un microsegundo después) son dos neutrones de alta
e nergía y uno de baja
b) Para las interacciones con un neutrón de baja energía, sólo se produce un neutrón de
cada nivel de energía
Si todos los neutrones hbres interactúan con los mkleos un microsegundo después de
su creación, encuentre las funciones de n tales que
a,. sea igual al número de neutrones de alta energía,
b,. sea igua] al nómero de neutrones de baja energía,
en el reactor, después de n microsegundos, n e: O.
En este caso tenemos ao = 1, b0 ;;;; O y el sistema de relaciones de recurrencia

ª " +l = 2a" + b,. (3)


b11• t =a,. + b,.. (4)
Sean!.,.= L:-oa.,r', Kx ;;;; L:Ob,.:c•
las funciones generacrices de las sucesiones {a..1 n~
O}, (b, In 2: O}, respecti,•ame nte.
De las ecuaciones (3) y (4), paran 2: O,

a,11.1r º 1 = 2a,.x 11 • 1 + b11.x11 • 1 (3)'


b11♦1.x 11• 1 = a11 x"+ 1 + b,, x "+1 (4)'
S i sumamos la ecuación (3) ~ para todo n ~ O, tenemos
. .
¿ a,,+ 1 x"♦1 = 2x L a,.x"+x L b,.x".
.
(3)'
11-0 11• 0 "'"'º
De manera análoga, obtenemos de la ecuación (4)'
. .
¿ bu1 x""'1 = x L a,.x"+x L b,.x".
- (4)"
.. -o 11-0 11-0

Utilizamos la función generatriz en este momento para obtener

f(x) - ao = 2xf(x) + xg(x) (3)'


g(x) - bo = xf(x) + xg(x), (4)"
un sistema de ecuaciones que relaciona las funciones generatrices. Al resolve r este siste-
ma, vemos que

f(x) = -
2
X
1-x (5+V5)(
-- = -
-10
1 )+ (5-V5)(
3x + i
- -
10
-1
6-
-) y
- --
y- X X

g(x)= - x -=(-5-3V5)(-I)+(-5+3V5)(-1),
2
x -3x+I 10 y-x 10 6-x
10.5 Un tipo de relación de recurrencia no lineal (opcional} 499

donde
3+v's 3-v's
-y=-2-· & = - -.
2
En consecuencia,

ª· = (5 ~;5)(3 -tr
(5 ~;5)(3 \vsr
+ y

b.=(-5~:vs)(3-/l'f' + (-5::vs)(3\vsf. n"'º·

EJERCICIOS 10. 4 1. Resuelva las siguientes reJacionei de recu.rrenda por el m6todo de las funciones ¡eneratriccs.
a) a,, .. 1 -a,,- 3'", n2:0, ao - 1 b) a.,. 1-a,, • n 1, n2::0, ao -1
e) a.,-3a..- 1 =S"- 1, n 2: l , ao-1
d) a.....1- 3a,..1+2a.... 0, n .i: O. ao• l , a,•6
e) a., ...1 - 2a,..1 +a,, =Z', n 2: 0, ac, ic l, a 1 :a 2
2. Paran objetos distintos, sea a(n, r) el mlmcro de formas enºque podemos seleccionar r de los n
objetos, OS r S n, sin repetición. En este caso. a(n, r).:: O cuando r > n. Use la relación de
n:cum:nciaa(n, r) • a(n - 1, r- 1) +o(11- I, r), donden ~ 1 y r~ 1, para 010$UUQUC/(x) =
(1 + :r'f genera a(n. r), r ~ O.
3. Resuelva los ¡iguientes si.stcmas de relaciones de recunenda.
a) a,.., =- 2a,. - 4b,. b) a... 1 • 2a,, -b,. + 2
b,. .. 1 =4a,.+6b,, b,.. 1 :. - a,. + 2b,. - l
n .!!: 0,ao • l, bo=0 n 2: 0, aoe0,bo= l

10.5
Un tipo especial de relación
de recurrencia no lineal (opcional)

Hasta ahora. nuestro estudio de las relaciones de recurrencia ha uatado de relaciones li-
nea1es con coeficientes constantes. El estudio de las relaciones de rccurrencia no lineales
y de las relaciones con coeficientes variables no es un tema que seguiremos con detalle,
excepto por una relación no lineal panicular que lleva por sí misma aJ mitodo de las
funciones generatrices.
Desarrollaremos el método en un problema de conteo de estructuras de datos. Antes de
eso, observemos que si /(x) = L,:Oa. ;xi es la función generatriz de Do. ª" DJ., •• • , entonces
[/(x))2 genera0oao.aoa1+a,ao.aoa: +a1a, + azao. ... ,Ooa. +a1a.-1 +aza.. _1+alao + · · · +
a,._1 a1 + a.,ao. ... , la convolución de la sucesión ao,, ai, a.2, .. . , consigo misma.

llianPlé>"1o4J En la sección 5.1 presentamos la idea de un diagrama de árbol. En general, un árl,o/ es un


grafo no dirigido, conexo y sin laz.os ni ciclos. Aquí analizaremos los árboles binarios con
rafa.
Capitulo 1O Relaciones de rec:urrencia

Figura 10.12

En ]a figura 10.12 vemos dos de estos árboles, donde el vértice con el cfrcu]o denola la
raíz. Estos árboles son binarios debido a que de cada vértice hay a lo más dos aristas
(llamadas ramas) que descienden (puesto que un árbol con raíz es un grafo dirigido) de
ese vértice.
En particular, estos árboles binarios con raíz son ordenados, en el sentido de que una
rama izquierda que desciende de un vértice se considera düerente de una rama derecha
que desciende del mismo vértice. Para el caso de tres vértices, la figura 10.13 muestra los
cinco posib]es árboles binarios ordenados con raíz. (Si no se prestara atención al orden,
entonces los últimos cuatro árl>oles con raíz serían la misma ~tructura.)

!\
(1)
I
(2)
{
(3)
\
(4) (5)
)
Figura 10.13

Nuestro objetivo es contar, paran 2: O, el nlhnerob,. de árboles binarios ordenados con


raíz de n vértices. Si conocemos los valores de b; para OS i S n. para obtener b,.+ 1 seleccier
narnos un vértice como la raíz y observamos, como en la figura 10.14, que las subestructuras
que descienden a la izquierda y la derecha de la raíz son árboles (binarios ordenados con
raíz) más pequeños, cuyo número total de vértices es n. Estos árboles más pequeños se
denominan subárboles de] árbol dado. Entre los posibles subárboles está el subárbol va-
cío, del que sólo hay uno ( = b0).
Consideremos ahora la forma en que pueden separarse los n vértices en estos dos
subárboles.
(l) O vértices a la izquierda, n vértices a la derecha. Esto produce un total de b0 b.
subestructuras por contar en b. + 1•
(2) 1 vénice a la izquierda. n - 1 vértices a la derecha. lo que produce b 1 b,. _1
árboles binarios ordenados con raíz, con n + l vértices.

(i + 1) i vértices a la izquierda, n- i a la derecha, para un total de b;b,._; hacia b• • 1•

(n + 1) n vértices a la izquierda y ninguno a la derecha, lo que contribuye conbft b0 de


los árboles.
10.5 Un tipo de relación de recurrencia no lineal (opcional) 50 1

Figura 10.14

Por lo tanto, para todo n ~ O,

b,..1 = bob,. + b1 b,._1 + b,,b,._2 + · · · + b,._,b1 + b11bo,

. .
L b•• ,x••1 = L (bob. + b 1 b._, + · · · + b._, b, + b. bo)x..1• (1)
11-0 ••O

Sea/(x) = L:.b. x" la función generatriz de b~ b1o b,, . .. Volvemos a escribir la ecuación
(!)como

(/(x)- b0)
...
= x L (b, b. + b,b._, + · · · + b.b,)x• =x[/(x)f.

Esto produce la ecuación cuadrática (en/tx))

x[f(x)J'-/(x) + 1 = O, y /(x)•[l~Yl-4x)/(2x).

Pero .JI-4.< = (1 -4x)112 =('¡')+(' f)(-4x) +(1f)(-4x)' +•••,donde el coeficiente de


x!', n ~ l. es

1/ 2)(- )" = (1/ 2)((1/ 2)- 1)((1/ 2)-2) · · · ((1/2) - n + 1) (- )"


( n 4 4
n!
= (- 1¡--' (1/ 2)(1/2)(3/2) · · · ((2n - 3)/ 2) (-4)"
n!
(-1)2"(1)(3) • • · (2n - 3)
n!
(-1)2"(n!)(l)(3) · · · (2n - 3)(2n -1)
(n !)(n!)(2n - 1)
(- 1)(2)(4) · · · (2n)(l)(3) · • · (2n -1) (- 1) (2n)
(2n - l)(n!)(n!) (2n - l ) n .
502 Capitulo 1O Relaciones de recurrencia

Enf(x), seleccionamos el radical negativo; en caso contrario, tendríamos vaJorcs nega-


tivos para b,.. Entonces

/(x) = 1._ [ 1 - [ 1 - ,.f-


2x
1
- ( 2n)x
,(2n-1) n
•]],
y b11 , el coeficiente de x" en/(x), es la mitad del coeficiente de x"" 1 en

¿• - 1-
..,(2n - l) n
(2n)
x•
.
Así,

b = ![ 1 ](2(n + l))= ~ = - 1- (2n)


• 2 2(n + 1) -1 n + 1 (n + l)!(n!) (n + 1) n ·

Los números b11 son los números de Catalan, en honor del matemático belga Eugene
Catalan (1814--1894), quien los usó para determinar el número de formas para colocar
paréntesis en la expresi6n x 1x1 x, . .. x•. Los primeros 11 números de Catalan son b0 = l,b1 =
1, b, = 2, b, = 5, b, = 14, b, = 42, b6 = 132, b, = 429, b, = ·1430, b, = 4862 y b., = 16796.

Continuamos ahora con una segunda aplicación de los números de Catalan, basada en
un ejemplo de Shimon Even. (Véase la página 86 de la referencia [5].)

Otra estructura de datos muy importante en las ciencias de la computación e s Japila. Esta
estructura permite guardar datos, con las siguientes restricciones:
1) Todas las inserciones se realizan en un extremo de la estructura, que se llama parte
superior de la pila, y el proceso de inserción se conoce como push (empujar).
2) Todas las eliminaciones de 1a pila (no vacía) también ocurren en la parte superior.
Llamamos a este proceso pop.
Puesto que el último (last) elemento insertado en (in) esta estructura es elprimer(first)
elemento que debe salir (out) de eUa, la pila se considera con frecuencia como una esttuc-
tura "last-in-first-out" (LIFO).
Los modelos intuitivos para este tipo de estructura incluyen una pila de fichas de póquer,
una pila de platos en una cafetería y la pila de descarte que se usa en algunos juegos de
cartas, como el gin-rummy. En los tres casos, sólo podemos ( 1) insertar un nuevo elemento
en la parte superior de la pila o (2) tomar (eliminar) el elemento de la parte superior de la
pila (no vacía).
Aquí usaremos esta estructura de datos, con sus procedimientos push y pop, para ayu-
darnos a permutar la lista (ordenada) l. 2, 3, . .. , n, paran E z•. El diagrama de la figura
10.15 muestra la forma en que cada entero de la entrada 1, 2, 3, •. . , ndebe empujarse hacia
la parte superior de la pila. en el orden dado. Sin embargo, podemos realizar una instruc-
ción pop para eliminar un elemento de la parte superior de la pila ( no vacía) en cualquier
instante. Pero una vez que un elemento es retirado de la pila. no puede ser devuelto a la
parte superior de la misma ni su valor puede ser utilizado de nue vo como entrada a añadir
a la piJa.
10.5 Un tipo de relación de rea,rrencía no lineal (opcionaO 503

'-/
-
- - -- -~Í:1.2.3.···,n ,,,,_

Figura 10.15

El proceso continll.a hasta que no hay elementos en la pila. Así. la sucesión ordenada de
elementos extraídos de la pila determina una permutación de l. 2. 3•. . .• n.
Si n = l. nuestra lista de entrada sólo consta del entero 1. lnscrtamos 1 en la parte
superior de la pila (vacía) y desputs lo sacamos. Esto produce la permutación 1 (poco
interesante).
Paran = 2. l<llelDOS dos permwaciooes posibles para 1, 2. y podemos obccner ambas por
medio de la pila.
1) Para obtener 1, 2,colocamos el I en la partesuperi~r de la pila(vacía) y desputs Jo
sacamos. ~ colocamos el 2 en la parle superior de la pila (vacía) y lo saca•
mos.
2) La permutación 2,1 se obtiene colocando el 1 en la parte superior de la pila (vacía)
y empujando luego el 2 hacia la parte superior de la pila (no vacía). Así, si sacamos
el 2 de la parte superior de la pila y Juego el 1, obtendremos 2. t.
En el caso n = 3, vemos que sólo podemos obtener cinco de las 3! =6 permutaciones
posibles de 1, 2, 3. Por ejemplo, la permutación 2. 3, J se obtiene mediante los pasos
siguientes.
• Se coloca el I en la parte superior de la pila (vacía).
• Se coloca el 2 en la parte superior de la pila (sobre el 1).
• Se saca el 2 de Ja pila.
• Se coloca el 3 en la parte superior de la pila (sobre el 1).
• Se saca el 3 de la pila.
• Se saca el I de la pila, y tsta queda vacía.
La razón por la que no podemos obtener todas las pcnnutaciones de 1, 2, 3 es que no
podemos generar la permutación 3, 1, 2 por medio de la pila, ya que, para que el 3 estt en
la primera posición de la permutación, debemos construir la pila inlroducicndo primero el
1 en la pila (vacía), lue go el 2 en la parle superior de la pila (sobre el 1) y, por último,
colocando el 3 en la parte superior de la pila (sobre el 2). Dcspuis de introducir el 3 en la
parte superior de la pila. lo tendremos como primer ownero de nuestra permutación. Pero
como el 2 está en la pan.e i¡uperior de la pila. no podemos sacar el 1 hasta haber sacado el
2. p0r lo que no podcnios generar la permutación 3. l . 2.
Sin=4, existen 12permutacionesdela lista(ordenada) 1, 2, 3,4quese pueden generar
por el mttodo de la pila. Las enumeramos en tas cuatro columnas de la tabla 10.4 de
acuerdo con la posición del 1 ca la permutación.
504 Capituk> 1O Relaciones de recurrencia

Tabla 10.4

1,2, 3,4 2 , 1, 3, 4 2,3, 1,4 2,3,4, 1


1, 2, 4,3 2, 1,4,3 3,2, 1,4 2, 4,3, 1
1,3, 2,4 3,2,4, 1
1,3,4,2 3, 4,2, 1
1,4, 3,2 4, 3,2,1

1) Existen cinco permutaciones con 1 en la primera posición, ya que después de meter


y sacar e] 1 de la pila. hay cinco formas de pennutar 2, 3, 4 por medio de la pila.
2) Cuando el l está en la segunda posición, el 2 debe estar en la primera. Esto se debe
a que metimos el l en la pila (vacía), después metimos el 2 sobre a y después
sacamos el 2 y luego el l. Hay dos permutaciones en la columna 2, puesto que 3, 4
pueden permutarse de dos formas.
3) En 13. columna 3, tenemos el 1 en la tercera posición. Observamos que los únicos
m1meros que lo pueden preceder son 2 y 3, que pueden permutarse en la pila (con el
1 en la pane inferior) de dos forma'i. Después se extrae el l y se mete el 4 en la parte
superior de la pila (vacía), para después re tirarlo.
4) En la dltima columna obtenemos cinco permutaciones. Después de colocar el l en la
parte superior de la pila (vacfa), hay cinco formas de permutar 2 , 3, 4 mediante la pila
(con el 1 en la pane inferior). Después se extrae el 1 de la pila para completar la per-
mutac ión.

Con base en estas observaciones, para 1 Si$ 4 , sea a; el número de formas de permutar
los enteros 1, 2, 3, . .. , i (o cualquier lista de i enteros consecutivos) por medio de la pila.
Además, definimos Go = 1, puesto que so1amente hay una forma de no permutar nada, por
medio de la pila. Entonces

donde
a) Cada sumando a1 ª* satisface j + k = 3.
b ) El subíndicej indica que existenj enteros a la izquierda del 1 en la permutación; en
particular, paraj ¿: 1, éstos son los enteros de 2 aj+ 1, inclusive.
e) El subíndice k indica que hay k enteros a la derecha del 1 en la permutación; para k
2: 1, éstos son los enteros de 4 - (k - I ) a 4.
Este problema de permutación puede generaliz.arse a cualquier 1l EN. de modo que

a.u 1= 0oa11 + ª 1 ª 11- 1 + ª1ª11- 2 + · · · + a...- 1a1+ a11 ao,


con ao = 1. De nuesuo ejemplo anterior, sabemos que

Continuaremos esca sección con otro ejemplo en que surgen los números de Catalan.
Históricame nte. éste es el primer caso conocido donde apareció esta sucesión de números.
10.5 Un tipo de rel.>ó6n de recurrencia no lineal (opcional) 505

1... Este ejemplo trata de UD problema propuesto por Leonard Euler. El problema analiza un
I!:::::!:::::..::.::=:;, pollgono convexo dado de ~ 3) lados; es decir, UD polígono de n lados que satisface la
siguiente propiedad: para cualquier par de puntos P,,P,del interior del pol/gooo, el seg-
mento de recta que une aP1 con P2 tambi~n está dentro del interior de} polígono. Dado un
polf¡ooo convexo de n lados. Eulcr queña contar el nl.1mero de formas en que podía
triangularse el interior del pol/gono (subdividirlo en triángulos) trazando diagonales que
no se intcrsccaran. Denotaremos el níimero de estas triangulaciones como r•.
En las partes (a) y (b) de la figura l0.16 encontramos un triángulo y dos cuadriláteros;
tstos áltimos han sido triangulados. La parte (e) de la figura muestra las cinco formas en
que podemos uiangular un pentágono convexo mediante diagonales que no se interscquen.
(En cada caso. tenemos dos diagonales que no se intersccan.) En consecuencia. r, :: 1, t4 =
2 y 11 = S. por lo que tenemos otra situación donde aparecen los números de Catalan. Sin
embargo, esta vez no usaremos una relación de recuncnc~ sino que estableceremos una
correspondencia uno a uno entre las ttiangulaciones del polfgono convexo (que queremos
contar) y los árboles binarios ordenados con raíz contados en el ejemplo 10.36.
Para el cason = 5, consideremos el pcntágooo convexo triangulado de la parte (a) de la
figura 10.17. Asociamos la raíz r de un árbol binario ordenado con nlz con el lado del
pentágono cuyos extremos están etiquetados como 1 y 2; nos referiremos a este lado como
el lado { 1, 2}.Ahora rotamos el lado {1, 2) hacia la diagonlll {5, 2), la primera (y única)
diagonal que podemos alcanzar mediante la rotación del lado { 1, 2}. Esta rotación en
sentido contrario aJ de las manecillas del reloj se designa en un árbol binario ordenado con
raíz mediante una arista trazada desde la raíz hasta el v~rtice u 1 al lado derecho de la raíz.
El vlnice u , corresponde con la diagonal del pentágono {5, 2} y el rosul!Jldo de la rotación
aparece en la figura l0. l 7(b). Si continuamos de esta forma, la roación en sentido contra-
rio al de las manecillas, de la diagonal {5, 2) hacia la diagonal {4, 2), corresponde a la .

1
2 2

<•>
D
3 2
(b)
LSJ
4 3
~
4 3

1 2 1 2 1 2 1 2 1 2

fj),@. f;J>. ~ t{),


4 4 4 4 4
(e)

Figura 10.1 6
capítulo l O Relaciones de recurrencia

(al (b)
v,
?/-\
(e)
.,

(d) (el (0
.,

/sJ)
(g)
4

(h)
\-! t/-Á
4

(i)
4

Figura 10.17

arista (Ut, u,} del árbol binarioordeoado con raíz de la parte (c)de la figura. En este caso,
el vátioc u, coc=ponde con la diagonal ( 4, 2}.
La situación de las partes (d) y (e) de la figura 10.17 muestra de nuevo la corresponden-
cia entre

1) el lado ( 1, 2} del pcnttgono y la raíz r;


2) la diagonal ( S, 2 J y el vtrtioc " • a la derocha de la raíz r, y
3) la rotación en contra del sentido de las manecillas de ( 1, 2} hacia (S. 2} y la arista
(r, " •} del árbol.
Sin embargo, cuando rotamos la diagonal (S, 2} hacia la diagonal (S, 3), obtenemos una
rotación en el sentido de las manecillas, como se muestra en la figura l0.17(f). Esta rota-
ción coc=ponde a la arista {Ut, u,} del árbol, orientada de modo que el vtnice u, quede
a la izquierda del v~nice '0 1•
P<>r 6ltimo, para el pcnttgono lriangulado de lapa& (g) de la figura 10. 17 tenemos dos
opciones para la primera rotación. Cuando rotamos la arista ( 1, 2} del pcnttgono hay dos
posibilidades: (1) una rotación en el sentido de las manecillas hacia la diagonal { 1, 4},
10.5 Un t ipo de relación de rtocurreocia no línNI (opcionaO 507

como se muestra en la figura I0.17(h); o (2) una rotación en sentido contrario al de las
manecillas hacia la diagonal {2, 4}. En este ca.so, primero aplicamos la rOUK:ión en sentido
de las maneeiUas y obtenemos el resultado de la parte (h) de la figura 10. 17. A continua-
ción, rotamos la arista { 1, 2) del pentigono hacia la diagonal {2, 4}, por medio de una
rotación en el sentido de las manecillas, como se muestra en la figura 10. I 7(i), donde
también encontramos el árbol binario ordenado con raíz correspondiente.
Este análisis muestra que para cualquier triangulación del pentágono convexo obtene-
mos un árbol binario ordenado con raíz tal que

1) la arista(!, 2} del pentigooocorresponde ala ralz rdel árbol;


2) cada diagonaJ que no se interseque corresponde con un vértice del árbol;
3) una rotación en el sentido de las manecillas de la arista { 1, 2} hacia una diagonal, o
de una diagonal hacia oua. corre.ipOnde con una arista del árbol en la que el vértice
inferior queda a la izquierda del vértice superior, y
4) una rotación en sentido contrario al de las manccilJas, de la arista { 1, 2} hacia una
diagonal, o de una diagonal hacia otra, corresponde con una arista del árbol en la
que el vértice superior está a la izquierda del vértice inferior.
Siguiendo estas ideas, encontramos las otras dos triangulaciones del pentágono con sus ár-
boles binarios ordenados con raíz correspondientes en la.figura 10.18. En las partes (a) y
(b) de la figura 10.19, se muestra este tipo de correspondencia para una de las 14 triangu-
laciones de un beúgono coovexo. Las pams (e) y (d) de la figura 10.19 muestran cómo
corresponde una de las 42 triangulaciones de un heptágono convexo con un árbol binario
ordenado con raíz de cinco vértices.
Resumimos y generalizamos el análisis anterior de la fonna siguiente: un polígono
convexo den+ 2 lados puede triangularse mediante n - 1 diagonales que no se intersecan,
de b. formas, donde b. es el n~simo número de Catalan.

~
1 2

.s -<
5 3
'

v,

Figura 1O.18
--,
l8 capitulo 1O Relaciones de rea,rrencia

1 2 1 2
'

•~
(a)
s
' •~
(b)
4
'
---- v,
(:.
1 2 1 2
'

,~

(e)
s
,
.!f\ -/·
(d)
s
v.
v,
i

F"tgura 10.19

Nuestro último ejemplo de esta sección es parecido al ejemplo 10.14. De nuevo, vemos
que debemos evitar obtener un resultado general sin un argumento general. independien•
temente de lo que puedan sugerir unos cuantos casos especiales.

Aquí partimos den objetos distintos y. paran~ 1, los distribuimos en a lo sumon rrcipitn•
tes idénticos. sin que haya más de tres objetos en cada recipiente y sin preocupamos por el
orden de los objetos dentro de los recipientes. Sea a,. el número de estas distribuciones y, a
panir de la figura 10.20, vemos que
-.-1, 01 = 1, a2=2, a,=5, y a, =14.
Parece que tuviiramos los primeros cinco tlrm.inos de la sucesión de nWlleros de Catalan;
sin embargo, el pallón se rompe y vemos que
a,= 46;, 42 (el sexto número de Callllao),
a,= lti§J' 132 (el séptimo número de Catalan), y
a,• 6521' 429 (el octavo número de Catalan).
(Las distribuciones de este ejemplo fueron estudiadas por F. L. Miksa, L. Mosec y M. Wyman
en la referencia (20).)

Otros ejemplos relacionados con los números de Catalan aparecen en la bibliografía del
capitulo.
10.5 Un tipo d• relación do recurrencia no lineal (opcionaO 509

u w wlJJ
(n - O) <n- 1) <n- 2)
wWj WJ WJ lJJJ
(n • 3)

lill ~ ltlJ l[Ll liliJ ~ ~ ~ AB AB ABC

~ 1A1D1~11s1c1~1 lslJ I l,1°1!1 LIJJJ


(n ~ 4)

Figura 10.20

EJERCICIOS 10.5 1. Para'°' úbo1C5 binarios ordenados con r>ltdel ejemplo 10.36, cakulcb,y trace las estructu-
ras de cuatro vtnices.
2. Verifique que para lOdo n ie: O,

H~~ 2
1)(~ :1 ) ·G: 1)( ~ )-

3. Muestre que pa,a todo n ao2, (2nn- l)-(2nn -- 21) - -(n '+-1)( 2n).
n

4 . ¿Cuáles de las siguientes permutaciones de t. 2. 3. 4 , s. 6, 7 , 8 pueden obtenerse usando una


pila (del ejemplo 10.37)?
a) 4,2,3, 1,5,6,7,8 e) 4,5,3,2,1,8,6,7
b) 5,4,3,6,2, 1, 8, 7 d) 3,4,2, 1, 7,6,8,5
5. Suponpmosquclosenteros t. 2. 3, 4, 5, 6, 7,Sse permutan usando la pila del ejemplo 10.37.
(a) ¿Cuántas permutaciones $00 posibles? (b) ¿Cuántas permutaciones tienen el 1 en la cuarta
posición y el Sen la octava? (e) ¿Cuánw permutaciODCS tienen el I en la sexta posición? (d)
¿CUántu pmnutaciones comienzan con 321?
6. Para un polígono convexo den ~ 3 lados, sea ,. el número de formas en que se puede triangular
el interior del polígono medianae diagonales que no se intenequen (como en el ejemplo 10.38).
a) Defina t1 • 1 y verifique que,•• , =12', + 1,1, _ 1 + · · · + ,,_1 + t,h-
b) Exp<e<e t, co función den.
7. Encuentre el árbol binario ordenado con níz., COCTCSpondicnte a cada una de las triangulaciones
(de un he~gono convexo) de la figura 10.21.

8. Paran 2!:0, b,.. = (n !


1
)(~) es el n~imo número de Catalan.

a) Muesttequcparatodon 2 0,b•• 1 = l(ln+ l) b..


(n+l)
b) Use el resultado de la pane (a) para escribir un programa (o desarrollar un algoritmo) que
calcule los primeros 15 nllmcros de Catalan.
11 1
capítuk> 1O Relaciones de recurrencia

2 2 2

'®.• '©• ':t¡;),


(a)
5
(b)
5
(e)
5

Figura 10.21

9. Paran ~ O. distribuya 2n puntos de manera uniforme sobre u.na circunferencia y n~relos en


fonna cíclica con los enteros 1, 2. 3, .. . • 1n. Sea a. el m1mero de formas en que estos 2n puntos
pueden emparejarse como n cadenas sin que &tas se intcrscqucn. (El cuo den • 3 se muC$tr3
en la figura 10.22.) Encuentre y rC5Uclva una relación de recurrcneia ¡,vaª•· n;?: O.

103 103 18/ '([) '0 3


6
2

5
4 6
2

s
4 6
2

5
4 6
2

5
4 6
2

5
4

Figura 10.22

10. En la figura 10.23 teocmosdosdclascincoformasenquepodcmostriangularel interiotdc un


pcn~ono convexo con diagonales que no se interscquen. En este caso hemos etiquetado cua•
tto de los lados coo las lcuas a. b, e, d, al i¡ual que los cinco vatices. En la parte (i) usamos las
etiquetas de los lados a y b para obtener la etiqueta ab para la diagonal que une los v&tices 2
y 4. Esto se debe a que la diagonal Olamadaab),junto con los lados a y b, nos proporciona uno
de los tri.ingulos interiores de la triangulación del pen(jgono convexo. Entonces. la diagonal
ah y el lado e dan lugar a la ct:iqueta ob(e) de la diagooal determinada por los ~ces 2 y S, y
los lados ob, e y (ob)c proporcionan un se¡undo triángulo interior para esta triangulación. Si
.se¡uimos de esta forma, etiquetaremos la arista base que une los v&tice I y 2 con ((ab)c:)d, una
de las cinco formas en que podemos introducir los paréntesis para obtena los tres productos
(de dos en dos números) nccc.sarios para calcular abcd. La triangulación de la parte (ii) de la
figura corresponde a.l producto con partntesis (ab')(.cd).
a) Determine cJ producto con parfntesis de a. b, e, d pan las otras tres triangulaciones del
pentágono convexo.
b ) Encuentre el p!Oductocoo paróltesi.s para cada uno de los hexágonos convexos triangulados
de las panes ( üi) y (iv) de la figun 10.23.
(De la pane (a). vemos que hay cinco formas de poner partntcsis a la expresión abcd. La pane
(b) mucsuadosde las 14 fonnas en que podemos inuoducirparmlOSis ponlacx¡nsiónabale. En
general, hay *f:jformas de poner entre paréntesis laCA:presión.r1x2.r1 • • • x._1 x.x• • 1• Tratan--
do de re.solver este problema, Eu¡cne Catalan descubrió la sucesión que lleva ,u nombre.]
10.6 Algoritmos divide y vencerás (opcional) 511

(-/5)
4 c 5 ~ •csd
3 5
3 6 3 6
d
• •
2 c.t,Xcd) 1 2 2
(í) (n) (iil') (iv)

F1gur.110.23

10.6
Algoritmos divide y vencerás
(opcional)t

Uno de los tipos de algoritmo eficiente, de amplio uso e Ímportanc:ia. es el basado en el


enfoque divide y vencerás. En este caso, la estrategia en general es resolver un problema
dado de iamallo n (n E z•¡ de la forma siguiente:
1) Se resuelve el problema directamente para un valor pcquefio den, en forma directa
(esto proporciona una condición inicial para la relación de rccunencia resultante).
2) Se descompone el problema general de tamaño n en a problemas pequefios, del
mismo tipo y (aproximadamente)del mismo tamaño, ya seaí•/blo Ln/b.h donde
a. b E Z •, 1 S a < n y 1 < b < n.
Luego resolvemos los a problemas más pequci\os y usamos sus soluciones para construir
una solución del problema original de tamaño n. Nos interesan particularmente los casos
en que n es una potencia de b y b = 2.
Estudiaremos los aJgoritmos "divide y vencerás", donde
1) El tiempo para resolver el problema inicial de tamaño ns 1 es una constante e~ O, y
2) El tiempo para separar el problema dado de tamaño n en a problemas similares pero
más pequeños, junto con el tiempo necesario para combinar las soluciones de estos
problemas pequeños y obtener una so1ución del problema dado es h(n), una función
den.

t EJ material de esta 5CCcióa puede omitirse sin perder La continuidad. s«i utilizado en. la secc:ión 12.3
pan dctumimr La fvaci6o de oompltjjdad co tiempo del al¡oritmo de oróent.nuento por fusida. Sin tmbar¡o,
el resultado tambim se obtcodn. comD un caso particular de ate aJ¡oritmo medimtt otro ~ que oo usad
mm:riaJ desanollado m esta seccioo.
* Pin, cada X E R. ~ que r.il
cbocad uc/to de z y l.i:J
d suelo de: .r. o la pame11itende .r.
donde

•>l•J • f•l••<.!""''e z
b) L.iJes el encero inmediato a La izqllierda de .i. para x e R - Z
e) rl
.i CI el en&cro inmediato. la derecha de: .i, para X e R - z.
Capítulo 1O Relaciones de re<UlfflKia

Nos interesa en este momento la función de complejidad en tiempo /(n) para estos
algoritmos. En consecuencia. usaremos aquí la notación/(n), en vez de la notación a~ con
subíndices utilizada en las secciones anteriores de este capftuJo.
Las condiciones que hemos establecido nos llevan a la siguiente relación de recwrcncia.

/(1) = e,
f(n) = af(n/b) + h(n), paran = b', k >'1.
Observamos que el dominio de/es {l, b,li', li'•... ) = !b'li e NJ e z·.
En nuestro primer resultado. la solución de esta relación de recurrencia se obtiene del
caso en que h(n) es la constante c.

oOREMA 10.1 Sean a. b. e E z·. b;, 2 y sea/: z• ➔ R. Si


/(l)•c, Y
f(n) • af(n/b) + e, para n =b',k ;,, ! ,

entonces para todo n = l, b, Ir, b', . .. ,

1) /(n) = c(log.n + I), donde a = I


c(an.,.'-1)
2) /(n)= ~ - - ~ , dondc aae2
a-1
Demostración: Para k ~ l y n =11-, escribimos el siguiente sistema de k ecuaciones. [Si se
empieza con la segunda ecuación, obtenemos cada una a partir de la anterior, (i) reempla-
zando cada aparición den en la ecuación anterior por nlb y (ii) multiplicando la ecuación
resultante en (i) por a.)
f(n) = af(n/ b) + e
af(n /b) = a' f(n /b') + a,;
• ' f(n l b') = •'f(n /b') + a'c
a'f(n l b') = a'f(n /b') + a'c

a'-'f(n /b'-') = a'- 1/(n /b'-1) + a'-• c


••-•t(nlb'- 1) = a'f(nlb') + a'-• e

Vemos que cada uno de los términosa/(n/b).a'/(nlli'),a'/(nlli'), ... ,a'- 1/(nJ/,'- 1) aparece
una vez como sumando en ambos lados de e.sw ecuaciones. Por lo tanto. al sumar ambos
lados de las k ecuaciones y cancelar estos sumandos comunes, obtenemos

f(n) = •'f(n /b') +[e+ ac + a'c +···+••-•e].

Como n = b' y /(1) = e, tenemos


f(n) = •'/(!) + c[I +a + a'+ •••+ a•-•¡
= c(l +a+ a2 + • • • + a.t-i + a•].
10.6 Algoritmos divide y.enceras (opcionaO 513

1) S i a= 1, entonces/(n) = =
c(k + 1). Pero n b'<c> lo¡,n =k,
por loque /(n) = c(log.n + 1), paran E lb'li E N f.
2) Cuandoa 2:2, entooces/(n) = =
c(l - a'•' ) c(a••• -1), a partir de la identidad 4 de
l- a a- 1
la tabla 9.2. Ahora, n = b'<c> lo¡,n = k, po< lo que
a' • a...,•= (b...,")...," = (b...,.)...,' =. ....,

c(an...,• - 1)
/(n) (a - l) , para n E {b'ji E N}.

a) Sa/: z• ➔ R, donde

/(1)=3, y
/(n) • /(n / 2) + 3, para n = 2', k E z•.

Por la parte 1 del teorema 10. 1, con e = 3, b = 2 y a = 1, se sigue que / (n) =


3(1og.n + 1), paran E ( 1, 2, 4, 8, 16, ... }.
b) Suponga que g: z• ➔ R, con

g(1)•7, y
g(n)•4g(n / 3) + 7, paran=3', k Ez·.

Entonces, con e• 7, b = 3 ya• 4, la pane 2 del teorema 10. l implicaqueg(n) =


(7/3)(4•'°"' - 1), si n E (1, 3, 9, 27, 81, . . . }.

Si analizamos el teorema 10.1, debemos observar, por desgracia. que aunque conoce,.
mos/para n E ( 1, b, b', .. . }, no podemos decir nada del valor de/para los enteros en z•
- { 1,b, Ir, .. .}.Así.en estc momcnto nopodemosanalizar/comouna funcióndecomple--
jidad en tiempo. Para superar esto, generalizaremos la definición 5.23, donde presentamos
la idea de dominación de una función.

Definición 10. 1 Sean/, g: z • ➔ R y S un subconjunto infinitodc Z•. Dccimosque g dcmínaa/enS (09ue


fes dominada por g en S ) si existen constantes m E R• y k E z• tales que IJC•) 1 S
m Ig(n) 1 para todo n ES, doode n 2: k.
En estas condiciones. tambi<!n decirnos que/ E O(g) en S.

Sa/: z· ➔ R definida po<

f(n)• n , paran E {1, 3,5, 7, ...} • S1,


f(n) = n', para n E {2, 4, 6, 8, ...} = S, .
i14 capitulo 1O Relaciones de recurrencia

Entonces/ E O(n) en S, y/ E O(n') en S,. Sin embargo, no podemos concluir que/ E


O(n). -

lllll'8 Del ejemplo 10.40 y la definición 10.1 tenemos que

a) /EO(log,n) en. {2'1kEN}


b) g E O(n""') en. {3'lk EN}

Con la definición 10.1, podemos considerar ahora los siguientes corolarios del teon:ma
10.1. El primero es una generalización de los resultados paniculares dados en el ejemplo
10.42.

:OROLARIO 10.2 Seana,b.c E z•conb:?2y/: Z• ➔ R.Si


/(!)=e, Y
/(n) = af(n/b} + e, paran= b', k >: 1,
entonces
1) /E O(log.n) en \b'lk EN}, cuando a= 1, y
2) /E O(n""'') en {b'lk EN}, cuando a 2: 2.
Demostración: Se deja como ejercicio a1 lector.

El segundo corolario cambia los signos de igualdad del teon:ma 10.1 por desigualda-
des. Como resultado, el codominio de/debe restringirse de R a R• U {O}.

:OROLARIO 10.1 Sean a, b, e E z·conb ;a 2 y/: z · ➔ R· U {O}. Si


/(!}Se, y
f(n)saf(n/b)+c, paran=!>', kaa:1,
entonces. para tOOo n = I. b, Ir, Ir, . .
1) /E O(log.n),dondea = 1, y
2) fe O(n""'), dondea 2:2.
Demostración: Consideremos la función g: z• ➔ R• U {O}. donde
g(l) =e, y
g(n} = ag(n/b) + e, paran e{!, b,b', .. .).
Por el corolario 10.1,
gEO(log.n) en {b'lkEN}, cuando a= 1, y
geO(n""'} en {b'lkEN}, cuandoa>:2.
10.6 Algoritmos divideyvenceras (opoonaO 515

Afumamos que/(n) S g(n) para todo n E {I, b, b', . .. ]. Para demostrar nuestra afir-
mación, baremos una inducción sobn: k. dondcn =b'. Si k= O, entonces n z 11° = 1 y /(1) S
e = g(l), de modo que el resultado es cieno para este primer caso. Si suponemos que el
=
resultado es cierto para algún t E N, tenemos que f(n) = f(b') S g(b') g(n) para n /f. =
Entonces, para k. = t + l y n = 11 =b'· 1. tenemos que

/(n) =f(b'• 1) s af(b,.'lb) + e= af(b') + es ag(b') +e = g(b'"') =g(n ).

Perlo tanto, poc el principio de inducción matemática.f(n)Sg(n)para 1odon E [ 1,b,b', .. . ].


I
En consecuencia,/ E O(g) en {11 k E N } y el corolario se sigue de nuestra afinnación
anterior sobre g.

Hasta este momento, nuestro estudio de los a1goritmos ..divide y vencerás" ha sido
principalmente teórico. Ya es tiempo de dar un ejemplo donde puedan aplicarse estas ideas.
El siguiente resultado confirma uno de nuestros anteriores ejemplos.

Paran al, 2, 4, 8. 16, . .. , sea/(n)cl número de comparaciones necesarias para encontrar


los eleme ntos múimo y mínimo en un conjunto S C R , donde IS 1== n y se usa el procc•
dimiento del ejemplo 10.26.
Si n = 1, entonces los elementos máximo y mínimo son el mismo eleme nto. Por lo
tanto. no se necesitan comparaciones y/( 1) = O.
Si n > 1[ entonces n = 2' para algún k E z • y separamos (dividimos) S como S, U S,,
donde IS, = S, =
1 1 = n/2 2' - '. Se necesitan / (n/2) comparaciones para encontrar el
máximo M1 y el mínimom; paracadaconjuntoS,. i = 1. 2. Paran~ 4, si conocemosm1, M1,
m 1 y M 1 , podemos compararm1 conm1 y M 1 con M: para detcnninar los elementos máximo
y mínimo de S. Por lo tanto,
f(n) = 2f(n / 2) + 1, sin =-2, y
f(n) = 2f(n /2) +2, sin• 4,8, 16, .. .
Por desgracia. estos resultados no son la hipótesis del teorema 10.1. Sin embargo, si
cambiamos nuestras ecuaciooes por las dcsiguaJdades
f(l) S2
/(n) s 2f(n /2) + 2, si n = 2·\ k 2: 1,
entonces. por el corolario 10.2. la función de complejidad en tiempo /(n), dada por el
número de compar:iciones hechas en este proceso recursivo. satisface/ E O(d"':l) = O(n).
para todo n = l. 2, 4, 8, . ..
Podemos analizar la relación entre tstc y el ejemplo 10.26 con más detalle. Por ese
resultado anterior, sabemos que si IS 1= n = it, k ~ 1, entonces el número de comparacio-
nes/(n) necesarias (en el proceso dado) para encontrar los elementos máx imo y mínimo
en Ses (3/2X2') - 2. (Nota: Nuestra proposición reemplaza la variable n del ejemplo 10.26
por la variable k.)
Como n =2 1, vemos que podemos escribir

f( l) =O
f(n) = f(2') = (3/2)(2') - 2 = (3/ 2)n - 2, paran = 2. 4, 8, 16 ...
Capítulo l O Relaciones de recurrencia

Por lo tanto,/ E O(n) para n E {2' 1k E N), como lo obtuvimos antes mediante el
corolario 10.2.

Todos nuestros resultados piden que n =11 para algún k E N, de modo que es natural
preguntarse si podemos hacer algo cuando permitimos que n sea cualquier entero positivo.
Para esto. presentamos la siguiente idea.

finición 10.2 Una función/: z· ➔ R· U {O} se denom.inamonótona creciente si para todos m,, n E z·,
m < n => fl.m) Sfl.n).

Esto nos permite considerar resultados para cualquier n E z+, en ciertas circunstancias.

)REMA 10.2 Sea/: z• ➔ R• U (O} monótona creciente y seag: z· ➔ R. Parab E z·, b2:2, suponga-
mos que/E O(g) para todon E S= (b'lk E NJ. En estas condiciones,

a) Sig E O(logn),entonces/E O(logn).


b) Si g E O(n log n), entonces/ E O(n log n).
<) Si g E O(n'), entonces/E O(n'), parar E R• U {O).

Demostr•ción: Demostraremos las partes (a) y (e) y dejaremos la parte (b) para los ejerci-
cios de la sección. Antes de comenzar, observemos que la base para los logaritmos de las
partes (a) y (b) es cualquier n6mero real positivo mayor que l.

a) Como f E O(g) en S y g E O(log n), al menos tenemos/ E O(log n) en S. Por lo


tanto, por la definición 10.1, existen constantes m E R.. ys E z· la.les que/(n) -=
lt(n) 1 S m I log n 1 = m log n para todo n E S, n 2: s. Necesitamos encontrar
constantes ME R· y,, E z· tales que/(n) S Mlog n para todo n 2:s., no sólo para
nE S.
Sear E z..,dondes <bt < t S: bhl (y log s::?; 1). Como/es monótona creciente y
positiva.

/(r) s/(b'.') sm log(b..')


= m(log(b') + logb]
= m log(b') + m log b
<m log(b') + m logb log(b')
=m(l + logb) log(b')
<m(l+logb) logr.

Así que si M = m(l + 1og b) y s1 = bt + t, vemos que para todo , E z·, sir~ Si,
entonces/(r) < M log r. Porlo tanto,/(r) S M log r y f E O(log n).
10.6 Algoritmos divide y vencerás (opcional) 517

e) Según las hipótesis de esta parte del teorema. existen constantes m E R♦• s E z•
tales que/(n)Sm(n') para todon EScuandon~s. Así. sir E z •cons<ll <tSIJl:+1,
entonces

f(t) -sf(b"' ) -s m(b.. 1)' =m [b'..'"]


= mb'(b')'
< m b't'.

Si M;;;; mb'y s1 ;;;;JI+ l . se sigue que para todor E z•, si t~s., entonces/(r) <Mt'
y / E O(n').

Ahora utiliz.aremos el resultado del teorema 10.2 para determinar la función de com-
plejidad en tiempo/ (n) para un algoritmo de búsqueda llamado búsqueda binaria.
En el ejemplo 5.71 analizamos un algoritmo en el que buscábamos un entero particular
Key en una listaA[IJ, A[2], A[3], .. . , A(n]. En ese ejemplo, los elementos de la lista no
tenían un orden particular, por lo que simplemente comparamos el valor de Key con el de
los elementos de la listaA[l ], A[2],A[3], . . . ,A(n]. Pero esto no seña eficiente si supiéra-
mos que A[I] < A(2] < A[3] < • • · < A(n]. (Después de todo, uno no busca el número
telefónico de una persona en la guía empezando en la primera página y analizando cada
nombre. El orden alfabético de los apellidos se usa para acelerar el proceso de búsqueda.)
Veamos un ejemplo panicular.

Consideremos la listaA[IJ, A(2],A[3], . . . ,A[?] de enteros, dondeA(l] = 2,A[2] = 4,A(3] =


5, A[4] = 7, A[5] = 10, A(6] = 17 y A[7] =20; además, sea Key =9. Revisamos la lista de
la forma siguiente:

1) Comparamos Key con el elemento del centro de la lista.. que en este caso es A{4];;;;
7. Como Key >A[4], nos concentramos en los demás elementos de la sublistaA[S],
A(6], A[7].
2) Ahora comparamos Key con e l elemento centra!A(6]. Como Key = 9 < 17 =A[6],
pasamos ahora a la sublista (de A[5],A(6], A[7]) que consta de los elementos meno-
res que A[6], que en este caso solamente es el elemento A[5].
3) Al comparar Key con A(5), vemos que Key ,'A(5], por lo que Key no está presente
en la listadadaA[I], A[2], A[3], .. . , A(7].

A partir de los resultados del ejemplo 10.44, hacemos las siguientes observaciones para
una lista general (ordenada) de enteros (o númerosreales). Sea A[IJ,A(2],A(3], ... ,A[n]
la lista dada y sea Key el entero (o número real) que estamos buscando.A diferencia de la
lista del ejemplo 5.7 1, aquí tenemos que

A(l] <A(2] < A[3] < · · · <A(n].

1) Primero comparamos el valor de Key con el elemento de 1a lista en o cerca dd


centro. Este elemento es A((n + 1)12] paran impar y A(n/2] paran par.
18 Capitulo 1O Relaciones de recurrencia

L
Ya sean par o impar, el elemento de la lista con el subíndice m = (n + 1)/2 es el J
elemento del centro, o cercano al centrO. Observe que 1 es el valor del límite infe.
rior de los subíndices de la lista y n es el límite superior.
2) Si Kcy es igual a A[m], hemos terminado. Si no, entonces
a) si Key es mayor queA[m], buscamos (mediante este proceso de división) en Ja
sublistaA[m + l],A[m+ 2], ... ,A[n];
b) si Key es menorqueA[m), entonces aplicamos el proceso de división y busca-
mos en la sublistaA[IJ,A[2], ... ,A[m- !].
Usamos las observaciones anteriores en el desarrollo del segmento de programa en
Pascal de la figura 10.24. En es,e caso, la entrada es una lista ordenada A[!], A[2], A[3], .. . ,
A[n] de enteros, o números rca1es., en orden ascendente. La lista y el valor del entero n se
dan antes en e l programa.junto con el va1orde la variableKey. Si los elementos de la lista
son enteros (números reaJes), entonces Key debe ser un entero (número real). Las varia-
bles Wwer y up~r s:on variables enteras que se usan para guardar los límites inferior y
superior, respectivamente. de la lista o sublista en la que se realiza la búsqueda. La varia-
ble entera cenler guarda el índice del elemento de la lista o de la sublista en o cerca del
centro de la lista o sublista. En general, center = L<Lower + upper) 12J Por último, la varia-
ble booleanaflag toma el valor falso (si no se encuentra Key) o verdadero (si se encuentra
Key).

Begin

flag : = false;
lower : = l :
upper : = n ;

While { lower <= upper) and (flag • false ) do


Begin
center := (lower+upper ) Oiv2;
If Key = A[center] then
nag : = true
Else
If Key < A(center] then
upper : = center - 1 ,{Se busca en la sublista infe rior.}
Else l ower : = ceriter + 1 {Se b usca en la sublista superior.}
End;

It flag = false then


Wri t e l n ( ' El valor ', Key, • no está en la lista. ' )
Else Writeln ( ' El val o r ', Key, ' se localiza en el elemento
cent~ ,·. ·)
End..
Figura 10.24

Queremos medir la complejidad del peor caso para el aJgoritmo implementado en la


figura 10.24. En este ej emplo, /(n) contará el máximo de comparaciones (entre Key y
A[centerl) necesarias para detenninar si el número dado Key aparece en la lista ordenada
A[I]. A[2], A[3], ... , A[n].
10.6 Algoritmos divide yvencerás (opcional) 519

• Paran= 1, Key se compara conA[l] y f(l) = l.


• Sin= 2, en el peor de los casos se compara Key conA[l] y después conA[2], por Jo
quef(2) = 2.
• Sí n = 3,f(3) = 2 (en el peor de los casos).
• Sin= 4, el peor caso ocurre cuando Key se compara primero con A(2] y después se
hace una bósqueda binaria en A[3], A[4). Esta última búsqueda requiere (en el peor de
los casos)f(2) comparaciones. Así.f(4) = 1 + /(2) = 3.
En este momento vemos quef(l) S/(2) $/(3) S/(4) y conjetwamos que/es una fun-
ción monótona creciente. Para verificar esto, usaremos el principio de inducción matemá-
tica en su forma alternativa. Supondremos que para todos i, j E { 1, 2, 3, .. . , n}, i < j ⇒
f(i) S.f(j). Consideremos ahora el entero n + l. Tenemos que analizar dos casos.

1) n + l es impar: en este caso caso escribimos n = 2k y n + 1 - 2k + l, para algún k E


z•. En el peor de los caso,f(n + 1) =/(2k+ 1) = 1 + f(k), donde 1 cuenta la compa-
ración de Key con A[k + l] y /(k) cuenta el máximo de comparaciones necesarias
para una búsqueda binaria en la sublistaA[l], A[2], ... ,A[k] oen lasublistaA[k+ 2],
A[k + 3], . .. , A[2k + l].
Entoncesf(n) =/(2k) = 1 + máx[f(k-1),f(k)). Comok-1, k <n, por la hipóte-
sis de inducción tenemos quef(k-1) Sf(k), de modo quef(n) = 1 + f(k) =f(n + 1).
2) n + 1 es par: en este caso, n + 1 = 2r, para algún r E z+ y en el peor de los casos,
f(n + I)= 1 +máx{f(r-1),f(r)) = 1 +f(r), por la hipótesis de inducción. Por lo tanto,

f(n) = f(2r - 1) = 1 + f(r - 1) s 1 + f(r) = f(n + 1).

En consecuencia. la función es monótona creciente.


Ahora vamos a determinar la complejidad en tiempo del peor caso para el algoritmo de
bilsqueda binaria. utilizando la funciónf(n). Como

f(l) = 1, y
f(n)=f(n/2)+1, paran=2', k2cl ,

se sigue del teorema 10.l (con a= l , b ~ 2 y e - 1) que

f(n) = log,n + !, /EO(log,n) paranE{l, 2,4,8, . ..}.

Pero si/es monótona creciente. obtenemos del teorema 10.2 que f E O(log2n) (para todo
n E z·). En consecuencia, Ja bllsqueda binaria es un algoritmo O(log2n), mientras que
el algoritmo del ejemplo 5.71 es O(n). Por lo tanto, cuando n crece, la búsqueda binaria es el
algoritmo más eficiente; pero entonces requiere una condición adicional: que la lista esté
ordenada.

Esta sección ha presentado algunas de las ideas básicas en el estudio de los algoritmos
"divide y vencerás". También extiende el material de complejidad computacional y análi•
sis de algoritmos presentado por primera ..,·ez en las secciones 5.7 y 5.8.
520 capítulo 1O Relaciones de recurrenda

Los ejercicios de esta sección incluyen algunas extensiones de los resultados desarro--
llados en la misma. El lector que desee estudiar más sobre el tema encontrará útil e intere-
sante la bibliografía del capítulo.

EJERCICIOS 10.6 1. En lossiguiencesejercicios.j: z• ➔ R. Encuenue/(n) r<SpectO del conjunto dadoS y determi•


na la forma "O mayi1scula" adecuada de/en S.
a) /(1) = 5
f(n)•4f(n/3)+5, n=3,9,27, ...
S={3;liEN}
b) /(1) = 7
f(n)=f(n/5)+1, n=S,25,125, . ..
S•{5'liEN}
2. Scana. b, e E z., b ~ 2 yd EN. Pruebe que la solución de la relación de rccurrcac:ia
/(l)•d
f(n) =af(n/b) + e, n = b', k"' 1
satisface
a) /(n) = d + e log,,n, paran= Ir. k E N, si a= l. •
b) /(n) = dn~• + d(a ~ l))[ni.-ie.• - l[, paran :::Ir, k EN, si a~ 2.
3. Determine las formas-o mayi1scula"adecu.adas para/en el conjunto {h"lke N ) delas partes
(a) y (b) del ejercicio 2.
4 . En los siguientes ejerciciosJ: Z .. ➔ R. Encuentre/(n) respecto al conjuntos dado y determine
la forma ··o mayúscula" adécuada para/en S. .--

a) /(1)=0
f(n) = 2/(n/5) + 3, n = 5, 25,125, ...
S={S'liEN}
b) /(!)= 1
f(n) = f(n/2) + 2, n = 2,4, 8, . ..
S={Z'liEN}
5. Consideremos un torneo de tenis para n jugadores, donde n = 1!-, k E z•. Ea la primera ronda
se juegan n/2 partidos y los n/2 ganadores pasan a la segunda ronda, en la que se realizan n/4
partidos. Este proceso de división poc la mitad continúa hasta detcnninar un ganador.
a) Paran= '2--, k E Z._ sea/(n) el mlmero total de partidos jugados en el torneo. Encuentre y
resuelva una relación de recurrencia para/(n) de la forma

/(l) = d
f(n)=af(n /2) +c, n=2,4,8, ... ,

donde a, e y d son constantes.


b) Muestre que la respuesta de la parte (a) también es una solución de la relación de recurrencia

f(l)•d
/(n)=/(n/2)+(n/2), n=2,4,8, ....

6. Complete las demostraciones del corolario 10.1 y de la parte (b) del teorema 10.2.
7. ¿Cuál es la función de complejidad del mejor caso en la Msqueda binaria?
8. a ) Modifi(lUC el procedimiento del ejemplo 10.43 de la forma s.i~ente: para cuaJ9uier S C R.
talque 1s 1 = n,dividimosScom0S1US1,donde 1s 11 = 1s11 paranpacy lsi1 = 1 + 1s21
10.7 Resumen y repaso histórico 521

paran impar. Muestre que si/(n} cuenta d número de comparaciones necesarias (en este
procedimiento) para encontrar los elementos máximo y mínimo de S. entonces/ es una
función monótona. creciente.
b) ¿Cuál es la forma ..Omayúscu]a.. adecuada para la función/de la pane (a}?
9. En d corolario 10.2 nos interesaba encontrar ta forma "O ma)'\lscula" adecuada para una fun.
ción/c z• -+ R• U {O} tal que
/(1) se, para e E z·
f(n )Saf(nlb)+c, paraa.bEZ•, conb~2yn=b',kEZ•.

En este caso, la constante e de la segunda des.igualdad se interpreta como la cantidad de tiempo


necesaria para separar el problema dado de tamaño n en a problemas similares más pequeOOS
de tamaño nlb y para combinar las a soluciones de estos problemas más pequeños a fin de
obtener una solución para el problema original de tamañon. Ahora analizaremos una situación
en la que este tiempo no es constante sino que depende de n.
a} Sean a. b. e E z• conb2'2. Sea/:Z· ➔ R· U {O} una función monótona creciente, tal que
f(J)sc
f(n)saf(n/b)+cn, paran •b', kEz•.
Use un argumento similar al dado (para igualdades} en el teorema 10.1 para mostrar que
para todo n = 1, b, b', b', ..

f(n)scn
..
±
, (a / b)'.

b) Use el resultado de la parte (a) para mostrar que/ E O(n log n), cuando a = b. (La base de
ta función log es cualquier número real mayor que l.)
e) Cuando a #b. muestre que ta parte (a) implica

f(n)s (-<
- )(••·• - b'.').
a-b
d) Utilice laparte(c) para demostrar que (i)/ E O(n)cuandoa <by (ii)/ E O(nlc>c>• ), cuando
y
a >b. [Nota: La forma ..O mayúscula" de/en cada una de las partes (b) (d) es para/en z·.
no sólo en {b' 1k E N}.

10.7
Resumen y repaso histórico

En este capítulo, la relación de recurrencia sllfgió como otra herramienta para la solución
de problemas combinatorios. En estos problemas analizamos una situación dada y des-
pués expresamos el resultado a11 en términos de los resultados de ciertos enteros no nega-
tivos más pequeños. Una vez determinada la relación de recurrencia. podemos encontrar
cualquier valor de a,, (dentro de Jímites razonables). Cuando tenemos acceso a un compu•
tador. tales relaciones son panicularmente valiosas, en especial si no se pueden resolver en
fonna explicita.
El estudio de las relaciones de recurrencia se puede rastrear hasta la relación de Fibonacci
F,.. 2 = F,.. 1 + F., n ;?:0, F0 =O, F 1 = 1, dada por Leonardo de Pisa (cerca del l 75--1250)en
1202. En su I.iher Abaci, aborda un problema relacionado con el número de parejas de
conejos que se producen durante un año sí se pane de una sola pareja que engendra otra
12 Capítulo 1O Re'aciones de recurrencia

Leonardo Fibonacci (c. 1175-1250)


Rts)(odl.>cido por cort6í.a de The Gritngef Cotltction, Nueva York

pareja al final de cada mes. Cada nueva pareja comienza a reproducirse en forma similar
un mes después de su nacimiento; y suponemos, además, que ninguna pareja muere du-
rante ese año. Por lo tano, al final del primer mes tenemos dos pares de conejos; tres pares
después de dos meses; cinco pares después de tres meses, y así sucesivamente. [Como
mencionamos en el resumen del capítulo 9,Abraham DeMoivre (1667- 1754) obtuvo este
resultado mediante el método de las funciones generatrices en 1718.] Esta misma sucesión
aparece en la obra del matemático alemán Johanncs Kepler (1571- 1630), quien la usó en
su estudio de la forma en que se ordenan las hojas de una planta o flor en tomo de su tallo.
En 1844, el matemático francés Gabriel Larné (1795-1870) usó la sucesión en su análisis
de la e ficiencia del algoritmo de Euclides. Posteriormente, F~ois Édouard Anatole Lucas
(1842-1891 ), quien popularizó el juego de las torres de Hanoi, obtuvo muchas propieda-
des de esta sucesión y fue el primero en Uamarla sucesión de Fibonacci.
Para un análisis elemental de los eje mplos y propiedades de los números de Fibonacci,
consúltese e l libro de T. H. Garland [9]. Se puede aprender más de los textos de V. E.
Hoggau. Jr. [13) y S. Vajda [27). El anículode UMAPporR. V. Jean [15) proporciona mu-
chas aplicaciones de esta sucesión. El capítulo 8 de la exposición matemática de R Honsberger
{14] proporciona una descripción interesante de los números de Fibonacci y de ta sucesión
relacionada con ella de los números de Lucas. El texto de R. L. Graham, D. E. Knuth y O.
Patashnik (11] incluye también muchos ejemplos y propiedades interesantes de los núme-
ros de Fibonacci y los números de Catalan; en el artículo de R. K. Guy ( 12] aparecen otros
contraejemplos para estos números, como los que están en los ejemplos 10.14 y 10.39,
respectivamente.
Un tratamiento similar del material presentado en este capítulo aparece en el capítulo 3
de C. L. Liu (19]. Para más detalles acerca del desarrollo teórico de las relaciones de
recurrencia lineal con coeficie ntes constantes, véase el capítulo 9 de N. Finizio y G. Ladas
[7}, el cual contiene una aplicación en óptica donde se usa una relación de rccurrencia para
determinar la trayectoria de un rayo que pasa por una serie de lentes delgados con una
separación uniforme entre ellos.
1O.7 Resvmen y repaso histórico 523

Las aplicaciones en teoría de probabilidades que tratan de los sucesos recurrentes, los
recorridos alearorios y los problemas de ruina pueden encontrme en los capítulo XII y XIV
del rexto cUsicode W. Feller(6). El módulo UMAPde D. Sbabcrt [23) presa,ta las ecuaciones
en diferencias e incluye una aplicación en coooomía cooocida como el teo~ma tu la u/ara-
ña. El rexto de S. Goldberg (10) tiene más aplicaciones para las ciencias sociales.
Las tknicas recursivas en la generación de permutaciones. combinaciones y panicio--
nes de enteros se desarrollan en el capítulo 3 de R. A. Brualdi [3] y el capítulo S de E. S.
Page y L. B. Wilson (21). El algoritmo que se presenta en la sección 10.1 para las
permutac:ioncs de { 1, 2, 3 ... , n} apareció por vez. primera en la obra de H. D. Steinhaus
[25) y se conoce como el algoritmo de ordenación con marcas adyacentes. Este resultado
fue ~ubieno más tarde en forma independiente por H. F. Tro«er [261 y S.M. Johnson
[ 16). Los ~todos eficientes de ordenación para las permutaciones y otras esttucturas
combinatorias se analizan con detalle en el texto de O. E. Knuth [171- La obra de E.M.
Reingold, J. Nicvcrgclt y N. Deo [22] trata tambitn de dichos algoritmos y de su
implementación en un computador.
Para quienes disfrutaron )os irboles binarios ordenados con raíz de la sección I 0.S, el
y
capítulo 3 de A. V. Aho, J. E. Hopcroft J. O. UUman 111 sera
inreresante. La base del ejem-
plo de las pilas es~ en la página &6 del texto de S. E ven [~1- El artículo de M. Gardner [SI
proporciona otros ejemplos en que surgen los números~ Catalan. Los aspectos compu•
tacionales para determinar los números de Catalan se analizan en el artículo de D. M .
Campbell (41; el capítulo 9 de la obra de R. Honsberger (141 trata la conexión entre estos
números y el principio de reflexión (que aparece en el ejercicio complementario 34 del
capítulo 1).
Por último. el tratamiento de los algorianos ..divide y vencerás" de la sección 10.6 está
modelado según la presentación de D. F. Stanai y D. F. McAllister en la sección 5.3 de
(24). El capítulo IO del texto de A. V. Aho, J. E. Hopcroft y J. D . Ullman [ 11 proporciona
más información sobre este tema. U na aplicación de este m~todo en un algoritmo de mu.1•
tiplicación de matrices aparece en el capítulo IO del texto de C. L. Liu [18).

BIBLIOGRAFÍA

1. Abo,Alfn,:t V., Jobn E. Hopcroft y Jeffrey D. Ullrrw,,Data Strucnu.s andAlgorilhnu, Reading.


Mass.• Addison•Weslcy, 1983.
2. Auluc:k, F.C.,·-on
Sorne NewTypes of Part:itionsAssociatcd wilh Generaliz.ed Ferrers Graphs",
Procudings of tM Cambridg~ Philosophical Socil!ty 47, l9S1, págs. 679-685.
3. Brualdi, RichatdA .. /ruroductory Q,,,,biniltorics, Nueva York, Elsevier North-Holland, 1977.
4. Campbell, Douglas M., ''"lbe Computation of Catalan Numben~, Marhematic,1 Magazint S1.
n~m. 4, septiembre de 1984, págs. 195-20&.
S. Even. Shimon. Groph Algori1hmJ. Rockville. Md.. Computer Sciencc Pres.s. 1979.
6. Feller. W tlliam. An /n1rod11C1ion 10 PrObobiliry Theory and /u Applicarions. \'OI. l. 3ª. ed.•
Nueva York, Wiley, 1968.
7. Fínizio, N., y O. Ladas,An ln.troducrion to DiffeunJial E,quarioru (wilh Diflerence Equations,
Fowier Series anti Partial Differmtial E,quarion.r), Belmoot, C.alif.. Wadsworth Publishing
Company, 1982.
S. Gardner, Manin, ··Mathemalícal Game.. Catalan nurnben:An lnreger Sequence that Materializes
in Unexpected Places", Scientific American. 234. m1m. 6,junio de 1976. págs. 12~125.
!4 Capítulo 10 Relaciones de recurrenóa

9. Garland,Trudi Hammcl,Fascinating Fibonaccis, Pa!oAJto, Calif., Dale Seymour Publications.


1987.
JO. Goldberg, Samuel. lrttrodMCticin 10 Dilferrnu Equarions (wirlt lllu.srrative Examplu from
&o,,omics. P,ycltology. and Sociology). Nueva Yodc. Wiley. 1958.
11. Graham, Ronald Lewis, Donald Ervin Knuth y Oren Patashnik, Concret~ Mathtmarics, Readin¡.
Mass., Addison-WesJey. 1989.
12. Guy, Richard K., "Tbe Sccond Siroo¡ Law of Small Numbeff", M a1/wnarics Magav,u 6),
m!m. 1, febrero de 1990, págs. l-20.
13. Hog¡a.tt, Vcmcr E., Jr., Fibonacci anti úu:aJ N~n. Boston, Mass., Houghton Miftlin, 1969.
14. Honsbcrgcr, Ross, Mmhantuical Gons 111 (Thc Dolciani Matbemat:ical Expositions. número
9). Wulnngton. o.e.. Tbe Ma:hcrmncal AssocilbOO of America, 1985.
15. Jean. Ro¡erV., "Tbe Fibonacci Seque,,ce", 111, UMAP Joumal 5, mlm. l. 1984, págs. ~ 7.
16. Johl\$0n, Selmer M., "Gcneration of Pennutations by Adjacem Transposition". Ma1hematics
of Ccmputarwn 17. 1963. págs. 282-285.
17. Knulh, Donald E., TMMt ofCompu,er l',og""""""gllblwn, 3 Sorting andS,,,rc/úng, Readina.
Mass., Addison-Wesley, 1m.
18. Liu, C. L, Ehm~ntr of Discret~ Mal~matics, 2- ed., Nueva York. McGraw-Hill. 1985.
19. Liu. C. L, ln.trodllCtion to Combinalorial Ma1hana.tics, NueV1. Ycd. Mc:Graw-Hill, 1968.
20. Miksa, F. L, L Moscr y M. Wyman. '"Restrictcd Partitions of Finitc Sets.., Canadian
MaJMma1ics Bulletin 1, )958, ~s. 87-96.
21. Pagc. E. S. y L B. Wi.Json. An lnrl'O®Ction 10 CompUlalÍ(Nt,(Ú Combinatoril:s. Cambridge.
Cambridge Univenity Pres.s, 1979.
22. Rci.ngokl, E. M.• J. Nicvergell y N. Deo, C.ombinatorial Algorithnu: Theory and f>ractitt.
En¡lewood Oiffs. N.J., Prentic:e-HaU, 1m.
23. Shcrbert, DonaJd R., Differenu &¡wations withApplicaJions, Módulo UMAP 322, Cambridge
Mass.• Birthanse,- Boston, 1980.
24. Stanat. Oooald F. y David F. McAllistcr, DisarteMOJhemalicsin Compuu.rSciaice,F.n¡lcwood
Oiffs, N.J., Prcnticc-Hall. tm.
2S. Steinhaus, Hugo D., One Hu.ndrtd Problmu in ElerMntary MatMmatics, Nueva York, Basic
Boola. 1964.
26. Trotter, llF.. -ACM Algoálhm lls-Pennutations", Communu;anons oft~ ACM 5, 1962.
pa&s. 434-435.
27. Vajda S., Fibonacci & Lucas Nwnbers, and tM Goukn ~c,ion, Nueva York, Halsted Press
(una división de John Wtley & Sons), 1989.

2. a) Paran2:0,scaB.clnllmc:rodeparticionesdc ( l,2.
EJERCICIOS
3, ... , n }. Sea Bo • 1 las particiones del vacío ti.
COMPLEMENTARIOS
Verifique que para iodo n 2!. O,

1. Paran E Z· y n 2: A: + 12: l, vcrifiqucal¡cbra.icamcnte


a í6nnula de rccursión
s..,-i,-o(n~t.)s,-i,.. (~)s,.
0 ,

[Los mlmcros B,. i 2: Ose conocen como númlros


tú B,11 en honor de E.ricTemple BeU (1883-1960).J
b) ¿Cómo se relacionan los números de Bcll y los nil•
;,ara los coeficientes binomiales. meros de Stirtina: de segundo tipo?
Ejercicios complementarios 525

3. Sean n. k e z· y sea p(n, k) d nCmcro de paniciooes a) Si n > 2. mueme que d. satisface la relación de
de" en exactamente i: sumandos (k un entero positivo). De- recurrcncia
mucstn: que p(n, k) = p(n- 1, k - 1) + p(n -k, k).
,. Pan n 2: l. sea a.. el nómcro de formas de escribir n d.. = (n -I)(d• • , +d•. ,), d,• 1, d, =O.
como b swm ordenada de enteros positivos impan:s. (Por
~.o.• 3, pues 4 • 3 + 1 = 1 + 3 • l + 1 + 1 + l.) b) ¿Cómo podemos defirúrd,,de modo qued rcsul!A-
F.ncueotre y resuelva una relación de recurrcncia para a... do de la parte (a) sea válido para n ~ 21
e) Escriba de nuevo el resultado de la panc (a) como
d,- nd,., = [d,.,-(n- l)d,_J. ¿Cómo podriamos
expresar d.--nd._1 en t&mioosded•-:h d._,?
d) Muestre que d, -nd,., = ( - I )'.
a) CalatleA.1,A.>yA.•. •l Sa/(x) = L~ ..Cd,x")ln!. Despu~ de multipHcar
b) Conjeture una fórmula general para A.•, n e z· y ambos lados de la ecuación de la parte (d) por x•Jn!
demuestre su conjetura por inducción matemática. y sumar paran "2, verifiqueque/(x) =(r)/(1 -x).
6. Seaoa•(l +./5')12yp=(l-./5')12. Por Jo tanto,
a ) Vaifiquequea2 •a+ I yfP • ~+ 1.
b) MuesucqueF,• (a' -Jl')l(a-P), parak2:0,doo-- d, - n![I-_!. +_!. _ _!. + ··· + (-I)"]-
de {F, 1k " OI es la succs;ón de f;bonacci. [Esta 11 2! 3! n!
fórmula fue publicada por primera vez.en 1843 por
Jacques Pbilippe Marie Binct. (1786-1856) y se 12. Paran~ o. tracen óvalos en el plano de modo que cada
conoce con frecuencia oomo la forma de Bi.net.) óvalo inteneque los demú en exactamente dos puntos y
<) Demuesuc que para cualqu;cr n ~ O, :r:.,(,)F, • que Qlalcsquiera tres de ellos no coincidan. Si a. denota el
F,.. nómcro de rcgiooes en el plano que resultan de estos n 6va•
d) Muestre que a'= 1 +2o:ytp• I +~. los, encuentre y resuelva una relación para a..
e) Demuestre que para cualquier n 2: O,L:-o(;paF. • 13. Para n ~ O, lanz.amos una moneda 2n veces.
F,... a) Si ª• es cJ número de sucesiones de 2n lanz.amien•
7. a) Pvaa=(l + /5Y2,.vcrifiqucquea1 + 1=2 +o: tos donde aparecen n caras y n cruces. mcuentrc a,
y que (2 + a)l • 5a 1 • en táminos de n.
b) Muestre que para P = (1 - ./5')12, P' + 1 = 2 + P y b) Encuentre las constantes r. s y, tales que (r + s.xY =
(2 + Pl'= SP'- ,. ,.
e) S111,m EN,demucstrcqueL,,,.(, )F~ •• = S-F,. ••
/(x) = r:.a.,•.
e) Sea b. el número de sucesiones formadas por los
te$Ultados de 2n lanzamientos en las que el o ~
8. a) Para a= (1 + ./5)/2 y p = (1 - Js)n. ¿cuámo vale ro de caras y de cruces sólo coincide por vez pri-
ajl? mera cuando se han completado los 2n lanzamien.
b) Verifique que a/(a + 2) = 1/(a- Pl y Pl(P + 2) = tos. (Por ejemplo. sin= 3, entonces CaraCara Cara
11cP-aJ. CruzCruzCruz y CaraCaraCruzCara CruzCruz se
e:) Sin EN. demuestre que ~"'(~"') Ff= S•F-,,.. ,. cuentan en b.. pero CaraCruzCanl CataCruzCruz
9. Paran E z•. sea F. d n-áimom1mero de Fíbonacci y y CvaCaraCnaCtu:CuaCruz no.)
defina Dcfmimo, bo= O. Muesuc que para todo n ~ l.
a,.• '1-0b,. + ª• b.. - 1 + · · · + a.. -1 b1+ a,.bo-
c.• F, F,. + F, F,._, + F,F,,- : + ·
L~..h•••·Muestrequeg(,)= 1-1/f(,)
+ F.-, F, - F.F, - i F;F••,.,. d ) S..g(x)=
y obtenga despu~ • ~ n 2 l.
14. (La nnna del jugado<)
a) Cuando Catalina y Julia juegan damas. cada una
F.ntonces C¡""' F¡F¡ =1 y C2 - F,F2 + F2F1= 2. Muestre que
para n ~ 3. c. = c•• 1 +c• • 1 + F•. de ellas tiene una probabilidad de ganar de un t.
Nunca hay un empate y las partidas son indepen-
10;. Paran 2: O. sea m z l (n+ 1)/2~- Demuestre que F •• J • dientes. en el sentido de que no importa el námero
k-.(.-:"'~ (Tul vez quiera consultar los ejemplos 9.lS y de partidu que se jueguen, cada una sc¡uirá tenien-
10. 10.) do un.a probabilidad de ½de ganar la si¡uiente par•
11. Paran E Z·, d. denota el número de desórdenC$ de tida. Oespu~s de cada juego, la perdedora da a la
{l. 2, 3, ... , n} , como e n la sección 8.3. ¡anadora una moneda de 25 centavos. Si Catalina
526 Capitulo 1O Relaciones de recurrencia

tiene $2.00 para jugar y Julia $2.50, y juegan hasta a(m, 1) = 1 •- •


que una pierde todo el dinero, ¿cuál es la probabi-
lidad de que Catalina quede am.ñn.ada? a(m,n) = n~ - L ("l ) a(m,i),
t•I
sim:iien>l.
b) Responda la pane (a) suponiendo que Julia está tan
cansada que sólo tiene una probabilidad de 1/4 de 17. Paran, m E z•.sa./(r,., m)el númerodeparticionesden
ganar cada juego. donde los sumandos forman una sucesión no creciente de
15. a) ¿De cuántas formas podemos separar nWDeros enteros positivos, sin que éstos sean mayores que
i) {1, 2, 3} en tres subconjuntos de tamaiio 1? m. Por ejemplo, si n=4ym = 2, vemosque/(4, 2)= 3, pues
ü) { 1, 2, 3, 4, S. 6} en tres subconjuntos de nos interesan las siguientes tres particiones:
tamaño2?
iii) { I, 2, 3, 4, 5, 6, 7, 8. 9} en tres subconjuntos 4=2 + 2, 4=2+1 + 1, 4=1 + 1+1+1.
de tamaño 3? a) Verifique que para todos n, m E Z•,
b) Paran E z•, sea/(n,3)el número de fonnasenque
f(n,m) =f(n - m,m) + f(n,m -1).
podemos dividir {l, 2, 3..... 3n - l. 3n} en tres
subconjuntos de tamaño n. Encuentre una relación b) Escriba un programa (o desarrolle un algoritmo)
de rccurrencia para/(n, 3) y establezca una fórmu- para calcu1ar /(n.. m), n, m E Z•.
la para/(n. 3) por inducción matemática. e:) Escriba un programa (o desarrolle un algoritmo)
e) Fijemvsk cn Z•. Paran EZ\ sca/(n, k)cl número para calcularp(n), el ndmero de particiones de cual-
de formas en que podemos separar ( 1, 2, 3, , .. • quier entero positivo n.
kn - 1, kn} en k subconjuntos de tamaño n. En-
18. OJando analizamos el dígito de las unidades de cada ná-
cuentre una relación de recuncncia para f(n, k) y
mero de Fibonacci F... n .?: O, vemos que estos dígitos forman
establezca una fórmula para/(n, k) por inducción una sucesión que se repite desputs de 60 términos. [Esto
matemática. fue demostrado por primera vez por Joseph-Louis Lagrange
16. SeanA,Bconjuntostalcsque IA! =m~n= IB I ysea (1736-1813).J Escriba un programa (o desarrolle un algoril•
i(m,n)el mlmcrodc funcioncssob~deA en B. Muestre que mo) para calcular esta sucesión de 60 dígitos.
PARTE

3
TEORÍA DE
GRAFOSY
APLICACIONES
7
11
Una introducción a
la teoría de grafos

C on este capítulo empcz.arcmos a desarrollar otro de los temas principales de este libro.A
diferencia de otras áreas de las matemáücas. la teoría de grafos tiene un inicio preciso:
un articulo publicado en 1736 por el matemático suizo Leonhard Euler ( 1707-1783). La
idea principal en que se apoya su trabajo surgió de un problema ahora muy popular, cono-
cido como los siete puentes de KOnigsbcrg. Examinaremos la solución de este problema, a
panir del cual Eulct desarrolló algunos de los conceptos fundamentales de la teoría de
grafos.
A diferencia de las gráficas continuas de los primeros cursos de álgebra. los grafos que
analizaremos tienen una estructura finita y se pueden utilizar para analizar las relaciones y
sus aplicaciones en distintas situaciones. Hemos visto ejemplos de aplicaciones de la teo-
ría de grafos en los capítulos anteriores (5-8 y 10). Sin embargo, el des.arrollo en esta parte
es independiente de esos análisis anteriores.

11 .1
Definiciones y eje mplos

Cuando utilizamos un mapa de carreteras, nos interesa ver cómo llegar de un pueblo a otro
por medio de las carreteras que se indican en el mapa. En consecuencia. tratamos con dos
clases distintas de objetos: pueblos y carreteras. Como hemos visto muchas veces antes.
podemos utilizar tales conjuntos de objetos para definir una relación. Si V denota el con-
junto de pueblos y A el conjunto de carreteras. podemos definir una relación 9i sobre V
comoa[ii:b si podemos viajar de a abusando solamente las carreteras de A. Si las carrete-
ras de A que nos llevan de a a b son de doble sentido. entonces wnbiln tenemos b9t a. Si
todas las carreteras son de doble sentido, tenemos una relación simétrica.
Una forma de representar cualquíer celación es enumerar los pares ordenados que son
sus elementos. Aquí. sin embargo. es más conveniente usar un diagrama, como se muestra
en la figura 11.1. Esta figura muestra las formas posibles de viajar entre seis pueblos
usando las ocho carreteras indicadas. Muestra que al menos hay un conjunto de carreteras
que conectan a cualesquiera dos pueblos (id~nticos o distintos). Esta representación gráfi-
ca permite trabajar más fácilmente que con los 36 pares ordenados de la relación~-
Al mismo tiempo. la figura 11.1 podría ser adecuada para representar seis centros de
comunicación. donde las ocho ··carreteras" se interpretan como enlaces de comunicación.

529
530 Capitulo 11 Una introducción a la teoría de grafos

Figura 11.1 Figura 11.2

Si cada enlace proporciona comunicación en ambos sentidos, nos interesara un poco más
la vulnerabilidad del centro a a riesgos como los fallos de equipo o los ataques del enemi-
go. Sin el centro a, ni b ni e se podrían comunicar con d, e o f
A partir de estas observaciones consideramos los siguientes conceptos.

Definición 11.1 Sea V un conjunto finito no vacío, y sea E~ V x V. El par (V. E) es un grafo dirigüw
(sobre V), o digrafot (sobre V), donde V es el conjunto de vinices, o nodos y E es su
conjunto de aristas. Escribimos G = (V, E) para denotar tal digrafo.

La figura 11.2 proporciona un ejemplo de un grafo dirigido sobre V = {a, b, e, d. e} con


E = {(a. a), (a, b). (a. d). (b, e)}. La dirección de una arista se indica al colocar una flecha
dirigida sobre ella , como se muestra aquí. Para cualquier arista, como (b, e), decimos que
la arista es incidente con los vénices b, e ; bes adyacente hacia e, mientras que e es adya-
cente desde b. Además. el véniceb es el origen, o fuente, de la arista (b, e) y el véniceces
el término, o vértice terminal. La arista (a, a) es un ejemplo de un lazo y el vértice e q ue no
tiene aristas incidentes es un vénice aislado.
Cuando no impona la dirección de las aristas. la cstructuraG = (V, E ), donde E es ahora
un conjunto de pares no ordenados sobre V. es un grafo no dirigido. La figura l l .3(a)
muestra un grafo no dirigido. Esta gráfica es una forma más compacta de describir el grafo
dirigido dado en la figura l l.3(b). En un grafo no dirigido, hay aristas no dirigidas, como
las aristas {a, b). {b, e}, {a, e }, {e, d} de la figura I l.3(a). Una arista como {a, b} repre-
senta {(a. b), (b, a )}.Aunque ( a, b) = (b, a ) sólo si a = b, te nemos que {a, b } = ( b. a } para
todos a, b. Podemos escribír fa, a} para designar un lazo en un grafo no dirigido, pero
fa. a } se considera igual a (a, a ).
En general, si no se especifica que un grafo Ges dirigido o no, supondremos que es no
dirigido. Cuando no contiene lazos. decimos que es un grafo sin lazos.

En las siguientes dos definiciones no nos preocuparemos por los lazos que puedan
aparecer en el grafo no dirigido G.

t Como la 1crminolog:ía de la teoría de grafos no es cstánd.ar, el lector puede cr,conlr:ll" algun.u diferen-
cias e ntre los tt'nninos usados aqui y en otros rexros
11.1 Oefinióones y ejemplos 531

d
(al lb)

Figura 11.3
\

Definición 11.2 Sean x. y vértices (no necesariamente distinros) de un grafo no dirigido G = (V, E ). Un
camino x-y en Ges una sucesión alternada finita (sin lazos)

de vértices y aris1as de G, que comienza en el vértice .:f y tennina en el vértice y y que


conlicne las n aristas e,= {x;_·,. X; }donde 1 :::si s n. ·
La longitud de un camino es n. el número de aristas que hay en el camino. (Sin = O, no
existen aristas,x :: y, y el camino se denomina 1rivial. Es1os caminos no se tendrán muy en
cuenta en nuestro trabajo.)
Cualquier caminox-y donde x = y (y n > 1) es un camino curado. En caso concrario, el
camino es abitno.

Observe que un camino puede repetir aristas y vértices .

....... 1-1;1- · Para el grafo de la figura 11 .4, tenemos, por ejemplo, los siguientes tres caminos abiertos.
Podemos enumerar solamente las aristas o solamente los vértices (si el otro queda deter-
minado claramente).
1) (a, b), {b, d), jd, <), (c.,),(,, d). (d, b): éSle es un camino a-b de longitud 6 en
el que se repiten los vénices d y b, así como la arista {b, d}( = (d. b )).
2) b ➔ e ➔ d ➔ e ➔ e ➔/: aquí tenemos uncaminob-/de longitud 5, donde se repite
el vértice c. sin que aparezcan las aristas más de una vez.
3) (/. e}. {e, e), {e, d}, {d. a}: en este caso, el camino/-a tiene longitud 4, sin repeti-
c ión de vlnices o aristas.

·~.
e Figura 11.4
532 Capítulo 11 Una introdu,óón a ta teoría de grafos

Como el grafo de la figura 11.4 no es dirigido, el resultado del camino 1 (pane 1)


también es un camino b-a (leemos las aristas, en caso necesario. como {b, d}, {d, e},
{e. e}, (e, d}. {d, b} y {b, a}). También se cumplen observaciones similares para los
caminos de las panes (2) y (3).
Por último, las aristas {b, e}, {c. d} y {d., b} proporcionan un camino b·b (cerrado).
Estas aristas (ordenadas de manera adecuada) también definen caminos (cerrados)c-c y d..J.

Examinaremos ahora algunos tipos especiales de caminos.

Definición 11.3 Consideremos un camino x -y en un grafo no dirigido G ;: (V, E ).


a ) Si no se repite ninguna arista en el camino x-y, entonces e l camino es un recorrido
x-y. Un recorridox-xcerrado es un circuito.
b) Cuando ningún vértice del cantinox-y se presenta más de una vez, el camino es un
camino simple x-y. El ténnino ciclo se usa para describir un camino simple cerrado
X-X.

Convenio: Cuando tratemos con circuitos, supondremos siempre la existencia de al me-


nos una arista. Cuando existe sólo una arista, entonces el circuito es un lazo (y el grafo
tiene entonces lazos). Los circuitos con dos aristas son multigrafos, concepto que definire-
mos más adelante.
El ténnino ciclo implicará siempre Ja presencia de al menos tres aristas distintas (del
grafo).

a) El camino b-f de la parte (2) del ejemplo 11. l es un recorrido b-f, pero no es un
camino simple b-/ya que se repite el vénice c. Sin embargo, el carn.ino/-a de la
parte (3) de aquel ejemplo es un recorridof-a y un camino simple/-a.
b) En la figura 11.4, las aristas (a, b}, {b, d), {d, c}, {e, ,},{,, d} y {d, a} dan lugar
al circuito a-a. El vértice d se repite, por lo que las aristas no nos dan un ciclo a-a.
e) Las aristas {a. b), {b, e), ( c. di y {d, a) proporcionan un ciclo a-a (de longitud 4)
en la figura 11.4. Cuando estas mismas aristas se ordenan apropiadamente, también
pueden definir un ciclo b-b, e-e o d-d. Cada uno de estos cic1os es también un
circuito.

Para un grafo dirigido usaremos el adjetivo dirigido, como -.e usa, por ejemplo, en
caminos dirigidos, caminos simples dirigidos y ciclos dirigidos.

Antes de continuar, resumiremos (en la Tabla 11.1) los resultados de las definícioncs
11.2 y 11.3 para referencia futura. Cada ocurrencia de "Sf' en las primeras dos columnas
debe interpretarse como "Sí, posible mente". La tabla 11. l refleja el hecho de que un cami-
no simple es un recorrido, que a su vez es un camino abierto. Además, cada cic1o es un
circuito y cada circuito (con al menos dos aristas) es un camino cerrado.
Considerando eJ número de conceptos que hemos presentado, es tiempo de demostrar
un primer resultado en esta nueva teoría.
11.1 Definiciones y ejemplos 533

T•bla 11.1

Vfrtke{s) Arista(s)
r<p<tido(s) ttpdida(s) Abierto Cerrado Nombtt

Sí Sí SI Camino
Sí Sí Sí Camino (cerrado)
Sí No Sí Recorrido
Sí No Sí Circuito
No No Sí Camino simple
No No Sí Ciclo

TEOREMA 11.1 Sea G = (V. E ) un grafo no dirigido. con a. b E V, a ~b. Si existe un recorrido (en G) de
a a b, entonces existe un camino simple (en G) de a a b.
Demostración: Como hay a1 ménos un recorrido de a a b, seleccionamos el que tenga la
longitud más cona.digamos {a. x.}, {x1,x1}, .. . , {.r_.,b}. Si este recorrido nocs un camino
simple, tenemos la situación {a. .r1}, {.ri, x 2}, ... , {.rt. 1, Xt}, {xt, Xt ♦ 1}. {.tt•I• .rh2},.
{x..,•.r"'}, {x_. .r"' .. 1}, ... ,{.t,.. b}, dondek < m y -x..:-=x... posiblemente con k • O y a(=.xo)
:;;; x .., o m • n + l y Xt-= b(~... 1). Pero entonces {a. .t1 }, {xi, x 2}, ••. , {x..,_1, Xt}, {x,.. x,,...1 },
... ,{.t~, b } es un recorrido simple más corto de a a b.

La noción de camino simple es necesaria en la siguiente propiedad de los grafos.

Definició n 11.4 Sea G =(V. E ) un grafo no dirigido. Decimos que Ges conexo si existe un camino simple
~ntre cualesquiera dos virtices distintos de G.
Sea G ~ (V. E) un grafo dirigido. Su grafo no dirigido asociado es el grafo ob<enido de
G si no se tienen en cuenta las direcciones de las aristas. Si se obtiene más de una arista no
dirigida de un par de vértices distintos de G, entonces sólo una de estas aristas se dibuja en
c:I grafo no dirigido asociado. Cuando este grafo asociado es conexo, considerarnos que G
es conexo.
Un grafo que no es conexo es discont!UJ.

Los grafos de las figuras 11.1. 11.3 y 11.4 son conexos. En la figura 11.2, el grafo no es
conexo ya que, por ejemplo. no hay un camino simple de a a t!.

q.iplo 11":;3 En la figura IU tenemos un grafo no dirigido sobre V= (a. b. c. d, <,/. g}. Este grafo no
es conexo ya que, por ejemplo, no hay un camino simple de a a t!. Sin embargo, el grafo
está compuesto por piezas (donde los conjuntos de vértices son V1 = {a. b, e, d }, Vl:;;; {e.
f. g} y los conjunlos de arisw son E, = ( (a. b}. (a. e}. (a. d}. (b. d} },E,= ( (<./}, (/. g}}
que. son conexos: estas piezas son las componentes del grafo. Por lo tanto, un grafo no
dirigidoG = (V, E) es disconexo si y sólo si V puede separarse en al menos dos subconjuntos
534 Capitulo 11 Una intrcx:tucción a la teoóa de grafos

rt-7"
,/ V d
V
e

f
g

Figura 11.5

V1, V2 tales que no haya una arista en E de la fonna {x. y} dondex E V1 e y E V1 • Un grafo
es conexo si y sólo si tiene solamente una componente.

Definición 11.5 Para cualquier grafo G :;; (V, E ), el m1mero de componentes de G se denota con K(G).

Para los grafos de las figuras 11. J, 11.3 y 11.4, K(G) ::: J. puesto que estos grafos son
conexos; K(G) = 2 para los grafos de las figuras 11.2 y 11.5.

Antes de cerrar esta primera sección, extenderemos nuestro concepto de grafo. Hasta
ahora hemos pennitido al menos una arista entre dos vértices; ahora examinaremos una
extensión.

Definición 11.6 Un grafoG = ( V. E ) es unmultigrafo si existen a, b E V. a"/=. b , con dos o más aristas de la
forma (a) (a. b){para un grafo dirigido), o (b) {a. b) (para un grafo no dirigido).

La figura 11.6 muestra un ejemplo de un multigrafodirigido. Existen tres aristas de aa


b, por lo que podemos decir que (a, b ) tiene multiplicidad 3. Las aristas (b. e) y (d, e)
tienen multiplicidad 2. Además, la arista (e. d) y cualquiera de las aristas (d, e) fonnan un
circuito (dirigido) de longitud 2 en el multigrafo.
Paran E z·.un multigi-afo es un n-grafo si ninguna de las aristas del grafo tie ne mul-
tiplicidad mayor que n. El grafo de la figura 11.6 es un 3-grafo dirigido.

Figura 11.6
11 . 1 Definiciones plos 535

Necesitaremos la idea de multigrafo en una sección posterior de este capítulo, d onde


íC$0lvcrcmos el problema de los siete puentes de KOnigsberg. Nota: Cuando trabajemos
con un multigrafo G, estableceremos explícitamente que Ges un multigrafo.

S 11. 1 1. Enumere tres situaciones, diíercn1cs de las vistas en esta sección. en que un ¡raro pueda ser
útil.
2. Pan. el ¡:rafo de la figura t 1.7. determine (a) un caminodeb ad que no sea un rtlCOO'ido; (b) un
rcc;onido b-d que no sea un camino simple; (e) un camino simple de b a d: (d) un camino
cerrado deb a b que no sea un circuito: (e) un circuito de b a b que no sea un ciclo; y, (0 un
ciclo deba b.

figura 11.7

3. Para el grafo de la figura 11.7. ¿cuintos caminos simples existen de b aj?


4. ¿,OJ!ntos caminos simples diferentes existen entre los vffliccs a y/ e n el grafo dado en la
figura 11.8?

Figur• 11.s Figura 11.9

5. SeaG = (V. E)cl grafo no dirigido de la figura 11.9.¿Cutruos caminossimplescxistcnenGde


a ah? ¿Cuántos de ellos son de longitud 5?
6 . Si a.b son vbticcs di:s1intos en un grafo G no d iri¡ido. la distancia de a a b se define como la
lon¡itud del camino simple más corto de a a b (si a = b, la distancia se define como O). Para el
grafo de la fi¡ura 11.10, encuentre las distancias dcd a (cada uno de) los vtrticcs de G.
7. Siclc ciudadcsa. b, e, d, e,Jy gcst:iln cooce1adas por un sistema de autopi$la.Scomosiguc: (1)
1-22 va dc a ac, pasando por b; (2)1-33 vadccadyentonces pasa porbycontinlla hacia/; (3)
1-44 va dcdpOr e hacia a; (4) 1-5S va de/a b, pasando por g; y (5) 1-66 va dc g a d.
536 Capítulo 11 Una introducción a la teoría de grafos

a) Use los vértices para las ciudades y las aristas dirigidas para los tramos de autopi5ta que las
unen, y ctibuje un grafo dirigido que modele esta situación.
b) Enumere los caminos simples de g a a.
e) ¿Cuál es el menor número de segmentos de autopista que tendrían que cerrarse para inte-
rrumpir el paso de b a d?
d ) ¿Es posible salir de la ciudad e y regresar a ella. visitando una sola vez las otras ciudades?
e) ¿Cuál es la respuesta de la parte (d) si no es necesario regresar a c'?
f) ¿Es posible comenzar en alguna ciudad y viajar por todas las autopistas exactamente una
vez? (Se permite visitar una ciudad mis de una vez y no es necesario regresar a la ciudad
donde se inició el recorrido.)
8. La figura 11.11 muestra un grafo no dirigido que representa una sección de unos grandes
almacenes. Los vértices indican el lugar donde se localizan las cajas; las aristas indican los
pasillos que hay entre ellas. Los almacenes necesitan instalar un sistema de seguridad. que
consiste en colocar guardias (vestidos d e civil) en ciertas cajas, de manera que cada cajero
tenga un guardia en su lugar o que haya un solo pasillo entre una caja con guardia y el cajacl
¿Cuál es el número mínimo de guardias necesarios?
9. Sea G = (V, E) un grafo no dirigido sin lazos y sea ia, b} una arista de G. Demuestre que la
arisca {a. b} es pane de un ciclo si y sólo si al eliminarla (com e rvando los vtrticcsa y b), G no
se vuelve disconexo.
1 O. Di6 un ejemplo de un grafo conex.o G tal que al eliminar cualquier arista de G se obtenga ua
grafo disconex.o.

·b· Ih

k Figura 11.11
11.2 Subgrafos. complementos e isomorfismos de grafos 537

1 1. Sea G un grafo que satisface la condición del ejercicio 10.


a ) ¿Debe G carecer de lazos? b) ¿Puede G ser un multignfo?
c) Si G tienen vfruces. ¿podemos determinar C'I.Wlcas aristas tiene?
12. a) SiG=(V,E)esun grafonodirigido con lvl z u, IE I = eynohayluos,demuesttequc 2,
su1 -u.
b) Establezca la desigualdad concspondiente e n caso de que G sea dirigido.
13 . Sea G : (V. E ) un ¡rafo nodiri¡ido. Definimos una relaci6o91: sobre Vcomoa!Jib si a •bo
si exi$le un camino simple en G de a a b. Demuestre que 9t es una relación de equivalencia..
Describa la partición de V inducida por 91:.
14. a ) Considere los tres grafos no dfrigidos conexos de la figura 11.12. El ¡rafo de la pa11e (a) de
la figura cons11 de un ciclo (con los vénices v 1• 111, v¡) y un vértice"' con arista.s (radios)
!ruadas desde w. a los ouos tres vMices. A este grafo se le llama rueda con tres radios y se
denota con R),, En la parte (b) de la fi¡ura cncontram0s el grafo R_., la rueda con cuatro
radios. La rueda Ri con cinco radios aparece en la fi¡ura l l.12(c). Determine eufntos
ciclos de longitud cuatro hay en cada uno de estos grafos.
b) En general. sin e z· y n .t: 3, entonces la l"llUIJ con II radios es el ¡ralo fonnado por un
ciclo de longitud II y un vfrtice adicionaJ que es adyacente: a los II vtrtices del ciclo. Este
grafo se denota con R•.
i) ¿Cuántos ciclos de longitud 4 existen en R.'!
ii) ¿Cuántos ciclos en R. tienen longitud n?

u, .,

0 ·W'©
u

(a)

Figu ra 11.12
v,

R, (b)
v,
V~ V1

R,
w,

(e)
w,

w,
w,

R,

11 .2
Subgrafos, co mplementos e
iso morfismos de grafos

En esta sección nos centraremos en las siguientes dos ideas:


a) ¿Q!,K; tipo de subestructura nos sirve para analizar un grafo?
b) ¿Es posible uazar dos grafos q ue parezcan distintos pero que tengan la misma es-
tructura subyacente?
Para responde r la pregunta de la parte (a) introduciremos la siguiente definición.
538 Capítulo 11 Una introducción a la teoría de grafos

Definición 11 .7 Si G: (V, E) es un grafo (dirigido o no), entonces G, : (V,. E,) es un subgrafode G si f


'i=~ V, y E1 e;: E, donde cada arista de E1 es incidente con los vértices de V1•

La figura ll.13(a) nos muestra un grafo no dirigido G y dos de sus subgrafos, G, y G,.
Los vértices a. b son aislados en el subgrafo G,. La parte (b) de la figura nos muestra un
ejemplo de grafo dirigido. Aquí el vértice w es aislado en G'.

.
(GJ (G,J (Gz) (G) (G')
b b b
' '
¡\
\V· .•
>0
e

I
r V

(a)
d d d
(b)
w
.
w

Figura 11.13

Algunos tipos especiales de subgrafos surgen del modo siguiente:

Definición 11 .8 Dado un grafo (dirigido o no) G: (V, E), sea G, : (V,. E,) un subgrafo de G. Si V,: V,
entonces G, es un subgrafo ~cubridor de G.

En la parte (a) de la figura 11.13, ni G, ni G, son subgrafos recubridores de G. Los


subgrafos G, y G, de la pane (a) de la figura 11.14 son subgrafos recubridores deG. Éstos

(G,J (G,J (G"J (G"'J

.\.!\.A/\
\l V·;···
d d
(a) (b)

Figura 11.14
11.2 Subgrafos. complementos e isomorfismos de grafos 539

son dos de los 2'• 16 posibles subgrafos recubridoresque existen deG. El grafo dirigido
G' de la parte (b) de la figura 11. 13 es un subgrafo, pero n.o un subgrafo recubridor, del
grafo dirigidoG dado en esa pane de la figura En la pane (b) de la figura 11.14 los grafos
dirigidos G " y G"' son dos de los 2' = 16 posibles subgrafos recubridores.

Definición 11 .9 Sea G = (V, E) un grafo (dirigido o no). Si 0 +- U~ V, el subgrafo de G inducido por U


es el subgrafo cuyo conjunto de vértices es U y que contiene todas las aristas (de G) de la
forma (a) (x. y), para x, y E U (si Ges dirigido), o (b) (x, y), para x, y E U (si Ges no
dirigido). Denotamos este subgrafo con (U).
Un subgrafo G ' de un grafo G=(V, E ) es un subgrafo inducido si existe 0-/; U~ V tal
queG' = (U).

Para los subgrafos de la figura l l.13(a), vemos que G 2 es un subgrafo inducido de G


pero el subgrafo G 1 no es un subgrafo inducido ya que no aparece la arista {a. d }.

a) SeaG = (V, E ) el grafo de la figura I l.l5(a). Lossubgrafosde las panes (b)y (c)de
la figura son inducidos. Para el subgrafo conexo de la parte (b), G 1 = (U,}para U 1 •
(b, e, d, e). De igual manera, el subgrafodisconexode la pane (e) esG, = (U,) para
U, = (a, b, e.f) . Finalmente, G,de la pane (d) de la figura 11.15 es un subgrafo de
G que no es un subgrafo inducido; tos vértices e, e están e n G3, pero la arista {e, e}
(de G) no cstí presente.
b) Observe que cada una de las componentes del grafoG de ta figura 11.5 (del ejemplo
11.3) es un subgrafo de G pero no un subgrafo recubridor de G. Una de estas com-
ponentes es inducid.a por V1 : { a, b, e, d }; la otra es inducida por V2 • { e, f. g}.

(G) (G,) (G,) {G 3)


e
' '

?). b1le ;; /
f f
' ~. f
(al (b) (e) (d)

Figura 11.15

Otro tipo especial de subgrafo se obtiene al e liminar cieno vértice o arista de un grafo
dado. Hemos formalizado esta idea e n la siguiente definición.
540 capítulo 11 Una introducción a la teoría de grafos

Definición 11.1 O Sea u un vértice en un grafo G = (V, E) (dirigido o no). El subgrafo de G denotado por
G - u tiene el conjunto de vénices V1 =V - { u} y el conj unto de aristas E, ~ E, ta1 que E1
conúene todas las aristas en A excepto las incidentes con el vértice u. (Por lo tanto, G - u
es el subgrafo de G inducido por V1 .)
De forma similar, si e es una arista de un grafo G=(V, E) dirigido o no dirigido, obtene-
mos el subgrafo G- e = (V1, E1) de G, donde el conjunto de aristas es E 1 :;;; E - (e}, y el
conjunto de vértices no cambia (es decir, V1 = V).

Sea G =( V, E) el grafo no dirigido de la figura l l . 16(a). La parte (b) de esta figura es el


subgrafo G1 (de G), tal que G 1 = G - c. También es e l subgrafo de G inducido por el
conjunto de véníces U, = (a, b, d, f. g. h}. y G, = (V - (e}) = (U,). En la pane (e) de la
figura 11 .16 encontramos el subgrafo G2 de G. donde G 2 • G - e, y e es la arista {e, d}. El
resultado de la figura l l .)6(d) muestra que las ideas de la definición 11. 10 pueden exten-
derse para e liminar más de un vértice (arista). Podemos representar este subgrafo de G
como G, = (G - b) - f= (G-f)-b=G-(b,f} = (U,), para U, = (a, e, d. g, h}.

<Gl (G,) (Gl) (G,)


..
:~. d-49 d-49
(a)
h
b o----A ª

(bl
h
b~

le)

h
d/ c
(di
rh

Figura 11.16

La idea de subgrafo nos ofrece una fonna de desarrollar el complementario de un grafo


no dirigido sin lazos. Sin embargo, antes de hacerlo definiremos un tipo de grafo de tama-
ño maximaJ para un número dado de vértices.

Definición 11 .11 Sea V un conjunto de n vénices. El grafo completo sobre V, que se denota con K,.. es un
grafo no dirigido sin lazos tal que para todos a, b E V, a #;b, existe una arista {a, b}.

La figura 11.17 proporciona los grafos completosKu para I S n S 4. Cuando ex.amine-


mos la idea de isomorfismo de grafos veremos que éstos son los únicos grafos c ompletos
posibles para el número dado de vértices.
Para detenninar el complemento de un conjunto e-n el capítulo 3, necesitábamos cono-
cer el conjunto universal en cuestión. Et grafo completo tiene un papel similar al del con-
junto universal.
11.2 Subgrafos. comp~entos ~ isomorfismos de grafos 541

• a a b

( K,)
.
a

<K1)
\ D -~
b e
( K,)
b
(K,)

Figur~ 11. 17

Dlfinici6n 11 .12 Sea G un grafo no dirigido sin lazos conn vértices. Elcomplementariode G, que se denota
con 'G, es el subgrafodeK4 formado por losn vértices deG y todas las aristas que no están
en G. (Si G: K,,. 7: es un grafo con n vértices y ninguna arista. A este grafo se le llama
grafo nulo.)

En la figura 11 .18(a) aparece un grafo no dirigido de cuatro vértices. Su complementa•


rio se muestra en la parte (b) de la figura. En el complementario, el vértice a está aislado.

a ~b ao r\
(a)
J ~ < d~

lbl
Figura 11.18

Una vez más hemos llegado a un punto donde hemos definido muchas ideas nuevas.
Para demostrar la importancia de algunas de ellas, las aplicaremos ahora en la solución de
un acertijo interesante.

L«u.ra instantánea. Para el juego de la locura instantánea se requie re n cuatro cubos. Cada
una de las seis caras de un cubo se pinta de un color: rojo (R), blanco (8), verde (V) o
amarillo (A). El objetivo del j uego es colocar los cubos en una columna de cuatro de modo
que aparezcan los cuatro colores (diferentes) en cada uno de los c uatro lados de la columna.
Consideremos los cubos de la figura 11. 19, numerados como se muestra. (Estos c1;bos
son solamente un ejemplo de este juego. Existen muchos otros.) Primero hay que estimar
el número de disposiciones posibles. Si queremos colocar e l cubo I en la pane inferior de
la columna. hay a lo sumo tres formas diferentes de hacerlo. En la figura 11 . 19 desdobla-
mos el cubo 1 y vemos que no hay diferencia si colocamos la cara roja o la cara blanca
Capítulo 11 Una introducción a la teoría de grafos

++
(1) (21

++
(3) (4)
Figura 11.19

opuesta sobre la mesa. Sólo nos interesan las cuatro caras restantes en la base de la colum-
na. Con tres pares de caras opuestas, habrá como máximo tres formas de colocar el primer
cubo como base de la columna. Ahora consideremos el cubo 2. Aunque se repiten algunos
colores, ningún par de caras opuestas tiene el mismo color. Por lo tanto, se tienen seis for-mas
de colocar el segundo cubo sobre el primero. Entonces podemos rotar el segundo cubo sin·
cambiar la cara de arriba del primer cubo o la cara de abajo del segundo cubo. Con cuatro
rotaciones posibles podemos colocar el segundo cubo arriba del primero de 24 formas
diferentes. Si continuamos con este razonamiento vemos que hay (3)(24)(24X24) = 41,
472 posibilidades por considerar. ¡Y podría no haber una solución!
Al resolver el acertijo, nos damos cue nta de la dificultad de mantener el registro (1 ) de
los colores en las caras opuestas de los cubos y (2) de las columnas de colores. Un grafo
(e n realidad, un multigrafo etiquetado) nos ayuda a visualizar la situación. En la figura
11.20 aparece un grafo con cuatro vénices R, B, V, A. Al considerar cada cubo, analizamos
sus tres pares de caras opuestas. Por eje mplo, el cubo 1 tiene un par de caras opuestas
pintadas de amarillo y verde, por lo que se traza una arista que conecta Ay V y se etiqueta
con 1 (del cubo 1). Las otras dos aristas de la figura etiquetadas con 1 representan los pares
de caras opuestas blanca y amarilla, y roja y blanca. Hacemos lo mismo con los otros

Figura 11.20
11.2 Subgrafos. complementos e isomorfismos de grafos 543

cubos para obtener el grafo de la figura. Un lazo con etiqueta 3 como el de B, muestra un
par de caras opuestas del mismo color.
A panir del grafo vemos que hay un total de 12 aristas en cuatro conjuntos de 3, según
las etiquetas de los cubos. En cada v~rtice. el número de aristas incidentes a (o de) dicho
vértice cuentan el m1mero de caras en los cuatro cubos que tienen ese color. (Contamos un
lazo dos veces.) Por to tanto, la figura 11.20 indica que para los cuatro cubos se tienen
cinco caras rojas, siete blancas, seis verdes y seis amarillas.
Con los cuatro cubos apilados e a una columna. examinamos dos lados opuestos de la
columna. Esta disposición da cuatro aristas del grafo de la figura 11.20, donde cada eti-
queta aparece una sola vez. Como cada color aparecerá una sola vez en un lado de la
columna. cada color debe aparecer dos veces como extremo de estas cuatro aristas. Si
podemos obtener el mismo resultado para los otros dos lados de la columna. habremos
resuelto el acenijo. En la figura 11.21(a) vemos que cada lado en un par de lados opuestos
de nuestra columna. tiene los cuatro colores si los cubos se ordenan según la información
proporcionada por el subgrafo mostrado aquí. Sin embargo. para que los otros dos lados
de la columna tambi~n cumplan la condición, se necesita un segundo subgrafo que no use
ninguna de 1as aristas de la parte (a). En este caso, sí existe un segundo subgrafo, como se
muestra en la pane (b) de la figura.

Figura 11.21

La fi¡ura 11.22 muestra la forma de acomodar los cubos, como lo indican los subgrafos
de la figura 11.21.
E.n general para cualesquiera cuatro cubos, construimos un multigrafo etiquetado y
tratamos de encontrar dos subgrafos tales que ( 1) cada subg:rafo contenga los cuatro v~rti-
ccs y las cuatro aristas, uno por cada etiqueta~(2) en cada subgrafo, cada vértice sea inci-

(1) (2) (3) (4)

Figura 11.22
544 C3pftulo 11 Una introducción a la teoría de grafos

dente con exactamente dos aristas (los lazos se cuentan dos veces) y (3) ninguna arista
(etiquetada) del multigrafo etiquetado aparez.ca en ambos subgrafos.

Ahora volveremos a la segunda pregunta planteada al inicio de la sección.


Las partes (a) y (b) de la figura 11.23 muestran dos grafos no dirigídos de cuatro v~rti-
ces. Como las aristas rectas y las curvas se consideran iguales. cada grafo representa seis
pares de vértices adyacentes. De hecho, probablemente pensemos que estos dos grafos soo
ejemplos del grafo K4 • Podemos establecer esto de una forma matemáticamente rigurosa
con la siguiente definición.

. ó m n

'~
(a) (b)
~ [SJ
y z
p

(e)
• Q 0
'
(dl
u

Figura 11.23

De f inición 11.13 Sean G 1 = (V., E 1) y G2 = (V2 , E2) dos grafos no dirigidos. Una función/: Vi ➔ V1 es un
isomo,jismode grafos si (a)/ es inyectiva y sobre y (b) para todos a, b E Vi , {a, b} E E,
si y sólo si {/(a).f(b)} E E,. Cuando existe tal función, G, y G, son grafos isomo,fos.

La correspondencia de vértices de un isomorfismo de grafos mantiene las adyacencias.


Puesto que el hecho de que los pares de vénices sean adyacentes o no es la única propie-
dad esencial de un grafo no dirigido, de esta fonna preservamos la estructura de los grafos.
Para los grafos de las partes (a) y (b) de la figura 11.23, la función! definida por

f(a) = w, f(b) =x, f(c) = y, f(d) = z

da como resultado un isomorfismo. [De hecho, cualquier correspondencia uno a uno en~
{a, b. c. d} y { w, x, y, z} será un isomorfismo, ya que ambos grafos son comple tos. Tam-
bién esto será cieno si cada uno de los grafos dados tiene solamente cuatro vértices aisla-
dos (y ninguna arista).] En consecuencia, en lo que se refiere a la estructura (del grafo),
e stos grafos se consideran iguales, cada uno es (isomorfo a) el grafo completo K, .
Para los grafos de las partes (c) y (d) de la figura 11.23 se necesita ser un poco más
cuídadoso. La función g definida por

g(m)=r, g(n) = s, g(p)=t, g(q) = u

es uno a uno y sobre (para los conjuntos de vénices dados). Sin embargo, aunque {m. q} es
una arista del grafo de la parte (e), {g(m), g(q)} = {,,u} no es una arista del grafo de la
11.2 Su rafos, complementos e isomorfismos de grafos S45

parte (d). En consecuencia, la función g no define un isomorfismo de grafos. Para mante-


ner la correspondencia de aristas se considera la función uno a uno y sobre h tal que

h(m) =s, h(n) • , , h(p) • u, h(q) = l.

En este caso tenemos las correspondencias de aristas


{m,n} ... {h(m),h(n)) = {s, r ),
{m,p) ... {h(m), h(p)) ={s, u},
{m,q) ... (h(m),h(q)} = {s, 1},
{n,q}-{h(n), h(q)} = {r,1},
{p,q) ...·¡h(p),h(q)} = {u, 1},
entonces hes un isomcwfismo de grafos. [Tambitn notamos, por ejemplo, que el ciclom ➔
n ➔ q ➔ m conesponde con el ciclos( = h(m)) ➔ r (=h(n)) ➔ 1( = h(q)) ➔ s ( = h(m)).]
Finalmente, como el grafo de la pane (a) de la figura 11.23 tiene seis aristas y el de la
parte (c) tiene solamente cinco aristas, estos dos grafos no pueden ser isomorfos.

Ahora examinaremos la idea de isomorfismo de grafos en una situación más complicada

En la figura 11.24 tenemos dos grafos, cada uno con diez vértices. A diferencia de los
grafos de la figura 11.23, no se ve de inmediato si estos grafos son isomorfos.
Se encuentra que la correspondencia dada por

a➔ q c➔ u t- r g-t>x i➔ z
b-+u d-+y f-+w h-+l J-s

preserva todas las adyacencias. Por ejemplo, [/. h} es una arista del grafo (a) y {w. 1} es h
arista correspondiente en el grafo ( b). Pero ¿cómo hallamos esta correspondencia? El Si•
guiente análisis proporciona algunas pistas.
Observemos que, como un isomorfismo preserva adyacencias, preserva subestructura!
de grafos como caminos simples y ciclos. En el grafo (a), las aristas (a ,/}. (f. i}, (i, d}
(d. t:} y {t. a} constituyen un ciclo de longitud S. Por Jo tanto hay que preservar esto a

•®· f g b
V
<ºw @
-
_ ,' ,Z $

i h Y

d u 1
(a) (b)

Figura 11.24
546 Capítulo 11 Una introducción a la teoría de grafos

buscar un isomorfismo. Una posibilidad para las aristas correspondientes del grafo (b) es
{q. w}, {w, z}. {z. y}, {y, r} y {r. q}. que también forman un ciclo de longitud 5. (Una
segunda elección posible está dada por ·las aristas del ciclo y ➔ r ➔ s ➔ t ➔ u ➔ y.)
Además, comenzando con el vénice a en el grafo (a), encontramos un camino simple que
..visitará" cada vértice sólo una vez. Expresamos este camino simple como a ➔ f ➔ h -4
e ➔ b ➔ g ➔ j ➔ e ➔ d ➔ i. Para que los grafos sean isomorfos, debe haber un camino
simple correspondiente en el grafo (b). En este caso, el equivalente es el camino simple
descrito porq ➔ w ➔ t ➔ u ➔ u ➔ .x ➔ s ➔ r ➔ y ➔ z.

Éstas son algunas de las ideas que pueden usarse para tratar de des.arrollar un isomorfismo
y detenninar si dos grafos son isomorfos. Analizaremos otras consideraciones en este ca-
pítulo. Sin embargo, no hay un método simple e infalible, es~ialmentc cuando tratamos
con grafos grandes G, = (V,. E,) yG, = (V,, E,), 1ales que IV, 1= 1V, 1y I E, 1= 1E,I.
Cerraremos esta sección con un ejemplo más de isomorfismo de grafos.

Los dos grafos de la figura 11.25 tienen seis vértices y nueve aristas cada uno. Por- tanto. es
razonable preguntarse si son isomorfos.
En el grafo (a). el vértice a es adyacente a otros dos vértices del grafo. En consecuencia.
si intentamos construir un isomorfismo entre estos grafos, deberíamos asociar el vértice a
con otro vértice análogo del grafo (b). por ejemplo, el vértice u. Una situación similar
existe para el vértice d con los vértices x o z. Pero. independientemente del vértice x o z
que se use, quedará un vértice en el grafo (b) que es adyacente a otros dos. Y no hay otro
vértice en el grafo (a) para continuar con la correspondencia uno a uno que preserve la
estructura. En consecuencia. estos grafos no son isomorfos.
Más aún, en el grafo (b) es posible comenzar en cualquier vértice y hallar un ciclo que
incluya las aristas del grafo. Por ejemplo, si se comienza en el vértice u, el circuito u ➔ w
➔ u ➔ y ➔ w ➔ z ➔ y ➔ x ➔ u ➔ u presenta esta propiedad. Esto no sucede en el grafo
(a) donde los únicos recorridos que incluyen cada arista comienzan en b o/y terminan en
/ o b, respectivamente.

<•>
Figura 11.25
&
X
(b)
y Z
11.2 Subgrafos, complementos e isomorfismos de grafos 547

:.IERCICIOS 11 .2 1. Sea Gel ¡raío no diri¡ido de la figura I J.26(a)


a) ¿ Cuántos subgrafos conexos de G licncn cuatro vértices e incluyen un ciclo?
b) Describa el subgra(o G1 (de G) de la parte (b) de la figura como un subgrafo inducido y en
lérminos de la eliminación de los vértices de G.
e) Describa el subgrafo G 2 (de G) de la parte (c) de la figura como un subgrafo inducido y en
lénninos de la eliminación de los vtrticcs de G.
d ) Trace el subgrafo de G inducido por el conjunto de vfflices U = fb. e, d,f. i,j}.
e) Para el grafo G, sea e la aris1a (e,/}. Trace el subgrafo G - e.
f) Sean e1, c 2 las aris1as fa, e}. fo. d}. respectivamente, del grafo G. Trace los siguientes
subgrafos de G: (i)(G - e,) - e:: (ii)(G- iti}- e,; y (iii)G - {e., t-2}.
2. a) Sea G • (V, E) un grafo no dirigido. con G 1::a (V., E,) un subgrafo de G. ¿En qué condicio-
nes C 1 no es un subgrafo inducido de G?
b ) Para el grafo G de la figura l l.26(a). encuentre un subgraío que no sea inducido.
3. a) ¿ Cuántos subgrafos recubridores exis1cn para el grafo G de la figura l 1.26(a)?
b) ¿Cuántos subgrafos recubridores conexos hay en la pan.e (a)?
e) ¿Cuántos subgrafos rccubridores generadores de la parte (a) tienen
i) el v~n.icc a como un vértice aislado?
i j ) el vértice b como un vértice aislado?

(G)

(G,J
. CG2)

rr2 r1'1 ~
b•

9 , 9 i
h j 9 h ' 1 j
<•> (b ) (e)

Figura 11.26

4. Si G; (V. E) es un grafo no dirigido. ¿cuántos subgrafos recubridores de G son 1ambi~n


subgrafos inducidos1
5. ScaG =(V, E)un grafo no dirigido.donde I vi ~ 2. Si todoslossubgrafos inducidosdcGson
conexos. ¿ pode mos identificar el grafo G1
6 . a) E.ncuen1rc dos grafos G = ( V. E ) y G, = (V,, E,>. con u e Yy u, e Vi. donde
• (G-u)=•(G) pero • (G, -u,) >•(G,).
b ) Encuenrre dos grafos G=(V, E) y G 1 = (V., E,), con e E Ey e, E E .. donde

• (G-,)- • (G) pero • (G, -,,)> • (G,~


7. Cada uno de los mulliirafos e1iquctados de la figura 11.27 surge en el anJlisis de un conjumo
de cuatro bloques para el juego de ..locura instantánea". Determine en cada caso si es posible
resoh·cr cJ ac:ertijo.
8 . Encuemrc lodos los grafos no diñgidos no isomorfos (sin lazos) con cua1ro vértices. ¿Cuántos
de estos grafos son conc1tos'!
548 Capítulo 11 Una introducción a la teoría de grafos

(a) (b) (e)

Figura 11 .27

9 . a) ¿Cuántos caminos simples de longitud 4 hay en el grafo completo K ( ! (Recuerde que W1


camino simple como U1 ➔ u 2 ➔ u, ➔ u. ➔ u, se considera ígual al camino simple u, ➔u,
➔ U, ➔ U2-+ u,.)

b) Sean m, n E conm < n. ¿Cuántos caminos simples de longirudm hay en el grafo comple-
to X.?
1 O. Paran 2: 3, sea R. el grafo de rueda con n radios. (Definimos estos grafos en el ejercicio 14 de
la sección 11.1). ¿Es alguno de estos grafos R. isomorfo a un grafo completo?
11. Para cada par dt grafos de la figura 11.28, determine si los grafos son o no son isomorfos.

a V
'

,❖··❖ @:~
h z z
(a) (b)

a U V

·♦ ·

d
e
{!J
f' z
(e)

Figura 11.28
11.2 Subgrafo:., complementos e isomorfismos de grafos 549

12. a) El entrenador Rodríguez debe planear un calendario para los cinco equipos de fútbol de su
liga. Si cada equipo juega contra otros dos, diseñe un calendario posible usando un grafo.
b) Aunque e n la pane {a} sea posible tener calendarios diferentes, muestre que estos calenda-
rios son iguales, excepto por una pennutación de los nombres de los e.quipos.
13. Sea G un grafo no dirigido (~in laz.os) con u vértices y ~ aristas. ¿Cuántas aristas hay en G?
14. Sea/; G 1 -+ G1 un isomorfismo de grafos. Si existe un camino simple de longitud 3 entre los
vértices.a y b de G,, demuestre que en G1 existe un camino simple de longitud 3 entre los
v~nicesf(a) y f(b).
15. a) Si G., G1 son grafos no dirigidos (sin lazos), demuestre que G,. G1 son isomorfos si y sólo
si Ci, y ~! son isomorfos.
b) Detenninc si los grafos de la figura 11.29 son isomorfos.

m
a . ,,

g e y u

X V

e w
Figura 11.29

16. a) Sea G un grafo no dirigido con n vé rtices. Si Ges isomorfo a su propio complemento e;.
¿cuintas aristas debe tener G? (Este grafo se conoce como aurocompl~mentario.)
b) Encuentre un ejemplo de un grafo autocomplementario con cuatro vértices y otro con cinco
vértices.
e) Si G es un grafo autocomplementario con n vén.ices, donde n > l .demue.strc que n = 4k o
n = 4k + l, para algúna J.: E z·.
17. Sea G un ciclo simple den vértices. Demuestre que G es autocomplementario si y sólo si
n = 5.
18. a) Encuentre un grafo G tal que G y G sean conexos.
b) Si Ges un grafo con n vértices. paran ~ 2 y G no es conexo, demuestre que Ges conexo.
19. a) Extienda la definición 11.13 a grafos dirigidos.
b ) Determine, salvo isomorfismo, todos los grafos dirigidos (sin lazos) de tres vértices.
e) Detennine si los grafos dirigidos de la figura 11.30 son isomorfos.

~1Jt
e figura 11 .30
)50 capitulo 11 Una introducción a la teoria de grafos

20. a) ¿Cuántos subgrafos H =(V.E) de K. satisfacen I vi = 3? (Si dos subgrafos son isomorfos
pero tienen conjuntos diferentes de vértices. considttelos distintos.)
b) ¿Cu:l.ntossubgrafos H = (V, E ) de X. satisfacen Ivi = 4?
e) ¿Cuintos subgnúos licne K.?

11 .3
Grado de un vértice: recorridos
y circuitos eulerianos

En el ejemplo l I .9 descubrimos que el número de aristas incidentes con un vértice podóa


utilizarse para mostrar que dos grafos no dirigidos no son isomorfos. Ahora veremos que
esta idea puede t<xlavfa ayudarnos más.

>efinici6 n 11.14 Sea G un grafo o multigrafo no dirigido. Para cualquier vértice u de G, e l graao de v. que
se denota grad(u), es e l número de aristas en G que son incidentes con u. En este caso, un
lazo en un vértice u se considera como dos aristas incidentes en u.

Para el grafo de la figura 11.31, grado(b) = grado(d) = grado(/) = grado(g) = 2, grado(c)


= 4, grado(e) =O y grado(h) = l. Para el vértice a se tiene que grado(a) = 3 ya que
contamos el lazo dos veces. Como h ticne~grado 1, se le llama vértice colgante.

Mediante la idea de grado de un vértice, se llega al siguiente resu1tado.

l:OREMA 11.2 Si G= (V, E) es un grafo o multigrafo no dirigido, entonces L_,grado(v) = 21 E 1


Oemostr•ción: Al considerar cada arista {a, b } del grafo G, encontramos que la arista
contribuye con una unidad a grado(a) y a grado(b) y. en consecuencia, con dos unidades a
r_,grado(v). Así. 21 El cuenta grado(V), para todo" E Vy r _,grado(v) = 2IEI.

Esce ceorema proporciona un indicio sobre el número de vértices de grado impar que
pueden existir en un grafo.
11.3 Grado de un vertice: recorridos y circuitos eulerianos 551

COROLARIO 11.1 Para cualquier grafo o multigrafo no dirigido, el número de vértices de grado impar debe
ser par.
Demostración: La demostración se deja al lector.

En el siguiente ejemplo se aplica el teorema ll.2.

Un grafo (o multigrafo) no dirigido donde los vénices tienen el mismo grado se de nomina
grafo regul.ar. Si grad(t>) ~ k para todos los vénices u, entonces el grafo es k•regular. ¿Es
posible tener un grafo 4•regular con 10 aristas?
Delteorema 11.2, 2 IEl I
=20= 4 vi, por lo que tenemos cinco vértices de grado 4. La
figura l l.32 proporciona dos ejemplos no isomorfos que satisfacen lo solicitado.
Si deseamos que cada vértice tenga grado 4, con 15 aristas en el grafo. entonces 2 IEl
= =41 1.
30 V por lo que resulta imposible tener dicho grafo.

(a) (b)

Figura 11.32

Ahora veremos la razón por la que Euler desarrolló la idea de grado de un vénice: para
resolver el problema de los siete puentes de KOnigsberg.

Los siete puentes de KOnigsberg. Durante el sigloxvm, la ciudad de KOnigsberg (en Prusia
Oriental) estaba dividida en cuatro zonas (incluida la isla de Kneiphof) por el río Pregel.
Siete puentes comunicaban estas regiones, como se muestra e n la figura I I .33(a). Se decía
que los habitantes hacían paseos dominicales tratando de encontrar una forma de caminar
por la ciudad cruzando cada puente exactamente una vez y regresando al punto donde se
había iniciado el paseo. ·
Con e l fin de detenninar si existía o no dicho circuito. Eule r representó las cuatro zonas
de la ciudad y los siete puentes con el multigrafo que se muestra en la figura J I.33(b).
Encontró cuatro vénices con grado(a) = grado(c) = grado(d ) = 3 y grado(b) = 5. También
enconcró que la existencia de ta1 circuito dependía del número de vé rtices de grado impar
del grafo.
552 capítulo 11 Una introducción a la teoría de grafos

(a)

Figura 11.33
t7·
(b)
e

Antes de demostrar el resultado gene ral, daremos la siguiente definición.

Definición 11.15 ScaG =(V.E) un grafo o multigrafo no dirigido sin vénices aislados. Entonces G tiene un
circuito euluiano si existe un circuito en G que recorre cada arista del grafo exactamente
una vez. Si existe un recorrido abierto de a aben G que recorre cada arista de G exacta-
mente una vez, este recorrido se llamará recorrido euleriano.

El problema de los siete puentes quedará resuelto si caracterizamos los grafos que tie-
nen un circuito culcriano.

TEOREMA 11.3 Sea G = (V. E) un grafo o multigrafo no dirigido sin vértices aislados. Entonces G tiene un
circuito euleriano si y sólo si G es conexo y todo vértice de G tiene grado par.
Demostración: Si G tiene un circuito culeriano, entonces para cualquier a, b E V existe un
recorrido de a a b; a saber, la parte del circuito que comienza en a y termina en b. Por lo
tanto, dc1 teorema 11. l se sigue que G es conexo.
Sea e el vértice inicial del circuito eule riano. Para cualquier otro vértice u de G. cada
vez que_ el circuito llega a U entonces partirá de ese vértice. Así, el circuito pasa por dos
aristas (nuevas) incidentes con u o por un lazo (nuevo) en u. En cada caso, se contribuye
con dos unidades a grad (U). Como u no es el punto inicial y cada arista incidente a u se
recorre una sola vez. obtenemos dos unidades cada vez que el circuito pasa por U, de modo
que grad (u) es par. Para el vértice inicial e, la primera arista dc1 circuito debe ser distinta
de la última, y como cualquier otro paso por e produce dos unidades para grad(c), tene-
mos que grad(c) es par.
Recíprocamente, sea G un grafo conexo tal que todos los vértice s tienen grado par. Si el
número de aristas de Ges l o 2, entonces G debe ser como los grafos de la figura 11.34.
Los circuitOS eulerianos son inmediatos en estos casos. Ahora procederemos por inducción
y supondremos que e1 resultado es válido para todas las sítuaciones con menos den aristas.
Si G tienen aristas, seleccionamos un vértice e en G como punto inicial para construir un
circuito euleriano. Como el grafo (o multigrafo) Ges conexo y cada vénice tiene grado
par. podemos construir al menos un circuitoC que contenga ac. (Verifique esto examinan-
do el recorrido más largo en G que comienza en c.) Si el circuito contiene todas las aristas
11.3 Grado de un vértice: re<:orridos y circuitos eulerianos 553

Figura 11 .34

de G. hemos terminado. Si no, eliminamos las aristas del circuito de G. asegurándonos de


eliminar cualquier vénice que haya quedado aislado. El subgrafo rcsrante K tiene todos los
vértices de grado par, pero puede no sér conexo. Sin embargo, cada componente de K es
conexo y tendrá un circuito euleriano. (¿Por qué?) Además, cada uno de estos circuitos
eulerianos tiene un vértice que está en C. En consecuencia. pode mos pan.ir de e y recorrer
C hasta llegar al vértice e, que está en el circuito euleriano de una componente C, de K.
Entonces se recorre e ste circuito euleriano y al regresar a c., continuamos en C hasta
llegar a un vértice c1 que está en el circuito eulerfano d~ la componente C1 de K. Como
el grafo Ges finito, podemos continuar este proceso hasta construir un circuito eulerian<;)
para G.

Si Ges conexo y no tiene demasiados vértices de grado impar, podemos hallar al menos
un recorrido euleriano en G.

COROLARIO 11 .2 Si Ges ua grafo o multigrafo no dirigido sin vértices aislados, e ntonces podemos construir
un recorrido euleriano en G si y sólo si Ges conexo y tiene exactamente dos vénices de
grado impar.
Demostración: Si Ges conexo y a y b son los vértices de G de grado impar, añadimos una
arista adicional { a, b} a G. Ahora tenemos un grafo G 1 conexo tal que todos sus vénices
son de grado par. Por lo tanto, G1 tiene un circuito euleriano C; cuando eliminamos la
arista {a, b} de C. obtenemos un recorrido eurcliano para G. (Así, el recorrido euleriano
comienza en uno de los vénices de grado impar y termina en otro vértice de grado impar.)
Dejamos al lector la demostración de~aso contrario

Si regresamos ahora al problema de los siete puentes de KOnigsberg. nos daremos cuenta
de que la figura l l.33(b) es un multigrafo conexo, pero tiene cuatro vértice s de grado
impar. En consecuencia. no tiene ni un recorrido euleriano ni un circuito euleriano.

Ahora que hemos visto que la solución de un problema del siglo xvm fue el inicio de la
teoría de grafos. ¿existe algún contexto contemporáneo donde apl icar lo aprendido?
Para responder esta pregunta (en forma afirmativa). estableceremos la versión di-
recta de l teorema t 1.3. Pero primero hay que extender la idea i nicial de l grado de un
vértice.
554 Capítulo 11 Una introducción a la teoria de grafos

Definición 11.16 Sea G =(V, E ) un grafo o rnultigrafo dirigido. Para cualquier u E V,


a) El grado de entrada de u es e) número de aristas deGque llegan au y se denotaCQD
ge(u).
b) El grado de salida de u e s el número de aristas de G que parten de u y se denota CQD
gs(u).

Si eJ grafo o multigrafo dirigido tiene uno o más lazos. cada lazo de un vértice dado v
contribuye con una unidad age(u) y a gs(u).

Los conceptos de grado de entrada y grado de salida para vértices nos llevan al sigujea.
te teorema.

TEOREMA 1 1.4 Sea G = (V, E) un grafo o mulitgrafo dirigido sin vé nices aislados. El grafo G tiene un
circuito euleriano dirigido si y sólo si Ges conexo y ge(u) =gs(u) para todo u E V.
Demostración: Dejamos la demostración de este teorema para que el lector la realice en el
ejercicio 24(a).

En este momento consideraremos una aplicación del teorema 11.4. Este ejemplo se
basa en un problema de telecomunicaciones planteado por C. L. Liu en las páginas 176-
178 de la referencia (24).

En la figura ll.35(a) tenemos la superficie de un disco de rotación dividido en ocho secto-


res de iguaJ área. En la parte (b) de la figura hemos colocado materia) conductor (sector
sombreado y el círculo interno) y no conductor (sectores sin sombrear) en el disco. Cuan- .
do las tres tenninales (mostradas en 1a figura) hacen contacto con los tres sectores dados~
el material no conductor no produce un flujo de corriente y aparece un I en la pantalla de
un dispositivo digital. Para los sectores con material conductor, se produce un flujo de
corriente y aparece un O en la panta11a. Si el disco se rotara 45 grados (en el sentido de las
manecillas del reloj), en la pantalla se leería l IO(de arriba a abajo). Así, podemos obtener

<•> (b)

Figura 11.35
11.3 Grado de un vértice: recorridos y circuitos eulerianos 555

al menos dos (por ejemplo, 100 y 110) de las ocho representaciones binarias de 000 (para
0) a 111 (para 7). Pero, ¿podemos representar los ocho nllmeros binarios al rotar el disco?
¿Se puede extender el problema a las 16 representaciones binarias de cuatro bits de C()()O
a 1111. y tal vez generalizar aún más los resultados?
Para responder esta pregunta del problema de la figura. se construye un grafo dirigido
G: (V, E) donde V: (00, 01. JO, 11) y E se conSlruye como sigue: Si b,b,, b,b, E V,
se traza la arista (b 1b 2, b 2b,). Esto produce el grafo dirigido de la figura l l .36(a), donde
1E 1 : 8. Se ve que este grafo es conexo y que para todo u E V. gt(U) : gs(u). En conse-
cuencia. por el teorema 11 .4, tiene un circuito euleriano dirigido. el cual está dado por

100 000 001 010 101 011 lit


<.'. 10-oo- 00- 01-10- 01---->11 - 11)

110

Aquí la etiqueta de la arista~ = (a, e), corno se muestra en la parte (b)dc la figura 11.36,
es la sucesión de tres bits x 1x~3• donde a = x 1x 2 y e =x~,. Como los vértices de G son las
cuatrosucesioncsdistintasdedos bits 00, 01, IOy 11, las etiquetas de lasocho aristasdcG
determinan las ocho sucesiones de tres bits distintaS. Así mismo, cualesquiera dos etiquetas
de aristas consecutivas en el circuito euleriano son de la fonnay 1J:Y, y J:))Y~·

000

01 10 01 10

111
(a) lb)

Figura 11.36

Partie ndo de la etiqueta de arista 100. a fin de obtener la siguiente etiqueta. 000,
concatenamos el último bit de 000, es decir, O. a. la cadena IOO. Entonces. la cadena resul•
tante 1000 proporciona 100 ( !.QQO) y 000( IQQQ). La siguiente etiqueta de arista es 001, por
lo que concatenamos e l 1 (el último bit en 001 ) a nucstr.1 c~cna actual 1000 y obtenemos
10001, lacual pcoporciona las tres sucesiones de tres bits disiintas IOO(!QQOl),OOO(IOQQ!)
y 001 (10001). Continuando de esta forma, llegamos .1 la sucesión de ocho bits 10001011
(donde el illÍimo I see,ivutlvt) y estos ocho bits se acomodan en los sectores del disco gi•
556 (.aµftulo 11 Una introducción a k, teoría de grafos

Figura 11.37

ratorio como en la figura 11.37, A partir de la cual se obtiene el resultado de la figura


11.35(b).AI rotar el disco de la figura l l.35(b), se obtienen las ocho sucesiones de tres bits
100, 110,111 ,011, 101, 010,00ly 000.

Para cerrar esta sección, no gustaría llamar la atención de l lector a la referencia (26] de
Anthony Ralston. Este artículo e s una buena fuente para obtener más ideas y generali:z.
ciones relacionadas con el problema analizado en el ejemplo 11.13.

EJERCICIOS 11.3 1. Determine I vi


para los siguícntes grafos o multigrafos G.
a) G tiene nueve aristas y todos los vértices tienen grado 3. -
b) Ges regular con 15 aristas.
e) G tiene IO aristas con dos vértices de grado 4 y los demás de grado 3 .
2. Si G = (V, E) cs un grafo conexo con IEl = 17 y grad(u) 2: 3 para todo u E V. ¿cuáles el valor
máximo para J vi?
3. Sea G = (V.E) un grafo conexo no d irigido.
a) ¿Cuál ese! mayor valor más grande posible para I vi si IEl =19 y grad(\)) ~ 4 para todo
\)E~
b) Trace un grafo para mostrar cada caso posible de la parte (a}.
4. SeaG =(V.E) un grafo no dirigido conexo sin lazos. que sea 3-regular(esto es, cada vénicede
Gtienegrado3).Si IEl=2 lvl - 6, ¿cuántovalcn lvl y IEI'
5. Sean G 1 =(V1 • E 1) y G 2 =( Vi, E:) los grafos no dirigidos conexos sin lazos d e la figura 11.38.

~~
g

Figura 11.38
h Y z
11.3 Grado de un vértice: recorridos y circuitos eulerianos 557

a) Determine lv, I. IE,l. lv,I IE,I .


y
b) Encuentre el grado de cada vértice de V1 • Haga lo mismo para cada vfrticc de V?.
e) ¿Son isomodos los grafos G 1 y G1?
6. a) Sea V = {a. b, e, d. t,f} . Dibuje tres grafos no dirigidos sin lazosG1 = (V, Ea). G¡= (V, Ei)
yG,= (V.E,)talesque, en los tres grafos. grad(a) = 3. grad (b) = grad(c) = 2 y grad(d)
= grad(e) = grad(/) = l.
b) ¿Cllmtos de los grafos de la parte (a) son conexos?
7. Sea G = (V, E) el grafo no dirigido de la figura 11.39.
a) Trace ~. e l complementario de G.
b) Trace el subgrafo de G inducido por los vértices (de G) de grado 3.
s. ScaG =(V.E) un grafo no dirigido sin lazos donde V= {U1, 'l>z, uh . ... u10}. Si grad(u,) = 2.
grad (u,) = 3, grad (u,) = 3, grad (U,) = 5, grad (U,) = 1, grad (u,) = 2, grad (U,) = ,5.
grad(Ui) = 2, grad (~)= 3. grad(u 1o) = 2, dcterminegrad(ujen el complementario G, para
todo 1 5 i !:> 10.
9. SiGe sungrafonodirigidoconn vérticcsy t aristas, scaó = míri..ev{grad <u)} ysca.6.:m~"
{grad(u)}. Demuestre que 6 s (t-/n) s 6..
1 0. SeanG = (V,E),H = (V' E' )grafos no dirigidos y/: V➔ V"un isomorfismo entre estos grafos,
a) Demuestre queJ- 1: V'-+ Ves también un isomorfismo para G y H.
b) Si a E V, demuestre que grad(a) (en G) es igual a grady(a)) (en H ).
1 1. Para todo k E Z.. donde k ;;:: 2. demuestre que existe un grafo no dirigido conexo sin lazos
G=(V, E) donde I v i = 21< y grad(u) = 3 para todo u E V.
12. Demuestre que para cualquier n E z· existe un grafo no dirigido sin lazos G = (V.E) tal que
1V 1 = 2n y que tiene dos vértices de grado i para todo 1 :S i S n.
13. Complete las demostraciones de los corolarios 11. 1 y 11.2.
14. Sea k un entero positivo fijo y sea G = (V, E) un grafo no dirigido sin lazos tal que grado(\>) i!:
k para todo u E V. Demuestre que G contiene un camino simple de longitud k.
15. a ) Explique por qué no es posible dibujar un grafo no dirigido conexo sin lazos oon ocho
vértices, tal que los grados de los vértices sean 1. 1, 1. 2, 3, 4, 5 y 7 .
b) Dé un ejemplo de un multigrafo no dirigido conexo sin lazos con ocho vértices. donde los
grados de los vénices son I, 1. I, 2. 3. 4. 5 y 7.
16. a) Encuentre un circuito euleriano para el grafo de la figura 11 .40.
b) Si se elimina la arista {d, t} de este grafo. encuentre un recorrido euleriano para el subgrafo
resultante.
17. Determine los valores den para los que el grafo completo K~tiene un circuito e uleriano. ¿Para
cuáles n tiene K~ un recorrido culeriano pero no un circuito euleriano?

ªIV\
d~ e

f
Figu,a 11 .39 Figura 1 1.40
558 capitulo 11 u na introducción a la teoríadegrafos.

18. A panir de l grafo de la figura J1.33(b). ¿cuál es el menor número de puentes que deben c1imi-
narse para que el subgrafo resulcantc tenga un recorrido euleriano. pero oo un circuito eulcriano1
¿Qué puente(s) deben eliminarse'?
19. Al visitar el museo de ciencias, Pablo y David intentan resolver si podñan pasar por las siete
habitaciones y el pasillo que las rodea sin cruzar ninguna puerta más de una vez. Si comienzan
des.de la posición del pasillo marcada con una estrella en la figura 11.41. ¿pueden lograr su
objetivo?

L * ....L _L_J
Figura 11.41

20. Sea G un grafo no dirigido sin lazos con n (~3) vértices. Si G tiene un solo vénicc de grado
par, ¿cuántos vértices de G tienen grado par?
21. a) Encuentre la longitud mbima de u n recorrido en
i) K,, ü) K,, W) K,,
b) Encuentre la longi rud máxima de un circuito en
i) Kt. Ü) K, W) K10 iv) K2,,,,nez· .
22. Sea G = (V. ~ un grafo dirigido tal que [ v ! = n y IEl = ~- ¿Cuáles son los valores para
L. .
v&e(U) Y 2.....vgs(u)?
23. Demuestre que para cualquier grafo o multig rafo dirigidoG = ( Y, E), Lvgs(v) = Lvge(U).
24. a) Sea G: (Y, E) un grafo o multigrafo dirigido sin vMiccs aislados. Demuestre que G tiene
un circuito euleriano dirigido si y sólo si G e s conexo y gs(u) = ge(U) para todo u E Y.
b) Encuentre un circuito culeriano dirigido para el grafo de la figura 11.42.
e) Un grafo dirigido esjuertemenreconao si existe un camino simple dirig ido dea a b para
todos los vértices a, b. cuando a 'fa b. Demuestre que si un grafo dirigido tiene un cicuito
euleriano dirigido, entonces es fuertemente conexo. ¿Es cierto el recíproco?

Figura 11 .42

25. Sea G un grafo dirigi~ c onn vértices. Si el grafo no dirigido asociado para Ges K., demuestre
que L,..dl [gs(\>))1 = l......v[ge{u ))2
11 .3 Grado de un vértice: recorridos y circuitos eulerianos 559

26. Si G =(Y.E) es un grafo o multigrafo dirigido sin vértices aislados. demuestre que G tiene un
recorrido euleriano dirigido si y sólo si (i) Ges conexo; (ii) g.s(V) z gt(o) para todo virtice n
excepto dos v&ticesx. yde V; y, (iii) g.s(.r) = gt(.r) + l. gt'(y) = gs(y) + l.
27. Sea Y = {000, 001, 010• . .• 110,111). Para cualquier sucesión de cuatro bits bib~J,,, trace
una arista del elemento b 1b:l>J al elemento b,l:,,J,, en V.
a) Trace el grafo G = (V.E) descrito.
b ) Encuentre un circuito euleriano dirigido para G.
e) Distribuya ocho ceros y ocho unos de modo uniforme alrededor del borde de un disco que
gira en el sentido de las manecillas del reloj, de modo que estos 16 bits formen una sucesión
circular tal que las subsucesiones (consecutivas) de longirud4 proporcionen las rcpre5enta-
ciones binarias de O. l. 2, ...• 14. IS, en algún orden.
28. Distribuya nueve ceros, nueve unos y nueve doses de modo uniforme alrededor del borde de
un disco que gira en el sentido de las manecillas del reloj, de modo que estos 27 símbolos
formen una sucesión circular tal que las subsucesiones (consecutivas) de longirud 3 proporcio-
nen las representaciones ternarias (base 3} de O. 1, 2 .. .. • 25, 26 en algún orden.
29. SeaG = ( V, E) un grafo no dirigido conexo sin lazos con I vi 2: 2. Demucstrt:qucG tienedos
v~rtices u, w donde grado(u) = grado(w).
30. Carolina y Ric-ardo van a una fiesta con otras ues parejas. En esta fiesta hubo muchos apreto-
nes de mano. pero ( 1) nadie estrechó la mano de su pareja; (2) nadie estrechó su propia mano;
y (3) nadie dio la mano~ de una vez a otra persona .Antes de s.aJir de la fiesta, Carolina
preguntó a las ottas siete personas a cuántas personas habían dado la mano. Ella recibió una
respuesta difer~nte de cada uno. ¿Cuántas veces dio Carolina la mano en esta fiesta? ¿Cuántas
veces lo hizo Ricardo?
31. Si G = (Y, E) es un grafo no dirigido con I vi =n y IEI = k.. usamos las siguientes matrices
para representar G.
Sea V= (0 1, U2, . ... u. }. Definimos la matriz de adyacencia A = (a;¡).,. . dondeaiJ = 1 si
{t>¡, \l¡} E Ey a, = O en otro caso.
Si E= {e1 ,e1 • • • • • e 1 }. la nu:itrizde incidencia I es la matriz n x k (b¡,). 11 t tal quebiJ = 1 si
e,
\l; es un vértice en la arista y b¡¡ = O en olfO caso.
a} Encuentre las matrices de adyacencia e incidencia asociadas con el grafo de la figura 11 .43.
b) Calcule A 1 y use las operaciones booleanas donde O + O = O, O + 1 = 1 + O 1 + 1 1 y O -= = 2
0 = 0 - l = 1 • 0=0.1 - 1 = l para demostrar que el elemento de la fila iy lacolumnajdeA
es 1 s i y sólo si existe un camino de longitud dos entre e l i-ésimo y f-ésimo vértices de- V.
2
e) Si calculamos A con la suma y multiplicación ordinarias, ¿qué indican de G los elementos
de la matriz?
d} ¿ Cuál es la suma de cada columna de A? ¿ Por qué?
e) ¿Cuál es la suma de cada columna den ¿Por q~?

Figura 11.43
,60 Capítulo 11 Unaintroducóóna lateoriadegrafos

11.4
Grafos planos

En un mapa de carreteras, las líneas que indican las carreteras y autopistas se intersecan
por lo general solamente en puntos de confluencia o en poblaciones. Pero hay ocasiones
en que las carreteras parece n intersecarse cuando una se localiza sobre otra. como en el
caso de un paso elevado. En este caso, las dos carreteras están en diferentes niveles o
planos. Este tipo de situación nos Lleva a la siguiente definición.

>efinición 11 .17 Un grafo (o multigrafo) G es plano si podernos dibujar Gen el plano de modo que sus
aristas se intcrsequc n sólo en los vértices de G. Este dibujo de G se conoce como una
inmersiónt de G en el plano.

Los grafos de la figura 11 .44 son planos. El primero es un grafo 3-regular, ya que cada
vé nice tiene grado 3; es plano pues ningún par de aristas se intersccan, excepto en los vérti-
ces. El grafo (b) parece un grafo no plano~ las aristas {x, z} y {w, y} se cruzan en un punto
distinto de un vértice. Sin embargo, podemos lra.Zar nuevamente este grafo como se mues~
traen la parte (c} de la figura. En consecuencia. K.. es plano.

.
12])
X

b
(a)
A
Figura 11 .44
e
-~
z
Cb)
y
(C)
z y

Al igua1 que K,, también K1, K 2 y K 3 son planos.


En la figura 11.45 se mue stra un intento de representar K~ en el plano. Si Ks fuera plano,
entonces cualquier i nmersión tendría que contener el pentágono de la parte (a) de la figu-
ra. Como un grafo completo tiene una arista por cada par de vértices distintos. añadimos la
arista {a. e} como se muestra en la parte (b). Esta arista está contenida completamente
dentro del pentágono de la parte (a). (Podríamos haber dibujado la arista en la región
exterior determinada por el pentágono. En los ejercicios, pediremos al lector la demostra-
ción de que en este caso se llega a la misma conclusión.) En la parte (c), se añaden las

t El tm?lioo embedding también se 1radue1: como ~cncajc".(N del E.)


11.4 Grafos planos 561

<•> (b)

~ e

~
(<) (d)

Figura 11.45

aristas {a, d} , {c. e} y {b, e}. Ahora examinaremos los vénices by d. Se necesita ]a arista
(b. di para obtener K,. El vtrtieedestá dentro de la región formada por el ciclo de aristas
(a. e}, (e,,} y{,. a}, mientrasquebestá fuera de laregión.Asf. al dibujarla arista {b, d),
hay que interscc:ar una de las aristas exisrentes al menos una vez. como lo muestran las
aristas punteadas en la parte (d). En consecuencia. K, no es plano. (Como esta demostra•
ción recurre al diagrama. definitivamente carece de rigor. Sin embargo, más adelante de•
mostraremos por otro método que Ks no es plano.)

Antes de caracterizar todos los grafos no planos, nccesitamOS examinar Olra clase de grafos.

Definición 11 .18 Un grafo G: ( V, E) esbipartitosi V: V, U v,. v, n v, : Oy cadaaristadeGesde la fonna


{a , b} con a E V1 y b E V2• Si cada vénice de V1 está unido con los vénices de V2~ se tiene
un grafobipanito completo. En este caso. si I v, I == m. 1Vl 1 =n, el grafo se dcnOla con
K~,-

La figura 11 .46 muestra dos grafos bipartitos. El grafo de la pane (a ) satisface la defi-
nición para V, : (a. b I y V, : (c. d. , ) . Si se aJladen las aristas ( b, d) y { b. e). el resultado
es e] grafo bipanitocompleto K2.~. que es plano. El grafo (b) de ta figura es K,,)- Sea V1 ==
{h1, h1 • hl} y V2 = {ui, u 1, ul} : interpretamos V1como un conjunto de casas y V2 como un
conjunto de servicios. Entonces KJ.J es el grafo de servicios. ¿.Podemos UJ"lir las casas con
los servicios. evitando que haya superposición de las líneas de servicio? En la figura l l .46(b)
parece que eslo no es posible y que KJ.., no es plano. (Una vez más, deducimos la no
planaridad de un grafo a panir de un diagrama. Sin embargo, más adelante , en el ejemplo
11 .19 de esta sección, verificaremos por otro método que K ). J no es plano.)
562 Capítulo 11 Una introduc:C':i6n a la teoría de grafos

(a) (b)

Figura 11 .46

Veremos que cuando estemos trabajando con grafos no planos. K, o K1.3 serán el origen
del problema. Sin embargo, antes de establecer el resultado general. necesitamos introou-
cir una última idea.

Definición 11 .19 Sea G =(V.E ) un grafo no dirigido sin lazos, tal que E ,f:. f. Una subdivisión elemental deG
resulta cuando eliminamos una arista e= {u. w } de G y entonces las aristas {u, u }, {u. w} se
añaden a G- e, donde"~ V.
Los grafos no dirigidos sin lazos G 1 = ( Vi, E 1) y G 2 = (V2, E2) son homeomorfos si son
isomorfos o si ambos pueden obtenerse del mismo grafo no dirigido sin lazos H por una
sucesión de subdivisones elementales.

a) SeaG ; (V, E ) un grafo no dirigido sin lazos con I El ;;,,


l. SiG'se obticnede Gpor
una subdivisión elemental, entonces el grafo G' ; (V', E') sa1isface Iv•I ; 1vi + 1
y IE'I ; IEI + l.
b) Consideremos los grafos G, G 1, G, y G3 de la figura 11.47. En este caso, G 1 se
obtiene de G por medio de una subdivisión elemental: se elimina la arista {a. b} de
G y se añaden las aristas {a, w} y {w, b}. El grafo G2 se obtiene de G mediante dos
sutxtivisones ele mentales. Por lo tanto, G 1 y G2 son homeomoños. Así mismo, ~

.[x]. .[)(].
(G) (G,) (G2)

.[S?J.
(G1)


(a)

Figura 11.47
d •
(b)
d TXT
Y


(e)
C X

d
y

e
Id)
. WZC X

d
11 .4 Grafos planos 563

puede obtenerse de G con cuatro subdivisones elementales, por lo que G~ es


homeomorfo a G1 y G2 •
Sin embargo, G, no puede obtenerse de G2 (o G2 de G 1) por una sucesión de
subdivisiones elementales. Además, el grafo G3 puede obtenerse de G 1 o G2 por una
sucesión de subdivisones elementa]es: seís (de tales sucesiones de tres subdivisiones
elementales) para G 1 y dos para G2• Pero ni G 1 ni G1 pueden obtenerse de 0 1 por una
sucesión de subdivisiones elementales.

Podría pensarse que los grafos homeomorfos son isomorfos excepto. posiblemente, por
los vértices de grado 2. En particular, si dos grafos son homeomorfos, son simultáneamen-
te planos (o no planos).
Estos antecedentes llevan al siguiente resultado.

EOREMA 11.5 Teorema de Kuratowski. Un grafo no es plano si y sólo si contiene un subgrafo que es
homeomorfo a K, o K 3. 3•
Demostración: (Este teorema fue demostrado por primera ve z en 1930 por el matemático
polaco Kasimir Kuratowski.) Si un grafoG tiene un subgrafo homeomorfo a K, o K l.S, está
claro que G no es plano. Sin embargo. el recíproco de este teorema es más difícil de
demostrar. ( Una demostración de esto aparece en el capítulo 8 de C. L. Liu [24].)

Demostraremos el uso del teorema de Kuratowski en el siguiente ejemplo.

La figura l l .48(a) es un grafo conocido llamado grafo de Petersen. La parte (b) de la


figura proporciona un subgrafo del grafo de Petersen que es homeomorfo a K3- 3• (La figura
11.49 muestra cómo se obtiene el subgrafo de Ku por una sucesión de cuatro subdivisiones
elementales.) Por Jo tanto, el grafo de Peterscn no es plano.

•ii;~f ' h
g b ~ •
,:

e
h
f b
d e 9
(al lb)

Figura 11.48

Cuando un grafo o multigrafo es plano y conexo obtenemos la siguiente relación des•


cubierta por Euler, para la cual necesitamos contar el número de regiones determinadas
par un grafo o multigrafo conexo plano, el número (de estas regiones) se define sólo
564 C.apltulo 11 Una introducción a la teoria de grafos

Figura 1 1.49

cuando se tiene una inmersión plana del grafo. Por ejemplo, la inmersión plana de K, en Ja
pane (a) de la figura 11.50 demuestra cómo esta representación de K, determina.cuatro
regiones en el plano: tres de área finita. Ri. R1 y R 3, y la región infinita R,. Cuando obser·
vamos la figura 11.SO(b) podría pensarse que K, determina cinco regiones, pero esta repre-
sentación no es una inmersión plana de K,,. Así, el resultado de la figura I l.50(a) es el
único que nos interesa.

li® -~·
d
(a) ••
e d
(b)
Figura 11.50

'EOREMA 11 .6 SeaG= (V.E) un grafo omultigrafo plano conexo con I vi =U y JEl =•·
Searel número
de regiones en el plano determinadas por una inmersión (o representación) plana de G~
una de estas re giones tiene un área infinita y se conoce como región infinita. Entonces
u-e+r=2.
11.4 Grafos planos 565

Cal
.
• o
(b) b (e)
e
Demostración: La demostración se hace por inducción sobre e. Si e -= O o
Figura 11.51

1, entonces Ges
isomorfo a uno de los grafos de la figura 11.51. El grafo de la parte (a) tiene u : 1, e: Oy
r = l;entoncesu - e+ r = 1-0t 1 = 2. Para el grafo(b), U= l, e = 1 yr = 2. El grafo (c)
tiene u= 2, e -= 1 y r: l. En ambos casos, u - e + r: 2.
Ahora, sea k E N y supongamos que el resultado es verdadero para cuaJquicr grafo o
multigrafo plano conexo con e aristas, donde O :s. e :S k. Si G : (V, E) es un grafo o
multigrafo plano conexo con u vénices, r regiones y e -= le+ 1 aristas, sean a, b E V con
{a. b) E E. Considere el subgrafo H de G obtenido al el iminar la arista {a, b) de G. (Si
Ges un multigrafo y {a, b} es una de un conjunto de aristas entre a y b, entonces la
eliminamos sólo una vez.) En consecuencia. se puede escribir H: G - {a, b} o G-= H +
{ a. b}. Consideremos los dos casos siguientes, que depende n de si Hes conexo o disconexo.
Caso 1: Los resultados de las panes (a), (b), (e) y (d)de la figura 11.52 muestran cómo un
grafo G puede obtenerse de un grafo conexo H cuando se dibuja el lazo (nuevo) {a, a}
como en las partes (a) y (b) o cuando la arista (nueva) {a, b} une dos vénices distintos en
H como en las partes (c) y (d). En todas estas situaciones, H tiene u vértices, k aristas y
r - l regiones, ya que una de las regiones de H se divide en dos regiones para G. La
hipótesis de inducción aplicada al grafoH indica que u - k + (r- 1)-= 2 y de esto se sigue
que 2 = u - (k + l )+ r=u-e + r.Así, en este caso e l teorema de Euleres cieno para G.
Caso 2: Ahora consideremos el caso en que G - {a.b} : H es un grafo disconexo [como
se muestra en la figura l 1.52(e) y (f)]. En este caso, H tiene u vénices, k aristas y r regio-

v-b)~ ~
Cal
,
,
'
\
--------

(bl ',, _ _ _ _,/


,
,'

(el
~ b

(dl

Figura 11.52
(e)
~ '•b
'0-~
1n
566 (apítulo 11 Una introducción a la teoría de grafos

nes. Así mismo, H tiene dos componentes H 1 y H2 , donde H, tiene.u; vértices, e; aristas y,,
regiones, para i = 1, 2. {La parte (e) de la figura 11.52 indica que una componente podría
ser un solo vénice aislado.] Además, U 1 + Uz :U, e 1 + e1 =k(• e- l) y r 1+ r2 = r + 1 ya que
H I y H2 determinan, cada uno. una región infinita. Cuando se aplica la h.ipótesis de inducción
a H 1 y H 2 vemos que
u1 - e, +r1 = 2
En consecuencia, (u1 + Uz)- (e 1 + e?)+ (r1 + r-0 = u-(e - 1) + ( r+ I ) = 4, y de esto se sigue
que u - e + r = 2, y así establecemos el teorema de Euler para G.

El siguiente corolario del teorema 11.6 proporciona dos desigualdades relacionadas


con el número de aristas en un grafo plano conexo sin lazos G con ( 1) el número de
regiones determinadas por una inmersión plana deG; y (2) el número de vértices de G. Sin
embargo. antes de examinar este corolario, observemos la siguiente idea. Para cualquier
región R en una inmersión plana de un grafo o multigrafo (plano), el grado de R. que: se
denota con grad(R), es el número de aristas recorridas en un camino cerrado (el más cono)
por (las aristas de) la frontera de R. Si G =(V, E)es el grafo de la figura 1l.53(a), entonces
esta inmersión plana de G tiene cuatro regiones donde
grad(R1) ; 5. grad(R,) ; 3, grad(R,) ; 3, grad(R..) ; 7.
(En este caso. grad(R,) =7, como lo detennina el camino cerrado a ➔ b ➔ g ➔ h ➔ g ➔
f ➔ d ➔ a.J La pane (b) de la figura muestra una segunda inmersión plana de G, otra vez
con cuatro regiones, y en este caso
grad(R,) ; 4, grad(R,) ; 3, grad(R,) ; 5, grad(R1) ; 6.
[El camino cerrado b ➔ g ➔ h ➔ g ➔ f ➔ b da grad(R,) ; 5.]
Se ve que I:,grad(R,); 18; L .,grad(R,) ; 2 • 9; 1E I. Esto es cieno en general ya
que cada arista de la inmersión plana es parte de la frontera de dos regiones [como {b, e }
de las partes (a) y (b)J o aparece dos veces en el camino cerrado alrededor de las aristas.de
la frontera para una región [como {g. h) de las panes (a) y (b)]. /

R, ••
R,
., g

d d
<•> (b)

Figura 11.53

Ahora consideremos to siguiente.


11.4 Grafos planos 567

COROLARIO 11 .3 Sea G: (V, E) un grafo plano conexo sin lazos con I V 1 : u, 1E 1 : , > 2 y , regiones.
Entonces 3, :S 2t y t :s 3u - 6.
Demostración: Como G no tiene lazos ni es un multigrafo. la frontera de cada región (in •
cluyendo la región infinita) contiene aJ menos tres aristas. por lo tanto. cada región 1iene
El
grado i?: 3. En consecuencia. 2e: 2 I = la suma de Jos grados de Jasrregionesdetenni•
nadas por G y 2, i!: 3,. Del teorema de Euler, 2 : u- ,+ r s u - , + (2/3),: u-( 1/3),,
porloque6 s 3u - ,o, s 3u-6.

Consideremos ahora lo que este corolario implica y Jo que no implica. Si G : (V. E ) es


un grafo conexo sin lazos con I E 1 > 2, entonces si e> lu-6. se sigue que G no es plano.
Sin embargo, si e :!:" 3u - 6, no se puede concluir que G sea plano.

J~18 El grafo K, no tiene lazos yes conexo con IOaris.tas y cinco vértices. En consecuencia. 3u
""'==-""'-"-'-' -6 =: 1:5-6 =9 < 10 ~ e. Por lo tanto, por el corolario 11.3. vemos que Ks no es plano.

11e19 El grafo Ku no tiene lazos y es conexo con nueve aris.tas y seis. vértices. En es.te caso.
~ ;._;e___ __, 3u -6 =: 18-6::;;; 12 2: 9 =e.Seria un errorconc1uir a partir de estoque K,. ) es. plano;
cometeríamos et error de estar argumentando al revés..
Sin embargo, KJ. ) noes plano. Si K u fuera plano, entonces como cada región del grafo
está limitada por al menos cuatro aristaS, tendríamos 4r s 2t. (Encontramos una situación
similar en la demostración del corolario 11.3.) Del teorema de Euler, u - e+ r:;;; 2 o r=
e - u+ 2 = 9 - 6-t 2::;;; :5. y 20=4r .s 2e = 18. De esta conuadicción se tiene que KJ.) no
es plano.

Usacérños el teorema de Euler para caracterizar los sólidos platónicos. [Para estos sóli-
d ris las caras s.on congruentes. y los ángulos sólidos (interiores) iguales.) En la figura
11.54 tenemos. dos de estos sólidos. La parte (a) de la figura muestra un tetraedro regu-
lar, con cuatro caras. cada una de las cuales es un triángulo equilátero. Si nos concentra•
mos en las aristas del tetraedro, nos fijaremos en s.u eslrUctura. Si se observa esta estruc-
tura desde e l punto ubicado directamente sobre el centro de una de las caras, se obtiene
la representación plana de la parte (b). Este grafo plano determina cuatro regiones (co-
rres.pondientes a las c.uatro caras); en cada uno de los cuatro vénices se intersecan tres
regiones. La parte (c) de la figura m uestra otro sólido platónico, el cubo. S u grafo plano
asociado está en la pane (d). En este grafo hay s.e.is regiones y en cada vénicc se intersecan
tres de éstas.
Con bas.e en las observaciones para cJ tetraedro regular y el c ubo. determinaremos los
otros sólidos platónicos por medio de sus grafos planos asociados. En estos. grafos G =
(V. E), tenemos u: 1 vi;,: 1El;,: e l número de regiones planas determinadas porG;
m :;;; el número de aris.tas en la frontera de cada región: y n = el número de regiones que se
encuentran en cada vénice. Así. las constantes m. n i?: 3. Como cada arista se utiliza en la
frontera de dos regiones y hay r regiones. cada una con m aristas. resulta que 2e = mr.
Contando los extremos de las aris.tas, se obtiene ll. Pero todos. estos extremos tambi~n
,68 Capítulo 11 Una introducción a la teoría de grafos

4 ~ [a) ~
(a)

Figura 1 1 .54
(b) (e) (d)

pueden contarse considerando lo que sucede en cada vértice. Como e n cada vénice se
encuentran n regiones, n aristas también se encuentran ahí, por lo que se cuentan n extre-
mos de aristas en cada vértice u. Esto da lugar a un total de nu extremos de aristas, de
modo que 2e =nu. Por el teorema de Euler, se tiene

2, t +-=e
0<2=u-e+ r=--
n
2,
m
(2m -mn +2n)
=c......;=...:....:::.:.
. mn
Con e, m, n > O, encontramos que 2m - mn +2n >O=> mn - 2m - 2n ;;;;;.- mn - 2m-2n +
4 < 4 => (m - 2)(n -2) < 4.
Como m, n :.?: 3, tenernos que (m - 2), (n - 2) E z• y solamente hay cinco casos por
considerar:
1) (m - 2) = (n - 2) = l; m = n = 3 (el tetraedro regular)
2) (m - 2) = 2, (n - 2) = 1; m = 4, n = 3 (el cubo)
3) (m - 2)=1,(n-2) = 2;m=3,n=4 (el octaedro)
4) (m - 2)=3,(n-2) = l;m=5,n=3 (el dodecaedro)
S) (m - 2) =1, (n -2) = 3; m = 3, n = 5 (el icosaedro).
Los grafos planos para los casos 3-5 se muestran en la figura 11.55.

V Octaedro
@T Oodecaedto Icosaedro

Figura 11.55

La última idea que anaJizaremos para grafos planos es el concepto de grafo dual. Este
concepto también es válido para grafos planos c:on lazos y para multigrafos planos. Para
11 .4 Grafos planos 569

Cal G= (V, E) (b) G'

Figura 11.56

construir un duaJ (respecto a una inmersión panicular) de un grafo o multigrafo plano G


con V = {a. b, e, d. e./}, colocamos un punto (vértice) deritro de cada región, incluyendo
la región infinita. determinada por el grafo, como se muestra en la figura l 1.56(a). Para
cada arista compartida por las dos regiones, dibujamos una arista que conecte los vértices
dentro de estas regíones. Para una arista que se recorre dos veces en el camino cerrado
alrededor de las aristas de una región, dibujamos un lazo en el vé rtice de esta región. En la
figura 11.56(b),G'es un dual del grafoG = (V, E ).Apanirdeesteejemplopodemos hacer
las siguientes observaciones:
1) Una arista en G corresponde a una arista en <16, y viceversa.
2) Un vénice de grado 2 en G origina un par de aristas en G6 que conectan los mismos
dos vénices. Por lo tanto, G" podría ser un multigrafo. (En este caso. el vértice e
proporciona las aristas {a, e }, {e, f} de G que originan las dos aristas que conectan
V y ,en G'.)
3) Dado un lazo en G, si el interior de la región (área finita) determinada por el lazo no
contiene ningún otro vénice o arista de G, entonces el lazo origina un vértice col-
/ gante en G". (También es cieno que un vértice colgante en G origina un lazo en G' .)
4) El grado de un vértice en G" es el número de aristas en la frontera del camino
cerrado en torno de la región en G que contiene ese vé nice.
(¿Por qué decimos que G4 es un dual de G y no el dual de G? En los ejercicios de la
sección veremos que es posible tener grafos isomorfos G1 y G1 tales que los duaJes respec-
tivos et. Gt no sean isomorfos.)
Para analizar con más detalle 1a relación entre un grafo G y un dual G' de G, presenta-
mos la siguiente idea.

Definición 11.20 Sea G =(V, E) un grafo o multigrafo no dirigido. Un subconjunto E' de E es un conjunto
de cone de G si aJ eliminar las aristas (pero no los vértices) en E ' de G, tenemos lC(G) <
K(G'), donde G' = (V. E - E'); pero cuando eliminamos (de E) cualquier subconjunto
propio E" de E' . se tie ne K(G) = ,c( G"), paraG' = ( V, E-E").
Capítulo 11 Una introducción a la teoría de grafos

Para cualquier grafo conexo, un conjunto de corte es un conjunto minimal de aristas de


disconexión. En el grafo de la figura 11.57(a), observe que cada uno de los conjuntos
((a. b}, (a, e}}. ((a, b ), (e, d)}, ((<. h) , (/, h). (g. h)} y ((d,/}J es un conjunto de
corte. Para e l grafo de la parte (b) de la figura, el conjunto de aristas ( (n, p). (r, p}. (r, s}}
es un conjunto de corte. Observe que no todas las aristas en este conjunto de corte son
incidentes en un solo vértice. En este caso, el conj unto de corte separa los vénices n, rde
los vértices p, s. El conjunto de aristas {{s, r}} también es un conjunto de corte para este
grafo: la eliminación de la arista {s, r} de este grafo conexo produce un subgrafo con dos
componentes, una de las cuales es el vértice aislado t .
Cuando un conjunto de corte para un grafo conexo consta sólo de una arista. esa arista
es un puente del grafo. Para el grafo de la figura l l.57(a), la arista {d. fJ es e l 11nico
puente; la arista {s, t} es el único puente de la parte (b) de la figura.

Figura 11.57

Regresemos ahora a los grafos de la figura 11.56, los hemos vuelto a trazar como se
muestra e n la figura 11.58, para enfatizar las relaciones entre sus aristas.
En este caso, las aristas deGscetiquetan 1, 2, ... , 10. El esquema de numeración para
G' se obtiene como sigue: por ejemplo, la arista etiquetada 4 • conecta los vértices w y zen
G'. Dibujamos esta arista ya que la arista 4 de G e ra una arista comlln de las regiones que

(a) G" (V. fl (b) G'

Figura 1 1.58
11.4 Grafos planos 571

contienen estos véniccs. De la misma fonna. la arista 7 es común a la región que contiene
a x y la región infinita que contiene a u. Por lo tanto, etiquetamos la arista de Gd que
conecta x y u como 7*.
En el grafo G, el conjunto de aristas etiquetadas 6, 7, 8 constituye un ciclo. ¿Qué hay
acerca de las aristas etiquetadas 6•, 7•, s• en~ Si se eliminan de O', entonces e l vértice
x queda aislado y por lo tantO G' es disconexo. Como no podemos desconectar G' elimi-
nando cuaJquier subconjunto propio de {6*, 7•, s• }, estas aristas fonnan un conjunto de
corte en G'. De igua1 modo, las aristas 2, 4, JO forman un conjunto de corte enG. mientras
que las aristas 2•, 4•. 10• forman un ciclo en G'.
También tenemos el conjunto de corte de dos aristas {3, 10} en G y vemos que las
aristas 3*. 10• proporcionan un c ircuito de dos aristas e n G'. Otra observación: el conjun-
to de corte de una arista {1•} en G' proviene de la arista 1, un lazo de G.
En genera], hay una correspondencia uno a uno entre los siguientes conjuntos de arisw
en un grafo plano G y un dual O' de G.
1) Los ciclos (conjuntos de corte) de n (~ 3) en G corresponden a los conjuntos de
cone (ciclos) de n aristas en G'.
2) Un lazo en G corresponde a un conjunto de corte de una arista en G'.
3) Un conjunto de corte de una arista e n G corresponde a un lazo en G'.
4) Un conjunto de corte de dos aristas en G corresponde a un circuito de dos aristas en
O'.
5) Si G es un multigrafo plano, entonces cada circuito de dos aristas en G detennina un
conjunto de corte de dos aristas de O'.

Todas estas observaciones teóricas son interesantes, pero nos detendremos en este mo-
mento para ver cómo aplicar la idea de un dual.

t}¡ü• Si consideramos las cinco regiones finitas de la figwa l l .59(a) como los países de un
-==-ce.. -' mapa y construimos el subgrafo (pues no se utiliza la región infinita) de un dual como se
muestra en la pane (b), tendremos entonces la siguiente relación.
Supongamos que tenemos el "problema del cartógrafo", según el cual hay que colorear
las cinco regiones del mapa de la parte (a) de modo que dos países con una frontera en
común reciban diferentes colores. Esta idea puede uaducirse en la noción dua1 de colorear
los véniccs de la pane (b) de modo que los vértices adyacentes se pinten con colores
distintos. (Estos problemas de coloración se analizarán con m'5 detalle en la sección 11.6.)

<•> (b)

Figura 11 .59
572 Capítulo 11 Una introducción a la téOña de grafos

El resultado final de esta sección proporciona una aplicación para una red eléctrica.
Este material se basa en el ejemplo 8.6 de las páginas 227-230 del texto de C. L. Liu (24).

En la figura 11.60 vemos una red eléctrica con nueve contactos (conmutadores) que con-
trolan cJ encendido de una luz. Se desea construir una red dual, en la que se encenderá (se
apagará) una segunda luz cuando se apague (se encienda) la luz de la red dada.

figura 11 .60

Los contactos (conmutadores) son de dos tipos: normalmente abiertos (como se mues-
tra en la figura 11.60) y normalmente cerrados. Usamos a ya'comocn ta figura 11.61 para
representar los contactos normalmente abiertos y normalmente cerrados, respectivamente.

.
---l 1-
a'

{a)
*
(b)
Figura 11.61

En la figura ll.62(a), un grafo con un par terminal representa la red de la figura 11.60.
Los vértices especia1es se etiquetan como l y 2. Estos vénices son las tenninales del grafo.
Así mismo, hemos etiquetado cada arista de acuerdo con su contacto correspondiente en la
figura 11.60.
Un grafo con un par terminal Ges un grafo plano con un par terminal si Ges plano y
el grafo resultante también es plano cuando se añade a G una arista que conecta las termi•
nales. La figura 1L62(b) muestra esta situación. Al construir el dual de la parte (b), se
obtiene el grafo de la parte (c) de la figura. Eliminamos la arista punteada para obtener las
tenninales t •, 2* de este dual, que es un grafo con un par terminal. Este grafo proporciona
la red dual de la figura l l .62(d).
Hare mos dos observaciones finales.
1) Cuando los contactos a. b, ese cierran en la red original (Fig. 11.60), las luces están
encendidas. En la figura l l .62(b), las aristas a, b, e, j forman un ciclo que incluye
las terminales. En la pane (c)de la figura. a*, b*. e• ,j* forman un conjunto de corte
que desconecta las terminales l *, 2*. Finalmente, si a', b ', e' están abienos, como
e n la pane (d) de la figura, no pasa corriente más allá del primer nivel de contactos
(conmutadores) y las luces se apagan.
11.4 Grafos planos 573

(a)

,.
._ ___-----,,,\
__

.. d•
,. :j• •

''
,, '
........'
2'
(e) (d )

F'agurai 11 .62

2) De manera similar, las aristas e, J. i, j forman un conjunto de corte que separa las
terminales de la figura l l .62(b). (Cuando los contactos en c.f. i est!n abienos como
en la figura 11.60, las luces están apagadas.) La figura l l.62(c) muestra que c•.r,
i*.j* forman un cíclo simple que incluye 1*, 2*. Si c',j, i' están cerrados como e n
la pane (d ). la corriente fluye por la red dual y las luces se encienden.

Ejercicios 1 1 .4 1. Verifique que la conclusión del ejemplo 11 .1.5 no cambia si la figura 11.45(b) tiene la arista
(a. e} traz.ada en el exterior del pcnt.ágono.
2. Muestre que si se elimina cualq-Jier arista de X,, el subgrafo l"C$Ultan1c es plano. ¿Es esto cierto
para el grafo Xu?
3. a) ¿Cuántos \l~rtíces y cu:lntas aristas tienen los ¡rafas bipa.nitos completos K., ,. K7 _11 y K •.••
donde m, ne z·?
b) Si el grafo K... u tiene 72 aristas. ¿cuán10 vale m?
4. Dcmuc$lrt que cualquier subgrafo de un grafo bipartito es bipartito.
5. Para cada grafo de la figura 11.63. determine si el grafo es o no bipartito.
6. Sean E z ·. n .!:: 4. ¿Cuántos sub¡rafos de K. son isomoños al grafo bipartho completo K,.J?
574 Capitulo 11 Una introducción a la teoria de grafos

• b . .
~
b

g
f

h
,~,
h
g
[BD e

<•> (G) (b) (G') (e) (G")

Figura 11.63

7. Seam.11 E z ·conm ~ n ~ 2.
a) Determine cuántos ciclos distintos de longitud 4 hay en.K•.•.
b) ¿Cuántos caminos simples difcrc nles de longitud 2 hay e n K•. •?
e) ¿Cuántos caminos simples difere ntes de longitud 3 hay en K._.?
8. ¿Cuál es la longitud del camino simple más largo en cada uno de los siguientes grafos?
a) K1,, b) K1_7 e) K,.1z d) K,,._,.,
donde m, n E z·
con m < n.
9. ¿Cuántos caminos simples de longitud máxima hay en cada uno de los siguientes grafos?
(Recuerde que un camino simple como u 1 ➔ "2 ➔ UJ se considera igual al camino simple u 3 ➔
Uz-+\11,)
a) K,., b) K1.1 e) K 1,1: d) K,.._,.,
dondem, n E z ·conm<n.
1 O. ¿ Puede un grafo bipartito contener un ciclo de longitud impar? Explique por qué.
11. &.a G = (V, E) un grafo conexo sin lazos con I vi= u. Si IE 1> (u/2)2• demuestre que G no
puede ser bipartilo.
12. a) Encuentre todos los grafos bipartitos completos no isomorfos G = ( V.E) donde vi = 6. I
I
b ) ¿Cuántos grafos bipanitos completos no isomorfos G = ( V, E ) satisfacen v i = n ~ 2?
13. a ) Sea X= ( I, 2. 3, 4. 5 }. Construya e l grafo no dirigido sin la:i-os G = (V.E) como sigue:
• (V): Cada subconjunto de dos elementos de X representa un vértice en G.
• (A): Si l.)1• u 1 E V corresponden a los subconjuntos (a. bl y (e, d i, respectivamente. de X.
entonces trazamos la arista (u1, ~} en G si (a. b} n {c. d } = 0.
b) ¿A qué grafo es isomorfo G?
14. Determine cuáles de los grafos de la figura 11.64 $0R planos. Si un grafo es plano. vuelva a
dibujarlo sin aristas solapadas. Si no es plano. encuentre u n subgrafo homeomorfo a K, o Ku.
15. Sea G = (V.E} un grafo no plano no dirigido sin lazos. ¿Cuál e s el valor más pequeño que
puede tener I E 1?
16. Demuestre que el grafo de Petersen es isomorfo al grafo de la figura 11.65.
17. Determine el número de vértices, aristas y regiones para cada uno de los grafos planos de la
figura 1l.66. Luego muestre que sus respuestas satisfacen el teorema de Eulcr para grafos
planos conexos.
11 .4 Grafos planos 575

.tlj· •••@
• b e d
• •
d b

• e

9 h 'I d ; e
<•> (b) (e)

(d)
'®' M •

d
e

uv
<•>
w xy z
9
e

en
k /
t

Ftgur.a 11 .64

Figur.a 11.65 Figur.a 11.66

18. Sea G: (V, E ) un grafo no dirigido conexo sin lazos. Suponp ademú que G es plaoo y que
determina 53 regiones. Si para alguna inmersión plana de G cada región tiene aJ menos cinco
arisw en su frontera. demuestre que \ V 1 ~ 82.
19. SeaG-= ( V.E)ungrafoplano4-regularconexosin lazos. Si IE l= l6.¿cutntas regiones hay en
una representación plana de G?
20. Suponga queG= ( V.E)esungrafoplano sinlazoscon l vl- u. JE I = e yK(G) •dnúmero
de componentes de G. (a) Establezca y demuesue una extensión dd teorema de Euler para este
grafo. (b) Demuestre que el corolario 11.3 sigue sieDdo vil ido si G no tiene luos y es plano,
pero no conexo.
576 Capítulo 11 Una introducción a la teoría de grafos

2 1 . Demuestre que todo grafo plano conexo sin lazos tiene un vértice u con grad(U) < 6.
22. a) I
Sea G =(V. E) un grafo conexo sin lazos con vi ~ 11. Demuestre que tanto G como su
complementario G deben ser no planos.
b) El resultado de la parte (a) es verdadero para I vi I
~ 9, pero la demostración para v i = 9,
es
10 mucho más difícil. Encuentre un contraejemplo de la parte (a) para I vi =
8.
23. a) Seak ez·. k2:3. SiG= ( V,E)esungrafoplanoconexocon lvl=u. lEl = eycadaciclo
tiene una longitud mínima dek. demuestre que e :S ( k~ )(u-2).
2
b) ¿Cuál es la longítud mínima de un ciclo en K 3-.,'!
e:) Use las partes (a) y (b) para concluir que K1., no es plano.
d ) Use la parte (a) para demostrar que el grafo de Peters.en no es plano.
24. a ) Encuentre un grafo dual para cada uno de los dos grafos planos y el multigrafo plano de la
figura 11.67.
b ) ¿Tiene el dual del multigrafo de la parte (e) algún vértice colgante? Si no. ¿contradice esto
la tercera observación hecha antes de la definición 11.20?

-~ ~
(a)

Figu ra 11 .67
h (b)
z

"~
25. Encuentre los duales de los grafos planos correspondientes a los cinco sólido$ platónicos.
26. a) Muestre que los grafos de la figura 11.68 son isomorfos.
b) Trace un dual para cada grafo.
e) Muestre que los duales obtenidos en la parte (b) no son isomorfos.
d ) Dos grafos G y H son 2-isomo,fos si uno de ellos puede obtenerse del otro al aplicar uno o
ambos de los procedimientos siguientes un número finito de veces.

7~ ~ e
y z

(a) {b)
Figura 11.68
11.4 Grafos planos 577

q
(G) (H)
Figura 11.69

1) En la figura J 1.69, dividimos un vértice r de G para obtener el grafo H no conexo.


(Cuando la división o separación de un vértice de un grafo produce un nuevo grafo con
más componentes, ese vértice es un virtice tk corte o pumo d~ articulación.)
2) En la figura 11.70, obtenemos el grafo (d) del grafo {a) de la manera siguiente:
i) primero separamos dos vértices distintos j y q, que hacen que el grafo sea
disconexo,
ii) después, reflejamos un subgrafo sobre el eje horizontal y
iii) luego identificamos el vértice j(q) en un subgrafo con el vértice q(;) en el otro
subgrafo.
Demuestre que los grafos duales obtenidos en la parte (c) son 2-isomorfos.

[E{3~~
npqrs
(a)
n
(b)
pqqrs

I]lLl~rE
npq¡
(C)
k m
(d)
pq km

Figura 11.70

t) Para el conjunto de corte { {a, b}, {e, b}, (d. b}} en la parte (a) de la figura 11.68. encuentre
el ciclo correspondiente en su dual. En el dual del grafo de la figura l l .68(b), e ncuentre el
conjun10 de cone que corresponde al ciclo {w. z:}. {t. x}. {x. y}. b w} en el grafo dado.
27. Para el grafo de la figura 11. 70{a}, encuentre dos conjuntos de corte que contengan la arista
{<, m).
28. a) Si G = (V.E) es un grafo conexo 3-regular sin lazos y con ocho vénicf';S, ¿cuánto vale E l?I
b ) Encuentre dos grafos G 1 y G 1 que satisfagan las condiciones de la parte (a) con G 1 plano y
G2 no plano.
578 Capítulo 11 Una introducción a la teoría de grafos

29. Encuentre la red duaJ para la red eléctrica que se muestra en la figura 11.71.

·~~
~~~ 1
- -=- Figura 11.71

11 .5
Caminos y ciclos hamiltonianos

En 1859, el matemático irlandés SirWilliam Rowan Hamilion (1805-1865) desarrolló un


juego que vendió a un fabricante de juguetes de Dublín. El juego era un dodecaedro regu-
lar de madera con 20 e squinas (vértices) en las que aparecían inscritos los nombres de
ciudades importantes. El objetivo del juego era encontrar un ciclo alrededor de las aristas
del sólido, de modo que cada ciudad estuviera e n el ciclo (exactamente una vez). La figura
11.72 es el grafo plano de este sólido platónico; dicho ciclo está formado por las aristas
negras más gruesas. Esta ilustración nos lleva a la siguiente definición.

Figura 11.72

~inición 11.21 Si G ~ (V, E) es un grafo o multigrafo con I vi


~ 3, decimos que G tiene un ciclo
hamihoniano si existe un ciclo en G que contenga cada vértice de V. Un camino hamihoniano
es un camino simple (y no un ciclo) de G que contiene todos los vénices.

Dado un grafo con un ciclo hamiltoniano, la eliminación de cualquier arista en el ciclo


produce un camino hamiltoniano. Sin embargo, es posible que un grafo tenga un camino
hamiltoniano sin que tenga un ciclo hamiltoniano.
Podría parecer que la existencia de un ciclo (camino) hamfüoniano y la exístencia de un
circuiro (recorrido) euleriano para un grafo son problemas similares. El ciclo (camino)
11.S Caminos y ciclos hamiltonianos 579

hamiltoniano tiene como objeto pasar por cada vértice de un grafo una sola vez; e1 circuito
(recorrido) e u1eriano recorre el grafo pasando por cada arista exactamente una vez. Por
desgracia. no existe una relación útil entre las dos ideas y, a diferencia de los circuitos
(recorridos) eulerianos, no existen condiciones necesarias y suficientes en un grafo G que
garantice n la existencia de un ciclo (camino) hamiltoniano. Si un grafo tiene un ciclo
hamiltoniano. e ntonces al menos será conexo. Existen muchos teoremas que determinan
condiciones necesarias o suficientes para que un grafo conexo tenga un ciclo o un camino
hamihoniano. Más adelante analizaremos varios de estos resultados. Sin embargo, en los
casos de grafos paniculares, recurriremos con frecuencia al método de prueba y error,
junto con algunas observaciones útiles.

Si Ges el grafo de la figura 11.73, las aristas {a. b], {b, e]. (e,/], (f. e ). ( e. d) . {d. g) ,
{g, h}. {h, i} forman un camino hamiltoniano para G. Pero, ¿tiene G un ciclo hamiltoniano?
Como G tiene nueve vénices, si existe un ciclo hamiltoniano en G, éste debe contener
nueve aristas. Comencemos en un vértice by tratemos de construir un ciclo hamiltoníano.
Debido a la simeuía del grafo, no importa si vamos deba e o a a. Iremos hacia c. desde
donde podemos pasar aj o i. Usando otra vez la simetría. vamos af Luego eliminamos la
arista {e, i} de nuestro análisis posterior. puesto que ya no podemos volver al vénice c .
Para incluir el vértice i en el ciclo. debemos ir ahora de/a i (ah y g). Con las ari stas {c./}
y {f. i} e nel ciclo. no podemos tener la arista {e, f} en el ciclo. (De otra manera. en el ciclo
tendríamos que grad(j) > 2.) Pero una vez en e, ya no podemos continuar. Por lo tanto, este
grafo no tiene un ciclo hamiltoniano.

d,.__ _ ..,__ _ _.

Figura 11.73

Este ejemplo muestra algunas sugerencias útiles para tratar de encon1rar un ciclo
hamiltoniano en un grafo G ~ (V, E).

1) Si G tiene un ciclo hamiltoniano, entonces para u E V, grad(u) 2 2.


2) Si a E V y grad (a) = 2, entonces las dos aristas incidentes con el vértice a deben
aparecer en cualquier ciclo hamiltoniano de G.
3) Si a E V y grad (a)> 2. cuando tratamos de construir un ciclo hamiltoniano. una
vez que hemos pasado por el vértice a, dejamos de tener en cuen1a las aristas no
utilizadas incidentes con a.
4 ) Al construir un c iclo hamiltoniano para G. no podemos obtener un ciclo para un
subgrafo de G a menos que contenga todos los vértices de G.
i80 (.apítulo 11 Una introducción a la teoría de grafos

El siguiente ejemplo proporciona una Jnteresante técnica para mostrar que un tipo par-
ticular de grafo no tiene un camino bamiltoniano.

En la figtlra l l .74(a) tenemos un grafo conexo G y queremos saber si G contiene un ca-


mino hamihoniano. La parte (b) de la figura proporciona el mismo grafo con un conjun-
to de etiquetas x, y. Estas etiquetas se usan como sigue: primero se etiqueta el vénicca
con la letra x. Los vénices adyacentes a a (b. e y d) se etiquetan Juego con la letra y.
Después etiquetamos los vénices no etiquetados adyacentes a b, e o d con x. Esto produce
la eti-queta x en los vénices e, g, i. Finalmente, se etiquetan los vénices no etiquetados
adyacentes a e. g o i con y. En este momento, todos los vénices de G están etiquetados.
Ahora, como f vi• 10. si G tuviera un camino hamiltonianodebería haber una sucesión
alternativa de cinco letrasx y cinco letras y. Solamente tenemos cuatro vénices etiqueta-
dos conx, por lo que lo anterior es imposible. De ahí que G no tenga un camino (o ciclo}
hamihoniano.
¿Por qué funciona este argumento? En la parte (c) de la figura 11.74 hemos vuelto a
dibujar el grafo dado y vemos que es bipartito. A partir del ejercicio 10 de la sección
anterior, sabemos que un grafo bipartito no puede tener un ciclo de longitud impar. Tam-
bién es cieno que si el grafo no tiene un ciclo simple de longitud impar. entonces es bipartito.
(La demostración se pide al lector en el ejercicio 9 de esta sección.) En consecuencia.
cuando un grafo conexo no tiene ciclo impar (y es bipartito). el método descrito anterior-
mente puede ser útil para determinar si el grafo no tiene un camino hamiltoniano. (El
ejercicio 10 de esta sección examina esta idea más ampliamente.)

¡® e y®xy
b
a
e
I
y
X
y
d

i d g X y x

h y
(a) (b) (e)

Figura 11.74

El siguiente ejemplo proporciona una aplicación que necesita los ciclos hamiltonianos
en un grafo completo.

En el campamento que el profesor Alfredo ha organizado con su grupo de antropología, 17


estudiantes comen juntos en una mesa circular. Como intentan conocerse mejor, tratan
cada tarde de sentarse con dos compañeros distintos. ¿Durante cuántas tardes pueden ha-
cer esto? ¿Cómo pueden hacerlo?
Para resolver este problema, consideramos el grafo K•• donde n ~ 3 y n es impar. Este
grafo tiene n vénices (uno para c ada estudiante:) y fil = n(n - 1)/2 aristas. Un ciclo
11. 5 Caminos cidos hamiltonianos 581

hamihoniano corresponde a una disposición de lugares. Cada uno de estos ciclos ti.ene n
aristas, por lo que se tienen como máximo (1/nXjJ = (n-1)/2 ciclos hantiltonianos sin que
dos de ellos tengan una arista en común.
Consideremos el círc.ulode la figura 11.75 y el subgrafodc K. que consta den vértices
y n aristas { l. 2), {2, 3).. .. , {n - 1,n}, {n, 1}. Mantenemos los vértices en la circunfe-
rencia fijos y rotamos este ciclo hamiltoniano en el sentido de las manecillas del reloj,
hasta el ángulo {1/(n - 1))(2n). Esto produce el ciclo hamiltoniano (Fig. 11.76) fonnado
podas aristas { 1, 3), {3, 5), {5, 2), {2, 7), .. . , {n, n - 3), {n - 3, n - 1}, {n -1, l }. Este
ciclo hamihoniano no tiene aristas en común con el primer ciclo. S i n ~ 7 y seguimos
rotando de esta manera el ciclo de la figura 11.75. hasta los ángulos {k/(n - 1)}(2>t), donde
2 s k ::s (n - 3)/2, obtenemos un total de (n- IY2 ciclos hamihonianos. sin que haya dos
aristas en común.
Por lo tanto. los 17 estudiantes que participan en el campamento pueden comer durante
[(17 - 1)/2} = 8 días an,es de que lengan que sentarse junto a otroes,udiante por segun-
da vez. Usando la figura 11 .75 con n ~ 17, podemos ob,e ne r ocho de esw posibles dis-
posiciones.

Figura 11 .75

Veremos ahora más resultados acerca de los caminos y c iclos hamihonianos. Nuestro
primer resollado fue establecido en 1934 por L. Redei.

TEOREMA 11.7 Sea K: un grafo dirigido completo; es decir, K; tienen v~rtices y para cualquier par de
v~rtices x. y distintos, exaclamenle una de las aristas (x, y) o (y, x) está en K:. Este grafo
(llamado tornto) contiene siempre un camino hamiltoniano (dirigido).
Demostración: Sea m ~ 2 y p. un camino simple con las m - l aristas(u1, ~). (Ui, o ,)• . . . ,
(u.. _ 1. u.). Si m ;- n. hemos tenninado. Si no. sea u un vértice que no aparezca e n p•.
Si (o. u ,) es una arista de ,C. podemos extender p. añadiendo esta arista. Si no, enton-
ces (u 1• u) debe serlo.Ahora supongamos que (u. u-.J está en el grafo. Entonces tenemos el
camino más grande:(u1, u), (U, U:?). (Ui. UJ). . . • (u .. _ 1, v .. ). Si (u. U:z) no es una arista de
K:. entonces (\>:. u) debe serlo . Al cominuar este proceso, sólo tenemos dos posibilida-
des: (a) para algún I s k s m - 1, las aristas(u,. u), (u. u,. ,) están en K; y podemos
reemplazar (Ua. u.. 1) por este par de aristas; o (b) (u.. u) está en t'. y aliadimos esta arista
a p• . En cualquier caso, obtenemos un camino simple p_, con m + l v~n.iccs y maristas.
Este proceso puede repetirse hasta obtener un camino simple p. con n vértices.
Capítulo 11 Una introducción a la teoría de grafos

En un torneo de ajedrez, cada jugador se enfrenta con cada uno de los demás jugadores
exactamente una vez. Queremos clasificar a los jugadores según el resu1tado del torneo.
Como podríamos tener una situación en que, para los jugadores a. by e, a derrote a by b
derrote a e, pero e derrote a a, no siempre es posible tener una clasificación en la que un
jugador que esté en ciena posición haya derrotado a todos los jugadores que han quedado
por debajo de dicha posición. Si representamos a los jugadores por medio de vértices,
construimos un grafo dirigido G con estos vértices, trazando la arista (x, y) six derrota a y.
Luego, por el teorema 11.7, es posible enumerar a todos los jugadores de modo que cada
uno haya derrotado aJ siguiente jugador de la lista.

"EOREMA 11.8 Sea G ; (V, E ) un grafo sin lazos, 1 vi ; n ;,: 2. Si grad(x) + grad(y) ;,: n - 1 para todos x,
y E V, x :/= y, entonces G tiene un camino hamiltoniano.
Demostración: Primero demostraremos que Ges conexo. En caso contrario, sean C.,C2 dos
componentes de G tales que x, y E V y x E Ca, y E C1 . Supongamos que C, tiene n;
vértices, i ; !, 2. Entonces grad(x) :S n1 - 1, grad(y) :s n, - 1 y grad(x) + grad(y) :S (n , +
n2) - 2 :S n - 2, lo que contradice la condición del teorema. Por lo tanto, Ges conexo.
Ahora construiremos un camino hamiltoniano para G. Para m :?: 2. sea p.,. el camino
simple {u1, \¾}, {u2, u 3}, ••• , {u,._ 1, u.,} de long:it1:1dm- l . (Podemos volver a etiquetar los
vértices en caso necesario.) TaJ camino existe, pues si m ""2, sólo necesitamos una arista. Si
u 1 es adyacente a otro vértice u distinto de 1->,;, u 3, • •• , u.,, añadimos la arista {u,\)¡ } a p., para
obtener p..,. 1• Realizamos el mismo proceso si v. es adyacente a un vértice distinto de U1,
½, .. . , u_1• Si podemos extender p., hasta p. de este modo, obtenemos un camino
harniltoniano. En caso contrario, el camino simple p.. {uhu,}, ... , {u~_ 1, u.} cumple que
u 1, u .. sólo sean adyacentes a los vé nices en p.. y m < n. Si esto ocurre, afirmamos que G
contiene un ciclo en estos vén ices. Si U1 y u. son adyacentes, entonces el ciclo es {u 1, U:z},
{Ut, u,}, ... , {u.. _1, u.,}, {u.,, u 1 } . Si u 1 y u . no son adyacentes, entonces v 1 es ad.yacente
a un subconjuntos de los vénices en {\¾, u,, . .. , u,._1 } . Si existe un vénice u, E Stal que
v,.es adyacente au,_1, entonces podemos obtener el ciclo añadiendo {u1, v,}, {u,_1, uJ ap,.
y eliminando {u,_1, u,} como se muestra en la figura 11.77. En caso contrario, sea si =k< I
m - l.Entoncesgrad(u1) ; ky grad(u.) s (m-1)-k, por lo que obtenemos la contradicción
grad(v 1) + grad(u.) :S m - 1 < n - 1. Por lo tanto, existe un c iclo que une a u ., u.i • .. . , u ..

~ v.

~
Figur" 11.77

Consideremos ahora un vértice u E V que no esté en el ciclo. El grafo G es conexo, por


lo que e xiste un camino simple de u a un primer vértice u, en el ciclo, como se muestra en
la figura 11.78(a). Si eliminamos la arista {U, _i, u,} (o {U¡, u, } si r"" t), obtenemos el
camino simple (más grande que e l original p.J que aparece en la figura 1 l.78(b). Repeti-
mos el proceso (aplicado a p,.) para e l camino simple de la figura 11.78(b) y seguimos
aumentando la longitud del camino simple hasta incluir todos los vénices de G.
11. 5 Caminos y cidos harTUltonianos 583

(a)

(b)

Figura 11.78

CDROIARIO 11 .4 Sea G = ( V.E) un grafo sin lazos con n(2:2) vtnices. Si grad(u) 2: (n - 1)/2 para todo v
E V.entonces G tiene un camino hamiltoniano.
Demostración: La demostración se deja al lector como ejercido.

Nuestro último teorema de la sección proporciona una condición suficiente para la


existencia de un ciclo hamihoniano en un grafo sin lazos. Esto fue demostrado por vez
primera por Oystein Ore en 1960.

TEOREMA 11.9 Sea G s (V, E) un grafo no dirigido sin lazos, con IV1= n 2: 3. Si gnd(z) + grad(y) 2: n
para todos los vtrtices x, y E V no adyacentes, entonces G tiene un ciclo h.amiltoniano.
~mostración: Supon¡amos que G no contiene un ciclo hamiltoniano. Añadimos aristas a
G hasta obtener un $ubgrafo H de K. tal que H no tenga un ciclo hamihoniano. pero que
para cualquier arista e (de K. ) que no esti en H. H + e sí tiene un ciclo hamiltoniano.
Como H ~ K,., existen vértices a, b E V la.les que {a, b} no es arista de Hpcro tal que H +
{a, b I tiene un ciclo hamiltoniano C. El grafo H no tiene dicho ciclo, por lo que la arista ( a, b)
forma pane del ciclo C. EnumeraretOOS los vénices de H (y G) sobre el ciclo C como sigue:
Cª (=u,)-+b ( = uz)-+u -+u.-+ · • •-+u.,_ -+u,. :>
3 1

Pan cuaJquicr 3 :!:ii: i :5 n. si la arista {b, U¡} está en el grafo H. entonces afinnamos que la
arista {a. u,_ 1 } no puedcscrunaaristadeH, ya que si ambas aristas están en H , para algún
3 :S i s n, obtenemos el ciclo ham.iltoniano

en el grafo H (que no tiene ciclos hamiltonianos). Por Jo tanto, para cada 3 s i .s n, como
máximo una de las aristas {b, \>¡}, ta, u,_i} está en H. En consecuencia,
grad .(a) + grad.(b) < n.
Capítulo 11 Una introduc;:ción a la teoría de grafos

dondcgrad,,(u)denotael grado del vértice u en el grafoH. Paracualquieru E V, grad,,(1>) 2:


grad,,(u) = grad(u). por !oque tenemos los vértices a, b no adyacentes (en G) que cumplen

grad(a) + grad(b) < n.

Esto contradice la hipótesis grad (x) + grad(y) :?. n para todos los vértices x. y E Vno
adyacentes. por lo que rechazamos nuestra suposición y vemos que G contiene un ciclo
hamiltoniano.

Ahora obtendremos los siguientes dos resultados del teorema 11.9. Cada uno de ellos
nos dará una condición suficiente para la existencia de un ciclo hamiltoniano en un grafo
no dirigido sin lazosG = (V, E). El primer resultado es similar a1 corolario 11.4 y analiza el
grado de cada vértice u en V. El segundo resultado analiza el tamaño del conjunto de
aristas A.

:OROLARIO 11.5 vi
Si G =(V. E) es un grafo no dirigido sin lazos, con I = n 2: 3 y si grad(I>) "2 n/2, para
todo u E V, entonces G tiene un ciclo hamiltoniano.
Demostración: Dejaremos la demostración de este resultadO para los ejercicios de la sec-
ción.

:OROLARIO 11.6 = =
Si G ( V. E) es un grafo no dirigido sin lazos. con I V 1 n 2: 3 y I E 1 2: 1";'1 + 2,
entonces G tiene un ciclo hamiltoniano.
Demostración: Sean a. b E V tales que {a. b} ft E. [Como a. b no son adyacentes, quere-
mos mostrar que grad(a) + grad(b) ~ n.) Eliminamos lo siguiente del grafo G: (i) todas
las aristas de la forma {a, x }. dondex E V; (ii)todas las aristas {y. b), donde y E V; (iii) los
vértices a y b. SeaH = (V', E') el subgraforesultante. Entonces, 1El = 1E'I + grad(a)+
grad(b), pues (a. b)_,;. E.
Como Iv• I = n-2,H es un subgrafodel grafocompletoK. _ ,, por lo que IE'I :S 1";'1.
Enconsecuencia. í, ') +2 :S IEI = IE'I +grad(a)+grad (b) S (•; ') +grad(a)+
grad(b) y tenemos que

1 2
grad(a)+ grad(b)ae(n; ) + 2- ( n ; )

= G)<n - l)(n - 2) + 2 -G)(n - 2)(n - 3)


= G)(n- 2)[(n -1)-(n -3)] + 2
= G)<n - 2)(2) + 2
= (n - 2) + 2 = n.

Por lo tanto, el teorema 11.9 implica que el grafo dado tiene un ciclo hamiltoniano.
11 .5 Caminos y cidos hamiltonianos 585

Un problema relacionado con la búsqueda de ciclos hamiltonianos en un grafo es el


problema del viajan1e: e n este caso, el viajante sale de su casa y debe visitar varios Jugares
antes de regresar. El objetivo es encontrar una forma de hacer su viaje más eficie nte (tal
vez en términos de la distancia total recorrida o del costo total). El problema se puede
modelar mediante un grafo etiquetado (]as aristas tienen distancias o costos asociados a
eUas), e n e l cual se busca el ciclo hamiltoniano más eficiente.
Las referencias de R. Bellman. K. L. Cooke y J. A. Lockett 17) ; M. Bellmore y G. L.
Nemhauser 18); E. A. Elsayed (15); E. A. Elsayed y R. G. Stem (16); y L. R. Foulds 117)
serán interesante s para e l lector que desee aprender más acerca de este importante proble•
ma de optimización.Así mismo, el te xto editado por E. L. Lawler, J. K. L enstra, A. H. G.
Rinnooy Kan y D. B. Shmoys {23] presenta 12 artículos acerca de diversos aspectos de
este problema.

1. Dé un ejemplo de un grafo conexo tal que


a ) no tenga circu.itos eulerianos ni ciclos hamiltonianos;
b) tenga un circuito euleriano pero no tenga ciclos hamiilonianos;
e) tenga un ciclo hamiltoníano pero no un circuito euleriano;
d ) tenga un ciclo hamiltoniano y un circuito euleriano
2. Caracterice el tipo de grafo en que un recorrido (circuito) euleriano es también un camino
(ciclo) hamihoniano.
3 . Encuentre u n ciclo hamiltoniano, si ei:is1e. para cada grafo o multigrafo de la figura 11 .79. Si
el grafo no tiene un ciclo hamiltoniano. detennine si tiene un camino hamiltoniano.

.
(~~-
(a)
d
¡
k

i
h

e
'~
h
f

lbl
g

;
i
(~)
(<)
h '

. •
-~·
b ( d

. EEEB¡ e d • b ( d •

, EEEE· f

g
k m n o
p q (

'
o

(d) (el (0

Figur.i 11 .79
586 capítulo 11 Una introducción a la teoría de grafos

4. a) Muestre que el grafo d e Pcterscn [Fig. I I.48(a)] no tiene ciclos hamiltonianos pero sí tiene
un camino hamiltoniano.
b) Muestre que si se elimina cualquier vátice(y las aristas incidentes atl) del grafo de Pctersen.
entonces el subgrafo resultante tiene un ciclo hamiltoniano.
S. Considere los grafos de las partes (d) y (e) de la figura 11.79. ¿Es posible eliminar un vMicc
de cada grafo de modo que los subgrafos resultantes tengan un ciclo hamiltoniano?
6. Si n ~ 3, ¿cuántos ciclos hamiltonianos distintos tiene el grafo de rueda R,,_?
7 . a) Paran?!: 3, ¿cuántos ciclos hamiltonianos distintos tiene el grafo completo K,,_?
b) ¿Cuántos ciclos hamiltonianos con aristas disjuntas tiene K21?
e) 19 alumnos de una guardcriajucgan diariamente a tomarse de las manos y formar un círcu•
lo. ¿Durante cuántos días pueden hacer esto de modo que no se tomen de la mano con el
mismo compañero dos veces?
8. a) Paran E z·.n2:: 2. muestre que el nómerodeciclos hamiltonianos distintos en el grafoK...
es(ll2)(n - l )!n!
b) ¿Cuántos caminos hamiltonianos distin1os tiene K•.•• n 2:: 1?
9. Sea C = ( V, E) un grafo no dirigido sin lazos. Demuestre que si G no contiene un ciclo de
longitud impar. entonces Ces bipartito.
1 O. a) Sea G = (V. E) un grafo no dirigido, bipartito. conexo, con V dividido como V1 U V?-
Demues1re: que si I V1 1t- 1V2 1. entonces G no puede tener un ciclo hamiltoniano.
b) Demuestre que si el grafo G de la parte (a) tiene un camino hamihoniano. entonces v, I 1-
l v, I =:1.
e) Dé un ejemplo de un grafo no dirigido conexo bipartito G = (V, E), tal que Y esté separado
como V1 U V? y I V1 I = 1 V: 1 - 1. pero tal que G no tenga un camino hamiltoniano.
11. a) Determine todos los torneos no isomorfos con tres vértices.
b) Encuentre todos los torneos no isomorfos con cuatro vértices. Enumere los grados de entra.-
da y de salida de cada vértice en cada uno de los torneos
12. Para el grafo dirigido completo (o torneo) x; = (V. E ), ¿cuáles son los valores de L vge(u) y
L ,ss<u)'
13 . Dé un ejemplo de un multigrafo no dirigido. conexo, sin lazos G = (V, E ) tal que =n I vi y
grad(x) + grad(J) ~ n - 1 paratodosx, y e V, perotalqueGnotengauncamino hamiltoniano.
14. Encuen1re un contraejemplo al recíproco del teorema 11.8.
15. Demuestre el corolario 11.4.
16. Demuestre el corolario 11.5.
17. Dé un ejemplo para mostrar que el recíproco del corolario 11.5 no necesariamente es cierto.
18. Elena y Dolores invitan a 10 amigos a comer. Entre el grupo de 12 personas. todos conocen aJ
menos a 6. Demuestre que todos pueden sentarse alrededor de una mesa circular de fonna que
cada persona conozca a las dos personas sentadas a su lado.
19. Sea G= (V. E } un grafo no dirigido. sin lazos. 6•regular. Demuestre que si /vi=11. entonces
C tiene un ciclo hamiltoniano.
20. Sea G= (V. E) un grafonodirigído. sin lazos.n-regular. tal que I vi ~ 2n + 2. Demuestre que
G (el complementario de G) tiene un ciclo hamiltoniano.
21. Paran~ 3, sea C. e l ciclo no dirigido con n vértices. El grafoC,.. el complementariodeC•• es
el cocido de n vértices. Demuestre que paran ~ 5. el cocido C. tiene un ciclo hamilloniano.
22. Se an e z·. n ~ 4 y sea v· el conjunto de vértices para el grafo completo K~- 1> {v1• u 1•
~- •••• l>• • 1}, Construyamos ahora el grafo no dirigido sin lazos C. = ( V, E):. partir deK•. 1 como
sigue: V= V' U {u} y E consta de todas las aristas en K• • 1 excep10 por la arista {01 • u 1 ) que se
reemplaza por el par de aristas {u1, u} y (U. u 2 J.
11.5 Caminos y ddos hamittonianos 587

a) Determine grado(.r) + grado()') para todos los vErtices no adyacentes x y y de Y.


b) ¿1iene G. un ciclo ha.rniltoniano?
e) ¿Cu.ti es la magnitud del conjunto de ari$W E?
d) ¿Contradicen los resultados de las partes (b) y (c) el corolario 11.6?
23- Paran e z·. n i!: 4, sea V'; {Ui, Ui, v,.. . , u._, ) eJ conjun.ode vb-1:iccs del grafo completo K,_,.
Comtruyacl grafo no di rigido sin lazosff. ; (V, E) apartirdeK•• , como s i¡ue: V; V' Utu)y
A consu de todas las aristas de K•• 1 junto con la nue,·a arist.a(u, U1}.
a ) Mue.1trc que H. tiene un camino hamiltoniano pero no un ciclo bamiltoniano.
b) ¿Cuil es la magnitud del conjunto de aristas E?
24. Sea n= 2 1 para k e z·.Usamos las n sc:cuencias de k bits (ceros y unos) para representar 1. 2.
3• •• . , n, de modo que para dos enteros consc:cutivos i. i + l. las secuencias correspondientes de
k bits sólo difieran en exactamente u.na componente. Esta rep,esc:ntación se llama c6digo Gray.
a ) Si A: = 3. use un modelo de grafo con V = {000. 001, 01 O, .. .. 111) para encontrar dkho
código para 1, 2, 3 ..... 8. ¿Cómo se relaciona esto con el concepto de camino ham.iltoniano?
b) Responda la pan:c (a) para k s 4.
25. Si G • ( V. E ) esun grafo no dirigido. un subconjunto / de Veswk~ndien1t si los vEnices de
/ no son adyaccnres. Un conjunto indq>endiemc / es maxima/ si no existe un vfrtice u que se
pueda agregar a/ de modo que / U {u} sea independiente. El númen:> dt in.dtpendtncia de G.
que se denota ~G), es el tamafto máximo de un conjunto independiente en G.
a ) Para cada grafo de la figura 11.80. encuentre dos conjuntos independientes muimales con
difcrcntcS tamal'tos.
b) EncuC"ntre ~G) para cada grafo de la parte (a).
e) Determine ~G) para cada uno de los siguientes grafos: (i) K 1,,; (ii) Ku: (iii) Ku : (iv) K,.,;
z•.
(v) K•.•; ( vi) K.., •• m, n E
d ) Sea / un conjunto independiente en G = ( Y, E). ¿Qué tipo de subgrafo induce / en~'?

Figura 11.80

26. Sean G • (V, E ) un ¡;rafo no diri¡;ido e / un conjunto independiente en V. Pan cada a E / y


cualquier ciclo hamiltoniano C de G. haW grad (a ) - 2 aris~n E incidentes con a que no
estln en C. Por lo tanto. habnt al menos 2-_, (¡rad(a) - 21 • L....., grad (a )- 2 ll l aristas en
E que no aparezc-an en C.
a) ¿Por quE son distintas las L,.., ll.
¡rad (a ) - 2 l aris1as'?
b) Sean u a I vi .t = 1El. Demuestre que si~- L grad (a) + 2 l 11
<\I, entonccs G no tiene:
nigún ciclo hamiltoniano.
e) Seleccione un conjunto independiente / adecuado y use la p,3rte (b) pari mostrar que el
¡:rafode la figura 11.8 1 (conocido como el grafo de Herschel) no tiene ciclos hamihonianos.

Finur;;a 11.81
588 Capítulo 11 Una introducción a la teoría de grafos

27. Consideremos los tres grafos bipanitos G 1 = (V1, E1), Gz = ( V2, E.J y G,= ( V1 • E3) de la figuni
11.82.
a) ¿Cuántos subconjuntos independientes de V 1 contienen un vértice? ¿Cuántos contienen dos
vértices'?
b) ¿Cuántos subconjuntos (incluyendo el conjunto vado) de V1 son subconjuntos tndepcn-
dientes de V17
e) ¿Cuántos subconjuntos de V1 son subconjuntos independientes de V2?
d ) Determine el número de subconjuntos independientes de V3•

·~~ ·~! · ·<}


(G,)

Figura 11 .82
(Gi)
5
CG,)
n- 1

11.6
Coloración de grafos
y polinomios cromáticos

En 1os laboratorios químicos JJ, Juanita se encarga del almacenamiento de los compuestos
químicos. Puesto que ciertos tipos de compuestos (como ácidos y bases) no deben
almacenarse juntos. decide que Juan, su compañero, divida el a1macfo en áreas índepen-
dientes de modo que los reactivos químicos incompatib1es se guarden en compartimentos
separados. ¿Cómo puede determinar el número de compartimentos que debe constn.1ir
Juan?
Si esta empresa vende 25compuestos químicos, sea {c1 , c2 , • •• ,c25 } = Vun conjunto de
vértices. Para cada I s i<} s 25, trazamos 1a arista {e;, e¡} sic; y e¡ deben guardarse en
compartimentos separados. Esto nos da un grafo no dirigido G = (V, E ) .
Presentarnos ahora el siguiente concepto.

>efinición 11.22 Si G = (V, E ) es un grafo no dirigido, una coloración propia de G ocurre cuando colorea-
mos los vértices de G de modo que si {a , b} es una arista en G, entonces a y b tienen
diferentes colores. (Por lo tanto, los vénices adyacentes tienen colores diferentes.) El nú-
mero mínimo de colores necesarios para una coloración propia de Ges el número cromático
de G y se escribe como X( G).

Si volvemos con Juanita al almacén, encontramos que e] número de compartimentos


que Juan debe construir es igual aX(G) para el grafo construido sobre V= {c., c2, .. . ,Czs}.
11.6 Coloraáón de grafos y polinomios cromáticos 589

¿Pero cómo calculamos X(G)? Antes de presentar cualquier forma de determinar e] núme-
ro cromático de un grafo, rcvísarcmos ]a siguiente idea.
En el ejemplo 11.22 mencionamos la conexión entre la coloración de las regiones de un
mapa plano (donde las regiones adyacentes tienen colores diferentes) y la coloración pro-
pia de los vértices en un grafo asociado. La determinación del número mínimo de colores
necesarios para colorear los mapas planos ha sido un problema de interés por más de un
siglo.
Cerca de 1850, Francis Guthrie (1831-1899) se interesó en el problema genera) des-
pués de mostrar la forma de colorear los condados de un mapa de Inglaterra con sólo
cuatro colores. Poco después, mostró el "problema de los cuatro colores" a su hermano
menor Frederick (1833--1866), quien era alumno de Augustos DeMorgan (1806-1871).
DeMorgan comunicó el problema (en 1852) a William Hamilton (1805-1865), quien no
se interesó en é l y por lo tanto permaneció en el olvido durante cerca de 25 años. En 1878.
la comunidad científica tomó conciencia del problema con el anuncio de Arthur Cayley
( 1821- 1895) en una reunión de la London Mathematical Society. En 1879, Cayley enun-
ció el problema en el primer volumen de los Proceedings o/the Royal Geographical SQciety.
Poco después, el abogado (y matemático aficionado) Sir Alfred Kempe ( 1849-1922) dise-
ñó una demostración que permaneció incuestionable durante más de una década. Sin em-
bargo, en 1890, el matemático británico Percy John Heawood ( 1861-1955) encontró un
error en el trabajo de Kempe. ·
El problema permaneció sin solución hasta 1976, cuando finalmente fue resuelto por
Kenneth Appel y Wolfgang Haken. Su demostración usó un complicado análisis
computacional de 1936 configuraciones (reducibles).

Aunque sólo se necesitan cuatro colores para una coloración propia de las regiones de
un mapa plano, necesitamos más de cuatro colores para una coloración propia de los vér•
tices de algunos grafos no planos.
Comenzaremos con algunos ejemplos breves. Después veremos una forma de determi-
nar X( G) a pan ir de subgrafos más pequeños de G, en cienos casos. [En general, el cálculo
deX(G) es un problema muy difícil.] También obtendremos el llamado polinomio cromático
de G y veremos cómo se utiliza para calcular X(G).

Para el grafo G de la figura 11.83, partimos del vértice a y junto a cada vértice escribimos
el número de un color necesario para una coloración propia de los vértices de G conside-
rados hasta ese punto. Al pasar al vénice b, el 2 indica que necesitamos un segundo color,

,,f-F0•;
e,1~

d. 2 Figura 11.83
590 Capítulo 11 Una introducción a la teoría de grafos

puesto que los vértices a y b son adyacente s. Seguimos en orden alfabético hasta/y vemos
que necesitamos dos colore s para una coloración propia de {a, b, e, d, e./}. Para el vértice
g, necesitamos un tercer color. el cual también puede usarse para el vértice h, puesto que
{g, h } no es una arista de G. Así. este método de etiquetado de una coloración secuencial
nos proporciona una c:oloración propia de G, por lo queX(G) :S 3. Como K 3 es un subgrafo
de G fpor ejemplo, el subgrafo inducido por a. b, g es (isomorfo a) K3], tenemos que
X(G) a: 3, por lo que X(G) = 3.

a) Para cualquier n ~ l, X(K~) = n.


b) El número cromático del grafo de Herschel (Fig. 11.81) es 2.
e) Si Ges el grafo de Petersen [véase la Fig. I l.48(a)], entonces X.(G) = 3.

Sea G el grafo que se muestra en la figura 11.84. Para U= {b,f. h. i). el subgrafo inducido
(U) de G es isomorfo a K,. por lo que;c(G) ~ X(K,) = 4. Por lo tanto, si podemos determi•
nar una coloración propia de los vértices de G con cuatro colores, entonces sabremos que
X(G) =4. Una forma de lograrlo es colorear los vértices e, f. gen azul; los vértices b. j en
rojo, los vértices e, h en blanco y los v6rtices a, d, i en verde.

~
h
Figura 11.84

Ahora veremos un método para determinar X(G). Nuestra e xposición se basa en la del
artículo de la referencia [27]. de R. C. Read.
Sea G un grafo no dirigido y sea A. el número de colores disponibles para la coloración
propia de los vénices de G. Nuestro objetivo es encontrar una función po)inomial
P(G, A), en la variable A, llamada polinomio cromático de G, que nos indique el núme•
ro de coloraciones propias diferentes de los vértices de G, usando un máximo de A.
colores.
En este análisis, los vértices de un grafo no dirigido G = (V, E ) tendrán etiquetas. En
consecuencia, dos coloraciones propias de tal grafo serán diferentes en el siguiente sentido:
una coloración propia (de los vértices de G) que usa como máximo Acolores es una función
/,condominio V y c-odominio { 1, 2, 3, ... , A), tal que /(u) i= /(u) para los vértices
adyacentes u, u E V. Las coloraciones propias serán diferentes si son diferentes como
funciones.
11.6 Coloración de grafos y polinomios cromáticos 591

I
a) Si G = (V, E) e s tal que vi = n yA= 6, entonces G 1iene n puntos aislados y por
la regla del producto, P(G, A.)= A.' .
b) Si G = K._. entonces debemos disponer de a1 menos n colores para obtener una
coloración propia de G. Entonces, por la regla del producto, P(G, A.) = A.(A.- l )(A.-
2)· • •(i..-n + l), que denotaremos con )..<-l. Para A< n, P(G, A)=Oy no existe una
coloración propia de K~. P(G, A)> O por primera vez cuando A,= n = X(G).
e) Para cada camino simple de la figura 11.85, consideremos el número de opcio-
nes (de los A colores) en cada vénice sucesivo. Si procedemos alfabéticamente,
veremos que P(G1, A.) = A.(A. - 1)3 y P(G,, A.)= A.(A. - I)'. Como P(G,, 1) =O=
P(G,, 1), pero P(G 1, 2) = 2 = P(G1, 2), se sigue que X(G,) = X(G ,) = 2. Si dispo-
nemos de cinco colores, podemos obtener una coloración propia de G 1 de 5(4)'
• 320 formas; entonces, G2 se puede colorear propiamente de 5(4)' = 1280 for-
mas.

·1J· "LI'
c,>i. - 1

(G,)
b,>i. - 1

(G,)
d,>i. - 1 c,>i. - 1

Figura 11.85

En general, si G es un camino simple con n vértices. e ntonces P(G, A) = }..(A - tt· 1•

d) Si G está formado por componentes G 1• G 2, •••• G1, entonces usamos de nuevo la


regla del producto para obtener P( G, A.)= P{G,. A.) ·P(G,. A.)· · ·P{G,, A.).

Como resultado del ejemplo 11.3 l(d), nos concentraremos en los grafos conexos. En
muchos casos de las matemáticas discretas, se utilizan métodos para resolver problemas
en casos de gran tamaño mediante la descomposición de éstos en dos o más casos más
pequeños. De nuevo utilizaremos este método. Para esto, necesitamos las siguientes ideas
y notación.
SeaG:;; (V, E) un grafo no dirigido. Para e= {a, b} E E, seaG, el subgrafodeG que se
obtiene al e liminare deG, sin quitar los v~rticesa y b; es decir, G, = G - e,comosedefinió
en la sección 11.2. A partir de G, obtenemos un segundo subgrafo de G, identificando los
v~rticcs a y b. Este segundo subgrafo se denota con e;.
capítulo 11 Una introducción a la teoria de grafos

La figura t 1.86 muestra G, y G; para el grafo G con la arista e dada. Observe que la
identificación de a y ben G; produce la identificación de los dos pares de aristas {d, b},
{d, a} y {a, e}. {b, e },

• e • e • <=b) e

d
IS]
G

Figura 11.86
b d
□ G.
b d
j
G;

Usando estos subgrafos especiales, veremos ahora el resultado principal.

EO REMA 11.1 O Teo~ma de descomposición para polinomios cromáticos. Si G = (V, E ) es un grafo conexo
y e E E, entonces

P(G,,,) = P(G,1.) + P(G;, 1.).

Demostración: Sea t -"" {a, b}. El número de coloraciones propias de los vénicesdeG,con
(a lo sumo) A colores es P(G,, A). Las coloraciones en que a y b tienen diferentes colores
son coloraciones propias de G. Las coloraciones de G, que no son coloraciones propias de
G aparecen cuando a y b tienen el mismo color, pero cada una de éstas corresponde a una
coloración propia de G;. Esta partición de las P(_G,. A) coloraciones propias de G, en Jos
dos subconjuntos disjuntos descritos produce la ecuación P(G~, A)"" P(G, A) + P(G;, A).

Al c alcular polinomios cromáticos. colocaremos corchetes en torno de un grafo para


indicar su polinomio cromático.

Los siguientes cálculos producen P(G, A) para un ciclo G de longitud 4.

P(G,1.) P(G.,l.) P(G;,,)


11.6 Coloración de grafos y polinomios cromáticos 593

Del ejemplo 11.3l(c) se siguequeP(G,, A) = A(A -1)'. Con G; =K,, tenemos P (G;,J.)
= A_m _ Por lo tanto,

P(G, ,) =l.(1. - 1)3 -l.(), - 1)(). -2) = l.(l. - l)[(i. - 1)2 - (1,_ - 2)1
=l.(). - 1)[1.2 -3l. + 3] = ).' - 4).3 + 6l.2 - 3l..

Como P(G,l) = O mientras que P(G.2) = 2 > O, sabemos que X(G) = 2 .

,.,, Aquí vemos una segunda aplicación del teorema 11.10

= (l.)(l.'' ') - 2l.'' ' = (l. -2) i.<•> = l.(l. - l)(l. - 2)2(l. - 3)

Para el grafo discone;<O


con las componentes K1 • K.

Para cada I sA s 3. P(G. A)= O, pero P(G, A)> O para todo A~ 4. En consecuencia,
el grafo dado tiene número cromático 4.

Los polinomios cromáticos dados en los ejemplos 11.33 y I t .34 indican los resultados
siguie ntes.

TEOREMA 11.11 Para cualquier grafo G. el ténnino constante en P(G, A) es O.


Demostración: Para cualquiergrafoG, X(G) > O, puesto que V=f.0. Si P(G. A) tiene término
constante a. entonces P(G, O) :;:: a t-0. Esto implica que hay a coloraciones propias de G
con O colores, una contradicción.

TEOREMA 11.12 Sea G = ( V, E) con I E 1 > O. Entonces, la suma de los coefi cientes de P (G, l.) es O.
Demostración: Como ] E 1 ~ 1, tenemos que X(G) ~ 2, por lo que no podemos obtener una
coloración propia de G con sólo un color. En consecuencia. P(G.1 ) =O= la suma de los
coeficientes de P(G. A.).

Puesto que podemos determinar fácilmente el polinomio cromático de un grafo com-


pleto. tenemos otro método para encontrar P(G, A). El teorema 11.10 redujo e l problema a
subgrafos más pequeños. En este caso. agregamos aristas a un grado dado hasta obtener
grafos comptetos.
594 Capitulo 11 Una introducción a la teoría de grafos

TEOREMA 11.13 =
Sea G (V, E), con a, b E V pero {a, b} =,
~E.Escribimos G; para el grafo que se
obtie nedeG a1 añadir la arista e= {a.b}.AI identificar los vértices a y ben G, obtenemos
el subgrafo G;+ de G. En estas circunstanc ias, P(G, A) = P(G;, A)+ P(G;+, A).
Demostradón: Este resultado se demuestra como el teorema 11.1 O. puesto que P(O:, "A.) =
P(G, A)-P(G;•, A).

- Apliquemos ahora el teorema 11. 13.

P(G,).) P(G; ,).) P(G¡',).)

En este caso, P(G, A)= ),!'' + ;.m = ~,. - IXA - 2)', por lo que X(G) = 3. Además, si
disponemos de seis colores, podemos obtener una coloración propia de los vértices en G
de 6(5)(4)' = 480 formas.

Nuestro siguiente resultado utiliza los grafos completos y los conceptos que aparecen
en seguida. Para los grafos G, = (V,. E,) y G, = ( V,, E,),
i) la unión de G1 y G2, que se denota con G1 U G 2 , es el grafo cuyo conjunto de
vértices es V1 U V2 y el conjunto de aristas es E1 U E?; y
ü) cuando V, n V2 i-0, la intersección de G 1 y G 2, que se denota con G 1 n G 2, es el
grafo con conjunto de vértices V 1 n V2 y conjunto de aristas E 1 () Ez..

rEOREMA 11.14 Sea G un grafo no dirigido con-subgrafos G 1, G 2• Si G = G 1 U G2 y G 1 n G2 = K,.,, para


algún n E z+, entonces

P(G, ).) = [P(G,. ).) · P(G,, ).)j/).'"'·

Demostración: Como G 1 n G 2 = K,.,, se sigue que K. es un subgrafo tanto de G 1 como de G 2


y que X(G 1) , X(G2) ~ n. Dados "A. c olores, K,., tiene },_(IIJ coloraciones propias. Para cada una
de e stas A.<11>coloraciones hay P(G1, "A.)/A,<11i coloraciones propias de los vénices restantes de
G 1• En forma análoga, hay P(G2 , A)A P-1 coloraciones propias de los vénices restantes
de G2 • Por Ja regla del producto.
11.6 Coloración de grafos y polinomios cromáticos 595

Consideremos el grafo de1 ejemplo 11.34. Sea G1 el subgrafo inducido por los vM.ices w,
x. y. i:. Sea G1 el ciclo determinado por u. w y x. Entonces G 1 n G1 es la arista {w, x}. y
G, nG2 =K1•
Por lo tanto,

P(G1,).) • P(G,,).) ¡_<•>. ¡_D>


P(G,).) = ¡_(l) ,:¡;;-
= ['-'(;. - 1)2(). - 2)'(;.- 3)]/ [).(). - l)]
= A(I. - !)(). - 2)2(). - 3),

lo que coincide con la respuesta obtenida en e1 ejemplo J1.34.

Podemos decir más acerca de los polinomios cromáticos; en panicular. hay muchas
preguntas sin responder. Por ejemplo. nadie ha encontrado un conjunto de condiciones
'- que indiquen si un polinomio dado en A es el polinomio cromático de algiin grafo. En el
articulo de R. C. Read (27] se dan más de<alles sobre este tema.

EJERCICIOS 11.6 1. El propietario de una tienda de mascotas recibe un envío de peces tropiCalC$. Entre las distintas
especies hay .a.1gunos pares en que una es depredadora de oua. En consecuencia. estos paces
de especies deben mantenerse en peceras distintas. Construya un modelo de este problema
como un problema de coloración de grafos y describa cómo determinar el menor número de
peceras necesarias pan. preservar todos los peces del envío.
2. Como presidenta de los cornil~ estudiantiles. Antonieta debe programar los horarios para
la reunión de 15 comitfs. Cada comitf se rcll.ne durante una hora a la semana. Las reunio-
nes de dos comitts con un miembro en comdn deben programarse a horas distintas. Mode-
le esto como un problema de coloración de grafos y describa cómo determinar el menor
mlmcro de horas que Antonieta dene que considerar para programar las reuniones de los
15 comitts.
3 . a) En tos laboratorios químicos JJ. Juanita recibe tres embarques que contienen un total de
sie1e sustancias químicas diferentes. Así mismo. la na1uralez.a de escas sustancias es tal que
para todo 1 :Si s 5. la sustancia i no puede almacenarse en el mismo companimento que
la sustancia í + 1 o la i + 2. Determine el menor número de companimcntos separados que
Juanita necesitan: para almacenar en forma segura CStaS siete suStanc::ias.
b) Suponga que además de las condiciones de la parte (a). los cuatro pares siguientes de las
mismas siete sustancias requieren tambifn compartimentos sepan.dos: 1 y 4; 2 y S; 2 y 6; 3
y 6. i,Cutl es el menor mimcro de compartimentos de .a.1maccnamiento que necesita ahora
Juanita para almacenar en forma segura estas siete sustancias?
4 . Dé un ejemplo de un grafo no dirigido G = (V, E) tal que X(G) = 3 pero que ningún grafo de G
sea isomoño a K,-
5. a) Determine PC.G. A.) de G= K,.,.
b) Paran E z·. ¿cuál es el poliñomio cromático de K ? ¿Cuál es su número cromático?
1••

6 . a) Considere el grafoKuquese muestra en la figura 11 .87, y sea.A E Z*cl número decolores


disponibles para una coloración propia de los ,·&tices de K2.,- (i) (,Cutnw coloraciones
propias de Ku tienen los vfflic:c5 a, b del mismo color? (il) ¿Cuántas coloraciol'JC$ propiu
de Ku tienen los vértices o, b de diferente color?
596 Capítulo 1 l Una introducción a la teoría de grafos

Figura 11 .87

b) ¿Cuál es el polinomio cromático de K2. 3?


e) ¿Cuánto vale x(K2. ,)?
d) Paran E Z ... ¡,cuál es el polinomio cromático para K2. .?
e) ¿Cuánto vale X(K2.~)?
7. Encuentre el número cromático de los siguientes grafos.
a) El grafo bipanito completo K•.•.
b) Un ciclo den vértices, n ~ 3.
c) Los grafos de la figura l 1.54(d), I l.57(a) y 11.80.
8 . Si Ges un grafo no dirigido sin lazos con al menos una arista. demucst"}que Ges biparti10 si
y sólo si X(G) = 2.
9 . a ) Determine los polinomios cromáticos para los grafos de la figura 11.88.
b ) Encuentre X(G) para cada grafo.
e) Si se dispone de cinco colores, ¿cuántas coloraciones propias de los vértices de cada grafo

,o- ~
existen?

t t

W
A
(a)
X

Figura 11.88
y z
(b)
y X
w

z
(el
y
'

10. a) Determine si los grafos de la figura 11.89 son isomorfos.


b ) Encuentre P{G,X) para cada grafo.
e) Comente los resultados de: las partes (a) y (b).

figura 11.89
11.6 Coloración de rafos polinomios cromáticos 597

11. Utilice el teorema t l .14 para determinu los polinomios cronú.ticos para cada uno de los ¡:rafos
de la figura 11.90.

b ~ e b ~

a~ d a ~
d g
l•l (b)

(e)
d g

(d)

Figura 11 .90

12. Sea C un grafo no dirigido sin laios. donde A z mix<>Ev{grad(u)}.


a) Demuestre que X(G) .S A + I.
b ) Encuentre dos tipos de grafos G iales que X(G) =A+ 1.
13.- Paran ~ 3, sea C. el ciclo de longitud n.
a) ¿Cuinto vale P(C),. A)?
b ) Si " ~ 4. muestre que P(,C.. A) • P(,P. - ,. A) - PC,C._ 1• A). donde P• • , denota el camino
simple de lon¡irud n - l .
e) Verifique que P(P. _,, ).) ""A.(A - 11 · 1, para cualquier n ~ 2,
d ) Establezca las relaciones

P(C.,A)-(A- l)" • (A - !)""' - P(C•• .,A), n ;o4,


P(C., A) - (A -1)" • P(C•• ,, A) - (),-!)"º', n >e 5.

e) Demuestre que para cualquier n ~ 3,P(C•. A)= (A - l "/ + ( - 1)'(). - 1).


14. Para n 2: 3. el ¡rafo de rueda. R.. se obtiene de un ciclo de longitud n colocando un nuevo
vértice en el ciclo y-&l\adiendo aristas (radios) de este nuevo ,·troce a codos los del ciclo.
a) ¿Qu~ relación hay entre X(C.) y X(R.)1
598 capitulo 11 Una introducción a la teoría de grafos

b) Utilice la pane (e) del ejercicio 13 para mostrar que

P(W.,J.)- , (J.- 2)" + ( - l)"J.(J.-2).

e) i) Si tenemOS k colores diferentes disponibles. ¿de cuintas formas podemos pintar las
paredes y el techo de un cuarto peru.a¡onal si debemos pintar con colores diferentes las
paredes adyaccnteS, así como cualquier pattd y el techo?
ii) ¿Cuál es el menor valor de k. para el cual es posible esta coloración?
iü) Responda las partes (i) y (ii) para un cuarto hexa¡:onal. (FJ lector puede comparar la
pane (e) de C$tC ejercicio con el ejercicio 6 de 10$ ejercicios complementarios deJ capí-
tulo 8.)
15. Sea G s (V. E) un grafo no dirigido sin lazos con polinomio cromáúco P(G. A) y I v i = n.
Utilice el teorema 11.13 para demostrar que P(G. A) tiene ¡rado n y coeficiente principal l .
16. Sea G • (V.E) un grafo no dirigido sin lazos.
a) Para cada a;raío de este lipo tal que I vi
~ 3. encuentre P(G. A.) y muesucquc sus términos
contienen polCncias consecutivas de A. TambiEn muestre q~ los coeficientes de estas¡»-
tcncias consecutivas alternan su signo.
b) Ahora considere G =(V.E) donde I vi •
n z: 4 y IEl •k. Demuestre por inducción
matemítica que los t~rminos de P(G. A) contienen potencias consecutivas de A y que 10$
coeficientes de estas poccncias consecutivas alteman su signo. (Como hipótesis de inducción.
suponga cr,uc el resultado es cieno para cualquier grafo no dirigido sin lazos G (V. E) =
donde(i) I Vl =n- lo(ii) l v l = n.p.,-o IEI
= k-1.)
e) Demuestre que si I vi
= n, entonces el coeficiente deA.•·1 en P(G. A) es el opuesto de El. I
17. Sea G • ( V, E ) un grafo no dirigido sin lazos.G es critico rcspec1o al color si X(G) > X(G-u)
para todo U E V•
.a) Ex plique por qué los ciclos con un número impar de vtrticcs son críticos respecto al color.
mientru que los ciclos con un número par de vértices no lo son.
b) Paran e z·.
n ~ 2. ¿cutJes grafos completos K. son críticos respecto al color?
e) Dcmues1re que un grafo critico respecto al color debe ser conexo.
d) Demuestre que si G es crítico respecto al color y X(G ) = k. entonces grad(U) z: k - 1 para
todo u e v.

11 .7
Resumen y repaso histórico

A diferencia de ouas áreas matemáticas, los inicios de la teoría de grafos pueden situarse
en una época y un lugar definidos: el problema de los siete puentes de KOnigsberg, resuel-
lo en 1736 por Lconhard Euler ( 1707-1783). En 1752 apareció el teorema de Euler para
grafos planos. (Este resultado fue presentado originalmente e n términos de poliedros.) Sin
embargo, después de estos desarrollos, hubo pocos avances en ~I área durante casi un
siglo.
En 1847. Gus1av KirchhofT (1824-1887) analizó un tipo especial de grafo llamado
árbol. (Un árbol es un grafo no dirigido sin lazos. conexo y sin ciclos.) KirchhofT utilizó
este ~oncepto en ciertas aplicaciones de redes eléctricas. al fonnular su e xtensión de las
leyes de Ohm para flujos eléctricos. Diez aftos más tarde. Arthur Cayley ( 1821-1895)
desarrolló el mismo tipo de grafos para contar los distintos isómeros de hidrocarburos
saturados C,!/:,, . !• n E z·.
En este periodo aparecieron también otras dos ideas importantes. La co,,j~rura de los
cuatro colores fue investigada por primera vez alrededor de 1850 por Francis Guthrie
11.7 Resumen repaso histórico 599

( 1831- 1899). En la s=ión l l.6n:latamos algo de la histonade este problema. que Kennelh
Appcl y Wolfgang Haken resolvieron mediante un complejo an'1isis computacional en
1976.
La segunda idea importante fue el ciclo hamihoniano, llamado así en honor de sirWilliam
Rowan Hamillon (1805-1865), quien en 1859 usó la idea pan, formular un acenijo que
usa las aristas de un dodecaedro regular.t No es muy d ifícil ha11ar una solución para este
problema, pero los matemáticos siguen buscando las condiciones necesarias y suficientes
para caracterizar los grafos no dirigidos que poseen un camino o ciclo hamfüoniano.
Luego de estos desarrollos, no hubo mucha actividad hasta después de 1920. La carac-
terización de los grafos planos fue resuelta por el matemático polaco Kasimir Kuratowski
(1896--1980) en 1930. En 1936 aparece el primer libro sobn: teoría de grafos, escrito por
el matemático húngaro DtnQ: KOnig (1884-1944), destacado investigador en este tema
Desde entonces, ha habido gran actividad en el área, especialmente durante las últimas
cqacro d«:adas, con ayuda del computador. Entre los muchos investigadores contemporá-
neos (que no se mencionan en la bibliograffa del capítulo) en tsta y otras áreas relaciona-
das, encontramos los nombres de Claude Berge, V. Chvátal, Paul Erdós, LaszloLovász, W.
T. Tuue y Hassler Whitney.
Un desarrollo simílar aJ de este capítulo aparece en los capítulos 6. 8 y 9 de C. L. Liu
(24]. Un estudio más avanzado"sc encuentra en los trabajos de M. Behzad, G. Chanrand y
L. 1.esniak-Foster [6), J. Bondy y U. Murty (10], N. Deo (14) y N. Hartsfield y G . Ringel
(22). El libro de F. Buckley y F. Harary (11) n:visa el tr.lbajo clásico de F. Harary (20) y
pone al alcance del lector los temas que aborda el trabajo original de 1969. El texto de G.
Chartrand y L. Lcsniak ( 12) presenta un enfoque más algorítmico. Una demostración del
teorema de Kuratowski aparece en el capítulo 8 de C. L Liu [24]. Otros desarrollos en el ·
campo de la teoría de grafos aparecen en los dos volúmenes del MM Srudies in Ma1hemo1ics,
editado por D. Fulkerson ( 18). El artículo de G. Chartrand y R. J. Wilson ( 13) desarrolla
muchos de los conceptos importantes de la teoría de grafos, enfocándose en un grafo
panicular, el grafo de Petersen. Este grafo (que se mencionó en la sección 11.4) recibe el

William Rowan Hami lton (1805-1865)


~PCWCOl'tfN Ot l h t G t _ , . . c ~ Huf!'r.llYcfl

t En realidad. el problem:1. del ciclo lwnihoni:mo fue propuesto originalmente por Euler. (N. dtl R.T.)
600 Capítulo 11 Una introducción a la teoría de grafos

nombre del matemático danés Julius Peter Christian Petersen ( J839-191O), quien analizó
el grafo en un anfculo de 1898.
En S. Seshu y M. Reed [30] es posible encontrar aplicaciones de teoría de grafos a las
redes eléctricas. En el texto de N. neó'(14], las aplicaciones en teoría de códigos, redes
eléctricas, investigación operativa. programación de computadores y química aparecen en
los capítulos 12 al 15. El texto de F. Roberts {28] aplica los métodos de teoría de grafos a
las ciencias sociales. El artículo de D. H. Rouvray [29] da aplicaciones más recientes de la
teoría de grafos en química
El artículo de R . C. Read [27) y e l capítulo VI de C. L. Liu [25) tie nen más información
acerca de los pol.inomios cromáticos. E l papel de la teoría de Polyat en la enumeración de
grafos se analiza en el capítulo 10 de N. Deo (14]. Una reseña completa de este tema se
encuentra en el texto de F. Harary y E. PaJmer [21 ].
N. Biggs, E. Lloyd y R . Wilson [9] proporcionan material adicional sobre el de sarrollo
histórico de la teoría de grafos.
Muchas aplicaciones de la teorfa de grafos implican grandes grafos que requieren el
uso intensivo de un computador y e] ingenio de los métodos matemáticos. El capítulo 11
de N. Deo (14] presenta algunos algoritmos computacionales que tratan varias propieda•
des de la teoría de grafos analizadas aquí. Siguie ndo la misma lfnea.. el texto de A. Aho.
l Hopcroft y J. Ullman [l] proporciona más detalles al lector interesado en ciencias de la
computación.
Como se mencionó al final de la sección 11.5, el problema del viajante está fuertemente
relacionado con la búsqueda de un ciclo hamiltoniano en un grafo. Éste es un problema de
teoría de grafos que resulta de interés en la investigación operativa y en Las ciencias
computaciona1es. El artículo de M. Bellmore y G . L. Nemhauser [8] proporciona una
buena introducción a los resultados de este problema. El texto de R. Bellman, K. L. Cooke
y J. A. Locken [7] incluye un análisis algorítmico del mismo problema junto con otros
problemas de grafos. La heurística necesaria para obtener una solución aproximada al pro-
blema aparece en el capítulo 4 del texto de L. R. Foulds (17}. El texto editado por E. L.
Lawler, J. K. Lenstra, A. H. G. Rinnooy Kan y D. B. Shmoys (23] contiene 12 anículos
que tratan varios aspectos de este problema, incluyendo consideraciones históricas y aJgu.
nos resultados sobre complejidad computacional. En los artículos de E. A. Elsayed [ 15] y
E. A. Elsayed y R. G. Stem [16] se analizan algunos desarrollos recientes en cuanto a
aplicaciones, como el de un robot que visita lugares diferentes en un a1macén automatita·
do para cumplir una orden dada.
La solución del problema de los cuatro colores puede examinarse con detalte a partir
del artículo de K. Appel y W. Haken (2]. Es posible apreciar el desarrollo de este trabajo
estudiando un articulo anterior de W. Haken {19]. El problema, junto con su historia y
solución, se examina en el texto de D. Bamette (5) y en el anículo de Scientific Amuican
escrito por K. Appel y W. Hak:cn (3). La demostración utiliza un análisis con computador
para un gran número de casos; el artículo de T. Tymoczko [31] examina el papel de estas
técnicas en las matemáticas pu.ras. En [4]. K. Appel y W. Hak:en examinan con detalle su
demostración a la luz del análisis computacional que utili:zaron.

t Prese1:11aremo-s la$ i<k,as básicas sobre este mttodo de enumeración en el capítulo 16.
'
11 .7 Resumen y repaso histórico 601

Finalmente, el artículo de A. Ralston [26] demuestra algunas conexiones entre la teoría


de la codificación, la combinatoria. la teoría de grafos y las ciencias computacionales.

BIBLIOGRAFÍA

t. Aho,AlfrcdV.. John E. Hopcroft y Jeffrey D. Ullman,Data Structu~s andA.lgorithms, Reading,


Mass., Add.ison-Wesley. 1983.
2. Appel, Kcnneth y. Wolfgang Hakcn, ''Every Planar Map Is Four Colorable", Bulletin of 1M
Amt rican Mathematical Socitt'j 82, 1976. págs. 711-712.
3. Appel, Kenneth yWolfgang Hakcn, "1bc Solution ofthe Four-Color-Map Problem",Scitnrific
American 237,octubre de 1977, págs. 10&-121.
4. Appel. Kenneth y Wolfgang Haken. ""Thc Four Color Proof Suffices". Ma1hemarical
Jnu:lligenur8, no. 1.1986, págs. 10-20.
5. Bamette, David, Map Coloring. Polyhedra, and tht Four-Color Problem (Thc Dolciani
MathematicaJ Expositions. núm. 8). Washington. O.C., The Mathcmatica1 Association of
America. 1983.
6. Behz.ad, Mehdi. Gary Chartrand y Linda Lesniak•Foster. GrapluandDigraplu. Belmon1. Calif.,
Waclsworth, 1979.
7 . Bellman, R., K. L. Cookc y J. A. Lockc n,.Algorithms. Graph.s, and Compuurs, Nueva York,
Acad.emic Press, 1970.
8. Bcllmorc, M. y. G. L. Nemhauser. '"lñe Traveling Salesman Problem: A Survey... O~ratioru
Ruearch 16.1968, págs. 538-558.
9. Biggs. N.. E. K. Lloycl y R. J. Wilson. Graph Theory (1 736-1936), Oxford, Inglaterra. Clarendon
Press, 1976.
10. Bondy, J. A. y U. S. R. Murty. Graph Theory with Applications. Nueva York:, Elsevier North-
Holland. 1976.
I J. Buck.ley, Fred y Frank Harary. Disrancein Graphs, Reading. Mass.. Addison-Wesley. 1990.
12. Chanrand. Gary y Linda Lesniak. GrapJu ami Digroplu. 21 ed, Montercy, Calif.. Wadsworth
ancl Brook.s/Cole. 1986.
13. Chanrand, Gary y Robin J. Wilson. "'Tbe Petersen Graph". en Frank Harary y John S. Maybee.
editores, Graphs and Applicarions (Procetdings of ,M Fir-1t Colorado Symposium on Graph
Theory), Nueva York, Wiley, 1985.
14. Deo. Narsingh, Graph Thecry with Applicatio11s to Enginu ring and Ccmputer Science,
Englewood Oiffs, N.J., Prentice-Hall, 1974.
t 5. Elsayed. E.A., "Algorithms for Optima! Material Handling inAutomatic Warehousing Systems'',
lnt. J. Prod.. R~s. 19,1981. págs. 525-535.
16. Elsayed, E. A. y R. G. Stem, "Computerized algoritluns for Order Processing ín Automated
Warehousing System". lnt. J. Prod. Res. 21. 1983, págs. 579-586.
17. Fould5. L R. CcmbinaLcrial Optimizalionfor Urukrgradwues, Nueva York. Springer-Verlag. t 984.
18. Fulkerson, D. R., editor, Studies in Graph Tlteory, Parts l. II. MM Studies in Mathematics.
vol. 11 y 12. The MathematicaJ Associatfon oí America. 1975.
19. Hak.en, Wolfgang. "An Anempt to Understand the Four-Color Problem", Joumal of Graph
Theory 1, núm. 3, 1977. págs. 193-206.
20. Harary, Frank, Graph Theory, Reading, Mass.. Addison-Wesley, 1969.
602 Capítulo 11 Una introducción a la teoria de grafos

21, Harary. Frank y Edgar M. Palmer. Graphical EnumeratWn, Nueva York, Academic Press.
1973.
22. Hartsfield. Nora y Gerhard Ringe l, Pearls in Graph Theory, Bostoo, Mass., Academic Prtss.,
1990.
23. Lawlcr, E. L., J. K. Lenstra, A. H. G. Rinnooy Kan. y D. B. Shmoys, cditoTcs, The Traveling
Salesman Problem. Nueva York. Wiley, 1986.
24. Liu. C. L, lntroduction to Combina.torio.l Mathemarics, Nueva York. McGraw-HiU, 1968.
25. Llu, C. L., Tapies in Combinmorial Marhematics, MathematicaJAssociation ofAmerica, 1972.
26. Ralston, Anthony, "De Bruijn Sequences -A MO<kl Example of the lnteraction of [)jscrete
Mathematics and Computer Sciencc'', Mathematics Magaúne SS. núm. 3, mayo de 1982.
págs. 131-143.
27. Read, R. C., ""An lntroduction to Cbromatic PolynomiaJs'', Journal o/Combinalorial ~ r y
4, 1968, págs. 52- 71.
28. Roberts. Fred S.. Discrere MatMmatical Modeb, Englcwood CJiffs, N.J., Prentice-Hall, 1976.
29. Rouvray, Dennis H., "Predicting Chemistry from Topology". ScientificAmerican 255, mlm. 3,
septiembre de 1986, págs. 40-47.
30. Seshu. S. y M.S. Reed, linear Graphs and Electrical Networks. Reading, Mass .• Addisoo-
Wesley, 1961.
31. Tymoczko, Thomas, ..Computers, Proofs and Mathematicians: A Philosophical lnvestigation
ofthe Four-ColorProof... Mathemarical Magazine 53, núm. 3, mayo de 1980, págs. 131- 138.

S. a) Muestre que los grafos G1 y G1 de la figura IJ.91


EJERCICIOS
son isomorfos.
COMPLEMENTARIOS
b ) ¿Cuántos isomorfismos diferentcsf G 1 ➔ Gi son
posibles?
1. Sea G un grafo no dirigido sin lazos con n véniccs. Si
G tiene 56 aristas y C:- tiene 80 aristas, ¿cuánto valen?
2. Un n-cubo es un grafo no dirigido G =(V.E) e n el que
los vértices se etiquetan con todas las n-uplas ordenadas de
ceros y unos de tal forma que para cada arista {u. w} E E.
las etiquetas de u,w difieren exactamente en una compo-
nente.
a) Demuestre que G es conexo.
b) Encuenue I v i y I El.
3. a) Si las aristas de K, se pintan de rojo o azul, de-
muestre que hay un triángulo rojo o un 1riángulo
azul que es un subgrafo. (G ,) (G¡)
b) Demuestre que en cualquier grupo de seis perso-
nas debe haber tres que sean completamente des- Figura 11.91
conocidas entre sí o tres que sean amigos mutuos.
4. ScaG = (V.E) un grafo no dirigido sin lazos. Recuerde 6. a) SeaV= {a.b,c, d,e,f.g}.Dibujeloscuatrografos
que G es autocomplcmcntario si G y G son isomorfos. Si G no isomorfos no dirigidos sin lazos G; = (V, E),
es autocomplcmentario y I V 1 = n, para 1 :s i s 4, tales que, en los cuatro grafos,
a) ¿cuánto vale IE 1? tenemos grad (a) = 5, grad (b) = 3, grad(c) =
b) demuestre que Ges conexo. grad(d) = grad (,) = 2 y grad(j) = grad(g) = l.
Ejercicios complementarios 603

b) ¿Cuintosde los grafos de la pane (a) son conexos? b) Vcrifique que lvl
eslasumadclnúmerodeinde~
7. a) ¿Cuántos caminos simples d e longitud 5 hay en el pendencia de G y su námcro de recubrimien10.
gn!o bipartito comple10 Kv? (Recuerde que un 12. Si G = (V. E)es un grafo no dirigido. un subconjunto
camino simple comou1 ➔ U?-+ u, ➔ V4-+ U,➔ u. D de V es un C<Hljw uo domiluuue si para todo u E V. o E
se considera igual al camino simple u, -+ 0¡ ➔ v4 Do u es adyacente a un v~nice en D. Si Des un conjunto
➔ U, ➔ \h-+ u •. ) dominante y nin¡ún s ubconjunto propio de D tiene esta
b) ¿CuMltos caminos simples de longitud 4 hay en propiedad~D es minimal. El tamafto de cualquier conjunto
K1,1! dominante con el mínimo mlmero de elementos en G se
e) Sean m.n,pE Z*,2m< nyl Sp.S 2m.¿Cuántos d eno1a con )l(G) y se conoce como número de dominación
caminos simples de longitud p hay en e l grafo de G.
bipartito completo K. _,.?
a) Si G no tiene vfrticcs aislados. demuestre que si
l. ¿Soo bipartitos los grafos planos para los cinco sólidos Des un conjunto dominante minimal, en1onces
. ' V - D es un conjumo dominan1e.
9. Use el teorema 11.14 para encontrar P(C. A) para el b) Si / ~ V e / es independiente , demuestre que / es
p>loGde la figura 11.92. un conjunto dominante si y sólo si / es indepen-
diente ma.timal.
e) M uestrcquey(G) s ll(G)yque lvl
s ll{GJx(G).

13. ScaG1 = (V1 • E1)dondc V, "" {a1.b1 ) y E, = f . Obtene-


mos G1 = (V2• Ei) de G 1 como sigue: V:= V1 U (a:, b:) y E1
= E1 U ({a1, v) Jo e V1) U ({o, b1} lv E V.}. Continuan-
do de esta manera. para rua.l.quier n e z·.
donde n ~ 2.
construimos el grafo G. ""(V•• E.) a panir del grafo G•. 1 =
(V._ 1, E._ 1) como antes:

V..={a.,b. )UV.- , y
Figura 11 .92
E.= E. -, U {{a.,u}I uE v._,¡ U{{u, b.H• E v.-,}-
10. Sea X = { J. 2, 3, .... n ). donde n ;o 2. Construya el aii) Para n E z-sea u. 1v. l.
= y
Encuentre resuelva
,grafo no dirigido sin luos G =( V. E ) como sigue: una relación de recurrencia para o •.
• (V): Cada subconjunto de X de dos elementos deter- b) Sea e.= IE, I paran E z·. y
Encuentre resuelva
mina un v&tice de G. una relación de rtturrcncia para e,..
• (E): Si u., v 2 E V corresponden a los subconjuntos e) Para n E Z♦• cuenta el número de sub¡rafos en
'~

la. b) y {e, dJ de X, respectivamente. dibuje la arista G. que son isomorfos a K3• Encuentre y resuelva
{u,, u,) en G si {a. b)n {c. di= 0. una relación de recurrcncia para,..
a) Muestre que G es un vfnice a islado si n = 2 y que d ) Demuestre que para todo n i!::: 1. X,( G. ) = n.
Ges disconcxo para n = 3 . 4. 14. a ) Paran~l.seaP. _1 elcaminosimpledcnvértices
,.) Mucwe que paran i!::: 5. Ges conexo. (De hecho, y n - 1 aristas. Sea a. el número de subconjuntos
pan.cua1guieru1,ViE V. lv1,UJ) EE oexisteun independientes de vén icc$ cnP•• 1, (El subconjunto
camino simple de longitud 2 que conecta u 1 y 'l.>l,) vacío se considera como uno de estos subconjun1os
e) Demuestre que G oo es plano para n i!::: S. independientes.) Encuentre y resuelva una relación
ti) Demuestre que para n i!::: 8. G tiene un ciclo de rccurrencia para ª~·
hamiltoniano. b) Determine el mlmero de subconjuntos indepcndien•
11. SiG= (V. E ) es un grafo no dirigido, un subconjuntoK tes (de vlnices) en cada uno de los ¡ratos C 1• G 2 y
• vcsrr:cubrimien1odeGsi para cada arista fa. b} dcG.a G 3 de la figura 11.93.
o lo estf en K. El conjunto K es un recubrimieruo m(nimo si e) Para cada uno de los grafos H1• Hi y H 3 dc la figura
~ - f.r} no recubre a G para iodo x E K. El número de 11.94. encuentre el número de subconjun10s indc•
.ertioesdcl recubrimiento más pequeño se llamanWl'l~rode pendientes de virticcs.
ac:alwimiett.to de G. d ) Sea C =(V.E') un grafo no diri¡ido sin lazos con
1) Demuc:scrc que si / ~ V. entonces / es un conjunto V = {u1• \>i••..• u,) y con m subconjuntos indepen-
independiente en G si y sólo si V - 1 es un dientes de virtices. El ¡rafo G' • (V". E') se cons-
recubrimiento de G. truye a partir de G como sigue:
504 Capitulo 11 Una introduC".ción a la teoría de grafos

·~~
(G,)
· ~ ~
Figura 11.93
... ~ :
CG,) 5 CG,)
n - 1

·+·
<H,)
.• ... ~
Figura 11 .94
<H,I s
,
<H,)
n
n-
.. ,
1

V' = V U {x1 • x2 , • • • , x1 }. donde ningún X; en V. a. cuenta el número de subconjuntos independientes en fx,,


para todo 1 :S i s s; y. E'= E U f(.t";, u1) [ I :Si ::s: x 1• • .• • x •. y1• y2• • ••• y. }. Encuentre y resuelva una relación
.s. 1 :S j :S r). ¿Cuántos subconjuntos de V' son de recurrencia para ª•·
independientes? 17. Considere los cuatro grafos de las panes ( i), ( ii), (iii)y
15. Sea G = (V.E) el ..grafo de escalera''. conexo. nodiri- (iv ) de la figura 11.97. S i ª• denota el número de
Pdo. que se muestra en la figura 11.95. subconjuntos independientes de {.x1, .r2, ••• , .r., J i, _\·~• .. . ,
a) Detennin< 1vi y IEI. y.} . ta1 que n ~ 1, e ncuentre y resuelva una relación de
b) Para n ~ O, sea a . el número de formas en que re.currencia para a•.
pueden seleccionarse n de las aristas en G de tal 18. Para n 2:: 1. sea ª• = (~!. el número de aristas en K,., y
forma que dos aristas no compartan un vértice en sea a0 = O. Encuentre la fun~ión generatriz/(.r) = L,:.Oa.x•.
común. Encuentre y resuelva una relación de recu-
rrcncia para a•. 19. Para el grafoG de la figura 11.98. responda las siguien-
tes preguntas.
a) ¿Cuánto valen )<G). P(G) y X(G)?
b) ¿Tie ne G un circuito e ul e riano o un ciclo
hamiltoniano"!
e) ¿Es G bipartito"! ¿Es plano"!
20. a) Suponga que el grafo bipanito completo K •.• tiene
16 aristas y satisface m .5 n . Determine m. n de
Y1 Yi Yi
modo que K.,_. tenga (i) un circuito euleriano pero
Figura 11.95
no un ciclo hamiltoniano; {ii) un ciclo hamiltoniano
y un circuito eulcriano.
16. Considere los cuatro grafos peine de las partes (i}. (ii). b ) Generalice los resultados de la pane (a).
iii) y (iv) de la figura 11.96. Estos grafos tienen I diente. 2 21 . Si G = ( V. E)es un grafo no dirigido, cuaJquiersubgrafo
llentcs. 3 dientes y n dientes, respectivamente. Paran ~ 1. de G quesea un grafo completo es unc/an de G. El número
Ejercicios complementarios 605

uw UL..JJ
x, x, x, x, x, x, x, x, x, x,,_, )(.,

1
Y, Y, Y, y, Y, Y, Y, Y, Y, y_, r.
(ij (i) (oi) (w) 1

Flguta 11.96

x, x, x, x, x, x, x, x, x, .r,,_, x•

~ ffi ~ --- ~
1
Y, Y, Y, Y, Y, r, Y, r, Y, y.,., y.
(,) {o¡ (iil) (N)

Figura 11.97

Figura 11.98
a
(a)
A tsJ
Figura 11.99
b
(b)
w x

• wáticcs en un clan maximal en Ges el núm.tro th clan de


e; J "' deoou con (>)(G). e) Demuestre que si G tiene un circuiro eulcriano, en-
1) ¿Cómo se relacionan X(G) y ro(G)? tonces L(G) tiene un circuito culeriano y un ciclo
b) ¿Existe alguna relación entre co(G) y J}(~)? hamil1oniano.
d ) Si G = "4. examine U,G) para mostrar que el recí-
•uta
ZZ. SiG=(V, E)esun grafo sin lazos no dirigido. el grafo
de G, denotado con U..G), es un gnfo con E como
Clllljaa(o de vMices. doodc unimos dos vMiccs ,t1• .t1 en
proco de la pan, (e) es falsa.
e) Demuestre que si G tiene un ciclo hamiltoniano.
e ntonces U,G) 1ambién.
U.G) si y sólo si ,t1, t: son aristas adyacentes en G
f) Analice L{G) para el grafo de la figura I l .99(b) y
1) Encuentre L(,G) para cada uno de los Jrafos de la
figun 11.99. muestre que el recíproco de la parte (e) es falso.
g) Verifique que L(G) no es plano para G • K, y G =
•l Si J vi •
n y lt;I • t, muestre que L(G) tiene t
K,.,.
vtni=y ( l/2)2:,.. ¡¡rad(u). [grad(U)-1]= ((1/
h) ™un ejemplo de un grafo plano G pero que U.G>
2 :E,..c¡¡rac1<u>P1 -. = I ..<"",'"'> arista,. no sea plano.
,06 (apítulo 11 Una introducción a ta teoría de grafos

!3. Explique porqu~cadaunode los siguientes polinomios 26. Al igual que cuando definimos A,.w al trabajar con cl
le A no puede ser un polinomio cromático. polinomio cromático de K.,. n ~ l, tambi61 podemos defi-
a) ).:'-5J?+7A1 -6A.+ 3 nir ~ =x(x- I)(x - 2)· · ·(x - 1'1 + 1). Esta multiplicación
b) 3).3-4A.2+1 produce un polinomio en x de grado "· Es.cribJmos este
e) A' -3)..1 + 5A.2-4A polinomio como L..os(n, i)x;; cada coeficiente s(n, ,), don-
!4. a) Para cualquier x, y E z·. demuestre que .t'y-ryl de O s i s ,,, es un núnuro de Stirling de primer tipo.
es par. Para todon, .(n,. n ) = l y .r(l'I, O)= O. Para i < Oo i> l'I.
b) Sea V= {I, 2, 3, .... 8, 9). Construya el grafo no !(.~ i) == O. Adcmás,!.:s(n + l ,i)~ =~• 11 = x(n)(x- n) =
dirigido sin lazosG = (V. E) como sigue: Param., n (2.,., .1(1'1, 1)x"](x-n). En consecuencia.. para cualquier 1 si
E Y, m ~ n, dibuje la arista {m. n} en G si S divide ::5 n, la comparación de coeficientes de potencias similares
m+nom-n. de x produce la relación de recurrencia
e} Dados tres enteros positivos distintos cualesquie•
ra, demuestre que existen dos de ellos. digamos.x y J(n + 1,i) =s(n, i - 1) - ns(n, i).
y, tales que 10 <livide a x'y-xy'.
!5. Un grafo n-parrilQ completo G =(V, E) es un grafo no a) Use la relación de recurrencia para encontrar una
lirigido sin lazos ta1 que tabla para $(1'1, ,), para todos 1 ::5 l'I ~ 5 y O S i ~ n.
b) Escriba un programa (o des.arrolle un algoritmo)
i) V= v 1 U Y1 U ••• U v.,con V;;l:0paratodo 1 ::s
para extenderla tabla de la parte (a) para todos,, s
i:Snyv1 nv.,::;0, para l :Si<j:Sn:y,
15y0Si::51'1.
ü) {u, u) E Esi y sólo si u E V;, u E V,,coni • i•
e) Para m, n E z·, definimos una función delta
a) Si lv;I = 1.paratodo I s i::S n,¿cuáJescl
d(m, 1'1) como l CIJ.andom """ y Ocuandom #n. Si
grafo resultante? · k =IIIÍX{m, 1'1}, entonces el m1merode Stirlingdel
b ) Si I v,I = p;,_para todo 1 :s i s n, determi- primer y segundo tipos se relacionan por
ne \VI y ÍE[.
e) Si V1 1 : p1, para todo l ::s i :s n. ¿cuánto vale
IC(G)? ¿Cuántas aristas tiene G?
! s(m, i)S(i, n) - 6(m, n) - ! S(m, i)r(i, n).
d ) Encuentre los valores den(>l) y P1o/>2, • • ,p.
Verifique estos resultados para los casos en que
para los que el grafo l'l-partito completo es pla-
m = n = 4 y m = 5, n = 4.
no.
12
Árboles

e ontinuaremos nuestro estudio de la teoría de grafos y nos centraremos en un tipo espe•


cial de grafo llamado árbol. Los árboles fueron utilizados por primera vez en 1847 por
Gustav Kirchhoff (1824-1887) en su trabajo de redes el6cllicas, aunque posteriormente
fueron desarrollados y definidos de nuevo porArthurCayley (1821-1895). En 1857, Caylcy
usó estos grafos especiaJes para enumerar los isómeros diferentes de los hidrocarburos
saturados C.H2,,. 2, n E z•.
Con la aparición de los computadores d.igitaJes se encontraron nuevas aplicaciones para
los árboles. AJgunos tipos especiales de úboles son muy .importantes en el estudio de las
cstrucruras de datos. las ordenaciooest, la teoría de codificación y en la solución de ciertos
problemas de optimización.

12.1
Definiciones, propiedades
y ejemplos

Definición 1 2.1 Sea G =(V, E) un grafo no dirigido sin la:ros. El grafo Ges un ároo/i si Ges conexo y no
contiene ciclos.

En la figura 12.1, el grafoG1 es un árbol, peroG2 no lo es, pues contiene el ciclo {a. b),
{b, e}, (e, a}. El grafo G, no es conexo, por lo que no puede ser un árbol. Si'\embargo,
cada componente de G} es un árbol; en este caso G1 es un bosque.
Si un grafo es un árbol, escribimos Ten vez de G, para enfatizar esta estructura.
En ]a figura 12.1 vemosqueG1 es un subgrafodeG,,, que G1 contiene todos los vértices
de G1 y que G1 es un árbol. En este caso, G1 es un árbol rccubridor de G2• Por Jo tanto, un
4rbol rec"1Jridor de un grafo concito es un subgrafo rccubridor que wnbifn es un árbol.
Podemos pensar en un árbol recubridor como aquel que proporciona una conectividad
minimal para el ¡:rafo y como un esqueleto mínima] que une los v~rtices. EJ grafoG 3 es un
bosque ncubridor para el grafo G,.

t .EJ ctnmDo SOfflll8 tambi& se UWSocc como ordeoamicoto. (J,I. IUI E.) ..,,,._
t Como en d caso de los p!os.. la ccrmioologfa en d estudio de los úboles DO es ettJDdar" y d lector
podña encontrar alpnu difemtciu de un libro de texto a otro, y de u.a país • ouo.

607
608 Capítulo 12 Árboles

}:( }:~:
~ :\
(G,)

Figura 12.1
f

(G1)
f
n

u~

(G3)
p
V

l
n

u~

(G,I
p
V

Ahora analizaremos algunas propiedades de los árboles.

TEOREMA 12.1 Si a,b son vértices distintos en un árbol T =(V.E). entonces hay un único camino que
conecta estos vértices.
Demostración: Como T es conexo. e xiste al menos un camino en T que conecta a y b. Si
hubiera más camínos de este tipo, por medio de dos de ellos, algunas aristas podrían for-
mar un ciclo, pero T no tiene ciclos.

TEOREMA 12.2 Si G • (V. E) es un grafo no dirigido, entonces Ges conexo sí y sólo si G tiene un árbol
recubridor.
Demostración: Si G tiene un árbol recubridor T, entonces para cada par a, b de vértices
distintos de V, un subconjunto de aristas de Tproporciona un (único) camino entre a y b;
por lo tanto, G es conexo. Por el contrario, si G es conexo y G no es un árbol, podemos
eliminar todos los ciclos de G. Si el subgrafo resultante G 1 no es un árbol, entonces G1
debe contener un cic1o C1. Eliminarnos una arista e 1 de C1 y sea G2 = G1 - e,.
Si G1 no
contiene ciclos. entonces G2 es un árbol recubridor de C , pues G2 contiene todos los
vértices de G, no tiene lazos y es conexo. Si G 2 contiene un ciclo C2 , entonces elimina-
mos una arista e2 de C2 y nos ocupamos del subgrafo G3 = G2 - e 2 = G 1 - {e1, e1 } . De
nuevo, si G, no contiene ciclos, entonces tenemos un irbol recubridor de G. En caso
contrario, continuamos este procedimiento un m1mero finito de veces, hasta obtener un
subgrafo recubridor de G , que sea conexo, sin lazos ni ciclos y que, por lo tanto, sea un
árbol recubridor de G.

La figura 12.2 muestra tres árboles no isomorfos para cinco vénices. Aunque no son
isomorfos, tienen el mismo número de aristas (cuatro). Esto nos lleva al siguiente res ulta-
do general.
12.1 OefinKiones. propiedades plos 609


( T,)

Figura 12.2
(T2)
'\ + ( T,)

MA 12.3 En cualquier árbol T= (1( E), 1VI = 1E l + l.


Demostración: La demostración se obriene al aplicar la forma alternativa de la inducción
matemática a I E 1. Si I E 1 = O. entonces el árbol está formado por un solo vtnice aislado,
como en la figura 12.3(a). En este caso. 1VI = 1 = IEI + l. Las panes (b) y (e) de la figura
verifican los resultados para estos casos en que IEJ = 1 o 2.
Supongamos que el teorema es cieno para cualquier árbol que cuente con un máximo
de k aristas, dondek ~ O.Ahora consideremos un árbol!= (V, E ), como en la figura 12.4.
tal que IEI = k + l. [La(s) arista(s) punteada(s) indica(n) que una pane del árbol no
aparece en la figura.] Si, por ejemplo, eliminamos de Tia arista con extremos y, z, obten-
dremos dossubárbo/,s, T, = (V1,E1)y T,=(V,. E,). tales que IVI = 1V, 1 + 1V,I y IE,I +
1E, I + 1 = 1El. (Uno de estos subárboles podría estar formado por un solo vfnice si, por
ejemplo, eliminarnos la arista con extremos w, .t.)ComoO :s; IE,1 :s k y O :s IE,I :s; k, de
la hipótesis de inducción se sigue que JE,I + 1 = 1V;I. para y = l. 2. En consecuencia, IVI =
IV,1 + JV,I =( JE,1 + l)+( IE,I + l) = (IE,1 + IE,I )+ 1)+ 1 = IEI + 1 yel teorema se
sigue de la forma alternativa de la inducción matemática.

(al
.
figur~ 12.3
/
(bl >
(e)

Figura 12.4

Al analizar los árboles de la figura 12.2 vemos tambiln que cada uno tiene al menos dos
véniccs colgantes; es decir~véniccs de grado l. Esto también es cieno en general.

TEOREMA 12.4 Para cualquier árbol T.: (V. E), si IVI ;;?: 2,entonces Ttiene al menos dos vérticcscolganlCS.
D<moruaci6n, Sea I VI =n a: 2. Por el teorema 12.3 sabemos que IEI =n - l. porloque.
del teorema 11.2, se s igue que 2(n - 1) = 2IEI = L,..vgrad(i,). Como Tes conexo, sabe-
610 Cap~ulo 12 Árt>oles

mosque grad(u) 2: 1 para todo u E V. Si Ttiene menos de dos vénices colgantes, entonces
grad(u) 2: 2 para todo 'O E V o grad(u*) = l para un único vértice u • E V. En e l primer
caso oblcncmos la contradicción

2(n - 1) = L grad(u) >e 21VJ = 2n•


.ev
Para e l segundo caso tenemos
2(n - 1) = L grad(u)"' 1 + 2(n -
.ev
1),
otra contradicción.

En la figura 12.5 tenemos dos árboles, cada uno con 14 vútices (con nombresC y H) y 13
aristas. Cada vértice tiene grado 4 (C representa un átomo de carbono) o grado 1 (H repre-
senta un átomo de hidrógeno). La parte (b) de la figura tiene un átomo de carbono (C) en
eJ centro del árbol . Este átomo de carbono es adyacente a c uatro vértices, tres de los cuaJes
tienen grado 4. No existe un vértice (átomo de carbono) en la pane (a) con esta propiedad,
por lo que podemos afirmar que estos árboles no son isomorfos. Sirven como modelos de
los dos isómeros químicos correspondientes al hidrocarburo saturadot CJ{10 • La pane (a)
representa al o-butano (antes llamado butano) y la parte (b) representa el 2-metil propano
(antes llamado isobutano).

H
1
H- C-H H

1 1
H- C - H H H- C- H H
1 1 1 1
H- C -H H - C - - C - - C-H
1 1 1 1
H-C - H H H H
1
H
<•> (b)

Figura 12.5

En el siguiente ejemplo damos un segundo resultado de química.

Si un hidrocarburo saturado [en panicular, un hidrocarbono acíclico (sin ciclos) con enla-
ces simples, llamado akano) tiene n átomos de carbono, mostraremos que tiene 2n + 2
átomos de hidrógeno.

t El adfetivo .SDIMrado se U$1. p.-. indicar que. para el aúmen:, de Momos de carl>oo.o presentes ea la
mol6cula. se tiene el tnlmero mhimo de ftomos de hidrógeno.
12.1 Definicíones, propiedades y ejemplos 611

Consideremos el hidrocarbono saturado como un árbol T= (V. E ) y seak el número de


vértices colgantes, o átomos de hidrógeno, en el árbol. Entonces, con un total den + k vé r-
tices, donde cada uno de los ,r átomos de carbono tiene grado 4 , vemos que

4n + k = ¿ grad(u) = 21EI = 2(jVj - 1) = 2(n +k - 1),


.ev

4n + k = 2(n + k- l) ⇒ k = 2n + 2.

Cerraremos esta sección con un teorema que proporciona varias formas de caracterizar
los árboles.

Las siguientes proposiciones son equivalentes para un grafo no dirigido G = (V. E) sin
lazos:
a) Ges un árbol.
b) Ges conexo, pero si se elimina cualquiera de sus aristas, G quedará desconectado
en dos subgrafos que son árboles.
e) G no contiene ciclos y I VI = IEI + l.
d) Ges conexo y IVI = IEI + l.
e) G no contiene ciclos y si a, b E V con {a, b} ft. E, entonces e1 grafo que se obtiene
de añadir la arista {a, b} a G tiene precisame nte un ciclo.
Demostración: Demostraremos que (a) => (b), (b) ⇒ (c) y (c) ~ (d); dejaremos al lector
las demostraciones de (d) => (e) y (e) => (a).
[(a) ⇒ (b}]: Si Ges un árbol, entonces Ges conexo.Así, sea e= {a, b} cualquier arista
de G. Entonces, si G - e es conexo, existen al menos dos caminos en G de a a b. Sin
embargo, e sto contradice el teorema 12.l. Por lo tanto, G - e es disconexo y podemos
separar sus vértices en dos subconjuntos: (1) el vértice a y todos los vértices que pue-
den alcanzarse desde a mediante un camino e n G - e; y (2) el vértice b y todos los
vértices que puede n alcanzarse desde b mediante un camino en G - e. Estas dos compo-
nentes conexas son árboles, puesto que un lazo o ciclo que estuviera e n cualquiera de
ellos estaría también en G.
[(b) ⇒ (e}]: Si G contiene un ciclo, sea e= {a . b } una arista de este ciclo. Entonces
G- e es conexo, lo que contradice la hipótesis señalada en (b).
Para demostrar que I v¡ = 1El + 1, consideremos dos casos. Si I VJ = 1, entonces
IEI = O, pues el grafo no tiene lazos. Así, !VI = 1 = O+ 1 = IEI + l. Si j VI > 1.
entonces I E 1 2: 1, puesto que el grafo es conexo. Sea e E E y consideremos e l subgrafo
G - e de G. De laparte(b) sabemos queG- e = 7 1 U 7 1 , donde Ti, T2 son árboles. Si T 1 ;;;
(V1.E1),T, = ( V,,E,),entonces ! Vil+ IV, I = IVI y IEil + IE,I = IEI-I;delteorema
12.3obtenemos IV;I = IE,I +1.parai= 1, 2. Eneonsecuencía, IVI ~ j Vd + IV,I =
(I Eil + l ) +(IE, I + l)= ( IEil + IE,I + l )+ 1 = IEil + l.
[(e)~ (d)]: Sea lC(G)-;;; r y sean Gi. G1 , . . . , G,las componentes de G. Para l s i 5 r.
seleccionamos un vértice u ; E G y añadimos las r - l aristas {u 1, ~}, { u,. u 3 } , • .
{u,_ " u,} a G para formar el grafo G' =(V', E '), que es un árbol. Puesto que G' es un
612 Capitulo 12 Árboles

árbol,sabemosque IVI = IE'I + 1 por elteorema 12.3.Perode (c), IVI = IEI + !. por
lo que IEI = IE I y r- 1 =0. Con r = 1, se sigue que Ges conexo.

EJERCICIOS 12.1 1. a) Trace los grafos de todos los árboles no isomodos con seis vértices.
b) ¿Cuántos isómCTos tiene el hcuno {~ 14)?
2. Sean T1 = ( V1, E 1) , 7 2 = ( V2, EJ dos árboles taJes que IEd = 17 y I V2 1 = 2IV.(. Determine
JV,I. JV,I Y IE,I .
3. a) Sea F 1 = ( V1, E1) un bosque de siete árboles con IA1 1= 40. ¿Cuánto vale I Vil?
b) Si F2 = (V2, f:i) es un bosque con I V11= 62 y IE11= 51, ¿cuántos árboles determina Fz?
4 . a) Si G = (V. E) es un bosque con I VI = V. IEI = e y J.COmponc ntcs (átboJes},¿qué relación
existe entre u, e y 1C'?
b) ¿Cuál es el número mínimo de aristas que debemos añadir a G para obtener un árbol?
5. ¿Qué tipo de árboles tienen e xactamente dos vértices colgantes?
6 . a ) Verifique que todos los árboles sean planos.
b) Obtenga el teorema 12.3 de la pane (a) y el teorema de Euler para grafos planos
7. Dé un ejemplo de un grafo no dirigido G = (V, E ) tal que I YI = IEI + 1 pero que G no sea un
árbol.
8. a) Si un árbol tíene cuatro vtrtices de grado 2, WlO de grado 3, dos de grado 4 y uno de grado
S. ¿cuántos vértices colgantes tiene?
b) Si un árbol T= (V, E ) tiene u 2 vértices de grado 2, u, vértices de grado 3, ... y u .. vértices
de grado m, ¿cuánto valen I VI y I El?
9. Si G = (V. E ) es un grafo no dirigido sin lazos, demuestre que G es un árbol si existe un único
camino e ntre dos vértices cualesquiera de G.
1 O. El grafo no dirigido conexo G = (V, E } tiene 30 aristas. ¿CUál es el máximo valor que puede
tener IVI?
11 . Sea T= ( V. E) un árbol con I VI = n ~ 2. ¿CUántos cam.inosdistintos existen (como subgrafos)
en T?
12. Sea G = (V, E) un grafo no dirigido, conexo, sin lazos. donde V = {u1 • 'l>t, u,, ... , u.} n 2:: 2,
grad(u1) = 1 y grad( u;) ~ 2 para 2 S i s n. Demuestre que G debe tener un ciclo.
1 3. Encuentre dos árboles recubridores no isomorfos para el grafo completo bipartitoK2.,. ¿Cuán-
tos irboles recubridores no isomorfos tiene Ku?
14. Sea G = ( V, E ) el grafo no dirigido sin lazos de la figura 12.6. (a) ¿Cuánto vale lC(G)? {b)
¿CuánlOs bosques recubridores distintos (aunque algunos podrían ser isomorfos) tiene el grafo
G dado?

Figura 12.6
12.1 Definiciones, propiedades ejemplos 613

1S. ¿Cuántos bosques recubridores distintos (aunque isomodos) tiene el grafo C.-(el ciclo con n
v&tices. n ~ 3)?
16. De<ennine el nllmero de bosque$ ra:ubridores distintos (aunque algunos podrwl ser isomoños)
para cada uno de los gnfos de la figun 12.7.

,n 1'.
dl_/\J • h
(1)

(3)

F't9ura 12.7

17. SeaG • (V. E)un grafo nodirigjdoconexo sin laz.osy Hun subgrafodeG. Elcompl~mtJUario
d,,t H ~n G es el subgrafo de G formado por las aristas de G que no e$l1n en H (junto con los
vhtices íncidentes a estas aristas).
a) Si Tes un '1'bol rccubridor de G. demuestre que el complemenwio de Ten G no contiene
un conjunto de tortc de G.
b) Si e es un conjunto de cone dcG, dcmuesuc que cJ compJcmenwio de e en G no contieoe
un 4tbol rccubridor de G.
18. Complete la demostración del teorema 12.5.
19. Un '1bol etiquetado es aquel en que los vtrtices estin etiquetados. Si el trbol tienen vfrticcs,
entonces {l. 2. 3, . .. , n} seusacomod conjuntodc etiqucw:. Vemos quedos árboles q ue sin
etiqucw: son isomorfos pueden volverse no isomorfos al etiquetarlos. En la figura J2.8, los
primeros dos trboles son isomorfos como trboles etiquetados. El tercero es isomorfo a los
ouos dos si no se tienen en cuen1.a las etiquetas; sin embargo. como árbol etiquetado, no es
isomodo a ellos.
El número de úboles DO isomorfos con n v&tiees etiquetados puede conwsc cstablecieodo
una correspondencia uno a uno entre estos Mboles y las n"- 1 sucesiones (con repetición) z.,
z1, • • • ,z,._2 de {l, 2, 3, ... , r1}. Si T es uno de esos irbolcsctiquctados. usamos el si¡uiente
aJ¡oritmo para establecer la correspondencia uno a uno. (En este cuo. T tiene al menos una
arista.)
Paso 1: Hacemos el contadorii¡uaJ a 1.
Paso 2: Sea T(Q s T.
'1,'1, '1,
614 üpítulo 12 Árboles

3 1 3 1 6 5

6 6 3
s 5 1
ro w (ii)

Figura 12.8

Paso 3: Puesto que un úbol tiene al meoos dos vtnic.es colgantes, seleccionamos el vMic:t
colgante en T(i) con la mínima etiqueta Y.r Ahora eliminamos la arista {x.. y,-) de 7'(1) y usamos
~ como la i-&ima componente de la sucesión.
Paso 4: Si i"" n - 2. tenemos la sucesión correspondiente al úbol etiquetado dado T(I). Si i ;
n - 2, incrementamos i en l , hacemos 7(,) i¡u.al al subárbol resulttnte en el paso 3 y regrcs.a-
mos al paso 3.

a) Encucotre la sucesión de seis dígitos para los úboles (i) y (lü) de la figura 12.8.
b) Si u es un v&tice de T. muestre que el ownero de veces que la etiqueta de u aparece en la
sucesión .r., .r1•.•. , x. _2 es grad(u) - 1.
e) ReconstNya el árbol etiquetado con ocho v~rtic:es asociado a la sucesión 2, 6, S, S. 5, S.
d) DesarroUe un algoritmo para rcconsuuir un m"bol a partir de una sucesióo dadax1,.r1, • •• ,Z..->

12 .2
Árboles con raíz

Ahora veremos Jos árboles dirigidos. Existen diversas aplicaciones para un tipo panicular
de árbol dirigido. llamado árbol con raíz.

tefinici6n 12.2 Si Ges un grafo dirigido, entonces Ges un 4rbol dirigido si el grafo no diri&ido asociado
con G es un árbol. Sí G es un árbol dirigido, G es un árbol con raít. si existe un único
v~n.ice r eo G, llamado la ra(z, tal que el grado de entrada de r ::ge(r ) = O y para todos los
demás vtrtices u, el grado de entrada de u es ge(u ) ; l.

El árbol de la parte (a) de la figura 12.9 es dirigido pero sin ralz; el árbol de la pane (b)
tiene r como raíz.
Trazaremos los árboles con raíz como en la figura 12.9(b), pero con el convenio de que
las direcciones van del nivel superior al inferior, de modo que no son necesarias las fle-
chas. En un árbol con raíz, un vtniceucon gradodesalidag.s(u);O es uruihoja (ovtnice
ttnn.inal). Los vtn.ices u. u,x. y,:: son bojas en la figura 12.9(b). Todos los demás vénices
son nodos de ramificación (o vtnices internos).
12.2 Árboles con raíz 615

1
f

<•>
g h

(b)
Figura 12.9

Consideremos el vtnices en el árbol con raíz [Fig. 12.9(b)]. El camino desde la raíz r
bastas es de longitud 2, por lo que decimos ques está en eJ nivel2 del árbol, oques tiene
númuo <Ú nivtl 2. En forma análoga, x está en el nivel 3, mientras: que y tiene número de
nivel 4. Decimos que ses un hijo den y n es padre des. Los vlnices w, y y t son desctn•
ditnll.$des, n y r, mientras c:iues, n y r son ascendien1ude w, y y t.. En gener:a.J., si U 1y Ut
son vfrtices de un árbol con raíz y u 1 tiene un número de nivel más pequeño, entonces u ,
es un ascendiente de U-z (o u,. es un descendiente de u 1) si existe un camino de u 1 a 0¡. Dos
vlniccs con un padre común son hermanes. Esco ocurre para los vértices q y s, cuyo padre
es el vútice n. Por úJtimo, si U1 es cualquier vtn..ice del árbol, el subárbol t.n u 1 es el
subgrafo inducido por la raíz u 1 y todos sus descendientes (aunque podría no tenerlos).

En la figura 12.JO(a) usamos un árbol con raíz para representar el índice de un libro con
tres capítulos (Cl , C2 y C3). Los vénices con el número de nivel 2 son para las secciones
de un capítulo; los del nivel 3 representan las subsecciones de una sección.

Libro Lib<o
C1

/1~ S1.1

;í\
Sl.2
C2
/\ C2 C3
53.1
S1.1 S1.2 S3.1/53.2\53.3 53.2
S3.2.1
S3.2.2
53.2.1 S3.2.2
S3.3
<•> (b)

Figura 12.10
616 Capítulo 12 Árboles

E1 árbol de la figura 12.IO(a) indica un orden para los vértices, si analizamos los
subárboles CI. C2 y C3 de izquierda a derecha. (Este orden volverá a aparecer en esta
se.cción, en un contexto más general.) Ahora consideraremos un segundo ejemplo que
proporciona dicho orden.

En el árbol T que se muestra en la figura 12.11, las aristas (o ramas, como se les llama a
menudo) que salen de cada vértice interno están ordenadas de izquierda a derecha. Por lo
tanto.T es un círbol ordenado con raíz,

Etiquetamos los vértices de este árbol mediante el siguiente algoritmo.


Paso 1: Primero asignamos a la raíz la etiqueta (o dirección) O.
Paso 2: A continuación, asignamos los enteros positivos l , 2. 3, .. . a los vértices del
nivel 1, de izquierda a derecha.
Paso 3: Ahora. sea u un vénice interno en el nivel n ~ l y sean u 1, U:z, •.• , U.t los hijos
de u (yendo de izquierda a derecha). Si a es la etiqueta asignada al vértice u, asignamos
las etiquetas a. l , a.2, ... , a.ka los hijosº" Uz, ... , u.h respectivamente.

En consecuencia, cada vértice de T. excepto Jaraíz, tiene una etiqueta de la forma


a 1.a2 .a 3 • • • • • a,4 si y sólo si ese vértice tiene m1mero de nivel n. Esto se conoce como el
sistema universal de direcciones.
Este sistema proporciona una forma de ordenar todos los vértices de T. Si u y u son dos
vértices de T con dmcciones by e, respectivamente, definimos b < e si (a) b;;; a 1.ai . .... a..
y e"" a 1.a2 • • a,..a,u 1 • ••• • a,4. con m < n~ o (b) b = a 1 .tlz .....a_.x1 ••••• y y e;:; a 1.lJ.:!

.. a,...tz ..... t. donde xlt x,_ E z+yxl <x,..

3.2

2.2.1

1.2.3.1 1.2.3.2
Figura 12.11
12.2 Árboles con raíz 617

Para el árbol que estamos examinando. este orde n produce

:s::,s:::,s ::s :: J'.,


1.1 122 12 32 2 2.2.1 3.2

Puesto que nos recuerda el orden alfabético de un diccionario, este orden se conoce como
l«icogrdfico o d~l diccionario.

Consideremos ahora una aplicación de un árbol con raíz en el estudio de las ci~ncias de
la computación.

a) Un árbol con raíz.esbUUJriosi para cada véniccu, gs(u) = 0. 1 o2;es decir, si u tiene
cuando mucho dos hijos. Sigs(u) = Oo 2 para todo u E v. entonces el árbol con raíz
es un árbol binario completo. Este árbol pue.de representar una operación binaria.
como en las partes (a) y (b) de la figura 12.12. P.ira evitar confusiones al trabajar
con una operación no conmutativaº, etiquetamos la raíz como o y pedimos que el
resultado sea a O b, donde a es el hijo izquierdo y bes el hijo derecho de la raíz.

l\+b) /\
(• -bl
,/ \b
<•>
b a
(b)
Fígu r a 12.12

b) En la figura 12.13 extendemos las ideas presentadas en la figura 12.12 para cons-
truir el árbol binario con raíz para la expresión algebraica

((7-a)/5) • ((a + b) t 3),

donde * denota la multiplicación y i la exponenciación. En este caso construimos


el árbol, como aparece en la parte (e) de la figura. de arriba hacia abajo. En primer
lugar construimos un subárbol para la expresión 7 - a en la parte (a) de la figura
12. 13. Esto se incorpora (como el subárbol izquierdo de/) en el árbol binario con
raíz correspondiente a la expresión (7 - a)/5 de la figura 12.13(b). Luego, de modo
similar, construimos los árboles binarios con rafa de las panes (c) y (d) para las
expresiones a + b y (a + b) i 3, respectivamente. Por último, usamos los dos
subárboles de las panes (b) y (d) como los subárboles izquierdo y derecho, respec-
tivamente, para•, con lo que obtenemos el árbol binario con raíz (de la Fig. 12.13(e)]
para ((7 - a)/5) • ((a + b) i 3).
518 Capltulo 12 Árboles

\•)
/\ /\
7
\el
b

~/>
7
(bl (d)
b
(e)

Figura 12.13

Hemos usado las mismas ideas en la figura 12.14. donde encontramos los~
les binarios con raíz para las e xpresiones algebraicas

(a - (3/b)) + 5 [en la parte (a)] a-(3/(b + 5)) [en la parte (b)).

1t
(al
3 b
(b)
b S

Figura 12.14

e) Para evaluar r + (uu)/(w + x -y) en un lenguaje por procedimientos, como BASIC,


escribimos la expresión en la format +(u• u)/(w + x-y i z). Cuando el compui.
dor evalúa esta expresión, realiza las operaciones binarias (dentro de cada paneque
aparece entre paréntesis) seg6n una jerarquía de operaciones, en la que la
exponenciación precede a la multiplicación y la división, las que a su vez preceden
a 1a suma y la resta. En la figura 12.15 numeramos las operaciones según el orden

t+(u•\l)l(w + x -y t z)
111 1 11
® ©® ®@(D Figura 12.15
12.2 Árboles con raíz 619

en que el computador las realiza. Para que el computador evalúe esta expresión,
primero debe analizarla a fin de realizar las operaciones en el orden dado.
No obstante, en vez de analizarla de ida y vuelta varias veces, la máquina con-
viene la expresión en una notación independiente de los paréntesis, conocida como
notación polaca. en honor del lógico polaco (ucraniano, en realidad) Jan Lukasiewicz.
(1878-1956). En este caso, la notaeión infija a O b para una operación binaria O se
conviene en o ab, la notación prefija (o polaca). La ventaja es que la expresión de la
figura 12.15 puede escribirse de nuevo sin paréntesis como

+t/•uv+w-x f yz,

donde la evaluación se realiza de derecha a izquierda. Cuando se halla una opera-


ción binaria. ésta se reaJjza con los dos operandos que estén a su derec-ha. El resul-
ta.do se considera entonces como uno de los operandos de la siguiente operación
binaria que se encuentra al recorrer la expresión hacia la izquierda. Por ejemplo.
con las asignaciones t : 4, u • 2, u"" 3, w = 1, x = 9, y = 2, z : 3, se realizan los
siguientes pasos para evaluar la expresión

+1/•uv+ w -x t yz.
o +4 / •23+1 - 9 l 23
2l3=8
2) +4/•23+ 1-98
9-8=1
3)+4 / •23+11
1+!=2
4) + 4 / • 2 3 2
2•3=6
5)+4/62
6 / 2=3
6) +4 3
4+3=7

Así. el valor de la expresión dada para las asignaciones anteriores es 7.

El uso de la notación polaca es importante para la compilación de programas y


puede obtenerse representando una expresión dada como un árbol con raíz. como
se muestra en la figura 12.16. En este caso, cada variable (o constante) se usa para
etiquetar una hoja del árbol. Cada vénice interno se etiqueta con una operación
binaria cuyos operandos izquierdo y derecho son los subárboles izquierdo y dere-
cho que determina. Si partimos de la raíz y recorremos el árbol de arriba hacia abajo
y de izquierda a derecha, como se muestra en la figura 12. 16, encontraremos la
notación polaca escribiendo las etiquetas de los vértices en el orden e-n que se en-
cuentran.
620 Capítulo 12 Árboles

,\
' \,!./\ Figura 12.16

Los dos últimos ejemplos muestran la importancia del orden. Existen varios métodos
para la ordenación sístemática de los vértices en un árbol. Dos de ]os más imponantes eo
el estudio de las estructuras de datos son el orden previo y el orden posterior. que definire-.
mos en forma recursiva a continuación.

Definición 12.3 Sea T = (V, E) un árbol con rafzr. Si Tnotiene otros vértices, entonces la misma raíz es el
ncollido en orrkn previo y orrkn posterior de T. Si I VI > 1, sean TI> T1, T3, ... , T. los
subárboles de T, de izquierda a derecha (como en la Fig. 12.17).

a) El recorrido en orden previo de Tvisita primero r y después recorre los vénices de


T1 en orden previo, despu~ los vértices de 12 en orden previo y así sucesivamente,
hasta recorrer los vértices de Ta en orden previo.
b) El recorrido en ortkn posterior de Trecorrrc en orden posterior 16s vértices de los
subárboles T" T2 , • • • , Tt para despu~ Uegar a la raíz..

Demostraremos estas ideas en el ejemplo siguiente.


12.2 Árboles con raíz 621 .

Consideremos el árbol con raíz de la figura 12.18.


a) Orden previo: Después de visitar el vértice l , visitamos el subárbol T1 con raíz en el
vértice 2. Después de visitar el vértice 2 seguímos al subárbol con raíz en el vértice
5 y luego pasamos al subárbol con raíz en el vértice 11. Este subárbol no tiene otros
vértices, por lo que visitamos el vértice 11 y después regresamos al vértice 5, desde
el cual visitamos, e n orden sucesivo, los vértices 12, 13 y 14.Al seguir esto retroce-
demos ( 14 a 5 a 2 a l) a la raíz y después visitamos los vértices del subárbol T2 en eJ
orden pre vio 3, 6, 7. Por último, después de regresar a la raíz por última vez. reco-
rremos el subárbol T3 en el orden previo 4, 8, 9, 10. 15. 16, 17. Por lo tanto, la lista
en orden previo de los vértices de este árbol es 1, 2, 5, 11, 12, 13, 14, 3, 6, 7, 4, 8, 9,
IO, 15, 16, 17.
En este orden, comenzamos en la raíz y construimos un camino lo más lejano
posible. En cada nivel vamos al vértice de la exuema izquierda (que no ha sido
visitado previamente) en el siguiente nivel, hasta llegar a la hoja t Después regre-
samos al padre p de esta hoja t y visitamos a su hermano e (y el subárbol determina- '
do por e) directamente a su dere.cha. Si no existe el hermano e, retrocedemos al
abuelo a de la hoja l y visitamos. si exíste, un vértice u que sea el hermano de p,
directamente a su derecha en el árbol. Continuando de esta manera, visitaremos (la
primera vez que encontremos cada uno) todos los v~rticcs que hay en el árbol.
Los vértices de la figura 12.lO(a), 12.11 y 12. 16 se visitan en orden previo.
b) Orden posterior: Para el recorrido en orden posterior de un árbol, comenzamos en
la raíz r y construimos el camino más largo. yendo al hijo que queda más a la
izquierda de cada vértice interno, cuando se pueda. Al llegar a la hoja R. visitamos
este vértice y después retrocedemos a su padre p. Sin embargo, no visitamosp hasta
después de visitar a todos sus descendientes. El siguiente vértice que visitamos se
encuentra aplicando en p el uúsmo procedimie nto usado e n r para obtener l, sólo
que ahora iremos primero de pal hermano de tque está directamente a la derecha
(de R.). En ningún momento se vísitaun vértice más de una vezo antes que cualquie-
ra de sus descendientes.
Para el árbol dado en la figura 12.18, el recorrido en orden posterior comienza
con un recorrido en orden pesterior del subárbol T1 con raíz en el vértice 2. Esto
produce la lista 11, 12, 13, 14, 5, 2. Pasamos al subárbol 7 2 y la lista en orden
posterior contim1a con 6, 7, 3. Luego para 73 encontramos la lista en orden posterior
8, 9, 15, 16, 17, 10, 4. Finalmente se visita el vértice l. En consecuencia, para este
árbol, el recorrido en orden posterior visita los vértices en el orden 11, 12, 13. 14. 5,
2. 6, 7, 3, 8, 9, 15, 16, 17, JO, 4, l.

11 12 13 14 Fígur a 12.18
522 Caphulo 12 Árboles

En el caso de los árboles binarios con raíz, existe un tercer tipo de recorrido del árl>ol:
el recorrido en orden simétrico. En este caso ne consideraremos los subárboles como pri-
mero y segundo, sino en términos de izquierda y derecha. La definición fonna1 es recursiva.
al igual que las definiciones del orden previo y el orden posterior.

Definición 12.4 Sea T ,.._ ( V. E) un árbol binario con raíz, donde r es la raíz;.
1) Si I VI = 1, e ntonces el vénice res el recorrido en orden simétrico de T.
2) Si I VI > 1, sean T1 y T0 los subárboles izquierdo y derecho de T. El recorrido en
orden simétrico de T recorre primero los vértices de T, en orden simétrico, después
visita la raíz r y luego recorre, en el orden simébico, los vértkes de T0 •

Vemos que un subárbol izquierdo o derecho podría ser vacío.Ade más, si u es un vértice
de dicho árbol, gs(u) = 1 y w es el hijo de u, debemos distinguir entre ser el hijo izquierdo
y el hijo derecho.

Como resultado de los comentarios anteriores, los dos áJ:boles binarios con rafz que se
muestran e n la figura 12.19 no se consideran igua1es. Como árboles binarios con raíz son
iguales. (Cada árbol tiene el mismo confunto de vénices y el mismo conjunto de aristas
dirigidas.) Sin embargo, al considerar el concepto adicional de hijos izquierdo y derecho,
vemos que, en la parte (a), el vénice u tiene a como hijo derecho, mientras que en la parte
(b), el vértice a es el hijo izquierdo de u. En consecuencia, cuando se tiene en cuenta la
diferencia entre el hijo izquierdo y el derecho, estos árboles ya no pueden verse como el
mismo.
Al visitar los vénices del árbol de la parte (a) de la figura 12.19, primero visitamos el
subárbol izquierdo de la raíz r en orden simétrico. Este subárbol está formado por la raíz u
y su hijo derecho a. (En este caso, el hijo izquierdo es nulo, o no existente.) Puesto que u
no tiene un subárbol izquierdo, visitamos en orden simétrico el vértice u y a continuación
su subárbol derecho, a. Una ve z recorrido e l subárbol izquierdo de r, visitamos ahora el
vértice r y después recorremos, en orden simétrico, los vértices del subárbol derecho der.
Esto hace que visitemos primero el vértice b (ya que b no tiene un subárbol izquierdo) y
luego el vé rtice e. Por lo tanto, la lista en orden simétrico para el árbol de la figura l 2. l 9(a)
esu,a, r, b, c.
Al considerar el árbol de la parte (b) de la figura, de nue vo comenzamos por visitar, en
orden simétrico, los vénices del subárbol izquierdo de la raíz r. Sin embMgo, en este caso,
este subárbol izquierdo consta del vénice u (la raíz del subárbol) y su hijo izquierdo a. (El

,~A
<•>
a e a
lb)
e
Figura 12.19
1 2 .2 Árboles con raíl 623

hijo derecho es nulo. o ine xistente.) Por lo tanto, este recorrido en orden simétrico visita
primero e l vértice a (el subárbol izquierdo de u ) y después el vértice u. Puesto que u no
tiene un subárbol derecho, terminamos visitando el subárbol izquierdo de reo orden simé-
trico. Después visitamos la raíz r y luego los conjuntos del subárbol derecho de r, en orden
simétrico. Esto produce la lista en orden simétrico a, u,,; b, e para el árbol que se muestra
en la figura 12.19(b).
Sin embargo, debemos observar que para el recorrido en orden previo ck este ejemplo
particular,t se obtiene el mismo resultado de ambos árboles:
Lista en orden previo: ,; u, a, b, c.
De la misma forma, este ejemplo panicular es tal que el recorrido en orden posterior de
cualquiera de estos árboles es e l siguiente:
Lista en orden posterior; a,u,c.b,r.
Sólo hay una diferencia en el recorrido e n orden simétrico, con sus distinciones enttc el hijo
izquierdo y derecho y entte los subárboles izquierdo y derecho. Para los árboles de las panes
(a) y (b) de la figura 12.19 encontramos que las listas en cxdcn simétrico respectivas son

(a) u,a,r,b,c (b) a,u,r,b,c.

Sí aplicamos el recorrido e n orden sím&rico al árbol binario con raíz que se muestra en la
figura 12.20, veremos que la lista en orden simétrico para estos vértices esp,j, q. f. c. k, g,
a, d, r, b. h. s. m, e, i, t. n, u.

p u Figura 12.20

t ¡Cuidado! Si intercambiamos el orden de los dos hijos existentes (de un padre dado) en un úbol binario
coa raíz. el cambio úecu los recorridos tl'l orden previo, posterior y simw:ico. No obsian1e, si un hijo es
''llulon. sólo se modifica el recorrido ca cxden simétrico.
)24 capítulo 12 Árboles

EJ siguiente árbol recubridor de un grafo conexo está relacionado con el concepto de


orden previo.
· Sea G == ( V, E) un grafo conexo no dirigido con r E V. A panir de r, construimos un
camino simple en G. lo más largo posible. Si este camino simple incluye todos los vértices
de V, entonces el camino simple es un árbol recubridorT de G y hemos terminado. En caso
contrario, sean x. y los dos últimos vértices visitados por este camino, con y como último
vértice. Después retrocedemos al vértice x y construimos un segundo camino simple en G
lo más largo posible, a partirdex, que no incluya los vértices ya visitados. Si no e xiste ta1
camino, retrocedemos al padre p de x y vemos lo lejos que pode mos 11egar a partir de p.
construyendo un camino simple (lo más largo posible. sin ir a vértices ya visitados) basta
una nueva hoja y 1• En caso de que todas las aristas que parten de p conduzcan a vértices ya
visitados, retrocedemos un nivel más y continuamos este proceso. Puesto que el grafo es
finito y conexo, esta técDlca, llamada retroceso o búsqueda en profundidad, detenninara
finalmente un árbol recubridor T de G, donde r se considera como la raíz de T. Por medio
de T podemos ordenar los vértices de G en una lista en orden previo.
La búsqueda en profundidad sirve como un marco de referencia en tomo del cual se
pueden diseñar muchos algoritmos para verificar ciertas propiedades de un grafo.AnaJiza-
remos con detalle uno de estos algoritmos en la .sección 12.5.
Una forma que ayuda a implementar la búsqueda en profundidad en un programa
consiste en asignar un orden fijo a los vértices del grafo ·dado G = (V. E). Entonces, si
e xisten dos o más vénices adyacentes a un vértice u y ninguno de estos vértices ha sido
visitado ante s, sabremos con precisión cuál de ellos debe visitarse primero. Este orden
nos ayudará a desarrollar la descripción anterior de la búsqueda en profundidad como
un algoritmo.
Sea G ==(V.E) un grafo no dirigido, conexo, sin lazos, tal que I VI =n y donde los
vértices están ordenados como Ui, U:?, l>:J, •• . , u •. Para encontrar el árbol recubridor en
profundidad, ordenado con raíz, aplicamos el siguiente algorilmo, donde usamos la varia-
ble U para guardar el vértice que se analiza en un momento dado.

Algoritmo de búsqueda - profundidad


, _ ,, Seasipav, a la ..;able11 y se illicializa T....,d
-deeste ffdice. (El-11, smilamzdol-recabridorqaese••
_qae_
m,llar.)

,-2: s+ ;....., ,..,¡sollfalice_.,......,i,2:5i:5a.talqae(1l.-..) Elf


ao ba sido'Visilldokldnfa.
Si ao s e - tal subfadice, ..,_se.,. al po 1. Ea..,_.
bacelosigaie,m 0 ) - l a a i s t a lv. v.l alflbol T:(2)__.,.11,a-e,;,
,egiesanosalposo2.
,-3: Si1>=11,,dflbol Tesd-nallllillcJr(.......,_a,anfz)4ei. . .
Pmo4' Si11l,11,,-ocedemosdeale11.Si ■esdimedolffdice .......a v •
T, ..,_...,.....aa11y,epesanosal¡mo2.
12.2 Árboles con raíz 625

Aplicaremos ahora este algoritmo al grafoG = (V.E) que se muestra en la figura 12.2l(a).
En este caso. el orden de los vértices es alfabético: a,. b, e, d., e, f. g, h. i, j.
Asignamos primero el vértice a a la variable u e inicia1iz.amos T sólo con el vénice a (la
raíz). En el paso 2, vemos que el vérticeb es el primer vértice tal que {a, b } E E que no ha
sido visitado todavía, por lo que agregamos la arista {a. b} a T, asignamosb at> y regresa-
mos a] paso 2.
Para u = b, vemos que el primer vértice ( no visitado todavía) y que proporciona una
arista al árbol recubridor es d. En consecuencia, agregamos la arista {b. d} a T, asignamos
da u y volvemos al paso 2.
Sin embargo, esta vez no existe un nuevo vénice que podamos obtener de d, puesto que
los vértices a y b ya han sido visitados, por lo que vamos al paso 3. Pero en este caso, el
valor de u es d, no a, así que vamos al paso 4 . Retrocedemos desde d y asignamos el
vértice b a u y después regresamos al paso 2. En este momento vemos que podemos añadir
la arista {b, e } a T.
Seguimos con el proceso y añadimos a continuación las aristas {e,/} y {e. h}. Pero
ahora, como el vénice h está asignado a u, debemos retroceder de h a e aba a. Cuando u
tiene asignado el vértice a por segunda vez. se obtiene la nueva arista {a. e}. Después
agregamos las aristas {e, g}. {g. i} y {g, j}. En e ste momento, hemos visitado todos los
vértices de G, por lo que retrocedemos deja g a e aa. Con u .= a de nuevo, regresamos al
paso 2 y de ahí al paso 3, donde termina el proceso.
El árbol resultante T =(V.E,) apare,;e en la parte (b) de la figura 12.21. La parte (e) de
la figura muestra el árbol T que resulta del ordenamiento de los vértices:}. i, h, g. f e, d,
c. b, a.

Figura 12.21

Un segundo método para buscar los vénices de un grafo no dirigido conexo sin lazos es
la búsqueda en anchura. Aquí designamos un vénice como la raíz y recorremos todos los
vértices adyacentes a la raíz. Desde cada hijo de la raíz. podemos recorrer los vértices (no
visitados) que son adyacentes a uno de estos hijos. Al continuar este proceso. nunca enu-
meramos un vértice dos veces, de modo que no se construye un ciclo; como Ges finito, el
proceso termina en cierto momento.
En realidad ya uti1iz.amos esta técníca anteriormente: en el ejemplo 11.25 de la sección
11.5.
626 Capítulo 12 .AJ-botes

Ciena estructura de datos es útil para desarrollar un algoritmo en este segundo m~


do de búsqueda. Una cola es una lista ordenada en la que los elementos se insertan en 111.
extremo (elfinal) de la Jjsta y se eliminan del otro extremo (el/nnu). Elprimerelcmen-
to insertado en la cola es el primero que sal.e de ella. En consecuencia, una cola se
conoc:e como una estructura FIFO ("'first-in, first-out", primero en entrar, primero ca
salir).
Como en la búsqueda en profundidad, nuevamente asignamos un orden a los vértices
de nuestro grafo.
Comenzamos con un grafo no dirigido conexo sin lazosG = (V.E), donde I VJ ;n y los
v6nices están ordenados como Ui. \½, 'U), . . . , u,.. El siguiente algoritmo genera el árbol
rccubridor en anchura (ordenado con raíz) Tde G para el orden dado.

Utilizaremos el grafo de la figura 12.2l(a) con el ordena. b, c. d. e,f. g. h, i.jpara ilustrar


el uso del algoritmo de búsqueda en anchura.
Partimos del vértice a. Insenamos a en Q e inicializamos T con este vértice (la raíz del
árbol resultante).
En el paso 2 eliminarnos a de Q y visitamos los vértices adyacentes a él: b, e, d. (Estos
vértices no han sido visitados previamente.) Esto permite añadir las aristas {a, b}, {a, e} y
{a, d}.
En et paso 3. insenamos b, e, d (en este orden) en Q y regresamos al paso 2. Ahora
eliminamos estos vértices de Q y visitamos los vénices adyacentes a ellos (no visitados
antes) de acuerdo con el orden dado de los vénices de G. De aquí obtenemos•los dos
nuevos vértices e, g y en las aristas (b, e }, {e, g} que agregamos a T.
Después vamos al paso 3 e insertamos e, g en Q. Regresamos al paso 2, eliminamos
cada uno de estos vértices de Q y encontramos, en orden, los nuevos vé rtices (no visitados
previamente)/. he i, j . Esto nos permite añadir las aristas{ <, /} , {<, h} y {g, i}, {g,j} al
árbol T.
De nuevo regresamos al paso 3, donde insertamos los vértices/, h, i,j e n Q. Pero ahora.
cuando vamos al paso 2 y eliminamos los vértices de Q. no encontramos vértices nuevos
(no visitados previamente). En consecuencia. el árbol T de la figura l2.22(a) es el árbol
12.2 Árboles con raíz 627

.
b~d

<•>
Ax
f h, j

(b) T,
Figura 12.22

recubridor en anchuradeG, para el orden dado. (El árbol 7 1 que aparece en la panc (b)de
la figura surge con el ordenj, i, h. g,f, e, d. e, b, a.)

Ahora aplicaremos las ideas de búsqueda en grafos en un ejemplo más.

Sea G = (V. E) un grafo no dirigido (con lazos), donde los vértices están ordenados como
Si la figura 12.23(a)es la matriz de adyacenciaA(G) deG, ¿cómo podemos
Ui, U:!, ..• , U¡.
usar esta representación de G para determinar si Ges conexo, sin dibujar el grafo?
Con u 1 como raíz. en la parte (b) de la figura analizamos el grafo por medía de su
matriz de adyacencia, usando una búsqueda en anchura. [Aquí hemos dejado de lado los
lazos ya que hemos pasado por alto los unos de la diagonal principal (desde e l extremo
superior izquierdo hasta el extremo inferior derecho).) Primero vjsitamos los vértices ad•
yacentes a U 1 y los enumeramos en orden ascendente segán los subíndices de los vértices
de A.(G). Continuamos con la búsqueda, y como alcanzamos todos los véniccs de G, C
resulta ser conexo.

., .,

T......l \ ~
V1 V2 \1) "'• V~ v, '11

v,
Vi
1 1 1 1 0 0 0
O 1 1 0 0 0 0
•. .,
A(G) - "•
v,
O 1 O O 1 O 1
0001010
'
.,
"'•
v,
0000100
1 0 0 1 0 O0
. ..
.......... Busq~
mprofund~
<•> (b) "''""""' (e)

Figura 12.23
628 C.pítulo 12 Árboles

Podemos obtener la misma conclusión mediante la búsqueda en profundidad de la parte


(e). Este árbol también tiene a u 1 como rafa. Al desarrollar el árbol para el análisis del
grafo, enumerarnos el primer vértice adyacente a u 1 según la fila de u, en A(G). De forma
análoga. después de UJ vemos que u, es el primer vértic.e nuevo en la revisión de A(G). El
vértice u, es una hoja en este irbol, puesto que no podemos visitar vértices nuevos a partir
de u,. Al ~troceder a u,, la fila 2 de A(G) indica que podemos visitar "• desde u,. Al
continuar este proceso, la panc (e) de la figura muestra que Ges conexo.

Ahora regresaremos a nuestro análisis de los árboles con raíz. La siguiente dcfinicióo
generaliza las ideas presentadas en el ejemplo 12.5.

Definici6n 1 2.5 Sea T = (V, E ) un árbol con ralz y sea m E z·.


Tes un árbol m-ariosigs(ll) ,s; m para todo u E V. Sim= 2, el árbol es unárbo/ binario.
Si gs(u) == Oo m para todo u E V, Tes un drl,ol m-ario completo. El caso especial m =
2 da como resultado un árbol binaruJ compkto.

En un árbol m-ariocompleto, cada vértice interno tiene cxactamcntcm hijos. (Ninguna


hoja del árbol tiene hijos.)
El siguiente teorema analiza algunas propiedades de los árboles.

TEOREMA 12.6 Sea T=(V. E ) un árbol m-ario completo con IVI =n. Si Ttiene l hojas eivértices intemos,
entonces(a) •=~+l;(b)l=(m-l)i+l.y(c) i= (l-1)/(m-l)=(n- l)/m.
Demostración: Esta demostración se ha dejado para Jos ejercicios de la sección.

Figura 12.24
12.2 ÁrbolM con raíz 629

El campeonato de tenis de Wimbledon es un torneo con eliminación simple, donde un


j ugador (o un equipo de dobles) queda eliminado despu6s de perder una vez. Si 27 mujeres
compiten en el campeonato de individuaJes, ¿cuántos encuentros deben realizarse para
determinar qui~n es la tenista nlhnero uno?
Consideremos el árbol de la figura 12.24. Si compiten 27 mujeres. existen 27 hojas en
este á.rbol binario completo. por lo que, a partir del teorema 12.6(c), el número de vtnices
internos (igual al nll!nero de encuentros) es;= (t- 1)/(m - 1) = (27 - 1)/(2 - 1) = 26.

.....,,'}!uj Una aula tiene 25 microcomputadores que deben conectarse a un enchufe de pared con
cuatro salidas. Se hacen las conexiones mediante cables de extensión con cuatro salidas
cada uno. ¿Cuál es el número mínimo de cables que se necesitan para poder utiliz.ar todos
los computadores?
El enchufe de pared se considera como la raít. de un árbol m-ario completo, con m = 4.
Los microcomputadores son las hojas de este ú-bol, por lo que l = 25. Cada vértice inter-
no, excepto la raíz., corresponde a un cable de extensión. Así, por la pane (c) del t.e0rema
12.6,existen (l- 1)/(m- l) = (25-1)/(4 - 1) = 8 v&tices internos. Por lo tanto, necesitamos
8 - 1 (el I se resta por la raíz)= 7 cables de extensión.

Definició n 12.6 Si T;;: (V. E) es un árbol con raíz y a es el número de nivel máximo de una hoja de T, Ttiene
altura a. Un '1bol con raíz T de altura a es e.quilibrado si e l número de ni vel de cada hoja
deTesa - 1 oa.

El árbol con raíz que se muestra en la figura 12.18 es un árbol equilibado de altura 3. El
r
árbol en la figura 12.21(c) tiene altura 7 pero no es equilibrado. (¿Por qué?)
El árbol del torneo del ejemplo 12.12 debe estar e.quilibrado para que el torneo sea to
más justo posible. Si no está equilibrado. algtln competidor ,endrá más de una oportun idad
para avanz.ar a la siguiente ronda sin jugar un encuentro.
Anles de enunciar nuestro siguiente teorema, recordemos que para cualquier x E R.
LxJ denota el máximo entero menor o igua) que x(el sucio dex). mientras que í x ldenota
el techo de x.

TEOREMA 12.7 Sea T = ( V. E) un árbol m-ario completo de altura a y l hojas. Entone.es l :S ~ ya e!:
r1og.,1 .
Demostración: Establecemos la demostración del :S m" por inducción sobre a. Si a= 1, T
es un árbol con una raíz y m hijos. En este caso, l = m ::;;; ~ y el resultado es verdadero.
Supongamos que el resultado es cieno para cualquier úbol de altura menor que a, y conside-
remos un árbol T con altura a y l hojas. (Los ntlmeros de nivel posibles para estas hojas son
t, 2. ... , a, donde al menosm de las hoJas están en el nivela.) Las l hoJas de Ttambi6n son
el tolal de hojastde losm subárboles T,. 1 :s; is m, de Tcon rafzcn cada uno de los hijos de
la raíz. Para l :s i s m, sea l., el número de hojas en el subárbol T,. (En el caso en que
coinciden la hoja y la raíz., ~;;: 1. Pero como m ~ 1 y a - l 2 O, tenemos nr-• 1 2 l = &;.) Por
530 úpitulo 12 Árboles

la hipótesis de inducción,,:, ::s TTJ6(Til ::s rrt'- 1,dondea(T¡} es la altura del subárbol T;y por lo
tanto, l = ~ + '2 + · · '.. :S m(rrt' - 1) = 2: 1 = rrt'.
Si f.,,, ::S m", vemos que log.,,t :s log.,(m") =ay como a E z--. se sigue que a ~ ÍJog,..tl.

:OROLARIO 12.1 Sea Tun árbolm-ario completo equilibrado con lhojas. Entonces la altura de Tes Ílog,.tl .
Demostración: Esta demostración se deja como ejercicio.

Cerraremos la sección con una aplicación que usa un árbol ternario (m = 3) completo.

(Árboles de decisión) Dadas ocho monedas (idénticas en apariencia) y una baJanu., si


exactamente una de estas monedas es falsa y más pesada que las otras siete, tenemos que
identificar la moneda falsa.
Numeramos las monedas como 1, 2, 3, ... , 8. Para usar la balanza y comparar los
conjuntos de monedas, debemos considerar tres tipos de resultados: (a) los dos lados están
equilibrados, lo que indica que ninguna de las monedas de ambos lados es falsa: (h) el
platillo izquierdo está más bajo, lo que indica que la moneda falsa está en este platillo: o
(c) el platillo derecho está más bajo. lo que indica que tiene la moneda faJsa.
En la figura 12.25(a), buscamos la moneda falsa comparando primero las monedas 1, 2,
3, 4 con 5, 6, 7 y 8. Si la balanza se inclina hacia la derecha, seguimos la rama derecha
desde la raíz para analizar las monedas 5, 6 comparándolas con 7, 8. Si la balanza se
inclina hacia la iz.quierda, comparamos las monedas 1, 2 con 3, 4. En cada nivel sucesivo
tenemos que verificar la mitad de las monedas, por lo que en el nivel 3 habremos identifi-
cado la moneda falsa.
El árbol de la pane (b) de la figura encuentra !amoneda más pesada en dos pesadas. La
primera pesada compara las monedas 1, 2, 3 con 6, 7, 8. Pueden ocurrir tres resultados: (i)

11, 2, 3, 41- [5, 6, 7, 81 11. 2, 31- 16. 7, 81

111 121 131 141 , 151

111 121 131 141 151 161 171 [81


Árbol binario~ decisión Árbol ternario de decisión
(a ) (Altura • 3 ) {b) (Altura "' 2)

Figura 12.25
12.2 Árboles con raíz 631

la balanza se inclina a Ja derecha, lo que indica que la moneda más pesada es 6. 7 u 8 y


seguimos la rama derecha desde la raíz; (ii) la balanza se inclina a la izquierda y seguimos
la rama izquierda para ver cuál de las monedas 1, 2, 3 es la más pe,ada; o (iii) los platillos
se equilibran y seguimos la rama central para ver cuál de las monedas 4, S es más pe,ada.
En cada v¿rtice interno, la etiqueta indica las monedas que se comparan.A diferencia de la
parte (a), podemos deduc ir una conclusión en la pane (b) cuando no se incluye una mone-
da en una pesada. Por último, aJ comparar las monedas 4 y S, puesto que no puede ocurrir
la igualdad, etiquetamos la boja del centro con e.
En este problema particular, la altura del árbol temario completo debe ser al me nos 2.
Si utilizamos ocho m onedas, el árbol tendrá al menos ocho hojas. En consecuencia. con l
~ 8, se sigue, del teorema 12.7, que a 2: Ílog,&1 ~ 3flog 8l
~ 2, por lo que se necesitan al
menos dos pesadas. Si se usan n monedas, el árbol temario completo tendrá l. hojas. donde
l ~ n, y su a1tura a satisface a ~ flog 1
3n

'8cíoos 12.2 1. Responda las .siguientes pregunw para el úbol de la figura 12.26.
a ) ¿Qut vtnice.s son las hojas?
b) ¿Qut v&tice es la raíz?
e) ¿Qut vtrtieecs el padredeg?
d ) ¿ ~ vtrtices son los descendientes de e?
e) ¿ ~ v&tiees son los hermanos des?
f) ¿Cuil es el número de nivel del vtrticcf!
g) ¿Qui. vfrticcs tienen nómcro de nivel 4?
h ) ¿Cu.'1 es la altura del árbol?
;¡ ¿Es eqwlibrado d úbol?
j) ¿E$ el árbol m-ario para algún m E z·
(mínimo)?
2. Sea T • (V. E) un árbol binario. En la figura 12.27, encontramos un subárbol de T con raíz en
el vtrticep. (La línea punteada que lle¡a al vhticep indica que el árbol T es mayor que como
aparceceo la figura.) Si el nllmerodenivcl del vtrticc 11: cs 37. (a) ¿cuües son los números de
rrivcl de IO$ vbticcsp, s, ,. u,~ x. y y 1,'!(b)¿Cuántos ascendientes tiene el ~ceM1(c)¿Cuintos
ascendientes tiene el váticc y?
3. a) Escriba la expresión ( w + :e - y')l(x • .?1) en notación polaca, mediante un árbol con raíz.
b) ¿Cu'1 es e l valor de la expresión (en notación polaca)
/ f a -be+ d • e/. si a'-' e • d e e = 2, b • fe 4?

p q S l
Figura 12.26 Agura 12.27
632 capitulo 12 Árboles

4 . Sea T= (V. E) un árbol con raíz ordenado mediante un sistema universal de direcciones.
a) Si el vértice u en T tiene direccción 2.1.3.6, ¿cuál es d ndmcro mínimo de bc:rmanos qain
debe tener?
b) Para el vlrtk:c u de la parte (a), encuentre la dirección de su padre.
e) ¿Cuántos ascendientes tiene el vfrticc u de la parte ( a)?
d) Con la presencia de 'U en T, ¿qut otras direcciones debe haber en el sistema?
5. Para el árbol de la figura 12.28, enumere los vtmces se¡dn u.o recorrido en orden previo.•
reconido en orden sim&rico y un recotrido en orden posterior.
6. Enumere los vó-tiCC$ del ftbol que se muestra en la figura 1229 cuando se visiUD en •
recorrido en orden previo y un recorrido en orden posterior.
7. a) Encuentre el 1rbol recubridot en profundidad para el srafo de la fi¡ura ll.67(a) si d«dm
de los vtttices es (i) a, b, e, d, ,,f. g, h; (ii) Ir, g,f. , , d, e, b, a; (iii) a, b, e, d, Ir, g,f. ,.
b ) R epita la panc (a) para el grafo que se muestra en la figura 11.80(.i).
8. Encuentre los árboles rccubridores en anchura para los ¡ratos y órdenes dados en el ejerócio 1.
9. Sea G = (V, E) un crilfo nodiria;ido con matriz de adyacenciaA(G) dada por
., . .. "' ., .. .. ..
,
o1 o o 1
., 1 o1 1 1 o 1
.. o o o 1 o 1 o

T
•• o 1 1 o o o o
., o 1 o o o o 1
•• o o 1 o o 1 o
.,
..
1 1 o
o o
o
o
1 o
o o
o
o !1
Utilice una bllsqueda en anchura c:on base en A(G) para deccnninar si Ges conexo.
1 O. Pata el grafo que se muestra en la figura 12.30. encuentre el úbol rccubridor en anchura.
cuando los vbtices se ordenan como (i) o. b, c. d. ~.f. (ii) ~ b. ~. c. d,f. (iii) ~ c. d, h. t./.

11 12 13

14 15 16 17
Figura 12.28 Figura 12.29
12.2 Árboles con raí1. 633

11. a) Dé un ejemplo de un '1-bol binario T1 = (V1o E 1)donde I V11 ~ 3 y las listas de orden previo
y orden sirn&rico de los vértices de V1 son las mismas.
b) Dé un ejemplo de un ;ubol binario T 2 = (V:, E,) donde I V21 ~ 3 y las listas en orden
posterior y orden simétrico de los vértices de V2 son las mismas.
e) ¿Qué ocurre con los resultados de las partes (a) y (b) si pedimos que T,, T2 sean árboles
binarios completos?
12. a) Sea T= (V, E ) un árbol binario. Si IVI = n. ¿cuál es la máxima altura posible de T?
b ) Si T= (Y, E) es un árbol binario completo y I VI = n. ¿cu.tl es la mhima altura posible de
Ten este caso?
13. Demuestre el teorema 12.6 y el corolario 12.1.
14. Con m, n, i, l como en el teorema 12.6, demuestre que
a)n=(m<-1)/(m-l) b) < =[(m- l )n+l]lm.
15. a) Un '1-bol ternario (o 3-ario) comple10 T= (V. E ) tiene 34 vértices internos. ¿Cuántas aristas
tiene T? ¿Owltas bojas?
b) ¿Cuántos vértices internos tiene un árbol 5-ario completo con 8 17 hojas?
16. Fl úbol binario completo T= (V. E) es tal que V = {a. b, e, ... , i, j, k}. La lista e n orden
posterior de Ves d., e, b, h, i,f,j, k. g. e, a. Obtenga un dibujo de Tmcdiantc esta información
si (a) ta altura de Tes 3; (b) la altura del subárbol izquierdo de Tes 3.
17. Para m ~ 3, podemos transformar un árbol m-ario completo·en un árbol binario completo
mediante la idea que se muestra en la figura 12.31.
a) Utilícc esta ttcnica para transformar el árbol de decisión temario completo de la figura
12.25( b).
b) Si Tes un árbol cuaternario completo de altura 3. ¿cu.tl es la altura mhima posible para T
después de transformarlo en un trbol binario completo?. ¿c:uál es la mínima altura'!
e) Responda la pane (b) si Tes un árbol m-ario completo de altura a.
18. a) En un torneo individual de tenis masculino cada uno de los 25 jugadores trae una lata con
pelotas de tenis. Al jugar un partido se abre y utiliza una lata. que es conservada por el
perdedor. El ganador lleva la lata no abiena a su siguiente encuentro. ¿Cuántas latas de
pelotas de tenis se abrirán durante el torneo? ¿Cuántos panidos se juegan en el torneo'!
b ) ¿En cuántos partidos jugó el campeón del torneo'?
c) Si cada encuentro es ganado por el primero que obtiene tres sets. ¿cuál es el numero máxi-
mo de sets que podrían jugarse (con todos los participantes) durante el torneo?

'

dffia :11\-·~::
, .x·-•
\6!
Figura 12.30 Figura 12.31
s..,_1
/ \,.
634 Capítulo 12 Árboles

19. ¿Cuál es el m1mero máximo de véniccs internos que puede tener un árbol cuaternario completo
de altura 8? ¿,Cuál es este número para el árbol m-ario complc10 de altura a?
20. El primer domingo de 1993, Ricardo y Francisco iniciaron una cadena de cartas, cada uno de
ellos e nvió 5 cartas (a diez amigos distintos). Cada persona que ceciba la carta. debe enviar S
copias a 5 nuevas personas e l domingo siguiente a la llegada de la carta. Después de los prime-
ros 7 dom.Jngos, ¿cuál es el número total de cartas de la cadena enviadas? <,Cuántas se enviarQD
los tres últimos domingos?
'
21. Utilice un árbol de decisión temario completo para repetir el ejemplo 12.14 para un conjunto
de doce monedas, donde exactamente una de ellas es falsa y más pesada que las demás.
22. Sea T= (V, E) un árbol m-ariocompleto equlibrado de altura a i!: 2. Si Ttiene h hojas y h•• 1
nodos de ramificación en el nivel a-1, e xplique por qué h = r,f- 1 + (m- l)b• • 1•

12.3
Árboles y ordenaciones

En el ejemplo 10.4 del capítulo 10 presentarnos la orde nación por el método de la bwbuja
en el estudio de las relaciones de recurrencia lineales. Virilos que el número de compara-
ciones necesarias para ordenar una Lista den elementos es n(n - 1)/2. En consecuencia.
este algoritmo determina una función/: z• ➔ R dada por /(n); n(n - J)/2. Ésta es la
función de complejidad en tiempo (para el peor caso) del algoritmo y la expresamos esc:ri•
hiendo f E O(,r}. En consecuencia, la ordenación por el método de la burbuja necesita
O(n1) comparaciones. Interpretarnos esto diciendo que paran grande, el número de com•
paraciones es igual a cri2, donde e es una constante de proporcionalidad que generalmente
no se especifica, pues depende de factores como el compilador y el computador utilizados.
En esta se.cción analizaremos un segundo métO<io para ordenar una lista dada de n
elementos en orden ascendente. Este métOOo se llama ordenación por inserción y veremos
que el orden de su función de complejidad en tiempo para el peor caso es O(n log2 n).
Haremos esto de la manera siguiente:
1) Primero estimaremos el número de comparaciones necesarias cuandon es una potencia
de dos. Nuestro métOOo utilizará un par de árboles binarios completos equilibrados.
2) Después analizaremos el caso para cualquier m, mediante el material opcional de
los algoritmos ..divide y vencerás" de la sección 10.6.
Sin es un entero positivo arbitrario, comenzamos considerando el siguiente procedimiento.
Dada una lista den elementos que deben ordenarse en forma ascendente, la ordenación
por inserción separa en forma recursiva la lista dada y todas las demás sublistas a la mitad
(o lo más cercano posible a la mitad) hasta que cada sublista contiene un solo elemento.
Después, el procedimiento intercala estas sublistas en orden ascendente hasta ordenar los n
elementos originales. Los procesos de división e inserción pueden describirse mejor me-
diante un par de árboles binarios completos equilibrados, como en el siguiente ejemplo.

(Ortünaci6n por inserción) Con la ordenación por inserción, la figura 12.32 ordena la lista
6, 2, 7, 3, 4, 9, 5, 1, 8. El árbol de la parte superior de la figura muestra la forma en que
el proceso divide primero la lista dada en sublistas de tamaño. 1-: Después se bosqueja el
proceso de inserción usando el árbol de la parte inferior de la figura
12.3 Árboles y ordenaciones 63S

6, 2. i, 3, 4 - 9. s. 1, 8

/ ~
6. 2. 7 -3, 4 9,5 - 1,8

/ 3\ - 4 9 - /S \1 - 8
6, 2-7

/\/\/\/\
6- 2 7 3 4 9 5 l 8

/\
6 2

6 2

V
2. 6 7 3 4 9 S l 8

\/\/\/\/
2, 6,7 3,4 5, 9 1,8

\ / \ s.s. /
2. 3, 4, 6, 7 1, 9

~ /
1, 2, 3,4, s. 6, 7,8, 9
Figura 12.32

Para comparar la ordenación por inserción con la ordenación por el m6todo de la bu.r·
buja. se requerirá detenninar su función de complejidad en tiempo. Necesitaremos e l si-
guiente lema para esta tarea

LEMA 12.1 Sean l 1 y l.,z dos listas ordenadas con números ascendentes, donde L; contienen; e lemen-
tos, j;;; J, 2. Entonces Lty¼ pueden intercal~en una lista ascendcntel con un múimo
de n 1 + "2 - 1 comparaciones.
Demostración: Para intercalar las listasl1, Li en la lista L. utilizamos el siguiente algoritmo.

Cada comparación de un número de Li con uno de L,_ hace que un elemento se coloque al
fina) de la listaL, por lo que no hay más den 1 + n1 comparaciones. Cuando una de las listas
636 Capítulo 12 Árboles

L, o L.: es vacía. no necesitamos más comparaciones, por lo que el número máximo de


comparaciones necesarias es n 1 + ~ - 1.

Para determinar la función de complejidad e n tiempo de ta ordenación po_ym'serción


consideremos una lista den elementos. Por el momento, no nos ocupare__...9).0S"dél problema
general y supondremos que n;;;; 2"'.t En el proceso de separación. primero dividimos la
lista de 2" elementos en dos sublistas de tamaño 211 - 1• (Éstos son los vénices de nivel 1 en
e l árbol que representa el proceso de división.) Al continuar e l proceso, cada lista sucesiva
de tamaño 2"-*, h > k, está en el nivel k y se divide en dos sublistas de tamaño (1/2)(2*-~ =
2"-*- 1• En el nivel h, cada una de las sublistas contiene 2•-•;;;; 1 elemento.
Al invenir el proceso, primero intercalamos las n: 2' hojas en 211 - 1 sublistas ordenadas
de tamaño 2. Estas sublistas están en el nivelh-1 y necesitan (1/2X2"') =2"'- 1 comparacio-
nes (una por cada par). AJ continuar el proceso de inserción, en cada uno de los 2 1 vértic.es
del nivelk, l :S k < h, existe una sublista de tamaño 2 11- 1 , que se obtiene al intercalar las dos
sublistas de tamaño 211 - t - 1 en su hijo (en el nivel k + l). Del lema 12.1. este intercalado
requiere un máximo de 2•-t - 1 + 211 - t - 1 - l = 2•- 1 - 1 comparaciones. Cuando se llega al
hijo de la raíz., existen dos sublistas de tamaño 2"- 1 (en el nivel 1). Para intercalar estas
sublistas en la lista finaJ se necesitan a lo sumo 2• - 1 + 211 - 1 - 1 = 2• - 1 comparaciones.
En consecuencia, para 1 :S·k :S h. en e l nivel k existen 2i. - 1 pares de vértices. En cada uno
de estos vértices hay una sublista de tamaño 2•-1, por lo que se necesitan como máximo
2-6-t• 1 - l comparaciones para intercalar cada par de sublistas. Con 2 1 - 1 pares de vértices en
el nivel k. el número total de comparaciones en el nivelkes cuando mucho de 2"- 1 (Z.' · 1 • 1 -1).
Cuando sumamos sobre todos los niveles k, donde l :S k s h, obtenemos que el número
total de comparaciones es

±2•-•c2'-'" - 1) L2'<2'-• - 1¡ L2• - L2•


.t• I
=
l .O
=
1 ..0 .t...O
= h. 2' - c2• - 1).

Sin= 211, tenemos que h = log:zn y


h · 2'-(2'-1) =nlog,n - (n -1) =n log,n-n + 1,
donde n Jog1 n es el ténnino dominante paran grande. Así, la función de complejidad en
tiempo (para el peor caso) de este procedimiento de ordenación es g(n) =n lofü n - n + 1
y g E O(n log1 n), paran= 2•,h E z•. Por lo tanto.el número de comparaciones necesarias
para ordenar por inserción una lista den elementos es dn lo~ n . para cierta constanle de
proporcionalidad d y para todo n ;?: no. donde no es un entero positivo particular (grande).
Para mostrar que el orden de esta ordenación por inserción esO(n log2 n) para todon E
Z♦•nuestro segundo método usará el resultado del ejercicio 9 de la sección 10.6, que
enunciaremos ahora:
Sean a. b, e E z·, b ;?:; 2. Sí g : z• ➔ R• U {O} es una función monótoná creciente tal que
g(!)sc,
g(n) s; ag(n/ b) + en, paran= b', h E Z',

t El resultado obtenido en el cuo n = 2\ h EN es verdadero todo n E z-. Sin emb3rgo. ncccsiwnos d


material opcio.o.a.l de la sección l0.6 paI11 deducir el resWtado ~ 11 arbitrari.a__ Ésa es la raz.6n por la que
incluimos cStc argumento de conteo en esta pan.e. en bcocficio de los: lectores que no estudiaron la sec:ció11
10.6
12.3 Árooles y ordenaciones 637

entonces, para el caso en que a =b. tencmosg E O(n log n), paratodon E z•. (La base de
la función Jogaritmo puede ser cualquier número real mayor que l . En este caso usaremos
la base 2.)
~ Antes de poder aplicar este resultado a la ordenación por inserción. queremos formular
~ te procedimiento (que se iluslra en la fi¡ura 12.32) como un algoriano preciso. Para
esto, llamaremos al procedimiento bosquejado en el lema 12. 1 el algoritmo de ..inserción'".
Desputs escribiremos ••insena (L,, LJ" para representar la aplicación de ese procedimicn•
to a las listas Li, 4 que están en orden ascendente.
El algoritmo para la ordenación por inserción es un proceso recursivo, pues podría
llamarse a sí mismo. En este caso, la entrada es una lista (llamada Lista) de n elementos
(números reales, por ejemplo).

La funcióng: Z• ➔ R• U (O} mocfuá la función de complejidad en tiempo (para el peor


caso) de este algoritmo, contando el nllmero máximo de comparaciones necesarias para
ordenar por inserción una lista den elementos. Paran• 2•. h E z•. tenemos

g(n) = 2g(n/ 2) + [(n/2) + (n / 2) - 1).


El término 2g(n/2) es el ~ultado del paso 2 del algoritmo de ordenación por inscn:ión,
y el sumando [(n/2) + (n/2) - 1) se sigue del paso 3 del algoritmo y del lema 12. 1.
Como g( l) = O. la ecuación anterior proporciona las desigualdades

g(l) = O,s 1,
g(n)-2g(n / 2) + (n - !) s 2g(n/2) + n, paran= 'l!', h E z•.
También observamosque g(I) =0,g(2) = l,g(3) = 3 y g(4) = 5, por loqueg(I ) :s g(2) :S
g(3) s g(4). En consecuencia. parece que g es una función monótona creciente. La de-
mostración de que es monótona creciente es similar a la demostración dada para la función
de complejidad en tiempo de la búsqueda binaria. Es10 ha seguido el desarrollo del ejem-
plo 10.44 de la scc:ción 10.6, por lo que dejamos los detalles de la demostración de qucg
es monótona creciente para los ejercicios de la sección.
638 Capitulo 12 Árboles

Con a= b = 2 y e = 1, el resultado establecido antes muestra que g E O(n log2 n) para


rodcn E z•.
Aunquen log2 n :S n2 para todon E Z+, el hecho de que la ordenación por el método de
la burbuja sea C>(n2) y la ordenación por inserción sea C>(n log2 n) no implica que el segundo
método sea más eficiente que el primero para todo n E z+. La ordenación por el método de
la burbuja requiere un menor esfuerzo de programación y tarda menos que el ordenamiento
por burbuja para valores pequeños den (según varios factores, como el lenguaje de pro-
gramación, el compilador y el computador). Sin embargo. conforme n crece, la razón
entre los tiempos de ejecución, medidos como (crr2-)/(dn log1 n) = (cld)(n/login) se va
haciendo tan grande como se quiera. En consecuencia, cuando la lista de entrada aumenta
su tamaño, el algoritmo O(n2) (ordenación por el método de la burbuja) tarda significati•
vamente más que el algoritmo O(n Iog2 n} (ordenación por inserción).
Para más detalles acerca de los algoritmos de ordenación y de sus funciones~ complejidad
en tiempo, el lectorpuederevisarlasreferencias [l), [3], [6) y [7) en la bibliografía del capítulo.

EJERCICI OS 12.3 1. a) Déun ejemplo dedos listasL,,Lz,cada unadelascualeses~enorden ascendente, conten-


ga cinc.o elementos. y donde se ne.cesiten nueve comparaciones para intercalar L1• Lz. me-
diante el algoritmo dado en el lema 12. l .
b) Generalice el resultado de la parte (a) al caso en que L, tiene m elementos,½ tienen ele-
mentos y se necesitan m + n - l oomparaciones para intercalar las dos listas.
2. Aplique la ordenación por inserción a cada una de las siguientes listas. Trace los árboles de
separación e inserción para cada aplicación del procedimiento.
•l - 1,0,2, -2,3,6, - 3, 5, 1,4
b) - 1, 7,4, 11,5, -8,15, - 3, - 2,6, l0,3
3. Un procedimiento más eficiente rdacionado con la ordenación por inserción es la onknación
r6pida. Aquí partimos de una lista L:. ai, a 2 , • • • , a. y usamos a 1 como un pivote pata el
desarrollo de dos sublistas L, y L:. de la forma siguie nte . Para i > 1, si a 1 < a 1• colocamos a; al
final d e la primera lista en desarrollo (es decir, L1 e n el final del proceso); en caso contrario,
colocamos a ; al final de la segunda lista½-
Despu6s de procesar a;, i > 1, colocamos a, al final de la primera Lista. Ahora aplicamos d
algoritmo de ordenación rápida en forma recursiva a cada una de las listas L, y Li para obtener
sublistas l.11 , l.12, L,1 y lu- Continuamos este proceso hasta que cada una de las sublistas
resultantes contenga un elemento. Ordenamos entonces las sublistas y su concatenación da la
ordenación buscada para la lista original L
Aplique la ordenación rápida a cada lista del ejercicio 2.
4. Demuestre que la función utilizada en el segundo método para analizar la función de comple--
jidad en tie mpo de la ordenación por inserción es monótona creciente.

12.4
Árboles ponderados y códigos
prefijo

Entre los temas a los cuales se aplican las matemáticas discretas, la teoría de codificación
es uno de aquellos en los que las diferentes estructúras finitas tienen un papel fundamen~
tal. Estas estructuras nos permiten representar y transmitir informacíón codificada en tér+
minos de los símbolos de u n alfabeto dado. Por ejemplo, la forma más frecuente de cocii•
12.4 Átboles ponderados y códigos prefijo 639

ficar o representar tos caracteres internamente en un computador es por medio de cadenas


de longitud fija, que utilizan los símbolos O y 1.
Sin embargo, los códigos desarrollados en esta sección usarán cadenas de diferentes
longitudes. La razón por la que una persona querría desarrollar tal esquema de codifica-
ción y la manera de construirlo son los principales puntos de esta sección.
Supongamos que queremos desarrollar una forma de representar las letras del alfabeto
(i nglés) mediante cadenas de ceros y unos. Puesto que existen 26 letras, debeóamos poder
codificar eslos símbolos en ttrminos de sucesiones de cinco bits, puesto que 2' < 26 < 2 5•
Sin embargo, en el idioma inglés (y cualquier otto). no todas las lettas aparecen con ta
misma frecuencia. En consecuencia. sería más eficiente utilizar sucesiones binarias de
diferentes longitudes. de modo que las letras de aparición más frecuente (como e, i, t) se
representaran mediante las sucesiones m ás cortas posibles. Por ejemplo, consideremos S =
{a, e. n. r, t} un subconjunto del alfabeto y representemos los elementos deSmcdiante las
I sucesiones binarias
a: 01 ,: O n: 101 r: 10 1: l.
Si debemos transmitir el mensaje "ata", se envía la sucesión binaria O11Ol. Por desgra-
cia, esta sucesión tambifo se transmite con los mensajes ..etn", "atet• y ..an...
C onsideremos un segundo esquema de codificación,·dado por

a: 111 ,: O n: 1100 r: 1101 t: 10.


En este caso, el mensaje "ata" está representado por la sucesión 11 110111 y no hay posi-
bilidades de confusión. Es mú, podemos usar el árbol binario completo etiquetado de la
figura 12.23 para decodificar la sucesión 11110111. A partir de la raíz. recorremos la arista
con el n6mcro 1 hasta el hijo derecho (de la raíz) . Continuamos a lo largo de las siguienles
dos aristas con la etiqueta 1 y llegamos a la hoja con la etiqueta a . Por lo tanto, el 1lnico
camino simple de la raíz al vtrtíce en a queda determinado sin ambigüedad por los tres
primeros unos en la sucesión 1111011 l. Oesputs de regresar a la raíz. los siguientes dos
símbolos de )a sucesión, esto es, 10. determinan eJ único camino simple a lo largo de la
arista de la raft a su hijo derecho, seguido por la arisla de ese hijo a su hijo izquierdo. Esto
tennina en el vtrtice etiquetado con t. De nuevo regresamos a la raíz y los tres últimos bits
de la sucesión determinan la letra a por segunda vez. Por lo tanto, el árbol "decodifica,.
1111011 1 como ata.
¿Por qut funcionó bien el segundo esquema de codificación si el primero era ambiguo?
En el primer esquema representamos rcomo 10 y n como 101. Si encontramos los simbo-

Figura 12.33
úpítulo 1 2 Árboles

los 10, ¿cómo.podemos determinar si los símbolos representan ro los primeros dos símbo-
los de 101, que representan n? El problema e s que la sucesión de res un prefijo de la
sucesión den. Esta ambigüedad no aparece en el segundo esquema de codificación, lo que
nos lleva a la siguiente definición.

)efinición 12.7 Un conjunto P de sucesiones binarias (que representa un conjunto de símbolos) es uo


cÓ<Íigo prefijo si ninguna de las sucesiones de Pes el prefijo de otra suce sión de P.

En consecuencia, las sucesiones binarias 111 , O, l 100, 1101, 10 forman un código pre-
fijo para las letras a. e, n, r, l, respectivamente. ¿Pero de dónde surge el árbol binario
completo de la figura 12.33? Para este problema necesitamos el siguiente concepto.

>efinición 12.8 Si T es un árbol binario completo de altura a, e ntonces Tes un árbol binario zotal si todas
las hojas de T están en el nivel a. /

Para el código prefijo P = {l 11, O, 1100, 1101, 10}, la sucesión binaria más larga tiene
longitud 4. Trazamos el árbol binario total etiquetado de altura 4, como aparece en la
figura 12.34. Los elementos de P se asignan a los vértices de e ste árbol de la forma si-
guiente. Por ejemplo, la sucesión 10 describe el camino de la rafa r a su hijo derecho e,,.
Después continuamos hasta el hijo i~uierdo de c0 donde el recuadro (marcado con un
asterisco) indica que la sucesión está completa. De regreso a la raíz, describimos las otras
cuatro sucesiones en fonna similar, lo que produce los cinco vértices que aparecen dentto
de los recuadros. A cada uno de estos vértices 1e quitarnos e1 subárbol que determina
(excepto a la raíz). El árbol podado resultante es el árbol binario completo de la figura
12.33, donde ningún "recuadro" es antecedente de orro ..recuadro" .

Figur a 12.34

Ahora veremos un método para determinar un árbol etiquetado que modele un código
prefijo, en el que se tenga en cuenta la frecuencia de aparición de cada símbolo en el texto
12 .4 Árboles ponderados y códigos prefijo 641

promedio; en otras palabras, un código prefijo donde usemos las sucesiones más conas
para los símbolos de aparición más frecuente. Si existen muchos símbolos, como las 26
letras del alfabeto. no resultará eficiente usar un método de prueba y error para la cons-
trucción de dicho árbol. Una elegante cosntrucción desarrollada por David A. Huffman
proporciona una técnica para la construcción de tales árboles.
El problema general de construcción de un árbol eficiente se pue.de describir como sigue.
Seap1,p2, ••• , p11 un conjunto de números positivos llamados pesos, donde p 1 .s l'l s
· • :S p 11• Si T = (V. E) es un árbol binario completo con n hojas, asignamos estos pesos
(uno a uno) a las n hojas. El resultado es un árbol binario compl.eto para lbs peses pi, Pi, . .. ,
p •. Elpuodelárl,ol, P(n, se define como I.: ,¡,,~p,), donde, para cada l :Si :S n, l(p,) es
el número de nivel de la hoja asignada al peso p;. El objetivo es asignar los pesos de modo
que P(T) sea lo más peque ño posible. Un árbol binario completo T' para estos pesos es un
árbol óptimo si P(T) s P(T) para cualquier otro árbol binario completo T con los mismos
pesos.
La figura 12.35 muestrá dos árboles binarios completos para los pesos 3, 5, 6 y 9. Para
el árbol T,. P(T,) ~¿:,¡,,~p.)~(3 + 9+ 5 + 6) • 2 ~ 46, puesto que cada hoja tiene número
I de nivel 2. En el caso de T2, P(T2) =3 • 3 + 5 • 3 +6 • 2 +9 · 1 =45, lo que veremos que es
óptimo.

Figur.i 12.35

1 2
Figura 12.36

La idea subyacente en la construcción de Huffman es que a fin de obtener un árbol


óptimo Tpara los n pesosp1,P2,PJ, ... ,p., consideramos un árbol óptimo T' para los n-
1 pesosp1 + p 2 ,p3, ••• ,p._. (No podemos suponer quep1 +p2 s p3.) En panícular. el árbol
r se transforma en Tal reemplazar la hoja u con pcsop1 + "2 por un árbol con raíz en ude
altura 1, con un hijo izquierdo de pesop1 y un hijo derecho de peso p 2• Para ilustrar esto, si
el árbol T2 de la figura 12.35 es óptimo para los cuatro pesos 1 + 2, 5, 6, 9, entonces el
árbol de la figura 12.36 será óptimo para los cinco pesos 1, 2, 5, 6, 9.
Necesitamos el siguiente lema para establecer nuestras afirmaciones.

lfMA 12.2 Si Tes un árbol óptimo para los n pesos p 1 S pz S • • • s p•• entonces existe un árbol
óptimo T en el que las hojas de pesos p 1 y Pz son hermanos en el nivel maximal (en r ).
Demo.stración: Sea u un vértice interno de Ten que el núme ro de nivel de u es maximal
para todos los vénices internos. Seanp,, y p., los pesos asignados a los hijos.x, y del vénice
u, con p]{ .s Pr Por la elección del vénice u , f.(pJ = l{p.,) ~ l(p 1), l.(p2). Consideremos el
642 Capítulo 12 Árboles

'
caso de p 1 < p2• (Si p 1 =P:n entonces podemos intercambiarp~ y p.,, y podemos considerar el
caso p1 < p.,. Aplicamos la siguiente demos1ración a este caso y vemos que p, y p 2 puedea
intercambiarse.)
Si !(p.)> !(p1), sea !(p,) > !(p 1) + j, para algúnj Ez·.
Enton<esp,'(p,) + pJ.(pJ: p1'.(p,)
+ p,['(p ,) + ,1 : p,i(p,) + p,j + p,i(p,) > p¡i(p¡) +. p¡j + p).(p¡): p,i(p,) + p,i(p,). Así,
P<n : p,l(p ,) + p,l(p.) + L.,, ,p;l(p,) > p ,l(p,) + p,i(p¡) + L..,.,p,{(.p;). En consecuencia,
a1 intercambiar las posiciones de los pesos p1 y p", obtenemos un árbol de menor peso.
Pero esto contradice la elección de T como árbol óptimo. Por Jo tanto, r.(_pz) = ~ 1) = tJ..p,).
De forma similar, se puede mostrar que l(p,) = i(p,), por lo que l(p,) : !(p,) : !(p,) :
r.{pz). Al intercambiar las posiciones del par p 1, p;r y el par Pl, p.,, obtenemos un úbol.
óptimo T , donde pi, [>l_ son hermanos.

Con este lema vemos que los pesos menores aparecen en los nive les superiores (lo que
produce números de nivel más alto) en un árbol óptimo.

TEOREMA 12.8 Sea T un árbol óptimo para los pesos Pi + "2• p¡, ... , p,,,, donde p 1 :S {'2 S: Pl S: • • · :S p..
En la hoja con peso p 1 + P2 colocamos un árbol binario (completo) de peso I y asignamos
los pesos P1t P2 a los hijos (hojas) de esta hoja anterior. El nuevo árbol binario 7 1 constnri•
do de esta forma es óptimo para los pesos Pi. P2, p,• ... , p•.
Demostración: Sea 72 un árbol óptimo para los pesos p., p,_, ... , p. , donde las hojas corres-
pondie ntes a los pesos Pi. />2 son hermanos. Eliminamos las hojas de pesos pi, p 1 y asigna-
mos el peso p 1 + Pi a su padre (ahora una hoja). Este árbol binario completo se denota coa
T,y P[T,):P(T, )+p, +p,. Además,P(T,) :P(D+p1 +p,. PucstoqucTcs óptimo, P(n s
P(T3). Si P(T) < P(T3) , entonces P{7 1) < P(72 ) , lo que contradice la elección de 72 como
óptimo. Por lo tanto, P(.7) = P(_T3) y, en consecuencia, P( T1) <P{T1 ). Así, 7 1 es óptimo para
los pesos pi, P2, .. . , p,..

Observación. La demostración anterior partió de un árbol óptimo T2 cuya existencia se


basa en el hecho de que sólo existe un m1mero finito de formas de asignar n pesos a un
árbol binario completo de n hojas. En consecuencia. con un n6mero finito de asignacio-
nes, existe al menos una asignación para l a cual P(T) es minimal. Pero el número finito
puede ser muy grande. Esta demostración estable.ce la existencia de un árbol óptimo para
un conjunto de pesos y desarrolla una forma para construir tal árbol. Para construir este
árbol (de Huffman) consideramos el siguiente algoritmo.
Dados los n(i? 2) pesos pi, p,¡, ...• p., procedemos como sigue:

Paso 1: Asignamos los pesos dados, uno a cada conjunto S de n vértices aislados.
[Cada vértice es la raíz de un árbol binario completo (de altura O) con un peso asignado
a El.]
Paso 2: Si IS 1> 1, hacemos lo siguiente:
a) Encontramos dos ~les T, T' en S con los dos pesos mínimos para la raíz p. p' ,
respectivamente.
b) Creamos el nuevo árbol (binario completo) T• con peso de la raír.p•.= p + p' y con
r
T, como subárboles izquierdo y derecho, respectivamente.
12.4 Árboles pond«ados códi os prefijo 643

e) Colocamos P enSy eliminamos Ty T . [Cuando JSI : 1, el árbol binario comple-


to e n S es un árbol de Huffman.)

Usaremos este algoritmo en el siguiente ejemplo.

Construimos un código prefijo óptimo para los slmbolos a, o, q, u, J\ z que aparecen (en
una muestra dada) con las frecuencias 20, 28, 4, 17, 12, 7, respectivamente.
La figura 12.37 muestra la construcción que sigue el procedimiento de Huffman. En la
parte (b), se combinan los pesos 4 y 7, de modo que podamos considerar la construcción
para los pesos 11, 12, 17, 20, 28. En cada paso [en las panes (<Hf) de la figura 12.37)
cream<>$ un árbol con subárboles con raíz en los dos pesos menores. Estos dos pesos me-
nores penenccen a v~rtices que antes o estaban aislados (un árbol solamente con una raíz)
o bien eran la raíz de un árbol anteriormente obtenido en la construcción. De este último
resultado, determinamos un código prefijo como
a: 11 o: 01 q: 0000 u: 10 y: 001 z: 0001.
Podemos obtener diferentes códigos prefijos a partir de la fonna en que se seleccionan los
árboles T, T y se asignan como subárboles izquierdo y derecho en los pasos 2(a) y 2(b) de
nuestro algoritmo, y a partir de las asignaciones de Oy I a las ramas (aristas) de nuestto

.
árbol (de Hulfman) fi nal.

.
12 17
. .
20 28

t:
51
(o)

11

/\ . . . .
(b)
7 12 17 20 28

7
A
17 20

J-..
(e)
7
. . .
17 20 28
(e)

88

37

J-..
4
(d)
7
A.
17 20 28
(f)
17 20

Figur.i 12.37
644 capítulo 12 Árboles

EJERCICIOS 12.4 1. Para el código prefijo de la figura 12.33. decodifique las sucesiones (a) 1001111101; (b)
10111100110001101; (e) ll0lll lllOOl0.
2 . Un código para {a, b. e, d. e} es a: 00 b: 01 e: 101 d: .dO e: yd, donde x. y. z E {O, I).
Determine .r. y y z de modo que el código sea un código prefijo.
3. Construya un código prefijo óptimo para los símbolos a, b. c. . . . , i, j que aparece (en una
muestra dada) con las frecuencias respectivas 78, 16, 30, 35. 125, 31. 20, 50, 80, 3.
4 . ¿Cuántas bojas tiene un árbol binario total s i su altura es (a) 3? (b) 7? (e} 12? (d) a?
S. Sea T = ( V. E ) un árbol m-ario completo de altura a. Éste es un árbol m-ario 10UU si todas sus
hojas están en el nivel a. Si Tes un árbol m-ario total de a1tura 7 y 279.936 hojas, ¿cuántos
vértices internos tiene T?
6. Sea L,, 1 s i s 4, cuatro listas de números, cada una de e llas dada en orden ase.endente. B
número de elementos de estas listas es 75, 40, 110 y 50. respectivame.nte.
a ) ¿Cuántas comparaciones se necesitan para intcrcalar estas cuauo listas, intercalando L¡ y
4. intercalando Li y Le y desputs intercalando las dos listas resultantes?
b) ¿Cuinw c.omparaciones necesitamos si primero intercalamos L, y L:. luego intercalamos
el resultado con L, y por -Oltimo intercalamos este resultado con L.?
e) Para minimizar el númao total de comparaciones en esta inserción de las cuatro listas, ¿q~
orden debe seguir la inserción1
d ) Extienda el resultado de la parte (e) a n listas ordenada! Li, Lz, . .. , L.,.
7. Con los pesos 2, 3, S, 10, 10, muestre que la alrura de un irbol de Huffman para un conjunto
dado de pesos no es única. ¿Cómo modificarla el algoritmo para obtener siempre un úbol de
Huffman de aJrura minimal para los pesos dados1

12.5
Componentes biconexas y puntos
de articulación

Sea G = (V.E) el grafo no dirigido conexo sin lazos que se muestra en la figura 12.38(a),
donde cada vértice representa un centro de comunicación. Una arista {x, y} muestra la
existencia de una linea de comunicación e ntre los centros x y y .

(a)

Fi ura 1 2.38
12.S Componentes biconexas y puntos de artkulación 645

AJ dividir tos vlniccs en e y/, en la forma indicada. obtenemos la colección de subgrafos


de la parte(a) de la figura. Estos vértices son ejemplos de lo siguiente.

Dlfinidón 12.9 Un vénice u en un grafo no dirigido sin lazos G = (V. E ) es un punto de art.iculacidn si
l<(G- u )> l<(G); •• decir, •I subgrafo G - u tien• más component<S que el grafo dado G.
Un grafo no dirigido conexo sin lazos y sin puntos de articulación es biconao.
Una componOlle biconua de un grafo es un subgrafo biconexo maximal (es decir, que
no está contenido propiamente en un sub¡rafo biconexo más grande).

El grafo d< la figura 12.38(b) tiene dos puntos de articulación e y/ y sus cuatro compo-
nentes biconexas se muestran en la pane (b) de la figura.
En ténninos de centros y lineas de comunicación, los puntos de articulación del grafo
indican los puntos donde el sistema es más vulnerable. Sin puntos de articulación, es más
probable que este sistema supere las interrupciones en un centro de comunicación, inde.
pendientemente de que tstas sean causadas por un fallo en un dispositivo t6cnico o por
fuerzas eXlemas.
El problema de encontrar los puntos de articulación en un grafo conexo ofrece una
aplicación para el árbol recubridor en profundidad. Et objetivo en este caso es desan-ollar
un algoritmo para determinar los puntos de aniculación de un grafo no dirigido conexo sin
lazos. Si no existen estos puntos. entonces el grafo es biconexo. Si existieran estos vérti-
ces. las componentes bicooexas resultantes podrían usarse para proporcionar información
acerca de las propiedades como la planaridad y el número cromático del grafo dado.
Necesitamos los siguientes antecedentes para desarrollar este algoritmo.

LEMA 12.3 = =
Sea G (V, E ) un grafo conexo no dirigido y T (V, E ') un árbol rccubridor en profundi-
dad deG. Si {a. b} E E pero {a, b} ft. E', entonces a es un ascendiente o un descendiente
d< b en el árbol T.
o.mostrad6n: Mediante el úbol rccubridor en profundidad T, obtenemos una lista en or-
den previo para los vénice.s de V. Para cualqWer u E v. sea ip(u) el índice en profundidad
del vértice u ; es decir, la posic ión de u en la lista en orden previo. Supongamos que ip(a)
< ip(b). En consecuencia, a se encuentra antes que ben el recorrido en orden previo de T,
por lo que a no puede ser un descendiente de b. Si, además, el vértice a no es un ascendien-
te de b, entonces b no está en el subárbol T. de T con raíz en a. Pero cuando retrocedemos
(por T.) hacia a, vemos que como fa, b) E E, en la búsqueda en profundidad podríamos
haber ido de a a b y usado la arista ( a, b) d< T.

Si G = (V, E) es un grafo conexo no dirigido sin lazos, sea T= (V.E') un árbol rccubridor
en profundidad para G, como se muestra en la figura 12.39. Por el lema 12.3, la arista
punteada {a, b},que no fonna panede T. indica una arista que pOdría estaren G. Tal arista
'i46 Capitulo 12 Árboles

...

Figura 12.39

se conoce como arista dt retroceso de T y a es antecedente de b. (En este caso, ip(a) = 3,


mientras que ip{b) = 6.) Una arista punteada, como {b, d} en la figura, no puede estar enG,
también debido al lema 12.3. Así, todas las aristas de G son aristas de To son aristas de
retroceso de T.
Para G y T definidas como antes, sea x E V ta1 que x no sea Ja raíz de T y sea T,. e el
subárbol quecons1adelaarista {x, e} (e hijo dex)junto con el árbol Te con raíz en c. Si oo
existe una arista de retroceso desde un descendiente dex en Tu a un ancestro dex (y xtiene
al menos un ancestro, la raíz de n. entonces la separación del vértice x produce la separa-
ción de Tu de G y x es un punto de articulación. Si no hay otros puntos de articulación de
G en Tu, e ntonces agregar a Tu las demis aristas de G determinadas por los vértices deT""
(el subgrafo de G inducido por los vértices en Tu) produce una componente biconexa de
G. Una rafa no tiene antece sores, por lo que la raíz es un punto de articulación si tiene IDti
de un hijo.
El árbol recubridor en profundidad proporciona un orden previo para los vértices de G.
Sea ip(x) el índice en profundidad de x en ese orden previo. Si y es un descendiente de x,
entonces ip(x) < ip(y). Si y es un ascendiente de x, ip(x) > ip(y). Definimos prof(x} =
mfn{ip{y) ly es adyacente en G ax o a un desc~ndiente de x}. Si z es el padrede x(en n,
entonces hay dos casos:

1) prof(x) .= ip(z): En este caso. T_.. el subárbol con raíz en x, no contiene v&tices
adyacentes a un antecedente de z por medio de una arista de retroceso de T. Por lo
tanto, z es un punto de articulación de G. Si Tz no contiene puntos de articulación.
entonces T.. junto con la arista {z. x } genera una componente biconexa de G (es
decir, el subgrafo de G inducido por el vértice z y los vértices de T.. es una compo-
nente bicone,i;a de G). Ahora eliminamos T., y la arista {z. x } de T y aplicamos esta
idea al subárbol restante de T.
2) prof(x) < ip(z): En este caso e,i;iste un descendiente de z que se une (mediante una
arista de retroceso) a un ascendiente de z.
12.5 Componentes biconexas y puntos de articulación 647

1: ..._,. elál>ol leCIDidlll"eu profundidad Tele G, para los Vfflices onle-


~lasfilas(o-)dcA(G). Seaoy1,y,. . . . , y.los vénices de Gen
plffio dado por T. EnllJaca ip(y,) = j, para todo 1 ,aj ,a n.
21: e.,.-.-,. ycoalÍDDIIDOS d e - = hacia Y•-h y._,, ...• Y•· y.,,
· WJ:i¡ puí(J¡). para 3 :S j :S n, eu la siguiente forma:
b)=aáfil'Wlusadyacmlloay,enG) Z0~
.•. • .._. . ._ 11ijos de-,,._,.. prof(y1) = mln(prol'(y,). proí(c,),
.. . , ..,,.c.)} (No baJ pablema eu este caso, ya que los Vfflices se
----•elClldalilnasoal..-!R"io-Eaconsecuencia, si c es un híjodcp,
_..,,.c)sedctmniaa-quep,oi(p).)
3: Sea w1 el padle de y, Si puí(J,) = ip(w¡), CllkJIICCS w, es un punto de anicula,
•G<J de 1),• - qae w¡ sea la nfzde Ty qae 11(, no -sa hijos en T distintos
JWemm.•........,caso. el .-bol - ilifz euy1janrocoa la arista ( w;,y1 ) es
_ . . . , . _ bicoaexadc G.

Aplicamos este algoritmo al grafo G ~ (V, E) dado por la siguiente matriz. de adyacencia
A(G). Los vértices están ordenados como X i, x:i. x 3•. •• , x 1o-

x, x, x, x. x, X. x, x, x, X ,o

x, o 1 1 1 1 1 o o 1 o y,
x, 1 o o o o o o o o o y,
x, 1 o o o 1 o o o o o y,
x. 1 o o o o 1 1 1 o o y,
x, 1 o 1 o o o o o o o Y•
x, 1 o o 1 o o o o o o Y•
x, o o o 1 o o o 1 o o y,
x, o o o 1 o o 1 o 1 1 y,
x, 1 o o o o o o o o y,
X,o o o o o o o o o o Y oo
Y1 y, y, Ys Y• Y• y, y, y, y,,

Podemos recorrer los I Ovértices de Gen la lista en orden previo dada en la figura 12.40(.a).
La parte (b) de esta figura proporciona un rcordenamiento de los índices del árbol recubridor
en profundidad mostrado, ip()j) ~ j, para todo t :s;¡ :s; 10. Ésus son lasyj, l :S j s 10, que
aparecen en los extremos de las filas y columnas en la matriz de adyacencia A(G). S i
partimos del vértice Yio. obtenemos de A(G) que prof'(y 10) = 8, puesto que Ya es adyacente
ay1o, pero para 1 :s; i S 7,y; noes adyacente ay10. Cony10 como hoja, se sigue que prof(y 1o) =
8. Como prof(y10) ~ ip(y8) y Ya es el padre de y1o, Y• es un punto de aniculación de G; Ja
arista {y1 , y 10 } es la componente bicone xa. En la figura 12.4l(a), para cada y,, 3 ::S j ::S 10,
648 Capítulo 12 Árboles

.,
.,
..
(a)

Figura 12.40

el par ordenado que queda junto a y1 es (pror (y¡), prof(y¡)). Hemos calculado esto median-
te e1algoritmo. mientras re trocedíamos de y 10a y9 a y1 , etcétera. a y,. Por ejemplo, al caJcu-
lar y1 , e n A(G) vemos que prof(y1) .. 5, puesto que y, es adyacente a y1 y para cada l :Si
s 4, y; no es adyacente a y,. Puesto que Yt es un hijo de y, y prof(y9 ) = 1. se sigue que
= = ==
prof(y,) l. Para y, y y,, prof(y,) prof(y,) 1 ip(y,), donde y, es el padre de y, yy,. Pcx
lo tanto. y 1 es un punto de articulación y los siguientes subárboles dan lugar a otras dos
componentes biconexas: (a) el subárbol con raíz en Yljunto con la arista {y1, y1 } ; y (b)el
subárbol con raíz en y,, con la arista {y,, y 10 } eliminada y la arista {yi, y,} agregada. Por
último, como y2 es una hoja (y un subárbol sin aristas con raíz en y 2), la arista {yi, y2 }
forma otra componente biconexa adicional.
La figura 12.41 (b) muestra los puntos de articuJación y 1 y Ya para G. Estos puntos deter•
minan las cuatro componentes biconexas de G que se muestran en la figura.

Y,

,,,,
,t
Y, ,,
,,
Y,

\'
Y,
y,(5,1)
\~~-
(a) (b)
',
Y,
\'"
Figura 12.41
12.5 Componentes biconexas puntos de articulación 649

100S 12. 5 1. Encuentre los puntos de articulación y componentes bicone:us del grafo que se muestra en la
figura 12.42.

g h Figura 12.42

2. SeaG• (V, E) un grafo no dirigido conexo sin lazos y sea: E Y. Demuestre que tes un punto
de aniculación deG si y sólo si e xisten z. y E Vdistintos tales qucx #- ::. y f'-:: y tales que cada
camino que une x y y conticoc el v6rtiee .t.
3. SeaT= (V.E)unátbolcoo IVl = nae2.
a) ¿Cuiles son los mlmcros múimo y m(o,jmo posibles de puntos de articulación de T? Des·
criba los Arboles en cada caso.
b) ¿OJ'ntas componentes bicoocxas tiene Ten cada uno de los casos de la parte (a)?
4. a) Sea T = ( V, E ) un Arbol. Siu E V,demucstrequeuesun puntodcarticulacióndcTsi y sólo
si ¡rad(u) > l.
b) Sea G z (V, E) un c;rafo no dirigido conexo sin lazos con IEI i:?. l. Demuestre que G tiene
al menos dos v&tices que no son puntos de articulación.
s. Si B1, B,. . . . , 8 1 son las componentes biconexas de un grafo no dirigido conexo $in lazos C.
¿cómo se rdaci~ xCG) con X(B¡). I s is 4?
6. Cada una de W siguialtes matrices de adyacencia represuua un P o no diri¡ido conexo sin
lazos G. En cada caso:
a) Detemúnc un '1h01 recubridor en profundjdad para G. con raíz en x 1•
b) Use el resultado de la parte (a) y aplique el algoritmo desarrollado en esta sección para
encontrar los puncos de articulación y las componentes biconexas de G.

z, z, z, z, z, z, z, z,
1 1 1 1 o o o
z, 1 o o 1 1 o o o
z, 1 o o o o 1 1 o

T
.... 1 1 o o 1 o o o
z, 1 1 o 1 o o o o
z, o o 1 o o o 1 o
z, o o 1 o o 1 o 1
z, o o o o o o 1 o
z, z, z, z. z, .... z, z,
1 1 1 1 o o
z, 1 o 1 1 o o o
z, 1 o
1 1 o 1 o

T
:r. 1 1 o
1 o o o
z, o o o
1 o o 1
.... o o 1 o o o o
z, o o o o
z, o o o o
1 o o
1 o 1 !J
650 Caphulo 12 Árboles

7. En el segundo paso del algoritmo para los puntos de articulación, ¿por qué no fue necesario
calcular prof(y 1) y prof(yJ'?
8. Sea G = ( V. E) un grafo no dirigido conexo sin lazos, con u E V.
a) Demuestre que C::U =G-o.
b) Si u es un punto de articulación de G. demuestre que u no puede ser un punto de articula-
ción de C,.
9. Si G =(V.E) es un grafo no dirigido libre de lazos, Ges critico nsptclo al color si x(G - u)<
X.(G) para todo u E V. (Ya analizamos antes tales grafos,cnel ejercicio 17 de la sección 11.6.)
Demuestre que un grafo critico respecto al color no tiene puntos de articulación.

12.6
Resumen y repaso histórico

La estructura que ahora llamamos árbol apareció en 1847 en la obra de Gustav Kirchhoff
(1824-1877) acerca de las redes eléctricas. El concepto también apareció en Geometrie
die lage, de Karl von Staudt (1798--1867). En 1857, los árboles fueron redescubiertos por
Anhur Cay ley ( 1821-1895), ·quien no conocía taJcs desarrolJos anteriores y fue eJ primero
en llamar "'árbol" a esta estructura; Cayley los utilizó en las aplicaciones relacionadas con
los isómeros químicos. También investigó la enumeración de cienas clases de árboles. En
su primer trabajo sobre árboles, Cayley enumeró los árboles con raíz no etiquetados; dc.s·
pués enumeró los árboles ordenados no etiquetados. Dos de los contemporáneos de Cayley
que también estudiaron los árboles fueron Car) Borchardt ( 1817-J 880) y Marie Ennemood
Jordan (1838-1922).
La fórmula rt' · 2 para el número de árboles etiquetados den vértices (ejercicio 19 del
final de la sección 12. J) fue descubierta en J860 por Carl Borchardt. Cay ley dio posterior•
mente un desarrollo independiente de la fórmula, en 1889. Desde entonce s se ha obtenido
de varias formas, las cuales se resumen en el libro de J. W. Moon [9].

Arthur Cayley (1821-1895)


12.6 Resumen y repaso histórico 651

El artículo de G. Polya [10] es pionero en la enumeración de los árboles y otras estruc-


turas combinatorias. La teorfa de enumeración de Polya, que veremos en e l capítulo 16.
fue de sarrollada en ese trabajo. Para más detalles acerca de la enumeración de árboles, el
lector debería consultar el capítulo 15 de F. Harary [4].
El computador digital de alta velocidad ha dado un gran impulso al descubrimiento de
nuevas aplicaciones de los árboles. La primera aplicación de estas estructuras fue en el
manejo de las fórmulas algebraicas. Esto data de 1951, en la obra de Gracc Hopper. Desde
entonces, las aplicaciones computacionales de los árboles han sido ampliamente analiza-
das. Al principio, los resultados particulares sólo aparecían en la documentación de
algoritmos espe.cíficos. El primer resumen general de las aplicaciones de los árboles fue
presentado en 1% 1, por Kenneth Jverson, como parte de un artículo más amplio acerca de
las estructuras de datos. Las ideas de orden previo y posterior pueden rastrearse hasta los
principios de la dtcada de 1960, como lo muestra el trabajo de Zdzislaw Pawlak, Lyle
Johnson y Kenneth Jverson. Fue entonces cuando Kenneth Iverson definió e l nombre y la
1
notación Í .x del techo de un número real ;c. El lector puede encontrar material adicional
acerca de estos órdenes y los procedimientos de implementación en un computador, en el
capítulo 3 del texto de A.Abo, J. Hopcrofty J. Ullman [ I J. En e l artlculodeJ. E.Atkins, J.
S. Dierckman y K. O' Bryant [2] se usa el concepto de orden previo para desarrollar una
ruta óptima para la eliminación de la nieve.
Si G = (V, E) es un grafo no dirigido sin lazos, entonces la búsqueda en profundidad y
la bl1squeda en anchura (dadas en la sección 12.2) proporcionan formas para detenninar si
el grafo dado es conexo. Los algoritmos desarrollados para estos procedimientos de bús-
queda también son importantes para desarrollar otros algoritmos. Por ejemplo, ta búsque-
da en profundidad surge en el algoritmo para determinar los puntos de aniculación y las
componentes biconc x.as de un grafo no dirigido conexo sin lazos. Si I Vj = n y IEl =t, se
puede mostrar que ambas búsquedas son () (máx {n, t} ). Para la mayoría de los grafos, e > n.
por lo que los algoritmos generalmente se etiquetan como C(e). Pero corno e s li)= (1/2)
(n)(n - 1), el orden de cada uno de estos algoritmos a veces aparece como O(n1 ). Por lo
tanto, decimos que los algoritmos tienen orde n lineal e n e y orden cuadrático en n. Estas
ideas se desarrollan con gran detalle en las páginas 172-191 del texto de S. Baase [3], que
incluye tambi6n un análisis de la función de complejidad en tiempo del algoritmo (de la
sección 12.5) que determina los puntos de aniculación (y las componentes biconexas). El
capítulo 6 del texto de A Abo, J. Hopcroft y J. Ullman [1] también trata la búsqueda en
profundidad, mientras que el capítulo 7 analiza la búsqueda en anchura y el algoritmo para
los puntos de aniculación.
El lector puede leer más acerca de las propiedades y aplicaciones computacionales de
los árboles en lasección 3 del capírulo 2 del libro de D. Knuth [6]. Las técnicas de ordenación
y su uso de los árboles se estudian con detalle en el capítuJo 11 de A. Abo, J. Hopcroft y J.
Ullman [l]. El tratamiento del tema en el volumen 3 de D. Knuth [7] está basado en una
amplia investigación.
La técnica de la sección 12.4 para el diseño de los códigos prefijo se basa en un método
desarrollado por D. Huffman [5].
Por último, el capítulo 7 de C. L. Uu [8] trata los árboles, ciclos. conjuntos de corte y el
espacio vectorial asociado con estas ideas. Al lector con ciertas bases de álgebra lineal o
abstracta este material Je parecerá de gran interés.
652 Capitulo 12 Árboles

BIBLIOGRAFÍA

l . Aho,AlfredV., John E. Hopc:mft y Jeffrey D. Vllman,Data Structurts andAlgorithms, Reading,


Mass., Addison•Wesley, 1983.
2. Atkins, Joel E.. Jeffrey S. D:ierela:na.n y Kevin O'Bryant, ..A Real Snow Job... The UMAP
Jounu,L o<oilo, núm. 3 (1990), págs. 231- 239.
3. Baase, Sara.Compweralgorithms: lntroducrion toDuignandAnalysis, red., Rcading, Mass.,
Addison•Wesley. 1988.
4. Harary, Frank. Graph 17ttory, Reading, Mass., Addison•Wcsley, 1969.
S. Huffman. David A., ..A Methoo for the Construction of Minimum Redundancy Codes".
Proc,dmgs ofth< /RE 40, 1952. págs. 1098-1101.
6. Knuth, Donald E.• TheArt ofú»nputer Programming, vol. 1, 2• w., Reading, Mass.,Addison.
Weslcy, 1973.
7 . Knuth, Donald E., TheArtofComputer Programm.ing, vol. 3, 2•ec1., Reading. Mass.• Addison-
Wcsley, 1973.
8. Liu, C. L., lntroduc.rion lD Combinatoria[ Malkmalics, Nueva York, McGraw-Hill, 1968.
9. Moon, John Wesley, Cowuing Labelled Trees. Canadian Mathematical Congress, Montreal,
Canadá, 1970.
1O. Poiya, George. "KombinatorischeAnzahlbestimmungen für Gruppen. Graphen und Chemisehe
Verbindungen", Acta Mathemalica 68, 1937, págs. 145-234.

a) Encuentre la sucesión característica del árbol


EJERCICIOS
binario completo de la figura 12.16.
COMPLEMENTARIOS b) Determine los árboles binarios completos para las
sucesiones características
i) 1011001010100 y
1. a) Sea G =(V.E) un grafo no dirigido sin lazos, con
ii) IOl lll0000101011000.
1VI = n. Demuestre que G es un árbol si y sólo si
e) ¿Cuáles son los dos últimos símbolos de la suce-
P(G,,)=1(•- 1¡--•.
sión cacacterfstica para todos los árboles binarios
b) Demuestre que X(G) = 2 para cualquier árbol con
completos? ¿Por quE?
dos o más v&tices.
e) Si G = (V, E') es un grafo no dirigido conexo con
4. Para k E Z\ $ea n = 2' y considere la lista L: ª" ai,
j VI = n, demuestre que para cualquier entero A~
O, P(G. ).) s 1(). - 1¡--•.
a,, . . • ª•· Para ordenar L en forma ascendente. primero
compare los elementos a; y a;. (Al2}, para cada 1 :S i s n/2.
2. E.n una empresa con 125 ejecutivos se establece un sis- Para los 2• - 1 pares ordenados resultantes. ordene primero
tema de comunícación telefónica. El sistema es inaugurado por inserción los pares ordenados i-ésimo e (i + (n/4))-ésimo,
por el presidente. quien llama a sus cuatro vicepresidentes. para cada 1 s i s n/4. Realice ahora una ordenación por
Cada vicepresidente llama a otros cuatro ejecutivos. algunos inserción de las cuaternas ordenadas i-ésima e (i + (n/8))-
de los cuales llaman a su vez a otros. ~ r a . (Un ejecutivo tsima. para cada I S i S n/8. Continúe el proceso hasta que
que haga una llamada hace cuatro llamadas en realidad.) los elementos de L esttn en orden ascendente.
a) ¿Cuántas llamadas se realizan para alcanzar un to- a) Aplique este procedimiento de ordenación a la lista
tal de 125 ejecutivos?
b) ¡,Cuántos ejecutivos, además del presidente, son L, 11, 3, 4, 6, -5, 7, 35. -2,
necesarios para hacer las llamadas? l. 23. 9, 15, 18, 2, - 10, 5.
3. Sea T un árbol binario completo con los vtrtices de T
ordenados mediante el recorrido en orden previo. Este reco- b) Si n = i-t, ¿cuántas comparaciones necesita como
rrido asigna la etiqueta I a todos los vtrticcs internos de T y máximo este procedimiento?
la etiqueta O a cada hoja. La sucesión de ceros y unos que 5. a) Si Tes un árbol binario total de altura 5. ¿cuántas
surge del recorrido en orden previo es la sucesién caracre- hojas tiene T ? ¿Cuántos vértices internos? ¿Cuán-
riszica del árbol. tas aristas (ramas)?
Ejercicios complementarios 653

b) Responda la parte (a) para un árbol binario total de Para cualquier grafo no dirigido conexo sin lazos G =
altura a. donde a E Z•. (V. E), d cuadrado de G, que se deno<a ron G', es el grafo
con el conjunto de vértices V (el mismo que G) y con el con•
6. SeaTunárbol binariototaldcalturia. Siu1, '\h. . .. , u.
junto de aristas definido como sigue: para a,. b E V distintos.
• las bojas de T, evah1e l:;:, 2-•, donde d.• es eJ námcro de (a, b) es unaaristadeG' si<(a, b) ,;; 2(enG). Fn las parte, (a)
IIÍ'l'd de la hoja U¡, para cada
1 :S i '!;'. m. y (b) de la figura 12.43, tenemos un grafo G y su cuadrado.
7. SeaG =(V.E) un grafo no dirigido sin lazos. Si grad{u) a) Encuentre el cuadrado del grafo de la parte (c) de
2 2 para todo u E V, demuestre que G contiene un ciclo. la figura.
a. Sea T= (V. E) un árbol con raíz r . Defina la relación~ b) Encuentre <Jl si Ges el grafo K1•1•
a Vcomo .x~y.para x. y E V.si x =yo si xestáen eJ e) Si Ges el grafo K1• • , paran .!:: 4. ¿cuántas aristas
camino de r a y. Demuestre que~ es un orden parcial. añadimos a G para construir G2?
d ) Para cualquier grafo no dirigido conexo sin lazosG,
9. Sea T = ( V. EJ un árbol con V= { u 1• v,: •.. . • u. } para demuestre que <? no tiene puntos de articulación.
■~ 2. Demuestre que el número de vtrtices colgantes en T
12. a) Sea T = (V. E)unárbol 6-ario completo de altura 8.
a iguala
Si Tes equilibrado, pero no total, determine los
2+ ¿ (grad(u,)-2) valores múimo y mínimo posibles de I VI.
p ..:..Jl:J
b) Responda la parte (a) si T =(V.E) es un árbol m•
10. ScaG = (V. E) un grafo no dirigido sin lazos. Defina la ario completo de altura a.
tdación91. en E como sigue: Si l!!!'i , t 2 E E, entonces 1!!!'1~ 1!!!'1 13. Los árbcl~s d< Fibonacci con raíz T,.. n ~ l, se defi-
sit1=e1 o si e1 y e2 son aristas de un ciclo C en G. nen en forma recursiva como sigue:
a) Verifique que~ es u na relación de e quivalencia 1) T1 es el árbol con raíz que consta solamente de la
en E. raíz;
b) Describa la partición de E inducida por 9l . 2) T2 es igual a T1 ~ también es un árbol con raíz con
11. Si G = (V.E) es un grafo no dirigido conexo sin lazos un único vértice; y,
ya.b e V, entonces definimos la distancia de a a b (o de 3) Paran ~ 3, T. es eJ árbol binario con raíz que tiene
b a a). que denotamos con d(a, b), como la longitud del a T._1 como subárbol izquie rdo y T. _1 como
camino más corto (en G) que une a con b. (Éste es el nú- subárbol derecho.
mttO de aristas del camino más corto que une a con b y es Los primeros scis árboles de Fibonacci con raíz aparecen en
cero si a= b.) la figura 12.44.

(a) G (b) G' (e)

Figura 12.43

T,
.
.
, A
~
p fa
r, T, T, r,
Figura 12.44
654 Capítulo 12 Árboles

(a) {b) {el r, r, r,

{d)
,~,

b
(el
,4J>, b

Figura 12.45

a ) Paran ~ l, sea t.~el número de hojas de T•. Encuen- 15. SeaG=(V. E }un grafo"cscalera"nodirigido,comoel
tre y resuelva una relación de recurrenc:ia para ¡:,_. que se muestra en al figura 12.46. Para n ?:: O, sea a,. d
b) Sea i,. el número de vértices internos del árbol T., número de árboles recubridores de G, y b. el número de
donde n ~ 1. Encuentre y resuelva una relación de estos árboles rccubridores que contienen la arisca {x1• y1 }.
recurrencia para i,.. a) Explique por qué a,.=a,._1 + b,..
e} De1ermine una fórmula para u,.. el número total de b) Encuentre una relación que exprese t,,. en ténninos
vértices de T,., donde n ~ 1. dea,. _1 yb,.- 1.
d) ¿Cuál es la altura del árbol T,., donde n .<?: 1? e) Use los resultados de las partes (a) y (b) paraestab)e.
14. a) Elgrafodelapartc(a)delafigural2.45tiene exac- cer y resolver una relación de rccurrcncia para a..
lamente un árbol recubridor: el propio grafo_ El 16. Sea T = ( V. E ) un árbol tal que IVI = u y IEI = <. El
grafo de la figura l2.4S(b) tiene cuatro árboles árbol Tes elegante si C$ posible asignar las etiquetas {t, 2.,
recubridores no idénticos, aunque isomorfos. En 3. . ... u} a los vértices de T de modo que el etiquetado
la parte (e) de la figura vemos tres de los árboles inducido de las ariscas (donde cada arista {i. j} recibe la
recubridores no idénticos para el grafo d e la parte etiqueta Ji -JI. para i, j E { L 2, 3.... , u~. if. ¡) hace que
(d). Observe que T2 y T, son isomorfos, pero 7 1 no las e aristas queden etiquetadas como 1, 2, 3,. . . e.
es isomorfo a T2 (o a 7 3 ) . ¿Cuántos árboles a) Demuestre que cualquier camino simple conn v,&-
recubridores no idénticos tiene el grafo de la figu- ticcs, n ~ 2, es elegante.
ra 12.45(d)? b) Paran E Z♦• n ~ 2. muestre que K •.• es elegante.
b} En la figura l 2.45(c) generaliz.amos los grafos de
las partes (a). {b) y (d) de la figura. de la forma
siguiente: paran E Z♦• G,. =(V.E) es el grafo no
dirigido sin lazos donde

tt!JJ
V= {a,b, 1, 2,3, ... ,n}, Y
E• {{a, 11,{l , b},{a,2},{2,b},{a,3},
{3,b}, ... ,{a, .i - l},{n-1,b},{a,n},{n,b}}.
(Nota: Para cada n E Z\ el grafo G,. es K2. •.)
Si t,. es el número de árboles recubridores no idén1i- Y, Y2 Y1 y,._, Yn
os de G,., encuentre y resuelva una relación de recurrencia Figura 12.46
iara 10 ,
Ejercicio-s complementarios 655

Figura 12.47

e) Si T = (V, E) es un árbol con4 :s I VI s. 6. muestre v6tices no colgantes tales que para todo v E V, v está en el
que Tes elegante. (Una conjetura seftala que todo camino Po u es adyacente a un vértice del camino P. Este
árbol es elegante.) camino se conoce como la columna de la oruga.
a) ¿CUántos conjuntos maximalcs independientes de
17. Para un grafo no dirigido G =(V.E), un subconjunto/ váticcs tienen las orugas de las partes (i) y ( ii) de
de V es vukpmdienu si ningdn par de v6ticcs de / son Ja figura 12.47?
adyacentes. Si, además, f U {x} no es independiente para b) Paran E Z•, sea a. el número de conjuntos maxi-
m. x E V - I, entonces decimos que 1 es un conjunto maJes independientes de vértices en una oruga T
atUimaJ uuúpendunt~ (de vtnices). cuya columna' contiene n v&tices. F.ncuentrc y re-
Los dos grafos de la figura 12.47 son ejemplos de un suelva una relación de recurrcncia para a •. (Quizás
tipocspc:cial de árboles llamados orugas. En general, un árbol el lector desee vo lver a analizar la parte (a) del ejer-
T= (V. E) es unacruga si ex.iste un camino (maximal) P de cicio complementario 14 del capítulo 11.)
13
Optimización
emparejamiento

E n el último capítulo de esta parte del texto usaremos las estructuras de .trboles y grafos y
presentaremos técnicas que surgen en el área de las matemáticas llamada investigación
opermiva. Esw ~cnicas opti,m izan algunos resultados analir.ando grafos y multigrafos
que tienen un número real positivo (Secs. 13. 1 y 13.2) o un entero no ne¡ativo (en la Sec.
13.3) llamado peso. asociado a cada arista del grafo o multigrafo. Estos números se reta•
clonan con información como la distancia entre los v~ces que son los extremos de una
arista. o tal vez como la cantidad de maierial que puede embarcarse de un vénice a ouo a
lo largo de una arista que representa una carretera o ruta Krea. Con los grafos como ma«:o
de referencia, desarrollaremos los métodos de optimización en forma algorítmica para
facilitar su implementación en un computador. Entre los problemas que analizaremos es-
tán los de deteffllinar:

1) La distancia más cona entre un v~nice dado Uo y cada uno de los demás véniccs de
un grafo dirigido conexo sin lazos.
2) Un .trbol recubridor del grafo o multigrafo dado donde la suma de los pesos de las
aristas del árbol sea minimal.
3) La cantidad máxima de material que puede transportarse de un punto inicial O•
fueote) a uo punto fmaJ (el sumidero). donde el peso de una arista indica su capaci•
dad para controlar el material transportado.

13.1
Algoritmo del camino más corto
de Dijkst ra

Comenzaremos con un grafo dirigido conexo sin lazosG; (V, E ). A cada arista e = (a. b)
de este grafo le asignamos un nl'.imero real positivo llamado el pü<> de t, que denotamos
con p(,) o p(a. b). Si .x; y E V pero (x. y) ~ E, definimos p(.x; y) = .,_
Paracualquicre=(a. b) EA, p{e) podríarepresentar (l) la longitud de una carretera de
a a b, (2) el tiempo que tarda recorrer esta carretera de a a b, o (3) el costo del viaje de a a
b por esta carretera.
Cuando demos un grafoG= (V, E)con las asignaciones de peso aquí descritas, diremos
que el grafo es un grafo ponderado.

657
capitulo 13 Optimízadóoyemparejamiento

En la figura 13.1,cl grafoponderadoG=(V, E)representa las rutas de viaje entre al


pares de ciudades. El peso de cada arista (x. y) indica el tiempo aproximado de un
directo de la ciudad x a la ciudad y.

Figura 13.1

En este grafo dirigido hay situaciones en las que p(.r, y)#- p(y. x) para ciertas aris~ (X.
y) y (y, x) de G. Por ejemplo, p(_c, /) = 6 #7 = p(f. e). Es probable que esto se deba a la
dirección del viento. Cuando el avión vuela de e a f. es probable que sea ayudado por 111
viento de cola el cual, a su vez., hará que reduzca la velocidad al viajar en dirección opues-
ta (de/ac).
Vemos que c. g E Vpcro(c, g), (g. e) ,tE, por lo q ue p(_g, e) = p(_c, g)= oo. Estotambiéll
es cieno para otros pares de vértices. Por otro lado, para cienos pares de vértices. comoa,
f. tenemos que p(a.f) = oo, mientras que J)(/, a) ; 11, un ntimero finito.

Nuestro objetivo en esta sección tiene dos partes. Dado un grafo ponderado G = (V.E).
para cada e ;;;; (a. b) E E interpretaremos p(e) como la longitud de una ruta dice.eta (por
automóvil, avión o barco)dea ab. Para cualesquiera dos vértices a. b E Vescribimosd(a,b)
como la distancia (más cona) de a ab. Si no existe talcamino(enG) dea ah, entonccsde-
finimosd(a, b) =00. Para cualquiera E V. d(a. a) ; O. En consecuencia. obtenemos la fun-
ción distanciad: V x V ➔ R• U (O, oo }.
Fijemos ahora u0 E V. Entonces. para cualquier u E V, de terminaremos

1) d(u,, u); y
2) un camino simple dirigido de Uo a u si d(Uo, u) es finito.

Para lograr estos objetivos, presentaremos una versión de l algoritmo desarrollado por
Edsger Wybe Dijkstra (1930-) en 1959. Este procedimiento es un ejemplo de algoritmo
voraz. ya que lo que obtenemos como e l mejor resuJtadolocalmente(para véruccs "cerca-
nos" a U:i) se convierte en el mejor resultado globalmente (para todos los vértices del
grafo).
Antes de establecer el algoritmo, nos gustaría analizar algunas propiedades de la fun-
ción distancia d. Estas propiedades nos ayudarán a comprender por qué funciona el
algoritmo.
13.1 Algoritmo del cambio más corto de Oijkstra 659

Conu0 E Vfijo(comoantes),seaS C Vcon'l)o ES y S = V-S. Definimos entonces la


distancia de Uo a S como

d(Uo,S)= ~{d(Uo,u)J.

Cuandod(llo, S) < oo, entonces d(llo, S) os la longitud mínima do un camino simple diri-
gido de \)ga un vértice en S . Así, existirá al menos un vértice 'U.,.,. 1 en S con d(Uo, 'S) =
d(Uot u,. .. 1) y u.,,. 1 estará en el extremo final de un camino simple mínimo. Así mismo, si
P: ('Oo, ll1) , (ll1, aj, .. . , (ll• . i, ll. ), (ll_ "• ••)es un camino simple dirigido más corto (enG)
de Uo a v 111 • i. entonces tenemos que

1) llo,\l,. . . .,ll.ESy
2) P": (llo, ll 1), (lli, v,), . .. , (ll,_,. ll,), es un camino simple dirigido más corto (en G)
de 'Uo a Ut, para cada 1 :S k s m.

(Pediremos la demostración de estos dos resultados en el primer ejercicio del final de esta
sección.)
De estas observaciones se sigue que
d(Uo,:S) = min {d(u,,; u) + p(u, w)},
donde el mínimo se evalúa sobre todos los u E s. w E S. Si aparece un mínimo en u = x
y w = y, entonces
d(Uo,Y) = d(Uo,x) + p(x, y)
es la distancia (más corta) de Uo a y.
La fórmula para d(llo, S) es la piedra angular do esto algoritmo. Partimos del conjunto
So = {Uo} y después determinamos

Esto nosdad(llo.
-~
d(Uo, S.)= mín{d(u 0 , u) + p(u, w)}.
.e:\,
So)= ~{p(llo, w)}, puosS0 = {Uol- Si Uo E So y d(llo, So)= p(llo, u,)
entonces ampliamos S0 a S1 = So U {U 1} y detenninamos

d(u, ,S'i) = ~ (d(Uo, u) + p(u, w)}.


..,,
Esto nos lleva a un vértice U:z en ~, con d(t>o, S1) = d(Uo. aj. Al continuar este proceso, si ya
hemosdeterminadoS;= {uo, U1, l.½, ...• U;} yu, .. , E 3\ con d(t>o, u, .. ,) =d(Uo, l ,), enton~
ces aumont.amosS, as,. , = s, U {11;.,J. Nos detenemos cuando!_, = 0 (donde n = 1vi)
o cuando d(llo, S,) = oo para algún Os is n - 2.
Durante este proceso colocamos diversas etiquetas sobre cada vérticeu E V. El conjun-
to final de e tiquetas que aparecen en los vértices tendrá la forma (L(u), u). donde I..(u) =
d(t>o, u) es la distancia de t>o a u y u es el vértice (si existe) que precede a uen un camino
simple más corto de Uo a u. Es decir, (u, u) es la íiltima arista en un camino dirigido de \)g
a u y este camino detenninad(Uo, u ). Primero etiquetamos u 0 con (O, - )y Jos demás véni-
ces u con la etiqueta ( oo, -). Conforme apliquemos el algoritmo, la etiqueta de cada u Uo +
carnbiará(a veces, más de una vez), de(~.-) hasta la etiqueta final (l.{ll), u) =d(llo, ll), u),
a menos que d(Uo. u)= oo.
Capitulo 13 Optimización y emparejamíeoto

Con estos antecedentes, podemos enunciar de manera formal el algoritmo.


=
Sea G (V, E) un grafo ponderado, tal que I vi =n. Para determinar la distancia más
cona de un vértice fijo u 0 a los demás vértices de G, así como un<:amino simple dirigido
más corto para cada uno de estos vértices, aplicamos el siguiente a1goritmo.

19gadlmo del cama, mis airtocle ~


-...J:ea-isdCXllllado<i=OyS.={,W. .
aa(•.-).
Si•= 1 , - V = {v.} yd,..,W,..Clli._..,_
Si•> 1, ef •rmd¡eso2.
. . .J:PlnC811a1) E 3'1,RR,.,tlAlll!fi!IIIDI-·
-cdqaolaliaol(L(v).y),dlJae

Procederemos a aplicar el algoritmo en el siguiente ejemplo.

Aplicaremos al algorinno de Dijkstra al grafo ponderado G = (V, E) de la figura 13.1, para


determinar la distancia más corta del vértice e(= Uo) a cada uno de los otros cínco vértices deG.
lni<ialización: (i =O). Sea S.= (e}. Etiquetamos e con (O, -) y los demás véniccs
deGcon (®,-J.
Primera iteración: (lo ;;;; {a, b,f. g, h} ). En este caso, i =Oen el paso 2 y encontramos.
por ejemplo, que

L(a) = mín {L(a), L (c) + p(c,a)}


= mín {oo, O+ ce;} = oc,
13. 1 Algoritmo del cambio más corto de Oijkstra 661

mientras que
L(j) = mfn {L(f), L(c) + p.(c,f)}
= mfn {oo,O + 6} =6.
Cálculos similares muesttan que Ll_b) =Ií.g) = 00 y 4h) = 11. Así,
etiquetamos el vértice/con (6,c) y el vénicehcon (11,c). Los demás
vénices de :SO siguen etiquetados eon (oo, - ). [Véase la fig. 13.2(a).J
Enel pa.so3vemosque/esel véniceu 1 en el "So máscercanoa~por
loqueasignamos aS1 elconjunto~U U1 = {c,J) e incrementamos el
contador i a l. Como i = 1 < 5( = 6 - 1), regresamos al paso 2. .

Figura 13.2

Segunda iteración: (:S, ={a, b, e, h}). Ahora, i =1 en el paso 2; para cada u E 5 1,


hacemos
L(u) = mío {L(u),L(u) + p(u,u)}.
.es,
de donde obtenemos
L(a) = mío {L(a), L(c) + p(c,a), L(f) + p(f,a)}
= mín {oo,O + "',6 + 11} = 17,
por lo que etiquetamos el vértice a como (17,/). De manera simi-
lar, vemos que
L(b) = mío {oo, O + oo,6 + oo} = oo,
L(g)=mfn{oo,0+oo,6+9} = 15, y,
L(h) =mín {!1,0 + 11,6 + 4} = 10.
{Con estos resultados obtenemos el etiquetado de la figura 13.2(b).]
En el paso 3 vemos que el vénice u2 es h, pues h E S1 y L(h) es un
mínimo. Entonces asignamos aS2 el conjunt0S1 U {h) = {c,J. h},
incrementamos el contador a 2 y como 2 < 5, el aJgoritmo nos
lleva de nuevo a1 paso 2.
662 Capítulo 13 Optimizaóón y emparejamiento

Tercera iteración: (~,={a, b, g)). Con i= 2 en el paso 2, ahora calculamos:

L(a) = ~ {L(a), L(u) + p(u, a)}


= mín{l7,0+"',6+ 11, 10+ 11)= 17
(así. la etiqueta de a no cambia)~
L(b) = mfn {"',O+ "', 6 + "', 10 + "'} ="'
Oa etiqueta de b sigue siendo oc>); y
L(g) = mío {15,0 + "',6 + 9, 10 + 4} = 14< 15,
por lo que laetiquetadeg cambia a(14,h), puesto que 14=L(h)+
p(h, g). Enrre los vértices de ~,.ges el más cercano a t>o, puesu,
que L(g) es un mínimo. En el paso 3, el vértice "3 se define como
g y S3 =S, U {g} ={e,/, h. g) ). Incrementamos el contador i a 3 <
5, y regresamos al paso 2.
Cuarta ilttac:ión: (~, = {a, b }). Con í = 3, detemúnamos lo siguiente en el paso 2:
L(a) = 17; L(b) ="'·(Así, las etiquetas no cambian durante esta
iteración.) Hacemos u, = a y S, = S, U {a) = { e, f. h, g, a)), en el
paso 3. Entonces incrementamos i a 4( < 5) y regresamos al paso 2
Quinta iteración: (~, = {b }). En este caso, i = 4 en el paso 2, y vemos que L(b) =
L(a) + p(a, b) = 17 + 5 = 22. La etiqueta de b cambia por (22, a).
Entonces, u, =ben elpaso 3,S,es {e,/, h, g, a. b) eiseincrementa
vi
a 5. Pero ahoraqueí= 5 = 1 -1, el proceso termina Obtenemos
el grafo etiquetado que se muestra en la figura 13.3.

Figura 13.3

De las etiquetas de la figura 13.3 obtenemos las siguientes distancias más cortas de e a
los otros cinco vértices de G:

=
1) d(c,f) L(f) 6 = =
2) d(c, h) L(h) 10 =
3) d(c,g) = L(g) = 14 4) d(c,a) = L(a) = 17
5) d(c,b) = L(b) = 22
13.1 Algoritmo det cambio más corto de Oi'kstra 663

Por ejemplo, para determinar UD camino dirigido más corto de e a b, partimos del vér-
tice b, que tiene la etiqueta (22,a). Por lo tanto, a es el predecesor deben este camino m'5
corto. La etiqueta en a es (17, /), por lo que /precede a a en el camino. Por último, la
etiqueta en/es (6,c), por lo que regrosamos al vértice e y el camino dirigido más cono de
e a b determinado por el algoritmo está dado por las aristas (e,/),(/. a) y (a, b).

Ahora que hemos demostrado una aplicación de este algoritmo, nos interesa el orden de
su función de complejidad en tiempo en el peor caso, /(n), donde • = 1 vi en el grafo
ponderado G • (V, E ). Mediremos la complejidad en ttrm.inos del nllmero de sumas y
comparaciones que se hacen en los pasos 2 y 3 durante la ejecución del algoritmo.
Despu~s del proceso de inicializacíón del paso 1, hay a lo más n - 1 iteraciones, pues
cada iteración determina el si¡uiente vértice más cercano a t>o y n - 1 = 1V - ( t>o} 1.
Si O s: i S: n - 2, entoDCCS en el paso 2 de esa iteración [la (i + 1}-ésima], vemos que
ocurre lo siguiente para cada u E J,.
1) Cuando O :s i :s n - 2, realizamos un máximo de n - 1 sumas para calcular

-
L(u) • mio {L(u), L(u) + p(u,u)}

lo que re¡msenta una suma por cada • E S,


2) Comparamos el valor actual de L(u) con cada uno de los números l(u) + p(• u)
(algunos de tos cuales podrían ser in.finitos), uno para cada u ES,., donde IS¡ ( :S
n - l , para determinar el valor actualizado de L(u). Esto requiere al menos n - 1
comparaciones. Por lo tanto, antes de Jle¡:ar al paso 3 hemos realizado cuando mu-
cho 2(• - 1) pasos para coda u E :!:,, con UD total de 2(n- l)' para todo u E :!:,.
Continuamos con el paso 3 y ahora debemos seleccionar el mínimo entre un
máximo den-1 númerosU.,u). donde u E 'S,. Esto requiere al menosn-2 compa-
raciones adicionales, en el peor de los casos.
En consecuencia. nin¡:una iteración necesita más de 2(n -1)2 + (n-2) pasos en
total. Es posible tener twta n - 1 iteraciones. por lo que

/(n) :s (n - 1)[2(n - 1)1 + (n -2)) E O(n').


Terminaremos esta sección con algunas observaciones que se pueden usar para mejorar
la complejidad en tiempo en el peor caso para este algoritmo. Primero debemos obsavar
que para O :s i S n -2, la (i + 1)-ésima iteración del algoritmo generó el (i + 1)-ésimo
v~nice más cercano a Uo,, el v~rticc U¡. 1. En nuestrQ: ejemplo, obtuvimos u 1 • /, '½ = h. u,
=g,u,=ayu,=b.
En segundo lugar, observemos las duplicaciones hechas al calcular L(u). Podemos ver
esto tipidamente en la segunda y tenxra iteraciones del ejemplo 13.2. Quisiéramos acor-
tar estos cálculos innecesarios, as{ que intentaremos usar un ~todo un poco distinto para
nuestro problema del camino mú cono. De nuevo comenzamos con un grafo ponderado
G s (V, E) con I V 1 = n y t>o E V. Denotamos con u. el i-~simo v~rtice más cercano a t>o,
donde O :si :s • - 1, S,= (\\>.u,, . .. , u,} y:!:,• V- S,.AI principio asignamos a cada u E V
el nllmero I.,.u) como sigue:

Lo(1>o) • O yaque d(1>o,1Jo)=O y


Lo(u) = 00, para u4u,,.
664 Capitulo 13 ()ptirnzaoón y emparejamiento

Entonces, para i ;,; Oy u E 3',, definimos


L,.,(u)= mío {L,(u), L,(u,) + P(U¡,u)},
dondeu1 es un vértice para el que~{u;)es minimal: un vértice que es i-ésimoccrcanoau.,
Vemos que

L,. 1 (u)== {d(t>o,v;) + P(v¡,u)}.

Veamos qué ocurre en cada una de las (a lo más) n- 1 iteraciones cuando utilizamos la
definición de L.. ,(u) que usa el vé rtice u,.
Para cualquier u E ~,. sólo necesitamos una suma [L,(u,) + p(u,, U)] y una comparaciól
[entre L,{u) y L,{u,) + p(u. u)] para calcular L.. ,(u). Como existen un máximo de n-1
vértices en~,. esto requirc a lo más 2(n-1) pasos para obt:ener L¡+ 1(u) para todbu El,.
Encontrar el mínimo de {L.. 1(u) lu E 3',} requiere cuando mucho n - 2 comparaciones,
por lo que en cada iteración obtenemos u, .. 1 (un vértice u E J, donde L¡ .. i(U) es WI
mínimo) en a lo sumo 2(n - 1) + (n -2) = 3n - 4 pasós. Desarrollamos como máximo n-
1 iteraciones, por lo que para esca versión del algoritmo de Dijkstra. la función de comple-
jidad en tiempo para el peor caso es O(n'). ·
A fin de determinar un camino más corto de Uo para cada u E V, u -¡i. Uo, vemos que
siempre que ~ .. 1(u) < L.{u), para cualquier O :S is n-2. necesitamos llevar un registro
del vértice y ES, para el que L.,. ,(u) = d(Uo, y)+ p(y, u).

EJERCICIOS 13.1 1. Su G = (V.E) un grafo ponderado la! que para cada arista,= (a. b) de E, p(a. b) C$ igual a b
distancia dea aba Jo Jargo de la arista e. Si (a, b) ~E.entonces p(a.b) • C10.
Fije 1>o = V y sea S e V, con 1'o E S. EntoDOC$, para S = V - S. definamos d(v., !) =
mfn,.¡(d(v., u)). Si u• ., E 'S y d(v., 'S¡ = d(v., u••,), entonces/': (V.. u,), (u,. u,), .. : ,
(u._i. u,.), (u,., u,.. 1)csun camino simple dirigido más corto (enG) dc~a u,.. 1 • Dcmuestreqoe::
a) \}o.U¡.Ui, ...• U,._1,u.ES
b) P': (\lo. u 1). (U1, uJ, ... , (u1_ i. uJ. es el camino simple dirigido más coito (en G) de Uo•
1.>., para cualquier 1 :s k :s m.

2. a) ApliquedalgoritmodeD:ijkstraal grafo ponderadoG =(V,E)dela figura 13.4ydetcnni-


ne la distancia lIW corta del v&tioe a a cada wio de los otros seis vbtices de G. & este caso,
p(t) = p(-<, y)= p(y, x) para cualquier arista t =(-<,y} de E.
b) Determine el camino más cono del v.!rtic:ea a los vátic:cs c,/ei.

Figura 13.4
13.2 Árbofes recubridores minimales: Los algoritmos de Kruskal y Prim 665

3. a) Aplique el algoritmo de Dijkstra al grafo de la figura 13.1 ydctc:n:ninc la distancia mis cor-
ta del vtmce a a los demú vtttices del grafo.
b) Determine el camino simple 0W cono del vi!rtice a a cada WlO de los v6ti.Cdt g y h.
4. Use las ideas desarrolladas al final de la sección para confirmar los resultados obtenidos en (a)
el ejemplo 13.2; y (b) la parte (a) del ejercicio 2.
5. Prudleorcfutelosiguiente, paraungrafopooderadoGa(V. E). donde V= {u,. u,."• ... •u,}.
y e 1 E E.con p(e1) < p(e)para todo e EE,el: e,. Si aplicamos el algoritmo de Oijkstra aG, y
calculamos la distancia más corta d(Uo, uJ para cada v&tice U;, 1 ::S i ::S n. entonces existe un
vht.ice u1, para alglin l S j S n. tal que la arista e1 se use en el camino más cono de Uo a u,.

13.2
les recubridores minimales:
algoñtmos de Kruskal y Prim

Hay que construir una red de cómputo con un acoplamiento vago para UD sistema de siete
computadores. El grafo G de la figura 13.S es un modelo de la situación. Los computado-
res se representan mediante los vértices del grafo; las aristas representan líneas de transmi-
sión que se tienen en cuenta. para enlaz.arciertos pares de computadores.Asociamos a cada
arista e de G un número real positivo p(,). el peso de<. En este ejemplo. el peso de una aris-
ta indica el costo previsto para la construcción de esa línea de transmisión particular. El
objetivo es enlazar todos 1os computadores minimizando el costo total de la construcción.
Para hacer esto, necesitamos UD árbol recubridor T, tal que la suma de los pesos de las
aristas en T sea minimal. La construcción de dicho árbol recubridor óptimo puede realizar-
se por medio de los algoritmos desarrollados por Joseph Kroskal ( l 928- ) y Robert Prim
(1921- ).
Como el algoritmo de Dijkstra. estos algoritmos son voraces; al usarlos, en cada paso
del proceso se hace una elección óptima (en este caso minimal) de los datos disponibles
restantes. De nuevo, si lo que parece ser la mejor opción localmente (por ejemplo, para UD
vértice e y los vértices cercanos a e) es también la mejor opción globalm~nu (para todos
los vénices del grafo), entonces el algoritmo voraz nos llevará a una solución óptima.

Figura 13.5
666 Capít¡¡lo 13 Clptimizaó6ny emparejamiento

Primero analizaremos el algoritmo de Krusk.al, el cual se genera del modo siguiente.


Sea G = (V, E) un grafo no dirigido conexo sin lazos tal que I
vi = n y cada arista •
tiene asignado un número real positivo p(t) Para encontrar un árbol recubridor óptimo
(minimal) para G, aplicamos el siguiente algoritmo.

Antes de establecer la vaJidez del algoritmo. consideremos el siguiente ejemplo.

Aplicaremos el algoritmo de Kruskal al grafo de la figura 13.5.

Inicialización: (i = 1) Puesto que sólo existe una arista (a saber, {e,g)) de peso
mínimo 1, comenzamos con T= { {e. g} }. (Al principio, Tes un ár-
bol con una arista, y despuésdecadaiteraci&i crece hasta ser un árl>ol ·
más grande o un bosque. Después de la última iteración, el subgrafo
Tes un árbol recubridor óptimo para el grafo dado G.)
Primera iteración: Entre las aristas restantes de G, tres de ellas tienen el siguiente
peso menor, 2. Seleccionarnos {d./), la cual satisface las condicio-
nes del paso 2. Ahora Tes el bosque {{,. g), {d./} } e incrementa-
mos i a 2. Como i = 2 < 6, regresamos al paso 2.
Segunda iteración: Dos de las aristas restantes tienen peso 2. Seleccionamos {d. e}.
Ahora, T es el árbol ( {,, g), (d,J), {d, ,}} e i toma el valor 3.
Como 3 < 6, el algoritmo nos envía de regreso al paso 2.
Tercera iteración: Entre las aristas de G que no están en T, la arista {/. g} tiene un
peso minimal 2. Sin embargo, si añadimos esta arista a T, el resul-
tado contíene un ciclo, lo que destruye la estructura de árbol que
buscamos. En consecuencia, analizamos las aristas {e, e}, {e, g} y
{d. g). La arista (d. g) produce un ciclo, pero {c. •} o {e, g)
satisfacen las condicione s del paso 2 . Seleccionamos {c. e }. T cre-
ce a { (, . g}, (d,f), {d, <},{c. e}} e i aumenta a 4. Regresamos al
paso 2 y vemos que las iteraciones cuarta y quinta nos proporcio-
nan los síguientes resultados.
13.2 Árboles recubridores minimale$: Le» algoritmos de Kruskal y Prim 667

Cuarta iteración: T; ( ( <, g}. (d,f), {d.<}, (e,<}, (b, el}; i aumenta a S.

Quinta iteración: T; {(<, g}, (d./}. {d.<}, {e,<}, {b, <}, {a, b}}; el contador i
toma el valor 6 = (número de vútices de G)- 1. Por lo tanto, Tes
un árbol óptimo para el grafo G y tiene peso 1 + 2 + 2 + 3 + 4 +
S; 17.
La figura 13.6 muestra este árbol recubridor de peso minimal.

Figura 13.6

El ejemplo 13.3 muestra la forma e n que el algoritmo de Kruskal realmente pft?d.uce un


árbol recubridor. Esto se sigue de las partes (a) y (d) del teorema 12.S, puesto que el
subgrafo resultante tiene n( ; 1V 1) vértices y n - 1 ariscas, además de ser conexo. En
general, si G; (V, E) es un grafo no dirigido conexo, ponderado, sin lazos y 'res el subgrafo
de G generador por el algoritmo de Kruskal, entonces T no tiene ciclos. Además, Tes un
subgrafo recubridor de G, ya que si u E Vy u no está e n T. entonces podemos añadir una
arista e de G a T de modo que e sea incidente con \l, y el subgrafo de G resultante no
tencfáa ciclos. Por último, T es conexo ya que, en caso contrario, tendáa al menos dos
componentes T1 y T2 y como Ges conexo, podríamos añadir a Tuna arista {x.y} de G, tal
que x esté en T1 y y esté en T2, sin que aparezca un ciclo en este subgrafo. En consecuencia.
el subgrafo T de Ges un subgrafo recubridor conexo de G sin ciclos (o lazos), de modo
que Tes un árbol recubridor de G.
El algoritmo es voraz; selecciona de las aristas restantes una arista de peso minimal que
no cree un ciclo. El siguiente resultado garantiza que el árbol recubridor obtenido es óptimo.

lEOREMA 13.1 SeaG; (V, E ) un grafo ponderado no dirigido, conexo y sin lazos. Cualquier árbol recubridor
de G obtenido mediante el algoritmo de Kruskal es óptimo.
Demostradón: Sea IV 1 ; n y sea T un árbol recubridor de G obtenido mediante el algoritmo
de Kruskal. Etiquetamos las aristas de T como e1 , e2 , •• • , e•• 1 de acuerdo con el orden en
que son generadas mediante el algoritmo. Para cualquier árbol óptimo T' de G, definimos
d(r) = k si k es el mínimo entero positivo tal que T y T contengan ambos a ei, e2, • • • ,
e,:.t, pero que el~ r.
SeaT1 un árbolóptimoparaelqued(Tt) =rsea maximal. Sir=n, entonces T= T1 y o~
tenemos el resultado. En caso contrario, r :s n - 1 y al añadir la arista e; (de 7) a 7 1
obtenemos el ciclo C, donde existe una arista e; de C que está en T1 pero no en T.
668 Capitulo 13 Optimización y emparejamiento

Partimos del árbol T1• Al añadir e, a T1 y eliminar e:. obtenemos un grafo conexo COI
vértices y n - 1 aristas. Este grafo es un árbol T1 . Los pesos de T1y T2 satisfacen la re
p(T,): p(T1) + p(e,)-p(<:}. •
Si seguimos la forma de selección de e 1, ~• • • • , e,_, en el algoritmo de Kruskal,
la arista e, de modo que ()(:e,.) sea minim.al y no obtengamos ciclo alguno al añadir e..
subgrafo H de G determinado por eh ~ . ... • e ,_ 1. Como e; no produce un ciclo al
añadido al subgrafo H, la minimalidad de p(e,) implica que p(e;) a: p(e,). Por lo
p(e,)-p(e:) s O, por lo que p(T,) s p(T1). Pero como T, es óptimo, debemos tener
p(T,) : p(T,), por lo que T, es óptimo.
El árbol 72 es óptimo y tiene las aristas ti, e2 , ••• , e,_ 1, e, en común con T, por lo
d{T2) ~ r + 1 > r = d(T1), lo que conttadice la elección de T1• En consecuencia, T1 = Ty
árbol Tproducido por el algorinno de Kruskal es óptimo.

Medimos la función de complejidad en tiempo para el peor caso f para el algoritmo


Kruska1 mediante las siguientes observaciones. Dado un grafo ponderado no diri ·
conexo y sin lazosG: (V,E), talque I v i :n y I El :m a: 2, podemos usarlaorde
por inserción de la sección 12.3 para enumerar (y volver a etiquetar, en caso necesario)i.
aristas de E como t1t e2 , •••• e,,.. donde p(e1) s p(e:) s · · · s p(e.). El número de compa--
rac:iones necesarias para hacer esto es O(m lo~m). Luego, una vez enumeradas las aristas
en ese orden (de pesos no decrecientes), realizamos el paso 2 del algoritmo un máximo de
m - 1 veces: una vez. por cada una de las aristas e2• e3• ••• , e,,..
Para cada arista e~ 2 :S is m. debemos determinar si e;provoca la formacióa de a
ciclo en-el árbol o bosque desarrollado basta ese momento (después de considerar las
aristas ea. e,z, .. . , e;_1), Esto no puede hacerse en una cantidad constante de tiempo [es~
0(1)] para cada arista. Sin embargo, todo el trabajo necesario para la detección de ciclas
puede hacerse como máximo en O(n log, n) pasos.t
Eo consecuencia, definiremos la función de complejidad en tiempo para el peor caso/,
para m ~ 2, como la suma de lo siguiente:

1) el nwnero total de comparaciones necesarias para ordenar las aristas de Gen ord,a
no decreciente, y
2) el número total de pasos rcaliz.ados en el paso 2 para detectar la formación de•
ciclo.

l.
A menos q;.e G sea un árbol, se sigue que I V 1 : n :s m : 1E puesto que Ges cooexo.
Como resultado, n log, • S m log, m y/ E O(m log, m).
También podemos dar una medida en términos de n, el número de vértices de G; n - t
s m, puesto que el grafo es conexo y m s (;): ( 1l2XnXn -1 ), el mlmerode aristas en X..
En consecuencia, m log, m s n' log, n': 2n' log, n y podemos expresar la complejidad en
tiempo del peor caso del algorinno de Kruskal como O(n' log, n).

t Para iús Wonnacióc accrca del adlisis del segmento relativo a la dctocción de ciclos, el leaor
deben revisar el capítulo 8 del texto de S. Baasc [2] y el capftulo 4 del texto de E. Horowiu y
S. Sahm !I>I.
13.2 Árboles recubridores minimates: los algoritmos de Kruskal y Prim 669

Robert Prim desarrolló una segunda técnica para la consttucción de un árbol óptimo.
En este algoritmo voraz. los vértices del grafo se dividen en dos conjuntos: procesados y
no procesados. Al principio, sólo hay un vénice en el conjunto P de los vértices procesa-
dos y los demás están en el conjuntoN de vérti<es por procesar. Cadaiteracióodel algoritmo
incrementa el conjunto P en un vértice, mientras que el tamaño del conjunto N decrece en
uno. El algoritmo se resume como sigue.
Sea G ~ (V, E) un grafo ponderado no dirigido, cone xo y sin lazos. Para obtener un
árbol óptimo para G, aplicamos el siguiente procedimiento.

Usamos este algoritmo para encontrar un árbol óptimo para el grafo de la figura 13.5.

El algoritmo de Prim genera un árbol óptimo de la forma siguiente.

Inicialización : i = 1; P= {a}; N= {b,c, d, e,f,g}; T = 0.


Primera iteración: T = {{a, b}}; P = {a, b}; N = {e, d, e,f, g}; i = 2.
Segun da iteración: T= {{a, b}, {b,e}}; P = {a, b, e}; N = {c,d, f,g}; i = 3.
Ter cera iteración: T = {{a, b}, {b, e}, {e,g}}; P = {a,b,e,g}; N = {c,d,f}; i = 4.
Cuarta iteración: T ={{a, b}, {b,e}, {t ,g}, {d, e}}; P ={a,b, e,g,d};
N ={c,f}; i = 5.
Quinta iteración: T ={{a, b}, {b,e}, {t,g}, {d, e}, {f,g}}; P ={a,b,e, g, d,f};
N = {c};i = 6.
Sexta iteración: T = {{a, b}, {b , e}, {e,g}, {d,e}, {f,g}, {c, g}};
P ={a, b, e,g,d,f,c}=V; N=0; i=7 = 1VI- Porlotanto, Tes
un árbol recubridor óptimo de peso 17 paraG. como se ve e n la
figura 13.7.

Observe que el árbol generador minimal obtenido aquí difiere del que se mue stra en la
figura 13.6. Así, este tipo de árbol recubridor no tiene que ser único.
;JO C.pótulo 13 Optimización y emparejamiento

figura 13.7

Sólo enunciaremos el siguiente teorema. el cual establece la validez del algorinno de


Prim. La demostración se deja al, lector.
_/
EOREMA 13.2 SeaG = (V, E) un grafo ponderado no dirigido, conexo y sin lazos. Cualquier árbol recubridor
de G obtenido mediante el algoritmo de Prim es óptimo.

Concluiremos esta sección con unas cuantas palabras y bibliografía acerca de la fun-
ción de complejidad en tiempo para el peor caso para e l algoritmo de Prim. Cuando apli-
camos el aJgoritmo a un grafo ponderado no dirigido, conexo y sin lazos G =(V, E ), donde
1V 1 = n y IE 1 = m, las implementaciones usuales requieren O(n') pasos. (Esto puede
revisarse en el capítulo 7 de A. Abo, J. E. Hopcroft y J. D. Ullman [ 1]; en el capítulo 4 de
S. Baase (2]; y en el capítulo 4 de E. Horowitz y S. Sahni (15].)Algunas implementaciones
m.is recientes de este algoritmo han mejorado la situación, de modo que ahora son necesa-
rios O(m log2 n) pasos. (Esto se analiza en los artículos de R. G·r aham y P. Hell [14]; de
D. B. Johnson [16]; y de A. Kershenbaum y R. Van Slyke [17].)

JERCICIOS 13.2 1. Aplique el algoritmo de l(,uskaJ y el algoritmo de Prim para detcnninar árboles recubridores
minimalc.s para el grafo de La figura 13.8.

Figura 13.8

2. La tabla 13. t proporciona información acerca de la distancia existente (en millas) emre pares
de ciudades en el estado norteamericano de Indiana. E.U.
Se construirá un sistema de carreteras para unir estas siete ciudades. Determine las carrete-
ras que deben construirse para mini miz.ar el costo de construcción. (Suponga que el costo de
construcción de una milla de carretera es el mismo entre cualquier par de ciudades.)
13.3 Redes de transporte: El teorema de flujo máximo y corte mínimo 671

/
Tabla 13.1

Evannille
Fort Wayne
c...,.
~
SoatbBeod
- 119
174
198
51
198
E_..

-
290
277
168
303
Fon
Wayae

-
-

132
121
79
Gary

-
-
-
153
58
-- -
-
-
-
140
-
-
-
-
-
Trrn: Haute 58 113 201 164 71 196

3. a) Responda el ejercicio 2. con la restricción adicional de que el sistema incluya una carretera
que enlace en forma directa Evansville con lndianápolis.
b) Si debe haber un enlace directo entre Fort Wayne y Gary además del que une Evansville
con lndianápolis, encuentre el número mínimo de millas de carretera que deben cons•
truirse.
4. SeaG = (Y,E)ungrafopondcradodirigido,conexoysin lazos. Paran E z~, sea {e1,e2, . ..• e.}
un conjunto de aristas (de E) que no incluyen ningún ciclo~ G. Modifique el algoritmo de
Kruskal para obtener un árbol recubridoc de G que sea m.inimal entre tod.os los ácboles
cccubridores de G que incluyen las aristas e1, e2, ...• e•.
S. a) Modifique el algoritmo de Kruskal para determinar un árbol óptimo de peso maximal.
b) Interprete la información del ejercicio 2 en términos del número de llamadas que pueden
realizarse entre pares de ciudades, por medio de la adopción de ciertas lineas nuevas de
transmisión telefónica. (Las ciudades que no estén enlazadas diccctamente deben comuni•
carse por medio de una o más ciudades intermedias.) ¿Cómo pueden comunicarse en fonna
minimal las siete ciudades y permitir que se realice un número máximo de llamadas?
6. Demuestre el teorema 13.2.
7. Sea G = (V, E) un grafo ponderado no dirigido. conexo y sin lazos tal que cada par de aristas
distintas ei, e1 E E. p(e1) 4:p(eJ. Demuestre que G tiene un llnico árbol cecubridor m.inimal.

13.3
de transporte: El teorema
flujo máximo y corte mínimo

Esta sección proporciona una aplicación de los grafos ponderados dirigídos al flujo de un
bien de una fuente a un destino dado. Ta1es bienes podrían ser litros de petróleo que fluyen
a travts de tubeóas o números de llamadas telefónicas transmitidas en un sistema de ro.
municación. Al modelar estas situaciones, interpretamos el peso de una arista en el grafo
dirigido como una capacidad que impone un límíte superior. por ejemplo. sobre la canti-
dad de petróleo que puede fluir por ciena parte de un sistema de tuberías. Estas ideas se
expresan formalmente en Ja siguiente definición.

13.1 Sea N = (V, E ) un grafo dirigido conexo sin lazos. Entonces N es una red. o ~ de trans•
pone, si se cumplen las siguientes condiciones:
72 capitulo 13 Optimización y emparejamiento

a) Existe un único vértice a E V tal que ge(a), el grado de entrada de a, es igual a O.


Este vértice es la fuente.
b) Existe un único vénicez E V, llamacj_osumidero, tal quegs(z),el grado de salida de
,. es igual a O. "-
e) El grafo N es ponderado, por lo que existe una función de E en el conjunto de
enteros no negativos que asigna a cada arista e = (u, w) E E una capacidad, que se
denota con c(e) :;::; c(u, w).

~· El grafo de la figura 13.9 es una red de transporte. El vértice a es la fuente. el sumidero es


el vénice z y las capacidades se muestran debajo de cada arista. Como c(a. b) + c(a, g) =
5 + 7 = 12, la cantidad.del bien que se transponadea az no puede ser mayor que 12. Como
c(d, z) + c(h, z) = 5 + 6 = 11, la cantidad queda restringida aún más, a no ser mayor de 11.
Para determinar la cantidad máxima que puede transponarse de a a z, debemos considerar
las capacidades de todas las aristas de la red.

·<1S1> g h Figura 13.9

Damos la siguiente definición para ayudamos a resolver este problema.

efinición 13.2 Si N: (V. E) es una red de transpone. una función/ de E a los enteros no negativos es un
flujodeNsi
a) f(e) S c(e ) para toda arista e E E, y
b) para cualquier u E V distinto de la fuente a o del sumidero t. L ....vf(w, u)= L ....,v
/(u, w). (Si no existe una arista (u, w).f(u, w) = O.)

La primera condic ión especifica que la cantidad de material transportado a lo largo de


una arista dada no puede exceder la capacidad de esa arista. La condición (b) pide que la
cantidad de material que fluye hacia un vértice u debe ser igual a la cantidad que fluye
desde ese vénice . Esto es cieno para tcxlos los vénices, excepto la fuente y el sumidero.

Para las redes de la figura 13.10, Ia etiquetax, y sobre cada arista ~ se determina de modo
que x. : c(e) y y es el valor asignado a un flujo posible f La etiqueta en cada arista r
satisface/(<) S c(e). En la parte (a) de la figura, el "flujo" hacía el vértice ges 5, pero el
..flujo" hacia fuera de l vértice es 2 + 2 = 4 . Por lo tanto, la función/no es un flujo en este
13.3 Redes de transporte: El teorema de flujo máximo y corte mínlmo 673

b 4, 1 d

)
I 6, 4

(a) g s. 2 h (b) 5, 3 h

f igura 13 .1 0

caso. La función/de la parte (b) satisface ambas condiciones, por lo que es un flujo de la
red dada.

Sea/un flujo para una red de transporte N = (V, E).


a) Una arista e de la red está saturada si/(e) =c(e). Cuando/(e) < c(e), la arista es no
saturada.
b) Si a es la fuente de N, entonces val(f) ;:; L ,.,.vf(a, u) es el valor del flujo.

Para la red de la figura 13.l O(b), {h,d} es la única arista saturada. Las demás aristas no son
saturadas. El valor de l flujo en esta red es

val(/)= L/(a,u) = f(a,b) + f(a,g) = 3+ 5 = 8 .


..v

¿Existirá otro flujo/1 tal que val({¡)> 8? La determinación de un flujo maximal (un flujo
que logra el máximo valor posible) es el objetivo del resto de la sección. Para lograrlo,
observamos que en la red de la figura 13.IO(b),

Lf(a, u)= 3 + 5 = 8 = 4 +4 = f(d,z) + f(h,z) = L/(u,z).


..ev ...ev

En consecuencia, el flujo total que sale de la fuente a es igual al flujo total que llega al
sumidero z.

La última observación del ejemplo 13.7 parece razonable , pero ¿ocurre en general? A
fin de demostrar el resultado para cualquier red, necesitamos el siguiente tipo particular de
conjunto de corte.

Definición 13.4 Si N = (V, E) es una red de transporte y Ces un conjunto de corte para el grafo no dirigido
asociado con N, entonces Ces un corte. o corte a - z. si la eliminací6n de las aristas de C
de la red produce Ja separación de a y z.
¡74 Capítvlo 13 Optimización yemparejamiento

Cada una de las curvas punteadas de la figura 13.11 indica un cone para la red dada. El
corte C, está formado por las aristas no dirigidas {a, g ), {b, d), {b, g) y {b, h). Este corte
divide los vértices de la red en los dos conjuntos P; 1a. b) y su compleme~o 7' ; {d, g,
h, z}. por lo que C1 se denota con (P, P). La capacidad de un corte, que se~enota con
c(P, 7'), se define como

c(P, l') ; L c(u, w),


...vee,
la suma de las capacidades de todas las aristas (u. w), donde u E P y w E P. En este
ejemplo,c(P, F) ;c(a, g) + c(b, c/) +c(b, h) ; 1 + 4 +6; 17. [Si consideramos las aristas
dirigidas (de P a 7') en el corte e, ; (P, F), a saber, (a. g), (b, el), (b, h), vemos que la
eliminación de estas aristas no produce un subgrafo con dos componentes. Sin embargo,
estas tres aristas son minimales en el sentido de que al quitarlas se elimina cualquier cami-
no dirigido posible de a a z.]
El corte C, induce la partición de vértices Q ; (a, b. g), l2 ; (d, h. z) y tiene capacidad
c(Q, l2); c(b, d) + c(b, h) + c(g, h); 4 + 6 + 5 ; IS.
Un tercer corte de interés es el que induce la partición de vérticesS; {a, b, d, g, h), 'S;
{z}. (¿Cuáles son las aristas en este corte?) Su capacidad es 11.

Figura 13.11

Por medio de la idea de capacidad de un corte, el siguiente resultado proporciona una


cota superior para el valor de un flujo en una red.

rEOREMA 13.3 Sea/un flujo en una redN; (V, E). Si C(P, 7') es cualquier corte en N, entonces val(!) no
puede exceder a e ; (P, 1').
Demostración: Sea el vértice a la fuente de N y el vértice t su sumidero. Como ge(a) = O, se
sigue que para cualquier w E V,f(w, a) =-O. En consecuencia,

val(/); L/(a,u)-L/(a,u)- Lf(w,a).


wev wev ...ev

Por la condición (b) en la definición de un flujo, para cualquierx E P, x,# a, L..vf(x, 1>)-
L..vf(w, x); Q_
13.3 Redes de transporte: El teorema de flujo máximo y corte mínimo 675

Si sumarnos los resultados de las ecuaciones anteriores obtenemos

...ev
...
val(/)= [ ¿,/(a,u)- 2./(w,a)]+ 2. [¿,f(x,u)- Lf(w,x)]
vev xe l' uE.V --EV

J = 2. f(x, u) - 2. f (w.x)
:,rf':P
vEV
xeP
,.,ev

= [ 2./(x,u) + L/(x ,u)] - [ 2.f(w,x) + 2./(w,x)].


xf!P xEP i E I' ieP
vEI' ..eP wE P weP

Como sumamos

. f /(x,u)
ve, ..·",
2. f (w,x)

sobre el mismo conjunto de todos los pares ordenados de P x P, estas sumas son iguales.
En consecuencia.

val(/) = 2. f (x , u) - ¿, f (w,x).
zfil' xEP
.eP ..,e;:

Para todos x, w E V, f (w,x) 2' O, por lo que

2. f (w,x) 2cO val(/) s 2. f(x , u) ,s 2. c(x. u) = c(P, P).


:!~ :,; 1/f.f
uEI" ~-E.,
l lli:.f

Del teorema 13.3 vemos que en una red N, el valor de cualqui~rflujo es menor o igual
que la capacidad de cualquier corte en esa red. Por lo tanto, el valor del flujo máximo no
puede exceder la capacidad mínima sobre todos los eones de una red. Para la red de la
figura 13.11, se puede mostrar que el corte formado por las aristas (d. z) y (h, z) tiene la
capacidad mínima 11. En consecuencia, el flujo máximo/de la red satisface val(/) s 11.
E1 valor del flujo m.Uimo es 11. La forma de construir tal flujo y la razón por la que su
valor es igual a la capacidad mínima entre todos los corte s serán e l tema del teorema
principal de esta sección.
Sin embargo, antes de analizar esta construcción, obse rvemos que en la demostración
del teorema 13.3, el valor de un flujo está dado por

val(/)=¿, f(x , u) - 2. f( w.x).


x€ P zlcP
vEP ~ p

donde (P, P) es cualquier corte de N. Por lo tanto, una vez que se construye un flujo en una
red. entonces para cua]quiercorte (P, P ) de la red, el valor del flujo e s igual a la suma de
los flujos en las aristas dirigidas de los vértices de P hacia los de P menos la suma de los
flujos en las aristas dirigidas de los vértices en Palos de P.
Esta observación conduce al siguiente resultado.
;75 capítulo13()ptirrizaóónyemparejamiento

:OROIARIO 13.1 Si/es un flujo en una red de transpone N = (V, E), eotonces el valor del flujo desde la
fuente a es igual al valor del flujo hacia el sumidero z.
Demo,t,..;;ón, Sean P = (a), 1' = V - (a) y S = V - (t), Q = (t). De la observa;u
anterior,

Lf(x,u)- Lf(w,x) = ~= L/(y,u) - Lf(w,y).


al(f
zef iEl. ,ea :,eQ
ve, ..e, vt!°Q weQ

Con P = (a) y g ,(a) = O, tenemos que I....-.••, /(w, x) = L /(w, a)= O. En forma
análoga. para Q = (t) y gs(t) = O, se sigue que I ,..,_•.af(w. y) = I,,. /(t. y)= O. El
0
consecuencia.

Lf(x, u)= L_f(a, u)= val(/)= L f(y,u) = L f(y,z),


zef, v'i;.I' :,EQ Jrl=Q
ve/' ueQ
lo que e stablece el corolario.

Pasemos ahora al resultado principal de la sección.

ºEOREMA 13.4 El teorema de/flujo máximo y cort, múiimo. Para una red de transponeN = (V, E). el flujo
máximo que se puede obtener en N es igual a la capacídad mínima sobre todos los cortes
de la red.
Demostración: Por el teorema 13.3, si (P, P) es un cone de capacidad mínima enN, entoD-
ces el valor de cualquier flujo en N satisface vaJ(/) s c(P, "'P). Para verificar la existencia
de un flujo/para el que val(J) • cf.P, 1'), utilizamos el siguiente algoritmo iterativo, llama-
do procedimiento de etiquetado.
13.3 Redes de transporte: El teorema de flujo máximo y corte mínimo

Puesto que un vértice y podría ser adyacente a, o de. más de un vértice etiquetado, los
resultados de este procedimiento podóan no ser únicos. Además. si .r está etiquetado, la
red podría incluir ambas aristas (x, y) y (y, x), lo que tal vez produciría dos etiquetas para y.
Pero e l procedimiento está diseñado para crear un flujo máximo. y podría haber mis de un
flujo de este tipo. No obstante. si un vértice puede ser etiquetado en más de una forma.
podemos hacer una elección arbitraria.
Al aplicar el procedimiento de etiquetado a los vértices de la red dada. repetimos los
pasos 4 y S mientras sea posible respecto del conjunto actual (aunque modificable) de
vénices etiquetados. En cada iteración debemos considerar dos casos:

Caso 1: Si el sumidero z se etiqueta como (X"', A(z)), entonces el flujo en la arista (.r, z)
puede aumentarse de/(x, z) af(x, z) + t.(z), según lo indique la etiqueta.
El vértice x puede etiquetarse como (u-, t.(x)) o (u-, t.(x)), donde t,(_x) ;,: t.(z). Para la
etiqueta (u•, 6(x)), podemos considerar el vértice u como la fuente para aumentar el flujo
en la arista (x, .z) en la cantidad 6(:z). En este caso, incrementamos de foíma análoga el
flujo presente en la arista (u,x), de/(u, x) a/(u, x) + t.(z)(no a/(u,x) + t,(_x)). Six tiene la
etiqueta (u-, t.(x)), entonces el flujo en la arista (x, u) cambia de/(x, u) a/(x, u)-t,(,z) para
proporcionar el flujo adicional de .dz unidades de x a z.
Conforme continúa este proceso de regreso hasta a, cada arista dirigida a lo largo de un
camínodea azve aumentado o disminuido su flujo [para un vértice (que quede en el cami-
no) con una etiqueta negativa] en 6z unidades. Al hacer esto. eliminamos todas las etique-
tas de los vértices. excepto (- , 00) para la fuente; el proceso se repite para ver si es posible
incrementar el flujo a1lo más.
Caso 2: Si el procedimiento de etiquetado se realiza hasta donde sea posible y el sumidero
z no está etiquetado. entonces se ha logrado el flujo máximo. Sea P el conjunto de vértices
de V que están etiquetados y Ji = V-P. Puesto que los vértices de JJ no están etiquetados,
los flujos de las aristas (x, y), donde x E Pe y E P, satisfacen/(x, y) = c(x, y). Ade!Dil,
para cualquier arista (w, u ) tal que w E JJ y u E P. tt.fU!rrws f(w. u) = O. En consecuencia,
existe un flujo para Ja red dada, tal que el va1or del flujo es igual a la capacidad del corte
(P, P). Por el teorema 13.3, tenemos que este flujo es un máximo.
Figura 13.12
5, O d 4,0 g

3, O 2,0

4, O 5, O h
(a)

d 4,0

2,0 2, O

(b)
.s(a·, 2) 4 , O e (s•, 2) 5,0 h(e•, 2) 4, 2 . 5, 2

b(a°, 7) 5, O di/>', 5) 4,0 b d 4, 3 g


7, O

1,0 3, O 2, O 2, O

s(b', 1) 4, 2 • Cd'. 21 5, 2 4, 2 5, 2
(el

b (a',4) 5, 3 d(b', 21 5, 4 d 4, 4 g

1,0 3, O 2, O ,V,',1) • 2, 0

(di
4, 2 e(d', 2) 5, 2 4, 2 . 5, 2 h

4, 4 5, 5 d 4, 4 g

2, O 1,0 3, O 2, 1

si/>', 1) 4, 2 e(d', 1) 5, 2 h(e·, 1) 4, 2 5, 3 h


(el

5, 5 d(e·, 1¡ 4, 4 b d 4, 4 g

1, O 3, O 2, 1 2, 1

s(b', 1) 4, 2 e (s ·, 1) 5, 3 h(e·, 1) 4, 3 5, 4 h
1n
)78
13.3 Redes de transporte: El teorema de flujo máximo y cone mínimo 679

Antes de demostrar el procedinúento de etiquetado enunciaremos un último corolario y


algunos comentarios relacionados con esto. La demostración del corolario se deja como
ejercicio.

COROLARIO 13.2 Sea N = (V, E) una red de transpone tal que para cada e E E, c(e) es un entero positivo.
Entonces existe un flujo máximo/para N, donde/(e) es un entero no negativo para cada
arista e.

las definiciones de red de transpone y flujo (en una red de transporte) pueden modificane
para admitir funciones de flujo y capacidad con valores reales no negativos. Si las capaci•
dades de una red de transpone son números racionales. entonces el procedimiento de eti•
quetado terminará en cieno momento y se obtendrá un flujo mú.imo. Sin embargo, si
algunas capacidades son irracionales, es posible que el procedimiento nunca 1ennine, pues
se obtendrían A(z) cada vez más pequeñas en cada iteración. Además, L. R. Ford, Jr., y
D. R. Fulkenon [12] han demostrado que el procedimiento podría producir un flujo tal que
no fuera un flujo máximo. Cuando aparecen capacidades irracionales, podemos usar la mo-
dificación hecha por J. Edmoocls y R.M. Karp [9], en la que el procedimiento termina (en
un nllmero finito de pasos) y se obtiene un flujo múim.o.

Utilizaremos el procedimiento de etiquetado para encontrar un flujo máximo para la red


de transporte de la figura 13.12(a).
En esta red. etiquetamos cada arista con un par ordenadox.y, dondex es la capacidad de
la arista yy= Oindica un flujo inicial para la red. La figura 13.12(b)demuestra la primera
aplicación del procedim.ien10 de etiquetado. En este caso, podemos hacer una elección en
el etiquetado del sumidero z. Elegimos (h", 2) como etique1a. en vez de (g•. 3) (¿qué otra
etiqueta podríamos utilizar para el vértice t?) Retrocediendo de z a h a t a s a a, e
incrementando el flujo en cada arista en 6(z) 1111 2. ob1enemos el nuevo flujo de la figura
13.12(b). las partes (c), (d), (e) y (f} de la figura muestran una segunda, tercera, cuarta y
quinta aplicaciones del procedimiento de etiquetado. Observe que el vértice g tiene una
etiqueta negativa en las partes (e) y (f). Además, la pane (f) proporciona una segunda
aparición de una etiqueta negativa. esta vez. para el vértice d. Aplicamos el procedimiento
de etiquetado por última vez para etiquetar la red de transporte como en la figura 13.13. En
esta ocasión. el sumidero z no csti etiquetado y debemos usar el segundo caso del proce•
= =
dimiento. Sean P (a,b J y 7' = [s, d. e, g, h, z]; vemos que c(P, 7') c(b, d) + c(b. s) +
= =
c(a, s) = 5 + 1 + 2 8 = 5 + 3 /(h, z) + /(g, z), el flujo a z, las aristas en N cruzadas por
la línea punteada son las arisw del conjunto de corte (no dirigido) asociado con el cone
(P, 1'), el cual consta de todas las aristas de la forma [x, y}, dondex E P, y E 7'.

Figura 13.13
580 capitulo 13 Optimización y emparejamiento

Cerraremos esta sección con tres ejemplos modelados con el concepto de red de trans--
pone. Después de establecer los modelos. la solución final de cada ejemplo se deja para
los ejercicios de la sección.

Las compañfasci, c2 , c1 fabrican microcircuitos (chips) en unidades de mil; dichos chips se


distribuyen a dos fabricantes de computadores,/1 y f¡, a través de la "'red de transpone" de
la figura 13.14(a). donde hay tres fuentes (c., e, y e,) y dos sumiderosJi yf,. La compañía
'• puede producir hasta 15 unidades, la compañía c2 hasta 20 unidades y la compañía C)
hasta 25 unidades. Si cada fabricante necesita 25 unidades, ¿cuántas unidades debe produ-
cir cada compañía para satisfacer la demanda de los fabricantes de computadores o al
menos proporcionar tantas unidades como permita la red'?
Para modelar este ejemplo mediante una red de transporte, introducimos una fuente a y
un sumidero z, como se muestra en la figura 13.14(b). Usamos la capacidad de prcxlucción
de las tres compañías para definir las capacidades de las aristas (a, c 1), (a, c2 ) y (a. c3) . Para
las aristas (J1, z) y (1,, z) usamos las demandas como capacidades. A fin de responder
nuestra pregunta. hay que aplicar el procedimiento de etiquetado a esta red para detenni-
nar el valor de un flujo máximo.

e,
e,
15 f,
e, 10
15 ,,
e,
e,
(a) (b)

Figura 13.14

La red de transporte que se muestra en la figura 13.lS{a) tiene una restricción más, ya que
ahora tenemos capacidades asignadas a los vénices distintos de la fuente y el sumidero.
Tal capacidad impone un límite superior a la cantidad del bien en cuestión que puede
pasar a travis de un vértice dado. La pane (b) de la figura muestra la forma de volver a
diseñar la red para obtener otra donde podamos aplicar el procedimiento de etiquetado.
Para cualquier vértice u distinto de a o de z, separamos dicho vénice en los vénices u 1 y Ui-
Trazamos la arista de u 1 a u, y la etiquetamos con la capacidad original asignada a u. Una
arista de la forma(u, w), donde u t- a, w -:j:. z, se conviene en la arista(~. w1) que mantiene
la capacidad de (u, w). Las aristas de la forma (a, u) se convierten en la arista (a,u1) con
capacidadc(a, u). Reemplazamos una arista como (W. t ) por la arista (w2, z), con capacidad
c(w, z).
13.3 Redes de transporte: El teorema de flu· múimo cortt mínimo 681

900) 15 h (I O)
(b)

Figura 13.1 S

El flujo máximo de la red dada se determina entonces mediante el procedimiento de


etiquetado aplicado a la red que se muestra en la figura 13.IS(b).

Durante una práctica militar. los mensajeros deben entregar la información del cuanel
general (vttticc a) a la comandancia de campo (vátice z). Puesto que algunos caminos
podrían estar bloqueados o destruidos, ¿cuin1os mensajeros deben enviarse de modo que
cada uno recorra un camino sin aristas en comdn con los demás caminos?
Ya que las distancias entre los váticcs no son importantes. el grafo de la figura 13.16
no tiene capacidades asign3das a sus aristas. El problema aquí es detenninac el nlhnero
máximo de caminos con aristas disjuntas de a a z. Si asignamos a cada arista una capaci-
dad de t. convertimos el problema en uno de flujo máximo, donde el número de caminos
con aristas disjuntas (de a a z) es igual al valor de un flujo máximo para la red.

·~·
g k s
Figura 13.16

EJERCICIOS 13.3 1. a) Para la red de la figw-a 13.17, sea 10 la capacidad de cada arista. s; cada arista,de la fl¡ura
se etiqueta con u.na función f. como se muestra. dc:tennioe los valores de s, 1, w. x y y de
modo que/ sea un flujo en la red.
b ) ¿Cuál es el valor de este flujo?
e) Encuentre tres cortes (P. JJ) en esta red que tcn¡an capacidad 30.
►82 Capitulo 13Optimízación y emparejamiento

Figura 13.17

2. Si N=(V,E) es una red de transporte. su/un flujo enNy (P, P) un corte. Demue·s treque el
valor del flujo/es igual ac(P, Ji) si y sólo si
a) f(,) = e(,) para cada arista,= (x. y), tal que z e P, y e -,, y
b) /(e)= O para cada arista e = (u, w), tal que u E 7'. w E P.
3. Encuentre un flujo máximo y el corte núnimo correspondiente para cada una de las redes de
transpone de la figura 13.18.

15 d d

i 8 j 12 k
(al Cbl
Figura 13.18

4. Aplique el procedimiento de etiquetado para encontrar un flujo máximo en los ejemplos 13.10,
13.11 y 13.12.
S. Demuestre el corolario 13.2.
6. En cada una de las siguientes "rales de transporte", dos compad!as c1 y c2 producen cierto
artículo utilizado por dos fabricantes / 1 y A Para la n::d de la parte (a) de la figura 13.19, la
compañía c1 produce 8 unidades y la compañíac2 produce 7 unidades; el fabricantc/j necesita 7
unidades yel fabricantcf26unidades. Enla red de la figura 13.19(b), cada compañía produce
7 unidades y cada fabricante necesita 6 unidadc:$. ¿En qué situación (o situaciones) safüfacen
las compañías las necesidades de los fabricantes?

b b 4 d

e, ,: e,
h

e, ,, e, 1,

9
(a) (b)

Figura 13.19
13.4 Teoría de empar ·amiento 683

7. Encuentre un flujo máximo para la red de la figura 13.20. Las capacidades de las aristas no
d.irigidas indican que la capacidad es la misma en cualquier dirccci6n. [Sin embargo, para una
arista no dirigida, el flujo sólo puede ir en una dirección a la vez, lo que es diferente de la
siwación de los v&tices b, 8 en la figura 13.1S(a).)

Figura 13.20

13.4
Teoría de emparejamiento

La escuela secundaria de cierta localidad debe contratar cuatro maestros para las siguien-
tes asignaturas: matemáticas (s 1), ciencias de la computación (s2 ) , química (s3 ), física (s,) y
biología (s5) . Hay cuatro candidatos interesados en impartir clase en la localidad: la srta.
Carranza (c1) , el sr. Ríos (e,), la sra. Conés (e,) y la sra. López (e,). La srta. Cananza es
diplomada en matemáticas y ciencias de la computación; el sr. Ríos en matemáticas y
física; la sra. Cortés en biología y la sra. López en química,. física y ciencias de la compu-
tación. Si la escuela contrata a los cuatro candidatos, ¿es posible que un maestro sea asig-
nado para impartir un tema diferente al de su especialidad?
Este problema es un ejemplo de una situación general, el problema de asignación. Usan-
do el principio de inclusión y exclusión, junto con el polinomio de torre ( véanse sccs. 8.4
y 8.5), podemos determinar de cuántas formas, si existen, podemos asignar a los cuatro
maestros a cada tema diferente a1 de su cspeclalidad. Sin embargo. estas técnicas no propor-
cionan un medio para configurar estas asignaciones. En la figura 13.21. hemos modelado
el problema por medio de un grafo bipartito O = (V, E ), donde V se separa como X U Y, con
X.= {c 1, c2 • c3 , e,} y Y = {si, s2, s3, s,, s,}, y las aristas de G representan la especialidad de
cada maestro. Las aristas {ci. s2 }, {c2 , s,}. {c.,. s 5 }, {c,,s3}, demuestran una de tales asigna-
ciones de X en Y.

e, ~ '',,
e,
,,
e,
,,
e,
s~ Figura 13.21

Para analizar esta idea con más detalle, presentamos los siguíentes conceptos.
i84 capítulo 13 Optirrizadón y emparejamiento

>efinición 13.5 Sea G = (V, E ) un grafo bipanito con V dividido como X U Y. (Cada arista de E úene la
forma (x,y), conx E Xy y E Y.)
a) Un emptJTejamiento de G es un subconjunto de E tal que ningún par de aristas
comparte un vértice en X o en Y.
b) Un emparejamiento completo de X en Y es un emparejamiento deG tal que cada
x E X es el extremo de una arista

En términos de funciones, un emparejamiento es una función que establece una corres•


pendencia uno a uno entre un subconjunto de X y un subconjunto de Y. Cuando el
emparejamiento es completo, se define una función uno a uno de X en Y. El ejemplo de la
figura 13.21 tiene dicha función y un emparejamiento completo.

Para un grafo bipanito G = ( V, E) con V dividido como X U Y, un emparejamiento


1.
completo de X en Y requiere que I X I s I Y Si IX I es grande, entonces la construcción
de tal emparejamiento no puede hacerse por prueba y error. El siguiente teorema. del
matemático inglés PhiHp Han (1935- ), proporciona una condición necesaria y suficiente
para la existencia de ta1 emparejamiento. Sin embargo, no usaremos la demostración del
teorema seguida por Hall, haremos una demostrae-i6n consiructiva que usa los resultados
que obtuvimos al e studiar las redes de transporte.

cOREMA 13.5 Sea G = (V, E ) un grafo bipartito con V dividido como X U Y. Existe un emparejamiento
completo de X en Ysi y sólo si para cada subconjuntoA de X, IA 1 :S I R(A) 1, dondeR(A)
es el subconjunto de Y que consta de los vértices adyacentes al menos a un vértice de A.

Antes de demostrar el teorema, lo aplicaremos en el siguiente ejemplo.

a) El grafo bipartito de la figura 13.22{a) no tiene un emparejamie nto completo. Cual·


quier intento por construir dicho emparejamiento debe incluir {xi, y 1} y {x2, y1 } o
{x1,y 3}. Si incluimos {~.YJ}, entonces no existe pareja parax3 • De la misma forma,

x,i y,x,~ y,
Xi

X1
X y

Y1

Y1
X1

Xi
X

Yi

Y3

x, Y, x, Y,

(a) Ys (b) Ys
Figura 13.22
13.4 Teoría de emparejamiento 685

Tabla 13.2
Á R(A ) IAI IRvt)I
0 0 o o
{.t,} {y.,y,,y,} 1 3
{.t,} {y,} 1 1
{.t,} {y,,y,;y,} 1 3
{.t,} {y,,y,l 1 2
{.t.,x,} {y.,y,,y,} 2 3
{.t.,x,} {y.,y,,y,,y,) 2 4
{.t.,x,} y 2 5
{.t,,x,} {y,,y,,y,} 2 3
{.t,,x,) {y,,y, ,y,} 2 3
{.t,,x,) {y,,y,,y,,y,) 2 4
{.t.,x,, x,) {y.,y,,y,,y,) 3 4
{.t.,x,,x,) y 3 5
{.t.,x,, x,) y 3 5
{.t,,x,,x,) {y,.y,,y•• y,} 3 4
X y 4 5

si incluimos {x3, y3 } , no podremos encontrar pareja ~ax2• Si A= {xh x2, x 3} ~ X,


=
entoncesR(A) {yi,y,J. Como l.t 1= 1
3 > 2 = R(A) l. e l teorema 13.5 implica que
no puede existir uo emparejamiento completo.
b) Para el grafo de la parte (b) de la figura, consideremos la lista exhaustiva de la tabla
13.2. Si el teorema 13.5 es válido, esta lista indica que el grafo contiene un empare--
jamiento completo.

Pasemos ahora a la demostración del teorema.


Oemostr.te.ión: Con V dividido como X U Y, sean X= {x.,.íi, /. . ,x,.} y y .. {y1,y2, •• • ,y,.}.
Construimos una red de transpone N que extienda el grafo G introduciendo dos nuevos
vértices, a Oa fuente) y z (el sumidero). Para cada vérticcx;, l Si S m. trazamos la arista
(a, X;): para cada vértice Yr 1 Sj S n, trazamos la arista (y¡, z). Cada nueva arista recibe una
I
capacidad de 1. Sea M cualquier e ntero positivo mayor que X J. Asignamos a cada aris~
tade G la capacidad M. El grafo original y su red asociadaN aparecen en la figura 13.23.
Se sigue que existe un emparejamiento completo en G si y sólo si existe un flujo máximo
en N que use todas las aristas (a, .x¡), 1 :s i :s m. Entonces, el valor de tal flujo máximo es
m= lxl.
Demostraremos que existe un emparejamiento comp1eto en G, mostrando que c(_P, 1')
;,: lx l
paracualquiercone (P, 7') de N. Si (P, 7') es un conearl>itrario en lan:d de trans-
poneN, definimos A = X n P y B= r n P. Entonces A~ X y escribimos A = {x1,.r,, . .. ,
.x;} para algún O :Si S m. (En caso necesario, volvemos a etiquetarlos elementos de X, de
modo que los subíndices de los elementos de A sean consecutivos. Cuando i =O, A .. 0.)
Ahora P consta de la fuente a y los vé rtices de A y el conjunto B !; Y, como se muestra en
la figura 13.24(a). (También etiquetamos de nuevo los elementos de Y en caso ne.cesarlo.)
586 Capirulo 13 Optimización y emparejamiento

(<,) (11)
X y

(a) (b)

Figura 13.23

.Así mismo, P = (X-A) U(Y-B) U {z}. Sie,tis1eunaariSla'{x, y} conxEAyy E¡Y- B),


entonces la capacidad de esa arista es un sumando cnc(P, -P) yc(P, 7');?: M> 1 X . Si no
existe tal arista, entonces de tenrunamos c{P, J5) mediante las capacidades de ( l) las aristas
de la fuente a a los vértices en X - A y ( 2) las aristas de los vértices de B al sumidero z.
Pue sto gue cada una de las arislas tiene capacidad 1, c(_P, P) = lx-A 1 + 1BI = lxl -
IAI + IB I. C-OmoB:lR(A),lcncmos IB I :a: IR(A) l ;ypuesto~uc IR(A)I :a: IA l. sc~-
IJ
gueque 1(B) 1a: IA l. Enconsccucncia,c<.P, 1')= lxl +d ni - (A :a: lx l. Por lo tanto,
ya que cada corte en la red N tiene capacidad al menos de IX1. por el teorema 13.4,

Figura 13.24
13.4 Teoría de emparejamiento 687

I 1.
cualquier flujo m:iximo para N tiene valor X Tal flujo da como resultado exactamente
1X J aristas de X a Y con flujo 1 y este flujo proporciona un emparejamiento completo de
X en Y.
Y viceversa, supongamos que existe un subconjunto A de X en elquelA 1> 1R(A)I. Sea
(P. 1') el corte que muestra la red de la figura 13.24(b), con P = {a} U A U R (A) y 1' =
(X-A) U (Y -R(A)) U {z}. Entonces c(P, 1') se determina por (1) las aristas de la fuente
a a los vértices de X -A y (2) las aristas de los vértices deR(A) al sumidor z. Porlo tanto,
c(P, 1')= IX -A I + 1R(AJl= IX l-,{IA 1-IR(A) l < IX I ,ya que IA I> IR(A)I .Laredtiene
un conede capacidad menor que !xi, por lo que, una vez más, por el teorema 13.14 se si-
gue que cualquier flujo máximo de la red tiene valor menor que lxl.
Por Jo tanto. no hay
un emparejamiento completo de X y Y para el grafo bipartito dado G.

Cinco e studiantes, e1, e2, e3 , e,. y es son miembros de tres comités, Ci. c2 y c3• El grafo
bipartito de la figura 13.25(a) indica los miembros de cada comité. Cada com.itt debe
seleccionar un representante para una reunión con el director de la escuela. ¿Podemos
hacer una selección de modo que cada comité tenga un representante diferente?

(al (b)
e,

Figura 13.25

Aunque este problema es lo bastante pequeño como para resolverse por inspección,
usaremos las ideas desarrolladas en la demostración del teorema 13.5. La figura 13.25(b)
proporciona la red para el grafo bipartito dado. En la figura l 3.26(a), aplicamos el proce-
dimiento de etiquetado (Teorema 13.4) por primera vez. el cual indica que la arista (c 1, e,)
es un posible inicio para un emparejamiento compJeto. (En las figuras hemos omitido
muchas etiquetas de las aristas para que los diagramas sean más sencillos. Cualquier arista
no etiquetada que comience en a o termine en z tiene la etiqueta 1, O; las demás aristas que
aparecen sin etiqueta tienen la etiqueta M, O.) Al aplicar dos veces más el procedimiento
obtenemos la red de la figura 13.26(b). En este caso, el flujo máximo indica que (c., e4 ),
(c1 , e1), (c1, e,) es una posible solución para elegir representantes.
Este ejemplo es un caso particular de un problema estudiado por Philip Hall, quien ana-
lizó una colección de conjuntos Aa. A2-. .. . • A,., donde los ele mentos ai. a2, ••• , a,, forman
un sistema de. nprrsentantes di.sn'ntos para la colección si (a) a, E A1 para todo 1 s i s n;
688 Capíl\Jlo 13 Optirriza<:ión y emparejamiento

(a)
es(Cj, l)
.,
e,

(b)

Figura 13.26

y (b)a¡# a¡, siempre que 1 :s; i <j :S n. Si parafraseamos el teorema 13.5 en este contexto,
veremos que la colección Ai, A1, •• . , A.. tiene un sistema de representantes distintos si y
sólo si, para todo 1 :Si :S. n:, la unióndeicualesquicrade los conjuntosAi. A2 , • • • ,A. tiene
al menos i elementos.

Aunque la verificación de la condición del teorema 13.5 puede ser tediosa. el siguiente
corolario proporciona una condición suficiente para la existencia de un emparejamiento
completo.

COROLARIO 13.3 Sea G = (V, E) un grafo bipartito con V dividido como X U Y. Existe un emparejanúento
completo de X en Ysi, para algún k E Z\ grado(x) 2: k 2: grado(y) para todos los vénices
xEXy y EY.
Oemostr~ci6n: Esta demostración se deja para los ejercicios de la sección.

a) El corolario 13.3 es aplicable al grafo que se muestra en la figura 13.25(a). El valor


adecuado de k es 2 .
b) En un grupo del illtimo año de bachillerato hay 50 estudiantes (25 mujeres y 25
hombres). Si cada mujer de la clase le agrada exactamente a cinco jóvenes y cada
13.4 Teoría de emparejamiento 689

joven disfruta la compañía de exactamente cinco mujeres del grupo, entonces es


posible que cada hombre vaya a la fiesta del grupo con una mujer que le guste y que
cada mujer vaya con un escudiante que se sienta atraído por ella. (Como resultado
de los problemas de este tipo. la condición del teorema 13.5 se conoce con frecuen-
cia en la literatura especializada como la condición dt matrimonio de Hall.)

Para los problemas como el del ejemplo 13.13(a). donde no existe un emparejamiento
completo. el siguiente tipo de emparejamiento suele ser interesante.

nidón 13 .6 Si G • (V, E) es un grafo bipartito con V dividido como X U Y, un ,mpar,jamíento ma.ximal


en G es aquel que relaciona el mayor número posible de vbtices en X con los vértices en Y.

Para analizar la existencia y construcción de un emparejamiento maximal, presentamos


la siguiente idea.

..tinición 13.7 Sea G = (V, E) un grafo bipartito con V dividido como X U Y. Si A (; X, entonces li(A) =
IA1- 1 1
R(A) es ladejiciou:iatú A. La túficienciatúl grafo G, que se denota con 6<,G),
está dada por li(G) = máx{li(A) IA ~ X}.

s = =y
Para S X, tenemos queR(S) S,p<>rlo queli(S) O li(G) ;,; O. Si li(G) > O, existe
un subconjunto A de X tal que I A 1 - 1R(A) 1 > O, por lo que I A 1 > 1R(A) I; el teorema
13.5 implica entonces que no existe un emparejamiento completo de X en Y.

M El grafo de la figura l3.27(a) no tiene un emparejamiento completo. [Wase el ejemplo


l3.l3(a).] Para A = = =
{x,. x,, x,f, vemos que R(A) {y,. y,f, y 6(A) 3 - 2 = l. Como
resultado de este subconjunto A, tenemos que S(G) ~ 1. Al eliminar uno de Jos vénices de
A ( y las aristas incidentes coo ti), obienemos el subgrafo de la parte (b) de la figura. Este

,, X y x, y
Y, Y,

,, r, '• Y,

,, r, '• Y,

,. Y, ,. Y,

r, Y,
(a) (b)
Figura, 13.27
690 capítulo 13 Optimización y emparejamiento

subgrafo (bipartito) proporcíona un emparejamiento completo de X 1 = {.t1 , .t), x..}. en r.


Las aristas {.ii,y1}, {x3, y3 }. y {x,,y,} indican uno de tales emparejamientos, que tamba
es un emparejamiento maximal de X en Y.

Las ideas desarroll~ en el ejemplo 13.16 conducen al siguiente teorema.

TEOREMA 13.6 Sea G = (V, E ) un grafo bipartito con V dividido como X U Y. El número máximo de
vénices en X que se pueden relacionarse con los de Y es / X 1 - 6(G).
Demostra<ión: Haremos una demostración constructiva, mediante redes de transporte, coroo
en la demostración del teorema 13.5. Al igual que en la figura 13.23, sea N la red asociada
con el grafo bipanito G. El resultado quedará demostrado cuando mostremos que (a) la
capacidad de cualquier cone (P, 7') de N es mayor o igual que I X 1- ll(G) y (b) existe ,a
cone con capacidad IX 1 - ll(G).
Sea(P, 7') cualquier corte enN, donde Pestá fonnado por la fuente a, los vénices de A=
P n X<;;; Xy los vénicesdeB=P n Y<;;; Y. [VéaselaFig. l3.24(a).] Como en la demos-
tración del teorema 13.5, los subconjuntos A, B pueden ser vacíos.

1) Sila arista(.<, y)estáen N, conx E Ar.y E Y-B, entoncesc(x, 1)es un sumando de


c(P, 7').Puestoquec(x,y)=M> lx ,scsiguequec(P, 1') > IXI l!: lxl -ll(G).
2) Si ooexiste uoaaristacomoen ( l ), entoncesc(P, 7')quedadeterminadopor las IX-A l
I I
aristas ele a aX-Ay las B aristasdcB az. Puesto que cada una de lasaristastieoc
capacidad J,vemosquec(P, 7')= lx-A I + ls l ; lxl - IAI + IBI.Ninguna
arista conecta uo vénice enA con uo vénice en Y-B, por lo queR(A) <;;;By I R(A)I
s IBI.Enconsccuencia,c(P, 1'J = <IXI -IAI)+ Is! ;,, (l x l-lAI)+ I R(A)I
= lxl-<IA I - IR(A)I) = lxl - ll<AJ;,, lxl-ll<G).

Por Jo tanto, en cualquier caso, c(P, 1') l!: 1X 1 -ll(G), para cualquier cono (P, 1') en N.

Para terminar la demostración. debemos escablecer la existencia de un cone con capa-


cidad lxl -ll(G). Puesto que ll(G) = máx{S(A) 1A<;;; X), podemos elegir un subconjunto
A de Xtal que ll(G)= li(A). Si analizarnos la figura l3.24(b), sea P = {a) U A U R(A).
Entonces 7' = (X-A) U (Y-R(A)) U {<}.No existe uoaaristaeotre los vénices deAylos
deY- R(A),porlocualc(P, 1')= lx- A I + I R(A)I = lxl -<IA I - IR(A)i>= lxl-
ll(A)= lxl -ll<GJ.

Cerraremos esta sección con dos ejemplos que tratan estos conceptos.

SeaG= (V, E ) un grafo bipartito con V dividido como X U Y. Para cadax E X, grado(x) l!:
4 y para cada y E Y, grado(y) s 5. Si Jx l s 15, debemos encontrar una cota superiorOo
nw pequeña posible) para ll(G).
Sea 0;óA <;;;Xy sea E, <;;;E, donde E, = {{a. b) la EA, b E R(A)}. Comogrado(a);,, 4
paratodoaEA, IE ,l ;,,4 JA I.Comogrado(b)S5paratodobER(A). I E,I s5IR(A)I.
Porlotanto,4 IAI s5IR(A)I yll(A)= IA I - IR(A)I s IAl-(415)IAI =(l/5)IAI.Co-
moA <;;;X.tenemos que IA I s IS. porloqueli(A) s ( l/5XIS) = 3. En consecuencia, li(G)=
13.4 Teoría de emparejamiento 691

máx(d(A)IA \;; X} s 3 y existe un emparejamiento maxímal MdeX en ftal que I MI le


lxl -3.

Nuestro último ejemplo se basa en el dado por C. L. Liu en las págs. 289-290 de la
referencia [20].

Se construirá una red de conmutación electrónica para encauzar las llamadas telefónicas
de las líneas de entrada a las líneas troncales de salida Existen 30 líneas de entrada. divi-
didas por igual en las tres categorías I, II y 111~y existen 24 líneas troncales de salida. Cada
línea de entrada de la categoría 1 está conectada de modo que puede conmutar con una de
las cuatro líneas troncales de salida Una línea de entrada de la categoría II puede conmu-
tar con cualquiera de dos líneas troncaJes de salida; una línea ele entrada de la categoría ID
sólo tiene una línea troncal de salida como conexión posible. Además. algunas restriccio-
nes de ingeniería hacen que el número de líneas de entrada que pueden conmutar con una
línea troncal de salida sea como máximo tres. Con base en esta información, si se realizan
llamadas telefónicas en las 30 líneas de entrada. queremos encontrar una cota inferior para
el número máximo de estas llamadas que al menos pueden conmutarse con las 24 líneas
troncales. Lo que buscamos es un emparejamiento maximal de las 30 líneas de entrada con
las 24 líneas troncales de salida. Así, consideramos el grafo bipartito G = (V, E), con V
dividido como X U Y que tiene 30 vértices en X, los cuales representan las 30 líneas de
entrada y los 24 vértices en Y que representan las 24 líneas troncales de salida. Si x E X. y
E Y, entonces {x.y} E E cuando la lineadeentradaxse puede conectar con la línea troncal
de salida y. Para determinar una cota superior de 6(G), consideremos un subconjunto arbi-
trario A de X tal que A lengam lineas de entrada de la categoría 1,n de la categoría II y p de
la categoóa III. (Otra alternativa es decir que A cons~ de m vértices de grado 4.n de grado
2 y p de grado l.) Vemos entonces que

8(A) = IA I- IR(A)l =m + n + p - IR(A)I-


Pero 3 I R(A) 1 le 4m + 2m + 2n + p, ya que cada vfrtice de Y [y R(A) \;; Y] tiene grado 3
como máximo. En consecuencia.

IR(A)I "'(l/3)(4m +2n + p),

por lo queO(A) "m + n + p-(1!3X4m + 2n + p) = (1/3)( -m + n + 2p). Por lo tanto, O(A)


=
alcanza el valormaximal (l/3X-0+ 10 + 20)= lOcuando m = 0,n 10 y p 10. Como A =
es un subconjunto arbitrario de X, se sigue queo(G) S 10. Así, independientemente de có-
mo se construya la red de conmutación, al menos 20 de las 30 líneas de entrada pueden
conectarse con las líneas troncales de salida.

EJERCICIOS 13.4 1. P..-aelgrafodc la figura 13.21,si scleccionamoscuatroaristasalazar,¿cuál es la probabilidad


de que proporcionen un emparejamiento completo de X en Y?
2. Catalina atrae a Alberto, José y Roberto; Juana a José y Daniel; Teresa aAlberto y José; Natalia
a Daniel, José y Francisco; y Carmen a Alberto, Josi y Roberto. (a) Construya un grafo para
modelar el problema de emparejanúento, donde cada hombre se relaciona con la mujer que le
692 capítulo 13 Optirrozad6n yemparejamiento

atrae. (b)Tracc la red asociada al grafo de la pane (a) y determine u.o flujo máximo para esa
red. ¿CUáJ es el emparejamiento completo que determina? (e) ¿Existe un emparej~
completo que relacione a Juana con Daniel y a Natalia con Francisco? (d) ¿Es posible dcltr-
minar dos emparejamientos completos tales que cada hombre se relacione con dos mujeres
diferentes?
3. En cierta escuela. el grupo del llltimo aoo de bachillerato es~ representado en seis comilél
escolares. por Ana María (a). Gastón (G). Julia (J). Carlos (C). Miguel (M). Norma (N). Pablo
(P) y Rosa María (R).quienes eswi en los oomitb de la forma siguiente: {A. G. J. P}. {G.~
C, R}.{A, M , N, P},{A, G, M, N, P}.{A. G, C. N. R} y {G, C. N, R}.
a) FJ consejo estudiantil llama a una reunión donde se necesita la presencia exactamente de ta
estudiante de dltimo año de cada comitf. Encue ntre una selección que maximice el nllllXIO
de estudiantes de último aoo que asistan a la reunión.
b) Antes de la reunión, las finanzas de cada comitf deben ser revisadas por un estudiame
de llltimo ~o que no cst~ en el comitf. ¿Puede lograrse esto? En caso afirmativo.
¿cómo?
4. Sea G= (V.E) un grafo bipartito con V dividido como X U Y,dondeX= {x1,x21 ••• ,X.} y Y=
{y1, y 2, ••• , y.}. ¿Cuántos cmpaccjanúentos completos de X en Y existen si
a) m•2,na4, y G•K..,,.? b) m = 4,tt ""' 4, y G=K.,.'!
e) m=S,n=9, y·G=,K...•? d) msn y G=K,,,..?
5. Si G =(V, E) es un grafo oo dirigido, un subgrafo recubrido< H de Gen el que cada vátía:
tiene grado 1 es un 1-factor(o empanjamjmlo perfecto) de G.
a) SiGtieneun 1-factor,demuestreque lvl espar.
b) ¿Tiene et grafo de Petcrscn un !-factor?
e:) En la figura 13.28, vemos el grafo X.. e n la parte (a), mientras que la parte (b) proporciona
los tres !-factores posibles para~. ¿C\Jjntos 1-factores tiene el grafo /4?
d) Paran E Z♦• seaª• eJ nllmero de 1-factores que existen para el grafo K:.- Encueotrc J
resuelva una relación de iecwrencía para a..

<•)
Figura 13.28
d - - -c
(b)
:7J:i \,
6. Demuestre el corolario 13.3.
7. Blanca se encarga de asignar trabajos de tiempo parcial a los estudiantes de su escuela. Tienr:
25 solicitudes de estudiantes y 25 diferentes trabajos de tiempo parcial dis¡xmibles. Cada soli-
citante esá calificado para realizar al menos cuatro de los trabajos, pero cada trabajo puede ser
realizado ¡x,r un máximo de cuatro de los solicitantes. ¿Puede Blanca asignar a todos los estu-
diantes trabajos para los que estén capacitados'! EJl.plique la respuesta.
8. Para cada una de las sigujentes colecciones de conjuntos, determine, si es posible, un sistema
de representantes distintos. Si no existe tal sistema. explique por qut.
13.4 Teorio de emparejamiento 693

a) A,• {2,3,4},A, • {3,4},A, • {l},A. • {2,3}


b) A, • A, =A,• {2,4,5},A.=A,•(1,2,3,4,5}
<) A, •(1,2}, A, • (2,3,4},A, = (2,3},A. • {1,3},A, •{2, 4}

9. a) D<tamine todos los sistanasde-disúnlos para la colca:ióa de0011juorosA, •


(1, 2},A, = (2, 3},A, = {3,4},A. • {4, I}.
b) Dada la colección de c:onjuntos A,• (1, 2}, A,= (2, 3}, A,= (3, 4} • .. . , A,= (n, 1),
determine cumtos sistemas de representaDtcs distintos Wsteo para dicha colección.
10. SUA,.A~ .• . ,A.. unacolcccióodeooojuntos tal que A, •A,=··· =A. y IA.I
= k>O para
todo I S Í S n. (a) 0emuestn: que la colccci6o - tiene UD sistema de- -
si y sólo sin s .t.. (b) Sin S k. ¿cúntos listcmas difcreotei tiene la colcoci6o.?
11. SuG• (Y, E ) ungraobipartitocon Y dividido como X u r. Si grado(a) a: 4para todoz ex
y grado(y) s 5 para todo y e Y, demuestn: que si lxl s 10, cntOOC<Sa{G) s 2.
r.
12. SUG= (V, E) un grao bipanitocoo VdivididocomoX u Para todoz e X, grado(a) a: 3 y
para todo ye r,grado(y) s 7. Si lxl s 50,mcucwnoaoocasup<riorOoaw P'QUdlapo-
s,ole) para a(G).
13. a) SUG•(V, E)el ¡rafo bipartito dela fi¡ura 13.29, con VdivididocomoXU Y. Dctemúne
a(G) y un emparejamiento maximal de X en Y.
r.
b) Para cualqw« grao bipanito G = (V, E ). con V dividido como X u si J!{G) denoca el
o4mero .,_. indcpendmcia ,te G, muestre: que Ir l
= J!(G) -a{G).
<) l>e1erminc un conjW>IO maximal independiente de vátices, lo 11W grande po$Íblc. para los
graos delas figuru l3.27(a) y 13.29.

X y
z, Y,

x, Y,

x, Y,

x, y,

X, f'"tgUril 13.29

14. Una red de coomutac:ión tiene nueve Uoeas de entrada y seis lineas troncaJes de salída. Debido
a consideracioocs de ingeoieda. cada línea de eouada de la red se cooecta con dos líneas tron-
cales de salida. micnuas que cada linea troocal de salida se oooecta con oo mú de tres líneas
de-
a) Sea A el coojUD(() den líneas de cnuada.. Muc:strc que en d grafo bipartito G asociado con
esta r<d de onrunutación, L,..grado(o) s l;...,,,gndo(b).
b) Para cualquier onnjunto A (como co Lt parte (a)) den líneas de cnlnda, demuestn: que
1R(A) 1 a: (2/3)n..
<) Para cualqw« conjunto A den llneat de entrada. muestre que a(,!) S (l13)n..
d) Ellpüquc por ~ siem¡R podemos ooooc<ar al mcoos seis líneas de entr>da a las lfneas
croocalcs de salida, indepcodicnternenie del disdlo de Lt r<d de conmutación.
694 capítulo 1 3 Optimizac:ión yernparEjamóento

13.5
Resumen y repaso histórico

Este capítulo ha ofrecido una muestra de la forma en que la teoría de grafos entra en •
área de las matemáticas Uamada investigación operativa. Presentamos cada tema en uaa
forma algorítmica que puede utilizarse eo la implementación computacional necessia
para resolver cada tipo de problema. Un tratamiento similar al de este material aparece aa
los capfrulos 10 y 11 del texto de C. L. Llu [20) y en el capírulo 4 de A. Tucker [26). Los
capírulos 4 y 5 de E. Lawler [ 19) ofrecen un amplio panorama de muchos otros desarrollos
acerca de redes y emparejamiento. Este texto proporciona una amplia gama de aplicacio-
nes e incluye bibliografía para lecturas posteriores.
En la sección 13.1 analizamos el algoritmo del camino más corto para grafos pondera-
dos. El desarrollo completo del algoritmo aparece en el artículo de E. W. Dijkstra [8].
La sección 13.2 presentó dos técnicas para encontrar un árbol recubridor minimalen 111
grafo ponderndo no dirigido, conexo y sin lazos. Estas t~cas fueron desarrolladas a fines
de la década de 1950por Joseph Kruskal [18] y Robert Prim [23]. Sin embargo, l o s _
dos de construcción de árboles recubridores minimales datan en realidad de 1926, con d
trabajo de Otakar BorB.vka acerca de la construcción de redes de energía eléctrica Aún
antes de esto ( 1909-1911), el antropólogo Jan Czckanowski, en su trabajo acerca de di ver•
sos esquemas de clasificación, estuvo cerca de reconocer el problema del árbol recubridoc
minimal y proporcionar un algoritmo voraz para su solución. El artículo de R. L. Graham
y P. Hell [ 14] menciona las contribuciones de Bordvka y Czekanowski y ofrece más infor•
mación acerca de la historia y aplicaciones de esta estructura.
La implementación computacional de todas las técnicas dadas en las dos primeras sec--
ciones aparece en los capítulos 6 y7 de A. Abo, J. Hopcrofty J. Ullman [!]; en el capítulo
4 de S. Baase [2]; en el capítulo 4 de E. H orowitz y S. Salm.i [15). Estas referencias tam-
bién analizan la eficiencia y velocidad de estos algoñtmos. Como mencionamos al final de
la sección 13.2, los artículos escritos por R L. Graham y P. Hell [14), por D. B. Johnsoo
[16) y por A. Kershenbaumy R Van Slyke [17) analizan algunas de las implementaciones

Edsger W. Oijkstra (1930- )


13.5 Resumen y repaso histórico 695

más recientes del algoritmo de Prim. Una aplicación interesante del concepto de árbol
recubridor minimal en el marco de la física aparece en el artfcuJo de D. Shier [25].
Como observamos en la sección 13.3, los problemas que abordan la asignación de
recursos o el embarque de bienes puede modelarse por medio de redes de transporte. El
trabajo fundamental de G. Dantzig, L. Ford y D. Fulkersoo que produjo el pr=dimicoto
de etiquetado para estas mies y el t<:orcma del flujo máximo y corte mínimo puede leerse
en sus artículos pioneros [6, 7, 10, 11]. El texto clásico de L. Ford y D. Fulkerson [12]
ofrece un tratamiento excelente del tema. Además, el lector puede leer el capítulo 8 del
texlo de C. Borgc (3] o el capílU!o 7 del libro de R. Busaclcer y T. Saaty [5]. El capítulo 10
de C. L. Liu [20] incluye una extensión al caso de redes en las que el flujo en cada arista.
está restringido por capacidades máxima y mínima. Para m.ás aplicaciones. el lector puede
analizar el artículo de D. Fulkerson en las p.áginas 139-171 de la rdercncia [13).
El último tema que analizamos fue el emparejamiento en un grafo bipartito. La teoría
subyacente fue desarrollada en primer lugar por Philip Hall en 1935, pero aquí usamos las
ideas de mies de transporte para proporcionar un algoritmo de solución. El capítulo 7 del
texto de O. Ore [22] ofrece una introducción al tema bastante comprensible. junto con
algunas aplicaciones. Para más detalles acerca de los sistem~ de representantes, el lector
debe examinar el capítulo 5 de la monografía de H. Ryser [24). Un segundo mélodo para
encontrar un emparejamiento maximal en un grafo bipartito es el método Jumgaro, que
ofrece en el capílulo 5 del 1ex1odeJ. Bondy y U. Murty [4] yen el capllU!o !O del libro de
C. Borge [3]. Además de su aplicación para resolver el problema de la asignación, la teoría
de emparejamiento tiene muchas aplicaciones combinatorias interesantes. El lector puede
aprender más sobre esto en el artículo de L Mirsky y H. Pcrfcct [21].

BIBLIOGRAFÍA

l. Abo,AlfredV.,JohnE. Hopcrofty Je/frey D. Ullman,DataS1nu:ouuandAlgoriJhms, Reading,


Mass., Addison-Wesley, 1983.
2. Baase, Sara. Compuleraigorithms: ln.rroducrion ,o Design andAnalysis, 2• ed., Reading. Mass.,
Addison-WesJ.ey, 1988.
3. Betge, aaude, The Theory ofGraphs and fu Applicalion.r, Nueva York, Wtley, 1962
4. Bondy, J. A. y U. S. R. Mutty, Graph 1Mory wilh Appücations, Nueva York. Elsevier North-
Holland, 1976.
S. Busacker. Robert G. yThomas L. Saaty, Finite. Graplu and Ne.tworlcs, Nueva York. McGraw-
Hill, 1965.
6. Danttig, G.B. y D.R Fulkerson. Computalion of Maximal Flows in Ne.tworJcs, 1be RAND
Corpontion, P-677, 1955.
7. Oantzig, G.B. y D.R. Ful.kerson. On the Mai Flow Min C,a TMoran, Tbe RAND Corpontioo..
RM-1418-1, 1955.
8. Dijkstra. E&ger W., ..ANote on Two Problems in Connexioo with Grapbs", Numerische
Ma1honatik l, 1959, págs. 269-271.
9. Edmonds. J. y R.M. K.arpo, '1beoretical Improvements inAlgoritbmic Efficíency forNetwork
Flow Problems", J. Assoc. CompUI. Mach., 19, 1972, págs. 243-264.
10. Ford, L. R. Jr., Nawork F/Qw Theory, Toe RANO Corporation, P-923, 1956.
11. Ford, L. R. 1L y D. R. Fulkcrson, "Maximal Flow Througb a N-or1:•, Canadúzn JoumaJ of
Mathunalic, 8 (1956), págs. 399-404.
696 Capitulo 13 Optimización y emparejamiento

12 Ford.. L.R. Jr_ y D. R. Fulkerson. Flows in Networla, Princeton, NJ., Princeton Universiry
Prcss, 1962.
13. Fulkenon. D. R, editor., S1ud.ies in Graph Theory, Parte l. MM SlUdUs in Ma1hematics, \IOl.
11, Thc Mathematical Association of America, 1975.
14. Graham, Ronald L. y Pavol Hell, ..OD tbe History of the MiIUmum Spanning Tree Problcm•.
AMalsoftht History ofúnnputing 1, núm. 1 (enero de 1985), págs. 43-57.
15. Horowitz.. EJlis, y Sartaj Sahni, Fundamental.so/ Computt r Algoriihm.s, Potomac. Md..
Computer Science Prcss. 1978.
16. Johnson, D. B., ..Priority Qucues with Update and Minimum Spanning Trecs", lnformatiotl.
Processing Let1ers 4, 1975, Págs,. 53-57.
17. Kershe nbaum, A. y R. Van Slyke, '"Computing Minimum Spanning Trees Efficicnlly"",
Proce.edingsofthe Annuo.l ACM Ccnfe.rence, 1972, págs. 518--527.
18. Kruska1, Joseph B. "On the Shortest Spanning Subtrcc of a Graph and lhe Traveling Salcsma
Problcm", Proceedings 0/1/ie AMS 1, ndm. l, 1956, págs. 48-SO.
19. Lawler, Eugene, Combinau'Jrial Optimi1,JllUNI...- Nerworlcs and Ma.rroids. NuevaYork, Holt.1976.
20. Liu, C. L., lntroduction to Comhinatorial Mathemalics, Nueva York, McGraw•Hill.1968.
2 J. Minky, L y H. Pcrfcct, ..Systems of RepresentativC$'". Joumal of MatM nuJtical An.alysis tnl
AppUcations 3, 1966, págs. 520-568.
22. Ore, Oystein, Thw ry o/Graphs, Providence. R.J., American Mathcmatical Socicty, 1962
23. Prim, Robert C., ..Shonest Connection Networks and Sorne Generalizations", &11 Systnt
úchnical Journal 36. 1957, págs. 1389-1401.
24. Ryser, Herbert J.• Combinatoria/ Ma1hematics, Caros Matbematical Monographs, número 14-,
Matbemacical Association of America, 1963.
2.S. Shier, Douglas R., 1'esting for Homogcneity Using Mínimum Spanning Trees", ~ UMAP
Jounwl3, núm. 3, 1982, págs. 273-283.
26. Tucker, Alan, Applied Combinatorics. red.. Nueva York., Wtley, f98S.

EJERCIOOS
COMPLEMENTARIOS

1. Aplique e l algoritmo de Dijkstta al multigrafo dirigido


ponderado de la figura 13.30 y encuentre la distancia más
cona del vbtice a a los otros siete vértices del grafo.
2. Para su clase de anilisis de algoritmos, Estela escribe
el siguiente algoritmo para determinar la distancia más cor•
ta de un v6rticc a a un vfrtice b e n un grafo dirigido ponde--
rado G = ( V, E).
Paso 1: Hacemos Distancia igual a O, asignamos un vértice
a ala variable u y T= V. Figura 13.30
Paso 2: Si u= b, el valor de Distancia es la respuesta al
problema. Si u ~ b, entonces 2) Hacemos Distancia igual a Distancia + p(u, w).
1) R<emplazarnos Tcon T-{u} y seleccionamos"' ET 3) Asignamos el vbtice w a la variable u y después
tal que p(u, w) sea minimal regresamos al paso 2.
E" ocios complementario~ 697

axrccto el algoritmo de Estela? En tal caso, demutstre-


Si no, dt un contraejemplo. B
0.2 0.1
=[ 0.4
º·~
0.5 0.1 ,
J. 1) Sea G = (V, E ) un grafo ponderado, no dirig;do, 0.4 0.4 o.
conexo y sin lazos. Si e 1 E E tal que p(e1) < p(it)
para las demás aristas e E E, demuestre o muestre Verifique que Bes doblemente estocástica.
que es falso que la arista e2 es parte de todo árbol e) Encuentre cuatro números Jeales positivos ci, cz,
recubridor minimal de G. e,. ye, y cuatro matrices de permutaciónPhPbPi.
b) Con G como en la panc (a), supongamos que exis- y P, talesquec1 +c2 +c, +ce= 1 yB= c 1P1 +C]Pz
ten aristas ei,e2 E E tales que p(it1) < p(et) < p(e) +c,,l\ +cJ',.
para las demás aristas e E E. Demuestre o pruebe d ) La parte (e) es un caso especial del teorema de
que es falso que la arista e2 es parte de cualquier Birkhoff•von Neumann; si 8 es una matri:z: doble-
árbol recubridor minimal de G. mente estocástican x n, entonces existen nt1meros
4. 1) Sea G = (V, E) un grafo ponderado no dirig;do, reales positivos c1, c2, •• • , Ct y matrices de
conexo y s.in lazos tal que cada arista e de G es L,~c;
J>Cl}°utación P1, P2, • •• , P.. tales que =1 y
parte de un ciclo. Demuestre que si e1 E E con L,.,.
e;P, = B. Para demostrar este resultado, procc,.
p(eJ> p(e) para las demás aristas¿- E E.entonces da de la manera siguiente: construya un grafo
Dingdf! árbol recubridor de G que contenga e1 pue- bipartito G = (V, E) con V dMdido como X U Y,
de ser maximal. donde X= {X1,X2, ••• ..x,.} y donde Y= b1,Yz. •···Y~}.
b) Con G como en la parte (a). supongamos que eh e1 El v&tice X;, para todo 1 :S i :S n, corresponde con
E E, con p(e1 ) > r ~ei) > p(e) para las demás aristas la i-6ima fila de IJ; el vútice y,. para todo 1 :S j :S
e E E. Demuestre o pruebe que es falso que la aris- n, corresponde con la j-6ima columna de B. Las
ta e2 no es parte de Dingón árbol recubridor minimal aristasdeGsondelafonna {X;, y1} si y sólo si bit>
deG. O. Afirmamos que existe un emparejamiento com•
pido de X en r.
5. Use el cooceptO de flujo en una red de tranSportC y En caso contrario, existe un subconjunto A de
-y•un multigrafodirig;doG = (V, E ), con V= {u, u, X,con IAI > IR(A) i. Esdecir,existeunconjunto
-~ y} yge(u) = 1, gs(u) = 3; ge(u) = 3,gs(u) = 3; ge(w) = de r filas de 8 que tiene elementos positivos en s
3.g,(w) = 4; y ge(,)= 5, ge(y) = 4, gs(y) = 2. columnas y r> s. ¿CuM es la suma de estas r filas
&. Hay que transmitir un conjunto de palabras {qs, tq, ut, de B? Al1.n así. la suma de estos mismos elementos,
19{ srt}, con un código binario para cada letra. al sumarse por columnas, e s menor o igual que s.
a) Muestrc que es posible seleccionar una letra de cada (¿Por~?) En consecuencia. tenemos una contra-
palabra. como un sistema de representantes distin• dicción.
tos de estas palabns. Como resultado del emparejamiento completo
b) Si se selecciona una letra al azar de cada una de las de X en Y, existen n elementos positivos en B que
cinco palabras. ¿cuál (;$ la probabilidad de que la aparecen de modo que dos de ellos no estío en la
selección sea un sistema de representantes distin- misma fila o columna (¿Por qué?) Si e, es el más
tos de las palabras? pequeño de estos e lementos, entonces podemos es-
cribir 8 = c 1P 1 + Bi, doode P 1 es una matriz de
7. Paran E Z•ypan I si S n, seaA, = {I, 2. 3, ... , n)-
permutación n X n tal que los UOO$ se localiz.an de
li}. ¿Cuántos
s.i~mas de representantes distintos existen
acuerdo con los elementos positivos de B que sur-
pnlacolccciónAi,A2>A1, ... ,A.? gieron del emparejamiento oompleto. ¿Cuánto vale
8. Este ejercicio bo~eja una demostración del teorema la suma de los elementos en las filas y columnas de
de Birthotf•von Neumann. B1?
a) Para n E z•. una matriz. n X n es una marriz tk e) ¿c.ómo termina la demostración?
permulación si existe exactamente un 1 en cada
9. SeaG = (V. E) un grafobipanitocon V dividido como
fila y columna y si todos los demás elementos son
X U r. Si E' ~ E y E' determina un emparejamiento com-
O. ¿Cuintas matric.es de permutación existen?
pleto de X en Y, ¿~ propiedad tienen los v&ti~ determi-
¿Cuintas n X n?
nados por E' en el grafo de línea L(G)? [El grafo de línea
b) Unamalrizn x nB esdoh/.t:menuestocástica sib1 2:
U,G) para un grafo no dirigido sin laz..os G se define en el
Opara todos 1 ::s i, j ::s n y la suma de los ciernen•
ejercicio complementario 22 del capítulo 11.]
tos en cualquier fila o columna es 1.
PARTE

4
ÁLGEBRA
MODERNA
APLICADA
14
·nos y aritmética
modular

n esta cuana y última parte del texto nuevamente baremos e nfasis en la estructura al
E analizar conjuntos de elementos que son cerrados en dos operaciones binarias. Los con-
ceptos de estructura y enumeración con frecuencia se refuerzan entre sí.Aquí veremos que
esto ocurre conforme vayan surgiendo las ideas de los capítulos 4, 5 y 8.
En el capfw.lo 4, cuando examinamos el conjunto z. lo hicimos junto con las operacio-
nes binarias cerradas de suma y producto. En este capítulo haremos hincapié en estas
operaciones, escribiendo (Z. +.•) en vez de solamente Z. Tomando como patrón algunas
propiedades de (Z, +,•)y definiremos la estructura algebraica llamada anillo. Sin saberlo,
beroos trabajado con anillos en varios contextos matemáticos. Ahora nos ocuparemos de
los anillos finitos que surgen en las aplicaciones de teoría de números y ciencias de la
computación. De particular inteRs en el estudio de las ciencias de la computación es la
fondón de dispersión, que nos ayudará a identificar los registros guardados en una tabla.

14.1
estructura de anillo: definición
y ejemplos

Comenzaremos con la definíción de la estructura de anillo. observando que la mayoría de


las definiciones abstractas, al igual que los teoremas, surgen del estudio de muchos ejem-
plos donde reconocemos la idea o ideas comunes presentes en lo que parece una colección
de objetos no relacionados entre sí.

"ci6n 14.1 Sea R un conjunto no vacío con dos operacionas binarias cerradas, denotadas con+ y·
(que pueden ser diferentes de la suma y producto usuales). Entooces (R, +, •) es uo anillo
si para todos a, b, e E R se cumplen las siguientes condiciones:

a) a+b=b+a Ley conmutativa de +


b) a+ (b +e) = (a + b) + e Ley asociativa de+

701
702 Capítulo 14 Anillos y aritmética modular

e) Existez E Rtal que Existencia de una identidad para +


a +.z =z +a para todo a E R.
d) Para cada a E R e xiste un eleme nto Existencia de inversos bajo +
b E R tal que a +b =b +a =z
e) a•(b•c)=(a •b)·C Ley asociativa de •
1) a-(b+c)=a -b+a • e Leyes distributivas de• sobre+
(b +e) -a =b • a +e• a

Puesto que las operaciones binarias cerradas de+ (suma en el anillo) y • (producto en d
anillo) son asociativas, no hay ambigüedad si escribimos a+ b + e para (a + b) + e oa+
(b + e) oa • b • e para (a •b) •coa • (b • e). Al trabajar con la operación binaria cerrada del
producto en un aniUo, escribiremos con frecuencia ah en lugar de a • b. Además, podemos
extender las leyes asociativas (dadas en la definición de un anillo) como en los ejercicios
8 y 9 de la sección 4.2. Por medio de la inducción matemática, podemos verificar que para
todos,; n E Z•, con n ?!. 3 y 1 ::s r< n,

(a1 + ai + · · · +a,)+ (a,♦1 + · · · +a.,.) = a1 + a2 + · · · +a,+ a, ...1+ • • • + a,..,

(a1a2· · · a,)(a,..¡ ··•a,.)= a1a2 · · · DrDr+t • • ·a11,


dondeª" az• . .. • ª" a,.. 1, ••• , ª• son elementos de un anillo dado (R, +, •). En forma
correspondiente. las 1eyes distributivas se generalizan como sigue:

a(b1 + b, + · · · + b.) = ab 1 + ab,+ · · · + ab.,


(b,+b,+ · ·· + b.)a =b ,a+b,a + · · · + b.a ,

para a. b¡, ~•. .. , b. elementos arbitrarios del anillo y cualquier n E z•, donde n ~ 3.
En la siguiente sección aprenderemos que la identidad de la suma (o elemento neutro)
es único, al igual que el inverso aditivo de cualquier elemento del anillo. Por ;1 momento,
veamos algunos ejemplos de anillos.

En las operaciones binarias (cerradas) de la suma y producto usuales, Z. Q, R y C son


anillos. En todos estos anillos, la identidad de la sumaz es el entero O y el inverso aditivo
de cualquier número x es el conocido -x.

Sea M2(Z) el conjunto de todas las matrices 2 X 2 con elementos enteros. [Los conjuntos
M,1.Q), M,I.R) y M,I.C) se definen en forma similar.] En M,(Z), dos matrices son iguales si
sus elementos correspondientes son iguales en Z.
Definimos + y • como

a b]+fe t ] = [ª+e b+f]


[e d [g h c+g d+h' [
ªe b]f•[g
d
t]=[ªe + bg af+bh]
h ce+dg cf+dh
14.1 La estructura de anillo: definición y ejemplos 703

En estas operaciones binarias (e.erradas), MiZ) es un anillo, con z = [8 8] y el inverso


aditivo de[ª b] es [-a - bJ.
ed - e -d
Sin embargo, aquí suceden cosas que no ocurren en los anillos del ejemplo 14.1. Por
ejemplo,

muestra que el producto en un anillo no tiene que ser conmutativo. Ésta es la razón por la
que hay dos leyes distributivas. Además,

[-11 -1][2
12 11]-[ºOO'º]
aunque ni[_: -~]ni[~ n son la identidad para la suma Por lo tanto, un anillo puede
tener divisores propios de cero; es decir, elementos distintos de cero cuyo producto es el
elemento neutro del anillo.

Ampliaremos ahora nuestro estudio de la estructura de anillo.

· "ón 14.2 Sea (R, +,-) un anillo.


a) Si ah = ba para todos a, b E R, entonces R es un anillo connwltltivo.
b) El anil1o R no tiene divisores propios de cero si para cuaJesquieraa, b E R, ah =z ===>
a= z o b =z:.
e) Si un elemento u E Res tal que u i= z y au = ua =apara todo a E R, decimos que
u es elemenJo unidad, o itúntidad para el producto, de R. R es entonces un anillb
con unidad.

De la pane (c) de la definición 14.2 se sigue que siempre que tengamos un anillo R
con elemento unidad, entonces R contiene al menos dos elementos. Además, si un anillo
tiene elemento unidad, veremos en la siguiente sección que éste es único.
Los anillos del ejemplo 14.1 son conmutativos, cuyo elemento unidad es el entero l . Nin-
guno de estos anillos tiene divisores propios de cero. Por otro lado, el anillo M.f..Z)
es un anillo no conmutativo, cuyo elemento unidad es la matriz (Ó ?] .Este anillo contie-
ne divisores propios de e.ero.
Además, siempre que queramos verificar que una configuración particular (R, +, •) es
un anillo conmutativo, una vez que sabemos queR es cerrado en ambas operaciones binarias
y que el producto es conmutativo, sólo debemos establecer una de las leyes distributivas
(ya que la otra se sigue en forma automática.)
Estudiaremos ahora otro ejemplo para continuar el análisis de las ideas plasmadas en
las definiciones 14.1 y 14.2.
capítulo 14 Anillos y aritmética modular

Consideremos el conjunto Z con las operaciones binarias de EB y 0. definidas como

xE!')y=x+y-1, x 0y = x+ y-xy.

Pueslo que la suma, resta y producto usuales son operaciones binarias cerradas para Z.
estas nuevas operaciones binarias (EB y 0) también son cerradas en Z. De hecho, veremos
que (Z, $, 0) es un anillo.
a) Para verificar que (Z. EB, 0) es un anillo, debemos establecer las seis condiciones
dadas en la definición 14.1. Examinaremos cuatro de tales condiciones y dejaremos
las otras dos para los ejercicios de la sección.
1) En primer lugar, como la suma ordinaria es una operación binaria conmutativa
en Z, vemos que paracualesquierax, y E Z,

xE!')y = x+ y -l= y+x -l= yE!')~

Así, la operación binaria E& también es conmutativa en z.lo que establece Ja


condición (a) de la definición 14.1.
2) Al analizar la condición (c), vemos que necesitamos encontrar un entero z tal
que a EB z = z EB a= a, para cada a en Z. Por·10 tanto, debemos resolver la.
ecuación a + z - 1 • a, lo que implica z=1. Por lo lanlo, el enlero l dístinto th
cero es el elemento neutro (o identidad de la suma) para EB .
3) ¿Qué ocurre con los inversos aditivos? En este momento, si lenemos un enlero
(arbitrario) a. queremos saber si existe un entero b taJ que a EB b =b EB a =t. De
la parte (2) y la definición de EB , sabemos que el entero b debe sarisfacer a + b
- 1 = 1, por lo que b = 2 - a. Así, por ejemplo, el inverso aditivo de 7 es 2 - 7 =
- 5 ye) inversoaditivode-42es 2 - (- 42) •44. Después de todo, en el caso de
7 vemos que 7 EB (-5) ; 7 + (-5) - 1; 7 -5 - 1; 1, donde 1 es la identidad
aditiva. [Nota: Puesto que hemos mostrado en (1) que E9 es conmutativa, tam-
bién sabemos que (-5) EB 7 = l.]
4) Para establecer la primera ley distributiva es necesario mostrar que para cua-
lesquiera a, b. e E Z,

a 0(bE!')c) = (a0b)E!')(a Oc).

Esto es cieno pues

a0(bE!')c) =a0(b +e-!) =a+ (b +e- 1)-a(b +c - 1)


=a +b +c-1-ab-ac +a.
mientras que

(a0b)E!')(a0c) = (a + b -ab)E!')(a + e - ac)


= a+ b-ab +a+ c-ac-1
= a + b + e - 1 - ab - ac + a.
(Nota: En vez de verificaren forma directa la segunda ley distributiva, demostrare.
mos y usaremos el hecho de que x O y ; y 0 x para todos x, y E Z.)
14. 1 La estructura de anillo: definición y ejemplos 705

b) Además, el anillo (Z , $, 0) tambifo tiene las propiedades adicionales dadas en la


definición 14.2. En particuJar. este anillo tiene elemento unidad (es decir, una iden-
tidad para el producto). Para determinar el elemento unidad, sea a cualquier entero
y consideremos el elemento u (i= z = 1) donde

a0 u=u 0 a=a.
Como a 0u =a+ u-au, resolvemos a +u-au =apara encontrar que u(l -a)=
O. Como a es arbitrario, estO debe cumplirse incluso cuando a t-1. En consecuencia.
el de integridad u = O es el elemento unidad del anillo (Z, $, 0).

Después de estos ejemplos de anillos infinitos, veremos ahora algunos anillos con un
número finito de elementos.

Sean '1.l ={1, 2} y R =~ ('\I). Definimos + y • sobre los eleme ntos de R como
I
A + B =At:. B= {x x E A oxE B, pero no ambos}
n B= la intersección de los conjuntos A, B ~G\l.
A• B -= A
Formamos las tablas 14.l(a) y (b) para estas operaciones.
Por medio de los resultados del capítulo 3, vemos que R satisface las condiciones (a),
(b), (e) y (j) de la definición 14.1 para estas operaciones binarias (cenadas) de "suma" y
"producto". La tabla de la "suma" muestra que Ses la identidad aditiva. Para cualquier x
E R, el inverso aditivo de.res el mismox. La tabla de multiplicación es simétrica sobre la
diagonal del extremo superior izquierdo al exlremo inferior derecho, por lo que la opera-
ción descrita por esta tabla es conmutativa. La tabla muestra también que R tiene ~lenunzo
unidad/JU . Así.Res un anillo conmutativo finito con elemento unidad. Los elementos {1},
{2} son un ejemplo de divisores propios de cero.

Tabla 14.1
+ (il) 0 {!} {2} 'lL . ( n) 0 {I} {2} 'lL
0 0 {!} {2} 'lL 0 0 0 0 0
{1} {!} 0 'lL {2} {1} 0 {1} 0 {1}
{2} {2} 'lL 0 {!} {2} 0 0 {2} {2}
'lL 'lL {2} {1} 0 . 'lL 0 {1} {2} 'lL
(a) (b)

Para R {a. b, e, d. e ), definimos+ y • mediante las tablas 14.2(a) y (b).


Aunque no las verificaremos aquí. las 125 igualdades necesarias para establecer cada
una de las leyes asociativas y distributivas son válidas, por lo que (R, +, ·) es un anillo
conmutativo finito con elemento unidad. sin divisores propios de cero. El elemento a es el
706 Capitulo 14 Anillos y aritmética modular

Tabla 14.2

+ a b e d e a b e d •
a a b e d e a a a a a a
b b e d e a b a b e d •d
e e d e a b e a e •b b
d d e a b e d a d •e e
e e a b e d • a e d b
(a) (b)

cero (es decir, la identidad aditiva) de R, mientras que b es eJ elemento unidad. En este
caso, todos los elementos distintos de cerox tienen un inverso multiplicativo y tal que .:cy=
yx = b, el elemento unidad. Los ele mentos e y d son inversos multiplicativos entre sí; bes
su propio inverso. lo mismo que e.

Definición 14.3 SeaR un anillo con elemento unidad u. Sía. b E Ryab =ba =u, entonces b es uninverJO
multiplicativo de q y a es una unidad de R. (El elemento b también es una unidad de R.)

En la sección 14.2 veremos que si un elemento del anillo tiene un inverso multiplicativo.
entonces tiene un dnico inverso.

Definición 14.4 Sea R un anillo conmutativo con elemento unidad. Entonces


a) Res un dominio de inugridad si R no tiene divisores propios de cero.
b) R es un cuerpot si todo elemento distinto de cero en R es una unidad.

El anillo (Z, +, ·) es un dominio de integridad pero no un cuerpo, mientras que Q, R y


C, con la suma y producto usuaJes, son dominios de integridad y cuerpos. El anillo del
ejemplo 14.5 es un dominio de integridad y un cuerpo.
De la pane (e) de la definición 14.2 se sigue que siR es un dominio de integridad o un
cuerpo, entonces I R 1 .e 2.

Como último anillo finito de esta sección, sea R ={s, t, u, w, x. y}, donde + y · están dados
en las tablas l4.3(a) y (b).
De estas tablas vemos que (R, +, •) es un anillo conmutativo con elemento unidad, pero
no es dominio de integridad ni cuerpo. El elemento res el elemento unidad y r y y son las
unidades de R.

t El tmn!DO fuld ta.mbitn se traduce como '"campo-. (N. del E.)


14.1 La estructura de anillo: defintci6n y ejemplos 707

Tabla 14 3

+ s t w y s t w X y
"
X
" s s
s s t w X y s s s s s
t t u "w y s t s l w X y
w
X
y s t s "X s u X
"w "w X
X
y s t "w s "w s w s w
y s t u "w s s X u
X
y
X
y s t w
X
y s
X
y "
X w t
" X
"
(a) (b)

Tambiin observamos que uu = uy, y aunque u no es el elemento neutro de R, no pode-


mos cancelar y decir que u= y. As(, un anillo general no satisface la ley de la cancelación
del producto que a veces damos por hecho. Analizaremos esta idea nuevamente en la
siguiente sección.

CIOS 14.1 1 . Eocuentrcel inverso aditivo de cada uno de losclemcntosde1os anillos de los ejemplo 14.5 y 14.6.
2. Determine si cada uno de los siguientes conjuntos es un anillo con la suma y producto usuales.
a) R = el conjunto de e nteros positivos y cero
b) R = el conjunto de todos los enteros pares
e) R = {/en I n E Z, k un entero fijo positi,10}
d) R ={a+ bVZJa, b E Z}
<) R={a+bV2 + cV31aE Z,b,cEQ}
3. Sea (R, +,•)un anillo y a. b, c. d elementos de R. Establezca las condiciones (a partir de la
definición de un anillo) necesarias para demostrar los siguientes resultados.
a) (a + b)+c = b+(c+a) b) d + a(b + c ) =ab + (d+ac)
e) c(d+b)+ab = (a+c)b + cd d) a(bc)+(ab)d = (ab)(d+c)
4. Para el conjuntoRdelej~plo 14.4.conserveA B = A n B pero defina A +B =A U B. ¿Es
(R. U. n) un anillo?
s. Considere el conjunto Z junto con las operaciones binarias EB y 0 dadas en el ejemplo 14.3.
a ) Verifique las leyes asociativas para EB y 0 y la segunda ley distributiva para terminar el
trabajo iniciado en la parte (a) del ejemplo 14.3. [Esto establece entonces que (Z. EB. 0) es
un anillo.}
b ) ¿Es conmutativo este anillo?
e) El!- la pane (b) del ejemplo 14.3 mosuamos que O es el demento w,jdad de (Z, EB, 0).
¿Cuáles son las unidades de este anillo?
d) ¿Es este anillo un dominio de integridad? ¿Es un cuerpo?
6. Defina las operaciones binarias$ y 0cn Z comoxEB y=x + y - 1,x 0y= x + y- :hy, para
cualesquiera x, y e Z. Explique por qué ( Z, EB, 0 ) no es un anillo.
7. Sean k. m enteros fijos. Eocuentte todos los valores de k. m para los que (Z, EB, 0) es un anillo
con las operaciones binarias xEB y =x + y-k,x 0 y = x+ y - nuy. dondex, y e Z.
8 . La tabla 14.4(a) y (b) bacende (R. +. •)un anillo, donde R= (s.,, z, y}. (a) ¿Qiál e,d ettode
este anillo? (b) ¿Cuál es el inverso aditivo de cada elemento? (e) ¿A qu~ elemento equivale
t(.s + .xy)'? (d) ¿EsR un anillo conmutativo? (e) ¿TieneR un demento unidad? (0 Encuentre un
par de divisores de cero.
708 (.apítulo 14 Anillos y aritmética modular

lllbla 14.4

+ s t % y s t % y
s y % s t s y y % %
t % y t s t y y % %
% s t % y % % % % %
y t s y % y % % % %

(a) (b)

9. Defina la suma y producto. EB y 0, respectivamente. en el conjunto Qde la manera siguiente.


Paraa. b E Q.a elb =a+b+ 7.a 0b =a +b +(abrT).
a) Demuestre que (Q. EB. 0) es un anillo.
b) ¿Es conmutativo este anillo?
e) ¿Tiene este anillo un elemento unidad?¿~ ocurre oon las unidades?
d) ¿Es este anillo un dominio de integridad? ¿Es un cuerpo?
1 o. Sea (Q. e. 0) el cue,po donde el y 0 están definidos como
a(j,b=a+b-k, a0b=a+b+(ab/m),
para le, m(J:0) elementos fijos de Q. Determine el valor de k,y el valor de m en lo siguiente.
a) El elemento neutro del cuerpo es 3.
b) El inverso aditivo del elemento 6 es-9.
e) El inverso multiplicativo de 2 es 1/8.
11. SeaR = {a+bi I a. b E Z,i' = -1},conlasumayel prodUdodcfinidos por(a+b,) +(c + di)=
(a +e)+ (b +d) i y (a.+bf)(c +di) = (ac-bd) + (be +ad)i, respectivamente. (a) Verifique que
Res un dominio de íntcgridad. (b) Detennine todas las unidades de R.
12. a) Determine el inverso multiplicativo de la matriz(l ~] en el anilloM.P,,); es decir, encuen-
tre a. b, c,dtalesquc

[! ;J[: :J=[: :m ;J=[~ n


b) Muestre que[! i]es una unidad en el anillo M,(Q) pero oo en el anillo M,(Z).
13. Si[~ ,J. E Mi R) demuesuequc[~ ~] es una unidad de este anillo SJ y sólo si ad-bel- O.
14 . -™ un ejemplo de anillo con ocho elementos. ¿Qu6 ocurre si tiene 16 elementos? Generalice
los resultados.
15. Para R = {s. t, x. y}. defina+ y• mediante la tabla 14.S(a) para+ y la tabla parcial 14.S(b)
para •. Esto hace de R un anillo.

Ta~a 14.S

+ s 1 X y s t % y

s s t % y s s s
t t s y % s t ?
% X y s t % s t y
y y % t s y s ? ?
(a) (b)
14.2 Propiedades; y subestructuras de un anillo 709

a) Use las leyes asociativa y distributiva para determinarlas entradas que faltan en la tabla del
producto.
b) ¿Es conmutativo este aniUo?
e) ¿Ttene un elemento unidad? ¿Qué ocurre c.on las unidades?
d) ¿Es este anillo un dominio de integridad?. ¿o un cuerpo?

14.2
l'lopiedades y subestructuras
de un anillo
En cada uno de los anillos de la sección 14. l hablamos de el elemento neutro del anillo y
el inverso aditivo de cada elemento del anillo. Ahora vamos a demostrar. entre otras pro-
piedades, que estos elementos son 11nicos.

MA14.1 En cualquier anillo (R, +, ·),


a) el elemento neutro z es único, y _
b ) el inverso aditivo de cada elemento del anillo es único.
Demostración:
a) Si R tiene más de una identidad aditiva. sean z1 , z2 dos de tales elementos. Entonces
z/'= Z1 + Zz =~1-
Y.i, que Z: es una Y.l que z, es una
identidad aditiva identidad aditiva

b) Para a E R, supongamos que existen dos elementosb, e E R tales que a +b=b +a=
z y a+ e= e+ a= z. Entonces b = b + z = b +(a+ e)= (b +a)+ e= z +e = c.
(El lector debe indicar la condición que establece cada igualdad.)

Como resultado de la unicidad de la parte (b), desde este momento denotamos el inver-
so aditivo de a E R como -a. Así mismo. hablaremos de la resta en el anillo, donde
entendemos que a -b =a+ (-b).
También del teorema 14.l(b) obtenemos lo siguiente para cualquier anillo R.

MA 14.2 (Las leyes de cancelación para la suma) Para cualesquiera a, b, e E R,

a) a + b=a+c9b=c, y
b) b+a = c+a9b = c.
Demostración:
a) Como a E R, se sigue que -a E R y tenemos
a+ b =a + c9(-a) +(a + b) =(- a)+ (a +c)9
[(-a)+ a] +b =[(-a)+ a] +c9
z+b=z+c9b = c.
b) Dejamos esta demostración similar al lector.
710 Capitulo 14 Anillos y aritmética modular

Observe que al analizar la tabla de la suma para un anillo finito, vemos que cada ele--
mento del anillo aparece una sola vez en cada fila y columna de la tabla. Esto es conse-
cuencia directa del teorema 14.2, donde la parte (a) controla las filas y la parte (b) las
columnas.

TEOREMA 14.3 Para cualquier anillo (R.+.•) y cualquier a E R, tenemos a:= w = z.


o.mostn,dón: Si a E R. entonces a.z = a(z + t) ya que z + z == z. Por lo tanto. z + a.z =a.z s:
a,:+ az. (¿Por qut?) Usamos la ley de cancelación para la suma y tenemos que,= az.
La demostración de que za• z se hace de manera similar.

El lector podría pensar que el resultado del teorema 14.3 es obvio. Pero no estamos
trabajando conZ, Q oM,f_Z). Nuestro objetivo es demostrar que cualquier anillo satisface
dicho resultado, y para obtenerlo sólo podemos usar las condiciones señaladas en la defi.
nición de un anillo y las propiedades que hayamos derivado para anillos arbitrarios hasta
este momento.

La unicidad de los inversos aditivos [de la parte (b) del téorema 14.1] implica ahora el
siguiente resultado.

TEOREMA 14.4 Dado un anillo (R, +,·)y a, b E R,

a) -(-a)•a,
b) a(-b) • (-a )b • -(ab ), y
c) (-a)(-b)=ab.

Demostración:

a) Por el convenio establecido despuü del teorema 14.1 , - (-a) deno<a d inverso
aditivo de -a. Como (-a)-t a= z. a es tambi~n un inverso aditivo de - a. En conse-
cuencia, por la unicidad de tales inversos, - (- a)= a.
b) Demostraremos que a(-ó) = -{ab) y dejaremos la otra parte al lec1or. Sabemos que
-{ab) denocaelinverso aditivodeab. Sin embargo,ab +a H>) =a[b + (-ó)) =az=
,. por el teorema 14.3, y por la unicidad de los inversos aditivos, a (-ó) = -(ab).
e) Aquí estableceremos una idea que hemos utilizado en álgebra desde nuestro pri-
mer encuentro con los númuos con signo. "Menos por menos es igual a más"; la
demoslnleión se sigue de las propiedades y la definición de un anillo. De la parte (b),
tenemos que (-a)(-ó) = -{a(-ó)J =-{-{ab)J y el resollado se sigue de la parte (a).

Para la operación de producto también tenemos lo siguiente, que es comparable al teo-


rema 14.1.

TEOREMA 14.5 Para un anillo (R, +, •),


a) Si R tiene un clemcruo unidad. entonces es ó.nico, y
14.2 Propiedades y subestructuras de un anillo 711

b) si R tiene un elemento unidad y x es una unidad de R. entonces el inverso multipH-


cativo de x es único.
Demostración: La demostración de estos resultados se deja al lector.

Como resultado de este teorema. denotamos el inverso multiplicativo de cualquier unidad


x de R comox-1• Además, podemos volver a enunciar la definición de un cuerpo: Un cuerpo
es un anilloconmutativoK con elemento unidad, tal que para todox E F, x:/:. z =>r' E F.
Con este concepto como ayuda. examinaremos más propiedades y relaciones entre los
cuerpos y los dominios de integridad.

MA 14.6 Sea (R.+, •) un anillo conmutativo con elemento unidad. Entonces Res un dominio de
integridad si y sólo si, para cualesquiera a,. b, e E R, tales que a#: z. ab = ac => b = c. (Por
lo tanto, un anillo conmutativo con elemento unidad que satisfaga la ley de cancelación
del producto es un dominio de integridad.)
Demostración: Si R es un dominio de integridad y x. y E R. entonces xy = z => x = Z o y =
z. Ahora bien. si ab= ac, entoncesab-ac =a(b-c) = z, y, como a:/:. z, se sigue que b-c
= z o b = c. Recíprocamente, si R es conmutativo con elemtnto unidad y R satisface la
cancelación para el producto, entonces sean a, b E R con ab = z. Si a = t. hemos termina-
do; si no, como az = z, podemos escribir ab = az y concluir que b = z. Así, no ex.isten
divisores propios de cero y Res un dominio de integridad.

Antes de continuar, observemos que la ley de cancelación para el producto no implica


la existencia de los inversos multiplicativos. El dominio de integridad (Z, +, ·) satisface ]a
cancelación para el producto, pero sólo tiene dos elementos que son unidades (l y - 1). Por
lo tanto, un dominio de integridad no necesariamente es un cuerpo. ¿Pero qué ocurre con
un cuerpo? ¿Es necesariamente un dominio de integridad?

lIOREMA 14.7 Si (F, + , •) es un cuerpo, entonces es un dominio de integridad.


o.mostr.Kión: Sean a, b E X con ab = .z. Si a = z. hemos tenninado; si no, a tiene un
inverso multiplicativo <r1 pues X e s un cuerpo. Entonces
ab = z ⇒ a- (ab) =a- 1 z ⇒ (a-
1 1
a)b = a- 1 z ⇒ ub = z ⇒ b = z.
Por lo tanto, K no tiene divisores propios de cero y es un dominio de integridad.

En e l capítulo 5 vimos que Jas funciones/: A ➔A podían ser inyectivas (o sobre) sin ser
sobre (o inyectivas). Sin embargo. si A erajinito, tal función era inyectiva si y sólo si era
sobre. (Véase el teorema 5. 11.) La misma situación ocurre con los dominios de integridad
finitos. Un dominio de integridad no tiene que ser un cuerpo, pero si es finito, dicha estruc-
tura es un cuerpo.

TEOREMA 14.8 Un dominio de integridad.finito (D, +,•)es un cuerpo.


Oemc>strKi6n: Como Des finito, podemos enumerar los elementos de D como {d 1, di, . ..•
d.). Parad E D, si d f, z, tenemos dD = (dd, dd,, ... , dd. ) !:; D , puesto queD es cerrado
712 capitulo 14 Anillos y aritmética modular

con el prodocto. Ahora, 1D 1 ~ n y dD s;; D, por lo que si pudiéramos demostrar que dD


tienen elementos, tendríamos dD =D. Si I dD 1 < n, entonces dd,,= dd1 para algún 1 :s. i <
j :s. n. Pero como D es un dominio de integridad y d ;i! z, tenemos que d1 = d¡, aun cuando
habíamos supuesto que eran distintos. Así, dD = D y para algún 1 S k :S n, ddt = u, d
elemento unidad de D. Entoncesddt=u ~des una unidad deD y comod es un elemento
arbitario, se sigue que (D, +,·)es un cuerpo.

A panir de la demostración del teorema 14.8 podemos ver que al trabajar con los ele-
mentos distintos tk cero de un cuerpo finito, la tabla de multiplicación de tales elememos
es tal que cada elemento del cuerpo aparece exactamente en una vez en cada fila y una vez
e n cada columna.
En la siguiente sección veremos algunos cuerpos finitos que son útiles en la matemárx:a
discreta. Sin embargo, antes de cerrar esta sección. examinaremos algunos subconjUDIDI
especiales de un anillo.
En el capítulo 6, cuando trabajamos con las máquinas de estados finitos, vimos algunos
casos donde los subconjuntos del conjunto de estados internos daban lugar a máquinas pm
sí (cuando las funciones de estado siguiente y de salida de las máquinas originales se
restringían de forma adecuada). A estas máquinas las llamamos submáquinas. Puesto que
las operaciones binarias cerradas son un tipo especial de funciones, encontramos una idea
similar en la siguiente definición.

Definición 14.5 Para un anillo (R, +,•),un subconjunto no vacío S de Res un subanil/o de R si (S, +, -)(es
decir, S con la suma y producto de R restringidos a S) es un anillo.

Para cualquier anilloR, los subconjuntos {z} y R son siempre subanillos de R.

a) El conjunto de todos los de integridads pares es un subanillo de (Z, +, ·).De hecho,


para cualquier n E z•, nZ = {nx I x E Z } es un subanillo de ( Z, +, •).
b) ( Z, +,•)es un subanillo de (Q , +,•),el cual es un subanillo de (R , +,•),que es un
subanillo de (C, +, ·).

En el ejemplo 14.6, lossubconjuntos S = {s, w) y T= {s, u, x) son subanillos de R.

El siguiente resultado caracteriza los subconjuntos de un anillo que son subanillos.

TEOREMA 14.9 Dado un anillo (R, +, •), un subconjunto no vacío S de Res un subanillo de R si y sólo si

1) para lodos a, b E S, tenemos que a + b. ab E S (es decir, Ses cerrado con las
operaciones binarias de suma y producto definidas en R), y
14.2 Propiedades y subestructuras de un anillo 713

2) para todo a E li, -a E S.


Demostración: Si (S, +,•)es un subanillo de R, entonces satisface todas las condiciones de
un anillo. Por lo tanto, satisface las condiciones I y 2 del teorema. Recíprocamente, seaS
un subconjunto no vacío deR que satisface las condiciones 1 y 2. Las condiciones (a), (b),
(e) y (f) de la definición de anillo son heredadas por los elementos de S, puesto que tam-
bién son elementos de R. As(, lo que debemos verificar es que S tenga un inverso aditivo.
Ahora bien S i: S. por lo que existe un elemento a E S; y por la condición 2, -a E S.
Entonces, por la condición 1, z =a+ (-a) E S.

Consideremos el anillo(Z. E9, 0) que analizamos en el ejemplo 14.3 y el ejercicio 5 de la


sección 14.1. En este caso, x e y= x + y - 1 yx 0 y= x + y -xy. Consideremos ahora el
subconjunto S = { .. . , - 5, -3, -1, 1, 3, 5, .. . } de todos los enteros impares. Como, por
ejemplo, 3 y 5 están en S pero la suma ordinaña 3 + 5 = 8 ~ S, este conjunto S no es un
= =
subanillode (Z, + , -).Sin embargo, 3 $5 3 + 5-1 7 E S. De hecho, para cualesquiera
a, b E Stenemos a $b =a +b - 1, donde a+ bes par y a+ b-1 es impar, por lo que a e
b E S.Además,a 0b =a +b - ab,dondea +bes paryab es impar, por loquea 0b E - -
S. Por último, -a [el inverso aditivo de a en el anillo (Z , e, 0)1 es igual a 2 - a, que es
impar si a lo es. En consecuencia, sia ES entonces -a ES y por el teorema 14.9,S es un
subanillo de (Z, $, 0).

Observe que (Z•, +, •)satisface la condición l del teorema 14.9, pero no la condición 2,
por lo que no es un subanillo de (Z, +, •).
El resultado del teorema 14.9 puede expresarse también como se indica a continuación.

MA 14.1 O Para cualquier anillo (R, +, -), si 0 ;as ¡;;; R.


a) entonces (S, +, •)es un subanillode R si y sólo si para todos a. b ES, tenemos que
a-b E Syab ES;
b) y si Ses finito, entonces (S, +, •) es un subanillo de R si y sólo si para todos a, b E
S, tenemos que a+ b, ab E S. (De nuevo, la ayuda adicional proviene de una con-
dición de ser finito.)
Demostración: Se deja aJ ]ector.

El siguiente ejemplo demuestra la forma de utilizar la primera parte del teorema ante-
rior.

a.ijj;:il Consideremos el anillo R = M,('T)¡ y el subconjunto

de R. Cuando x =y= O, se sigue que [8 8] ES y S la 0. Así, ahora analizamos cualquier


714 Capitulo 14 Anillos y aritmética modular

par de elementos de S; es decir. dos matrices de la forma

[r:y r;r] [u~w u:w].


doade .:t, y, u, w E z. Tenemos que
(r- u)+(y- w)]
[r:y r;r ] - [,,:w u:w]=[(r-u):(:-w) x-u ,
por lo que Ses cerrado en la resta. Al pasar a la multiplicación, tenemos

r r+y][ u u+w]
[;c+y x u+w v

= [ru+(r+y)(u+w) r(u+w)+(r+y)u]
(r +y)u+r(u+w) (r+y)(u+w)+ru

- [X\>+xv+ yu+xw+ yw
- xu+yu+xo + xw
xu+xw +xo+ yu
xu+yu+xw+yw+xu
l
=[ ru+ru+yu+rw+yw (ru+ru.+yu+rw+yw)+(-yw)l
(xu+xu+yu+xw +yw)+(-yw) xu+.ru+yu+xw+yw '
por lo que S también es cerrado en la multiplicación.
Recurrimos entonces a la pane (a) del teorema 14.10 y tenemos que Ses un subanillo
de R. .

Ahora destacamos un tipo importante de subanillo.

Definición 14.6 Un subconjunto no vacío/ de un anilloR es un ideal de R si para todos a, b E /y todor E


R, tenemos que (a)a -b E /y (b)ar, ra E/.

Un ideal es un subanillo, pero el recíproco no siempre se cumple: (Z, +,•)es un subanillo


de (Q, +,•)pero no es un ideal, ya que, por ejemplo, (ln)9 lt Zaunque (1/2) E Q, 9 E Z.
Por otro lado, todos los subanillos del ejemplo 14.8(a) son ideales de (Z, +, ·).
Si observamos de nuevo el ejemplo 14.10, veremos que sia E S, x E Z.entonces a 0x=
a +x...:ax ( =x 0a) y sixes par(puesto que ya analizamos el caso en quexes impar en el
ejemplo 14.10), entonces a + x es impar y ax es par, de donde a + x - ax es impar. F.n
consecuencia. para t<Xlos a E S y x E Z, a 0 x y x 0 a están en S, por lo cual S es un ideal
del anillo (Z, al, 0).

EJERCICIOS 14.2 1. Complete las demostraciones de los teoremas 14.2, 14.4, 14.S y 14.10.
2. Sia, byc sondementos albittariosdeunanillo(.R,+, ·),demuestreque(a)a(b-c) =ab-(oc)=
ab- ac y (b) (b - c)a = ba-(ca) = ba-ca.
3. a) Si Res un anilJo con elemento unidad y a. b son unidades en R. demuestre que abes una
unidad de R y que (abt1 = b-1a-1•
14,2 Propiedades y subestructuras de un anillo 715

b) Parael anillo Mt.,Z). cncuenueA- , B"'\ (AB}- , (BA,' y S- A"' si


1 1 1

A-[1 ;]. B•[~ n


4 . Demuestre que una unidad en un anillo R no puede ser un divisor propio de cero.
5. Si a es una unidad en un anillo R, demuestre que - a también es una unidad en R.
6. Si a,b son unidades en un anillo R, ¿es a+ b necesariamente una unidad en R?
7. a) Encuentre todos los subanillos del anillo dado en la tabla 14.4.
b) ¿Es alguno de los subanillos de la panc (a) también un idean
8. a) Verifique que los subconjuntos S = {s, w} y T = {s, u, x) son subanillos del anillo R del
ejemplo 14.6. (Las operacioocs binarias para los elementos de S, T están dados en la tabla
14.3.)
b) ¿Son los subanillos de la parte (a) ideales de K!
9. Para R = Mt.,Z), sea Sel subconjunto de R ta1 que

s-{~ ;]l,,yez}.
Demuestre que Ses un subanillo de R.
1 O. Sea R = M,(L) y sea S el subcoojunto de R dado por

Demuestre que Ses un subanillo de R.


11. Sean S y T subanillos de un anillo R. Demuestre que S () Tes un subanillo de R.
12. Repita el ejercicio t 1, rccmplllando ..subanillo" con "'ideal".
1 3 . Sea (R, +,·)un anillo. Si S, T 1 y T1 son subanillosdeRy S !;;;; T1 U T2 , demuestreque S!:; T 1 o
S!: T,.
14. a ) Sea (R, +.·)un anillo finito, conmutativo, con elemento unidad u. Sir E R y r no es el
elemento neutro de R. dcmUCSII'C que res una unidad o un divisor propio de cero.
b ) ¿Siguen siendo válidos los resultados de la parte (a) si Res infinito?
15. a) ParaR=M,<Z),demuestreque

es un subanillo de R.
b) ¿Cuál es el elemento unidad de K!
e) ¿Tienc S un elemento unidad?
d) ¿Tiene S propiedades que Roo tenga?
<) ¿Es Sun ideal de R?
16. Sean S y T los siguientes subconjuntos del anillo R = Mt.,Z):

s-{[: ~]1•,b,cez}. r-{[t ~]l•,b,c,dez}.


a) Verifique que Ses un .subanillo de R. ¿Es un ideal?
b) Verifique que Tes un subanillo de R. ¿Es un ideal?
17. Sea (R, +, ·) un anillo conmutatívo, y sea .t el elemento cero de R. Para cada elemento fijo a E
R, definimosN(a) = {r E RIra =z}. Demuestre que N(a) es un ideal de R.
716 Capitulo 14 Anillos y aritmética modular

18. SeaR un anillo conmutativo con elemento unidad u y sea /un ideal de R. (a) Si u E l. demues-
tre que J = R. (b) Si/ contiene una unidad de R, demuestre que/= R.
19. Si R es un cuerpo, ¿cuántos ideales tiene?
20. Sea (R, +,•)el anillo (finito) conmutativo con elemento unidad, dado porlas tablas 14.6(a) y(bi

Tabla 14.6

+ z u a b z u a b
z z u a b z z z z z
u u z b a u z u a b
a a b z u a z a b u
b b a u z b z b u a
(a) (b)

a) Verifique que R es un cuerpo.


b) Encuentre un subanillo de R que no sea un ideal
e) Sean.r,. y incógnitas. Resuelva el siguiente sistema de e.cuaciooes lineales en R: bx. + y =11;
x+by=z.
21. Sea R un anillo conmutativo con elemento unidad u. .
1) Par1. cualquiera E R (fijo), demuestrequcaR = {arl re R} es un ideal de R.
b) Si los tínicos idca1cs de R son {d y R, demuestre que Res un cuerpo.
2 2. Sean (S, +, ·)y (T, +', -')dos anillos. Si R:S x T, definimos la suma ..~.. y eJ produa:o ..0"
como

(s1,t1)@(.t-J,t2) = (s1 +s2,l1 +'t2),


(s1, t1) 0 (s2,t2) = (s, ·s2,t1 ·' ti),

a) Demuesire que con estas operaciones binarias cerradas, R es un anillo.


b) Si S y T son conmutativos. demuescrc que Res conmutativo.
e) Si S tiene elemento unidad Us y Ttiene elemento unidad u 7, ¿cuál es el elemento unidad deR'!
d) Si Sy Tson cuerpos, ¿es R tambi~n un cuerpo?
23. Sca(R,+,·)unanilloconclementounidaduy l R I =8.ParaR'=R XR XR x R,definimos
+ y • como lo indica el ejercicio 22. En el anillo k'. (a) ¿cu1ntos elementos tienen exacwneme
dos componentes distintas de cero? (b) ¿cuántos elementos tienen componentes d.istinw de
cero? (e) ¿existe un elemento unidad? (d) ¿Cl.Wltas UJUdades existen si R tiene cuatro unidades?
24. Consideremos el anillo (Z3, e, 0) donde definimos la. suma y eJ producto como (a, b, e) e
(d, ,,f) = (a+ d, b +e, e+ f) y (a, b, e) 0(d, , ,/)=(ad, be, cf). (En eslc caso, por ejemplo,
a + d y ad se calculan mediante las operaciones binarias normales de suma y producto en Z.)
SeaSel subconjuntodeZ3,doodeS= {(a. b, e) l a=b +e) . OcmuestrequeSnocs unsubanillo
de (Z', EB, 0).
25. ScaR=Z x Z X Q. Defina las operaciones binarias de suma(+) y multiplicación ( • ) paraR
de la forma siguiente: Para (a, b, q), (e, d, r) E R. (a, b, q) + (e, d., r) =(a+ e, b + d, q + r) y
(a. b, q) • (e, d. r) = (ac, bd. qr). (F.n este caso, las sumas a+ e, b + d denotan la suma oorma1
de enteros y q + r denota la suma de números racionales; los productos ac, bd denocao b
multiplicación común de enteros y qr el pnxlucto de números racionales.) En forma similar al
resultado del ejercicio 22, (R, +,•)es un anillo con estas operaciones binarias.
a) Dt ejemplos de cuatro divisores propios de cero en R.
b) Caracterice cuándo un elemento (m, n, s) de R es un divisor propio de cero.
e) Dt ejemplos de cuatro unidades de R.
d) Caracterice cuándo un elemento (m, n.. s) de Res una unidad.
14.3 los enteros módulo n 717

26. Sea (R, +, •) u.o anillo y a E R. Definimos Oo = z. la= a y (n + l)a •na+ a, para cualquier
• e z·. (Aqul eswnos multiplicando los demontosde R porelc..-os de z. por lo que ahora
tenemos una nueva operación diferente del prowctom Z o en R.) Paran> O, definimos (-n)a •
n(-.,), as! que, por ejemplo, (- 3)a = 3(-.,) • 2(-.,) +(-.,) = [(-.,) +(-.,)]+(-.,)=[-(a +a)]+
(-.,) •-{(a+ a) + a] • -{2a + •J = -(la).
Para cualesquiera a, b E R y cualesquiera m. n E Z, demuestre que
•> ma +no • (m + n)a b) m(no) = (mn)a
e) n(a + b)-no + nb d) n(ab)=(na)b • a(nb)
<) (maXnb) = (mnXab) • (11a)(mb)
27. a) Parad anillo (R, +,•) y aialquiera E R.definimosd"' •aya-• 1 ~a-a para todo ne z•.
Demuesttt que paracualesqui0111m, • e z·, (a")(a') .,.. •• y (a")" =~-
b) ¿Puede usted indicarcómodefiniraOoe', n e Z+, incluyendo las condiciones que necesa-
riamenteR debe satisfacer para que estas definiciones tengan sentido?

14.3
Los enteros módulo n

¡Por el momento tenemos suficiente teoóa! Ahora nos conceQJ:raremos en la consttuccióo


y el uso de anillos y cuerpos finitos especiales.

Sea n E Z♦• n > l. Para a. b E Z. decimos que a es congruente con b mótht/o n, y


escribimos a ., b(modn), sin 1(a-b); o, en formaequivalente,a = b +.bi para alg(ink E Z.

Vemos que (a) 17 = 2(mod 5); (b)- 7 • -49(mod 6).

La cooa;ruencia módulo n es una relación de equivalencia sobre Z.


Oemoruo<i6n: La demostración se deja al lector.

Como uoa relación de equivalencia ~ un conjunto induce una partición de ~e.


paran ~ 2, la congruencia módulo n divide a Z en las n clases de equivalencia
[O) = { ... , -2n, -n,O,n,2n, .. .) ={O+ nx lx EZ}
[!) = { .. . , -2n + 1, -n + 1, 1,n + 1,2n + !, . ..} = {! + nxlx E Z}
[2) • { .. . , -2n + 2, -n + 2,2,n + 2,2n + 2, ...} ={2 + nx lx EZ}

[n -1) •{. . . , -n -1, -l,n - l,2n - 1,3n -1, ...}


• {(n - 1) + nxlxE Z}.
P:n cualquier z e Z, por el algoritmo de la división (de la sección 43), podemos
escribir t = qn + r, donde O :S r < n, por lo que t E [r], o [t] = [r]. Usamos la notación z.
718 capítulo 14 Anillo~ y aritmética modular

para denotar {[O], (l], [2), ... , [n - !]]. (Cuando no haya peligro de ambigüedad, CXlO
frecuencia reemplazaremos [a] con a y escribiremos z. =e {O, 1, 2, ... , n - l }. Nuestro
objetivo es definir operaciones binarias cerradas de suma y multiplicación sobre el con-
junto de clases de equivalencia Z., de modo que obtengamos un anillo.
Para [aj, [bJ E z.,
definimos+ y· como

[a]+[b]=[a+b] [a} · [b] = [a][b} = [ab}.


Por ejemplo, sin= 7, entonces [2] + [6) = [2 + 6) = [8] = [IJ, y [2) [6) = [12) = [5).
Antes de aceptar con rapidez estas definiciones, debemos analizar si estas operaciones
están bien definidas, en el sentido deque si [a] = [e] y [b] = [dJ, entonces [aj+ [b] = [e]+
[d] y [al[b] = [e][d). Como [aj= [e] puede ocurrir sin que a ¡l,c, nos preguntarnos si los
resultados de nuestra suma y producto dependen de 1os representantes elegidos de las
clases de equivalencia. Demostraremos q ue los resultados de las dos operaciones no de-
penden de la elección de los representantes de las clases y que las operaciones están biea
definidas.
En primer lugar, observemos que [al= [e)=> a =e +sn para algúns E Z y [bJ = [d] =>
b ; ; ; d + rn, para algún 1 E Z. Por lo tanto,

a + b = (e +sn) + (d + tn) =e +d+ (s + t)n,

de modo que (a+ b) = (e + d)(mod n) y [a+ b] =[e+ d ). También se cumple

ab =(e+ sn)(d + tn) = cd + (sd +et+ stn)n

=
y ab cd(mod n), o [abJ = [cd].
F.ste resultado nos conduce al siguiente.

TEOREMA 14.12 Paran E z+. n > l. Z ,. es un anillo conmutativo con elemento wúdad igual a [1) en Jas
operaciones binarias cerradas definidas antes.
Demostración: Se deja al lector. La verificación de las propiedades del anillo se sigue de
las definiciones de suma y producto de z . y de las propiedades correspondientes del
anillo (Z. +, •).

Antes de enunciar más resultados. veremos dos ejemplos paniculares. ~ y ~ En las


tablas 14.7(a) y (b), y 14.8 (a) y (b), escribimos a en vez de [a].

Tabla 14 .7

Zs + o 1 2 3 4 o 1 2 3 4
o o 1 2 3 4 o o o o o o
1 1 2 3 4 o 1 o 1 2 3 4
2 2 3 4 o 1 2 o 2 4 1 3
3 3 4 o 1 2 3 o 3 1 4 2
4 4 o 1 2 3 4 o 4 3 2 1
(a) (b)
14.3 Los enteros módulo n 719

labia 14.8

+ o 1 2 3 4 5 o 1 2 3 4 5
o o 1 2 3 4 5 o o o o o o o
1 1 2 3 4 5 o 1 o 1 2 3 4 5
2 2 3 4 5 o 1 2 o 2 4 o 2 4
3 3 4 5 o 1 2 3 o 3 o 3 o 3
4 4 5 o 1 2 3 4 o 4 2 o 4 2
5 5 o 1 2 3 4 5 o 5 4 3 2 1
(a) (b)

En ~. todo elemento distinto de cero tiene un inverso multiplicativo, de modo que Zs


es UD cuerpo. Peropara~ l y 5 son las únicas unidades y2. 3,4 son divisores de cero. Por
ocro lado, en ~ 3 • 3 = 3 • 6 = O. así que para que Z., n > 1. sea un cuerpo, se necesita más
que UD módulo impar.

MA 14.13 Z. es un cuerpo si y sólo si n es primo.


Demostración: Sean un primo y supongamos que O<a< n. Entoncesmcd(a. n)= 1, por lo
que (de laSec. 4.4) sabemosqueexisten enteross, t la.les que as +m = l,Así,as = l (mod
n), o [a][s] = [ l) y [a] es una unidad de Zr el cual es por lo tanto un cuerpo.
Recfprocamente, sin no es primo, entonces n =n1n,, donde l < n 1, ~ < n. Así, [n1] ~ [O]
y [n,J +[O] pero [nJ[n,J = [n1n,] = [O], y Z, ni siquiera es un dominio de integridad, por lo
que no puede ser un cuerpo.

En Z., [5] es una unidad y [31 es un divisor de cero. Buscaremos la fonna de reconocer
cuándo [a] es una unidad en Z., si n es compuesto.

noREMA 14.14 En Z., [a] es una unidad si y sólo si mcd(a, n)= l.


Demostración: Si mcd(a, n) = 1, el resultado se sigue como en la demostración del teorema
14.13. Para el recíproco, sea [a] E Z. y [a)"' = [s]. Entonces [as) = [a)[s] = [1], por lo que
as"' l(mod n)y as= 1 + m, para algún I E Z. Pero 1 =as +n(-t) ~mcd(a, n) = l.

Encuentre [25J-1 en Z,,.


Como mcd(25, 72) = 1, el algoritmo de Euc lides implica

72 = 2(25) + 22, 0<22 < 25


25 = 1(22) + 3, 0<3<22
22 = 7(3) + ! , 0<!<3.

Como l es el último resto distinto de cero, tenemos que

1 = 22 - 7(3) = 22 - 7[25 - 22] = (- 7)(25) + (8)(22)


= (- 7)(25) + 8(72 - 2(25)l =8(72) - 23(25).
720 capítulo 14 Anillos y aritmética modular

Pero

1 = 8(72) - 23(25) =} 1 ~ (- 23)(25) ~ (-23 + 72)(25)(mod 72),


por Jo que (J] = (49][25] y [25J-• = (49] en Z,,.

Ahora bien. [25] es una unidad en Zn. pero ¿existe una forma de saber cuántas unida-
de s tie ne este anillo? Del teorema 14.4. si 1 :S a< 12. entonces [a J- 1 existe si y sólo si
mcd(a, 72) ~ l . En consecuencia, el número de unidades en Zn es el número de enteros•
tales que I s a< 72 y mcd(a, 72) = l. Usando la función phi de Euler (Ej. 8.5), veremos
que este valor es
4>(72) = 4>(23 32) = (72)[1- (1/ 2)][1 - (1 / 3)) = (72)(1/ 2)(2/ 3) = 24.

En general, paran E z•, n> 1,existen4>(n) unidades yn- l -$(n)divisores propios de


cero en z•.

El último ejemplo de esta sección presenta una aplicación a la recuperación de infonnacióa.

Cuando se busca en una tabla de registros guardados en un computador, cada registro tiene
asignada una posición de memoria. o dirección, en la memoria del computador. El propio
registro está formado a su vez por cuerpos (que no tienen nada que ver con las estructuras
de anillo). Por ejemplo, Una sección escolar mantiene un registro de cada estudiante; cada
registro contiene información acerca del número de la seguridad social, el nombre y d
área de especialidad del estudiante, lo que representa un total de tres cuerpos.
Al buscar el registro de un estudiante particular, podemos usar su número de seguridad
social como laclave para el registro, puesto que ésta identifica en forma única ese registro.
Como resultado, desarrollamos una función del conjunto de claves al conjunto de direc--
ciones de la tabla.
Si la escuela es lo suficientemente pequeña, es posible que Jos primeros cuatro dígitos del
número de seguridad social basten para la identificación. Desarrollamos una función de
disper,i6nhdel conjunto de claves (que siguen siendo los números de la seguridad social)al
conjunto de direcciones, determinada ahora por los primeros cuatro dígitos de la clave. Pcx
ejemplo, h(OSl-37-649S) identifica el registro que está en la dirección asociada con 0813.
De esta forma podemos guardar la tabla con un máximo de 10,000 direcciones. Todo estara
bien mientras h sea inyectiva. Si un segundo estudiante tiene como número de la seguridad
social 081 •39-()207, entonces h ya no identificaría en forma única el registro de un estudian-
te. Cuando esto pasa, se dice que ocurre u.na colisión. Puesto que el incremento del tamaño
de la tabla guardada produce por lo general un mayor espacio de almacenamiento no utiliza.
do, debemos comparar el costo de este espacio con el costo de1 control de colisiones. Se han
diseñado técnicas para resolver colisiones, las que dependen de las estructuras de datos (como
ve.ctores o listas lineales enlazadas) que se usan para guardar los registros.
Algunas de las diferentes funciones de dispersión que se han diseñado son las siguientes.
a) El mitodo de la división: Á.quf restringimos el número de direcciones que puedan
utiliz.arse hasta un entero fijo n. Para cualquier clave k (un entero positivo). defini-
=
mos h(k) = r, donde r k(mod n) y O s r < n.
14.3 los enteros módulo n 721

b) El método de plegado se implementa con frecuencia; en éste, la clave se separa en


dos partes, las cuales se suman para obtener h(clavc). Por ejemplo, h(081-37-6495) =
081 + 37 + 6495 = 6613 utiliza el plegado; si sólo queremos direcciones con tres
dígitos, podemos suprimir el primer dígito 6 y tener h(081-37-6495) = 613.
No es posible hacer suficiente énfasis en cuanlO a la imponancia de elegir una función
de dispersión adecuada, si al mismo tiempo intentarnos mejorar la eficiencia en términos
de uoa mayor velocidad y menos espacio de almacenamiento utiliza.do.
Con el concepto de módulo, podemos desarrollar una función de dispersión h, usando
la misma clave anterior. donde

con
Y1 2 x1+ x2 +x3 (mod5), Osy1 <5
y, • r,+r, (mod3), Osy,<3
y,-x.+r, +r,+r, (mod7), Os y,<7.

En este caso, por ejemplo, h(081-37-6495) = 413.

1. Enumere cuatro elementos en cada una de las siguientes clases de equivalencia


•) [IJ en Z, b) [2] en z., <)110] en Zn
2 . Demuestre que si a. b. c. n E Z con a. n :> O y b = e (mod n), entooces ah = ac(mod an).
3. Scana, b, m, n EZconm, n:>O. Demuesucquesia = b(modn)ym ln. cotoncesa = b(modm).
4. Demuestre que para todo entero n, exactamente u.no de los enteros n, 2n -1 y 2n + 1 es
divisible entre 3.
5. Sin E z• y n > 2, demuestre que

•-• . { O (mod n), n impar


l;•= 1 (modn), n par

6. Complete las demostraciones de los teoremas 14.11 y 14.12.


7. Defina la relación 9'l en Z .. como a9'i b si t(a) = t(b), donde 't(a) es el número de divisores
(enteros) positivos dea. Por ejemplo, 29i 3 y 491: 25 pero 5~9.
a) Verifique que 9t es una relación de equivalencia en z•.
b ) Para las clases de equivalencia [a] y [b] inducidas por$, defina las operaciones de suma y
producto como [a]+ [b] =[a+ b] y [a][b] = [ab]. ¿Están bien definidas estas opc:racioncs?
Es decir, ¿al,! b, b91 d=> (a+ b)91 (e+ d),(ab)!?/! (cd))?
8. Encuentre el inverso multiplicativo de cada eJemcnto de Z 11, Z 0 y Z 11•
9. Encuentre [at' en z,,., para (a) a= 17, (b) a= 100, y (e) a = 1n.
1 O. a ) Encuentre todos los subanillos de Z11, Z11 y Z:i..
b) Construya el diagrama de Hasse para cada una de estas colecciones de subanillos, donde el
orden parcial surge de la inclusión de conjuntos. Compare estos diagramas con los del
conjunto de divisores positivos den (n = 12; 18; 24), donde el orden parcial proviene ahora
de la relación de divisibilidad.
e) Encuentre la fórmula para el número de subanillos en Z., n > l.
722 capitulo 14 Anillos y aritmética modular

11. ¿Cuántas unidades y cuántos divisores (propios) de cero tiene (a) Z n? (b) Z 111? (e) Z u111
12. Demuestre que en cuaJquier lista den enteros consecutivos, uno de ellos es divisible entre a.
13. Si se eligen tres enteros distintos a1 azar del conjunto (1, 2, 3 , ... , 1000},¿ruil es la pn:>b&
lidad de que su suma sea divisible entre 37
=
14. a ) Pan. e, d. m,. n E Z con n > J y m > O, demuestre que si e d(mod n), entonces me al =
(modn) y<"= <f'(mod n).
b) Si x..x~-i• .. x,.to =x. • Hr + · · · +x1 -10 +x.odenotaun entero den + l dígitos, dcmUCSR
entonces que
x,.x,,-, ... x,.ro-x,. +z.,-1 + · · · + X1 + Xo (mod9).
15. a) Demuestre que paratodon E N, JO'= (-J)"(mod JI).
b) Considere el resultado para mod 9 en la pane(b) del ejercicio 14. Enuncie ydemucstrcm
resultado análogo para mod 11.
e) Para los números siguientes, determine el valor del dígito d de modo que el resultado sea a
entero divisible entre 11: (i) 2d6S3874~ (ü) 37d64943252.
16. Delermine el último dígito en 3".
17. ¿Cuál es el último dígito de 9 19ti?
18. Para p primo, determine todos los elementos a E Z, tales que a 2 = a.
19. Para a. b, n E Z .. y n > 1, demuestre que a = b(mod n ) ⇒ lllcd(a. n) = mcd(b, n).
20. a) Muestre que para cualquier (a] E 'L,, ,i (a] 1'(0], entonces (a]' = J.
b) Sean E z· con mcd(n, 7) = J. Demuestre que 7 l(n'- 1).
21 . Para la función de dispersión que aparece al final del ejemplo 14.14, encuentre (a) h(I23-04-
=
2275); (b) un número de la seguridad social n tal que h(n) 413, lo cual provoca una colisiól
con el nómero 081.37-649S del ejemplo.
22. Escriba un programa(o desarrolle un algoritmo) para implementar la función de dispersión de:!
ejercicio 21.

14.4
Homomorfismos e isomorfismos
de anillo

En esta última sección examinaremos las funciones (entre anillos) con propiedades espe--
ciales que dependen de las operaciones binarias cerradas en los anillos.

Consideremos los anillos(Z,+, •)y (Z., +, •),donde la suma y el productodeZ.se definea


como en la sección 14.3.
Definimos/: Z ➔ Z. como/(x) = [x). Por ejemplo,/(!) = [IJ = [7) =/(7) y /(2) =/(8) =
/(2 + 6.t) = [2], para cualquier k E Z. (Así,/ es sobre pero no inyectiva.)
Para 2, 3 E Z,/(2)= [2),/(3) = [3] y tenemosque/(2 + 3) =/(5) = [5] = [2] + [3] = /(2)+
/(3) y /(2 · 3) = /(6) = [O] = [2][3] = /(2) ·/(3).
De hecho, para cualesquiera x , y E Z,

f(x +y)= [x +y] = [x] +(y] = /(x) + / (y),


! !
Suma en Z suma en z.
14.4 Homomorfismos e isomorfismos de anillo 723

y
f(x ·y)= [.ry] = [x][y] = f(x) · f(y).
t t
Producto en Z Producto en Z.

Este ejemplo nos lleva a la siguiente definición.

14.8 Sean (R, +, •) y (S, EB. 0) anillos. Una funciónf: R ➔ Ses un homomorfismo de anillbs si
para todos a. b E R,

a) f(a + b) = f(a)lif)f(b),
b) f(a · b) = f(a)0f(b).

Se dice que esta función preserva las operaciones de anillo por las siguientes razones:
coosideremosf(a +b) =f(a) EB/(b). Sumando primero a. b ,n R y encontraododespués la
imagen (mediante/) de esta suma, obtenemos el mismo resultado que cuando determina-
mos las imágenes (mediante/) en S de a. by luego sumamos las imágenes en S. (Por lo
tanto, la operación de función y las operaciones aditivas conmutan entre sí.) La misma
observación también se cumple con las operaciones multiplicativas en los anillos.
Para los anillos z. y Z. definimos la función f: z, ➔ z, como ./{[al) = [a]'(= [a']).
Entonces, para todos [a], [b] E Z., tenemos

f([a][b]) = /([ab]) = [ab ]' = ([al[b])' = [af[bf = f([a])f([b]).


t t
ProductOffl4 ProductOffl ~

En consecuencia, esta función/ preserva las operaciones de multiplicación en los anillos.


= =
Sin embargo, para [l), [2] E Z., vemos que/([1] + [2)) =/((3)) [3)' [l], mientras que
= =
/([l]) +/([2]) [1)' + [2)'= [l] + [4) [5) (#[l] enZ,.)Así.fnopreserva las operaciones
aditivas en los anillos, por lo cual/ no es un homomorfismo de anillos.
La función g: z.
➔ Z. dada por g([a)) = 3[a], preserva las operaciones aditivas, pero
no las multiplicativas, e n los anillos.

ºción 14.9 Sea/: (R. +, •) ➔ (S, E!l. 0) un homomoñismo de anillos. Si fes inyectiva y sobre,
entonces/es un isomorfismo de anillos y decimos que R y S son anillos isonwrfos.

Podemos pensar que los anillos isomorfos surgen cuando nos referimos al "mismo"
anillo en dos idiomas diferentes. La función/ proporciona entonces un diccionario para
traducir sin ambigüedad de un lenguaje a otro.
Los términos "homomorfismo.. e ..isomorfismo" provienen del griego, donde rrwrpho
se refiere a la forma o estructura, homo significa similar e iso significa idéntico o igual.
Por lo tanto. podemos pensar que los anillos homomorfos son aquellos que tienen una
estructura similar y los anillos isomorfos son réplicas (abstractas) de la misma estructura
724 Capítulo 14 Anillos y aritmética modular

En la definición 11.13 definimos el concepto de isomorfismo de grajos. Establecimos


entonces que los grafos no dirigidos G1 = (V1o E1) y G2 = (V2• Ez.) son isomorfos si podemos
encontrar una función/: V1 ➔ V2 tal que

a) / sea inyectiva y sobre, y


b) {a, b} E E1 si y sólo si [f(a),/(b)) E E-,.

A la luz de nuestras observaciones acerca del isomorfismo de anillos, otta forma de pensar
la condición (b) es que la fuociónfpreserva las estructuras de los grafos no dirigidos G,y
G2. Cuando I V1 1= 1V2 1, no es difícil encontrar una función/: V1 ➔ V2 que sea inyectiva
y sobre. Sin embargo, para un conjunto dado V de vénices, lo que determina la estructura
de un grafo dirigido G = ( V, E) es su conjunto de arislas (donde se definen las adyacenci>sdt
los vértices). Por lo tanto. una correspondencia biyectiva/: V1 ➔ V2 es un isomorfismo
de grafos si preserva la estructura de G, y G2 preservando las adyacencias de los vértices.

Ejemplo 14.16 Para el anillo R del ejemplo 14.5 y el anillo Z,, la fuoción/: R ➔ z, dada por

f(a) = [O], /(b) = [!], /(e)= [2], /(d) = [3], /(e) = [4]

es un isomoñismo de anillos.
Porejemplo,/(c + d) =/(a)= [O]= [2] + [3] + /(c) + /(d), mienttasque/(be) =fl.• ) = [4)=
[1 ][4] =/(b)/(e). (Como no disponemos de otros métodos y teoremas, hay que verificar
25 igualdades de este tipo para que se preserven las operaciones binarias.)

Puesto queexisten5! = 120 funciones biyectivasdeR en ~.¿existe algo que nos ayude
a determinar si una de estas funciones es un isomorfismo? El ejemplo 14.16 da lugar al
siguiente teorema. el cual nos proporciona formas para al menos comenzar a determinar si
las funciones entre anillos pueden ser bomomorfismos o isomorfismos [Las partes (e) y
(d) de este teorema se basan en los resultados de los ejen:icios 26 y 27 de la sección 14.2.J

TEOREMA 14.15 Si/: (R, +, •) ➔ (S, $ , 0 ) es un homomorfismo de anillos, entonces


a) /(W = Zs, donde z.t, Zs son los elementos neutros de R, S, respectivamente;
b) /(-a) = -/(a) para cualquier a E R;
e) /(na) = n/(a), para cualquier a E R, n E Z;
d) /(a')= [f(a))' , para cualquier a E R, n E Z•; y
e) si A es un subanillo de R, entonces/(A) es un subaniUo de S.
Demostrac.ión:

a) z, $ /(z,.) = /(z.) = /(z,. + z.) = /(z.) El)/(z.). (¿Por qué?) Asf, por la ley de cancela-
ción para la suma. tenemos que /(z.) = z,.
b) z, =/(z.) =/(a+(-a))=/(a) $/(-a). Como los inversos aditivos deSson únicos y
/(- a) es un inverso aditivo de /(a), se sigue que /(-a) = -/(a).
e) Sin = O, entooces/(na) =/(z,.)=z,=nf(a). El resultado también es cieno paran = 1,
por lo que suponemos que la propiedad es cierta para n = k (.!: 1). Por induccióa
matemática, examinamos el caso en que n = k + l. Por los resultados del ejercicio
14.4 Homomorfisrnos e isomorf"tsmos de anillo 725

26 de la secxión 14.2, obtenemos/((k+ l)a) = /(ka+ a) =/(ka) fB/(a) • kf(a) e


/(a) (¿por qué?) = (k + 1)(/(a)) (¿porqué?). (Nota: En este caso tenemos tiu tipos
diferentes de sumas.)
Cuando n > 0,/(-M) = -nf(a). Esto se sigue de nuestra demostración anterior
por inducción, de la parte (b) de esta demostración y de la parte (b) del teomna 14.1,
puesto que/(-na) +/(na)= /(n(-a)) +/(na) = nf(-a) + n/(a) = n[f(-a) + /(a))=
n[- /(a) +/(a)] = nt, • t,. Por lo tanto, obtenemos el resultado para todo n E Z.
d) Dejamos la demostración de este resultado al lector.
e) ComoA ¡la f,/(A) ¡la e. Si.i, y E /(A), entoncesx =/ (a), y= /(b)paraalgunos a, b E
A. Entooces x fB y=/(a) fB/(b) =/(a+ b), y x 0 y=/(a) 0/(b) = /(ab), con a + b,
ab EA (¿por qué?), porloquex(By,x0y E/(A).Aslmismo, six E/(A),entonces
x = /(a) para algún a E A. Así, tenemos /(-a)= -/(a)= -x y como - a E A (¿por
qué?), tenemos - x E /(A). Por lo tanto,/(A) es UD subanillo de S.

Cuando el homomorfismo es sobre, obtenemos el siguiente resultado.

14.16 Sí/: (R, +, 0) ➔ (S,e, 0)es UD homomorfismo de anillos deR sob~ s, donde 1s1 > 1,
entonces
a) si R tiene elemento unidad "'•• entonces / (u.) es el elemento unidad de S;
b) si R tiene elemento unidad"'• y a es una unidad enR, entonces/ Ca) es UDa unidad en
S y f(tr 1) = (/{a)J- 1:
e) si R es conmutativo. entoDCCS Ses conmutativo; y
d) si / es UD ideal de R, entoooes /(/) es UD ideal de S.
Demostración: Demostraremos la pane (d) y dejaremos e l resto al lector. Como / es un
subanillo de R, se sigue que/(/) es un subaoillo de S por la pane (e) del teorema 14.15.
Para verificar que/(/) es UD ideal, seax E/(/) y s E S. Entonces x = /(a) y s =/(r) para
alguoosa E/, rE R.Asl,s0x=/(r) 0/(a)=/(ra), con ro E /y tenemos ques0x E
/(/). En forma análoga, x 0 s E /(/), por lo que/(/) es UD ideal de S.

Estos teoremas refuerzan la forma en que los bomomorfumos e isomorfismos preser•


van la estructura. Pero ¿qu6 uso tieoen estas f'w>ciones, además de usarse para demostrar
más teoremas? Antes de terminar, consideremos otros dos ejemplos.

Sean C el cuerpo de los números complejos y Sel anillo de las matrices reales 2 x 2 de la
forma[_¡; !): definimos/: C ➔ S como /(a + b,) = [ -b!],para a + bi E C. Entonces

por lo que/ es una función inyectiva. Tambi6n es sobre. (¿Por qui?)


726 Capítulo 14 Anillos y aritmética modular

Adenw;,
f((a + bi) + (x + yi)) =f((a + x) + (b + y)i)

a +x
= [ - (b+y)
b+y] [ ª bl
a + x = -b a
+ [ .r
-y
y]
X

= f(a + bi) + f(x + yi),

f((a + bi)(x + yi)) = f((ax -by) + (bx + ay)i)


- [ ax-by bx + ay]-[ a
- - (bx + ay) ax - by - -b a
b][ -yx y]x
= f(a + bi)f(x + yi),
por lo que/es un isomorfismo de anillos.
Pero ¿dónde podemos usar este isomorfismo?
Supongamos que debemos encontrar el valor del número complejo

. (5.2-7.li)'
(-8.3+9.91)+ (l.3+3.7i)''

Si disponemos de un computador con una implementación de BASIC que incluya open-


ciones con matrices, entonces. usando este isomorfismo, podríamos traducir el problema
de números complejos, no implementados en BASIC, en un problema de matrices qae
puede controlar el computador. Aquí buscamos el resultado

-8.3 9.9] + [5.2 -7.1]'([ 1.3 3.7]') - '.


[ -9.9 -8.3 7.1 5.2 -3.7 1.3

Con el programa en BASIC de la figura 14.1. encontramos }a respuesta (con dos cifras
decimales) al problema de aritmética compleja como 8379.98 + 15122.7i.

Ampliaremos la idea desarrollada en el ejercicio 22 de la sección 14.2; sea R el anillo


Z, X z, z,.
x Entonces IR 1 = 1Z, 1 • 1 z, y
1 · 1Z, 1 = 30 las operaciones de suma y
producto en R se definen como sigue:
Para cualquier (a., a,, a,), (b., b,, b,) E R donde a., b, E Z,, a,, b, E Z, y o,, b, E Z,,

(a1, a2, a3) + (bi, bi, b1) = (a1+ b1, ªz + bz, a1 + b3)
t t t 1
Suma en R Suma Suma Suma.
en~ en Z, en Zs

(a.,a,,~(b,,b,,~,) = (a, i-b.,~b,,a, t!;,>·


Producto en R Producto en Z1 Producto en ZJ Producto en ~
14.4 Homomorfismos e isomorfismos de anillo 727

10 Dim A(2,2 ) . 6 (2,2). C(2,2 ). 0( 2 , 2 ), E( 2.2 ), F(2,2 ) ,


G( 2.2 ). H( 2.2 ). K( 2.2 ). 11(2, 2 )
20 Kat Read A. E. K
30 Data l.3,3.7,-3.7,l.3.5.2,-7.1.7.1.5. 2 .-8 . 3 . 9.9.
-9.9, - 8 . 3
40 ICat B•A
50 For I 2 l To 3
60 ltat C ., A• B
70 llat A=C
80 Next I
90 llat O• lnv( A)
100 llat F • E
110 For J • l To 6
120 Mat C=E•F
130 llat E•G
140 Nex t J
150 Kat H • O• E
160 llat ll • K + H
170 Print ' 1 La solución al problema es 11 ; 11( 1,1 ) ;
"+ " ; 11 ( l.2 ); "1"
180 End
La soluci6n al problema es 8379.98 + 15122.7 i
Figun1 14.1

Definimos la función/: Z,. ➔ R comoj{x) s (.,,.x,,x,). donde


x •x1 (mod2), Osx, s 1
x • x,(mod3), Osx, s2
x • x3 (mod5), Osx, S4.
En otras palabras, .x1, A'? y x:, son los restos que resultan de dividir x entre 2, 3 y 5. respec-
tivamente.
Los resultados ele la tabla 14.9 muestran que/es una función inyectiva y sobre.

lllblo 14.9
x (enZ.,) /(x)(en R) x (en Z.,) ((x) (enR) x (en Z.,) 1/(x) (enR)
o (0,0,0) 10 (0, 1,0) 20 (0,2, 0)
1 {l, 1, 1) 11 (1,2, 1) 21 (1, 0, 1)
2 (0, 2,2) 12 (0,0, 2) 22 (O, 1,2)
3 {1, 0,3) 13 (1, 1,3) 23 (1, 2,3)
4 (O, 1,4) 14 (0, 2,4) 24 (0, 0,4)
5 (1,2,0) 15 (1,0,0) 25 (1, 1,0)
6 (0,0, 1) 16 (O, 1, 1) 26 (0,2, 1)
7 (1, 1,2) 17 (1,2,2) 27 (1, 0,2)
8 (0,2,3) 18 (0, 0, 3) 28 (O, 1, 3)
9 (1,0,4) 19 (1, 1, 4) 29 (1, 2, 4)
728 Capítulo 14 Anillos y aritmética modular

Para verificar que/ es un isomorfismo, sean x, y E Z» Entonces

f(x +y)= ((x + y)mod2,(x + y )mod3, (x + y)mod5)


= (xmod2,xmod3,xmod5) + (ymod2,ymod3,ymod5)
= f(x ) + f(y),
y

/(zy) = (xymod2,zy mod3,xy mod5)


= (xmod2,x mod3,x mod5) · (ymod2,y mod3,ymod5)
=f(x)f(y),

por lo que/ es un isomorfismo.


Al examinar la rabia 14.9 vemos que, por ejemplo.

1) /(0) = (O, O, O), donde O es el elemento neutro de Z,. y (O, O, 0) es el elemento neutro
deZ,x z,x z,.
2) /(2 + 4) = /(6) = (O, O, 1) = (O, 2, 2) + (O, 1, 4) = /(2) + /(4).
3) El elemento 21 es el inverso aditivo de 9 en Z» mientras quc/(21) = (1, O, l )cseJ
inverro aditivo de (1, O, 4) =/(9) en Z, x Z, X Z,.
4) {O, 5, JO, 15, 20, 25} es un subanillodeZ,., con {(O, O, OX =/(O)), (1, 2, OX =/(5)).
(O, 1, OX= /(10)), (l. O, OX= /(15)), (O, 2, OX= /(20)), (1, 1, OX = /(25))} como
subanillo correspondiente en Zi x Zl x ~-

Pero ¿qué más podemos hacer con este isomorfismo entre Z30 y Z:z x ~ x Z,? Supon-
gamos. por ejemplo, que necesitamos calcular 28 • 17 en Zx.. Podemos transferir el proble-
ma a Z, X Z, x Z, y calcular/(28) -/( 17) = (O, 1, 3) - ( 1, 2, 2), donde los módulos 2, 3 y
5 son menores que 30y es~ fácil trabajar con ellos. Puestoque(O, 1, 3) • (1, 2, 2)=(0 - 1,
1 · 2, 3 · 2) = (0, 2, 1) y /"'(O, 2, 1) = 26, se sigue que 28 · 17 (en Z,.) es 26.

El isomorfismo del ejemplo 14.18 es un caso particular de un resultado más general


llamado el teorema chino del resto. Lo enunciaremos en este momento pero no lo demos-
traremos. Por el teorema fundamental de la aritmética. sabemos que para cualquier n E z•,
n > 1, podemos factorizarn comopj'Pi- · · · pt. dondep1,i>z, ... ,p., sonkprimos distintos.
k ;,, 1 y n,. n,, ... , n, E z•. El teon:ma chino del resto dice que los anillos z. y Z.., x Z.., x
· · · x z., son isomorfos para m1 = p¡1• mi = p'f, .. . , m., ;:;;; p;--. Para et lector interesado en
el tema, debemos mencionar que en las páginas 344-359 del texto de J. P. Tremblay y R.
Manohar [7], el teorema chino del resto se eswdiajunto con aplicaciones de la aritmética
de los restos a los computadores.

EJERCICIOS 14.4 1. Si Res el anillo del ejemplo 14.6, construya un isomorfismof: R ➔ ~


2 . Complete las demostraciones de los teoremas 14.15 y 14.16.
14.4 Homomorfismos e isomorfismos de anillo 729

3. Si R. Sy Tson anillos y/: R ➔ S,g: S ➔ Tsoo homomorfismos de anillos, demuestre que la


composición g o/: R ➔ Tes un homomorfismo de anillos.

4. Si [82) aeR}, entoDCCS Ses un anillo en las opcraciooes de suma y producto de matrices.
Demuestre que R es isomorfo a S.
S. a) Sean (R, +.•)y (S, EB,0) anillos con elementos oeutros z,,y ts. respectivamente. Si/: R ➔
Ses un homomorfismo de anillos, seaK= (a eRJ/(a) =<,). Demuestre qucK es un ideal
de R. (K es el núcko del homomorflsmo de anillos/) ·
b ) Encuentre el núcleo del homomorfismo del ejemplo 14.15.
e) Sean/, (R, +, ·) y(S,$,0) comoenlapane(a). Demuestre que/es inyectiva si y sólo si el
mlc:leo de/e,(<,).
6. Use la infonnación de la tabla 14.9 para calcular lo siguiente en z.
• l (13)(23) + 18 b) (11)(21) - 20
<) (13 + 19)(Z7) d) (13)(29) + (24)(8)
7. a) Construya una tabla (como en el ejemplo 14.18) para el isomorfismo/: Z,. ➔ Z. X Z,.
b) Use la tabla de la parte (a) para calcular lo siguicotc en Z»
1) (17)(19) + (12)(14) ü) (18)(11)- (9)(15)

8. Seann,,; s E z•conn,,; s ;;i:; 2,n =rsy mcd(,; s) = l. Si/: Z. ➔ Z.. x Z,esunisomorlismo de


aoilloslalquc/(a) : (l,O)y/(b):(0, l),dcmuesuequcsi (m, r) EZ, x Z.,tWl«!esf'(m, r) a,
ma+ tb(modn):
9. a) ¿Cuántas unidades hay en el anillo Z.?
b) ¿Cuántas unidades hay en el anillo~ X ~ X Zi?
z.
e) ¿Son isomorfos los anillos y~ x Z i x Z 1?
10. a) ¿Cumtas unidades tiene Z15? ¿Cuántas tieoe Z:, x Z,?
b) ¿Son isomorfos los anillos Zu y Z:, x ~ ?
11. Sean p, q, r primos distintos. ¿Cuántas unidades tiene cada uno de los siguientes anillos?

a) Z,.i b) z, z,
X e) zfHI
d) z, xz,, e) z,1 f) Z, 1 X z,
g) z, xz,x z,. lt) Z,l,, i) Z,1x z,,
j) z,. x z, xz,, k) Z,,,, 1) z,"xz,
m.) Z, X Z,,, a) z, xz,,xz, o) z,.xz.. x zr, donde ~b.cez+
12. ¿Oláles de los anillos del ejercicio 11 son isomorfos?
13. ¿Son isomorfos z. y el anillo del ejemplo 14.4?
14. Si/: R ➔ Ses un homomorfismo de anillos y J es un ideal deS, demuestrequeJ1(J) = {a E
R j /f.a) E J) es UD ideal de R.
15. En la solución del ejemplo 14.17, parte del problema pedía calcular

1.3 3.7]')_,
([ -3.7 1.3

Algcbraicamente, esto es igual a

1.3 3.7]-')'.
([ -3.7 1.3

¿Existe alguna ventaja. respecto a los cálculos con computador, al usar una forma y DO la otra?
730 Capítulo 14 Anillos y aritmética modular

14.5
Resumen y repaso histórico
Este capítulo nos presentó un sistema matemático llamado anillo, hacendo hincapié ea
estructura inducida por dos operaciones binarias cerradas. En todo el desarrollo de
matemáticas. el anillo de los enteros ha desempeñado un papel central. En la rama de
matemáticas llamada teoría de números, analizamos las propiedades básicas de (Z, +. •)
de los anillos finitos (Z.. +, •).Los anillos de matrices nos proporcionan ejemplos •
dos de anillos no conmutativos.
Este capítulo contiene el desarrollo de una teoáa abstracta. Con base en la definicióll
anillo, establecimos principios del álgebra elemental que hemos usado desde nuestros ·
meros encuentros con la aritmética, los números con signo y el manejo de in
Quizá al lector Je hayan parecido tediosas algunas de las demostraciones, ya que justifica.
mos todos los pasos en su derivación. Frente al reto que representa demostrar un resu1tailt
en las matemáticas abstractas, deberíamos seguir el consejo del retórico romano Maca
Fabio Quintiliano (siglo to.e.): "En Jugar de buscar con ahínco que se nos entienda•
deberíamos hacer todo lo posible por que no se nos malentienda...
Un famoso problema de la teoría de nWDeros. conocido como el último teomna 4,
Fermat. afirma que la ecuación~+ y' = t'. n E z •.n > 1, ·no tiene soluciones en z• sia>
2. En 1637, el matemático francés Pierre de Fermat (1601- 1665) escribió que había do-
mostrado el resultado pero que la demostración era demasiado lmga como para incluidl
en el margen de su manuscrito. Por desgracia. hasta el momento existe una infinidad•
valores de n para los cuales no se ha demostrado este teorema.t a pesar del tremeodD
esfuerzo de matemáticos como Leonhanl Euler (1707-1783), Peter Gustav Lejeuoe DiricbJd
(180.5-1859), Car! Friedrich Gauss (1777- 1855), Sophie Germa.in (1776-1831), Adra,
Marie Legeodre (1752-1833), Niels HenrikAbel (1802-1829), Gabriel Lamé (1795-1810)
y Leopold Kronecker (1823-1891). Sin embargo, los intentos por resolver este problea
han traído como consecuencia nuevas ideas y teorías matemáticas.

Pierre de fflmat (1601-1665) 5ophíe Gennain (1776-1831)

t& 1994,Andzew Wlles.. con lacolaboraci6a de Richard Taylor,denl0$tr6porfia el último1e0ft:ml.*


Fermat. como cocsecoeocia de sus resultados sobre CW'Vti elípticas modulares. (N. MI R.T.)
14.5 Resumen y repaso histórico 731

Al tratar de demostrar el último teorema de Ferma~ el matemático alemán Emst KU1IUI1Cr


(1810-1893) desarrolló las bases para el concepto de ideal. Su compatriota Richard
Dedekind (1831-1916) formuló. dio nombre y utilizó este concepto posteriormente en su
estudio de los ahora llamados dominios de Dcdekind. Sin embargo, el uso del término
"anillo" parece atribuible al matemático alemán David Hilben (1862-1943).
Los bomomorfi.smos de anillos y su relación con los ideales fueron ampliamente estu•
diados por la maiemática alemana Emmy Noetber (1882-1 935). Su gran genio recibió
escasa remuneración. financiera y de otros tipos, del gobierno de su país. debido a la
discriminación sexual que existía en las universidades de su q,oca. Sin embargo. el talento
de Emmy Noether fue reconocido por sus colegas; fue elogiada en la edición del New York
Times del 3 de mayo de 1935 porAlben Einstein (1879-1955), quien reconoció la influen-
cia e importancia de su trabajo para el desarrollo de la teoría de la relatividad. Además de
sufrir la discriminación sexual, como judía fue obligada a salir de su tierra natal en 1933,
cuando los nazis Uegaron al poder. Pasó los últimos dos años de su vida asesorando a
jóvenes matemáticos en Estados Unidos. Para más información acerca de esta fascinante
persona, consulle la biogralla escrita por A. Dick [2].

Emmy Noether (1882-1935)

Los anillos especiales llamados cuerpos surgen en los sistemas de los números raciona--
les, reales y complejos. Pero también vimos algunos cuerpos finitos interesantes. Exami•
naremos estas esttucturas nuevamente en el capítulo 17. en conexión con los diseños
combinatorios. La teoría de cuerpos desarrollada por el genio francés Evariste Galois
(1811-1832) respondió preguntas acerca de la solución de ecuaciones polinomiales de
grado> 4. Estas preguntas han impresionado a los matemáticos durante siglos y sus ideas,
conocidas ahora como teorfa de Galois, siguen constituyendo una de las teorías matemáti•
cas más elegantes jamás desarrolladas. En el texto de O. Zariski y P. Samuel [9] aparecen
más detalles acerca de la teorfa de Ga1ois.
Como lectura complementaria en cuanto a la teoría de anillos en un nivel introductorio,
el lector inieresado debe consultar los capitules 13-17 de J. A. Gallian [3], el capítulo 6 de
732 capítulo 14 Anillos y aritmética modular

V. Lamey [4] y los capítulos 6, 7 y 12 de N. McCoy y T. Berger [5). Un tratamiento a


poco más avanzado aparece en el capítulo 4 de) texto de E. A Wall:cr [8].
El desarrollo de congruencia modular, junto con muchas ideas afmcs, se debe princi-
palmente a Car! Friedricb Gauss. El texto de l. Niven y H. Zuckerman contiene n,js dda-
lles acerca de la solución de congruencias.
Por último, el tema de la disper>íón puede estudiarse en el capítulo 2 de J. Tremblay y
R. Manohar [7]. El capítulo 4 deA.Abo, J. Hopcroft y J. U11man (l] incluye un análisis de
la eficiencia de las funciones de disper,íón y un estudio probabilístico del problema de
colisión que surge para estas funciones.

BIBLIOGRAFÍA

l. Abo,Alfr<dV.,JohnE. Ho¡,croft y JeffrcyD. Ullman,DaraStructuns andAlgoriJhms. Reading.


Mass., Addison-Wesley, 1983.
2. Dick,Auguste, Fmmy Noerho(}882-1935J, trad. Hcidi Blocher, Bostoo. Bid:bOuser-Bosloo. 1981.
3. Galliao, JosepbA., Conremporary Abstroct A lgebra. 2ª ed., Lexington, Mass., D.C. Heath aad
Company, 1990.
4. Lamey. Violet Hachmeister, Abstraer Algebra: A Fint Course, Boston, Prindlc, Weber .t;
Schmidt, 1975.
S. McCoy, Ncal H. y'Tbomas R. Bergcr,Algebra: Groups, Rings and Orhu Topks, Boston. AllJI
and Bacon, 1977.
6. Niven, lvan, y Herbert S. Zud.:ennan,An Iruroducricn to r~ Tht:ory o/NumlH.ri, 4" ed., Nucq
York, Wtley. 1980.
7. Tremblay, Jean-Paul y R. Manchar, Disc~te Marhematical Scruc.nues tiVirh ApplicaJion.s ro
Cmnpu:er Sciena. Nue"a York. McGraw-HiU, 1975.
8. Walkec. ElbertA., ln.trotl1'Ction 10.4.bstractAlgebra, Nue"YaYork, Random HouseBirtlwJscc, 1987.
9. Zariski, Osear y Pietre Samud. CommutativeA/gebra. vol. 1, Prince!on, NJ.• Van Noslrand. 1951-

~) Ningún subanillo de un dominio de integridad tie-


EJEROOOS
ne divisores propios de cero.
COMPLEMENTARIOS f) Cualquier $Ubanillo de un cuerpo es un cuerpo.
g) Un cuerpo sólo puede tener dos subanillos.
b) Lafuncióof. (Z. +, ·) ➔ CZ. +, ·)dadapor/{x):21
1. Determine si las siguientes proposiciones son 'Verda-
es un homomorfismo de anillos.
deras o falsas. Para cada proposición falsa. dé un contra-
i) Todo cuerpo finito tiene un número primo de ele-
ejemplo.
mentos.
a) Si (R.+, ·) es un anillo y 8 'FS C R con S cerrado
bajo+ y •, entooces Ses un subanillo de R. j ) Elcue,po(Q,+,·)tieneonainfinidaddcsubanillos.
b) Si (R, +,•)es un anillo con elemento unidad y Ses 2. Demuestre que un anillo Res conmutativo si y sólo si
un subanillo de R. entonces S tiene un elemento (a+ b)'= a'+ 2ab + b', para todos o. b E R.
unidad. 3. Un anillo Res bockano si a2 =apara codo a E R. Si R
e) Si(R. +. -)es unanilloconelementounidadu., yS es booleano, demuestre que (a) a + a = 2a = z para todo a e
es un subaoillo con elemento unidad Us, entonces R; y (b) Res conmutativo.
Uit=U.s. 4. a) En el cuerpo C de los números complejos. el co.
d) Todo cuerpo es un dominio de integridad. jugado de un mímero complejoz = x + tyestá dado por?=
Ejercicios complementarios 733

i-iy. Por lo tanto, 2+1, = 2 - 3i y Si= - Si, mientras que junio parcialmente ordenado en la inclusión de coojunt0$.
7=7. Si z, Zt, Zz E C demuestre que Si Ay B900 idealesdeR,coninf(A,B} =A n Bysup(A.B}=
A + B. el conjunto parcialmente ordenado es un recículo.)
1) z1 + z2=Ti +'i; 10. a) Si pes primo,demuestrequepdivide a (r)paratodo
i) ZiZi = z;z; O<k<p.
il) (z)" =(:'), para lodo n E Z- b) Sia, bEZ,demuestteque(a+b)' "'d' +b'(modp,
lv) (z)- 1 = (z-1), para todo z +O 11. Dados n enteros positivos x,,X:.t, . . . ,x.., oo necesaria-
v) z + ZER
..,
vi) zfeR•,paratodozi'O

b) Sa/: (C, +, ·) ➔ (C, +, ·)dada por/(,J=T. De-


mente distintos, demuestre que n 1 (x1 + x 2 + · · · + x,¡) para
algún 1 :si :Sn, oque existen I S i<j S ntalesquen 1(%...1
+ · · · +x>-, +x¡).
muestre que/ es un isomorfismo. 12. a) ¿Cuántas soluciones tiene laccuaciónx+ y+z=O,
S. Sí(R, +, ·) esunanillo,demuestrcqueC= Ir E R lar= si.x,y.zEZ,?
b) Responda la parte (a) si x. y. z E 4
,._para todo a E R} es un subanillo cleR. (El subanilloC es
dCffllro de R.) 13. a) ¿De cuántas formas podemos seleccionar dos ente-
6. Dado un cuerpo finito K, sea Ml.,K) el conjunto de las ros positivos m. n, no llCCC$Mlamcntc distintos, de modo
que 1 :S m. n s 100 y que el último dígito de.,. + 3• sea 8?
albices 2 x 2 con elementos de K. Como en el ejemplo
14.2,(Mi._K), +.•) se convierte en un anillo no conmutativo b) Responda la parte (a) para el caso I .s m. n .s 125.
CCII demento unidad.
e) Si seleccionamos m. n al azar [como en la parte
a) Dctcnnineel ndmerodeelementosdeMi,K) siKes (a)j, ¿cuál es la probabilidad ahora de que 2 sea el
último dígito de 7 ..+ 3"?
i) Z2 ü) Z1 iü) Z,..p primo
14. Sean ez•,conn> l.
b) Como en el ejercicio 13delasección 14.l,A= [~ ~] a) Si n = 2k, donde k es un entero impar, demuestre
E M, (Z,) es una unidad si y sólo si ad - be #z. que k' " k (mod n).
Esto ocurre si la primera fila de A no contiene dni- b) Si n = 4.1: para k E z•. demuestre que (2k}' s O
camente ceros y la segunda fila oo es un mllltiplo (modn).
(por un elemento de z,) del primero. Use esta ob- e) Demuestte que
servación para determinar el número de unidades en
i) M,!_7,J il) M,!_Z,) ili) M,<Z,), p primo
"'i, ;1 ...{l (modn),
1
paranparconfimpar
7. Dado un dominio de integridad (D, +,•)con elemento ,., O (mod n ), de Olro modo.
-,ii.seana, b E D tales queab,. z. (a) Si a' =b'ya'=
f,demuestre que a= b. (b) Sean m. n E z• tales que m. n 15. Paran EZ♦,seaS. = 12+31 +52 + ··· +(2n-1)2=
• primos relativos. Si d" = b'" y a"=,,., demuestre que a = b. (½)(nX2n - IX2n + 1). [Establecimos esia fó!Ttlula para la
suma de los cuadrados de los primeros n enteros positivos
l. SeaA =R•.Dcfinimosey 0enA comoa EBb = ab, el
impares mediante inducción matemática en la parte (a) del
,rocmctocomúndea, b;ya0b=(i0f,t.
ejercicio 1 de la sección 4.1.j
a) Verifique que (A., e, 0) es un anillo conmutativo
con elemento unidad. Detenninc el último dígito de (a) S...,; (b) s,,,.,; (e)
b) ¿Es un dominio de integridad o un cuerpo? S171x, y (d) S., donde n es un múltiplo de 5.
9. Sea R un anillo con ideales A y B. Definimos A+ B = 16. Escriba un programa (o desarrolle un algoritmo) que
fa+bla E A, b E B}. Demuestre que A +Bes un ideal de invierta el orden de los dígitos en un entero positivo dado.
l. (Para cualquier anillo R, los ideales de R forman un con. Por ejemplo, la entrada 1374 debe producir la salida 4731.
15
Álgebra booleana
y funciones
de conmutación

e nuevo nos enfrentamos a un sistema aJgebraico en el que la estructura depende princi-


D palmente de dos operaciones binarias cerradas. A diferencia del tratamiento dado a l
te ma de los anillos, al estudiar las álgebras booleanas pondremos más énfasis en las apli-
caciones que en la naturaleza abstracta del sistema. No obstante. examinaremos con cui-
dado la estructura de un álgebra booleana y veremos resultados un tanto diferentes de los
de los anil1os. Entre otras cosas. un álgebra booleana finita debe tener 2" elementos, para
algún n E z•. Por otto lado, conocemos al menos un anillo para cada m E z•, m > l. el
anillo (Z~ +, ·).
En 1854, el matemático inglés George Boole publicó su obra monumental An
Jnvestigation of the Laws of7ñought. En esta obra. Boole creó un sistema de lógica mate-
mática que desarrolló en términos de lo que ahora llamamos un áJgebra booleana
En 1938, Claude Elwocxl Shannon desarrolló el álgebra de las funciones de conmutación
y mostró la forma e n que su estructura se relacionaba con las ideas establecidas por Boole.
Como resultado de este trabajo, un ejemplo de las matemáticas abstractas del siglo XIX se
convirtió en una disciplina matemática aplicada en el siglo x:x.

15.1
Funciones de conmutación:
Formas normales disjuntiva
y conjuntiva

Un interruptor eléctrico puede encende~ (pennitiendo el flujo de corriente) o apagarse


(evitando e l flujo de corriente). En forma análoga. en un transistor, la corriente pasa (con-
ductor) o no pasa (no conductor). Éstos son ejemplos dedis¡x,sitivos con dos estados. (En la
sección 2.2 vimos que el interruptor eléctrico se relacionaba con la lógica con dos valores.)
Para analizar estos dispositivos con dos estados, abstraemos conceptos como "'verdade-
ro" y ~also". "encendido.. y "apagado", de la forma siguiente.

735
736 Capitulo l S Álgebra booleana funciones de conmutación

ScaB = {O, 1). Definimos la swna, producto y complemento para los elementos
como

Una variablexes una variabk boo/.eana six sólo toma valores de B. En e


x + x = x y x2 = x • x = :a= x para cualquier variable booleana :c.
Si x, y son variables booleanas, entonces

1) x+y=0siysólosix=y=0,y
2) .ry= 1 siysólosix=y = l.

Sin E z •,entoocesB"= {(b.,b,, ... ,b.)lb,E {O, 1 ),1 s; Sn}. Unafunción/: 8'
es una función th conmutacWn. o booleana. de n variables. Las n variables se en
escribimos/(x1, ~••••• xJ, donde cadax., para 1 :Si sn es una variable booleana.

Sea/: B' ➔ B, don<le f(x. J\ z) = .ry + z. (Escribimos .ry en vez de x • y) Esta


booleana queda determinada evaluando/para cada una de las ocho posibles asi
a las variables x. y. z. como lo demuestra la tabla 15.1.

Tabla 15.1
X y z xy f(x,y,z)= xy +z
o o o o o
o o 1 o 1
o 1 o o o
o 1 1 o 1
1 o o o o
1 o 1 o 1
1 1 o 1 1
1 1 1 1 1

Definición 15.1 Paran E z .,n ;, 2, seanf.g: 8"->Bdos funciones booleanas de lasnvariables booleaaa
x1, x2 , ••• ,x•. Decimos que/y g son iguales y escribimos/=g si las columnas paraf.g(ea
sus respectivas tablas de función] son exactamente las mismas. [Las tablas muestran qae
f(b,. b,, ...• b) =g(b,, b,•... , b.) para cada una de las 2• posibles asignaciones de Oo 1
a cada una de las n variables booleanas x1, x1 , • . • , .x,..]

Definición 15.2 Si/: g. ➔ B. entonces el complemento de f. que se denota con j, es la función booleau
definida sobre B- como

!(b,. b,, .. . , b, ) = f(b,. b,, . .. , b.).


15.1 Funciones de conmutación: Formas normales dísjuntiva y conjuntiva 737

Sig: B' 48, definimos/+ g,f · g: B' ➔ B lasumay producwdef, g, respectivamente,


como
(f + g)(b,, b,, .. . ,b.) =f(b,, b,, . .. ,b.) + g(b1, b,, .. . ,b.)
y
(/· g)(b1,b2, ... , b.) = f(b,,b,, . .. ,b.) ·g(b1,b,, .. . ,b.).

En la tabla 15.2 resumimos dieileyes (consecuencias importantes de estas definiciones).

Tabla 15.2

1) / =/ x=x Ley del dobu


complemento
2) f+g=jg x"+y=xy Leyes de DeMorgan
"fi=j+g xy=x+y
3) f+g=g+f x+y=y+x Leyes conmutativas
fg=gf xy=yx
4) f+(g+h) = (f+g)+h x+(y+z)=(x+y)+: Leyes asociativas
f(gh) = (fg)h x(yz) = (xy)z
S) f+gh=(f+g)(f+h) x +yz = (x+y)(x+z) Leyes distributivas
f(g +h) = fg +fh x(y +z) = xy+xz
6) f+f=f x+x = x Leyes de idempotencia
ff=f xx=x
7) f +O=f x+O=x Leyes de iden1úiad
f· l =f x·l=x
8) f_+l= I x+X=l Leyes de los inversos
ff =O u=0
9) f+ l=I x+l=I Leyes de Mminación
f· O=O ,<·0=0
10) f+fg=f x+xy=x Leyes de absorción
f(f+g)=f x(x +y)= x

Como con las leyes de la lógica (en el Cap. 2) y las leyes de la teoría de conjuntos (en
el Cap. 3), las propiedades que aparecen e n la tabla 15.2 son satisfechas por las funciones
booleanas arbitrarias/, g, h: B' ➔ B y porlas variables booleanas arbitrarias x, y, z. (Escri•
bimos/g en vez de/· g.)
El símbolo O denota la función booleana constante cuyo valor es siempre O, y 1 es la
función cuyo úníco valor es l. (Nota: O, 1 'l. B.)
De nuevo, la idea de dualidad aparece en las propiedades 2 - 1O. Si s representa un
teorema acerca de la igualdad de las funciones booleanas, entonces s'. el dual de s, se
obtiene al reemplazar en s todas las ocurrencias de + ( •) por • ( +) y todas las ocuncncias de
0(1) por 1(0). Por el principio de dualidad (que analizaremos en la sección 15.4), la propo-
sición s' también es un teorema. Lo mismo es cierto para u.o teorema que trata de la igual-
dad de las variables booleanas, excepto que reemplazamos en este caso los valores booleanos
O y 1, no las funciones constantes O y l.
738 Capitulo 15 Álgebra booleana y funciones de conmutación

El principio de dualidad es útil para establecer la propiedad 5 de la tabla 15.2


funciones y variables booleanas.

La ley distributiva M +sobre·. Las últimas dos columnas de la tabla 15.3 muestru
f + gh =(j+ gXf+ h). También vemos que x + )'t =(x + yXx + z) es un caso
esta propiedad, si/, g, h: B' ➔ B, conf(x, y. <) =x, g(x.y. <) =y y h(x, y.<) =z. Porlo
no necesitamos más tablas para establecer esta propiedad para las variables booleaa&
Por el principio de dualidad, tenemos que /(g + h) = fg + fh.

Tabla 15.3

f g h gh f+g f +h f+gh (/+g)(f+h)


o o o o o o o o
o o 1 o o 1 o o
o 1 o o 1 o o o
o 1 1 1 1 1 1 1
1 o o o 1 1 1 1
1 o 1 o 1 1 1 1
1 1 o o 1 1 1 1
1 1 1 1 1 1 1 1

a) Para establecer la primera ley de absorción para las variables booleanas, en vez
basarnos en la construcción de una tabla, argumentamos de la forma siguicate:

x+.:ry=xl+xy Ley de identidad


=x(I + y) Ley distributiva de • sobre +
=xi Ley de dominio (y ley conmutativa de+)
=x Ley de identidad

Este resultado indica que algunas de las leyes pueden deducir,e de otras. La
pregunta es entonces qué propiedades debemos establecer con tablas para ol>teas
las demás. como lo hicimos aquí. Analizaremos esto posteriormente, en la sccci6I:
15.4, cuando estudiemos la estructura de un álgebra booleana.
Por el momento. demostraremos que los resultados de la tabla 15.2 pueden ma-
se para simplificar otras expresiones booleanas.
b) Para las variables booleanas w, x, y y z. tenemos que

.Razones
wy + xy + wz + xz = (w + x)y + (w + x)z Ley conmutativa de • y la ley
distributiva de • sobre+
=(w + x)(y+z) Ley distributiva de • sobre+
l S. l Funciones de conmutación: Formas normales disjuntiva y conjuntiva 739

e) Simplificaremos la expresión wx +xi+ (y + z), donde w. x. y y z son variables


booleanas.

Razones

-+s+~ + n - - + ~+n+~+n Ley de De Morgan


- wx + (x + z) +(y+ f) Ley del doble complemento
- [(wx + x)+z] + (y+ z) Ley asociativa de +
- (x + z) + (y + z) Ley de absorción (y las leyes
conmutativas de + y ·)
-x+(z+z)+y Leyes conmutativa y
asociativa de+
=x + Z+y Ley idempotente de +

Hasta este momento, hemos repetido para las funciones booleanas lo hecho con las
proposiciones. Al dar una función booleana (en términos algebraicos). construimos su
tabla de valores. Consideremos ahora el proceso inverso: dada una tabla de valores, en-
contraremos una función booleana (descrita en términos algebraicos) para la cual sea la
tabla correcta.

Dad$ tres variables boolcanasx, y, z encontraremos las fórmulas para las funciones/, g, h:
B3 ➔ B de las columnas dadas en la tabla 15.4.
Para la columna que está debajo de f. queremos un resultado que tenga el valor 1 cuan-
= =
do x =y O y z l . La función f(x, y, z) = X,Y.t es una de esas funciones. De la misma
forma, g(x, y, z) - xyz da el valor 1 para x - 1, y - z - O y O eo los demás casos. Como f y
g tienen el valor l solamente en un caso y estos casos son distintos entre sí, su suma/+ g
toma el valor I exactamente en estos dos casos. Así, h(_x, y, z) = f(x. y, z) + g(,x, y, z) =xyz. +
:ijí tiene la columna de valores dados bajo h.

Tabla 15.4

X y z f g h

o o o o o o
o o 1 1 o 1
o 1 o o o o
o 1 1 o o o
1 o o o 1 1
1 o 1 o o o
1 1 o o o o
1 1 1 o o o

Este ejemplo nos lleva a la siguiente definición.


740 Capítulo 15 Álgebra booleana y funciones de conmutación

Definición 15.3 Para cualquier n E z·, si/es una función booleana sobre las n variablesx1,:S:, .. . ,z.
a) cada término X; o su complemento X;, para 1 s i S n es una literal;
b) un ti rmino de la forma Y1 Y2 ···Y~ donde cada y,= X; o X;, para l s i s n, es
conjunción~ntal; y
e) una representación de/como una suma de conjunciones fundamentales es una
ma nonna/ disyuntiva (f.n.d.) de f

Aunque no daremos una demostración formal, los siguientes ejemplos indican que cml-
quier función/: B" ➔ B,f;, O, tiene una única representación (excepto por el orden de 111
conjunciones fundamentales) como una f.n.d.

Encuentre la f.n.d. de/: B' ➔ B, donde/(:,, y. z) = xy + xz.


De la tabla 15.5, vemos que la columna de/tiene cuatro unos. los cuales nos indican la
cuatro conjunciones fundamentales necesarias en la f.n.d. de/, de modo que.f{x. y, .z:)=
iyz+ Xyz+xyZ+xyt.

Tabla 15 s

X y z xy xz f
o o o o o o
o o 1 o 1 1
o 1 o o o o
o 1 1 o l. 1
1 o o o o o
1 o 1 o o o
1 1 o 1 o 1
1 1 1 1 o 1

Otra forma de resolver este: problema consiste en tomar cada término producto e intto-
ducir de alguna forma todas las variables faltantes. Usamos las propiedades de estas varia-
bles para obtener xy + xz = xy(z + z) + x(y + y)z (¿por qué?)= xyz + xyz
+ xyz + xyz.

Encuentre la f.n.d. de g(w. x, y, z) = wxy + wyz + xy.


Examinaremos cada término como sigue:
a) wxy = wxJ (z + z) = wxJz + wxyz
b) wyI = w(x + x)yz = wxyz + wxyz
e) xy = (w + w)xy(z + z) = wxyz + wxyz + Wxyz + Wxyz
De la propiedad de idempotencia de + se sigue que la f.n.d. de g es

g(w,x,y, z) = wxyz + wxyz + wxyZ + wfyZ + wxyz + Wxyz + WxyZ.


15.l Funciones de <::onmutación: Formas normales disjuntiva y conjuntrYa 741

Consideremos las primeras tres columnas de la tabla 15.6. Si acordamos enumerar las
variables booleanas en orden alfabético. veremos que los valores dex, y, zen cualquier fila
determinan una etiqueta en binario. Estas etiquetas en binario para O, 1, 2, . .. , 7 surgen
para las filas 1, 2, ...• 8, respectivamente, como se muestra en las columnas 4 y 5 de la
tabla 15.6. [Observemos, por ejemplo, que la primera fila tiene número de fila 1 pero
etiqueta en binario 000(::;;; O). De la misma forma, la séptima fila. dondex= 1, y = 1, z =0,
tiene número de fila 7 pero etiqueta en binario 110(= 6).) Como resultado. la f.n.d . de una
función booleana no nula se puede e xpresar en forma más compacta. Por ejemplo, la
función f del ejemplo 15.5 puede darse como f = Lm(I, 3, 6, 7), donde m indica los
mi.nténninos (es decir, las conjunciones fundamentales, en este caso, con tres literales) en
las filas 2, 4, 7. 8, con las respectivas etiquetas en binario 1, 3, 6 1 7. Usamos la palabra
minténnino para enfatizar que la conjunción fundamental toma el valor 1 un número minimal
de veces (a saber, una vez) sin ser idénticamente nula. Por ejemplo. m(l) denota el
minténnino para 1a fila con etiqueta en binario 001(=1), donde x::;;; y ::;;; O y z-= l; esto
corresponde a la conjunción fundamental iyz, la que toma el valor 1 para exactamente
una asignación (donde x =y= O y z ::;;; 1).

Tabla 15.6
X y < ~ e n binario Nómero de fila
o o o 000(=0) 1
o o 1 001 (= 1) 2
o 1 o 010 (= 2) 3
o 1 1 011 (=3) 4
1 o o 100(=4) s
1 o 1 101 (= 5) 6
1 1 o 110 (= 6) 7
1 1 1 111 (= 7) 8

Aun sin una tabla podemos representar la f.n.d de la funcióng del ejemplo 15.6, pagamos
por caso, como una suma de minténninos. Para cada conjunción fundamental c 1c:_ey:.,
donde c 1 ::;;; w o W, . .. , c4 ::;;; z o Z, reemplazamos cada C;, l ::s i ::s 4, por O, si e; es una
variable con complemento, y por 1 en caso contrario. De esta forma obtenemos la etiqueta
en binario asociada con esa conjunción fundamental. Como suma de mintérminos. vemos
queg = Lm(6, 7, 10, 12, 13, 14, 15).

La forma nonnal conjuntiva,. que analizaremos antes de cerrar esta sección, es dual de
la forma normal disyuntiva.

Seaf: B1 ➔ Bdadaen la tabla 15.7. Un término de la forma e,+ c 1 + c 3,dondec1 = xoi, c1


::;;; y o Y y c 1 = z o Z es una disyunción fundamental. La disyunción fundamental x + y + z
toma el valor l en todos los casos, excepto donde el valor de cada x.. y, z es O. En forma
análoga,x+ Y +z toma el valor 1 exceptocuandox= z = 0yy =- l. Como cada una de.estas
disyunciones fundamentales toma el valor O solamente en un caso y estos casos no ocurren
742 Capitulo 15 Álgebra booieana y funciones de conmutación

Tabla 15.7
X y z f
o o o o
o o 1 1
o 1 o o
o 1 1 1
1 o o 1
1 o 1 1
1 1 o o
1 1 1 1

en forma simultánea. el producto (x + y + z)(x + Y + z) toma el vaJor O precisamente en los


dos casos dados. Si seguimos de esta forma podremos representar/ como

f= (x +y+ z)(x +y+ z)(.f +y+ z)


que es la forma normal conjuntiva (f.n.c.) de f.
Puesto que la disyunción fundamental x + y + z toma el valor 1 un número máximo da
veces (sin ser idénticamente 1), es un maxti rmino, particularmente cuando usamos UII
etiqueta de fila en binario para representarla. Usamos las etiquetas en. binario para indeur
las filas de la tabla y e scribir f = IT M (O. 2, 6), un producto de maxrérminos.
*
Esta representación ·existe para cualquier f 1 y es íinica salvo por el orden de la
disyunciones fundamentales (o mutérminos).

Sea g: B' ➔ B tal queg(w. x. y, z) = (w +x + y)(x + 5' + z)(w + y). Para obtener la f.n.c. do
g, escribimos de nuevo cada disyunción en el producto como sigue:

a) w+x+y=w+x+y +O=w+x+y+zz
=(w +x +y+ z)(w +x + y +z)
b) x +y+ z= ww +x +y + z =(w + x +y+ z)(w+x +y+ z)
e) w+y = w + x.i' +y= (w +x +y)(w +.f+y)
= (w + x +y+ zz)(w + .f +y + zz)
= (w +x + y+ z)(w + x +y+ f)(w +x +y+ z)(w + x +y+ z)

En consecuencia, por la ley de idempotencia de •, tenemoo g(w, x; >: z) = (w + x +y+ zXw +


x+ y+ z)(w +x+ Y +zXW+x + Y +z)(w+x+y + z) · (w +X +Y +zXw +X + Y +z).
Para obtener g como producto de maxt6rminos, asociamos a cada disyunción twKt.
mental d, +d2+d1 +d.el número binariob1 bz b3 b., donde b1 = O si d 1 = w; b 1 = l si d 1 =1i;
... ; b,= O si d.= z; b,= I si d.= z. Como resultado, g = ITM(O, l , 2, 3, 6, 7, 10).

Nuestro último ejemplo de la sección repasará lo aprendido acerca de las formas de


representar una función booleana no constante/(es deciI./-1'0 y /-1'1).
15.1 Funciones de conmutación: Formas normales dis·untiva y conjuntiva 743

Sih(w, x. y, z) = wx+ Wy +Xyz, entoncespodemosescribirnuevamentecadasumandodeh


como sigue:
i) wx = wx(y + y)(z + z) = wxyz + wxyz + wxyz + wxyz
ü) wy =w(x +x)y(z + z) = wxyz + wxyz + wxyz + wxyz
iü) xyz = (w + w)xyz = wfyz + wfyz
Usamos la ley de idempotenc~ para+ y vemos que la f.n.d. de hes
wxyz + wxyZ+ wxyz + wxyz+ Wxyz + WxyZ+ Wfyz + WXyZ + wXyz.
Si consideramos cada conjunción fundamental en la f.n.d. de h, obtene mos las siguien-
tes etiquetas binarias y números de mintérminos:

wxyz: 1111 ( = 15) wxy z: 1100 (= 12) wxyz: 0011 ( = 3)


wxyz: 1110 ( = 14) wxyz: 0111 (= 7) wxyz: 0010 (= 2)
wxyz: 1101 (= 13) wxyz: 0110 ( = 6) wxyz: 1011 ( = 11)
En consecuencia, también podemos escribir h = :Em(2, 3, 6, 7, 11, 12, 13, 14, 15). De
esta representación. usando mintérminos. tenemos h = il M (O, l, 4, 5, 8, 9, 10), un pro-
ducto de maxtérminos.
Por último, tomamos la etiqueta en binario de cada ~término y determinamos su
disyunción fundamental correspondiente:

0=0000: w+x +y+ z 8= 1000: w +x+ y+ z


1=0001: w+x + y+z 9 =1001: w + x +y+z
4=0100: w + x +y+z 10 = 1010: W + X + y + Z
s =0101: w + x+ y+ z
Esto nos dice que la f.n.c. de h es
(w + x+ y+ z)(w + x+ y+z)(w+x+ y+ z)(w +x+ y+ z)·
(w + x + y +z)(w +x +y + z)(w+x + y+z).
Por lo tanto,
wxyz + wxyZ + w.tyz + wxyz + Wxyz + WxyZ + Wf'yz + WXyZ + wXyz =
km(2,3, 6, 7, 11, 12, 13, 14, 15) = IlM(0, 1, 4, 5,8, 9, 10) =
(w + x +y + z)(w+x + y + z)(w +x+ y + z)(w+x + y+ f) ·
(w +x + y+ z)(w+x +y+ z)(w + x +y+ z).

EJEROOOS 15.1 1. Enwentrc el valor de cada una de las siguientes expresiones booleanas si los valores de las
variables booleanas w, .x. y y 4 son 1, l, O y O, respectivamente.
a) fji + Xy b) w+fy
e) wx+j +yz ~~~~ d) wx+xy+yz
<) (wx+yz)+wy+(w+y)(x+y)
2. Sean w, .r y y variables booleanas, donde .x toma el valor 1. Para cada una de 1~ siguientes
expresiones booleanas, determine, sí es posible, el valor de la expresión. Si no pue<k detenni-
nar el valor de la expresión, encuentre entonces el número de asignaciones de valores de w y y
tales que producen el valor 1 para la expresión.
a) .r+.xy+w b) xy+w e) Xy + xw d) Xy+w
744 c.apítulo 15 Álgebra booleana y funcione:1 de c.onmutación

3 . a ) ¿Cuántas filas se necesitan para construir la tabla (de función) de un.a función booleana de
n variables?
b ) ¿Cuántas funciones booleanas diferentes den variables existen?
4 . a) Encuentre la conjunción fundamental formada con las variables w. x. y, z o sus comp~
tos. de modo que el valor de la conjunción sea J exactameote cuando
O w= x =0,y =z= I. U) w=0, x = l , y =l, z 2 0.
W) w = O,x = y = z=l. iY)w2:x-ysz =O.
b) Re sponda la parte (a), esta vez para las disyunciones fundamentales en vez de las con►
ciones fundamentales.. donde el valor de cada disyunción fundamental es O precisamca:
para los valores dados de w. x. y. z.
S. Suponga quef: B ➔ B cst.tdada porf(.x. y. z) = (x+ y)+(iz).
3

a) Determine la f.n.d. y f.n.c. de/


b ) Escriba/ como una suma de minténninos y como un producto de maxU:nninos (usando
etiquetas en binario).
6. Seag: S- ➔ Bdadapor g(w. x. y. z) = (wz +xyz)(:x+"zyi).
a ) Determine la f.n.d. y f.n.c. de g.
b) &criba g como una suma de minté:rminos y como un producto de maxtérminos (usando
etiquetas en binario).
7. Sea F6 el conjunto de todas las funciones booleanas/: fJ4 ➔ B.
a) ¿,Cuánto vale JF,I?
b ) ¿Cuántas conjuncioDC$ (disyunciones) fundamentales existen en F6 ?
e) ¿,Cuántos mintáminos {maxtmninos) tiene F6 ?
d) ¿Cuántas funciones/ E F6 toman el valor 1 cuando (exactamente) dos de sus variables
toman el valor I? (En los demás casos. el valor de/puede ser Oo 1.)
e) ¿Cuántas funciones/E F6 toman el valor I cuando al menos dos de sus variables toman el
valor l? (En los demás casos, el valor de/puede ser O o 1.)
O Sean u, \l, w, x. y y z las seis variables booleanas para las funciones en F6 . ¿CUántasdeesus
funciones son independientes dex [es decir,/(u. u, w, x. y, z) =;/(u. U. w. X, y, z))? ¿Cuá:lb&
son independiente$ de las tres variables booleanas x. y, z.?
8. Sea/:~ -+ B. Encuentre la fonna normal disyuntiva de/ si
a) r '( l ) • {0101 (es decir, w =0,x = l,y • 0,z • l), OllO, 1000, 1011}.
b) r'(0) • {0000,0001,0010,0100, 1000, 1001,0llO}.
9- Sea/:B"-+ B. Si la fn.d.de/tienem conjunciones fundamentales y su f.n.c. tieneidisyundoa:s
fundamentales. ¿cuál es la relación entre m, n y K!
1O. Si x. y y z son variables booleanas y x +y+ z. =xyz. demuestre que las tres tienen el mismo valot
11. Simplifique las siguientes expresiones booleanas.
a) .cy+(x+y)l'+y b) x+y+(x +y+ z)
e) yz + wx + z + [wz(xy + wz)I d) X1 + i"1X2 + i"if2.r, + i"1i"2i", .r" + · · ·
12. Encuentre los valores de las variables booleanas w, x. y. z. que satisfagan el siguiente sistema de
ecuaciones (booleanas) simultáneas.
x + fy = O iy=iz iy+i"Z+ zw = Zw
13. a) Puaf.g,h,B" ➔ B,,xmucsr.requefg+fh+gh =fg+fhyquefg+fg +fg+fi= I.
b) F.nuncie el dual de cada resultado de la parte (a). ·
14. Sean/. g: B" ➔ B. Defina la relación "S" en F., el conjunto de todas las funciones booleanas
den variables. como/ S g si e l valor de ges I al menos cuando e) valor de/es l.
a) Demuestre que esta relación es un orden parcial en F..,
b) Demucsttequefg sfyf s f+g.
15.2 Redes de puertas: Suma minimal de productos y mapas de Kama h 745

<) Pan n s 2. trace el diagrama de Hasse pan, las 16 funciones de F, ¿En ~ parte del
diagrama se localizan los mint&minos y los maxté:rminos? Compare este diagrama con el
del conjunto potencia de {a. b. e, d}, ordenado parcialmente con la relación de inclusión.
15. Defina la operación buwia cerrada ED(O exclusiva) en F. oomo/ED g =fi + fg, dootk/, g:
~ ➔ B.
a) Ddennine/ED/, ¡e/. / ED l y /ED 0.
b) Dcmuesue o refute lo siguiente:
i) f $ g • O=>f g 2 11) / Ef) (g $ h)•(/Ef)g)$h
lli) f$g= J $g IY) f ~ = ( [ @g)(!Ef) h)
•J f(g@ h) =fg $ P, .,, <7ei> = f$ g •f$ g
vil) /13:) g-/13:)h=>g • h

15.2
de puertas: Suma minimal
de productos y mapas
de Kam augh

Las funciones de conmutación de la sección 1S. l presentan una teoría matemática interc,.
sanie. Su importancia radica en su implementación poc medio depu, rta.s /6gicas (disposi-
tivos de un computador digital que realizan tareas especÍÍJCaS en el procesamiento de da-
tos). Los componentes elkaicos y mecánicos de tales puertas dependen del estado actual
de la tecnología; no nos preocuparemos aquí por las cuestiones relativas a los equipos.
La figura 15.1 oonticne las puertas lógicas para la negación (complemento), conjunción y
disyuncióo en las partes (a). (b) y (e). respectivamente. Puesto q ue las operaciones booleanas+
y • son asociativas, podrfamos lellCr más de dos entradas para una puataAND o una puena OR.

,-[>--, ;=Q--xy ;~ x+y


0>)PueuANO
"'"""""' {c)PutrtaOR

Figurii 15.1

La figura 15.2 muestra la red /6gica o de puertas, par.o la expresión (w + x) • (y + .t<).


Los símbolos que aparecen en una lfnea a la izquierda de una puena (o inversor) son las
entradas. Cuando están en un segmento de recta a la derecha de la puena, son las salidas.
Hemos dividido la línea de. entrada de x, para que pueda servir como entrada en una puerta
ANO y en un inversor.

(w+iX,y+xz)

Figura 15.2
746 Capítulo 15 Álgebra booJeana y funciones de conmutación

Los ejercicios proporcionarán algo de práctica en el trazado de una red lógica para
expresión booleana y en el paso de la red a la expresión. Por el momento. subra
algunas características de estas redes.

1) Una línea de entrada puede se¡,= para servir de entrada a más de una puma.
2) Las líneas de entrada y de salida sólo se juntan en las puertas.
3) No se puede retroceder; es decir, la salida de una puerta g no puede usarse
entrada de la misma puerta g o de cualquier puerta que (directa o in ·
lleve a la puerta g.
4) Supondremos que la salida de una red de puenas es una función instantánea de
entradas presentes. No existe dependencia del tiempo y no damos importancia a
entradas anteriores. como en el caso de las máquinas de estados finitos.

Con esto en mente. anali zaremos la suma de números binarios en un computadcx

Al sumar dos bits (dígitos binarios), el resultado es una sumas y un acarreo c. F.n tres
cuatro casos, el acarreo es O, por lo que nos centraremos en el cálcu1o de 1 + l. Si e
namos las partes (b) y (e) de la tabla 15.8, consideramosJa sumas y el acarreo e
funciones booleanas de las variablesxy y. Entonces e = xy y s= Xy +.x'j =xEB y= (.r +
(xy). (Recordemos que e denota la O exclusiva)

Tabla 15.8
X y SUma en binario X y Suma X y Acarreo
o o 0+0=0 o o o o o o
o I 0+1=1 o 1 I o I o
I o 1 + 0=1 1 o I I o o
I I I+ I = 10 1 1 o I 1 1
(•) (b) (e)

La figura 15.4 es una red de puertas con dos salidas. Se conoce como una red desala
múltipl.e. Este dispositivo, llamado semisumadbr, implementa los resultados de la tabla
15.8 (b) y (e). Si usamos dos semisumadores y una pucna OR, consttuimos el sumador
compl.eto que se muestra en la figura 15.4(a). Si x =x,.x_ 1 ••• x2 X 1 Xo y y= y. y.,..1 •• • JJ1t
Yo. consideremos el proceso de sumar los bits X; y y; para determinar la sumax + y . Ea cSle
caso, c.,..1 es el acarreo de la sumadex;_1 y y;_1 (y de un posible acarrcoc;--2-) Laeotradac...,

s = (x+y)(x'"y)

El semiscmador
Figura 15.3
15.2 Redes de puertas: Suma minimal de productos y mapas de Karnaugh 747

junto con las entradas x,. y y;, produce la suma s, y el acarreo C; como se muestra en la
figura. Por último, en la figura 15.4(b), se combinan dos sumadores completos y un
semisumador para obtener la suma de los dos números binarios XJX1Xo y y 2 y 1 Yo cuya suma
eSC1 S2 S1S0-

e_, X,
,.
Yo s,
x,
x, Y, s,
Y, e; - x,
X,Y, + (,_1(x, $ y,) Y, e,
(a) Et wmad°' completo (b)

Figura 15.4

El siguiente ejemplo presenta el tema principal de esta sección: la representación con


u.na suma minimal de productos para una función booleana.

Tenemos que encontrar una red de puertas para la función booleana

f (w,x,y,z) = ~m(4 , 5, 7, 8, 9, ll).


Consideremos el orden de las variables como w, x. y, z. Podemos determinar la f.n.d. de
f escribiendo cada número de miotérmino en notación binaria, para encontrar después su
conjunción fundamental correspondiente. Por ejemplo, (a) 5 ;:; 0101. lo que indica la con-
junción fundamental w.tyz:; y (b) 7 ;:; 0111, lo que indica W.ry.z. Si seguimos de esta fonna,
tenemos/(w, x. y, z);:; w.tyz + w.t)'z + wiyf + wXJz + w.Xyz + WX)'Z.
Usamos las propiedades de las variables booleanas para obtener

f=wxz(y+ y)+wxy(z+ z) + wxyz + wxyz


=Wxz + wxJ + wXyz + WxJiZ= Wx(z + yz) + wX(Y + yz)
=wx(z + y) + w:X(y + z) (Why?) = wx(y+z) + wx(y + z),
así,
a) f(w,x ,y,z) = Wxz +Wxy + wXJ+ wXz ; o,
h) f(w,x,y, z)=wx(y+z)+wY(y+z).

En el ejemplo 15. 11, el resultado

f(w, x,y,z) = Wxz + Wxy + wXJ + wXz


se conoce con frecuencia como una representación mediante una suma minima.l de pro•
duetos para la funciónf(w, x, y, z) ; :Em (4, 5, 7, 8, 9, 11). Vemos que esta represeniación
748 capítulo 15 Álgebra booleana y funciones de conmutaáón

es una suma de cuatro productos, donde cada producto está formado por tres li
adjetivo minimal se refiere a dos cosas:

1) CuaJqu.ier posible modificación posterior producirá una representación que


una suma de ta1es productos; y
2) Si/ puede representarse en una segunda forma como una suma de productos.
literales). entonces tendremos al menos cuatro términos de productos, cada uno
un minimo de tres literales.
[Nota: Una suma minimal de productos para una función booleana dada/(# 8)
tiene que ser única, como veremos en el ejemplo 15. 15.]

En este texto, el análisis de esta idea será un tanto informal. No intentaremos dem
que cada función booleana no nula tiene tal representación como suma minimal de JIO"
duetos. En vez de ello. supondremos la existencia de esta representación y simpl
continuaremos nuestro estudio acerca de la forma de obtener tal TCsultado.
A partir de este momento, consideraremos una entrada de la forma W como una entllda
exacta, que no ha pasado por puena alguna. en vez de considerarla como el resultado
introducir w y pasarla por un inversor.
En la figura 15.S(a), tenemos una red de puenas que implementa la f.n.d. de la funci6o
del ejemplo 15. 11. La pane (b)de la figura es la red de puertas para/como unasumamia-
mal de productos. La figura 15.S(c)tiene una red de puenas para/= wx(y +,) +w.i'(y +z).
La red de la parte (c) sólo tiene cuatro puenas lógicas, míentras que la puerta de la pare
(b) tiene cinco. En consecuencia, podríamos pensar que la red de la parte (e) es mejar
respecto a la minimización de costos, puesto que cada puerta adicional incrementa el COSlt
de producción. Sin embargo, aunque hay menos e ntradas y puertas para la implementacióa
de la pan.e (c). aJgunas entradas (corno Y y z), deben pasar por tres niveles de puerta.saa:s
de producir la salida f Para la suma minimal de productos de la parte (b), sólo hay das
niveles de puenas. En el estudio de las redes de puertas, las salidas se consideran como
funciones instantáneas de la entrada No obstante, en la práctica. cada nivel de puenas
añade un retraso en el desarrollo de la función f En el equipo digital de alta velocidad
queremos minimizar el retraso, por lo que optamos por más velocidad con un costo maf(I'
de fabricación.
Esta necesidad de maximizar la velocidad nos hace desear la representación de ta
función booleana como una suma minimal de productos. Para lograr esto en el caso de
funciones con no más de seis variable s, usamos un método gráfico llamado mapa llt
Karnaugh, desarrollado en 1953 por Mauricc Karnaugh (1924- ). Los mapas de Karnauglo
siempre producen formas con un máximo de dos niveles de puertas y veremos que la f.n.d.
de una función booleana es la clave que está detrás de esta técnica
Para simplificar la f.n.d. de/en el ejemplo 15.11, combinamos las dos conjunciooes
fundamentales Wxy"z y WX)'Z en el término producto Wxz. pues wxy"z + W.xyz = Wxz(y +y)=
W.XZ( 1) = Wxz. Esto indica que si dos conjunciones fundamentales difieren exactamente ea
una literal, entonces se pueden combinarse en un término producto con ese literal faltanae.
Para g: B4 ➔ B dada por g(w. x, y, z) = w.tjif + w.tjiz + w.xyz + wxyf, cada conjuncióa
fundamental (excepto la primera) difiere de su antecesor exactamente en un literal. Pode,.
mos entonces simplificar g como g = wxj(z + z) + wxy(z + z) == w.xj + wxy= wx(J + y) =
wx. También podríamos escribir

g = wz(yz+ yz + yz + yz) = wz(y +y)(z + z) = wz.


15.2 Redes de puertas: Suma minitnal de productos mapas de Kamaugh 749

f(.w.x.y.z}

y
f(w,x,y,rJ

NiYe( 2

figura 15.5

La clave para este proceso de reducción es el n:conocimiento de los pares (cuaternas,


2•·uplas) de conjunciones fundamentales tales que dos términos adyacentes difieran exac•
tamente en un literal. Si h: lJ4- ➔ By la f.n.d. de h tiene 12 términos. ¿podemos mover estos
t6rminos para reconocer las mejores reducciones? El mapa de Kamaugh organiza los tér•
minos por nosotros.
Comencemos con el caso de dos variables, w y x. La tabla 15.9 muesua los mapas de
Kamaugh para las funciones/(w. x) = wx y g(w. x) = w + x. (Suprimimos los ceros en las
tablas de estos mapas.)
En la pane (a). el 1 interior de la tabla indica Ja conjunción fundamental wx. Ésta apa-
rece en la fila de w = 1 y la columna dex = l. e) único caso en que wx = l. En la parte (b)
hay tres unos en la tabla. El 1 de la parte de arriba corresponde a WX, que tiene el valor 1
750 capitulo 15 Álgebra booleana y funciones de conmutación

exactamente cuando w = O, x = l. Los dos unos de la parte inferior corresponden a wi


wx. como se Ice en la fila inferior de izquierda a derecha.

wm
labia 15.9

(a) wx (b) w +x

La tabla 15.9(b) representa la f.n.d. WX +wX + wx. ComoresuJtadode su adyacencia


la fila inferior, la tabla indica que wi y wx difieren exactamente en un literal y
combinarlas para obtener w. Por la ley de idempotencia para la suma (que es crucial
trabajar con mapas de Karnaugh), podemos usar la misma conjunción fundamental wx
segunda vez en este proceso de reducción. La adyacencia en la segunda columna de
tabla indica la combinación de Wx y wx para obtenerx. (En la columna dex aparecen
las posibilidades de w; a saber. w y W. Ésta es una forma de reconocer x como el resulrallll
decsacolumna) Así, la tabla 15.9(b) muestra que Wx+ wX + wx = Wx+ wi' + wx+w.i=
(wx +wx) + (wx+ wx) ; w(x +x) +(w+ w)x; w(l) + (l)x ; w+x.

Ahora. consideremos tres variables booleanas w, x, y. En la tabla 15.10, la primera idea


nueva que encontramos aparece en los encabezados de las columnas paraxy. No son iga-
les a los encabezados q-ue usamos para las filas en las tablas de funciones. Aquí vemos. al
ir de izquierda a derecha, que 00 difiere de OI en una cifra, 01 difiere de 11 exactamente ea
una cifra, 11 difiere de 1O en una cifra y, para cerrar el ciclo, 1O difiere de 00 exactamelle
en una cifra.

labia 15.10
w xy 00 01 11 10

Sif(w. .x; y); ~ m (0, 2.4, 7),entonces,comoO; OOO(wxy"), 2 ;010, (w.ry¡, 4; IOO(w.r,>
y 7 = 11 l(wxy), podemos representar estos términos colocando unos como se mUCSlra ea
la tabla 15. 10. El 1 de wxy no es adyacente a ningún otro 1 de la tabla. por lo que esd.
aislado; tendremos entonces a wxy como uno de los sumandos en la suma minimal de
productos que representa af El 1 de ~ ( en el extremo derecho de la primera fila) no esd.
aislado, ya que si consideremos que la tabla se envuelve, este l es adyacente al l de wxy
(en el extremo izquierdo de la primera fila). Esto se combina (mediante la suma) para
damos w;tj' + wif = Wj(x +x) = wf(l) = wy. Por último, los unos de la columna dex=y
= O indican una reducción de w.xy + wxy' a (W + w)xy = (l)iy = xf. Por lo tanto, como
suma minimal de productos,/= wxy + wy + Xj.
15.2 Redes de puertas: Suma minimal de productos y mapas de Karnaugh 751

De las partes respectivas de la tabla 15.11, tenemos

a) /(w. .r,y) = l:m(0, 2,4,6)= l: m(0,4) + l:m (2,6)= (wxy + wxy) + (w-'5'- + wxy¡ =
(w+ w)x y + (w + w)zy= (l)xf + (l)zy = xy+.ry = (x + x'fy= (l )y = y, la única
variable cuyo valor no cambia al considerar los cuatro términos designados por los
unos. [En este caso, el valor de y es O, por lo que /(w, x, y)= y.]
b) /(w,x,y)=l:m(0,1,2,3)=iHY+wfy +wxy+wxy = w(x y+xy +xy+xy) =
w(x + x)(y +y) = w(l)(l) = w.
e) f(w, x,y) = l: m(l,2,3,5,6, 7) = l: m(l ,3,5, 7) + L m(2,3,6, 7) =y+ x.

Tabla 15.11

~xy 00 01 11 10 w xy w\ xy 00 01 11 10

o
1
(a)
~ 1
o
1
(b)
o
1
(c)
m
Para pasar al caso de cuatro variables, consideremos el siguiente ejemplo.

Tenemos que encontrar una representación como suma mi.nimal de productos para ]a fun-
ción /(w, .r, y, z) = l: m (O, 1, 2, 3, 8, 9, 10).
El mapa de Karnaugh para/ de la tabla 15.12combina los unos de las cuatro esquinas
(adyacentes) para obtener el términowxyz + W.iyl + wxyz + wXyf =Xi(ji:y + ~ + wy +
wy) =xz. Los cuatro unos de la fila superior se combinan para damos w.f. (Si solamente
utilizamos ]os dos unos de en medio, no haríamos uso de tQdas las adyacencias disponibles
y obtendríamos el ténnino w.íz. que tiene un literal más que Wi".) Por 6.ltim.o, el 1 en la fila
(w :;;:: 1, x= O) y la columna (y = O, z: = 1) se puede combínarcon el 1 a su izquierda. y asu
vez se puede combinar con los dos primeros unos de la fila superior para obtener WXyf +
WXj'z: + wXy'z + wxyz::;;:: Xj'. Por lo tanto, como suma m.inímal de productos.f(w, x, y, z:) =
xf+WX + Xy'.

Tabla 15.12

El mapa de/(w, x,y,z) = l: m(9, 10, 11, 12, 13) aparece en la tabla 15.13. El único 1 de la
tabla no combinado con otro término es adyacente a un 1 a su derecha (esta combinación
produce w.iz:) y a un I sobre él (esta combinación produce wjiz:). En consecuencia. pode-
752 Capitulo 15 Álgebra booleana y funciones de conmut ación

mos representar/ como suma minim.al de productos de dos formas: w.ty + wX)' + wii
wxy + wfy + k-'Jz. Entonces, este tipo de representación no es única Sin embargo,
mos observar que en cada caso aparece el mismo número de términos de producto y el
mismo número total de literales.

Tabla 1 5.13

wx yz 00 01 11 10
00
01
11
10
QJ1

Hay una forma correcta y otra incorrecta de usar un mapa de Kamaugh.


Sea/= (w,,; y, z) = ~m (3, 4, 5, 7, 9, 13, 14, 15). En la tabla 15.14{a), combinamos•
bloque de cuatro unos para formar el término .xi. Pero si nos fijamos en los otros cuaao
unos, hacemos lo que aparece en la pane (b). Así, el resultado de la pane (b) ¡,rodv,%/
como una suma de cuatro términos (cada uno con tres literales), mientras que el méuNID
descrito en la pane (a) añade el término adicional (innecesario) .tt-

Tabla 15.14

wx\ yz 00 01 11 10 wx\ yz 00 01 11 10

~b
00 00
01 01
11 10 1 11
10 10
(a) (b)

Las siguientes sugerencias de uso de los mapas de K.amaugh se basan en Jo hecho hasla
ahora. Las enunciamos en este momento para poderlas usar en mapas más grandes.

1) Comience por combinar los térmioos de la tabla doode haya como máximo ma
posibilidad para la simplificación.
2) Verifique las cuatro esquinas de la tabl~ pues pueden contener unos adyaceases
aunque parezcan aislados.
3) En todas las simplificaciones, intente obtener el bloque máximo posible de uaos
adyacentes para obtener un término producto minimal. (Recuerde que los unos~
den usarse mís de una vez, en caso oecesario, debido a la ley de idempotencia de+.)
4) Si existe una opción para simplificar una entrada en la tabla. intentamos usar UDOS
adyacentes que no hayan sido utilizados en una simplificación anterior.
15.2 Redes de puertas: Suma minimal de productos y mapas de Kamaugh 753

Si/(u, w,x,y,z) = ~m (I, 5, 10, 11, 14, 15, 18, 26, 27, 30, 31),construimosdostablas
4 x 4, una para u= O y otra para u= l. (Tabla 15.15)

Tabla 15.15
wx yz 00 01
00
01
11
©
10

(u=0) (u= I)
AJ seguir e l orden de las variables, escribimos, por ejemplo, 5 = 0010 l para indícar que
es necesario un 1 en la segunda fila y la segunda columna de la tabla para u = O. Los otros
cinco unos en la tabla de u= O corresponden a los minténninos de l. 10, 11, 14, 15. Los
mintérminos para 18, 26, 27, 30, 31 se representan mediante los cinco unos de la tabla de
u ::::: 1. Después de llenar todos los unos, vemos que el uno de la primera fila, cuarta colum-
na de la tabla para u= 1 puede combinarse con otro término solamente de una fonna (con
uwXyz) para obtener el producto ufyz. Esto también es.cierto para los dos unos de la
segunda columna de la tabla (u = O). Esto nos da el producto liwyz. El bloque de ocho
unos produce wy, y tenemos que/(u, w, x,y, z) = wy + i>wyz + UX}'Z.

Una función/de las seis variablesr, u, w, X. yy zrequierecuatrotablas para cada uno de


los casos (a)t = O, u= O; (b)t = O, u= l ; (c) r= 1, u= 1; y (d) r= 1, u= O. Con más de seis
variables. el método se vuelve demasiado complicado así, se puede usar ouo procedimien-
to: el métO<ÍJJ de Quine•McCluskLy. Para un número grande de variables, el método es
tedioso para hacerlo a mano, pero es un procedimiento sistemático adecuado para su
implementación computacional, panicularmente para computadores con cíerto tipo de
comando de "comparación binaria". (En el capítulo 7 de la referencia 3 hay más detalles
sobre esta técnica.)

Cerraremos esta sección con un ejemplo relacionado con el concepto dual: el producto
minimal de sumas.

Paiag(w, x, y, z) = flM (I, 5, 7, 9, 10, 13, 14, 15), esta vez colocamos un Oen cada una de
las posiciones para los equivalentes en binario de los maxtérminos enumerados. Esto pro-
duce los resultados que se muestran en la tabla 15.16 (donde suprimimos los unos).

Tabla 15.16

wx\yz 00 01 11 JO
00
01
11
JO
o o ~
754 capítulo 15 ÁJgebra booleana y funciones de conmutación

El O c:n la esquina inferior derecha sólo puede combinarse con el O que esté arriba de
él, por lo que tenemos (W+x +Y +z)(W + X+ Y +z) =(W + y+ l) +.xi= (W + y +z)+O=
W + Y+ z. El bloque de cuatro ceros (para los maxténninos de 5, 7, 13, 15) se simplifica
como X+ z.mientras que los cuatro ceros (para los maxtérminos de 1, 5, 9. 13) en la
segunda columna producen y +f. Así,g(w, x,y, z) =(W+Y +z)(i +z)(y+f), un produclt
minimal de sumas.

EJERCICIOS 15.2 1. Use los inve=res y las puertasAND y OR para constnm las puertas de la figura 15.6.

;=tr=)-f(x,y) ;=:t=)-g(x,y) ;:[>- h (x. y )

f(x. y ) = x (i;)y g (x,y) • iy h(x, y) aa.x +y


Puetta O-EXCWSNA P\JtrYNANO Puetta NOR

Figura 15.6

2 . Use solamente puertas NANDt (vtase la Fig. 15 .6) para construir el inversor, la puena AND
y la puena OR.
3 . Responda el ejercicio 2, reemplazando NAND por NOR.
4. Use los inversores. las puertas ANO y OR para construir las redes de puertas para
•) f(x,y, z) = xz + yz + x b) g(x,y, z) = (x + f)( y + z)x
e) h(x,y,z) = (.ry$yz)
S. Para Ja red de la figura 15.7, exprese/como función de w,x, y, z.

Figura 15.7

6. Implante el semisumadordc la figura 15.3 usando solamente (a) pumas NAND; (b) poenas NOll
7. Para cada una de las redes de la figura 15.8, exprese la salida en términos de las variables booieaam
x. y o de sus complementos. Utilice despu6s la expresión de la salida para simplificar la red dada.

t La puerta NANO se consuuye en forma seociUa con transistores, tanto c:on la tecnol.ogia anterior de
sc:micooductorcs, como con las técnicas más recientes de fabricación de microcircuitos (chips). Adcl'lm. la
mayoría de las redes de puenas que representan lo que realmente ocurre dernro de los computadores actmles
contienen u.na gran cantidad de estas puert.aS NANO.
15.2 Redes de puertas: Suma minimal de productos y mapas de Kamaugh 755

;i+J>E??-p-
(a)

(b)

Figura 15.8

8. Para cada una de las siguientes funciones boolell03$/. di.se& una red de puertas: de dos niveles
para/ como una suma m.inimal de productos.
a)/: 81 ➔ B. dondc/(x.y, .t)z 1 si y sólo si exactamente dos delas variables tienen el valor l.
b) /: B' ➔ B, doode/Cx,7, u=1 si y sólo si al menos dos de las variables tienen d valor l.
e)/: B' ➔ B. doode/(w.x.y,.t) = 1 si y sólo si un número impar de variables tienen el valor l.
9 . Encuentre una representación mediante una suma minimaJ de productos para

a) f(w,x,y) • Lm(l,2,5,6)
b) /(w,x.y)• ílM(0,J,4,5)
<) f(w, x,y,z) = Lm(0,2,5, 1,8, 10, 13, 15)
d ) /(w, x,y,z) = Lm(S,6,8, 11, 12, 13, 14, IS)
• l f(w,x,y, z) • Lm(7, 9, 10, 11, 14, IS)
f) /(u, w,x,y,z) = Lm(l, 2,3,4, 10.17, 18, 19,22,23,27,28,30,31)

10. Oblenga una rcpresentaci6n mediante una suma minimal de productos para / (w, x, y, z) •
ílM(0. l. 2. 4, 5, 10. 12. 13, 14).
11. ~/: B' ➔ B una función de las variables booleanas x 1• xb ...• ..r,.. Dcterm.ine n si el nómero
de unos necesarios para expresar ax, en el mapa de Karnaugh de/es (a) 2; (b) 4 : (e) 8; (d) 21,
paraA::EZ-,conl sA:::sn-1.
12. Si g: B1 ➔ Bes una función booleana delas variables booleanasz1,zl, .. . ,z,. ¿cuiru.os unos
se oecesitan en e1 mapa de Kamaugh deg pan represmtar el tbmino producto (a)z1; (b)z.,r:,;
(c)z1 i"2.r,; (d)z, z 1.r,.r..,?
13. F.n los siguientes ejercicios,/: B' ➔ B. donde las variables booleanas (en orden) 50D w, z, y y
~ Determine J¡-'(0)I y J r 'Oll si. como suma minimal de productos,/se reduce a
a) X b) wy e) wy"z
d) x + y e) zy +z f) xyf+w
756 Capítulo 15 Álgebra booJeana y funciones de conmutación

15.3
Aplicacio nes adiciona les:
Condiciones de indiferencia

Ahora nuestro objetivo es usar las ideas desarrolladas en las dos primeras secciones en 1111
variedad de aplicaciones.

Como responsable de un bazar de beneficencia. Paula deja su trabajo una tarde para bomcar
un pastel que será vendído en el bazar. Los siguientes miembros del comitl del bazar
ofrecen donar los ingrcdícntes necesarios, como se muestra en la tabla 15.17.

Tabla 15.17
Harina Leche Mant,quilla NU«<S Huevos
Susana X X
Dolores X X

Berta X X

Teresa X X

Rulh X X X

Paula envía a su hija Sarita a recoger los ingredientes. &cribiremos una expresiól
booleana para ayudar a Paula a determinar e l conjunto de voluntarias que debe tener ea
cuenta para que Sarita pueda recoger todos los ingredientes necesarios (y nada más).
Sean s, d. b, t y r cinco variables booleanas correspondientes, respectivamente, a las
cinco mujeres enumeradas en la primera columna de la tabla. Para obtener la harina, Sarila
debe visitar a Susana o Berta En la terminología booleana, podemos decir que la harina
determina la sumas+ b. Este término será parte de un producto de sumas. Para los demás
ingredientes, las siguientes sumas denotan las opciones disponibles.

le.che: b + t + r mantequilla: s + d + r nueces: d + r huevos: t

Para responder la pregunta planteada, buscamos una suma minimal de productos para
la función/(s. d. b, t, r) = (s + b)(b + t + r)(s + d + r)(d + r)t. Podemos obtener la respuesu
multiplicando todo y simplificando después los resultados. o mediante un mapa de
Karnaugh. Esla vez usaremos el mapa (que aparece en la labia 15.18).

Tabla 15.18

db\tr 00 01 11 JO db\tr 00 01 11 10
00 o o o o 00 o o 1 o
01 o o 1 o 01 o o 1 o
11 o o 1 1 11 o o 1 1
10 o o o o JO o o 1 1
(s =O) (s = 1)
15.3 A¡,licaciones adicionales: Condiciones de indiferencia 757

Comenzaremos con f como un producto (no mini:mal) de sumas. En consecuencia. pri-


mero debemos llenar los ceros de la tabla como sigue: En este caso. por ejemplo, s + b
queda representado por los ocho ceros de las filas primera y cuan.a de la labia, para, = O;
~ son las ocho asignaciones paras,~ h. t, r dondes+ b tiene el valor O; para t necesi-
tamos los 16 ceros en las primera, dos columnas de ambas tablas. Despufs de llenar los
ceros de las otras tres sumas del producto, colocamos un 1 en los nueve espacios restantes
para llegar a la labia mostrada. Ahora necesitamos una suma minimal de productos para
los nueve unos de la labia. Vemos que el resultado es ,n + sdJ + bn + dbt. (Vcriffquelo.)
Por lo tanto, Sarita puede ir a recoger los ingredientes de cuatro formas: puede ir con
Susana. Ruth y Teresa; o bien, con Dolores, Bena y Teresa; o bien, seguir cualquiera de sus
otras dos opciones.

En nuestta siguiente aplicación analizaremos cierta propiedad de los ¡ralos. Ya hemos


presentado esta propiedad eo el ejercicio complementario 12 del capítulo 11. En este caso,
sin embargo. nuestro tratamiento DO depende de dicha presentación anterior.

· · n 15.4 Sea G = (V, E) un grafo (no dirigido) con conjunto de virtices V y conjunto de aristas E.
Un subcoojuntoDde V es unconjwuodominanlt paraG si para todo u E V, u E Dou es
adyacente a u.o vértice de D.

Para el grafo que se muestra en la figura 15.9, los conjuntos {a, d), {a, e,<) y {b, d, <,/)
son ejemplos de conjuntos dominantes. El conjunto {a. c. e} es un conjunu, dominante
m.immal, ya que si se elimina cualquiera de los tres vértices a, e o e. los otros dos ya no
dominan el grafo. El conjunto {a, d) tambitn es mínima!, pero {b, d, •• f) no lo es, pues
{b, d, <) ya domina G.

Figuro 15.9

Para el grafo de la figura 15.9, los v~rtices representan ciudades y las aristas carreteras.
Queremos construir hospitales en alguna de estas ciudades, de modo que cada ciudad
tenga un hospital o sea adyacente a una ciudad que lo tenga. ¿De cuántas formas podemos
realizarlo, construyendo un número minimal de hospitales en cada caso?
Para responder esta pregunta. necesitamos los conjuntos dominantes minimales paraG.
Consideremos el v~rtice a. Para garantizar que a cumple nuestro objetivo, debemos cons-
ttuir un hospital ena. b, d of(pueslo queb, d y /son adyacentes aa). Por lo tanto, teoemos
un ~ a + b + d +f. Para que b satisfaga nuestras condiciones. generamos el ~rm.ino
a+b+ c + d.
758 (aphulo 15 Álgebra booleana y funciones de conmutación

Continuando con las otras cuatro localidades vemos que la respuesta es entonces una
sentación con una suma minimal de productos para la función booleanag(a, b. c. d, e,
(a+ b + d + fXa + b +e+ d')(b +e+ d')(a + b +e+ d + <'/(d + • + fXa +, + f). Usamos
propiedades de las variables booleanas para obtener

••Ca+6+4+J)(t+c+.O·
fil+•+J1P+•+J>
=«•+bc+(6+'Jlm+(-+J)]

"'f«+fa+6+. .+e+_n
m__,+..,.+tlll;f+,.,,,,+-,.,.~Jtl
+Mt+6ir+lf+.W.+4ii+.,
- ""+(tlll;f+~+of+tlj
+ (oal+a,l+-,+/M~+•+~

=""+ef+.,,+•+6/+M+,#

En consecuencia, en seis de los casos podemos lograr nuestro objétivo construy


solamente dos hospitales. Si a y e tienen más habitantes y queremos que los h
es~n en estas ciudades, entonces también tendríamos que construir un hospital en e.

La siguiente aplicación que analizaremos presenta las condiciones de indiferencia.

Las cuatro líneas de entrada para la n:d de puertas que aparece en la figura 15.10 propor-
cionan los equivalentes en binario de los dígitos O, 1, 2, . .. , 9, donde cada nwnero•
representa como abce (e es la cifra menos significativa). Construiremos una red de puerta
con dos niveles de puenas tal que la función de salida/sea igual a 1 para la enttadaqm
representa los dígitos O, 3, 6, 9 (es decir,!detecta los dígitos divisibles entre 3).

Figura 15.10

Antes de concluir que/= O para los otros 12 casos. analicemos la tabla 15.19, doade
aparece una ..X.. para el valor de/en los últimos seis casos. Estas combinaciones de eatradl:
no aparecen (debido a cienas restricciones externas), por lo que el valor de/en estos casos
es indifmmtt. Para tales casos, las salidas son no especificadas y/ está especificada tlt
manua incompleta. Por lo tanto, eseribirnos/=l:m(O, 3,6, 9) +d(JO, 11, 12, 13, 14, IS),
donde d(IO, 11, 12, 13, 14, 15) denota las seis condiciones de indiferencia para las fila
con las etiquetas en binario para 10, 11, 12. 13, 14, 15. Para buscar una rcpresentaciól
como suma minimal de productos para/, podemos usar cualquiera o todas estas condicio-
nes de indiferencia en el proceso de simplificación.
1 5.3 Aplicaciones adicionales: Condiciones de indiferencia 759

Tabb 15.19
a b e • f a b e • f
o o o o 1 1 o o o o
o o o 1 o 1 o o 1 1
o o 1 o o 1 o 1 o X
o o 1 1 1 1 o 1 1 X
o 1 o o o 1 1 o o X
o 1 o 1 o 1 1 o 1 X
o 1 1 o 1 1 1 1 o X
o 1 1 1 o 1 1 1 1 X

Utilizamos el mapa de Kamaugh de la tabla 15.20paracscribir/como unasumaminimal


de productos y obieocmos

¡ . ;;¡;c,; + k + be,+ ae.


El primer ,umando d e / ~ O; bce =noce 3, ya que representa 0011 (abce),
puesto que 1011 (abe•) no aparece. De la mis111J1 forma, usamos be, para =nocer 6, y
aepara9.Lafigura 15.11 proporciona los delalles iniemos (sin los invcrsorcs) dela figura
15.10. (Obocrve que en la tabla 15.20 hay condiciones de indiferencia no utilizadas.)

Tabla 15.20

ab \ ce

Q)@@
00 O! 11 10
00
01
ll X X X
10 1 X X

b - - . . r- -...
,--..__ _,
e

Figura 15.11

Terminamos la sección con otro ejemplo del uso de las condiciones de indiferencia.
Capítulo 15 Álgebra booleana y funciones de conmutación

Encontraremos una representación como suma mini.mal de productos para la func:i6I


booleana especificada de manera incompleta

f(w, x,y, z) = im(0, 1, 2,8, 15) + d(9, 11, 12).

Tabla 15.21

00
01
11
10

Consideremos el mapa de Kamaugb de la tabla 15.21. Como en el ejemplo anterior, calo


mintérmino está representado por un 1 en la tabla~cada condición de indiferencia se desi,-
na con una x. El l que representa WXyZ (en el extremo derecho de la primera fila) puede
simplificarse solamente de una forma, usando el 1 "adyacente" para w..r y z . Esto nos da
WXyf + WXyT = wi'f(y +y)= w..rz. De la misma forma. el 1 de la conjunción fundamemal
w.xyz sólo es adyacente a una X. la correspondiente a la condición de indiferencia W%)'l.
Esla adyacencia se simplifica como w..ryz + w.Xyz = ""'.YZ· Por último, los unos restantes de
las conjunciones fundamentales WXYz y w..ry z pueden usarse con el mintérmino para O
(a saber, WXyz) y la condición de indiferencia wiyz. Esto nos da Wiyz + w:iyf+ wxy,; +
wxy"z = (WZ + wt + WZ + wzJXY • (w + w)(z + z}ty" = xy.
[Observe lo siguiente:
l. En la tercera simplificación usamos la conjunClón fundamental wx'y'I por segunda
vez. También la habíamos usado en la primera simplificación, pues es adyacenrc a
z.
la conjunción fundamental W.Xy Sin embargo, no presenla un problema debido a
la ley de idempotencia de+.
2) No utilizamos la condición de indiferencia para wxyz'.)
En consecuencia. como suma minimal de productos,/(w,. ..r, y, z) = L m(0, 1, 2, 8, 15) +
= =
d(9, 11, 12) l: m (O, 1, 2, 8, 15) + d(9, 11) wxz + wyz + xy.

EJERCICIOS 15.3 1. Parasudécimocumpleai\os, Marta quiere regalara su hijo Juan algunos sellosdecorrcospwa
su colección. En la tienda encuenua seis paquetes diferentes (que llamaremos"'· u, w, x. y, l).
Los tipos de sellos de estos paquetes se muestran en la tabla l 5.22.
Determine todas las combinaciones minimales de paquetes que Mana puede comprar de
modo que Juan reciba algunas sellos de todas las regiones geograficas.
2. Vuelva a hacer el ejemplo 15.20 usando un mapa de Kamaugh con seis variables.
3. Determine si cada una de las siguientes proposiciones es verdadera o falsa Si es falsa, tW: UD
contraejemplo.
Sea G = (V, E) un grafo no dirigido con D1, D 1 ~ V.
a) Si D,. D, son conjuntos dominantes de G, entonces tambiái lo es D1 U D1-
n
b) Si D,. Dz son conjuntos dominantes de G, entonces tambitn lo es D 1 O¡.
e) Si D 1 es un conjunto dominante de G y D1 !:: Di, entonces D2 domina G.
d) Si D 1 U D 2 domina G. entonces al menos uno de los conjuntos Di, D2 domina G.
15.3 Aplicaciones adicionales: Condicione> de índfferencia 761

Tabla 15.22
-Unido& Europa AJia Áfric:a
u ✓ ✓
V / ✓

w ✓ /
.X /
y / /
z / /

4. Determine todos los conjuntos dominantes mini.males para el grafo G de la figura 1S.12.

g F'tgun1 15.12

5. Encuentre una repre$CObcióo como su.ma minimaJ de produaos para


•J f(w,x ,y, z) = l:m(l, 3, S, 7, 9) + d(IO, 11, 12, 13, 14, IS)
b) f(w, x,y, z) e l:m(O,S,6,8, 13, 14) + d(4, 9, 11)
e) f(v, w,x, y, z) = l:m (0,2,3,4,S,6, 12, 19,20, 24, 28) + d(l, 13, 16,29,31)
6. Las cu=o Uncas de cnttad.a para b n:c! de puertas de la fipra I S. 13 propo<cionan los equiva-
lentes en binario de los ni1metos O. I, 2, . •• , 15, doode cada nómero se representa como aba.
donde e es el bit menos significativo.
a ) Determine la f.n.d. de/, cuyo valor es 1 para abce primo y O en los dcmú casos.
b) Thbuje la red de puertas de dos niveles para/como una suma minimal de productos.
e) Nos informan que la red dada forma parte de una red más grande y que, por tanto, los
equ.ivale:otes en binario de los oómeros IOa 15 nunca son entradas de nuestra red. Disetie
una red de puenas de dos Divdcs para/en csw ci.reunstancias.

~
e ~ ~....,....
.,,... f

• "'""" Figura 15.13


762 Capitulo 15 Álgebra booleana y funciones de conmutación

15.4
La estructura de un álgebra
booleana (opcional)

En esta tíltima sección analizaremos la esuucrura de un álgebra booleana y determinare-


mos aquellos m. E z• para los que existe un álgebra booleana de m elementos.

Definición 15.5 Sea~ un conjunto no vacío que contiene dos elementos especiales O (el cero o eleIDCJ:a>
neutro) y 1 (el uno o elemento unidad), sobre el cual definimos las operaciones binarias
cerradas+,• y una operación monaria (o unaria)-. Entonces (91, +, ·,-,O, ! )es un6/g,b,a
bookana si se cumplen las siguientes condiciones para todos x. y, z E 9!.

a) .r+y=y+x a)' .ry=yx Leyes conmutativa


b) .r(y +z) =xy +xz b)' .r+yz=(.r+y)(.r + z) Leyes distributivas
e) x +O=x e)' .rl =.r · 1 =.r Leyes de identidad
d) .r+x=l d)' .ü=x·x=O Leyes de inversos
•l o+ 1

Como hemos visto en la definición 15.5. escribiremos con frccuencia.xy en vez dex ·1·
Cuando conocemos las operaciones y los elementos Oy 1,escribimos!i en vez de (!i, +, ·,
-.o, 1).
Por la experiencia anterior tenemos los siguientes ejemplos.

Si O\l es un conjunto (finito), entonces 91 =~(Oli) es un álgebra booleana, donde para A, 1


~O\l. tenemosA+B=A U B,AB=A n B,A = elcomplementodeA(enO\l)ydonde fes
el elemento cero y <u. es el elemento unidad.

Paran E z·, F. = ( /: /J' ➔ B). el conjuntodefuncionesbooleanasdenvariablesbooleaaas,


es un álgebra booleana, donde +,- y - están dados en la definición 15.2, donde e l cero es la
función constante Oy la función constante 1 es el uno.

Consideremos ahora un nuevo tipo de álgebra booleana.

Sea~ el conjunto de todos los divisores enteros positivos de 30: !il_= { l . 2. 3, 5, 6, 10, lS.
30}. Para cualesquiera.<; y E 9!, definimosx + y =mcm(x, y); ry = mcd(.t; y); y x = JOIL
Entonces, 1 es el elemento neutro y 30 el elemento unidad y podemos verificar que (!i, +,
·, -, 1, 30) es un álgebra ·booleana. Estableceremos una de las leyes distributivas para CSII
álgebra booleana y dejaremos las otras condiciones para que el lector las verifique.
. 15.4 La estructura de un álgebra booleana (opcional) 763

Para la primera ley distributiva. queremos mostrar que

mcd(x, mcm(y. z)) = mcm(mcd(x, y), mcd(x, z))

para x. y, z E ~. Para esto, escribimos


x=2t131<25«>, y=T"lJ"':5"'3, z = 2"1 3"15"3 ,
donde O s k;. m;. n; ::s 1 para todo 1 :s i ::s 3.
Entonces mcm(y, z) == 211 3~ Y•, dondes; == máx{m;, n;} para tcxJ.o 1 ::si ::s 3 y entonces
= =
mcd(x, mcm(y, z)) 2'• 3• 5", donde l; mín{k;, máx{m, n,} }, para todo 1 s i s 3.
Además, mcd(x, y) = 2"• 3~ 5~. donde u;= mín{k, m,} cuando 1 s i s 3 y mcd{x, 4) = 2.,
30, 5.,, con U; = mín{k,, n,}, para todo 1 s i s 3. Asf, mcm(mcd(x, y), mcd(x, z)) = 2"l 3•,
5...,, donde w; == máx{u;. U;}, para todo 1 si :5 3.
Porlotanto, para cada i E{l, 2, 3}, w;= máx{u;, u ,} = máx{mín{k,, ,n,}, mín{k;, n,} } y
t1 == mín {Je.. máx{m;, n1}}. Para verificar el resultado, necesitamos mostrar que w; = t; para
todo 1 sis 3. Sik.=0, entoncesw, ==0 ==l,- Sik;= 1, entonces w1 -=máx{m;. n;} = t ;. Esto
abarca t<Xlas las posibilidades, de modo que w1 == t, para 1 s i s 3 y

mcd(x, mcm(y, z)) = mcm(mcd(x, y), mcd(x, z)).

Si analizamos un poco más este resultado, vemos que podemos reemplazar 30 ¡x,r cua1-
quier número m =p 1 p2 p3 , donde p 1, p¡, p 3 son primos distintos. De hecho, el resultado es
válido para el conjunto de todos Jos divisores de Pi Pz ... p., un producto de n primos
distintos. (Observe que dicho producto no tiene cuadrados; es decir, no existe k E z•.
k>
1, tal que le' lo d1v1da.)

Haremos una observación en cuanto al cálculo proposicional. Si p, q son dos proposicio-


nes primitivas, podríamos pensar que la colección de todas las proposiciones que se obtie-
nen de p, q usando V, /\ y - deberían formar un álgebra booleana. Después de todo, sólo
hay que ver las leyes de la lógica y la forma en que se corresponden con los resultados de
la teoría de conjuntos y las funciones booleanas. Existe una diferencia fundamental: en
nuestro estudio de la lógica vimos. por ejemplo, que p/\q <=> q /\ p, no que p/\q == q/\p.
Para evitar esto, definimos una relación ~ en el conjunto S de todas las proposiciones
obtenidas de esta forma a partir de p. q, donde s 1~ s1 sis1 <=> sr Entoncesgj: es una relación
de equivalencia enSy divide a S, es este caso, en 16 clases de equivalencia. Si definimos
+,..:....:J - en estas clases de equivalencia como [s1] + [s 2] = [s1 V s 1 ], [s11[s2] == (s1 /\s2]
y {s1 ] = [Si] y si reconocemos [TJ como el elemento uno y [FJ como el elemento cero,
entonces obtenemos un álgebra booleana.

La definición de álgebra booleana tiene nueve condiciones, aun cuando en las listas de
propiedades de la teoría de conjuntos, la lógica y las funciones booleanas ya enumeramos
19 propiedades. ¡Podría haber más! Sin duda, hay una forma de obtener las demás propie-
dades y otras no incluidas entre las 19, a partir de las dadas en la definición.

MA 15.1 Las leyes de idempotencia. Para x E ~. un álge bra booleana, (i) x + x = x; y (ii) xx = x.
764 Capítulo 15 Álgebra booleana y funciones de conmutación

Demostración: (A la derecha de cada igualdad que aparece en la demostración, bemol


anotado la letta de la condición de la definición 15.5 que la justifica.)

1) x=x+O e) Ü) x=x·l e)'


= x+ñ d)' =x(x + i') d)
= (x +x)(x +i') b)' =..u+.ü' b)
=(x+x) · I d) =.u+O d)'
=x+x e)' =.u e)

AJ demostrar este teorema, podemos obtener la demostración de la pane (ii) a partir de


la correspondiente en la parte (i). cambiando todas las ocurrencias de+ por• y viceversa.
así como todas las ocurrencias de Opor 1 y viceversa.Así mismo. las justificaciones de los
pasos correspondientes forman un par de condiciones en la definición 15.5. Como antes.
estos pares son duales entre sí; la condición (e) es autodual. Esto nos lleva al siguiealt
resultado.

TEOREMA 15.2 El principio de dualidad. Si s es un teorema relativo a uñ álgebra booleana y s puede


demos= a partir de las condiciones de la definición 15.5 y de ottas propiedades obfe.
nidas a partir de las mismas concLiciones, entonces su dual r" también es un teorema
Demostración: Seas tal teorema, Si dualizamos todos los pasos y razones en la demOSU>
ción des (como en la demostración del teorema 15.1), obteoemos una demostración para,'.

Ahora enumeraremos algunas otras propiedades de un álgebra booleana. Demos~


mos algunas de ellas y dejaremos el resto de las demostraciones al Lector-.

TEOREMA 15.3 Para cua1quier álgebra booleana .fi, si x.y E .fi, entonces

a) x·O=O a)' X+ 1 = 1 Leyes de


dominancia
b) x(x+y)=x b)' x + xy=x Leyes de
absorción
e) [xy=xz y fy=i'z) ⇒y = z Leyes de
cancelación
c)'[;c+y=x + z y i'+y=i' + z] ⇒y=z

d) x ( yz)=(xy)z d)' x+(y+z)=(x+y)+z Leyes


asociativas
•> [x+y=l y xy = o) ⇒y = i' Unicidad de los
inversos
C) i=x Ley del doble
complemento
g) iy=i'+y g)' x+y=xy Leyes de
De Morgan
15.4 La eslructura de un á booleana (opcional) 765

b) O= 1 b)' 1=0
1) xy =O si y sólo l)' x+Y • l si y sólo
si.ey • x if.r + y•z
Demostración:
a) x·0=0+x·0, por la definición IS.5(c), (a)
=x·f'+ .t·O, por la definición 15.S(d)'
= x ·(1'+0), por la definición 15.S(b)
=.t·:f, por la definición 15.S(c)
=O, por la definición 15.5(d)'
a)' Se sigue de la pane (a) y del principio de dualidad.
e) En estecaso,y = 1-y= (x+:i)y=xy+ i'y=xz+i'z=(x+i')l = l ·z=z(Verifique
todas las igualdades.)
e)' Bte es el dual de la parte (c).
d) Para esw>lecer el ttSUltado, usamos (e)' y llegamos a la conclusión mostrando que
x + [x(yz)] = x + [(xy)l] y i' + [x(yz)] = i' + [(xy)l]. Usando la ley de absoo:ión
vemos que x + [x(yz)J =x. De la misma fonna. x + [(xy)l] = [x + (xy)J(x + z) =
x(x + z) = x. Entonces i' + [,(J?)l = (i' + i)(i' + J?) = 1 • (i' + yz) = i' + yz, mientras
quei' + (w)z] = (i' +xy)(i' +z) = ((i' + x) · (i' + y))(x.+ z) = ( l · (i' + y))(i' +z) =
(i' + y)(i' + z) = i' + J? (verifique todas las igualdades.)
El ~ultado se sigue entonces de la ley de cancelación de la parte (e)'.
d)' Por fortuna. tste es el dual de la pane (d).
e) En este caso tenemos X =X+ O=X +.ty• (x +x)(x +y)• l· (x +y)= (i' +y) · 1 =
(i' + yXx +y)= i'x +y= O+ y= y (Verifique todas las igualdades.)
Observamos que la proposición (e) es autodual. La proposición (f) es un coro-
lario de (e), puesto que i y z son complementos (inversos) de X.
e) Este resultado se sigue de la parte (e) si podemos mostrar que i' +yes un comple-
mento dexy.

ry + (l' +Y) = (ry + 1') +y = (x + l')(y +x) +y


= l·{y +x)+Y • (y +y) +x = 1 +l'• 1.
ademú, ry{.f + Y) • (ry.f) + (.IYY) = ((.ü)y) + (x( yy)) = O• y + x •O=
0+0 • 0.
En consecuencia, i +Yes un complemento dcxy, y por la unicidad de los complemen-
tos, se sigue que xj = X + y.

¡Ya tenemos suficientes demostraciones por el momento! Ahora vamos a analizar la


fonna de imponer un orden sobre los elementos de un álgebra booleana. De hecbo, quere•
mos un orden parcial, por lo que veremos ahora el diagrama de Has.se.
Comencemos por analizar los diagramas de Has.se de las siguientes ilgebras
booleanas.

a) @'('11), U, n. -. e, '11) donde 'll= {l. 2, 3) y el orden parcial es inducido por la


relación de inclusión.
766 Capítulo 15 Álgebra booleana y funciones de conmutación

b) ('::/, +, •, -, 1, 30), donde El= ( l. 2, 3, 5, 6, 10, 15, 30), x +y = mcm(.t, y). Z1


mcd(x, y) y X= 30/x. En este caso. el cero es el divisor 1 y el e lemento unidad es
divisor 30. La relación~ en ~. definida como x 51: y si x divide a y hace de';:/
conjunto parcialmente ordenado.

La figura 15.14 muestra los diagramas de Hassc de estas dos álgebras booleanas. Si•
prestamos atención a las etiquew en los véffices de cada diagrama, vemos que las eSIJ\l0o
turas subyacentes son iguales. Por lo tanto, esto indica el is.orru:,rjisrno de álg~bras boolmn&
Estos ejemplos también implican otraS dos ideas.

1) ¿Podemos ordenar parcialmente cualquier álgebra booleana finita?


2) Si observamos la figura 15.14(a), vemos que los elementos no nulos justo por ani-
ba de f!I son tales que cualquier otro elemento distinto de fJ puede obtenerse C<a)
sumabooleana decstostres. Por ejemplo, {! , 3) = { I} U {3} y {I, 2, 3) = ( 1) U
{2} U (3). Para lapane(b), los números 2, 3 y 5 son tales que todo dívisordislia
de 1 se obtiene como la suma booleana de estos tres. Por ejemplo, 6 = mcm(2, 3) J
30 = mcm(2, mcm(3, 5)).

¡,,,.3}:q,

6&15
30

1, . , ¡ w¡,_3¡

¡,¡~ ¡3¡ 2~5

1
(a)
1 1- 0 (b)

Figura 15.14

Ahora comenzaremos a trabajar fonnalmente con estas ideas.


En el capítulo 3, al trabajar con conjuntos, relacionamos las operaciones U, n y- coa
la relación de inclusión por medio de la equivalencia de las siguientes proposiciones: (a)A
(;; B; (b) A n B =A; (e) A U B =By (d) ii e; A, donde A, Be; '1.1. Abora usaremos bs
partes (a) y (b) para tratar de ordenar un álgebra booleana.~

Definició n 15.6 Si x. y E ~. definimos x :S y si xy = x.

Así, hemos definido un nuevo concepto ( S) en ténninos de conceptos que teníamos en


~ (· y el concepto de igualdad). ¡Podemos entonces construir definiciones! Pero, ¿sirve
esto de algo?
15.4 la estructura de un álgebra booleana (opóonaQ 767

MA15.4 La relación ":S" recitn definida es un orden parcial.


Demostración: Como .a = x para cualquier x E 91. tenemos que x :S :e y la relación es
reflexiva Para establecer la Sl'ltisimetría. supongamos que x. y E !i con x :s y y y s x.
Entonces.ry= xy JX "" y. Por la propiedad conmutativa..ry• yx y x = y. Por11ltimo, si:c s
y y y :5 z. eotonces .I)' = x y y¡:= y, podo que x = iy = .t(y¡:) = (iy)t = x,; como x = ,z,
tenemos que x s t y la relación es transitiva.

Ahora podemos dar un orden parcial a 10da álgebra booleana y observamos que para
cualquier x eo un álgebn booleana. O :5 x y x :5 l. (¿Por q~?) Sin embargo, aotes de
continuar, consideremos el álgebn booleaoa formada por los divisores de 30. ¿Cómo apli-
camos el teorema 15.4 en este ejemplo? En este caso. el orden parcial eslá dado por x S y
si .I)' = x. Como iy es mcd(x, y), si mcd(x, y) = x, entonces x divide a y. Pero ktc era
precisamente el orden parcial que tenía el álgebra booleaoa al empezar_

Con este conc:epto de orden parcial, regresaremos a las observaciones que hicimos an-
tes acerca de los elementos en los diagramas de Hasse de la figura 15.14.

Sea O e l elemento neutro de un álgebra booleana 9J. Un elemento x E 9J, x f. O, es un


4tomode 9J si para todo y E 9J, y :5 x => ya O o y = x.

a) Para el álge bn booleaoadetodoslossubconjuntosde"U = { 1, 2, 3), lositomos son


{l }, {2} y {3).
b) Al trabajar con los divisores enteros positivos de 30, los átomos de esta álgebn
booleana son 2, 3 y 5.
e) Los átomos del álgebn booleaoa F.= lf: 8" ➔ B} son los miotirmioos (o coojun-
ciones fundamentales).

Los átomos de un álgebn booleaoa finita satisfacen las siguientes propiedades.

TEOREMA 15.5 a) Si x es un átomo de un álgebra booleana 9J, entonces para todo y E ,11, .I)' • O o xy = x.
b) Six., x2son átomos de B y x, t-x:. entonces x.x1= O.
Demostración:
a) Parax, y E 9J,xy :5 x, pues(xy)x=.l(y;c) =.l(xy) = (.a)y ~xy. Si.res un '1omo,xy :5
x=>xy•Ooxy=x.
b) Esto se sigue de la parte (a). El lector debe proporcionar los detalles.

lEOREMA 15.6 Si xi' x1 , • . . • x. son todos los {tomos de un álgebra booleana !i y x E fl con xx1 = Opara
todo I s i ~ n, entonces x = O.
768 Capitulo 15 Álgebra booleana y funciones de conmutación

Demostración: Si xf,O, sea S = {y E 9110 <y:,; x}. (O <y denota O:,; y y Of, y.) Como
x ES, S #- 0. Como S es finito. podemos encontrar un clementoz en ~ tal que O< z :!. xsi:a
que otro elemento de ~ esté entre O y z. Entonces z es un átomo y O = xz: = z > O. F..511
posibilidad nos lleva a una contradicción. por lo que no puede ocurrir que x-:/= O~ es decir.
x=O.

Esto nos lleva al siguiente resultado acerca de una representación.

TEOREMA 15.7 Dada un álgebra booleana 9J con átomos xt' X,, .. , x., entonces cualquier x E 93, x #- O,
puede escribirse como una suma de átomos de manera dni~ salvo el orden.
Demostración: Comox f,O, por el teorema 15.6, S = {x,1.o:,f, O} f,6. Sean S = {x,, x,,, ..
. ,x..} yy=x;1 +~ + • • • +x.t. Entoncesxy=.x(.x.1 +xi: + • - • +x;) =xx,1 +.ui: + • • •+~
= x,1 + xi: + · · · + X;,, por el teorema 15.S(a) y xy == y.
Consideremos ahora (X)'Jl'., para cada I sis n. Six, ~ S, entonces.o;= Oy (X)'JX,=0.
Par3.X;ES, tenemos(xy}X;=.a,(x,. + xiz +· · ·+ x.¡) =.n,(X,¡ Xi:! · · · i i.t) =x(.x¡ X,Xz),doode.t
es el producto de los complementos de todos los elementos-en S - {x.}. Comox;:X1 ;:;0, se
sigue que (xyµ,= O. Así, (X)'Jl'., = O para todo x, tal que 1 :,; i :S n. Por el teorema 15.6,
tenemos que xy = O.
Comoxy = y y xy = O, se sigue quex =X· 1 = x(y + y¡ = X)'+ xy =xy +O= y =x,, +.r;,
+ • • • + x,•• una suma de átomos.
Para mostrar que esta ·representación dex es llnica, salvo el orden, supongamos que .r =
Xj1+X,: +•··+X;r
Si xii no aparece como sumando en X;1 + xi: + • • • + X¡1, entonces x¡, = X¡1 • X¡1 = Xj-1 (xii +
xh + • • • +xit[por e1 teorema 15.5(b)] =xifx =x1i<x.1 +xi: + • • • + x;~) = O [de nuevo, pord
teorema 15.5(b)]. Por lo tanto, X;¡ debe aparecer como sumando en x,1 + xiz + • • • +x,._,al
igual quex>,i, . .. ,x~. Asf, f 5 k. Por el mismo razonamiento, obtenemosk. :s f y vemos que
las representaciones son idénticas, salvo el orde n.

Por este resultado, vemos que si !i es un álgebra booleana finita con átomos X1, x:,
x
... ,x11, entonces cada E~ puede escribirse en forma única como k.,c,x,,
donde cada
c; E {O, 1}. Si e,= O, esto indica quex, no está en la representación der. e,= 1 indica que
sí está. En consecuencia,cadax E 91 tiene asociada unan•upla (c1, c2 , •• • ,e,.) y ex.isten2"
de tales n•uplas. Por lo tanto, hemos demostrado el siguiente resu1tado.

TEOREMA 15.8 Si 9J es un álgebra booleana finita con n átomos, entonces l911 = 2•.

Debemos resolver una última cuestión. Si n E Z♦• ¿cuántas álgebras booleanas difereo.
tes de tamaño 2• ex.isten? Si observamos los diagramas de Hasse de Ja figma 15.14, verc•
mos dos imágenes diferentes. Pero si hacemos caso omiso de las etiquetas de los vértices,
las estructuras subyacentes son las mismas. Por lo tanto, estas dos álgebras booleanas soa
idénticas o isomorfas en un sentido abstracto.
15.4 La estructura de un Algebra booleana (opcionaO 769

· ·ón 15.8 Sean (si,,+ . •, - .O, 1) y (si,,+,•, - , O, 1) álgebras booleanas. Entonces 911,, 911, son
isor,wrfassiexiste unacorrespondenciabiycctivaf: ~ 1 ➔ !i2 talque paratodo.r.. y1 E ~ 1,

a) f(x, + y,) = f(x,) + f(y,)


1 1
(e,,a,1 t-i-,J
b) f(x, ·y,)= f(x,) •f(y,)
1 1
{«1.,) (fl'lll:z)

e) //J,) = /(x,) [En/(x,) tomamos el complemento en 911,, mientras que para /(x,)
tomamos el complemento en ~-1

Tal función/p~sen,a las operaciones de tas estructuras algebraicas.

tS..a Para las dos álgebras booleanas de la figura 15.14, definimos/como


f : 0-+I f: {2}-+3 f: {1, 2}-+6 /: {2, 3}-+ 15
f:{1) -+ 2 f: {3}-5 f: {l,3}-+ 10 /: {1,2,3}-+30
Observemos Jo siguiente:

a) Los elementos neutros se ~ n mediante/, al igual que los elementos unidad.


b) f({I} U {2}) = f({I, 2}) = 6 • mcm(2, 3) = mcm(/{ 1 }), /({2}))
e) f({l,2Jn {2,3}) • f({2}) • 3 = mcd(6, 15) = mcd(/{ 1, 2}), /((2, 3}))
=
el) f(IiJ) • f({l,3}) 10 = 30/ 3 3 f({2}) = =
e) Laimageo decu.alquier átomo((!}, {2}, (3})es un átomo (2, 3, 5, respectivamente).

Esta función es un isomorfismo. Una vez establecida la correspondencia entre los ele-
mentos neutros y los itomos respectivos. las demás correspondencias quedan determina•
das por el teorema 15.7 y la preservación de operaciones mediantef.

A partir del ejemplo anterior obtenemos nuestro último resultado.

TEOREMA 15.9 Cualquier álgebra booleana fioim 911 es isomorfa a un álgebra booleana de conjuntos.
Demostración: Como 911 es fioita, si tienen átomosx, 1 :Si :S • y 19111 = 2•. ScanGU= {1,
2, ... , •I y 9'~ el álgebra booleana de los subconjuntos de 6ll.
Definimos/: 911 ->9'('\Qcomo sigue. Para cadax E 911, se sigue del teorema 15.7 que
podcmosescribirx= 4.,c x
1 1, donde cadac, es Oo l. [En estecaso,c1 E {0,1 }( =B) y para
cualquier átomo a en gi, c1a =O(el elemento neutro en~) si e;= O, mientras que C; a = a
si C¡ = l.] Entonces definimos

f(x)•{il l si:Sn y c,•I}.


770 (ap~ulo 1S Álgebra booleana y funciones de conmutación

[Por ejemplo, (1)/(0) = 0; (2)/(x,) = {i) paracadaátomox,. donde 1 sis n; (3)/(x1 +


{ 1, 2); y (4)/(.x,+x, + .<,) = (2, 4, 7).J Consideremos ahorax,y E !i, conx = 4-i<i-',
y= 4-Ax,, dondec,,d,E {O, 1) para todo 1 s i s n.
Primero vemos quex+ y= };..s;x;, dondes¡=c, +d, paracada 1 sis n. (R.ec:onl►.
mosque 1 + 1 = 1 en este caso.) En consecuencia,

/{x+y)={il lsisn y s1 =1}


={i J! sisn y, c,=lo d,=1)
={i /lsisn y c1 =l}U{i/ lsisn y d, =1}
= f(x) Uf(y).

El teorema 15.5 (b) señala que

X· y = ±t¡X¡,
,_,
donde t; = C; d¡ para todo 1 s i s n y de manera similar obtenemos

f(x ·y)= {i /1 si sn y r, = 1}
= {i /lsisn y, e,=1 y d,= 1)
= {i /lsisn y c, =l}n{i/ls isn y d1 =1}
= f(x) nf(y).

Para completar la demostración de que/es un isomorfismo, primero debemos observar


que si.x= 4'...
c¡X¡, entonces X = 4-
1c;.x;. Esto se sigue de los teoremas 15.3(e) y 15.S(b).
pues
11 11 11 11

Lc,x,+ L cix,= L(c,+ ci)x,= Lx,= 1


i• l i• I l •I i•I

(¿Por qué es cierto esto? Véase el ejercicio 17 de esta sección.) y

Ahora vemos que

/(i')={i /lsis n y ci = l}
={i /l SiSn y c,=O)
= {i / lsisn y c, = l}
=f(x),
por lo que la función/ preserva las operaciones en las álgebras booleanas ~ y ll'('U).
Dejamos al lector los detalles de mostrar que/ es inyectiva y sobre, lo que establece que
/ es un isomorfismo.
15.4 la estructura de un álgebra booleana (opcional) 771

1. Verifique la segunda ley distributiva y las leyes de identidad y del inverso para el ejemplo
15.25.
2. Complete la demostración del teorema 15.3.
l . Sea~ el conjunto de los divisores enteros positivos de 210 y defina+,• y para 8 como
x+ y = mcm(x. y),x · y =xy = mcd(x. y) y X= 210/x. Determine lo siguiente:
a) 30+5 -7 b) (30+5) ·(30+7) e) (14+ 15)
d) 21(2 + W) e) (2+ 3) + 5 f) (6 + 35)(7+ 10)
4 . Vunos que para un álgebra booleana 8 , la relación "S" en B dada por x s y si xy = :e, es un
orden parcial. Demuestre que (a) si .x :S y, entonces x + y= y; y (b) si x s y entonces Y:s i.
5. Sea (B, +, ·,-,O, 1) un álgebra booleana parcialmente ordenada por s.
a) Si w E 9$ y wsO. demuestre que w = O.
b) Si w E 9$ y Isx. demuestre que w = 1.
e) Si~ z E 93, con y s zey s Z, demuestrequey=O.
6. Sea (91, +, ·, - ,O, l)unálgebrabooleanaparcialmenteordenadapor s. Si w. x, y, z E~. con
w s xy y s z. demuestre que (a) wy s xz; y(b) w + y s x+ z.
7. Si 93 es un álgebra booleana. parcialmente ordenada por s y x. y E 93, ¿cuál es el dual de la
proposición .x S y"'?
04

8 . Sea F. = lf: Ir ➔ B} el álgebra booleana de todas las funciones booleanas de n variables


booleanas. ¿Ctúntos '1omos tiene F.1
9. Verifique el teorema 15.5(b).
10. Dada un álgebra booleana 9l, un subconjunto no vacío 9111 de 9J es una subáJg~bra si para
todosx, y E 91,, cenemosquex + y.xy•.i E 93 1•
a) Demuesueque0,1 E 91 1 para un subálgebra 9:1 1 de 91. (Por lo tanto, J9J 1 ( 2: 2.)
b) Encuenue dos ejemplos de subálgebras en el ejemplo 15.25.
e) Para 'U= {1, 2, 3} y 9l el álgebra booleana de subconjuntos de ~ ¿cuántas subálgebras
diferentes podemos encontrar?
d) Muestre que la definición anterior puede reducirse si sólo pedimos que cuando x, y E fi1o
entonces x + y y X E fi1.
e) Si 931, ~ son subálgcbras de 91, ¿es~, n !i:z una subálgebra?
11. Dada un álgebra booleana 91, sea 0 'f'- 911 S: 93. Si O, 1 E 931 y fii1 es cerrada en + y ·, ¿es §11 una
subálgebra de 91?
12. Si 93 es una álgebra booleana.demuestre que el cero y el uno de 93 son únicos.
13. Sea f: 91, ➔ !i, un isomorfismo de álgebras booleanas. Demuestre lo siguiente:
a) f(O) = O.
b)f(l) = l.
e) Six, y E 931, conx s y, entoncesf(x) :S/{y)en 93-i.
d) Six es un ál:omo de 91., entonccs/(x) es un átomo en Si½.
e) Si 9'1 es una subá1gcbra de 911, entonces/(:f1) es una subálgebra de !ilz.
14. Sea 9J1 el áJgcbra booleana de todos los divisores enteros positivos de 2310. y~ el álgebra
booleana de los subconjuntos de {a, b, e, d. e}.
a) Definaf: 91, ➔ 91,comof(2) = {a},f(3)= {b}.f(5)= {c}.f(7)= {d},f(ll)= {•}. Para que
/sea un isomorfismo, ¿cuáles deben ser las imágenes de 35. 110. 210 y 330?
b) ¿Cuántos isomorfismos diferentes podemos definir entre ::J.11 y 932?
15. a) Si 91,. 91, soo~ehras booleanas yf: 9l, ➔ 9l,es inyectiva.sobre y tal quef(x+ y) =f(x) +
/(y) y/~ =/(x). para toda x, y E 93., demuestre que/es un isomorfismo.
b ) F.nuncie y demuestre otro resultado análogo al de la parte (a). (¿Qué principio aplicamos
aquf?)
772 capitulo 15 Álgebra booleana y funciones de conmutación

16. Demuestre que la función/del teorema 15.9 es inyectiva y sobre.


17. Sea 9J un álgebra booleana finita conn átomosx1, X:. . •• , x.. (Así, 1911=2•.) Democstreqae
1 •x, + x2 + · · · +z•.

15.5
Resumen y repaso histórico

El concepto moderno de álgebra abstracta fue desarrollado por George Boole en su est.
dio de los sistemas abstractos generales, opuestos a los ejemplos particulares de tales m-
temas. En su obra publicada en 1854, An /nvestigation oftire laws ofTlwught, fonnuló la
estructura matemática que ahora llamamos álgebra booleana. Aunque en el siglo XIX tenía
u.na naturaleza abstracta, el estudio del álgebra booleana fue analizado en el siglo xx pr
su valor en varias aplicaciones.
A partir de 1938, Claude Elwood Shannon (1916-) hiro su primera contribución im-
portante al álgebra booleana aplicada en (8]. Diseñó el álgebra de las funciones de
conm.utaci6n y mostró su relación con el álgebra de la lógica. Durante las décadas de 1940
y 1950 se hicieron más desarrollos en esta área, en el artículo de C. E. Shannon [9] y ea d
informe del laboratorio de computación de la Universidad de Harvard [10]. (El término
computacional bit fue acuñado por Claude E. Shannoo, quien también fue uno de los
primeros en representar Ja información en términos de bits.)

CJaude Elwood Shannon (1916- )

Vl.Dlos que podemos representar las funciones de conmutación por medio de sus formas
normales conjuntiva y disyuntiva. Estas formas nos permiten escribir tales funciones ea
fonna compacta por medio de etiquetas en binario. El proceso de minimización nos mos-
1S.S Resumen y repaso histórico 773

tró cómo representar una función booleana dada como suma minimal de productos o como
producto minimal de sumas. Con base en el método de mapas de E. W. Veitch (11], desa-
rrollamos la modificación de Maurice Kamaugh [4] como un método gráfico para la sim-
plificación de las funciones booleanas. Otra úcnica que meocionamos en el texto fue el
algorinno de tabulación conocido como el método de Quine-McCluskey. Este método fue
desarrollado originalmente porWtllardVan Orman Quine(1908-) (6,7] y modificado por
Edward J. McCluskey, Jr. (1929-) [S]. Es muy útil para funciones con más de seis varia-
bles y se presta para su implementación en un computador. El lector interesado en los
mapas de Karnaugh puede leer el capítulo 6 de F. Hill y G. Peterson [3]. El capítulo 7 de
[3] ofrece un excelente tratamiento del método de Quine •McCluskey. A. Friedman y P.
Menon [2] analizan las redes de puertas a la luz de la tecnología contemporánea. mientras
que T. Booth [ 1] analiza aplicaciones más específicas del diseiio lógico en el esrudio de los
computadores.
Aunque la mayor pane de este capítulo fue de naturaleza aplicada, la sección 15.4 nos
mostró un análisis de la estructura de un álgebra booleana. A diferencia de los anillos
conmutativos con elemento unidad, que tienen todos los tamaños posibles, vimos que un
álgebra booleana sólo puede tener 2" elementos, donde n E z •. La unicidad de la represen•
tación surgió aJ encontrar los átomos de un álgebra booleana, que se usan para construir el
resto del álgebra (excepto por el eeto). El álgebra booleana de lps conjuntos, que estudia-
mos en el capítulo 3, representa todas las álgebras booleanas, en el sentido de que un
álgebra booleana con n átomos es isomorfa al álgebra de textos los subconjuntos de {1, 2,
3, ... ,n}.

BIBLIOGRAFÍA

1. 800th, TayJor L .• Digital Nttworks and Computtr Sysrtm.s, r ed.• Nueva York:, Wtley, 1978.
2. Friodnwl, Arthur D., y P. R. Mcnon. Theory ond Ik.sign of Switching Circuits. Woodland
Hills, California Computer Science Prcss, 1975.
3. Hill, Frcderick J. y.(Jerald R. Peterson, lnrroduc.tion to Switching Theory and Logical Dtsign,
3• cd.., Nueva York, Wiley, l 981.
4. Kamaugh. Maurice, "'1bc Map Method for Syntbesis of Combinatoria] Logic Circuits",
TronsacrWns o/ tM AIEE, parte l. vol. 72, núm. 9, 1953, págs. 593•599.
S. McCluskey, Edward J. Jr., ..Minimization of Boolean Functions" ,Bell Sy~em Tuhnkal Joumal.,
35, núm. 6, noviembre de 1956, págs. 1417-1444.
6. Quine, Willard V., "'Tbe Problem of Simplifying Truth Functions", American Malll4matical
Mon1hly. 59, núm. 8 octubre de 1952, págs. 521-531.
7. Quine. Willard V., ..A Way to Simplífy Truth Functions", ÑMrican Mathematicol Monzhly,
62, núm. 9, noviembre de 1955, págs. 627-631.
8. Shannon, Claude E., ..A Symbolic Analysis ofRelay and Switching Circuits.., Tran.soction.s of
th< AJEE, vol. 57, 1938, págs. 713-723.
9. Shannon, OaudeE., 'íhc: Synthesis oíTwo-tcrminal Switching Círcuits",&USysttur1 Tuhnical
Jou.mal, vol. 28, 1949. págs. 59-98.
10. Personal del laboratorio de computación. Syn1Msis of Ehctronic Compuríng and ConJrol
Ci,cuirs, Annals 27, Cambridge, Mass., Harvard Univcnity Pres.s. 1951.
11. Veitch. E.W., ..A Chart Mcthod for Simplifying Truth Functions". Procetdings ofthe. ACM.
Pittsburgh, Penn., mayo de 1952, pág~ 127-133.
774 capítulo t 5 Álgebra booleana y funciones de conmutación

10. En la red de retroalimentación de la figura 15.15, -


EJERCICIOS mos un retraso unitario (una de las mkluinas de estados fi-
COMPLEMENTARIOS nitos del capítulo 6). Este dispositivo hace que el co_mplc-.
mento de la salida/al tiempo t se retro.alimente en lapoc:111
ANO, junto con nuevos valores de las entradas x. y ea 11
1. a ) Sean ~ 2. Si x. es una variable booleana para 1 S
siguiente unidad de tiempo o pulso de reloj . t + l.
i s n, demuestre que

i) (x1 +x2 +···+x,.)=X1i2. · · · X,.


Ü) (X1X:·· · x,.)= i"1+ X2. +· ·· +%,.
=
üi) a : '- 1X;><Il7-1X.-) X1:Í2 + X2 i"J + XJX•
+ • •• + x,._1 z,. + x,.X1
b) Enuncie el dual dela proposicióo (ili)de la pane(a).
2. Seanf.g: B' ➔ B funciones booleanas, donde/;l:m(I,
2, 4 , 7, x) y g; l:m(O, 1, 2, 3, y, z, 16, 25). Si/ ,s; g, ¿qué
sonx.,y,z? ..,,..,
3. Elena organiza una fiesta y tiene que decidir si invita a unitario
cinco amigas. Fig u ra 15.15
a) Si invita a Margarita., también debe invitar a Juana.
b) Si invita a Catalina, también tiene que invitar a Complete la tabla 15 ."23, la cual proporciona la salida/
NataJia y a Margarita. en cada instante t para los valores dados de x y y.

e) Puede invitar a Carmen o a Juana. pero definitiva-


mente no a ambas. Tabla 15.23
-
d ) Ni Catmen ni Natalia irán si oo invita a las dos.
1 o 1 2 3 4 5 6
e) Debe invitar a Catalina, a Nacalia o a ambas.
Dctemtine los subconjuntos de estas cinco amigas que
X o o 1 o 1 1 o
EJcna puede invitar cumpliendo las condiciones (a) a (e).
y o 1 1 o o 1 1
f 1 1 ? ? ? ? ?
4. Sean.f. g: B' ➔ B tales que/; l:m(2, 4, 6, 8) y g ;
l:m(I, 2, 3, 4, 5, 6, 7, 8, 9, 11, 13, 15). Encuentre una fun-
ción h: B'- ➔ Btal quc/=gh. 11. Paran variables booleanas, hay zt' funcioncsboolcanal,
S. Sea 9! un álgebra booleana parcialmente ordenada por cada una de las cuales puede representarse mediante om
S . Six, y, z E 9J, demuestre quex+ y S z si y sólo si.r :s z tabla de función.
yy s l.. a) Una función booleana/en n variables.x1. ~, ••• ,.t,
6. Enuncie y demuestre el dual del resultado en el ejerci• estmlOdual si/(.x1• Xi• .. .• x) = f(x1,X:?, ••• ,.r.~
cio 5.
¿Cuántas funciones booleanas en n variables son
7 . Sea ~ un álgebra booleana parcialmente ordenada por autoduaJes?
:S. Para x, y E ~. demuescre que
b) Scaf:B' ➔ B.fcs sirntlricasif (x.y, ,) ; f(x. ,,,l
a) x.S y si ysólosii + y= l;y ; f(y,x, <) ; f (y. z.x) ;f(z,x,y) ; f(z,y,x).As!,d
b) x :s .Y si y sóJosi:cy = O. valor de/ no se altera al reordenar las tres col111D-
nas de valores enumerados bajo x, y y len la tabla
8 . Sean x, y ele mentos del álgebra booleana g¡_Dcmucs-
para f. ¿Cuántas de estas funciones existen para d
tre que x = y si y sólo si .xy" + iy = O.
caso de tres variables booleanas'! ¿Cuántasexi.slea
9. Use un mapa-de Karnaugb para encontrar una repre- paran variables booleanas?
sentación como suma minimal de productos de
12. Las cuatro líneas de enuadade la red de la figura 15.16
a) f(w,x,y,z); ~m(O, 1,2,3, 6, 7, 14, 15) proporcionan los equivalentes en binario de los m1meros O.
b) g(v, w,x,y, z) =Il M(I, 2, 4, 6, 9, 10, 11, 14 , 17, 1. 2. . . . , 15, donde cada nwnero se representa como abce y
18, 19, 20, 22, 25, 26, V, 30) e es la cifra menos significativa.
775

números 10al 15nuncascdancomoentradas,cli-


señe una red de puertas de dos niveles para g en
este caso.
13. Para (a) n = 00, y (b) n = 120, explique por qu6 los
Figura 15.16 divisores positivos den no forman un álgebra booleana. (En
estecaso,:c:+y =mcm(x,y),xy=mcd(:c:,y),i =n/:c:, 1 esel
a) Encuentre la f.n.d. de g. cuyo valor sea l exacta- elemento neutro y n es e l elemento unidad.)
mente cuando abe~ sea el equivalente en binario 14. Sea !i 1 el álgebra booleana de todos los divisores ente-
del,2,4u8. ros positivos de 30030, y sea fil e l álgebra booleana de los
b) Dibuje la red de pucrtaSdedos niveles paragcomo subconjuntos de {u, u, w,:c:,y,l}. ¿Cu:tntosisomorfismos/:
una suma minimal de productos. =
91, ➔ 91, satisfacen/(2) = (u} y /(6) fu, u}1
e) Si esta red es parte de una red mis grande y, en 1 S. Sean a, b, e E ~. un álgebra booleana. Demuestre que
consecüencia, los equivalentes en binario de los ab + e = a(b + e) si y sólo si es a.
16
GNpos, teoría
de la codificación
y método
de enumeración
de Polya

A I esbldiar las eslI1>CIUr.lS algebraicas analizamos las propiedades compartidas por siste-
mas mateimtioos particulares. Oesputs geoeralizamos nuestros hallugos para estudiar
la estructura subyacente comWI en estos ejemplos particulares.
En el capítulo 14 hicimos esto con la estructura de anillo, la cual dependfa de dos
operaciones binarias cerradas.Ahora estudiaremos una estructura con una operación binaria
cerrada. Esta estructura se llama gn,po.
Nuestro estudio de los grupos analizara muchas ideas similares a las de los anillos. Sin
embargo, trataremos principalmenie los aspectos de la estructura necesarios para la ieoría
de la codificación y un !Mtodo de conleO desarrollado por George Polya.

16.1
Definiciones, ejemplos
y propiedades elementales

· "ón 16.1 Si Ges un conjunto no vacío y oes una operación binariaenG, entonces (G, o) es un grupo
si cumple las siguientes condiciones.
1) Para todos a. b E G, a• b E G. (Ges cerrado mediante •.)
2) Para toda a. b, e E G, a• (b •e) = (a • b) • c. (Propiedad asocwiva.)
3) Existe e E G tal que a o es e o a = a. para todo a E G. (Existencia de un elemento
identidad o neutro.)
4) Para cada a E G existe un elemento b E G tal que a • b = b • a z t. (Existencia de
inversos.)

Si, ade~. a o b =boa para todos a, b E G, entonces Ges un grupo conmutativo o


abtliano. El adjetivo abtliano es en bonoc del matemitico noruego Niels Henril: Abel
(1802-1829).

n 1
778 Capitulo 16 Grupos, teoría de la codificación y método de enumeración de Polya

Observemos que la primera condíción de la definición 16.1 puede omitirse si sólo


mos que la operación binaria en G sea una operación binaria ce"ada.

Si revisamos la definición 14.l (para un anillo). mencionamos cómo las leyes


para las operaciones binarias cerradas de+ (suma en el anillo) y • (producto en el
podían extenderse mediante la inducción matemática El mismo tipo de situación SUIF
los grupos. Si (G, 0 ) es un grupo, y ,; n E z+ con n ~ 3 y 1 ::5 r < n, entonces

(0¡ 0 02° ••• oa,)o(a ttlº ••• oa.,)=a1°D2". · • 0 D, 0 Qttl º •• • ºlli,i,

donde a1, ai•. . .• ~ Gr+ h ••• , a. son elementos de G.

Con la suma ordinaria, Z. Q , R y C son cada uno un grupo abeliano. Ninguno de ellm
un grupo mediante la multiplicación, pues O no tiene inverso multiplicativo. Sin
Q•, R* y e• 0os eleme ntos no nulos deQ, R y C, respectivamente) son grupos
multiplicativos.
Si (R, +, •) es un anillo, entonces (R, +) es un grupo abeliano; los elementos distintos
cero de un cuerpo forman un grupo abcliano multiplicativo.

Paran E z+, n > 1, tenemos que (Z.., +) es un grupo abeliano. Sip es primo. (Zt ·)es
grupo abeliano. Las tablas 16. 1 y 16.2 muestran lo anterior paran= 6 y p = 1. (Recordl>-
mosque en z...
escóbimos con frecuencia a en vez de [a] : {a+ ht lk E Z}. Usamos la
misma notación en Z ;.)

Tabla 16.1 Tabla 16.2

+ o 1 2 3 4 5 1 2 3 4 5 6
o o 1 2 3 4 5 1 1 2 3 4 5 6
1 1 2 3 4 5 o 2 2 4 6 1 3 5
2 2 3 4 5 o 1 3 3 6 2 5 1 4
3 3 4 5 o 1 2 4 4 1 5 2 6 3
4 4 5 o 1 2 3 5 5 3 1 6 4 2
5 5 o 1 2 3 4 6 6 5 4 3 2 1

Definición 16.2 Para cualquier grupo G, el número de elementos de Ges el onkn de G que se dcnoracoa
1G j . Cuando el número de elementos de un grupo no es finito, decimos que G tiene ordoa
infinito.

Paracualquiern E z•, l(Z.., +)I =n, mientrasquej(Z~ •)I =p-1 para cualquier primo p.
16. 1 Definiciones. . p1os y opóedad,s elementale, 779

Comencemos conclanillo(Z,. +, •) y consideremos el subconjunto U,= {a e~ laes una


unidad en Z,} s {a E Z,la-' existe}= { 1, 2, 4, 5, 7, 8} = {a E Z•I 1 :s as 8 y mcd(a. 9)=
1 }. Los multados de la tabla 16.3 muestran que U, es cerrado medianle la multiplicación
en el anillo (Z,. +, •); a saber, la multiplicación m6dulo 9 . Además, tambi~n observamos
que 1 es el elemento neutro y que cada elemento tiene un inverso (en U,). Por ejemplo. S
es el inverso de 2, y 7 es el inverso de 4. Por '1ltimo, como cualquier anillo es asociativo
mediante la operación de multiplicación (en el anillo), se sigue que a • (b • e) =(a• b) • e
para todos a. b, e E u► En consecuencia, (U~ •) es un grupo de orden 6; de hecho, es un
grupo abeliano de orden 6.
Engeneral, paratodon E z·. talque•> l, si U. = (a E (Z.. +, ·)la es una unidad} =
(a E z•p :S a s • - 1 y mcd(a, n) = 1}. entonces ( U., ·) es un grupo abeliano con la
operación binaria (cerrada) de multiplicación módulo•· El grupo (U., •) es el grupo de
unida<ksdel anillo ( Z., +,•) y tiene orden +(n), donde +(n) denota la función phi de Euler
de la sección 8.1.

'll>bla 16.3

1 2 4 s 7 8
1 1 2 4 s
7 8
2 2 4 8 1 s 7
4 4 8 7 2 1 s
5 5 1 2 7 8 4
7 7 5 l 8 4 2
8 8 7 s 4 2 1

De aquí en adelante, escribiremos la operación del grupo en forma multiplicativa. a


meoos que se iDdique lo contrario. Asf, denotaremos a o b con ab.
Todos los grupos satisfacen las sigujentes propiedades.

1EOREMA1 6.1 Para cualquier grupo G.


•> el ne·urro de Ges único.
b) el inverso de cada elemento de Ges único.
e) si a, b, e E G y ab = ac, entonces b z c. (Propiedad cancelativa por Ja izquierda)
d) si a. b, e E G y ba =ca, entonces b s e. (Propiedad cancclativa por la derecha)
Demostración:
a) Si ei, e1 son neutros de G. entonces e1 = e 1 e1 = e1 . (Justifique cada i¡ualdad.)
b) Sea a E G y supongamos qucb, e son ambos inversosdea. Entoncesb=be = b(,ac)
= (ba)c • t e; c. (Justifique cada igualdad.)

Dejamos la demostración de las propiedades (e) y (d) al lcctoc. (Debido a estas propieda-
des, cada c1emento del grupo aparece exactamente una vez en cada fila y en cada columna
de la tabla para un grupo finito.)
780 capítulo 16 Grupos. teoría de la codificación y método de enumeración de Pt>lya

Con base en el resultado del teorema 16.l(b), denotaremos el inverso único dea
cr1• Si escribimos el grupo en forma aditiva. usaremos -a para denotar el inverso (acli ·
dea.
Como en el caso de la multiplicación en un anillo, tenemos las potencias de los eJeme:a.
tos de un grupo. Definimosd'=e,a 1 = a.fil = a• a, yen general,d'-" 1 =<r- •a, paratodo11E
N. Puestoquecadaelementodel grupo tiene un inverso, paran E Z• delinimos<l"'=(a')'.
Entonces et' está definido para todo n E Z y podemos mostrar que para textos m. n E Z..
el". el'= (I"·· y (do/= el'".
Si la operación del grupo es la suma, entonces los múltiplos reemplazan las potencias J
para cada m. n E Z y a E G tenemos que

ma+na=(m+n)a m(na) = (mn)a.


Para un grupo abeliano G. también tenemos que para todo n E Z y todos a. b E G.
(IXab)"= d'b", cuando escribimos Gen forma multiplicativa; y (2) n(a + b) =na+-,
cuando usamos la notación aditiva para G.

Ahora analizaremos un subconjunto especial de un grupo.

Sea G = (Z., +). Si H = {O, 2, 4}, entonces H es un subconjunto no vacfo de G. La tabla


16.4 muestra que (H , +) es también un grupo mediante la operación binaria de G.

tabla 16.4
+ o 2 4
o o 2 4
2 2 4 o
4 4 o 2

Esta situación da sentido a la siguiente definición.

Definición 16.3 +
Sea G un grupo y 0 H ,;;, G. Si H es un grupo mediante la operación binaria de G,
entonces H es un subgrupo de G.

a) Todo grupo G tiene como subgrupos a {e}, G. Éstos son los subgrupos trivialesde
G. Los demás se llaman no triviales, o propios.
b) Además deH = {O, 2, 4), el subconjunto K= {O, 3} también es un subgrupo propio
deG=(Z.,+J.
e) Cada uno de los subconjuntos no vacíos {l. 8} y {1,4, 7} es un subgrupo de(U, -i
d) El grupo (Z, +) es un subgrupo de (Q, +), que a su vez es un subgrupo de (R, +~
Sin embargo, Z* no es un subgrupo de (Q*. •) mediante la multiplicación. (¿Por
qué?)
16.1 OefinKiones, plos y propiedades elementales 781

Para un grupo G y 8 f. H i;; G, lo siguiente nos indica cuando Hes un subgrupo de G.

16.2 Si Hes un subconjunto no vacío de un grupoG, entonces Hes un subgrupode G si y sólo


si (a) para todos a. b E H, ab E H y (b) para todo a E H. a-' E H.
t>emostrKi6n: Si Hes un subconjunto de G, entonces. por la definición 16.3. Hes un
grupo mediante la misma operación binaria. Por Jo tanto, satisface todas las condiciones
de un grupo, incluyendo las dos mencionadas. Recíprocamente, sea 8 f.H i;; G tal que H
satisface las condiciones (a) y (b). Para todos a. b, c E H, (ab)c = a(bc) en G, por lo que
(ab)c = a(bc) en H. (Decimos que H "bemla" la propiedad asociativa de G.) Por último,
comoHf. e. sea a E H. Por la condición (b),a-1 E H y por la condición (a), aa-'=, e H,
por lo que H contiene el elemento neutro y es un grupo.

Una condición de finitud modifica la situación.

16.3 Si Ges un grupo y 0 f. H i;; G, con H finitc, entonces H es un subgrupo de G si y sólo si H


es cenado mediante la opaaeión binaria de G.
Demostnción: Como en la demostración del teOr<ma 16.2. si Hes un subgrupo de G.
entooccs H es cerrado mediante la operación binaria de G. Recfprocamcnte, sea H un
subconjunto finito no vacío deG, que es cerrado. Si a EH, entonccs aH= {ahlh EH}~
H debido a 13 condición de cerrado. Por la cancelación en G, oh 1 =ah2 ~ h1 =ki, de modo
que laHI = IHI. Con aH !;; Hy laHI = IHI, corno H es finito, tenemos que al/ = H.
Comoa E H. existe b E Htal que ab=a. Pero (en G)ab =a= a,, por lo que b =• y H
contiene al neutro. Puesto que e E H = ah, existe un elemento e E H tal que oc = e.
Entonces (ca)'= (caXca) = (c(ac))a = (ce)a =ca= (ca),, por lo que ca=• y c = ,r' E H.
En consecuencia, por el teoccma 16.2, Hes un subgrupo de G.

Eo el teorema 16.3, la condición de finirud es crucial. Tanto z• como N son subconjuntos


cerrados no vacíos del grupo (Z. +), pero ninguno tiene los inversos aditivos necesarios
para la estructura de grupo.

El siguiente ejemplo presenta un grupo no abeliano.

Consideremos el primer triángulo equilátero de Ja figura 16.I(a). Cuando rotamos este


aiingulo 120" (dentro de su plano). en sentido cootrario al de las manecillas del reloj, en
tomo de un eje perpendicular a su plano y que pasa por su centro C, obtenemos el segundo
aiingulo que se muestra en la figura 16.1 (a). Como resultado, el vlnice etiquetado origi-
nalmente como 1 en la figura 16.l(a) está ahora en la posición que al principio tenía el
nWDero 3. Oc la misma forma. 2 está en la posición que ocupaba el 1, y el 3 donde estaba
el 2. Podemosdescribirestomediantela función x, :( 1, 2, 3)➔( l. 2, 3), tal qucx 1(1) = 3,
lt1(2) = 1, ><1(3) = 2. La notación mis compacta U¡ D , donde escribimos it1(1) debajo&: ;
para cada 1 S ; S 3, eofati1.a que Jt1 es una pcrmuración de ( 1, 2, 3). Si x, denota la
rotación en sentido contrario al de las manecillas del reloj con 240°, entonces x-,: = Bi ~.
782 capítulo 16 Grupos. teoría de la codrficacíón y método de enumeración de Polya

A _r., ► A ---A _,, ► A


~~~~
1 3 2 1 l 3'2 3
(a) (b)

Figura 16.1

Para el neutro no; es decir. la rotación de n(360º), paran E z. escribimos 1to = (l l l


rotaciones se conocen como movimientos rígidos del triángulo. Son movimi
bidimensionales que conservan fijo el centro C y preservan la forma del triángulo. Pcr
tanto, el ttiángulo se ve igual que cuando empezamos, excepto por un posible
miento de las etiquetas en algunos de sus vértices.
Además de estas rotacion~. podemos reflejar el triángulo a lo largo de un eje que
por un vértice y por el punto medio del lado opuesto. Pa,a el eje diagonal que biseca
ángulo derecho de la base, la reflexión da por resultado la figura 16.1(b). Represe
esto con r, = (! f ID- Una reflexión similar a lo largo del eje que biseca el ángulo izq .
de la base produce la permutación r2 ;;;; (l j ~)- Cuando reflejamos el triángulo a Jo largo
SU eje vertical, tCDCffiOS T3 ; u~n. Cada T¡, para 1 S i :S 3, CS UD movimiento • •
tridimensional.
Sea G = {Xo. 1t., 7½, r1, r1 , r,} el conjunto de movimientos ñgidos (en el espacio)
triángulo e.quilátero. Hacemos de G un grupo definiendo el movimiento rígido ap, para a.
~ E G, como el movimiento obtenido al aplicar primero a y después~. Así. por ejeqi).
n.1r 1 = r3• Podemos ver esto de manera geométrica, pero será más fácil considerar 111
r
permutaciones como sigue: :lt1r1 .::: Gi DO ~ donde, por ejemplo, 7t¡(l) =3 y r ,(3) = 3'
escribimos l ~ 3 -!L.+ 3. Así, 1 ~ 3 en el producto 1t1 r 1• (Observe que el orden•
el que escribimos el producto 1t1 r1 es e l opuesto al de su función compuesta, como la
definimos en la sección 5.6. La notación de la sección 5.6 aparece en el análisis, mieob'IS
que en álgebra hay una tendencia a utilizar este orden opuesto.)Además, como 2 ~
I ~ 2 y 3 ~ 2 ~ l.sesigue que 1t1 r 1 ={ll~ ==r3 •
La tabla 16.5 verifica que G es cerrado mediante esta operación binaria. con neutro s.,
Además, 1t11 ='J½, 1ti 1 = 1t1 y cualquier otro elemento es su propio inverso. Puesto que los
elementos de G son en realidad funciones, la propiedad asociativa resulta del teorema S.6
( aunque en orden inverso).

labia 16.5
,,, T¡ ,,
r,

,,, "'
,,, ,, ,,

"º "º ,,,
,,, ,,, r, ,,

,,, ,,, "º
.,., ,, ,, T¡
r, T¡

r, ,, "º "• ,,,
r, ,, r, T¡ ,,, "º "•
r, ,, T¡ r,
"• "' "º
16.1 Definiciones. ejemplos y propiedades elementales 783

Calculamosnir1 como r3,pcrode la tabla 16.5 vemos que r1 7t¡ = r2. Como7t1r1 • r3 'F-r2 =
r1Xt, Ges no abeliano.
También podemos obtener este grupo como el grupo de permutaciones del conjunto
{ 1, 2, 3} mediante la operación binaria de composición de funciones. Lo denotamos con
s, (el grupo simétrico de 3 símbolos).

El grupo simétricoS,constadelas24permutaciones de(!, 2, 3, 4 ). En este caso,"-= (:l l !)


ese1-~u -. s·1a -(""\ ~-1? 1 2 ◄ ,en"'~'"'+'-
- 2 1 4 3, •-1!2") •-""'-(""\ ""'-(""I
13 4 2,.pero ¡.,u,- 421 3ft por Ioque S,.es
no abeliano. Además, ¡}-1 = U3 f :) y r,.2 = Xo= 1}3 • En S4 existe un subgrupo de orden 8 que
representa el grupo de movimientos rígidos de un cuadrado.

Analiza=los aloa una construcción pera formar grupos giandes a partir de grupos pequeño6.

Sean (G, •) y (H, *)grupos. Definimos la operación binaria• en G x H como (gi, h 1) •


(g,, h,) = (g1 • g,. h1 • h,). Entonces (G x H, ·) es un grupo llamado producto din,cto de G y H.
Demostncóln, Dejamos al lector la verificación delas propíedadesdegrupode(G x H, •).

Consideremos los grupos (Z,, +), (Z~ +). En G = Z, X z,,


definimos (a1, b 1) • (a,, bi) =
(a 1 + a,, b 1 + b,). Entoncés, Ges un grupo de orden 6, donde el neutro es (O, O) y el inverso
del elemento ( 1, 2) es ( 1, 1), por ejemplo.

1. Para cada uno de los siguientes conjuntos, deten:nine si el conjunto es o no un grupo mediante
la operación binaria dada. De ser así, determine su oeuuo y el inverso de cada elemento. Si no
es un grupo, formule las condiciones que viola de la definición.
a) {-1, l} bajo la multiplicación
b) {-1, I} bajo la suma
e) {-1,0, I } bajo la suma
d) {10.lnEZ}bajolasuma
e) El oonjuntodc todas las funciooes/:A ➔ A,dondeA = {1, 2, 3,4}, mediante lacomposi•
ción de funciones
1) El conjunto de todas las funciones inyectivas g: A ➔ A donde A= {l. 2, 3, 4}, bajo la
composición de funciones
g) {al2"ia. n E Z. n 2' O), bajo la suma
2. Demuestre las panes(<) y (d) del t,orerna 16.1.
3. ¿Por qut el conjunto Z no es un grupo bajo la resta?
4. Sea G = {q E Qlq F-1}. Definimos la operación binaria O en G como x o y= x +y+ zy.
Demuestre que (G, o) es un grupo abcliano.
5, Defina la operación binaria o en Z como z o y= z +y+ l. Verifique que (Z. o) es un grupo
abeliaoo.
6. Sea S = R• X R. Defina la operación binaria º en S como (u. 'U) º (z,, y) = (Ja, ,u: + y).
Demuestre que (S, o) es un grupo no abeliano.
784 capítulo 16 Grupos, teoña de la codificaóón y método de enumeración de Polya

7 . Encueotrc los elementos de los grupos U» y Uu. los grupos de unidades de los anillos CZ. +, •)
y (Z,.. +. •), respectivamente.
8. Paracualquierg,:upoG. demuestre que Ges abelianosí y sólo sí (ab")' =a'b' para todos a, b EG.
9. Si G es un grupo, demuestre que para todos a. b e G.
a) (a- 1 ) -1 =a b) (ab )-1 =b- 1 a - 1
1 O. Demuestre que un grupo Ges abeliano si y sólo si para todos a. b E G. (ab,'- = a-"b4 •
11. Encuentre todos los subgrupos de los siguientes grupos.
a) (Z,., +) b) (z;,, ·) e) s,
12. a ) ¿Cuántos movimientos rígidos (en dos o tres dimensiones) tiene un cuadrado?
b ) Elabore una tabla de grupo para estos movimientos rígidos, como la tabla 16.S para rJ
triángulo equilátero. ¿Cuál es el neutro para este grupo? Describa en forma geonmcad
inverso de cada elemento.
13. a ) ¿Cuántos movimientos rígidos (en dos o tres dimensiones) hay para un pentigono regular!
Descríbalos en forma geomEtrica
b) Responda la parte (a) para un n-ágono regular, n ~ 3.
14. En el grupo S,, sea

¡ 2 3 4 5)
a - (2 3 1 4 5
P-(12 21 3 34 5)
5 4.

Detennineajl, il<X, "'· ll', Cl""', p-•, (ClP)~. (Jl<x)~ y P~Cl""'.


15. Si Ges ungrupo, seaH= {a E Glag = gapara todo g EG}. Demuestre que Hes unsubgrupo
de G. (El subg,:upo H es el cenm, de G.)
16. Sea ID el número complejo(ll.J2X I + 1).
a) Muestre queO'J= 1, pero 01" 'Fl paran E z.,
1 ::S n S 7.
b) Verifique que {<if'Jn E Z... 1 s ns 8} es un grupo abeliano mediante la multiplicacióll.
17. a ) Demuesttc et teorema 16.4.
b ) Extienda la idea desarrollada en el teorema 16.4 y c l ejemplo 16.9 al grupo Zi x ~x Z.=
Zl y responda lo siguiente.
i) ¿cu.il es el orden de este grupo?
z:
if) F.ncucntre un subgrupo de de orden 6, otro de orden 12 y uno más de orden 36.
üi) Detennine el inveno de cada uno de los elementos (2, 3, 4), (4, O, 2), (5, 1, 2).
18. Si H. K son subgrupos de un grupo G. demuestre que H n Ktambién es un subgrupo deG.

16.2
Homomorfismos, isomorfismos
y grupos cíclicos

Ahora regresaremos a analizar las funciones que preservan la estructura.

l-N Sean G = (Z, +) y H = (Z., + ). Definimos f: G ➔ H como


f(x) - [x]-{x + 4klkEZ}.
Para todos x, y E G.

f(x +y)= [x + y]= [x] + [y]= f (x) + f(y).


1 1
t . a ~ eo G t . ~ eo H.
16.2 HomomoñtSmOS, isomorfismos y grupos óclkos 785

En este caso,/ preserva las operaciones de grupo y es un ejemplo de un tipo particular de


función que definiremos a continuación.

ilnición 16.4 Si (G, •)y(H, •) soo grupos y/: G ➔ H, entooces/es unhomomo,fismoth gn,possi para
todos a. b E G, /(a• b) =/(a) • /(b).

Cuando sabemos que las esuucruras dadas son grupos, decimos simplemente que la
función/ es un homomorfismo.
En el siguiente teorema damos algunas propiedades de los homomorfismos.

lllREMA 16.5 Sean(G, •).(H. •) grupos con neutrosiespectivas,0 y,,. Si/: G ➔ H es un homomorfismo,
entonces
a) f(eo) = ••·
b) f(tr') = {f(a)t' para todo a E G.
e) /(a.) = {f(a))' para todo a E G y todo n E Z.
d) ft.S'J es un subgrupo de H para cada subgrupo S de G.
Dt:mo.straci6n:
a) ••• /(<o) =/(<e) =/(<c••c) =/(<e) • f(e0 ), por el teorema 16.l(d), se sigue que
/ (<e)=•• ·
b) y e) Dejaremos al lector la demostración de estas panes.
d) Si Ses un subgrupo de G, eotonces Sp 9, por lo quef(S¡,f,f. SUnx, y E f(S¡.
Entoncesx = f(a) ,y : f(b), para algunos a, b E S. Comos es un subgrupo deG,
se sigue que a • b E S. por lo que x • y = f(a) • f(b) = f(a • b) E /(S'J. Por
dltimo, r ' = {f(a)J-' =/(tr') E/(S¡, puestoquetr' E Ssia E S. Enconsecueo-
cia, por el teorema 16.2,/(S'J es un subgrupo de H.

Definición 16.5 Si/: (G, •) ➔ (H, • ) es un bomomodismo,fes un isomo,fismo si es inyectiva y soba. En


este caso. G y H son grupos isomorfos.

Sea/: (R+, •) ➔ (R. +)tal que/(x) = log,o(, ). Esta función es uno a uno y sobne. (Verifique
estas propiedades.) Para cualquier a. b E R·.J(ab): log.,(ab): log,0 a + log,0 b =/(a)+
/(b). Por lo tanto.fes un isomorfismo y el grupo de números reales positivos mediante la
multiplicación es igual (en un sentido abstracto) al grupo de los nwncros reales bajo la suma.
En este caso, la función/traduce un problema en la mu1tiplicación de los m1meros reales
(un laoto dif!cil sin calculadora) ea un problema que trata de la swna de mlmeros neales (un
considerando aritmttico más sencillo). Ésta era una razón fundamental para el uso de los
logaritmos antes del advenimiento de las calculadoras.
capítulo 16 Grupos, teoría de la codificación y método de enumeración de Potya

Consideremos el conjunto G = (a+ bJ'S la. b E Q). Mediante la operación binaria dela
suma ordinaria de los números reales, este conjunto es un grupo abeliano. En este caso.
a + bJ'S = e + dJ'S para a. b. c. d E Q. si y sólo si a = e y b = d. (Pediremos al lector qoe
establezca este hecho en los ejercidos del final de la sección.)
A continuación, consideremos e) conjunto H = {[g ]a.beQ},
3f que es un gn1p1
abeliano bajo la operación binaria de suma de matric.es. Para cualesquiera a + M, e+
dJ'S E G.

de modo que la asignación de a + b../3 a [ b3} ] nos proporciona una función/: G ➔H


inyectiva Además, dado un elemento arbitrario de H, una matriz de la forma [; 3]}
donde x, ye Q, vemos que x + yJ'j E G y f(x + y,fj) = [ ; 3 ¡]- Por lo tanto, la funcióof
también es sobre. Por úlumo,

f((a + bV3) + (x + yV3)) = f((a + x) + (b + y)V3) =


a+ x 3(b +y)] = [ª 3b] + [x 3y] = f(a + bV3) + f(x + V3).
[b+y a+ x b a y x Y

Así, la función/preserva las operaciones de gi:upo de G y H. y (G, +) y (H.+) son grupos


isomorfos.
[Nota: También tenemos que

f((a + bV3)(x + yV3)) =/((ax+ 3by) +(ay+ lix)V3) =

[ax+ 3by 3(ay + bx)] =


ay+bx ax+3by b a y x

3b][x 3y] = (f(a + bV3))(f(x + V3)).
Y

Así, los anillos (G, +,•)y (H.+,-), donde la multiplicación en Ges la multiplicación
(ordinaria) de los números reales y la multiplicación en H es el producto de matrices,
también son isomorfos.]

Sea Gel gi:upo de números complejos ( l, - 1, i, -i) mediante el producto. La tabla 16.6
muestra la tabla de multiplicar del grupo. Si H::: (Z., +), consideremos la función/: G ➔
H dadapor

/(1) = [O] /(-1) = [2] f(i ) = [1] f( - i) = [3].

Entonces f((i)( - i)) = /(1) = [O] = [1] + [3] = f(i) + f( - i), Y /((-1)(- i)) = f(i) =
[!] = [2] + (3] =f(-1) + /(-i).
Aunque no hemos verificado todos los casos, la función es un isomorfismo. Observe
que la imagen baj o/ del subgi:upo ( l, -1} de Ges ((O], [2]), un subgi:upo de H.
16.2 Homomorfismos, isomorfismos y grupos ádlcos 787

Tabla 16.6

-1 -i
1 1 - 1 i -i
-1 -1 1 -i
i -i - 1
-i -i -1

Analicemos más de cerca el grupoG. En ese caso, ;i =i, P-= -1, í3=-i e i'= 1, de modo
que todo elemento de Ges una potencia de i; y decimos que i genera G. Esto se denota con
G = (i). (También es cierto que G = (-i}. Veriffquelo.)

La última parte del ejemplo anterior nos lleva a la siguiente definición.

!linidón 16.6 Un grupo Ges cíclico si existe un elemento x E G tal que para todo a E G, a : x• para
alglln n E Z.

a) El grupo H -= (Z,, +) es cíclico. En este caso, la operación es la suma, por lo que


tenemos múltiplos en vez de potencias. Tenemos que (l] y [3] generan H. Para el
caso de [3], tenemos que l · [3] = [3], 2 • [3] = [2], 3 • [3] = (l] y 4 · (3] = [O]. Por lo
taDto, H = ([3]) =([!]}.
b) Consideremos el grupo multiplicativo U,= ( l, 2, 4, 5, 7, 8) que analizamos en el
ejemplo 16.4. Aquí tenemos que 21 ; 2, 22 = 4, 23 - 8, 24 = 7, 25 = 5, 26 = 1 por lo que
U, es un grupo cíclico de orden 6 y U,= (2). También es cieno que U,= (5) puesto
que 5'= 5, 5'= 7, 53 = 8, 5' = 4, 5'= 2, 5'= l.

El concepto de grupo cíclico da lugar a una idea afin. Dado un grupo G, si a E G,


consideremos el conjuntoS = {a 1 1k EZ}. Del teorema 16.2, sabemosqueSes un subgrupo
de G, Uamadoelsubgrupo generado por a, que se denota con (a). En el ejemplo 16.13, (,)
=(-<) =G; además,(-!}= {- 1,1) y(I}= { 1 }. Para la parte(a) del ejemplo 16.14,conside-
ramos los múltiplos en vez de las potencias y vemos queH = ([11} = ([31), ([21) = ( [O], (2])
y ([O]) = {[O]). Si analizamos el grupo U, e n la parte (b) de ese ejemplo, vemos que U, =
(2) (o ((2]}) = (5), (4} = {!, 4, 7}= (7), (8) = ( l. 8), y (1) = ( 1 }.

!finición 16.7 Si Ges un grupo y a E G, el oTikn d, a, que denotamos con o(a), es 1(a} 1- (Si 1(a} 1 es
infinito, decimos que a tiene orden infinito.)

En el ejemplo 16.13, o(l) = !, o(-1) = 2, mientras que i y -i tienen orden 4.

Revisemos la idea de orden para el caso en que 1(a) 1 es finito.


788 (.apítulo 16 Grupos, teoría de la codificación y método de enumeración de Pofya

Si l(a}I = 1, entonces a=,, puesto que a= a' E (a) y•= a'E (a). Si l(a)I es finitopm,
a i , , entonces (a)= {a"lm E Z} es finito, de modo que {a, a', a', ... )= {a"lm E Z'}
tam.bífo es finíto. E n consecuencia. existen s, t E z+ tales que l ::s. s < t y tr = a, de lo qs
se sigue que á - 1 =e, con r - s E z+. Como e E {a"'Jm E Z'""}, sean el mínimo entera
positivo tal que a'=,. Afirmamos qoe (a)= {a, a', a'.... , a'-•, a'(= e)),
Primero observemos que 1{a,a',a', .. . ,a'-',a'(= e) ) 1 =n. En caso contrario, tenmit-
mosa"= a" para u. u enteros positivos, tales que 1 s u< u s n~ y entonces a"-•== t,c:oa
O< u - u < n. Sin embargo, esto contradice el hecho de que n sea mínimo. Así, abom.
sabemos que 1(a) 1 a: n. Pero para cualquier k E Z, el algoribno de la división implicaqoe
k=qn+r,dondeO S r<n,J)Ol'loquea'=a".,=(a')'(a') = (<')(a') = á E {a,a', a', .. .• ~-•.
a'( = • = a")). Porlo tanto, (a) = {a, a', a', . . . , a'- •. a'(=e)) y rambif n podemos demir
o(a) como e l mínimo entero positivo n tal que a"= e. Esta definición a1ternativa parad
orden de un elemento del grupo (de orden finito) es muy ótil en el siguiente teorema.

TEOREMA 16.6 Sea a E Gcon c(a) =n:,. Si /e E Z y~= e, entonces n lk.


Demostración: (De nuevo) por el algoritmo de la división, tenemos que k =qn + r, para OS
r < n, y entonces se sigue que e = ti'= d"• • = (a')'(a') = (<')(a') = a'. Si O < r < .l.
contradecimos la definición den como c(a). Por lo tanto,.r::::: O y k: qn.

-
Ahora analizaremos algunos resultados adicionales relativos a los grupos cfclioos. B
siguiente ejemplo nos ayudará a motivar la segunda parte del teorema 16.7.

De la parte (b) del ejemplo 16.14 sabemos que U, = { 1, 2. 4, 5, 7, 8) = (2). Usamos..,.


hecho para definir la función/: U, ➔ (Z.. +) como sigue:

/(1) =[O] /(2) = [1) /(4)= [2]


/(5) = /(2') = [5) /(7) =/(2') = [4] /(8) =/(2') =[3].
Así. en general, para cada a E U9 escribimos a : 2.t para algún O s: k s 5 y tenemos qae
f(a) =/(2') = [k]. Estafuncióo/es inyectiva y sobre; vemos, J)Ol'ejemplo, que/(2 •5)=/(1)=
[O] = [I} + (5) = /(2) + / (5) y /(7 · 8) =/(2) = [1 ] = [4] + [3] = /(7) + /(8).
En general, para a,b E U, podemos escribir a: 2• y b: 2•, donde O s: m, n :S 5 y se
sigue que

f(a ·b) = /(2'"· 1.") = /(2M") = [m +n] = [m] + [n) =/(a)+ f (b).
En consecuencia. la función/es un isomorfismo y los grupos U, y (4.. +) son isonw:riOL
[Observe la forma en que la función / relaciona los generadores de los dos grupos
cíclicos. O bserve también que la función g: U,-► (4. +), donde

g(l) = [O] g(5) =[1) g(7) = g(5') = [2]


g(S) =g(5') = [3) g(4) = g(5') = [4] g(2) =g(5') = [5]

es otro isomorfismo e ntre estos dos grupos cJclicos.]


16.2 Homomorfosmos, isomo<fismos y grupcs ódicos 789

TEOREMA 16.7 Sea G un grupo cíclico.


a) Si IGI es infinito, entonces Ges isomorlo a (Z, +).
b) Si IGI =n,donden> l,entoncesGesisomorloa(Z.,+).
Demostndón:
a) Para G =(a)= {a'lk E Z ), sea/: G ➔ Zdada por/(a') =k. (¿Podñaocurnrquea'=
a'si 1#! En esecaso,/no sería función.) Para a'", a" E G ,/(a'"· a") •f(a'" ..) = m +
n =/(a") +/(a"), por lo que/es un bomomorfismo. Dejaremos al lector que verifi.
que que fes inyectiva y sobre.
b) SiG=(a)= la.a', ... ,a"·',a"= •l entonces la función/: G ➔ Z.dada por/(a'J=[kJ
es un isomorfismo. (Verifique esta afinnaci6n.)

Si G = {g), Ges abeliano, pues g"· g"= g" .. = g"••= g" · g"para todos m, n E Z. Sin
embargo, el recíproco es falso. El grupoH de la tabla 16.7 es abeliano, y o(e) = 1, o(a) =
o(b) = o(c) = 2. Puesto que DO tiene elementos de orden 4, H DO puede ser cíclico. (El
grupo Hes el mínimo grupo no cíclico y se conoce como el grupo cuatro tk Kkin.)

'Dibla 16.7

, ,
• a b e
a b c
a a •e ,e b
b
c
b
e b a ,a

Nuestro 11.Jti.mo resultado se refiere a Ja estructura de 1os subgrupos en un grupo cíclico.

TEOREMA 16.8 Cualquier subgrupo de un grupo cíclico es cíclico.


Demostración: Su G = (a). Si Hes un subgrupo de G, cada elemento de H tiene la forma
a', para algún 1 E z. Si H 1, 1,), sea I el mínimo entero positivo tal que a' E H. (¿Cómo
sabemos que existedicbo entcro?)AfirmamosqueH = (a'). Como a' EH, por la propiedad
de cierre del subgrupo H , (a') !: H. Para la inclusión opuesta. sea b E H, con b = a" para
algtln s E z. Por el algoritmo de la división, s = qt + r, donde q, r E Z y O :S r < t. En
consccuencia,a'=a"'· • y a'= a1'a'= (a')-<b. H es un subgrupo de G, por lo que a' EH~
(a')-< E H. Entooccs. con (a')-<, b E H, se sigue que a'= (a')-<b E H. Pero si a' E H coo r >
O, contradecimos el becbo de que I sea mínimo. Por lo tanto, r = O y b • a"= (a"/' E (a'),
por lo que H = (a'), un grupo cíclico.
790 Capítulo 16 Grupos, teoria de la codificación y método de enumeración de F\:,lya

EJERCICIOS 16.2 1. Demuestte las parteS (b) y (e) del te()rema 16.S.
2. Si/: G ➔ H y g: H ➔ Kson homomorfismos. demuestre que la funcióncompuestago/: G➔
K dada po, (g • [Xx) =g(f(x)) es un bomomodismo.
3. Sea (G, +) el grupo {a+ bJ'i¡a, b E Q }, como en el ejemplo 16.12. demuestre que para a+
N3, c+d,Í3 E G. a+ b,Í3 =e +d,Í3 si y sólo si a=cy b = d.

4. SeaA•[-~ ü
a) DetermineA1,A3 yA'.
b) Verifique que {A, A2, A', A' } C$ un grupo abeliano en el producto ordinario de matrices.
e) Demuestre que el grupo en la parte (b} es isomodo al grupo de la tabla 16.6.
5. Si G:: (~ +),H= (Z,, +} yK= (Z¡, +), encuentre un isomorfismo para los gruposH x K yG.
6. Sea/: G ➔ H un homomorfismo sobreyectivo de grupos. Si Ges abeliano, demuestre queff es
abcliano.
7. Sea (Z x Z. EB) el grupo abeliano tal que (a. b) EB (e, d) = (a+ e, b + d) (en este caso,
calculamos a+ e y b + d mediante la suma ordinaria en Z) y sea (G, +) un grupo aditivo. Sif.
Z x Z ➔ Ges un bomomodismode grupos tal que/(!, 3) =g, y/(3, 7) = g• exp,ese/(4,6)
en ~ de 81 y K2•
8. Sea/: (Z x Z.EB) ➔ (Z , +) la función dadapo,f(,; y) =x- y. [En este caso, (Z x Z. EB)esd
mismo grupo del ejercicio 7, y (Z, +) es el grupo de los enteros con la suma ordinaria.]
a) Demuestre que/es un homomorfismo sobreyectivo.
b) Dcterminctodoslos(a, b) É Z X Z talesque/(a, b)=O.
e) Determine r (7).
1

d ) Si E = {2n (n E Z }. ¿cuinto vale ¡-'(E)?


9. Encuentre el orden de cada elemento del grupo de movimientos ñgidos de (a) el triángulo
equilátero; y (b) el cuadrado.
1 O. E n S5, encuentre un elemento de orden n. para todo 2 S n S 5. Determine también el subgrupo
(dclioo) de S~ que genera cada uno de estos elementos.
11. a) Encuentre todos los elementos de orden 1Oen (Z.0. +).
b) Sea G = {a) un grupo cíclico de orden 40. ¿Quf elementos de G tienen orden 10?
12~ a) Deternrine UH, el grupo de unidades del anillo (Z 14, +, ·).
b) Muestre que U1, es cíclico y e ncuentre todos sus generadores.
13. Verifique que (Z:, •) es cícUco para los primos 5, 7 y 11.
14. Pata un grupoG, demuestre que la función/: G ➔ Gdada por/(a) =a-1 es un isomorfismo si
y sólo si Ges abcliano.
15. Demuestre o refute la siguiente proposición: Si Ges un grupo tal que todo subgrupo propio de
Ges cíclico, entonces G es cíclico.
16. Para ro= (11.fiXI + i). sea Gel grupo multiplicativo {ro'(n E z•, 1 s ns 8].
a) Muestre que Ges cíclico y encuentre los elementos x E G tales que (r} = G.
b) Demuestre que Ges isomorfo al grupo CZ., +).
17. a) Encuentre todos losgcnerado«:s de los grupos cíclicos CZ1c +). (Z 1" +) y (Zu, +).
b} Sea G = {a}, con e (a)= n, Demuestre que et, k. E z·, genera G si y sólo si k y n son primos
relativos.
e} Si Ges un grupo cíclico de orden n, ¿cuántos generadores distintos tiene?
18. Sea/:G ➔ Hunbomomorfismodcgrupos. Sia E G, c,(a) =n y o(/(a)) =k (enH),demucstre
quek(n.
16.3 Clases laterales y el teo,ema de L.agrange 791

16.3
Clases laterales y el teorema de
Lagrange

En las últimas dos secciones, para todos los grupos finitos G y subgrupos H de G, obtuvi•
mosque IHI divide a IGI. En esta sección mosir.=mos que esto no fue casualidad, sino
que es cierto en general. Para demostrarlo necesitamos una idea nueva.

Definición 16.8 Si Hes un subgrupo de G, entooces para cualquier a E G, el conjunto aH = {ah I h E H)


es una cuis, /at,ra/ izquierda de H en G. El conjunto Ha= {halh E H} es una e/as,
laleral derecha de H en G.

Si la operación en G es la suma. escribimos a + H en vet de aH, donde a + H-= {a +


hlhEH).
Cuando utilicemos el tirmino clase lateral en este capítulo. nos referiremos a una clase
lateral izquierda. Para los grupos abelianos no necesitamos distinguir entre las clases late,.
rales izquierdas y derechas. Sin embargo, ~te no es el caso para grupos no abelianos.

Si G cs el grupo del ejemplo 16.7 y H= {lfo. x 1, it¡}, la clase lateral r 1 H = {r1lfo, , ,,.., r 1",} •
{ri. r 2, r,}. De la misma forma tenemos que r1 H= r, H = {ri, r1 , rJ} mientras que 1toH •
n ,H= 1t,H= H.
Vemos que laHI = IH I para todo a E G y que G =HU r1H es una partición de G.
Para el subgrupo K = {lfo, r 1} tenemos que r, K = {r,, "2} y r,K = {r,. 1t¡). De nuevo
surge una partición deG : G • K U r1 K U r 3 K. (Nota: Kr1 • {1tor1, r1 r1} = {r1, 1t1 } i=r1 K.)

~1'-11) Para G = (Z,,, +) y H = {[O]. [4], [8]}, tenemos que

(O] + H • {[O], (4], (8]} • (4] + H • (8] + H = H


(1] + H - {(l],[5],(9]} = (S] + H = [9) + H
[2] + H = {[2], (6],[10]} = (6] + H = [10) + H
(3] + H ={[3],(7), (11]} • (7) + H - (11] + H ,

y HU ((1) +H) U (12] + H) U ([3} + H) es unapanición de G.

Estos ejemplos nos preparan para los siguientes resullados.

LEMA 16.1 Si H cs un subgrupo del grupo finitoG, entonces para lodosa, b E G, (a) laHI = IHI ; y
(b) aH =bHoaHnbH = e.
a) ComoaH= {ahlhEH},cstoimplicaque laHI :S IHI.Si laHI < IHl, tenemos
que ah,= ah¡, con h,, h¡ elementos distintos en H. Por la cancelación izquierda en G,
obtenemos la contradicción h;= h,. de modo que laHI = 1H I.
792 capítulo 16 Grupos, teoría de la codificación y método de enumeración de Po!ya

b) SiaH n bHj0, sea e =ah,= bh, para algunos h., h, E H. Six E aH, entoncesx=ah
para algún h E H, de modo que x = (bh,h,'")h = b(h,h;'h) E bH, y aH bH. En s
forma análoga, y E bH =>y= bh, para algún h, EH=> y= (ah1h¡1)h, = a(h,h¡1h,)
E aH, de modo que bH !;;;; aH. Por lo tanto, ali y bH son disjuntos o idénticos.
(Observemos aquí que si g E G. entonces g E gH, puesto que e E H. Además.
por la parte (b), podemos separar a Gen clases disjuntas entre sí.)

En este momento estamos listos Qara demostrar el QrinciQal resultado de esta se<:.-
ción.

TEOREMA 16.9 Teorema de Lagrange. Si G es un grupoñnitode orden n y Hes un subgrupo de ordenm,


entonces m divide a n.
Demostración: Si H =G se tiene el resultado. En caso contrario,m < n y existe un elemento
a E G-H. Como a '1- H, se sigue queaHjH, demodoqueaH n H= 0. SiG=aH UH,
enton<:es IGI = laHI + IHI = 2 jH j y se sigue el teorema. En caso contrario, existe un
elementobEG- (HUaH), talquebHnH= 0=bHnaHy lbH I = jHj.SiG=bHUaH
U H, tenemos que IG / = 3 IH /. En caso contrario, obtenemos un elemento e E G tal que
e <l. bH U aH U H. El grupo Ges finito, de modo que este proceso termina y vemos queG
=a1H U a,H U••• U aJf. Porlotanto, IG I = k lHI y mdivide an.

Otro método para demostrar este teorema aparece en el ejercicio 14 de esta sección.
Terminaremos la sección con dos corolarios. Pediremos su demostración en los ejerci-
cios de la sección.

COROlARIO 16.1 Si G es un grupo finito y a E G, entonces o(a) divide a IG 1.

COROlARIO 16.2 Cualquier grupo de orden primo es cíclico.

EJERCICIOS 16.3 1. Sea G =s•. (a) Para c:r =(Ji ] t), encuentre el subgrupo H = (a). {b) Determine las clases
izquierdas de H en G.
2. Responda el ejercicio l para el caso en que a se reemplaza por p = B1r :).
3. Si "t = Ui ! ;)E S
4, ¿cuántas clases lateral« determina (y)?
=
4. Para G {Zu, +), encuentre las clases laterales determinadas por el subgrupo H =([31). Haga J
lo mismo para el subgrupo K = ((4)).
5. Sca.G= R x R, con la operación de grupo+ dadapor (a. b) + (c. d) = (a +e, b+á), para a, ..
e, d E R.. (F.n este caso. calculamos a+ c. b + d mediante la suma ordinaria en R.)
a) Si H = {(a, Ola E R L demuestre que Hes un subgrupode G.
b) Dé una interpretación geométrica de las clases laterales de H en G.
6. Sea R un anillo con elemento unidad u. Demuestre que las unidades deR forman un grupo bajo
la multiplicación del anillo.
16.4 Elementos de la teoria de la codificación 793

7. Sea G un grupo con subgrupos H y K. Si IGI =660, IKI =66 y K CH C G, ¿cuáles son los
posibles valores de IHI?
8. Sean Pi , pz, Pi. p, cuatro primos distintos y sea G un grupo con subgrupos H y K. donde K C
H C G. ¿CUántos valores posibles tiene IHI si
•) IGJ= p1pip;p·;--; ·1Kj = p1¡>ip3?
b) IGl:P!f:,ei~!YlKl=p~if}P:J - +
e) IGI -P1 Pz p, p,. a lJC1 - p 1 p 2 p 3803p,. , donde n1,n2,n:t .n.E Z • m1,m2,m3, m.
EN,y m. sii, para todo ·1 S_is 4?
9. Sea G = S,., el grupo simttrico de 4 símbolos y sea H el subconjunto de G dado por

a) Construya una tabla para mostrar que Hes un subgrupo abcliano de G.


b) ¿Cu.mtas clases laterales iu¡uierdas de H existen en G? _
e) Considereelgrupo(Z, X Z,. EB) doode (a, b) EB (c. á) = (a +e, b +á) y las sumas a+c,b+
d se ca]culan con la suma módulo 2. Demuestre que Hes isomodo a este grupo.
1 O. Si Ges un grupo de orden n y a E G, demuestre que a-= e.
11. Sea p un primo. (a) Si G tiene orden 2p, demuestre que todo subgrupo propio de Ges cíclico.
(b) Si G tiene orden p,i, demuestre que G tiene un subgrupo de orden p.
12. Dem~tre los corolarios 16.1 y 16.2.
13. Sean H, Ksubgruposdc un grupo G, donde e es el neutro de G.
a) Demuestn: que si IHI = 10 y IKI = 21, entoncesH n K= {,}.
= =
b) Si IHI m y IKr =n, con mcd(m, n) 1, demuestte que H K n = {e}.
14. Lo siguiente proporciona una forma alternativa de demostrar el teorema de Lagrange.
Sea G un grupo de orden n y sea H un subgrupo de G de orden m.
a) Defina larelación91 cnGcomosigue: Si a. b E G.cntoncesa91 b sitr1b EH. Demuestre
que 91 es una relación de equivalencia en G.
b) Paraa, b E G, demuesueque a9'1: b si y sólo si aH = bH.
e) Si a E G, demut:Stre que [a], la clase de equivalencia de a bajo9i:, satisface (a] = aH.
d) Para cualquier a E G,demuestreque laHI = IH I.
e) E,tabl=• ahora la conclusión del teorema de Lagrange; es deciI, que IH I divide a IGI.
15. a) Teoronack Fermat. Sip es un primo, demuestre que a'= a(modp} para cualquiera E Z.
(¿Cómo se relaciona esto con el ejercicio 20(a) de la sección 14.37)
b) Te.o~ de Euhr. Paratodon e z •ytodoa E Z. demucstreque si mcd(a. n) = 1,cntonccs
d"'' "'
l(mod n).
e) ¿Cómo se relacionan los dos teoremas de las panes (a) y (b)'?
d) ¿Existe alguna relación entre estos dos teoremas y los resultados de los ejercicios 6 y 10'?

16.4
Elementos de la teoría
de la codificación

En ésta y las siguientes seis secciones presentamos un área de las matemáticas aplicadas
llamada teoría algebraica tk la codificacíón. Esta teoría se inspiró en el artículo funda-
mental de Clallde Shannon (1948) y en los resultados de Marce! Golay (1949) y Richard
794 capítulo 16 Grupos, teoría de la c:odificación y método de enumeración de Polya

Hamming ( 1950). Desde entonces se ha convertido en un área de gran interés donde las
estructuras algebraicas, la probabilidad y la combinatoria desempeñan un papel importante.
Nuestro estudio será de nivel introductorio, mientras buscamos un modelo de la trans-
misión de infonnación representada por cadenas de las señales O y 1.
En la comunicación digital surgen algunos problemas cuando la información se trans,.
mite en forma de cadenas de ceros y unos. la presencia de "ruido" en un canal, cuando se
transmite cierta señal, puede provocar que se reciba una señal diferente, lo que hará que el
receptor tome una decisión equivocada. Por lo tanto, queremos desarrollar técnicas que
nos ayuden a detectar, e incluso a corregir, los errores de transmisión. Sín embargo, lo único
que podemos mejorar es la probabilidad de una transmisión correcta; sin más garantías.
Nuestro modelo utiliza un canal similrico binario, que se muestra en la figura 162. EJ
adjetivo binario aparece debido a que cada señal se representa mediante uno de los bits O
o 1. Cuando un transmisor envía la señal O o 1 en ese canal. a cada señal se asocia um.
probabilídad (constante) p de transmisión incorrecta Si esa probabilidad pes la misma
para ambas señales. el canal es simitricc. En este caso. por ejemplo. tenemos la probabili-
dad p de enviar un cero y recibir un uno. La probabilidad de enviar la señal O y recibirla en
forma correcta es entonces l - p. Todas las posibilidades aparecen en la figura 16.2.

O
z l-p 0
0 '

~M~""' :~~ """

1 1- p 1

El canal simétrico binario Figura 16.2

Consideremos la cadena e= 10110. Consideramos a e como un elemento del grupo~.


formado a panir del producto directo de cinco copias de (Zz, +).·Para abreviar la notacióo,
escribimos 10110en vez de (l. O, 1, 1, O).Al enviar cada bit (señal individual) dec a través
del canal sirn~trico binario, supondremos que la probabilidad de transmisión incorrecta es
de p = 0.05, de modo que la probabilidad de transmitir sin errores es (0.95') 0.77. =
En todo nuesto análisis de h. teoría de la codificación, supondremos que la transmisión
de cualquier señal no depende de la transmisión de las señales anteriores. En consecueo•
cia, la probabilidad de ocurrencíade todos estos sucesos independientes (en su orden pre--
detenninado) está dada por el producto de sus probabilidades individuales.
¿Cuál es la probabílidad de que la parte receptora de l mensaje de cinco bits reciba la
cadena r = 00110; es decir, el mensaje original con un error en la primera posición? La
probabilidad de la transmisión incorrecta del primer bit es de 0.05, así que con la hipótesis
de sucesos independientes, (0.05)(0.95)' =
0.041 es la probabilidad de enviar e = 10110 y
recibir r= 00110. Si e= l(X)()(), podemos escribir e + e = r e interpretar r comocl resultado
de la suma del mensaje original e y el palr6n de error particular e= 10000. Puesto que c.
r, e E ~y - 1 = 1 en Z::?, también tenemos e+ r =e y r+ e = c.
16.4 Elementos de la teoría de la codificación 795

Al transmitir e = 10110, la probabilidad de recibir r= OOIOOes

(0.05)(0.95)'(0.05)(0.95) = 0.002,

por lo que es poco probable que ocurra este error múltiple.


Por último. si transmitimos e= 10110, ¿cuá1 es la probabilidad de que rdifiera de e en
exactamente dos lugares? A fin de obtener la respuesta,. sumaremos las probabilidades
para cada patrón de error formado por dos unos y tres ceros. Cada uno de estos patrones
tiene una probabilidad de 0.002. Existen (!)de tales patrones, por lo que la probabilidad de
dos errores en la transmisión está dada por

C)co.os)'(0.95)' = 0.021.

Estos resultados dan lugar al siguiente teorema

MA 16.1 O Seac E z;:. Para la transmisión dec a través de un canal simétrico binario con probabili-
dad de transmisión incorre.eta p,
a) La probabilidad de recibir r= e+ e, donde t. es un patrón de error panicular, forma-
do por k unos y (n -k) ceros, c:s P'(l - p'r'-
b) La probabilidad de que se comentan k errores en la transmisión es ~)¡1(1 -ir'.

En el ejemplo 16.19, la probabilidad de cometer cuando mucho un error en la transmí-


sión de e= 10110 es (0.95)' + 11X0.05)(9.95)' = 0.977. Así, la probabilidad de tener
errores múlñples en la transmisión será considerada insignificante en el análisis de este
capítulo. Dicha hipótesis es válida si p es pequeña. En la realidad. un canal simétrico
binario se considera ''bueno" si p < 10"". Sin embargo. independientemente de lo estipula-
do. siempre pediremos que p < l fl.
Para mejorar la precisión de transmisión en un cana) simétrico binario, pueden usarse cier-
tos tipos de esquemas de codificación en que se proporcionan algunas seilalc:s adicionales.
Para m, n E z•, sean> m. Consideremos 0 'fo W ~ 'Li"'. El conjunto W consta de los
,mnsajes por transmitir. Añadimos a cada w E W n - m señales adicionales para formar la
palabra codificada e, donde e E z;. Este proceso es la codificaci6n y se representa me-
diante la función E: W ➔ z;. EntonccsE(w) = e y E(W) = C i; z;. Puesto que la función
E sólo añade bits a los mensajes (distintos), el proceso de codificación es inyectivo. Des-
pu~ de la transmisión,cserecibe comoT(c), dondeT(c) E z;. Por desgracia. Tnoes una
función, pues T(c) puede ser diferente en distintos tiempos de transmisión (ya que el ruido
en el canal cambia con el tiempo). (Véase Fig. 16.3.)

..,,.,
Mensaojew
(un elemento
Palabra codificada
asociada e • E{w)
{un elemtnto de Z5)
Lapalabta re<ibida
T (e) (un demento
de ZD
El resultado
decodifie.Jdo
(un ~todeZ1)

fiQ1,.1ra 16.3
96 capítulo 16 Grupos, teoría de la codificación y método de enumeración de Polya

Después de la recepción de T(c), queremos aplicar una función de decodificacióa &.


z; ➔ 7)f para eliminar las señales adicionales y, esperamos, recibir el mensaje original•·
Lo ideal sería queD • .T • E fuera la función identidad sobre W, con D : C ➔ W. Puesto 'l'I'
no podemos esperar esto, buscamos funciones E y D de modo que haya una probabi1idlll
alta de decodificar correctamente la palabra recibida T(c) y volver a tomar el memlje
original w. Además, queremos que larazónm/n sea lo más grande posible, de modo que•
se añada un o6mero e.xcesivo de señales a w para obtener la palabra codificada e= E(w).
Esta razón mln mide la eficiencia de nuestro esquema y se denomina razón de codifa.
ción. Por último, las funciones E y D deberían ser algo más que resultados teóricos; deba,
ser prácticos en el sentido de que puedan implementarse en forma electrónica.
En dicho esquema, las funcionesE y D son las funciones decodificación y decodificaa,I,,,
respectivamente, de un c6digo de b/qqu, (n,m).
Ilustramos estas ideas en los dos ejemplos siguientes.

Consideremos el código de bloque (m+ l, m) param = 8. Sea W= ZJ. Para cualquierw=


w1w2 ••• w1 E W,definimosE: ~ ➔ ~ comoE(w)=w 1w1 . . . w 1w9 , dondew,= J.~.. W;J
1
la suma se desarrolla módulo 2. Porejemplo,E(ll001101) = 110011011 y E(00110011)=
001100110.
Para toda w E Z!, E(w) contiene un número par de unos. Asf, para w = 11010110 y
E(w) = 110101101, si recibimos T(c) = T(E(w)) como 100101101, del número impar de
unos en T(c), sabemos que ha ocurrido un error en la transmisión. Por lo tanto, sí podemos
detectar errores sencillos en la transmisió~ pero parece que no hay manera de corregirlos.
La probabilidad de enviar la palabra codificada 110101101 y cometer a lo sumo m
error en la transmisión es

Los ~
~ --------
(1 -p)'+ (l)p(I -p)' .
biU se transmiten
en forma COtl'teta.
un bit ha cambiado en la transrmión
y st detecta un ffl'OC.

Para p =0.001 obtenemos (0.999)' + i; }(0.OOIX0.999)' 0.999964. =


Si detectamos un error y podemos retransmitir una señal de regreso al transmisor para
que repita la palabra codificada, y continuamos este proceso hasta que la palabra recibida
tenga un nwnere> par de unos, entonces la probabilidad de enviar la palabra codificada
I 10101101 y recibir la ttansnúsión correcta es aproximadamente de 0.999964.t
Por desgracia. si hay un número positivo par de errores en la transmisión, T(c) se acepta
como la palabra codificada correcta e interpretamos sus primeras ocho componentes como

t Patap = O.OOI. laprobabilid3d deque ocurra un número impar de errott:s eu la tnmsmisióu de la palaba
codificada 110101101 es

P- =(l)(0.999)'(0.00IJ + (l)(0.999)'(0.00I)' + (l)(0.999)'(0.00I)' + (1)(0.999)\0.00I)' + (l)(0.001)'


,¡,_ 0.008928251 + O.OOOOJ0083 + o.ocx:oxx:m + O.OOXMXXXJO + O.OOXMXXXJO- 0.008928334.

Si q, la probabilidad de la transmisión oorrccta de llOIOIIOl, es (0.999)' . l a ~ de que esta paWn


codificada se transmita y redba en forma corre,cu aa estas oondicioacs (de tttransmisióo) está dada por

q + Pa p,,r · q + (p impu'fq + (pimplr)'q + · · · ,,. q/(1 - p ~') .;. 0.91»964 (coa seis cifras decimab).
16.4 Elementos de la teoría de la codífk ación 797

el mensaje original. Este esquema es el c6digode verificaci6n de paridad (m + l, m) y es


adecuado solamente cuando no es probable que ocurran varios errores.
Si enviamos el mensaje 11010110 a través del canal. tenemos una probabilidad de
(0.999)1 = 0 .992028 de transmisión correcta. Mediante el código de verificación de pari-
dad, podemos incrementar las probabilidades de obtener el mensaje correcto hasta aproxi-
mamos a 0.999964. Sin embargo, se envía una señal adkional (y es posible que se necesi-
ten transmisiones adicionales) y la razón de codificación de.crece de 1 a 819.
Pero supongamos que en ve z de enviar ocho bits enviáramos 1(,() bits, en cadenas suce-
sivas de longitud 8. La probabilidad de recibir el mensaje correcto sin un esquema de
=
codificación sería (0.999)'" 0.852076. Con el método de verificación de paridad envia-
mos 180 bits, pero la probabilidad de una transmisión correcta aumenta a (0.999964'j1D-:;
0 .999280.

El código de triple ~petici6n (3m, m) es aquél en que podemos detectar y corngir errores
simples en la transmisión. Sim = 8 y W= Z!, definimos E: Z! ➔ Zl' comoE(w,w2 • •• w1w.)
=w1w1 ••• w,w1w1 ••• w, w,w2 ••• w,.
Por lo tanto, si w = 10110111, entonces e= E(w) = 10 1101111011011110110111.
La función de decooificación D: Zf ➔ Z! se guía por la regla de la mayoña. Por
ejemplo, si T(c) = 1010011100110111!0110110, entonces tendríamos tres errores, que
aparecen en las posiciones 4, 9 y 24. Decodificamos T(c) analizando las posiciones 1, 9 y
17, para ver qué señal aparece más veces. En este caso es l (que aparece dos veces). así
que decodificamos la primera entrada en el mensaje decodificado como 1. Si continuamos
con las entradas de las posiciones 2, 10 y 18, el resultado para la segunda entrada del
mensaje decodificado es O (que aparece las tres veces). A medida que continuamos,
recapturamos el mensaje correcto, 1011011 l.
Aunque tenemos más de un error de transmisión en este caso, todo está bien a menos
que aparezcan dos (o más) errores y que e l segundo error ocurra ocho o 16 espacios des-
pués del primero; es decir, si ocurren dos ( o más) transmisiones incorrectas para el mismo
bit del mensaje original.
Ahora bien, ¿cómo se compara este esquema con los otros métodos a nuestro alcance?
Si p = 0.001, la probabilidad de decodificar cocr<ctamente un solo bit es de (0.999)'+ (!) •
=
(0.001) (0.999)' 0.999997. Así, la probabilidad de recibir y decodificar correctamente
el mensaje de ocho bits es de (0.99999Tf= 0.999976, apenas un poco mej or que el resul-
tado del método de verificación de paridad (con el cual podríamos vemos obligados a
retransmitir, lo que incrementa el tiempo global de transmisión). En este caso. transmiti-
mos 24 señales para e l mensaje. así que nuestra razón es ahora de ½.
El precio de esta
mayor precisión y la capacidad para detectar y comgir los errores simples (Jo que no
podíamos hacer en los esquemas anteriores), es un incremento en el tiempo de transmi-
sión, pero no perdemos el tiempo con retransmisiones.

EDOCI0S 16.4 1. Sea e un conjuoto de palabras codificadas, donde e C ~- En los ,;gwentes ejercicios se dan
los datos de dos de las cantidades e (patrón de error), r (palabra recibida) y e (palabra codifica-
=
da), con r e + ~- Determine d tercer término.

a) c=l0!Oll0, ,=1011111 b) e • 1010110, < = 0101101


e) < = 0101111, , = 0000111
798 Capitulo 16 Grupos, teoría de la codificación y método de enumeración de Polya

2. Un canal s~co binario tiene una probabilidad p = O.OS de transmisión incorrecta. Si


transmite la palabra codificada e =0110ll101, ¿cuál es la probabilidad de que (a) rtttl>amos
011111101? (b) recibamos r = 1110111007 (e) ocuna un solo error? (d) ocurra un error
(e) ocurra un error triple? (t) se presenten tres errores. ninguno de los cuales sea
con otro?
3. Sea E : Vi ➔ Z1 la función de codificación para el código de repetición triple (9. 3).
a) Si D: Zl ➔ Zl es la función de decodificación com:spoodientc, aplique D para
las palabru reeitridas (i) 11110 1100; (ii) 000100011; (iii) 010011111.
b) Encuentre tres palabras recibidas diferentes r para las que D(r) = 00>.
e) Paracadaw EZl,¿cuántovale ILr'(w)I?
4. El código de cinco repeticioocs (Sm, m) tieoe la función de codificación E: Z'l ➔ 'Jlr,
E(w) =k'WWWW. La decodificación con D : 'O{- ➔ 2;' se realiza mediante la regla de la
(Ea este caso, podemos corregir los mores senciUos y dobles hechos en la transmisión.)
a) Sip = 0.05, ¿cuáles la probabilidad de la transmisión y decodificació n correcta de la sdlallt
b) Responda la parte (a) para el mensaje 110 en ,..ez de la sei'iaJ O.
e) Para m = 2, decodifique la palabra recibida r = 0111001001.
d ) Si m =2, encuentre tres palabras recibidas r tales que D(r) =00.
e) Si m =2 y D : 'ZJ,,º ➔ Z½, ¿cuánto vale ID-1(w)I para cada w E ~

16.5
La métrica de Hamming

En esta scccíón desarrollaremos los principios generales para el análisis de las capacidl-
des de detección y corrección de errores de un esquema de codificación. Estas i ~fe.
ron desarrolladas por Richard Hamming.
Comencemos porconsideraruncódigoC ~~.tal quec1 =0111.c2 = 1111 E C. Enesae
caso. tanto el transmisor como el receptor conocen los elementos de C. As(. si el transmi-
sor envía c 1 pero la persona que recibe la palabra codificada recibe T(c1) como 1111,ea-
tonces esta persona cree que se transmitió c2 y toma la decisión ( equivocada) que implica
e¡. En consecuencia, aunque sólo se cometió un error de transmisión, los resultados p)-
drfan ser inconvenientes. ¿Por qué ocurre esto? Por desgracia, tenemos dos palabras codi-
ficadas casi iguales. Son cerc:anas entre sf, ya que sólo difieren en una componente.
Describimos esta idea de cercanía con mayor precisión en la forma siguiente.

Definición 16.9 Paracualquierelemento.r• .rJXz ... x. E Zi donde n E z+, el peso de x. que se denota cm
p(x), es el número de componentes x, de x, para 1 s i s n. tales que x,= 1. Si y E Zi, la
distancia entre x y y, que se denota con d(x, y), es el número de componentes tales que .r¡ #-
y;,. para 1 :S i :S n.

Paran = 5, seanx = 01001 y y = 11101. Entonces p(x) = 2, p(y) = 4 , y d(x,y) =2.Ademm,


x+ y= 10100, de modo que p(x +y) = 2. ¿Es casualidad que d(x, y)= p(x + y)? Para cada
1 :Si :S S,x;+ y;contribuye con una unidad ap(x+y) <=)X;#y; <:=)X;,y,contribuyeconUD1
16.5 La métrica de Hamming 799

unidad ad(.r, y). [En realidad, es10 es cierto para todo n E z•, por lo que p(.r +y)= d(.r, y)
para todos x, y E ~ -1

Si .r, y E ~. escribimos d(x, y) = I,;'., d(.r,, y1) , donde

para cada 1 s i s n, d(.r,,yJ = {º1 si.•,: y,


SIZ¡ -,-y; .

.EMA16.2 Para todos x, y E ~ . p(.r + y) S p(x) + p(y).


- llemostramnos el lema mcdimie el análisis. para cada I s ; s n, de las
compoocotesx;, J;.X;+ y, de x, y y x + y , respectivamente. Solamente una situación haría
que es1a desigualdad fuera falsa: six,+ y, = 1 mientras quex,= Oy y1=0, para algún I si
s n. Pero es10 nUDCa <>eurn, pues x, + y1= 1 implica que CllClamODte una de las variables
,x.o y, es l.

En el ejemplo 16.22 vimos que

p(x + y) = p(lOIOO) = 2 s 2 + 4 = p(OIOOI) + p(IIIOI) = p(.r) + p(y).

rEOREMA 16.11 La función diS1aDCÍI d definida en~ x ~ satisface lo siguicnle para todosx, y, z E ~-

a) d(.r,y)'<0 b) d(.r,y)=0e>.r • y
e) d(.r,y) = d(y, .r) d) d(x, z) s d(.r,y) + d(y,z)

Demostroción, Dej aremos al lector las tres primeras part<s y demos=mos la parte (d).
En~. y + y = O, por lo que di_x, z) = p(.r + z) • p(.r+ (y + y) +,)= p((.r +y) + ( y+ z))
s p(.r+ y)+ p(y+z), por el lema 16.2. Comop(.r+y) =d(x, y) yp(y+z) =d(y,z), se sigue
el resullado. (Es1a propiedad se conoce por lo general como ladnigualdad triangular.)

Cuando una función satisface lti cuatro propiedades enumeradas en el teorema 16. 11.
es una función distancia o métrica y decímos que (Zi, d) es un espacio mitrico. Por lo
tanto. d (dada arriba) se conoce con frecuencia como la mitrka ch Hamming.

Definición 16.10 Paran, k E z•y .r E z ; , laes_fera de radiokcon centro cn.rsedefinccomoS(x, k)= (y E


~ ld(x. y) s k].

- Paran=3y.r= 110 E Zj,S(.r, I)= {110,010,100,111) yS(.r, 2)= {110,010,100, lll,


000, 101,011}.

Con estos antecedentes podemos pasar a los dos resultados principales de esta sección.
800 Capítulo 16 Grupos, teoña de la c;odificad6n y método de enumeración de Polya

TEOREMA 16.12 '. Sea E: W ➔ C una función de codificación con el conjunto de mensajes W k z; y el
conjunto de palabras codificadas E(W) = C k z¡, donde m < n. Para k E z·. podemos
detectar los errores de transmisión de peso s k si y sólo si la distancia mínima entre las
palabras codificadas es al menos k + l.
Demostración: El transmisor y el receptor conocen el conjunto C, por lo que si w E W es el
mensaje y se transmitec=E(w), sea e# T(c) = r. Si la distancia mínima entre las palabras
codificadas es al menos k + 1, entonces la transmisión de e puede producir hasta k errores
y r no estará enumerado en C. Por lo tanto, podemos detectar todos los errores e tales que
p(e) s k. Recíprocamente, sean ci, c2 palabras codificadas tales que d(c1, ei) < k + 1.
Entoncesc2 = c 1 + e, donde p(e) s k. Si envíamos c 1 y T(c1) = c2, entonces pensaríamos que
fue enviado c 2, lo que no serviría para detectar un error de peso s k.

¿Qué podemos decir de nuestra capacidad para corregir errores?

TEOREMA 16.13 Con E, W y C como en el teorema 16.12, y k E z._


podemos construir una función de
decodificación D: ~ ➔W que corrija todos los errores de transmisión de peso S k si y
sólo si la distancia mínima entre las palabras codificadas es al menos 2k, + l.
DemoWación: Parac E C, consideremosS(c, k)= {rE z¡ ld(c,r):,; k). DelinimosD: z¡ ➔
Wcomo sigue. Sir E Zl y r E S(c. k)para algunapalabracodificadac, entonccsD(r) = w,
donde E(w) =c. [En este caso, e es la (única) palabra codificada más cen:ana a r.J Sir 'l.
S(c, k) para todo e E C, entonces definimos D<.r) = wo, donde w 0 es un mensaje arbitrario
que permanece fijo una vez elegido. El único problema que podría surgir en este caso es
que D no fuera una función. Esto ocurrirá si existe un elemento r en z; tal que r esté en
S(c,.k) yS(c,, k) parados palabras codificadas distintas e,. e,. Peror E S(c,.k) =,d(r, e,):,
k, y r E S(c,, k) =, d(r, e,):, k, de modo qued(c,. e,) :S d(c,. r) +d(r, c2 ):, k +k<2k+ l.
En consecuencia. si la distancia mínima entre palabras codificadas es al menos 2k + 1,
entonces Des una función y decodificará todas las palabras recibidas posibles, corregirá
cualquier error de transmisión de peso s k. Recíprocamente, sic1o c2 E Cyd(c1, ci) :S 2k,
entonces podemos obtenerc2 a panir de c 1 haciendo un máximo de 2k cambios. Empezan-
do en la palabra codificada Ci, hacemos aproximadamente la mitad (exactamente, ld(ci,
c2 ) nJ) de estos cambios. Esto implica que r = c 1 + ei, con p(e1) s k . Si continuamos a
partir de r, hacemos los cambios restantes para llegar hasta c2 y ver que r + e 2 = c2, con p(e2)
S k. Pero entonces, r; c2+ e 2. Ahora, como e,+ e 1 = r = c2+ e 2 y p(e1), p(ei) S k, ¿cómo
podríamos decidir cuál es la palabra codificada de la que surge r? Esta ambigüedad surge
de un e rror posible de peso s /c. que no puede corregirse.

- SiW=Z~,seaE:W➔ Z~dadapor
E(OO) = OOOXJO E(lO) = 101010 E(0l) = 010101 E(11)=111111.

Entonces la distancia mínima entre las palabras codificadas es 3, de modo que podemos
detectar los errores dobles y corregir los sencillos.
16.6 La verificaó6n de paridad y matrices generaderas 801

Si

S(OOOOOO, 1) =(x E Z!ld(OOOOOO,x) s !}


= {000000, HXXXXJ, 010000, OOllXXl, IXXll OO, 000010, OOIXXll},
la función de decodificación D: Z! ➔ W da D(x) = 00 para todo x E 5(000000, 1).
En forma análoga,

S(OlOlOI, 1) = (x E Z!Jd(Ol0101,x) s l}
= {010101, 110101, IXXllOl, OlllOl, OllXXll, 010111, 010100},
y en este casoD(x) =01 para todox E S(0IOI0I, 1). En este momento, la definición deD
sirve para 14 elementosdcZ !. Si seguimosdefiniendoDpara los 14 elementosdeS(I0 I0IO,
1) y S( IIIIII, 1), todavía falta considerar otros 36 elementos. Definimos D(x) = 00 (o
cualquier otro mensaje) para estos otros 36 elementos y tenemos una función de
decodificación que corregirá los errores simples.
Respecto a la detección, sic= 010101 yT(c) =r= 111101. podemos detectar este error
doble. pues r no es una palabra codificada. Pero si T(c) = r 1 • 111111, ocurre un error
triple, así que podríamos pensar que e= 111111 y decodificar en forma incorrecta r 1 como
11, en lugar del mensaje oon<cto 01.

16.6
La verificación de paridad
y matrices generadoras

En esta sección presentamos un ejemplo en el que las funciones de codificación y de


decodificación eswi dadas por matrices sobre Z,. Una de csw matrices nos ayudara a
localizar el código más cen:ano a una palabr:a recibida dada. Esto será de particular utili-
dad si el conjunto C de palabras codificadas crece.

Sea
l O O 1 1
G= O 1 O O 1 1
[
º]
O O 1 1 O 1
una matriz 3 X 6 sobre ~- Las primeras tres columnas de G forman la matriz identidad
3 X 3 / ,. Si A denota la matriz formada con las últimas tres columnas de G, escribimos G
= [/, JA] para denotar su csttuctun. La matriz (dividida) Ges una matriz g,n,radora.
Usamos G para def>nir una función de codificación E: Z! ➔ Zl como sigue. Para w E
ZJ, E(w) = wG es el elemento en Z! obtenido al multiplicar w. considerado como un
vector fila tridimensionaJ, por la matriz G de su derecha. A diferencia de los resultados
sobre el producto de matrices en el capírulo 7. en estos cálculos tenemos 1 + 1 =O, no 1 +
l=I.
(Aunque el conjunto W de mensajes no sea todo de Zl, supondremos que todo Zl está
codificado y que tanto el transmisor como el receptor conocerán los mensajes reales de
importancia y sus correspondientes palabras codificadas.)
802 Capítulo 16 Grupos, teoría de la codificación y método de enumeración de Pofya

Por ejempJo, vemos que

E(llO) = (llO)G = [110] rn ~ ~! i !] = [110101),

E(OlO) = (OlO)G = [010] [~ ~ ~ ~ ~ ~] = [010011).


O O 1 O 1

Observe que podemos obtener E(IIO) sumando las dos primeras filas de G, mientras
que E(OIO) es simplemente la segunda fila de G.
El conjunto de palabras codificadas obtenido mediante este método es
C = {000000, 100110, 010011, 001101, 110101, 101011, 011110, 111000} ¡;; Zl,
y podemos volver a tomar el mensaje correspondiente eliminando las tres últimas com~
nentes de la palabra codificada. Además, la distancia mínima entre las palabras codifica-
das es 3, por Jo que podemos detectar errores de peso s 2 y corregir los errores simples.
Para cualquier w: w1w2w3 E Zt E(w) = w1w2w1w..w,w, e Zf. Puesto que

l O O 1 1
E(w)= [w1 w2 w3] O 1 O O 1 1
[
º]
001101
= [w, w,w,(w1 + w3)(w1 + w,)(w, + w3)],

tenemos w,.= w 1 + Wp w,= w 1 + w 1 • w6 = w 2 + w 3; estas ecuaciooes son las ecuaciones tk


verific.aci.ón ch paridad. Como W¡ E Z, para cada 1 S i :S 6. se sigue que w, = -W; y
entonces podemos escribir nuevamente las ecuaciones como

w1 +w3+w,. =O
w1 +w2 +w, =O
w2 + w3 + w6 =0.
Así, tenemos que

w,
[!
o 1 1
o o 1
1 o o
o
~] w,w,
w, = H · (E(w))•= m
....
donde (E(w))" denota la traspuesta de E(w). En consecuencia, si r = r1 r, ... r, E Z!,
podemos identificar r como una palabra codificada si y sólo si
16.6 La verificación de paridad y matrices generaderas 803

SiescribimosH= (BIIJ, observa.mosque si intercambiamos las filas y columnas deB,


obtenemos A. Poc lo tanto, B=Aw.
Por la teoría desarrollada antes acerca de la corrección de errores. y como la distancia
mínima entre las palabras codificadas de este ejemplo es 3, podemos desarrollar una fun-
ción de decodificación que corrija los errores simples.
Supongamos que recibimos r = 110110. Queremos determinar la palabra codificada e
que sea el vecino más cercano de r. Si existe una larga lista de pal.abras codificadas que
debemos confrontar con r, sería mejorexaminar primero H · ~. que es el síndrome der. En
este caso,

1 o o 1 o] ~11l =[1],
101100 O
H·ru = [l
011001 1
o
de modo que r no es una palabra codificada. Poc lo tanto, al menos detectamos un error. Si
revisamos de nuevo la lisia de palabras codificadas, vemos qped(IOOll0, r) = l. Para las
demís e E C, d(r, e) a: 2. Si escribimos r= e+<= 100110 + 010000, vemos que e l error
de transmisión (de peso 1) aparece en la segunda componente der. ¿Es coincidencia que el
síndrome H • rtr baya producido la segunda columna deH? Si no. entonces podríamos usar
este resultado para ver que si ocurrió un solo error de transmisió~ éste tuvo lugar en la
segunda componente. Si cambiamos la segunda componente de r, obtenemos e; e l mensa-
je w comprende las tres primeras componentes de c.
Sea r = e + e, donde e es una palabra codificada y e es un patrón de error de peso 1.
Supongamos que l está en la i-ésima componente de e, donde 1 S i s 6. Entonces
H · ru=H · (e+ e)1r = H · (c 11 + e"} = H • c 1r + H · eu.
Como e es una palabra codificada, se sigue que H - c'6 = O, por lo que H • rw = H • ¿r:: i-
tsima columna de la mattiz H. Así. e y r difieren solamente en la i~sima componente y
podemos determinar e al intercambiar simplemente la i-ésima componente der.
Nos interesan principalmente las transmisiones en que los errores ml'.iltiples son raros,
por lo que esta técnica tiene gran valor. Sin embargo, si pedimos más, estaríamos esperan-
do obtener demasiado.
Supongamos que recibimos r = 000111. Calculamos el síndrome

H · r" = 1] O
[
1 1 O O [ 1]
1 O O 1 O O= 1 ,
º]
011001: 1

y obtenemos un resultado que no es una columna de H. Sin embargo, podemos obtener


H , rw como la suma de dos columnas de H. Si H • ,-V provino de las columnas primera y sexta
de H, al corregir estas componentes en r obtenemos la palabra codificada 100110. Si su-
mamos las columnas tercera y quinta de H para obtener este síndrome, después de cambiar
las componentes tercera y quinta de r obtenemos una segunda palabra codificada, 001101.
!04 capitulo 16 Grupos, teoria de la codificación y método de enumeración de Folya

Así, no podemos esperar que H corrija los errores múltiples. No debe sorprendemos que la
distancia mínima entre las palabras codificadas sea 3.

Resumimos los ~ultados del ejemplo 16.25 para la situación general. Para m, n E z• con
m <n. la función de codificación E: Z; ➔ z; está dada por una matrizm x n G sobreZ,. Esta
matriz G es la matriz generadora del código y tiene la forma [I,. IA], donde A es una matnz
m X (n- m).Enestccaso,E(w)=>,(,paracadamensajewE ZrYelcódigoC= E(Z;)C z;.
Lamarriide verificacüin tU paridadasociadaHes una matriz (n-m) x n de la forma
[A•jf. _.] . También podemos usar esta matriz para defuúr la función de codificación E,
puesto que si w = w 1w1 • .• w,. E l.;', entonces E(w) = w1w1 ••• w_..,w,.. 1 • • • w., donde
podemos determinar a w• • 1, •• •• w. del conjunto de n-m ecuaciones (de verificación de
paridad) que swgen de H • (E(w))"= O, el vector columna den- m ceros.
Esta única matriz de verificación de paridad H también proporciona un esquema de
decodificación que corrige los errores simples de transmisión si:
a) H no contiene una columna de ceros. (Si la i-mma. columna de H tuviera solamente
ceros y H · r" = Opara una palabra recibida r, no podrlamos decidir si r era una palabra
codjficada o una palabra recibida aiya i-ésima componente fue transmitida en fonna
incomcta. No queremos comparar rcon todas las palabras codificadas si Ces grande.)
b) Ningún par de columnas de H son iguales. (Si la i-ésima y j-ésima columnas de H
son iguales y H •r 11 es igual a esta columna repetida.. ¿cómo decidir cuál componen•
te de r debemos cambiar?
Cuando H satisface estas dos condiciones, obtenemos el siguiente algoritmo de
decodificación. Para cualquier r E Zi, si T(c) = r. entonces
1) Si H · r" = O, pensaremos que la transmisión fue correcta y que r es la palabra
codificada que fue transmitida. El mensaje decodificado consta entonces de las pri-
meras n componentes de r.
2) Si H · ,.Ues igual a la i-ésima columna de H. pensamos que hubo un error simple en la
transmisión y cambiamos la i-ésima componente de r para obtener la palabra codifi-
cada c . En este caso, tas primeras m componentes de e producen el mensaje original.
3) Si no ocurre ninguno de los dos casos anteriores, pensamos que hubo más de un error
de transmisión y que no podemos dar una forma confiable de decodificación en esta
situación.
Terminaremos la sección con un comentario final acerca de la matriz H. Si partimos de
una matriz de verificación de paridad H = [B 11. _.] y la usamos en la forma descrita arriba
para definir la función E, entonces obtenemos el mismo conjunto de palabras codificadas
generado por la única matriz generadora asociada G = [/. j B").

:JERCICIOS 16.5 1. Parael ejemplo 16.24, enumere los elementos en S(lOlOIO, 1) y S{llll 11.1).
'16.6
2. Decodifique las siguientes palabras recibidas. en el ejemplo 16.24.
a) 110101 b) 101011 e) 001111 d) 110000
3. a) SixE Z\' ,detonninc IS{;c, 1)1 , IS{;c,2) J. JS(x.3)1 .
e e
b) Paran. k z· con 1 s k s n. si x Zi, ¿cuál es el valor de IS(x, k) I?
4. Sea E : ~ ➔ zp una función de codificación en que la distancia mínima entre tas palabras
codificadas es 9. ¿Cuál esd máximo valor dektal que podamos detectar errores de peso :S k?
Si queremos corregir errores de peso s n, ¿cuál es el valor máximo den?
16.6 La verificacióo de paridad matrices eraderas 805

5. Para cada una de las sl¡plimle$ ~dedeoodí6c:acióo, eorucmre la distaociamlnimaemre las


palabras codificadas. Anllitt la capacidad de deu<dóo y c:omccióG de errores de cada código.

a) E:Z}-+~ b) E: z~-z;o
00-+00001 01-+ 01010 00-+ IXlOOIXlOOOO Ol-+0000011111
10-+ 10100 ll -+ 11111 10-+ 1111100000 ll-+1111111111
e) E:Zf-+ ZÍ d) E:z~-zt
000-+ 000111 001-+ 001001 000--->00011111 001-+ OOlllO!O
OI0-+010010 Oll-+011100 0!0-+ 01010101 Oll-+01110000
100-+ 100100 101-+ 101010 100-+ 10001101 101-+ 10101000
110-+110001 111-+ 111000 110-+ 11000100 lll--->11100011

6. •J Use la maaizde verificacióodeparidadH del ejemplo 16.25 paradecoctificarlas sl¡wentes


palabras recibidas.
0 lll lOI i) 110101 i,") 001111 iY) 100100
• J 110001 n") llllll ril") 111100 "rili) 010100
b) ¿Esún detcnniruodos de forma llnica los resultados de la pane (a)?
7. La funcióo de coojficacióo E : Z! ➔ Z! esú dada por la maaiz geocradora

I O 1 1
G • [ O I O 1 1.
º]
a) Determine todas las palabras coojficadas. ¿Qut puede decir de la capacidad de delecciónde
erro,es de este código? ¿Qu~ puede decir de su capacidad de com:ccióo de enores?
b) Enalentrc la maaiz de vcrificacióo de paridad H a,oáada.
<) Use H para decoctificar cada nna de las ,ig,rientes palabras r<cibídas.
0 11011 11) 10101 111) 11010
iY) 00111 V) 11101 ri) 00110
8. Defina la función de codificación E: Zl ➔ Z! por medio de la maaiz de verificación de pañdad
I O 1 1 O O]
H= 1 1 O O I O .
[
1 O I O O 1
a) Determine todas las palabras codificadas.
b ) ¿Cooige este código todos los errores simplC$ de transmwón?
9. Encuentre las matrices generadora y de verificación de parid¡d para el esquema de oodifica..
ción coa verificación de paridad simple (9, 8) del ejemplo 16.20.
1 O. a) Muestre que la matriz I X 9 G = (1 1 1 . . . 1) es la maaiz geocradora del código de m,eve
repeticiooes (9, 1).
b) ¿Cllil es la matriz de verificación de paridad H asociada. a este caso?
11. Para nn código (n, m) C coo IDllriz geocradoraG = (I.IAJ y matriz de verificación de paridad
H = WII, .•J. el código (n, n-m) C' con matrizgcnendora [I, .• !A"J y matriz de verificación
de paridad ¡A I/.J es el código d""1 de C. Muestre que los códigos de los ejercicios 9 y 10
forman nn par de códigos duales.
12. Dadon e z·. seaM(ft. k) ¡;; Zl d coojuntoque cootimed mlmero múimode palabras codificadas
de longitwl n. dende la distancia mlnima emre las palabras codificada.< es 21: + 1. Demuestre que

2" 2"
k~(:) " IM(n,k~SL;_(:)'
806 Capitulo 16 Grupos, teoría de la codificación y método de enumeración de Polya

(La cota superior sobre lM(n. k)I es la cota de Hamming; la cota inferior se conoc:c comoa
d< Gilb,rt.)

16.7
Códigos de grupo: Decodificación
con líderes de clase

Ahora que hemos analizado el materia1 introductorio a la teoría de la codificación, es la


de ver cómo entra la estrucrura de grupo en el tema

Definición 16.11 SeaE: Z; ➔ Z¡ una función de codificación. El código C= E(Z;) es uncódigod<i..


si e es un subgrupo de Z¡.

Recordemos la función de codificación E: z¡ ➔ Z! (del ejemplo 16.24) tal que


E(OO) = 000000 E(lÓ) = 101010 E(Ol) = 010101 E(ll)=llllll.
En este caso, Z~ y z¡ son grupos bajo la suma componente a componente módulo 2; el
subconjunto C= E(Zl)= (000000, 101010, 010101, 111111} es un subgrupode Zjypat
lo tanto, es un ejemplo de un código de grupo. (Observe que C contiene a 000000, 11
elemento cero de Z f.)
En general, cuando las palabras codificadas forman un grupo, veremos que es m.is fá:i
calcular la distancia mínima entre las palabras codificadas.

TEOREMA 16. 14 En un código de grupo, la distancia mínima entre palabras codificadas distintas es el mlli-
mo de los pesos de los elementos distintos de cero en el código.
Demostración: Sean a, b, e E C tales que a f.b, d(a, b) es mínimo y e es distinto de cero,
con peso mínimo. Por el cierre del grupo C, a + b e s ~ codificada. Como d(a. b) =
p(a + b), por la elección de e tenemos que d(a, b);;, p(c). Recíprocamente, p(c) = d(c, 1).
donde O es una palabra codificada pues Ces un grupo. Entonces, d(c. O) ;,: d(a, b) por la
elección de a. b, de modo que p(c);,: d(a, b). En consecuencia, d(a, b) p(c). =

Sí Ces un conjunto de palabras codificadas y ICI = 1024, tenemos que calcular('~')=


523,776 distancias para determinar la distancia mínima entre las palabras codificadas.
Pero si podemos reconocer que C tiene una estructura de grupo. sólo necesitamos calcular
los pesos de los 1023 elementos distintos de cero de C.
¿Hay alguna forma de aseguramos que las palabras codificadas forman un grupo? Pur
el teorema 16.S(d), si E: z; ➔ z; es un homomorfismo de grupos, entonces C = E(Zl')
será un subgrupo de z;-. Nuestro siguiente resultado usará este hecho para mostrar que los
códigos obtenidos al usar una matriz generadora G o una matriz de verificación de paridad
H son códigos de grupo. Además. la demostración de este resultado vuelve a confirmar la
observación hecha (al final de la sección anterior) acerca de el código que surge de una
matriz generadora G o su matriz de verificación de paridad H asociada.
16.7 Códigos de grupos: Decodificación c:on líderes de dase 807

16.1 5 Sea E: Zf ➔ Zi una función de codificación dada por una matriz generadora G o la
matriz de verificación de paridad H asociada. Entonces C = E( Zi') es un código de grupo.
Demostración: Estableceremos el resultado demostrando que la función E que swge de G
o H es un homomorfismo de grupos.
Si x; y E Zl', entonces E(x +y)= (x + y)G = xG + yG = E(x) + E(y). Por lo tanto, E es
un homomorfismo y C = E(ZJ') es un código de grupo [por la pane (d) del teorema 16.5].
Para el caso de H, sixes un mensaje.entoncesE(x) =x1x2 •• • x.,.x'"'. 1 ••• x. , dondex = x1
x, . • . x., E z; y H • (E(x)"f'= O. En panicular, E(x)quedadeterminadademanera única por
estas propiedades. Si y es un mensaje. entonces también lo es x + y y E(x + y) tiene (.t1 + y 1) ,
(x, + y,), ... , (x.+ y.,) como primeras m componentes, al igual que E(x) + E(y). Además,
H · (E(x) +E(y)"f'= H · (E(x)"+E(y)") =H · E(x)"+ H · E(y)"= 0 + 0 = 0. Como E(x+ y)
es el Onico elemento de Zi con (x1 + y1). ( ~ + yz), . .. • (.t.,+ y..) como sus primeras m
componentes y tal que H • (E(x + y))"= O, se sigue que E(x +y) = E(x) + E(y). Por lo tanto,
E es un homomorfismo de grupos y, en consecuencia. C = {e E ~ 1H · e"= O) es un
código de grupo.

Ahora usamos la esuuctura de grupo de C, junto con sus ciases en Zi, para desarrollar
un esquema de decodificación. Nuestro ejemplo usa el código desarrollado en el ejemplo
16.25, pero el procedimiento se aplica a cualquier código de grupo.

Desarrollaremos una rabia de decodificación de la manera siguiente.


1) Primero enumeramos en una fila los elementos del código de grupo G, empezando
con la identidad.
000000 100110 010011 001101 110101 101011 011110 111000.
2) A continuación seleccionamos un elementox de Zi (Zi, en general) de modo quex
no aparezca en la tabla desarrollada hasta el momento y que tenga peso mínimo.
Después enumeramos los elementos de la clase x + C. de modo que x + e quede
directamente debajo de e para cada e E C. Para x = 1()(X)()() tenemos

000000 100110 010011 001101 110101 101011 011110 111000


100000 000110 110011 101101 010101 001011 111110 011000.
3) Repetimos el paso 2 hasta que las clases induzcan una panición de Z! (~. en
general). Esto produce la tabla de decodificacüm que se muestra en la tabla 16.8.
4) Una vez construida Ja tabla de decodificación, para cada palabra recibida r encon-
tramos la columna que contiene a r y usamos las tres primeras componentes de la
palabra codificada e de la parte superior de la columna para decodificar r.
A panir de la tabla vemos que las palabras codificadas para las palabras recibidas

, , = 101001 ,,= 111010 ,,=001001 ' • = 111011


son
e,= 101011 e, = 111000 e,= 001101 e,= 101011 ,
808 CapítlJlo 16 Grupos. teoria dela codificación y método deenumeraóón de Polya

respectivamente. De estos resultados. podemos concluir que los mensajes respectivos sea

W¡ = 101 w, = 111

Tabla 16.8
Tabla de decodificación para el código del ejemplo 16.25

000000 100110 010011 001101 110101 101011 011110 111000


100000 000110 110011 101101 010101 001011 111110 011000
010000 110110 000011 011101 100101 111011 001110 101000
001000 101110 011011 000101 111101 100011 010110 110000
000100 100010 010111 001001 110001 101111 011010 111100
000010 100100 010001 001111 1io111 101001 011100 111010
000001 lOOlll 010010 001100 110100 101010 011111 111001
010100 110010 000111 011001 100001 111111 001010 101100

Las entradas de la primera columna de la tabla 16.8 se llaman los lúkres de clase. Pan
las primeras siete filas, los líderes de clase son iguales en todas las tablas, con alguaa
posible permutación de las filas. Sin embargo, para la última fila. podríamos usar l<XXX)I
o 001010 en vez de 010100 puesto que también tienen peso mínimo 2. Así, la tabla no
tiene que ser única [Como resultado, no podemos corregir todos los errores dobles pueslO
que podría no haber una única paJabra codificada a una distancia mínima para cada reo la
última clase, la correspondiente al líder de clase 010100. Por ejemplo, r;;;; 001010 tiene
tres palabras codificadas más cercanas (a distancia 2): 000000, 101011 y 011110.]
¿Cómo nos ayudan realmente Jos líderes de clase? Parece que hubiéramos utilizado las
palabras codificadas en la primera fila para deccxlificar r1, r2 , r 3 y r•.
Consideremos ]as palabras recibidas,1= 101001 y r 2 = 111010 en la sexta fila. dondcel
líder de clase es x =00001 O. Al calcular los síndromes, tenemos que

H · (r,)" = m = H ·(r2)" = H ·r•.

Esto no es coincidencia.

TEOREMA 16.16 Sea C !:; Zj un código de grupo para nna matriz de verificación de paridad H y sean T¡, r, E
z;. Para la tabla de ciares de C en Z¡, r1 y r2 están en la misma clase de C si y sólo si H ·
(r1) 11= H · (r 2Y'-
Demostn.ci6n: Sir1 y r 2 están en ]a misma clase. entonces r 1 =x+ c 1 y r 2 =x+c2 , donde:xes
el üder de clase y c 1 y c2 son las palabras codificadas de la parte superior de las columnas
respe.ctivas parar1 y r 2• EntoncesH · (r1'f=H · (x + c1"?= H ·xr1+H · (c,)tt=H •xa:+ O=H·
x•. puestoquec1 es unapaJabracodificada. De la misma forma.H • (r2Y'= H · xtt, de modo
que r i, r 2 tienen el mismo síndrome. Recíprocamente, H • (r1"JZ= H • (r2 Y, ⇒ H • (r1 + r.¡f=
O ⇒ r1 + r 2 es una palabra codificada c. Por lotanto,r1 + r2 =e, de modo quer1= r2 + cy , 1 E
rz+ C. Puesto que , 2 E r 2 + C, tenemos a ri, r 2 en la misma clase.
16.7 Códi<>os de orupos: Dtcodificaci6n con Ude<es de clase 809

Al decodificar las palabras recibidas y usar la tabla 18.6, debemos buscar en los 64
elementos para encontrar una palabra recibida dada. Para C !;:;: ~. existen 4096 cadenas,
cada una con 12 bi1S. Tal proceso de búsqueda es ledioso, as( que es probable que deba
pensarse en un computador para que realice la búsqueda. Por el momento, parece que esto
significa guardar toda la tabla: 6 X 64=384 bi1Sdealmacenamientopara la tabla 16.8; 12 x
4096 = 49 152 bits para C ~ Zl1 . Quisiéramos mejorar esta situación. Sin embargo, antes
de que mejoren las cosas, empeorarán si agrandamos la tabla 16.8, como se muestra en la
tabla 16.9. Esta nueva tabla incluye a la izquierda de los líderes de: clase (las traspuestas
de) los síndromes de cada flla.

Tabla 16.9
1-bla de decodificación 16.8 con sindromes

000 OO(XXJ()
100110 010011 001101 110101 101011 0llll0 111000
110 100000 000110 110011 101101 010101 001011 111110 011000
011 010000 110110 000011 011101 100101 111011 OOlll0 101000
101 001000 lOlll0 011011 000101 111101 100011 010110 110000
100 000100 100010 010111 001001 110001 101111 011010 llllOO
010 000010 100100 010001 001111 ll0lll 101001 0lllOO 111010
001 000001 100111 010010 001100 110100 101010 0lllll 111001
111 010100 110010 000111 0llOOl 100001 llllll 001010 101100

Ahora podemos decodificar una palabra recibida r mediante el siguiente procedimiento.


1) Calculamos el síndrome H · r•.
2) Encontramos el líder de clase x a la derecha de H • r".
3) Sumamos x a r para obtener c. (La palabra codificada e que estamos buscando en la
parte superior de la columna que contiene a r satisface e+ .r =ro e = x + r.)

En consecuencia. lo único que necesitamos de la tabla 16.9 son las dos primeras colum-
nas, cada una de las cuales necesita (3X8) + (6)(8) = 72 bi1S de almacenamiento. Con 18
bits de almacenamiento adicionales paraH podemos guardar lo necesario para este proce-
so de dcicodificación. llamado thcodificacién nwl.ia.nJe líderes CU clase, en 90 bits de
almacenamiento, en oposición a la estimación original de 384 bits.
Si aplicamos este procedimiento a r= 110110. encontramos el síndrome

H·r"•m
Como0ll está a la iu¡uierdadel líder de clasex = 010000, la palabra codificada e= x + r=
01000 + 110110 = 100110, de la que podemos recuperar el mensaje original, 100.
En este caso, el código es un código de grupo tal que el peso mínimo de las palabras
codificadas distintas de cero es 3, por lo que esperamos poder encontrar un esquema de
decodificación que conija los errores simples. Realizamos esto debido a que los patrones
de error de peso 1 son todos líderes de clase. No podemos corregir todos los errores do--
bles; sólo un patrón de error de peso 2 es un líder de clase. Todos los patrones de error de
peso 1 o 2 tendrían que ser líderes de clase antes de que nuestro esquema de decodificación
pueda corregir los errores simples y dobles en la transmisión.
810 Capítulo 16 Grupos, teoría de la codificación y método de enumeración de fl>lya

A diferencia de la situación del ejemplo 16.25, donde usamos los síndromes también
para la decodificación, ahOTa las cosas son diferentes. Una vez que tenemos una tabla
completa que enumere todas las clases de C en z~.
el proceso de decodificación mediante
los líderes de clase nos dará la respuesta para todas las palabras recibidas, no sólo para
aquellas que son palabras codificadas o con síndromes que aparecen entre las columnas de
la matriz de verificación de paridad H. Sin embargo, debemos observar que todavía existe
un problema, puesto que la última fila de nuestra tabla no es única A pesar de ello, como
lo afirma nuestro último resultado, este método proporciona un esquema de decodificación
mejor que cualquier otro.

TEOREMA 16. 17 Si decodificamosmedianle Uderesdeclase,r E~ es unapalabra recibidayrsedecodifica


como la palabra codificada e• (que después decodificamos para recuperar el mensaje),
entonces d(c• , r) s d (e, r) para todas las palabras codificadas c.
Demostración: Sea x el líder de clase para la clase que contiene ar. Entonces r = e*+ x, o
r +e* = x, de modo que d(c*. r) = p(r + e*) = p(x). Si e es cualquier palabra codificada.
entonces d(c. r) = p(c + r) y tenemos e + r = e + (e*+ x) =(e+ e*) + x. Como C es un
código de grupo, se sigue que e + e• E C y por lo tanto e + r está en la clase x + C. Entre
los elementos de laclasex + C, el líder de clasex se elige como el elemento que tiene peso
mínimo. por lo que p(c + r) :?: p(x). En consecuencia. d(c• . r) = p(x) ::s p(c + r) ,.., d(c, ,).

16.8
Matrices de Hamming

Hemos visto que la matriz de verificación de paridad H es ótil para corregir los errores
simples de una transmisión cuando (a) H no tiene una columna de ceros y (b) no hay dos
columnas de H que sean iguales. La matriz

H-
[
l 1 O 1 1 O
1 O 1 1 O 1 O
º]
0 11 1 0 0 1
satisface estas dos condiciones y tiene el nómero máximo posible de columnas para el
número de filas (r = 3). Si ali.adimos otra columna. H ya no nos servirá para corregir los
errores simples.
La matriz generadora G asociada con Hes

G -r~ ~ ~ ~ :~ ~]-
00 l O O l l
0001111
En consecuencia, tenemos un código de grupo (7, 4). La función de codificación E : ~ ➔
z; codifica mensajes de cuatro bits como palabras codificadas de siete bits. Observamos
que, como H está detenninada por tres ecuaciones de verificación de paridad, hemos
max.ím.izado el nómero de bits que tenemos en los mensajes (según nuestro esquema de
codificación actual). Además, las columnas de H, leídas de arriba hacía abajo, son los
equivalentes en binario de los enteros del 1 al 7.
16.8 Matrices de Hamming 811

En general, si partimos de r ecuaciones de verificación de paridad. entonces la matriz


de verificación de paridad H puede tener hasta 2'" - 1 columnas y servir para corregir los
I
errores simples. En estas circunstancias,H = [B Ir], dondeB es una matriz r x 2'- 1-ry
I
G;;;; [I. B'1], con m = 2'- 1 - r. La matriz de verificación de paridad H asociada con un
código de grupo (2' - 1, 2'- 1 - r) es una matm d, Hamming y el código se conoce como
un c6digo de Hamming.

Sir= 4, entonces 2'- 1 = 15 y 2'- -1 - r= 11. La 6nica matriz de Hamming posible parar=
4 es

1 1 1 1 1 1 1 O O O O 1 O O
11 1 1 0 0 0 1 1 1 0 0 1 0 0 .
º]
[ 110011011010010
101010110110001

De nuevo, las columnas de H contienen los equivalentes en binario de los enteros del 1 al
15 ( = 2'-1 ).
Esta matrizH es la matriz de verificación de paridad de un código de Hamming (15, 11)
cuya razón de codificación es 11/15.

Respecto a la razón de codificación de estos códigos de Hamm.ing, para cualquier r e?: 2.


la razón de codificaciónm/n dedichocódigoestádadapor mln= (2'- l -r)/(2'- l)= 1-(r/
(2' - !)]. Conforme rcrece, r/(2'- !) tiende a cero y la razón tiende a l.

Concluimos nuesto análisis de la teoría de la codificación con una última observación.


En la sección 16.6 presentamos G (y H) en lo que llamamos laforma sistemática. También
son posibles otras disposiciones de las filas y columnas de estas matrices, las cuales dan
códigos equivaJ.entes. (El lector puede ver más detalles de esto en el texto de L. Domhoff
y F. Hohn [2].) Mencionamos esto puesto que es una práctica común enumerar las colum-
nas en una matriz de Hamming de r filas de modo que las representaciones binarias de los
en~ros de 1 a 2' - l apare.zcan como las columnas de H si se leen de izquierda a derecha.
Para el código de Hamming (7, 4), la matriz H mencionada al inicio de esta sección toma-
ría la fonna (equivalente)

o o o
H, = O 1 1 O O 1 1 .
1 1 1 1]
[1010101
En este caso, la identidad aparece en las columnas primera, segunda y cuana y no en ]as
últimas ttes. En consecuencia, usaríamos estas componentes para verificar la paridad y ver
que si enviamos el mensaje w = w1w 2w3w4 , entonces la palabra ccxlificada correspondiente
E(w) será C1C2W1C3W2W3W4, donde

C¡ = W¡ + W2 + W,
Cz= W¡ + W3+ W ,
C3= W2+ W3+ W4,

de modo quo 11, • (E(w))"= O.


81 2 Capítulo 16 Grupos, teoría de la codificación y método de enumeración de Polya

EJERCICIOS 16.7 1. SeaE: Zl ➔ 'JJf la funcióndecodificacióndeuncódlgo C. ¿C\wltos cálculos sonn«esarios


Y 16.8 para determinar la distancia núnim.a entre las palabras codificadas? ¿Cuántos cálculos son
necesarios si E es un homomorfismo de grupos?
2 . a) Use la tabla 16.9 para decodificar las siguientes palabras recibidas.

000011 100011 111110 100001 001100 011110 001111 111100


b ) ¿Cambia alguno de los resultados de la parte (a) si usamos un conjunto diferente de üderes
de clase?
3. a) Construya una tabla de decodificación (con síndromes) para el código de grupo dado por la
matriz generadora

1 O 1 1
G= [ o 1 O 1 1 ·
º]
b ) Use la tabla de la parte (a) para decodificar las siguientes palabras recibidas.

11110 11101 11011 10100 10011 10101 11111 01100


e) ¿Corrige este código Jos errores simples de transmisión?
4. Sea

l 1 0 1 1 0
H= 1 O 1 1 O 1 O
[
º]
O 1 1 1 O O 1

la matriz de verificación de paridad de un código de Hamming (7, 4).


a) Codifique los siguientes mensajes: 1000 1100 1011 1110 1001 1111.
b) Decodifique las siguientes palabras recibidas: 1100001 1110111 0010001 0011100.
e) Construya una tabla de decodificación que conste de los síndromes y Udcrcs de clase del código.
d ) Use el resultado de la parte (e) para decodificar las palabras recibidas dadas en la parte (b).
5 . a) ¿CuJles son las dimensiones de la matriz generadora del código de Hamming (63, S7)?
¿Cuáles son las dimensiones de la matriz de verificación de paridad H asociada?
b) ¿Cuál es la razón de codificación de este código?
6. Compare las razones de codificación del código de Hamming (/, 4) 'i del código de triple
repetición (3, 1).
7. a ) Seap =0.01 la probabilidad de transmisión incom:cta para un canal simtuico binario. Si se
cnvíad mensaje 1011 por medio del código de Hamming (J, 4), ¿cuál es la probabilidad de
una decodificación correcta?
b ) Responda la parte (a) para un mensaje de 20 bits enviado en cinco bloques de longitud 4.

16.9
Enumeración y equivalencia:
Teorema de Bumside

En esta sección y las dos siguientes desarrollaremos una técnica de conteo conocida como
e l m6todo de enumeración de Polya. Nuestro desarrollo no será demasiado riguroso. Con
frecuencia, sólo enunciaremos los resultados genera1es de la teoría vistos en la solución de
un problema específico. Nuestro primer encuentro con e l tipo de problema al que se aplica
esta técnica de conteo aparece en el siguiente ejemplo.
16.9 Enu~aci6n equivalencia: Tmrema de Bumside 813

Tenemos un conjunto de palillos. todos de la misma longitud y del mismo color. y un


segundo conjunto de discos de plástico. Cada disco tiene dos agujeros, como se muestra
en la figura 16.4, en doode se insertan los paliUos para formar diferentes figuras, como un
cuadrado. (Vtase la Fig. 16.5.) Si cada disco es rojo o blan<:o, ¿cuán10s cuadrados dislin-
lOS podemos formar?
Si el cuadrado se considera fijo. entonces los cuatro discos se localizan en cuatro posi-
ciones diferentes; en cada posición usamos un disco rojo o blanco. Así, existen 2' = 16
configuraciones diferentes, que se muestran en la figura 16.5, donde un cfrculo oscuro
indica un disco rojo. Separamos las configunciones en seis clases, &(I), cf(2), ... , cf(6),
de acuerdo con el nWDero y posición relativa de los discos rojos.
Supongamos ahora que el cuadrado no está fijo sino que se puede mover en el espacio.
A menos que marquemos los Vfflices (discos) de alguna manera. cienas configuraciones
de la figura 16.5 senln indistinguibles desputs de tales movimientos.

Figura 16.4

□ □□□□
c( (1)
e,
c((2)
e, e, c. e,

□□□□ □□
c((3)
c. e, e, c.
C((4 )
e,, e,,

c((S)□□□□ □
Figura 16.5
e,, e,, e,. e,,
ct(6)
e,.
814 Capítulo 16 Grupos, teoria de la axtificao6n y método de enumeracióo de Polya

Para poner estos conceptos en un contexto más matemático, usaremos e l grupo no


abeliano de movimientos rígidos tridímensionales de un cuadrado para definir una rela•
ción de equivalencia sobre Jas configuraciones de la figura 16.5. Puesto que usaremos este
grupo en ésta y las siguientes dos secciones, daremos ahora una descripción detallada de
sus elementos.
En la figura 16.6 tenemos el grupo G = {1t.o, X i, 1½, 1t), ri, r2 , r3, ,.} de los movim.ícntos
rígidos para el cuadrado de la parte (a), donde hemos etiquetado los vénices como 1, 2, 3
y 4. Las panes (b) a (i) de la figura muestran la forma en que se aplica cada elemento de G.
Hemos expresado cada elemento del grupo como una permutación de { 1, 2, 3, 4} y en una
nueva forma l1amadaproducto de ciclos disjuntos. Por ejemplo, en la pane (b) vemos que 1ti
= (1234). El ciclo (1234) indica que si comenzamos con el cuadrado de la parte (a), des-
pués de aplicanti, vemos que 1 se ha movido a la posición que ocupaba en un principio 1,
2 ha pasado a la de 3. 3 a la de 4 y 4 a la de 1. En general, si xy aparece en un ciclo,

1 2 4 1 3 4

4
□ 3
Posición inicial
deta.,adrado
3
□ 2
Rot.lci6n de 90• en el wntido dt
las manecillas del reloj
2
O] 1
Rotaci6n d e 180" en el sentido de
las manecillas del reloj

'lT¡ "" ( ~~~ )"" (1 234) 11'2 "" <jfri> = (1 3)(24)


(a) (b) (e)

2 3 1 2 4 3

1
~ 4
Rotación de 210• en tl s.entido
4
[Q] 3
Rotación de 360" en el sentido de ~
- El-1 2
r ~ t o a la h orizontal
de las manecillas d el reloj las manecillas d el m◊j

'IT1 - c~gj>: (1 432> .., -qm> - (1)(2)(3)(4) ,, = (fü~>- (14)(23)


(d) (e) (ij

ttJ'
3 4

3 , 4
Reflexi6n resl)Kto a la vertical
4 C2T
~eóón respecto a la diagonal
1
TSJ
2
'!
Reflel(!Ón respecio a la diagonal
q~ ~ u.por 10svéfticl!'J 2 y 4 quepaiapo,IOSvtr'ti«,.ly3

'2 .. <iUj> "" 0 2><34> f3 • <j~~: ) = (1 3)(2)(4) ,. -<m;> = (1)124)(3)


(g) (h) (,)

Figura 16.6
16.9 Enumeración equivalencia: Teorema de Burnside 815

entonces x pasa a la posición que ocupaba originalmente y. Además. para un ciclo en


donde x y y aparecen como (x . .. y1 y pasa a la posición que ocupaba x al aplicar el
= =
movimiento descrito por este ciclo. Observe que (1234) (2341) = (3412) (4123). De-
cimos que cada uno de estos ciclos tiene longitud 4, el número de e lementos del ciclo. En
el caso de r1 en la parte (f) de la figura, partimos de 1 y vemos que r, manda 1 a 4, por lo
que tenemos (14 ...) como inicio de nuestto primer ciclo en la descomposición de r 1• Sin
embargo, r 1 manda 4 a 1. por lo que hemos completado una parte, a saber. (14) de toda la
descomposición. Entonces elegimos un vértice que no haya aparecido; por ejemplo, el
vénice 2. Puesto que r, manda 2 a 3 y 3 a 2. obtenemos un segundo ciclo (23). Esto
termina con todos los vértices y entonces (14)(23) = r,. donde los ciclos (14) y (23) no
tienen v~ces en común. En este caso. cada una de las ex.presiones (14)(23) = (23)(14) =
(23)(41) =(32)(41) proporciona una representación de r, como producto de ciclos disjuntos,
cada uno de longitud dos. Por llltimo, el elemento del grupo r, = (13)(2)(4), el ciclo (2)
indica que 2 queda fijo por, o invariante bajo, la permutación r3• Cuando se conoce el
número de vértices incluido~ también podemos escribir la permutación r 3 como ( 13),
donde se entiende que los elementos faltantes quedan fijos. Sin embargo. escribiremos
todos los ciclos en nuestras descomposiciones, ya que esto será útil en nuestto análisis.
Antes de continuar con nuestro análisis principal acerca de los discos y los palillos,
analizaremos otro resultado relativo a los ciclos disjuntos.
En el grupoS6 de todas las permutaciones de { l. 2. 3, 4, 5,.6}, sean =Hf::~.
~ Como
producto de ciclos disjuntos,

.,, = (123)(4)(56) = (56)(4)(123) =(4)(231)(65).


Sia ES6'cona== U ¡ i: l U.entonces
1 2 3 4 5 6)(1 2 3 4 5 6)
ª = (l24)(356) = (2 4 3 1 5 6 1 2 5 4 6 3 '
de modo que podemos pensar cada ciclo como un elemento de S6 •
Por ültimo, si a= (124)(3)(56) y~= (13)(245)(6) son elementos de S., entonces

afl = (124)(3)(56)(13)(245)(6) = (143)(256),


mientras que
Jla = (13)(245)(6)(124)(3)(56) =(132)(465).

Regresamos a tas 16 configuraciones, o coloraciones. de la figura 16.5 y ahora analiza-


mos la forma en que cada elemento del grupo G, de la figura 16.6, actúa sobre estas con-
figuraciones. Por ejemplo, n 1 : ; {i J11) permuta los números de acuerdo con una rotación
de 90º en el sentido de las manecillas del reloj, para el cuadrado de la figura 16.6(a), lo que
produce el resultado de la figura l 6.6(b). ¿Cómo actúa una rotación de este tipo sobre S =
{ C1, C2 , ••• , C16} nuestro conjunto de coloraciones? Usamos Ttt' para distinguir entre la
rotación de 90º en el sentido de las manecillas del reloj para { 1, 2, 3, 4} y la misma
rotación aplicada a S-= { C 1, ½ •. . . , C,6 ) Tenemos que
816 Capitulo 16Gn,,pos, teoría de la codificación y método de enumeración de Polya

Como producto de ciclos disjuntos,


1Tj = (C,)(C,C,C,C,)(C.C,C, C.)(C10C11)(C.,c.,c.,C.,)(C..).
Observemos que, bajo la acción de 7t~, no se modifica ninguna configuración en otra gue
esté en otra clase.
Como segundo ejemplo, consideremos la reflexión r ¡ de la figura l6.6(b). La acción de
este movimiento ógido sobre S está dada por

rj =

= ( c.)( C,)(C, C,)(C,)(C, C,)(C, C.)(C10)( C11)(C,, C.,)( Cu)( C.,)(C,6).


De nuevo, ninguna de las configuraciones es transformada por rt en otra que está fuera de
la clase en la que estaba anteriormente.
Usamos la idea de que el grupo G actúa sobre el conjunto S para definir la siguiente
relación 9? sobre S. Para las coloraciones C;. C1 E S donde 1 ::5 y, j ::5 16, escribimos C,~
½si existe una permutación O' E G tal que o•(C;) = C¡, Es decir, como o• actúa sobre las
16 configuraciones de S, C; es transformada en C¡. Esta' relación 9'l es una relación de
equiva1encia, como verificaremos a continuación.

a) (Propiedad reflexiva) Para toda C; E S, tal que 1 :s ; :s 16, se sigue que C;9/1 C,
puestoqueGcontiene la pennutación identidad. ('ff:J'(C;) = C, para todo 1 s is 16.]
b) (Propiedad simétrica) Si C¡9i C1 para C;.C¡ ES entonces O'*(CJ = C¡. para algún o
E G. Ges un grupo, por lo queo·1 E G y vemos que (o•) - 1 = (0- 1)•. (Verifique esto
para dos opciones de o E G.) Por lo tanto, C;= (<r')*(C,) y g¡ C;, c,
e) (Propiedad transitiva) Sean C;. CJt Ct ES tales que C, 911: Ci --y C; ::iJl C.,. Entonces C1=
O*(CJ y C.= t •(C¡), para aJgunos O', t E G. PorelcierreenG, ert E G y vemos que
(O't)*=O'*-t*, donde O' se aplica antes enat yo• antes eno*t*. (Verifique esto para
dos permutaciones específicas a, t E G.) Entonces Ct= (<Tt)*(C;) y9ies transitiva.
[Es probable que el lector baya notado que C,= t*(C¡) =t"(o*(C,)) y pensado que
deberíamos escribir (<rt)* = t*o*. De nuevo, ha habido un cambio de notación para
la función compuesta que habíamos ya definido en el capítulo 5. Aquí escribimos
o*-t* en vez de (ot)•, y aplicamos primero o•.]

Como~ es una relación de equivalencia sobre S, 91: divide a Sen clases de equivalen-
cia, que son precisamente las clases c{l(l), cf(2), . .. , c,(6) de la figura 16.5. En conse-
cuencia, existen seis configuraciones no equivalentes bajo la acción del grupo. Así, entre
las 16 coloraciones originales, sólo 6 son realmente distintas.
Podemos generalizar lo ocurrido en este ejemplo de la manera siguiente. Si S es un
conjunto de configuraciones, sea G un grupo (de permutaciones) que actúa sobre S. Si la
relación g¡ se define sobreS comox ~ y si 1t*(x) = y, para alguna 7t E G, entonces 911: es
una relación de equivalencia.

Si sólo se dispone de discos rojos y blancos para unir los palillos, podríamos haber
obtenido la respuesta de este ejemplo a panir de los resultados de la figura 16.5. Sin em-
bargo, desarrollamos un poco de matemáticas para responder la pregunta. Si nos referimos
16.9 Enumeración y equivalencia: Teorema de Bumside 817

a S como e] conjunto de 2-coloraciones de ]os vértices de un cuadrado, comenzamos a


preguntarnos acerca del papel de 2 y a buscar el número de configuraciones no equivalen-
tes si los discos vienen en tres o más colores.
Además, podríamos observar que la funciónf(,; w) = r'+ r3w + 2r2w2+ rw3+ w' es la
función generatriz (de dos variables) para el número de configuraciones no equivalentes
de S. En este caso, el coeficiente de r'w'-;. para OS is 4. nos proporciona el ntimero de
2-coloraciones diferentes que tienen i discos rojos y ( 4 - J) blancos. El coeficiente de rw2
es 2debidoa las dos clases deequivalenciad(3) ycl(4). Porúltimo,/(1, 1)=6,el mlmero
de clases de equivalencia. Esta función generatriz}{,; w) es el inventario de patranes de las
configuraciones. La examinaremos con más detalle en las siguientes dos secciones.

Por ahora. presentamos una versión ampliada de nuestros resultados actuales en el si-
guie nte teorema. (Una demostración de este resultado aparece en las páginas 136--137 de
C. L. Liu [ 12).)

16.18 Teorema de Bumside. Sea S un conjunto de configuraciones sobre las que actúa un grupo
finito de permutaciones. El número de clases de equivalencia e n que S se divide por la
acción de G está dado entonces por

donde 'V(7t*) es el número de configuraciones en S fijas por n•.

Para poder aceptar la validez de este teorema, primero analizaremos dos ejemplos en
los que conocemos las respuestas.

En el ejemplo 16.28 vimos que V(1tf) = 2, puesto que solamente C1 y C16 están fijos. o
invariantu, bajott~. Sin embargo, parar3 E G, V(r:) = 8 debido a que Ca, C1, C,, C1~ C11,
C13, C1, y C16 permanecen fijos bajo esta acción de) grupo. De manera similar, ~{í-t) = 4.
o/(11:) ; 2, o/(it:) = 16, o/(rrl = '1'('1) = 4 y \jl(rt) = 8. Como I G 1 = 8, el teorema de
Bumside implica que el número de clases de equivalencia, o las configuraciones no equiva-
lentes. es ( l/8X16 + 2 + 4 + 2 + 4 + 4 + 8 + 8) = (J/8X48) = 6, la respuesta original.

¿De cuántas formas puede n sentarse seis personas en tomo de una mesa redonda si dos
disposiciones se consideran equivalentes cuando una se puede obtener de la otra mediante
una rotación en el sentido de las manecillas del reloj.con unánguloi • 60°. para O :si :s 5?
En este caso, las seis personas distintas deben sentarse en seis sillas en tomo de la mesa,
como se muestra en la figura 16.7. Nuestro grupo de permutaciones G consta de las rota-
ciones lt; en el sentido de las manecillas del reloj con un ángulo de i • 60º, donde O :s i :s
5. En este caso, las reflexiones no son importantes. La situación es bidimensional, ya que
sólo podemos rotar el ciclo (que representa la tabla) en el plano; el círculo nunca sa1e del
plano. El mlmerototal deconfiguraciones posibleses6!. Vemos quc \jl(it;) = 6! y quc 'I'(<)
=O, para 1 s i :S 5. (Es imposible mover a varias personas y que al mismo tiempo éstas
permanezcan fijas en una posición dada.)
818 capitulo 16 Grupos, teoría de la codificaoon y método de enumeración de Polya

602
5
4
1

3
Figura 16.7

En consecuencia. el número total de disposiciones no equivalentes para sentarse es

(....!....) ¿ .¡,(a') = (!)c6!+0+ 0 +0+0+0) = 5!,


IGI oEG 6

como vimos en el ejemplo 1.16 del capítulo 1.

Ahora analizaremos una situación en la que es más clara la potencia de este teorema

¿De cuántas formas podemos 3-colorear los vértices de un cuadrado, si el cuadrado debe
moverse en tres dimensiones?
Ahora tenemos los palillos del ejemplo 16.28, junto coa discos rojos, blancos y azules.
Si observamos el grupo de la figura 16.6, tenemos lo siguiente:
'V(7tl) =3". puesto que la identidad fija 81 configuraciones en el conjunto S de posibles
configuraciones.
'V(1tr) =: 'V(x:)- 3, para cada1tt, rtt deja invariantes solamente las configmaciones con
todos los vértices del mismo color.
'V(,ct) - 9, ya que X: puede fijar solamente aquellas configuraciones en las que los
vértices opuest0s (en forma diagonal) tienen el mismo color. Consideremos un cuadra-
do como el que se muestra en la figura 16.8. Hay tres opciones para colocar un disco
con color en el vértice 1 y después una opción para relacionarlo con el vértice 3. De la
misma forma. existen tres opciones de colores en el vértice 2 y después uno para el
vértice 4. En consecuencia. hay nueve configuraciones invariantes bajo X:.
'V(rf): w(r!) : 9. En el caso de rf, para el cuadrado que se muestta en la figura 16.8
tenemos tres opciones para colorear cada uno de los vértices 1 y 2 y después debemos
hacer corresponder el color del vértice 4 con el color de vértice 1 y el color del vértice
3 con el del vértice 2.
= =
Por último, ,¡,(,t) IJl(,1) 27. Para ,1, tenemos nueve opciones para colorear los dos
vértices en 2 y 4 y tres opciones para el vértice 1. Entonces existe solamente una opción
para el vértice 3. puesto que debe coincidir con el color del vértice l.

4 3 Figura 16.8
16.9 Enumeración equivalencia: Teorema de 8umside 819

Por el teorema de Bumside, el nl1rnero de configmacioncs no equivalentes es (1/8X3' +


3 + 31 + 3 + 3 2 + 32 + 3l+ 3') :;: 21.

1. Consideremos la configuración que aparece en la figura 16.$.


a) Dctennine n:,ltf,,-l' y ,:.
b) Verifique que(.;')• = (X:~ y (r¡')' = (,:J-'.
e) Verifique que (K,. r1) • = X: ,t y (X,. rJ • •.,: ,:.
2 . E.lprcse cada uno de los siguientes elementos de S1 como producto de ciclos disjuntos.

~= (! ! ! 2

&= (! ; ~ 4 ~)
3 . a) Determine d orden de cada uno de los demeDIOS del cjcn:icio 2.
b) Enuncie un resultado general acerca del orden de un demento en s.
en támioos de las
longitudes de los ciclos en su descomposición como producto de ciclos disjuntos.
4. a) Detcnnine el mlmero de formas distintas coque podemos colorear los v&tices de un tri4n.
guJo equilátero usando los colora rojo y blanoo, si podemos mover el uitngulo libremente
en tres dimensiones.
b ) Responda la parte (b) si tambitn $C dispone del color azul.
5. Responda las preguntas del ejercicio 4 para un pentágono regular.
6. a) ¿Cu.iotas formas distintas hay de pintar las aris,as de un cu.adrado con tres colores dife,.
rentes'?
b) Responda la pane (a) para las aristas de un pentágono ne¡ular.
7. Hacemos una pulsera para niiia con cuatro cueow colocadas en un alambre circular. Los colo-
res de las cuenw son rojo, blanco, azul y verde y existen al menos cuatro cucnw de cada
eolor.
a) ¿CUAntas pulseras distinW se pueden hacer de esta forma si las pulseras se pueden rour
pero no reflejar?
b) Respooda la pane (a) si las pulseras se pued<n row- y r<flejar.
8 . Un bastón se pinta coo tres bandas c:ilíndric:as de color (no necesariamente distintas) y cada
banda tiene la misma loogirud que las demás..
a) ¿CUánw decoraciones distintas pueden hacerse si hay tres colores disponibles de pintura?
b) Responda la parte (a) para bastones con aJatro bandas cilíndricas.
e) Respooda la pane (a) para baslOnes eon n bandas cilíndricas.
d) Responda las partes (a) y (b) d las bandas cilíndricas adyacentes deben tener colores
distintos.
9. ¿De cuántas formas podemos 2-colorcar los vétices de las configuraciones que se muestran en
la figun. 16.9 si &tos son libres de moverse (a) en dos dimensiones'? (b) en tres dimensiones?

F'tgura 16.9
820 capítulo 16 Grupos, teoría dela codificación y método deenumeraóón de l'olya

1 O. Una pirámide tiene una base cuadrada y cuatro caras que son triángulos equiláteros. Si pode-
mos mover la base libremente (en tres dimensiones), ¿cuántas formas DO equivalentes hay de
pintar las cuatro caras si tenemos pintura de cuatro colores distintos? ¿Cuántas formas hay si el
color de la base debe ser diferente de los colores de las caras triangulares?
11. a) ¿De cuántas formas podemos pintar las casillas de un tablero de ajedrez. 3 x 3 usando
pintura azul y roja? (El fondo del tablero es negro.)
b) ¿De cuántas formas podemos construir un tahlerodeajedrez.3 x 3 wúendo {conpegamen.
to) cuadrados de plástico transparente con tintes azul o rojo7 (Tenemos nueve cuadrados
disponibles de cada color.)
12. Responda el ejercicio 11 para un tablerodeajedrez4 x 4. [Reemplace cada "'nueve.. en la parte
(b) con 'ºdicci~is".J
13. ¿De cuántas formas podemos pintar los siete caballos (id6nticos) de un carrusel. usando pintu•
ra_negra. marrón y blanca'?
14. a) Sea S un conjunto de configuraciooc:s y G un grupo de permutaciones que acroa sobre S. Si
x E S. demuestre que {1t E G ln:*(z ) =x} es un subgrupo deG Olamado el~rabiliuulorde
x).
b ) Determine los subgrupos estabilizadores de la parte (a) para cada una de las configuracio-
nes C, y C 1, de la figura 16.5.

16.10
El índice de ciclo

Al aplicar el teorema de Burnside, nos enfrentamos al cá1cu1o de V(x") para cada 1t E G,


donde Ges un grupo de permutaciones que actúa sobre un conjunto S de configuraciones.
Si el nWDero de colores disponibles aumenta y las configuraciones se vuelven más com-
plejas. tales cá1cu1os se complican un poco. Además, parece que si podemos determinar el
número de 2-coloraciones para un conjunto S de configuraciones. podríamos usar algo del
trabajo realizado en este caso para determinar el número de 3-coloraciones, 4-coloraciones,
etcétera. Encontraremos alguna ayuda si regresamos a la solución del ejemplo 16.28. Esta
vez prestaremos más atención a la representación de cada permutación 1t E G como pro-
ducto de ciclos disjuntos. Resumiremos nuestros resultados en la tabla 16.10.
Para ,.-., el neutro de G, escribimos Jto= (IX2X3X4), un producto de cuatro ciclos
disjuntos. Representaremos esta estrucrura de ciclo en forma algebraica como xt, dondex1
indica un ciclo de Iongirud 1. El término X: es la repnsentación ~ la estructura (U ciclo
de Jto.. En este caso. interpretamos ..disjunto" como "independiente", en el sentido de que
el color utilizado para pintar los vértices de cualquier ciclo no influye en la selección de
los colores para los vértices de otro cic1o. Míentras los vértices de un ciclo dado tengan el
mismo color, encontraremos configuraciones que son invariantes bajo X:. (Definitiva-
mente, parece que estamos exagerando el uso de las matemáticas, puesto que Jet fija todas
las 2-coloraciones del cuadrado.)Además, como podemos pintar los vértices de cada ciclo
con rojo o blanco, tenemos 2' configuraciones y vemos que (r + b)'= r4+ 4r'b + 6121:r+
4,t,3+ b' genera estas 16 configuraciones. Por ejemplo, del término 6r21r observamos que
existen seis configuraciones con dos vértices rojos y dos blancos, como se ve en las clases
ce(3) y cf(4) de la figura 16.5.
Analizamos ahora1t 1 y vemos que1t1 =(1234), un ciclo de longitud 4. Esta estructura de
ciclo queda representada por x4 y aquí solo hay dos configuraciones invariantes. El hecho
16.1O El índice de cido 821

-.........
11a 16.10

Rcpram-
Conligandon« lad6nd<•
mSqutson enosln>ctura lnvmtario de eooftpracloDcs que soa
- . . . . G) lmarialltcs bajo

r,= (1)(2)(3)(4 2'' : Todas las confi-


r
.
d<dclo

x, (r + w)'
bmoriaot<sbojo . .

= r" + 4r3w + 6rw2 + 4rw3+ w◄


guraciones en S

"'=(1234) 2: C11 C 16 x, r' + w"' =,.. + w'

¡,,,= (13)(24) 22: C1. C 10, Cu, C16 x; (r' + w')' =r" +Zr'w' +w'

~ = (1432) 2:C1, C 16 x, r"+w" =r" + w'

, = (14)(23) 22: c i. c 7, e,, c ,6 x,' (r'+ w')' = r" + Zr'w' +w•

¡r, =(12)(34) 2 2: c ., c6, e,. c .6 x/ (r' +w')' =,.. + Zr'w' + w'


r,= (13)(2)(4) 2•: e,, e,, e,, c10, X2X1
2
(r' + w')(r + w)' =r' + Ír'w + 2r'w + 2rw' + w•
2

Cn, Cu, Cu, C16

,. = (1)(24)(3) 23: C 1, ~. C,, C ,o, X2X1 ' (r' + w')(r + w)' =r' + 2r w + 2r'w' + 2rw' + w•
3

Cu, C,2, C,., C16

PG(X1,X2,X3, x,.) = Inventario}


completo = Sr" + 8r3w + l6r2w2 + 8rw3 + 8w"
f
(x~ + 2x. + 3x1 + 2x2x~)

de que la estructura de ciclo para1t1 sólo tenga un ciclo indica que para que una configura-
ción sea invariante bajo xr, cada vértice en este ciclo debe pintarse del mismo color. Si
podemos elegir e ntre dos colores, sólo existen dos configuraciones posibles: C 1 y C16• En
este caso, el término (r" + 11') genera estas configuraciones.
Proseguimos con r 1 y tenemos que r1 = (14X23), un producto de dos ciclos disjuntos de
longitud 2; el t&mino Xi representa esta estructura de ciclo. Para que la configurac16n
quede fija con rf, los vértices 2 y 3 deben tener el mismo color; es decir, tenemos dos
opciones para colorear los vértices e n (23). Tambif n tenemos dos opciones para colorear
los vértices de (14). En consecuencia, obtenemos 22 configuraciooes invariantes: C1(r"),
C,(r'b'), Cl.r'b') y C,.(b').{(r '+ /,')' = r' + 2r'b'+ b'.J
Por último, en el caso de r3 =(13)(2X4). tenemos que x2x{ indica su descomposición en
un ciclo de longitud 2 y dos de longitud 1. Los vértices en 1 y 3 deben pintarse del mismo
color si la configuración debe quedar fija con ,-r. Con tres ciclos y dos opciones de color
para cada ciclo, tenemos 2'configuraciones invariantes, que son C1(r"), C 2(r 3b), C,.(r 3b),
C,o<.r ' b'), C 11 (r'b'). C13(rb'), C,,l.rb') y C,.(b'). La función generatriz para estas configura-
ciones es (r 2 + b1)(r+ b'f, ya que cuando consideramos el ciclo (13) tenemos dos opciones:
ambos vénices en rojo (r 2) o ambos vértices en blanco (b1). Esto nos da ( r 2 + b2). Para
cada vénice único en los dos ciclos de longitud 1, r + b proporciona las opciones para cada
822 Capítulo 16Grupos. teoóa de la codificación ymétodo de enumeración de Polya

ciclo, (r + b"f las opciones para ambos. Por la independencia de la elección de cokla
cuando pasamos de un ciclo a otro. (r2+ lrXr+ bf genera las 2lconfiguraciones invariaalel
bajo ,1.
Argumentos similares proporcionan la información que aparece en la tabla 16.I0¡a
las permutaciones 'Jt.z, n,, r 2 y T4.
En este momento vemos que lo que determina el número de configuraciones i n ~
bajo n:•, paran E G, depende de la estructura de ciclo de 7t. Dentro de cada ciclo, debemaa
usar el mismo color, pero podemos seleccionar ese color de las dos o más opciones~
nibles. Para r 1, teníamos dos ciclos (de longitud 2) y 2 2 configuraciones. Si disponemos ele
tres colores, el m1mero de configuraciones invariantes es 32• Para m colores, el número•
m2. Si sumamos estos términos para todas las estructuras de ciclo que pueden surgir. ote-
nemos L.ov'1'(1t*).
Ahora queremos enfatizar las estructuras de ciclo, así que definimos el índice tk ciclo.
Pv, para el grupo G (de permutaciones), corno

1
Pa(xi.x2 ,x 3 ,x4 ) =- L (representación de estructura de ciclo de 1t).
IG1""º
En este ejemplo,

Si reemplazamos cada x1• x2• x3 , x. por 2, vemos que el número de 2-coloraciooes •


equivalentes es igual a

P0 (2, 2, 2, 2) = (1 /8)(2' + 2(2) + 3(2') + 2(2)(22)) = 6.

Resumiremos nuestros hallazgos en el siguiente resultado.

TEOREMA 16 .19 Sea S un conjunto de configuraciones en el que actúa un grupo de permutaciones G. (Ges
un subgrupo de s., el grupo de todas las permutaciones de { 1, 2, 3, ... , n} y el índicem
ciclo Pa= (x1, x 2, X), • . . , x.) de Ges

(1 / IGIJ ¿ ..0 (representación de estructura de ciclo de 1t).)

El número de m-coloraciones no equivale ntes de S es entonces PG= (m, m, m, . .. , m).

Terminamos la sección con un ejemplo que usa este teorema.

¿De cuántas fonnas distintas podemos 4-colorear los vértices de un hexágono regular que
puede moverse libremente en el espacio?
Para un hexágono regular, existen 12 movimientos rígidos: (a) las seis rotaciones de 00,
60º, 120•, 180º, 240º, 300", en el sentido de las manecillas del reloj; (b) las tres reflexio-
nes respecto a las diagonales que pasan por )os vértices opuestos; y (e) las tres reflexiones
respecto a las rectas que pasan por los puntos medios de los lados opuestos.
·o,
16.1 O El índice de cido 823

(1) (1)(2)(3)(4)(5)(6) ,: 1
(7) (1)(26)(35)(4) xtKJ
(2) (123456) ., (8) (13)(46)(2)(5) xtx~
(3) (135)(246) •l (9) (15)(24)(3)(6) X~ X~

(4) (14)(25)(36) •l (10) (12)(36)(45) •l


(5) (153)(264) •l 5 3 (11) (14)(23)(56) •l
(6) (165432) ., 4
(12) (16)(25)(34) •l
Agura 16.10

En la figura 16.10 hemos enumerado cada elemenlo del grupo como un producto de
ciclos disjuntos, junto con su representación de la esttuctura de ciclo. En este caso,
Pa(x.,x,,x,,x,,x,,x,) = (1/ U){xl + 2x6 + 2x\ + 4,n + 3x¡x\),
y entonces existen
Pa(4,4, 4,4,4,4) - (l/UX4'+ 2(4) + 2(4') + 4(4') + 3(42)(4'))- 430
4-<x>Joraciooes no equivalentes de un hexágono regular. (NoULAunque nix, nix, aparecen
en una representación de estructun de ciclo, podemos enumerar estas variables entre los
argumentos de Po-)

IEROOOS 16.1O 1. ¿Owltas S-coloraciooes c:üstinw e.listen para los v&tices de un cuadrado que puede moverse
libremente en (a) dos dimcosiooes? (b) tres dimensiones?
2. Responda el ejercicio 1 para UD peoúaooo rq¡ular.
3 . Encuentre el m1mero de 4--co1oracioDC$ oo equivalentes de las configuraciones que aparecen
eo la figura 16.11 cuando tstas pu«leo movene libranente en (a) dos dnnensiooes; (b) t=
dimensiones.
4. a) ¿eutnw 3-<oloncione$ clisúow eaistea para los vtrtices de UD bexí¡ooo rqular que
puede moverse libremente en el espacio?
b) ~ un argumento eombilworio para mosavque para todom e Z♦• (m'+ 2nt + 2nr+ 4m'+
3m') es c:üvisible entre 12.
5. a) ¿Cu.úllas 5-coknciooes distintas existen para los vtrtices de un bcú¡ono re¡uhr que
puede moverse libremente en dos dimensiones?
b) Responda la pano (a) si d hex.lgooo puede movene libremente en t= dimensiones.
e) Encuentre dos 5-coloraciooes que sean equivalentes para el caso (b) pero distintas para el
caso (at
6. ¿Orantas 3-coloraciooes existen para las ari.Jtas de las configuraciones que se mucsuan en la
figura 16.11 si &w pueden moverse libremente en (l) dos dimensiones? (b) tres dimensiones?
824 Capítulo 16Grupos. teoría de la codificaóón y método de enumeración de Polya

7. a) ¿Cuántas 3-coloraciones distintas existen para las aristas de un cuadrado que puede mover.
se libremente en tres dimensiones?
b ) ¿Cuántas 3-coloraciones distintas existen para los vfflices y las aristas de dicho cuadrado?
e) Si un cuadrado se puede mover libremente en tres dimensiones, sean k, m y n el número de
3-<:oloraciones distintas de sus vértices (llnicamente), de sus aristas (únicamente) y de S1JS
vértices y aristas, respectivamente. ¿n. = km?~ una explicación geométrica)

16.1 1
El inventario de patrones:
Método de enumeración de Polya

En esta última sección regresaremos al ejemplo 16.28 y nos concentraremos en el inventa•


rio de patrones y la forma de obtenerlo a partir del índice de ciclo.
Para 1to E G, toda configuración en Ses invariante. Esta (representación en) csttuctura
de ciclo para no está dada por xf. donde para cada ciclo de longitud 1 tenemos la opción de
colorear el vértice en ese ciclo rojo (r) o blanco (b). Si usamos + para representar la o
exclusiva, escribimos (r + b) para denotar las dos opciones para ese vértice (ciclo de Ion•
gitud 1). Con cuatro de estos ciclos, (r+ b)' genera los patrones de las 16 configuraciones.
En el caso de 7t1 = (1234), X. denota la estructura de ciclo y en este caso los cuatro
vénices deben tener el mismo color para la configuración si deben permanecer fijos bajo
1ti. En consecuencia, los cuatro vértices son rojos o los cuatro son blancos, y expresamos
esto en forma algebraica como r' + b'.
En este momento, observamos que para cada una de las permutaciones consideradas, el
número de factores en el inventario de parrones para esa permutación es igual al número
de factores en e l inventario de patrones en su (representación en) estructura de ciclo. ¿Es
esto solamente una coincidencia?
Si proseguimos con r1= (14)(23), encontraremos la esttuctura de ciclo xi- Para el ciclo
(14) debemos pintar ambos vértices 1 y 4 rojos o blancos. Representamos estas opciones
r
como + b2• Puesto que hay tales ciclos de longitud 2, la función generatriz para el inven-
tario de patrones de configuraciones de S fijos mediante r~ es (r2 + b2)2. De nuevo, el
nt1mero de factores en la éStructura de ciclo es igual al mímero de factores en el término
correspondiente de la función generatriz.
= =
Por último, para r1 (13)(2)(4), la estructura de ciclo CSXzXÍ xfx2. Para cada uno de
los ciclos (2) y (4), r + b representa las opciones para cada par de vénices, mientras que
(r + b"r cuenta las cuatro coloraciones del par. El ciclo (13) indica que los vértices 1 y 3
deben tener el mismo color; r1 + b2 cuenta las dos posibilidades. Por lo tanto, el sumando
de la función generatriz para estas configuraciones es (r + b)2(r2 + ir) y tenemos tres
factores en la estructura de ciclo y el sumando correspondiente de la función generatriz.
Pero aquí surgen más detalles.
Si observamos los términos en las estructuras de ciclo, vemos que, para 1 :s i s n, el
factor X; en la estructura de ciclo corresponde con el término r' + 11 en el sumando asociado
de la función generatriz.
Continuando con las estructuras de ciclo para lti, 1th r1 y r, veremos que podemos
obtener el inventario de patrones reemplazando cadax; en P<;{x1, x2, x 3, x.) con ri + 11, para
l :s i s 4. En consecuencia,

P<;{r + b, r 1 + b2, r 3 + b3, r' +b') =r' + r3b + 2r2b2 + rb1 +b'.
16.11 El inventario de patrones: Método de enumeraci6n de l'o1ya 825

(Esto produce una octava parte del inventario completo enumerado en la tabla 16.10.)
Si ruvitramos tres colores (rojo, blanco y azul), la sustirución para X; serla r' + /1 + d,
donde 1 :s i :s 4.
Generalizamos csw observaciones en el siguiente teorema.

:JREMA 16.20 Mitodo tk e.numuaci6n de Polya. Sea S un conjunto de configuraciones sobre las que
s.
acn!a un grupo de permutlciones G, donde G es un subgrupo de y G tiene Indice de
ciclo PG = (x1, Xi. •.. , x.,). En1onces la función generatriz del inventario de patrones de las
m--coloraciooes no eqWvalentes de S está dada por

Pe - Ld,
(LC;,
•l
- ... ,¿c}',
l•l
-) l• l

donde c1• c1, • ..• c. denotan los m colores disponibles.

Aplicaremos este teorema en los sjgujentes ejemplos.

Uoa pulsera para nida se forma con tres cuenw (roja, blanca y azul) en un alambre al que
se te da la forma circular. Las pulseras son equivalentes si podemos obtener una de ocra.
mediante una rotación (plana). Encuentre el inventario de patrones de estas pulseras.
En eslc caso, G es el grupo de rotaciones de un triángulo equilá1cro, de modo que G =
((IX2X3), (123), (132)}, donde 1, 2, 3 denotan los vátices del triángulo. Entonces P0 =
(,t., x,, .r,) =
(1/3X.r/ + 2.r,) y el inventario de patrones está dado por (1/3)((r + b + a1+ 2(r'+
b'+a')J = (l/3)[3r'+ 3r'b + 3r'a + 3rb'+6rba + 3ro'+ 3b'+ 3b'a+ 3ba'+3a'J = r'+ r'b +
r'a + rb' + 2rba + ro'+ b' + b'a + ba' + a'. Interpretamos este resultado como sigue:

1) Para cada sumando, distinto de 2rba, el coeficiente es l, puesto que sólo existe una
única poisera (distinta) de este tipo. Es decir, existe una poisera con tres cuentas
rojas (para r'), una con dos cuentas rojas y una cuenta blanca (para r'b), etcéten,
para los otros siete sumandos con coeficiente 1.
2) El sumando 2rba tiene coeficiente 2 debido a que existen dos pulseras no equiva-
lentes con una cuenta roja, una blanca y una azul, como se muestra en la figura
16. 12.

Si tambitn podemos reflejar las polseras, entoncesG es {(IX2X3), (123), (132), (IX23),
(2X13), (3X12)} y el inventario de patrones de este caso es el mismo que el anterior, con
una excepción. En este caso tenemos rba, en vez de 2rba, puesto que los patrones (de
rotaciOO) no equivalentes de la figura 16.12 se vuelven equivalentes si se permiten las
reflexiones.
lo 16 Grupos. teoria de la ccdifiaKión y método deenumeraci6n de Fblya

Consideremos las 3.coloraciones de las configuraciones del ejemplo 16.28. Si los


colores son rojo, blanco y azul, ¿cuántas configuraciones no equivalentes tienen
mente dos vértices rojos?
Dado quc Po(xi.x1, X1t.X.) = (118Xxt + 4+ 3xl + 2x'1xl), la respuesta es lasumade
coeficientes de r'b', r'á' y r'ba en ( l/8)[(r + b + a'¡'+ 2(r'+ b'+a') + 3(r2 + b'+ á'f + 2(
b' + á')(r + b + a)'].
En (r+ b + a)', tenemos el túmino 6r2b2+ 6r2a2+ 12r2ba. Para3(r2 +b2+af,
interesa el término 6r1lr+ 6r1a2, mientras que 4r1b2+ 4r1a2+ 4r1ab surge en 2(r2+
a')(r + b + a)'.
Entonces, (l/8)[6r2b'+ 6r'á'+ 12r2ba + 6r' b'+ 6r'á'+ 4r'b'+ 4r'á'+ 4r'ab] =
2r2Ct' + 2rlab, el inventario de las seis configuraciones no equivalentes que tienen e
mente dos vértices rojos.

Nuestro siguiente ejemplo trata del inventario de patrones para las 2-coloracioncs
los vértices de un cubo. (Los colores son rojo y blanco.)

Para el cubo de ta figura 16. 13, veremos que su grupoG de movimientos rígidos comta
lo siguiente.
1) La transformación identidad, con estructura de ciclo ..rf.
2) Rotaciones de 90°, 180º y 2700 respecto a un eje que pasa por los centros de
caras opuestas: De la figura 16.13(a) tenemos
Rotación de 90°: (1234X5678) Estructura de ciclo: xl
Rotación de 180': (13)(24)(57)(68) Estructura de ciclo: xt
Rotación de 270': (1432X5876) Estructura de ciclo: xi
Puesto que hay otros dos pares de caras opuestas. estas nueve rotaciones se•
nen en cuenta en el término 3;4 + 6x1 en el índice de ciclo.
3) Las rotaciones de 180' respecto a un eje que pasa por los puntos medios dedos.,;,,.
opuestas. Como en la figura 16.13(b), tenemos la permutación (17)(28X34X56),CIIJI

'
'''
7 !-- - -- 6

90"', 180", 27(1'


120", 2411'
(a) (b) (e)

figur~ 16.13
16.11 El inventario de patrones: Método de enumeración de Po a 827

estructura de ciclo está dada por xf. Como existen seis pares de aristas opuestas.
estas rotaciones contribuyen con el término 6~ al índice de ciclo.
4) Rotaciones de 1200 y 240º respecto de un eje que pasa por dos vértices diagonalmente
opuestos. De la pane (c) de la figura, tenemos
Rotación de 120': (168)(274)(3)(5) Estructura de ciclo: xf xj
Rotación de 240': (186)(247)(3)(5) Estructura de ciclo: xfxJ
En este caso, hay cuatro pares de taJes vértices, lo que da lugar al tértruno 8.x?xf en el
índice de ciclo.
Por lo tanto, Pc(Xi, x2• • • • , x8) :;;;: (l/2.4Xxl + 9rj + 6x] + 8xfxI) y el inventario de
patrones para estas configuraciones es

(l/24)[(r+ b)' + 9(r' + b')' + 6(r' + b''j' + 8(r + b'j'(r' + b')']


= r8 + r'b + 3r6b2 + 3,:1b3 + 71'4b4 + 3r3b5 + 3,:IP + rb' + b1
Si reemplaz.amos r y b por 1, encontramos 23 configuraciones no equivalentes.

Puesto que el método de enumeración de Polya fue desarrol)ado en un principio para


contar los isómeros de los compuestos orgánicos, cerraremos la sección con una aplica-
ción relativa a ciena clase de compuestos orgánicos. Esto se basa en el ejemplo de C. L.
Liu. (Véanse las páginas 152-154 de la referencia [12).)

En este caso nos interesa estudiar las moMculas orgánicas de la forma que se muestra en la
figura 16.14, donde C es un átomo de carbono y X denota cualquiera de las siguientes
componentes: Br (bromo), H (hidrógeno), CH, (metilo) o C,H, (etilo). Por ejemplo, si
reemplazamos cada X por H. obtenemos el compuesto C~ (metano). La figura 16. 14 no
nos debe confundir. La estructura de estos compuestos orgánicos es tridimensional. En
consecuencia, utilizamos el tetraedro regular para modelar dicha estructura. Podemos~
locar el átomo de carbono en el centro del tetraedro y col ocar nuestra selección de X en los
vértices l. 2, 3 y 4, como se muestra en la figura 16.15.

/l\
X

1
x - - c - -x

1
·~· .4,
(a)
~ 1
1
3

12rr, 240"
(b)
3
1

2x
18~

X
Figura 16.14 Figura 16. 15
828 capirulo 16Grupos, teoria de la codifi<:aóón y método de enumeración de fl:>lta

El grupo G que actúa sobre estas configuraciones está dado por:

1) La ttansfonnación identidad (1X2X3X4) con estructura de ciclo xf.


2) Rotaciones de 120º o 240º respecto de un eje que pasa por un vértic.e y el centro de
la cara opuesta. Como lo muestra la figura 16. lS(a), tenemos
Rotación de 120º: (1)(243) Estructura de ciclo x,x,
Rotación de 240°: (IX234) Estructura de ciclox,x,
Por simetría, existen otros tres pares de vértices y caras opuestas, por lo que estos
movimientos rígidos contribuyen con el término 8xi-t3 en Pc;(X1, x 2, x 3, x.).
3) Rotaciones de 180º respecto a un eje que pasa por los puntos medios de dos aristas
opuestas: el caso que se muestra en la parte (b) de esta figura está dado por la
permutación (14X23) cuya estructura de ciclo es xf. Como existen tres pares de
aristas opuestas, obtenemos el término 3xf en P<;(Xi, x 2, x 3, ~).

= =
Por lo tanto, Pa(x,, x,, x,, x,) ( 1/12)[.t/ + 8x,x,+ 3xJ] y P.(4, 4, 4, 4) (1/12) • [4'+
8(42) + 3(42)] = 36, de modo que existen 36 compuestos orgánicos diferentes que pueden
formane de esta manera.
Por último, si queremos saber cuántos de estos compuestos tienen exactamente dos
átomos de bromo, sean w, x, y y z los "colores" Br, H, CH3 y ½:Hs, respectivamente;
encontramos la suma de los coeficientes w2:c2, w2y2, w2:z:1, w2~. w2xz y w2yz en el inventario
de patrones
(l/12)[(w +x +y+ z)'+S(w +x + y +z)(w'+x'+ y'+ z') + 3(w'+x'+ y 2 + z')'J.
Para (w + x +y+ z)', el término de importancia es 6w2x2 + 6wlyl+ 6w2.z2+ l2w2xy +
l 2w2xz + 12w2yz. El sumando intermedio del inventario de patrones no da lugar a ninguna
de las configuraciones deseadas, mientras que en 3(w2 + x2 +Y+ f)? encontramos 6w2x2 +
6w'y'+ 6w'z'.
En consecuencia, el inventario de patrones de los compuestos que contienen exacta-
mente dos átomos de bromo es

(l/!2)[!2w' x' + 12w'y2 + 12w2 z' + 12w' xy + 12w'xz + 12w'yz]


y existen seis de tales compuestos.

EJEROCIOS 16.11 1. a) Encuentre el inventario de patrones para l:u 2-coloraciones de las arü,tas de un cuadrado
que se puede mover libremente en (i) dos Wmensiones; (ii) tres dimensiones. (Considere
que los colcrcs son rojo y blanco.)
b) Responda la parte (a) para 3-coloraciones, donde los colores son rojo, blanco y azul.
2. Si un pentágono regular puede moverse libremente en el espacio y podemos utilizar los colores
rojo, blanco y azul para sus vértices. ¿cuinw configuraciones no equivalentes tienen exacta-
mente tres v&ticcs rojos? ¿Cuántas de ellas tienen dos v~ces rojos. uno blanco y dos azules?
3 . Suponga que en el ejemplo 16.35 utilizamos una 2-coloración para las caras del cubo, el que
podemos mover libremente en el espacio.
a) ¿Cuántas 2~loraciones distintas hay en esta situación?
b) Si los colores disponibles son rojo y blanco. determine el inventario de patrones.
e) ¿Cuintas coloraciones no equivalentes tienen 3 caras blancas y 3 rojas?
16.12 Resumen y repaso históri<:o 829

4. Para losoompuestosorgánioosdcl ejemplo 16.36, ¿cuánlos tienen al menos un átomo debromo?


¿Cuántos tienen exactamente tres átomos de hidrógeno?
5. Encuentre los inventarios de patrones para las 2-coloraciones de los vértices de las configura-
ciones en la figura 16.11. cuando podemos mover éstos libremente en el espacio. (Considere
que los colores son verde y oro.)
6. a) ¿De cuántas formas podemos pintar los siete caballos (idáJ.ticos) de un canuse1 con pintura
negra. marrón y blanca de modo que baya tres caballos negros. dos marroB:S y dos blancos?
b) ¿De cuántas formas babrfa igual m1mero de caballos negros y marrones?
e) Dé un argumento combinatorio para verificar que para tooo n E z·, Ir+ 6n es divisible
entre 7.
7. a) ¿De cuántas formas podemos pinw los ocho cuadros de un tablero de ajedrez de 2 x 4.
utilizando los colores rojo y blanco? (El fondo del tablero es de color negro.)
b) Encuentre el inventario de patrones para las coloraciones de la pane (a).
e) ¿C\wltas de las coloraciones de la parte (a) tienen cuatro cuadros rojos y cuatro blancos?
¿.Cdntas tienen seis cuadros rojos y dos blancos?
8. a) ¿Cuintas 2-coloraciones existen para l a s ~ regiones del molino que se muesua en la
figura 16.16 si se usan los colores negro y oro, y si el fondo de cada región es gris?
b) Responda la parte (a) para las 3-coloraciones posibles, utiliz.ando pintura negra, oro y azul
para colorear las regiones.
e) Para las coloraciones de la pane (a), ¿cuántas tienen cuairo regiones negras, dos oro y dos
azules?

Figura 16.16

9. Sean m. n E z• con n ~ 3. ¿Cuántos sumandos distintos aparecen en el inventario de patrones


de las m~loraciones de los vértices de un poUgono regular de " lados?

16.12
Resumen y repaso histórico

Aunque el concepto de grupo de transformaciones evolucionó en forma gradual en el


estudio de la geometría. el principal impulso en el desarrollo del concepto de grupo proví-
no del estudio de las ecuaciones polinomiales.
Los métodos de solución de ecuaciones cuadráticas eran concx:idos para los antiguos
griegos. Después, en el siglo xvr, se hicieron algunos avances en la solución de ecuaciones
polinomiales de grado 3 y 4, con coeficientes racionales. Leonhard Euler (1707-1 783) y
830 (.apitulo 16Grupos, teoria el<, la codificación y método d<,enumeraóón el<, Polya

Joseph-Louis Lagrange (1736-1813) prosiguieron con los polinomios de grado may<Io


igual que cinco e intentaron resolver la ecuación general de quinto grado. Lagrange obser-
vó que debía existir una relación entre el grado n de una ecuación polinomial y el grupo de
permutaciones s•. Sin embargo, fue Níels HenrikAbel ( 1802-1829) quien demostró final.
mente que no era posible encontrar una fórmula para resolver la ecuación general de quia-
to grado utiliz.ando solamente operaciones de suma. resta. multiplicación. división o ex-
tracción de rafe.es. Durante el mismo periodo, el problema de la existencia de una condi-
ción necesaria y suficiente para la solución por radicales de un polinomio de grado n ~ S
con coeficientes racionales fue estudiado y resuelto por el ilustre matemático francés Evarisle
Gal oís (1811-1832). Puesto que la obra de Galoís utiliza las estructuras de grupos y cuer-
pos, diremos algo más de ella en el resumen del capítulo 17.

Niels Henrik Abel (1802-1829)

Si analizamos las páginas 278-280 de J. Stillwell [20), veremos que e l concepto de


grupo y. de hecho, la misma palabra actual "grupo" apareció por vez primera en la obra de
Galois Mémcire sur les conditions de résolubiliti <ks équarions par radicau.x, publicada
en 1831. La asociatividad., el neutro del grupo y los inversos eran consecuencía de las
hipótesis de Galois, ya que él trabajó únicamente con un grupo de permutaciones: de un
conjunto finito y su definición de grupo sólo pedía la propiedad de cierre. Fue Arthur
Cayley (1821-1895), en su anículo de 1854 On th, Th,ory o/Groups, as Depending on
the Symbolic Equa.ríon & = 1, quien vio por primera vez la necesidad de enunciar la pro-
piedad asociativa para los elementos del grupo. La primera mención real de los inversos
en la definición de grup:, aparece en el artículo de 1883 GruppenthEonttischen Studien II
de Waltber Franz Anton von Dyck (1856-1934).
El concepto de clase lateral, que presentamos en la sección 16.3, fue des.arrollado tam-
bién por Evariste Galois (en 1832). El término fue acuñado en 1910 por George Abram
Miller (186~1951).
Después de los logros de Galois, la teoría de grupos afectó muchas áreas de las mate-
máticas. Por ejemplo, a fines del sigloXDC, el matemático alemán Felix Klein (1849-1929),
16.12 Resumen y repaso histórico 831

en lo que ha dado en llamarse el programa de Erlanger, intentó codificar todas las geome-
trías existentes de acuerdo con el grupo de transformaciones en las cuales eran invariantes
las propiedades de la geometría.
Muchos otros matemáticos, comoAugustin-Louis Cauchy (l 78~1857),AnhurCayley
( 1821-1895), Ludwig Sylow (1832- 1918), Richard Dedekind (1831- 1916) y Leopold
Kronecker (1823-1891), contribuyeron al desarrollo de ciertos tipos de grupos. Sin em-
bargo,fueenl900cuandoseestablecióuna lista de condiciones para definir un grupo abs-
tracto general.
Durante el siglo xx. se han realizado muchas investigaciones en un intento por analizar
la estructura de los grupos finitos. Para los grupos abelianos finitos, se sabe que cualquier
grupo de este tipo es isomorfo a un producto directo de grupos cíclicos cuyo orden es una
potencia de un primo. Sin embargo, se ha visto que el caso de los grupos no abelianos
finitos es más complejo. A partir de la obra de Galois. se ha prestado panicular atención a
un tipo especial de subgrupo llamado subgrupo normal. Para un grupo G, un subgrupo H
(de G) es normal si para todo g E G y todo h EH, ghg-• E H. En un grupo abeliano, todo
subgrupo es normal, pero éste no es el caso para grupos no abelianos. En todo grupo G,
{e} y G son subgrupos normales, pero si G no tiene otros subgrupos normales decimos
que es simple. Durante las últimas cinco décadas, los matemátjcos han buscado y determi-
nado todos los grupos simples finitos y analizado su papel en la estructura de todos los
grupos finitos. Entre los principales promotores de este desarrollo están los profesores
Walter Fei~ Jobn Toompson, Daniel Gorenstein, Micbael Aschbacher y Roben Griess, Jr.
Para más detalles acerca de la historia y efectos de este monumental trabajo, el lector
puede coosultar los artículos de J. A. Gallian [3], A. Gardiner [5], M. Gazdner (6], R.
Silvestri (19] y, particularmente, el de D. Gorenstein (8].
Existen muchos textos a los cuales recurrir para un estudio posterior de la teoría de
grupos.A nivel introductorio, los textosdeJ.A. Gallian [4] yV. Lamey (11] proporcionan
un análisis que va más allá de la introducción dada en este capítulo. El texto de l. Herstein
[10] es una fuente excelente e incluye material relativo a la teoría de Galois.
Los inicios de la teoría de la codificación algebraica pueden seguirse hasta 1941. cuan-
do Claude Elwood Shannon comenzó sus investigaciones de los problemas en las comuni-
caciones. Estos problemas surgieron de las necesidades de la guerra. Su investigación
produjo nuevas ideas y principios que fueron publicados posteriormente en 1948. en el
artículo (18]. Como resultado de su obra, Shannon es reconocido como fundador de la
teoría de la información. Después de esta última publicación, aparecieron los resultados
de M. Golay (7] y R. Hamming (9], las cuales dieron un fuerte impulso a la investigación
en el área. Las 1478 referencias enumeradas en la bibliografía que aparecen al final del
volumen II de los textos de F. MacWilliams y N. Sloane (13] deben dar una idea de la
actividad en el área después de 1950.
Nuestro tratamiento de la teoría de la codificación sigue el desarrollo del capítulo 5 del
texto de L. Dornhoff y F. Hohn (2]. El texto de V. Pless (14] proporciona un tratamiento
agradable de los temas en un nivel intermedio. Un trabajo más avanzado relativo a la
codificación aparece en los libros de F. MacWilliams y N. Sloane ( 13] y de A. Street y W.
Wallis (21]. Una aplicación interesante del uso del principio del palomar en la teoría de fa
codificación aparece en el capítulo XI de (21].
En las secciones 9, 10 y 11 de este capítulo tratamos una técnica de enumeración cuyo
desarrollo se atribuye al matemático húngaro George Polya (1887-1985). Su artículo (15]
proporciona las ti6cnicas fundamentales para el recuento de clases de equivalencia de
isómeros químicos, grafos y árboles. (En cierta medida, las ideas de este trabajo fueron
832 O,pitulo 16Grupos. teoría de lacodificaóón y método deenumeraó6n de Polya

anticipadas por J. Redficld [ 17].) Desde entonces, estas técnicas han tenido un valor·
culable en los problemas de recuento de áreas como la realización electrónica de las
ciones booleanas. El teorema fundamental de Polya fue generalizado por pñmera vez
artículo de N. DeBruijn (1] y existen e n la bibliografía otras extensiones de estas ·
artículo de R. C. Read [ 16] relata la profunda influencia que el ieoo:ma de Polya ha ·
en los desarrollos en análisis combinatorio. (El número de la revista que contiene
artículo incluye también varios otros artículos relativos a la vida y obra de George
Nuestro tratamiento del tema sigue la presentación dada en el anículo de A
[22]. En el capíntlo 5 del texto de C. L. Liu aparece una presentación más águrosade
método.

BIBLIOGRAFÍA

1. DeBruijn, Nicolaas Govert, "Polya'sTheoryof Counting". capítuloSdeApplid


Matlu!malics, editado por Edwin F. Beckenbach. Nueva York, Wtley, 1964.
2. Dornhoff, Larry L y F ~ Hohn., Appl ied Mod.tmAlg,b,a. Nueva York, Macmillan, 1971.
3. Gallian, JosepbA., -rbe Searcb forFinite Simple Groups". MaJhemmics MagaziM49, 19'1\
págs. 163-179.
4. Gallian, JosephA., Comemporary Abstract Algebra, red., Lexington, Mass., D.C. Headl . .
Company, 1990.
5. Gardiner, Anlhony, "Groups of Monst=", N,w Scientist, 5 de abril de 1979, pág. 34.
6. Gatdner, Martin, '"!be Cap<wc ofthe Monstcr.AMathematical Group with a Ridiculous Numbs
of E!ements", Sci,nrific American 242(6), 1980, págs. 20-32.
7. Golay, Mare<I J. E., "Notes on Digital Coding", Procu mngs ofthe IRE 37, 1949, pág. 657.
8. Gorensteín, Daniel '1be Enormous Toeorom", Sci,ntific American 253(6), 1985, págs. 104-
115.
9. Hamming. Richard W., "Error Detecting and Error Correcti.ng Codes". &ll System Tedatial
Joumal 29, 1950, págs. 147-160.
10. Herstein. lsrael Nathan. Topics inAlgebra, red., Lexington. Mass.• Xerox College ~
1975.
11. Lamey, Violet H., AbstractAlgebra: A First Course, Boston. Prindle, Weber & Schmidt, 1975.
12. Llu, C. L , Introduction 10 Combina:lorial Mathemmics. Nueva York, McGraw-Hill, 1968.
13. MacWilliams, F. Jessie y Neil J. A.Sloane, TM Theory ofError-Correcting CO<US, vols. I y O.
Amscerdam. Nonh-Holland, 1977.
14. P1ess, Vera. Jntn>ducrion ro tire Tht!ory of Error-Co"ecting Codes, Z' cd., Nueva York, Wiley,,
1989.
1S. Polya, Gcorge, ·'Kombinatoriscbe Anzahlbestimmungcn für Gruppen, Grapben und Cbemisbe
Verbindungcn", Acta Mathematica 68, 1937, págs. 14~254.
16. Rcad, R.C., ..Polya's Thcorem ani:1 Its Progeny", Malhemalics Mag<JZiM 60, nó.m. 5, diciem-
b<e de 1987, págs. 275-282.
17. Redficld, J. Howard, 1be Theory of Group Reduced Distributions". American Joumal, tf
Ma,hematics 49, 1927, págs. 433-455.
Ejercicios complementaríos 833

18. Shannon, Qaudc E, 1be Mathematical 1beory of Communication.., lkll System Technical
Joumal 27. 1948, págs. 379-423. 623-656. Reimpreso en C. E. Sbannon y W. Weaver, The
Mat~ma1ical Theory ofComnumicalion (Urbana. University of lllinois Press. 1949).
19. Silvestri. Richard, ..Simple Groups of Finite Order", Arrhiveforthe, Historyof Exact Scknces,
20. 1979. págs. 313-356.
20. Stillwcll, Jobn. Ma1hematics and lts History. Nueva York. Springer-Vedag, 1989.
21. Strcct,Anne Penfold yW.D. Wallis, Combinatoria/ T/t¿ory: An Jntroduction, Wmnipeg, Cana-
dá. Toe Charles Babbage Research Ccnter, 1977.
22. Tucker,Alan. '"Polya's Enumeration Formula by Example", MatMltUUics Magazine. 47, 1974,
págs. 248--256.

e) Enuncie y demuestre una generalización que in-,


EJERCICIOS
cluya los resultados de las partes (a) y (b).
COMPLEMENTARIOS
9. Sea(G, •)ungrupotalque
x • a oy = b oa oc:>.roy = b oc,
L Sea/: G ➔ H un homomorfismo de grupos, donde e,,
,la identidad en H. Demuestre que para todos a, b, e, .r, y E G. Demuestre que (G, o) es un
a) K= (x E G lt<x) =es) es un subgrupodeG. ( K es grupo abeliano.
el núcleo del homomorfismo.)
10. Parak.,nEZ·conn.~k~ l,seaQ(n..,k)elnllJ:nerode
b) si g E G y x E K. entonces gxg-1 E K.
pcnnutaciones1t E S., tales que cualquier representación de n
~ SiG, HyKson grupos y G=H x K,demuestrcqueG como producto de ciclos disjuntos no contiene ciclos de lon-
aic:oc: subgrupos que son isomorfos a H y K. · gitud mayor que k. Verifique que
L Sea G UD grupo ta1 que a2 = e para todo a E G. De-
aestre que Ges abcliano. Q(n + l,k)= H()
L
,-o
n (i!)Q(n - i,k~
J
L Si Ges UD grupo de orden par, demuestre que existe un
1
siento a E G tal que a t-e y a= a- • 11. Parak,nEZ·talesquen~2yl Sk::S:n,seaP(.n..,k)
~ Sea/: G ➔ Hun homomorfismo de grupos. Si Tes un s.
el número de permutaciones x E que tienen k ciclos. (Por
bgrupo deH, demuestreque/~'<n = (a E G l/(a) ET) ejemplo, (JX23) se cuenta en P(3, 2), (12)(34)secuentaen
P(4, 2) y (1)(23)(4) se cuenta en P(4, 3).)
a subgrupo de G.
a) Verifique queP<..n + l , k) = P(.n, k-1) +n.P(,n. k).
~ Demuestre que los grupos ~ y Z 1 x Z 3 no son
IDOcfos. considerando los órdenes posibles de los ciernen-
b) Determine L;_,P(n, k ).
12. Paran~ l. si o . -r ES"' definimos la distanciad(o, t)
entre oy-rcomo
'. Seaf: G ➔ H un homomorfismo sobreyectivo de gro-
s. Si G es un grupo dclico, demuestre que H también es d(o, , ) = máx{ la(i)-T(i)j I I ,;¡ ,s;n}.
:lico. a) Demuestre que d satisface las siguientes propie-
e a) Considemnos el grupo (Z, x Z,, $ ) donde, para dades.
a, b, e, d E Z,, (a. b) $ (e, d ) =(a+ e, b + d); las 1) d(a, t) a, O para todas a , t e s.
sumas a + e y b + d se calculan mediante la suma ii) d (o, t ) =O si y sólo si a= t.
módulo 2. ¿Cuál es el valor de (1, O) EB (0, 1) e iiil d(a, t) = d(a, t ) para todas a, t e s,.
(1, l ) enestegrupo? iv) d(p,t ) sd(p,a)+d(o,t) poratodosp,a,tES,
b) Consideremos ahora el grupo (Z:i x ~ x Z:i, EB), s.
b) Sea E el elemento identidad de (es decir, E(i) i =
donde(a. b. e) $(d. e./)= (a +d,b+e, e+/). (En para todo l si s n). Si 1t E s. y d(1t, E) ::s: l , ¿qu'=
este caso, calculamos las sumas a+ d. b + e, e+ f se puede decir de ll(n)?
usando la suma módulo 2.) ¿Qué obtenemos cuan- e) Para n .!: l , sea ª• el número de permutaciones 11.
do sumamos los siete elementos distintos de cero en s.
tales que d(it. E) s l. Encuentre y resuelva
(o distintos del oeutro) en este grupo? una relación de recurrcncia para a,..
834 Capitulo 16 Grupos, teoria de la codifi<ao6n y método de enumeración de Fl:,l¡a

13. La prueba de un proyectil consiste en dispararlo a un b) En realidad utiliza cxactammte cuatro de loscirico
blanco. Su¡xmga que la probabilidad de acertar en cualquier colores disponibles?
prueba simple es de 0.75 y que los resultados de los dispa•
ros sucesivos son independientes.
a) Si disparamos cuatro proyectiles. ¿cuál es la p~
babilidad de lograr (i) exactamente dos aciertos?
(il) al menos dos aciertos?
b) ¿Cuántos proye.ctiles debemos disparar para que la
probabilidad de tener (al meoos) un acierto~ al
menos de 0.95?
14. ¿De cuántas formas puede Nina pintar las ocho regio-
nes del cuadrado que se muestra en la figura 16.17 si
■) dispone de cinco colores?
Figu r¡,, 16.1 7
17
Cuerpos finitos
y diseños
combinatorios

· A hora es tiempo de recordar la estructura de anillo del capítulo 14 para estudiar los anillos
de polinomios y su papel en la consttucción de cuerpos finitos. Sabemos que para cual-
quier primo p, (Z¡.. +,•) es un cuerpo finito, pero aquí veremos otros cuerpos finitos. Así
como el orden de un álgebra booleana finita debe ser una potentia de 2. los órdenes posi-
bles para los cuerpos finitos son p--, donde p es primo y n E z•. Como aplicación de estos
cuerpos finitos analizaremos algunos diseños combinatorios, por ejemplo, los cuadrados
latinos. Por 6.ltimo, investigaremos la estructura de una geometría finita y veremos la rela-
ción entre este tipo de geometrías y los diseños combinatorios.

17.1
Anillos de polinomios

Recordemos que un anillo (R, +,•) consta de un conjunto no vacío R, tal que (R, +) es un
grupo abcliano, (R, •} es cerrado bajo la operación asociativa• y las dos operaciones están
relacionadas mediante las leyes distributivas: a(b +e) = ab + ac y (b + c)a =ba + ca, para
a, b. e E R. (Escribimos aben vez de a· b.)
Para preseqtar el concepto formal de un pollllomio con coeficientes en R. sea x una
indeterminada; es decir, un símbolo que no es elemento del anillo. Utiliz.amos este símbo-
lo x para definir lo siguiente.

efinidón 17.1 Dadoun anillo(R. +, •),u.na expresión de la formaf(x);:;;a,.x• +a,._ 1x •- 1 + · · · +a,x1 +<Jor'.
donde a; E R para todo O :S i s n, es un p0Iincmio en la indete rminada x con coeficientes
en R.
Si a. no es el elemento cero de R, entonces a. es el coeficiente principal de f(x) y
decimos que f(x) tiene grado n. Por lo tanto, el grado de un polinomio es la máxima
potencia dex que aparece como sumando del polinomio. El término ao;iJ es la constante. o
término constante, de f(x).
Si g(x ) =b. X"' + b__ 1x-- 1 + • · • + b 1x1 + b0.i' es otro polinomioenx sobreR, entonces
f(x) =g(x ) si m =n y a, = b, para todo O :S i ,; n.

835
836 Capítulo 17 Cuerpos finitos y diseños combinatorios

Por último, usaremos la notación R[x] para representar el conjunto de todos los
polinomios en la indeterminada x con coeficientes en R.

a) Sobre el anillo R = (Z.. +,•),la expresión Si'+ 3r -U' es un polinomio de grado


2, con coeficiente principal S y término constante -2,!J. Como antes, en este caso
usamos a para denotar [a) en z..También podemos escribir este polinomio caDO
Si'+ 3x' + 4.x" ya que [4) = [-2) en z..
b) Si z es el cero del anillo R. entonces el polinomio cero z,!J = z es también el cero de
R[x]; decimos que este polinomio no tieM grado y no tiene coeficiente principal.
Un polinomio sobre R que sea el cero o tenga grado Oes un polinomio constanu.
Por ejemplo, el polinomio 5.x" sobre Z, tiene grado O y cocficienle principal 5, pcr
lo que es un polinomio constante.

Presentamos ahora las operaciones (cerradas binarias) de suma y producto de estos


polinomios para obtener un nuevo anillo. Para un anillo de coeficientes (R, +, •), sean

+
/(JC) =a.x" + a.-,x"-1 + · · · + a,x' a.r'
g(JC) = b.r" + b.- ,...--, + · · · + b1JC1 + b,JC',
dondea¡,b1 E RparaO sis n,O sj :S m.
Si n ?:: m, definímos

..
f(x) + g(JC) = ~(a;+ b,}x',
, (1)

donde b¡ = z para i > m, y

/(JC)g(JC) = (a.b.)r''• + (a.b.- , + a.-,b.)r''•-1 + · · ·


+(a,b,+a.b,)x'+(a.b,)xº. (2)
En la definición de/(x) + g(x), el coeficiente (a,+ b,), para cada O s i s n, se obtiene al
sumar elementos en R. Para/(x)g(x), el coeficiente de x' es L,~a,-t bn donde todas las
sumas y productos ocurren dentro de R y O :S t :S n + m. El siguiente es un ejemplo del tipo
de cálculos utilizados.
Scan/(x) =4x' + 2x' + 3x' + lx'y g(x)=3x' +r +U polinomios de Z,[x]. En este caso,
a1=3, a,= l ,
y
b,=3, b1 = l, b,=2.
Para todo n ~ 4 tenemos que a.= O. Si m e?! 3, tenemos que b. = O. Usamos las definicio-
nes de las ecuaciones (l) y (2), donde las sumas y productos de los coeficientes son módu-
lo 5, para obtener
f(x) + g(x) = (4 + O)x' + (2 + 3).r' + (3 + l)x1 + (1 + 2)x0
= 4.r'+ ar+ ""'+ 3x'= 4.r'+ ""' + 3r'
17.1 Anillos de polinomios 837

f(x)g(x) = (ta,_,b,y (ta,_,b,~• (ta,-,b,Y


+ +

+ (*.a,_,b,y+ (*.a,_,b,~' + (ta.-,b,r


= (0·2 + 0· l + 4·3 +2·0 + 3·0+ l ·0)x'
+ (0·2+4 · l + 2·3 +3:0+ l·0)x'
+ (4 · 2 + 2 · l + 3 · 3 + l · 0)x'
+ (2 · 2 + 3 · l + l · 3)x' + (3 · 2 + l · l)x' + (1 · 2)x"
=~+~+~+~+~+~=~+~+~+~
Las operaciones cerradas binarias definidas en las ecuaciones (1) y (2) están diseñadas
para obtener el siguiente resultado.

TEOREMA 17.1 SiR es un anillo, entonces (R[x], +,•),con las operaciones de suma y producto dadas en las
ecuaciones (1) y (2), es un anillo, llamado el anillo tk po/úwmú,s, o anillo po/úwmia/
sobre R.
Demostración: Las propiedades de anillo para R[x] se basan en las de R. En consecuenci~
aquí demostraremos la ley asociativa del producto como un ejemplo, y dejaremos las de-
más demostraciones al lector. Sea h(x) = r:..c.xh y
con/(x) g(x) definidos como antes.
Un sumando úpico de (f(x)g(x))h(x) tiene la forma A.t', donde O s r :s (m + n) + p y A es
la suma de todos los productos de la forma (a;b,)c., con Os i :s n, O :Sj :S m, O :S k :S p ei
+ j + k = t. En .l{x)(g(x)h(x)), el coeficiente de res la suma de todos los productos de la
formaal.,bicJ, de nuevo con Os i :S n., O :Sj :S m, O :S k s pei + j +k=t. Puesto queR
es asociativo la multiplicación, (a;b1)c, = a,.b1c,) para cada uno de esos términos, y entonces
el coeficiente de r en (f(x)g(x))h(x) es igual al de f(;cXg(x)h(x)). Por lo tanto, (j(x)g(x))h(x) =
f(x)(g(x)h(x)).

COROLARIO 17 .1 Sea R[x] un anillo de polinomios.


a) Sí Res conmutativo, entonces R[x] es conmutativo.
b) Si R es un anillo con elemento unidad. entonces R[x] es un anillo con elemento
unidad.
e) R[x] es un dominio de integridad si y sólo si Res un dominio de integridad.
Demostr.Kión: Dejamos la demostración de este corolario al lector.

Desde este momento, escribiremosx en vez de x'. Si R tiene elemento unidad u, defini-
mos xO = u y para cualquier r E R escribimos ,,!>como r.

Seanf(x), g(x) E Z.[x], con/(x) = 4x' + 1 y g(x) = 2x + 3. Entonces/(x) tiene grado 2 y


g(x) tiene grado 1. Por nuestras experiencias anteriores con los polinomios. esperamos que
838 Capitulo 17 Cuerpos finrtos y diseños combinatorios

el grado def(x)g(x) sea 3, la suma de los grados de/(x) y g(x). Sin embargo, eo este caso,
f(x)g(x) = (4x' + IX2x+ 3)= 8x' + 12x'+ 2x+ 3 =4x' + 2x+ 3, puesto que [8] = [O] eoZ,.
Así, el grado de/(x)g(x) = 2 < 3 = gradof(x) + grado g(x).

La causa del fenómeno del ejemplo 17.2 es la existencia de divisores de cero en el


anillo z..
Esta observación nos lle va al siguiente teorema.

TEOREMA 17.2 Sea (R, +,•)un anillo conmutativo con elemento unidad u. Entonces Res un dominio de
integridad si y sólo si para todosf(x),g(x) E R[x], sif(x) y g(x) oo son el polinomio cero,
entonces

gradof(x)g(x) = grado f (x) + grado g(x).

Demostración: Seao/(x) =L;',. a, x', g(x) =I,;_.b, x' , cona,, -j,.z , b. t- z_Si Res un dominio
de integridad, entonces a.b.-;,. z, de modo que el grado de f(x)g(x) es n + m = gradof(x) +
g:radog(x). Por el contrario, si Rnoes un dominio de integridad. sean a. b E Rtalesquca+.
z, b-j,.z peroab = z. Los polinomios/(x) =ax+ u, g(x) =bx +u tienen grado 1, pero/(x)g(x)
=(a+ b)x + u y grado f(x)g(x) :S 1 < 2 = gradof(x) + grado g(x).

SeanRunanilloconelemcntounidaduy/(x) = a.,t' + · · · +a 1.r+ao E R(.r]. Si re R,


entonces/(r) =a/ + · · · + a1r + ao E R. Nos interesan particularmente los valores de r
tales que/(r) =z.

Definición 17.2 Seao Ruo anillocooelemeoto unidad u y/(x) E R[x], con gradof(x) ;,, l. Si rE Ryfl.r) =z.
entonces res una raíz del polinomio / (x).

a) Si/(x) =x' - 2 E R[x] , entonces/ (x) tiene raíces,/2 y - ,/2, puestoque(,/2)'-2 =


O = (-,/2)'- 2. Además, podemos escribir/(x) = (x-,Í2Xx + ,/2), coox - ,/2, x+
,Í2 E R[x]. Sin embargo, si consideramos / (x) como elemento de Q[x], entonces
f(x) no tiene raíces, pues .fi y -Jz son números irracionales. En consecuencia, la
existencia de las raíces de un polinomio depende del anillo correspondiente para los
coeficientes.
b) Para/(x) =x' + 3x + 2 E 7.Jx], vemos que

/(O) =(0)2 + 3(0) + 2 = 2 /(3) = (3)2 + 3(3) + 2 = 20 = 2


/(1) = (l )' + 3(1) + 2 = 6 = O /(4) = (4)2 + 3(4) + 2 = 30 = 0
/ (2) = (2)2 + 3(2) + 2 = 12 = O /(5) = (5)2 + 3(5) + 2 = 42 = O

En consecuencia,/(x) tiene cuatro rafees: 1, 2, 4 y S. Esto es más de lo que esperá-


bamos. En nuestras experiencias anteriores, un polinomio de grado 2 tenía cuando
mucho dos raíces.
17. 1 Anillos de polinomios 839

En este capítulo trataremos principalmente de anillos de polinomios K[x], donde K es


un cuerpo (y K(x] es un dominio de integridad). En consecuencia. ya no analizaremos
situaciones en las que grado /(x)g(x) < grado /(x) + grado g(x). Así mismo, a menos que
digamos lo contrario, denotaremos el elemento cero de un cuerpo con Oy usaremos 1 para
denotar su elemento unidad.
Como resultado del ejemplo I7.3(b), desarrollaremos ahora los conceptos necesarios
para que un polinomio de grado n tenga cuando mucho n raíces.

finidón 17.3 Sea Kun cuerpo. Si/(x), g(x) E K[x] y /(x) no es el polinomio cero,/(x) es un divisor(o
factbr) de g(x) si existe h(x) E K[x] tales que/(x)h(x) = g(x). En este caso, también deci-
mos que f(x) divide a g(x) y que g(x) es un múltiplo de /(x).

Esto nos lleva al algoritmo de la división para polinomios. Sin embargo, antes de de•
mostrar el resulr.ado general, examinaremos dos ejemplos particulares.

En cursos anteriores de álgebra aprendimos la división larga de polinomios con coeficien-


tes reales. Dados dos polinomios /(x), g(x) con grado /(x) S grado g(x), organizamos
nuestro trabajo en la forma
q 1(.x) + q2(.x) + · · · + q,(x) ( = q(x))
/(x))g(x)
/(x)q,(x)
g(x) - /(x)q1(x)

r(x)
donde continuamos dividiendo hasta que

r(x) =O o grado r(x) < grado /(x).

Entonces, g(x) = q(x)/(x) + r(x).


Por ejemplo, si/(x) =x-3 y g(x) = 7:r'-Zr + 5'- 2,entonces/(x),g(x) E Q[x] (o R[x],
o C(x]) y vemos que
7x' + 19.x + 62 (= q(x))
x - 3J7x1 - 2.? + 5x - 2
7r' - 2lx'
~ + 5x - 2
19x' - 57x
62x- 2
62x -186
-----¡g;¡: ( = r(x))
Si verificamos estos resultados, tendremos

q(x)/(x) + r(x) = (7x' + 19.x + 62)(.x - 3) + 184 =7x3 - 2x2 + 5x - 2 = g(x).


840 Capítulo 17 Cuerpos finitos y diseños combinatorios

La t6cnica del ejemplo 17.4 también se aplica cuando los coeficientes de los polinomios se
toman de un cuerpo finito.
Si/(x) = 3r + 4x + 2 y g(x) = 6x' + 4x' + 5r
+ 3x + 1son polinomios en Z,[x], entooca
del proceso de división obtenemos los siguientes cálculos.
:zr'+ X+ 6(=q(x)}
3x' + 4x + 2)6x' + 4?+ 5x' + 3x + 1
6x' + x'+ 4x'
3x3 + x'+3x+l
3x' + 4x' + 2x
4x'+ x+I
4x2 +3x + 5
Sx + 3 (= r(x))
Si realizamos (como antes) toda la aritmética en Zi. veremos que
q(x)f(x) + r(x) = (2.r' + x + 6)(3x' + 4x + 2) + (Sx + 3)
= 6x' + 4x'+ Sx' +3x + 1 = g(x)

Pasemos ahora a la situación general.

TEOREMA 17.3 Algorirmo de la división. Sean f(x), g(x) E K[x] y /(x) no es el polinomio cero. Existen
polinomios únicosq(x), r(x) E K(x) tales queg(x) =q(x)/(x) +r(x),donder(x) =O o grado
r(x) < grado /(x).
Demortradón, SeaS= {g(x)-l(x)/(x)ll(x) E K[x)}.
Si O E S, entonces O= g(x)- l(x)f(x) para algún l(x) E K(x]. Entonces, con q(x) = r(x)
y r (x) = O, tenemos que g(x) = q(x)/(x) + r(x).
Si O "'S, consideremos los grados de los elementos de S y sea r(x) = g(x) -ql_x)f(x) un
elemento en S de grado mínimo. Como r(x) t O. el resultado se sigue si grado r(x) < grado
/(x). En caso contrario, sean

r(x) = a,.x" + a..,_ 1x"- 1 + · · · + G.JX2 + a1 x + Clo, a,+o.


f(x) = b.x'" + b.-,x'"-1 + · · · + b,x' + b 1x + b0 , b.;óO,
con n ~ m. Definimos

h(x) = r(x) - (a,b;' x'-•Jf(x) = (a, - a,b;' b.)x" + (a,_, - a,b;' h.-,)x"-1
+···+(a,,_.,. - a,.b;1 b0 )x"-"' + a,._.,._ 1 .r"'-•- 1 + · · · + a 1x + ao.

Entonces h(x) tiene grado menor que n, el grado de ,(.x-). Aún más imponante, h(x) = [g(x)-
ql_x)/(x)] - [a,b~•r•Jf(x) = g(x)- (ql_x) + a, b~•x"-•]f(x), por lo que h(x) ES, lo que
contradice la ele.cción de r(x) por tener grado mínimo. En consecuencia, grado r(x) <
grado /(x) y obtenemos la parte de existencia del teorema
Para la unicidad, seag(x) =q.(x)f(x) + r 1(x) = q,(x)f(x) + r2(x), donde r1(x) = O o grado
r1(x) < grado/(x), y r,(.r) = O o grado r-f.x) < grado/(x). Entonces [ql.x) - q,(x)Jf(x)) = r,(x)
-r,(x) y siq2(x)-q1(x),'O, entonces grado([q-f.x)-q1(x))/(x)) a: grado/(x), mientras que
17.1 Anillos de polinomios 841

r,(x) - r,(x) = O o gn,do [r,(x ) - r,<x)J S máx{grado r,(x),grado r,<x)) < grado f (x ). En
consecuencia, q,(x ) = q,(x) y r,(x) = r,(x).

El algoritmo de la división nos proporciona los siguientes resultados acerca de raíces y


factores.

E/teor,mad,I restb. Paraf(x) E K[x] y a E K,elrestodeladivisióndef(x)entrex-aesf (a).


Demostración: Del algoritmo de la división, f(x) = q(x)(x - a)+ r(x), donde r(x) = O o
grado r(x) < grado(x-a) = l. Por Jo tanto. r(x) = res un elemento de K. Si sustituimos a
por x, vemos quef(a) = q(a)(a- a)+ r(a) =O+ r= r.

El teor,ma delfacu,r. Sif(x) E K[x] y a E K, entoncesx - a es un factordef(x) si y sólo


si a es una raíz de j(x).
Demostración: Si x - a es un factor de f(x), entonces f{x) = q(x)(x - a). Como f(a) =
q(a)(a - a)= O, a es una raíz de /(x). Recíprocamente, supongamos que a es una raíz de
f(x). !'oc el algoritmo de la división, f(x) = q(x)(x- a) + r, doode r E K. Como f(a) = O,
tenemos que r = O, de modo que f(x) = q(x)(x - a) y x- a es un factor def(x).

a) Seaf(x) =x' -6.x' + 4x'-x' + 3x- 1 E Q[x]. Del teorema del resto, se sigue que al
dividir / (x) entre x- 2. el resto es
f(2) = 2' - 6(2') + 4(2') - 2' + 3(2) - 7 = - 5.
Si fuéramos a dividirf(x) entre x + l, el resto seriaf(-1) = - 2.
b) Si dividimosg(x) = x' + 3x' +x' +x' + 2x + 2 E Z,[x] entrex- 1, entoncesel resto
en estecasoesg(l)-= 1 + 3 + 1 + I +2 +2=0(enZ,). En consecuencia.x- i divide
a g(x) y por el teorema del factor,
g(x) = q(x)(x - 1) (donde grado q (x) = 4).

Con la ayuda de los resultados de los teoremas 17.4 y 17.5, plantearemos ahora la
ú1tima de las ideas principales de esta sección.

lEOREMA 17.6 Si/(x) E K[x] tiene grado n 2: 1, entooces/(x) tiene un máximo den raíces en K.
Demostración: La demostración es por inducción matemática sobre el grado de/(x). Si/(x)
tiene grado 1, entoncesf(x) =ax +b, para a, b E K,a # O. Comof(-a-'b) = 0.f(x) tiene al
menos una raíz en K. Si c 1 y '1 son dos raíces. cntonccs/(c1) -= ac 1 +b = O= ac2 +b =fl.c2).
Por la cancelación en un anillo. ac1 + b =ac2 + b => ac1 =ac1• Como K es un cuerpo y a 'F
O, tenemos que ac1 -= ac2 ~ c 1 = c2, por Jo que /(x) só1o tiene una raíz en K.
Supongamos ahora que el resultado del teorema es verdadero para todos los polinomios
de grado k(2: 1) en K[x]. Consideremos un polinomíof(x) de grado k+ l. Sif(x) no tiene
842 Capítulo 17 Cuerpos finitos y diseños combinatorios

raícesenK, el teoremaquedademosttado. Encaso contrario, sear E K tal que/(r) =O.lw


el teorema del factor.f(x) = (x- r)g(x), donde g(x) tiene grado le En consecuencia, perla
hipótesis de inducción, g(x) tiene a lo sumo k rafees en K y /(x) tiene, a su vez, k + 1llÓIZI
enK.

a) Sea/(x) = x'-fu + 9 E R[x]. Entonces/(x) tiene como máximo dos rafees en l.a
saber, las raíces 3, 3. Además,/(x) = (x- 3)(x- 3), una factoázación en dos factma
de primer grado, o lineales. Así, en este caso, 3 es una raíz de mub.ipücidad2.
b) Para g(x) = x' + 4 E R [x], g(x) no tiene rafees reales, pero el teorema 17.6 nose
contradice. (¿Por qué?) En C[x], g(x) tiene las rafees 2i, -2i y se puede facu:irim
como g(x) = (x-2i)(x + 21)
e) Si h(x) = x' + 2x +6 E Z,[x],cntoncesh(2) = 0,h(3) = Oy éstas son las únicas ralea
del polinomio. Además, h(x) = (x - 2)(x - 3) = x' - Sx + 6 = x' + 2x + 6, pues [-5] =
[2] en Z,.
d) Como vimos en el ejemplo 17.3{b). el polinomio :t' + 3x + 2 tiene cuatro rú:es.
futo no contradice el teorema 17.6, pues Z, no es un cucrpo.Además,xl + 3x+ 2=
(x + IXx + 2) :;: (x + 4Xx + 5), dos factorizaciones distintas.

Terminaremos con una última observación, sin demostración, acerca de la idea de


factorización enK[x]. Sif(x) E K(x] tiene gradan y ri, r2 , ••• , r. son las raícesde/(z)ea
K (donde las raíces pueden repetirse.es decir, r, = r¡para algunos 1 Si <j S n}, entonces
f(x) = a.(x - r,)(x - r,).· · · (x - r.), donde a. es el coeficiente principal de /(x). Em
representación dc/(x) es única saJvo por el orden de los factores de primer grado.

EJERCICIOS 17.1 1. Sean /(x1 g(r) E z,{x], donde /(x) =U+ 2x' + 3r + x + 4 y g(r) = 3x' + 5x' + 6r + l.
Determine /(x) + g(_x),/(r) - g(x) y /(x)g(r).
2. Determine todos los polinomios de grado 2 en Z:?[x].
3. ¿Cuántos polinomios de grado 2 hay en Z 11[xJ? ¿Cuántos de grado 3? ¿Cuántos de grado 4'?
¿Cuántos de grado n, paran EN?
4. a) F.ncucntrc dos polinomios no nu1os/(x), g(x)en Z 1Jx], tales quc/(x)g(x) = O.
=
b) Encucnt,c polinonrios h(x), k(r) E Z.,(x] tales que grado h(x) 5, grado k(x) =2 y gndo
h(x)k(r) = 3.
S. Tennine las demostraciones del teorema 17.l y el corolario 17.1.
6. Si/(r)=ax' +br +a+d,g(_x)=5x'+ 3x- 7 E Z(x) y/(x)= (3x+ l)g(_x),encuenttca, b, cyd.
7. a) Si/(.r) =x'- 16, encuentre sus rafees y factorización en Q[x].
b) Responda la pane (a) para/(x) E R(x].
e) Responda la pane (a) para/(x) E C(x].
d) Responda las panes (a), (b) y (e) paraf(x) =x:' - 25.
8. Para cada uno de los siguientes pares/{x), g{x), encuentre q(x), r(x) tales queg(x) = q(x)/(x) +
,(x ), donde ,(%) = Oo grado ,(x) < grado /(x).
a) f(x),g(x) EQ(x], f(x) =x• - 5x' + 1x,g(x) =x' -2x' + 5x - 3
b) f(x),g(x) E Z,(x] , /(x) =x' + l,g(x) =x• + x' + x' + x + 1
e) f(x),g(x) E Z,[x] , f(x) =x'+ 3x + l,g(x) =x•+ 2x' + x + 4
17.2 Polinomios irreducibles: Cuerpos finitos 843

9. En lo siguiente, encuentre el resto que resulta de dividir f(x) entre g(x).


a) /(x),g(x)EQ[x), f(x) =.r' + 7x' -4x' + 3x' + 5x' - 4, g(x) = x -3
b) /(x),g(x)E z,¡x],f(x) =x'00 + x'° + x"' + x'° + 1, g(x) =x - 1
=
e) /(x),g(x)EZu[x], f(x) 3x' -8x' + x' - x' + 4x -7, g(x) = x + 9
10. a) Encuentre todas Las raíces de/(x)= x' + 4x ,;¡e,¡ E Zu[x].
b) Encuentre cuatro polinonúos linealcx <listintosg(x), h(x).s(;;), l(x) E Z,,lx] tales quef(x) =
g(x)h(x) = ,(.r)l(x).
e) ¿Contradicen los resultados de la parte (b) las proposiciones establecidas en el párrafo
posterior al ejemplo 17.77
11. ¿E.i v.ilido el algoritmo de la división (para polinomios) en el dominio de integridad Z[x]?
Explique por qué.
12. Para cada uno de los siguientes polinomios/(x) E Zdx]. determine todas las raíces de Z,. y
cscriba/(x) como producto de polinomios de primer grado.
a) f(x) = x'+5x'+2x+6 b) f(x)=x' -x
13. Eoc:ueotrcunpolinomio/~)enZ{x ]cuyas raíces sean 1/3 y- Vl. [Observequecomo(Z.+. •)
ex un subanillo de (Q, +.•).podemos connderar af(x) como un elemento de Qlx ].J
14. Dado un cuerpo K, snf(x) E K[x], dondc/(x)= a,.r- + a_.¡X"'· 1 + · · · + a)'r +a1x + ao,
a ) Demuestrcquex - 1 es factordc/(x) si y sólo si a. +a,,. 1 + · · · + a:+ a 1 + ao= O.
b) Sin es par,demuestreque x + 1 csfactordc/(x)si y sólo si a,..+a•. 2 + · · · +a:+~=a,,. 1 +
a,..3+ ··· +a3 +a1.
15. ¿Cuú.tas unidades tiene el anillo Z,[.rJ? ¿Cuántas tiene Z,.Cx]? ¿Cuántas tiene z,(x], con p
primo?
16. Si Res UD domitúo de integridad. demuestre que si/(x.) es una unidad en R[x], entonc:esj(x) es
una constante y es una unidad en R.
17. Verifique qucf(x);-2r+ 1 es una unidadenZ,dx]. ¿Cootradic:e esto el resultado del ejercicio 16?
18. Paran E z·,n
z:: 2,sea/(x) E Z.[x]. Demuestre que si a. b E Z ya•b(mod n),enroncesf(a) •
f(bl (mod n).
19. Sean R, S amnos y sea g: R ➔ S un homomorfismo de anillos. Dcmucsuc que la función G:
Rlx] ➔ S[x] dada por

es un homomorfismo de anillos.
20. Sea (R, +. •) W1 anillo. Si Tes un ideal de R, demuestre que /[x]. el conjunto de todos los
polinomios en la indcten:ninada x con coeficiente$ en f, es un ideal en R{x].
21. Si K es un cuerpo, sea S ~ K[x], donde/(x) =a.:r-
+ a__¡X" - 1 + · · • +a2J! + atX +aoE S si y
sólo sí a. +a,,_ 1+ · · · +a2 + a 1 + ao= O. Demuestre que Ses UD ideal de K[x].

17.2
Polinomios irreducibles:
Cuerpos finitos

Ahora queremos construir cuerpos finitos distintos de los del tipo (Z,. +, ·), donde pes
primo. En la construcción usaremos los siguíentes polinomios especiales.
844 Capítulo 17 Cuerpos finitos y diseños combinatorios

~ nición 17.4 Su./(x) E K[x), con Kun cuerpo y grado/(x) 2: 2. Dedmos quc/(x) es n,ducible (sobre
K) si existen g(x), h(x) E K[x), tales que/(x) = g(x)h(x) y cada uno de los polinomios g(x),
h(x) tiene grado :!: l. Si/(,t) no es redUCil>le, entonces es irrtducible, o primo.

El teorema 17.7 contiene algunas observaciones lltiles acerca de los polinomios


imducibles.

TEOREMA 17.7 Pan polinomios en K[x),

a) cualquier polinomio no nulo de grado S 1 es irreducible.


b) si/(x) E K[x] con grado/(x) = 2 o 3, entonces/(x) es reducible si y sólo si/(x) tiene
una raíz en el cuerpo K.
0emostt>ción: Se deja al lector.

a) El polinomio x' + 1 es irreducible en Q [x]y R[x), pero en qx) vemos que x' + 1 =
(x+ i)(x -1).
b) Sea/(x)=x' +2x' + 1 E R[x).Aunque/(x) no tiene rafees reales, esreducible,pucs
(x' + 1)2 = x' + 2x' + l. Por lo tanto, la parte (b) del teorema 17.7 no se aplica a los
polinomios de grado > 3.
e) En Z,[x),/(x)=x' +x'+x + 1 es reducible pues/(!)= O. Perog(x)=.r'+x+ 1 es
imducible pues g(O) = g(l) = l.
d) Su. h(x) = x' + x' + x' + x + 1 E Z,[x). ¿Es h(x) reducible en Z,[x]? Como h(O) =
h(l) = 1, h(x) no tieoe factores de primer grado, pero podríamos encontrar a, b, e, d E
Z, tales que (x' +a,+ bXx' + a +d) =x' +.r' + x' +x + l.
Desarrollamos (x' +ax+ bXx' + ex+ d) y comparamos los coeficientes de las
potencias similares de x. con lo que obtenemos a + e = 1, tu: + b + d = 1, ad+ be =
1 ybd= I.C<E!Obd= 1, teoemosqucb= 1 yd= 1,po,loqueac+b+d= 1 =>ac=
l =>a =e= 1 =>a +c =O. Esto contradice el becbodequea + e:: l. En coosecuen--
cia. h(x) es imducible en Z,[x].

Todos los polinomios del ejemplo 17.8 comparten una propiedad común, que defume.
mos a continuación.

Definición 17.S Un polinomio/(x) E K{xJ csmónico si su coeficiente principal es 1, el elemento unidad


deK.

Algunos de nuestros siguientes resultados (basta el análisis del ejemplo 17.11 inc:lusi--
ve) nos recordaran los capítulos 4 y 14.

~nición 17.6 Si/(x), g(x) E K[x), entonces h(x) E K[x] es un máximo común divisordo/(x) y g(x)
17.2 Polinomios irreducibles: Cuerpos finitos 845

a) si h(x) divide a/(x) y a g(x); y


b) si k(x) E Kjx] y k(x) divide a/(x) y a g(x), entonces k(x) divide a h(x).

Estableceremos ahora los siguientes resultados relativos a la existencia y unicidad de Jo


que llamaremos el máximo común dívísor. que abreviaremos como mcd. Además. existe
un m~todo para determinar este divisor, conocido como el algoritmo de Euclides para
polinomios. La demostración de este primer resultado se bosqueja en los ejercicios de la
sección.

Sean /(x~ g(x) E K[x], con al menos uno de /(x) o g(x) no nulo. Entonces, cualqlrier
polinomio de grado mínimo que se pueda escribir como combinación lineal de/(x) y g(x)
(es decir, en la forma s(x)f(x) + l(x)g(x), para s(x), l(x) E K[x]) sera un máximo comt\n
divisor de /(x) y g(x). Si pedimos que el mcd sea móníco, entonces sera único.

Algoritmb de Euclides para pa/inomios. Sean /(x), g(x) E K[x], con grado /(x) S grado
g(x) y /(x) +
O. Sí aplicamos el algoritmo de la división, escribirnos

g(x) = q(x)f(x) + r(x), grado r(x) < grado fi:x)


f(x) = q ,(x)r(x) + r 1(x), grado r,(x) < grado r(x)
r(x ) = q,(x)r,(x) + r 2(.t), grado rlx) < grado r 1(x)

r,_,(x) = q.(x)r,_ 1(x) + r.(x), grado r.(x) < grado r._,(x)


r,_,(x) = q,.,(x)r.(x) + r,.,(x), rh 1(x)=O.

Entonces r.(x). el ó1timo resto distinto de cero. es un máximo común divisor de/(x) y
g(x) y es un múltiplo constante del máximo comt\n divisor mónico de /(x) y g(x). [Si
multiplicamos rt(x) por el inverso de su coeficiente principal, obtendremos el ónico
polinomio mónico al que llamaremos el máximo común divisor.]

Definición 17.7 Si/(x), g(x) E K[x] y su mcd es 1, entonces/(x) y g(x) sonprimosentrui.

Los íiltimos resultados que necesitamos para construir nuestros nuevos cuerpos finitos
son el análogo de la construcción desarrollada en la sección 14.3.

lEOREMA 17.10 Seas(x) E K[x],s(x)+O. Definimos la relación ~enK[x] como/(x) ~ g(x)si/(x)-g(x)=


l(x)s(x), para algt\n l(x) E K[x]; es decir, s(x) divide a/(x) - g(x). Entonces ~ es una rela-
ción de e.quivalencia en K[x].
846 Capítulo 17 Cuerpos finitos y diseños combinatorios

Demostración: La verificación de las propiedades reflexiva,. simétrica y transitiva de 9i se


dejan al lector.

Cuando ocurre la situación del teorema 17.10, decimos que/(x) es congruente con g(.t)
m6du/o s(x) y escribimos /(x) = g(x) (mod s(x)). La relación ffi se conoce como COII·
gruencia m6dulo s(x).
Analizaremos las clases de equivalencia para tal relación.

Sea s(x) = x' +x + 1 E Z,[xJ. Entonces

a) [O]= [.t'+x + 1]= {0,x'+x + .l,x'+x'+x,(x + l)(x'+x + 1), ...}


= {r(x)(x' + x + l ) lr(x) E z,{x]}
b) [l] = {l,x2 +x,x(x 2 + x + 1) + 1, (x + l)(x2 +x + 1) + 1, ...}
= {r(x )(.t' + x + 1) + Jlr(x) E Z 2(x]}
e) (x] = {x,x'+ l,x(x'+x + 1) +x,(x + l)(.t'+x+ 1) +x, .. .}
= {r(x)(x' +x + 1) + x jr(x ) E z,{x]}
d) (x + 1) = {x + l ,x',x(x1 +x + l)+ (x + 1),(x + l )(.t'+x + 1)
+ (x + 1), ...} = {r(x)(.t' + x + 1) + (x + l)lr(x) E Z,[x]}

¿Son éstas todas las clases de equivalencia? Sif(x) E Zz[x], entonces por el algoritmo
de la división.f(x) = q(x)s(x) + r(x), donde r(x) = Oo grado r(x) < grado s(x). Como /(x) -
r(x) =q(x)s(x), se sigue que/(x) = r (x) (mods(x)), por lo que/(x) E [r(x)I. En consecuen-
cia, para determinar todas las clases de equivalencia. consideramos las posibilidades para
r(x). En este caso, r(x) = O o grado ,{x) < 2, por lo que r(x) = ,Ú + b, donde a, b E Z,.
Como sólo tenemos dos opciones para a o para b, hay cuatro posibles opciones para r(.r):
O, 1,xy x + l.

Ahora damos una estructura de anillo a las clases de equivalencia del ejemplo 17.9. Si
recordamos cómo hicimos esto, en el capítulo 14, para Z.., definimos la suma como lf(x)) t
[g(x)] = lf(x) + g(x)]. Como grado(f(x) + g(x)) ,s máx(grado/(x), grado g(x)), podemos
encontrar la clase de equivalencia de lf(x) + g(x)) sin mayor problema. En este caso, por
ejemplo, (xi+ [x + I] = (x + (x + 1)] = [2x + 1] = (11, pues 2 = O en Z,.
Para definir la multiplicación de estas clases de equivalencia, tenemos una ligera difi.
cultad. Por ejemplo, ¿cuál es el valor de [x)[x) en el ejemplo 17.9? Si, en general, defini-
mos lf(x)][g(x)J = lf(x)g(x)), es posible que el grado f(x)g(x) ;,, grado s(x), así que no
podríamos encontrar fácilmente [/(x)g(x)] en la lista de clases de equivalencia. Sin embar-
go, si el grado /(x)g(z) ;,: grado s(x), entonces, usando el algoritmo de la división, pode-
mos escribir f(x)g(x) = q(x)s(x) + r(x), donde r(x) = O o grado r(x) < grado s(x). Como
f(x)g(x) =q(x)s(x) + r(x), se sigue que/(x)g(x) = r(xXmods(x)), y definimos lf(x)g(x)J =
[r(x)J, donde [r(x)] sí aparece en la lista de clases de equivalencia.
A partir de estas observaciones construimos las tablas 17.1 y 17.2 para la suma y el
producto, respectivamente, de {[O], [l], [x], {x + l]}. (En estas tablas escribimos a en vez
de (a].)
17.2 Polinomios irreducibles: Cuerpos finitos 847

~bla 17.1 tabla 17.2

+ o 1 X .r+l o 1 X .r+l
o o 1 X .r+l o o o o o
1 1 o .r+l X 1 o 1 X .r+l
X X .r+l o 1 X o X x+I 1
.r+l .r+l X 1 o .r+l o x+l 1 X

A partir de la tabla de multiplicación (Tabla 17.2), vemos que estas clases de equivalen-
cia no solamente forman un anillo, sino también un cuerpo.donde [lt' = [!], [xJ-1 = [x+ l ]
y [x + lt' = [x]. El cuerpo de orden 4 se denota con Z,[x]/(r + x + 1) y observamos que
contiene (una copia isomorfa de) el subcuerpo Z,. [En general, un subanillo (R, + , •) de un
cuerpo (K, +, •)es un subcuerpo si (R, +,-)es un cuerpo.)Además. para los elementos no
nulos de este cuerpo, vemos que [.r]' = [x], (x]'= [x+ !), [.r)' = [l], de modo que tenemos
un grupo cíclico de orden 3. Pero los elementos distintos de cero de cualquier cuerpo
forman un grupo bajo la multiplicación y cualquier grupo de orden 3 es cíclico, así que
¿para qué distraemos con esta observación? En general, los elementos no nulos de cual-
quier cuerpo finito forman un grupo cíclico bajo la multiplicación. (Una demostración de
este hecho aparece en el capítulo 12 de la referencia (10].) ·
Resumimos la construcción anterior en el siguiente teorema. En los ejercicios de la
sección aparece un bosquejo de la demostración.

lEOREMA 17.11 Sea s(.r) un polinomio no nulo en K[x].


a) Las clases de equivalencia de K[x] para la relación de congruencia módulo s(x) for-
man un anillo conmutativo con elemento unidad en las operaciones binarias cerradas

[/(.r)] + [g(x)] = [/(.r) + g(.r)], [f(.r)][g(.r)] = [/(x)g(x)] = [r(.r)],

donde r(x) es el resto que se obtiene al dividir f(x)g(x) entre s(x). Denotamos este
anillo con K [x]/(s(x)).
b) Si s(x) es irreducible en K [x], entonces K[x]/s(x) es un cuerpo.
e) SilKI = q y grado s(x) = n, entonces K[x]/s(x) contiene q"elementos.
Antes de continuar queremos enfatizar que si s(x) es irtcducible en K[x}. los elementos
del cuerpoK[.r]/s(x) no son simpÍes polinomios (enx). ¿Cómo puede ser esto, consideran-
do la presencia del sfmboloxencada uno de los elementos [x] y [x + l] del cuerpo Z,[x]/
(x' + x + 1) del ejemplo 17.9? Para aclasas este punto, consideremos un ejemplo infinito
que resulta ya conocido para nosotros.

En este caso, seaK =(R, +, -),el cuerpo de los números reales y consideremos el polinomio
r
irreducible s(x) = + 1 en R(x]. De la parte (b) del teorema 17.11 vemos que R[.r]/s(x) =
R[x]/(r + 1) es un cuerpo.
Para cualquier/(x) E R(x], obtenemos del algoritmo de división que

f{.r)=q(.rJ(x'+l)+r(.r),donde-r(x)=O o Osdegr(x)sl.
848 Capítulo 17 Cuerpos finitos y diseños combinatorios

Por lo tanto,

R(xV(.r +!)={la+ bx]la,b ER},

donde podemos mostrar que [a+ bx] = [a]+ [bx] =[a]+ [b][x].
Entre la infinidad de elementos de R[x]/(x' + 1) están los siguientes:
1) [!] = [! + l(xXx' + l)lt(x) E R[x]}, donde encontramos los elementosx' + 2 yli'
+ 3x + 1 (de R[xl);
2) [r] = {r+ t(x)(x' + l)lt(x) E R[x]J,donde r es un número real arbitrario (pero fijo~
3) [-1] = (-1 + t(x)(x' + l)lt(x) E R[x]J, donde encontramos el polinomio-!+
(IXx' + 1) = x'; así, [x][x] = [x'J =[-!];y
4) [J2x-3] = ((J2x-3) + t(xXx' + l)lt(x) E R[x]}.
Consideremos ahora el cuerpo (C.+. •)delos números complejos y la correspondeocia

h: R(xV(.r + I) ➔ e'
donde h([a + bx]) = a+ bi.
Para cualquier [a +bx], [e+ dx] E R{x]/(x' + 1), tenemos [a +bx] = [e +dx] <c>(a +bx)-
(c +dx) = t(xXx' + 1), para algún t(x) E R [x] <e> (a-e)+ (b-d)x =t(x)(x' + 1). Sit(x)ao
es el polinomio nulo, entonces tenemos (a - e) + (b - d)x, un polinomio de grado IDe8II'
que 2, igual a t(xXx' + 1), un polinomio de grado mayor o igual que 2. En consecuencia,
t(x) = O, de modo que a+ bx: e+ dx y a= e, b;;;; d. Esto garantiza que la correspoodencia
dada por h sea en realidad una función. De hecho, h es un isomorfismo de cuerpos. (V&se
el número 24 de los ejercicios que aparecen al final de la sección.) Para establecer que i
preserva la operación de multiplicación, por ejemplo, observemos que

h([a + bx][c + dx]) = h((ac + adx + bcx + bdx'])


= h((ac +(ad+ bc)x] + (bd](x2])
= h((ac +(ad+ bc)x] + (bd](-1])
= h(((ac - bd) +(ad+ bc)x])
= (ac - bd) +(ad + bc)i =(a+ bi)(c + <Ü)
= h([a + bx])h((c + dx]).
Como R[xV(x2 + l) es isomorfo a C, la correspondencia h({.t]) = i nos hace pensar ea
[x} como un número en R[xV(x2 + l)y no como un polinomiodex(enR[x]). El número[x)
representa una clase de equivalencia de polinomios en R{x] y este número (x] se comporta
como el número complejo i en el cuerpo (C, +, ·). También debemos observar que para
cualquiernúmero real r, h([r]) = ry ([r]jr E R } es un subcuerpo de R [x]/(x' + !), qu,:es
isomorfo al subcuerpo R de C.
Por último, si ahora identificamos el cuerpo R[x]/(x' + !)con el cuerpo (C , +,·),podo-
mos resumir lo ocurrido hasta e] momento como sigue: Partimos del polinomio irreducible
s(x) =x' + 1 en R(xÍ, que no tenía raíces en el cuerpo (R, +, ·).Después ampliamos(R, +, •)
a(C, +,•)y enCenconuamos laraizi(y laraíz-,)paras(x),qucpodemos factorizarabma
como (x + i")(_x - i) en C[x}.
17.2 Polinomios irreducibles.: Cuerpos finitos 849

Puesto que eo este capímJo nos interesan principalmente los cuerpos finitos, analizaremos
ahora otro ejemplo de cuerpo finito que surge gracias al teomna 17.11.

En Z,lx), el polinomios(x) =x' +x + 2es irreducible pueu(O) = 2,.t(I)= 1 y .t(2) = 2. En


consecuencia. Z3[x]/(s(x)) es un cuerpo que contiene todas las clases de equivalencia de la
forma(a.r +b], doodea.b E Z,. Éstas surgen de los restos posibles al dividir un polinomio
/(x) E Z,lx) entre,µ). Las nueve clases de equivalencia soo [O],[!], [2], [x], [x+ 1), [x+ 2),
[2x), [2x + 1) y [2x + 2).
En vet de construir toda una tabla de mu1tiplicación. examinaremos cuatro productos a
manera de ejemplo y haremos dos observaciones.
a) [2x)[x} = [2x'J = [2x' +O}= [2x' + (x' +x + 2)) = [3x' + x + 2) = [x+ 2) pues 3 = O
eo Z,.
b ) [x+ l}[x+ 2] = [x' + 3x+2} = [x'+ 2) = (x' + 2+ 2(x' +x +2)} = (2x}.
c) [2x + 2]' = [4x' + 8x + 4] = [x' + 2x + 1) = [(-x- 2) + (2x + !)], puesto que x'"
(-x - 2Xmod s(x)). En consecuencia. [2x + 2]2 = [x- l] = (x + 2).
d) Con frecuencia escnl>imos las clases de equivalencia sin ettt:betes y 005 coooentramos
en los coeficientes de las po<encia.. dex. Por ejemplo, esc¡ibimos 11 en vez de [x + 1J y
2 1 representa [2x+ 1). En consecuencia, (21) · (12)= [2x + l][x+ 2) = [2x' +5.r+ 2) =
[2x' + 2x+ 2) = [2(-x-2)+ 2x+ 2] = (-4+ 2) = [-2) =[!]por lo que (21)-1 =(12).
e) Tambiin observamos que

[x]' = [x] [r]'=[2x +2] [r]' =[2x] [x]'=[x +l]


[r]'=[2x+l] [xr=[2) [xr= [r +2] (r)'=[l]

Por lo tanto, los elementos no nulos de Z,[x]/(.t(x)) forman un grupo cíclico bajo la
multiplicación.
O Por último, cuando consideramos las clases de equivalencia [O], (1) y [2], vemos
que nos dan un subcuerpo de Z,lx]/(.t(x)), subcuerpo que identificamos con el cuer-
po ( Z~ +, •).

En el ejemplo 17.9 (ye! anilisis posterior)y en el ejemplo 17.11, construimos cuerpos


finitos de órdenes 4 (= 2') y 9(= 3'), respectivamente. Ahora cerraremos esta sección in-
vestigando otras posibilidades para el orden de un cuerpo finito. Para hacerlo necesitamos
la siguiente idea.

Sea (R. +,•) un anillo. Si existe un entero positivo mínimo n tal que nr = z (el cero de R)
para todo r E R, decimos que R tiene caractufstica n y escnl>imos car(R). Cuando no
existe tal entero. R tiene caracteristica O.

a) El anillo (Z,, +,·)tiene característica 3; (Z,, +,•)tiene caracter!stica4; en general,


(Z., +,•)tiene característica n.
b) Los anillos (Z, +,•)y (Q. +,•) tienen, ambos, característica O.
850 (.apítulo 17 Cuerpos finitos y diseños combinatorios

e) Un anillo puede ser infinito y tener caracter!stica positiva. Por ejemplo, Z,[x] es•
anillo infinito pero tiene caracterfstica 3.
d) El aníllodel ejemplo 17.9 tiene característica 2. En el ejemplo 17.11, la cara:taf,.
tica del aníllo es 3.Adíferencia de los ejemplos de la parte (a), el orden de un anilo
finito puede ser distinto de su característica
Sin embargo, los ejemplos 17.9 y 17.11 son más que simples aníllos. Son.,...
pos con característica un número primo. ¿Podría ser verdadera esta propiedad para
todos los cuerpos finitos?

TEOREMA 17.12 Sea (K, +, •) un cuerpo. Si car(K) > O, entonces car(K) debe ser primo.
Demostración: En esta demostración escribimos el elemento unidad de K como u para que
se distinga del entero positivo l. Sea car(K) ; n > O. Si n no es primo~escribimos n =-,
dondem, k E z • y l <m, k <n. Por la definición de caracterfstica.nu =z,el cerodeK. Ptr
lo tanto, (mk)u = ,. Pero

(mk)u =(u + u + · .. + u)= (u+ u+···+ u)(u +u+···+ u)= (mu)(ku).


mtsumandos

Como K es un cuerpo, (muXku) = z => (mu) = z o (ku) = ,. Supongamos, sin ~rdida de


generalidad, que ku =z. Entonces, para cualquier r E K, kr=~ur) = (ku)r=zr=z. !oque
contradice la elección den como la característica de K. En consecuenci~ car(K) es primo.

(La demostración del teorema 17.12en realidad sólo nccesitaqueK sea un dominio de
integridad.)

Si K es un cuerpo finito y m = IKI , entoncesma = z para todo a E K, pues (K,+)es UD


g,upo aditivo de orden m. (Véase el ejercicio 10 de la sección 16.3.) En consecuencia, K
tiene característica positiva y por el teorema 17.2, esta característica es prima. Esto oos
lleva al siguiente teorema.

TEOREMA 17.13 Cualquier cuerpo finito Ktiene ordenp', dondep es primo t E y z·.
Demostración: Como K es un cuerpo finito. sean car(K) =p, p primo, u el elemento unidad
y z el cero. Entonces So;;;; { u, 2u, 3u, . .. , pu =z} es un conjunto de p elementos distintos
enK. Encaso contrario,mu =nupara 1 s m <n :S p y (n-m)u =t. con O<n - m <p.Asf.
para todo x E K, tenemos que (n - m)x: (n - mXux) = [(n - m)u]x = zx = z. y esto
contradice quecar(K):p. SiK=So,entoncesj Kj = p' y se sigueelrosultado.Si no, seaa E
C -.5;,. EntoncesS, = {ma +nujO<m,n s p] es un subconjuntodeKcon IS,! S p'. Si
1S1 1<p2, entoncesm1 a+n1u=m;:a+~.conO<m1,m-i,n:1,n, s pyalmenosunadelas
cantidades m1 - '"2, n:,. - n1 =/O.Si m1 -"'2 = O, entonces (m 1 - "'2)a = z = (n.i-n1)u, con
O< jn, - n, I < p . En consecuencia, para todo x E K ,jn, - n, lx = jn,- n,l(ux) = (ln, -
n,l u)x = zx=z, con O< jn,-n,1 <p =car(K), ouacontradicci6n. Si n, -n, =O, entonces
(m1 -mi)a=zcon 0< lm1 -'"21 <p. ComoKes uncuerpoya-J:.z, sabemosquea-1 E K,
por lo quelm1-m:ilu= lm, - m2laa-1 = za-1 =z, con O< lm1 - m1 1<p, una contradicción
17.2 Polinomios irreducibles: Cuerpos finitos 851

más. Por lo tanto, ni m 1 - 171:z ni n1 - nz se anulan. Entonces, (m1 - 171:z)a - (nz - n 1)u j:. z.
Elijamosk E Z•lalqueO <k<py k(m, -m,) = 1 (modp). Entonces a= k(m, - m,)a =
Jc(,nz-n1)u, ya E So, otra contradicción. Por lotanto,!S1 1=fTy si K = S 1, el teorema queda
demostrado. Si no, continuamos el proceso con un elemento b E K -S,. Entonces Si - {fh +
ma +nulO <f. m, n !S p} tendráordenp3. (Demuéstrelo.) Como K es finito, llegamos a un
punto en el que K =S,_, para t E z· ylKI = IS,_, 1=p'.

Como resultado de este teorema. no existen cuerpos finitos con órdenes como 6, 10,
12, ... Adenras, para cada primo p y cada I E z•, existe mumente UD ünico cuerpo de
orden ¡f. Cualquier par de cuerpos finitos del mismo orden son isomorfos. Estos cuerpos
fueron descubiertos por el matemático francés Evariste Galois (1811- 1832) en su trabajo
acerca de la no existencia de fórmulas para resolver las ecuaciones polinomiales generales
de grado 2: 5 en Q. Como resultado, uo cuerpo finito de orden p' se denota como GF(p'),
donde las letras GF indican cuerpo de Galois (Ga/ois field).

CICIOS 17. 2 1. Delcrmine si cada uno de los siguientes polinomios son írreducibJes o no en los cuerpos dados.
Si es reducible, proporcione una factorización con factores irreducibles.
a) , ' + 3x - 1 sobre Q, R, C b)x' - 2 sobrc Q, R, C
e) , ' + x+ 1 sobre Z,, Z,, Z, d)x' + x' + 1 sobre z,
.r
e) + x + 1 sobre Zs f)x2 + 3.r2- x + 1 sobre Z,
2. Dé un ejemplo de un polinomioj(x) E R[x] tal que/(x) tenga grado 6, sea reducible, pero que
no tenga raíces reales.
3 . Determine todos los polinomios/(x) E Zz[x] tales que 1 ~ grado/(x) S 3 y f(x) sea irreducible
(sobre z,).
4. ~/(x) = (U+ I X5x'- Sx+ 3X4x - 3) EZ,Cx]. Escriba/(x) comoel producto de una unidad
y tres polinomios mónicos.
5. ¿Cuántos polinomios mónicos de z,{x] tienen grado S?
6. Demuestre el teorema 17.7.
7. A CQntinuación damos un bosquejo de la demostración del teorema 17.8.
a) ~ S = (s(x)f(x) + t(x)g(x)lt(x), t(x) E X(x]}. Sel=ioneun clementom(x) de grado mlni-
mo en S. (Recuerde que el polinomio nulo no tiene grado. así que ~stc no se selecciona.)
¿Podemos garantizat que m(x) sea mónico?
b ) Mues~ que si h(x ) E K[x] y h(x) divide a/(x) y a g(x), entonces h(x) divide a m(x).
e) Muestre que m(x) divide a/():). En caso conttario, use el algoritmo de la división y escriba
f(x) = q(x)n(x) + r(x), donde r(x) 'F- O y grado r(x) < grado m(x). Muestre luego que r(x) E
S y obtenga una contradicción.
d) Repita el argumento de la parte (e) para domostrar que m(:c) divide a gµ).
8 . Demuestre los teoremas 17.9 y 17.10.
9 . Use el algoritmo de Euclides para polinomios y encuentre el mcd de cada par de polinomios,
sobre el cuc,po correspondiente K. Desputs escriba el mal como s(x)f(x) + t(x)g(x), donde
t(x), t(x) E X(x].
a) f(x)=x 2 +x - 2, g(x)=x~-x•+xJ+x2 -x-l enQ[x}
b) f(x) = x' + x' + I, g(x)=x' + x+I enz,¡x]
e) f(x) == x' + 2x2 + 2x + 2, g(x) = 2x] + 2x2 +x + 1 en z,{x}
10. Si K es un cuerpo arbitrario, seanf(x),g(x) E K[x]. Si/(x) y g(.r) son primos entre sí, demues•
tre que no existe un elemento a E K tal que/{a) === O y g(a) = O.
852 Capitulo 17 Cuerpos finitos y di~ños combinatorios

11, Sean/(x),g(x) E R{x] con/(x) = x'+2x' +ax- b,g(x) = x' u'- bx+a. Determinelosvakns
de a, b de m<>do que el mcd de /(x) y g(x) se.a un polinomio de grado 2.
12. En el ejemplo 17.9, determine la clase de equivalencia que en cada caso contiene al potioomio;
a) x"+x'+x+l b) x>+.r2+1 e) x•+.r3+.r2+1
13 . A continuación damos un bosquejo de Ja demostración del teorema 17.11.
a) Demuestre que las operaciones dadas en el te.orema 17.1 l(a) están bien definidas. JDOSb'»
=
do que si JT,x) f,(xXmod s(x)) y g(x) = g,(xXmod s(x)), entonces /(x) + g(x) =/,(Je)+
=
g,(xXmod s(x)) y /(x)g(x) /,(x)g,(xXmod s(x)).
b) Verifique las propiedades de anillo para las clases de equivalencia en K[x)/(s(x)).
e) Sea/(x) E K[x], coo/(x) +o ygrado jl,x) < grado s(x). Si s(x) es irreducible en K [x), ¿por
qut impüca esto que I es el mcd de /(.x) y s(z)?
d) Use la parte (e) para demostrar que sis(x) es imducible en K[x), mon=K(xY(s(x)) es wi a,e,po.
•l SilKI = q y grado s(x) = n, determine el o<dende K[x]/(s(x)).
14. a) Muestre que S'(.%) = :r!- + 1 es reducible en Z.[x].
b) F.ncuentre las clases de equivalencia para el anillo Z:{x)/(s(x)).
e) ¿Es Z,[x]/(s(x)) un dominio de integridad?
15. Para eJ cuerpo del ejemplo 17. 11. encuentre Jo siguiente:
a) [x +2](2:c +2] +[x + ! ] b) [2:c + l ] -[x + 11[2:c + ! )
e) [2:c+lf[z+2] d) (22)-' =[2:c +2r'
16. Seas(.x)=x'+x'+IE Z,lx]. ·
a) Demuestre que s(x) es irreducible.
b) ¿CuAl es el 0<den del cuerpo 2,ix]/(s(x))?
e) Encuentre [T+x+ lJ--1 en Z.[x]/s(x). (Sug~rvrcia: encuentre a. b. e, d E Z. tales que{.i2+
x+ I] · [ax' +bx' +ex+ d] = [!].)
d) Determine [x' + x + 1J[r + 1) eo 2,ix]/(s(x)).
17. Si pes primo. sea s(x) irreducible de grado nen z,t:x].
a) ¿Cuántos elementos tiene el cuerpo Z,CxV(s(x))'?
b) ¿CUántos elementos de Z.,lx Y(s(x)) generan el grupo multiplicativo de elementos no nulos
deesteCUCf]X)?
18. Dé la característica de cada uno de los anillos siguientes:
a) z., b)Z.,[z] c)Q[x]
d) Z[45) = [a +/,✓ 5 la.b E Z J, en las operaciooaordioarias de suma y producto de números
reales.
19. En cada uno de los anillos siguientes, las operaciones de suma y produao se realizan -
a oomponmte. como en el ejercicio 22 de la sección 14.2 Detenninc la carac:tcñstica en cada~
a) Z, x Z, b) z, x z. e) z.x 4
d) Z.,xZ.,param,ne z -,m,nac 2
e) z,x z f) z x z g) Z x Q
20. Parael teorema 17.13,demuestteque JS,I =¡Y.
21 . Encuentre los órdenes n de todos los cuerpos GF(n), tales que 100 :s n :s JSO.
22. Construya un cuerpo finito de 25 elementos.
23. Construya un cuerpo finito de 27 elementos.
24. a) Demuestre que la funciónh enel ejemplo 17.IOes uno a uno y sobre y preserva la opera-
ción de suma.
b) Sean (C.+.•). (K, ES, 0) dos cuerpos. Si g: C ➔ K es un isomorfismo de anillos y a es un
elemento no nulo deC (es decir, a es una unidad de q, demuestrequeg(a-') =[g(a)J-1• (En
consecuencia, esta funcióng establece un ísomorfismo de cuerpos. F.n particuJar, la función
h del ejemplo 17.IOcs una función de este tipo.)
17.3 Cuadrados latinos 853

25. a) Sea Q (.Í2] • (a+ h'21a. b E Q). Demuestre que (Q(.Í2], +,•)es un subanillo del cuerpo
(R.+.·). C& =
caso, las operaciones binarias en ll y Q(.Í2] son las de suma y producto
ordiDarios de oCmeros reales.)
b) Demuestre que Qf..Í2] es un cuerpo y que Q(zY(.t'- 2) es isomorfo• Qf..Í2i
26. Seapunprimo.
y
a) ¡Cuántos polinomios cuadrllticos (de erado 2) mónícos r + bz + e en Z,Cz] podemos
factorizar C00 factores lineales CD z,{x)? (Por ejemplo. SÍ p: 5. CDtOnct$ d polinomio + r
2x + 2 en Z,C.r) seda uno de los polinomios cuadratico$ que debemos contar, en estas c:oo--
dicionc,.)
b) ¿Cuántos polinomios cuadráticos ai1 + bx + eco Z,(.r] podemos factorizar coo f-actores
liocales en Z,(.r]?
e) ¿o.1'ntos polinomios cuadtiticos mónicos,! + bx-+-cen 7.,J.r] soo irreducibles sobr'c Z,?
d) ¡Cuántos polinomios CWlddticos ar+ bz + e en z,¡..-J son uroducibles solxe Z,?

17.3
Cuadrados latinos

Nuestra primera aplicación del capítulo se relaciona con la estructura llamada cuadrado
latino. Tales configuraciones surgen en el estudio del diseiio combinatorio y cumplen un
papel en estadística, en el diserio de experimentos. Presentaremos esta estructura en el
siguiente ejemplo.

Una compañia petrolera esá interesada eo probar cuarro tipos de aditivos para gasolina
para determinar su efecto en el kilometraje. Para esto, un equipo de investigación diseña
un experimento en el que cuatro automóviles diferentes, denotados A, B, C y D, correo
sobre una pista determinada en un laborarorio. Cada recorrido usa la misma cantidad dada
de combusnl>le. con uno de los aclitivos presente. Para ver el efecto de cada aditivo e.o cada
tipo de auto, el equipo sigue la planificación de la tabla 17.3, donde numeramos los aditi-
vos como 1, 2, 3 y 4. Esta planificación ofrece una forma de probar con precisión cada
aditivo en cada tipo de automóvil Si un aditivo produce los mejores resultados en los
cuauo tipos. el experimento revelará su capacidad superior.
La misma corporación también está inleresada en probar cuatro aditivos desarrollados
para la limpieza del motor. Una planificación similar para estas pruebas es la que se mues-
tra en la figma 17.4. donde también denotamos los aditivos para la limpieza del motor
como 1, 2, 3 y 4.

Tabla 17.3 Tabla 17.4

Amo Lw,, .... Día


Miér• ,... Auto

A
Lw,.

1
....
2
Día
Mlér.

3
,...
4
A 1 2 3 4
B 2 1 4 3 B 3 4 1 2
e 3 4 1 2 e 4 3 2 1
o 4 3 2 1 o 2 1 4 3
854 Capítulo 17 Cu erpos finitos y d iseños combinatorios

Además, el equipo de investigación está interesado en el efecto conjunto de


tipos de aditivo. Se necesitan 16 días para probar los 16 posibles pares de aditivos (
para el mejoramiento del kilometraje. el otro para 1a limpieza del inotor) en cada
vil Si se necesitan los resultados en cuatro días, el equipo de investigación debe diseñar
programación de modo que cada par se pruebe en algún auto. Existen 16 pares
en ( 1, 2, 3, 4} X ( 1, 2, 3, 4 }, de modo que esto puede hacerse en el tiempo dado si
superponen las programaciones de las tablas 17.3 y 17.4 para obtener la programacióo
la tabla 17.5. En este caso, por ejemplo, la entrada (4,3) indica que el martes se usó d
C para probar el efecto conjunto del cuarto aditivo para el mejoramiento del kilometraje
el tercer aditivo para limpieza del motor.
Tabla 17.5

Auto Lun. ......


Día
Miér • Jue.
A (1, 1) (2, 2) (3,3) (4,4)
B (2,3) (1,4) (4,1) (3,2)
e (3,4) (4,3) (1,2) (2, !)
D (4,2) (3, 1) (2,4) (1, 3)

Lo ocurrido en este caso nos lleva a los siguientes conceptos.

Definición 17 .9 Un cuadra.de latino n x n es una tabla cuadrada de símbolos, por lo general 1, 2, 3, .. . ,l.
donde cada símbolo aparece exactamente una vez en cada fila y en cada columna de la tabla.

a) Las tablas 17.3y 17.4son ejemplosdecuadradoslatinosde 4 X 4.


b) Para n 2:: 2, podemos obtener un cuadrado latino de n X n a partir de la tabla del
grupo (Z., +) si reemplazamos las ocurrencias de O por el valor de n.

A partir de los dos cuadrados latinos del ejemplo 17.13, podríamos producir todos los
pares ordenados en S X S, para S = { 1, 2, 3, 4}. Ahora nos preguntamos si podemos o no
hacer esto para cuadrados latinos de n X n e n general.

Definición 17.10 Sean L.= (a 1), L 2 =(b•) dos cuadrados latinos de n x n, donde 1 s i,j s n y cadaair, b, E
( 1, 2, 3, . . . ,n). Si losn'paresordenados (a,. b, ), 1 ,;; i,j,;; n, son distintos, entoncesL,.L,
es un par de cuadrados latinos ortogonales.

a) No existe un par de cuadrados latinos ortogonales de 2 X 2 pues las t1nicas posibi-


lidades son

1 2 2 1
L,: 2 1 L,:
2.
17.3 Cuodrodos latinos 855

b) En el caso 3 X 3, enconttomos el par ortogonal


1 2 3 1 2 3
L,: 2 3 1 y L,:312
3 1 2 2 3 1
e) Los dos cuadrados latinos de 4 x 4 del ejemplo 17.13 fonnan UD par ortogonal. El
cuadrado latino de 4 X 4 que se muestra en la tabla 17.6 es ortogonal a cada uno de
los cuadrados latinos de ese ejemplo.
labia 17.6

1 2 3 4
4 3 2 1
2 1 4 3
3 4 1 2

Podñamos continuar enumerando algunos cuadrados latinos más grandes, pero por el
momento ya hemos visto bastante de ellos para plantearnos las siguientes preguntas:
1) ¿Cuil es el primer valor den> 2 para el que no cxiSIC un par de cuadrados latinos
ortogonales de n X n?
2) Paran> 1, ¿qué podemos decir acerca del nllmero de cuadrados latinos de n X n
que podemos construir de modo que cada par de ellos sea ortogonal?
3) ¿Existe UD método para ayudamos a construir un par de cuadrados latinos ortogooalcs
de n X n para algunos valores de n > 21
Para de poder analizar estas preguntas, nccesiwnos una forma estándar para nuestros
resultados.

Dlfinid6n 1 7 . 11 Sil es un cuadrado latino den x n, eotoocesLestáeo suforma. estándar-si su primera ítla
esl23 .. . n.

Excep<o por el cuMndo latinol, del ejemplo 17.l S(a), todos los cuadrados latinos que
hemos visto en la sección están en su forma estándar. Si un cuadrado latino no está. en su
forma estándar. es posible ponerlo en esa forma intercambiando algunos de los símbolos.

El cuadrado latino de 5 x 5 que apa,ece en (a) no está en forma csdndar. Sin embar¡o, si
reemplazamos cada ocwrencia de 4 con l, cada ocurrencia de 5 con 4, y cada ocurrencia de
1 con 5, entoocesel resultado es el cuadrado latino (cstándar)de 5 X 5 que se muestra en (b).
42351 12345
13542 5 3412
3 4 2 1 5 3 2 S 4
2 S 3 4 2 4 5 3 1
5 4 2 3 4 5 1 2 3
(a) (b)
856 Capítulo 17 Cuerpos finitos diseños combinatorios

Con frecuencia es conveniente trabajar con cuadrados latinos en forma estándar.¿


taña esto de algún modo nuestros resultados acerca de pares ortogonales?

TEOREMA 17.14 SeaL,, L, un par ortogonal de cuadrados latinos de n x n. Si L,, L, se estandarizan


I:. Lt entonces~. L: son ortogonales.
Demostroción: Se deja al lector.

Necesitamos est.as ideas para )os resultados principales de la sección.

TEOREMA 1 7. 15 Sin E z•, n > 2, entonces e l máximo número posible de cuadrados latinos de n X •
son ortogonales dos a dos es n - 1.
Demostraci6n: Sean Li, L,_, • .• , Lt k cuadrados latinos distintos den X n que está
forma estándar y son onogonales dos a dos. Escribimos a!..>para denotar el elemento ea
i-isima. fila y j-ésima columna de L., donde 1 :S i, j :S n, 1 :S m s k. Puesto que eSIEII
cuadrados latinos están en forma estándar, tenemos que af¡-> = l , af;> =2, . .. , y af: 1:
para todo l S m :s Je. Consideremos ahora '4¡> para todo ·1 s m :S /c.. Estos elemc:olol
la segunda fila y la primera columna están debajo de af¡> = l. Asf, '4¡>i= 1, para todo 1 $
m :s k, o la configuración no es un cuadrado latino. Además, si existe 1 :s f < m :S k tal q111
afl l = att>, entonces c:I par Li. L. no puede ser ortogonal (¿Por qu6 no?) En consecucm:ia.
hay cuando mucbon - 1 opciones para los elementos ~ 1 de cualquiera de nuestros cuaik>
dos latinos de n x n y el resultado se sigue de esta observación.

Este teorema impone una cota superior sobre el número de cuadrados latinos den X •
que son ortogonales dos a dos. Veremos que para algunos valores den. podemos alcannr
esta cota.Además. nuestro siguiente teorema proporciona un método para la construcciá
de estos cuadrados latinos. aunque inicialmente no est6n en forma estándar. La c::onsno,
ción usa la estructura de un cuerpo finito. Antes de demostrar este teorema para el caso
general. examinaremos un caso particular.

SeaK= (f,¡ 1 sis 5} = Z,, con/,= l ,f,=2,/, = 3,f.=4 yf, = 5, el cero de Z,_
Para Is k s 4,sca41amatriz5 x 5 (~"), donde I s i, j s 5y

•l? =J.J,+J;.
Sik= l , construimosl1 = (a!I)) como sigue. En este caso, a f =/./; +/¡ ={; + Jí para Is
i, j :s; 5. Si i = 1. calculamos la primer fila de Li como sigue:
•H' =t,+J, =2 •W=f ,+f,=3 •W=J,+f , =4
• i!l = J, + J, = 5 •W=f, + !, = 1
calculamos los elementos de la segunda fila de L 1 cuando i = 2. Tenemos entonces

aí.P =f, +f, =3 a'fJ.' =f,+f,=4 a!¡}=f,+f, = 5


a'r} =f, +f.=1 a'fi=f , +f, =2
17.3 Cuadrados latinos 857

Seguimos con nuestros cálculos para obtener el cuadrado latino L,, como

2 4 S
3 4 5
4 S 2 3
s 2 3 4
2 3 4 S
=
Para k = 2, los elementos de L 2 están dados por la fórmula aill JJ¡ + Íi =2J; +h- Para
obtener la primera fila de L 2, hacemos i = 1 y calculamos

aW=2Jí + f,=3 afi=2f, + f,=4 a'?}=2f,+f,=S


af.l=2f,+f, =! af}=2f, + f, = 2
Si hacemos i igual a 2, calculamos los elementos de la segunda fila de L 2 como sigue:
af.'=2f,+f, = S afJ=2f,+f,=1 afJ=2!,+f,=2
af} =2f,+f.=3 afJ=2f,+f,=4
Usamos cálculos análogos para i = 3, 4 y 5. los cuales producen el siguiente cuadrado
latinoL 2:

3 4 5 2
1 2 4
2 3 4 S
4 S 1 2
2 3 4
Podemos verificar directamente que los dos cuadrados latinos Li y L 2 son onogonales.
En el ejercicio 5 (al final de esta sección) pediremos al lector que calcule L, y r... Nuestro
siguiente resultado verifica que las cuatro tablas Li, L 2, L, y L. son cuadrados latinos
ortogonales dos a dos.

IEOREMA 17.16 Sean E Z', n > 2. Si pes primo y n = p', para t E z-, entonces existen n - 1 cuadrados .
latinos de n x n. onogonales dos a dos.
Demostracóón: Sea K = GF(p'), el cuerpo de Galois de orden p' = n, Consideremos K = (/i,
h, .. . ./.} donde/1 es el elemento unidad y f. es el cero.
Construimos n - 1 cuadrados latinos como sigue.
Para cualquier 1 s k s n-1.scaLa la tablan X n (ab.1;)), 1 s i.j s n. t.alque a'(> =.hh+/¡.
Primero mostramos que cada 4 es un cuadrado latino. En caso contrario, existen dos
elementos idfoticos de K en la misma fila o columna de 4 Supongamos que la repetición
aparece en una columna; es decir, a!¡> = a!f>, para 1 :s: ,; s s n. Entonces a~t = ftf, +ÍJ =
f,f. +.f; = a;f'. Esto implica que/,/,=l,fn por la cancelación para la suma en K. Como k l,n,
se sigue queft-1= J., el cero de K. En consecuencia.A es invertible, de modo que/,=f, y r = s.
Un argumento similar muestra que no existen repeticiones en cualquier fila de 4 (Deja-
mos esta demostración como ejercicio.)
858 capítulo 17 Cuerpos finitos y diseños combinatorios

En este momento. tenemos n- 1 cuadrados latinos, Li, L,,.• •• . , L.,_1. Ahora demostta-
remos que son ortogonales dos a dos. En caso contrario, sea l s k < m :S n - 1 tal que
lS.i,j, r,ss n, (i,j) "f (r,s).
(Entonces, el mismo par ordenado aparece dos veces si superponemos Lk y L.-) Pero

a!?=a~~"'>f,f;+f¡=t.f,+f,, Y
a!;" =al;l,;,,f.f; + Í; =f.f, + f,.
Al restar estas ecuaciones, vemos que (1,-f,Jf; = (/,-f,Jf, Comok /am., (1, - f,J noesd
cero deK, por lo que es invenible y tenemos/¡ = f,. Si sustituimos esto de nuevo en cual-
quiera de las ecuaciones anteriores, vemos quejj :/r En consecuenci~ i = ry j=s, Pork,
tanto, para t I= m, los cuadrados latinos 4 y 1....,. forman un par onogonal.

El primer valor de n que no es potencia de un primo es 6. La existencia de un par de


cuadrados latinos ortogonales de 6 X 6 fue analizada primero por Leonhard Euler ( 1700-
1783) cuando buscaba una solución para el "problema de los seis oficiales". Este probl~
ma trata de seis regimientos diferentes; de cada regimiento se seleccionan seis oficiales.
cada uno con un rango distinto. (Hay solamente seis rangos posibles.) El objetivo es onJe.
nar a los 36 oficiales en una tabla de 6 X 6 de modo que en cada fila o columna de la tabla,
cada rango y cada regimiento estén representados exactamente una vez. Por lo tanto. cada
oficial. en la tabla cuadrada corresponde a un par ordenado (i. 1). donde 1 s i, j :s 6 e i
representa el regimiento y j el rango. En 1782, Euler conjeturó que el problema no podía
resolverse. que no exiSte un par de cuadrados latinos onogonales de 6 X 6. Él fue un poco
IIlll allá y afirmó que para todo n E z•, si n = 2 (mod 4). entonces no existe un par de
cuadrados latinos onogonalcs de 6 x 6. En 1900, G. Tarry verificó la conjetura de Euler
mediante una enumeracíón sistemática de todos los cuadrados latinos posibles de 6 X 6.
Sin embargo, en 1960, gracias al esfuerzo combinado de R.C. Bose, S.S. Sbrikbande y E.
T. Parker. se demostró que el resto de la conjetura de EuJer era falsa; este traba.jo mostró
que sin E z+.n = 2(mod4)yn>6,entoncesexisteunpardecuadrados latinosonogooales
den X n.
Para más detalles ~rea de este resultado y los cuadrados latinos en general, el lector
deberá consultar la bibliografía del capítulo.

EJERCICIOS 17.3 1. a ) Escriba nuevamente el siguiente cuadrado latino de 4 X 4 en forma estándar.

1 4 2
3 2
2 4 3
4 2
b ) Encuentre un cuadrado latino de 4 x 4 en forma estándar que sea ortogonal al resultado de
la pane (a).
e) Aplique el proceso inverso eo la parte (a) al resultado de la pane (b). Muestre que su
respuesta es onogonal al cuadrado latino dado de 4 x 4.
2. Demuestre el teorema 17.14.
3. C.Omplcte la demostración de la primera parte del teorema 17.16.
17.4 Geometrías finitas y planos afines 859

4. Los tres cuadrados latinos de 4 x 4 en las labias 173, 17.4 y 17.6 son onogonales dos a dos.
¿Podría encontrar otro cuadrado latino de 4 x 4 que sea ortogonal a estos tres?
S. Complete los cálculos del ejemplo 17.17 para obtener los dos cuadrados latinos de S X 5 L, y
L,. Escriba de nuevo, en forma estándar, cada cuadrado latino L;, para t .S i ~ 4.
6. Encuentre tres cuadrados latinos de 7 X 7 que sean ortogonales dos a dos. Vuelva a escribir
estos resultados en forma estándar.
7. Amplíe el experimento del ejemplo 17.13 de modo que el equipo de investigación necesite eres
cuadrados latinos de 4 x 4 que sean ortogonales dos a dos.
8. Un cuadrado latino Les auwortogonal si L y su traspuesta L• forman un par on.ogonal.
a) Muestre que no existe un cuadrado latino autoortogon.al de 3 x 3.
b) Dt un ejemplo de un cuadrado latino de 4 x 4 que sea autoortogonal.
e) Si L = (~) es un cuadrado latino autoonogonal de n x n, demuestre que todos los clcinen-
tos a;;. para 1 .s i .s n , deben ser distintos.

17.4
Geometrías finitas
y planos afines
En la geometría euclídea del plano real, vemos que (a) dos puntos distintos determinan
una única recta y (b) si f es una recta en el plano, y P es un punto que no está en f, entonces
existe una única recta f' que contiene a P y es paralela a e. Durante los siglos xvm y XIX se
desarrollaron las geometrías no euclídeas que estudiaban las alternativas a la condición
(b). No obstante, estas geometrías contenían u.na infinidad de puntos y rectas. El concepto
de una geometría finita sólo apareció al final del siglo XIX en la obra de GinoFano (Giomale
di Matemariche, 1892).
¿Cómo construir una geometría de este tipo? Para esto, regresamos a la ya conocida
geometría euclídea Para describir los puntos y las rectas de este plano en forma algebraica,
introducimos un conjunto de ejes de coordenadas e identificamos cada punto P me.diante
un par ordenado (c,d) de números reales. Esta descripción establece una correspondencia
u.no a uno entre los puntos del plano y el conjunto R x R. Si usamos la idea de pendiente,
podemos representar cada recta en este plano de forma única como (1) x = a, donde Ja
pendiente es infinita. o (2)y =mx + b, donde mes la pendiente; a, m y b son números reales
arbitrarios. También vemos que dos rectas distintas son paraJelas si y sólo si tienen la
misma pendiente. Cuando sus pendientes son distintas. las rectas se inte~an en un único
punto.
=
En vez de utilizar los números reales a. b, e, d, m para el punto (e, d) y las rectas x a.
y= mx + b, pasamos a una estructurafinita comparable con ésta, el cuerpo finito. Nuestro
objetivo es construir lo que llamaremos un plano afín (finito).

)efinición 17.12 Sea~ un conjunto finito de puntos y sea':! un conjunto de subconjuntos de ti', llamados
rectas. Un plan.o afín (finito) sobre los conjuntos~ y':! es una estructura finita que satis-
face las siguientes condicíones.

Al) Dos puntos distintos de~ están (al mismo tiempo) en un único elemento de~; es
decir, están en una única recta.
860 Capítulo 17 Cuerpos finítos y diseños combinatorios

A2) Para cualquier I E :t, y cualquier PE~ con P 1/. t existe un único elemento( E
;/ tal que.P E f' y e, f' no tienen puntos en comán.
A3) Existen cuatro puntos en~. donde tres cualesquiera de ellos no son colineales.

La raz.ón de la condición (A3) es evitar situaciones poco interesantes como la que se


muestra en la figura 17.1. Si sólo tenemos en cuenta las condiciones (Al) y(A2).entonces
este sistema seña un plano afín.

Figura 17.1

Regresemos ahora a nuestra constIUCCión. Sea K = GF(n), donde n = p' para algúa
primo p y t E z•. Para construir nuestro plano afín, que denotamos con PA(K), sea~;;;
{(e, d)lc, d E K},Asl, tenemosn' puntos.
¿Cuántas rectas debemos tener para el conjunto ;l?
Las rectas están en dos categorías. Una recta de pendiente infinita tiene la ccuación.x =
a, donde a E K.Así, tenemosnde estas «rectas verticales"'. Las otras rectas están dadas cu
forma algebraica como y = nu + b, donde m. b E K. Con n opciones param y n parab,se
sigue que tenemos ,r1 que no son "verticales". Por lo tanto,l<:I.I = n2 + n.
Antes de verificar que Jll(K), coo ~ y :t según se construyeron, es un plano afio,
haremos dos observaciones más.
En primer lugar, para cada recia e E '5t, si I está dada porx =a, entonces hay n opciones
parayen 1 = {(a, y)ly E KJ.Asl,!cootieneexactamenten puntos. Si lestádadapory =mr+
b, con m. b E K, entonces para cada elección de x tenemos determinadoy de manera única
y de nuevo t tiene n puntos.
Consideremos ahora cualquier punto (e, d) E f1>. Este punto está en la recta x = c.
Adeffiti, en cada recta y = mx + b de pendiente finita m., d - me determina b de manera
única Con n opciones para m, vemos que el punto (e, d) está en las n rectas de la forma y
= mx + (d- me). En total, (e, d) está en n + 1 rectas.
Hasta este punto de la construcción de Rt(K), tenemos un conjunto 11 de puntos y un
conjunto:! de rectas talque (a)l~I =n'; (b)l:ll =n' +n;(c)cadal E:! contienen puntos;
y (d) cada punto en ,i' está en exactamente n + 1 rectas. Ahora demostraremos que PA(K)
satisface las tres condiciones para ser un plano afin.
Al) Sean (e, d), (e,f) E ,i'. Usarnos la fónnula de dos puntos para la ecuación de una
recta y obtenemos

(e - c)(y -d) = (f-d)(x - e) (!)

como una recta en la que están (c. d) y (e,/). Cada uno de estos puntos está sobre
n + l rectas. ¿Podría haber una segunda recta que los contuviera?
El punto (e, d) está sobre la rectax= c. Si (e,f) también está sobre esa recu,
entonces e= e, perof#=d, puesto que los puntos son distintos. Sie: e, la ecuación
( 1) se reduce a O= (f-dXx-c), ox = e pues/-d*O, de modo que no obtenemos
una segunda recta.
17.4 Geometrías finitas y planos afines 861

Si e f e y (e, d), (e, f) están en una segunda recta de la forma y= mx + b,


entonces d= me+ b,f= me+ by (/-d)= m(e- e). Nuestros coeficientes son
elementos de un cuerpo y e ,. e, de modo que m = <f-d)(e - et' y b = d - me=
d-(f-dXe-ct'- En consecuencia, esta segunda recta que contiene a (e, d) y (e,
f) es
y= (f-d)(e-cf' x + (d-(f -d)(e -c)-1 c)
o bien, como la multiplicación cnK es conmutativa, (e-cXy-d) = (/-d) • (x-
c), que es la ecuación (1).Asf, dos puntosde9'están solamente en una recta y se
satisface la condición (Al).
A2) Para verificar esta condición, consideremos el punto P y la recta f que se mues•
tran en la figura 17.2. Puesto que existen n puntos en cualquier recta. sean P11
P,, ... , P, los puntos de t (µtos son los Wlicos puntos de 1, aunque la figura
podóa sugerir nw.) El punto P no está en t de modo que P y P; determinan una
única recta f¡, para cada 1 :S i s n. Antes mostramos que cada punto está sobre
n + 1 rectas, por lo que ahora existe una recta adicional e• con P sobre e' y e' no
interseca a t

Figura 17.2

A3) La última condición utiliza el cuerpo K. ComolK I 2: 2, en K están el elemento


unidad l y el elemento cero O. Considetemoslos puntos (O, 0),(1, O), (O,!), (1, l);
si la recta 1contiene tres cualquiera de estos puntos, entonces dos de ellos tienen
la forma (e, e), (e, d ). En consecuencia, la ecuación de eestá dada por x = e, que
no es satisfecha por (d, e) o (d, d). Por Jo tanto, tres cualesquiera de estos puntos
no son colineales.
Hemos mostrado lo siguiente.

TEOREMA 17.17 Si K es un cuerpo finito, entonces el sistema basado en el conjunto ft de puntos y el


conjunto :i de rectas descritos arriba es un plano afín que se denota con Jl4.(K).

Mediante algunos ejemplos particulares veremos una conexión entre estas geometrías
fin.itas, o planos afines, y los cuadrados latinos de la sección anterior.

ParaK= (Z,, +, •), tenemosn = IKI = 2 . El plano afín de la figura 17.3 tienen'=4 puntos y
n' + n = 6 n,,;tas. Por ejemplo, la m:ta t. = {(!,O), (1, 1)) y e, no contiene otros puntos
862 Capítulo 17 Cuerpos finitos ydiseños combinatorios

Figura 17.3

como podría sugerir la figura Además, E5 y f6 son rectas paralelas en esta geometría finita.
pues no se intersecan.

SeaK = GF(2') el cuerpo del ejemplo 17.9. Recordemos la notación del ejemplo 17.ll(d)
y escnoamos K= {00, 01, 10, ll}, con la suma y el producto dados en la tabla 17.7.
Usamos este cuerpo para construir una geometría finita con n1 = 16 puntos y n1 + n = 20
rectas. Podemos separar las 20 rectas en cinco clases paralelas de cuatro rectas cada una.

Tabla 17.7

+ 00 01 10 11 00 O! 10 11
00 00 01 10 11 00 00 00 00 00
01 01 00 11 10 01 00 01 10 11
10 10 11 00 01 10 00 10 11 01
11 11 10 01 00 11 00 11 01 10

Clase 1: Aquí están las rectas de pendiente infinita. Estas cuatro rectas "verticaJcs'"
están dadas por las ecuacionesx =00, x = 01, x = 10 y x= 11.
Cwe 2: Para la clase "horizontal", o clase de pendiente O, tenemos las cuatro rectas y=
00,y =01,y= 10 y y= 11.
O.ase 3: Las rectas con pendiente 01 son aquellas cuyas ecuaciones son y= 01.x + 00, y=
0lx+0l ,y =0lx+ lOy y=Olx+ 11.
Clase 4: Esta clase consta de las rectas con ecuaciones y = lOx+ 00, y = 10.x + 01,y-=
IClx+!Oyy=!Ox+ll.
Clase S: La última clase contiene a las cuatro rectas dadas por y = llx + 00,y = 11.r+
01, y = llx+ lO yy = llx+ 11.

Puesto que cada recta en PA(K ) contiene cuatro puntos y cada clase paralela contiene
cuatro rectas, veremos ahora la forma en que tres de tales clases paralelas dividen los 16
puntos de l¾(K).
Para la clase con m = 01, tenemos cuatro rectas: (1) y= Olx + 00; (2) y= Olx + 01; (3)
y= Olx + 10 y (4) y = Olx + ll. Sobre cada punto de PA(K) escribimos el número corres-
17.4 Geometrías finrtas y planos afines 863

~ X ✓
(00,11) (01,11) (10,11) (11,11)

V": ~
(00,10) (01,10) (10, 10) (11,10)

/\:
(00,01) (0 1,01) (10,01) (11,01)

~
(00,00) (01,00)
~
(10,00) (11,00)
/ '
Figura 17.4

diente a la recta sobre la que seencuentta (v&se la Fig. 17.4). Podemos dar esta configu-
ración por medio del siguiente cuadrado latino:

4 3 2 1
3 4 1 2
2 1 4 3
2 3 4

Si repetimos este proceso para las clases 4 y 5. obtenemos las particiones que se mues-
tran en las figuras17.5 y 17.6. respectivamente. En cada clase se enumeran las rectas, de
la pendiente dada, en eJ mismo orden que la figura 17.4. Dentro de cada figura está el
cuadrado latino correspondiente.
Estas figuras producen cuadrados latinos de 4 x 4 que son ortogonales dos a dos.

. • .
4
(00,11)
2
(01, 11)
1
( 10,11)
~
( 11, 11)

. . .
3
(00,10)
1
(01,10)
!
( 10.10)
4
( 11.10)

. . . .
2
(00.01)
4
(01,01)
3
( 10.01)
1
( 11,01}
4 2 1 3

.1
(00.00)
~
(01.00)
. .
4
( 10.00)
2
( 11.00)
3 1 2
2 4 3
1 3 4 Fig willl 17.5
864 capítulo 17 Cuerpos finitos y diseños combinatorios

. .
4
(00.11)
1
(01,11)
;
(10.11)
!
(11,11)

;
(00,10)
!
(01,10)
. .
4

(10,10)
1
(11,10}

. . . .
2
(00.01)
3
(01 .01)
1
(10.01)
4
(11 .01)
3 2

. . .
1
(00.00)
4
(01 .00)
2
(10.00)
;
(11 .00)
4
1
1 4 2
1
4
3 Figura 17.6

Los resultados de este ejemplo no se deben a1 azar, como lo muestra el siguiente teorema.

TEOREMA 17. 18 = =
Sea K GF(n), donde n 2: 3 y n p', p primo. Los cuadrados latinos que surgen de R!(K)
para las n - 1 clases paralelas, donde la pendiente no es Q ni infinito, son onogonales dos
a dos.
Demostración: Bosquejamos una demostración de este resultado en los ejercicios de la
sección.

EJERCICIOS 17.4 1. Complete la siguiente tabla que trata de los planos afines.

Número de
Número de rectas por UD
Cuerpo Número de puntos Númerodertttas puntos en una rttta punto
25

GF(3')

56

17

31

2. ¿Cuántas clases paralelas determina cada uno de los planos afines del ejercicio J'! ¿Cuántas
rectas hay en cada clase?
3. Construya el plano affn PA(~). Detennine sus clases paralelas y los cuadrados latinos corres.-
pondientes a las clases con pendiente finita distinta de cero.
4. Repita el ejercicio 3 con Zs en vez de ~ -
S. Determine cada una de las siguientes rectas.
a) La recta en R4.(Zi,) paralela a y= 4x + 2 y que contiene a (3, 6).
b) La recta en /M(Z11 ) paralela a 2x + 3y+ 4 = O y que contiene a (10. 7).
e) La recta e n ffl(K). donde K = GF(2~. paralela a IOy = 11.r+ 01 y que contiene a ( 11. 01).
(Véase la Tabla 17.7.)
17.5 Diseliosdebloqueyplanospr 865

6. Suponga que intentamos construir un plano afín PA.(ZJ como lo hicimos en esta sccción.
a) Determine adles de las coodiciooes (A 1), (A2) y (A3) fallan CD esta situación.
b) EocuCDm: el nt!men> de rectas que cootiCDCD un punto dado P y el nt!men> de puntos que
esWl en una recta dada f. co esta ..gcometr(a...
7. A continuación damos un bosquejo de la demosuación del teorema J7 .18.
a) Consideremos una clase paralela de rectas, dada por y = nu- + b, donde m E K, m -;:. o.
Muestre que cada recta en esta clase i.n.teneca a cada recta --vcrtical.. y a cada recta ..bori_
zootal" CD ouc:tamCDtc un punto de /l\(K). As!, la coofi¡uracióo obtenida al etiquetar los
puntos de/l\(K) como CD las figuras 17.4, 17.5 y 17.6 <$Un cuadrado !aúno.
b) Para mosuar que los cuadrados latinos correspoodienres a dos clases diferentes (djstintas
de las clases de pendiente nula o infinita) son onogonales, supongamos que existe un par
ordenado (i. 1) que apatt.ee más de una vei cuando un cuadrado se su.perpcme sobre otro.
¿Cómo obtenemos de esto una contradicción?

17.5
Diseños de bloque
y planos proyectivos

En esta 11ltima sección examinaremos un tipo de diseño combinatorio y veremos su rela-


ción con la estructura de una geometría finita. El siguiente ejemplo ilustra ese diseño.

• Ei;¡;¡:.:11::..::~:.:.J David (d) y ,uesposa María (m) viajan a Nueva York con '"' cinco hlj0$: Ricardo (r),
Pedro (p), Cristóbal(<), Bealriz (b) y Julia(}). Durante su estancia de una semana reciben
tres pases cada día. para visitar el Empi~ State Buiúling. ¿Podemos hacer un programa
para es1.a familia de modo que todos visiten esta atracción el mismo nWDCfo de veces?
El si¡uientc programa es una p0$ibilidad.

1) b,c, d 2) b,j, r 3) b,m,p 4) c.j,m


S) c, p, r 6) d,j, p 7) d, m,r

Aquf obtuvimos el resultado por ensayo y error. técnica que podemos utilizar para un
problema de este wnaño. Sin embargo, en gcoeral se necesita una estrategia más eficaz..
Adenw, al pedir cieno programa, podñamO$ estar pidiendo algo que no existe. Por ejem-
plo. en este problema. cada par de miembros de la familia están juntos llnicamcntc en una
visita. Si la familia recibe cuatro pases cada día, no podríamos construir un programa que
conservara esta propiedad.

Generalizaremos la situación del ejemplo del siguiente modo.

Definición 17.13 Sea Vun conjunto con u elementos. Uoacolección {8 1, B:, . . .• B.J de subconjuntos de V
es un diseíw de b/qque incompkto equilibrado, o diseíw (1', b, r. k, A.), si satisface las
siguientes condiciones:
866 Capítulo t 7 Cuerpos finitos y diseños combinatorios

a) Para cada 1 :S: i :S b, el subconjunto B1 tiene k elementos, donde k. es una con:stme


fijayk<U.
b) Cada elemento x de V está en r( :S b) de los subconjuntos B;, 1 :S i ::S b.
e) Cada par x, yde elementos de V aparecen juntos en A( :S b)de los subconjuntos S.
1,; i,; b.

Los elementos de V suelen denominarse variedades, debido a que sus primeras aplD
ciones en el diseño de experimentos fueron en pruebas de fcrtiliz.antes y plantas. Los b
subconjuntos B,. B,, . .. , B, de V son los bloques, donde cada bloque contiene k variedo,
des. El número r es el número ~ réplica del diseño. Por último, A. es la covalmcia dd.
diseño. Este par.imetto equilibra el diseño en el siguiente sentido. Para los diseios de
bloque generales, tenemos un número ),,, para cada par x, y E V; s.i ),,, es el mismo pm
todos los pares de elementos de V, entonces A representa esta medida común y el diselD
está equilibrado. En este texto trabajaremos solamente con diseños equilibrados.

a) El programa del ejemplo 17.20es un ejemplo de un_diseño (7, 7, 3, 3, 1).


b) Para V= {I, 2, 3, 4, 5, 6}, los diez bloques

1 2 4 134 156 236 346


1 26 135 235 245 456

son un diseño (6, 10, 5, 3, 2).


e) Si K es un cuerpo finito ylKI =n, entonces el plano afín lll(K) produce un diseio
(n', n' + n, n + 1, n, 1). En este caso, las variedades son los n' puntos de PA(K); las
Ir + n re.etas son los bloques del diseño.

En este momento tenemos cinco parámetros que determinan nuestro diseño. Ahora
veremos la relación entre estos parámetros.

TEOREMA 17.19 = =
Para un diseño (u, b, r. k, 1..), (1) ur bk y (2) A(U - 1) r(k- 1).
Demostraci6n:

1) Coo b bloques en el diseño y k elementos por bloque, enumeramos todos los ele-
mentos de los bloques y obtenemos bk símbolos. Esta colección de símbolos coasta
de los elementos de V donde cada elemento aparece r veces, para un total de u,
símbolos. Por lo tanto, ur = bk.
2) Para esta propiedad. usamos la matriz A de incidencia dos a dbs de este dise6o.
Sil V I =u, sear=(J) el Dlllllerode pares de elementos de V. Construimos lamalrizr
X bA = (a,) dada por a, = 1 si el i-t simo par de elementos de V está en elj-és.imo
bloque del diseño; en caso contrario, a1 =O.
17.5 Diseños de bloque y planos proyectivos 867

B, B, B,
x,x,
ª" ª" ª"
X 1X¡
ª" a,, a,.

... .,__,,.., ...


x,x. 0.-11
X:z_X¡
a..-1t,

x.-,x. a., a,, ª·•

Ahora contaremos el número de unos en la matriz A de dos formas.

a) Consideremos las filas. Puesto que cada par X;, X¡, para 1 ::s i < j ::S \'I aparece en A
bloques. se sígue que cada fila contiene A unos. Con i filas en la matriz, el número
de unos es entonces Al; Av(u - 1)/2.
b) Consideremos ahora las columnas. Como cada blcxiue tiene k elementos, esto deter-
mina m;:; k(,k- 1')12 pares y éste es el número de unos en cada columna de la matriz
A. Con b columnas, el número total de unos es b~k - 1)/2.
= = =
Entonces, 1..1>(,i - 1)/2 b~k- 1)/2 ur(k- 1)/2, de modo que A(u - I ) r(k - 1).

Como ya mencionamos antes, si n es una potencia de un primo. podemos obtener un


diseño (n', n' + n, n + 1, n, 1) del plano affn M(K), donde K = GF(n). En este caso, los
puntos son las variedades y las rectas son los bloque s. Ahora presentaremos una construc-
ción que amplía Ri(K) a lo que se llama un plano proyectivo finito. A partir de este plano
proyectivo podemos construir un diseño (rr2 + n + l, ,il + n + l. n + l, n + l , 1). Primero
veamos cómo se comparan estos dos tipos de planos.

Definición 17.14 Sift' es un conjunto finito de puntos y:i' es un conjunto de rectas. cada una de las cuales
es un subconjunto no vacío de~•. entonces el plano (finito) con base en !t' y :t• es un
planc proyectivo si satisface las siguientes condiciones:

Pl) Dos puntos distintos en~• están en una sola recta


P2) Cualesquiera dos rectas de ;i' se intersecan en un único punto.
P3) Existen cuatro puntos de~• tales que tres cualesquiera de ellos no son colineaJes.

La diferencia entre los planos afín y proyectivo radica en la condición que trata de la
existencia de rectas paralelas. En este caso, las rectas paralelas del plano afín basado en 3>
y ;i se intersecan cuando el sistema se amplía al plano proyectivo basado en ft' y':/'.
La construcción es la siguiente.
868 Capítulo 17 Cuerpos finrtos y diseños combinatorios

Comenzamos con un plano afín PA(K), donde K = GF(n). Para cada punto (x, y) E !J,
escribimos nuevamente el punto como(x,y. 1). Entonces pensamos los puntos como temas
ordenadas (x, y, z), donde z = l. Escribimos de nuevo las ecuaciones de las rectas x =e y y=
mx+ben PA(K) comox=cz y y = mx +In, donde z= l. Seguimos con nuestro plano a&
original lll(K), pero con un cambio de notación.
Añadimos el conjunto de puntos ((1, O, O)} U ((x, 1, O) lx E KJ a ft para obtener el
conjuntoft'. Entooces,lft'J = n' + n + l. Sea f. el subconjunto deft' que consta de estos
nuevos puntos. Esta nueva recta puede darse mediante la ecuación z = O. con la coodicióa
de que nuoca x = y = z = O. Por lo tanto, (0, O, 0) ~ ft'.
Analicemos ahora estas ideas para el plano affnPA(z,J. En este caso,ft = {(O, O), (1, 0),
(O, 1), (1, l)}, de modo que
ft' = {(O, O, 1), (1, O, 1), (O, 1, 1), (1, 1, 1)} U {(1, O, O), (O, 1, O), (1, 1, O)}.
Las seis rectas en :l eran. en un principio,
x =O: ((0,0),(0,1)} y =O: ((0,0),(1,0)} y = r. {(0,0),(1, 1)}
X= 1: {(1,0),(1,1)} y= 1: {(O, 1), (1, 1)} y =x + 1: {(O, 1),(1,0)}
Volvemos a escribir esto como
x=O y=O y=x x=z y=z y=x+z
y añadimos una nueva recta e_ definida como z : O. Éstas constituyen el conjunto ':t' de
rectas de nuestro plano proyectivo. A partir de este momento consideramos a z como
variable. En consecuencia, larectax=zconstade los puntos (O, 1, O), (1, O, 1) y (1, 1, 1).
De hecho, cada recta de se que contenía dos puntos contendrá ahora im; puntos al ser
considerada en :l'. El conjunto ':l' consta de las siguientes siete rectas:

x = O: {(O, O, 1), (O, 1, O), (O, 1, 1)} y = z: ((1, O, O), (O, 1, 1), (1, 1, 1)}
y = O: {(O, O, 1), (1, O, O), (1, O, 1)} y= r. {(O, O, 1), (1, 1, O), (1, 1, l)}
x = z: {(O, 1, O), (1,0, 1), (1, 1, 1)} y= x + z: {(O, 1, 1), (1, 1,0), (1, o, 1)}
z =O(€.): {(1, 0,0),(0, 1, O), (1, 1,0)}

(0,0,1)

Figura 17.7
17 .5 Diseños de bloque y planos proyectivos 869

En el plano afín original, las rectas x = O y x = 1 eran paralelas. pues ningún punto del
plano podía satisfacer estas ecuaciones en forma simultánea. En este nuevo sistema.. :r = O
y x :;:;: z se intersecan en el punto (O, l , O), así que ya no son paralelas e n el sentido de
Jl4.(7.,). De la misma forma. y = x y y =x + 1 eran paralelas en Jl4.(7.,), mientras que ahora
las rectas y = :r y y = :r + z se intersecan en (O, 1, O). Ilustramos este plano proyectivo
basado e n:J'' y:i' en la figura 17.7. En este caso.el "círculo" que pasapor(l, O. 1). (1, 1,0)
y (O. 1, 1) es la recta y :;::;: x + z. Observe que cada recta interseca a e..
que se llama con
frecuenci:i la ~eta del in.finito. Esta recta está formada por tres puntos del infinito. Definimos
dos rectas como paralelas en el plano proyectivo si se intersecan en un punto del infinito (o
en e.).
Este plano proyectivo nos proporciona un diseño (7, 7, 3, 3, 1) como el desarrollado
por ensayo y error en el ejemplo 17.20.

Generalizamos los resultados del ejemplo 17.22 como sigue: Sean una potencia de un
primo. El plano afín PA(K). para K = GF(n). nos proporciona un ejemplo de un diseño
(n2, n2 +n,n + l , n, 1). En M (K), lasn2 +n rectas caen enn + 1 clases paralelas. Para cada
clase paralela. añadimos un punto del infinito a Jl4.(K). Añadimos el punto (O, 1, O) a la
clase de rectas x = cz, e E K y el punto (1, O, O) para la clase de rectas y = bz. b E K.
Cuando m E K y m :/:- O, e ntonces añadimos el punto (m""1, 1, O) para la clase de rectas y= m.--c
+ bz. b E K. La recta del infinito l. es entonces el conjunto den + 1 puntos del infinito. De
esca forma obtenemos el plano proyectivo sobre GF(n), que tiene rt2 + n + 1 puntos y tr +
n + 1 rectas. En este caso, cada punto e stá sobren+ 1 rectas y cada recta contiene n + 1
puntos. Además, dos puntos cualesquiera de este plano están en una sola recta. En conse-
cuencia, tenemos un ejemplo de diseño (n2 + n + 1, n2 + n + 1, n + 1, n + 1, 1).

EJERCICIOS 17.5 1. Sea V = {l. 2. . .. , 9}. Detmninc los valores de u, b, r, ky l. para el diseño dado por los
siguientes bloques.

12 6 1 47 2 3 4 2 7 9 3 78 468
135 189 2 5 8 369 4 59 5 6 7
2. F.ll.cuentre un ejemplo de diseño (4, 4, 3, 3, l ).
3. Encuentreunejemplodedisefio {7, 7,4,4,l).
4. Complete la tabla siguiente de modo que los parámetros u, b, r; k. A de cualquier fila sean
posibles para un diseño de bloque incompleto equilibrado.

u b r k l.
4 3 2

9 12 3

10 9 2

13 4 4

30 10 3
870 Capítulo 17 Cue<pos finitos y diseños combinatorios

5. ¿Es posible tener un disello (u, b, r, l, l.) tal que (a) b = 28, r = 4. k = 3?
(b) U= 17, r = 8, k = S?
6. Dado un diseño (u. b, r. Je. l) con b = u, demuestre que si u es par, entonces les par.
7. Un disefto (u,b,,; k. l) es unsisuma. triple si k= 3. Sil= 3 y l = 1, decimos que el ·
un sisuma. tripk ~ Stdner.
a) Demuestre que en todo sistema triple de Steiner, l(u- 1) es par y lu(u - l) es di ·
entre 6.
b ) Demuestre que en todo sistema triple de Steincr, ues congruente coo 1 o 3 módalo6.
8 . Verifique que los siguientes bloques forman un sistema triple de Stcincr de nueve
des.

128 14 7 234 279 389 4 6 8


1 35 1 69 25 6 36 7 459 51 8
9. En un sistema triple de Steiner con b =- 12. encuentre los valores de u y r .
10. En los siguientes ejercicios,~• es un conjunto de puntos y ':f.' un oonjunto de rectas, cada
de los cuales es un subconjunto no vacío de~'. ¿Cuáles de las condiciooes (PI), (P2) y (P3)
la definición 17.14 se cumplen para los conjuntos~• y ':l' dados?
a) ~• •{a,b,c}.
~: ={{a,b),{a,c},{b,q).
b) ~•={(x,y, z) lx,y,zER}= R'.
~ • es el conjunto de todas las re.etas en R3•
e) !f• es el conjunto de todas las rectas en R J que pasan por (0, O. O).
~• es el conjunto de todos los planos en R 3 que pasan por (0, O, O).
11. De un grupo de IS estudiantes de primer aiio de bachillerato, se fomwi equi¡,os de bolos caa
cinco estudiant.es cada uno. Cada uno de los estudiantes está en el mismo ntimero de equipos r
cada par de estn<liantes están juntos en dos equipos. (a) ¿Cu.lntos equipos hay en total? (b) ¿Ea
cuintos equipos diferentes está cada estudiante?
12. La sedorita Martínez dio a su grupo de ciencias de la computación una lista de 28 problemas e
indicó a cada estudiante que escribiera los algoritmos para resolver exactamente siete de eslOS
problemas. Si cada estudiante hizo to indicado y si para cada par de problemas había exacta-
mente un par de estudiantes que escribieron el algoriuno para resolverlos. ¿cuántos esrudiantes
tiene la señorita Maitfnez en su grupo?
13. Consideremos un disello (U, b, r, k. l.) sobre el conjunto V de varicdooes, donde I VI = u " 2. Si
x. J E V,¿Cll.mtos bloques del diseñocontieoenaxo ay?
14. En una clase de programación. la profesora Magda tiene un total de n estudiantes y desea
asignar equipos de m estudiantes a p proyectos de computación. Si cada estudiante debe ser
asignado al mismo mlmero de proyectos. (a) ¿en cuántos proyectos participarác a d a ~
te? (b) ¿En cuántos proyectos participará cada par de estudiantes?
15. a ) Si un plano proyectivo tiene seis rectas que pasan por cada punto, ¿cuántos puntos tiene el
plano proyectivo en total?
b ) Si hay 57 puntos en un plano proyectivo, ¿cuántos puntos esbin en cada recta del plano?
16. Al constnrir el plano proyectivo a partir de Bt(Zv en el ejemplo 17.22. ¿por q~ no incluimos
el punto (0. O, O) enel conjunto~'?
17. Determine los valores de u, b, ,; k y A para el diseño de bloques incompleto equilibrado asocl•
do al plano proyectivo que swge de M(K) para las siguientes opciones de X : (a) Z-,, (b) Z, y
(<)GF(8).
18. a) Enumere los puntos y rectasde ~ (Zi). ¿Cuántas clases paralelas tiene esta geometría fini-
ta? ¿Cuáles son los parámetros del diseik> de bloques incompleto equilibrado asociado?
17.6 Resumen y repaso histórico 871

b) Enumere los puntos y rectas del plano proyectivo que surge de ma;,. Determine los pun•
tos en L. y úselos para determinar las clases "paralelas" de esta geomctrla. ¿Cuáles son los
panimetros del disefto de bloques incompleto equilibrado asociado?

17.6
Resumen y repaso histórico

En el capítulo 14 habíamos desarrollado la estructura de cuerpo. En este capítulo analiza-


mos los anillos de polinomios y su papel en la estructura de los cuerpos finitos, fijando la
ateoc:ión en las aplicaciones a las geometrías finitas y los diseños combinatorios.
En el capítulo 15 vimos que el orden de un álgebra booleana finita sólo podría ser una
potencia de 2.Ahora vimos que el orden de un cuerpo finito sólo podía ser una potencia de un
primo y que paracadapprimoycadan E Z♦.sóloexiste un cuerpo, salvo isomoñimos. de
orden P". Denotamos este cuerpo con GF(pn), en honor del matemático francés Evariste
Galois (1811- 1832).
En el capítulo 14 obtuvimos los cuerpos finitos (Z,. +, •) parap primo, por medio de la
relación de equivalencia (congruencía módulon) definida en Z. Con estos cuerpos finitos.
desarrollamos aquí los dominios enteros Z,[x]. Luego. si s(x) e s un polinomio irreducible
de gradon en Z,Cx], una relación de equivalencia similar (la congruencia módulos(x)) nos
dio un conjunto de P" clases de equivalencia, que denotamos con Z,[x]/(s(x)). Estas P"
clases de equivalencia so~ los elementos del cuerpo GF(p"). (Aunque no demostramos los
resultados posibles en gene~ se puede demostrar que sobre cualquier cuerpo finito Z,.
existe un polinomio irreducible de grado n para cada n E z+.)
La teoría de cuerpos finitos fue desarrollada por Galois en su obra acerca del problema
de las soluciones de ecuaciones polinomiales. Como .m encionamos en el resumen del ca-
pítulo 16, el estudio de las ecuaciones polinomiales fue un área de investigación matemá-
tica que representó un reto para muchos matemáticos de los siglos XVI al XIX. En el siglo

Evariste Galois (1811-1832)


872 Capítulo 17 Cuerpos finitos y diseños combinatorios

XIX, Niels HenrikAbel (1802- 1829) mostró por primera vez que la solución de la CCUaciÓII
general de grado 5 no podía dme por radicales. Galois mostró que para cualquier polinomio
de grado n sobre un cuerpo K, existe un grupo correspondiente G que es isomorfo a ua
subgrupo de s.. el grupo de permutaciones de { l. 2, 3, ...• n}. La esencia de la obra de
Galois es que tal ecuación polinomial se puede resolver mediante (suma. resta, multiplicar
ción, división y) radicales si su grupo correspondiente es ~solub~. Pero ¿qué hace a UI
grupo finito resoluble? Decimos que un grupo finito es resoluble si tiene una cadena de
subgrupos G = K1 ::, K2 ::, K 3 :> ... :J K, = {~}. tales que para cada 2 s j s t, K; es UI
subgrupo normal de K;. 1 (es decir, .l)'Z"1 E K; para cada y E K; y cada x E K, _1) y IK;. ,1 e
1K;I es primo. Se puede ver: que todos los subgrupos de S;, para 1 s i s 4, son resolubles,
s.
pero para n ;?:: 5 existen subgrupos de que no lo son.
Aunque parece que la teoría de Galois trata principalmente de los grupos, no hemos
mencionado aún lo relativo a la teoría de cuerpos. Como consecuencia del trabajo de
Galois, las áreas de la teoña de cuerpos y la teoria de grupos finitos se convirtieron en
temas de gran intel'ts matemático.
Para más detalles acerca de la teoría de Ga/Qis, un buen lugar para comenzar es el
capítulo 6 del texto de V. Larney (8] y el capítulo 12 del libro de N. H. McCoy y T. Be,¡er
[10]. El capítulo 5 de L N. He rsteio (6] tiene oras sobre el tema y una presentación detalla-
da aparece en el texto de O. Zarislá y P. Samuel [16]. El apéndice E del texto de V. Larney
{8] incluye un interesante relato de la vida de Galois; el lector puede revisar más sol"re su
vida en e l relato un tanto ficticio de L. lnfeld (7]. El artículo de T. Rothman (11] proporcio-
na un análisis contemporáneo de las imprecisiones y mitos que rodean la vida, y en espe•
cial la muerte, de Galois. Las notas biográficas en las páginas 287-291 del texto de
J. Stillwell [13] relatan más acerca de la vida y obra de este gran genio.
Los cuadrados latinos, los diseños combinatorios y las geometrías finitas de las últimas
secciones del capítulo nos mostraron cómo interviene la estructura de cuerpo finito en los
problemas de diseño. El estudio de los cuadrados latinos ortogonales data de la q,oca de
Leonhard Euler (1707-1783) y el problema de los 36 oficiales .ha sido desarrollado en
forma considerable desde 1900; particularmente, desde 1960, con la obra de R. C. Bose, S.
S. Sbrikbande y E. T. Parker. El capítulo 7 de la monografía de H. Ryser (12] ofrece los
detalles de sus logros. El texto de C. L. Liu [9] incluye ideas de la teoría de la codificacióo
en su análisis de los cuadrados latinos.
El eswdio de las geometrías finitas puede rastrearse hasta la obra de Gino Fano, quien.
en 1892, consideró una geometría finita tridimensional con 15 puntos, 35 rectas y 15
planos. Sin embargo, estas geometrías sólo adquirieron importancia en 1906, cuando
O. Veblen y W. Bussey comenzaron su estudio de las geometrías proyectivas finitas. Para
más detalles sobre este tema. el lector observará que los textos de A. Albert y R. Sandler
( I] y H. Dorwan [4] son interesantes. El texto de P. Dombowslá (3] proporciona un amplio
tratamiento del tema para quien busque un estudio más avanzado.
Por último, el concepto de diseño fue estudiado en un principio por los estadísticos, en
el área llamada diseño de experimentos. A través de la investiga::•i6n de R A. Fishcr y sus
seguidores, esta área ha llegado a tener un papel i.mponante en la teoría moderna del
análisis estadístico. En nuestro desarrollo analiz.amos las condiciones en las cuales podría
existir un diseño (u, b, ,; k. A) y la forma en que estos diseños se relacionan con los planos
afines y los planos proyectivos finitos. El texto de M. Hall, Jr. (5] presenta más detalles
sobre el tema, al igual que la obra de A. Street y W. Wallis [14]. El capfculo XIII de la
referencia [14] incluye un material relativo a los diseños y la teoría de la codificación. En
la obra de W. Wallis [15] aparece una reseña muy completa del tema de los diseños, mientras
873

que el texto edilado por J. H. Dinitz y D. R. Stioson [2] omce al lectoc una recopilación de
los trabajos más recientes en el área.

BIBLIOGRAFÍA

l . Albcrt,A.Adriany R. Saodler,An lntrodJIC.ti.on. toFin.iu ~ctive Planu, Nueva York. Holt.


1968.
2. Dúúiz, le!frey H. y Douglu R. Stinson. edi10r<s. Conwnporc,y Duign TMory, Nueva Yodt.
Wtley, 1992.
3. Dombows.kJ, Peter, Finüt GtonutriLs, Nueva York, Springer•Verlag. 1968.
4. Dorwan, Harold L, 7M ~ oflncid<nce, Englewood Oilfs, NJ.• Preotice.Hall, 1966.
5. Hall, Marsball. JL, Combinatoria/ TMory, Waltbam. Mass.• Bwsdell. 1967.
6. Herstein, Israel Natban, Topics inA/gehra, 2" ed.., Lcxington. Mass.• Xerox Collqe Publisbing,
1975.
7. lnfeld, Leopokl, Whom di, Gods Lave, Nueva YO<k. M<Graw-Hill, 1948.
8. Lamey, Viole< H.,AbstroctAlgebra.cA FfrstCaw-se, Boston.Prindle, Wcbe<&Sclmúdt, 1975.
9. Li.u, C. L.,Topics in Combinau;,rial Mazlaemarics, MathematiealAssociation ofAmeric:a..1972.
1O. McCoy, Neal H. yThomas R. Berger, Algtbra; Groups, Rings and Olher Topics, Boston. Allyn
and Bacon, 1977.
11. Rodlman. Tooy. "'Genius and Biograpbers: Tbc Fictionaliz.atioo of Evariste Galois'". Tlze
Am,ri<,v, Modwnancal MONhly 89, m!m. 2. 1982, págs. 84-106.
12. Ryser. Herben J., CombiMu)ria/ MaJ/wnali<s, Caros Mathematical Mooographs, nOmero 14,
Malbcmalical Association of America, 1963.
13. Stillwell, Jobo, MaJMmali<s and lts History, Nueva Yodt, Springcr-Yerla¡, 1989.
14. Sl=1, Annc Ponfold y W. O. Wallis, CombiMu)ria/ TMory, An /ntrod,,ction, WIDDipeg, Ca-
nadi, 1be Charles Babbaac RCSC3>Ch Ccnter. 1977.
l 5. Wallis, W. D., Combin<uorial Duigns., Nueva York, M=l Detker, !ne., 1988.
16. Zarisld, Osear, y Pierre Samucl, Commwarrll Algtbra. vol l. Nueva York. Van Nostrand,
1958.

EJERCICIOS b) Encuentre las r>lces nociooales. si existen, de los


,íguíeates polinomios sobre Q. Faaorice /(z) en
COMPLEMENTARIOS
Q(z].

O /(z) = 2s' +3z' -2z - 3


1. Determine n si sobre GF(n) hay 6561 polinomios
m6nicos de grado 5 sin rtrminos COOSWlleS. Ü)/(z)=.r• + z>-,rl-2x - 2

2. a) Sea/(.x)=a.,z"+ · · · +a,x+ .. e Z{z]. SirlsE W) /(z)~Jz'-2.r-St'-4.r-4


Q, con. mcd(,; J)'"' 1 y /(r/s) • O, dcmucwe que e) MUC$tro que el polinomio/(.x) = z"" -r"' + z'° + x'
sia.y , ¡.._ + 1 no tiene raíces racionales.
874 Capítulo 17 Cuerpos finitos y diseños combinatorios

3. a) ¿Para cuántos enteros n tales que t :s n :s 1000, 12. Un plano proyectivo tiene como coordenadas los de-
podemos factorizar /(z) = x:2 + x - n como el pro- mcntos de un cuerpo K. Si esta recta contiene 91 recm,
ducto de dos factores de primer grado en Z[z]? ¿cuál es d Vlllo,- de JKJy car(K)?
b) Responda la parte (a) para/(x) =r' + 2r- n.
13. Sean V= {.x1,.x2••••• :e:.} d conjunto de variedades y
e) Responda la parte (a) para/(x) = r' + Sx - n.
{Bi, Bi. . ..• B. } la colección de bloques para un discfto(,a.
d) Seagt<)=r'+Jo:-nEZ{x],paral :Sn:S 1000.
b, r. .I; A). Definimos lamatri4 d, incidmciaA parad dise-
F.ocuentre el entero positivo mínimo l: tal que g(x)
ño como
no pueda factorizarse mediante dos factores de pri•
mer grado en Z{xl para todo 1 :s ns 1000.
4. Verifique que el polinomio /(x) = ,:' + ,:l + x + 1 es A= (a,)..-, 'donde a . =
11
{1,O, si .X; EB¡
en caso coouano.
.
reducible sobre cualquier cuerpo K (finito o infinito).
5. Si p es primo, demuestre que en Z,[x]. a) ¿CUántos unos hay en cada fila y columna de A?
x' -x ~
....
IJ (x -a).

6. ParacualquiercuerpoK,sca/(.r)-=.r' + a,,_.,r- - 1 + . .. +
b ) Sea J••• la matriz m x n tal que cada elemento a
l. En vez de J.x.a, escribimos J.. Demuestre que
para la matriz de incidcnciaA,A · J•= r y J. ·A=i·
J,,.,.
a¡.t" + '1(J E K[x}. Sir., rb . ..• , . son las raíces 0/!/(x) y r1 E K e) Muestre que
para todo 1 :s i :s n, demuestre que
a) -a,.-1=r1+ r2+· · ·+r,,.
b) (-trao= r1r2···r,..
7. Sea/(;<) E R!x]. Sía +bi E C yf(a+b,)= O,demues-
trcque/(a+b,)=/(a-bi)=O. (Las propiedades que obser-
vamos en el ejercicio complementario 4(a) del capítulo 14
podñan servir en este caso.)
,,..¡¡ ).
,
).

).
l.
).

).
...
...
...
...
... '.]
8. Sea R un anillo conmutativo con elemento unidad. ¿F.o •(r-,)1.+).J~
quf condiciones es verdadera cada una de las proposiciones
siguientes? donde l .. es la identidad (multiplicativa) u x u.
a) Para todos a. b E R, (a+ b)' = a'+ Ir. d) Demuestre que det(A ·A')=
b) Paratodosa.bER,(a+b"}' = a 3 +l,,!. (r- A)►'[r+ (u- 1)).J = (r - 1.)..,rk.

9. Cuatro delos siete bloques en un diseño (7. 7, 3, 3, 1) 14. Dadoundiseño(u, b. r, Je. A) basado en las u varieda-
son{! , 3, 7}, {!, 5, 6}, {2, 6, 7) y {3, 4, 6). Detennine los des de V, reemplazamos cada bloque B;,, para 1 :s i s b, por
otros tres bloques. su complemento B¡ = V - B,. Entonces, la colección {J..
10. Encuentrn los valores dcb y rpara un sistema triple de
Bi, . .. , B.} proporciona los bloques para un disdio (u, b,
r', K, A'), tambi&l basado en e l conjunto V.
Stciner tal que u= 63.
a) Enaienueestcdiseilo (u,b, r,
k', A')complcmm-
11. a) Si un plano proyectivo tiene 73 puntos. ¿cuántos tario COITCSpondiente al dise:i'io dado en el ejen::i--
puntos están sobre cada recta? cio 1 de la sección 17.S.
b ) Si cada recta de un plano proyectivo pasa por 10 b ) & general, ¿cómo se relacionan los parámetros r.
puntos, ¿cuántas rectas hay en este plano? k', ,.. del diseño comple=tario y los parameaos
u. b, r. .t; A del diseño original?
Apéndice 1
Funciones
exponenciales
y logarítmicas

E n el estudio de las matenwicas y de la ciencia de la computación. nos eocootramo5 con las


funciooes cxpooencialcs y Iogarfmlicas. Ya presentamOS el concepto de funcido en la sección S.2.
y en la parte (d)del ejercicio 15 de esa sección encontramos la funcióo/: R ➔ R• taJ que/(x) =e'
par.u e R. &to es un ejemplo de una funcióo expoooncial. I>es¡,u&. en el ejemplo 5.61, n:laciooa-
mos esta función con una función lopritmica. denotada In .:r, dcolc .x e R•. Más adelante, ea el
ejemplo 5.73 del mismo capítulo, 002 funcióo logu(lmica (a saber. Iog,,,. pan n e Z•) aparoció en
el análisis de un algoritmo. Como estos tipos de ecuaciones aparecieron en otros capítulos, propor-
cionamos ahora este aptndice para repasar algunas de las propiedades fundamcotales de estos dos
tipos de funciones.

Comencemos con la idea de los exponentes enteros positivos. Por ejemplo, sabemos que la
expresión 37 indica la multiplicación de siete trcscs; es decir,

3'= 3 . 3 · 3 · 3 · 3 · 3 · 3 = 2187.

En este ejemplo. el nllmero 3 es la base de 37; el mlmero 7 es el upon.tnu o ¡x,una.a. En general,


cuando el exponente es un entero positivo, la base (que llamaremos b) puede ser cualquier mí.mero
real (incluido O). Al trabajar coo un expooeote dado por un entero negativo. usamos La si¡uicnte
dcfmicióo.

Dl'finición A1 .1 Para cualqwer n'1mcro real no nulo by cualquier n e z-, tenemos Ir'= llb".

~ A1.1 i OeladefimcióoAl.l ienemosque

•> 3-• • 1/3' • 1/2187


b) (1/2¡-' = 1/(1/2)' • 1/(1/64) • 64
e) (-3/5)-' • 1/(-315)' • 1/(- 243/3125) • - 3125/243

A-1
A-2 Apéndice 1 Funciones exponenciales y logarítmicas

Por llltimo, cuando e1 exponente es el entero o. definimos b°= l, para cualquier número real no
nwo b.t
Podemos resumir las ideas anteriores de la manera siguiente, donde usamos la idea de definición
recursiva (que presentamos en la sección 2 del capítulo 4) en la primera parte:

Para cualquier b E R,

1) b1 =byb'=b ·b ..1 paran EZ'talquen> 1;


2) Si H O y n E z·. entonces b~ = 1111'; y
3) Si b -J: O. entooces bº= l.

Para pasar de los exponentes enteros a los exponentes dados por números racionales, r-ccorde-
mos, de nuestra experiencia en álgebra. que si q E z•, donde q > l y bes cualquier nl.1mero real oo
negativo, entonces la expresión blf.f denota la q-ésima raíz de 6. Por lo tanto, b-U. es el nwnero real
a tal que fil= b. Por ejemplo,

32"'= 2 pues 2'= 32, y (118)"'= 112 pues (112)' = 1/8.

Pero cuando nos enfrentamos a las ecuaciones 22= 4 y (-2)2;, 4, debemos prcgun.tam0s lo que
entendemos por 4 112• La convención establecida llama raíz positiva a la representada por 4112, de
modo que 4 112 = 2, no-2 o ± 2 De la misma forma. 911.! = 3, 16112= 4 y para rualquier r E R. (r2)112:=
Jr J, el vaio,- absoluto de r, no la propia r. Además, aunque 2'= (-2)'= (2i)'= (-2i)'= 16, cuando
encontramos la expresión }61.M, &ta denota la raíz cuarta positiva. es decir, 2.
Si b es un mlmero real negativo y q es un entero positivo impar. nuestra definición anterior de Ir"
sigue teniendo sentido. Encontramos, por ejemplo. que (-8)"' = - 2, pues (-2)'= -8 y ni.ng,ln otro
cubo de un mlmero mJl produce -8. Sin embargo, para el ca.so en que q. =2, la expresión (- 4)w:
deno<a un nllmero complejo que no es real; evitaI-emos este tipo de situaciones aquf.
Por último, sin entrar en un análisis detallado en el desarrollo de los mhneros irracionales, coin-
cidircmos en que Jos numeros reales (aunque irracionales) oomo 2in = .fi y(- 5)1-''= !Ff, existen
y, en general, para q E z· y r E R , tambiin existen los siguientes números reales:

,v.= 11r. para ,2:0 ,.Uq = 1tr,para r< Oy q impar.

Ahora que hemos planteado este tema de los exponentes (o potencias) de la forma 1/q, donde q
es un entero positivo mayor o igual que 1, pasamos a la siguiente definición.

Definición A1 .2 Seab e R y sean p. q e z·. Entonces

1) b-''=(bv')',para baO;
2) b-,,, • (b"')-• ~ 1/((bv')"], para b> O;
3) b"•=(bv")', para b<O yqimpar: y
4) b-,,, = (bv')-• = 1/((bv')'], para b <O y q impar.

Ilustramos esca definición en el siguiente ejemplo.

t LaHptt:Sión <Pes unafqnna ind~t~rmmadtJ pues su Vllloc podría ser distinto en diferentes siruaciooes..
Esta idea se estudia en d ~ o . en d contexto de la regla de L'H8pital.
Apéndice 1 Funoones exponenciales y logañtmicas A-3

•> (8)"'•8"'•(8"')"•2'• 4(•64"'•(8'}"')


bl (8W'"• (81"')-'=r' • 1/J'= 1/Tl(• (3"')'•[(81v'¡-'J'= [(8W"')')
e) (- 1/32)"' • [(-1/32)"']' • (-1/2)'- -1/8
d) (-102A)· "' • [(-102A)"'¡'• (-4)-' • l/(-4)'= l/16(• 1/(-102A}"')

El óltimo resultado que aparece en la parte (a) del ejemplo anterior indica lo siguiente, lo cual es
CÍClto CD geoeraJ.:

Las ouas panes de la definición Al .2 wnbitn pueden ampliar como

b _..,. • W'l"' • (1/b')"" • (1/b"'), b > O, p, q E z•.


b"<• (b'J"", b <O, p,qez·. q im¡,a<
b-"·- (b-')""• (lfb')"" • (1/b"'). b <O, p ,q e z·. q impar.

Usando 2 como base, de las ddinicioocs A 1.1 y A 1.2 tenemos que

y que

Z-"' =(2"')-' • (VÍ¡-' • (I/V2)' • l/(2V2) • 0.3535534


2"' • (V2)' • 2V2 =2.8284271 ( - (2')"' • v'ii~
Sin embargo, ¿cómo trabajar con expresione:J como 2.fi, donde nos enfrentamos ahora a una
potencia irraciooal? Usando el hecho de que .fi • 1.7320508 ... , podemos evaluar las siguicn1c$
po<cncias raciooalcs:

21 a2
2'·' ( • 2.,,,. • (2")"" • 131072"") • 3.2490096
2L7J • 3.3lms:2
2'·"' • 3.3218801
2Ln>O ,¡, 3.3218801
2L""" "' 3,3219952

Con la ayuda de una calc:uladorao un c:omputadoc. vemos que 2¡;, b.asla siete cifras decimales.
esti dado poc 3.3219971. Si queremos ,cr mú J)ffi:ÍSO$. podemos decir que el número rw 2 6 es el
llmilede la sucesióa 21, 21·', 2 1•71, 2 1-132, 2 1-mo, 21-l'IIIS, • •. (Estas ideas se estudian en el dk:ulo y en
la introduc::ción al wl.isis.)
De maneta similar tratamos la expresjóo b'. donde b e R· y r e R.

Usamos los resultados vistos huta ahora acerca de los exponentes para establecer las siguicole$
propiedades, sin demostrarlas.
A-4 Apéndice 1 Funciones exponenciales y logaritmicas

TEOREMA A1.1 Las propUdades tk los e.xp<,Mntu. Para a.b E R• y i, y E R,

1) (b')(b") = b' · b' = b'•',


2) (b')/(b') =b"/b' =b' -',
3) (b')' = b~ =b'' = (b"f, y
4) (abf = (a')(b") = a'·b".

Ilustramos las propiedades en el teorema Al.l como sigue.

1) 3Yi. 3:t12= 3'(Y.l)1--(J/2)) = 3112 =3' =81


2¡ (7"')/(7'"') = -¡1,.,,,_,,.,,,,
=r""' = r' = 1/7' = 1/49
3) ((V2)'f = (V2)' =(2"'}' =2'""" =2' = 8
4l (3\/5)' =3'(VS)' = (81)(25) = 2025

Hemos terminado con los antecedentes necesarios para definir una función exponencial.

Definición A1 .3 Para un número real positivo fijob. lafunción/: R ➔ R•dada por/(x) =Eres lajunci6n uponmcial
tk bo.u b. [A veces denotamos Ir con exp.{x).]

- ·· a) En la figura A l. 1 aparecen las gráfü:as de cuatro funciones:

/1: R - R .. , f 1(x)=x 2 /,: R-,R+, /,{_x)= Z-


/,: R-,R, /,(x) =x' /.; R-, R•, /,(x)=3'

Las funciones/, y fj son funciones polinomiaJes. ne funciones exponenciales. Por lo tanto,


cuando analizamos L1s funciones exponenciales.Ji y f. observamos que existe una diferencia
entre las exprcsiones..r'(para/1) y 2' (para/,) y entre lasexprcsionesi1 (para/;) y 3x (paraf.).
Las funciones exponenciales Ji y/, son tales que
1) f,<x) >0 y f.(x) > O, para todox E R; en panicular,f,(x) > J y J.(x) > 1 para todo x> O,
mientras que O</i.x) < 1 yO </ix) < l, para todox< O.
2) para cualesquiera,; y E R,x <y ⇒ f-1.x) < f,<y) ( y f.(x) <f.( y)). (Esto es cierto para
cualquier función exponencial tal que la base b > 1. Es decir, si b > l y .t < y, entonces
b'< b'.)
3) si u ,w E R y /f.,u) = /f..w), entonces u= w. [Esta propiedad tambiái es cierta para
cualquier función cxponencial/(,x) = 11' con b > l. Así, para u, w E R y b > l. tenemos
que b• = b"'⇒ u== w.J
b ) La gráfica de la función/,: R ➔ R♦• dadapor-/,(x) = (l/2Y• 2""'. aparece en la figuraAl.2.
Esta gráfica muestra las siguientes propiedades, que son ciertas para todas las funciones
exponenciales/: R ➔ R• tales que/(x) = Ir para O< b < 1.
1) En este caso,J,(x) > Opara todo.t E R, pero ahora vemos que/,(.t) > I parax< Oy
/,(x) < l six>O.
Apéndice 1 Funciones exponenciales y logarítmicas A-5

1,(,1¡

-3-2-1 1 2 3 -2
(1,) (1,)

1,(,1¡ l,(x)

28 (3. 27)

-3-2- 1 1 2 3
(1,) (IJ

FiguraAl .1

-4 -2
(1,) Figura A1.2

2) Six;y E R conx<y, entoncesJ;(x) > J;(y).


3) Paru.y E R , si/,(z) =/,(y). en1oncesx= y.
e) Si lw>lamos de /a función expooencial. nos .eferimos a la función/: R ➔ R• tal que/(;<)=
e' para el D1lmero imciooal e=
2.71828. Esta función aparea: como J. en la figura Al.3,
doode usamos las aproximaciones e' = 7.38906 y e' = 20.08554. La función/, ( - en
la Fig. Al.3) es la función exponencial talque/,(x) =r.
A-6 Apéndice 1 Funciones exponenciales y logarítmicas

f,.(x) f,(x)

•' •'

•' •'
(O. 1) (1, e) (- 1. e) (O. 1)

- 3- 2- 1 1. 2 3 4 - 4 -3-2 - 1 1 2 3
( fJ ( f,)

Figura A1.3

Aprendimos, de la prop;edad (3) en las panes (a) y (b) del ejemploAl.5, que pan cualquierb E
R· y x,. y E R. si b i- I y ~ = bY entonces x = y. Esta observación oos ayuda a resolver la ecuacióa
exponencial.

¿Para qui números reales n se aimple que (In~ = (118)"-<'°" •ffl?


Podemosescncires,a «uación como~= S"º""" pues (112)""' = ((112)"']'"' = -r-'y ( 118)"<'º""'=
[(J/8/1Jl'°'•-M = guO.•-M. F.ntonccs

~ = gc:u:.t..-•YJ=> '16"'2 = (l3)(10.-•)ll::;,,'J!"'2 = z(10.H) =>


6n' = !On +4~3n's5n +2~
3n' -5n -2 = (3n + l)(n -2) = O~n = - 1/3 or n =2.

Ahora que hemos anali7.ado la función exponencial, dedicaremos nuestra atención a un segundo
tipo de función que va de la mano con la función exponencial. &ta es e l logaritmo o funcióa
Jogarftmica. Sin embargo. antes de presentar esta función. daremos un repaso a las propiedades
fundamentales de los logaritmos. Primero consideremos la ~!ación precisa entre los exponentes y
los logaritmos, según la siguiente definición.

Definición A1.4 Seab un número real positivo fijo distinto de l . SixE R...,escribimos lot..,xparaindicareJ/Qgarimw
<U xaz base b (o el logaritmo en bascbdcx), que es el (único) nO:mero real y que satisfaceb'=.x.
Podemos volver a fonnular esta idea como sigue: log.x- es el exponente (o potencia) a la qoc:
debemos elevar la base b para obtCDCí .r. Por lo tanto,

y= log.xsi y sólo si .r= b".

11::iM!í:i Obtenemos los siguientes resultados debido a la definición anterior:


=
a) Como 2J 8 , tenemos que lo&l 8 =3.
b) Encontramos que log,(1/81)= - 4, pues 3"'= 1/(3') = 1/81.
Apéndice 1 Funciones exponenciales y logarítmicas A-7

e) Para cualquier b E R., tal que b # l, se sigue que


i) log.b = l pues b' = b,
ü) log.1>'=2 puesb' = b'.y
ül) log.(1/b) = - 1 pues bº = lib.
d) Como = 7"' = .fi. tonemos que log, .fi = 1n.

Supongamos qucb, .x E R..,quebestá fijo yes distinto de l. Si log..x=6,¿cuflesel valor de l~.:t?


Sabemos que log..x=6 <=> h'-= .x, de modo que.x = {ir)l. Y .:t= {b7)J<=> log.:;x= 3. {De manera
análoga, vemos que lo~.x = 2 y Jog,,,,x= l.)

Junto con las propiedades (1), (2) y (3) de los exponentes. que apa=en en el teorema Al.!, las
siguientes propiedades pertenecen a los logaritmos.

IEOREMAA1 ,2 Sean b, r, s e R·. tales que bes fijo y distinto de l. Entonces

1) log, (n) = log.r + log.s,


2) log, ( r/s) = log, r - log,s, y
3) log, (,') = s log, r.

DemostrK'6n: Demostraremos la parte (1) y dejaremos la parte (2) para los ejercicios al final de
este ~odie.e. Para la parte (3), sólo pedimos (en los ejercicios) la demostración para el caso en que
ses un entero no nulo, pero aceptaremos {sin demostración) y usaremos la proposición general que
aparece aqul.
Supongamos que.x= log,.ry y = log.s. En,ooccs, comox-= Jog.r<=>lr= ry y=log.s <=> ~ = s
se sigue dela parte (1) del tcoremaAl.l que rs-= (b')(b') =br ♦ ,. Como rs= b·..'<=> I<>Urs) = x +
y, hemos mostrado que

log,(rs) =x +y= log.r + log.s.

En nuestro siguiente ejemplo veremos que podemos us.ar los tres resultados del teorema Al.2
para calcular los logaritmos.

Antes de la llegada de los computadores y las calculadoras, se usaban los logaritmos como ayuda
para el dlculo de productos. cocientes y potencias y para exttaer raíces. Con frecuencia. la base de
estos logaritmos era 1O y se disponía de tablas de estos niimcros para el trabajo con los logaritmos.
[Los logaritmos fueron ideados por el matem.á1.ico cscocés John Napier(ISS0-1617). Los navegan.
tes y astrónomos los usaron en el siglo xvu para reducir el tiempo e n que se realizaba una multipli•
cación o una división.]
Por ejemplo. como log10tO = J y log10)00= 2, vemos que 1 < log10 3 1 < 2 . De hecho, log10 3i =
1.4914. De la misma fonna. tenemos 2 < I081o137 = 2.1367 < 3. Se sigue del tcoremaAl.2quc

1) log.,4247 = log.,(31 · 137) ~log.,31 + log., 137 = 1.4914 + 2.1367 = 3.6281,


2) log.,(137/31) ~log.,137 - log.,31 = 2.1367 - 1.4914 =0.6453, y
- 3¡ log., ~ = log.,137'" = (l/3)1og., 137 = (1/3)(2.1367) ~ 0.7122.
A-8 Apéndice l Funciones exponenciales y logarftmicas

En el cálculo, CDCOlllnUnos un uso para los logaritmos de base, ,;, 2.71828, llamados logari-
narurales y que denotamos por lo general con ln x. para .t E R•. Cuando trabajamos con el amlisil
de algoriunos en la ciencia de la computación.los logaritmos de base 2 con frecuencia mostraroaa
utilidad. Pero esto no significa que debamos preocupamos por trabajar con logaritmos de m,
diversas bases. Muchas calculadoras proporcionan logaritmos de base 10 y de base e. F.a nucsa
siguiente resultado veremos que si podemos obtener los logaritmos en una base, podremos usarb
para obtener logaritmos en cualquier otra base.

TEOREMA A 1.3 l.afómuda d< cambi<> d< base. Sean a. b E R•, ambos distintos de l. Para cualquier x E R•,

log.x
log.x = log.a.

Demostración, Sean e = log.x y d = log.x. Entoo= Ir = x = a' y log,x = log. a'= d log.a =
0og.x)(log.,a). En consecuencia. log.:c: = log., z/log.a.

Por medio de una labla o calculadora vemos que log,2 = In 2 = 0.6931 y log,10 = In !O = 2.3026.
Por lo tanto, por el teorema.Al.3, Iog2 IO = ln 10/ln 2 =2 .302~.6931 ¼ 3.3222.

Si x = b como resultado del teorema Al.3 aparece una fórmula especial. En este caso tenemos qoe

Una vez que hemos repasado los antecedentes necesarios, es hora de definir la función logarítmica.

Definición A1 .5 Sea b ,< l un número =1 positivo fijo. La función g, R• ➔ R dada por g(x) = log,x es Jajiaocióo
Wgaritmica ~ base b.

a) Consideremos las funciones logañtm.icas

Las gráficas de estas funciones aparecen en la figura A 1.4. Escas funciones son tales que
1) 11(.t) ~ O y g?(x) .!: O para todox ii?: I. mientras que g¡{.i) <0 y g./,.x) <Opara todo.r< l.
(Esto es cierto para toda función logarítmica Jog.r tal que b > 1.]
2) para todosx.y E R♦. x< y ~ g 1(x)<g 1(y) [y gi..i)<g:z(y)). (Denucvo,cstocsciertopan
toda función logarítmica log.x tal que b > l.]
3) si "· u E R· y g 1(u) =g 1(U), entonces u= u. (Debecbo, si b> J, tenemos log.u= log.1>
=
~u= u pues w = 1086,u e> u =b•y w log.u =- u = b•·. )
Apéndice 1 Funciones exponenciales y k>garftmicas A-9

(8.3)

(g,) (g,)

Figura A1.4

b) La gráfica de la figura Al.5 conosponde a la función g,: R• ➔ R dada por g,(%) = lol!on)X.
Esta grifica ilustra las siguientes propiedades, que son ciertas para toda función logarítmica
log..xtalqueO<b< l.
1) En este caso,g,(x) ~ Opara todox S 1, aúentras qucgl..x)< O para todox > l.
l ) Para todosx. y E R•, si x <y entonces gJ,.x) > g,(y).
3) Si u, u E R• y g,<u); g3(u), entonces u; u. [La demostración C$ igual a la dada en la
S<Cción (3) de la parte (a).]

g/;,ó

-4
(g,) Figura At.5

c) En la pane (a) de la figuraAl.6 tenemos las gráficas de las funciones/: R ➔ a•, donde/(x)
=2r y g: R• ➔ R, dondcg(x) = lo&2x. Estas gráficas son s~tricas (entre si) con respecto de
la recta y = x(csdccir, si doblamos la figura a lo largo de la recta y= x. entone.es las gráficas
de / y g coincidirían). Aquí observamos tambifn la correspondencia entre los puntos de
ambas gráficas. Por ejemplo, el pun10 (2. 4) de la gráfica de/correspoode al punto (4,2) de
la gráfica de g. En general, cualquier punto (x. 2') sobre la gráfica de/ corresponde con el
puruo (2', x(= log,2')) sobre la gráfica de g y(%. log,.t) sobre g cornspoode coo (log,x, x(=
2"'')) sobre/
d ) Las gráficas de las funciones

h: R-+r, h(x) = (1/2)'

aparec:cn en la parte (b) de la figura A 1.6. Como en la parte (e), estas funciones tambitn son
simftricas respecto a la recta y = x. En este caso. cada punto (x.( l/2Y) sobre la g:rffica de h
corresponde al ponto ((1/2)', ,{= log,.,.(1/2)')) sobre la gráfica de k, y (x, log,,n,X) sobre k
corresponde a (lo&;Lmz, z(= (112>1°'<~) sobre h. (Estas dos gráficas se intersecan sobre la
rec:tay=xenel puntoz ,,;,,0.6412.)
A-10 Apéndice 1 Fvnóones exponenciales y logaritmicas

(8,-3)
(b)

Figura A1 .6

e) Tal vez el lector desee examinar, o volver a examinar, lu gráficas de las funciODC5 y= r y y=
In x que se muestran en la figwa 5. 9 de la sección 5.6. En esa sección esrudiamos la relación
desimetrfa de las funciones respecto de la rectay = x [mencionada en las panes (e) y (d)] en
relación con las ideas de composición de funciones y la ipversa de una función.

EJERCICIOS A-1 1. Escriba lo siguiente en forma exponeocial, para z,y E R·.


a) viJ' b) ~81.r-'y' •> 5,;ls,•y-•
2. Evalde lo siguiente.
a) 125-"' b) 0.027"' <) (4/3)(1/8)-"'
3. Detennine lo siguiente.
7"'
a) (5"')(5''") b) .,,, e) (5"')(20"')
7
4. En los SJguientes ejercicios, CDCUCDtrt: los m1rneros realesx para los cuales son válidas las ecuacioocs.
a) S"' = 5,.., b) 4.-0 = (1/2)~-• e) (1125)'-• = (1/125)'

5. Escriba cada una de las siguientes ecuaciones exp::>ncnciales como una ecuación logarltmica.
1
a) 2 = 128 b) 125"' =5 o) 10
4
=1/10,000 d) :Z- = b
6. Enruentre cada uno de los siguientes logaritmos.
a) log,.100 b) Iog,.(1/1000) <) log,2048 d) log, (1/64)
e) log.8 f) log,2 11) log,ol b) log,,9
7. Determine x en los siguientes ejercicios.
a) log. 243 = 5 b) log,x = -3 e) log,. lOOO =x d) log..32=5/2
•) Iog.z = O f) log,z = 2/3 11) log.x- -2 h) log.. Y5 =1/2
8. Demuestre la parte (2) del teoremaAI.2.
9. Sean b, r E R\ donde bes fijo y distinto de l.
•) Para cada n E z•. demuestre que log.,.. = tt log.r.
b) Demuestre que log.r= (-n)log.r para todo n E z·.
1 O. Aproxime lo siguiente, suponiendo que log;,.S = 2.3219 y 10117 = 2.8074 {htita cuatro cifras
decimales).
a) log,10 b) log,100 e) log,(7/5) d) log,175
Apéndice 1 Funciones exponenciales y logarítmicas A-11

1 1. Dado que In 2 =0.6931, In 3 = 1.0986 y In S = 1.6094 (has,a cuatro cifras decimales), aproxi-
me lo siguiente.
a) log,3 b) log,2 e) log,S
12. Detennine el valor de x en cada uno de los siguientes ejercicios.
=
a) log,.2 + 1og,.s log.ox
e) log.3 + log.x = log. 7 - log.S
13. Determine z en lo siguiente.
a) logioz + log,06 • l b) lnx - ln(x - l) • ID3
e) ~z'- Iog.z =4 d) log,(r H x +4) - log,(2x - S) • 2

14. Determine el valor dcz si logiz = (l/3){logz3 - lo~S] + (2/3)1~6 + log:17.


15. Sea b un número real positivo fijo distinto de l. Si a, e E R.., demuestre que~~ =~ •.
Apéndice 2
Matrices,
operaciones
con matrices
y determinantes

A partir del capítulo 7, y en varios capítulos posteriores. presentamos ciertos tipos de matrices.
Desde el punto de vista ttistórico, c:sw estructuras matemáticas fueron desarrolladas y analizadas
en el siglo XJX por el matemático inglts Arthur Cayley ( 1821- 1895) y su colaborador norteamerica-
no (nacido ea lnglatcaa) James Joscph Sylveste, (1814-1897). El trabajo de Caylcy acerca del
álgebra de matrices, presentado en 1858, proporciona otra situación en la que la investigación en
matemáticas abstractas demostró des:pub su importancia en muchas áreas aplicadas; por ejemplo,
en la teoría aántica de la fisica y el análisis de datos en psicología y sociología.
Para los lectores que no hayan estudiado la\ matrices en otros cursos o que sólo deseen rcpuar
el álgebra de maaiCC$ que usamos en este texto, el material de este apmdice será de utilidad. (No
demostraremos todos los resultados en general, sino que los establ«:eremos junto con un ejemplo.
Para un trataD»enf.o más riguroso, el lector deberá consultar alguna de las referencias al final de este
apb>dlce.)
Eo primer lugar, comencemos por lo siguiente.

Definiáón A:J..1 Para m. n E z•, una matriz m x n es una disposición rectangular de mn nó.meros ordenados en m
filas (borizonta1cs) y n columnas (verticales).
Una matriz m x n se denota con A = (a,).,. .. donde 1 :S i :S m y 1 '5 j :S n; el nwnero a, es el
elonLnto (~1) (es decir, el demento que aparece en la i--éwna fila y laj~ima columna de A). Una
matrizm x 1 se llama con frecuencia una.matriz.columna (o vector columna); una matriz 1 x n es
una matrkfila (o vectcrfila). Si m = n, la matriz es cuadrada.

- Sean A=(a,,),.,=u HB=(b,,),.. =[: ~ -~ ~], yc=[iñ 1].


En este caso.A es uoa matriz: 3 x 2. tal que a 11 = 1. au= 2. a 21 = O, an= 3, a 31 =-S y a 52 =4. La
matriz B tiene dos filas y cuatro columnas, donde, por ejemplo, tenemos los elementos b13=O y
b,. = 7. F.o la matriz cuadrada C 2 x 2 vemos que los elementos de la matñz pueden ser nómeros
racionales o irracionales.

A-13
A-14 Apéndice 2 Matrices. operaciones con matrices y determinantes

[Nota: Aunque los elementos de una matriz pueden ser incluso m1mcros complejos, en eSle
apEndice sólo trataremos las matrices cuyos elementos son n11Ineros reales.}

Como ocurre con otras estructuras matenáti.cas. una vez definida la estructura necesitamos deo.
dir cuándo dos de ellas son la misma Ahora indicaremos e1 mttodo para tomar esta decisión.

Definición A2.2 SeanA : (a1 ),.. ,.. y B:::: (b,)..-:,,: 1 dos matrices m x n. Decimos que A y B son iguales, y escribimos
A =B,si~= b, paratodos 1 :s: i Smy 1 Sj:S: n.

En la definición A2.2 vimos que dos matrices son iguales cuando tienen el mismo número de filas.
el mismo n11mero de columnas y los mismos elementos correspondientes. Como resultado, si

A=["'0 23 º]
X
B= [-1O y º]
z 4 '
para que A y B sean iguales, debemos tener w =-1. x: 4, y= 2, z = 3.

Sí recordamos nuestros primeros J>as0$ en la arittMtica, des~s de aprender a contar comenza...


mos a combinar los enteros por medio de la suma y después con cJ producto. Seguiremos la misma
línea y veremos ahora la forma de combinar las matrices.

Definición A2..3 SiA=(aiJ)• •• y B = (b¡i),.x. son dos matricesm xn, su suma, que se denota con A + B, es la matriz
m xn C=(cv).,..tal que cil=af +b, para todos 1 :s i s m, I SJ s n.

Dela definicióoA2.3 vemos que sólo podemos sumar matrices del rriismotamano (que tieoenel
mismo número de filas y el mismo nllmero de columnas). Además. la suma de dos matrices se lleva
a cabo sumando sus elementos correspondientes.

~1 Consideremos las matrices

y C=
[1-1]
-7
3 -4 .
6

I + 2 3+(-1) 4 + 6] [3 2 10]
Aquí vemos que A + B =[ 2 + 3 O+ 1 6 +7 = 5 1 13 . De hecho, también tene-
1 + 4 1+2 3+2 5 3 5

mos 8 +A = [! : !~] ,
5 3 5
lo que ilustra el siguiente resultado general.

Para dos matrices cualesquiera m x n E y F. E+ F ;- F + E. Por lo tanto, la suma de matrices es


un ejemplo de operación (binaria) conmutativa.
No podemos cleterminarlasumaA + CoB + CporqueA y Btienen tres columnas yCsolamcnte
dos. Sin embaigo, podemos determinar la suma
Apéndice 2 Matrices, operaciones con matrices y determinantes A-15

C+C =
[
-7 6
-Ji [ 1-li [ 2-2]
3¡ -4 + 3 -4 =
-7 6
6 - 8.
-14 12

En la última parte del ejemplo A2.3, vimos que podríamos obtener el resultado C + C multipli-
cando cada elemento de C por el número 2. Esto nos lleva a la siguiente idea general.

Definición A2.4 Si A = (0¡1).,..y r E R ,elprodw:toporun escalarrA es la matrizm x ncuyoelcmento(i.J) es ra.


paratodosl SiSm,l :Sj:Sn.

.
a) S1A =¡10 _ 61 _ 4]
3
, entonces

3A• 3[ 1 - 61 -3
4] [ 3·1 3-6 3 -4 ] [3 18 12]
= 3·0 3 · (-1) 3-(- 3) = O -3 -9 '
0

b) ParaA•[l 6
O - 1 -3
4] y B = ¡ -5
-2 2] , tenemos que 3B
1
=[ _-6
1
5
O 6]
3 21 'Y

J(A + 8)=3 m 6 4]
¡- 2 o
-1 -3 + -5 1 m
=3¡-1 6 6]=[-3 18 18]=[3 18 12]+[ - 6 O 6]
-5 O 4 - 15 O 12 O -3 -9 -15 3 21
=3A +3B.
e) Podemos generalizar el resultado de la parte (b) como sigue: para dos matrices cualesquiera
E, F m x n y cualquier r E R. r(E + F) =rE + rF. l;.$t'e principio es la ley dis1riburiva dt la

-
multiplicación por tscalare.s sobrt ÚJ swna dt matrices.

a) Sea A = (a,,)1• , una matriz 3 X 2 aroilnlria, y sea Z rn n = Entonces

A +z= [:;:
ª>1
::t[g
a:J
g] =[:;: :g:::g]
O O an+O
a >1
::LA.
a:l
+0
= [:;:
a,1

Decimos que Z es el ntuPO aditivo (o ctro) de todas las matrices 3 x 2.

b) Si "A= [ ~
-4
-!]
5
y B= [=! 4
-!].
-5
scsigueque

A+ B = [~ :)=~( ::t/~]= ¡gO


(- 4)+4 5+(-5)
g].
0
En consecuencia, decimosqueB= (-l)A esel inversoadiírllodcA y tambiál escribimosB = -.4..
A-16 Apéndice 2 Matrices, operaciones con matrices y detenninantes

Espcmnos que lo bccho basta ahora haya mostrado ser de inletts. Pero lo que hace el e,n,d;odo
las matrlte$ algo realmente interesante es la operación de multiplic.aci.ón de matrices. Si i n ~
definir esta operación como la operación componente a componente de la suma de matrices., el
resultado es poc.o interesante. En vez de esto. e l producto de matrices se basa en una multipücaci61
de filas y columnas y una suma; por ejemplo.

Por lo tanto, en un caso particular, tenemos

(-1 4 3]m=(-1) -2+4 -1+3·7=-2+4+21 =23.

F.n general. si a= (a.:,)1s1s.es un vector fila 1 x n y b = (b;)1"1>J: ■ es un vector columna nx t.


cmooccs d, = };_ a¡ b;. Este t'e$ultado, que es un nwnero real, es el producro escalar de los
1
vectores (o matrices) a y b. Esta idea es la clave ncccsaria para la siguiente definición.

Definición A2.5 DadaslasmatricesA=(a;j).,.yB= (b,J-r elpmducro(dematrices)ABeslamatrizC=(c,J• . , . -

c..1:=0i1bu+4-'2"2..,+ ··· +41.b_.=;0i,b,.1:, paratodosl:s:is~t:s:k:s:p.

Por lo tanto, el elementoc,1 en la i ~ fila y lak~ma columna.de la matriz m. xp C seotiocnc


mediante el producto escalar del vector fila i de A y el vector columnaj de B.

Lo siguiente ilustra el resultado dado por Ja definición A2.S.

... ...... .,. bu b., b,,

1
...
AB
ª" "'· b,, b,, b,,

._, ._, .... b., b., b.,


e,. c., ... e,.
...

• ::]
e,, Cu
''"
=C=
e,, e,, -·· ... e,,
...
c., c., ... "- . .. c.,

1 27
a) eo..,deremos
_ .
las mamcesA = (a,1),., = [121] y B = (b1,),,., = [ ~ ~ ~] . Entooces

~= :].
3 0 4
AB - C=(eu)2x3• [ : donde
Apéndice 2 Matrices, operaciones con matrices y determinantes A-17

C11ª1•1+2·1+1•0=3
ffll 2
3 3 ,
1 1
7]

C12= 1 ·2+2•3+ 1· l =9
ffl[i Ifl.
2
Cu= 1 •7 + 2 · 3 + 1 • 1 = 14
ffl[i 1· 3
1

c,,-3·1+0·1+4-0=3 [w.JI fl.


e,,= 3 ·2+ 0-3 +4-1 = 10
lW.i][i 1fl.
c,, • 3 · 7 +0 ·3 + 4 · l =25 lW.i][i i ,.
f.o. consec:uencia.

9 25
AB•C• [ 33 10 14].

b) Si A y B son como en la parte (a). intentemos formar el producto matricial BA =

[i i U[1 ~ !]- Pan dot=ninar el elemento de la primera fila y la primen columna

de BA.. queremos fonnar el producto escalar

(1 2 7]rn - 1-1+2·3+7·"?".

Por desgracia. no teoemos los elementos suficientes en la primen. columna de A, por lo que
no podemos formar este producto escalar Di el producto matricial BA..
Ahora no.s ddatcmos a pensar por qut pudimos formar el producto AB pero no el pro-
ducto BA. Si 0011Sidenmos de nuevo el producto BA. vemos que la dificultad rw.tica en d
hecho de que la primera columna de A no terúa el mismo nt1mero de elementos que la prime-
ra fila de B. El m!mero de elementos de la primera fila· de· -S-es 3, que es el oúmcro de
columnas en B. El mlmero de elementos en la primera columna de A es 2,. que es d número
de fila., de A. Estas observaciooes oos llevan al siguiente resultado geaeral.
Si Ces una matriz m x n y Des una matriz p x q, c:ntooces el producto CD se puede
=
formar cuando n p; es decir, cuando el nwnen> de columnas en C (la primera matriz) es
igual al m!mero de filas de D (la seguooa matriz). Y cuando n = p, el producto CD resultatle
tieoc m filas y q columnas.

Analicemos con más detalle la multiplicacióo de matrices.


Apéndice 2 Matrices, operaciones con matrices y determinantes

a) Si A=[! f] yB=[1 Ó l].enton=AB=[! ! !] anentrasqueBA=[! ~]-


consecuencia. aunque podemos formar ambos productos de matrices AB y BA. no oca
que AB = BA. De hecho, estos productos ni siquiera tienen el mismo tamaiio.
b) SiA=[-J -lJys=[l i].vemosqueAB=[8 8] yBA=[l ::].Así,enesle-
AB y BA tienen el mismo tamai\o pero AB -1= BA.
e) Por íiltimo. analic.c las macric.cs

A= [l 1 3]
4 -1 5 ' B=[-3~ -1
t],
F.n este caso vemos que

As-[! _: ~JU
j]=[=~ =~] Y

[-7 -11¡1 2] (-10 -10]


(AB)C= -7 -2 3 -4 • -13 -6 '

mientras que

BC=U _¡][~ _¡)=[J =~] y

A(BC} • [l 13][
4 -15 -6
; -~] = ¡-10 -10]
-6 .
-2 -13
Por lo tanto, (AB)C =A(BC).
En general, si m, n, p, q E z· y A= (a,¡)...., B = (b¡J.., y C = (cu), .... entonces
(AB)C=A(BC),

por lo que el producto de matrices es asociativo (cuando se puede realizar).

De los resultados de las partes (a) y (b) del ejemplo A2.7 tenemos dos hechos importantes:
1) La operación de producto de matrices no es conmutativa en general.
2) Es posible detcmtinar dos matrices no nulas C = (c11). • • ( cu f: O para algunos J s i s m. J
=
Sj S n) y D (d1J.,., (d,: '#t. O para algunos I Sj s n, 1 s k s p), tales que CD =Z=
(0). x,.

En resumen, la multiplicación de matrices no necesariamente se comporta como la muhiplica-


ción de níimeros reales.

Ahora que hemos hecho algunas comparaciones entre la multiplicación de matric.cs y la multi~
plicación de números reales. seguiremos un poco más.
Apéndice 2 Matrices, operaciones con matrices y determinantes A-19

a) Si consideramos las matrices cuadradas (en particular, las matrices 2 x 2), vemos que

[• b][l º] = [1 º][• b] •[•


cdOl Oled cd·
b]
Encomecuencia.lamatriz/2= [~ ?] cselMuJromulliplicativodetodaslasmatriccs2x2.
En general, para un entero positivo fijo n > l, la matriz

csi=j
1. = (&.,)••••
esi+;

es cl Mutro multiplicativo de todas las matrices n x n.


b) De regreso a los mlmeros reales, recordemos que para cada .x E R, si x "I=- O, entonce$ existe
y E R tal quc:ry= y.x= l. Este número real ese) invenomulti.plicativo de.xy se denota con
frecuencia como r.
Quisiframos saber si se presenta una situación similar para las matrices cuadradas; nos
concentraremos en las matrices 2 x 2.
SiA=[~ ~].doodea. b.c. dson ru.1mcrosreales fi~.¿podemosdetenninarunamatriz
B = [; 1] tal que AB = BA =/ ? (En este ca.so, w, x, y. z son los números reales desconoci•
1

dos y queremos determinar sus valores eo tmninos de los mlmeros reales dados a, b. c. d.)
Formamos el producto AB y vemos que

Para que AB sea igual a / 1 ; es decir, para que

aw+by
[ cw+dy cx+dz
ar+bz]=[l 1o]O

de la definición de la igualdad de matrices se sigue que

(1) aw+by=l (3) ar+bz=O


(2) cw +dy =O (4) cx+dz • l.

Centrándonos en la$ ecuaciones (1) y (2), si multiplicamos la ecuación (1) por d y la


ecuación (2) por b, tenemos que

(!)' adw+bdy=d (2)' bcw + bdy = O.

Restando la ecuación (2)' de la ecuación (l)', vemos que adw - bcw= (ad- be)w = d, de
modo que w = dl(ad - be), si ad - be t O. Otros cálculos similares producen x = --bl(ad -
be), y= ---d(ad -be), z =a/(ad- be) y estas fórmulas wnbién son v1ilidas si ad -be e/e O.
[Nora: (1) El mlmero real ad - be es eldeunninantede lamatrizA. (2)Aunquedetcrmi•
namos los valores de w, x, ~ .t a partir de la ecuación AB = 12, se puede demostrar que se
obtienen los mismos resultados cuando trabajamos con la ecuación BA = / 2.]
A-20 Apéndice 2 Matrices, operaciones con matrices y determinantes

e) Usando los multados de la parte (b), sea

A=[: :]=[~ il
Entonces, como ad-be= 1 · 1-2 · 0 = l('F 0), se sigue que w = 1/1 = l,z= -2/1 =-2.y=
-<Vl=O,z=lll=l,y

r~ m~ -iJ ¡~ ~J ¡~ -m~ i]-


= =

. ·~-- A"' = O
. escnvuuvS
En estas CU'CUD.StancJas, [¡ -2]l .
d) Considet<mos la matriz A, -[~ !]. donde el delerminantedeA,=3 • 2- 1 .¡ = 5(# O~

EnestecasotenemosqueA;'=[~ ff'=[-t~ -!~~] =1/5[_:


t) De las partes (b), (e) y (d), podemos decir que si A=[: :].
-n
entonces A 4 =,

_ l_ [ d -b] = determinante de A= ad -
si det(A) f O. be
det(A) -e a '
f) Para la matriz.4. =[l ~]. vemos que el
2 deA = l · 6-2 • 3 = O,
determinante 1 de modoqoc

en este caso no existe un inverso multiplicativo; es decir. no existe A; 1•

Hasta el momento hemos desarrollado algunas ideas fundamentales acerca de las matrices, y el
lector podria preguntarse cómo podrían usarse estas estructuras matemáticas. Por lo tanto, regresa-
remos de nuevo a los nómeros reales y a algunas de 13$ ideas de álgebra elemental.
Al resolver la ecuación 2x = 3, podemos escribir la sigujente lista de ecuaciones:

(!) 2%=3
(2) G)(:ZX) = G)(3)
(3) [(!)2Jr =3/2
(4) l·z=J/2
(5) z=3/2

Al resolver C$ta ecuación, el mlmero real 112(= Z-1), que utilizamos en la ecuación (2), es lo que
necesiwnos para "despejar la incógnita", cuando vamos de los pasos (3) y (4) al paso (S). As!, t11
general, si partimos de los nwneros reales fijos a. b doade a 'F O. entonces la ocuación ax= b tiene
la solución X = d"'b.
Consideremos ahora el sistema de ecuaciones lineales:

(")

que podemos representar en forma matricial como


Apéndice 2 Matrices, operaciones con matrices y determinantes A-21

{Esta forma de representar un sistema de ecuaciones lineales es útil para comprender la razón que
subyace en la definición del producto de matrices. ya que el lado_derecho de cada ecuación en(*) es

matriz[~ 1]con lamatrizcoJumna ~].]Si hacemos


el producto escalar de una fila de la

A=[! i]• X=[;] B=[n


entonoes buscamos una solución de la ecuación (matricial} AX= B. ¿Podríamos tener aquJ' la soJu.
s. =
ción X = ,4,~1 si C011$ideramos que tsta era x a--1b en la ecuación anterior ax= b?
Puesto que el ddenninantedeA = 3 • 2-1 • i°=5;, O, a partir de la pane (e) del ejemploAl.8
sabemos que

A-'=(1/5)[ - 21 -!]=[
3
2/5 - 1/5]
-1/5 3/5 .
Entonces tenemos que

(!)' [! m;J=m
(2)' [_t; -~nm m;D-[-t; --~m~J
(3)' ([_t; -~;fü i])[;] = [~t;]
(4)' [~ ~][;l=[~t;J
(5)' [;J =[ ~tn
Dela definieiónA2.2 se sigue entonoesde la solución ~] =X=A-1B = [~~~que x =-1/S y
y= 1815.

a12
En general, si A=[~: ~] y B =1~1. cona 11,an, a:i,a:u, b1, b2
Ozi #- O. entonces la solución del sistema de ecuaciooes lineales,.
E Ry det(A) = a 11 a 12-

aux + ai:z1 =b,


al}%+ O::ij = b-,.,
esti dada por

Y
1_[ -a,.a,, - •n•.,][b•]=[(l/det(A))(anb,-aub,) ]
x-[z]-A_,8__det(A) b, (1/det(A))(-a,.b,+a.,b,) ·
Además, aunque no podemos demostrar nuestro siguiente resultado, lo siguiente es cieno para
n e z•, n ~ 2.
=
Si A = (°'J).u es una matriz real (COD inverso multiplicativo}, y B (b.}1,.;,u, X= (xJ1,u ""'°º
matrices columna n x 1 (como las definklas antes para n = 2), entonces el sistema resultante de
ecuaciones lineales

AX=B
A-22 Apéndice 2 Matrices. operaciones con matrices y determinantes

tiene ta solución

Ahora. aunque no trabajaremos coo las inversas de matñc:es mayores que 2 x 2.


ap&dice coo unos cuantos resultados más de detemtinantes mayores.
Ya sabemo, que paraA=[~ ;]. el detcmúnan1edeA,det(A) • ad-bc. Eldet(M se

lo geoenJ conl~ :~Pata trabajar con los detemunantes de matrices mayores oecesitm
i¡uíeote idea.

Definición A2.6 Sea A = (a;) • • ., con n ~ 3. Para cada J s i :s n y 1 s j :s "· el menor asociado coa cr,
determinante (n - t)x (n - 1) obtenido dela matriz.A desputs de eliminar lai-áim.a ftlay b ·
columna de.A.

a) Para A • [ !~ !),
-1 3 7
1<nemos que

1) el menorasociadoconOseobdenedeA sí eliminamos su primera fila y segunda

1 O2]
[-13 34 67 00$ lleva a;

2) paraaz,=6, elmeoores 1::: ::J•L! ~j.


b) Dadalamatriz4x4

B=[
-6
!~ -3 - 2
9
~
4
-1]
S '
O
el menor asociado con 3 es el determinante de 3 x 3

2 o 61
- 3 -2 S,
19 4 O

que se obtiene de la matriz 8 al eliminar la segunda fila y la primera colunma de B (y


reemplazando los corchetes de la malriz por las barras verticales de los determinanlcs).

Dada una matriz A = (a¡1>,. >• para todos 1 :S i S 3, 1 S j S 3, denotamos con M, el memr
asociado con ª•·
Entonces
Apéndice 2 Matrices. operaóones con matrices determinantes A-23

- aulª12
an ªni_•ul°"a,, º"I + o,.Iº"
ª» a» a, °"1•
1 a32

y decimos que evaluamos detGA) mediante un tksarrollo por ~no~s.


De esta forma. reducimos el problema a los determinantes de 2 x 2 que ya sabemos evaluar.
Examinemos un ejemplo.

•> li i - i l -2(-l)'··I: ~I +4(-l)'· •I~ ~I +(-7)(-l)'·•I~ ! I

• 2(8-0-2·6) -4(3 · 0 - 2·S)-7(3 •6 - 8·S)


• 2(-12) - 4(-10) - 7(-22) • 170.

[Nota: En este desarrollo poc mcnon:s encoolramos una suma que usa cada clcmen10 au,,
para 1 ,; /,; 3, ca la primera fila del dc!amioanu: y cada uno de tale, dcmcal0$ se multipli•
ca por dos 16minos:
1) (-1)1..-, donde el exponente 1 + j es la suma del námero de füa y el m1mero de columna
dcav; y,
2) su menor asociado M.,.
b) El lector podña ¡,,cg,,nwsequt tic:nedc especial la primen filado un dc!aminan!c, pues si
dcsam>llamos d determinan!< de la panc (a) por la t = columna, d desarrollo resultanle
seña

t. 0n(-ly♦'M,. • (- 7)(-l)'"'I~ :1+ 2(-l)!"'I; :1+ 0(-l )'••I~ :1


• (-7)(3· 6-8· S)-2(2·6-4 · S) + 0(2·8- 4 •3)
-(-7)(-22)-2(- 8) • 170.

e) Lo ocurrido en las partes (a) y (b) DO es mera coincidencia. En general, para una mauiz.A.
3 x 3, podemos evaluar d dclemlinant< de A desarrollwlolo a lo latgo de cualqlrie< fila
(fij,) o de cualquie< columna (fija). Este mtcodo se extiende a ma!ri= cuadradas más grm-
de:5; es decir. para n e Z♦• donde n ~ 4, podemos desarrollar un dclerm.iname n x n, a lo
largo de cualquiera de sus n filas o de sus n columnas, como n sumandos, cada uno de los
cuaJcs tiene un detc:rm.inante (n - 1) x (n - 1).
Si A. = (GJ.... donde n 2: 3, entonces

det(A) -Ío;;(-ly♦1M,, [desarrollo a lo lM¡o de la i-tsima fila (fija)]


;-1

Í o,,(-ly♦1M,,
=,., [desarrollo a lo largo de laHsima columna (fija)].

d ) De la parte (e). vemos que si A= (a;i)... pan cualquier n ii!: 3 y A. tiene una fila o columna
con todos los elementos nulos. se sigue que eJ determinante de A es O.
A-24 Apéndice 2 Matrices, operaciones con matrices y determinantes

BIBUOGRAFfA

Las ideas presentadas en este apEndicc (y sus ejercicios c:onespondicntes) debewl bastar cxi-.
base necesaria para el uso de las matrices y determinantes en este texto. Para el lector que desee
aprender más acerca de esta área de las matemáticas, cualquiera de los siguientes libros podrtsaff
como un buen punto de partida.
l. Anton, Howard y Cuis Rorrcs. Ekmmt.ary l..irwzr Algtbra wilhApplkalwns, Nueva York.
Wiley, 1987.
2. Strang. Gilben. linau Alg,bra and lts Applications, 3' ed., San Diego. Calif.• H"""""
Brace Jovanovicb, lnc., 1988.

EJEROCIOS A.2
1.ParaA=(a,1),.,=[-: i -: =!],ddernúne
2 7 5 6
a) la segunda fila de A; b) la teoccracolumna de A;

Iwl~x
2.Detenninew,x,y.ztalesque w~~]=[2,!3z
1
~/].

3.SeanA=[_: ¿ ;],s=[: ! !]. y e = [~ ! _;J. Encuenuelosiguí-


•> A+B b) (A+B) + C e) B+C d) A+(B+C)
e) 2A f) 2A + 3B &) 2C + 3C ~) 5C( = (2+ 3)C)
0 2B - 4C(•2B + (-4)C) j) A +2B-3C k) 2(3B) 1) (2·3)8

. [• b] [
4.Encuenttea,b, c,ds1 3< d + 4_ 1 -2] [1 º]
3
_
2
a2
5 3
.
5. Realice las siguientes multiplicaciones de matrices.

•> (1 3 7][-~] b ) [: ;](i ;]


d) [241 -~ - ~][-~
O -5
:]
7 7 2
~ e)[~ nm z ~]
6.ScanA=[-¡ ~].s=[: ; -~]. YC=[-~-~ -n Muesttequc

(a)AB +AC=A(B +C); y(b) 8A + CA=(B+ C)A.


[En general, si A es una matriz m x n y B, C son matrices n x p, entonces .4B + AC =
A(B + C). Para las matrices B. C nxp y una matrizAp xq, sesiguequeBA +CA = (B +C),4,.
Estos dos resultados son las leyes disrribwivas para la multiplicaci6n. de manicu soh~ la
suma tk matrices.]
7. F.ncucntre el inverso multiplicativo de cada una de las siguientes matrices, si &te existe.

a) [! :] b) [~ ¿] e)
-3
[ -6 2
¡] d) [ 7-3¡
-2 1
Apéndice 2 Matrices. operaciones con matrices y determinantes A-25

8 . Resuelva cada una de lM siguientes ecuaciones matriciales para la matriz A 2 x 2.

9 . Si A =[~ ~] Y B =[ =~ i]. detcnnine lo siguiente.

a) A-1 b) s-• e) AB d) (AB¡-'

10. En la parte (b) del ejemplo A2.8 obtuvimos la fórmula para A·1 considerando la ecuación

matricial AB = li. donde A=[~ ~}B=[; 1] e / =[Ó 2 ?J Muestre que obceoemos las
mismas soluciones para w, z, ;>; z si trabajamos con la ecuación matricial BA = / 2•
11. Resuelva los siguientes sistemas de ecuaciones Jinea.Jes por medio de matrices:
a) 3.r-2y=5 b) 5'+3y •35
4x-3y=6 3.r - 2y=2
12. Evaló.c los siguientes determinantes 2 X 2:

•> l !I b) 1~1~1 dl ¡s10¡


15 20

13. Sean a, b, e, d E R tales que rc ~ :;: 7. Determine el valor de lo siguiente.

a) 1~ ~I b) 1~ :1 <) li ~I d) ,~ ~I
14. Sea.A una matriz 2 x 2 con dct(A) = 3 1. ¿Cuál es el valor de det(2A)?, ¿e l dedet(SA)?
1 S. Dcsarrol1e tos siguientes determinantes a lo largo de la ftla y columna dados.

a) ,~~ 4 1
=ii;fila 2,columna3
2
b) ,~
O 5
~ _¡ , ;filal. columna2
7
16. Desarrolle los siguientes determinantes a lo largo de cualquier fila y columna.

•> 1
6 -2 1
o 21 b) 4 27 O º1 <)
1O 21 -4,O
14 3 2 13 6 2 13 3 2

17. Despejex:n - ; J=s


18. a) Evahle cada uno de los siguientes determinantes 3 x 3:

i)
1
1 1 3
111 4
1 21
m) 1~i -!I
b) Enuncie un resultado general sugerido por las respuestas de la parte (a).
19. a) Evalúe cada uno de los siguientes determinantes 3 x 3.

1)
1
O -1
2 -111 il)
s 2 11
O - 1 - 1
12 3 O 110 3 O
A-26 Apéndice 2 Matrices, operaciones con matrices y determinantes

b) Seana,b,c,d, e,f,g,h,iE R. Sil: ; {1=17, cvaldc

1)
3a b
3d e f
el 24 2b 2,1
11) 3d 3e 3f
13g h i 1Sg Sh 5i
20. Sean A= (a,).~ y B = (b¡;),~ dos matrices. Al formar el producto matricial AB, seg,lo la
definición A2.S, ¿cuántas multiplicaciones (de los elementos) se realizan? ¿Cuántas suma
(de productos de elementos) se realizan?
Apéndice 3
Conjuntos
numerables y no
numerables

E D d ejemplo 3.2 de la S<Cci6n 3.1 mencicoamos de modo míonnal las Mleas de lo que pensamos
es un conjunto finito y un conjunto infinito. Esle óltimo ap!odice tnUa estos temas de manera
más rigurosa y nos ayudará a dar cierto significado a IAI (d tamaoo, o cardinal, de un conjunto A)
cuando A es un coojuoto infinito. Para desmollar estas oociones C:00 más precisión. recordemos el
siguiente coocepc:o. que preseownos por primen: vez en la seccido 5.6.

Dofinici6n A3.1 Para dos conjuntos cualesquiera A, B, la fuoo.00/: A ➔ B es una correspon,tkncja biy«tiva si/es
inyectiva y sobre.

s.ao,1 = z· 1 B• 2Z·= (2.tlt e z·1. 12.4, 6. ... l-1.a fuocioo/:A ➔ s. dada poc/Crl = 2x,
es una correspondencia biyeaiva.
1) Paraa1, a 1 E A, tenemosquef(a1)~/(aJ ⇒ la,= 2a1 => a 1 = a 1,demodo que/es inyectiva.

= =
2) Si b E 8 , entonces b 2a para un (wlico)a EA y/(a) 2a •b. lo que ba<:ede/uoa función
sobre.

El resultado del ejemplo A3.l nos lleva a coosiderar lo siguiente.

Definición A3.2 Si A. B son dos conjuntos, decimos que A tiole el mismo tamaño, o cardinal, qJU B, y escribimos
A - B, si existe um. c:oucspoodeoci.i biyectiva/: A ➔ B.

Del ejemplo A3. I, vemos que z• tiene el mismo tamaño que 2Z•, aunque parezca que '27,· tiene
menos dememos que z•; ~ de rodo, sabemos que w c z·.
Si definimos g: B ➔ JI (para B = 2Z· y A= z•¡ como g(2t) • k, emonces
1) g(2k1) • g(lkV ⇒ k1 • k1 ⇒ 2k1• 2k1t lo que establece el hecho de que ges inyectiva; y
2) para cada k E A, tenemos que 2t E 8 , con g(2t) = k, de modo que g cambitn .. una función
sobre. En coosecuc:ocia,. ges una wttes¡x:udeocia. biyectiva y B -A.

A-27
A-28 Apéndice 3 Conjuntos numerables y no numerables

Así, al menos en el caso de A = z+ y B = 2Z• vemos que A - By B - A (aunque B CA). Fa


realidad, lo ocurrido en esta siruación es cierto en general. pues la función g reci~ definida es.•
realidad, la fu.nción/-1 de/en el cjemploA3.l. En el teorema 5.8 aprendimos que una funcióaa
invertible si y sólo si es inyectiva y sobre. F.n consecuencia. siempre que dos conjuntos no vacfosA,.
B cumplan que A - B. entonces se sigue del teorema 5.8 que B - A, de modo que podemos decir qR
A y B tienen ti mismo cardinal.,quc denotamos con IAI IB!.
= (Nota: Nocs necesario que-A=B.)
Consideremos otro ejemplo.

Para 8 = 2Z·= (2.tlk E z·¡ y C = 3z•= (3kjk E z•¡, la función h: 8 ➔ C dada por h/.2.t) = lt
establece una correspondencia biycctiva entre By C. Por lo tanto, tenemos que B - C (y C -B,de
donde IBI = ICl).Además. si usamos la funciónf:A ➔ Bdefinidaen el ejemploA3.I, donde A•
z +, por el teorema 5.5 sabemos que h of: A -+ Ctambitn es una correspondencia biyectiva.Asf.
A - e <Y e-A, de donde IAI = IC I>-

Podemos resumir lo aprendido hasta el momento como parte del siguiente resultado.

TEOREMA A3.1 Para c:ualesquiera conjuntos no vados A, B. C,

a) A -A;
b) si A -B, entonces B -A; y
e) siA-ByB-C,cntonccsA-C.
Demostración:

a) Dado un conjunto A no vacío, A -A se sigue del hecho de que la fuoción identidad l'-:A ➔
A es una correspondencia biycctiva.
b) SiA-B,entonces existe una correspondeocia biycctiva/: A ➔.B. Peroentonccs¡-1: B ➔A
es también una correspondencia biyectiva y tenemos que B -A.
e) Só A -By 8-C.exiSIMcorrespondenciat,;yeaivaf: A ➔ By g: B ➔ C. C.O.OOgof:A ➔ C
también es una correspondencia biyectiva, se sigue que A - C.

Ahora usaremos las ideas desarrolladas hasta el momento para definir lo que entendemos por
conjunto finito y conjunto infinito.

Definición A3.3 UnconjuntoAesfinitosi A = 0osiA ... {1, 2. 3•... ,n} paraalgúnn e z+. Cuando A= ttdecimos
que A no tiene elementos y escribimos IA
1= O. En el segundo caso. decimos que A tienen elemen-
tos y escribimos jA1= n. Si A rw es finito, decimos que es in.finito.

Con esta definición. podemos ver que si A es un conjunto finito no vacío existe una correspoo-
dcncia biyectiva g: {l, 2, 3, ... , n}➔ A para algún n E z•. &ta función g proporciona una
enumeración de los elementos de A, como g(l), g(2), ... , g(n); enumeración en la que podemos
contar un primer elemento, un segundo elemento, ...• etMera, hasta un n--ésimo {último) clcmemo.
Además, cuando A es un conjunto infinito, vemos que no existen E z+ para el que podamos
encontrar una correspondencia uno a uno/: A ➔ {l. 2, 3, ... • n}. Pero si A, B son conjuntos
infinitos. ¿podemos concluir en forma automática que 1= IA IB 1{es decir, que existe u n a ~
pondencia biyectiva entre A y B)? Ésta es la pregunta que contestaremos, en sentido negativo, al
continuar nuestro análisis. Por ahora, presentaremos ()(ro tipo particular de conjunto.
Apéndice 3 Con·untos numerables no numerables A-29

nición A3.4 Un conjunto A es n"1Mrabú (o contable) si (1) A es finito o (2) A - Z...

Hemos vmo que 2Z• - z · y 3z· - z•. y como z• - z•. se sigue que los 00llju.ntos z·. 22• y 3z •
,on conjuntos nuinenl>lcs. De z·
becbo, para cualquier k E Z , k I O, la función/: ➔ tZ· dada por
/(x) = a.es una correspondencia biye<tiva.de modo que kZ• es numerable (y ikZ• i = ¡z• ¡). En
consecuencia. el conjunto de todos los enteros negativos (es decir, (-1 )Z·) es un conjunto numerable.
Además. si A es infinito y A - Z.., tambióitenemosqucZ·-A. porloquecxisteu.nacorrespon-
dcucia biye<tiva/: z• ➔ A que proporciona una enumeración de los dementos de A;• saber,/(().
/(2)./(3), ... ; de es<a fonm. podemos e= loselementos de A (pero nunca terminar de hacerlo).
Por ó.!timo, como observamos antes. si A - r. tenemos que Z.. -A. En coosecuencia, podemos
demostrar que un conjunto dadoA es infinito numerable (es decir, infinito y numerable) si cncontr1.-
mos una corrcspondeocia biyectiva/: A -+ z• o una correspondencia bíyectiva /: z• ➔ A.

Como z•t - l)Z• y {O} soo numenbles, ¿es numerable Z = z• U (-l)Z' U {O}?
Consideremos la función/: Z.. -+ Z dada por

x/2. para • par


1
f(x)• - C,- - 1)12. paru impar

Aquí vemos, por ejemplo, que

/(4)=4/2 - 2 y /(3)- -(3-1)/2 - -212- - 1.

Afumamos que/es una comspoodencia biye<tiva tal que/(2Z1 = z • y /(Z• -2:l') = (- l)Z• U
{O}. pues supongamos que a, b E z• oon/(a) =f(b).
1) Si a, b ,on pa,es. enlo11CC$/(a) =/(h) = a/2 •b/2 =a = h.
2) Sia, hson impa,es,CDIOOCeS/(a) = /(h) =-<• - 1)12=-{h-1)12 =• -1 • h-1 =• =h.
3) Si aes par y b impar. entooces/(a) = /(b) = d2 =-(b- 1)12 =•=-h +I = • - 1 = -h.
con a - 1 ~ l y - b < O. Por" lo tanto. este caso no puede ocurrir~ al igual que el caso en que
aesimparybpaL

F.o. consecuencia, la función/ es al menos inyeaiva.


Adcmá.<. para cada y E Z.
1) si y=O. CDIOOCC$/(I) • O;
2) si y> O, CD100CeS 2y E z • y /(2y) = 2}·/lz y. y
=
3) si y< O, entonces -2y + 1 E Z- y /(-2y + 1) -{(-2y +1)- IV2 =-{- 2y)l2 • y.
Así,/es tambi~ una función sobre y/: z•-+ Z es una correspondencia biyectiva. Por lo tanto,
Z es numerable.

Aunque todos nuestros ejemplos de conjuntos infinitos numerables han sido subconjuntos de Z,
tambil:n existen otros conjuntos infinitos numerables posibles.

•l Sea A= {l. 112. 1/3, 114•... } = 11/nln E z·1. La funcido/: z· ➔ A dada por/(n) = 1/n
establece una correspondencia biyectiva entre z·
y A. Por lo tanto, IZ· 1 • IA I y A es
numcDble.
A-30 Apéndice 3 Conjuntos numerables y no numerables

b) La función g: z+ ➔ Q•ta1 queg(11)= l/(3n) es inyectiva y aunque podriano scrunacomt-


pondencia biycctiva. tenemosqueg1: z+-+ {l/3. 116, 1/9, . . }(C q), dondeg1(n) =,W
esunacorrespondenciabiyectiva,demodoque {l/3, 1/6.1/9.... } csunconjuntonwncnt&.

Pana avanzar un poco más en nuestro desarrollo de los conjuntos numerables, presentamos la
siguiente definición.

Definición A3.5 Paran E z•, una sucesión finita tkn 1in1'1.UWs es UD?. función/cuyo dominio es {l. 2, 3•. .. ,n}.
Escribimos esta sucesión por lo general como un conjunto ordenado {xi.Xi,XJ, .. . ,.r,.} ta1 que.X¡ =
1(1) para todo 1 :s i s n.
Una suusi6n infinita es una función g que tiene z• como dominio. Por lo general, denotamos
este tipo de sucesión mediante el conjunto ordenado {..r.hu· o {xi.x:.xJ, . .. }, donde.r;=g(l)para
todoi EZ•.

a) Podemos pensar que el conjunto { 1, 1/2, 1/4, 1/8, 1/16} es una sucesión finita. dada por b.
función/, A ➔ <r =z-••.
tal que A = {l. 2. 3, 4, 51 y /(n)
b) El conjunto A del ejemplo A3.4 también puede expresarse como { 1/n}..a-, una sucesión
infinita dada por la función g: Z" ➔ Q.., tal que g(n) = 1/n para cada n E z•.
e) Los táminos de una sucesión no tienen que ser distintos entre sí. Por ejemplo. sea/: z•➔ z.
dondex.=/(n) = (- 1),...1, para cada entero positivo n. Entonces {x.}. ..r = {x.,xz, x,,.x...
x 1, • • • } = {l,- 1, I, - 1, 1, ... } pero la imagen de/está formada solamente poreJ conjunto
con dos elementos,{l, - 1}.

Nuestro siguiente resultado relaciona los conceptos presentados en las definicionesA3.4 y A3.S.

TEOREMA A3.2 Si A es un conjunto numerable no vacío, entonces podemos escribir A como una sucesión de ele-
mentos distintos.
Demostración: Debemos considerar dos casos.
1) Si A es finito, entooccsA - {l, 2, 3, ... , n } (y {l, 2, 3, ... , n} -A) para algán ne z•. Por
lo tanto, existe una correspondeocia biyectiva/: {l. 2. 3, ...• n} ➔ A.
Definimos a;=.j(1) para cada 1 S i S n. Entonces, como/es inyectiva y sobre, {a1,a2,
aJ, ...• a. J es una sucesión de los n elementos distintos de A.
l) Si A es infinito existe una correspondencia biycctiva g: z· ➔ A.
Definimos a.= g(1) para todo i E Z". Comog es inyectiva, los elementos de la sucesión
infinita {ai, a 2, a}, . .. } son distintos; {ai, az. a}, ... } = A pues ges sobre.

Antes de continuar, regresemos un poco para recordar que Z ... es numerable, al igual que los
subconjuntos (de z•) 2z• y 3z•. Esto sugiere la posibilidad de que todo subconjunto de un conjunto
numerable sea tambitn numerable. Para analizar esta posibilidad. presentamos las dos ideas si-
guientes.

Definición A3.6 1) La sucesión infinita {a1,a2, a3, •• • J = {a;);u· es unasubsucesiónd~z· si paratodoi e z....
a;E Z•ya, < a;. 1 •
2) Sean {x.}. u • y b.}. u.·dos sucesiones infinitas. Decimos que {y.}~u• es un.a subsuasión
c1cz·
de tx~}•t::t· si existe una subsucesión {a1 hez• tal que para cadak Ez•.
tcnemos)'l=X..,·
Apéndice 3 Conjuntos numerables y no numerables A-31

a) {1 , 3, 5, 7, ... } es una subsuces;ón de Z', al igual que (1, 2. 4, 7, 11, 16, . .. }. La primera
subsuccsión está dada por la función/: z+➔ z+tal queª•= f(n) = 2n - l. Podemos generar
la segunda subsucesión en forma recursiva. como
1) c,=h(l) = l ;y,
l) c..1 = h(n+ l ) = h(n)+n=c. +n.paran~ l.
b) Sean {x.}.u-• y {y.l.t:rdos sucesiones tales que paracadan E z•, x.=/(n) =(-1)'"+ (l/n)
y y. = g(_n) = 1 +(l/(2n)). Así, (x,J,.,.. = (O, 312, -1/3, 5/4, - 4/5, 7/6, -6/7, 9/8, . .. } (y
{y. J.u• = {312. 5/4, 7/6, 9/8, . . . }) y'y.= zi.para toda n E z+. Para la subsuC:CSión {a,Jtn·
(de Z } tal que a.,= 2k paracadak E z+, tenemosquey. =z:.. = z.,_para cadan E z•, loque
muestra que {y.}.u• es una subsucesión de {x,,}.er•
e) Paran E z ·, sean x.=
l/n y y.= l/(3n). Entonces {x.l.er = { 1, 1/2. 113. 1/4 , 1/5, 1/6,
in, . . } y {y.}.u• = {113, 1/6, 1/9, ... }. Consideremos ahora la subsuccsión {at}u: r (de
Z } tal quea1 = 3k para cada k E z·.
Entonces, para cada n E Z"",y. = l/(3n) = .rJA-= x.,_, de
modo que {y.}.&cr es una subsuccsión de {x,,}. u•.

Pasemos ahora al siguiente resultado relativo a los conjuntos numerables y sus subconjuntos.

TEOREMA A3.3 Si Ses un conjunto numerable no vacío y A !: S. entonces A es numerable.


Demostración: Si A es finito, entonces, por la definición A3.4, sabemos que A es numerable. Así,
podemos suponer, de aquf en adelante, que A es infinito. Como S es numerable, podemos utilizar el
teorema A3.2 para enumerar los elementos de S como una sucesión infinita de t&minos distintos.
por lo que escribimosS= {.ri, s2,si, ... }. Ahora. definimos una subsuccsióo {a.}..u· de z+ como
sigue:

ª• = nún{nln e z•. y s. e A}
Oz = mfn{nln e Z..., n > a 1 y s. E AJ
a,= mfn{n jn E z.. n > a2 y s. E A}

En genera], una vez seleccionados ai. a2, a1, ... , a ., defirúmos a,.1 = mfn{njn E z+, n > a, y s. E
A}. Consideremos la "'función" F: z• ➔A dada por F(n) =s._. Sim, n E z•. vemosquem=n ⇒ ª•
=a. ⇒ s. .. : s._ ⇒ F(m) = F(n), por lo que no hay duda de que F es una función. Para terminar la
demostración de que A es numerable, necesitamos mostrar que Fes una correspondencia biyectiva
Supongamosquem,n e z •con F(m) = F(n). EntonccsF(m)=F(n) ⇒ s.,_ =s•• .⇒ a.= a. puestO
queloselementosdela succsiónS = {si, s2,s1, . . . } son distintos.Además.a.= a.⇒ m =n, porque

los elementos de la subsuc.esión {a.},.g• de también son distintos. En consecuencia. esta función
F es inyectiva.
Ahora. seab EA. Como A C S={s1, s 2,s,, ... } podemos escribirb = s. para algúnm E z+. Si
m = ai, entonces F(l) = s'"' = s.= b. Si m #-ai, entonces. como a1 < a1 < a3 < ...• existe un r E z•
mínimo tal quea,_1 <m s a,. De ladcfi.-i.iciónde tasubsuoesióo {a,,}.ez-sabemosquea,= mín{tlt EZ♦•
t>a,. 1 y s, EA}(ycomom > a,._ 1 ys. EA,tenemosquear S m).Ahora,a, S m ym .s a, ⇒ a,=m,
por lo que F(r) :s., =s.=b. En consecuencia. la función Ftambitn es sobre.

Deducimos del teorema A3.3 que un conjunto dado no vacío S es numerable si y sólo si S tiene
el mismo cardinal que un subconjunto de z•. Así, si existe una función inyectiva/: S ➔ z+ (no
necesariamente una correspondencia biyectiva), esto basta para afirmar que S es numerable, ya que
p
S - /(5Xo I S 1= 1/(S) y /(Sj es numerable.
A-32 Apéndice 3 Conjuntos numerables y no numerables

Hasta este momento, todos los conjuntos infinitos analizados han sido numerables. ¿Sen que
todos los conjuntos infinitos son numerables, y que para cualesquiera dos conjuntos infinitos A. B
tenemos que IA 1-= IBI? El siguiente resultado aclarará este punto.

TEOREMA A3.4 El conjunto (0, I] = {zlz E R. y O<x s 1} ooes numerable.


Demostración: Si (0, l] fuera numerable, entonces (por el teorema A3.2) podnamos escribir este
conjunto como una sucesión de ~rminos distintos: (0, 1) = {r i, r2, r,, r, , ... }. Para evitar dos
representaciones, coincidiremos en escribir los mlmeros reales en (0, 1) como O.S como 0 .499 ... ,
de modo que OÍDgún elcmen10 de (0, IJ quede represemado por un desarrollo decimal !inilO. S,
escribimos este desarrollo decimal para ri, r1, r,. ... , obtcocmos

T1 = O. l211 0 1.2 l21J l214 • • •


r1 = 0.ou a22 a:,ai,. ...
T3 = 0.0 J1 On 12D 123' •••

r,. = O.a,.101120,-,a,.. . ••

donde .. E (0, I, 2, 3, ...• 8. 91 para IOdos i.j E z•.


Consideremos ahora el n'l1mero real r = O.b1 bz b, . . .• donde para cada l E z•,

b,•{3, si o., ,'3


7, SI Ou • 3.

Entonces, r E (0, l], pero paracadak E z.,


tenemos r ;,. rb por loquer f. {r., r 2, , ,. r,, ... }. Esto
contradice nuestra hipótesis de que (0, l] = {r., , 2, ,,, r,, . .. }.

La ~ c a utilizada en esa demostración (del teorema A3.4) se coooce por lo general como la
con.rtrutti6n diagOMl tk Ca,it()r, en honor del matemático alemtn (nacido en Rusia) Georg Cant«
(184►1918). quien presentó la idea en diciembre de 1873.
Cuando un conjunto noes numerable, decimos que csno nwnerable. Así, (O, 1] es no numerable-
Si un conjunto A. es no numerable, entonces (1) z· y
A. no tienen el mismo tamai\o, o cardinal, de
modo que Z•f A ye! cardinal de A es mayor que elde Z~esdecir, IAI > Jz ·¡. aunque A y Z·sean
conjuntos infinitos.
El siguiente corolario presenta otro ejemplo de un conjunto no numew,tc.

COROLARIO A3 .1 FJ conjunto R (de todos los mlmcros reales) es un conjunto oo numerable.


Demostra<ión: Si R fuera numerable.entonces por el teoremaA3.3, el subcoojuntO (0,1) dcR sería
numerable.

Antes de continuar con algo nuevo, diremos unas cuantas palabras acerca de este conoepco de
conjunto no numerable.
1) En primer lugar. debemos observar que eJ corolarioA3. J es un caso particular del resultado
general: Para cualesquiera conjuntosA,B, si A es no numerable y A ~ B. entooccs B es no
numerable.
Apéndice 3 Conjuntos numera~ y no numerables A-33

2) A cfilettnda del r=ltado del teorema A3.3, no ocurre por lo geoeal que los subconjuntos no
vados de los conjuntos DO numerables sean DO numerables. Incluso tenemos un subconjunto
infinito A de un B DO numerable ta1 que A es numerable; por ejemplo. sean A • Z y B ;; R..
3) Siguícndod -=aA3.3obscn-amosqucc:uandolellemos unconjullloAypodemoo~
unafunci6o iny,,;,iva/:A ➔ z •,...,,._da,njwmA tienequcwmm:nble. Nopod,mos
r
in..,,n-los papdesdcAyZ-eu la función/. Si bayunafunci6o ÍDy«IÍ. .g: ➔ A,elcoojunlo
A podrfa ser llO lllllll<nblc. Basta coosidctv g: z•➔ R tal que g(.t) = X para cada X E z•.
4) Consideremos los puntos en el plano cartesiano sobre d círculo unitario x2 + (y - 1)2 = 1.
¿De qut tamaJlo es este coajunto S = {(x.J) 1.r, y E R y x'+ {y-1)'= 17, es decir, ¿es S
numerable o no numerable?
En ta figura A3.1 tenemos un drculo unitario (en el plano) centrado en C(O,l). Este d:rculo es
tangente a la recta num&ica real (o ejex) en el punto dondez =O.El punto P , sobre la circunferen-
cia, tiene coordenada$ (0, 2).

y
PI.O, 2)

u· -2 -1 p• o T' 2 3 Q'
Figuro Al.1

Sea (z. y) cualquier puntO sobre la circunferencia del clrcwo unitario. distinto del punto P(_O. 2).
Por ejemplo. d pwuo Q es uno de taJes puntos y Res otro. Trazamos la recta determinada por P y
Q. Esta rocta intcrseca el ejez en Q'. De la misma forma. la recta dctenninada por P y R in1crseca el
ejex en R'. Rcáprocamc:me. axuideremos los puntos sobre deje x. e,tcepto el punto tal que z = O.
Dos de estos puntos son T y U'. La recta que pasa por P y T inlerseca el círculo unitario en T. El
punto U es el punto de intc:rseccióo (sobre la circunferencia) determinado por la recta que p&a por
P y U'. Por óltimo, observemos que P' (sobre el eje x, donde x == O) se corresponde consi¡o mismo.
De esta forma, obtenemos una correspondencia biyectiva entre los elementos deS y el conjunto R.t
Por lo w,10, IS
1= 1R 1· de modo que Ses om, conjunto 110 numerable.
Resunúcndo lo que sabemos de IZI IR1, c,decir, que IZI< 1RI, ahora queremos dctcnni-
y
narsi IQ ZI IQ IR!:
1= 1 o 1= o, tal vez.¡z¡< IQI < IR!. Para <$to demostraremos un punto
mú general a panir de lo siguiente.

TEOREMA Al.5 El conjunto z •x z+es numerable.


Demostrodón: Dclinímos la funcióo/: z •x z• ➔ z • «<00/(a, b) ~ 2'3'. El r=ltado scticlcttosi
podemos dcmostruquc/es inyectiva. Para(m, n), (14 u) E z -x z •. /<,,,, n) =/(14 u) => 2"3• = 2'3u
:=- m;; u, n = u, por d teorema fundamental de la ari~tica. En consecuencia,/ es inyectiva yz·
x z• es numerable.

t Obsavcmos que la circwtfcrencia wñt1ria compúro (Cl decir, la qoc incluye el punto P (O, 2) sólo
coatieDc un punto mis que el conjwuo S. Se puede demosnr que. de hecho. el cardinal de la cin:unfereoc:ia
compldaosipaJa ¡s¡.(N.d,/~)
A-34 Apéndice 3 Conjuntos numerables y no numerables

Antes de hacer una afirmación acerca del tamaño, o cantinal, de Q , necesitamos considenr
primero el subconjunto Q n (O, !]=(sis E Q yO<s s !} de Q.

TEOREMA A3.6 El conjunto Q n (O. I) es numerable.


Demostración: Primero debemos establecer que escribiremos cada s de Q n (0,1] en la forma
(Wlica) plq, dondep. q e z+y no tienen divisores comunes distintos de I.Ahora definimos/: Q n
(0,1] ➔ z • xz•corno/(plq) = (p. q), y sea Kla imageo def Paraplq, uJ-o E Q n (O, ! ], tenemo,
que f(plq) = f(u/-o) ⇒ (p, q) = (u, u) ⇒ p = u y q = u ⇒ p/q = ulu, por lo que/ es una funci6II
inyectiva. En consecuencia. Q n (0. 1] - K , un subconjunto del conjunto numerable z • x z•. Del
teorema A3.3, sabemos que el conjunto Q n (0, I J es numerable.

A medida que continuemos con nuestra tarea de determinar I Q 1, necesitaremos las dos defini-
ciones y el teorema siguientes.

Definición A3.7 Sea <; una colección de conjuntos de un universoGU.. La unión de todos los conjuntos en':T, que se
escribe UA 0 A, se define como {xl.r E '\l. y .x E A, para algún A E 5"}.
Si<; es una colección numerable, es decir,~= {A1, A:,A,, .. , }. podemos escribir LJAnA. =
U:,A.. = U,_A,..

llll!lí:lid En los siguientes ejemplos, el universo '\l. es R.


a) Para cada n e Z.... sea A. = (n - l,n). Entonces, por ejemplo. A1 = [O, l),A 2 =[l, 2) y A. 3=
(2,3). Para~= (A,.A,, A, ... } ={A, 1iEZ'}tenemosque U,..A = U:,A.. = U..,.A,..=
10, + ~l-
b) Dado cualquierq E Q·, sea A,= (q-112.q + 1/2). En este caso, por ejemplo, A 112 = (0, 1),
A. = (712, 9/2) yA,.0 = (19/6, 25/6). Si~ = [A, 1q E Q•}, entonces LJ, 0 A = LJ...,.A, =
(- 1/2, •-).

Definición A3.8 Sea "3 una colección de conjuntos tomados de un universo <i\1. La colección ~ es una colecci6n
disjunJa si para todos A., B de 'il", si A -:/; B, entonces A n B = 0.

~ ' Si volvernos a examinar las dos colecciones del tjemploA3.7, veremos que la colección de la pane
(a) es la única colección disjunta.

Los conceptos de conjunto numerable y de colección disjunta de conjuntos se rtúncn ahora en


nuestro siguiente resultado.

TEOREMAA3.7 Sea~ una colección disjunta numerable de conjuntos, cada uno de los cuales es numerable. Enton-
ces UAU A tambitn es un conjunto numerable.
Apóndice3 ·untos numerables no numerables A-35

Dtmostrad6n: Como~ es una colección disjunta numerable, podemos escribit(.J = {Ai, Ai, A» ...}
doodeA.,nA,•. para todosi,j E z•. si i P,j. Ademis, paracadan E z •. A. es numerable y puede
exprosarse como {a... a.,,.......' una sucesión de !&minos cnstintos. Pan mostrar que u••
.A es
numerable.consideremos cadaz EU,.u A.
Como LJA.,.A=lJ:_1A,.. tenemos quex E A. paraalgánn E Z"' (fijo); e5te n es ónicodcbido a
que~ es una coleccióo ctisjuota. Ademú, X E A. =>X= . . . para algw, k E z·
(donde k esú fijo y
es ónicoi Definimos ahora/: U,..A ➔ z•x z• como /(,e)• /(a..,} = (-. k). Del -=a A3.S
sabemos que x z· z• es nu.merable. por lo que ta imagen de/ es o\l.JDCm)lc. En consecueocia. et
resultado será válido una vez que demostremos que/es inyectiva. Esto es fácil de demostrar, ya que
six=a..,y=a,, E U,.,A, con/(x) =/(J),entonca/(a.,J =/(a,,) => (n. k) • (p. q) => n=p, k=
q=>a,.=a,,,=>x=y.

Observe que la demostración del teorema A3.7 es vtlida sá ~ es finito (y recmplaumos - con
1~ p o si uno o más de los conjuntos A• i E z•.
es finito.

Como rm,11-!o del teo«ma A3.7, podemos irabajar ahora con el cardinal de Q.

TEOREMAA3.8 El conjunto Q (de todos los ru1meros racionales) es numerable. ·


Demostradón: Comencemos pou<eordarqueAo• Q n (0, 1] es mnnerable (teo<emaA3.6).Ahora
bien. paracualquierc:nteron distinto de cero. seaA.• Q n (n.n + I] y definamos/.: A.➔ Aocomo
f,l..q) • q - n. Entonces J.(q1) = f.(q,J => q 1 - n = q 1 - n ~ q1 = q2, por lo que f. es inyectiva En
consecucncia, A. - /.J.AJ !:: Ao y poc el teorema A3.3, tenemos que A. es numerable. Ademis, para
cualesquiera,._ ne z.,. ,/, n =>A. nA.= e. DclejemploA3.3 sabemos que Z esnumenble. poc
loque';f = {4A.1,A.0 A1,A.:. ... } es unacolcccióa disjuntanumerablcdeconjuntosnumerabJC$.
Por)o tanto, por el teorema A3.7, se si¡ucque UA.,.A. = U...xA. = Q es numerable.

Asf, ahora sabemos que z•, Z y Q son infinitos y z• - z - Q, mientras que Res infinito y R +
z•.Recordemos que cualquier conjunto infinito A., tal que A. - z•,
es infinito numerable; ahora
denowemos el cardinal de un conjunto de este tipo como IA =
1 K,. usando la letra hebrea aleph.
coa el subín<tioc O. para denotar el primer nivel de infinito. El cardinal de R es mayor que Ko y 5C
denota genenlmentc con e, de conrinUQ.

En nuestto siguiente teorema mejoraremos c1 resultado del te0remaA3.7. El siguiente lema oos
ayudad a mejorarlo.

lfMAA3.1 Sea~= {A.1oA:,A.:,, ... } una colccci6n numerable de conjuntos (de un univcrso"1). Sea"°'= {B1o
8 1, 8,. ... } la colección numerable de conjuntos tales que B1= A 1 y B0 =A. -u;::A.,
paran~ 2.
Entooces ~es una colección disjunta numetable y lj._ , \ 2
1
U;.
1 Bt.
DemosttKión: Primero estableceremos que la colcccióo numerable~ es disjunta. Par2 esto, debe-
mos mostrar que para cada i. j E z• tal que i 'F-j. tenemos 8 1 n B1= t. F.n caso contrario. sea i <j tal
que B1 n B1= t . Para cualquier x E B, n B,. tenemosqucx E B,•A.1 - LJ;'.:~At ⇒ x ~ A.,. pues 1
:Si :Sj-1. Pcrotambim.ocuncqucxEB1=A.,- lÍ.:,'1A.- ⇒xeA.,. puesA;-U:,11 .¡\ CA.r(Nota;,
U:, A. = t s; i • 1.) La contradkción <,e 11c A, y x E AJ inctica que 8, n B¡= f para todos i. j E Z"
tal que i /:- j. Así. '"1; es un.a colección numerable disjunta de conjuntos.
A-36 Apéndice 3 Conjuntos numerables y no numerables

Para la segunda parte {es decir. para LJ;. 1.\- =LJ:'-1 Bt)• comencemos conx E u;_, t\--
x E A. para algúnn E z•y scamel mínimodetalcsn. Sim= l,cntonccs.x EA1=81 C LJ;_1a._.
m > J,entonccsx (tA1 para todo 1 :sj :S m-1 y de este modox E A.-LJ;:t\- =B. C U:.18,;.&
cualquier caso, x E u;. 1
~ y LJ;'.,1..\- Cu;. 1
13.t. Para la inclusión opuesta, tenemos que y E
LJ;. = B.t y EB.,para algt1nn E z• (Onico)= y EA,.paraeste mismon E Z♦, pues81 =A1 y B,:
A; -
1
u::! = LJ:.
A.t C A;. para todo i ~ 2. Entonces y E A.. y E 1
,\,, de modo que LJ;.1~ t

LJ;.1, \ - En consecuencia, LJ:'..1, \ =u;_1s*.

Como en el caso del teorema A3.7, la demostración del lema A3. l es válida si t:.; es infiniro (y
reemplazamos ® por ¡..-
1).
Del lema A3. I. vemos que p<Xlemos debilitar las hipótesis del teorema A3. 7: la colección':/ no
tiene que ser disjunta. Esto se establece formalmente como sigue.

TEOREMAA3.9 La unión de cualquier colección numerable de conjuntos numerables es numerable.


Demostración: Si ~ = (A1, A:tt A,, . . . ) es una colección numerable de conjuntos numerables,
construimos la colección numerable~ = {B1• Bz. 8 1• • • • } como en el lcmaA3.l. Para cadak E z·,
Bt C Ai. por Jo que (por el teorema A3.3) cada Bi es mJmerable. El lema A3.l sei\ala que
LJ:;..1" =LJ:.1 1J.t y, por el teorema A3.7. sabemos que LJ;_,8- es numerable. Por lo tanto,
UAuA =LJ;.1" es numerable.

De nuevo, si-;; es finito. la demostración del teoremaA3.9 sigue siendo válida (reemplazando cada
ocurrencia de ® por ¡..-p.
I ¡ IR
Despu!s del teorema A3.8 mencionamos que z· = Me y 1 = e, donde Me< c. Aunque aún
hay mucho que decir de los conjuntos infinitos. cerraremos este a~ndice mostrando que éstos DO
son los únicos nómeros cardinales infinitos. De hecho, existe una infinidad de n(imeros cardinales
infinitos.

TEOREMA A3.1 O Si A es cualquie, conjunto. entonces IA 1< \:'i'(A) 1.


Demostración: Si A = 0. enconces IA 1= Oy :'i'(A) 1 = 1:'i'( 0) 1= l 10} 1= l , po, lo que el re,ultado
es cierto en este caso. Si A#- 0, seaf: A ➔ ~(A)dada porj{a)={a} para cada a E A. La función/es
IA IA
una función inyectiva y se sigue que 1 = 1/(A) 1 S l:'i'(A) 1· Para mostrar que 1 ~ l:'i'(A)I,
debemos probar que ninguna función g: A ➔ ~(A) puede ser sobre. Así, sea g: A ➔ 1(A) y consi•
I
deremosB= {a a EA ya ~g(a)). Rccordemosqueg(a) CAyqueB CA. SiB E9'(A) y ges una
función sobre, debe existir a· E A tal que g(a') = B. ¿a' E g(a') o a'~ g(a')? Exactamente uno de:
estos dos resultados debe ser verdadero.
Si a' E g(a') = B. entonces, por la definición de 8 lenemos que a· ft g(a '). y la contradicción a'
E g(a·) y • · E g(a·). Por otro lado, si • • E g(a). entonc<s • • E 8, pero B =g(a), por lo que
nuevamente obcenemos la misma contradicción.
Por lo tanto, no existe a' E A tal que g(a') = B. por lo que g no puede ser sobre y por lo tanto
IA I< ¡3'(A)J.

Como consecuencia del teorema A3. IO. vemos que no existe un número cardinal infinito máxi-
mo. ya que si A es cuaJquier conjunto infinito. entonces jA J< l~(A)I < 1;1@>(A))I <.. Sin
embargo. existe un número cardinal infinito mínimo que, como mencionamos antes. es Xo-
Apéndice 3 Conjuntos numerables y no numerables A-37

BIBLIOGRAFÍA

Como todavía hay más que decir de los conjuntos numerables y los no numerables, el lector intere-
sado podría buscar más información en los siguientes libros.
1. Endcrt~ Herbert B., Elmwus ofSet Theory, Nueva York. Acadcmic Press, 1m.
2. Halmos, Paul R., Naivt Set 'IMory, Nueva York, Van Nostrand. 1960.
3. Hcnle, James M., An Ou1line oj Set T'Mory, Nueva York, Springcr-Verlag, 1986.

EJEROCIOS A.3 1. Determine si las siguientes proposiciones son verdaderas o falsas. Para las partes (d)-{g), ~
un contraejemplo si la proposición es falsa.
a ) El conjunto Q"' es numerable. b) FJ conjunto R• es numerable.
I
e) Existe wia conespondcocia biycctiva emrelos conjuntosN y 2Z = {2k k E Z }.
d) Si A., B son conjuntos numerables., entonces A U Bes numerable.
e) Si A, B son conjuntos no numerables., entonces A. n Bes no numerable.
f) Si A, B son conjuntos numerables, entonces A - Bes numerable.
g) Si A. B son conjuntos no numerables., entonces A - B es no numerable.
2. a) Sea A = (n' 1n E z•¡. Encuentre una corn,spoodencia biyectivaentre z •y A.
b) Encuemre wiacorrespondencia biyectivaentre z• y {2. 6. 10.14• ... }.
3. SeanA, Bconjuntos tales que A. es no numerable. Si A~ B,.demuestrcqueB es no numerable.
=
4 . Sea f {r E R lr es irracional} = R -Q. ¿Es /numerable o no numerable? Demuestre su
afirmación.
S. Si S, Tson infinitos y numerables, demuestre queSx Tes numerable .
6. Demuestre que r xrx Z-= ((a. b. ella. b, e E r ¡ es numerable.
7. Demuestre que el conjunto de todas las soluciones reales de las ecuaciones cuadráticas ar 2
+ b.r + e = O, donde a, b, e E Z, a I=- O, es un conjunto numerable.
8. Determine una COrTCSpondcncia biyectiva entre el intervalo abierto (0, 1) y los intervalos
abiertos (a)(O, 3); (b)(2. 7); y (c)(a. b), donde a. b E ll y a < b.
FÓR MULAS
n! n factorial: O!= l; n! = n(n - 1) . .. (3)(2)(1), n E z +
P(n,r) el número de permutaciones de n objetos tomados de
ren tr, OS r:S::n. [P(n, r) =n!/(n - r)!]
C(n, r) = (;) el número de combinaciones o selecciones de n objetos
tomados de ren r, OS rS n. [C(n. r) = n!/[r!(n - r)!J]
el número de combinaciones o selecciones de n objetos
tomados de r en r, permitiendo repeticiones (r ~ O)

El teorema del binomio: (x +y)" = (¡)xºy" + (!)x'y•- 1 + • • • + (:)x"yº

(•;') = (:) + (,=,).


S(m,n) = (1/n!) ¿,-o(-1)'(.=,)(n - k)"', un número de Stirling del segundo tipo.
S(m, n) es el m1mero de formas de distribuír m objetos distintos entren recipientes
idénticos sin que ninguno de los recipientes quede vacío.
(-:) = (-!)'("';-•). n,r ez+
f(x) = tlo + a,x + a,x' + a,x' + · · · :f(x) es la función generatriz (ordinaria) para
la sucesión ao. ª" ai, a3, .. .
ParaaER;m,ne z +

(1 + x)' = (i) + (!)x + (¡)x' + · · · + (:)x'


(1 +ax)'=(;)+ (;)ax+ (¡)a'x' + · · · + (:)a"x"
(1 + x")" = (¡) + (;)x"' + (l)x"' + .. . + (:)x-
(l - x"")/(1 - x) = 1 + x + x' + · · · + x"

1/(1-x)=l+x+x' +r' + ··· =Z:x'


i• O

1/(1 - x)" = (-0•) + (1")(- x) + ( 2')(-x)2 + ( 3')(- x)' + • • •


. .
= L ("i')(-xY=L ("';-•)x'
f•O i•O

g(x) = tlo + a ,(x/1 !) + a 2(.ri/2!) + a,(x'/3!) + • • • : g(x) es la función generatriz


exponencial para la sucesión ao. a1, a2, a3o .

x' r'
e"= l+x +-+ - + ···
2! 3!

(-21) (.-'+,-')=l+ -2!x' +-+


x' ...
4!
- e"-e -,) =x+-+-+
G)( r' x' •··
3! 5!
F.,n a:cO el n-ésimo· número de Fibonacci: F0 = o. F1 = I y
F.= F.-1+ F.-2, n ~2
b,..n2::0 el n-ésimo número de Catalan:
b. = (.¼,)(~), n "'O
Índice de materias

t(elt«Jjutovado).100 priDdpio de dlwidad, 764 psa b . ~ d e ~ - 4 6 7 . 4 6 8


i!(G)."1 propiedlda de UA. 764, 165 recllf'IÍW>, 467. 481,489
,CG).603 __,_,-..n1 recwsivo4dmúimo~ciril«. 469
O(G).689 leetU!Mldit~ 761.ffl voru. 6S&. 66S. 667. 669. 694
~ ) (la f\lAd6a pbi de & .->. 409--411. 720. alsorism. 231 Üln. di a l.ft,ol COA raíz. 629
779 al¡oritmo(I). 44, 4S, 227. 238. 239, 29), 294. A-riC4JI JOJUMl o/ Motlwmalia, •21
X(G). S81-'9I 297,305, SIi. Sl2 ~ Zdtfoulturit', JJO
o<(G),'34 amlisis . . l. 2-U. l:60. 296--305. 310. lll. ..UOC•l. 73.S. 778. 779.135. 846

"'-~-.,.
). (la cadena vma). 316 485.416,,A-S booJt.alt6. n1.732
1.• .s91 compwdomks y propiedades en la leOrÚI COll t"lcmem:ouni.dad, 70J,8JJ.8J.8
~-I.. ~ .316 de p-a/0$, 600 ~vo.103.137
t , r .111 ~voeoo~mudad. 773
•G>.""
-m
biuria.Sl7-Sl9 de .-rica. 702.10). 730
enancbura,6U de polillOIDios. 83.S. 837, 871
A•.A•, A•,A•. m d<-.0. <S
• •Hllltld•). 717 C9i -..11&& ao ~ )OO. )01
• r s ~ c : o n l, m6daJoa, 717 d e ~ dcu sbol deHmfma., 642. M hvun,otitM ;. t:w LllWf of n-1-". oa
Abd , Níeb ~Drit. 730. m.m. 172 643 Mid A.re Foudt d rM llorlt,_ticol
-.216 de co,ueo de irbolu etiqii;ctado, !lo r-orK1<,fl.og~udPf'flbobility. 138

-
-=-reo.lJl. 746 isomorfo.. 611,614 A..al~ofdwllWlofnc...,1t1. J16.
~350 - ~ I 0 4:...Ut--'6ttecrla m.m
Ackema=. Wilbdm. 260 al¡ebnic.t « la ~ ANStfORTRAN. 14.S. 33'
de d e ~ dc pu.DIOS de aniealad6n. -.._400
dade. 363. 530 647, 651 Arlloa,How.d.A,24
MCD.363.5)0 de Dijbn. 657. 660. 665 .-.Ptu...171
Abo. Al&td v.• )9S, 396,523,600.601, 6SJ. «t.c:lide:t.. ap&ad69. 2S1: ~ ~ fuci6o
6S2. 670,694,695 pmaella'0&.226,229,l93.468.469.S22.7l9 Appel. KüDcdi, S89, 599-601
ll-jalr.231 paapoliaoajos.S4S Aquib.139
Al-lQlowlrizmt. A11u lli'far Moll.ammtd itl• deupoaacilci6o..30J átiot. 24. ◄99. 598. 607-612. 6 14-631. 6.lA-
Mild. 231. 239 dcuaserddaderbüc.~635 6'8. 650-65.S. 651. 665-670. W
Albm, A. Adl'i&D.
alcaDo.610
uq,b.3<9-310
m. 873 deKtustal,665-668
de bdivili6L 213-218, 22S-227
pan, «IIUOS.. 215-2 11. 225. 226. 231.
..........
"'- a-..6211
• • (alepb teto).A,ll,A,36 ?SJ. 276. ?71. 29), 717. 7SS. 7t9 a-624

- "'-
alfu:ielo,23,316 pan potiaomios., 839-141. M5-147 de Knllbl. 666-668
dcCllCfalda.321 de ordtuci6n po, ~ 6.37
deAlida.lll ooa 1lm'CII adyacnlel. '467. 468. .Sll de Prim. 669. 670. 69S
p0f mcm6a,631.63' i-:ad-...:aodejrt,o,JesCliql,eado&.613.
d e las h:rláoca d e ~ 735 ~ns.m.394 614
deklsdraailOildeCOIUlllltacióa. n2 de Plim. 669,670.695 pait.t-ltiJ.1~u.riiYt"tU!dedireocioacs.
d e ~ 6 1, 63, 64: 'HGM ~ del CEDO mil corto. 657-664. 69-, 616
k,adebJ6sb. cid mwmo 00G11m di\'UOlt ?ZS-231 psalaOClllllnlCCade•jrt,d deHaffmm.
Alldn(s)boolcam(s). ns. 762-no. 772. m. ddprocedimiea(ode~676.6TI 642.60
135.871
6lc:codc:u. 767. 773
del proceso de mi1timi.z.aci6c, 389
dd li$remauivenaldedireicdoDcs.6l 6 .,,
para,lolpu.oiosdearóc:ulación.6◄7-6.sO.

.._.,.
de5.mci6e de. 762 diYKk-, ~ SJI-S20. 523, 63'
daaldcu, 764 kenñvo. 299, 676 aisiadeMr'OCa0.646
isomorfu, 768-769. 773 pan t-1 ll'bol ~ IJJWIIW. 66S. 669 ~ IIUll.615
itoalorfiuaodeua. 766, 76&. 769, 77) paradprooesodeml:nbaiución(~Mde binario.628
onb pwdal de u. 76S-767 csradol 6.ailos). l89 camplao. 6 17, 621. 6-o

,.,
-2 Índice de materias

eqailibrado.,634 dcttdlo.617 bicoodicioA&I. ,3, 56. 62. 121-123


Cliqrlc:Cado,639 lZqUic:rdo.617 Bigp, NonDID L. 44, 46, 600, 601
pan 1111. OOlllljlWO de peso&. 641 mcesi6Acanctms6ca. 6S2 Binet. Jac.ques P!iilippt. MMic,. S2S
OOIIII ralz. 499. SOO. 617 Y&tiee(s) bit(s).,,m

'""""""·""' «>1,-.609 bwlco (espacio).317. 318

-6'0
COR~ SQS.507. 523

--""
inter'.IIOS, 614 Blocbcr. Hc:idl. 732
cmnmal.614 b1oqoo,17
arco, 330,363 de uoapanieióa, 382
~.<:111 arpmano. SI. s&. 77. 79. 30, 124 dirfflo.17
combwtorio, -403, 404 CDUdiJalo.866
caaDCbura.62S-6Z7. 651
ca profudidld. 624. 628. 651
c&i.¡o ptffijo, 640. 641, 643. 651
dcpenaxuiadeu:aek.mtmo. 147, 15&, 160,
161 -.,,
iDditecto, 17

dueao.133 ttipkmmc..17
colmno&(deSMoru¡;t). 655 DO válido, &S, 93. 94, 127 Bouaocio.4'6
c:ompooellle bkoDeu. 645--648 poda Ul.vena, &S, 128 80Ddy, J. A..S99.60l.69S
coanú.61◄,619.628.629 por la redproc:a.&S. IT7 Boolc. George, 138, 139, 176, 178. 735,
de deeisi6o,. 630. 631 ~SI.SS. 77,79-80,81.8'-93. 127, 129; m
de Fibomoci. 653 vloM tmnbi-11' del:r!o$traci6G Boom. Tayb L. 773
coatm.653 adsta(s), 363, S30 ~ . Catl Wilhdm., 6SO
dmaid6G.6'11 de retroceso de un itbol, 646 Bortvta. Otüar. 694
de Haffmu.. 642. 643 dirigida. 330. 363 Bose. Raj Cllaodra. 858.. sn
~.61S ~530 bo.43
ciripdo,614 · DO dirigida. 36J ....... (117
c:iepM,e, 6S4-6S5 aosannda. 6TI ~.«J'l
eqailibrado.,629 satmada.673 Boya'. Carl Beojamill. 179
eóqucudo, 613 Arlsl6ce:les. In, 138,233 Braille (ffltema).29
bermaoo.61' Arithr,,w-rica lnus,a. 45 Broobbeat, J. Gleml, 343,344
llijo,61$ Aridl,,wric.:,, 239 Bruldi.RktiardA., 523
detoobo.617 AnC;.j~i.45 Buctlcy. Fm1. ,99. 601
izqmcrdo.617 ascendientes, 615 bualordcu.382
ook>.622 Aschbacber.Mkbad.131 Bus.acta, Robc:rt G., 695

_...
boja.614
-ario,628
compiclo. 628. 644

ai.veJ.615
Atkiu. Jod E.. 65t, 6Sl
itomo de u. .ilzd,n booleana. 161.
Auptai.StaD, 239,241
Aubd.f. C..476.523
aitodaal..164.165
m
búsqueda. '17
bi.ma. 517-S19
en~62S--627. 6SI
ea.p(C)fv;Ñdad.624, 628. 651
Bussey. W. H., 241, 8n
DOdosderuni6c.lcióo.,614 uiomas de Peano. 240 blltuo,610
IIWDCrodcnivcl,615 Byron. A11.p$tl Ada, 239, 240
(:,pcimo.641.642 Baae.S...3tl,6Sl , 6Sl.668.670.694,695 B)'f'Oll.Locd,239
8 ~ Clwk:$, 239 bytc, S,219
""'"
del dieciourio. 617 Bacbmaml, Pau.1 Gastav He:illlicb, 310
k ~ . 6 17 Bmx:ac.Darid.600,601 e: (CCIGWIUO). A -35. A-36
paralos'YMiCddelllltrbol.616, 617,621). Ba-aier. Wilham J.• 3-43. 344 c.c•.uJ
623 Barwise,JOtl,l39,140 ltv). 19,44
pouerior {recorrido). 620, 651 bue,A-1 d.,P, P').614
¡rtvio (rccorrido). 620. 624. 651 bue de 4atoJ relKional rrz. 311 c:adeoab). 23, 3lfi-320. 325-327, 399
am&rioo (reconido), 622 base d e l a ~ 1Myt86, 18&, 189. 196 caraacrbdca. 391-393
«dcucióa, 634-638, 651 base de UD SU.tema DCUDl!rico_ 218 rniaimal,391-393
ripd&.638 basel.218-220 CODCaCeDacióllde, 318
ordeaadocooraú,616 baseS. 218 de: paridad pa:, 342
-655 base 16. 219,220 dcRbgrupos. 872

_,..
pde-615 base de 1ILlll defimci6n recursiva. 202 firaitat,.317
pe$0deUllljJt,ol,64J BASlC. 6. 55. 60, 253.618. 726 igualdad de, 317
pesosi-a,t1:111irbol69ómo,64I Bedenba,;h.&h,,i.af.,832 iDVCQO, 325
pall(Ode~.64S.647,651 Bem.ad, Mebdi, 599, 601 l.(kcadcmvaáa).316
~614 Bell, Eric Temple, 524 kxl¡ituddc11..111, 316.318
tamM.616 Bdlmaa., R.. 585. 600. 601
reaaaidor, «n. 624,657. 665
CD profuACtidad, 64S-647
~ M.• SM. 600. 601
BerJe. Claude. 599. 6"
....-...m
¡,o{C!lciudc:. 318
~ - 66S-670. 694. 695 Ber¡cr, 1bomas R., 732. 872. 873 ptt:fijo,319
6pimo. 665,666.669, 670 Bernays.. Paul. 138
rmoceso,621.624
siseemallllÍ"Cnaldedireocioaes.616
8emoulli.J.w:ib. 4'
Bernouili, Johum, 308
.-..,20
propio. 319

propia.320
abirbol. 609. 615 Bcrtrand. Josepb Louis F ~ .50 sllfijo.319
Índice de materias · ¡.3

pn,pio,319
-vada(l.). 316 ............
secumcia1. 315; )o/tJM ,-,bii,c mlqaim de .......... 296
caackitica,296,304
dlcilloproposicioDal,76)
......
ClaraJt.Alem. 309
,
<Olria.296

_..
Cablut of /,r.f11Y11U, N~uuar, ad P,4';,a. exponencial,296.489
bk,W. 138 a.t.DeaaS., 311 """"'1.296
cambio de bue. 218-223 oh,<(,) liDeal,296.298.)04

--
camiDO. 531. 532 de equi.valeDcia. 383. 384, 395 1..-.prftmiel. 296
abicno, 531 lalerü791.!fI1,830 polinomial. 296
aleatorio, ID wcraldm,cba.7'91 complejos conjugados. 478
Ol!l'lalio, .Sll. 566, 569 ,....
lalenl izquierda. 791

-
de llll pío, S4(), S41
cirigido.532
bamiltoaiao. .S78,.S8S. 599 ,,.~ deo:11wf>stafoes.ungrafo.613
paraontcgisrro,720 00111plemt:ato
=--365 pimaria.27.S a 1IDOS. 220-223
dirip5o.S32
ai 11D gnd'o, 365,532,533 -"'
cocido.586
de oa flmci6a boolcai,a, 736
de llD COAjulll:O, 1.5 9, 289. 290, 540
relativo.159

_-
u:ivial.531 coc:ieDle.llS,216
Campbell.DDuglm M.. 523 c:odi6cai:i6a, J9S.796,IIOl,804;Wa.ulaJltbi/a
tccda al¡dnica de la eodifi.cacióa
- bicoaeu. 64S-647,
•) 6S1
campo(deu.uatiuededllo,ttlacional).m
~enmi.ic¡:istro.720 c:ddi&o(J); ~ ,a.,,cbi/11 ICffla algd:niea de de un: snfo, 366. 36&. 533-534
C&llalsimi!:tricobimrio. 79-C;v61U,-b,;,/,ca- lacodüicaci6G
riaaigebnicadelacodificlcióo d,bloq,x,196 deCDteroS.37-39, 150. 151
de bloque (11.,1111). 796 defuDciom:s(t/).:281-283,285
caacdaci6urmltipticatiTteaZ.213
Ca11tor. Gcorg.156, 176-171. 309-311,A-32 deCUIOOrepeticioacs. 798 ,
derdaciones..351.358
~ orginkos., 827. 828
capacidad,671,672,679 deGny.587

-
deo.oaarista,612 d,g,upo.806
de WI corte, 674 de Hammi:Dg. 811 oomúa dmsot. 225
c:ar(R).S49
cataeterlsóca de 1111 amJ1o. 849, 850
c:ardiaal (de GO cooj\tolo), 144, A-71. A-23
c.rou.Lewis..139
_..,
denueverepeticiooe¡.805
detcpetkióulriple.797
de~deparidad.797
ooa,únmúlliplo, 230

de cadttm, 318
deleag,;lajes..320,321

-
CICalao., &igCDC'.. 502. 510 equivalemes.811 coochasióc, SS. 77. 124, 121, 129
c-.:hy.A~l.oua.831 prli;o, 333. 640. 641. 643. 651 CODclesa de Lovdaoe, 239, 240
Cayley. Arthut. 421. SS9, .598, «11. 650. 830, codOlrtimo.2:52 c:ondióóa(es)
831.A-13 de froakR. '62
b iriomial,.71,45, 153 de u:idi.ferc.Dcia. 758-760
"""'
demanoria,, 5
dellBl.panicióo. 382
u:.ahi:Domw.28
priacipal,835
de matrimollio de Hall. 689
imcial(es).462
Cohe,,,o.ücl.LA.,45,46.310,312 oec,esaria.52
""""'
deuuillo. 733 oob,626 ~51
dellD.grupo. 784 cokcdóadisju.acadeQ)lljllQl'.os.A•34 coe,,e,aivas Wsicas. S2-S4, 75, 100, 101
eolisi6a (ai la dispeni6n). 720, 732 cuantific:adore 100
'""""'
de Klecoe de UD lapaje. 322. 330 ~ Mary Jota, 241
oohurm (de ua oruga). 655
,..._ 7>
nep:i6a(no).52
eaugnipo.m
positiva de UD lenguaje. 322
CbttaDd. Gary, 599,601 ,..,.._,..
cobaci6ll de 1111 ,nto. 588-ffl DOC', 75
o (disyuDcióa). 52
0u Sbi-üe. 171 colonciéa fija (illvaiaooe). 817 oexcli.lsiva.52
QvataJ. V..599
ci<lo
combmaci6D(es).19-21, 26.44, 45. 423. m
COD repetición. 33-J6. ,44, 354, 440, 496, ,_,.
o inchwva (disyunción). 52

si ... (DlOIIOtl (implicacióa). 52, S4


dirijido. 36.S, 373,532 497
ea a grúo, 365, 532, 571-573, 578. 651 de 11 e11r, 19 si y sólo si {bicoodiciooal). 52. S4

_,,o
bamilcomaDO. 578-535. 599. 600 ""'1
de emetOS.. 21,4, 226
y(CODjuncióD).52
CCIIICICbvas 16gicu: w~cooectivas Wsicu
congruencia. 395
rq,d-lllllil.61,189 depoliDOfflioa..845

-
cienda(s) de la oomputaci6o. 3, 4, 44. SS, 10, t«Dbiaal~ 14). 207. 601. 79'4 detriáDglllos,61
104. 241,245,248, 251-25'3, 260. 331,333, ~decoeficienu:s.441 mócll}o,c,717.732
363. 395,396,472, 502.,600.601.617. 701, coaipiladcll",n.38l m6oaJo $(.1}. 846, 871
A-1,A-8 de fOJtTRA?,1. 252 CODjetun de EuJa- (cuachdos laiDOs). 858
circuito(s). 532,571 CODja¡ado de va Pimero ~ 478. 732.
oombiulorio,315 c ocnpmadoml, 293-296. 511-520. 600 733
decc:.rrol.315
nkriaao. 552. 57!, 579
del cuo promedio. 300. 301
del mejor ca.!0, lOO, 301 _,40
conju:Dcióa.52

dirigido. 554, 555 del pec!f" caso. 300, 301 .aislada, 750
lópeo.(cbip}l71 eDtiempo conjwilO(s)
1-4 Índice de materias

bim ordmado, 134. 312 glu de ir,JeffiKia, llld11eci6a mar.etá1ica,


de cone, 569, 571-573. 651 iaduccióauia:amtia:fanm..alllettlMm
defndices.166 de Gilbert, 806; vlase tambib1 teoría COlnbiDatoria, 10, 40,!il, 143, 149, 2'9, 264
detimdo eo forml l'fflniq, 209.149. 2.50. al¡d,mca de la codificad6II duuu.133.134
323-32> de H.amm.i:ag, 806; vlase UJmbiiA teorb. iDdim:u, 1.14
disjQDIOS, 158, 171;vlauta,11bil11conjwlto$ ~deacodificació9 porcomradioci6o, 81, 91, 9:S, 11.3. 133, l:M,
~disjm;os infcrior.378

...
14&. 153,184,233, 29S

-
~603.757 mperior.378 Deo. Nasingb. S23, 524, 599-(i()l
miDimal,603, 757
finito. 144, 149, A-28
....-.. desam::llloportDe11Cltts..A•23
a:iplopfla.241 desc.eodieuk.,615
ifl&pendiaue de váti<:a.. 587,655 cuadrado de UD grafo., 65)
muimM.. 587,655 cu.ahdo(s) latmo(s). 835, 85J...359, 861,863, d ev;oaaunda f,pan.11..U.red de piataS). 741,
izlfmito. 144. 309, 311.A-2S 864.m 746
Dltameole-disjw:llo5.158,.J71 IUIOOl1ogooal.8S9 dciinapem:utaeióa.815
D O ~ 310,A-?7, A-32. A-33
lllllimnble. 309,A-'Z7, A-29, A-37 _........_m
eafonm.esciadar,85S en &acciones s.impk:s. 441, 495. ◄96
desi,paldad ÑDgular, 799

-
ordeaado.A-30 awitifiCaciócúq:,tk:iia.112 desordea, ◄1&.419. 429
pa:rci.almcllte ordenado. 372 awitificadores, 100, 1()2, 103. 11D-125. 138. dc:tea:ióGdcerrcxesenaDcódigo. 80(►.W!IIJlt
poteDcia. 148 145, 146, 166. 185, 29S ZtllllbU11rcorfa.algebraicadclaaxlifiaci6e
IOCalmeme Of'detlado, 374 ~ 100, 101 dettnnia.ute, 478. 479. A-19-A-21
IUU-..cna!.S40 z.100
vado(f).148

-_,..
Lt, 120 d csbol. 247,248, 34), ◄91)
: o o j v m o ~ otdenado, 372-3?9. 381. exisleDcial, 100,107 de estado. 329
394. 397,399.400 existeDCiallUlko.120 dcfbijo. 19◄, 19:S.363
algurianodeordcnacióolopol6gk:a.37S-ln, ux,plfcito., 102. 103

-......
de Hwe, 373-,376. l!M, 765-768
394 anivasal, 100, 14!i de Veaa, 162-16.S. 169. 170, 174. 178. 403,
◄08.414.428
bici «deaado, 312 Oick. Aupne. 731, 732
""""-'"
lon,gilndde 1U1a. 399
maximal,399
libre. 100
aibo, 561, S68
Diclaoo, Leooard Eugair.. 240,241
Diad:11m. Jdfrty s.. 651. 652
cuerpo(,). 706. 711, 719, 731, 77&, &30, 839, difercoeialilD!aica.157
cocaimerior.378 &!i0.871. DiJital Equipmeii,t ~ 5
~superior.378 de GaJoiJ., 851, 871-872 dfgitolbi~(bits).5
diagrama de Hasse, 37:J.376. 394 fin:iro(s). 83S, S40, 843,847,849. a,o, 851, digrafo.363.530; PltucUllllhchtpfodiri,-
"""""'º
mwmal. 376
811. 872
isocooños.SSI
DijbR. Edsgc- Wybe, 658, 694, 695
lmlitt, Jdfzj H.. 873
múimo,m curva de IXlp) de nieve de Koch, 486 DiofaDto., 230,239
mi..aimal, 376 <=zeboowsti.Jao.694 di=d6a
ml'.nimo,ln deawrob)1el, 5
iDmersi6A. 397 ~a..b).653 de dos bytes. 5
mtxima COlll imcrior (1.llf}, 378. 379 ~ 7). 798; viau tambiln fleet{aal,ebnicadc CII 1a meux,ria del coq,utador. 5, 720
~(O(a~(~).378,379 Ltcodific:aici(iu CII U, siuemawzi~de ~ 616
orde:a IOta1, 374--376. 394 Daoais. G. B., 69S ~odecuatrobyrel.S
-.io.:rn l)a[e.C.J.• 311,312 Dmdlkt,hlc:rGuluv Lejeun,e,300,310. 730
OClfflDle (c&mino) de 011 polioomi.o, 835 O.bas.JoscpbW.arre3,lJl,312 duefkl(s}
OGSUOllt de Planck. 43 David.~ Ni¡htinp)e, 179 combiaa1orio(s). 731. 835. 853, 865. 871,
OG$U'U0ddadeHllffmaopwtirboles6pcimos. DeAru<:.omhiNaaria., 1)7 872
641-643 DeBniiju. Nicobas Gow:n. 832 cuadrado5 latinos. 835, &53-859, 861, 863,
Mle0.3.4.10 DEC COigital F.qv;ipmeat C,orpowioo). S 864.872
Drltl!:Oe:r.oe,.i-.,o, 23-25.428 ~ ¡nedia:ue Hdcfts de CWC:, 1109; debloque,1165-869
IX'd:Íl!:oo., A-lS
oottad:icci6c. 58. 64, 86. 156
lita# tafflbil11 !ecrlaalgci>nicade laeodifi- 'º'ºº~"""·••ilihndo..., ........
acióo de uperimcmos. 853, &56, s.n
ottnejc:mplo. 94, 95, 101-103. 107, 132, 134 Dcddiud. Richard. 240,309,731,831 gwmeuiafinita.83S.&s9.862.865.871. m
oattapociti"" 69, IOS-107, 376 (1>,b, , • .t,l.), 865-867.ffl.874
oavergenc:ia,437 de UI conjunto de Y&tiee$, 689
:,avoll.,cióa de~-.
cole, K. L. 515,600,601
443, 4S4, 499 de oagrafo.689
comp~874
plaoo afin. &59-86◄, 866,.868, s.n
defiDicióa. 56, 121, 121 plano ~ v o . 867-869, rn, 873
:wobrio.124 ~ v a , 202,.20,I.. 206-208, 254. 284. 316. di.s:posic:iótl, ~12. 19-23. 33, 35. 4◄. 316. 418,
mecci6a de erttlfU ea u c6di¡o, 800: Wa- 318,324, 32',461, 620, 622 ◄28. 449, ◄50; ..,fu.u Uzmbibr. pcnrMacióo
H Ulllfbiffl teoña a l ~ ca de la codifica_. de Laplacc,. Pietre-Simoo. 173, 4S7 circular. 11. 12
ció, Oclong. Howa-d. 139, 140 coo posiciooc:s prohi.Di.das, 424-426
~ bi)"'Ctiva, Z80, 309. 442. 44&. OcMoivn:,Abnhatn.. 310,42.8,456. S22 lineal.f>.-12
505, 684, A,27
lnC(l-:.,673
Oc Mor¡ao.Augu.stu._ 138. 176, 239. 241,589
dcmo5ttadón. SI . 139. 14(►. v'4.u lllmbib:. re- .........
dupositlvo de lb ewdos., 735
Índice de materias ¡.5

ailareoria.i,et,nicadelacodific:aci6a.798: deaál,Jebrabooieana. 762 """"<•)


wa.J,UllttflM,tleOl'fa algebnicadelacodi- ck 1111 aailk). 702 -,,,340
fiaoáóo dewi~l43 de-ifa.43
al WI (Tifo. 535. 6$3
disuibucióe bioomial. 173 ..... ,,.
mwmal de ua ooojwlto ~ orde,.

mbimo de 11D OODjU:1110 parcialmalllt orde,.


de sumidero,, 341
detraasici6a.340. 341
equivakDICS (J.Es.). 3.50. 388
distribltcioaa.,33,lS,36. ·U
dis,-ci6Cl.S2 -.m i:Dicial.329
flmdameDtll.741
diY,6(),)02
divide
..... ,,.
minimal de 11D coDjul@ ~ orde,.

mlaimodemiooo.j!SOIObie:11.otdemdo.1&4
DICCrD0I,, 3IS, 328,349.388

_,..
J:--eqainlez11e1. 350, 388

pan(SIIC:rol,,213 ámmoeom1coojwllopan:ialmemordc:a- s.ipieote, 328


pn po1i.cmios, 839 do.m 1-cquivalemes ($1 Et.ri). 388
dividmdo. 216 esnlla de David. 486. 487
divisióo 1arp de potiDomios. m. 840 CIIWl¡ni:¡)O.m esncran(J}
divisor(a} plR + CII UD ui.Do. 702 al¡,ebnicu. 794
ck:cero,83& paraua~]l8 delldlivo. 17
propios. 213. 703 pan anaoperaci&ablama. 269. 270 de dalo$. 24S. 362. 39.5. 396. 499, 607. 620.
panmtaw.213.216 Wlidad de 1a11 am1b. 703 626.651. 1'20
UllO de UD il¡ebra booleana. 762 de dedsi6a, S5

-~-
pn~Ol.839
4.,418-420 demcmo(i.¡)deou.rnaatt.A-1] .si-entoDces.SS
doble nepdóa. 64 Ekmc:Dt°', 21S, 23:Z. 233, 2:38 »~S5
DoariM of ClraAcu. 428 Elsayed. E. A.. 585, 600, 601 desde<x:i6a,S5

... ARJ,626

-
dodctacdn,.S68.S78.599
Dod:lowm. Pew, mm fin1-U16n1-out,626
dommac:i6G ck fwldoae1,, 294 mwmal. 689, 695 Juc-i•fint~502.
perfecto,692 UF0.502
d cDedekiad. 731 EAdenoa.liertlfflB., 179,A-37 ctiqam~741
deiDleg:ridad. 706. 711,837-&39.&SO enf~~44 mpl:11 de WII ábol dq;m,e. 6S4
de wa base ck dalos reladonal. 273 &i,-..l45 &lclides.45. 21.5, 226. 232. 233. 238,239
-•>
_,,.
deanafwlc:ióa.252 EuJc:r. uonard. 309. 396,456. 4.57. SOS. S29,
Dambolf. laty L. 343. 344,811.831, 832 ~ 2 1 4.223 551,563. 598. no. 830. 8S8, m
Dm,r,st_ H.old L, m. 873 .impar.131 E-, ShimoD, 503, .52:3
2.~610 mód8Jo"- 717-nl Ew:s.. Howard, 139,140.311. 312

........
dnaldeum.propocicióo.65.161, 737, 764 par,131 'h, 100
3!x., 120
deu.41gdnbooleaDa.m.164
C::OlalllecXÍl.deOOGjwltot,, )61
ail6gíca.65
0yct. Wdhef fram AlltOll YOII, l30 - primo(s). 134.18'3.214. 719
rdacivot.226

c1e 1111 algoritmo.


~252
m. 293
pan 11M m6qui.ade esaados finitos. 328
pan wia poerta. 74S. 146
e:q,erimeDto. 172,247
eq,ooenle.A•l•A-4,A~
un.sióa de llllt fctaci6cl,, 257

/:A ➔ B,251
/(l,.).2S2
--.m
ft. E., 388

ffll.lCi6A(a)
eaamend6L 3, 23. 44. 119. 409, 428, 433
eauzaci..do(i).Sl,317
/(,,;) •l\l)(mods(.i)).8-46
/(,,;) es congrueJlte Q;Ja ,V-) (m6dalo s(l)). S46
cmc1Cristiea. 471 fw. Sasam. S., 139, 140 F. (coarradiCC:KlG). SS, 86
r.,.,
-829
-829

"""'-129
difamciaks. 461
dioflncica.230.240
EQUIVALENCE. eouncildo de ANSI
FORTRA."i. 334. 3&5
eqaivalmcia l6pc... 61.63,64,68
Enl6sce.a.239
&d0$,'-I.S99
f"(BJ,287
/E(j(z).294
/E(i(f)mS• .513
/ a domiuda por,.
294
aacl!aeaciu.461,52:l~viw,kutlbilnrela- E:rialtp, Pro111UM1. 831 /esdor:aiAlldapor , ea.s. 513

-
dODeSde recwratci.a arordedabordamia:lto,222 f.a.c., 742
pan vaü'laldóa de paridad. 802;'J/as, lalll· esfera S(~ .t). 799; Yiau 1a•bib1 1eoria f.a.d., 740
Ah leoriaalgebnica de lacodific:ldO. al¡dnica de la codificaci6c fatWf de UD polinomio, 839, 841
poli,x,a,i,1.829 liueal.842
quCDtica,830,m mári<o.799 factooal.7,206,298
Edmoods.J.• 679 1D1CJtraJ. rn. 247.262.•tS.426 factoriz.aci6G de UI poliDomio, 842
eficieaciade u esqoema decocificaci61l. 796: e,pocific,dóo falada.85.128
~ ,_¡,;,¡,,, teorla al¡dnica de. ccdfi. exiUmcial,136 Fuo. GiDo. &59, 872
""6o uni\o'U1al.124 r.:it,Waket,831
Eia:sleul.Albat. 731 esquemasdctodifkad6D.. 19S-791;vlastt4R- Feiler, Wiltiam, 458, .52:3
"""6o,43 biin ll!IOrÍa al¡dni,ca de la codmeaci6a f'eDdc:I.Danid.140
-•>
cero.A- IS
estabilmdot.820
escadfsóca.853
Fermal.PicrTcde.,239.240, 730
fetmiollr,s.43
1-ó Índice de materias

::em:t. Ncnwi MacL.eod, 457 tilCnl. 740 rda::i6D de reainmcia DO liDeal, SOi•.S02
=iliusBooaccü.456 roapideK.aruu:Jh, 748-7S4, 159,160 Ubladeidelltidades,440
inal, (tami..Dal). 363, 530 maxt&mino, 742 ~ de dlcwo, 43&-343
:=itúz.io,Normau.,522. S23 m&odo de Qg:iae-M<:Cmb:y, 7Sl idcDódld. 280
=isher.R.A.,872 milllérmiDo, 741 iguldad,281
lujo eD uu red de trampcxtc, 672. 619 Ollmerodefilas.741 imagec,2S2

-
!ujo mu.imal, 673 oexclusivs_ 145 dcuc:oojtul,to. 256
~ Leslet' Ra:odolpb. J~ 679, 695, 696 prodaao.737 de Wl elemew,, 251
dema,.lffll:liAo$. 742 INT, 253
altunaóva de l a ~ matcmáóca; íovma, 285,A-I0
~iDdoedóo1%11temáica,farmaahemaava c omo prodllao minimal d e ~ 7Sl. iovc:rtibie. 284-286. 290
atin&wde uc:oadrldolati.oo. 855 7S4, 773 ioyecóvL 2S5, 282. 28.S, 290, 684
deBixt.525

---_,.,
como swm mimmaJ de productos. 747. lof;lrimlict,A-1,A-8
de tabla.SI 7S6, 158-160, 773 moD6coaac:m::iente..Sl6
ioddamiaada.A-2 simlmca. n4 DO(acióa,251.252
-.m
"""""
C011.juntiva(f..o.c.). 741,742, n2 deminlfrmíDOJ. 741
O ma.yúsatla. 294-296

diS)"WIÓVI (f.JLd.,). 740, m ~313 b maria,261


t.istcm.itic;a, 811; viase tambiiJt: 1corfa c:cdificu, 79$,801,804 uociai.va.268
algebrlliel de la codificacióll codominio. 252 eenada: 267, 269--271
;omt41 Lofit:; or. tlK Cah,bu r;,f l,ifrre11ct, ~ltjidad.245 eomr,utativa. 268
N«x:aaryaAdProbabk, 138 compa,&ici6a de,. 281-285, 287, A-10
5nrmla(J)
bieu formada, 212 _,.,
compoesu(t o/). 281
,.,
ordec de un. 294
de: illvcni6a de M'5bius. 428 ~•bi)'CCtin.2l30.309.S05,684 paneentera,252,253,406.629

-
deStirliDg.310 aecienle,313 phl de&ler,-409-411. 720. 779
dd c:ambio de base, A-3 dcaccao..2SJ,2S9 ~ de u-. 2S4
exp21'd.ta. 201. 202 de Acm?IWlll. 259. 260 pred~).314
pan;lasuma,40,SI. 186, 187,2"5,259, 296 IX ~ 795: Wtl# raaibfh tema
On,uJ,arioJlmllaNaico. 240 alpaieadc lacodifiQcióa d e !ali coajumo. 287-290
Of<TRAN. 145, ™· 385 de complejidad de !ali dcmcG«>. 251
)(ODel, 4 ) enespacio.294 prorec:ci6o. 271-213
Oll}ds, L R., SSS, 600. 601 CD 6empo. 294,304,464. 466. 488. 5 12, pwitOfijo,419
ourier.kmBapdsteJoscph.309 513. 515. 511-519. 663,664,668.670: rc,aa-sjva, 260. 461. 469

-
re¡e.Goaliieb, 138 viau '°"""""'0011"lrjidad~ JUUicdóa.. 2S7

_,,.
ricdman,AnbutD_m panlaordemcióupordmialdodela.bur· '8lida. 329
bu.ja. 464. 466 sobre, 260,262-265,271,m,282, 285,290,
d e waa arista diripda. 363. 530 panila«deaaci6apor-~636-638 355,384,403.407, 408, 42&,453, 526
e.riiman:d.6n decomnwacióa, 73S-743, 745, m sobreyectiva.260
11.lkmoa. D. R., S99, 601,679,695,696 de dec:odifica::ióG, 796. 804: vlase t.ambiin IUOC($U(d(lf), 314
1Dci6D(es). 113. 202. 2-45, 251-257. 259--265, lb;lriaal~delacodificaci6a
267-269, 271. 272. :zsG.290. 294-296.. 30$. dedispemóG. 101,720, n1, 712 fui.ita de II ll&miDos. A-30
308,309,315,317, 409-411,418, 419. 516. de salida. 252. 329 i.nfmita,25.S
701, ~722., 732, 7JS.743. 745. 754,758, ddc:$111doiiguialie, 329 IIXJo. 252. Í.Sl, 629
772, ffl. 796, 799,801,804,A-l.A-4-A-6. del mayor entero. 252. 2SJ tedio. 2SJ. 254,269.629, 651
A~,A-1O,A-30 dda.606 tnmc(amieldO). 253
aplicación, 251 di,u..ncia. 799; viase tambii• 1e0rfa fwldamcmosdelas~ 139. 343
~va.280 algd,n:icadeb.codific:adóa
bookana, 736-743, 74S.7S4. 159. 160. 1n. domiAaci6n. 294--296. 353,513 C.S41
m domimo.252 to/, 281
"""""'1.774 exporierac:w.418,A- I.A-4.A-6 zdomim.a/, 294
~.736 alemión,257 t domiua/ai S. 513
coodic:ioaes de iftdiferei,cia. 758-760 fillita. 245. 254, 255, 262, 290, 308, 3,43 G-e(eWta).540

__
coojQDc:ióa fuodamtmal, 740 geni:ral?U.310. 433-443, 446-450. 454-458, G - 'I> ('1> vátioe). S40
di.iyucióa fu.ndamcncal, 741 466, 493-499. SOi, S22. 817. 820. 821. &24. Galiko,309
~'l)Ccifieadadema11eni.ncoaiplcu,7S8 825 Galliac,.Josq,bA.. 731, 732.831,832
etiqueta binaria, 741 OOGYOJaci6D de .suoe5iooes, 443
,
f..n.c., 742
Ú>-4.. , . ,
demomea.101$,457,458
Galoi$, EvwiMe. 731. &30, 831. 851. 871,872
G.dmet,Alld>oay, 831, &31
aponeDcial. 449-453. 457. 4S8 Gaibicr.MatiLS23,831,832
operador de suma, 4S4, 455 Ciar1alld. Tnldi KammeL 522, 524
coajm:i.tiva. 741, 742, m

...
<lfd.ocia..449 Gauu., Calt Friedrieb. 395, 730. 732
di~va. 740. n2 panresolverrelac:í<:u$dertevm:Dda.49> to!(v).554
ipald,ld. 736 gaiaadorde1111.pupodclio). 7S7
~deWllvariable.boole:ai:la, 744 particiooesdeetillel'OS,, 445-448 gcuealizadóa
Índice de materias

msaacia1. 136 .t--«fUl,w."I HdJ. Pwoi. 670. 694. 696


uivaul, 121 11,partitO c:ompldo. 606 lk:Dlc,JamaM..179,A~37
~ dd priDcipio de ladu.A6D. 1 aodiripdo. 364,411.412..531. 724 ~615
Qdw:sir6a,41)..417

,_.,,
,mlrlco.121 ...,,.,
,o-'60,'63

,._,,,..,,,,
Ha:sli!:ia.kndN--.831,&32,ffl.l?l
~--.b.591.61/1,610,61)
_.,, .....,...,.,,
COIIIUpll'IIC::l'Uliml..572
bijo.615
dm:cbo.617
fhm. 835. 8.59. 862. 865. 871, 172; vi4H
_.,,
,-1,ihpbao,a& ...,.....,, iz.qu:icrdo.617
ooJo,622
ldlb:ml,365,530 Wbert, DMd. 139, 178, 260. 343. 131
prG1CClin.fbira,ffl
0-,,,,di,,.,.,.,6'0
Gumm.Sophio.730
.......
Onbam.~L,:o.-is.lll,312.'22.524.610.

GreJory. 0wua. 176


HiB.Ftedmc:tJ.. m
bip6ajs.. 52
-~186.197
Gencio&, Jlldi.dl L._ 343, 344
(if(p'),. &51.157. 871
Griess.Robut.Jr~l31
~•lkrll~ll9 ._.......,,,..
biSlOri-1 de -- ~ 44

(iill.Al'tm.3-U.344 ppo(d, ffl ffospa.'tibwicrE., h..'22.nt

-..
~lliMtaUIMllir:M,859 lbdbm. 1n. rn
OISdd.Kmt.ln
Golay.MarcdJ. E.. 793,831.832
...-m
dclxo. 187. 789.847.849
~~)94
Hola, Fnu.E.. 343,344,111,131.132
tio;■,614
Golclq.Samoel.52l.524 cc:irmmtatiWt.ffl
0ormAeia. Dmiel. 131. 132
.,_,_,..
p:ad(o),550
cmaodieKkia,.789
de~dpb .._.,.,
deuillm. ~725. DI

.,...
de canda de 11D vmice. 554
· - ~114
dcmQbo.826,127
de1111bw,ploreplar.122,123
Hombapr. Ros,., 5?2·.524
Hopcrcft.JobaS..J.4:3. 344.. 391. 396.523.600.
601,651,6.Sl.670,694,ffl. 732
desalidade•Ya1icc..5S4 deulE:lnelhJqlllz.177.121
...................,.,.......
Happcr, Orace. 651

·-~"°
drupotillollllio.Sl5

de ua rcpóa. 566
dc••tabla.27)
po(J)
·•traplo~ 71¡.713
.......,,_
• ~ 7 1 . 1 . 1 1.5; . . . ...t.

denmformlCioacs.829.131
_,.,
de 1lllidade&. 779
Mdfzma. DmdAbn. 343,34$. )96.641.651,
6S2
Haygcm.. Cbrbdau. 4.S

2.....:rlot,,S76
_,..
__ -
IIOdado DO dlidplQ. 364. 533 qw.::da...,uCClllljcmto.116 mMPCJRT.5
_,._...,
IUtOCClmp~S49 ,_m
timmicol,SaJ.713 ktca1, 71,4, 72:5. 731
bipatico. 561. sao. 683. 684. 689 simpk.131 idcaodad combwcoria..149. 151. 290
-'61 G~~ $bJia 11. '30
coaapldD (6), 366. 368. 491. 540. SSO. SIi
~364.5»
p(o). 5S4
,.,
Gmil:ir.,Frma.Slf, 599
dec■de:all.. 317
de~lll
cc.apalenaiMl,m defmacioaelboolc:aa. 736
afóco~del c:oior. 598.6SO G.iy. Riebard K.. 522. 524 defflllrica.702.A-14
de acalua. 604, 654 denáznerosreales.61

..-....
deFarer.441.457
de Herxld. 517, S90

depeial!:.604,óm
~ Wolt'PII, 519, S99-601
Kall.Manllal.h..42'.8'73
Kall,Plüp,.614..617.695
Humo&. P•U•• 179.A-37
depomiomb.135

-2>1
CD1rC ooajueoa. 14.S. 1<46,, 164, 250. 321

deucm¡.-..256

-
dePelenea, 56), 590. 600 HaikoD. Sir William Rowa. 176. S7&. ffi, de IUI demauo, 251

_
·~364 >99 implica. 52
deraedl..537 HamraiDJ, RlcbnWaJ,ey.193. 7t4. 791.&31, ~71.105
demlidad..561 IJ2 implicacidD.52.56..s&.121·123
diripdo. l3l. 349. 357. 363-361. m 313.. 16pca.79. l().104
394. 395. 530. .533. 657 códigttdc.111 /.,361
ltCDS.l:30.363 coc.ade.806
..., iodcpeadcada (y variabb booleau). 744

-'"
mstu..363.530 matriz de, 111: viase t1a-'til• teoría ~135

__
~dellcodi5c&i6e ladice,.166
m&riQ de. 799; ./ase 1.-.-bli• 1cod1 dec:ido,-a•gnpo.122
~coouo.365.551 al&dx:aicl.de:acodmcac:i61: deaoprodraclo.23.5
W0..363.530 Haasoll. Omb. 429 dc:aasam■.2:2

...
-
DOdo&.363,530 H,nty. Fnak. 59M02. 651,652 CllPQfllndidad.645
vmk:a..363.530 8-dy.Godfrey K■rold. 241. 429 UldKci6a lllltal::l6óca. 95. 11>111. 1~19',
~o.366.533 Harufidd.Nan.599.60'2 196. ~210. 240. 241,322. 323, 325,382.
. . . S6&..569.S71
...,.,
bocnorfos...544
Ham.Tiac:lmst.ittk.44.46 _,,.
SIS, 565. 629.610. 724. 721. ICI, 142

Hcawood. Pwcy ldm. 589 doble. 313


-8 Índice de materias

.... _
forma alrenaativa. 196,.199, 20S, 209. 233. Kempe. Sir A1fred, 589 k)Uas,od.;:vas
303.519, 552. .566, 609 Kcpk:r. Jobaues, 522 de la l6gica, 64. 108
:if {mfimo). 378, 379 Kaihenbaum.. A- 610. 695. 696 dc:bmuhiplicaci6o.dcmMria::a,A- l8
afdd. Lcopold. 872. 873 """-238
mmo; ~ múiina eou menor Kimlboff, Gustav, 598. «n. 650

-
deemeros. 132
l:geueriael6ctrica.333 KiU0"1-jahrw"al•wqirabolcl.239 d e ~ IQks., 111
mac:ni611,YJ7,S60 KlceDC:, Slcpbm Cok, 139, 140, 322 debieoriadcoocijuntos.160
~ (fwK:ióD múimo emero). 2.Sl IOeia,Fdix,831 demuoil),>, 701. 702
illf:mMiolwBU:SiDCUMactiiracJCorponóoQ.5 IC,...!161
- ")
abierto,72
K.. 366,491,540
Kild.phof, 551
e ft UD araiUo, 702
pan 1\,203

""""'·"
~.72.73
...-..
Kauch, Domld.F.nia. 311. 312. 39', 396, 522-
52.4, 651. 652
panU,204
pan. la Jmia de IIWDml$ ~ 206
Kolaavi. z~. Jo. :ws. 396 pan.imgnrpo.778
dccoojl:UltOJ. 157, 159 K(i.ai.¡,D&itis,599

-
panflmciones~737
~ 166 Kliai.gsbet¡.lossic11:puc:g1a;de,J96.529,535, para 12D álgebra booleaA, 764
dc¡rúos.5~ 551-553.,598 paravviablesbook:aou, 737
XroDccta-. Leopold. 238. 730.131 lcy(ei)tonn:IUWiva{s)
a bkm. 153 K'.tuwl.beph Benaa,I,. 665. 694, 6915 de ♦ CllllllamllO, 701
<:enado-153 Knmmer. Ena$t, 731 dela)ógica,64
aemiabierto.153 Kuralowst:i, Kas.imir, 563, 599 delasamadcmatriccs.,A-14
"""'41,u;t;o inAMlrw, hfoul.Onull. •S6
,_,.,

_
delateomadc:o:,ajllZl10$., 160
IYViiuue (elemento i11variu1e bajo una parafunc:ionesboobo.t$., 737
~ ) . 815,817 l.(G),60S paralamultiplieac:ióodeDÚIDm>Srealel, JII
l\tellllrio.121 taboraorio de computacile de la Universidad
-.12, dc~ard, 772. 773
~lasvmadc:amros,132
pan. la su.ma de ~ leales. 111

.,....
de patrooes, 817. 824-828 laboratorio de software de UNIX. l 7 panmi.ilgdnboo~ 762
,versa dc waa. rei.:ióo.. 284 l.adu.Garasimos..S22,S23 pan.variablesbook:aus., 737
,veno(,) l..agnzlge.Josepb-Louis,S26,830
adiciV'0.280 LamE.Gabrici.S22,730 dcla ló¡ica,6.5
deQamatlU,A-15 l.aDdau. ~ 310 de la tema de C'Olljtlnl:Ol, 160
dcu~,280 Laplai;:e. Pian; Simoa de, 173,457 dclasfuoeiooeibookaoat..737
dcund.tauorcal.120 l.a,q. YJOklH.adlmcisaer,241, 731. 732.831, delas vaiables booleanu. 737
bajo ♦ CD UD aDiJJo, 702 832.872,873 de UD ilgdn bookaJla. 764
deatlSeadeoa.325 Lmoa. Harold J., 4S& leya,dccaooelacióa
aa m1 grupo, m, 830 Ltwk:r, Eugeue L. 585,600.602, 694, 696 para 1a mu.ltiplicacióa, 213,700.711
multiplicativo, 280 luo. 363. 365. 367. 368. m , 530. 543, SSO. para la smnt. (eo ua anillo). 709
de llDII malrU.A•l9 569. 571 pcaUl1 Algebra booicana. 764
deu11olizmtott:llci$úDk.JdcQm>, l20,280 Lqa;idre.Adrieit-Mañe. 730 paraugrvpo.rn
eauaaillo. 706 Lnb.ai%,GoufriodWithdm.137, 13!.308.309 k)U de De Motpn
vawr, 745 lema.21S dclal6gica,63.64,66
vutip:ión Of"Dliva. (,()0. 657. 694 leRguaje. 202. 319-323, 343,349,350 dclaleOfiadcCODjWXO$, 160, 162,163, tn
r'/VtlCipiidi~.39' de progna¡acile Pascal. J.4. 44, SS, 60, 61, delaleiorl•dccoojumo.,(ampliada).20S. 206

-
{'1)).645 104, 2S3, 287, JOI, 314. 3&5. 4611 genenliDdas., 167
~610 p o t ~. 618 panfuxionesboolc:ai:w, 737
~químicos, 607,610, ~ . 827,332 vado, 320 parawiQgebnbookana,764
Lcastra. J. K., 585,600.602 paravmuihksbooieaDu, 737
d e'lJctir.bookaau..766.768.769.m 1.eorlltdodc Piu. 4S6, 521, 522 k)'tS dedomwci6ti
dcuilloi. 723.12:6. 728 Lesmak, Linda, 599, 601 delalógia.65.67
decuerpos fillitoa, 8.:51 Lesaialt-Fostcr.LiDda,599,601 de la ieoria de coajUDltl$.. 160
c1e grar°'. 540, 544-546. m. 724 l..e\bqge. William Jacboa., 241 para Ñaciones booleagu, 737
dc,,.,.._785 Lewis. HmJ R.. 343. 345 pan UD ilgebra ~ 764
--~651 Lcwis,JamesT_JII paravambles~7l7
leies del doble cot?4)k:mctlt0
m. Rogtt V.. 522. 524
llllsoa. O. B.. 670, 695, 696
bmon. Lyle, 651 .
"""
de Kin:bbolf. 474
de: la doble nep:ión. 64, 6S
de la tleCria de COGj\11lt0$, 160
paia fu.DcioDes ~ 737
lmsoa., Sebner M..vtiD. 523, 524 dclalógiea. 64,65.68., 73, 8&, 94. 160, 737. pea WI ilgebta booieaDa, 764
763 para vwablcs booleuw. 737
rdu. Maic E.lmemoDd, 650
dclasfwacicmesboolc:uu,737,763 leya.ddia~
,,. 561,562 dclateoliadccoajumos, 160,165.737. 763 dclalógica,65
nauP.M.•148.m de Obm,474 de la ieori• de eoajwiros. 160
n:llKlgb,mapade. 748-7S4. m paradflujod6:trico.598 p a n ~ - boolealw. 737
~deiocbfetc:OCia. 758,.760 dc:hik,gwDo. 82, 84, 88, 14!, 147 panwtál,ebraboobm. 762
rp, R.M.. 679 de las variables~ 737 pan qn-,1es boolelcw., 737
Índice de materias 1-9

lq,udclM:lllrO cuamiAcadoaa. 91-114.120-134 iaccrao&.32&,329,lU


delalósica.6S
de la tecna de conjm,.toc. 160
demostncióo.121, 123--134
equivalellci.a lópca. 6 1, 63, 64, 68 _....,._,..
Nqllivak:Dlc:I_ 350. 3&8

pan faac:ic:ees boolww. 737 impli,ad61 l6cka, 79, al. 104 lipieoee. 328

-
pana~booleana. 762 Ieyes.,64, 6.5 nDSilorio.340.341
psa'llriablaboobaal..737 ~139 1~350,381
te,acSaaíb.tivu qaeióa.de~~109. ----c:oDC11,,34l . l6.5
delalllllhiplicacióapotacabrasobrc.la 110, 11), 114
IWllldcrmrrioes,A-1' prisacipiodedlaalidad.65 d tNlida.319

-
de .. mmdplicmóatobtel.llWlla proposidoaes()'COGedi•u). 51-S4 del ewdo li¡llieme, 329
c.■ lolc.~132 ~~•~- 62.

-
~I)
ulol ■dmcr-o6~63 64, 104 de Metly. 343
dlltl6pca.64.65 de raruodeda. ■llidldtt.. 339. 340
dc.ltleOdadeco.jucol., 160 d eid'eftKia.9).93 dl:mr-.odel>Cllidlda.UP.343
deSdiNQÓll.66.67 dl:~deaaaid■d.)39
to. la leOrla de coojutoc. 169
......
simbólica.137 detutirts..344

-
Cll WI llliDo, 702 primEr lliYel de accaibilidad.. 3.50
par■ t.::kiiiia bookDu. 737 de R"gla de. iaíueDcia. gg póoclpio del paloalm', ))8
,... 1a amlrip1icaci6i■ • mamca d n 1a •~53.56-SS.61~ procao * mi1limi.Dci6e. 381--393

--
IIMl»dr.maaica.A-24 l.Ofldo■ Madlem■dc:alSocier)'.5&9 ~ d e ..eaioon. )1'. 34)
pwauQ¡d:nboolcul.762 ...... )2.
pwauuillo. 702 d eaucadcu,23, 316.,31& NIC--=iade~341
pan variables booleatw, 737 de wu. i;adeaa (ea ucoajmito ~ ~ -~l de ■ccaibítidad. 350
ordeaado), 399 mbl:D6quio.a. 341
d&lal,6p:,a,65 deuCIIIIÚ0,,.531 ADl!dorbilladoa tcric, )32
de&a...,750. 7$2. 7fO deuaC...suaple.365 ~ (aado).341
dt la IOOÑ. de coci;.--. 160 deu.c:ido tabladl::ata;b.329
para fudoac:s booleatlu, 7l7 n!IC'Odldesnq,oa.&15 tabladl::nmicida.329
pan u 61¡ctn boo1euL 763 eoasnfo.365 \IOIVff&IJTaDCS..330
pata v■l'labb boolwt.u. 737 l.oTW..Laulo..599 MariaadeúaadosUDidot..372.394
liMrAbod, 456. Sll l.(wdaoe, A-,um Ada Byroa. 239, 2AO mamDtical fi:a.allciens. 4&5
libn,del■ Q"C■C. . d,. 44 Loumty. Edwri 311. 312 --.Wmiollable..491
lfdu de clu,c.., SOS; Wuc. ,...¡,¡¡. teorla L--.Fr:u;out-.dA.-olt.411. 522 ~AMl,-a/to,ic.&ilfl• &trt
~deb.c:ocifiadda ~J-.619 ,ow,.,,bae.abholl{~~
limó~ Dava de c.-;-.o. 144 dw. 138
deW11tuoó6oreal,ll),ll4 IIIMl'iz.A-13-A,24
deua1ocaióadeo4mer0freales.. 121,A-3 cero (c:lemeDlo). A-1.5
líffllireia!erior- l,lj.C).J,l.j.Q), .Wi(R)• .W:,(%), 702 cao-ao {O. 1). 2"'.5. 3.57, 359-362. 39S
cola IOlaci6a panel proda,clD.135 Madam.Cob.310 ~A-13
cla ■oraci6apanb.-.22 MacWilliaml.,F. Jaur.,131, lll

--239
alllhda, A-13
lfmite-..puior Maoohar. lL 721, 731 delldyac:eadade•pfo.367."9
. . . .-.22 tmp1- de 1C.--p.747S4.m de Hammj•g. SIi: wi• u tuUlii• leoria
CIII la IIOUICi6- de prodvao, lJj al¡etnia,delaeodifieac:ióo
.~/(JI') • L. 113, 11, ~•deatadotfi.Qiffl.,31.5.329-332.33.5- de:ltl.ddmcia
341. 343. 349. 350. 388--393. 396. 712. 746 de U1 disdo. 874
.~.,. • L..121
- )
dead)-aoaacia,391
........
acc:$1iilidld.3$0

de anda. )21, 319


••Jrllfo.ffl
-•-866.867
·~fR1

-
defadica..YTI de salida. 321, )19 de rdac.i6L " '· 362. 367
dobleroea11eealazadas.»$ dt~depniad.804.806.&17.SIJ :
nlaucw,)95 """'"º
aristadiri&idll..330 vlau tambibt tecria ~ de taco,.
liaeab caiu.adu. 720 cadeu~mfDima.391-39)
lilenl. 740 defiaicid& )29 ddiaid6■• A-13
Ua.C.L. ◄.5. ◄6, 42S,429, ◄s&. .522-S24, .5$4.. diloamadtatado&..329,l)O dc.Nrtoilopcw---a. A,23
.56J. mm.ti00.602.6.5t. m.691.694- E.381.319 ~ A -19-A-23
696.S11. 827. 872.173 E,.'81 doblemale estoeisóc:a. 6'17
Uoyd, E. K.. 600. 601
Ulquc.1- rom,pd,j""Nitdl'ts. 1)9
Loctm., J. A..~- 600, 601
.......,,.
E..38&

equi~336
c:lc:metl.lO(i.j). A-ll
c:lemeoto DUlo, A-15
fila.A,13
loca1'i..cudDea.S41-S44
losarit.:a.A4A-I
-•>
....,,..._,.. lpaldld. A-14
iaveno ailth-'o, A · 1.5
-•I.A.. detrao.dem.cia, )40, 341 aa.eno..ióplicaaTO.A-19
iópca..51-140 eqnivabties. 350. 388 kyaoi.iatinde larauh:iJ,licaci6D., A-18
CUICCCi.-.(16pcal)b6sicas..5M4, 75,100.101 íoicial, 329 kyco1unutatin de la soma.A-14
10 Indice de mat"erias

ley distributiva deaepc:i6a..8'3 11!, fónrllllade aproximacióa de Stirtin¡. 310


• • a:altipl.ic:aci6cl por C$Calam ditt la de ~ d e Polya, 600. 6SI, 812, 824- IH)UWID, 610
sumadematrioe:&,A-15
de la muJtiplicaó6D de macrices so«e la
828. 832
depk:¡ado(paralacispc:ni6a).721
•""""'-""
,sf.actorial. 7

_,..
~delDllrioes.A-24 de Qlue-McCtusby, 753, 773 n•irafo,534
gaaadon.801,806,807;WO#,_¡,;¡.le0- diqomldeCanrtt,]10,A-32 •-246
daal&dnicadelacodificaci6D
meoot.A·22
.......,_..,
abamliYO. 123, 124 11-upla. 246,272
D&lld(cooectiva}. 75
riatroaditiW>,A-15 iadireclodedemosrrad6a. 93 Napet,Jobn,A-7
neattomalr:iplicltivo.,A•l9 nqaci61l,52
prodlllccodc matrices.A-16.A-17 de Rammiq, 799; Villff IUtlJillf: teorfa deproposkioaescuamificadas. 109,110,113.
podDc:to por u acabr,A-15 al¡dnicadeo6dip 114
fila.A-13 Me)ff. Paa1 L. 179 Nemhal.tser, (i_L, S&S, 600,601
&iSlmla deec:oaoone8 liDeala, A-20 miembro (de e coajanro), 143 neottoaditivo
soma,A-l4 Mitlbuu.Alulabetla.239 paralasmaaiccs.A-1.5
Vfflorcohmllll.A-13 Milkr, Gcor¡e Atnham. 830 para los OVmeros reaJes. 120
vectotfila,A-13 mío, 235 oeulfQl!llQ.ltiplicarivo
auocylus.fn:Dc::ucc.240 míDi.ma cota supaior (svp). 378 dekl511úmerolrales, 120
u.235 míDimo común múltiplo. 230, 231 de um maaiz. A-19
WJDaal(aw'erior(w), 378
úimocoml14divii«
para adCrOl. 22S-22&. 231, 231
miDtámmo.741
-.i......695.696
mod. 228,469
.....
cnwaamllo. 703
.
paralamulti¡:,licaci6nde11t\mett:astt:aJc:s.l20
para pobomk,s.. 844, 845 mod•. 717 paralasumadeDÚ!m:rolreales.119.120
axltnniilo. 742. moddo DmrODeS. 43,497.498
:.yt,,e,e,JobtlS.,601 de B<ne-Ewteua. 43 ~ e.ron v.. 119. 140. Jn. 312
:cAllister. David F., 139,140,343,345,396. deftnni.Dirac.43 Newton. Sit Isüc., 310
523.524 deMu.wcU-Botamami.43 Nie\--ugek, Jurg. 523. S24
:coishy. E. J.• Jt.. 773 Mod,u PotlfflS. 80, 84. 8&. 126., 127. 323 nivd.6IS
:c<:.o,. ~ a. n2. m n1 ModruTolkn.r. 83.34, 86, SS, 126, 127 aívdes de Winito, 309
cd(zmximocom4adi.viscr) momeMOS angn1atet. ·O nivel.es de puMaS. 748
panemeros.226.235 Moatmon. Picre Remoad de. 428 Nil'Cl. lvaa. 240. 241. 4S&. m
pan poliDomios;, 845 Mooa. Jobz!Waiiey, 650, 652 aodo, 363, 530; l'N# Ullffbihl Yátice(s}
cm (aú:imo com1n mlikiplo). 230, 235 Moorr.. Edwsd Fonest. 343. 345. 396 nodot 17

-..
caty, George H., 343, 3-45 Monsh, Roaald P., 140 nodosderamiñcacida..614
~ea cúatica. 43 Moser, L . S08. 524 Noedla, Emmy, 731
ecáic:autadlsúca. 43. (S movu:meulosdgidos nor (CODCdiva). 7.5
.._410 de u cudrado. 814
,_,,
¡_,;,., su l.u CONiirio,is~ rúolllbililida de llll albo, 326. 827
l'lfUllUNU par radioaia, 830 de 11..11 be.ú¡oDD te¡nw', 822. 82) beudecimll, 219. 220
~principal,S dcllDtctraocW.827,828 infija,249.619

-
endeboG, Ellion.139. 140 dc11Dtri.in¡oJocqviJttcro. 731-783 O mayú$cvl&. 294-296. 310
-,P.R..m n..lDJ:l'ÚO, 363, 534. 542. 657 pi(s). 235
cact.A-22 eóquaado.S42 polac:a,619
~nnjes. 794, 795: 11/as~ ttuftbil,e tcorfa ptt,fija.619
algdiraica.dclacodifieaci6n de<:la$esdeequivak-ocia A,p:11(%).22

-
=soaes pi,43 de ea1ttOS (ea Z..). 718
tdtotbuDiffertflliaUs.310 " depoliDomios., 346,847 ""'""
de UD bomcmodismo de uil1os. 729

d eúirmadón.80
de polinomios.. 836
multiplieidad _,,
de 1:111 bomomodismo de pupos, 833

de c:ocficiemes iode1Cfmimdm. 483 detmaarisfa,534 ~ 1&3, 192, 193,200,206.201


de oomplc:mcmo a en. 2»223 deumil'm,842 c:ardiAal ttamñnito, 309
rk ~ 133, 134 múltiplo(s} canbalc:s infinitos. A-35, A-36
de~icióa.134 deUDCD$m).21.) comp¡q.cnBASIC, 125-Tn
d e ~ p o t ~ 8 6 , 133.134 deunpolinomio.839 o:ia.sipo,710
de descemo ímairo. 240 de elementos de ll.ll gnipo, 780 aom4tioo,43!, S8,8.59:S

.,..._308
iSe diYJSióa (J,ara dispcnióa). 720 Mmty. U. S. IL, 599, 601, 695 de Bdl,524
deCatalan.502, 504.505. 5'17, 508, $22,.523
:Sciaducci6a~ 196 N.1'3 de clan de un grafo. 60S
:le la columna pria,:ipal. 259 (: ) ....... 45. 438 de dominacióo de OD gafo. 6JO
klatTiba.428 de Fiboaacci, 18'3. 207-209. 299,300.456.
k la fila principal, 253, 259 (7)• >0.438 461. 472. 476, "81,488,489, $22,. $26
:k 1a-1tiplicaci6nanidada.308
:klufwiciones~493-499,501,
(,..,"1_:,......). 28 de filas. 741
de iDdepcDdmciade ungnfo. 581
502 1t!, 7,206.298 de Lucas. 183. 208. 209,461.512
deaivd.615
de recubrimicmo de UD grafo, 603
Índice de materias

f/{.•.11).elmimetodepanicioaesde•eneuc-
tameate11S8i:DIIDdospositivos,4S7,458
_,..
poligooo COOV'CM).. 505
1-11

dertplic&. 866
de StirliJag del primer tipo. 606
fl{.•ü. el aúmcro de pa'ticiocld de 11. 445, 456
P<,,i,r).8.44 _,,.
polill0ffli()(1)

de Scirlio&: del segundo tipo, 36. 264, 26',


310,311,386
1'(7).641
Ñ) (en lcorfa de la codificacióD). 798; v'4#
""""""'836
CTOmióco, 431. 589-595. 600
lliaDgalar,493 Uzml,i¿n t.eorfa algdlnxa de la codificación detorTe. 421..-425,429. 434.683
p(.i-) (pelo de.- c-aicu .t), 665 e11 laioc6goi.taJ1,., 83S
o(«-ectiva).S2 PA.(K).860 inedtk:ibk.844
C(z) (wdea de 1). 294 patlh.615 móa.ico. 844
(i(z)eaS.SJ3 hge. E. S.• 523. 524 primo(•). 844

,..
o eu:luva. Sl. 62. 434. 82A
o achtsiva(e) pan fllfldones booleanas, 745.

oiaclusiva,52
pabbn{s). 317
codificad.a, 79S; 11lou to111bil11 teod1.
llgdlraic:adelacodificacióD
recibida. 795;viostra-bii.111t(Jr{aaJ¡dnica
relltivo~845
redl.tci.ble,844
5lll grado.836
Polya. Geuxc:, 651. 652. m.
832
O'Bryat.Ke-via.651.652 delacodificacióa l'op.502
-.S68
O. tlw 17woryofG""'Pt. alkpnulvi1o«tJu
Sy,,tbotk Eq""1tioft 8' s 1.830
""""""'"
pslfndromo. 16, 327
Palmer. Edg:ar M~ 600. 602
posicióv de memoria. 384, 385
posruladosdePeano.240
po1eodal:S)
opcioae• permisibles. 99 ~.Cbrisc<»H..343. 34S de UD alfl.bcto, 316
optf'DÓo. lS7, IS8 F d e ~ latiDoit Q"tOgoaaks. SSC-858 de WI dc:mc:alo de WI grupo. 780
bwiria...157. 202. 214,267, 61?. 619 parofdc:oado, 246. 250,251 de u k:Gguaje, 322
uoc:iañva, 268 VJl... 100.145 de UD número rc:al, A•l
bie:a dr:6nida.. 718 pa-adoja.de RllSSdl. 156. 177 d,e,ma~318
cerrada. 157,267, 269-271. 318,701.730. pareju deelecttoDa de CODduoci6a. 43 de12111.fwlci6a.284
7lS. 762. m . m. 836. 831 Parttt, F.mest Tlldc:n, 8S&. 872 deUAa reladóa. 358,359,316

-
~va.268 pwticióa., 382,386,388. 39S. 717.191 t..316
t-:wia,313 de emeros. 36. 37. 445-448. 456-451. S23 ¡na,d<.361
coamia::iva, 313 pa:tfailu Rlbatómieas. 43 pnfijo. 319. 350. 640
mouria, IS&. 267, 762 Puca.l, Bbix. 4S. 179,240 propio.319
llenlsia.313 puoUdJCtivo.18.S. 186. 196
1&flaria, 151.267. 762 ~Oren,311,312,S22.524 d e u11 coajllDto, 287-290
opendor de suma, 454, 455 pmóadeeffOttSeDWlcódigo. 794;'1>io.u14111• de mi. elemellllo. 2S 1
opcnado&. 157 bil11ieoñaaJ.gdniea.dclacodifM:acióo premisa. SS, 77. 80. 124. 129
ópóc-.522 Pl.wlak. Zdzislaw. 651 Prim. Roben O.y. 665. 694. 696
opti.m.iucióa. 44, 248, 333, (i[J7 POPll.5 primer!Uvel
Peacoct. Gecqe, 176 de accesibilidad, 350
""""
de,. tX,). 294
de 105 v&tíc:a de 11D trbol. 620. 621
Peno. Giuxppe. 240, 395
Pdrte.OwbSaudcn.138
de illfiaito.A- 35
PrúccipiaUothonatia,,l].8,ln
de v;aa rdaciÓII de ~ beal, 471 Ptrlect. R, 695,696 principio
deaa elernii:mo dd ¡rapo, 787, 788 pm:ai-deacceso. 17 dedasifiaci6eiauzacb, ◄28
de a grupo, 778 penllllQCióa,6-9.19, 20,44.45.408,"°9,428. de discribuci6D de Diricblei, 309. l'ff# taM•
dddicdonario. 617 449,467, 502-504. 523; Wou UUISbün dis,. biin principio del palomar
ua.tiol.616.617 po¡jQÓa de o.widad
lwcog:rá&o. 617
p.dal.349,3S3-356.372-379.394,39S;via- ,..,
p,n,.54 delalóp:a.65
culateoriadecoajuatos..161
#"11WnfflcoctjuDlopcdalmeal:cmSaiado de UD úbol, 641 para fDDciooes booleanas, 737. 738
pa'aWl~boolcaa., 765-767 dcua~6$1 pan UD '1¡d,n. boolem&. 764
pos&mol'(re,:cmdo),620,651 de una cadena., 23 pan. variables boole:anl.s. 137, 738
previo (m:orrido). 620,624,651 de:x(eatcoria.dc lacodificacióD). 198;vi0# deekcci6a.S
linuk:o(m:orrido).622 U#l0fi1tlCffll.aJ,gebnicadell.codi.6cación de iochmóayachssióa. 261. 401-412. •418.
toul.374-376,]9,4 pan \ID átiot 6p<imo. 641 424, 428, 43J, 683
~6J4..63&.65I pes Sllficiaue pan q. 52 de ioducci6a fiaita. 191: vias~ 1aahil11
pord aJ¡oritmode la tat,uja. 463-466. 470. Pcuncn. Gen.kl R•• T13 íoductióo rm.iemidca
6.34. 635,638 ~Juliu.sPc:terOmtiu.600 de iDduccióo fulita. fonaa al'lenlativa.. 196-

..,.....,.
por imerció&, 6 ~ . 668 pila. 502-504, 523
pbl:iaridad de Ull grafo. 366, 645; vio.u ia..-
199; wos~ llurlhiln mdoeci6a matamtica.
fomia.altemativ1.
tq,olóJica. 375
Ore,.OySleia,583,69S.696 .....
bib1 g:rafo pll.Do dcinduccióalllllltmática., 184:llioulmnhibi
induccióD malemWCa. iadueci6o masem>-
,......,,
orip (de IIDII arista). 363. 530 alúa. 859-864, 866-868
finito. 859-864
tica.: fon:oa altemativa
dellldncci6am:aternárica~.196
proyfflivo. 867-869 de rellw6a, 49, S23
p(t)(pesop.-..vmaristat).66.S fWto. 867. 872.. 873 del buea ordea, 184, 21S, 2lS
P(G.).).590 Plr:ss, \m..831.831 del pa)omar. 27S-278, 290,309.311. 338. 831
-12 Índice de materias

robllbilidad, 3. 45. 1'72-174 , 176, 178. 179. c1efmldoaa.m cpmmc:a.600.610,&27,828

,..
24s.,1s.•2S.•26.•2S.•S1.•sa.<CSO. sn. derelac:iooc:s, 358
~~w,porladerecha(enuap
Qaine. W. V~ 624. 773
QmriliaDm, M.ms Fabius, 730
diszribocióobinomial, 173 ,.~m
prueba de BcnKJulli. 173 ¡wopicdld. cucelativa por la izqaierda (ai 1m l{C,x).421
apaciomuestr11, 1n R,R•, a •. U3
- ~T1'J
experimearo. In propiedadesdelosairaos,183-243 radios (m IUII grafo de rueda). 537
raukado iDdepeDdieDre. 173 algoritmo de Eaetides, 226-229 rafcacanctaísticas,471
.-::ao(s).173 .?goritmodeladivisióo.21)-.218.22S-ffl raizdeusbol.614
""'-1.173 lldlcciórtmatemitica,183-199,233 rlllfz de a jrt:,ol crdemdo bimrio. SOO
i~s,79-4 múimc>00IZIID dhi50L 22S-229, 231 raíz de u polinomio. 838, SU, &44
-•>
de asipaci6a,, 683, 69S
aúimo Comlm mlilti.p)o. 230, 231
pri1D05, l8:l, 214, 21.5, '132,.234
raizmllliple.842
R.abt<l8. A¡¡d,ogy, 5.56. 601. 602
de decw6a de Hilbcrt, 344 priDcipio del buen ordm, IM nmas.S00,616
dceqaivak:ucia,396 1e10ram.fa:DdameDtaldelaaritm&:ica, 183. Ramxy. Fnnk P!mnpc:Qa. 311
dc~S2J 232,236 ruóafmea,481
de b QlllrO coba.. S89, 598-600 ~debó~A.-4 ruóa comd..D (ea UH prog.resióu o 1erie
delos36o6ciab,&5&.872 propiedadesdeloslogaritmOl,A-7 ge,om&ric:l).'61
ddcanógrafo.571 proposicióD(es), 51, .5l, 55. 56. 5&, 6U9. 78- ru6D.deeodificlci6a. 796.811;vid#f4IWJila
del viajaDre, SS!i, 600 &8, 98-114, 121-134, 137, 139, J&J

____
ICUia ll,etni,cl de b codi6cdóa
del YOto,so abiena(s), 99, 122, 123, 143., 147, 184. 1&5 tUCIDIIIQiieaeodcduc;tivo,137
~Ml~k.428 i6giwneate eqoivak:Atcs, 1O, 9t" 6:avena dc la i-dac:i.. 3}, 284
~o coq,caesta.Sl,53 reacttlr,497

..-..,,. ...
deeoquctado. 676. 6T1, 695
recanivo.637
,.. coottadica6D.5&
00alnpo$.idn.,69 ...,,_.
Read,R.C..590,595.600.602.832

cmntificada,98-114, 121-134 de11n1.implieacilm,69,85, IQS.107


defiDidoaa,,56, 121-123 de am iq,lkacióa cumifkada. 122
d<-300 da.al de ua popoñdóa. 6S m:ooocedol"deACaioaes.335.342
de mimmizacióra, 349, 383-393 cquivaleocia l6pca. 63, 64 recooocirniento de aa rdacroo. por computa-
recarsivo, 202 esnaura de decw6a si-aatODCeS, 55, 56 «K, 357-362
m;xmdo.532.533
"""""
ClrlC:WDO, 246--248. 2SO. 271,321
eR111CtuRdedecisióa 5 ' ~ 55. 56
itraplicac:i6Dl6gic:a, 79-88 eaOl'lbsitD&rioo..622

_
de cicb disjuntos.. &14 implica l6p;amu,te. 7& ewllriaDo. 552. 553, 57&. 519
de fuoeiooes booleuas.. 737 ii:rrers:a.69 ~aliafimto.869
de-maaioes.A-16,A-17
de~.742 _..,,,
l6gicameoce eqllivüemes.. 62-68. 104 ~odewgnfo.603
~~ck1111gm"o.60.l
dueeto de ¡rup05, 783

..
DC"pci6adcproposicioaescuaati6cadas, 109, recuento ex.::aivo, 23, 25, 42:&
~ « iatormlcióa. no

-58
miDima1 de AUmS. 753, 7S4, 773 110,113,114
porae~A-15.A-16 primión., 52
...
m:lnióa.202

-~h,c,l
og:ramaenBASlCpan,laari~dclOls
a11maos compk:jos.. m
para e1 alJorirmo de &elides, 229
11001Cma. 121.123.124.12&. rn.137.139. un
prorones,42
deOOOllllOCión. 72-74
depoc:nas.11s. 745, m
d e ~ 774
pan.dalgoritmodeEadi.dts(recun.ivo).-469
Ja:adc:ilculodeloJDdar:rosdtFíbomcá.299 ,.,,.,.
proyecci6D.'nl-m delllid&m1Jl:ipJe. 746
de traspone. 333. 671-681. 684. 685. 690.

-
pan.dc:&a.lo dc losDWDGOl ckFiboGacci ~ 10, 40, 51. 143, 149, 2$9, 264 694,695
(re,cun.il'0).489 deBenou.lli.173 lristaDDsatlltldl,673
pcalab6sqoedabwria.5l8
pan la flmd6a pbi de Eulc:r. 410
paralaordeuci6apord m&ododelabur-
,....,.,.,.
deGOdd.178

puertalópca.745
lristasatm-1a.673
c(P. P). 674

baja. 464 A.1'fil. 171. 745 d ell&llristl,671,672


op'&mafuelKe.252.308 NAND, 754 de W1 corte, 674
QgfmJaobjeto.2.S1.308 NOR. 754 a,nc,673
opwmc:i6a de ccmpuudores. " · 600 OR. 745 a-:.613
ogJmlllcióllesaucnnda.194
opsi6D g,eomtu:ica. 461. 462 ,...,,,,
ORac:hasi.n., 754 ddimcióa, 671
fhljo ai ima red. 672. 679
opiedad(dcuardlcióG}
aatisim&ric:a. 352,353,367,373,394
de artica.laci6o. 577, 645-647.651
de 11.11 rede.lo, 279
,.._.,,
flujo mwmaJ. 673

rdlwva, 350. 351. 367,394,395 deliDfuüto.869 proc:edimienlode~,676.&n,6"


Ñmtcrica. 3.51. 367. 394. 395 fijodcu..DafuDCióo.419 ..._.n
trUSitiva, 352,367.373. 394, 39S
opiedadasocimva
CIIIIDgntpO. m,83()
"""·'°'
Q,Q'.Q"'. 1'3
5WlidmJ.6n
teorema de flujo mwmo 1 carte mfnimo.
676,677,679,695
pan. ta compolici6fl qes!le0el.lriopan.p.52 vab"dewaflajo.673
Índice de materias 1-13

-
du1. S12. S13 re!ad6a.202,245,24S,249.251,284.309.349- 1-equjvaleocia. 350,388
dfc:trica,. sn. S13, s98. 600. 6'11, 650 362. 364. 366-368.. 372,.379, J82-.386. 388. ~.420,.428
enterie.73 394. 39:S. 397. 399,400,529 repeticióG, 146
liDtal de resine:ocias.. 473 urisimhriQ. 352.362. 367
16pca.74S binaía,248 deuuaestrv<:tm"adecido.820,322,824,826-
paralela, 73 c:ompoac:i6nderdac:ioaes.3S7.358 828
PfXl',372,.394 c:oq,ocsta.35&.360 de a.na lista de adya,cmda. 39S, 397
t6cmcadeevatoacióayrevisi.6nde11Dpro- coojuoto parcialinc:ll.ie or&n.to, 3n-374, R.e$et,Diaoe,140
¡rama.372.394 37H79 resUi(eou.aaDillo), 7C1J
mmponc.671 dewxaibilidad, 350 resto. 21S. 216
Redei,L.S81 deequivak:iacia. 349. 355. 356,368, 382-386. rauxcióa de Ulllfuci6o. 257
Redfield. J. Howvd.132 381. 394-396. 717. 816,845. 871 rewhadoiDdtpeu,dim1e,l7'
bdaaiaad.U.S.nlara:,S'J, 1"8 """-382 mícül.379,733

_,.,
R«d.M. B.• 600.602
Recd. Roben D., 45, 46
refinamicato(deGDapanicióa). 391
-782
cla5e de cquiYale:Dcia. 383. 384, 39:S
IMb:Dow de Scidi.Dg del sepodo tipo. 386
p1Jtici6o.382-386.39S
deFibcaacci.472.S21
de ipalda,d. 155. 356. 382
retraso, 343
Wlitario,714
reuooeso, 621. 624
Riagcl. Gcrmrd. S99, 602
RiQllOO)' Ku. A.. H. G .. S&S. 600,602
c1e iocklsióa. 249. 353. 373, n6. ns. 765, do~SSI
-.no
infinita.S64
166 Riord& Jolm. 429. 458

_
rqiuro CII UDa base de cbtoc rdaciolw, 273 deordeupscial.353.372:vtos~J-1,j/,co,. r,.r.(C).421
tqla(s) .
de lmplific:lci6o dis)'\IDtiva. 88, 1S7
dea patial,«iDjVIJlo ~ordenado
d e ~ 461-464. 471--480, 482-499. "-'"
Robcru. Frcd S.• 45, 46. 600, 602
de c:oamdiccióa,, 86-88 Sil, S12. 521,522 R.om::s.Cbris.A•24

...._ .
de~traci6DCODdiciocal.88
de demosrradóD por casos, 88

exisltOcial, 136
condcióaimcial.462
CODdiciooadefrown.462
~~471

..,.....
~odo de coeficientes iAdetenniudo$.
R0$1CD, KeDlll:tb H.. 45, 46. 241
Rou.K~~ 139,140
Rota. Giu Ca:lo, 429, 458
rotaciooes. 781. 782
Rotbman. ToQJ, S72, 873
W-..mal. 124-132. 147
mftododef'l1Dciooes~493-499 Rochschild. Bruc:e L, 311, 31.2
aisteDCial, 136 r a f c : e s ~. 471 Rouvray. Damis H.. 600. 602
Wliv-enal.12&-132.147 de Fibomcd.472. 521 Roy. R. R.. 45, 46
deiukm>cia, 80-83,85. 86. 88. 94, 9S, 124. bomogf:nea..463,471,483 nrido (eo UD c,anal sim&rico bi.o.ario), 794
132.137, 147 lineal.463 ruina ddjugador, 525
de aq)lifieadóD dis)'Ulltiva. 88 deprimerordm,461-467 RusclL Lmi Bauud Atlh.ir William, 138,
de dcmostraci6a ~ 88 de KgUlldo orden, ~1-480 156.171
de demostnci6n por el m&odo de coorra- nobomogiDea.463.,471,482-492. Rysa-, Hatcn JohG, 45, 46,42.8, 429,695.696.
d:i.cdón. 87 u,olioeaJ..499--504 872.873
deespecificadóaanivenal. 124-132.147 sistl!ml.derelacioDesderecorracia.498.
de geaeraliucióo 11111Uverul, 128-132, 499 S(pc,,1).264
147 sohlcióa(es) S(¡-,.t). 799
de la 001111.jundÓII, &5. 88 _,.¡,462 ~s.. 783
de separación. 80.81 linealmcllli: ~ i e s . 471. 471 Saa(y. lhomu L.. 695
de r.ú:npfilicación OODjaDtiva, 88 paniailar. 483 S&tmi, Sanaj, 668, 670, 694, 696
del dilema coostnletivo, 88
del dikma dcsuuecivo, 88
del silogismo disyu:ad"WO, 86. 88
tabtade ioluciODC$~late$ panel l'DE•
lodode~iDdeterminados,490
defimcióa de una, 248, 349
"""'
deunalgociODO, m, 293
de una miquml de estados fi.aitOl. 328
ley deJ silopmo. 82 divide. 373, 766 de UDl puerta. 745, 746
ModaPOMIU,80 ffl:ados eqci.Yakutes, 3.50. 388 DO especificada, 758
Moda Tolk,u, 83 CSlado5 t. .vtlentes.. 350. 388 SamueL Pic:rrc, 731. 7.32. 872. 873
kdw:.tio od AJs,mJ-, 87 bomoséoc.11 asociada. 483,484,489,490 Sazidkr,R.. 872.873
tabladerqlasdeiDfCff:Dcla.88 lllva'Jidew,a,2.84 Scbroder,~138
de ta c:oojuDcióa. &5, 88 indle-.ü.-a.357 w:cuenciadelWISicióo. 341
de ta ma)"Oda. 797. 798; ~·lo.u tambiln leo- lc:y uoci.ebVll de l.t composici6D, 358 S,ef~r Yn:irob (El lib10dt lo cnaci6fl). 44
da algd,nic:a de la codificaci&i matriz ccro-wao. 357. 359-362,. 39S JCgllOOO Di.-el de xicesibil:idld. 350
de la 5WDa,. 3, 4, 6. 20, 23. 14S. 151. 170.
263, 265. 276
matriz de, 359-362, 367
módulo"-.3.50
......._
~lccc:i6n, l9•22.24--27,44;vfllk'lalllbibrcom-

de L'Ht,pital, A,2 orde4 parcial, 3SJ.3S6, 372-379, 394 c:oo repeticióD, 440, 496. 497
del pro,:hw::to. 3, s-8, 12, 19-24, 35. 36, 145, poa,cias de 110&. 358-. 359 ~UIPX, 746,747
163,234.246. 254. 2Ss. 261. 276. :m. m. pritner nivel de a&::Celibilidad. 350 serie de Maclaurin. 310,418
3S4,419,S9I.S94 ref'laiYa. 3.50, 351, 362, 367. 394, 39S pa:ux.418
de $UScituciórl (en 16¡ica), 66-68 segundo nivel de accesibilidacl, 3.50 5Cric de p0f,tociu, 435,436.445, 457,496
Reiqold. Edftnl MMtin.. 523, 524 s.ímtaica. 351,362. "367. 394. 39:S Kriegeomftrica.437,440.481
-.382 1!1DSiriva. 351. 362,367,373,394,395 Sesbu. s.. 600. 602
14 Índice de materias

:y&th. ICllu, 4'29 Jubdivuión demeutal. 562 de decodificacióA. 807, 808; W4M 14.bilA
~ C1aude Elwood. 735, m. m. 793, snb,:rafo.538 leOC'!aalgel:nja.delaaic:li&aci6a
831. 833 indDcido.539 c:on síndromes. 809
~ Doaald R., 523,524 deatados..329
tli•tic.Chi.178
"""'"""'-""
nbgrvpo. 780, 781 de formas O m:ay6satla. 296
MU. Doogl.as R.• 695. 696 ~ por ILfl elemento del ¡:ropo, 717 de identidade5 p:an. fwlciona gemnDioa.
MDOyl. O. B.. SSS, 600,602 DO trivial, 780 440
mkhaDde., s. s.. 858, m oonnai.831,872 de mimeros de Stirliug del scpDdo tipo. 264
i.53 propio. 780 d e ~ 163.164
llvesai, Richard.831.&32 aivial, 780 dete¡Jasdeinfetencia.88
mbolodcl..&oda,,310 submiqttiDa. 341, 712 de reglas para negar proposici<Ma c:on ua
DCtromt.803;via.uwmbiin~algdnica submarino Pobris. 372 CUlllificador, 110
delacodificacióo ~A-30 desolucioncJ~pataelrnftodode
D grado,836 sublabltto. 421,422 ~ ~ 490
p,entonoe:1 q,52 subcabletosdisjuDlos,421 de ttusici6a. 329
-•)
deccaadoaatiDeale1.A•20
subtabkTos ~di.sjumos..422
RICC:$l6o.2S4
de verdad. 53. 56. 57, 61-65. n, 1&. 80. 94,
163
de Dllmeros binarios..21s.:no ~ 6S2 parannabuededalosrelacional, 212,m
de tdacioaes de recurrellCÍa, 497-499 QUllenllria,245 tablero deajccka. 420-426
de repraeatama di51:imos, 687, 695 de: ñborulcci,522 tama6odeunc:oojmllo, 144.A-27
Otul(bueS).218 de tnnsfeffllci.a.·341 Tary, G., 858
~ UNIX, ·11. 18 fi1'liu de II t&minos, A-30 ~ N k mlo. 178
tripk,870 infillita.A-30 taatolopa. 58, 64
de Sleioer. 870 IUCCS0, 173 i6caka de ordcmci6o. 463--$66
u.oivenaldc:~616 ~ t a l, 173 1ie11ic.u de dkalo pan las faoc:iooes
y$61os.i,53
ioane, Neil James Akundu. 831, 832 """"""'-"'
suoesor.2-40
~ 436-443

,.,
~ ,). 121, 123. 124, 128, 131. 137. 139.
llidi, Kem, JollD Scepbco, 239 AIC.eSQS~es, 794
l. 783. 830 sufijo, 319, lSO cbiDo del rest0. 728

-2$7
:>ifcr,Alcuoda, 311,312 propio, 319 de Bilkboff-voa Neumaim. 697
>lidos plalómcos. 567, 568 ,-22 . deBun:iside.817. 820
)low, Daniel. 140 de Cobweb. S23
llucióa(es) debiu, 746 de coagrue:oci.a de &Jet. 793
de e:uciooe1 polinomiales. 871 de tlasc:s de equivabcia de COGgNCDCia de: Fuma, 7'3
geocnJ de una relación de recam:aciL 462 demeros(mZJ, 718 deDeMoim. 199,4n
partiallar. 483. 484, 489, 490 depolinoini.Ol.846 d e ~ para poliDomios e:rowri-
iudepeudiaatcs;vhstiofucioaesüt:almenfe de fuDcicmes booleatlas, m .,,._ 592
~Dfe:S de mmica. A-14 de flujo mwmo y tone míDimo. 676. rn.
liDealmaa~independianes. 411.•n d e m i ~ . 741 679, 695
¡,a,ocf,JoelH..311,312 de.mlmerosbimri~ 746 deKmalO'l,W,$63,599
taD1t. Oouald F.. 139.140, 3A3, 345,396, 523. et1: t1.11 compm,dot, 746 del...agraoge.m
524 depoliDomios. 836 de bpafospllDOS0011QOfde Ec. 564, '9S
tuley, Rk:blrd Peta-. 458 dobk. 32 de Jq)reileUtlciÓII pwa 1,111 ilJdn bookaia
teiahaus.. Hugo Dynoiey. 523, 524 índi:cc,22 filliu,768.m
i:em, R. G., 58.5. 600, 601 límiteillferior. 22 del biDOmio. 26-28. 37, 45, 124. 150, 178.
lifel.Micbel.-4S límite superior. 22 405. 438.. 449. 4S1
liU.ell.Jotm. 830. 833,872.873 l'llill.itnal de prodbw)s. 747, 748, 756, 758- del bulomic,~439
twoo.~g.1.asR.,873 760. m delfad:or.841.842

-
tirliQg,lame$. 310 del resto.MI
10D. Robcrt R .. 139, 140 binarioenseric.332 flmdamaital de ta ari~ca., 183, 232-234,

_,
tow, 746,747

-771
ti-aoa.,Gilbttt,A-24 236,241, 2S4. 265,276.321, 354.1SS. 409.
~ AAae Penfold. 831, &33. 873 728.. A-33
al UI m6qu;iDa de ffladcs fiml0$, 341 flmdamc:acaldelalgebra.171
abanillo, 712-714, 714,725 tDlllll.rcddetruisporte, 672 mau::mitico,121
lMtbol, 500,609,615 sup (mínima «>Ca superi«). 379 m. mlllriDomial, 21, 37. 124
derecho, 622. 623 1upmno; ~ múJima rota SGpCrior
ilquierdo..622.623 ~ o. lS8 deat6matas.343
1bcadma. 320, 350 Suppc:s,PIDicltC., 179 deGalois. nl,83l
propi&.320 Sylow, l..udwi¡. 831 de baDdas semic:oocluctoras., 43
11x:oujomo.144-149, ISJ, 159,162.163, 247, Sy~oes;.JuxsJosepb.418.A-13 de la a1difkad611. 3.44. 173. 333, c,QQ601.
2,48,436,442 s,wtbt:>iit: LAtic. 11s 600. 638. S72. 873; W4St 14,.¡,¡¿,. ~
deudememo, 1'8.173 algebraiade la.::odi&aci6n
propio. 144-146 To(tautok,gía). 58 de c:6di.gos.: "6digo ~jo. 63s-643
l~,$47,$48 -•l de ta relatMdad. 731
Índice de materias 1-15

·~343.3'9
de-i-o., 35,214,215, 238-241, 309,310.
409.421,445., 701. 7)0
...
tcodademillix, 701-7'.!l, 792,13$,8)1,tl&,

uillo(s)
ll.lboDajwllo.1'4
deu:aelmlal&o.141
propio.144
*Polyaall~deploa.600 -rn ~ 1$8

_-723
•ita.e,.311 cmaelmmlll>lllidld. 7CB,137,0I lallladepenac:acia. 163.16'
•~in co.maavo. 703 Clmlllodeuc:oejulo, 144.A-27
le0f(a IJ&d:nica de la COltiicadÓL 23, 17:J. delllllrk:u. 702.103 ui6acle~l57
m.793-811.831 •~w.m ui6II generalizada die ooojalol, 166
~•dl!lcodi1k:ac:iaoc
call~biolrio.?N ...
demamotl.1Q2.,703,730 uiveno, 143. ª"
uiwcnodledisc:alo,141.1'4

.._,._,..
oo4ficaó6o. 195. 801,104
e6diJO(•> cacrv de •aaillo. 733
~11!!16d1Üc)11, 717
IICda de ploc.. 1111. 24&. 330. 333. 362-361.
372-376. J!N-J97. 411. 412. 421. 529-SJS.
537-546. S49-"6. 5ss...m. 516-595. S98-

..,...,.....
ckbloqoe(-.a).796 caerpo. 706, 711. 719, 731 f1H1, 613. 645-647, 653-655, 6$1, 6.S&. 683,
dec:iacorq,edciooa,.798 ddíak:i6a,701.101 "'· 619, 692. 6M. 7S7. &32
~pvpiol:decao,703 ,.____,,irboia_lCOriade~
deHarm:aio&,111 doaúio de ~106.71I mato.rcdts•~
demeverq,eódoa,et.805 ~aaidad.703 i(G).639
delripkrq,eddda.797 Cllltl'Olmódalo•,717-nl x (G). d D6mcro cromilico de G, S88-.591

_..,,

__
dewrificaci6adeparidld.797 p-.po dr.: uidada. 792 a(G). cl .a-rvdcdadeG.60S

--
~ m.m. n1 -tC),. d ....-O d e ~ de G. 534
·~•11
0Clffecd6a de cmira.lOO
Weal.714.725,731
i a , , r n o ~ 706
'J(G).dll6maode~d,;(;.603
l'(CJ,.dama'Odle~deG,587
cocadt.Gilbert.806 bomorfismo..713,726.728
coca de Hansi:ü-,. 106 le)'deca.:eld6adelaa:akiplbd6a. 707. d d c:miiao lMI conodeOijblra. 657. 665

de·-- .........
_,..
~,l.791 711 paiabJllllllCltde~6'7.6.SI
~796.-.W
~--"-Idcradldaac.l09
,., lit)' decmcdacim
..rro ~ 703
10P, 724
~ffl.500
l6cle:o• • 11ocriomomm1o. m omnú,ffl,500
dneicci6o de erroru, 800 ptOpiedadu:dtU.llol.. 709-714 ordeMdo coa raC&. 500. ~507
diJUDCU. 798 lllbmlJo. 112-114. 724. ns arco.330.363
~ · vai6caci6a.dela paridad. I01 .-isa.363.SlO
c&ieaciade•aqia,caade~796
aiaa(S(.r..t)),799
......
Z.,.717
'°" -)31).)6)
ao 41::i.pda. 363
apedo m&rico. 799 -.ortade c:oa;,i.a.. 99.143-149, 1$6-167, 171. camioo.531,532
~•c:odiftc:ar:.&ia.79S-ffl m.
11~179. 184, io. 2A6. 24. 309-111. atlic:rto.Sll
fuma~III )IS, 317, 34l.A-27,A-30.A-34 OttTO. 531. S66. S69
t.ci61.4mada.799 ..-•~de-.---.1,n dióJido,5)2
Gola,.Matd. 793 c:adal.14',A,27.A-21 buállolliaao. 571., 599
Haa:laíoa. Ricbard. m. m. m ~ di u COljllmo. 159 simple. 36$. 532. 533
tsda"dtci.t.808 ~ t d a i v o , 159 =--36$

__
mci.l de Halnalba. 811 CODjwllo(s) diripdo.532
~dewri6caddlide11paódld..l04.106. bic:aordeudo.l&,, a:i\'ial.Sll
107.111 ~ del'.ndic:ca.166 cidD ~ S1S. S99. 600

_,..
.-.ú:~801.a>6.I01 ciJjalo&.IS&,171 cidadiri¡ido,365.m.m

__
......._,.,
mi:maje.794,79$ ftaico, 144, A-21 ciraülo.532.571

a:nicade~ 799
,W.791
,,, wizlitG.144,A-28
~di.;..o..,1sa.111
n.icriuo...SS2.S78.S79
diripdo. 55', 55.S
a o ~ 3 l 0 . A-30,A•37
"'"''°' de...,__
................,.,,.,
--
alo(t). 141

plUÓlldeer:n:it 794
amaable. D,A-27.A--29
pok:Dcia.14& ,,_,..
colorm6a 5111-'95

pe:tc>de~ 798 Vld'o(I). 141


ddimd6a Ultlliri,... de u ccmjlmlo. 143 • llll púo. $40, S4I

--
ruda de codi6caci6a, 796. 811 ~•V....162.16.S dr:uabpdonaap:afo.61)

......,.. .......,.._,.,
m

__...
rcatadelamayodl.797.798 ~simáócl. lS7 ooq,cwatt. .366. J6&.
ck:me::alo.143 t:ikoaaa.6'5-647,651
.!!,.l), 799
. . . . OumEtwood.793 ~dtcoajT,,Dcot..157 d ecorte,569,S71-S73
-.803
IKaOl~794
1i1t1k d e ~ I07, IOI
k,a de la
lit:~ de ~
teaía.
iMaw:ic:a6a paaúzada dt ~ 166
~
1'4
160
dominaalt..603.151
mimm1L 603. 151
~•~m.6Ss
~ 143 muimal,,517,6"
wdJIOIIIMcaQII0.803 micmbro.143 cucbclodcaanfo.6.SJ
·16 Índice de materias

d(a,b).658 K...,.561. 562 llmóo de pafol:. 594


defidencia,689 ~ •• 561 l-fac((lr.692
~ bistdrico, 600 c. '66.S40 Vfflice(s). 363. 530
~de&ue.373-376 ICOai¡sbcr¡. 105 siele ~ de, 396, S.Sl- adyaoemes,. 363,530
digafo. 363, 530 SSJ, 598 aislackl, 363, 374, 530
distaDda. 535,653 UGJ.00.S colpnce.550.569

-~396
~amieato perfcao, 692 la%o, 363,365.367. 373,530.543, SSO. 569.

=-
decorte.Sn

-
~deuwboldepate,6.St

foc:olc de wia arista. 363, 530


"'
listadeadyxax:ia.397
liJtadciadica,397
fuw (leffllitial), 363, 530
ieoriadegrupos. 777-834

a.jo loonwwitáDta.541-5'4 clase weral. 791, 830


G -tt (t! IIIMamQ).S40 c:lase lalCl'll derceha. 791
G-u (1>evértiee),S40 de lllD camiDo, 53 J clasela&etali:.QuimS&.791
~u).S.54 deucan:lioo$imple.365 ddi1lic:i&ldewaarapo. m

.,..,
ge:ne:racióa de un subpfo, 538 detlll cicl,o, 365
mmiz de adyacencia. 367. 559
descomposicióo de lltla pcnmnaci6n. 815
demento inVllriaDie bljo vmi pamatláóD.
de enaada de u vértice, 554 ~ d e ~ 559 815.817
dcsalidadeunvértice.SS4 Dllti¡rafo, 363. 534. 542 estabilizador.820

-.......,m
de UA \oÚtice.. 550 dirigjdo, 542 (ijo(io.vm&Dle).817
de UDa rqidca. S66 imltiplicidad (deu.naarista), 534 ~ d e l l D ($Ub)grapo.787
¡rad(u).SSO ,-<Ol,o,602
pd(R).566 n-irafo,534
pfo(s) DOdos.363,SlO ádieo.7i7. 789
asociado DD diri_pdo, 364, 5)3 ~IIQ.m
-=-complmx:DWio. 549 crom4tico, .SSS-591 c:muo de KkiD. 789
biconexo.645 deelaa,,605 depermataciones. 783,815,872
bipstilo. 561, 580.683, 684,689 dedominaci6ca.603

_,.,
cletn.o:Sformacioa829.831
bipanito~kto,561 deiadepc:Ddencia. S87 de~779
COfq>leto, 366. 368. S40, SSO. SSI de~D&0-,60] que:acttlasolft11.11c:onjnmo, 8 l6
COIIVDparlfflD.Dll,Sn orip (de ua lrisu.). 363. S30 =<>h>l>I<.m
eooexo. 364. 533 P(G.l.).590 simttrico, 783
afticorespectoalcolor,596,6'0 peso de mia aruta. 657 ~le, 831
deacalera, 604,654 plmaridad de píos.. )66 bomomorfismo, 78.S
de Hendid, 587. 590 poliuomioaomitioo.st9-59S,600
"'"-'°' 60S
de peioe, 604,
problema de loc cutto colores, 589. 598-600
p1)blema.ddviajme.S35,600
loDg:itixlde11Dcido,,8I5
mtiodode aawm:nd611.de Polya. 812.-828
de Pelcrtea, 563. 590. 600
d e ~ 364
den,c:da.S37
""""'"°
puntO de anieulación.
fadios.537
sn. 645-647. 651
tnO\'ÍlmeDtosñgidos
dewa~.814
de llD albo. 826. S27
de wrvicioi.. 561 rt.e:OCrido.531.532.533 de UD he:ú¡ODO ttgular, 822. 82.J
dirigido. 363,, 364. 372. 373. 394, 39', 530 eukriano. 552. S.S3. 578. m dellll.terratldrorep.lar.827.821
Ci0cqlkto.S81 JeCUbrimimo de 11D gafo, 603 de UD aimguJo equiÜ!ffl).. 781-783
etiquetado, 333 mi.llimal. 603 mdlriplo$ de e ~ del gnipo. 780
dbcooao, 366. 533 mkld6c.uica.598.600 lll1deo de l,UI ~ m a o . 833
2-isomcdos, S76 ttgiÓllñ:lfilat&,~
"'""'

--
dlMl. 568. 569,511 ttgiooes (en e ¡rato plano). S63 de uo elcmoJco dd pupo. m. 788
t - e COt1C10,., 365. SS8 repcesemacióil de um lim de adyacaicia. dcpgnipo. m
bomeomorfos. 562 395-397 poleDciu de demeatos dd ¡rvpo. 780
R..537 producto dirCCIO de ¡rapo&. 783

_
,t.rqv.1.v.sst sólidos plal:ócioo5, S67, 568 pocbx:tosdcddosdisju!ltos. 814
~tOc:ocnp)eto, 606 subdivisióo dcmelltal. 562 s. 783
DO diriJido., 364, 530 su.bg:rafo.538 sobgnipo, 780. 781
DO plano, 560-56) iaducido.539 DOtriYW_ 780
llll)o,541 Iffll.bridor.538 normal 831. 872
- de descomposic:i6D
•)
..
pi-,. 560,645 propio.780
COD IUI pat lleS?aiml, 572 para polia otllios trivial. 780

_,.. ..
"""""""-592 - •>
¡x,od<ndo.657
n:gula,r, 550
lill&Jm.36S.S30 ,
de Eok:r- pan gafos plaoa: COlleXO$, 56'•
cleEuJer, 793
~u).554
acideocia.530
de Kunt~ 563. 599
ieoría. de ~ o . de Polya. 600 "'""""'-""
de l...agaag,e. 792
terminales. S72 ieoriaalgebnic:adelacodificaci6a.806-810
lllCneeci6adepfos,.594 lfflnioo (de una arista), 363. 530 ~ dd empsejamieolO. 683-691, 694, 695
somcrlismo de gnfos, 540, 544-S46 """"'581 6(.G),689
Índice de materias 1-17

-
C0lldici6D de mammoaio de Hall,~

d e 11D CODjQDCo de 'IÚÓ(:a, 689


dem rn'o,689
empa,ejamieofo,684
complao, 68-4
muu:ml,689.~
pm. tan grupo. 779
Wl:idad de mi il¡dn boolcua. 762
UDÍdad de un uillo, 706
llllióa de OCGjUIUOll. 157, 159. 250, 260, 289,
290,A-34
mióa de¡rafos. 594
ui&tp:mnlwidadee:oajwlloa.166
faem:. 363. 530
itllenlm.614
qc:u.363.530
tcraial1. 363,530,614
voh'er1arrucar. 3J.0
'\boEltiDpauscD.Nldreas.4!i
\ba.Koc:h.Htlge.,486
'\boStmdt.Kld,650
prol:,lemadeuiguc.i6a.618.695 vaiveno. 99. 102. 103, 143, 144
l,l$fQDl d e ~ disciciUll. 687, 69S IIJlliw:no de cmcano. 99, 143. 144 ¼>rle.auagen i«r ~ Alpbra d.n Logik. 138
tc:nDiaak:1,,572 UDiwno DO vado, 102
Walb::r,.Elbat A.,732
t&miAoaislado.750

_,_,..
~c:scadlsóca.43.45 '·""
1-eqaivalmcia. 350
l -faclor.69'2
Wallis,W.0.,831.833,m
Waad.Mi1clicl.241
ICttaedro.567,568 aranio.497 Wcavu. W.. 833
Tb/OÑ .4Jlal}'tiqw ikspombililh. 173 Westoa. J. lhdey. 429
Thlo~ ANll)'tiqw da P'°"'1bilila. 457 Vajd&,S.• 522.524 Whildleld.Ali'redNonh.138.ITI

.._,.,
1bompson. Jobn. 831 validez de 111D qameDIO, 87 Whimey. Hasdc:r. 599
Whitworth. WilliamAlku.45,46,42:S.429

_., .
Y»ordeuaflll.}o.673
10ITe,. 420-422 Y-.lordeverdad.S3 Wikb:, lbymond L., 139,140.311, 3l2

_
10ITeS de HaDoi, 484. 485, 522 V. Slyte, R., 670, 695. 696 Wilf,Hctbet1S.. 4S8
nsposic:ida de Wl gnfo de ffflcr._ 448 ..-ariabk(s).98--100 WUSOA, L B.. 523. 524
uupuesca, de ltJl.l tnaitm. 361 ac:ocada.100.112 W"llsoc, RobiD J., S99-6(1J
cra)"tCIOria es,calomda, to. 149, 151, 152 ~474 Wood. Oeru:k.. 343, 345
TtutiuoosAlgebr4, 116
Tmnblay, ~Pal. 728. 732
ll'iaaplaci6o,(dcP-poigmoCCIIIW10).50S,S0'1
Vet,len.O.,m
...
libre. 100,123
Wri¡bt. O.les R. B., 139, 140
Wri&bt, E4ward Maidud. 241, 429
~ ) (deunaeadi=a).23
Wymu.M..508.52-4
iriúga.lodePascal. 152, 153.155, 178
Troaa, H.F.. 523, 524- w,::iAomiscacm;,.803
Tucto:.Aim.45.46.428.429,458.694. 696.&13 YCCIOr columaa.A-13
Turila¡. AlaD MJdtisoo. 344
Tune,W.T.. 599
_,.,,,
vect«fila.A·IJ.

Tymoczko. Tbomas. 600. 602 Ve:ildl.· E. w .. n3 y,52


\íeDD,, Jola, 178 Youtc. Bevaa te.... 241
UibmP. kfffey David,,343,344, 395,396,523, l'aificaci6o de u.n pror,ama. 194 ~ 316. 318

600. 601. 651,652,670, 69-4, 695, 732 ~ s) de ~ pío. 363,37-4, 530


4lrimoteorc:mldc Fumar:. 730. 731 adyaoemts. 363,530 z.z·,153
WliddaddeiD~ aislado.363.374,530 7.ariski. Ose«, 731. 732. 872. 873
~ aa ilgdn bc>obaa. 764 colgaDCe. 550, 569, 6(J() Z..153, 717
pcauasinillo,109,711 .. -.m "l.uctamsi_Hett,,enS-1,240,241 .45&,ID
NOTACIÓN

LÓGICA p,q enunciados (o proposiciones)


~p la negación de (la proposición) p: no p
p/\q la conjunción de p. q: p y q
pVq la disyunción de p, q: p o q
p ➔ q la implicación de q por p: p implica q
p .... q la bicondiciooal de p y q: p si y sólo si q
sii si y sólo si
P ""q implicación lógica: p implica /ógica=nk q
p <;,>q equivalencia lógica: p es lógicamenu equivaknte a q
To tautología
Fo contradicción
'!fx para tooo x (el cuantificador universal)
3x para algún x (el cuantificador existencial)
~ORIADE
CONJUNTOS xEA el elemento .x es miembro del conjunto A
X ftA el elemento x no es miembro del conjunto A
"U el conjunto universal
A !;B, B;;iA A es un subconjunto de B
A C B, B::>A A es un subconjunto propio de B
A </:. B A no es un subconjunto de B
Aq;B A no es un subconjunto propio de B
IAI el cardinal, o tamaño, del conjunto A; es decir, el número
de elementos de A
9= {} el conjunto vacío o nulo
*(A ) el conjunto potencia de A; es decir, la colección de todos los
los subconjuntos de A
An B la intcnccción de los conjuntos A, B: {x fx E A y x E BJ
AUB la unión de los conjuntos A, B: {xlx E A ox E B)
A t::. B la diferencia simétrica de los conjuntos A, B:
{x lx EAoxE B,perox~AnB)
A el complemento del conjunto A: {xlx 'U. y x lt A)
A-B el complemento (relativo) del conjunto Ben el conjunto A:
{xfx EAyxftB)
U,e,A, {x lx E A, para al menos una i/), donde J es un conjunto de fndíccs
n;e1A¡ {xlx E A, para toda i E[), donde/ es un conjunto de índices

NÚMEROS afb a divide ab, para a, b E Z, a ,fo O


afl, a no divide a b, para a, b E Z, a #O
mcd(a, b) el máximo común divisor de los enteros a, b
mcm(a, b) el mínimo com6n múltiplo de los enteros a, b
+(n) función phi de Eulcr paran E z•
LxJ el máximo entero menor o igual que el número real r. la pane
entera de .r. el suelo de x
íxl el menor entero mayor o igual que el n6mero real r. el techo de x
ª " b(modn) a es congruente con b módulo n
NOTACIÓN

RELACIONES AxB el producto cartesiano, de los conjuntos A, B:


{(a,b)la EA.bEB)
911!;;:AxB 911 es una relación de A a B
a911 b; (a, b) E911 a está relacionado coa b
a~ b; (a, b) fj!: 911 a no está relacionado con b
9'I' el inverso de una relación 911: (a, b) E 91 sii (b, a) E 91!'
911,y la relación compuesta de 91 !;; A X B, Y !;; B x C:
(a, e) E 91, Y ,i (a, b) E 911, (b, e) E Y para algún b E B
sup{a, b} la mínima cota superior, o supremo, de a y b
inf{a, b) la máxima cota inferior, o ínfimo, de a y b
[a] la clase de equivalencia del elemento a (respecto de una
relación de equivalencia911 sobre un conjunto A): {x EA I x9'1 a)

FUNCIONES f:A-->B fes una función de A en B


/(A,) paraf:A -->By A, !;;; A,f(Al)es la imagen deA 1 mediante
f.esdecir,{f(a)la A,)
f(A) para/: A -->8,/(A)es la imagen de/
f:AX A-->B / es una operación binaria en A
f:AXA -->B(!;;;A) fes una operación binaria cerrada en A
1.:A -->A la función identidad sobre A: l.(a)= a para cada a E A
flA, la restricción de/: A--> B aA, S: A
g,f la función compuesta de f: A --> B, g: B --> C:
(g, f)a =g(f(a)), para a E A
r' la inversa de una función f
r'(B,) la preimagcn deB1 !;;; B paraf:A--> B
/EO(g) fes "O maymcula" deg; fes de orden g

B.ÁI.OEIIIIA
LASCADENAS ,.
:E un conjunto finito de símbolos llamado alfabeto
la cadena vacía
llx ll la longitud de la cadena x
:¡;• {x,x, . . -x.lx, El:},n E z·
:Eº ¡,.¡
:i:· ;;J_. :t•: el conjunto de todas las cadenas de longitud positiva
...U :t•: el conjunto de todas las cadenas finitas
A es un lenguaje
la coocatcnación de los lenguajes A, S: B ~•:
{abla EA,b EB )
{ a,a, . .. an lai E A!;;;:¡;•¡, n E z·
¡,.¡
U A"
..,,.
M= (S, ,, O, V, 0>)
...
U A•: la clausura de Klccnc del lenguaje A
Una máquina de estado finito M con estados internos S, alfabeto
de entrada ,. alfabeto de salida -0, función de siguiente estado
v:sx, ➔ Svfunción desalidaro:S ➔ i ➔ O
NOTA C IÓN
CONJUNTOS z el oonjunto de eateros: {O, 1, -1, 2.-2. 3, -3, ... }
ESPECIALES N el conjunto de los enteros no negativos o de los nómeros
DE NÚIIEJIOS naturales: {O, 1, 2, 3, .• . }
z· el coojunto de los enteros positivos: {l, 2. 3, ... } =
{xEZlx>O}
Q el oonjunto de números racionales: {alb Ia, b E Z, b f. O }
Q· el coojunto de números racionales positivos
Q• el coojunto de números racionales distintos de cero
R el oonjunto de números =les
R• el conjunto de números =les positivos
R• el conjunto de números =les distintos de cero
e el conjunto de números complejos: {x + yi x, y E R, i' = -1}
c- el oonjunto de nllmeros complejos distintos de cero
z. {0, 1, 2. ... ,n -l}paranE z•
[a,.b) el intervalo cerrado dea a b: {x E RI as x S bl
(a. b) el inrervalo abierto de a a b: {x E RI a< x < b}
[a,b) un intervalo semiabiertodea ab: {x E RI a S x < b}
(a, b) un intervalo semicerrado de a ab: {x E RI a <x :s b}

(R.+.·) R es un anillo coo open,cio... binarias+ y•


R(x) el anillo de polinooúos sobre el anillo R
grado/(x) el grado del polinomio/(x)
(G, o) Ges un BJUPO bajo la open,cióu binaria •
s. el BJUPO simttrico den símbolos
aH una clase lateral izquierda del subgrupo H (en el grupo G):
{ahlh EH}
(91,+,· .-.0.11 el '1gebra booleana 91 coo operaciones binarias + y · , la
operacióu unaria- , y los elementos neutros O (para+) y !(para•)

G •(V. E) Ges un grúo coo un coejunto de vtrtices Vy un coojunto de


aristas E
K. el grúo completo con n vtrtices
G el complemento del grúo G
grad(V) el grado del vtrtice u (eo un grafo no dirigido G)
od(v) el grado de salida de un vtrtice u (en un grafo dirigido G)
id(v) el ¡rado de eatrada del vtrtice v (en un grafo dirigido G)
K(G} el nllmero de componentes conexas del grafo G
K.,. el grafo completo bipartito en V= V, U V,. donde
V, n V,= 1,1V,I= m,I V,1=n
ll(G) el número de independencia de G
x<GJ el nllmero cromático de G
P(G,A) el polinomio cronwico de G
y(G) el nllmero de dominacióu de G
L(G) el grafo de línea de G
T= (V,E) T es un '1bol con un conjunto de vértices V y un conjunto de
aristas E
N=(V.E) N es una red de lraDSporte coa un conjunto de vátices V y un
coniunto de aristas E
Capítulo 1
Principios fundamentales
de conteo

cienes 1.1 y l . a ) 13 b) 40 e) La regla de la suma en la pane (a); la regla del producto en la parte (b)
.2-pág. 12 3. a) 288 b) 24 -
5. 2 x 2 x 1 x 10 x 10 x 2 = 800 placas diferentes
7. 29
9. a) (14)(12) = 168 b) (14)(12)(6)(18) = 18,144 e) 73,156,608
11. a) 12 +2= 14 b) 14 X 14 =196 e) 182
13. a) P (S,8) = 8! b) 7! e) 6! d) 2(6!)
15. 4! = 24
17. 26 + 26(36) + 26(36) 2 + ... + 26(36)7 - 36 = 21•• 26(36) 1 -36
19. a) 7! = 5040 b) (4!)(3!) • 144 e) (5!)(3!) • 720 d) 288
21. a) 8!/3! = 6720 , b) 6! = 720
23. a) 12!/(3 12!2!2!) b) 2(11!/(3!2!2!2!)) e) (7!/(2!2!))[6!/(3!2!)]
25. 12!/(4!3!2!31) = 277,200
27. a) 11 = 10 b) n = 5 e) n = 5
29. a) (10!)/(2!7!) = 360 b) 360
e) Sean x. y y• números reales cualesquiera y sean,n, n y p enteros no negativos cualesquiera.
El número de trayectorias de (x. )\ z) a (x + m, y+ n, z + p) seglln lo descrito en la pane (a),
es (m + n + p )!/(m!n!p!),
31. a) 576 b) La regla del producto
33. a) 9 X 9 X 8 X 7 X 6 X 5 = 136,080 b) 9 X 10'
(i) (a) 68,880 (b) 450,000
(ü) (a) 28,560 (b) 180,000
(iij) (a) 33,600 (b) 225 ,000
1 10
35. a) 2 • b) 3
37. a) 6! b) 2(5!) = 240
39. a) 10 + 128 + 1281 = 16.522 bloques: 8,459.264 bytes
b) 10 + 128 + 1281 +·128>= 2.,Ll 3.674 bloques: 1,082,201 ,088 bytes

ección 1.3 - pág. 29 l . (~) =6 1/(2!4!) = 15. Las selecciones de tamaño 2 son ab. ac, ad, ae, af. be, bd. be. bf, cd, ce, cf.
de, dfy ef
3. a) C(lO, 4) = 10!/(4!6!) = 210 b) (',') = 12!/(7!5!) = 792
e) C(14, 12) = 91 d) (lij) = 3003
5. a) P(S. 3) • 60
b) a , f, m a, f, r a, f,t a, m, r a, m, t
a, r, t f_, m,r f, m, t f, r, t m, r, t
7. a) (~) = 125,970 b) (t)(t) ª 44.100 e) r:.
1(,21221)(~)
d ) "'
¿.,'"
.. , ('")(
i 'º )
12-, e) }:;2s (',º)(il'!.,)
9. a) G) 28 = b) 70 e) (!) = 28 d) 37
JI. a) 120 b) 56 e) 100

S-1
S-2 Soluciones

13. a) (~')(l)(l)(l) = 12!/ [(3!)'] = 369,600 bl 207,900

15. (:)(4~!,) = 7350


17. a) ('¡') = 105 b) (';') = 2300; (';');f.') = 12,650
1
19. a) }:le,!1 b) i,.,-k ,

d) ~,(-!Yº'¡'= }:¡. ,(-l)'º'k'

e)
~ " !..:!:.!_
..,..• + ¡
~• (-l)'[n+~
g) .,,,.. (2i)!J
21. a) (';') + ('!')(l) + (',") = 220 b) (~") + (',") + ('!'}(l) + ('!')(n • 705
e) 2'°(}:/.. (1\'))
23. a) (;) b) (;)-n- n (n-4),na,4
25. ■) (~ b) (~')(2') e) (~}(2')(- 3)'
27. a) (,.1.,) • 12 b) 12 c) (,.1.,)(2)(-1)(-1)'• - 24
d) -216 e) (,.l,.,)(2')(- 1)'(3)(-2)'= 161,280
29. ■) z1 b) 210 e) 310 d) 4~ e) 410
31. (2n) ( 2n ) (2n)! (2n)!
n +\n - 1 =~+ (n -l)!(n +l)!
ª (2n)!(n + !)' + (2n)!(n)(n + 1)
(n + l)!(n + l )! (n + l)!(n + l)!
= (!c\[(2n)!(2n + 2)(n + 1) + (2n)!(2n + 2)•]
V (n + l)!(n + I)!

33. m+} (m+n)! (m+n)!


-me::) (m+n)!
n ( m =n m!n! =m!(n-1)! = (m + I) (m + l)(m!)(n -1)!
(m + n)! (m +n)
=(m + 1) (m + l)!(n -1)! (m + l)'v,, + 1

3S. Coosi<l=mos los desarrollos de (a) ((1 + x) - xr; (b) ((2 + x) - (x + l))'; y
(c)[(2 + ,)-x}' .
Y'T. a) a3-0c, b) a, - a,; e) ;a,-! =;i'

5ecci6n 1.4 - pág. 40 l. •l G:) b) (l} c)Ci) 3. ~ ) 5. a) 2' b) ?"


7. a) (ll) b) (',l) e) C'i) d) 1 e) (!;l) fJ (ll)-(l)
9. n = 1
11. a) (\') b) ('/) + 3(~") + 3(l) + (!)
13. ■) (l) b) i /_.(;:i)
IS. ~)(24!) 17. a) (lí) b) S 12
19. ■) (S')(ll) b) (~) + 4'(:l) + 4'(:!) + 4'(:)] + (4'(ll) + 4(ln + 4'(:!) + 4'0) + 4')
21. 0') 23. 24,310 • ~-, i (paran = ('f))
25. a) Consideremos el siguiente segmento de programa co Pascal, donde i, j, A; m, n y suma son
variables enteras.
su::ia:::0;
For i: "'l tondo
For J:-ltoido
Fork:•l tojdo
Form : - 1 tokdo
suma: :a:: suma + l;
Soluciones S-3

0espu& de ejecutar el segmento, el valor de suma es(":>). el n11mero de formas de


seleccionarí,j, l. mdc {l. 2. 3, ... , n}. permitiendo las repeticiones y 1 :s m :s A: s j :S
; :s n.
{1)
Para cada valor de ;, los ciclos For de;, .t y m producen 1 ) tjccuciooes de la proposi-
cióD suma;= suma+ t. F.o. comecuencia.(": )::
3
I:.(~ 1
}
b) Del cjacicio 24 sabemos que í:/ • (1!6)(nXn + IX2n + 1). Usaremos esto en el paso,
del a,¡¡umcmo siguieote:

(1) (",') - ¡;_, (';')


(2) (1/4!)(n + 3)(n +2)(n + l)(n) • I;.1 (1/6)(i + 2)(i + !)(,)
(3) (1/4)(n + 3)(n + 2)(n + l)(n) • I;.1 (i'+ Ji'+ 2,)
(4) (1/4)(n + 3)(n + 2)(n + l)(n) • I;. ,i' + (3)(1/6)(n)(n + 1)(2n + 1)
+ (2)(1/2)(n + l)(n)
(S) (1/4)(n + l)(n)((n + 3)(n + 2)-2(2n + 1)- 4) = I:..,i'
(6) (1/4)(n + i)(n)[n' + Sn +6 - 4n -6) • I;.,i'
(7) (1/4)(n+l)'n' • I:..,i'.

rT. a) Colocamos uno de los m objetos - en cada uno de los n recipientes distintos. Es,,
deja m - n objetos idtnticos por colocar en los n recipientes distintos, lo que produc
("":::H) =(::!)=(::;') distribuciones. .

L (l)(l) + (l)(l) + (;)(l)


Ejercicios
3_ a) 36+36'+···+36'• I/., 36' b) (36+36'+ ···+36')
complemtntaños- <) (51 +SI'+···+ 51')(36')
pig. 46 5. a) 10" b) (10)(11)(12) · · · (34) • 34!/9! <) (25!~)
7. a) C(i2,8) b) P(l2,8)
,. ((7!/2!)(!)](6)
11. a) 12 b) 49
13. (1/10){10!/ (4!3!3!))
15. •> (i) (l)+(l)(l)+(:) (ü) (!)+(OO)+(l) (iü) (!)+(OO)+(l)-9
bl (i) (l)(l) + (l)(l) (u) and (w) (l)(l) + (l)(l)
11. •l 2(!) + G) • 343 bl (2('!)-9) + [('f)-1) • 1200
1'. a) (5)(9!) b) (3)(8!)
21. Gl) - 5(!)
23. 2(••;::-•) + (n + r-k - 1)("•;::· ')
25. 0 • (1 +(-l)r • (:)-(;) + (;) - (;) + · · · + (-1)"(:).
so(: )+(:)+(:)+ ... •(;)+(;)+(;) + ...
rT. a) P(20, 12) • 20!/8! b) (~')(12!)
2'. a) CT) + ('l') + · · · + (l!) • ~-o(:;,'.) b) I!..('..')
e) n •2k + 1, k:2::0: ~ 0 (*t/;')
n • 2k, ka, 1: ¡,'..(';')
31. •>(~<::;>· •¡- (:--~ = (;:I) bl ~• • (;:l) • ("ó') + (";') + · · · + (::l) ~ r·•
33. a) 11!/(7!4!) b) (11!/ (7!4!))- (4!/ (2!2!))14!/ (3!1!))
<) (11!/(7!4!)) + (10!/ (6!3!1!)) + (9!/(5!2!2!)) + (8!/(4!1!3!)) + [7!/(3!4!))
( en la parte (a))
{111!/(7!4!)) + (10!/(6!3!1!)] + (9!/(5!2!2!)] + (8! /(4!1!3!)] + (7!/(3!4!)D
-[{14!/ (2!2!)] + (3!/ (1!1!1!)] + (2!/ 2!1} X {14!/ (3!1!)) + (3!/ (2! i !)IlJ
( en la panc (l¡))
S-4 Soluciones

C..pltulo 2
Fundamentos d• lógica

sección 2.1 - pág. 58 t. Las oraciones de las partes (a). (d), (e) y (b) son proposiciones. Las otras cuatro oraciones DO
lo son.
3. a) O b) O e) 1 d) O
S. a ) Si el ttiángulo ABC es equilitc:ro. entonces es isósceles.
b) Si el triángu.loAJJC no es isósceles, entonces no es equilátero.
d) El triángulo ABC es isósceles. pero oo es equilátero.
7. a ) Si Daniela practica su scivicio diariamente. entonces teDdra una buena posibilidad de ganar
el tom«> de tcws.
b) Si usted no arregla mi aire acondicionado, entonces yo no p ~ La renta.
e) Si a Maria se Je permite subir a la motocicleta de Luis, entonces debe usar su casco.
9. Las proposiciones (a), (e), (O y (b) soo rautolog{as.
11. a) 2'= 32 b) 'Z'
13. a) p:O; r : O;s:O
b) La proposición sdebe tener el valor de verdad l . pero la proposición p puede tener el valor
de verdad Oo 1, al igual que la proposición r.
15. a) m=3, n =6 b) m=3, n=9 e) m= l8, n•9 d) m=4,n=9
t) m =4, n=9 f) m= 4, n=9 &) ma4,n = l9
17. V 19. Rodrigo

Sección 2.2 - pág. 74 ! . a) (i) p q r qA r p➔ (q A r) p➔f p➔ r (p➔q) A(p➔ r)

o o o o 1 1 1 1
o o 1 o 1 1 1 1
o 1 o o 1 1 1 1
o 1 1 1 1 1 1 1
1 o o o o o o o
1 o 1 o o o 1 o
1 1 o o o 1 o o
1 1 1 1 1 1 1 1

(üi) p q r , v, p ➔ (q Vr) p➔ q 7 T-+(p -+ f )

o o o o 1 1 1
o o 1 1 1 1 1
o 1 o 1 1 1 1
o 1 1 1 1 1 1
1 o o o o o o
1 o 1 1 1 o 1
1 1 o 1 1 1 1
1 1 1 1 1 1 1

b) [p ➔ (q V r))<>(,r ➔ (p➔ q)] De la parte (iii) de la parte (a)


<>(-ir- (,pV q)] Por la segunda regla de sustitución.
y(p ➔ q)<e>hp v q)
Por la primera regla de sustitución,
y (, ➔ t) <=> (-.r -+ -,s) para cualesquiera
proposiciones primitivas s, l
Soluciones S-5

<c>((,-,p/\-.q)➔ ,j !'orla ley de DcMorgan, la regla dela doble


ncgacióD y la segunda regla de sustituci6o
<e> ((p 1\-.q )➔ ,j Por la rc¡la de la doble ocpcióa y la segunda
regla de sustitucióo

J. a) Paracualquicrpropoocióoprimitivat, t V--.,.,_ T• R,emplacecadaocurrmciadetpor


p V (q A r), y d resultado se sigue por la primera regla de SWlitucióo.
b) Paracualesqu.icraproposiciooes primitivau, t, tenemos(s ➔ t) c::,. (-,t ➔ -.1). Reemplace
cada ocurrencia des por p V q y cada ocwrcnciade, por,. y el resultado es consecuencia
de la primera regla de SUSÓDlaÓll.
e) Para cualcsqwert proposicioocs primitivas a.by e, la ley di!.tributiva de V sobre A csta-
blcoc que a V (b II e) <e> (a V b) 11 (a V e). El resultado dado aqul et una tautolog!a
cuando aplicamos la primera regla de sustiruci.óo con los reemplaumientos: a por ((p V q)
➔ ,J; bporr. y cporr.
5. •>Xarina poso sus estudios ames que su inlcrfsco se,..udladecinc. pero DO batcmdo (aalo)
una buena educación.
b) Norma no esti haciendo su tarea de matc:mtticas o Oaudia no cid practicando sus leccio-
nes de pi.ano.
e) Lorcmo se fue de VIC8CÍOOd y no se preocupó por viajar ca avióo, pero (ú i) DO se dMene.
d) Homero aprobó su curso de Pascal y tcnnin6 su proyecto de estructura de datos, pero no 5C
graduó al fioal del scmcsn..

7. •> , f c, , v f) /\(p /\{p /\f )) p fl.f


o o o o
o 1 o o
1 o o o
1 1 1 1

b) (-.p /\q) y (p y (p y q))<c>p y q

a) Si hoy ct d día del trabajo, entonces mallaoa es martes. (VERDADERO)


Co,uraposiliva: Si maft.ana no es martes, entonces hoy no es el día del trabajo (VERDADE-
RO)
Rttfproca: Si maft.ana es manes, entonces boyes d día del trabajo.
Inverso: Si hoy oo es el día del trabajo, entonces maftana no es martes.
Coosidcrcmos cualquier hmcs del allo, cxccp<o d día dd trabajo. Entooc:cs la rccfproca
y la inversa son FALSAS. Sin embat¡o, para cualquier otro día. la reáproca y la inversa
soo VERDADERAS.
b) Si - 1 <3 y 3 + 7 = IO,cotonccs seo(\!). (VERDADERO)
R«fpro= Si sco('f) = -1, cntooccs-1 <3 y 3 + 7 = 10. (VERDADERO)
fnv,na: Si - 1 >: 3 o 3 + 7 ,< 10, cntDOCct sco('f),< - 1. (VERDADERO)
Contrapositiva: Si scu(f) P. - 1, cntooces -1 ~ 3 o 3 + 7110. (VERDADERO)
e) Si Paco vive en Nueva Inglaterra. entonces Paco vive ea Vcrmont.
Conrraposi.dva: Si Paco DO vive en Vcrmoot, entonces no vive en Nueva In.&Ja.tt:na
Si Paco vivien en Vermoat o en cuaJquicr parte fuera de Nueva mgbtt:n-a. enlODOCS la
implic:ac:ióo dada y su cootrap05itiva serían VERDADERAS. Si Paco vive en Nueva ln&la-
terra pero oo en Vennont. entonces la implicación dada y su concrapositiva serfan FALSAS.
Recíproca: Si Paco vive en Vennont. emooces Paco vive en Nueva lnglatetra. (VERDA-
DERO)
lnv~rso: Si P-aco no vive en Nueva Inglaterra, entonces Paco DO vive en Vermoot. (VER-
DADERO)
S-6

13. z Figura 2.l(a) Figura 2.l(b)


•>2 20 10+1=11
b) 6 20 10+5=15
e) 9 20 10+8=18
d) 10 20 10+9=19
•> 15 20 10+10=20
O n>lO 20 10+ 10= 20

15. p q r [(p-q)/\(q-r) /\(r-p)] ¡e,... ,¡/\ e,... , ¡/\¡, ...,M


o o o 1 1
o o 1 o o
o 1 o o o
o 1 1 o o
1 o o o o
1 o 1 o o
1 1 o o o
1 1 1 1 1

17. a) (p j p) b) (p i p) f (q j q) e) (pi q) t (pi q) d) P t (q 1 q)


e) (r f s) f (r f s), donde rrcpresentap f (q f q) Y, n:¡msentaq f (p f p)

19. p q ,e, i q) (, p t , q) ,¡p i q) (,pi , q)

o o o o o o
o 1 1 1 o o
1 o 1 1 o o
1 1 1 1 1 1

21. •J p V [p /\ (pvq)) Razones


<'>p V p Leydeabrorción
<'>p Ley idempotente de V

e) [(,pv,q)-(p/\q/\r)) Razones
<c>,(,p v ,q) V (p/\q /\, ) S ➔ l<=:>-,s VI
c,,(,, p /\ ,,q) V(P /\q /\ r) Leyes de DeMorgan
<'>(p/\q) V (p /\ q /\ r) Ley de la doble negación
<'>p /\ q Ley de absorción

d) p /\[(,q ... (r /\ r)) v ,[q V ((r /\ s) V (r /\ ,,)))) Razooes


,:,,pi\ [(,q ... r)v,[q V ((r /\s)V(r/\,s)))) Ley idempotente de /\
<'>p /\[(, q ... r)v ,[q V(r /\(, v ,,)))) Ley distributiva de /\ sobre V
<'>p /\[(,q... r)v,(q V(r /\ T,))) Ley inversa
<'>p /\[(,q-,) v,(qvr)) Ley del neutro
<'>p /\ [(,,q v , )v,(q v r)) t ➔ u~-.tVu
<'>p /\[(q v ,Jv ,(q v r)) Ley de doble negación
<:>p /\ To Ley inversa
<'>p Ley del neutro
Soluciones

Sección 23 - 1"'9. 95 l . o) JI r ,-, <P V t) <Pv,i-,


o o o' 1 o 1
o o 1 1 o 1
o 1 o 1 1 o
o 1 1 1 1 1
1 o o o 1 o
1 o 1 o 1 1
1 1 o 1 1 o
1 1 1 1 1 1

La v.alidez cid ar¡umcnto se si¡uc de los rc,ultados de la 1lltima fila. (Podemos pasar poc
alto las primeras siete ftlas.)

e) p r , v, P V(t Vr) .,, , v,


o o
'o o o 1 o
o o 1 1 1 1 1
o 1 o 1 1 o o
o 1 1 1 1 o 1
1 o o o 1 1 1
1 o 1 1 1 1 1
1 1 o 1 1 o 1
1 1 1 1 1 o 1

Los rosultados de las mas 2, S y 6 esublcccn la validez del a,¡¡umcoto dado. (Podemos
pasar poc alto los resultados de las ocnu cinco mas de 1a tablL)
3. a) Si p tiene d valor de verdad O. tambi& p /\ q.
b) Si p V q dcoc el valor de verdad O, entonocs el valor de verdad de p (y el de q) es O.
e) Si q tiene valor de verdad O, entooecs el valor de venlad de ((p V q) /\ ~Pl es O, sin
importar el valor de verdad de p.
d) La proposición q V s tiene valor de verdad Osólo cuando cada una de las proposiciones q,
s tienen el valor de verdad O. Eotooces (p ➔ q) tiene el valor de verdad 1 si p tiene el valor
de verdad O; (r ➔ J) tiene el valor de verdad 1 si r tiene el valor de verdad O. Peto eotooces.
(p V q) tiene el valor de venlad O, no l.
e) Para (-.p V-. r), el valor de verdad es Ocuandop y rtiencn valor de verdad l. Est0 obliga
emoooes a qoeq.s tomen el valor de verdad l. para que (p ➔ q). (r ➔ s) tengan valor de
verdad l . Sin embar¡o, esto produce el valor de verdad Opara (-.q V -.J).
S. a) Regla de la simplificación disyuntiva
b) No válido; es un inlentO de argumentar por la reclproca
e) Mod,,s Tolkns
d) Regla cid silogismo <fisyundvo
e) No válido; intento de argumentar por la reciproca
1) Ley del silogismo
7. 1) y 2) Premisa
3) Pasos ( 1) y (2) y la regla de separación
4) Premisa
5) Paso(4) y (r ➔ -..q)<=> (...,.....q ➔ -.r) ~ (q ➔ -.r)
6) Pasos (3) y (5) y la regla de separoción
7) Premisa
SOiuciones

8) Pasos (6) y (7) y la regla del silogismo disyuntivo


9) Paso (8) y la regla de amplificación disyuntiva
9. a) 1) Premisa (la negación de la conclusión)
2) Paso(l ) y-.(-,q ➔ s) e::> --,{--,-,q V s)<=>-.(q V s)<::::>-,q A -,s
3) Paso (2) y la regla de la simplificación conjuntiva
4) Premisa
5) Pasos (3) y ( 4) y la regla del silogismo disyuntivo
6) Premisa
7) Paso (2) y la regla de la simplificación conjuntiva
8) Pasos (6) y (7) y Modw Tollens
9) Premisa
10) Pasos (8) y (9) y la regla del silogismo disyuntivo
11) Pasos (5) y (10) y la regla de conjunción
12) Paso (11) y el método de demoslracióo por contradicción
b) 1) p-q Premisa
2) ,q - , p PasoOJy(p ➔ q)<e>bq ➔ ~pJ
3) p vr Premisa
4) ,p _r Paso(3)y(p Vr).,.hp ➔ r)
5) , q_ , Pasos(2)y(4)ylaleydelsilogismo
6) -ir vs Premisa
7) , -s Paso (6)y(-,rV s)<=>(r ➔ s)
8) .·.,q-s Pasos (5) y (7) y la ley del silogismo
e) 1) 1p++q Premisa
2) (,p-q) /\(q-,p) Paso(l)y(~p <->q) <e>((~p ➔ q) /\ (q ➔ ~p)]
3) •p - q Paso (2) y la regla de simplificación conjuntiva
4) q-r Premisa
5) ,p _ r Pasos (3) y (4) y la ley del silogismo
6) Premisa
7) :.p Pasos (5) y (6) y Modus ToU,ns
11. a) p:l q :0 r:1
b) p:0 q:O r:Ool
p:O q:l r:l
c)p. q, ,:1 s:O
d) p,q, r : 1 s:O

Sección2.4- pág.114 1. a) Verdadero b) Falso e) Falso d) Veroadero •l Falso


1) Veroadcro g) Falso h) Falso i) Falso
3. Las proposiciones (a), (e) y (e) son verdaderas y las proposiciones (b). (d) y (f} son falsas.
S. a) 3r (m(r)/\c(r)/\j(r)} verdadero
b) 3r(s(r)/\ c(r)/\,m(r)} verdadero
e) Vr(c(r)- (m(r)Yp(r))} falso
d) Vr[( g(r)/\c(r)) - ,p(r)}, o verdadero
Vr((p(r)/\c(r))- , g(r)}, o
Vr((g(r) /\p(r))- ,c(r)}
e) Vr((c(r)/\s(r))-(p(r) l! e(r ))} verdadero
f) 3r(g(r)/\,m(r)/\,p(r)}, o
3r(g(r)/\,(m(r) vp(r))] verdadero
7. a) (i) 3rq(r)
(ii) 3r(p(r) /\q(r)]
(iii) Vr (q(r)- -.r(r)]
Soluciones S-9

(iv) Vi[q(x)--,r(x))
(v) 3.x [q(x)/\t(x)]
(Y!) Vx [(q(x) /\ r(x))-s(x)]
b) Las proposiciones ú), (ú), (v) y(vi) son verdaderas. Las proposiciones (iii) y(v) soo falsas;
x-= 10 es un contraejemplo de cada u.na de estas proposiciones.
e) (i) Si x es un cuadrado perfecto. cntooces x > O.
(li) Si x es divisJ.l,le entre 4, cn100CCS x es par.
fui) Si z es divisible entre 4, cn1onces x oo es divisible eotrc: 5.
(iv) üistc un entero que es divisible entre 4, pero no es un cuadrado perfecto.
d) (i) Sea x • o. (iii) Sea x = 20.
, _ a) (i) VC!dodero (ü) Falso =
Considere x 3.
(IÜ) Vc:rdadcro C,v) Vc:rdadcro
<) r,) Vc:rdadcro (ii) Vc:rdadcro
(IÜ) Vetdadero r,v) Falso =
Patax 2 o 5, el valor de verdad dep(x) es l.
mientras que el de r(r) es O.
ll. a) Las variables.,; y soo -a<las; oo hay variables libres.
b) En este caso, la variable x es li~. mientras que las variables y, z soo acocadas.
e) W variables x. y son ac:otaim; la variable z es libre.
d) La variable :r H acotada. pero la variable y es libre.
13. a) p(2,S) yp(3.S)Vp(S.S)
b) p(2,3)/\p(3,3) /\p(S,3)
d) [p(2,2) V p (2,3) V p(2,5)] V [p(3,2) V p(3,3) V p(3,5)] V [p(S,2) V p(S,3) V p(S,5)]
e) [p (2,2) /\p (2,3) /\p(2.S)] /\ [p(3,2) /\ p(3,3) /\p(3,5)] /\ [ p(S,2) /\ p(S,3) /\ p(S.S)]
15. a ) La negación propuesta es correcta y es un.a proposición verdadera.
b) La nepci6n propuesta es incorrecta. Una versión correcu de la negación es: Para todos tos
nClmeros raciooaJcs x. y, la sumax + J es racional. E.su vttSi6D correcu. de la ne¡acióo es
una proposición verdadera.
d ) La negación propuesta es incoaecta. Una versión correcta de la oepción es: Para todos los
enteros .x, y, si x y y son impares, entonces X'J es par. La proposición (ori¡inal) es verdadera.
e) La oegaci6o propuesta es i.ocoaecta. Una versión correcta de la negación es: Existe un
nt!mero raciooaJ cuyo <nadt>do es uracional. La proposición (original) .. vcnladen.
17. a) Existe un entero n tal que n no es divisible entre 2 pero n es par (es decir, no es impar).
e) Existen enteros k. m. n t.aJcs que k - m y m - n son impares y k - n es impar.
e) Existeunllllmerorealxt.alque lx-3l<7yx s -4ox>: 10.
1'. a) Proposición: Pan.todos loscaaos positivosm. n.sim >nentooccs,,,,.>tr1. (VERDADERA)
&cíproca: Para todos los enteros positivosm. n. sim2>rr1.eotoocesm > n. (VERDADERA)
Inversa: Para todos los enttros positivos in. n. si m :s "• entooccs ml S rr1. (VERDADERA)
Con.rropositiva: Para todos Jos CDlCl'OS positivos m. n. si m2 s n1• cntoooesm s n. (VER-
DADERA).
b) Proposición.: Para todos )os enteros a. b. si a> b. entooces tr>lr. (FALSA; sean a• l y
b•-2.)
Reciproca: Para todos los enteros a. b. si a1> Ir, entonces a > b. (FALSA; sean a •-S y
b•3.)
lnvma: Pan todos los enteros a, b. si a S b, enroocesa' :5 b'. (FALSA; sean a= -5 y b = 3.)
Conrraponrivo: Para todos los entero!a. #,_ sitr ~ #r,eotoocesa s b. (FALSA; sean a= 1
yb =-2.)
e) Proposición: Para cualesquiera enteros m, n y p. si n, divide a n y n divide a p, entonces m
divide a p. ( VERDADERA)
Recíproca= Para todos los enteros m, p, si m divide a p, entonces para cu.alquicr entero n se
sigue que m divide a n y n divide ap. (FALSA; scmm • l, n = 2 y p• 3.)
Inversa: Para eualesquiera.cnterosm, n yp, simno divide an on nodivideap.entoncesm
no divide ap. (FALSA; ~an m = J,n = 2 y p = 3.)
S-10 Soluciones

Contrapositiva: Para cualesquiera cnterosm y p. si m oo divide ap, entonces para cualquier


enteron se sigue que m no divide ano n no divide ap. (VERDADERA)
e) P,oposicién: Cualquiereru,rodivisiblecme 12131Dbitnesdivisil>le cme4. (VERDADERA)
Rtcíproca: CUalquier entero divisible entre 4 también es divisible entre 12. (FALSA; COD-
sidere el entero 8.)
lnvusa: Si un entero no es divisible entre 12, entonces no es divisible entre 4 . (FALSA;
considere el entero 8.)
Conrrapo.sitiva: Si un entero no es divisible entre 4. entonces no es divisible entre 12.
(VERDADERA)
f) P,oposicwn: V,t(r+4x-21 > 0) ➔ [(x> 3) V (x<-7)]] (VERDADERA)
R,c(proca: Vx[[(x > 3) V (x <-7)] ➔ (r+ 4x- 21>0)] (VERDADERA)
lnvusa: V,t(x'+ 4s-21 :5 0) ➔ [(x :5 3) /1 (x a: -7)]), o Vx[(r+4s - 21 :5 O) ➔ (-7 s
x s 3)] (VERDADERA)
Conzro¡,osiriva: Vx[[(x S 3) /1 (x a: -7)] ➔ (x'+4s-21 SO)), o Vx[(-7 S x s 3) ➔ Cx'
+4s-21 s O)J(VERDADERA)
21. a) Verdadera b) Falsa e) Falsa d) Verdadera e) Verdadera f) Falsa
23. a) Va3b[a+b=b+a=0] b) 3u\fa[au=ua=a] e) Va+03b[ab=ba=I)
d) La proposición de la parte (b) s;gue siendo verdadera, pero la proposición de la parte(<) ya
no es verdadera en este nuevo uru verso.
25. a) 3x3y[(x>y)J\ (x - ys0)) e) 3x3y[(x<y) /\Vz[x;,,z v za:y)]
e) [VxVy((x >0)/\ (y >0))) J\ [Vz(xz sy)) ·
'1:1• .!!?.!'•"L~3E>0Vk > 03n[(n > k) J\l r. - L I "'•]

Seai6n 2.5-pág. 134 1. Aunque podr!amos escribir28 =25 + 1 + 1 + 1 = 16+4+4+4, no hay formadccxprcsar28
como la suma de como mnimo tres cuadrados perfectos.
3. 30=25+4 + 1 40=36+4 50=25+25
D=M+M ~•25 + M+I D=36+M
34 • 25+9 44=36+4+4 54=25+25+4
36=36 46=36+9+1 56=36+16+4
38=36 + 1+1 48=16+16+16 58•49+9
5. a) FJ. nwncro real a: no es un entero.
e) Todo$ los dircctorcs administrativos saben cómo delegar autoridad.
d) El cuadrilárero MNPQ no es cquiangular.
e) Greta evita comer bfgado.
7. a) Cuando la proposici6n 3x [p(r) V q(x)J es verdadera. existe al menos un elemento e en el
universo dado tal quep(,c) V <l,.c)es verdadera. Por lo tanto. aJ menos una de las proposiciones
p(c),q(c) lieucel valor de verdad 1, por lo que al menos una delas proposiciones3xp(x)y
3xq(x) es verdadera. Por lo tanto, se sigue que 3xp(x) V 3x q(x) es verdadera y 3x [p(x) V
q(x)J => 3x p(x) V 3x q(x). Rec!procamcntc. si 3x p(x) V 3.rq(x) es verdadera. entonc<s al
menos una de las proposiciones p(.a) y q(,b) tiene valor de verdad 1, para algunos a. b del
universo prescrito. Supongamos. sin p&dida de generalidad. que es p(_a). F.ntonces p(_a) V
q(a)ticne valocdcverdad l,demodoque3.r[p(r) V q(x)] cs unaproposici6n verdadera.y
3xp(x) V 3x q(x) => 3x [p(x) V ~)].
b) Primero consideremos cuJndo una proposición Vx [p(x) A q(x)] es verdadera. Esto ocwrc
cuandop(.a) A q(a) es verdadera para cada a del universo dado. Entoooesp(a) es verdaden.
(al igual que q(a)) para todo a del universo, de modo que las proposiciones Vx p(s) y Vx
q(x) ,oo verdaderas. Por lo tanto. la proposici6n Vx p(x) /1 V.rq(x) es verdadera yVx [p(s)
/1 q(x)] => Vxp(x) A Vxq(x). Rccfprocamcnte,,upongamosque Vxp(x) /1 Vxq(,)es una
proposición verdadera. EntoncesVxp(x), Vx q(x) son ambas verdaderas. Ahora, scac cual-
quier elemento del universo prescrito. Entonccsp(c), q(c) y p(c) /\ q(c) son todas vmlade-
ras. Y como elegimos e en forma arbitraria. se sigue que la proposición Vx [p(x) /\ q(x)Jes
verdadera, y Vx p(x) /1 Vx q(x) => Vx [p(s) /1 q(x)].
Soluciones S.11

9. 1) Premisa
2) Premisa
3) Paso (1) y la regla de especificación univemu
4) Paso (2) y la regla de especificación univemu
S) Paso (4) y la regla de simplificación conjuntiva
6) Pasos (5) y (3) y ModJu Pon,ns
7) Paso (6) y la regla de simplificación conjuntiva
8) Paso (4) y la regla de simplificacióo conjuntiva
9) Pasos (7) y (8) y la regla de conjunción
10) Paso (9) y la regla de generalización univemu
11. Consideremos las proposiciones abiertas
w(x): .:e: trabaja para la unión de cr61ito
1 (%): x elabora solicitudes de pmtamos
c(x): x sabe COBOL
q(x): X sabe Quattro
y sean re; las letras que representan a Roxana e ln6s, respectivanlCntc.

En forma simbólica, el argumento dado es el siguiente:

Yx[w(x) -+c(x))
Yx[(w(x) A t(x))-+ q(x)]
w(r)A-.q(r)
q(i)A-.c(í)
.-.-,((r)A-.w(i)

Los pasos (y razones) oecesarios para verificar el aigumento son los siguientes.

1) Yx[w(x)-+c(x)] Premisa
2) q(i)A -.c(i) Premisa
3) -ic(i) Paso (2) y ta regla de simplificación c.onjuntiva
4)w(i)-+c(i) Paso (1) y la regla de especificación universal
S) -.w(i) Pasos (3) y (4) y ModJu To/Jau
6) Yx[(w(x)At(x))-+ q(x)] Premisa
7) w(r)A-.q(r) Premisa
8) -iq(, ) Paso (7) y Ja regla de la simplificación conjuntiva
9) (w(r)A t(r))-+q(r) Paso (6) y la regla de especificación univemu
10) -.(w(r)At(r)) Pasos (8) y (9) y ModKS Tollau
11) w(r ) Paso (7) y la regla de la simplificación conjuntiva
12) -.w(r) v-.t(r) Paso (10) y ley de De Morgan
13) -.t(r) Pasos (11) y (12) y la regla del
silogismo disyuntivo
14) .-.-,t(r) A -.w(i) Pasos (13) y (5) y la regla de conjunción
13. •) Conrrapositiva: Para todos tos enteros k y I, si k y I no son ambos impares, entonces kf no es
par O, para todos los enteros k y f. si al menos uno de ellos es par. entonces kf es par.
Demostración: Supongamos (sin p&dida de generalidad) que k es par. Entonccsk = 2c para
= =
a1gán entero e, debido a la definición 2.8. Ento~ kf (2c)f 2(d), por ta ley asociativa
de la multiplicación de enteros, y des un entero. En consecucocia. kf es par, de nuevo. por
ta definición 2.8. (Observe que este resultado no pide nada especial acerca del entero f.)
S-12 Soluciones

15. Demostracwn.: Suponpmos que para algán entero n, n' es impar mic:otras que n DO lo es.
Entonces n es par y podemos escnbir n = 2a. para algón entero a, por la definición 2.8. En
comc.cueocia.n2 = (2a)2 = (2aX2a) =(2 • 2)(a • a), por las leyes conmutativa y asociativa dela
multiplicación de enteros. Por lo talllO, podemos escribir"'= 2(2a'), doode 2a' 05 un entao, lo
que significa que n 2 es par. Así. hemos llegado a una cootradicción, ya que tmemos que 7r es
impa,(al principio) y tambi6t pat: Estacootradic:ciónsurgede la hipótesis falsa de quen noes
impar. Por lo tanto, para todo entero n, se sigue que n2 impar~ n impar.
17, l><mosrracién:
(1) e.amones impar, tenemos que n =2a + l para aJgón cntc:ro a. &tonces n + 11 = (2a + 1) +
11 =2a + 12== 2(a +6), doadea +6es unentero. As(.porladefinición28, se s:iguequen+ 11
es pa,.
(2) Si n + 11 no es par, entonces es impar y tenemos que n + 11 =2b + 1 para algtm entero b.
De este modo, n =(2b + 1) -11 ='lb- 10 =2(b-S), doode b-S ""un entero y se sigue
de la definición 2.8 que n es par, es decir, no es impar.
(3) En este caso, cooservamos la hipótesis (que n es impar) y suponemos tambifn que n + 11
no es par; es decir, es impar. MI~ podemos csaibir n + 11 = 2b + 1 para algwl entero b.
Esto implica entonces que n • 2(b - 5), para el entero b - S. Por la definición 2.8, se sigue
que n es par. Pero sin es par {como hemos demostrado) e impar (por hipótesis), llegamos
a una contradicción. Así, nuestra hipótesis era incorrecta y se sigue que n + 11 es par para
cualquier entero impar n.
19. Este resultado no es cierto en general. Por ejemplo, m =4 =: 2 2 y n = 1 = 12 son dos enteros
positivos que son cuadrados perfectos, pero m + n =22 + 12 = S DO es un cuadrado perfecto.
ll. Demo1troción: Demostraremos el resultado dado al establecer la vcnlad de su contrapositiva
(que es lógicamente equivalente).
Consideremos la negación de la conclU$ión; es, decir. .x < 50 y y< SO. Así, se sigue que .x +y<
SO+ SO= 100 y tenemos la negación de la hipótesis. El resultado dado se sigue entonces por
este método indirecto de demostracióo (por coorrapositiva).

Ejercicios l. I
complementarios- r ,(,vr)
p f s 9 /\ r ((9 /\,)-,(, V r)J p++I
pág.140
o o o o o 1 1 o
o o o 1 o o 1 o
o o 1 o o o 1 o
o o 1 1 o o 1 o
o 1 o o o 1 1 o
o 1 o 1 o o 1 o
o 1 1 o 1 o o 1
o 1 1 1 1 o o 1
1 o o o o 1 1 1
1 o o 1 o o 1 1
1 o 1 o o o 1 1
1 o 1 1 o o 1 1
1 1 o o o 1 1 1
1 1 o 1 o o 1 1
1 1 1 o 1 o o o
1 1 1 1 1 o o o
So'uciones S-13

3. •> , f
, ,.., , ...e, ...,> <, ...,> c,++f)++r
o o o 1 o 1 o
o o 1 o 1 1 1
o 1 o o 1 o 1
o 1 1 1 o o o
1 o o 1 1 o 1
1 o 1 o o o o
1 1 o o o 1 o
1 1 1 1 1 1 1

b) Las asignaciones de valores de vermdp: O; q: O; r. Oprodu<enel valor de verdad 1 paro


(p ➔ (q ➔ ,)]y el valor de vermd Opara [(p ➔ q) ➔ ,].En consecuencia, esw proposi-
cioocs DO 50D lógicamente equivalentes.
S. (1) Si catalina no pncuco sus lecciooes de piano.- no podñ ir al ciDe.
(2) Si catalina picma ir al cine."""""" dd>at ~ sus lecciooes de piano.

1. •> (, p v , qJ/\(IWpJ /\p


b) (,p v,q)/\(/i,Vp)/\p
co(, p v ,q)/\(p /\p) F0 V p~p
co(, p v ,q)A p Ley iclem¡,otalte de "
;;,,p /\ (, p v,q) Ley coomutativa de /\
e>(p /\,p)V(p /\,q) Ley diSll'ibotiva de /\ sobr< V
e>Ii,V (p /\,q) p /\ -.p~Fo
e>p /\ ,q F• es d oeuuo para V .
,. o) =apositiva b) inversa e) ccampositiva d) invcna e) inversa
1) c:ootr>positiva &) l<Cfproca

11. , f
, , t, <, t f) t , f t, , t (f t ,)
o o o 1 1 1 1
o o 1 1 o 1 1
o 1 o 1 1 1 1
o 1 1 1 o o 1
1 o o 1 1 1 o
1 o 1 1 o 1 o
1 1 o o 1 1 o
1 1 1 o 1 o 1

De !u columnas 5 y 7 vemos que (p t q) t , ~ p t (q t ,)


13. (! ) , f , ,.!+, ,.!+, {p4f) /\(f4T) ,.!+, [(p4f)A (t4 r)].!+(1.!+r)

o o o o o o o o
o o 1 o o o o o
o 1 o o o o o o
o 1 1 o 1 o o o
1 o o o o o o o
1 o 1 o o o 1 o
1 1 o 1 o o o o
1 1 1 1 1 1 1 1
S-14 Soluciones

La tabla aos muestra~ laJX'OlX)Sicioo [(p .!+q) I\ (q.!+r)] .!+(p.!+ r) noes ma tautologfa
(2) Considercmoslasasignacionesdevaloresdeverdadp: O,q: 1 yr: O. En este caso, tenemo.5
los valores de verdad rc$Ultantes:

p4q: o q2-r: o p4,: ¡


[(p2'q)l\(q 4r)J4(p4r): O
En consecuencia., la proposición [(p !tq) /\ (q .!+,)].!+(p.!+,) noes una tautología.
(3) Para este llltimo caso, supongamos que cada una de las prop:>Siciones p. q y r tieoeo la
uignaci6n de valor de verdad O. Entonces (p !+q),(q !+r) y (p !+ r) tienen el mismo valor
de verdad. a saber, O, lo mismo que la proposición [(p.!+ q) /\ (q !+ r}l !+ (p !+ r), de
modo que ésta no es una tautología

15. •> p 9 T pYq (p Y q)Yr q ll. r p Y(q Y r )

o o o o o o o
o o 1 o 1 1 1
o 1 o 1 1 1 1
o 1 1 1 o o o
1 o o 1 1 o 1
1 o 1 1 o 1 o
1 1 o o o 1 o
1 1 1 o 1 o 1

Se sigue de los resulrados en las columnas 5 y 7 que [(p Yq) Y,) <e> [p Y (q Yr)J.
b) Las proposicioocs dadas DO son lógicamente equivalentes. Las asignaciones de valores de
verdadp: 1, q: O, r: O proporcionan un contraejemplo.
e) Si p, q y r tienen el valor de verdad O, entonces. p ➔ (q Y. r) es verdadera, mientras que
(p ➔ q) Y: (p ➔ r) es falsa. de donde la implicación no es una implicación lógica.
17. a) ¼rdadcTa b) Falsa e) Verdadera d) Verdadera e) Verdadera
r¡ Falsa g) Falsa b) Falsa 1) Falsa j) Verdadera

Capitulo 3
Teoria d• conjuntos

Sección 3 .1- l. Son todos el Drismo conjunto.


pág. 154 3. Las partes (b) y (d ) son falsas y las demás vcnladcras.
5. a) {0,2} b) {2,2½,3½,5!, 7!} e) {0,2, 12,36, 80} el) {j,,';,i1,t,,,l,,,l,}
7. a) Vx[xeA-xEBJ /\ 3.,;[xEB l\xl!;AJ
b) 3.x[xEA l\xéBJ v Vx[xéB v xEAJ
OR, 3.x [xEA l\ xéB]VVx[xEB-xEA]
9. a) IA l =6 b) 1B1=7
e) Si B tiene 2ª subconjuntos de cardinal impar.entonces IBI = 11 + l.
11. a) 31 b) 30 e) 28
13. a) ('l') b) (':) e) ffl +(':')+ ffl+(':')
15. Sean W={ l},X={{IJ,2JyY={X,3].
17. e) SixEA,cntoncesA C: B ⇒ x EByBC C=>xE C. Porlotanto,A. s;: C. ComoBC C,
existe-, E C tal que y~ B. Además, A s;: By y f. B =>y~ A. En consecuencia.A C: C y
y E Ccony~A=>ACC.
d) Como A e B. se sigue que A. C: B. El resultado se sigue entonces de la pane (e).
S-15

19. a) Paran, k e z· con n .?: k + l, consideremos el beú¡ooo coo centro en (:). &te tiene la
forma
(:: :) (".')
(,!,) (:) (.:,)
(".') (:!:)
donde las dos temas alternantes,(:::), ( .:1) ,(''t') y(~).(:!:). (i,), satisfacen
(:::)(.:.)("':')-(";') (:::)(i,).
b) Par> n..t E z• roo• ;, k+ l.

(; =:)(i,: 1)(•; l) = [(k-<;);!>~ t)!L +1)!(:'-k -l)!][k!(~"++/~'kJ


-[k!(~·_-/~'k)!L };);!>~k)](k-1)!(:'_ +1J=(·;1)(;: :)(k ~1)·
k
JI. • - 20
23. ScaA. •{.t.~ ai.a~ .... a.}. El m1mero de subconjuntos de A de tamafto res (-;2 } Podemos
separarlos en cuatro clases: (1) los(:) subconjuntos~ oo contienen a .r ni a y. (2) los(.!l)
subconjuntos ~cootieoen a.rpero DO ay. (3) los(r=.) subconjuntos que contienen ay pero
oo a .i; y (4) los{~) subooojuntos que cootieoeo a .r y a y.
15. •>Si SE S,aitooces, comos= IAIA ~Al, rmemosqueS ~ s.
b) Si S ~ S, eo!ODCCS. poc la dcfinici6o de S, tmemos que SE S.

Stcción 3.2- l . a) {l,2,3,5) b) A e) y d) 'U -{2}


pig.167 e) {4.8) r¡ {l ,2,3,4,5,8) 11) f
~) {2,4,8} 1) {1,3, 4,5.8)
3. B •{a,b,d}o B ={a,b,<I
s. 9
7. •>1•1s l AnB lslB lslAuBlsl~I
e) l•l slA-BlslA lslAUB l s lA l+IB l "' l~ I
,. a) Vcrdaden b) Vctdaden e) Vctdaden d) Falsa e) Verdadera
1) \lcrtladcn 11) Vcrdaden b) Falsa i) Falsa J) Falsa
11. •> Seaa'II = {1,2. 31,A • {11, B = {21 yC= (31. Entonces A n C=B ne- f pero A ,1, B.
b) Par>'U = {l. 21.A = {l},B = {2} y C='U, tenemos que A U C= BU CperoA,; B. (De
w partes (a) y (b) vemos que no tenemos leyes de cancelación para n ni para u. Eslo
difiere de lo que sabemos de R. donde si a. b, e E R (i) ah ~ oc y a '1- O ⇒ b =e; y (ii) a +
b•a+c ⇒ bac.J
e) :rEA ⇒ .reA u c ⇒ .reBu C.Asl,.rEBo.rE C.Si.rEB.eor.oncesA. C:B. Si.rEC.
entoncesxE A n C = B n Cy.rEB. En ambos casos.A.~ B. De la misma forma.y E B ⇒
y EB u C=A u C.demodoqueyEA oy E c. Siy E C,cntonce,yE B n C=A n c.
En cualquk:r caso• ., e A y B ~ A. Por lo tanto, A ,: B.
d) Seaz E A. Consideremos dos casos: (l)z E C:::) z f: A ll C => z f; B ll C=>z e B.
(2)i f C=>i EA ll C=>i E B ll C=>iE B (pucu~ q. Encualquiercaso,iE B,de
modo que A(; 8 . De 11w1c1uitnilar, ses;gucqueB (;Ay A= B.

~ : ~¡1- •(A U B)n(A U B) n(AUB) n(AU B) b) A •AU(A ns)


<) A n B -(AUB) n{A u ii)n(AU B) d) A • (A n B) U(A n'U)
17. a) Sea '11 • (1, 2. 31,A • {11 y B= {21. Entoooes {l. 2) E~(A U B) pero
{l. 2} ~~(A) U~(B)
b) XE~(An B)~X(;A nB~ X(;A y X!:;B~XE~A) y
XE~(B)~XE~(A)n~B). por lo tanto ~{A ns) - ~A)n~(B}
S-16 Soluóones

19. a) '1.' b) 2"


e) En la tabla de pcrtc:nencia. A C B si las columnas de A y B son tales que siempre que
aparezca un I en la columna de A. existe un 1 correspondiente en la columna de B.

d) A B e AUif (AnB) U(Bnc)


o o o 1 1
o o 1 1 1
o 1 o O 1
o 1 1 o o
1 o o 1 1
1 o 1 1 1
1 1 o 1 1
1 1 1 1 1
21. a) A n(B-A)=An( B n A)=Bn(AnA)•Bn,-,
b) [(A nB) u (A n B nen D)] u CA n B)=(A n B) u CA n B}porla ley deabsordón =
(A UA)nB="ll nS=S
d) .iÍ U.BU(A n s nl')=(An1l/U[(A n B)nl']= [(A ns) U(A n B)]n
((A ns) ut) =((Añ"Eiul'] •.iÍU .B UG'
e) xu (A n.B) = CAUA) nCAUll)=AU.11 = .rns;Añ"B u [(A nB) n CJ =Añ"ñ uc
= A nene. Continuamos de esta forma para obtener A nsncnon ....
23. u:_,B.=S,={2,3,4, ...}= Z- - {ll, n!:.,B.=B,, = {13, 14,15, . ..1. u:_, B.=S,,
n;_,B. =B.
15.xEA U (f\618¡) <=> x EA o (xE B1paratodoi El)<=> Paratodoi E /,x EA o.r E B, ~ Para
IOdo i E /,x EA U S,<a>x E Íle,(AUSJ [A n <lJ..,S,l=U,p, ílB,) sesigueeotonces
por dualidad.]

Secciones 3.3 y
3.4- pág. 174 l. a) 24! + 24! - 22! b) 26! - (24! + 24! - 23!]
3. 9! +9! - 8!
S. a) 55/216 b) 5/54
7. a) ffi =i'l b) 2/15 e) 3/35 d) 1/2
9. 3/ZB
11. h (A ) = 1/3, h(B) =7/ 15, h (A n B) =2/15, h(A U B) =2/3;
h(A UB)=2/3= 1/3+7/15-2/15= h(A) + h(B)-h(A n B)
13. a) (13!/(2!)' ] - 3(12!/(2!)'] + 3(11!/2!) - 10! b) Divida el r<sultado de la parte (a)enu.
[13!/(2!)'].
15. 2/39
Ejercicios l. Supongamos que (A - B) ~ C y x E A - C. Emonces. x E A pero x f. C. Si x f. B. entonces
complementarios- [x E A A x f. BJ ~ x E (A - B) ~ C. Ahora tenemos que .r f. C y x E C. Esta contradicción
pág.179 indica que x E B, por lo que (A - C) ~ B.
Recíprocamente. si (A - C) ~ B, sea y E A - B. Entonces. y E A pero y f. B. Si y f. C.
entooees [y E A /1. y "' C] => y E (A- C) t: B . Esta contradicción (es dedr, y "' S y y E B)
indica que y E C. de modo que (A - B) !'; C.
3. a) Loseonjuntos'll= ( I, 2. 3],A = ( 1, 2}.B = (1} y C= [2] proporcionan uneontraejemplo.
~A=An•=An~ u~ = ~nqu~n~-~nqu~-0
=~nqu~-0=~nqu~n~=Bn~u~=Bn• =B
S. a) 126 (si 1os equipos usan diferentes uniformes); 63 (si los equipos son indistinguib1C$)
IJ2 (si los e.quipos usan diferentes uniformes); 56 (si los equipos son indistinguibles)
b) 2" -2; (1/2)(2" -2). 2" - 2 -2n; (1/2)(2" - 2 - 2n).
Soluciones S-17

7. •) 128 b}IA 1 = 8.
9. Supongamosque(A n H) u C=A n (BU C) y quex E C. EntoncesxE C => x E (A n B) U
C=> x E A n (BU C) s;A, de modo quex E A y C _; A.
Recíprocamente, supongamos que C C A.
(1) s; y E (A n H) u c. entOD<CS y E A n B o y E c.
(0 yEA<18=>y E (A<1H)U (A <1C)=> y EA<1(B U C).
( ü) y E C:> y EA,puesCs;A.Además,y E C=> y EB U C,demodoquey EA (1
(BUC).
En el caso ¡;¡ o el caso (ü ), tenemos que yE A n (B u C). de modo que (A n B) u e .;
A<1(B U C).
(2) Ahora, sea z E A n (B U C).
Entonces z EA n (Bu C) = (A n HJ u (A n C).; (A n B) u c. puesto que A ne.; c.
De las partes (! ) y (2) se ,;gue que (A n B) U C = A n (BU C).
11. a) Supongamos que A U B = <,! y sea z E A. Entonces x E A ~ x f A ~ x E B. puesto que
A U B ='ll. En consecuencia.A.; B.
Recíprocamente, sea y E .SU.. Si y E A, hemos terminado. Si no, entonces y , A, por lo
que y e.A C B. Encualquiercasotenemosy EA U B. porloque<;J. C A U B. Pero siempre
tenemos A U B CüU., por lo que se sigue que A U B=~-
13. •) [O, 14/3] b) (O, 9/ 5] e) {O} d) {O} U (6, U]
•> (O, +~) 1) (O, +®) g) {O} b) ~

15. •> A B A<1 B Como A !:: B, sólo consider.unos las filas 1, 2 y 4. Para estas
filas.AnB=A.
o o o
o 1 o
1 o o
1 1 1

e) A B e (A<1 B J U(B nCJ A n C Para C C B CA, solamente considera-


mos las filas 1, 5, 7 y 8. En este caso.
o o o o o (A n B) u (B n C)=A n C.
o o 1 o o
o 1 o 1 o
o 1 1 o o
1 o o 1 1
1 o 1 1 o
1 1 o 1 1
1 1 1 o o

d) A B e A f;.8 A t;. C B t;.C Si A ó. B = C. consideramos las filas


l , 4, 6 y 7. En estos casos.
o o o o o o Af;.C = ByB t;. C=A.
o o 1 o 1 1
o 1 o 1 o 1
o 1 1 1 o
1 o o 1 o
1 o 1 1 1
1 1 o o 1
1 1 1 o o
S-18

17. 1) (-::,') (msr+ l ) b) (•-:••) (21:s n +l)


1,. 1) 23 b) 8
21. 1) (lnt(::) b) (~)/(l:J
23. 7"-3(3") + 3
25. ('¡)(\')10') =0.3483
27 ) ""' (>NM) .... (i+S)!
• ■ ~~ l l \&-l = -',,t-oJ1J1(8-'1!
b) (i) (';)(~'>1w...~1·n1 (ü> ~><'i')1w...mc1:i>J
(iii) [(':) + (';)(~') + (!)(~ + (l)r:) + (gJG)]IW-,.~¡:l)]

Capitulo 4
Propiedades de los ent..-os:
lnducd6n matemática

Sección 4.1- J. b) COOlO l · 3 = C1X2X9)1(6),d r=hadoes venladeroparan = l.Supongamoo quedrcsulll-


pág. 199 does vc:roadero para•= l(a< 1): 1 · 3 + 2 - 4 + 3 · S + · · · +l(k+ 2) = l(k + 1X2k+ 7)16.
&tooces. consideremos el caso en quen = .t-+ 1: [1•3~ 2•4 + • • • + k(i: + 2)) + (.t + I)
(k+ 3) = [l(k+ 1X2k + 7)16] +(k + lXk+ 3)= [(k + 1)16J[l(2k+ 7) +6(k + 3)] =(k+ 1)
(2.t'+ 13k+ 18)16 = (k+ lXk+ 2X2k+ 9)16. Podo tanto, el =u!tado se sigue para todo.
n E z••por el principio de inducción finita.

e) S(n): ±- +1)
- =-"-
n+l
, .. , .i(i
1

1
1 1 1
S(l): ~ i(i + l) = l(Z) = 1+1' por lo que S(l) es veroadera.

Supongamos S(k): ±- 1
- = - k _ _ ConsideromooS(k+ 1).
,_, i(i + 1) k + 1

' f _ l _= ±-1- + 1 k +- - - -
•- d(i + l) ,-,i(i +l) (k+l)(k +2) (k+l) (k+l)(k+2)
= (k(k + 2) + 1)/[(k + l)(k + 2)) - (k + 1)/ (k + 2),

de modo que S(k) => S(k + 1) y d resultado se sigue para todo n E Z-, por d principio de
inducción finita.
3. a) 7626 b) 627,874
S. Ikmostroci.6n.: Denotamos los números sobre la circ:unfereocia comoxi. X:z. x,. . . . •~x,oo,x1.
Si la conclusión no es verdadera. entonces

X1+x2+x3 + x, < 202


X2+ X3+X,+Xj< 202
X3+x,+xs+zt <202

Xi7+.r'il8+x,,+x100 < 202


.r911+x,,+x1oo+X1 < 202
x,,+x1oo+xi +x2 < 202
X100+.t1+XJ+ X3 <202
Soluoone S-19

Sumando los resultados en ambos lados de csus 100 de$i¡uald3des obtenemos que

4 ! z,< 100(202)
o

20,200>4 !x, •4 ! i =4[100(l01)/2) • (200)(10l) • 20,200


¡.1 ,-1

~ cootradicción.
7. a) 506 b) 12,144
9. Paran= 11, 11-2=9<9¼• ( ll'-11)112. Supoagamooquecl resullaOoes verdadero paran=
i( a: 11): k-2 <(k'- k)/12. Ow>don= k+ l,k-l<(k'-k)/ll=>(k-2)+ l<((k'-kYll)+ 1
=> (k+ 1)-2 <(k'-k + 12)/12. Parak>6, l b 12 y b 12-k, de modo que (k + 1)-2 <
(k'+ k)/12= [(k+ l)'-(k+ 1))/12. Enconsecuencia,el resultado se si¡ue pau todon a: 11 por
el principio de inducción finita.
ll. Paran• s. 2'a 32 > 2S = S2• Supongamos cieno el resultado par2n • .t( ~ S): ~> k!. Paralc>
3,1:(k-2)> 1 ok'> 2k+ l. 2'>k'=> 2' + 2'>k'+k'=>1.'''>k'+k'>k'+(2k + 1) =(I:+ 1)'.
Por lo tanto. el resultado se sigue para todo n ~ s. por el princip;o de inducción materMtica.
13. b) Si c:omem.amos con n • l tenemos que

,.,±jH
1• H, • l • ((2)(1)/2)(3/2)-((2)(1)/4] • 1(2Xl)/2)H, - ((2)(1)/4).
Supooemos que la proposición (abierta) dada es wodadcra para• = k y tenemos que

i,¡H
,., 1• ((k + l)(k)/2)H,., - ((k + l )(k)/4].

Paran = k + 1 tenemos que

'-i;¡H,• ÍiH;+(k + l)H,.,


f-1 J-1
• ((k + 1Xk)/2JH,., - ((k + lXk)/4) + (k + l)H,.,
• (k + 1)(1 + (k/2)]H,., - ((k + l)(k)/4]
• (k + 1)(1 + (k/2)1(H,., - (l}(k + 2)))- ((k + l)(k)/4)
• ((k + 2Xk + 1)/Z)H,., - [(k + l)(k + 2)V(2(k + 2)] - ((k + l XkJ/4)
• [(k + 2)(k + 1)/Z)H,., - ((l/4)(2(k + 1) + k(k + 1)))
• [(k + 2)(k + l)/2)H,.,-((k + 2)(k + 1)/4).
f.o coosccuencia. por el principio de inducción ma1emática. se sigue que 1a proposición
(abierta) dada es cierta para todo n E z·.

15. SupoopmosqueS(k)es vcrdadt:ra. ParaS(k+ 1),tcncmosque 4,'i :(Ik+(l/2))'/2) + (k + 1)=


(k'+k+(U4)+2k+ 2)/2-[(I: + l)'+(k+ l)+(U4JY2= ((l+ l) + (U2))'12. DemodoqucS(k)=>
S(_lc + 1). Sin embargo, oo tenemos un primer valor de /c. tal que S(k) sea verdadera: para todo k .t
1, 4,.,i
= (kXk+ 1)12 y (kXk + 1)/2• [k + (112)]'/2 => O• 1/4.
17. C...j<ftmL• Sean EN. Entooces

(2n + l)' + (4(0 + 1 +2 +3 + · · · +n))' = (4(0+ 1 + 2 + 3+ · · · +n) + l]'


Donostracitín.: P-ar'l.c:ualquic:r mímeronarural n, (O+ 1 + 2 +-3 + • • • +n) =n<.n + IYl. por lo que
(2n+l)'+(4(0+1+2+3 + ··· +n))'
= (4n' + 4n + 1) + 16{n(n + 1)/2)'
• (4n 1 + 4n + 1) + 4n:::(n + 1)1
• (4nz T 4n + J) + 4n' ♦ 8,zl + 4n:t
-4n'+8n.,+ 8nz+ 4n + l.
S-20 Soluciones

Además, [4(0 + 1 + 2+ 3 + · · · n) + 1)2 = [[(4nXn + 1)(2)+ lf= [2n(n + 1)+ IJ'= [2n'+ 2"+
1)1 =4n•+ 8n1+ 8n1+ 4n + l.
19. a) Para n = 1, [sen(2"8)]/(2' sen 8) = (sen 28)/(2 sen 8) = (2 sen 8 cos 8)/(2 sen 8) •
cos e. de modo que el resultado es cierto en este caso.
Supongamos que el resultado es verdadero paran= k( ~ 1):

(cos q)(cos 28) · · · (oos(2"' )8) = [sen(2'6))1[2'sen 8). CUando n = k+ 1, cnlonccs


(cos 8Xcos 28) · · · (cos(2"')8Xcos(2'6)) = [scn(2'8)cos(2'8)]1[2'sen 8)
= [2 sen(2'8)cos(2'8))1[2'·'sen 8)
= [sen 2(2'6))1[2"'sen 8)
= [sen(2"' 8))/(2''' sen 8).
En consecuencia. el resultado es verdadero para todo n 2: l , por el principio de induccidl
matemática
b) Sin = 1, la (Koposición es cos 8 = (sen 28)/(2 sen 8), que es verdadera pues (sen 28V(2 sea
8) = (2 sen 8 cos 8)/(2 sen 8) = cos 8.
Paran = k<. 2: 1), suponemos que la proposición siguiente es vcrdadcra::
cos 8 • cos3q + cosS8 • · · · + cos(2k- 1)8 = (sen 2k8)/(2 sen 8).
Cuando n = k + l, tenemos (cos 8 + cos38 + · · · "fr cos{2k- J)8) + cos{2k + 1)8 =
[(sen 2k8)/(2 sen8)) + cos(2k + 1)8= [sen 2k8 + 2sen 8 cos(2k+ 1)8)/( 2 sen 8).
Para establecer este caso, necesítamos mostrar que
sen(2k + 2)8 = [sen 2k8 • 2sen 8 cos(2k + 1)8). Tenemos que

sen(2k + 2)8 = sen 2l9 cos 28 + cos 2k8 sen 28


= sen2k8[1 - 2sen2 8) + 2 sen 8 cos 8 cos 2k8
= sen2k8- 2 sen 8[sen 8 sen 2k8- oos 8 cos 2k8]
= scn2k8 • 2 sen 8cos(2k + 1)8.
El resultado se sigue ahora paran 2: l, por el principio de inducción finita.
21. Sea S(n) la siguiente proposición (abiena): Pata .x, n E z•. si el programa llega a la parte
superiOI' del ciclo Whilc. después de que las dos instrucciones del ciclo se ejecuten n(>O)
veces, entonces el valor de la variable entera Answer es .r(n!).
Consideremos primero S(1 ), la proposición para el caso en que n = 1. En este caso, d
programa (si alcanza la parte superior del ciclo Wbile) produce una ejecución del ciclo Wbile:
i ~ el valor z · 1 = .r(l !) y el vaJor den decrece a O. Con el valor den igual a cero, el ciclo
no vuelve a procesarse y el va!orde la variable Answeresx( I !). Por Jo tanto.S(l)es verdadera.
Supongamos ahora que la proposición es verdadera paran= k (~ 1). Para .r, k E z·, si el
programa llega a la parte superior del ciclo While. entonces, despu6s de Al.ir del ciclo. el valor
de la variableAnsweresx(k!). Para verqueS(k+ l ) es verdadera. si el programa llega a la parte
superior del ciclo While, entonces OCWTt lo siguiente durante la primera ejecución:
El valor asignado a la variablezesx(k + 1).
El valor den decrece a (1 + 1)- 1 = k.
Pero entonces, podemos aplicar la hipótesis de inducción a los enteros x(k + 1) y k. para afu.
mar que al salir del ciclo While correspondiente a estos valores. el valor de la variableAnswer
cs(.t(k + l )Xk!) =x(k+ IJ!
En consecuencia, S(n) es verdadera para todo n ~ t , con lo que hemos verificado que ei
segmento de programa es correcto. mediante el principio de inducción matemática.
23. ParaP(n) = ( 1000!./s)[((I • $)(2)"''--((I - $)(2)"''), tenemos (después de sustituir 1 y 2 eo
vez den) queP(l) = IOOOy P(2) = 2000, de modo qucP(n) es válida paran= 1, 2. Suponga-
mos la validez de P(IJ, P(2), ... , P(k- 2), P(k- 1), para k ;,, 3. Entonces
Soluciones S-21

P(k) • P(k - 1) + P(k -2) • (IIXXl/VS)[((l + VS)/2)' - ((! - VS)/2)'


+ ((1 + VS)/2)H -((1 - VS)/2)'- 'I
• (!IXXl/VS)[((l + VS)/2)'-'-[l + ((1 + VS)/2))
- ((l - VS)/2)'-'·[l + ((1 - VS)/2)1]
• (IIXXl/VS)[((I + VS)/2)'-'·((l + VS)/2)'
- ((l - VSJ/2Y-'·((! - VS)/2)']
• (llXXl/VS)[((l + VS)/2)"' - ((1- Y5)/2Y-'J,
de modo quel'(n) es dlida para todo n >, 1, po.-inducciól1 maleDWica (formulternativa).
15. Sea T• {n E Z•I• a, -.y S(n) es falsa). Como S(-.). S(-.+ 1), S(-.+ 2), ... , S(n,) soa
vcrdaderas,sabcmos qucno,,no+ l,no+ 2, ... ,n1 ,t. T. Si Tf&c tli, entonccs Ttieoc un elemento
mínimo r, pues T!; z·.
Sin embargo, como S(no), S(,,.+ 1)•. .. , S(r-1) son veroaderas, se
sigue que S(r) es verdadera. Poc lo tanro. T s t y se sigue el resultado.

Sección 4.2- l. a) c,=1;yc..1 =c.+1.paran.!: l.


pjg. 210 b) c 1 •7;yc..1=7c.,paran2:: l.
e) c1 ::z IO;yc_1 =c.+3,paran 2:: J.
d) c1=3;yc-1=c.+ U.paran 2:: l.
e) c1 ::o:7;yc..1 =c.,parani!:: l.
f) C1=l;yc..1=c.+2n:+l,paranó?: l.
3. a) Sea T(n) la siguieotc proposición: Paran E z•, n 2:: 2. y W proposicioDes p, q1, 'lb . . , q_

P V(q, /\q,/\-. -1\ q.) ..(p V q,) A(pv q,)/\·. -A(pvq.}

La proposición T(2)es verdadera en virtud de la ley distributiva de V sobre A . Si supone-


mosque T(k) es verdadera para todok 2:: 2. analiZ2mos ahora la situación de las proposicio-
nes p, q.. <lb •.• , qb qM. Tenemos que p V (q1 A q2 I\ • · · /\ q, I\ q...1)
..pV[(q,/\q,/\ .. -1\q,)/\q,.,J
.. (pV(q,/\q,/\· · ·/\q,)JA(pVq,. ,)
.. [(pvq,) A (p v q,) /\-. -A(p v q,)J A (pVq,. ,)
.. (pv~ A(pv~ A ---A(p v~A(pv~~
Se sigue entonces del principio de induccióo matcm4tiea que la proposición T(n) es verdadera
para todo n 2:: 2.
s. a) (i) Laintenec:c:ióndeA1,A2CSA1 nA::-
(Ü) La intersección deAa.A:. ... ,A•. A.. 1esttdada porA. 1nA2 n ••• nA. nA.,..1= (A 1n
A2 n • • • n A.J nA.,.¡, la intc:rsecci6nde los dos conjuntos A, nA2 n • • • n A. yA.,.1•
b) SeaS(n) la proposición (abierol) dada. Eruoncos. la Vffllad deS(3) se sigue de la ley asoáaliva
den. Suponemos queS(.t)es verdadera parak ó?: 3 y consideramos el caso dek + 1conjwu:os.
(1) Sir= k.entonces

(A , nA2n · · · nAj.)nA.. , = A, nAzn • • • n A.nA..i♦ i,


de la definición recursiva de la parte (a).
(2) Para 1 S r <k. teoemOS

(A, nA,n ---nA,)n(A,., n ... nA,nA,-,)


• (A,nA,n -. -nA,) n((A~, n --. nA,) nA,-,J
- ((A, nA,n -••nA,)n (A~, n ---nA,)J n A,.,
= (A, nA2n•••nA,nA,...1n• -•n A,..)nA ..... ,
= A, nA2n• •· n A,nA,....1n •·· nA..,nA....1,
y por el principio de inducción matemátic:a, S(n) es verdadera para todo n ~ 3 y todo 1 ::$: r < n.
S-22 Soluciones

7. b) SeaS(n) la proposición(abiena)dada. Sabemos queS(2)cs verdadera por la ley distributiva


de U sobre n . Suponemos ahora que S(k) es verdadera para k 2: 2 yconsideramos S(J: + 1)
para losconjuntosA,B1,B2•••• , BhB,.1- Tenemos que
A urs,ns,n- .. ns,n s,.,J
=A U[(S,nB,n- .. ns,)ns,.,)
=[A U(S,ns,n .. -ns,)) n(A us,.,)
=[(A u s,)n(A UB,)n- --n(A US,)J n(A us,.,)
=(A US,)n(A u s,)n .. -n(A US,) n(A u s,.,).

y entonces S(n) es verdadera para todo n 2: 2. por el principio de inducción matemática.


9. a) (i) Paran = 2. la expresión X¡.li denota el producto usual de los nt1mcros reales .x:,, y-¾·
(ü) Sean E z•, con n 2: 2. Para los números reales Xi, x2, •••• x,,, x,..1 definimos

el producto de los dos números reales X1, Xt, ••• , x,, y .r_.1 •
b) El resultado vale para n-= 3 por la ley asociativa de la multiplicación (para los nwnero:S
reales). De modoquex1(x2xJ= (.r1. r ~ y no hay ambigüedad al escribir..r.xi%1- Suponemos
que el resultado es verdadero para algún k 2: 3 y todo 1 s r < k y analizamos el caso para
k + 1( 2:: 4) números reales. Tenemos que (l) sir= k, entonces Cx,x.i · · · Xt>xt+i= .r1z 2 • • •
z..rt.i. por la defuúción recursiva dada en la parte {a); y (2) si 1 :S r <le.entonces (xth • • •x,)
ex.... · ' ·XtXt+1)= <x1.r2' · ·.r,X(.r,.., ·' ·AA+,) = ((X1X2· ' ·.r,)(.:C,.¡ • • ·xJ)x-..1=(.r1.r2• ' ·X,.l'.,.¡
· · ·xJ.r..,=x1 x2 • • ·Z..Xn-1 • • •XtX1♦bdemodoqueelresultadoes verdadero paratodon 2: 3
y todo 1 :S r < n, por el principio de inducción matemática.
11. a) Sea ai. ªt. a,- .. . esta sucesión de enteros. Entonces podemos definir esta suecsión de
forma recursiva como:
(1) a, = 2; y (2) •-•=(a.)', paran >: l.
13. DmwstracWn (por la forma alternativa del principio de inducción matenWica): Paran= O, l,
2, tenemos

(n =O) ao., = a,= l>c(V2)º;


(n=l) a,., =a,=a,+a.=2a,\/Í=(V2)'; Y
(n = 2) a2...2 =a.• a, + 4 1 =2 + 1 = 3 ~2 = (VÍ)2.
Por lo tanto, el resultado es verdadero para estos tres primeros casos y tenemos la base de la
demostración.
A continuación, paraalgán k ~ 2. suponemos que el resultado es verdadero para todon =O,
1,2, .. . ,k.Sin = .t+ l,te~que

°<'••>••=a.., =a,., +a, a,(\/Í)' + (V2)'-'= [(V2)' + 1)(\/Í)M = 3(\/Í)'->


= (3/2)(2)(\/Í)'-' = (3/2)(\/Í)' >e (V2)'-', pues (3/2) = 1.5 > V2( 1.414). =
Esto proporciona el paso inductivo de la demostración.
De los pasos base e inductivo, se sigue , por la forma alternativa del principio de inducción
matemática, que a..,2 2: (/2, )" para todo n E N.
15. DmwstracWn (por inducción matemática): Para n = l tenemos
1
2!,
F¡F;-1 = F1F0 + FiF1 =
= =
(1)(0) + (1)(1) 1 12= Fj, por lo que el resultado es válido en este primer caso.
Supongamos ahora que el resultado es verdadero para algiln k 2: J. Esto nos da
1
F¡F¡_1 = ¿:
F,t. Paran = k + 1, tenemos entonces que
2(.UI) 2k

L
i •I
F;F,-1 = L F,F,-1 + Fu.1Fu + Fu.Ju• t
i•I

• Fi,. + F21r.1Fu + FlA•zFu.•1 = Fa.(F21r +Fu.a)+ F'lk+2FVt•1


= Fa.F21r.1 + Fu+-:F21r... 1 • Fu.-Á_F21r + Fu+1) = Fi.2.
Soluciones S-23

Como el resultado es cierto paran= 1, y la verdad en n = .t implica la verdad en n = k + 1. se


sigue del principio de inducción matemática que Z:
F¡Ff-l = Fi,. para todo n E z•.
1
17. Demostración (por inducción matemática):
POS(} base: Cuando n = l. tenemos que

± F,., •F,.12 ~0~ 1-(2/2)= 1 -!:!_= 1- F,:',


.•,2' 2 2
por lo que el resuJtado es cierto en este primer caso.
Paso inductivo: Suponemos que la proposición (abierta) dada es verdadera para n = k y tene,.
mosque 4 1
~
1
=1 - ~ . Cuando n = .t + 1 tenemos que

~ I F,-1 ~ F1-1 F1c 1 F1c-+2 F1c


t t -~ t +r+1 "" - +r· 1 r
= 1 + (l/2'"')[F, - 2F,.,J = 1 + (1/2'.,)((F, - F,.,) - F,.,]
= 1 + (112"')[-F,., - F,.,] = 1 - (l~')(F,., + F,.,) = 1 - (F,.,/2',.').
De los pasos base e inductivo se sigue, por el priocipio de inducción matemática. que

Yn E Z- ! (F,.,/2') ~ 1 - (F•• ,/1:').

19. Demostración (por la forma alternativa del principio de inducción matemática): El resultado
esválidoparan=Oyn= l,pucs

(n= O) SF..,=SF,=S(l)=S=7 - 2•L. - Lo•Lo.,-Lo; y


(n = 1) SF,.:z=SF,=S(2) =10• 11-1 = 4-L1 = L 1• • -L1 •
Esto establece el paso base de la demostración.
A continuación. $Uponemos la hipótesis de inducción; es decir, para algdn k( i?: 1), SF..2 =
4...-4. para todon = O. 1, 2, ... ,k-1. k. Se sigue entonces que paran =k + 1,
SFt1c♦1>•:z = SF.,.., =S(F,.....2 + F.1:.1) = 5(F1r•2 + F<1r-1>..2) = SF,.i + SFc1r- l)....2
= (L.,.._ - L,) + (l.c;.t-1)+4 - L,1;-1) = (L1c... - L1c) + (L1c..., - L.,,- 1)
=(L, ..... + L,.,)-(L.t + L1c,-1) = L1c+s - l1c♦1 =~k..1)-• - L.1:-1,

donde usamos las defmiCM?nes rccursivas de los números de Fibonacci y los números de Lucas,
para establecer la segunda y octava desigualdades.
Se sigue entonces de la forma altecr.ativa del principio de inducción matemática que
"In EN SF,....2 = L...... -L,..

11. a) Pasos lluooes


! ) p,q,r, To Parte (1) de la definición
2) (pVq) Paso (1) y parte (2-ii) de la definición
3) (,r) Paso (1) y parte (2-i) de la definición
4) (To/\(,r)) Pasos (1) y (3) y parte (2-iii) de la definición
5) ((pvq)-(To/\(,r))) Pasos (2) y (4) y parte (2-iv) de la definición
<) Pasoo lluooes
! ) p,q,r,s Partc(l)deladefinición
2) (p -r) Paso (l)y pane(2-iv) de la definición
3) (pVq) Paso (1) y pane (2-ii) de la definición
4) ((pvq)-s) Pasos (3) y (1) y parte (2-iv) de la definición
5) ((p-r)/\((pVq)-s))Pasos (2) y (4) y parte (2-iii) de la definición
S-24

Secci6n4.3- l. e) Si a lxy a ly, entonces x = acy y= ad.,con c,d E Z. Así, z: = x-y=a<.c-d)y ali.
pág. 223 demostración de los otros casos es similar.
g) Se sigue de la parte (O po,- inducción matemática.
3. Como q es primo, sus únicos divisores positivos soa 1 y q. Como p es primo, se sigue qae
J. Por lo tanto, pJq => p = q.
S. DenwstracWn (porcontrapositiva): Supongamos que albo a je. Si a Jb, entoocesai=b
alg,lnk E Z. Pcroak=b => (ak)c=a(kc) =bc=>albc. Obtenemos un resultado similar sial~
7. a) Scana;l , b = S,c=2.0troejcmploesa=b=5,c=3.
b) Demostración:
(i) 31 l(Sa + 1b+ lle) =>31 l (lOa+ 14b+22c).Además, 3l l(31a+31b+31c),de-
que 311((31a + 31b + 31e) -(10a + 14b + 22c)]. Por lo tanto. 31 l(21a + 17b+!lcl.
(ii) 31 l(Sa+ 1b+ lle) y 31J(31a+31b+31c),podoque 311[2(31a +31b+31c)-5(5-♦
1b + lle)], o 31 l(37a + 27b + 1c). Como 31 J31a.se sigue que 31 l[(37a + 27b+ 7<1-
3la]; es decir, 31 l(6a + 21b + 1c).
9. [bla y b l(a + 2)] => b l[ar+(a + 2)y] para todosx, y E Z. Scanx = -1,y = J. Eruoocesb>t
ybl:Z.po,- loqueb= 1 o2.
ll. Sean a = 2m + 1 yb = 2n+ l, conm,n e N. Entonccsa1+1r= 4(m2 +m+nl+n)+2,de
que 2l(a'+ b') pero 4,((a' + b').
13. Para n = Otenemos r - 4• = 7°- 4° = l - t =Oy 3 IO. Asf, el resultado es verdadero para•
primer caso. Suponemos que es verdadero paran= k( 2:: O) y tenemos que 3 IC7t_ 4i. Pasaa:
al caso en que n = k + 1 y vemos que 71♦1_4M= 7(7~ -4{4'} = (3 +4)(-¡t}-4(4~ =3(7')+
4(7'- 4~. Como 313 y 31(7'- 4~ (po,-la hipótesis de induccióo), se sigue de la parte(Odd
teorema 43 que 31[3(7~ + 4(7'-4~]; es decir, 3 1(7"'-4"'). El principio de induccióo-.
mática implica ahora que 3 j(7• - 4'") para todo n E N.
15. llas<lO llas<2 llas<l6
a) 22 10110 16
b) 527 IOOOOO!lll 20F
e) 1234 10011010010 4D2
d) 6923 l!0llOOOOI0ll IBOB
17. llas<2 llas< lO llas<l6
a) l!OOlllO 206 CE
b) 00110001 49 31
e) llllOOOO 240 FO
d) 01010111 g¡ 57
19. Máximo entero Múúmo entero
a) 7 = 2'-'-1 - 8• - (2')
b) 127 - 2'-I -128= -(2')
15
e) 2 - 1 -(2")
d) 231_1 - (2'')
e) r-
1
-1 - (2"- ')
21. ax=ay =:>a.x-ay = O=:>a{_x- y)= O. Ene) sistema.de los enteros. sib,c E Z y be = O,entooces
b= Oo c = O. Como a(.T- y) =0 y a ,J; O. se sigue que (x-y) = Oy x = y.
27. a) Como2 l l0' paratodorEZ\2lnsiys6losi2 jr~
b) Se sigue del hecho de que 4 110' para I a, 2.
e) Se sigue del hecho deque 8Jl(j para t ~ 3. Engcoeral,
1
2" jn sí y sólo si 2'""1!(,, · 10'+ · · · +r1 -10+ ro).

>ección 4.4- l. a) mcd(1820, 231) = 7 = 1820(8) + 231(-63)


pág. 231 b) mcd(2597, 1369) = 1 = 2597(534) + 1369(- I013)
e) mcd(400l , 2689) = 1 = 4001(-1117) + 2689(1662)
d) mcd(7983, 7982) = 1 = 7983(1) + 7982{- I).
Sr25

3. mcd(a, b)• d=> d •ax+ by, cooz,y E Z.


mcd(a, b) • d => ald. bid E Z
1 • (a/d'}, + (bld)y => mcrJ(.ald, bid) = l.
5, Se.in mcd(a, b) = h y mcrJ(.b, d) = g.
mcd(a, b) •h => [hja y hjbJ =>hi(a · 1 +be) => h jd.
(h jb y h jdJ => hjg.
mcd(b, d) = g => (gjb ygjdJ => gl(d · 1 + b(-<)) => 1la.
(glb, ,la y h= mcd(a, b)J => 1lh. hlg,glh. con g. he z· => g =h.
7, a) Si e E z •, aitooces e= mcd(a, b) si (y sólo si)
(1) cja y clb; y
(2) Vd E Z((dla) A (djb)J => dic.
b) Sic E z•, en10DC<$ el mcd(a, b) si (y sólo si)
(1) c,taoc,tb;o
(2) 3d E Z((dla) A (dlb) A (d,t'c)J.
9. mcd(a, b) • 1 =>ax+by• 1 para_. y E Z. Eoloo=ac.r+bcy = c. ajacz,albcy(puesalbc) =>
aje.
l l. Se.ind,= mcd(a, b) y d,= mcd(a-b, b).
d, =mcd(a-b, b) =>ld,j(a-b) A d,lbJ =>ld,l[(a-b) +bJ] porlapartc(f)del -cma4.3 =>
dila y [d,ia A d,lbJ => d,ld,.
d,• mcd(a, b) =>(d,la A d,lbJ =>d,l(a+(-l)bJ,porlapartc (f)del teorema4.3. Por lo tanto,
d,l(a - b).
Comod,l(a - b) y d, lb, se sigucqued,ld, En coosecucncia, tcocmo, qoc [d,ld, A d,ld, A
d,. d,>OJ => mcd(a, b) • d,= d,• mcd(a-b, bi
13. Vemos que para todo n E z·. (Sn + 3)(7) + (7n + 4X-5) = (3Sn + 21) - (3Sn + 20) = l. En
c:onsecucoci&. se sigue que mcd(Sn + 3, 1n + 4) = l, o que Sn + 3 y 1n + 4 soo primos rc]ativos.
15. Nobaysalucióoparac,ó 12, 18. l'anc = 12,lassalucioncssooz• 118 - 165.t.y = -10+ 14k.
k E Z. Para e= 18, las soluciones sonz= tn - l6Sk.y•-lS + 14t. k E Z.
17. Seana, b, e e z ·. Si ax+ by= e tiene unasolución.ro,y0 e Z. entoocesaxo+ by0 = e, y como
mod(a. b) divide a a y a b, se sigue que mcd(a, b)Jc. Recíprocamente. supongamos que
mcd(a, b)lc. EDlooces e= mcd(a. b'¡d para algún d E Z. Como mcd(a, b) • as+ bt para
al&Uooss, re Z,tencmosquca(sd) +b(tá) =mcd.(a. b')d=coa.io+ by.o= cy ax+by• e tiene
una solución en Z.
19. b • 40,425
21. mod(n,n + 1) = I; mcm{n, n + 1) • n(n + 1)

Sección 4.S- l . a) 2'·3'·5'·11 b) 2'·3·5'·7'·11' e) 3'·5'·1'·11-13


pig. 236 3. a) m 2 = p;"1p~p~···p~
3
b) m •p~ p~ p~-- -p~'
S. n • 19

7. (La demostración es similar a la del ejemplo 4.38.) Ea caso contrario, tenemos que /i •alb,
doodea,b E Z'y mcd(a.b)= l. Entoocu,ÍP=alb =>p=a"b'=>pb'=a'=>pla'=> pja(por
d lema 4.2). Comopja, sabemos que a= p.t para algdn k E z·
y pb'=a'• (pi()'= p'k'. o b'=
pk'. Por lo tanto, plb' y cnton=plb. Pero si PI• y pjb, enton= mcd(a,b);, p> 1, lo que
contradice nuestra afirmación anterior de que mcd(a, b) = l.
9. b) Sea log~p• a/b, donde a, b E z•, mcd(a. b) = l. EnlODCcSp = l rl""o¡/'= 10'= 2' · S',lo
que contradice el teorema fundamental de la aritmttica.
11. a) 96 b) 270 e) 144
13. 660 I S. 176,400 17. n = 2-3·5'·7' = 7350
19. a) S b) 7 e) 32
d) 7+7+5+25+20+20• 84 e) 84
S-26 Soluciones

2 1. 1061(= 512 + 256 + 293)

23. a) !] 2' = (2')(2')(2')(2")(2') = 2•·••>+~, = 2" = 32,768


[V= <2')(3')(4')(5') = (5!)' = 120'= 14,400
b)

~·ñn
,.,
a'= al • a2 ,a, •• ·ti' =a1-+2+J+ ••· •• = a-<-•1)(2
Q(i1) =Qll. Q']) . cr2 .. . ~ =Qtl• 22+ Jl-+ · · ·-+--2 = a""'•l)(z..+l),'6

2S. lhmcsrrocién (por inducción matemática): Paran= 2, tenemos quc[!(1-7")=(1-i)


= (1- ¼) = 3/4 = (2 + 1)1(2 · 2),de modo que el toSultadoes cierto para este primer caso, lo
que estable.ce el paso base de nuestra demostración inductiva A continuación., supongamos

cierto el resultado para alg,ln k E z•, donde k a, 2. Esto nos dafi~,(1 -f) = (i + 1)1(21).
Cuando consideramos el caso paran = J: + l. obtenemos el paso :inductivo, ya que tenemos

1
'fi (1 -l)= rn(1 - l))(1 - -- ,) ·
1-2 l \J-2 l (k + 1)
1 ] [k+l][(k+l)' -1]
• [(k+ 1)/(2k))[ 1 - (k+ !)' • (k+1)'2k
k'+2k
= (2k)(k + l) = (k + 2)/(2(k + 1)) = ((k + !) + 1)/(2(k + 1)~
El resultado se sigue entonces para todos losenterosp01itivosn 2! 2porc1 principiodeinducci6n
matemática.
27. a) Los divisores poSltivosde 28son 1,2. 4, 7.14y28, y 1 +2 +4 + 7 + 14 +28=56==2(28),
de modo que 28 es un entero perfecto. Los divisores positivos de 496 son l, 2. 4, 8, 16, 31,
62. 124,248 y 496, y 1 + 2 + 4 + 8 + 16 + 31 +62 + 124 + 248 +496 = 992 = 2(496), de
modo que 496 es un entero perfecto.
b) El teorema fundamental de la aribmtica implica que los diviso~ de 2'9'"-1(2--1), para i--1
primo, 500 1, 2. 2'. 2', ... , z--• y (2"-1), 2(2'"-1), 2'(2'"• 1). 2'(2'"-1), ... , y 2'""(2"-1).
Estos Wvisorcs suman [l + 2 + 2 2 + 21 + ... + z-i] +(2•-1)[1 + 2 +22 + 23 + ... + 2--1] =
(2'"- 1) +(2"- IX2"- I)= (2•-1)(1 + (2"-1)) =2"(2•- 1) = 2(2...,(2'"- l )J, por lo que
2-1 (2• - 1) es un entero perfecto.

Ejercicios l . a+(a+d) + (a +24) +· • • +(a+(n-l)d)=na+[(n- I},td'Y2 Paran= l,a=a+Oyclrcsultado


complementarios- es verdadero en este caso. Suponiendo que
pág.242
! [a+ (i - !)d] =lea + ((k - l)kdj/2,

tenemos que

I [a+ (i - l)d]-(/ca + [(k -1)/ca']/ 2) +(a+ kd)- (k + l)a + [k(k + l)dj/2,


de modo que el resultado se sigue por inducción para todo n E z·.
3. Conjetura:4'.,c-1)'+1¡2 =(-1r1 paratodonez·. k.,i
Demostración (por el principio de inducción matemitica): Si n = l, la conjetura da como
resultado .k. 1( -
1 2
1)'• ; = (- 1)1• 1(1)2 = l = (-1)1• 1(1) = (-t)i.i k.,i. que es una proposición
Soluciones S-27

verdadera. Esto establece la base de la demostración. Para confirmar el paso inductivo, supon-
dremos que la siguiente proposicióo es verdadera:

i (-IY,'i' = (-1)'•' i i
t-1 i•l
para algón k e!! l. Cuando n =k + 1, tenemos que

¼(-ly♦t¡'-= (! (-1):••,.,.) +(-1)<••11•1(k + 1)2

= c-1i•·• I; + c-1r'<k+1J'= c-1>•··ckxk + 1)12+ c- 1)..,Ck + 1i'


= Hr'Í(k + l )' - (k)(k + 1)/2)
= (- l r'(l/2)[2(k + l)' - k(k + 1))
= (-1r'(l/2)(2k' + 4k + 2 - k' - k]
= (-lr'(l/2)[k' + 3k + 2] = ( - l)..'(l/2)(k + l)(k + 2)
=(-1t· 2 ¼i,
de JDOOOque la verdad del resultadocnn=kimplica la verdad del mismoenn ::k+ l,con lo
que obcenemos el paso inductivo. Se sigue entonces del principio de inducción matemática que

±(-ly-'i' =
1•1
( - !)""' ±
i•l
i,
.
para todo n E z•.
2 2
5. a)• 11 +n+41 n • +,r+41
1 43 4 61
2 47 5 71
3 53 6 83
b) Paran=39,n1+n +41:: 1601,unprimo. Peroparan::40,nl+n +41 ::(41)2,dcmodoquc
$(39) :f; $(40).
7. Paran=!,};_/ = 1 = [(1)(2)(3XS)]l.l0,demodoquc la fónnula cs verdadera en este primer
caso. Supongamos ahora que cJ resultado es cieno paran :: k(i!:: I ); es decir,
~-/= [k(k+ IX2k + 1)(31<'+ 3k- l)]l.l0. Cuando•=
k + 1, tenemos
"¡J:,' ,• = [k(k + 1)(2k + !)(3k' + 3k - l)]/30 + (k + l)'
= ((k + l)/30l[k(2k + 1)(3k' + 3k - 1) + 30(k + l)')
= ((k + 1)/301(61:' + 39k' + 91k' + 89k + 30)
= ((k + l)/30l[(k + 2)(2k + 3)(3k' + 9k + 5))
= [(k + l)/ 3011(k + 2)(2(k + 1) + 1)(3(k + !)' + 3(k + 1)-1)),
de modo que la verdad del resultado en n :: k implica La del caso n = k + 1. En consccucocia. el
resultado se sigue ahora para todo n E z• por el principio de inducción matemitica.
9. a) Paran = O, 221oo1 + 1 = 2 + 1 = 3 , de modo que el resultado es cieno en este primer caso.
Suponiendoque3 divide a 221-o• + I paran = k(2:0) E N,cons.idercmos el caso den = k+ l.
Como 21tt♦1),ot + 1 = 22M + 1 = 4(2zt♦1) + I = 4(2111♦1 + t) -3, y 3 divide a 22t•1 + 1 y a 3, se
sigue que 3 divide a l1U•1,.. 1 + t . En ccOSCOJcncia, cJ resultado es verdadero para n = k + 1
siempre que lo sea paran= k. Por el principio de inducción finita. el resultado es cierto para
todonEN.
e) Sin =0. (n'n)+{n'l3)t{l ln/21) = O, un entero.Así, Ja proposición (abierta) es verdadera en
este primer caso. Suponiendo que la proposición es verdadera paran = k ~ o. tenemos que
(lln) + (.t3/3) + (I lk/21) E Z. Pasamos ahora al caso en quen =k + I.Aquítenemos que
(k+ l )'n + (k + l)'/3 + ll(k + 1)121) =[(k'l7) + (k'/3) + (llk/21))
+ [(7k'+ 21.t'+ 35.t'+ 35.t'+ 21.t'+ 7k)l7) + [(31<'+ 3k)/3) + [(117) + (113) + (1 lnI)), donde
S-28 Soluciones

el primer sumando es un entero por la hipótesis de inducción y el segundo y tercer sumandos


son enteros puesto que cada uno tiene un nwneradoc divisible entre el denominador dado.
Así, la verdad de la proposición paran = k + 1 radica en el becbodequecl último sumando
=
sea o no un entero (pero esto es cierto, pues (lfl) + (1/3) + ( llnl) = (3 + 7 + 11)121 1 E
Z. Por lo tanto, el resultado es verdadero para todo n E N, por el principio de inducción
matemática
ll. Paran = 2 tenemos que 21 = 4 < 6:::: (;) < 16 = 4 2, de modo que la proposición (abierta) es
verdadera en este primer caso. Suponiendo que el resultado es cierto paran= k 2'.. 2 (es decir,
'1!< (2/) < 4-t), consideremos ahora lo que ocurre paran = k + 1. Aquí tenemos que

2(k+ l)) (2k+2) [(2k +2)(2k+l)] (2k) (2k\


( k + 1 - \ k+ 1 - (k+ l)(k + 1) k • 2[(2k + l)/(k + !)]\ k}
>2[(2k + 1)/(k + 1)]2' >2"••,
ya que (2k+ 1)/(k+ 1) = [(k+ 1) +kV(k+ 1) > !.Además, [(k+ 1) +kV(k + 1)<2, porlotanlo
(2fti2) = 2l(2k + IY(k + OJ(2t) < (2X2)(~) < 4M_ Eo consecuencia. d resultado es cierto
para todo n ~ 2. por el principio de inducción matemática.
13. a) Primero observemos que el resultado es cierto para todo n E Z .. taJ que 64 s n s 68. Esto
se sigue de los cálculos

64 ~2(17) + 6(5) 65~13(5) 66 ~3(17) + 3(5)


67=1(17)+10(5) 68=4(17)

Supongamos ahora que el resultado es cierto para todon tal que 68 S n s k y consideremos
el entero k + 1. Entonces k + 1 = (k-4)+ 5 y como 64 s k - 4 < k, podemos escnOir k-4 =
o(l7)+b(S) paraalguoosa,b EN. Enconsecuencia.k+ 1 =a(l7)+(b + 1X5) yel resultado
se sigue para todo n ~ 64. por la forma alternativa del principio de inducción matemática.
b) La demostración de este caso es similar a la de la pane (a). Los síguíentes cálculos son
necesarios para mostrar que el resultado es verdadero para 108 s n s 117.

108 ~6(13) + 3(10) 109 = 3(13) + 7(10) 110 = 11(10)


111 • 7(13) + 2(10) 112 ~4(13) + 6(10) 113 -1(13) + 10(10)
114 = 8(13) + 1(10) 115 ~5(13) + 5(10) 116 = 2(13) + 9(10)
117 - 9(13)
15. a) r=ro+r1·10+rr102 + ··· + r,.•1(r
= To+ 71(9) + r1 + r2(99) + r2 + · · · + r,.(99 •.• 9) + r,.
n9',
= [9r1 + 99r: + · · · + (99 ... 9)r,.) +(ro+ r1 + , 2 + · · · + r.).

Por lo tanto, 9l rsi y sólo si 9l(r0 + r1 + TJ+ · · · + rJ.


e) 3Jtparax= 1 o4o7;9 frparax = 7.
17, a) (',') b) (!) e) m para la parte (a); {;) para la pane (b).
19. a) l, 4, 9 b) 1, 4, 9. 16, ...• k, donde kes el máximo cuadrado menor o igual que n.
21. a) 1000 b) 43S e) 12
23. a) Paracualquiern E Z\n ~ 3, 1 +2+3+ • • · +n=n(n + 1)12. Si {1, 2, 3, ... ,n} = A U B
consA= s.., entonces 2s..t= n(n + 1)12, o 4s,t= n(n + 1). Como 4 ln{n + 1) y mcd(n, n + 1):::
1,entonces4jno4J(n+ l).
b) Aquí verificamos la rccfproca de nuestro resultado de la parte (a).
(í) Sí 4ln,escribímosn =4k.Aqul<=>os {!, 2. 3, ... ,k, k+ I, ... , 3k,3k+ 1, . . ., 4k) =
A U B, donde A = {!, 2, 3, ... ,k, 3k+ 1, 3k+ 2, ... ,4k - 1, 4k) y B= {k + l,k+2.
. . .. 2k, 2k+ ! , 3k- l. 3k),cons,=(I +2 + 3+ · · · +k) + [(3k+ ! ) +(3k+2)+ · ..
~
+ (3k + k)) = [k(k + 1)12] + k(3k) + [k(k + 1)12] k(k + 1) + 3k'= 4k'+ k y s,=
[(k + !) +
Soluciones S-29

(,t+2)+· - -+ (l+l)J+({2k+ 1)+(2k+2) + · · - +{2k+l)J =l(l) +[.t(l+ 1)12] +.t(2k)


+ (.t(l+ 1)12] = 31'+.t(l+ lJ =4.t'+ l.
(ü) Ahora consideraremos el cason + l •4.t. F.ntoncesn •4.t-1 y tenemos (1, 2. 3, . . . •
1 - 1, .t, ... , 31-1, 3.t, ... , 4.t-2.41-1] =A U 8, cea A= {I, 2. 3, ... ,1-1, 3.t,
31+ 1, .. . , 4.t- 1) y B= {.t,1+ 1, ... , 2k-l, 2.t, 2k+ 1, ... ,31- 1). Vemosqoe
s,•[I +2+3 + · · · + (1-1)) +[31+{31+ 1)+ · · · +{31+(1- 1))] = [{1-IX1)12] +
.t(31)+[(1- IX1)12J• J.1'+1'- 1 = 4.t'- lys,= [1+(,t+ ll+· - - +{1+(1-1))] +(2k+
{2k + 1) + · · · + {2k+ (1- 1))] = 1'+ ((1 -1 )(1)12] +1(2k)+[(l- 1)(1)12) =31'+
(1 -1)1=4.t'- 1.

sección 5.1- 1, A x 8 • {(1, 2), (2, 2),(3, 2), (4,2), (1,5), (2, S), (3, 5), (4, 5)}
~g.250 B XA • {(2, 1),(2,2),(2, 3),(2,4),(5, 1),(5, 2),(5,3), (5,4)}
A U(B XC) • {l,2,3,4,(2, 3),(2,4).(2, 7), (5,3),(5, 4), (5, 7))
(A U B) x C - {(1, 3),(2, 3), (3, 3), (4, 3), (5, 3),(1, 4),(2, 4),(3, 4). (4,4).(5,4). (1, 7).
~7).~7).~7).~m - ~xC) u ~xC)
3. Six=-2y y •7.
5. a) 9 b) '2' e) 29 d) 2' e) (:) r) (;) +(:) +(:)
7. a) Supon¡amos que Ax B !:; C x D y sea.na E A, b E B. Eolooces (a.b) E Ax By como
Ax a¡;; ex D,ICDCmOSqoe (a,b) E ex D. Pero{a,b) e ex D=>a e eybED. Por
lotanlO, a EA=-• e e . dcmodoqoeA ¡;; eyb e B=>b eD,pocloqueB¡;; D.
Rcdprocamerue. supoopmosqueA ¡;; ey a¡;; D,yqoe (.r. y) EA X B. Emooces{.r.y) e
Ax B=>zEAyyEB=>zE C(puestoqueA ¡;; C)yyED~qucB ¡;;D)=>(<,y)
E e X D. En cons=encla. A X B ¡;; e X D.
b) Aunque alguno de los conjuntos A, B. e, D ,ea vado, seguimos terucndo qoe

[(A ¡;;C)/\ (B¡;;D)]~[A xB ¡;; C x DJ.


Sinemba,¡o, no tiene que valerlaiedproca. Portjcmplo, sean A= f,B= {I, 2], C= {1,2}
y D • ( 1 ) . Enlooccs A X 8 = ti; <21 caso cootrario, exi5'e un par ordenado(<. y) <21 A X By
CSIO significa que d coaju,Dlo vado A c:oooeoe un elementox. Y c:ntooc:es A X 8 "' fi t: C X
D, peroB = (1, 2) t.{)} = D.

'-©:-·,. . ,
ll. e) (z,y) e (A n B) x c=zeA n Byye C= c,eA yz e B)y ye c=c,eAyye
C)y(<EB.,. y E CJ= C<.y) EA X Cy(x. y) es X e= (z, y) E(A XC) n {B XC)
13. e,. y) EA X (B - CJ= z eAy y es- e =zeAy ()' EBy y ~ CJ=C<eAyy E B)y
(< EA yy ~CJ= (.r.y) EA X By (.r.y) ~A X C=C,. y) E (Ax 8)-(A x C)
u. •> (IJ (0, 2) e 9l; y
{2) Si (a. b) E91:, entonces (a+ l,b+ 5) E 9t.
S-30 SOiuciones

b) De la parte (1) de la defunción, tenemos que (0, 2) E Sil. Por la parte (2) de la defilljcióa,
tenemos entonces que
(i) (0, 2) E!!t*(O+l, 2+5)= (1,7)E!il;
(ü) (l,7)E9t*(l+l,7+5)=(2,U)E9l;
(iü) (2, U ) E!il*(2+ ! , U + 5) = (3, 17)E!il; y
(iv) (3, 17)E9t*(3+ 1, 17 + 5)·= (4, 22)E9t.

Sección 5.2- l. a) Función; imagen= {7, 8, 11, 16, 23, ... } b) Relación. no e, función
pág. 258 e) Función; imagen = R d) y e) Relación, oo es función
3. a) (!) {(l,z),(2,z),(3,z ),(4,z)} (2) {(1,y), (2,y),(3,y),(4,y)}
(3) {(1, z),(2,z),(3, z), (4,z)} (4) {(l,z),(2,y),(3, z ),(4, y)}
(5) {(1,z),(2,y),(3,z),(4,z)}
b) 3' e) O d) 4' •) 24 f ) 3' g) 3' b) 3'
S. a) {(1,3)} b ) {(- 7/2, -21/2)}
e) {(- 8, - 15)} d) R' - {( - 7/2,-21/2)}={(z, y) iz# - 7/2oy #-21/2}
7. a) [2.3-1.6]=[0.7] =0 b) [2.3] - [l.6]= 2-1=1
e) [2.3] -[1.6] =2 - 2=0 el) [3.7]+[7.3]=3 + 8=11
•) [3.41[6.2] = 4 · 6 = 24 f) (3.4][6.2] = 3 · 7 • 21
g) [2,:] =6 b) 2(1rJ = 6 i) 2f1r]=8
9. a) ... [- !, - 6/7)U[O,Jn)U[l,8/7)U[2,15/7)U ...
b) (!, 8/7) e) Z d) R
11. a) ···U (-5/2, -2] U (-3/2,-1] U ( - 1/2,0] U(l/2, 1] U(3/2,2] U ·
u-z(m-1/2,m]
b) · · · U ( - 7/3, -2] U (-4/3,-1] U (-1/3,0] U (2/3, !] U (5/3,2]U · · · =
u-<m-1/3,m]
e) · · · U((- 2n - 1)/n,-2] u((-n - 1)/n, -l] U(- 1/n,OJ U((n- 1)/n,l] U
((2n -l)/n,2]U · · · = U~u(m -1/n, m]
13. a) Demostración m, Si a E z•,cnconcc, f •l = a ylfaVaJ = llJ = l. Si a~ z·. escribimos
a =n +e, donde n E z· yO<C< l. EntoDCeS falta = (n + l)l(n +e) = 1 + (1 - c')/(n + e),
donde 0 < (1 - c)/(n +e) < l. Por lo canco, lfaVaJ = LI+(l-c)/(n+c)J = l.
b) Consideremos a= 0.1. Entonces
(i) l[aVaJ = (1/0.1] = [10] = 10 # l; y
(ü) íl• V• l = [0/0.ll = O#!.
1
De hecho, (ii) es falso para todo O<a< 1, yaqucfLaJ/a = Opara todos esos valores dea.
En elcasode(i), cuando0<a s 0.5, se sigue que
r
para o.5 <• < 1, a11. =11., donde 1 <11. < 2
f
. ,,."
yent=Lr
2 ylfaVaJ "2+ l. Sinemba,go,
•V•J =1 para o.s <a< 1.
15. a) Inyectiva; la imagen es el conjunto de todos Jos enteros impares.
b) Inyectiva; la imagen es Q.
e) No es inyectiva: la imagen es {O, :t6, ±24, ±60, ... } = (n3 - n In E Z }.
d) Inyectiva; la imagen es (O,+ oo).
e) lnyectiva;laimagcnes[-1 ,1].
O No es inyectiva; La imagen es [O, IJ.
17. 41
19. a) /(A, UA,) - {y EB iy =/(z), zEA, UA,} - {yEBiy =f(z),xEA , o
z E A,} = {y E B!y = f(z),x EA,) U{y E B !y = f(z),x EA,)= /(A,) U/(A,)
<) Delaparte (b)./(A, nA,J !:/CA,) nf(A,). Recfprocamcnte,y E f(AJ n/(A,) => y=/1.x,) =
f(x.;J parax, EA 1,x1 EA2 ~ y=/(x1) yx1 =x2 (pucstoque/csinycctiva)~ yE/(.,4., nA.;J.
Asf,/inyectiva =>/CA, n A,) = /CA,) nf(A,>.
21. a) /(a,1) = 12(i - 1) + j b) /(a,1) = lO(i - 1) + j e) f(a,1) - 7(i - 1) + j
23. a) (i) f(a,,)-n(i - 1) + (k - l)+j (ü) g(a,1) =mU -1) + (k-1) +i
b) k+(mn - l) sr
Soluciones S-31

:IS. a) A(l, 3) = A(O,A(l, 2)) = A(l, 2) + 1 = A(O,A(l, 1)) + 1 - [A(l, 1) + 1] + 1 =


A(l , 1) + 2 =A(O,A(l,O)) + 2 = [A(l,O) + 1] + 2-A(l, 0) + 3 =A(O, 1) + 3=
(1+1)+3=5
A(2,3) =A(l,A(2,2))
A(2, 2) =A(l,A(2, 1))
A(2, 1) = A(l,A(2,0)) = A(l,A(l, 1))
A(l, 1)-A(O,A(l,0)) =A(l, O) + 1 = A(O, 1) + 1 = (1 + 1) + 1 - 3
A(2, 1) =A(l,3) =A(O,A(l,2)) =A(l, 2) + l =A(O,A(l, 1)) + 1
= [A(l, 1) + 1] + 1 = 5
A(2,2) =A(l,5) =A(O,A(l,4)) =A(l,4) + l •A(O,A(l,3)) + 1 =A(l, 3) + 2
= A(O,A(l,2)) +2=A(l,2) + 3•A(O,A(l, 1)) + 3 = A(l, 1) +4 = 7
A(2,3) =A(l, 7) =A(O,A(!,6)) •A(l,6) + 1 = A(O,A(!,5)) + 1
=A(0,7)+ 1 • (7 + 1) + 1 =9
b) ComoA(l,O) =A(O, 1)=2=0+2,el resu!Wloes vilidoparaelcason=O. Suponemos que
la proposición (abíerta)es vetdadeta paraalg1ink( >: O) y teoemosA( l,i) =k + 2. Entonces
tenemosqueA(l , i+ l)=A(O,A(l,k)) =A(l,k)+ 1 =(i+2)+ 1 =(k+ 1)+2,demodoque
la verdad enn =kimplicala verdad~n =k+ t. En consecuencia.A.(l. n) =n + 2 para todo
n E N por el principio de inducción m.ueimtica.
e) AquftmemosqueA(2,0)=AO, I)= 3 (po<el resu!Wloen·l aparteb).Así.A(2,0) = 3 + 2 ·
Oy la proposición (abierta) dada es verdadera en este primer caso. AcontiDuación, supone-
mosque el resu!Wlo es cierto paraalg1ink( >: O); es decir, suponemos queA(2,i) = 3 + 2k.
Para!+ 1 tenemosenronces queA(2,h l)=A(l,A(2, k)) =A(2,i) + 2 (por la parte b)=
(3 + 2!) + 2 (por la hipó<esis de inducción)= 3 + 2(! + 1). En consecuencia, para todo ne
N, A(2, n) = 3 + 2n. por el principio de inducción matemática.

Sección S.3- l. a) A= [1,2,3, 4), B = {u, w,x, y,z),/= {(l,u), (2,u), (3, w),(4,x)J
~g. 266 b) A,B oomo en (a),/ ={(1,u),(2,x), (3,z),(4,y)}
e) A ={1,2,3,4,S}, B ={w;x,y,z},f= {(!, w),(2, w),(3,x),(4,y ),(S,z)}
el) A= {l,2,3, 4), B = {w,x,y,z},f= {(l, w),(2,x),(3,y),(4, z)}
3. a), b), e) y 1) son inyectivas y sobre.
d) No es inyectiva ni sotm:; imagen = [O, + oo)
e) No es inyectiva ni sobre; imagen=[-¼,+ 00)
S. (Par.lelcason=5,m=3):
~..(-1)'(,~,)(5 - k)' = (-l)'(l)s' + (-I)'(l)4' + (-I)'(l)3'+ (-l)'(l)2'
+ (-1)'(:}1' + (-!)'(!)o'
= 125 - S(64) + 10(27)-10(8) + 5 • O
7. a) (i) 2!S(7,2) (ü) G}[2!S(7,2)] (üi) 3!5(7,3)
(iv) (:)[3!5(7, 3)] (v) 4!5(7,4) (vi) (!)[4!5(7,4)]
b) (:)[k!S(m, k)]
9. Para todo r E Rcxistc al menos una E R talquciT-2a2+a - r=O, puesto que el polinomio
;;_ 'lr+ x- rtiene grado impar y coeficientes reales. En consecuencia./e¡ sobre. Sin cmbar-
go./(0) =O= /(1), de modo que/no es inyectiva.

3 10

25S
511
3025
9330
7770
34105
6951
42525 = 2646 462
5880
36
750
1
45
S-32

13. a) Como 156,009 = 3 x 1 x 17 x 19 x 23, se sigue que hay S(S, 2) = 15 factorizacionesoo


ordenadas con dos factores de 156,()()1), donde cada factor es mayor que 1.
b) S(S,3) +S(5,4) + S(5,5) = 25 + 10 + 1 = 36
e) ~-,S(5,i) =l5 + 25+10+1=51 d) k;_, s (n,i)

Secció n 5.4- l. Aquítmemos, portjemplo,que/(f(a. b), e) = /(a. e)= e, mielllrasque/(a./(b, c)) =/(a. b) = a.
pág. 273 de modo que/ no es asociativa.
3. a), b) y d) son conmutativ~ y a.wciativas; e) no es conmutativa ni asociativa.
S. a) 25 b) S" <) s~ d)S'°
7. a) SI b) SI e) No
9. a) 1216 b) p''q"
11. Por el principio de buen orden,A tiene un elemento mínimo y este mismo elemento es el neutro
para f. Si A C$ finito, entonces A tendrá un elemento máximo, y este mismo elemento será d
neutro para g. Si A es infinito, entonces g no tendrá un neutro.
13. SI
15. •>
5 b) A, A, A, e) A1,A2
25 25 6
25 2 4
60 40 20
25 40 10

Sección 5.5- t. Las palomas son los calcetines; los nidos son los colores.
pág. 278 3. a) 7 b)IJ <) 6(n - I)+ 1
s. 26'+1 = 677
7. a) Paracadax E {l, 2, 3, ... ,300}, escribimosz= 2" • m, donde n ;;e; Oy mcd(2. m) = l;M
tiene ISO posibilidades: 1, 3, 5, ... , 299. Cuando seleccionamos ISI números de {l, 2.3,
.. . , 300), deben existir al menos dos números de la formax = 2' • m, y= 2' • m. Si x<y,
cntonccsxly; encaso contrario.y< xy y¡x.
b) Si seloccionamos n + 1 enteros del conjunto {l, 2, 3, .. . , 2n}, entonces dicb.! seleccióo
debe tener dos enteros tales queily o ylx.
9. a) En este ca.so. las palomas son los enteros l.~ 3, ... , 2S y los nidos son los 13 conjuntos
{ I, 25), {2, 24), ... , {11, 15), {12, 14 ), {13). Al selecciona, 14 e nteros, obtenemos los
elementos de al menos un subconjunto de dos elementos, cuya suma es 26.
b) Si S= { 1, 2, 3, ... , 2n + t }.dondenesunentero positivo. entonces cualquiersubc:onjunro
de wnaoo n + 2 de S ~ CO!ltentt dos elementos cuya suma sea 2n + 2.
11. a) Parai..-ualquier t E {l. 2, 3, . . . , 100}, vemos que 1 :S Ji :S 10. Cuando seleccionamos 11
<lemcotos de {I, 2, 3 •... • 100), debemosteoer <!os (digamo,, x y y) tales quel,.G = J
L-51• de modo que o <1-Jx- J;J< 1.
b) Sean E Z ... Si seleccionamos n + 1 elementos de {l. 2. 3• . . .. 112}. entonces existen dos
(digamos. i y y) tales que O<!Ji- .J,J< l.

13. Dividimos el interior del cuadrado en cuatro cuadrados


congruentes más pequedos. como se muestra en la figu.

EB·
ra Cada cuadrado pequCOO tiene 11,Ji como longitud
de la diagonal. Sea R 1 la región del interior del cuadrado
AEKH junto con los puntos del segmento EK. sin incluir
el punto E. La regiónR2 es el interior del cuadradoEBFK
junto con los puntos del segmento FK, sin incluir los
puntos F y K. Las regiones R3 y R. se definen de manera
análoga. Entonces, s.i elegimos cinco puntos del interior
del cuadrado ABCD, al menos dos de: ellos estén en R.;
Soluciones S-33

pan algw¡ 1 sis 4 yatos p<JDIOS csán a unadiSlallcia


menor de lt.fi (unidades) entre sr.

15. Considcr<mos los subconjuntos A de S tales que 1 s IA I s 3. Como ISI = S, emten


U)+(n+(n =2S de tales subconjuntos. ScasAla sumade losclcmentosdeA. Entonces 1 :S
sA :S 7 + 8 + 9= 24. Mí. por el principio del palomar, existen dos subconjuntos de S cuyos
elementos producen la misma wma.
17. 12
19. a) Como ISI .e 3 existen.,, e Stales que;,,y son ambos pa= o ambos impares. En todo
caso, x + y es par.
b) 5•2' + 1 <) IS l>,9 =2' +1
d) Paran E z.,sea S • {(oh a,. . .. , aJJa; E Z\ 1 Sis n}. Si ISI ~ 2"'+ l, entonces S
cootieoc do$ n-uptas ordenadas (x1, .ri. . .. ,x.). (y1, y,. . . . , yJ tales que-"+ y1 es par para
todol :Si:Sn.
e) S, <OlDO en la parte (b).

Sección 5.6-- l. hes sobte <e> pan todo b E B. d E D, existen a E A, e E C tales que h(a, e) = (b, d) <e> para
pág. 290 todo b E B, d E D existen a E A, e E C tales quef(a) • b, g(c) = d <e>J, g son sob<c. hes uno
a uno<=> (para todosa,a1 EA, c,c1 E C, /r(a. e) = J(.a1,cJ =>a = a1, c=c1J<=> [para todos a.
cz. E A,c, c1 E C,/(a) =-/(a 1) =>a a ª r y ,Ce) =g(c1) =>c=-c1) <=>/.1 son inyectivas.
3. g'(A )=g(Tn(SU A )) • T n(SU(Tn(SUA)))
• Tn[(SU T) n(SU(SU A))] - Tn((SU T)n (SU A)]
• [Tn (Su T))n (S U A) = Tn (S UA) •g(A)
S. a=3,b•-l;a•-3,b•2
7. a) 7!-6!•4320 b) n!-(n-l)! • (n-l)(n - 1)!
9. a) (b,a) E (91, U91,y e> (a,b)E91, U91, e> (a,b)E91, o(a,~) E 91,e> (b,a) E !lt; o
(b,a) E!ll;e>(b, a) E 91; U91;
b) (b,a) E (91, n91,)' e> (a,b)E91, n91, e> (a,b) E91, y (a,b)E91, e> (b,a )e91;
Y (b,a)e91\ e> (b,a) e9t;n91;
e) (a, b) E (91:Y<c>(b, a) E (91:)<c>(a,b) €91,
u . a) r '(z) • (l/2)(1nz - S)
b) Para z E R•,
(f•r'Xz) • f((l/2)(1nz -S)) - ,~,,,,,.._ n>•• =....-,., -··· =z.
Paraz E R,
<r'•J)(z) • r'(,-') • (l/2}(1n(,-')-5] • (1/2)(2' + S - 5] •z.
S-34

13. f. g invertibles=:) cada función/, g es inyectiva y sobre~ g o/es inyectiva y sobre~ g o/es
invertible. Como (g o/) o U-1 o g-1) = le y (,f4 o g--t) o (g o/)= }.4, se sigue que~ o g4 es un
inverso de g o/ Por la unicidad de los inversos, tenemos que /4 o g-1 = (g o¡r•.
15. •l r'(-10)={- 17) ¡-'(0)={-7,5/2}
¡-'(4) = {-3, 1/2,5} ¡-'(6)={-1,7)
¡-'(7) = {O, 8) ¡-'(8) = {9)
b) (i) (-U, -8] (ü) (-12, - 7] U[5/2,3)
(üi) (-9, -3)U[l/2,5] (iv) ( - 2,0)U(6,11)
(v) (12, 18)

17. 31 • 4 1 = 576 funciones


19. a) Laimagende/={2.3. 4 •... J =Z•- {l}.
b) Puesto que l no está en la imagen de/, estafuncióo no es sobre.
e) Para todosx,y E z·,/(x)=/(y) =>x+ 1 = y+ 1 =>x= y. de modo que/no c., inyectiva.
d) La imagen de g es z•.
•l Como g<_z•¡ = z·. el codominio de g, esta función..,~
f) En este caso. g(I) = 1 = g/_2) y I j 2. de modo que g no c., inyectiva.
g) Para cualquierx E Z". (g •J)(x) = g(_/(x)) =g(x+ I)= máx{l. (x + 1)- 1} = máx{l.x} = x,
pues X E z•. Por Jo tanto, g o/= t.-
b) (/•g)(2)=/(max{l,l}}=/(1)=1+1=2
(/•g)(3) • /(max{l,2}) = /(2) = 2 + 1 = 3
(/•g)(4) =/(max{l,3}) =/(3) = 3 + 1 = 4
(/•g)(7) =/(max{l.6}) =/(6) = 6 + 1 =7
(/•g)(l2) = /(max{l, 11)) = /(11) = 11 + 1 = 12
(/•g)(25) =/(max{l,24}) =/(24) = 24 + 1 =25
i) No, pues las funcionesJ. g nosoo invenas entre sí. El cálculo de la parte (h) podría indicar
que/ o g = t~. pues (fo g)(.r) = x para x ~ 2. Pero tambifn vemos que (fo gXl) =
/(máx{l, O})=/(!) = 2. de modo que(/• g)(l) j 1 y, en con=uencia./• g j 1~.
21. a) /(9. 11) = S = /0. {ll) y (9, 11) # (9. {I J). por lo que/no c., inyectiva.
g({l J. {2}) = {l, 2} =g({l. 2}. {2}) y ({l}, (2J)i'({l. 2}. (2)). porloquegesinyectiva.
1,((1 }. (2)) = ( 1, 2} = 1,((2), ( 1J) y (( 1}, {2}) SF ((2}, ( 1J) por lo que h c., inyectiva.
b) Para cualquier A subconjuntodeZ•./(A.A)=g(A.A)=h(A. 9) =A.de modo que cada una
de las tres funciones f. g y h es una función sobre.
e) De los resultados de la pane (a) se sigue que ninguna de estas funciones es invertible.
d) Los coojuntos/"'(9). h·'(!ll,/"'(( 1 }). h-'((3))./"'({4, 7}) y h·'((5. 9}) son todos infinitos.
•l g·'(0J; {(S,0).Ylg· •(0)1= 1
,-'((2}) = {(0. {2)),((2}.0). ({2},{2})}, Y ¡g-'({2})1= 3
¡g - '({8.12})1 =9
Soluciones S.35

23. a) ae¡-'(B, n B,)<.:>f(a)EB,nB,<.:>f(a)EB, Yf(a ) EB,<.:>a e¡-"(B,)


Y• e¡-'(B,)<.:>a e¡- '(B,) n¡-'(B,)
e) • E¡-'(B,)<.:>/(a)EB,<.:>/(a)fÉ B,<.:>a r/i¡-'(B,)<.:>ae¡-1(B,)
25. a) Supoogamos quez,. z, E Z y f(zJ •.{(.aj. Eolooccsf(z,),.f(.aj soo ambos pares o ambos
impares. Si ambos soo. pares. entonces ~J = /(xJ => -2r1 = -lxi => x1 • X:- Ea caso
cootrario,/(zJ.f(z,) soo ambos impares y/(z,) =.{(.aj=> 21:,-1 • 21:,-1 => 21:,=lz,=>
xt : .t,:. En coosccuencia. la función/es inyectiva.
Para demostrar que/es una función sobre, sean E N. Sin es par. entonces (-n./2) e Z,
(....-2)< O yf(-,,/2)•-2(-,,/2) = n. Parad ca,o en que" es impar,._ que (n + 1)'2 E Z.
(n + l°)/2>0 y /(C:,, + 1)'2) = 2{(n + 1)'2] - 1 =C:,, + 1)- 1 = n. Pl><lo tanro,/es >Obro.
b) J-': N ➔ Z. donde

r'(z) = {(!)(•- z +/2!),, %- 1,3,5, 7 ...


z•0,2,4,6, ..•
27. a) Para lodon EN, (z •IX• ) = (h •IX•)• (kolXn) = "·
b) Los resultados de la parte (a) nocootradicenel teorema S.7. Ya que au.nqucg o/= lto/•
k • J• 11'1, observamos que
(1) (fo gXl)=/Q.1/JJ) =/(0) = 3 • O •O+ I, de modo que/ o g;. 1.:
(U) (f, hXI) =Jq_7,13_p =f(0) = 3 · 0 • 01' 1, de modo que/, h i' l.; y
(iü) (fokXl)=JQ.3(3.P=/(1)=3· 1 •Hl,demodoquc/oHI•.
En COOS«U<Dcia. DiaguDa de las funciooes g, ~ o k es la i1lvcna de f (Desputs de todo,
como/no es sobre. no es invertible.)

Sección 5.7- l. a) /EO(n) b ) /EO(l) e) /EO(n"¡ d) /E O(n')


pág. 297 e) /EO(n"¡ I) fe O(n') s) /EO(n')
3. a) Para todo n E z•, O :5 log¡n <n. Asl, seank = 1 y m • 200en la definición S.23. Entonces
1/(•ll = 100 log,n = 200(½ log,n) < 200( ½•>
= 2001,<n) I, por lo que/E O(z).
b) Paran• 6, 2'-=64 < 3096:: 4096- 1000= 212 - 1000 • z:.._ 1000. Si suponemos que~<
2"- IOOOparan al>: 6. cenemosque2 < 2'=> 2(2') < 2'(2"- 1000)< 2'2'- 1000, 02"'<
iv-• - 1000, por lo que /(n) < g(n) para todo " >: 6. PI>< lo WllO, si l • 6 y m = 1 en la
definición 5.23, v=os que para " >: k. 1/(• l I s "'1g(•JI y JE O(z).
5. Pan .,.,.....que/E O(z), ,cak= 1 ym•4en ladelimcióo S.zl. En!ooccs. paran>: k.1/(nJI •
n2 + n :5 nl + nz= 2n1 :5 2n>: 4((1/l)n') =- 4lg(n)I y/es dominada por g. Para mostrar que
g fÉ O(f), seguimos la idea del ejemplo S.66: es decir,
Vm E R· Vk E z•3n E z· [(n 2'k) /\01(n~ >ml/(n)0 J.
A>í. DO importan los valore> de m Y k. ele¡imos " > mú{4m, k). Eruooce$ li<• JI ={½)•'>
(½) (4m)r• m(2"') >: m(n'+n) = m 1/(n) 1, de modo que r fÉ O(f). La altenwiva es ver que si
g E O(f). entODC<S 3m E R· 3k E z· v. z·
E 1 Hl•'I s mi•'+"'· o(t)n' s m(n + 1).
nl nlnl n
Entonces 2(n+ l) s m =>O<¡;;< 2(n+I) s m ⇒ ¡ s m, una contradicción. pues n es
variable y m coastante.
7. l'llra ,odon >: l,log,n S n.as/quesik• 1 ym= 1 culadefinici6o5.zl,tenomOS l,<n)I = log,n S
• =m · " • m lf(•ll- PI>< lo tanto. g E 0(/). Para mostrarque/fÉ O(z), observamos primero
que Um _n_ = + oo. (Podemos establecer esto mcdjantcla regla del cálculodeL'Hospital.)
..... 1011"
Como lim - · -'"' +CD, tenemos que para cada m E R.· y k E z .. exiSlt ne z· w que
n ..... k,g2n
log,n > m, o 1/(n)I • "> mlog,n = mlg(nll- Podo tanto,/ t O(z).
S-36 Soluciooes

9. Como/E O(g).exis<enm E R',k E z·wesque 1/(n)i s mlg(•)I para todo n >: k. Poro
entonces 1/(•)I S [mllclJlcg(n)I paratodon >: i ,porloque/E O<_cg).

secóón s.a- l. a) /E O(n') b) fE O(n') e) /E O(n') d) /E O(log,n) e) /E O(n log,n)


pjg. 305 5. a) Aquí tenemos cinco sumas y (2 + 3 + 4 + S + 6) = 20 multiplicaciooes. ·
b) Para el caso gencraJ, tenemos n sumas y (2 + 3 + · · · + n + (n + 1)) = (lnXn + tXn + 2}-1
multiplicaciones.
7. Para• = I. tenemos que a,=O =LOJ= L1og,1J, por lo que el resultado es verdadero en este
primer caso. Supongamos ahora que el resultado es cierto para todo n = l. 2. 3, . .. , k, donde
k ~ 1 y consi.derenl0$ los casos paran = .t + l.
(i) n =k+ 1 = 2•,dondem E z•: En este caso.a.= 1 +al.-'2-.f 1 + az-.-i = t+Ltog2 2.-tj =
1 +(m -1) =m =Llog2 2" J = Liog,nJ; Y
(ii) n = .t + 1 = Z-+r, doDdem e z•y 0< r< Z-: En este caso 2'"<n < rt, por lo que
(!) r·'<(n/2)<2"';
(2) r-• =¡r- •J s l•/2J < [2"'J = 2"'; Y
(3) m -1 = log,2"'· ' s!og,[n/2] <log,2"' =m.
En consccucncia, LIog,[n/2JJ =m -1 ya.= I + ªL,¡,J= I +L Iog,[n/21] = I + (m-1) =
m=[!og,•J ·
1og2 nJ
Por lo tanto, la forma alternativa del principio de inducción mat:emMica implica que a.= L
para todo n E z•.

Ejercicios l. a) (i) Si algunodelosconjuntosA o Bes vacío,entooces:A x B = e =A n B ycl resultado


complementarios- es cierto. Para A, B no vacíos tenemos que:
pjg. 312 (x,y) E (A X B) n (B X A) ⇒ (x, y) EA X By (%.y) EB X A ⇒ C, EA y y E B)y
(zEB y y EA) ⇒ zEA n Byy EA n B ⇒ (x,y)E (A n B) X (A n B);y
í,%.y)E (A nB) X (A nB) ⇒ (z EA yz E B)y(yEAyyEB) ⇒ (%,y) EA X By
r,,.~eBxA ⇒ í,%.~ E(AxB)n~x ~
En consocuencia, (A X B) n (B X A) = (A n B) X (A n B).
(ü) Si alguno de los conjuntos A o Bes vado, entonces A X B = f!I =B x A y el resultado
es verdadero. Sí no, sea(%, y) E (A X B) U (B X A). Entooees
(x,y)E (A X B) U (B X A) ⇒ (z,y) EA X Bo(z,y) E (B XA) ⇒ (< EA y y EB)o
(z EB yyEA) ⇒ C,EAoz EB)y(yEAoyEB) ⇒ .r.yEA U B ⇒ (%,y)E(A U
B)x(AUB).
b) Para~= {I, 2, 3,4), sean A={!, 2} y B= {3}. Enl0oc<$ (A U B) x (A U B) = {I, 2, 3) X
{1,2,3), mientrasque(A x B) U (B x A)= {(l. 3),(2, 3)) U ((3, 1),(3, 2)). Como 1(A U
B) x (A UB)I =9y l(A xB) U(B x A)I = 4,nopuedeoc:wrirque(A UB) x (A UB)~
eontenido en (A X B) U (B X A).
3. a) ~> b) oo

:L :L
O

e) 2u-4J
l 5 (AJ O 1 5 (AJ
Soluciones >-37

s. a) (.r.y) E (/1 nB) X (Cn D) é>xEA ns.y e cnD<c>(.r EA,yE C)y(.r EB,yED)
é> (x,y) EA X Cy(.r. y) EB X D é>(:r, y) E(A XC) n (8 X D)
b ) (.r.y) E(A u B) X (C U D)<c>xEA UB,yE cu D<c>(xEAo,e B)y(y E Coy E
D)<c> (.rEAyy E C)o (.r EB yy ED)o (.r E A yy E D)o (xEBy y E C) é>((.r, y) E
A X C)o((.r,y)EB X D)o((.r,y)EA X D)o((.r,7) EB X C)<c>(.r,7) E(A XC) U
(B X D)U(A X D)U(B XC)

7. Como9t !: z· x Z',debemos verificarquez• x z· !:9t. Si z· x z·!t9t, seaS= ((m. n)Jm,


• E z • y (m, n) E9t}. Dela partc(l)deladcfiniciónrccunivadada, sabemos quc(l. 1) E S.
Si S f, f, entooce5 podemos elegir de los elemauos de S.., par O<deoado (m, n) para el que
m +n sea minimal. En este caso. al menos uno de los m1maosm.n es mayor que l. (i) Si m > t.
eotonoes m-1, n E z· y(m- l)+ n < m + n. de modoquc(m- l, n) f: S. EDCOllSCCDCIICia.
(m - J, n) E9ty por la primera parte del apanado (2) de la definición recursivaoblcnemosque
((m - 1) + 1, n) = (m. n) E9l. (ti) En ca.wcontrario, param a l , tenemos n > l , n -1 E z• y
m +(n-1) <m+n. Pwlo tanto,(m,n - l)E S.Asl, (m,n-1) E9ly por la segunda parte del
apanado (2) de la definición i=niva t,,iemo, que (m, (n - 1) + J) • (m, n) E 9t. Las dos
cootrldicdooes cu las - Ql y fúl sdlalan que s = f y 9t ::i z· x z ·.
9. • = u.fi y, = .,/3j2 ll, a) (7!Y(2(7')]
13. Para 1 :S l :s 10, scai:;el n6mcrodecartas escritas en el día i. F.4tonccs.r1 +.r2 +z,+ •••+.ta,
Xt-Xt0• 84,o.r1 + · · · + zt•S4. Suponpmosquex1 +x2 +z,< 25,-Xi+x,+z..<25, ... ,zt+xt
+x.<2.S. &tooces.r1 +2x2 +3(;t,+ · · · +.r..}t- 2K,.+.r. <8(2S) =2CO, o lC.r,+· · · +x.)< 160.
<-!f.= t-
F.o ooosecuenc:ia. obtenemos la coorradioci6rl S4 =.r, + · · · + .1c 53
15. Para que n;., (i - ~) sea impar, (i- iJ debe ser impar para lodo 1 ,: i s n; es decir, ono de
los nllmerosk.i. debcserparyd otro impar. Como n es impar,n-= 2m + 1 y la lista I, l. ... , n
debe contcncr m enteros pares y m + 1 enteros impares. Sean 1, 3, S, ... , n las palomas e ih
;,. 4. ... , i. son los nidos. 01.ando mucho. m de los nidos pueden ser enteros pares. de modo
que (i-iJ <Jebe,c,par para al menos 1111 i= J, 3, 5, ... , n. En coosecuencia.ft..(i-~)
es par.
17. Sean x1o xi, ... , x. los n objetos distintos. Colocamos x. en un recipiente. Ahora tcnemo$ dos
rccipientesdistwos. Paracadax1.x2,, ... ,x..i baydosopdooes. loqueda 2~ distribucioocs..
&trc &w hay una tal que.r1,x2,, ...• x.... están en el recipieotc coox.. por lo que eliminamos
esta diSlribución y tmcmos que~. 2) = 2*'- J.

19. a) y b) m!S(n, m).


21. Fijemos m= 1. Paran • 1, el resultado es cierto. Supongamos que/oP =P o/y consideremos
/•/"'. f •JM =f • (/•/') =/• (/'•/)• (/•/') •f•JM •f. Porlo WJto,/•/'=/'•/pan.
todon E Z-. Su¡,oopnooabora..,.porul¡1!nt a, l ,f'•f'• f ' • f ' . - r' •/'=(/•/')•
/'=/•(/'•/') =/o(/'•/')•(/•/')•/'•(/' •/l •/' =/'•(/• r> =f' •r', de modo'IJl"r•r
= r • rparatodosm,n ez·.
13. Cualquier n E z· tal que n sea impar.
25. x<y~/(.r) </(y) (ple$/., ctecicnle)~ g(f(.r)) <g(/(y)) (pues ges crocicu1<). Pw lotanlO,
g o/ es acc:icnte.

rl. f •g • {(x,z).(y,y). (z,i)}; g•/• {(i,i),(y,z).(z,y)J;r' • {(i,z).(y,,).(z,y)};


g-• • ((i,y),(y,i), (z,z)}; (g•f)-• • {(i,i).(y,z),(z, y)} • r'•g-•;
g-'•r' • {(x,z).(y,y), (z,,)}.
29. 2'. 2' · 3'~ m6 funciones
JI. a) (,roa)(,)= (a•-.)(x) • i
í b) .,,...(,)•x-n; a"(i)•i+ n (na,2)
el ..-•ci>••+•;a-•(•l••-• (•"'2)
S-38 Soluciones

33. a) T(n)=(e,+1)(<, +l)···(e,+1)


b) k = 2: 7(2) = 7(3) = T(5) = 2 k = 3: 7(2') = • (3') = T(S') = 3
k = 4: 7(6) = 7(8) = 7(10) • 4 k = 5: 7(2') = 7(3' ) = 7(5') = 5
k = 6: 7(12) = 7(18) = 7(20) = 6
e) Para todo k > 1 y todos los primos p, r '(p'"') = k
35. a) S(8, 4) b) S(n, m)
37. a) Seanm = 1 yk= l. Entonces,paracualqulern >: k lf(n)I s 2<3 s Jg(n)I = mlg(n)l,po,
lo quef E O(g).
39. Observemos primero que si log.n = r ,eruooccs n = d y log.n = iog.(a') = nog.a = (log.a)(log.n~
Ahora, seanm = (los.a) y k= l. Enton=, para cualquier n >: k. l,<n) I = log,n = (log.a)(log.n) =
mlfCn)l, porloque g E O(f). Por último, si m = Oog.a)4 = log.b y k= l. vemos que para
cualquiern >: k. lf(n)I = log.n =Oog.b)(log.n) =m lg(• )I. Por lo tanto,fE O(g).

Ca pitulo6
Lenguajes: Máquinas de estados
finitos

Sección 6.1- l. a) 25; 125 b) 3906 3. 12 5. 780


p.lg. 38 7. a) {00,11,000, 111,0000,llll} b) {0,1}
el r-¡,,oo,11 , 000, 111,0000,1111} d) ¡0,1,00,11¡
•J {OO,Ol ,10, ll}•{w ll w l =2} or
gJ l:" -{O, 1, 00, 11}= {l,01, 10}U{wll wl ac3}
h) l:" - {X,O, 1, 00,01, 10,11,000, 111,0000, 1111)
1) I' -{X,O, 1, 00, 11,000, 111,0000, 1111)
9. a) x EAC=x= acparaalglln.a EA.e E C=::r E BD.pucsA s:;; B. C~ D
b) SiAf ,< 8, seax E Af.x E Af =»= yzparaalg,1ny EA,, E 0. Pero, E 8 es imposible.
EntoncesA8 f 0. [De manera similar, 8A= 8.J
11. La única forma en que es,o puede ocurrir es que A=¡;.¡.
13. a ) En este caso, A• consta de todas las cadenas x de longitud par, donde $i x ?= i..., entooces x
comienza con O y termina con 1, y los símbolos (0 y 1) se alternan.
b) En este caso.A• consta precisamente de aquellas cadenas formadas por 3n ceros, paran E
N.
e) Aquí, una cadena x e A• si (y sólo si)
(i) x es u.na cadena den ceros, paran EN; o
(ü) x es una cadena que comienza y termina en cero, y tiene al menos un J y al menos dos
ceros entre dos unos cualesquiera.
15. SeaI:unalfabcloC00 e "A ¡;;I;•. Si IAI = 1 yxEA.c:ntoooes.tt= x,pues A'=A.Pero 11.nll =
2llxll = llx ll ⇒ llxll =0 ⇒ x E 1... Si IAI> 1, S<a x E A tal que llxll> O pero llxll es
minimal. Entoncesx E A'⇒ x=>~, para y, , EA. Como llxll = IIYll + ll,11. si IIYII , 11,11>
O, entonces una de las cadenas y, z está en A con una longitud menor que Ilxl 1- F.n consecuen-
cia. uno de los valo= IIYII o 11,11 es cero y A EA.
17. Si A= A.1, entonces se sigue por inducción matemática que A = A'" para todo n E z•. Por lo
tanto,A =A.♦• Por el ejercicio 15,A = A1 ⇒ A. E A. Porlo tanto, A = A•.
l!J. Por la definición 6.11,AB = {abla EA,b E B} y como es posibletencra1b 1 . a 2 b,; cona1,a2 E
A, a, #a, y b,, b, E B. b, lb, se sigue que IABI s IA x BI = IA I IBJ.
21. a) x E A(U.,,B,J <'>X= ab para algún a E A , b e u.,s; <'>x=ab, donde a E A y b E B,
para algún i E / ea> x E AB, para alg,ln i E / ea> U.,Al\-
23. a) Las palabras 001 y 011 tienen longitud 3 y están en A. Las palabras 00011 y 00111 tienen
longitud S y también están en A.
b} Del paso (1) sabemos que I E A. Entonces, aplicando el paso (2) tres veces, obtenemos
(i) 1 EA ⇒ Oll E A;
Soluciones S-39

(ü) 011 EA => 00lll E A;y


(iü) OOlll E A => 0001 lll E A.
e) Si 000011 11 estuvia a en A. entonces. del paso (2) ve mos que esta palabra seda generada
porOOOlll (enA). Delamismaforma.OOOlll enA =>0011 estáenA =>01 está en A. Sin
embargo, en A no hay palabras de longitud 2; de hecho, no hay palabras de longitud par
enA.

2$. a) PasClo Ruoaes


1) () está en A Parte (1) de la definición recursiva
2) (())está en A Paso (1) y parte (2-ü) de la definición
3) (( ))( ) estA en A Pasos (1) y (2) y parte (2-i) de la definición
b) PasClo Ruoaes
1) () estA en A Parte (1) de la definición recursiva
2) (( )) csá en A.
Paso (1) y parte (2-ii) de la defürición
Pasos (1) y (2) y parte (2•i) de la definición
3) (( ))( ) estA en A .
Pasos (1) y (3) y parte (2-i) de la definición
4) (())()()está en A.
27. (1) l. E AysEAparatodos E l:;y
(2) Para cada x E A y s E l:, la cadena= está también en A
[Ninguna otta cadena de i:• csá en A.]

sección 6.2- t. a) 0010101; ,, b) 0000000; ,, e) 001000000; So


~ . 333 3. a) 010110 b) ,.o , .o

..,
5. a) 010000; ,, b) (s,) 100000; ,,
(s1) OOOCKX)¡ S1
e)

o
. ..
1 o 1
(s,) 110010; s,
,, o o
So
s,
So
s, ,, 1 1
s, s, s, o o
..
s, So s,
s, s,
o 1
o 1
d) s, •l x - 101 (único)
7. a) (i) 15
9. a)
(ü) 3"
. .. (iii) 2" b) 6"

o 1 o 1
,. ,, o o
So
s, ,, s, o o
s, ,, ,, o 1
s, s, ,, o o
,. s, s, o o
s, s, s, 1 o
Soluciones

b) Sólo hay dos posibilidades: x= 1111 ox=(X)()().


e) A = {111}{1)' U {000}{0) •
d) En este caso, A = { lll II l{ 1 )' U {00000}{0)' .

Sección 6.3- l. a) b)
p.lg. 342
'·º

5. b) (i) 011 (ü ) 0101 (ili) 00001


e) La núquina tiene como salida un Oseguido por los primeros n -1 símbolos de la cadena de
entrada x con n símbolos. Por lo tanto, la máquina es un retraso unitario.
d) Esta máquina realiza las mismas tareas que la de la figura 6.13 (pero sólo tiene dos esta-
dos).
7. a) Los estados de transición sonso- s 1• El estados, es un estado de sumidero. {s1, s1, s,. s,,ssJ,
{s,} y {s1• s3, s,} (con las restricciones correspondientes sobre la función dada v) son
submáquinas. Las submáquinas fuertemente conexas son {s..} y {s1, .r» s,}.
b) Los estadoss,. s, son de transición. El linico estado de sumidero ess,. E1 conjunto {.ro,s., .r,,
s,,} proporciona los estados de una submáquina; {.ro. s 1} y {s,} son submáquinas fuerte..
mente conexas.

Ejercicios l . a) Verdadera b) Verdadera e) Verdadera d) Falsa e) Verdadera


complementarios- 1) Verdadera g) Verdadera b) Verdadera
p.lg.34 5 3. Seanx E I: y A = {x). Entonces A'= {.u) y (A'J' = {).,x',x', . .. ).
A•= {A. x. .r'.r, ... } y {A*) 1 =A"', de modo que (Al)• -;,. (A"')1•
5. 0..={1,00}' {0}
O,,= {0}{1,00}' {0}
On = ~
0..={1,00¡• -M
O,,= {l}{I , 00}' U {10}{1,00}'
7. a) Por el pñncipio del palomar, e xiste un primer estados que se encuentra dos veces. Seay la
cadena de salida que resulta. la primera vez que se encuentras, hasta llegar a este estado por
segunda vez. Entonces, a partir de ese punto, la salida es yyy .
b) n e) n
9.

~
'·º
Soluciones s-41

U . Supongamos que podemos consuuir esa máquina y que tienen estados, paran E z •. Para la
cadena de entrada O' 1..1, queremos que la salida sea fi"-1. Sin embargo, al procesar los unos de
esta cadena de entrada, obtenemos n + 1 estados S¡, Sz, • .• , s., s..1 de la función v. En conse-
cuencia, por el principio del palomar, cxi$lCD dos cstadoss;, s1 tales que i <i peros,= s,- Como
resultado, si eliminamos los j - 1 unos de los estados s,.., s1,.:, • •• , s,. tenemos que la máquina
~ ela sucesiónO"t-1-<rl,doIJden + l -U-1) :S n. Sinembargo,O" 1..1-(H) fl_A.

13. a)

o
. 1 o
.. 1

(s.,s,) (s.,s,) (si.s,) 1 1


(s.,s, ) (s.,s,) (si.s,) o 1
(si.s,) (s1,s,) (sz,.s,) 1 1
(si.s,) (s1,s,) (sz,s,) 1 1
(sz,s,) (s2,s,) (s.,s,) 1 1
(sz,s,) (s,,s,) (s.,s,) 1 o
b) m((So, ss), 1101) = JI 11; M1 esd en el estado .so y M 2 en el estados,.

capitulo 7
Relacionff: La MgUnda vuelta

sección 7.1- l . a) {(I, 1), (2, 2), (3,3), (4,4), (1, 2), (2, !), (2, 3), (3,2))
~ g:356 b) {(1,1),(2,2),(3,3),(4,4),(1,2)} e) {(1,1),(2,2),(1,2),(2,1)}
3. a) S..01,,f,,f, E ':f conf,(n) = • + l.f,(n) = Sn, y f,(n) = 4n + 1/n.
b) Sean ga, 81, g3 ES con g 1(n) = 3. gi,n) = 1/n y g,t..n) = sen n.
S. a) Reflexiva, antisim&rica, transitiva
b) Transítiva
e) Reflexiva, simwica. transitiva
d ) Simttrica
e) Caso par: reflexiva, sim&nca. transitiva; ca.w impar. s~ca.
O Caso par. reflexiva. simttrica. transitiva; caso impar: simétrica.
&) Simttrica
h) Reflexiva, sim6trica
i) Reflexiva, transitiva
j) Reflexiva. simétrica, transitiva
7. a) Para todox E A. (x, x) E9ti,9i?> de modo que (x, x) E!Jl1 n Wt2 y!:11 n fJl:1 es reflexiva.
b) (i) (x. y) E 91:1 n 9t2 ~ (.x. y) E 911,!:Vlz= (y. x} E 911o9?1= (y,x) E9t1 n91z y91 1 n 912
essi~ca.
=
(ü} (x. y),()'. x) E911 n 9't1 (x, y). (y. x) E 91i.9h. Por la antisimetdadc9t1 (of/W,x =
y n
y 91:1 !Jlz es antisimétrica.
(iii) (x.y),(y, V E 911n9Jl:1=> (x.y),(y. V E 9ii,9't2=>(x. z) E91:1.~2(propiedadtransitiva)
~ (x, z) E 91 1 n 912, de modo quc!!A:1 n g\2 es tramitiva.
9. a) Vetdadaa b) Falsa: s,anA = (1, 21 y91 = {(I, 2), (2, 1)).
e) (i} Reflexiva: verdadera
(ii) Simétrica: falsa. Sean A= {l. 2}, 91, = {(l. 1)1 y 91,= {(I, 1), (1, 2)1.
(iii) Antisimttrica y transitiva: falsa. S... A= { 1, 21. 91, = {(1,2)1 y 91,= {(I, 2), (2, 1)).
d) (i) Reflexiva: falsa. S... A= {I, 2),91, = {(I, 1)) y91, = ((1, 1), (2, 2)).
(ii) Simétrica: falsa. S... A = { l. 2}. 91, = {(I, 2)) y 91,= {(I, 2). (2. 1)1.
(iii) Antisi~ca: verdadera
Soluciones

(iv) Transitiva: falsa. SeaDA= {l, 2),9t,= {(!, 2). (2, 1)) y
91,= {(l, 1), (1, 2), (2, 1), (2, 2)).
•> Verdadera.
11. a) ('•t•)('•¡-') =mm= 9 b) 18
e) (••¡-')('•¡-•)•mm= 30 d) f,(l
•)81 f) m
13. Podñacxistirun elemento a EA tal que para todo b EA, (a. b) y (b, a) no esttn eo9t.
1.$, r - n cuenla los elementos en 9t de la forma (a, b), a ...¡. b. Como 9t es si~ca. r- n cs par.

Sección 7.2- l. i!t,9' = {(l, 3), (1, 4)}; 9'•!!1 = {(!, 2), (1, 3), (1,4), (2,4)};
pág. 368 9t' =9t' = {(1,4), (2,4), (4,4)}; 9" = 9" = {(I, 1), (1,2), (1, 3), (1, 4)}
3. (a, d) E (9t1 o9t2' 091:1 ⇒ (a, e) E91: 1 °91:2.(c,d) E 9t,para algónc E C ⇒ (a. b) E9li, (b.c)
E!ll~ (e, d) E!ll, paraalgllnb E B,c E C ⇒ (a, b) E!ll.,(b, d) E!ll,•!ll,⇒ (a.d) E!ll,• (91,•
~ y (9't1 09W o9t, ~9t1 o (91:1 o9l:J)
5. Esto se sigue deJ principio del pal0mat. En este caso, las palomas son los 'Z"" + 1 enteros entre
Oy 2. inclusive, micntra1 que las palomas son las ?J relaciones sobre A..
7. 2n
9. Considttcmos d elemento de la i~ima fila y laj~sima columna de M(9t1 o 9tJ. Si este ele-
mento es 1, entonces existe b1 en B tal que l s k s n y (a.. bJ E 9t., Cho c1) e 9t2- En
eonsecuencia. el elcmentodelai-fila y l a k - columna de M(!ll,l cs 1, y el elemento
de la k-fila y_Hsima eolumoa deM/[11.) es 1. Esto prowceun I eo la i~ima fila y j-ésima
columna eo el producto M(!ll,) · Mf/I<,).
Si el elemento en la fila i y la colomnajdeM(!Jt1 o9t:) es O, entonces para cadab.t, tal que 1
S k S •• (a,. bJ ~ !JI, o (b0 e¡¡ ~ !JI~ Esto significa que en las matrices M(!ll,). M/[11,) si el
elemento de lai-áima fila y k-fflma columna de Mi$..) es l, entonces d demento de la .t-ésima
fila y j~ima columna deM(!ll,) es O. Por lo tanto, la enlnlda de lai-fila yj-ésimaeolumoa
de M(!ll,) · M(!ll,) cs O.
11. d) Sea,.,
el elemento de la fila (z) y columna (y) de M. Entonces•~ apan:ce en la fila (z) y la
columna(y)deM-.9tcsantisimttrica(:;;)(.r.Ql=s,r= 1 ⇒ x=y) c::::>M nM' S /~
13.

·~·w , ..

15. W9t-~~~~~~~~~4~~~4~~~~~~~~~4
(d,f),(f,á)):
(b) (e) (á) (•)

~
(a)
(a) o 1 o o 1
(b) 1 o 1 1 1
M(i!t)-(c) o 1 o o o
(á) o 1 o o 1
(•) 1 1 o 1 o
(f) o o o 1 o
Soluciones s-43

Para las partes (ü) y (iv), las filas y columnas de la matriz de relación esún indexadas
como en ta parte (i).
(ü) 91 ={(a, b),(b,,), (d,b), (d,c), (<,/)}:

1 o o o
o o o

·~•·[!
1
o o o o
1 1 o
o o o
o o o
o
o
o !]
(iv) 91 ={(b, a),(b, e), (e, b), (b, ,), (c,d), (<, d)}:

M(91) -
o o
1 o
o 1
o o
o o
o o
o
.. ']
1 O 1 O
o 1 O O
o o o o
o 1 O O
o o o o

17. Para cada u E V, sl la fila de u en M y la columna de u en M sólo tienen caos, entonces u es un


- aislado CD el grafo dirigido G.

19. 9 1 ~ 91'~

~ 4

21. a) ~ b) 2"

23. a) 91,: ¡: : ~ ~ ~1 ¡: 91,: ~


O O 1 1 O 1 1 1 O O
O O l 1 O O O O 1
o o o O I O O O 1

b) Dada una relac:ido de equivalencia 9t sobre un conjunto fmito A. enumeramos los elcme:n•
tos de A de modo que los elementos"' la muma celda de la partición (vtasc la See. 7.4)
sean adyacentes. La matriz. de relación resultante tcodrí entonces bloqUC$ cuadrados de
unos a lo largo de la diagonal (del extremo superior izquierdo al extremo inferior derecho).
25. (s,) a,. 1:
(s,) b e• 2:
(s3 ) a:"" a +3;
(s,) e:• b:
(s,) 1:• 2•a-l;
(s,) be• a •e:
(s,) e,. 7:
(s,) de • c+2:
5-44 Soluciones

Sección 7.3- l.
pág. 380

3. Para todo a E A. b E B. tenemos que a91, a y b!ll, b, de modo que (a, b)91 (a, b)y 91 es
reflexiva. (a. b)9!: (e, d). (e, 4)91: (a. b) =>a9!:1 c,c9t1 a y b9iid, d9!:2 b =>a= e, b = d ⇒
=
(a, b) (e, d), por lo quc91 es aotisi_,ca. (a, b) 91 (e, d), (e, d )91 (<,f> ~ a91, e, c91,, y
b9t2 d. d9l,f=> a9't1 e, b~d=> (a. b)9!: (~/). y esto implicaque9'1: es transitiva
S. f< {I} < {2} < (3} < {!, 2} < {l,3} < {2,3) < {l, 2,3}
(Existen ottas p0$lbilidadcs.)

7. a) / • b) 3<2<1<4 3<1<2<4 . e) 2

' 2
"-,/
U. Seanx, y.supremos. E.ntoocesx91:y, pues yes una cotasuperimyzes un supremo. Dela misma
forma. y 91: .:c. 9l antisimttrica ⇒ x =y. (La demostración para el inf es similar.)
13. Sean '11= {l, 2), A =ll'~ y sea 91 la <dación de inclusión. Entonces (A,91) es un conjunto
parcialmeme onlenado pero no un onlcn total. Sea B = {f, { l}}. Entooccs (B x B) n 91 es un
orden tola!.
15. n ♦(;)
17. a) Los n elementos de A se ordenan a lo largo de una recta vertical. Si A = {ah ai. . .. , a.J,
donde a 1 9t ~ 9t a,91: • • · 91: ª•• podemos trazar el diagrama como sigue:

r·-
.i•••

b) n!
t~
1'. sup inf sup inf
•l {1,2)
• d) {l,2,3) {l}

•• ••
b) {1,2,3) t) {l,2,3)
e) {1,2) O {1,2,3)
21. Para cualquiera E Zse slgue quea9!a, puesa - a=O, un entero par no negativo. Por lo tanto,
9!: es ,efkxiva. Si a. b, e E Z ooo a 91: b y b 9!: e, entoooes
a-b = 2m, para a!glln m EN
b-c = 2n, para algw,n EN
y a- e= (a - b) + (b- e)= 2(m + n), donde m + n E N. Por lo tanto, a 91: e y 9't es transitiva.
Soluciones 5-45

Por llltimo, supongamos que a9tb y b9ta pan algunos a, b E Z. Eruooccs a - b y b - a soa
ambos enteros no negativos. Como esto sólo puede ocurrir para a - b = b - a • O, vemos que
[a9tb A b9ta] ⇒ a= b. por lo (11C9t es antisUMtrica.
En c:omccueoda. la rdaádo 91 es un «den parcial pan Z. Pm> ""es un «den toCaL Por
cj<mplo,2. 3 E Z y DO<>=reque2913 oí 3912,pue< - 1 y 100,,>1.,,.aos paes oonepli"°'-
13. ■) 2 b) 10
25. b) y e) En este caso el elemento mlnimo (y ilnico elemento minimal) es (O, O). El elemento
(2. 2) es el elemento mwmo (y ilnico elemento maximal).
d) (0,0)91 (O, 1)91(0, 2)91(1, 0)91(1, 1)91 (1,2)91(2,0)91(2, 1)91(2,2)
27. a) Falsa. Sean~ = { 1, 2), A = ¡,(-11), y 91 la rdaci6n de ínchw6n. Enlooccs (,1, 91) es un
mfculotalque panada S. TEA, ínf{S, T} • SU Tesup{{S. T} = S n T. Sill embargo,
{ 1} y {2} DO CSWl relaciol1ados, por lo que (,1, 91) DO es UD «den total.
29. ■)a b)a c)c d) e e)z ll• l)v
(A, 9U es un retículo donde t es d elemento mbimo (y ónico elemento maximal) y a d ele•
mento mlnisDO (y ilnico elemenro minimal).

;«ción7.4-- t. a) & este caso, la colección A1, Az- A, proporciona una partici6o de A.
~g. 387 b) AunqueA=-=A1 U A,; UA, UA4 , tcnemosA 1 nA_i: # t.de modoquelacolecciónA1,ArA1 ,
A, no proporciona una panic:i6o de A
e) La colecci6oA 1, A.21 A, proporciona una partición de A.
3. 91 = ((1, 1), (1 , 2). (2. 1). (2. 2), (3, 3), (3, 4), (4, 3). (4, 4), (S. S)}
5. 91 DO es transitiva pue< 1 912 y 2 91 3 pc<O I si 3.
7. a) Paratodo"-y) EA,x+y=x + y=>(,.y)9t(z.y).
(,r,,Yt)9t C,t,.,yi) =xi + :,.•.ti+ 11⇒ x2+ )'i•X1 +y1 ⇒ (.r2,Yi)911: (x1oyJ.
(%¡,y.)9t~y,J.(rl!o-,.J9t(x,.y.J ⇒ .r1+)'t•X2+h,X:+)'-z:z.t,+'h,px'loc:,Je.r1 +Y1 =.X3+Y,y
ex,, yJ!ll «~JJ. Cano91 es rdlexiY3. simtlrica y trllDSitiY3. es uoardaádo deequivalmcia.
b) ((1, 3)] • {(1,3), (2, 2),(3, I)}; ((2,4)] • {(I, S), (2,4), (3, 3), (4, 2), (5, 1))
((!, I)] • {(I, I)}
e) A -{(I, l)}U{(l,2), (2, l)}U{(l,3),(2,2),(3,l)}U{(l,4),(2,3), (3, 2),(4, l)}U
{(I, S), (2, 4), (3,3), (4, 2), (5, 1)) U{(2, S), (3, 4), (4,3), (5,2)} u
{(3, S), (4, 4), (5, 3)1 U {(4, S), (S, 4)) U {(S, S)}
,. a) Para todo (a. b) E A, tenemos que ab = ab, de modo que (a. b) 91 (a. b) y 91 es rdlexiV>.
Para.,., que 91 es simttrica, supongamos que (a. b), (c. d) E A y que (a, b) 91 (c. d).
Entonces, (a. b)9t(c. d) =>ad=bc =>cb•da => (e, d)9t (a, b), po,-lo que9tes simlaica.
Porllltimo,sean (a. b), (c. d),(,./) EA tales que(a. b)!ll (c. d) y(c, d )!ll (,./). Entonces
(a. b)91 (e, d) =>od•bcy (c. d)9t(,./) => c/=de,demodoqueadf• bcf• bdeycomo
d ,; O, tenesDOSa/= be. Pm>af= be=> (a, b)!ll(,,/) y enc:omccueoda.91 es transitiV>.
Se sigue de loanteriorque9t a una relación de equivalencia sobre.A..
b) ((2, 14)] • {(2, 14)}
((-3, -9)] • {(-3, -9),(- 1, -3),(4, U)}
((4,8)] • {(-2,-4),(1,2),(3,6),(4,8))
e) Existen cinco o,ldas en la partición; de bocho,

A• ((-4, -20)] U ((-3, -9)) U ((-2, -4)] U ((-1, - 11)] U[(2, 14)].

ll. a) P:ancadaXC A, tenemos B n X= B n X, por loqueX!ll Xy91 es reflexivL s; X,YC A,


entoncesXa Y~Xn Ba YnB~ Yn B= X nB:-> Y9tX,dcmodoque9tcssim&rica.
Porllltimo,si W, X, YCAcoo W91Xy X91 Y.entonces Wn B•XnByXnBz YnB.
Por lo tanto. w n B = rn B, W9iYy91: es transitivL En consecuencia,9t es una relación
de equivalencia-¡,(,!).
b) {f,{3}}U{{l},{l,3}}U{{2},{2,3}}U {{!,2},{1,2,3))
e) [X]= {{i ,3},{l,3,4},{l ,3,5},{1,3,4,5}}
d ) 8, uno por cada subconjunto de B
13. a) (!) (:) b) 4(:) e) 2(!)
d) (!) (:) + 4(:) + 2(:) + (:) + (:)
15. 300
17. Sea {A,}.vuna partición de un conjunto A. Definimos 91: en A como x9'l y si para algwl i E /
tenemos .x;. y E A.,. Para cualquier x E A, x, x E A. para algiln i E ,. por lo que x g¡ x y 91 es
reflexiva. z9i:y ⇒ z, y E A,. paraalgd.n i E/ ⇒~ X E A¡ paraalgúni E J ⇒y9tz.de modo
que 9l es simftrica. Si x 9t y y y 9t z. entonces x, y E A. y~ z E A1 para i, j E /. Como A. n A1
contiene ay, por el teorema 7.6(c). se siguequeA.=A1, porloqaci =J. Por lo tanto• ..r. z e A;.
de modo que x 9i z y 9i es uansitiva.

Sección 75 -- l. a} s1 y s5 son equivalentes. b) s1 y s 5 son equivalentes.


pág. 393 e) s1 y s1 son equivalentes; s_, y s, son equivalentes.
d) s1 y s1 son equivalentes; s. y s, son equivalentes.
b) (i) O O O O
(ü)
(iü)
o
oo M, o
. o..
1 1
s, s, s, 1 o
s, s, s, 1 o
s, ... s, 1 o
s. s, s. o o
... s, s, 1 o

Ejercicios 1. a) Falsa. Sean A = {l, 2}, Y= {l. 2}, 91, = {(l. l)} y 91,= {(2, 2)}. F.ntonce.UiEl91, es
complementarios- reflexiva. pero ni9i1 ni9l2 lo son. Sin embargo, recíprocamente. si91: 1 es reflexiva para todo
pág.396 (en realidad. para al menos un) i E /, entonces UiEl~res reflexiva.
3. (a. e) E 91:209i:1 ⇒ para algán b EA, (a, h) E 9b (b, e) E9i1. Como9!:i,91:2son simétricas,
(b, a) E9!:1, (c. b) E911, demodoque(c, a) E911o912 ~91:20911• (e, a) E 9t2o9i:1=> (c. d) E
91,, (d, a) E 91, para alg,ln d E A. Entonces (d, e) E 91s (a. d) E 91, por simctrla y (a.e) E 91,
0911, de modo que911 2 o9/: 1 ~ 91 1091:2 y se sigue el resultado.
S. (e, a ) E (91:1 091:,JCc=-(a, e) E 91:1 o 9h ~ (a. b) E 91:1, (b, e) E91:2 para algún b E B e=- (b. a)
E ~. (c. b) E~paraalgúnb E Bc=-(c, a) E~ 091:f.
7. Sean~= ( !, 2. 3. 4. S}. A = il'~ - (~. f}. En la ,elación de inclusión, A es un conjunto
parcialmente ordenado con cinco elementos minimales {x}. J :S x :S S. pero sin elemento
mínimo. Además,A. tiene cinco elementos maximales, los cinco subconjuntos de~ de tamaño
4, pero no tiene elemento máximo.
9. •= 10
11. a) Paracualqwer/E',, 1/(n)I ,< 11/(n)I paratodona, I.demodoque/91/y91esretlexiva.
F.n segundo lugar. sif.g E'3. entoo=/91g ""' (/E O(g) y g E 0(/)) ""'(g E 0(/) y/E
O(g)) ""'g91f. de modoque 91es simétrica. Por último, =nf. g. h E', con/!ill g,g91f.g
91: h y h 91 g. Entonces existen mi, m2 E R" y ki, k2 E z· tales que 1/(n) 1 :S m1 1g(n) 1 para
todon 2: k1 Y lg(n)I S m1lh<n)I para todon ~ k 1• En consecuencia.para todon 2: máx{k1♦
k,}, tenemos 1/(n)I s m,lg(n)I S m, m,lh(n)I, por lo que/E O(h). De manera análoga,
h E O(J). Así,f91: h y 91 es transitiva
b) Para cualqujer/E',.fesdominadaporsím.isma,.de modoquel/]9' [/] y9' es reflexiva.
F.n segundo lugar.,; (g}, [h] E '3'. con (g] 3' [h} y [h] 3' (g], entonces g 91 h. como en la
pane (a), y (g] = [h}. F.n consecuencia, 3' es antis;métrica. Por último, si[/], (g], [h] E '3 ',
con[/] 3' (g] y [g}3' [h}, entonces/es domfaada por g y ges dominada por h. Así. como
en la parte {a)./es dominada por h y [/] 9' (h), lo que hace de 9' una relación transitiva
-- -
Soluciones $-47

Lista de Ustad< Listad< Lista d< Listad< Ustade


13. a) b) e)
ad}'ll(eocia h>dku Índl= 1odlcos
1 2 1 1 1 2 1 1 1 2 1 1
2 3 2 2 2 3 2 2 2 3 2 2
3 1 3 3 3 1 3 3 3 1 3 3
4 4 4 5 4 5 4 4 4 4 4 6
5 5 5 6 5 4 5 5 5 5 5 7
6 3 6 8 6 6 6 1 6 8
7 s 7 4

15. b) Las celdas de La partición son Las component,. conexas de G.


17. Un orden posible es 10. 3. 8. 6. 7. 9. l . 4, 5, :Z. donde el prog,,una 10 se ejecuta primero y el
prog,,una 2 al llltimo.
19. a) IAI =2'=32
e) Existen seis clases de equivalencia. una por cada uno de los pesos O, 1, 2. 3, 4 y 5. El
mlmero de elementos en cada clase de equivalencia est.i dado como sigue; peso O, ( ~);
pesol, (/); peso:Z. (n;peso3, (l};peso4, (!);y peso 5, n).
d) Reemplazamos S por n, donde n E z•. Entonces existen n + 1 clases de equivalencia. una
porcada uno delos pesos O, 1, 2, .. . ,n. Si O :s: k :s n, entonces existen(:) elementos de
A en la clase de equivalencia de peso k. ·
21. 4• - 2(3") + 2"
23. •l (i) B 9!A 9!C; (ü) B 9!C9! F
B 9't A 9t C 9l F es una cadena maximal. Existen seis de estas cadenas maximalcs.
b) En este caso, 119' 385 es una cadena maximal de longitud :Z. mientras que 2 9' 691 l2 es de
Joogirud 3. La longitud más grande de una cadena para este conjunto parcialmente ordena-
do es 3.
e) (i) 8 <;;{1}<;; {1, 2}<;;{1, 2,3),;; <lt (ü) 8 <;; {2}<;;{2,3}<;;{1, 2, 3) !:'lt
Hay 4! ::: 24 de tales cadenas maximalcs.
d) n!
25. Sea a 1 9't a 1 91: • .• 9i: a...i 91 ª• una cadena (maximal) de máxima longitud en (A, 91!). Entooces
a. es un elemento maximal en (A, 9t) ya1 91: a 2 9't . .. 91:a..1 es una cadena mawnal en (8, 9t').
Por lo tanto, la longitud de una cadena más grande en (B, 91:') es al menos n - 1. Si existe una
cadenab1 91:' b,91:' • • • 91:' b. en (8, gr) de longitud n, entonces ésta es tambi~ una cadena de
longitudn en (A.,!?)?). Pero entonces, b. debe ser un elemento marimal de (A. 9t), lo que contr.a-
dicc que b, E B.
27. Sin= l,entonccs para cualquier x, y EA. si x el y, cntooces.r~ y y yf;. x. Podo tanto, (A, 9ti)
es una anticadeoa, y se sjgue el resultado. Supongamos ahora que el resultado es cierto paran : k 2:
1 y sea (A, 91) un conjunto parcialmeDle ordenado tal que La lqtud de una cadena más grande es
k+ l. SiM es el conjunto de todos losclemen!os maximalcs de (A, 91), eoronccs M ~ 8 y M es una
anlicadena.., (A. 91). Además. por el eje,cicio 25. CA - M. 9''). con9'' = ((A - M) x (A-M)) n 9',
es un coojunto parcialmente oroenado tal que k es La longitud de una cadena de longitud máxima.
Así, por la hipótesis de inducción. A. - M = C1 U C2 U · · · U C¡,. una partición en k anticadenas.
En ~ a , A : C1 U C1 U • · · U Ct U M , una partición en k + 1 anticadenas.

C..pltulo 8
El principio de Inclusión
yuclusl6n

Sección 8.1- l. a) 534 b) 458 e) 76


~- 412 3. 4,<lé0,400 s. cm - <n<m + G>CD- G)(l)
7. (IS!)[(::) - G)(':) + n}(:)] ,. 26! - (3(23!) + 24!) + (20! + 21!)
Soluciones

11. [6'- (1)5' + (04' - (,'}3' + (l)2' - (:}]/6'


13. 9!/((3!)'] - 3{7!/[(3!)'JJ + 3(5!/3!) - 3!
15. 651/7776 =0.08372
17. a) 32 b) 96 e) 3200
19. a) z--• b) z--•(p - 1)
21. a) 1600 b) 4399
23. «17) = «32) = «48) = 16
25. Si 4 divide a ~n), entonces debe cumplirse uno de los casos siguientes:
(1) n es divisible entre 8;
(2) n es divisible entre dos (o más) primos impares distintos;
(3) n es divisible entre un primo impar p (como 5, 13 y 17), donde 4 divide ap-1; o bien,
(4) n es divisible entre 4 (y no 8) y al menos un primo impar.
27. Si un ruimeron es divisible entre un primop, laejocucióndel ciclo While separa cadaocurreo-
=
cia de p. El primer ciclo es para p 2, el siguienlC para p = 3 y el siguiente ciclo es para los
primosp > 3.
=
Al trabajar con primos. una vez que pasamos de p 5, no tiene sentido analizar los enteros
pares como candidatos posibles a ser primos.

Sección 8.2- =
l . Eo 768; E, & 205; E,= 40; E,= 10; E,= O; E,= l. ~E,= 1024 N. =
pág. 417 3. a) [14!/(2!)'] - (:}[13!/ (2!)'] + (l){l2!/(2!)'] - (l)[ll!/(2!)'] + (l}[I0!/2!] - (l)[9!]
b) E,= (l)[l2!/ (2!)'] - (n(l)[ll!/(2!)'] + (l)(:)(10!/2!] - (l)(l}[9!]
e) L, =(l)[ll!/ (2!)']- G)(l)[l0!/2!] + (l)(l)[9!]
s. E,= 6132; L, = 6136
7. a) !l:/-.(-1)1(:}("ü"')]/(ll) b) ~-,(-lr'(i)(:)("ü"')]/(ll)
ci 1mcm -3C:>cm1,cm

Sección 8.3- l. 10! - (l)9! + (l)s! - (l)7! + (l}6! - (l)5! 3. 44


pág. 419 S. a) 7! -d, (d, =(7!)<- ') b) d,. =(26!)<- '
1. n = 11 9. (lO!)d., • (10!)'(,-')
11. a) (d,0) ' =(10!)' ,-, b) ~ ~(-1)'(':')[(10 - i)!f
13. Para cada n E Z♦• n! cuenta el total de permutaciones del , 2. 3•... , n. Cada una de estas
permutaciones tendrák elementos desordenados (es decir, k clementosx1,x 2, •• ,X!: en (1, 2, 3,
...• n} tales quex1 tU>estáen laposiciónz1,XJnocstácn la posición X?, ••• , y Zt1W~enla
posiciónxJ y n -!elementos fijos {es decir, los n -k clementos Yto Yll ... ,Yw. en {l, 2. 3, .. .•
y,_
n) - {z1, ZJ. •. . ,xt} $00 talesquey1 está en laposiciónyi,y2 está en la posición y:- . • . ,
está en la posición y-).
Los n - k elementos fijos se pueden elegir de (.,:t)
formas y los k elementos rcstaotes se
pueden permutar entonces {es decir, desordenar) de t4 formas. Por lo tanto, existen ( • .:,Jd.
=
(; )d. permutaciones de 1, 2. 3, ...• n con n -k elementos fijos {y k elementos desordenados).
CUandokvañadeOan,contamostodaslasn! permutaciones de 1, 2, 3, . .. , nde acuerdocoo
el número k de elementos desordenados.
En consecuencia,

n! = mdo + (;)d, + (;)d, + · · · + (:)d. J, (;)d,.


&

IS. (!)(n- 1)! - (;)(n - 2)! + (¡)(n - 3)! - · · · + (-1r'(.~,)(0!) + (-1)"(:)


Soluciones S-49

ecciones 8.4 y
!.S-pág. 426 3. •> O+ (jar+ C}<s-7).t' + C)<S·7·6)r' + {j<s•7-6·S)x' + • •. + {!)(s!)r' =
! (j P(s. ,)x'

b) i (~)P(n,i)x'
, _. 1

S. a) (i) (!+2r)' (ü) 1+ 8r + 14x'+<lr'


(üf) 1 +9.r + 2Sr+2Lr' (iv) 1 +Sr+ 16r + 7r'
b) Si d tablero e consta den pasos y cada paso tiene t bloques, cnrooces
,(C,x) = (1 + .b:)".
7. 5! -8(4!) + 21(3!)-20(2!) + 6(1!) • 20
,. a) 20 b) 3/ 10
11, (6!/ 2!)-9(5!/ 2!) +27(4!/ 2!)-31(3!/ 2!) + 12 • 63

:'.jffcicios 1, 134
;omplementarios-
¡,ág. 430 3. ((24!)/(6!)'1[ G!)-(1)G~) (~)Wl
+
s. ~ (- !y0)(8- i)!
7. 9! - (l)(2)(8!) + (l)(2')(7!) - (l)(2')(6!) + (l)(Z')(S!) - (l)(2')(4!)
9. Sea T • (13!)/(2!)' .
a) ([(!)(10!)/ (2!)'] - [(:)(l){9!)/ (2!)] + [(l)(!)(8!)D/ T
b) [T- (E, + E,)YT,
donde E, • [(l)(9!)/ (2!)] - [(!)(l)(8!)] Y E, • (!)(8!)
U . a) (;.::)
13. a) Si ncs par.cotooc:cs. poccl teorema fundamen1al dela aritmética(reorcma. 4.11). podemos
escribirn s :itm, dondek ~ 1 y mes impar. Entonces 2n • ~'my~2n)= (?•'X l- f)9(m)
= = = =
2'9(m) 2(2'X½>«m) 212'{1- t>+<mn 2[+(2'111)) 2+(n). =
b) Cuar>:lo • es impar, tenemos que +<2n) = (2n)(l-½) íl,1,(1 -~) , donde d producto se

toma$0bretodoslos primos (impares)que dividenan. (Si n s I. cntooces n~(1 -.!.) es


~ - p

(2n)(l - l)Il,.. (1 -;)=•Il,.. (1-;)-~•~

Capítulo 9
Fundones generatrices

sección 9 .1- l . a) El c:ocficientedcr'cn(l+..r+.r + •· ·+..r')'.


~ - 435 b) Eloodicicncdc..t»co(l + x+:t2+ ·· · +.r'ff} +.t'+x'+ ·· · +r'jo(I +.r+.r2+ ·. -1(1 +r+
.r+ •. ·)'.
e) El coeficienredcx"cn (x'+x'+.r)(.r+.r+ • . . + x''f.
d) ElcocficientcdcrDcn(l +.x + zl+ · · · +.x•)'(l +.t'+.r'+ .. +x'°}-{x + .r'+.r'+ · .. +x8)
o (1 + .r+.r1+ · · ·)3(J +.t'+x• + · · -)-(z+x'+.C+ · • •).
3. a) El coeficicmcdcx10 cn (1 +x +xl+x" + •• •Y,.
b) El coeficientedc.r' en (1 + ..r + .xl+x'+ · · · )".
5. La respuesta es d coeficiente de x'1 CD la función generatriz

(1 +x +x2 + x' + · ··)'(l + x +xl + ... + .r 1°).


S-50

l . a) (l+x)' b) S(l+x)' e) (l+xr' d) .r'/(1-x)


•l 6.r'/(l+x) O (1-,')-• &) (1-2")- ' b) .r/(1 - ,u)
3. a) g(x) •/(x)-a,,' +lr'•f(x)+(3 - a,)r'
b) g(x) • f(x)+(3-a,)r'+(7-a,)r'
e) g(x) • :1/(x) + (1 - 2a,)r + (3 - 2a,)r'
d) g(x) • 7/(x) + [5/(1- x)J + (1 -2a, -S)r + (3-2a,-s)r' + (7-2a,- S)r'
5. a) ('l) b) (";')
1. (l:) - 5(t)+(l)
,. a) O b) (l:) - 5(l:) <) (ll) + 4(U) + 6(:;) + 4(g} + (t:)
u . e;¡ - 4(l:) + 6(ll)
13. [G:) -('OOl) + ('t)ri) - ()'))/(6"}
15. (1/8){1 + (-1)1 + (1/4)(":') + (1/2)(":')
11. (!-, -.r-.r' - ,"-,'-,'J-• -11 -(, + .r2+ · ·· + x'W'
-1 + (% +x'+ · · · +x') +(x +.r2+ · · · + .r')'+(x +.r2+ · · · + x')'+ · · ·,
Ulliltudl clostitadb trts~

donde d 1 se cncuga del caso c:o que no se tira el dado.


t,. a) Las diferencias son 2, 3, 2, 7 y O, lo que suma 14.
b) {3,5,8,15} e) {l+a,l+a+b,l+a+b+c,l+o+b+c+d}
21. C.t=IJ-oi(k - i'f=k2'IJ.oi -2kl:!.oi2 + ~-oi3
- (k'){k(k + 1)/2J-2k(k(k + 1)(2k + 1)/61 + [k'(k + 1)2/4J
• (1/12)(k')(k'-1)
23. a) (1 +x +r+x'+x')(O+x +2.r+lr'+ ···)-~c,.r' donde eo•O, e, ~!,
c2 • l +2•3, c, • 1 +2 +3s6, c.•1 +2+3+4 • 10, Y
c.• n + (n -1) + (n -2) + (n -3) + (n - 4) • Sn - 10 para todo n "5.
b) (1-x+r-x'+ ···XI - x+r- r'+ · · ·) •
0
:,'j' •(1 +x)"",la función gencralriz dela

sucesión (-.'W.'W.'). (-,'), .... Por lo tanto, laconvolución del par dado de ,ucesiooos
ese,, c.,c~ ... ,doodcc,•(-.') :(-1>"('7') : (- 1)(";1)s (-l)"(n + 1),n EN. [ é.raes
la ,ucesión a1temada l. -2. 3, -4, 5, -6. 7, . .. J
e) (1 +x+r+.r')(O + x+2x'+lr'):O+x+(l +2)r+(l +2+3)r+ (1 +2+3)<'+(2+3),'+
3%', de modo que la ,ucesión o, l . 3. 6, 6. 5, 3. O, O. o.... es la coavoluci6n de lu
'1JC<Si-.1, 1, 1, 1,0.0,0, ... yo, 1,2,3.o.o.o•...

Sección 9.3- l. 7; 6+ l ; 5+2; 5+! +!; 4+3;4+2 +1; 4+1 + 1 + ! ; 3+3+ ! ;


~ . 448 3+2+2; 3+2+1+ !; 3+ 1 + 1 + 1 + !; 2+2+2+ !;
2+2 +1+ 1+1;2 +1+1+1+1+1; 1+1+1+1+1+1+1
3. El nwnero de particiooes de 6 en 11DOS. doses y u..es es 7.
5. a)yb)

(1 + r2 + x• + x' + · · ·)(1 + x• +x*+ · · ·)(l + x'+xu + · · ·)· · · -n- 1


1-r5

7. S<a/1,r) la función ga,cratriz del m!mcro de particiones den e z · tales que un ""1W>do no
aparece mú de dos veces. Elllonces

,.,
/(x)•íl(l+.r'+x").
Sea g(,) la funcl6n ga,cratriz del námero de particiooes de n tales que niD¡\lD =--- es
divisible eorre 3. En este caso,
1 __ _2__ ..
g(x)•-1_ __ 1__ _ 1_ __
1-x l-x2 1-x• 1-x' 1 - x'
Soluciones S-51

Pero
/(x) = (1 + x + x')(l +.r' +x')(l +x' + x' )(l +x' + r') · • •
1- .C 1 - x' 1-x' 1 - x 11
1- x 1- x2 1-x3 1-x•
1 1 1 1 1
-1-x l -x1 1 - x• 1 - x' 1 - x 1
=g(x~
9. Este resultado se sigue de la correspondencia uno a uno entre los grafos de Ferrcr con sumandos
(lilas) que oo soc superion:s a m y los grafos uaspucslos (que wnbitn son grafos de Fcm,,)
que tienen m sumandos (filas).

Secci6n9.4-
l. •> ,-· b) ,,, <) , - d) ,.,. •>ar o uh
3. o) g(x) • /(x) + ((3 - a,)/3!}x'
~9- 453 b) g(x) • /{z) + ((-1- a,)/3!).r' =•'" - (126r'/ (3!))
e) g(z) • 2f(z) + )2-24,)z + ((4-24,)/2!).r'
''
5. o) (1+z)'(1 + x+~}
2

b) (l+x)(t+x+~(t+z+~+~+:._\
2) 2 3! 4!}

~
1
7. La respuesta es el coeficiente de ? en (~ + + ·· ·+ x º)'.
2.S! 3! 4! 10!
f. o) (1/2)(3"'+ 1)/(3") b) (1/4)(3" + 3]/(3") <) (1/2)(3"- l]/(3")
d) (1/2)(3"'- 1]/(3") e) (1/2)(3'° + 1)/ (3")

u.•> (~ +~+---){fi+fi+···f
b) (x+~+~+···)'[ex>'~
2! 3!
+~+~+:._'\ +f::._)f~ +~+:._'\
U, 3! 4! s!J \2, \:J! 4! s!J
+ (~)(fi + ~) + (~)(~)]
Sección 9.S-
3. ac,, a1 - ao,a2 -ai,a.J-a2 , •••
~g. 455
5. f (~'.,),•• f ff a,-a.L=f ff.,-a.)x•
-- -~- r -~-
= .U!••k-.t-,.,.r" c /(z)/(1 -z)-f(z)
= [/(x) - /(x)(l -x)V(l -z) •.rf(z)/(1 - z)
7. /(x)=(e'/(1-z)]
Ejercicios
complementarios- l . o) 6/(1 - z) + 1/(1 - x)' b) 1/(1-ax) e) 1/(1- (1 + a)x]
d) 1/(1- x ) + 1/(1 - ax)
~9- 4S9
3. mn - mm mr +
5. Su./(x) la función generatriz. parad nárnero de particiones den E z• en las que no se repiten
los sumandos pares (un sumando impar puede o oo repetirse). Entonces
/(x)= ( 1 +x +x2 +x'+ · · ·)(l +x2)(1 +x'+x•+x' + · ·)(l +x•)· · ·
1 1
• ___l_·(l+z')----(I+z')•----·
1- x 1-x' 1-.r'
S-52 Soluciones

Seag(x) la función generatriz para el nllmero de particiones den e z• en las que no se repiten
los s.umandos más de tres v~s. EntODCCS
g(x) = (1 + x + x' + x')(l +x' +x'+ x' )(l + x'+ x' + x')- · ·
• ((1 +x)(l + x'))[(l + x')(l + x'))[(l +x')(l +x')] · · ·
= [(1 -x')/(1-x)](l + x')((l - x')/(1-x')](l + x') ·
((1- x')/(1 -r')](l + x') • • •
= (1/ (1- x))(l +x')(l/(1 -x'))(l + x')(l/(1-x'))(l + x') · · · = f(x~
7. a) l,S,(5)(7),(5)(7)(9),(5)(7)(9)(11), .. . b) a=4,b =-¡
9. n(2"-')
11. a) m b) (!}'/(~')

~pftulo10
Relaciones de recurr.ncia

Sección 10.1-
1. a) a.,=Sa..-1on~1,ao=2 b) a,,=-3a..-i.n.i::l,ao=6
pág. 470
<) a.= (l/3)a._,,n., 1,ao= 1 d) ••= (2/S)a.,_,,n2' 1,a.=7
3. d= %(3/7) S. 141 meses 7. a) 145 b) 4S
9. a) 21345 b) 52143, 52134 <) 21534, 21354, 21345
d) 21543eslal13, 35421esla67, 31524esla43.
Sección 10.2-
l. a) a,,=(3/ 7)(-1)"+(4/7)(6f,n2'0 b) a,,=4(1/2)"- 2(5)", n2'0
pág. 480 <) a,,=4+3(-1/3)",n 2'0 el) a,,=3sen(ni,/2),n2'0
•l a,,=2"[<:0S(n-./2)+(1/ 2)sen(ni,/2)), n2'0 O a.=(5-n)3",n2'0
g) •• = (V2)"(oos(3"n/ 4) + 4sen(3r.n/4)), • "' O
3. a,,=(1/IO)(T-(-3)"],n2'0
S. a) F, -¡;;- ¡¡,
F, =F.-&
Fs =Ft.-F.

Fz.._1,;, &-F..-2
Conj~tura: Para todo n E Z.. , F1 + F3 + F, + · · · + F~ = F211 + Fo= F'J,,.
Dmwstración.(porinducciónmatcnútica): Paran = t tenemosF1=Fz.. yes.toes cierto pues
F1 = 1 = F2- En con.sccucncia. el resultado es cierto en este primer caso (y esto establece la
base de la demostración), A continuación suponemos que el resultado es verdadero para
n = k(:?!: I); es decir, suponemos que

¡;¡ + F, + F, + .. . + &-, = F,,.


Si n = k + l tenemos que

F._+ fi + Fs + "• . + Fu- 1 + &:.t•l)-1


= (,<; + F, + F, + ... + &-,) + F,,., = F,, + F,,., = F,,., = ¡;¡,..,,.
Por lo tanto, la verdad del resultado paran implica la verdad paran= k+ l y por el principio
de inducción matenú.tic"a se sigue que para iodo n E z•
fj + fj + Fs + • • • + Fi..-1 = Fi..
1. a. = (1/VS)[((l + VS)/2¡-♦' -((l -VS)/2¡-♦'),n2'0
9• • • = 1<5 + .i1i1c2.i1>ll<3 + .i1>12r- 1<5 - .i1>1c2.i1>11<3 - .i1>12r
JI. •·= [(8 + 9V2)/16)[2 + 4V2r + ((8-9V2)/16][2- 4V2r. n "'º
Soluciones S-53

13. a.= 2'•, donde F. es el n<Simo m1mero de Fibonacci paran i!:. O


15. .r.•4(2")-3,n,:O
17. o.= Y51(4")- 35,n,:O
19. Como me<l(F,. Fo)= 1 = me<l(F,, FJ, consideremos n a,; 2. Entonocs
i.-F,+1\(-1)
F.-l;+F,
~-F.+~

F... 1 .;.F.+ F..-1


Si invertimos el orden de estas ec:uaciooes. tenemos los pasos del algoritmo de Euclides parad
cálculo del mcddeF..1 y F.,n 2:: 2. Como el 11JtimorcstodisiintodcceroesF1 = t ,scsigucquc
me<l(FM,. FJ = 1 para todo na,; 2.

Sección 10.3- l. a) o. -(n+I)', n,:O b) o.•3+n(n-l)', n>eO


p.lg. 492 e) o. -6(2")-5, •"'O d) o.=2"+n(2"-'), n >eO
3. a) o. • a• • ,+ n, • "'1, .. -1 o.=l +(n(n+l)]/2, n2'0
b) b.•b•• ,+2, na,2, 1,,-2 b.•2n, n2cl, bo- 1
5. a) o.•(3/4)(-1)"-(4/S)(-2)"+(1/20)(3)", n >c O
b) o.• (2/9)(-2)"-(5/6)(n)(-2)"+ (7/9), n>eO
e) o.• (S/4) - (1/4)(- 1)"- (l/2)scn(n,r/2)
7. a.=A + Bn + Cn' - (3/4)n' + (5/24)n'
,. P= $117.68
. 11- a) a.• ((3/4)(3)"-5(2)"+(7n/2)+(21/4)J"', n>eO
b) a,,• (1/2)((-1)"+ l)n!, n,:O
e) a..-2, n..!:0

Secóón 10.4- 1, a) a,,• (1/2)(1 + 3"), n2cO b) a,,= 1 + (n(n -1)(2n - l)J/6, n;,,O
p.lg. 499 e) a,, • (1/2)(3"+5"], n 2cO d) a. • 5(2")-4, n "'º
e) a.• 2", ""'º
3. a) o.•2"(1-2n), b.•n(2""'), n2cO
b) a.• (-3/4) + (1/2)(n + 1) + (1/4)(3"),
b.• (3/4) + (1/2)(n + 1)-(1/4)(3"), n >eO
l. b, • (8!)/((5!)(4!)] • 14
Sección 10.5-
p.lg. 509
1./-<:-I<<.~
A'\'1\>i<
3. 2n - 1) (2n - 1) [ (2n - 1)!] [ (2n - !)! ]
( n - n - 2 = n!(n - !)! - (n - 2)!(n + !)!
(2n - l)!(n + I)] [(2n - l)!(n - !)]
• [ (n + l)!(n - !)! - (n - l)!(n + !)!
(2n - I)! ]
= [ (n + l)!(n - !)! ((n + 1) - (n. - !)]
(2n - 1)!(2) (2n - 1)!(2n) (2n)!
(n + l)!(n - !)! (n + l)!n! (n + l)(n!)(n!)

= (n ! !)(~)
5. a) (1/9X':) b) [(1/4)(;)]'
e) [(l/6)('l')l[(l/3)(l}] d) (1/6)(',")
7.
'

-/'
1

.~-
.. '

0'
1
'

-~·
'
-+-+
~

.
"' '

;<· '
1

-+-+
~-•.
,. a.. •aoa.-1 + a1a..- 2 + a:za,._, + · · · + O..-:z0:1 + 0..-10o
Comoac,= 1, a 1= 1, a2 = 2 y a,• 5, tenemos que a. es igual al n-áimo mlmcrodeCatalan.

Sección 10.6- L a) / (n) • (513X4n"'~-1) y /E O(.-,.¡ paran E {31li EN)


p.ig. 520 b) /(n) = 7()og,n + 1) y/E O(log,n) paran E {S'li EN)
3. a) /E O(log.n) sobR: {btlk E NJ b) /E O(~ sobre {b'lk E NJ
5. •l /(1) = o/(•l • 2/(n/2)+ 1
Del ej=icio 2(b),/(II) • n - l.
b) La ecuacióo/(n) =/(n/2) + (n/2) se-... de la mancn si¡uiente; scjucpn n/2eocueo-
aos"' la primera ronda. Dcsputs rcsw, n/2 jugadores, de modo que occesiwnos f(n/2)
encuentros acücionalC$ para dctaminar al ganadoc.

7. 0(1)
9. •l /(n)saf(n/ b) + en
af(n/ b) s rrf(n/b') + «(n/ b)
rrf(n/b') s rr'f(n/b') + rr'c(n/ b')
a' f(n /b') :' a'f(n/b') + a' c(n/b')

• •-
1
s
f(n/b'-') a'f(n/ b") .¡. r• c(n/b•-•¡

l'oclo Wllo,/(n) s a'.f(nlb,t) +at{I +(o/b) +(a/b)'+· .. + (a/bJ""J=a'/(l)+cn{I + (a/b) +


(alb'r + · · · + (o/b)"'),yaquen =b'. Como/(1) S cy(nl/1) • 1,tenemooque/(n) s cn{I +
(alb) + (alb')' + · · · + (alb't" + (alb)'J = (en) "¿,_.(a/bY,
Soluciones S-55

e) Pallla p b.

~ [1-(a/ bf.. ]
en{:;:. (a/bY • en l - (a/b)

• (cXb'J[-1-_ (a_/b_f_••¡ . c[-b'_


- ~(a'_•_'lb~)]
1 - (a/b) 1 - (a/ b)
• c(-b'• •_-a'_•· ¡. c(-a'·•_-b_•••¡.
b - o. a-b

d) Dela ¡,ane(c)./(n) :5 (cl(a - b))(.,.., -b' J= (cal(a - b))a'- (cbl(a - b)'¡b'. Pt:ro a' =,.....•
4

,;-, y b'• n, de modoque/(n) :5 (cal(a- b))n..,.-(cbl(a-b))n.


(i) Cuando a < b, entonces log.a < 1 y/ E O{n) sobre z ·.
(ü) CUandoa > b, eotooces log,a> 1 y /E O(d°<') soorc z·.

f jerdáos
complementarios- l. ~:1)• (k+l)!(:!-k-l)!•~:::; ·k!(n•~k)!•~::)C)
pág. 524 3. Debemos considerar dos casos. Caso l (1 es un sumando): existen p(n - 1, k - 1) formas de
separar n - 1 en exacumentek- 1 sumandos. Caso 2 ( 1 DO es un sUIIWldo) : cada sumandos1,
si- . . . , sl> 1. Para 1 ::!i i ::!i k.Jea r,= s,- 1:?:: l. Entooccs 11,tz-· . . , rtpropordonan una
partición de n - ten exacwncote k sumandos. Estos casos son exhaustivos y disjuntos, de
modo que por la regla de la suma,p(n,k) • p(n - 1. k - 1) + p(n - k, k).

5. • )

b) Conj,nuu:Palllnez·.A·• [: ... l ]. doodeF,denotad•-n4mcrodeFibooacci.


. • [1-:) J.
Demostroci6n: Paran = l.A • A1= 1 0 = [F. F.]
fo ,demodoqueelresultadoescicrtoen
este caso. Supoop.mos que el resultado es verdadero para n = k ~ I ; es decir, At :a

[1••.....
~ ].Paran=k+I,

A"• A••• =A'·A=[F,.,F11.


F, ][1 1]
F1r-1 1 O
• fF,.,+ F, F,.,¡ . [F,., F,.,¡
LF. + F,,,_, Ft F.,.. Ft. ¡

En consecuencia. el resultado es verdadero para todo n E z•, por el principio de ioduccióc


matemática.
7. a) Comoal•a + l , se sigue que al+ 1 =2 +ay (2 + a)2 =4+ 4a: +a2• 4(1 +a)+ (re Sa2•
b) Como P'= P+ l. tenemos que P'+ I = P+ 2 y (2 +P>'=4 + 4jl +P'=4(1+ P> + P'= sp•
S.56 Soluciones

,. ( 2n)
e)•~ k F21t..,. ... -!
- ,. (2n}[ª"'" - p"··i
k a. - p

= (1/(a-p))L! i
(t} (a')•~ - (t} (p')'p•]
• (1/(a - p))[~(l + a')"' - P"(l + p')"')
= (1/(a - P))[~(2 + a)"' - P"(2 + P)"')
= (1/(a - p))[a"((2 + a )'}" - 1!"'((2 + P)'l1
• (1/(a - P))(a"(Sa'}"- jl"'(Sp'}"]
= 5"(1/(a - P))[a"'•• - p",."] = S"F,.••
9 . Aquftcoemosquec...i +c..,, +F. = (F,F.,_, +F2F-:_+F~w+· ··+F..,, F2 + F.,.1 F,) +(F1 F,..:z +
FiF.,.3 +F~-• · · · + F..,.1 F 2 + F JJ + F. =F1(F...1 +F.,_:)+ F-1.,F..,, +F..,V + F-J•.F..1 + F-) +
· · · + F...:1..F:+ FJ + F.., F 1 +F,.= F 1F.+ F2 F.,_1 + F3 F..:i+ · · · + F-:F, + F~ F2 +F.F1, pues
F2= F,= l. F.n consecuencia,
c.-,+c..- 2 +F,.=c., para no:!::3.

11. a) Para cualquier desorden, 1 se coloca en la posición i. donde 2 :S i :S n. Pueden ocurrir dos
cosas: Caso 1 (i está en la posición 1): los otrosn- 2 enteros se desordenan ded,....2 formas.
Con n -1 opciones para i, esto produce (n- l)d_2 de tales desón1enes. Caso 2 [i no está en
la posición 1 (o la posición,)}: consideremos 1 como la nueva posición natural para i, de
modo que hay n - 1 elementos por desordenar. C.oo n - 1 opciones para i. tenemos (n - l)d,,..
desónienes. Puesto que los dos casos son exhaustivos y disjuntos, el resultado se sigue de la
regla de la suma.
b) do= 1 e) d. - nd•• , = d•. , - (n - 2)d•. ,
13. a) a. =(':), na:O b) r=l, s=-4, t= - l / 2
d) b. = (1/(2n -1))(':), na: 1; bo=O
15. a) (i) 1 (ii) (l)(l) = 5!/(2!)(3!) = 6!/(2' •3' •2!)
(üi) (!)(l)(l) = ((8)(7)/2)(6!/(2' · 3' · 2!)) = 9!/(2' · 3' -3!)
b) /(•, 3) = ('";V(n - 1, 3),n a: 2,/(1, 3) = 1
Conj~tura: f(n, 3) = (3n)!/(2"3•n t) _
Dcnomacwn(porínducción matemática): Paran = 1,/(1, 3)= 1 y 3Y(2!3!1!)= l,demodo
que el resultado es cierto en este primc:r caso y establece la base de nuestra demosttación.
Vamos ahora al paso inductivo. dondesuponemos que el resultado es verdadero paran =
l(~l); es dcc:ir,/(1. 3) = (3t)!/(2'3't!). Sin= t+ l , tenemos

f(t + 1, 3) = (3(1 +;>- l)t(t, 3) = ( 31 ; 2)1(,,3)


= [(3t + 2)(31 + l)/2)[(3t)!/(2'3't!))
• ((31 + 3)(31 + 2)(31 + 1)/(2-3 · (1 + 1)))[(3t)!/(2'3't!))
= (3t + 3)!/(2"'3"'(1 + 1)!) = (3(t + 1))!/(2"'3"'(1 + 1)!~
Puesto que la vero.ad de esta fórmula en n = t implica la verdad de la misma paran= t + l,
el principio de inducción matemática implica que

f (n,3) = (3n)!/(2"3"n!) para todo n e z·.


e) f(n,k)=(7:~i1}f(n-l,k), na:2, /(1,k)=l.
Conjenu-ao f(n, k) = (kn)Y(2"3' · • · k'n!)
Demonración (por inducción matemática): Cuandon = l.J(t, .l:) = I y k!/(2131 • • • k'l!) =
1, por lo que este primer caso es verdadero y establece la base de nuestra demostración
inductiva.
Soluciones S-57

Si suponemos que el resultado C$ cierto para n = I(~ l), tc:ncmos nuestra hipótesis de
= =
inducc:i6n:/(t. k) (kr)V(2'3' • · · k't!). Y cuando n t + 1, se sigue que

/(1+ 1 k)• ( k (1 + 1)-1) f(~k) •(kt +k- l)/(1,k)


' k-1 k-1
• [(kt + k -1)!/(k - t)!(kt)!)[(kt)!/(2'3' · · · .t'1!))
• ((kt + k)Vk(1 + l)(k - !)!)11/(2'3' · · ·.t'1!)]
• (kt + k)!/(2'.,3"' · · · ,t-•'(1 + !)!)
• (k(t + 1))!/(2"'3"' · · · .t"'(1 + !)!~
En coasecococia. d resultado dado C5 vadadcro pan todo n E z• por d principio de
ioducddomalel!Wica.
17. a) Las particiones que se cucotao cn/(n., m) caen en dos categorías:
(1) Las particiones en las quemes un sumando. :Éstas se cuentan cn/(n - m, m), ya qucm
puede apan,c:er más de una va.
(2) Las particiones en lasquem oocsuosumando.de modo quem-1 es el sumando n1':s
grande posible. Esw parúcioocs se cueman CD /(n, m - 1).
Como estas dos catc¡orW son exhaustivas y disjuntas, se sigucQJJe/(n, m) •f(n-m. m) +
/(n,m-1).

C..pltulo 11
Una lntrocluccl6n • la teorfa de grafos

Secci6n 11.1- l . a) Representar las rutas a&cas recorridas entre cierto ooojunto de ciudades por una aerolmea
pjg. 535 particula<
b) Representar una r<d el6caica. En es,e caso, los v.!mces pueden r,presenta, intenuptor<S,
transi~. ctc&n.; y una arista (x. y) indica la cxi!tencia de un cable que cooccta x y y.
e) Ha¡amos que los vbtices represcmen UD conjunto de solicitantes de empleo y UD conjunto
de puc$tOS vacantes en una empresa. Trazamos una arista (A, b) para denotar que d solici-
Wlte A csá C3lificado pua d puesto b. Entonces. todos los puestos vacantes pueden OCU•
parse si d pafo resultaDte proporciona un emparejamiento corre un subcoojunto de los
soliciWlteS y los puestos abiertos.
3.6 5.9;3
7. a) 1-22 • 1-22 b) {(g,d),(d,<),(e,a)};
{(g, b), (b,c), (c. d), (d,,), (<,a)}
e) Dos: uno de {(b, e), (c. d)} y
uoo de {(b,.I), (/, g). (g. d)}.
d) No
e) Sf. Recorra el camino
{(c,d), (d,e), (<,a), (a, b), (b,f), (/,g)}.
,...
f) SI. Utilice d recooido
((g, b), (b,f), (f,g), (g, d), (d., b), (b, e), (e, d). (d., e),(<, a), (a, b )}.
9. Si {a, b) noes parte de un ciclo,entonces su eliminación provoca la desconexión dca y b (y
G). Si no, existe un camino simpJe Pdca a b, taJ que P junto con {a. b} proporciona un ciclo
qoe coolienc a la. b). Rec!procamcruc, si la climinacióo de la. b) de G descooccta a G,
cntooces cxistenx., y E V tales que dánico camino ñmpk P dcx a y contiene a~~ {a, h}. Si
• fuera pone de un ciclo C, cntooces las ari,tas CD (P - {<}) U (C -{,}) comcodñan un
segundo camino simple que u.ni.ría x coa y.
11. a) Sí b)No c)n-1
13. La partición de V inducida por 9t produce las componente$ (conexas) de G.
S-58 Soluóones

Sección 11.2- !. a) 3 b)G, =(1/).doadc U={a.h.d,f,g,h,,j);G,=G-{c)


pág. 547 <) G,=(W). dondc W= {h, c. d,f, g, í.j); G,= G-{a,h)
d)

e)

(1 h

o (i, ü, y iñ) • .-----:

3. a) 29 =SU
rrz.
g ,.,

b) 3
1 j

e) (i) 7!' (ü) 7!'

7. (i) .
S. Ges(ocsisomorfoa)K•. donden= IV!.

V□A R□v A□R


A
'
,[J,
V (ü) Sin solución

(iñ) .• V
V
A
•A

R□ 8
A
•Ov
'
'
A□R v[J•
V .
9. a) Hay (1/2)(7)(6)(5)(4)(3) • 1260 caminos de longitud 4 en K,.
b) El número de caminos de loogitudm coK., paraO<m < n,es (l/2)(n)(n- l)(n-2) · • • (n - m).
11. (l) No (2)No (3)Sf.Hacemoscorrcspondcracon.w.bconw,cconx.dcony,ecoou
y/con z.
13. W-•
15. a) Si G1= (V1, E1) yG1 =(V1 , EJ sonisomorfos.entoocesexisteuna función/: V1 ➔ V1 quees
inyectiva y sobre y prcsc:rva las adyacencias. Si :c. y E V1 y {x, y} ~ Ei, entonces if(.:r),
/(y)) ,t E,. Por lo 1a1110, la misma función preserva las adyacencias para "ci;, o,
y puede
usarse para definir un isomortismo para 'a;. a;.
El rccfproco puede cstabJecerse en forma
similar.
b) No son isomorfos. El complemento de un grafo que contiene al v&tice a es un ciclo de
longitud 8. El complemento del otto grafo es la unión disjunta de dos ciclos de long:irud 4.
17. • Si Ges el ciclo con ansias {a, h). {h, e), {e, d), {d, e) y {e, a),

@
entonces f; es el ciclo C<!_D aristas {a, e}. {e, e}, {e, b}, {b, d} y
~ e {d. a). Por lo tanto, G y G son isomorfos. Recíprocamente. si Ges
un ciclo sobren v~ces y G, Gson isomoños,entoocesn =½(;) o
n = ¼(n)(n -1),y n = S.
d
Soluciones S-59

19. a) En este caso,/tambitn debe conservar las direcciones. Asi, (a. b) E E, si y sólo si (/(a),
/(b)) E E,.

b)
I.Lb.~.4)~
L b. /;;,.
L tJ\ /;;,.
() . {J\ /;;,.
e) No son isomorfos. Consideremos el vhticea cnel primer grafo. Es incidente aun v&ticee
incidente desde otros dos fflliccs. Ningwl v&tice del o<ro grao tiene esta propiedad.

Sección 11.3- 1. a) IVJ=6 b) IVl=lo2o3o5o6ol0o1So30


pág. 556 (En los primeros CIWro casos, G debe ser un multigrao; cuando IVI • 30, Ges di.scooexo.)
•> IVl=6
3. a) 9
b) ~ -- -- - -, -- - -- - --~

'@·
.. . ' ~ :
1
. g 1
• w
, ,
1

S. a) IV,l =8• IV, l; IE,l=l4 • fE, I


b) l'alll V, teaemos que gn,d(a) z 3, grad(b) =4,gn,d(c) = 4, ¡¡rad(d) = 3, gr.id(,)= 3, fPd{./) = 4,
= =
pd(g) = 4 y g,ad(h) = 3. l'alll v, teaemos pad{s) = 3, ¡¡rad(r) 4, g,ad(u) 4, g,ad(u) • 3.
¡rad(w) = 4, ¡rad(.r) = 3, grld(y) = 3. grad(z) = 4. Por lo W>tO, cada uno de los dos .,.ros
tiene cuatro fflliccs de pilo 3 y cuatro de pilo 4.
e) A pesar de los resultados de las partes (a) y (b), los gnfos G1 y G, no son isomorfos.
En el grafo Gt los cuatro vátic:es de grado 4 (t. u. w y z) esún en un ciclo de lon¡itud 4.
l'alll el grao G,. Ios Yátiocs b. c.fy g (cada uno de pilo 4) oo estin en un ciclo de
loo¡itud 4.
Una segunda vfa para ver que G 1 y Gi no soo isomocf'05 es considerar de nuevo los
v&tices de grado 4 en cada grafo. En G1, estos v&ticcs inducen un sub¡rafo dis.coocxo que
constl de las dos aristas (b, e} y {f. g}. Los cuatto v&ticcs de pilo 4 en el gnfo G,

·0 ·e·
iDducc:o un subpafo cooexo qoc tiene cinoo aristas. cualquier arista posible excepto {u. z).
1.

1
' • •

Ca) b ~ I
S-60 Soluciones

9. 61 VI s r.,.. grad(u) s Al v¡. Como 2IEI =l:..v grad(u). se: sigue que 61 VI s 2IEI ,s
A !VI. podo que 6 s 21.dn) ,s A.
JJ. Comenzamos con un ciclo u, ➔ u,; ➔ 'O) ➔ ••• ➔ ~ ➔ ~ ➔ u1• Des~ trazamos las i:
aristas {uh 1>!.1}, {t>z, tlt+2l, ... , {u,.. u....}, ... , {u.,~}. El grafo resultante tiene 21: vátices.
cada uno de grado 3.
13. (Corolario ll.l). Sea V = V, U V,, donde V,(V,) contiene todos los v&ticcs de grado impar
(par). Entooccs 2IEI - L..v, grad(u) = l:..v, grad(u) es un entero par. Si IV,I es impa,.
entonces l:.c:v1 grad(u) es impar.
(Corolario 11.2). Para la n:clproca. sea G = (V. E) un grao con un recorrido culcriano, tal
que a, b ,oo los vtr1ices inicial y final. Aftadimos la arisla {a, b} a G para formaron grao más
grandeG1=(V, EJ. dondeG1 tiene un recorridoeulcriano. Por lo tanto, G1 es cooexo y cada vátice
de G, tiene grado pac Al climlnar la arisla {a, b} deG., los vhtices deG lendránel mismo grado par
exccp!Oa. b; gra<lc(a)• grad,;,(a) - l,gra<lc(b) = grad,;,(b)-1,de modo"'°los vhticesa, btieoea
grado impar en G. Además., como las aristas de G forman un reconido de cuJeriano, Ges cooexo.
IS. a) Sean a, b. e, x. y E V tales quegrad(a) = grad(b) = grad(c) = 1, gradC,) = 5 y grad(y) = 7.
Como grad{y} = 7,y es adyacente a los otros (siete) vtrticcs en V. Por lo tanto, el vbticex
no es adyacente a los váticcsa,. h y c. Comox no puede ser adyacente as! mismo, a [QeDOS
que tengamos lazos, se sigue que grad{x) ~ 4 y no podemos uazarun grafo con las condi-
ciones dadas.

17. nimpar;n = 2 19. SI


21. a) (i) 13 (ü) 25 (iü) 41 (iv) 2n' - 2n+l
b) (i) 12 (ii) 24 (iü) 40 (iv) 2n' -2n
23, L..vgs(U)= IAI . l:,.,ge(U)
25. Del ejercicio 23, r.,..¡gs(u)-ge(u)] =0. Para cada u e V, gs(u)+ ge(u) = n - l , por lo que
O= (n - 1) · O= ¿ (n - l)[gs(u}- ge(u)]
= ¿ [gs(u) + ;u)][gs(u}-ge(u)]
.ev
= ¿ [(gs(u))' - (ge(u))'],
.ev
y se sigue el resultado.
rl. a)yb)

e)
o
o
o···
' o
'
'
o
'''
SOiuciones s-61

=• 2: 2. Como Ges conexo sin lazos, para todo" E Y tenemos que 1 ,s gr>d(;r) ,s n
29. Sea I VI
- !.Aplicamos d principio del palomar. donde losn vbtica soa las palomas y losn - 1 ¡nidos
pooblcs son los mdos.
31.a) •.
t'z e, t. e, 4 e, e,,, eioeu

H: :] . [l
"11'2'UlV.Us

1100000000]
., o
A=:[! u. O 1 1 O 1 "' o
0111100000
/ •v, 1 0 0 0 1 0 0 1 0 1 1
0000101100
"' 1 1 1 1 1 ... o 1010010110
b) Si existe un camino de longiNd dos entre U; y u.,. lo denotamos con {u,. i.:>t}, {u., 'U1 }.
E.ntooccSa.•a,¡= 1 eo.A. ycl clemcruoi.jeo.A2 es l. Recíprocamente. si el dememo i.j
deA2 es l , ml0oce$existealmenosunva1orde1. l s k .Sn.talquea.-=~= l y esto indica
la existencia de un camino {u.,. Ua}, {ub u,} eottc los vtnioes i-áimo y j-ái.mo de V.
e) Para l :S i, j s n, el elemmto i. j de A2 cuenta el n11mc:ro de caminos distintos de longitud
dos aitre Jos vbtices i-áimo y j-ánno de V.
d) Si u CSli en la parte superior de la columna. la suma de los clcmcnlos de~ es d gr1do de
u si oo hay wi lazo en u. En caso contrario, ¡rad(u) • [(suma de la columna para u)- I] +
2(mhncro de lazos en U).
e) Pan. cada columna de /, la suma de sus elemmtos es 1 para un bzo y 2 para una arista que
no sea un lazo.
l.
Sección 11.4- Ell esa, situaci6a, d vbtic, b CSli en la región focmada poc las
~ - 573 aristas (a, d}. (d. cJ, (e, al y d vbtie< <csti fucn de esca no-
gióo. Po, lo tuno, la arisu (b, ,¡ cruzaríunade las aristas (a, dJ,
{d. e) , o {a, cJ (como se mucstn).

3. •> Grafo Número dt •&tices Número de aristas


K.., 11 28
K 1.11 18 71
K_. m +n mn
b) m • 6
S. a) Bipartito b) Bipartito e) No bipartito
7. a)(;')(;) b) m(;)+nfn=(l/2)(mn){m + n - 2]
e) (m)(n)(m - l)(n -1) •4{~;)
9. a) 6 b) (l/2)(7)(3)(6)(2)(5)(1)(4) • 2520 e) 50,295,168,000
d) (1/2)(n)(m)(n - l)(m - l)(n -2) · · ·(2)(n -(m + l))(l)(n - m)
ll. Scpan,mos Y como v, U Vbcoc, IY,I = m, IV,I =u-.._ Como Ges bópanito, d Dllmcto
máximo de ariswquc puede tcncTesm(u-m) •-(m - (1>'2)1'+ (u/2)'. una funcióodem. Para
un valor dado de u, si u es par. m • u/2 maximiza m(u - ns)= (ul2)[u - (u/2)] a (u/2)2• Para u
~.m=(u-1)/2om •(U+ l)/2maxúruzam(u-m)=((u-1)12)1u - ((u-1)12)] • ((u-


J<
l)/21((u + 1)/2] •((u+ 1)12](u-((u + 1)12)] : (u'-1)14 =[Cu(2)' (1>'2)'. PI>< lo tuno, si
1El > (ull)', G no puede ,.. bópartito.
13. a) a : {1,21 /: {4,5}
b: {3,4} 1 : {2,5}

®.
1
e: {1,5} h: {2,3}
• ¡ 9
' d : {2,4} i: {1, 3}
<: {3,5} j: {1,4}
;
d
S-62 Soluciones

b) Ges (isomoño a) el grafo de P«ersen. [Vwe la Fig. ll.48(a).J


15. 9
17. a) Hay 17 v&ticcs. 34 aristas y 19 regiones; yu - e + r= 17-34 + 19 :: 2.
b ) Tenemos 10 vátic.es, 24 aristas y 16 regiones; u- e+ r = 10- 24 + 16 = 2.
19. 10
21- En casooonlnrio.grad(u) 2' 6 para todo UE V. Entonces 2e= :E..,,grad(u) >: 61 VI. de modo
que e >: 3IVI, lo que contradíce es 3IVl- 6 (Corolario 11.3).
23. •l 2e2'!:7 =k(2+e-u)~(2- k)<2'k(2-u)~ es[k/(k-2))(u-2)
b) 4
e) EnK,. tenemose = 9yu = 6. [.l/(k-2)](u- 2) =(412X4) = 8< 9= e. ComoK.,esconexo.
debe ser no plano.
d) En estecaso. k= S. u = 10,e= IS y [.l/(k-2))(u- 2) = (Sl3X8) =(400) < 15 =e. El grafo
de Pe.tersen es conexo, por lo que debe ser no plano.
25. El dual del t<Uaedro [Fig. ll.54(b)) esel grafomí$mo. P-arael grafo (cubo)dela figura ll.54(b),
su dual es el octaedro y viceversa. De la mísma forma. el dual del dodecaedro es el icosaedro
y viceversa.
27. (i) {{k,m},{m,,}} (ü) {{i,k),{k,s},{k,m}}
29.

Sección 11.5--
1. •l
71 b)@
p;!g. 585

3. a) Ciclobamiltoniano:a ➔ g ➔ k ➔ i ➔ h ➔ b ➔ c ➔ d ➔j ➔ J ➔ e ➔ a
b) Ciclohamiltoniano: a ➔ d ➔ b ➔ e ➔ g ➔j ➔ i ➔ J ➔ h ➔ e ➔ a
e) Ciclo hamiltoniano: a ➔ h ➔ e ➔J➔ g ➔ i ➔ d ➔ c ➔ h ➔ a
d) Camino hamiltoniano: a ➔ e ➔ d ➔ b ➔ e ➔/➔ g
e) Camino bamiltoniano:a ➔ b ➔ e ➔ d ➔ e ➔j ➔ i ➔ h ➔ g ➔J➔ k ➔ l ➔ m ➔ n ➔ o
O Ciclohamiltonfano:a ➔ b ➔ c ➔ d ➔ e ➔ j ➔ i ➔ h ➔ g ➔ l ➔ m ➔ n ➔ o ➔ t ➔ s ➔
r ➔ q ➔ p ➔ k ➔ /➔ a
5. d) Si eliminamos cualquiera de los vértices a. b o g, el subgra(o resultante tiene un ciclo
bamiltoniano. Por ejemplo, a1 eliminar el v&tice a encontramos el ciclo hamiltoniano b ➔
d ➔ c ➔f➔ g ➔ t ➔ b.
e) Existe el siguiente ciclo hamiltoniano si eliminamos el vértice g: a ➔ b ➔ e ➔ d ➔ e ➔
j ➔ o ➔ n ➔ i ➔ h ➔ m ➔ J ➔ k ➔ f ➔ a. Ocune u.na situación simttrica al eliminar el
vtrticei.
7. a) 012Xn-1)! b) JO e) 9
9. Sea G = (V, E) un grafo no dirigido, sin lazos ni ciclos impares. Suponemos que Ges cooexo
(en caso contrario, trabajamos con las componentes de G). Seleccionamos cualquier v&tice de
SOiuciones S-63

Vy sean V1 = {u E Vld(x, u), la longitud del camino máscorto cntrexy ues impar} y Y1=
{w E Vld(x, w), lalongituddcl camino máscortoentrexywes par}. Observe que (i)x E Y2t
(ii) V= V1 U V1 y(ili) V1 n V1 = e. Afumamos que cada arista {a. b} co A. tiene un v&tice en
V1 y el otro en V:- Para ver esto, suponemos que l!! ={a, b} E E cona, b E V1• (La demostración
para a.b E V1 es similar.) Sean E.= {{a, u,}, {u1, UJ}, .. . , {u_i, x}}, las m arisw en un
camino más corto dea ax y E.= { {b, 1>i1, {u~ ui], .. . , {U~t,X}} las n aristas en un camino
más corto de ba.x. Observequem yn son ambos impares. Si {u" U:, .. . , u- } n (u~ '0:2• .•
. •u.:,..1} = tt entonces el conjunto de aristas E'= { { a, b}} U E. U E. proporciona un ciclo impar
en G. En caso contrario, sea w( 4=- x) el primer v&tice en que se encuentran los caminos y sea E"
el conjunto
{{a, b}) U {{a, v,}, fu,, u,}, . .. , !u,, i,i} U{{b, v;}, {u;, vi}, . . . , {vj, i,i},
para algunos l s i s m - 1 y 1 :s j :s: n - l. EntoOCCS'" E" proporciona un ciclo impar para G o
E- E" contiene un ciclo impar para G.
U. a) • •

,D.. ,D.
b)•~
·~
d <

gs(a) = 3 ge(a) = O gs(a) = 3 ge(a) = O


gs(b)=2 ge(b) = 1 gs(b) = 1 ge(b) = 2
gs(c) = O ge(c) = 3 gs(c) = 1 ge(c) = 2
gs(d) = 1 ge(d) = 2 gs(d) = 1 ge(d) = 2

.[8].
gs(a) = 1 ge(a) = 2 gs(a) = O ge(a) = 3
gs(b) = 1 ge(b) = 2 gs(b) = 2 ge(b) = 1
gs(c) = 2 ge(c) = 1 gs(c)= 2 ge(c) = 1
gs(d) = 2 ge(d) = 1 gs(d) = 2 ge(d) = 1
13. Para el multigrafodelafiguradada, IVI = 4,grad(a) = grad(c) =grad(d) = 2 y grad(b)=6. Por
lo tanto, grad(x) + grad(y) ~ 4 > 3 =4 - 1 para dos vtniccs .i; y e V no adyacentes, pero el
multigrafo no tiene un camino bamiltoniano.

15. Para todos.t,y E V.grad(x) + grad(y) a: 2((n-1)12] =n- 1, de modo que el resultado se sigue
del teorema 11.8.
17. Paran~ 5, sea C.=(V. E)el ciclo conn v&tices. Entonces C.tieneun ciclo hamiltoniano (en
realidad, es un ciclo hamiltoniano), pero para todo u E V, grad('O) = 2 < n/2.
5-64 Soluciones

19. Esto se sigue del teorema 1t .9, puesto que para todos los v&ticcs x,. y E V (no adyaccotcs),
gr.,d(x) + g,ad(y) = 12 > 11 = 1v¡.
21. Sin = S, los grafos C, y C, son isomodos y ambos son ciclos ham.iltonianos con cinco v&tices.
Para n 2: 6, sean u,.u v&tices no adyacentes en C.. Como grad(u) = grad(U) = n - 3, tenemos
quegrad(u)+ grad(u) = 2n - 6.Además, 2n - 6 2: n ~n 2: 6,de modoquesesiguedel teorema
11.9 que el cocido C. contiene un ciclo hamiltoniano sin;?:. 6.
23. a) El camino simple u ➔ u 1 ➔ U: ➔ \>J ➔ • • • ➔ u..1 prop:,rciona un camino bamiltoniano
para H•. Como grad(u) = 1, el grafo no puede tener un ciclo hamiltoniaoo.
b) F.n este caso, IEI = ("';1) + l. (Así, el número de aristas necesario en el oorolario 11.6 no
puede disminuirse.)
25. a) (i) {a, c,f,h), {a,g} (ü) {z}, {u, w,y)
b) (i) il(G) = 4 (ü) ~(G) = 3
e) (i) 3 (ü) 3 (iü) 3 (iv) 4 (v) 6 (vi) El m!ximo de m y n
d) El grafo completo con III vértices
27. a) S; 6 b) 17 e) 33 d) 1 + 2"

Sección 11.6- l . Trazamos un v&tice por cada especie de pez. Si dos espccicsx, y deben mantenerse en peceras
pág. 595 . distintas, trazamos la arista {x,. y}. El nómcro nú!Umo de peceras nctcsarias es entonces el
mlmero cromático del grafo resultante.
3. a) 3 b) 5
5. a) P(G, l.)= A(l. - 1)'
b) Para G = K,~ tenemosqueP(G, 1.) = A(l.-1)'.
z(K.J = 2
7, a) 2 b) 2 (n par); 3 (n impar)
e) Figura 1l.54(d): 2; Figura ll.57(a): 3; Figura ll.80(i): 2; Figura ll.80(ii): 3
9. a) (!) X(X-l)'(X - 2)' (2) X(X - l)(X-2)(X' - 2X+2)
(3) X(X - l )(X - 2)(X' - SH 7)
b) (!) 3 (2) 3 (3) 3
e) (!) 7'1fJ (2) 1020 (3) 420
11. a) X(X - !)'(X - 2)'
b) X(X-l)(X-2)'(X'-4X'+6X-3)
e) X(X - l)(X -2)'(X' - 4X' + 6X - 3)
d) X(X-l)(X- 2)' (X'-4X'+ 6X-3)
13. a) X(X - l)(X - 2) b)Sesiguedelteorcma 11.10
e) Se sigue de la regla del producto
d) P(C.,X) = P(P.,X)- P(C.- ,. X)= X(X - !)"-• -P( C.-,.X)
=[(X -1) + l j(X- o·-•- P(c._,. X)
= (X - 1)"+ (X - t¡--' -P(C.- ,,X),
as! P(C., X) - (X - 1)" = (X - 1¡--• - P(C. - ,. x).
Si reemplazamos n con n - 1 obtenemos

Por lo tanto.
P(C.,X)-(X-1)" = P (C.-,, >-)-(X- 1)"- '.
~) Si continuamos a panir de la parte (d),
P(C.,X) • (X - 1)" + (-1)"- '[P(C,,X) - (X - 1)']
= (X -1)" + ( - 1)--,[X(X- l)(X-2) - (X - l)'J
= (X - I)" + (-!)"(X- !).
~uciones 5-65

15. Del teorema 11.13,el desarrollo del'{G, A) coorendrtexXWD<Dte uoaocuamc:iadcl po1ioomio


cromático de K.. Puesto que no aparece un grafo más grande, este t&mioo determina el grado
como n y el coeficiente principal es 1.
17. a) Paran e z•. n 2: 3, sea C. el ciclo conn v&tices. Sin es impar. enlQDCeSX(CJ ::.3. Ptto
para rualquierveo c . el subgrafo c.- u es un camino simplecoon- l vmi.ces y x(C,- u)=
2, por lo que pan• ;mpar, c. es aftico r.specu,
del color.
Sin embargo. si n es par. tenemos que x(C.) • 2 y para cualquier u en Ch el subgrafoC,- u
es un camino simple coa n - 1 ~ces y x(C. - u)• 2. En consecuencia. los ciclos 0011, uo
ndmero par de vMic:es no soo críticos respectO del color.
b) Pan: cua1quier grafo completo K., con n 2: 2. tenemos x(KJ • n y para cualquier váticc u
eo K., K,- u es (isomorfo a) K--t, de modo que X(K,- u)• n- l. En c:ons«:UCDCia. (odo
grafo completo con al menos una arista es critico rcspcctO del color.
e) Supoo¡amos que G oo es conexo. Sea G, una componente de G tal que x(G,l =x(G) y sea
G2 cual.quier otra componente de G. Entoooes X(G1).?: X(GJ y para todo u en Gz. ttDclnOS
que x(G- u)= x(G1) 3 x(G), de modo que G oo es crítico respecto del color.

Ejercid os l. n = 17
complementarios- 3. a) Etiquc:Lamos los vmiccsdeK,comoa, b, • .. ,f De lascincoaristassobrea,aJ menos tres
~ g. 602 tieneo el núsmocolor,ctigamos,elrajo. Sean {o. b}, (o. e}, {o. d} ....,amw, s; las aristas
(b. e}. (<. d}, {b. d} son todas azules, se sigue el resultado. s; no. una de estas aristas.
ctigamos {<. d},cs roja. Entooces, las aristas (o.<}, (o. d},J<. d} producen un trián¡ulo rojo.
b) Consideremos a las seis personas como v~rticcs. Si dos personas soa amigas (descoooci-
du). dibujamos una arista roja (azul) que conecte sus v&tic:es respectivos. El resultado se
sigue eotooc:cs de la parte (a).
S. a) Podemos volver a dibujar G2 como

b) 72
7. a) 1260 b) 756
c)(Caso l : pes impar, u = 2k + 1 parak e N.)Tencmosmncaminos simplesde longitudp= 1
(cuandot=O) y(m)(n)(M - l)(n-1) · · · (m - t )(n - t) caminos sunples de lonaótudp • 2k +
1 2: 3.
(Caso 2: pes par, p • 2k para k e z•.) CUando p < 2m (es decir, k < m), el nllmero de
caminos simples de longirudp es (l/2Xm)(n)(m - l Xn - 1) • • • (n - (k - l ))(m - k) +
(112)(n)(m) (n - l)(m -1) · · · (m-(k- l))(n -k). l'>np = 2m lene1DO$ (l/2)(n)(m)(n- l)
(m - 1) • • • (m - (m - l))(n - m) canúoo, ,;mplcs de longótud (mb.hna) 2m.
9. lQ. - l)(l. -2)'0.'-3A+3)
U . a) Sea/independiente y {a. b} E E. Si a y b no están en V-/, entonces a, b E/ y como son
adyaoerues. / no es independiente.. Rcáprocamcnte. si / í:: V con V -1 un recubrimiento de
Ge / no es independiente, entonces existen vtrticcs x,y e/ ules que {x. 1} E E. Pero
(.r,y} EE =>xoyCSUcnV - 1.
b) Sea/ un conjunto maximaJ: independicntc. de tamano mllimo. en G y K un recubrimiento
mínimo. Dela panc (a), IXI :s IV-1I s IVHI I e 111 " IV-XI ~ IVI-IXJ.o JXJ+J/J
2: JvJ 2: JXl+IIJ.
13. a) v,, • u,.- 1 + 2,n~2,u,_=-2
u..•2n, n.t:1
b) e. • e..- 1 + ~ -1 • e .. - 1 + 4n -4, n .2: 2, e1 =O
t,. • 2n(n - l), n ~ 1
Soluócnes

e) ,. - ,,._1 + 2t,._1 =t..-1 + 4(11 - l)(n - 2), n ~ 3, t 2 =O(= t1)


t. = (4/3)n' - 4n' + (813)n, na, 2 (o na,!)
15. •> IVJ=2n; IEl=3n - 2 (na,t)
b) a.. =a..- , +a..- 2, ao=a1 = 1
a.,:: F.,.i,cl (n + l~imoollmerodeFibonacci
17. a,.=a.-1+2a.-2. a1=3,ai=S
a.=(-1/3)(-1)"+(4/3)(2"), na,!.
19. a) -,(G)=2;11(G)=3;x(G)=4
b) G no tiene un recorrido culcriano, ni un cimlito euleriano; G tiene un bamiltoniano.
e) G no es bipartito, pero es plano.
21. a) x(G) "' Ol(G). b) Son iguales.
23. a) El término constante es 3, no O. Esto oootradicc el teorema 11.11.
b) E coeficiente principal es 3, no l. Esto contradice el resultado del ejercicio 15 de la sección
11.6.
e) La suma de los coeficientes es-1, no O. Esto contradice d tc:orcma 11.12.
25. a) K, b) !VI= }:.".,P;, !El =I,,.,.,Mp;p1 c)i(G)=n;Gticnc¿;'_,(';') aristas.
d) n=2:p1=l, Pz.1:l
p1=2, P2.¡:2
n =3:p1 = 1, P2• l, ~~ 1
p1=l, pz=2, J>3=2
p,=2, p,=2, p,=2
n=4:p1 = 1, Jn.=l, J>J=l, p,=1
p1=l, Jh.=l, p3=l, p,=2

capitulo12
Árbolff

5ecd6n 12.1- l. a)
p¡lg. 612

b) 5
3. a) 47 b) 11

.
5. Los camioos simples
7.

.D, ..
9. Si existe un ll.n.ico camino simple entre cada par de ~rt:iccs de G, entonces G es conexo. Si G
contiene un cicJo, cntooce5 existe un par de v~ces.x, y con dos caminos simples distintos que
los unen. Por lo tanto, Ges un grafo no dirigido, conexo, sin lazos, sin ciclos y. por lo tanto, es
un árbol.
u. (;)
13.. a) En laparteú) dela figura dada encontramos el grafo completo bipartitoK,. l- Las partes (ü)
y (iü) proporcionan dos úbolcs rccubridores no isomorfos para K,_ 1•
b) Salvo isomorfismo, ~tos son los 11n.icos árboles rccubridorcs de Ku.
Soluciones

15. n
"
:~: :~: ..:~: ..
17. a) Si el complemento de T condeoe UD conjunto de corte, entonces la eliminación de estas
arl$tas desconecta a G y existen vbtice$ x. y sin un camino que los una. Por lo tanto, T no
es UD árbol recubridor de G.
b) Si e l complemento de C contiene un úbol recubridor, entonces cada par de v&tices en G
tiene un camino simple que los une:. y este camino DO incluye aristas de C. Por lo tanto, la
e e
eliminación de lu aristas en de G no desconecta a G, y así, no es un conjunto de conc
deG.
19. a) (i) 3. 4,6,3,8,4 ("ú) 3,4,6,6,8,4
b ) Ninglin v&tioe colgante del irbol dado aparece en la sucesión. por lo que el resultado es
cierto para estos v&tices. Si se elimina una arista (,¡:, y } y y es un vM:ioe colgante (del úbol
o de alguno de los suWrbolC$ ruultanres}, ¡rad(.:r) se decrementa en 1 y se coloca .:rea la
sucesión.Al continuar el proceso, (i) este v&dcc.rse ooovienc en un v&ticccolgan.teeIJ UD
sub4rbol y es eliminado, pero ya no vuelve a registrarse en la suoesión, o (ü) el vátioe.:r se

c)rl,
deja como uno de los d.ltimos dos v&tices de una arista. En ambos casos,.x se cnumc:n. en la
succsiOO (¡rad(.r}-1) veces. ·

2 6 S :

1 •

d) De la • - dada se <OD<JCe d grado de cada vtnice en el irbol


Paso 1: Hacemos el cootador i igual a 1.
~ 2: Delos váticcsdegrado 1, sclccciocwnos d vtniccu coa la ctiqucumlníma. Esto
detennioa la arista {u, .1;-}. Eliminamos u del conjunto de etiquetas y reducimos el
grado de..,..,
l.
Puo 3: Si ; < n - 2. incrementamos; en 1 y regresamos al paso 2.
Puo 4: Si ; = 1t - 2, los v&tiocs (etiquecas) ~ .:r...i cst4D unidos por una arista Sl .rw f< .:r..1.
(En este caso, el úbol es completo.}

Sección 12.2- l . a) f, h,k,p, q,s,t b) a e) d


~ g. 631 d) ,,f,j, q, s, t e) q, t f) 2
e) k , p, q, s, t 4•> 1) No J) m - 3
3. a) / + w -.ry •" f z3 b) 0.4
5. Ordc:o pttvio: r,j, h. g, e, d, b, a. c.f. i. k. m. p. s. n. q, 1, u, '"'I u

7. a) (i)y (úi) !):


Orden sim!trico: h. e, a. b, d, c. 1, f. j. i. r. m. s. p, k. n. u, t, w. q, u
Orden posterior: a, b, c. d, e,f. g, h. i.j, s, p, m., u, w, 1, u, q, n. k. r

(ü) ,0••
..
< d <

,
Soludones

b) (í)

l[ ,. l
l.'
9. Ges conexo.

11.

~. ./ '
~ '• / ,. ·•-• .¡\..
"' (b)
"'
a) El árbol que se muestra en la pane {a) de la figura tienen vtnices y las enumeraciones en
orden previo y si.mftrico de estos véti.ccs son 'O¡, "2, u,. ... , u,...i, u ..
b ) F.n la parte (b) de la figura tenemos un árbol binario conm vtniccs y las enumeraciones en
orden posterior y simétrico de estos v~rtices son u .. U-a, ... , U:s. Ui. '01,
e) Si T1. T2 deben ser átboles binarios completos, ya no podemos tener I Y1l, IVil i?: 3. Debe-
mos ser más espedficoscn relación con el número de véti.ces; en particular, 1V1 1 y I V2I no
pueden ser pares. Pero inclu.so esto ,w es lo suficientemente bueno, ya que cualquier árbol
binario completo con tres o más vtrtices debe contener un subárbol como c1 que ¡,e muestra
en la parte (e) de la figura. Y cuando consideramos la forma de enumerar estos ues vtttices:
Orden prnio: ,; a. b
Orden poswior: a, b, ,
Orden .simétrico: a. ,; b,
vemos que en un árbol binario completo ya no es posible tener una lista en orden s~co
igual a la lista en orden previo o posterior.
13. Teorema 12.6
a) Cada v&tice interno tiene m hijos, de modo que hay mi. vértices que son hijos de algún otro
vhtioe. Esto sirve para todos los vértices del árbol excepto la raíz. Por lo tanto, n =mi + l.
b) l,.+i=n = mi + 1 ~l.= (m - l)i + 1
e) 1=(m-l)i+ I =>i=(l-1),'(m-I)
n=mi+ l ~i=(n-lYm.
Corolario 121
Como el árbol es equilibrado, ,,,._1< t :s n'f' por el teorema 127.

m"-- 1 <t::S m4 ~log.,(m"- 1)<Iog..(t)slog,.(m")


C:,(h - l)<log,.tshc;>h =nog,.<J
Soluciones S-69

15. a) 102; 69
17. a) b) 9; 5 <) h(m -1); (h -1) + (m -1)

19. 2184S; 1 + m + m' + · • • + m""' ~(m' -1)/ (m - 1)


D, {L ~ ~~-1~ ~~ ~

121 131 f {41 191 1 f10J {11} f {121 (SI 11 {6) (71 f {11

Sección 12.3- l. a) L ,: 1,3,5, 7, 9 L,:2, 4, 6,8, 10


pig. 638 b) Supongamos que ,n < n.
L i : 1,3, S,7, ... ,2m-3, m+n
L,: 2, 4,6, 8, . . . , 2m-2, 2m-l, 2m, 2m+l, . . ,m+ n-1
3. a)
f-2,-3.-11
{- 3. - 21
l - 1l
1-Jf 1- 21

Sección 12.4- l. a) tear b) tatener e) rant


pig. 644 3. a: 001 e: 1000 e: 01 g: 00001 i : 11
b: 000001 d: 0001 /: 1001 h: 101 ;: 000000
S. SS,981
7.
lO 30

1-"
2 J
'
o
~
2 3
o

Mejore la pane (a) del paso 2 para el algoritmo del úbol de Huffman como sigue. Si exista:
n(> 2) de tales árboles con pesos n:úrtimos en la raíz p y p', entonces
S-70 Soluciones

(i) si p <p' y n-1 de es.tos árboles ticnenp' como peso en la raíz. seleccione un árbol (coa
peso p' en la raíz) con peso mínimo; y
(ü) sí p = p' (y los n úbolcs tienen el menor peso en la raíz), seleccione dos árboles (de peso
p en la raíz) de mínimo peso.

Sección 12.5- l. Los puntos de articulaci6n son b, e, f. h. j. k. Las componentes biconexas son B1 : { {a. b} );
pág. 649 B,: {{d,e}}; B,: {{b,c},{c,f),{f,e},{e,b)); B,: {{f,g},{g,h},{h,fl};
B,: {{h,i},{i,j},{j,h}}; B.: {{j,lcj}; B,: {{k,p),{p,n},{n,m),{m,/cj,{p,m)).
3. a ) T puede tener tan pocos puntos de articulación como n - 2. Si T contiene un vátice de
grado (n - 1), entonces este vátice es el único punto de articulación. Si Tes un camino
simple con n vtrtices y n- l aristas, entonces los n - 2 vértices de grado 2 son todos pun10s
de aniculación.
b) En todos los casos, un árbol con n vhtic.es tiene 11- 1 componentes biconexas. Cada arista
es una componente biconexa
5. x(G) - máx{x(B,)llsis/cj.
7. Siempre tenemos prof(yJ = prof(y1) = l. (Hoza: Los vtrtices y 1 y Yz siempre· están en la misma
componente biconexa.)
9. Encasocontrario,seau E VunpuntodearticulacióndeG. Entonccs.:(G-u) >lC(G)= l. (Por
el ejercicio 17 de la sección 11.6, sabemos que Ges conexo.) Ahora, G - u es disconex.o, coa
componentes H i, H 2, •• • , H, para, 2: 2. Para 1 :S is 1, sea U; e H¡. F.ntonccs H;+ u es un
subgrafodc G-u,., n(H,+ u) s z<G-u,.,)<z(G). (F.ncstccaso, u,., = u,.)Ahora.scax(G)=
ny sea {c1,cz. ....c.} un conjunto den colores. Para cada subgrafoH,+u, l s ; s :,podemos
dar una coloración propia a los vtrtices de H, + v con a lo sumo n - t colores y podemos usar
c1para colorear el vátice u para todos estos I subgrafos. Entonces podemos unir estos r subgrafos
en el v&tice u y obtener una coloración propia de los vbtices de G doode usamos menos de
n(= X(G)) color,s.

Ejercicios l. a) Si G es un árbol, lo consideramos como un árbol con raíz. Entonces hay Aopciones para
complementarios- colorear la raíz de G y (A - 1) OpciOOC$ para colorear cada uno de sus de$CCndicnle$. El
pág. 652 resultado se sigue entonces de la regla del producto.
Recíprocamente, si P(G, A)= l(). - 1)8""1, entonces, como el factor 1 aparece solamente
una vez, el grafo G es conexo.
P(G.~) - ~(~ -¡¡--•
= ~• - (n - l)X"- ' + · ·· + (-1)"-• ~c;,G ciene n vértices
y (n - 1) aristas:. Por lo tanto, Ges un árbol (por la parte (d) del teorema 12.5).
b) De la patte (a), P(G, 1) = O y P(G, 2) = 2 > O, de modo que z(G) = 2.
e) Para cualquier grafoG = (V,E), si e E E, entooccs P(G, l.) = P(G0 l.) - P(G~ l.) de modo
que P(G. l) S P(G,. l). Si Ges conexo pero no es un árbol. eliminamos una arista de cada
ciclo deG hasta que el subgrafo obtenido sea un árbol recubridor T de G. F.ntonces P(G, l) S
P(T, A) = A(l - n•-
1
• [Recordemos que P(G, A) es el námcro de coloraciones propias de los

,.,h
vátices de G con Acolores. Por lo tanto, P(G, l) s P(T, A) se interpreta aquí como una
desigualdad entre dos números, no como dos polinomios.]
3. a) 1011001010100

.. r o ~
Soluciones S-71

e) Puesto que los dos Oltimos v&tioes visitados m un recorrido m orden previo $00 hojas. los
llltimos dos símbolos en la sucesióo caract<ds<icadecualquier "1>ol binario compldo son OO.
5. a) 32;31; 62 b)2';2'-1;2(2"-1)
7. Suponemos que G = (V.E) es conexo (en caso CODITario, trabajamos con""" componente de
G). Como Ges cooexo.y ¡rad(u)" 2 para todo u e v. d -=o 12.4 implicaqueG ooes ua
úl>oL Ptro cualquier grado oo dirigido conexo sin Lazos que oo sea un úbol debe comeocr un
ciclo.
9. Para l ~ i(< n), scaz.= d nómerode vétioesutalesque grad(u) • í. F.ntonccsx1 +%J+ - • • +z..., •
IVI • IEI + t.demodoque 2IEI = 2(- t +x,+x,+ · ··u..,). Ptro2IEI = I...grad(uJ•
(x1 + 2:c,+ lr1 + • · · + (n - l)l:...J. Despejamosx1 en 2(- 1 +x1 +xt+ · · · +x..J•x1 + 2.r:i+ · · · +
(n- llr... yoll<aJcmooqucx,•2+x,+2>.+ lx,+ •· • + C• - 3lr...•2+:r_.,,.,(Jrad(u,)-2].
U. a) O es isomorfo a K,- b) O es isomorfo a 14
e) G' es isomorfo a K*'• de modo que el ndmerode aruus nuevas es ("i')-n .(;).
d) Si GJ- tiene UD punto de articulación x, entollce$ existen u, u E V tales que cada camino
simple (en G') de u a u pasa por .x. (Esto se sigue del ejerc:icio 2 de la sección 12.S.) Como
Ges conexo, existe UD camino simple C (en G) de" a u . Si x no csd. en este camino simple
(que tambiáJ a, UD camino simple en G'). entonces eontrldecimos d hecho de quex sea UD
punto de uticulación en G'. Por lo tan10. d camino C (en G) pasa porx y podemos cscnl>ir
C: u ➔ 111 ➔•••➔..,,_,➔ U. ➔ x ➔ U. ➔ U... ➔· ·· ➔ U1 ➔ u. Petoentoooes, en cP,
aiiadimos la arista {11., u.}.con lo que el camino simple C (en G')dado porC: 11 ➔ 111 ➔
• • • ➔ "--4 ➔ M. ➔ U. ➔ u-, ➔ • · · ➔ Ut ➔ uoopasapx x. Porlotanlo, xooes un punto
de aniallacióo en G'. y G'oo ti<nc pwuos de aniculxi6o.
lJ. a ) ~= r... + C.., paran " 3 y l, • ~= l. Pucslo que &u es pn,c:isamellle la ldaci6a deroamaicia
.=
de Fibonacci, teoemos que I F.. el n-':simo ntlmeco de Fibooacci. paran it: l.
b) i..•i..-1+i..-2+1, n C::3, i1 ei2:::;0
;,. • (l/VS)a"-(I/VS)ll"-1 = F. -1, n ~ 1
d) La altura de 7 1 es O; paran ~ 2. la altura de T. es n - 2.
15. a) Pan los irl>oles r..:ubridores de G hay dos casos exhaustivos y mutuamente exduyaires:
(i) La arista {x1 ,y1 } esd.en d árbol rccubridor. estos irboles n:cobódores se c:ucntan en b.,
(ti) La arista {xi.y1} noestten el árbol rcc:ubridor. en este caso, las aristas {.r1o.r2}, {y1,y,}
CSWlambasenel árbol recubñdor.Aleliminarlasaristas {.r,..r1}, {yi, y,) y (x1,y1} del
&!'fo escalera origillll. ahora necesitamos un irl>ol recubrido< para d &!'fo escalcra
resultan1c 0011 • - 1 peldaños. En este caso, t<nemos a.., úboles rccubridcns.
b) b. • b.- 1 +2a..-1o n it: 2
e) a.. - 4a..-1+ a..-2•0, n~2
a. • (l/(2V3))[(2+ VJ)" - (2 - VJ)"], n ~ O
17. •l (i) 3 (ü) s
b) a.. • a..-l+ a..-2, n ~ 3. a1=~ a,: • 3
a.= F..,, el VI+ 1)-61,imo admero de Fibonaccci

c.ftulc,1J
Optlmiucl6n y empa,.¡~mlento

Sección 13.1- l. a) Encaso contrario.sea u, eStalque 1 :S; Smeieselmfnimodetalessu~Emonccs


~- 664 d(u., u,)< d(1>o, u ••1) ycootradecimos la eleccióodc u••, como ua vótioe ueaS para cual
d(u., u) es UD mínimo.
b ) Supoogamos que hay un camino simple diri¡kio aw cono (caG) deUoa u 1. Si este.camino
simple pasa por un v&tíce en S, entonc:es tenemos una contrac:licción por la parte (a). En
;..72 Soluciones

caso contrario, tenemos UD camino simple dirigido más corto C" de Uo a Ut y C" 56lo pasa
por vértices de S. Peroentooces C" U {(u., U..1), (u1.1, Ut.J, .. .• (u..i, u.,.), (u•• U.,1)} es
un camino simple dirigido (en G) de 1li) a u_, y es más corto que el camino C.
= =
3. a) d(a,b) = 5; d(a,c) 6; d(a,f) 12; d(a,g) = 16; d(a,h) = 12
b) f: (a,c),(c,f) g: (a,b),(b,h),(h.g) h: (a,b),(b,h)
S. Falso. Consideremos el siguiente grafo ponderado. • • ~,,

'"'/'
.,

«ción 13.2- l. El algoritmo de Krustal genera la siguiente sucesión (de bosques), que termina en un árbol
>ág. 670 rccubridor minimal T de peso 18.
(1) F, = {{,, h}} (2) l';=F, U{{a,b)) (3) 11 = li U {{b, el}
(4) F.=/, U{{d,<}} (5) F. = F. u H•,fll (6) I'; = F, U{{a,<}}
(7) l; = l';U {{d,g)} (8) l'i=T=l;U{{.f.i}}
3. a) Evansville-lndianápolis (168): Bloomington-lndianápoljs (51): Soutb Bend-Gary (58);
Tern: Haute-Bloomington (58); Soutb Bend-Fort Waync (79); lndianápolis--Fort Waync
(121).
b) Fort Wayne-Ga,y (132); Evaosville-lndianápolis {168): Bloomington-lndianápolis (51);
Gary-SOUtj Bend (58): Tem: Haute-Blooming<on (58); lndianápolis--Fort Waync (121).
5. a) Para determinar un árbol óptimo de peso m.ax.imal, rttJil!)lazamos las dos ocurrencias de
''pequeño" en el alg0ritmo de Kruskal por "grande...
b) Usamos las aristas: Soutb Bend-Evansville(303); Fort Wayne-Evansville (290): Gary-
Evaosville (277); Fon Wayne-Tem: Haute (201); Gary-Bloomington (198): lndianápolis-
Evansville {168).
7. Cuando todos los pesos de las aristas son distintos, se selecciona una única arista en cada paso
del algoritmo de Kruskal.

1. a) -s=2;t=4;w=S;x=9;y=4 b) 18
ección 13.3- e) (i) P = (a,b, h, d,g,i); P = {z} (ü) P ={a,b. h,d,g); P = (i,z)
ág. 681 (iii) P=(a,h);P={b,d,g,~z}
3. (1) b 15. 1A. d (2)

h li 6 f

Elfloiomaxin?.a' es 32. El f\Jjo maximal es 23,


que es c(J>, P) para queesc(/',P)para
,. - {~. b. d. g, h}yP - {i, Z} P• {a}yP- {b.g. U d. h. .t z}.

5. En este caso, e(~) es un entero positivo para cada e E E y el flujo inicial está definido como
f(e) = O para todo e E A. El resultado se sigue, pues en cada aplicoción del prncedimiento de
etiquetado el incremento en el flujo proviene de la segunda componente de una etiqueta, donde
dicha componente es siempre un entero positivo; cuando UD vfrtice tiene una etiqueta negati•
va, el decremento en el flujo nunca produce un valor negativo para dicho flujo.
Soluciones S-73

J 6.4 k .._, m

Sección 13.4- l. 5/(l) = 1/14


pág. 691 3. Sean. e 1• el' ... , e, los comitts, de acuerdo con la forma en que fueron enumerados en el
ejercicio.
a) Seleccionamos los miembros como sigue: c 1 -A; e2 - G; e, - M ; e, - N; e, - C; e, - R.
b) SdecciODamos losqueooSCID miembros romo sigue: C1 -C;c2 -A; e, -G;c.-J; c,-M; 4 -P.
=
S. a) Un 1-factor para un grafoG {V, E) consta de aristas que no tienen vtttices en común. Así,
el 1-factor contiene un mlmero par de vértices., y como recubre a G, debemos tener que I VI
es par.
b) Consideremos el grafo de Petenen como se muestra en la figura 1l.48{a). Las aristas

{,,a} {b,c) {d,i) {g,í) {f,h)


proporcionan un 1-factor para este grafo.
e) Hay (5X3) = 15 ract= uno para /4.
d) Etiquetamos los v6tices de K211 como 1, 2, 3, . .. , 2n - 1, 2n. Podemos asociar 1 con
cualquiera de los demás 2n - 1 vMices; entonces, en el caso n ~ 2. debemos encontnr un
1-factor para el grafo K11,,.:l: En consecuencia.

a.. • (2n - l}a..-i. 01 = l.


Tenemos que
a.= (2n - l}a.- , = (2n -1)(2n -3)a.._, • (2n -1)(2n -3)(2n - 5}a.-, = ...
=(2n - 1)(2n - 3)(2n - 5) · · · (5)(3)(1)
(2n)(2n - 1)(2n -2)(2n -3) · · · (4)(3)(2)(1)
(2n)(2n -2) · · · (4)(2)
(2n)!
= 2"(n!)
7. Sí, Blanca puede hacer tal asignación.. Sea X el conjunto de solicitantes y Y el conjunto de
trabajos de tiempo parcial Entonces para cada X E X, y E Y. trazamos la arista {%. y) si el
solicitantezestácalificado para el trabajo de tiempo parcial y . Eruonces grad(r) >: 4 >: g,ad(y)
para todosx EX,y E Yyel resultado se sigue del corolario 13.3.
9. a) (i) Scle.ccionamos i de A¡ para 1 s i :s 4.
{ü) Seleccionamos i + 1 de A¡ para l si :s 3 y 1 de A..
b) 2
ll. Para cualquier subconjunto A de X, sea G.A. el subgrafo de G inducido por los vl:rticcs en A U
R(A). Sieesel númerodearistasenGA>entonccse 2: 4IA I, puesgrad(a) 2: 4 para todo a EA.
Dela misma forma,,,; SIR(A)i, pues g,ad(b),; 5 paratodob E R(A).Asf, SIR(All >: 4IAI
y li(A) = IA 1- IR<A)I s IA J- (4/SJ IAI =(1/S}IAI ,; (1/S)IXI =2. Entonces, como li(G) =
máx{d(A)IA i;; X), tenemos queli(G) S 2.
13. a) li(G) = l. Un emparejamiento mazimal de X eo Y está dado por
{{z,,y,}, {z,,y,J, {z,,y,J, {z,,y,}).
b) Si li(G) =O, existe un emparejamiento ma.timal de X eo Yy Jl(G) = I Yl,o I YI = Jl(G)-
li(G). Si li(G) = k> O,scaA !;; X talque IAI - IR<All = .t. EntoocesA U (Y-R(A)) es un
-74 Soluciones

conjunto iodcpendíente maxima! detamallo máximoenG y ll(G) = IA 1+ 1Y-RC,4.)1 = 1YI


+ ( IAI-IRCA>I >= I YI +8(G),demodoque IYJ =ll<GJ-8(G).
e) Figura 13.27(a): Xi, X1, X1t Y2t J4, Ysl~Figura 13.29: {x,. .X., )'2, y,. J4}.

ie'cicios l. d(a,b) = 5 d(a,c) = 11 d(a,d) = 7 d(a,,) = 8


omplementarios- d(a,f) = 19 d(a,g) = 9 d(a,h) = 14
ág. 696 [Observe que el lazo en d v&ticc g y las aristas (e, a) de peso 9 y(/. <!) de peso S no son
importantes.]
3. a) La arista t 1 se selecciona sícmpre ca el primer paso del algoritmo de: Krusb1.
b) Utilizamos de nuevo el algoritmo de Kruskal y seleccionamos la arista t 2 en la primmi
aplicación del paso 2, a menos que cada una de las aristas eh e 2 sea incidente con los
mismos dos vhtices; es decir, que las aristase1, ~ formen un circuito yG sea un multigrafo.
5.

7. d., el nwn,ro de desórdenes de {l. 2. 3, .. .• •l-


9. Los vátices [en el grafo lineal I.(G)] de<aminados por E' forman un conjunto indcpcndienle
maximal.

capltlllo14
Anlllos y arltm6tlca modular

ección 14.1- l . (Ejemplo 14.5): -a= a. -b =e,~= d, -d = e, -e= b


.lg. 707 (Ejemplo 14.6):-s =s,-1= y, -u=x, -w= w, -x=u,-y = t
3. a) (a+b) +c = (b+a)+c Lcyconmutativade+
= b + (a + e) Ley asociativa de +
= b + (e+ a) Ley conmutativa de+
b) d + a(_b +e) = d + (ab + ac) Ley distributiva de· sobn: +
= (d + ab) + ac Ley asociativa de+
= (ab + d) + ac Ley conmutativa de+
=ab +{d + ac) Ley asociativa de +
e) c(d + b) + ab = ab + c(d + b) Ley conmutativa de+
=ah+ (cd +cb) Ley distributiva de · ~ +
=ab + (cb + al) Ley conmutativa de+
=(ab +cb}+ cd Ley asociativa de+
= (a+ c)b + cd Ley distributiva de · sobre +
5. a) (i) La operación binaria cerradae es asocialiva. Patacualcsquicraa, b,c E Z. tenemos que
(a$b)$c=(a + b-1)$c= (a + b -l)+c-1 =a +b + c-2,

a$(b$c)-a$(b +c-1)-a +(b +c-1)-1-a +b +c - 2.


ycualquier a. b, e E z. tenemos
(ü) Pata la opc,ación binaria ccrnda 0
(a0b)0c =(a+ b -ab)0c - (a + b -ab) + e -(a+ b -ab)c
- a +b-ab +c-ac-bc +abe =a +b +c -ab - ac-bc + abe,
Soluciones S-75

y
a0(b0c) • a 0 (b +c-be)• a + (b +c-be)-a(b + e-be)
• o +b +c-bc - ab -ac +abc • o +b +c-ab-ac-bc +abe.
Eo coosecuencia, lo operación binaria =rada 0 tambitn es asociativa
(ili) PO< 111timo, dados los enteros a, b, e, .....,.,. que

(bEBc)0• • (b+c-1)0• • (b +c -1) +a -(b + e- !)a


• b +c-1 +o -ba - c.a +a •2a +b +c-1-ba-co,
y
(b0a)EB(c0a) • (b + a -ba)EB(c + a -ca)
• (b + a -ba)+ (c+a-ca)-l •2a +b +c - 1-ba-ca.
Poc lo tanto, tambi& se cumple la segunda ley distributiva.
e) Fuera del cero, la dnica unidad es 2, pues 2 0 2;: 2 + 2 - (2 • 2) = O, el elemento \lllldad para
(Z, $ ,0).
d) fur. anillo es un domiaio de inl,gridad, pero no un c:ue,po. Para awesquicra a, b e Z
vemos que a 0b= 1 (delemem>cao) =>a +b- ab= 1 =>o(l - b) s (l - b) => (a- l XI -
b) •O= a= 1 o b • l, de modo que no hay divisores propios de cero en (Z. e, 0 ).
7. Pocdejc,ácioatt<rior, sabcmosqueoecaitunos-lascoodiciones sollrek,mporalasque
s e ~ lasleyosdisttibulivas.Como0 esCODm1131iv2,nos.,...,,...,.,.couoasoiadeCSl3sley<S.
Si .r. )l. z E Z. entoooes
x 0 (1Ellz) • (x 0 1) EB(x 0 z)*
x 0 (1 +z-k) • (x + y - mzy)Ell(x +z - mu)
*x +(y +z -k)-mx(y +z -k) •(x +y -mzy) + (x +z-mxz)-k
=>.i + y +z - k -mxy -mz:z + mla •x + y - mry +x +z - mxz -k
*mkx = x*mk • l*m • k • 1 o m = k • -1, pu,s m,kEZ.
9. a) Veri5.,.,..... una de las leyes cüstnoutivas. Si a, b, e e Q, entonoes
• 0(bE!lc) = a0(b +e + 7)
=a+ (b +e + 7) + [a(b +e+ 7)]/7
=a+ b + e + 1 + (ab/1) + (oc/7) + a,
mientras que
(a 0 b)Ell(a0c) =(a 0 b) + (.Oc)+ 7
=• + b + (ab/7) +a +e+ (ac/7) +7
•a+ b +e +1 + (ab/1) +(ac/7) + a.
Ademú. el número racional - 7 es el elemento cero, yel inverso aditivo decualquiun1lJne.
ro racional a es-14 - a.
e) Para c:ualq\Uer a E Q. a"" a 0 u = a+ u +(tu,/7) = Jl[I +(a/7));: O="• O, puc:su> que a
es arbitrario. Poc lo tanto, el nllmero raciooa1 Oes el elemento unidad de este anillo. Ahora,
sea a E Q, tal que a f' - 7, el elemento cero del anillo. ¿Podemos encontrar b E Q tal que
a 0 h•O;esdccir,talquea +b + (abfl)•0?Enestccaso.a+ b + (ab/7)•0 =bCl +a/7))
= - • => b = (- a)l[I + (af7)J. PO< lo lalllO, l0do mlmero raciooal distinto de - 7 es una unidad.
d) PO< lo parte (e) sabemos que (Q, e , 0) es un c:ue,po. Para verilicat que tambó!n es un
dominio de integridad. seana.b E Q tal~que a 0 b =--7. Tenemos entooces quea 0 h=
- 7 => a+ b + (abnl •-1 => a(I + (b/7)] = -b - 1 => a(1 +bJ =(-1)17 + bJ(7) =>(a+ 7)
(b + 7) • 0 ~ a-+- 1 = 0 ob + 1 = 0= a = -1 oh = -1. En consecuencia, no existen divisores
p-opios de cero (el nómero racion.al -7) y (Q, e , 0) es un dominio de int<gridad.
JJ. b) 1, - 1, ~ - i

13. [•
e d
b1-• • (1/ (ad- be)) [ -ed -bJo ' ad -be +O
,76 Soluciones

15. a) .u=x(t+y)=.rt+.ry =t+y =x


yt= (x +t)t=xt+a=t+t=s
yy =y(t+x)=yt+yx = s +s s 2

tr = (y +.r).t =yx +x:c =s +.r =x


ty =(y +x)y =yy +.ry =s +y =y
b) C.Omo tt=x /:- r=xt. este anillo noes conmutativo.
e) No hay elemcnlo unidad y, por lo tanto, tmlpoco unidades.
d) El anillo no es dominio de integridad ni cuerpo.

ección 14.2- l . Teorema 14.IO(a). Si (S, +, ·) esun subanillodcR,entonccsa-b, ab E Sparatodosa,b ES.


~. 714 R.ocípnx:amente, sea a E S. &tooces a - a = z E S y z- a = - a E S. Ademis, si b E S,
eatonces-b E S,de modo quea - (- b) =a + b E Sy Ses un subanillo por el teon:ma 14.9.
3. a) (abXh~.r') = a(hlr').r'=awr'=a.r'=•y(lr'.r'Xab) = b-'(...a)I, =h-'ub=lr'h=•• por
lo que abes una unidad. Como el inverso multiplicativo de una unidad es Wlico, (abr1 =
l,-4trl.

b) A-'= -1 [ 2 -7)4 a-• - [-21 -2)


5 (AB)- • =[ _ 4 -
9 3415)
(BA)-•=[~: -:J s-•A-•=[_: -~J
5. (-a¡-•=-(•-')
7. a) Los subaoillos SOO (x}, (.r, y} y (.r, y, s, I}.
b) Todos los ,ubanillos de la parte (a) son tmlbién ideales.
9. El conjWl10 S no es v,áo puc>[g gJ ES, parax•y = O.
Para[; ~}[; :] e s,doodex.y.u,wEZ,teoemosque

(i) [; ;]- [ : :]=[::: ;::].unetementodeS,y


(ii) [; ;][~ :]=[:!~ :!;::'.],quetambiálesundemmtodeS.
u. z E S,T=?t e sn T=sn T,;, f.a,h es n T =?a,Ó E Sya,h E T=a+h,ab E S y
a+b,ab E T⇒a+b,ab ES n T.a ES n T~a E Sya E T⇒ -a E S y -a E T⇒ -a E
S n T. Por lo tanto, S n Tes un subanillo de R.
13. Eo caso contrario, existen a, b e S tales que a E 7 1• a ,t. T1 y b E T:z. b f: 7 1• Como Ses un
subanillo de R. esto implica que a+ b E S. Por lo tanto, a + b E T1 o a + b E T2•
Supongamos sin p&dida de gencnlidad que a + b E T1 • Como a E Ti, tenemos que - a E Ti,
de modo que por ser la suma cenada en 71 tc:oemos entorx:es (-a)+ (a + b) = (- a +a) + b =
be T1, una contradicción. Por lo tanto, S C T1 U 72 ~ S C T1 o S C 72 •

15. b) [~ ~] e) [~ ~]
d) Ses un dominio de integridad, núentras que R es un anillo DO conmutativo con elemento
unidad.
17. C.omoza=z, se siguequcz E N(a)y N(a) ,f:. t. Si r i,r2 E ~a).entooccs(r,- r,Ja=ria-r2a=
z-z = z. de modo que r1 -r2 E N(a). Por último, si re N(a) y s e R,entonccs (rs)a = (sr)a=
s(ra) = sz = z, por lo que rs.sr E N(a). Por lo tanto, N(a) es un ideal, por la definición 14.6.
19. 2
21. a) a = au. EaRpuesu E R,dcmodoqueaRf. 11. Siar1.ari E aR.,cntooccsar1 -ar2 = a(r1 - rJ E
all.Además, paraar1 E aRy rE R. tenemosqucr(arJ=(arJr= a(r1r) E aR. Ptrlotanto,aR
es un ideal de R.
Soluciones

b) Sea.a e R,a # z. Entonces a = aw EaR, porloqucaR:s R. Comou e R= aR, u zarpara


llgw, r E R y r = ,r. Por lo WllO. R CS 110 cuerpo.
23. •l f;\(49) bl 7'
e) ~r, d . - 0 (1<, -. -. •l d) 4'
25. a) Los demenlos (1, 1, 0), (0, 1, 2/3), (-S, O, 2) y (0, O, -In) son todos cüviscrcs propios de
cero en R. Por ejemplo, en d caso de(), l. O), COl1Sidenmos delementodistinto deCffl> (O,
O, 1). En este caso, (1, 1,0) ·(O.O, ll• O· 0. 1 · O, O· 1) = (0, O, O).
b) Sea (m. n, s) e R.. f.ntooces (m. n,. s) es un divisor propio de cero si y s6lo si al mmos UDO
de los ad.meros m. n, s no es cero y al menos UDO de los nlimc:ros m. n, s es cero. La
lltenwivaes verque(m, n. ,)es uncüvisorpropiodecerosi y sólo si 1ml + 1• 1 + lsl ~ O
y mns a-0.
d) Sea (m. n. s) e R. Entooces (m. n, ,) es una unidad de R si y sólo si'"• ±1, n = ±1 y s t- O.
(Ademú, ,i (m. n,. s) es un.a wtidad deR teuemosque (m. n. s)"' = (m...1,n...i, r') ==(m. n,
,...).]
'n. b) Si Rtiene un elemento unidad11, dcfinimos á'= 11, para a e R, a,j. i. Si a es una unidad de
R, definimos ,r como (,•-,, para• e z•.

Sección 14.3- l . a)~ 1, 8, IS b)-9,2.13,24 c) - 7, 10,27,44


~ . 721 3. Como a • b (mod n), podemos escribir a = b + kn pan. algiln k e Z. Y'" 1n :;;;) n = lm para
l1gw, 1 e Z. En consecuencia, a= b + .bi • b +(k~m y a ■ b (mod m).
5. Sin es impar, coosid«emos los n - J nl1mefos 1, 2. 3,. : .. n- 3, n-2 y n-1 como (n - 1)12
pares: 1 yC,,-1), 2y(n-2), 3 y (n-3), ... , n -('j') -1 y n-('j'). Luumadecadaporcs
que es ooogruente coo Omódulo n. Por lo tanto,
11,. 4' 1
i • O (mod n). Si n es par, comidera-
mos losn -1 ntlmeros 1, 2, 3, ... , (n/2)-1, (n/2), (n/2) + 1, ... , n -3, n -2, n -1 como
("'2)-1 pares; a saber, 1 yn-1, 2yn-2.3 yn-3, ... , ("'2) -1 y ("'2) + l,yclnwnero
<,,12). Para cada por, la suma es n. o Om6dulo n. de modo que -Z:,'1 ■ (n/2) (mod n).
7. No, 291 3 y 391 S, pero S 918. Además. 2913 y 291 S, pero 49115.
9. a) [17]-• - [831] b) [1oor• = (111] e) ¡m¡-• - (735]
11. a) 16 unidades, Ocüvisores propios de cero
b) 72 unidades, 44 cüviscrcs propios de cero
e) 1116 unidades, Ocüviscrcs propioo de cero
13. [('r) + 2('l') + F,)F,)'}/('l")
15. a) Para n =OICncmOS ICl'• I • 1(-1)', de modo que ICI' ■ (-l)'(mod 11). [Como 10- (-1) =
11. 10 ■ (-1) (mod 11), o 10' ■ (- l l' (mod 11). Porlo tanU>, d n:sultado tambimcs cierto
pan n = l.] Supongamos que es cieno el resultado pwa n = k ~ 1 y consideremos d
caso para k+ l. Entooccs, pucsu>quc 10' ■ (-l)'(mod 11) y 10 ■ (-l)(mod 11), l<DemOs
10'-' = 10' • 10 ■ (- l )'(-1) = (-1)''' (mod 11). El rosultado se cumple cntonca para todo
• e N. por el principio de inducción matemática.
b) Six..r... ... Z2X1 Xo= X.. • IO"+x...,, • 10'""'+ • • • •~ • 102 +.r1 • 10+.rodenota un entero c.on
(n + 1) dí¡itos. mtooces
.r.x.-1 ... .r, .r,x.,• e-ir.r. + <-o·-,.r.-, + -.. + .r2: - .r, +Xo cmoc111).
Donos1raci6n:
.r,.z_, ... z,.r, z.o• z..· 10- + X.-1. ur-1 + ... + .r2· 102 + .z, · 10+.to
•z.(-1r +z..-,(-1,' + · · · + z,(-1)' + z,(-1) + %o
- (-1rx. + (-ir-• z._, + · · · + z,-z , +x. (mod ))~
e) (i) da ) (Ü) d a 4
17. 9
1'. Seant = mcd(a.•),h =mcd(b,n). (a ■ b (mod •ll => [a•b +kn, pora~k e ZJ => [8lb y
0

hlaJ. !,lby 11•1 => 1lh; [hl• Yhl•l =>hit-Como g, h >0, se si¡ucquc g = h.
21. a) 112 b) 031-43-3464
S-78 Soluciones

sección 14.4- l . s-+O, t-+ 1,v-+2, w-+3, x-+ 4,y-+ S


pilg. 728 3. Sean ()?, +, ·), (S, EB, 0) y (T, +', ·? los anillos. Para cualesquiera a. b E R, (g • IX• + b) =
(g(f(a + b)) =g(f(a) e f(b))=g(f(a)) +' g(j(b)) = (g •IX•)+' (g olXh).Aá:mis, (g •IX•· b) =
(g(j(a · b)) =g(J(a) 0/(b)) =g(J(a)) ·' g(f(b)) =(g •IX•)·' (g o/Xb). Poc lo tanto,g o/es un
homomorfismo de anillos.
S. ■) Como/(%,) = z,, sesigucquez,E Ky KH. Si.r. y E K,entonces/(z-y) =/(x + (-y))=
/(x) EB/(- y) =/(x)e/(y) =z,9z,= z,, de modo quez-y E K. Pocóltimo, siz E Ky ,e
R, entonas/(n) =f(r) 0/(z) =f(r) 0Zs=Zs yf(xr) =/(x) 0/(r) =z,0/(r) =z,, de modo
que rx. xr E X. En consecuencia. K es un ideal de R.
b) Elmlcl<0es (6n I • EZ }.

7. •> z (cnZ,.) f(z) ( cnZ. x Z,) z (cnZ,.) /(z) ( cnZ. x Z,)


o (0,0) 10 (2, 0)
1 (1, 1) 11 (3,1)
2 (2,2) 12 (0,2)
3 (3,3) 13 (1,3)
4 (0,4) 14 (2,4)
5 (1,0) 15 (3,0)
6 (2,1) 16 (0,1)
7 (3,2) 17 (1,2)
8 (0,3) 18 (2,3)
9 (1, 4) 19 (3,4)

b) (i) /((17)(19) + (12)(14)) = (1,2)(3,4) + (0,2)(2,4) = (3,3) + (0,3) = (3, 1), y


¡-'(3,1)= 11
9. •) 4 b) 1 <) No
11. ■) 4,{i1) =p'(l -(Íl) = p(p - 1)
b) 4,(p)·$(p)= p-1)'
<) $(pq) = pq(l -~)(1-¼) = (p - l)(q - 1)
el) $(p) · $(q) = (p - l)(q - 1)
e) $(p')=p'(l-})=p'-p'=p'(p-1)
r) p(p-1')'
g) (p - 1)'
b) (p' -p)(q -1)
l)(p' -p)(q-1)
D (p - l)'(q - 1)
k) (p - l)(q - l)(r -1)
1) (p-l)(q-l)(r-1)
m) (p - l)(q - l)(r- 1)
D) (p- l)(q- l)(r-1)
o) (p· - p•-')(q' -q•-•>v- r•¡
13. No. Z. tiene dos unidades, mientras que el anillo del ejemplo 14.4 sólo tiene una.
15. Es mejor usar la forma del programa. puesto que el producto de matrice:5 es DW preciso que la
inversión de una matriz. Si se calcula primero la inversa. entonces el error de redondeo se
puede incrementar al elevar la inversa a la cuarta potencia.

jercicios l. ■) Falsa.S=R=ZyS=Z•.
omplementarios- b) Falsa. SeanR=Zy S= (2z I z e Z}.
ilg. 732
e) Falsa.S=R=M,(Z)yS= {[~ gJ¡.ez}-
S-79

d) y e) Verdaderas.
f) Falsa. El anillo (Z. +,•) .. UD subanillo (pero no UD cuerpo) en (Q. +, -).
1) Falsa. Puacualquierprimop, (ai(p')la. n E Z , n,: O} O$ UD subanillode (Q. +, ·).
h) Falsa./(6) = 12 = /(2 · 3), pero/(2) ·/{3) = (4X6) 24. =
O Falsa. Coosidc=nos d cuerpo de la 1ab1a 14.6.
D Verdadera.
3. a) [a+ a • (a +a"f=a2+ a1+ tl+a1• (a +a)+ (a+ a))=> [a+ a• 2a =z;J. Por lo tanto,
-a =a.
b) Para.cualquiera E R,a +a= z ⇒ a • -a. Paraa,.b e R, (a +b)• (a+b)2aa2+ab+ba+
IJ• a+ ah+ ha+ b => ab + ba = z => ab = -ba • ba. por lo que Res conmutativo.
5. Como az • z• ui para todo a e R. teocmos que z e e y e-;, e. Si x. y e e, auonces ex+
y)a •
:uz +ya •<tr+ay=a(<+ y), (-"l'la =x()'a) = ,(ay) • (xa)y= (ax)y • a(xy) y (-z)a•-Cxal=
- (<tr)= a(- z), para todo a E R.de modoquez+ 7,,ry,-z E C. Encomecacncia, Ces un
subanillo de R.
7. Como m. n son primos relativos, podemos escnl>ir 1 = wu + ni donde 1,1 E Z. Como m. n > O.
se sigue que uno de los nllmcros so I debe ser positivo y el otro neptivo. Supon¡amos (s.in
ptroida de gcnenlidad) que, es negativo, de modo que 1 - ms =n1 > O.
Entonca d'= lt' => (a'f • (lf'Y =>Ir'• lt" => - • b'- => a(d'f" = b<.ll'f". l'ao como
-s>Oy ti'= 11', icncmos (a"'f" = (ll't". En c:oosca,cncia,
(1(.-¡<-•>• (b~)<- •> ,j zJ/\(o(a-)'-''.• b(bj<-•>J)~• • b,
ya que podemos usar la ley de canccbci<la parad producto m UD domimo de int.cgódad.
9. Seanx=a1 +b.,y=a,+b,.cooa"a2 EA y "1, "2 E B. Entoocesx- 1=(0 1- az) +(b,-bJ E
A. +B. Si rE Ry a +b E A +B. oona E A y b E B,entooccs m EA, rb E By ,(a+ b) EA
+ B. De manera similar, (a+ b)r E A +By A.+ Bes un ideal de R.
u. Considctemos los námc:ros z,, .X¡ + X2, x, + Xz+ X1o ••• ' X¡+ Xz + x, + ... + X.. Si uno de estos
nt1meroses ooogruc:nte coa O módulo~ se sigue el resultado. Si no. existen 1 :Si <j :s n tales
que (%1 +.ti+ • • • +.rJ • (%1 + • • • +x,.+z" 1 + · · · +.r;)(modn). Porlo tanto. n divide a<r.,.1 +
···+•~-
13. a) 1250 b) 19S3 e) 3/16
U. a) O b)S c)O
d ) 5, sí d óltimo dígito den es 5;
O, si el dltimo dígito de n es O.

capitulo 15
Álgebr• _...,.. y funciones
de conmutKl6n

Sección 15.1- 1.a) l b)l e)! el)! e)l


pag. 743 3. a) Z' b ) 2""'
La siguiente tabla muestra el accimiento del m1mero de ñmciO!le$ booleanas respecto del
crecimiento del rulmero n (de variables booleanas).

n 2""'
1 4
2 16
3 256
4 65,536
5 4,294,967;196
6 18,446,744,U73,7®,SSl,616
Soluciones

S. a) f.n.d. xyz +;cyz +ryZ+xyZ+XyZ


f.n.c. (z+y+z)(z+y+z)(z+y + z )
b) /= ¿m(2,4,5,6,7) • IlM(0,1,3)
7. a) r' b) T e) T d) '1."' e) 2' í) 2"; 2"
9. m+k=2"
11. a) y+xZ b) .x+y e) w.r+ z d) .r1 +.r1 + x,+x,+•· ·

13. •> (i) f g h fg Jh gh fg+Jh+gh fg +Jh


o o o o o o o o
o o 1 o 1 o 1 1
o 1 o o o o o o
o 1 1 o 1 1 1 1
1 o o o o o o o
1 o 1 o o o o o
1 1 o 1 o o 1 1
1 1 1 1 o 1 1 1

La altem.ativa es fg + Jh = (fg + J)(.fg + h) = (/ + /)(g + J)(/g +h) =


l (g + /)(fg +h) • fgg + gh + lfg + Jh =fg +gh + Og+ Jh =fg+ gh + /h.
(ii) fg + fg + Jg + ¡g=f(g + g)+ J<.g +g) =/-l + /-1=/ + / = 1
b) (i) (f+g)(/+h)(g+h)•(j+g)(/+h)
(ii) (/ + g)(/ + g)(/ + g)(l + g) = o
15. a) /$/=t;/$/= l ;/$1=/;/(ijt=/
b) (i) /e g = O.,.Ji+ Jg =O=> fi =Js =O. [/= 1 y .fí =O] => g = l. [/= Oy ]g = OJ =>
g = O.Por lo tanto,/= g.
(iü) JEf¡g=/g + ls= /g + fg= fg+ Jg =f$g
(iv) &te es el ónico rcsuJtado queoo es verdadero. CUando/ticned valor 1, g ticned
valor Oy h el valor 1 (og tiene el valor l y h el valor 0), entonces/$ gh tiene valor 1
pero (f/Ii g'IJIII h) tiene valor O.
(v) fg ®fh =ftfh +tifo=(]+ g)fh + fg(j + h) =lfh + fgh + f!g + fgh =
fgh + fgh = f(gh + gh) = f(g$h)
(vi) /$g =¡g + fg =fg+¡g =f(i¡g
f$g = fg+ Jg=(/+g)(j+g) • ¡g +fg • /$g
ección 15.2- l. a) z (ijy • (z+y)(ly) , ~ _
~g. 754
~ x@:,

b) xf ; =[)-----C>--- ;¡ e) X+ y ; =[)-----C>-- rr,

3. a) , ---Q>--,

e) , ~
•• y
y
Soluciones S--81

S. f(w,x,y, z)•W.í)'Z+(w +x+y)z

a) La salida es (x + Y}(% + y)+ y. Esto se simplifica a x+ (Yy) + y =x +O+ y =x + y y nos


proporciona la red equivaJence. 11W sencilla. de la parte (a) de la figura.
b) Encstecaso,lasalidaes (z+y)+(:ry + y),qucsesimplifica a xj + r¡ + y = Y:y+r¡ + y •
?(y + Y) + y = :r(I) + y = :t +y. Obtenemos la r«I equivalente. mis sencilla, de la pane (b) de
la figura.
9. a) /(w, z,y)•xy +zy b) /(w,z,y)• z
e) /(w, z,y,z) =zz + ff
el) f(w, x,y,z)=wyz+zyz + wyz+zyl
e) f(w,x,y, z) = wy + w.íz + .ryz
() /(u, w,x,y,z) =v'w.ry.f +uw:rI +v.í)'Z + WXz +\IW)' +vyz
11. a) 2 b) 3 e) 4 d) k +1
13. a) lr'(O)I- lr '(l)l • 8 b) lr'(O)I- 12, lr'(l)I = 4
e) lr'(O)l• 14, lr'(l)l- 2 d) lr'(O)l• 4, lr '(l)l= l2

Sección 15.3- l . uu+woy+uxz+uy:+wz


~g. 760 3. Las panes (a) y (e) soo ciertas.
b) En la figura 15.12. sean D,= (c. di, D,• la. g}.
d) SeanD,= (a), D,• (d) en la figura 15.12.
S. a) /(w,x,y, z)•z
b) /(w,x,y, z) =:iY? +zyz + zyz
e) /(u,w,x,y,z) - vY?+ Wfyz+ üWZ+üry

sección 15.4- 3.a)30 b)30 c)I d)2l e)30 1)70


~g. nl S. a) w s O ⇒ w ·O= w. Pero w · O• O. poc la pane (a) del teorema lS.3.
e) y s z ⇒ Y?=- y y y s? = y? ., y. Por lo tanto, :,• yz = ()'?)z • y(?z) s y• O= O.
7. y Sx
9. Porcl teorema IS..S(a), si X i.X: $00 átOIDO$ distintos yX1X2 1, o. COI.onces Xi •X1X2 • X2X1 = X21
una contradicción.
ll. No. Sean 'U= (a. b, e) y 9l • ~(¾ U . n .- . t . 'Uj. Si 9!,= lt, 'U, la}, {bl, la. bl).
entonces!11 satisface las condiciones dadas, pero no es una su~cbra de SI. Por' ejemplo, {a}
E !i,.
13. a) /(O) •/1.,d) paracadax E 9!,./1#) •/(z)/(%1 • /(z)/(x) = O.
b) Se sigue de la panc (a) por dualidad.
e) xsy~zy =x~/(zy) •/(x)~/(z)/(y)•/(x)~/(z) :s/(y)
e) S. ;. lf ⇒/(S1) -1, e. Seanxhy1 E / (S1)conx.,y1 E S1 y/~1) = x1,/C,1) = }'?. Entonces
/(x1+~ =
/(x1) + ft:11) Xt + ) '2 Y X1 + Y1 E S1. Por lo tanto, xi+ Y2 E /(S1). Además,
Yi' = f(x1 ) = /(Y1) y 00mor1 Es,. se sigue quer 2 E f(S1). Porel ejercicio lO(d).f(S 1) es
una suW!gcbra de 9l,.
15. a) /(zy) • /(i +y) • /(i +Y)• /(i) T /(y) • /(i) ·/(Y) • /(f)·/(y) • /(x)·/(y)
b) Sean SI,, .!iz álgebras booleanas tales que/: !l1 -+ Si2 cs UDO a uno y sobre. Entonces/ es
un isomorusroo si)m = /(z) yf(zy)=f(x)/(y) para todos.r.1 E ~,- (Se sigue de la pane
(a) por dualidad.]
17. Para cada 1 !Si :S: n, (x1 +Xz + · · · + ~ ax1x,+xiX¡-+ • · • +x,.1 .r1+:C.X,·+X1o,1.r, + ··· +X.X.,=
0 + O+ · ·· + O + .r; +O+· ·+O= .r;. por la pane (b) del teorema 15.5. En consccucncia, el
teotema 15.7 implica que (x1 + x 2 + · · · +x.)x=x para todox E SI. Como el demento uoo es
dnico (a partir del ejercicio 12). concluimos que I a x1 + x~ + • • • + x,..
S-82 Soluciones

Ejercicios l. a) (i) Cuandon = 2,x1 +-,denotalasumabooleanadex1 y-,:. Paran 2: 2, deíinimos.z.+:iz+


complementarios- + · · · +x. + x..1 de manera recursiva como (.r1 •~+ · · · +x.) +x..,. (Podemos dar una
pág. 774 definición similar para el producto booleano.) Paran= 2. x 1 +X:2 =X1 X¡ es cieno; &la
es una de Jas leyes de DcMorgan. Supongamos que el resultado es citi'to paran = .t(2: 2)
y consideremos d caso n = l. + l.
(x, +x2 + •"• + X.t + X,t+t)= (.:r1 + X2 + ••"+ X.t) + x •• ,
= (X1 + X:+ · · · + X.t)X.t...1
=xiii· · ·zil';':i"
En conseaiencia. el resultado es verdadero para todon 2: 2. por el principio de indueción
finita.
(ii) Se sigue de la parte (,) por dualidad.

(iü) {I:,.1x;)(TT:..1x;) =(.:r1+.:r1+ · · · +zJ(Xi'" +i;'+···+i;). Esto tiene el valor Osi.x;-=


Opara todo 1 :Si :s n,o si :ii' =O para todo 1 S: i :s; n. En ambos casos, x 1i;' +x2zi +
· · · + x,, Xi tiene el valor O. En los demás casos, la expresión tiene el valor 1 y existen
x.,. x,.1, 1 :s i s n - 1, tales que uno tiene el valor O y el otro l, o x. tieae el valorO(l)
mientras que x1 tiene el valor 1(0). En estos casos. .r1 zi' +.:r1 ij + · · · + x.ii también
tieoe el valor l.
b) (11".1.r,)(L'_1X1) = (z1 + Xz)(x:z +xi)··· (z..- 1+ X.:)(x,. + xi)
3. FJ.la sólo puede invitar a Natalia y a Carmen.
S. Six :S z y y s z., entonces, por el ejercicio6(b)de la sección 15.4, teocmosx + y s .t + z. Por
la ley de idempotencia. tenemos z + z = z. Recíprocamente, supongamos que x + y :s z. Tene-
mosque x .:s x + y. pues .t(;r + y)= x + ry (por la ley de idempotcncia) = x (por la ley de
absorción). Comox :S x+ yyx+ y S z.tenemosx :s z. pues un orden parcial es transitivo. (La
demostración de que y :s z es similar.)
7. a) xsy:::)x+Xsy +.i:::>lsy+ X:::>y + .i=.i+y=l. Recíprocamente,
z+ y= l~x(.\'+y) =x · 1~.ti'(•0) +xy =x~.ry =x~x sy.
b) xsy~xy =x~xy =(xy)y •x(yy)=x·0=0. Recíprocamente,
xy =O~x =x-1 =x(y +y) =xy +.ty =zyy x = .ty~x sy.
9. a) f(w,x,y,z)=wz+xy
b) g(u,w,x,y,z) = VWyz+xz+wJZ+XJZ
11. a) 2""-,, b) 2'; 2'.. '
13. a) Sin =60,existcn 12div:isores, y ningán álgebra booleana contiene 12 elementos., pues 12
oo es potencia de 2.
b) Sin= 120, existen l6div:isores. Sin embargo, six=4, entonces X= 30y X · X= mcd(x,x) =
mcd(4, 30) = 2, que no es el elemento cero. Asf, no $C satisfacen las leyes de los inversos.
15. Si es: a ,cntoncesac =c,de modoqueah+c=ab +ac=a<.b +e). Recíprocamente, siah+c=
a(b +c)=ab +ac,cntonccsac =ac+O=ac +(ab + 00) = (ab+ac)+ab = (ab +c)+aó =e+
(ab+ ® ) =e, yac=c ⇒ c :s a.

C:,,pltulo16
Grupos, teom de la coclific.oci6n
y método de enumeración de Polya

ección 16.1- l . a) Sí. El neutro es 1 y cada elemento es su propio inverso.


ág. 783 b) No. El conjunto no es cerrado bajo la suma y no hay neutro.
e) No. El conjunto no es cerrado bajo la suma.
d) Sí. El neutro es O; el inverso de ICn es 10(-n) o - lOz.
e) No. La función/: A -+A tal que/(x) = 1 para todox EA no es invertible.
f) Sf. El neutro es IA y d inverso de g: A ➔ A es g"": A ➔ A.
el sr. El neutro es O; el inverso de a/(2') es (- a)/(2').
3. La resta no es una operación binaria (cerrada) asociativa en Z. Por ejemplo. (3 -2) - 4 = - 3 fl-
5 • 3 - (2 - 4~
S. Como.r, 1 E Z ~ .r +y+ 1 E Z. la opc:ncióo es una opcncióo binaria c:cmda (o Z es cerndo
bajo •). Para w.~yE Z, wo(.ro y)sw o (.r+ y+ l)=-w+(.r+y+ I)+ l • (w+.r + l)+ y + 1 :z
(w o.r) o y, de modoquelaoperac:ióobia.ariaesasociativa. Ademá.s,zo y=.r + y+ 1 =y+.r+
l z: y o .r, para rodos .:c. y E Z, por lo que o tambitn es conmutativa. Si.re Z, entonces x o (-
1) •x + (-1) + 1 = x( • (-1) oz), de modoque-1 es el elemento neutro de o, Poróltimo,,para
c:adax E Z, tcDcmOSque-.r-2 E Z y.ro (-.r-2) =-.r+ (-.r-2)+ 1 =-1[ = (-z -2)o z), por lo
que-.r-2 es el ínvenodezbajo º· Encoosec:ueocia. (Z. o) es un grupo abcliano.
, . u,.-¡1,3, 7,9,11,13, 17,19)
u,.• {1,5, 7, 11, 13, 17, 19,23}
9. a) El resultado se sigue del teorema 16.l(b) pues (~)-1 y a son inversos de a-1•

b) (b-, • -')(ab) • b-'(•- '•)b • b-'(•)b: b-'b ••y (ab)(b-•• -')


- a(bb- 1)a- 1 - o(e)a- 1 zoo- 1 - t
Asf, b""a-• es un inverso de ab; por el teorema 16.l(b), (ab)~-b~a~.
11. a) {O}; {0,6}; {0,4,8}; {0,3,6,9}; {0,2,4,6,8,10}; ~"
b) {l}; {l,10); {1,3,4,5,9); U,
e) {,ro}; {1ro, 1:r1,1ti}; {1ro,r1}; {1ro,r:}; hro.rJ.}; S,
13. a) Hay 10: cinco ro<aciooes 000 un ángulo i(.7Z'). O ,: i ,: 4, y cinco rellexiones r,sJ)<dO de
recw que contienen un ~ce y el pun10 medio del lado opuesto.
b) Para un n.ágooo re¡ular (n ~ 3), ex.iStCD 2n movimientos rígidos. Existen n rotaciones con
M¡Ulos i(3(:Jrln). O S i ~ n - l. &istc:o n reflexiones. Paran impar, cada reflexióa es
res.pecto de una recta que pasa por el vbt:ic:e y por d punto medio del lado opuesto. Paran
par, aislen nl2 reflexiones respeao de rmas que pasan por los vmiees opuestos y nn.
reflex.iooes íe$pectO de rectas que pasan por los puntos medios de los lados opuestos.
IS. C,,mo,g:g,paratodog E G,so,;guequc, E Hy H + • . Si.,; ye H,cntoocesxg =gxy yg=
gypara IOdog E G. En consccu,ncia, (z¡,)g=-'(yg) =>(gy)=(zg)y= <,z)y:g(zy) para IOdo
g E G, y leDeDlOS que.tyE H. Poráltimo, pan. cada.r E H,1 E G,.rg~•g-1.L De este modo,
<zr'>~• e,~ z,' o gr=:r1 y.re H. Por lowno. Hes W1 subgrupo de G.
17. b) (i) 216
(ii) H,= {Cx. O, O)iz E Z.} es Wl sub¡rupode orden 6
H,= {(z.y. O)lz. y E Z,.y• O, 3} es Wl subgrupo de orden 12
H,• {(%,y, 0)lz.1 E Z.l tiene orden 36
(üi) -(2, 3,4)•(4,3,2); - (4,0,2) • (2,0, 4); -(5, l , 2) = (1,5, 4)

Sección 16.2- l . b) /(a-') ·/(a) =/(,r'-a) •/(•,l=•,y/(a) ·/(,r') =/(a • a-') •/(•,l =•,,. de modoque/(a-')
pjg. 790 es u.o inversode/(a). Por la unicidad de los inversos (feorema 16.Jb), se sigue que/(a-') •
[/(a))"'.
3. Sía + b./3 = c+d ./3 .en1oocesa-c=(d-b)./3. Pand- b+ O, tenemosque./3 = (a-e)/
(d-h), lo que contradice el hecho de que .fj sea irracional. Por lotanto,d-b • O.de modo que
d•bya-c=(d - b)./3 •O ⇒ a=c.Reclprocamente. {(a•c) A (b=d)} ⇒ ((a•c) A (b./3 =
d.,/JJJ ⇒ a+ b./3 •e +d./3.
S. /(0) •(0,0) /(1)•(1,I) /(2) • (2,0)
/(3) • (O, 1) /(4) • (1, O) /(5) • (2, 1)
1. /(4,6) • -5g, + 3g,
5-84 Soluciones

9. a) o(1r,) = 1, o(-.,) =o(1r,) =3, c,(r,) =c,(,,) =o(r,) =2


b) (Vtase la Fig. 16.6) c(lG,) = 1, t.(x,) = e(",)= 4, t.(">) = c(r,) = c(r,l = c(r,) = c(rJ = 2
ll. a) Los elementos de orden 10 son 4, 12, 28 y 36.
13. Z: = (2) = (3); :U = (3) = (5); :U, = (2) = (6) = (7) - (8)
15. Este resultado es falso. El grupo cuatro de K1ein del ejemplo 16.16 proporciona un
contraejemplo.
17. a) (Z,,, +) = (1) = (5) = (7) = (11)
(Z.♦, +) = (1) • (3) =(5) = (7) = (9) = (11) = (13) a (15)
(Z,., + ) = (1) = (5) = (7) = (11) = (13) = (17) = (19) = (23)
b) Sea G= (a'). Como G = (a), tenemos que a= (ao/ para algán s E Z. Enlonces a'--=•de
modo que 1-k.s= m. pues c(a) = n. 1-k.s=m => 1 =k.s+ o,=> mcd(k,n) = l. Rcdproca-
merue, seaG= (a),doodea' E Gy mcd(k.n)= l. Entooces (a') C G. mcd(k,n) = 1 => 1 =
k.s +"'·paras, I e z =>•=•'=a"· •= (a')'(.do/= (a')'(.eY= (a')' e (a'). Porlo tanlo, Ge
(a'). Así, G =(a') o a' genera a G.
e) .,;n).

iección 16.3- 1. •> {(½ i ! t). O 1 t :). (! f l ;), G i i :)}


~g. 792
b> o i : ;)H=W i 1 :),C! 1 i n.o ; i ;),u f : ;n
C! i 11)H=W i : n.o 1 t ;).ci f 1 :),(! i ~ -nl
G i ! ;)H-{0 i t :),(~ ! l t),(! f J n.o i ! 1)}
(l i l !)H- W ¡ i n.o f ! l).(! i t ;),G i l !)}
G 1 1 ;)H={U f ~ !).0 i ! 1),(! i t ;),(l 1 l D}
(: i J !)H=H ·
3. 12
+
5. a) Como (0, O) EH, se sigue que H f. Sean (a. O), (b, 0) E H. Entonces (a. O)+ (b, O) =
(a+b, O) E H. Además,(-a,0) E Hpancada(a, O) E H. Porlo Wlto,H es unsubgrupo.
b) Cada el.ase consta de los puntos de una recta borizontal.
7. Porel tcor=adel.agrange,sabemosqoe IKI =66(=2 ·3 • ll)dividea IHI y que IH I divide
a IGI = 660( = 2'· 3-5 · 11). En consecuencia,comoK# Hy H+ G, tenemos que IHI es 2(2·
3 · 11) = 132 o 5(2 · 3 · 11) = 330.
9. a) SeanE={:ill),a =0~~).~=(~{;)ya=n~~).

. .• . a p 6

.•
a p 6
a a 6 p
p p 6 a
6 6 p a
Por el teorema 16.3, Hes Wl subgrupo de G. Y como los elementos de la tabla anterior son
simhricos respectO de la diagonal que va del exm:mo superior izquierdo al extremo infe-
rior derecho, tenemos que Hes un subgru¡x, abeliano de G.
b) Como JGI =4! = 24 y JHI = 4, existen 2414 = 6 clases larerales w¡uierdas de H en G.
e) Consideremos la función/: H ➔ Z, x Z, definida po,
/(•) = (0,0), /(a)=(l,O), /(jl)=(O,l), /(6)=(1,1).
Esta función/es inyedivay sobre; pancada.t,y e H, tenemos que
f(x· y)= /(z)@/(y).
En consecuencia./es un isomorfismo.
~uciones S-85

(Nota: Podemos dar otras respuestas posibles en este caso. De hecho. podemos definir seis
posibles isomorfismos.)
U. •> SiHesuosuhgrupopropiodcG,cruooces.po,el-=adcLagrance, IHI es2op.Si
IH I • 2, cotooccs H = {<. x},dondcr= •• de modo que H =(,). Si IHI =p, sea y E H.
y t t. Emooces <(y) = p. de modo que H = ()').
b) SeazEG,z#e.Emooces G(,) =pn o(x) =J>'. Si c(,) =p.cnxooc:<$ 1(,)1 = p. Si o(z)=
J>',cntonces G =(,) y (,')es 1111 subgrupode G de orden p.
13. b) Seax EH n K. Si d orden de.res r.entonccsrdebedividir a m y an. Como mcd(m, n),.
1, se si¡uequer• I, de doDde.r=e y H n Kz {e}.
15. •l En <Z:, •),
w-p-1 elementos, asf, po<el cjcn:icio 10, pancada (x) E <Z:. -),
[zr-'•
[1] o.r.,..1 • l(modp), o:x' • x(modp). Para cada a E Z , sipla, entonces a •0(modp) y
a's0•a(modp). Sip f a,eruooces awb(modp)donde 1 $ b Sp - l y a'•b'• b •
a(modp).
b) En el pupo G de unidades de z. hay +(n) unidades. Si a E Z y mdc(a, n) = 1, entonces
[a) E Gy [al""• [l)o.,..• l(modn)
e) y d) Estos resultados se siguen de los ejercicios 6 y 10. Son casos particulares del ejerci-
cio 10.

Sección 16.4- t. a) < = 0001001 b) , = 1111011 e) c = 0101000


pág. 797 3. •l (i) D(l11101100) = 101 (ü) D(000100011) = 000
(iii) D(0l0011111) = 011
b) 000000000, 000000001, 100000000 e) 64

Secciones 16.5 y l . S{l01010, 1) -{101010,001010, 111010, 100010, 101110, 101000, 101011)


16.~g. 804 S(ll1111, l) • {111111,011111, 101111, 110111, 111011, 111101, 111110)
3. •> 1,S{x, 1)1• 11; IS(x,2)1• 56; IS(x,3)1• 176
bl IS(x,k~ • 1 + (;) + (;) + · · · + (:) - ~ .. (:)
5, a) La distancia mínima entre las palabras oodificadas es 3. Fl código puede detectar lodos los
errores de peso s 2 y puede~ todos los errores simples.
b) La di5wlcia mfmmacutrc Las palabns codificadas es 5. El c6diSo puede clctt,ctar todos los
cmxa de peso :S 4 y puede ccrregir todos los cmns de peso s 2.
e) La distancia mínima es 2. El código detecta todos los errores simples pero no tiene capaci•
dad de corrección.
7. •l C= {00000, 10110, 01011, 11101). La di,w,cia mfmma entre las paW,r» codificadas es
3, de modo que el código puede dctcctar todos los moccs de peso :s 2 y pocdc comgir
todos los errores simples.

b) H = ¡: ~ ~ ~ ~1
O I O O 1
e) (i) 01 (ü) 11 (v) 11 (Y!') 10
Para (üi) y (iv), el síndrome es (111)9. que noes una columna de H. Si suponemos un doble
error. si (111)"= (110)"+ (001)", eniooces la palalndccodific:ada recibida es OI (pan (úi)J
y 10 (par:a(iv)J. Si ( lll )"z (011)"+ (100)", obtenemos IO (pan (úi)J y 01 (pan (iv)t
9. G = {/1 IA.J, donde la es la matriz identidad multiplicativa 8 x 8 y A es u.na columna de ocho
uom. H • WI IJ = (111111111 IJ.
U . Compa,e la matriz geocn,dor> (para la verificacióa de paridad) del ejereicio 9 coa la matriz
generadora (para la verificación de paridad) del ejercicio IO.
5--86 Soluóones

>ecciones 16. 7 y l. F,); 255


16.8-pág. 812 J. a) Síndrome Uderde clase
000 00000 10110 01011 11101
110 10000 00110 11011 01101
011 01000 11110 00011 10101
100 00100 10010 01111 11001
010 00010 10100 01001 11111
001 00001 10111 01010 11100
101 11000 01110 10011 00101
111 01100 11010 00111 10001
(Las últimas dos filas no son únicas.)

b) Palabra recibida Palabra codirkada Mensaje decodifica.do


11110 lOÜ0 10
11101 11101 11
11011 01011 01
10100 io110 10
10011 01011 01
10101 11101 11
11111 11101 11
01100 00000 oó
s. a) Ges57 x 63;Hes6 x 63 b)Latasaes ¾f.
7. •l (0.99)' + G)(0.99)'(0.01) b) ((0.99)' + G)(0.99)' (0.01)]'

ección 16.9- l. a) -rrr =-


~g.819
(c, c, c, c. c, c, e, c, c. c., Cu Cu c., c,. c., c,j
c, c. c, c, c, c, c. c, c, c,. Cu c,. c., c., c., c,
,: -
(c, c, c, c. c, c. e, c, c. c.. Cu c., c., c,. c., c'j
c, c. c, e, c, c. e, e, c. c,. Cu Cu c.. c,. c., c,
b) (1rl 1t=

(C·c, c, c, c. c, c. e, c, c. c,. Cu c., Cu c,. c.,


c, e, c, c. c. c. c, c, Cu c,. c,. c., c., c,. c'j
c,
=c1rn-1
<) 1r; r: =
(c, c, c, c. c, c. c, c, c. c,. Cu c., c., c,. c., C,j
c, c, c. c, e, c. c. c, c, Cu c,. c., Cu c., c,. c,
ª<1r,,.r
J. a) c(a) • 7; o(~)= 12; ch)= 3; o(~)• 6
b) Sea a E S., con a= c 1 c2 ••• cb un producto de ciclos disjuntos. Entonces ll(a) es el mcm
de f(c,), f(c,l, ... , f(cJ, donde f(cJ es la longitud de c. para 1 si s l:.
s. a) 8 b)39 7. a) 70 b) 55
9. Figura triangular. a) 8 b)8
Figura cuadrada: •l 12 b) 12
ll, a) 140 b) 102 13. 315
Soluciones 5-87

SKá6n 16.10- l. a) 165 b) 120


pjg. 823 3. Figura triangular. a) 96 b) 80
Figura cuadrada: a) 280 b) 220
Figura bcugooal: a) 131,S84 c)70.l44
5. a) 2635 b) 1505
.
·o·
<) •

:o:
. 1


V

7. a) 21 b) 954
+
e) No: l:::z 21 ym =21, por loqucbn= 441 954 =n. En. este caso, la posicióo de cierta arista
debe c:casidcnne rospcdo de la posición de los v&ticcs. Por ejemplo,

.:□··.. no es equivalente a IR□: aunque R


os csequivalente

·o· Ro.• I
1 1 N 1 N

·a esequivalcntca
1 N

Secá6n 16.10- l. a) (i) Y (ü) r' + w'+ r'w+:zrlw'+rw'


pjg. 828 b) (i) (1/4)[(, + b + w)' + 2(r' + b' + w') + (r' + b' + w')'J
(ü) (1/S)[(r + b + w)' + 2(r' + b' + w') + 3(r' + b' + w')'
+ 2(r + b + w)'(r' + b' + w')J
3. a) 10
b) (1/24)[(, + w)' + 6(r + w)'(r' + w') + 3(r + w)'(r' + w')'.+ 6(r' + w')'
+ 8(r' + w')'J
e) 2
5. Sea v = verde y o= oro.
Figura triangular. (l/6}((v + o)'+ 2(v + oXv'+ o')+ 3(v + ol'(v'+ o')]
Figura cuadrada: (l/8){(v + o,S + 2(v + o)(v"+ o')+ 3(v + o)(v1+ o-1)2
+ 2(v + o)'(v'+ o')]
Figura beu¡onat (l/4X(• +o'f+ 2(v + oXv'+o')'+ (v + o)'(v'+o')'J
7. a) 136
b) (1/2)((, + w)' + (r' + w')' ]
<) 38; 16
,. e··:·•¡

Ejercicios l . a) Como/(t.,)=e.. se~i\l<qu<<c E Ky KH. Si,; y E K,cruonces/(;c) =/(y)= ••yf(;cy)=


complementarios- /(.x)/ó) ze•e•= e,r por loquezy E K.AdclIW, parax E K,/(,;c1) a V(x)t 1 =- e; 1 ze., por
pjg. 833 ,Jo que .r' E K.. Por Jo tanto, K es un subgrupo de G.
b) Si x E X, entonccs/(.r) • e•. Para todo g E G,

Por lo WllO, para todo x E K, g E G. leocmos que gzg"' E K.


3. Sean a. b E G. Entooccsa'b'=ee = e• (ab)'=abab. Peroa'b'= abab => aabb= abab =>ab=
ba. por lo que G es abcliano.
Soluciones

S. Si Tes un subgrupo de ff, entonces e11 e T y f(ec) = e11, por Jo que ec E 1


r
(T). Por lo tanto,
J"'(T) # 0. Si x, y E /"'(T), entooc,sf(x),f(y) E T. Como Tes un subgrapo de H ⇒ f(x)f(y);
f(xy) E T ⇒ xy E/"'(T).AdemAs,x E f"'(T) ⇒ J()<) ET ⇒ [f(x)]4 E T ⇒J(,c') E T⇒
x-1 E J-1(7). En consccuencia.f"1(n es un subgrupo de G.
7. ScanG;(g) y h ;f(g). Si h, EH,encooc,sh,;f(g") paraalgánn E Z, ya quejes sobre. Por
lo tanto, h,; f(g"); (f{g)J' ; li' y H; (h).
9, Paraa, b E G,

(aoa- 1)ob-1 ob = b ob- 1 o(a- 1 oa)=:>


a•a-1 •b =b• a- 1 oa=:>a•b =b•a,
de lo que se sigue que (G, o) es un grupo abeliaoo.
11. a) Consideremos una permutación a que se aicnta en P<..n + 1, .t). Si (n + 1) es un ciclo (de
longitud 1) en CJ, entonces o se aienta en PC,n. k - 1). En caso contrario, consideremos
cualquier permutación t que se cuente en P(n, k). Para cualquier ciclo de 't. digamos (a, a1
... a,.), existe n r posiciones en las cuales colocar n + 1: (1) entre 0 1 y a1; (2) entre ai y a1;
. • . ; (r- 1) cntrea,..1 y a,; y (r) entre a, y a 1 • Por lo tanto, tenemos un total den posiciones
para colocar n + l en t. En consecuencia, P(n + 1, k) = p(n, k-1) + nP(n, k).
b) ¿.t-l P(n. k) cuenta todas las permutaciones en S., el cual tienen! elementos.
13. o) (i) (;)(O.75)'(0.25)'
(ii) (l)(O. 75)'(0.25)' + (:)(O. 75)' (0.25) + (!)(O.75)'
b) Al menos 3

Capítulo 17
Cuerpos finitos y diseños
combin.illtoños

ección 17.1- l. f(x)+g(x)-2x' + Sx' + x'+5


~9- 842 /(x)g(x) =6x' + 2r6 + 3x~+ 4x' + 2x1 + .r2 + 4x + 4
J. (10)(11)'; (10)(11)'; (10)(11)'; (10)(11)"
7. a) y b)f(x); (r+ 4)(.t-2Xx + 2); las ralees son ±2.
e) f(x) ; ()< + 2,)(x- 2,)(x-2Xx + 2); las ralees son ±2, "2i.
d) (a) f(.x); (r - 5)(x'+ 5); no hay ralees racionales.
(b) f(x) ; (x - .,/sXu.,/5)(.t'+ 5); las ralees son ±.,/5.
(e) J(x); (x-.,/sXx +./sXx-.,/s1)(x+.,/s1); las ralees son ±.,/5, ±i.,/5.
9, a) /(3) ~ 80tl0 b) f(l) ~ l e) f(-9)-/(2);6
11. Como (Z. +. ·) es un dominio entero pero M un cuerpo. el algoritmo de la división para
polinomios no se cumple en Z(x].
13, f(x) ; (3x-lX7x - l);2Jx'-4x -l
15. 4;6;p-1
17. F.o Z.[x], (2:r + 1)(2.:c + l ) = 1, de modo que (lr + 1) es u.na Wlldad. Esto no contradice el
ejercicio 16 pues (Z., +,•) no es un dominio de integridad.
19, Scanf(.x) ; I :.Oa."''y h{x) ; 4-ob,.i', donde a, E Rpara0 s; s m,b, E R para O ,s; S ky
ms k. Entonces/(x) +h(x) = Z-o<a. +b;).t', dondea_ =a.,.¡ = . .. =a.t=z:,cJ cerodeR, de
1

modo que G(f(x) + h(x )) ;G(}:'..,.(a, + b,)x1) ; ~g(a1 +b;)x'; ¿:.. [g(a1)+g(b¡)]x' ;
I:.. g(a,)x' + k ..g(b,)x' ; G(f(x)) + G(h(x)). Además, f(x)hi,x) ; Z::,\x', donde e,;
a.bo+ a~1b1+ · · · + a1b,_, + aob; y
G(f(x)h(x))- GQ;;'_.'c,x~; ~...•g(c,)x'.
Soludones 5-89

Como g(cJ • g(aJg(bó) + g(a,.,)g(b,) + • • • + g(aJg(b,.,) + g(a,)g(b,), se sigue que

¿;-..;'g(c,)x' • a;."'..g(a,~ g ( b,)r') • G(f(z)) · G(h(.r)~


e,
En c:onsecuencia. Rl•l ➔ .11•1 es un bomomomsmo de anillos.
21. Observemos primero que para/(%) •a..r"'+a....r1 + · · · +a~+a,x+ aq. tenemosa.+a_1 +
• • • + a2 + a 1+ ao= Osi ysólo si/(1) =O. Puesto que el polinomio nulo está en S, el conjunto S
es no vado. Con/~) c:omocl polinomio dado, seag(.r) .s b,.r"+b-..1,i'H + • • • +b,r+ b.x + bo
e S. (Encsteeaso,m s n y param<n teoemosb-1=b..1• · · · :::;b.=0.) E.ntonccs/(1) - g(l)
• 0-0=0,demodoque/t,)- g(z) es.
Consideremos ahora h(r) • ¿'~r,.r' e K{,]. En este caso. h(z)//;c) E Kl•l y h(l)/(1) •
h(I) •O= O, por lo que h(r)Jt,) e s.
En consecueocia, Ses un ideal co X[.r).

Sección 17.2- l . a) ,'+ 3, - 1 es meduciblc sobre Q . Sobre R o C ,


pig. 851
.r' + 3x -1 = [z - ((-3 + Vl3)/2)][z -((- 3- VJJ)/2)].
b) x'-2 es i!TCduciblc sobre Q .
Sobrc R.x'-2=/;c - fíXx+ fí'Xx'+.fi);
r - 2=1;c- :/2 x,+ tí x, - :/ii x, +:/ii >sobre c.
e) , ' + x+ 1 • /;e + 2X, + 2) sobre Z,. Sobre Z,. ,' +z + 1 es ineduciblc;
x'+ x+ 1 = (x+ S)(;c + 3) sobn: z,.
d) x' + ,' + 1 es i!TCduciblc sobre Z,.
t) x1 + z + 1 es irreducible sobre 4
1) , '+ 3.r-• + 1 es irmlucible sobre Z,.
3. Gradol:.r.:c+I Grado2:.r+z+I Grado 3:.t'+x2+J;.r1+.l"+I
5. 7s
7. a) Sf, puesto que los coeficientes de los polinomios se tomaroa de un cuerpo.
b) h(,)ljt,), g(,) ⇒ /t,) = h(z)ul;c), g(x) = h(r)u(z) para M/;c). U/;c) e Kl•J.
m(;c) =,f,;,)/t,) +J(z)g(,) paras(,). J(z) e K{z], de modoqucm(z) •/,(r)(,l;c),,(z) +1/;c)U/;c)]
y h(z)jm(;c).
e) Sim(z),/' //;e), cnt_,.//;c) = q(z)m(;c) + ,(,).donde 0< grad ,(,) < gradm(z).
ml;c) = sl.z)/t,) + J(z)g(x). po< Jo que,(,)=//;c)-q(x)(J(z)//;c)+ J(x)g(,)]
• (1 - q(x)s(.x)) / /;e) - q(x)l/;e)g(x) y allOClCe$ ,(,) E S.
Como grad ,(,) < grad m(;c), cootradecimos la dcccióo de m(;c). Por Jo tanto. ,(,) = O y
m(,)jjt,).
,. a) El males /;e- 1) = (l/17)(x'-x'+ x'+.r-z-1) -(1/17)(.ru-2X.r-2.r+ Sz-8).
b) El mcd es 1 • /;e+ lXx'+x'+ 1) + l;c'+.r+zX.r+z + 1).
e) El mcd es,,'+ 2x+ 1 • (;e' + 2.r+ 2x+ 2) + /;e+ 2X2x'+ 2.r+ x+ 1).
U. a=0.b•O;a=0.bz 1
13. a) /(z) •/;/;cXmod s(z)) ⇒ /t,) =f,(z) + h(z)s(z) y g(z) • g,l;cXmod s(z)) ⇒ g(;c) • g,(z) +
l(z)s(.x). Por lo Wll.O.//;c) + g(,) =f,/;c) + g,(;c) + (h(z) + l(;c))s(.x), de modo quc/t,) + g(z) •
f,l;c) + 1,l;cXmod s<.x» y Jtx)g(xl = f,l;c)g,l;cl + C/,l;c)l(;cJ + g,l;c)l,(.rJ + h/;c)t(•l*ll*). de
modo que/t,),<,) a /,l;cl1,l;cXmod ,(,;,)).
b) Estas propiedades se siguen de las propiedades correspondientes de K{.:rJ. Por ejemplo, paca
la ley distributiva,
1/(x)J<Ig(x)) + [h(z)D • [/(z)][g(z) + h (z)) = [/(zXg(z) + h(x))]
• [/(z)g(z) + /(z)h(z)] =1/(z)g(x)] + 1/(z)h(z)]
• [/(z)J[g(z)] + [/(z)][h(z)].
e) F.n caso connario, existeg(x) E F[x] la! que gradg(x) > Oy g(x) lf(x), .r(x). Pero entooces,
s(x) seña reducible.
d) Unelememodislintode=deK!x]/(,(x)) tiene lafonna [f(x)],doode/(x)-;, Oy ¡¡rad/(x) <
grad .r(x). Como /(x) y ,(x) son primos relativos, existen r(x), l(x) tales que 1 = /(x)r(x) +
,(x)l(x),de modo que t -.11,)r(x)(mods(x))o [l]=(f(x)][r(x)J. PorlolalllO, (r(x)] = (f(x)]"'.
•> q"
15. a) [2x + t] b) [x + IJ e) [2x + 1] el) [2x]
17. a) p" b) «J," - 1)
!ll. a) 6 b) 12 e) 12 cl)mcm(m,n)
•> o !) o g) o
21. 101, 103, 107, IO!I, 113, 121, 125, 127, 128, 131, 137, 139, 149
23. Para,(x) =x'u'+x+ 2 E Z,(x), vemosqne,(0)=2.s(l) = 2ys(2)= l. Entonccs.porlapane
(b) del teorema 17.7 y las pa,teS (b) y (c)del teorema 17.ll,z,¡xY(sCx))cs un cuerpo finito con
31 = '1:1 elementos.
25. a) Como O = O+ o./2 E Q[./zJ, el conjunto Q[2] no es vacío. Para a+ b./2, e+ dJz E
Q[ ./zJ, tenemos (a+ b./2)-(c+d./z) = (a - e)+ (b-d"¡ ./2, con (a -c).(b-d"¡ E Q; y
(a + bJz)(e + d./2¡ = (ac • 2bd"¡ + (ad + be) Jz, con ac + '1bd. ad + be E Q.
F.n consecuencia. por la pane (a) del teo<ema 14.10, tenemos que Q[ Jz J es un subanillo de R.
b) Para mostrar que Q(./2) es un subcue,po de R dcbcmos_cnconttar en Q[./2] un inveno
multiplicativo paracadaclcmcntodistintodecerocnQ(./2]. Seaa+b./2 E Q[./2) talque
a+ bJ2 +O.Si b = O, entonces a+ O,,,-, E Q y ,r + O -./z E Q[./zJ. Para b O, +
necesitarnos encontrar e + d./2 E Q(./2] la! qne

(a+ bVÍ)(c + dV2) = 1


Ahora bien, (a• b./2 )(e +d./2) = 1 => (ac + 2bd"¡ +(ad+ be)./2 = 1 => ac + 2hd= 1
yad +be= O=>e= -adlb y a(-ad/b) + 2hd= 1 =>- a'd+ Ui'd= b =>d= b/(W-a') y
e =-al(W - a'). (Nota: W-a' +Opoes./2 es im>cioruil.) F.n coosccucncia. (a+ b./z )""' =
[-a/(2b'-a')J + [b/(2b' - a'}) Jz. con [- a/(2b'-a')J, (b/(2b' - a')J E Q. As/, Q(./zJ es un
subcuerpo de R.
e) Como s(x) = .r' - 2 es reducible sobre Q, por la pane (b) del teonem.a 17.ll sabemos que
Q(xY(x' - 2) cs un cuerpo.
Definimos la correspondencia/: Q(xY(x'- 2) ➔ Q(2] como

Jaa+bx l)=a+bV2.
Mediante un argumento similar al dado en el ejemplo 17.10 y la pane (a) del ejercicio 24 se
sigue que/es un isomorfismo.

~ción 17.3- l. a) 1 2 4 b) 1 2 3 4 e) 1 4 2
ig. 858 2 1 4 3 3 4 1 2 4 1 3
4 3 2 1 2 1 4 3 3 1 2 4
3 4 1 2 4 3 2 1 2 4 3 1
3. a':/ = a~ ⇒J,f,+ /;=/,/,+ /;⇒f;=/;⇒i=j
s. L,: 4 5 1 2 3 L.: s 1 2 3 4
2 3 4 s 1 4 s 1 2 3
s 1 2 3 4 3 4 s 1 2
3 4 s 1 2 2 3 4 s 1
1 2 3 4 s 1 2 3 4 5
Soluciones S-91

En la forma e5tándar, los cuadrados latinos L.. 1 s i s 4, son


L;: 1 2 3 4 5 L;: 1 2 4 5
2 3 4 5 1 3 4 1 2
3 4 5 1 2 5 1 3 4
4 5 1 2 3 2 3 5 1
5 1 2 3 4 4 5 2 3
L;: 1 2 3 4 r.:: 1 2 4
4 5 1 2 5 1 3 4
2 3 4 5 4 5 2 3
5 1 2 3 4 3 4 1 2
3 4 5 1 2 2 3 5 1
7. Utilizamos un tercer factor, como cuatro tipos de líquido de transmisión o cuatro tipos de
neumáticos.

Sección 17.4-
pág . 864
l.
Cuupo Número dt- putos Número de rectas ............
Númuo de pantos Númeroderectu
sobtt llll pa.nto

GF(S) 25 30 5 6
GF(3') 81 90 9 !O
GF(7) 49 56 7 8
GF(2') 256 272 16 17
GF(31) 961 992 31 32

3. Hay nueve puntos y doce rectas. Estas rectas están en cuatro clases paralelas.
(i) Pendiente O: y = O; y = 1;y= 2
(ii) Pendiente infinita: X= ();X : 1; X= 2
(iii) Pendiente 1:y=r.y=x+ l;y = x + 2
(iv) Pendiente 2 (como se muestra en la figura): (1) y= 2x (2) y= 2x + 1 (3) y = 2x + 2

El cuadrado latino correspondiente a la cuarta cJase paralela es .


3 1 2
2 3 1
1 2 3
5. a) y=4x+l b) y=lr +!Od2x+ 3y +3a0
e) y = !Ox o !Oy = llx
7. a) Recta vertical: x = e. La recta y =mx + b interseca esta recta vertical en el punto único
(e, me+ b). Cuando b toma los valores de K, no existen dos entradas de columna (sobre la
recta x = e) que sean iguales.
Recta horizontal: y = c. La recta y= mx + b interseca esta recta horizontal en el punto
ónico (m-1(c-b), e). Cuando b toma los valores de K, ningún par de filas (sobre la recta
y = e) son iguales.
S-92 Soluciones

Sección 17.5- l. v=9, b =12, r=4, k=3, ).= 1


pág. 869 3. ).=2
1 2 3 4 1 3 5 1
1 2 S 6 1 4 6 7 S 4 3 4 S 6
S. a) No b) No
7. a) ).(v-l)=r(k-1) =2r ⇒ ).(v - l)espaL
Av(v - 1) =w(k - 1) = bk(k-1) =b(3X2) ⇒61 ).v(v-1)
b) (X = l) 6 l).v(v-1)*6lv(v - 1)*3lv(v-1)*3lvo3l(v-1)
A(v -1) par* (u -1) par *v impar
*
3 lv v= 3', 1 impar* v=3(2s + 1) • 6., + 3 a v &3(mod 6)
3 l (v -l) ⇒v-1 =3', 1 par ⇒ v -1 =6r ⇒v=6r + 1 a'v • l(mod6)
9, v=9,r=4 11. a) b = 21 b) r=1

13. Hay A. bloques que contienen ax y y. Y como res el m1mero derq)lica del diseño. se sigue que
r - ). bloques contienen ax, pero no a y. De la misma forma. hay r- ). bloques que contienen
a y, pero no a z . En consecuencia. el número de bloques en el discfto que contienen a y o ax
cs(r-A.) + (r-A)+I= 2r-A..
IS. a) 31 b) 8
17. a) v =b=31;r=k=6;). =J b) v=b=51;r=k = 8;).=1
e) " =b = 73¡ r= k =9; X= 1

Ejercidos l. n • 9
complementarios- 3 . a) 31 b) 30 e) 29 d) k= 1000
p~g. 873 S. Para todo a E z,,
~= a [V6uc la panc (a) del ejercicio 15 al final de la sección 16.3.], de
modo que aes una raízdex'- xy x-a es un factordex' -x. Como cz,. +, ,)·es un cuerpo, el
polinomiox' -z puede tener un máximo dep raíces.. Por lo tanto,x'-x = fl.e:., (x-a).
7. Por la parte (a) del ejercicio complementario 4 del capítulo 14, sabemos que para Zi, z2 E C,
re R yn e N, tenemos (1) ~=Z¡ +zi'; (2) TZ¡=TZJ ==r~; y (3)@. Como/(z)E R{.r],
podemos escnlrir f(x) = a,.%"'+ a_. x-4 + • • • + a,il + a.,x: + ao. donde a, e R para O .:s j :S m.
Oxoof(a + M=O para a+ b; E C, ~ que L ;-o•i<• +biY= O. Por lo WllO, O =0 =
,...a1(a+biY = ¿"
¿" ,... a1 (a+b,)1 =¿J<')a;(a-biY y (a- b,) es una
,... a1(a+biY = L'
ralzde/(:c).
9. {l,2, 4},{2, 3,5), {4,5,7}
11. a) 9 b) 91
13. b) A· J,,es una matriz u X bcuyoelemento(i,J)es r, puesto que hay runos en cada fila de A
y cada demento de 1. es l. Por lo tanto, A • l •= rl..,..,._ De la misma fonna, 1• . A es una
mattiz u x b tal que su elemento (i, ]) es k. puesto que hay k unos en cada columna de A y
cada elemento de 1. es 1. Por lo tanto, l . · A =k • l . ,,, ►
e) El elemento (i, 1) de A • A• se obtiene mediante eJ producto componente a componente de
las filas j y j de A. Si i = j. esto produce el nómero de unos en la fila i, que es igual ar. Para
i :/- j, el nllmero de unos es el nllmero de veces que .x; y x, aparecen en el mismo bloque.
ndmero dado por A. Porlo tanto.A •A• = (r - A)J. + iJ,..
d) ). ~ X ~
). X X
A A A A
A A r .. . ).

A A ). A .•• r
Soluciones S-93

>.-r >.-r ). - r >.-r


). r->. o o o
). o , -). o o
O)
o o r-). o
). o r - >.

r + (v-1)>. o o o o
). r - ). o o o
~
). o r - ). o o
). o o r -). o
). o o o r -).
= [r + (v - 1)>.](r - >.¡-• = (r - >.)-'[r + r(k -1)] = rk(r - >.)-'

Oave: (1) Multiplicamos la columna 1 por-1 y la sumamos a las demás u- l columnas.


(2) Sumamos de la fila 2 basta la u a la fila 1.

Af>'ndke 1
Funciones uponendal
y logañtmlc.a
3y"'
~ g.A-10 l . a) Vxy =x"',,a b) ,els1.r-'y' = 3x-'"y"' • x"'
e) 5 ~ • 5(8"'x'"y-"') = 5(2r'y-'") = IOx'
y"'
3. a) 625 b) 1/343 e) 10
S. a) log,128=7 b) log,,,5•1/3 <) log,olJI0.000=-4 d) log,b • •
7. a) 3 e) 3 e) 1 1) 1/36
9. a) Demostración (por inducción matemMica):
P'aran = l, laproposici6oes log.rl = log..r, por lo que el resultado es cierto en este primer
caso. Supongamos que el n::sultadoes cierto paran:::-l:( ~ l)y tenc:m0$quelog.rt =lcl<>atr,
Ahora. paraelcasoenquen=k+ l,tenemosquelog..rM:::-Jog.(r· r)= log..r+log.,rA[por
la pane(l) del t«>remaAl.2] = log.r+ klog.r(porla hipóresisde induccióo) = log.(r •r') s
(1 + A:) log.r = (k + 1) log.. r. Por lo tanto, el íe$ultado se cumple para todo n E z• por el
principio de inducción matemática.
b) Para cada n E z.. log,r◄ = log,(1/r') = log, 1 - log, ,' [por la parte (2) del teo<ema.Al.2] =
O- n log. r [por la parte (a)] = (-n) log. r.
11. a) 1.5851 b) 0.4307 e) l.4ó50
13. a) 5/3 b) 3/2 e) 36 d) 4 e) 7
15. Seanx=...,..y y=c"'".Enwn<es

x = •-' ⇒ log.x = iog.[•-•J = (log.c)(log.a). y


y = c"'<- ' ⇒ Iog.y = log. (.-1 = (log.a)(log.c~

En consecuencia, tenemos quelog.x = log;y, de lo que se sigue que x = y.


S-94 Soluciones

Apéndlce2
Matrices, operaciones con matrices
y determinantes

~ g . A- 24 l. a)

.
[- 1 3 1 -8]

e) ~a21 =a21 +an+an+a24 = - 1 +3+ 1-8= -S


1;1
d) La,-=au+au+a34=-3-8+6=-5

3
3. a) A+8=[ü ~] b) (A+ 8 ) + e-[~
6 ~]
e) 8+C= [! 62 i] d) A+(8+C)~n
3
6 ~]
t)
2 8]
2A = [-i O 6 f) 2A + 38 = ¡1 s
1. 6 18
lll
&) 2C+3C=[ O S
2S 20 - IS
10] h) se=[ o s
2S 20 -IS
10]

o
[
Q 2B-4C= - 1s · - 12
2 -2
;gJ j) A+2B - 3C • [ - 1
4 -8 ~]
k) 2(38) = [6
6 126 z!] 6 6]
Q (z.3)8=[: 12 24

s. •J [12], o 12

d)[~ ~~ ~6]
- 23 -35
t) [ ~ :
3g 3h 3i
,]

7. a) (-1/s)[_; -i] b) [~ ~] e) Lainvenanoexiste. d) [! ~]

d) (A8)-' • (1/10)[2 - 3]
6 -4
Soluciones S-95

11. •> [! =;l~l-m


[;J-[! =rm-H>[=! ;füJ=m
b) [~ -m;J = [3;]
[;J-[~ _ff'[ ~J-<- >[=; -m3;J-m
3
1119

13. a) 21 b) 21 e) 21 d) 63

+c- 1x-1r•I! ~l• -3(2)+(10)+1=5

13 o -21
1 -1 =(-2)(-1)'· ·13 11+ (-l)(-l)'··ll º I
l 4 1 4 1
14 2

+2(-1¡•··1! ~1 - -2(-1)+1+2(1)•5

b> li i -;I = 1c-1¡•··I; -;1+ 1c-1rl~ - ; 1

+ 2(- 1¡•·•1~ ; I- c21 + 20)- (14) + 2(10) - 47

21 31 -421 - l(-1¡•·•1 -◄1 + 3(-1)'·'1 21


2 1
10 5 7 07 07

+ S(-1)'·•1~ _¡ , -(-1)(14) + 3(7)-S(-8) • 47

17. X= %2
S-96 Soluciones

19. a) (i) 1~ -i -:1


2 3 O
=2(-1)'·'1-2 _ 11+3(- 1)'•'11
1 1 O -1
11
=
2(-2 -(-1))-3(-1) = 2(- 1) + 3 = l.
(ü) 5 (iü) 25 (iv) 5 (v) 25 (vi) 125
b) (i) 51 (ü) 306 (iü) 510

Apéndlce3
Conjuntos numerables y no
numerables

~g.i><-37 l. a) Verdadera b) Falsa e) Verdadera d) Verdadera


e) Falsa: SeanA= Z U (0, l] yB= Z U (1, 2). EntoncesAy Bsooambos no numerables, pero
A n B =Z es numerable.
1) Verdadera
g) Falsa: Sean A= z· y
U (O. 1] B= (O, 1). Entonces A y B son ambos no numerables, pero
A -B = {2, 3, 4, ... } es numerable.
3 . Si B fuera numerable. entonces, por el teorema AJ.3, A seña numerable. Esto nos lleva a una
contradicción, pues antes obtuvimos que A es DO numerable:
5. Como S, T son infinitos. numerables, por el teorema A3.2 sabemos que podemos escribir S =
(s1,sbs)t • • . } yT= {ti, 12, I¡. . . },dossucesiones(in.finitas) delffllli.nosdistintos. Definimos
la función

f:Sxr-z•
comof(s,., tj) = 2'3', parai, j E z•. Si i,j, k, I E z• y/(s;, I¡) =f(sntJ, entoncesf(s¡, t) = f(St,IJ
⇒ 2'3'= 2'3' ⇒ i = k.,j = I (por el teorema fundamental de la aritm6tica) -:> s,.=S1 y t1= 11 =>
(si. t~ = (sn tJ. Por lo tanto.fes una función inyectiva y S x T- f (S x T) e z•. Así, por el
teorema A3.3, sabemos que S x T es numerable.
7. Como Z - {O} C Z sabemos que Z - {O} es numerable. Por la parte (b) del tjercióo 6,
sabemos que (Z- {O}) x Z x Z es numerable. Ahora. para cualquicr (a, b, e) E (Z- {O}) x
Zx z. existen cuando más dos soluciones reales (distintas) de la ecuación cuadrárica ar+ bx +
e= O. FJ teorema A3.9 implica entonces que el coojunto de todas las soluciones reales de las
ecuaciones cuadráticas aJ! + bx + e = O. tales que a, b, e E Z y a -/:. O, es numerable.

Você também pode gostar